Solution Manual For Finite Mathematics and Applied Calculus, 8th Edition by Stefan Waner,q Steven Co

Page 1


Solutions Section 0.1 Section 0.1

1. 2(4 + (−1))(2 ⋅ −4) = 2(3)(−8) = (6)(−8) = −48

2. 3 + ([4 − 2] ⋅ 9) = 3 + (2 × 9) = 3 + 18 = 21

3. 20∕(3 * 4) − 1 = 20 − 1 = 53 − 1 = 23 12

4. 2 − (3 * 4)∕10 = 2 − 12 = 2 − 65 = 45 10

5.

3 + ([3 + (−5)]) 3−2×2 3 + (−2) 1 = = = −1 3−4 −1

7. (2 − 5 * (−1))∕1 − 2 * (−1) 2 − 5 ⋅ (−1) = − 2 ⋅ (−1) 1 2+5 = +2=7+2=9 1

6.

12 − (1 − 4) 2(5 − 1) ⋅ 2 − 1 12 − (−3) 15 = = =1 16 − 1 15

8. 2 − 5 * (−1)∕(1 − 2 * (−1)) 5 ⋅ (−1) =2− 1 − 2 ⋅ (−1) 5 11 −5 =2− +2=2+ = 1+2 3 5

2 × (−1) 2 2 2×1 2 = = =1 2 2

10. 2 + 4 ⋅ 3 2 = 2 + 4 × 9 = 2 + 36 = 38

11. 2 ⋅ 4 2 + 1 = 2 × 16 + 1 = 32 + 1 = 33

12. 1 − 3 ⋅ (−2) 2 × 2 = 1 − 3 × 4 × 2 = 1 − 24 = −23

13. 3^2+2^2+1 = 3 2 + 2 2 + 1 = 9 + 4 + 1 = 14

14. 2^(2^2-2)

9. 2 ⋅ (−1) 2∕2 =

15.

3 − 2(−3) 2 −6(4 − 1) 2 3 − 2 × 9 3 − 18 = = −6 × 9 −6(3) 2 5 −15 = = −54 18

17. 10*(1+1/10)^3 1 3 = 10(1 + ) = 10(1.1) 3 10 = 10 × 1.331 = 13.31

2

= 2 (2 −2) = 2 4−2 = 2 2 = 4

16.

1 − 2(1 − 4) 2 2(5 − 1) 2 ⋅ 2 1 − 2(−3) 2 1−2×9 = = 2 × 16 × 2 2(4) 2 ⋅ 2 1 − 18 17 = =− 64 64

18. 121/(1+1/10)^2 121 121 = = 2 1 1.1 2 !1 + " 10

121 = = 100 1.21


−2 ⋅ 3 2 [ −(4 − 1) 2 ] −2 × 9 ⎤ −18 = 3⎡ = 3[ 2 −9 ] ⎣⎢ −3 ⎦⎥ =3×2=6

19. 3

Solutions Section 0.1

1 2 2 21. 3⎡1 − (− ) ⎤ + 1 ⎣⎢ 2 ⎦⎥ 1 2 = 3[1 − ] + 1 4 3 2 9 = 3[ ] + 1 = 3[ ] + 1 4 16 27 43 = +1= 16 16 2 = # 12 $ − 12 = 14 − 14 = 0 2

8(1 − 4) 2 20. − [ −9(5 − 1) 2 ] 8×9 72 = −[ = −( ] −9 × 16 −144 ) 1 1 = −(− ) = 2 2 1 2 2 2 22. 3⎡ − ( ) ⎤ + 1 ⎣⎢ 9 3 ⎦⎥ 1 4 2 = 3[ − ] + 1 9 9 −3 2 −1 2 = 3[ ] + 1 = 3[ ] + 1 9 3 1 3 4 =3 +1= +1= 9 9 3 2

= 22 − # 21 $ = 21 − 41 = −2

23. (1/2)^2-1/2^2

24. 2/(1^2)-(2/1)^2

25. 3 × (2 − 5) = 3*(2-5)

26. 4 +

27.

3 = 3/(2-5) 2−5

3−1 = (3-1)/(8+6) 8+6 Note 3-1/8-6 is wrong, as it corresponds to 3 − 18 − 6. 29.

31. 3 −

33.

4+7 = 3-(4+7)/8 8

2 − 𝑥𝑦 2 = 2/(3+x)-x*y^2 3+𝑥

35. 3.1𝑥 3 − 4𝑥 −2 −

60

𝑥2 − 1

= 3.1x^3-4x^(-2)-60/(x^2-1)

1

28.

4−1 = (4-1)/3 3

30. 3 +

32.

5 = 4+5/9 or 4+(5/9) 9

3 = 3+3/(2-9) 2−9

4×2 2

!3"

34. 3 +

= 4*2/(2/3) or (4*2)/(2/3)

3+𝑥 = 3+(3+x)/(x*y) 𝑥𝑦

𝑥2 − 3 2 = 2.1x^(-3)-x^(-1)+(x^2-3)/2

36. 2.1𝑥 −3 − 𝑥 −1 +


37.

2

Solutions Section 0.1

2

!3"

5 = (2/3)/5

38.

3

!5"

= 2/(3/5)

39. 3 4−5 × 6 = 3^(4-5)*6 Note that the entire exponent is in parentheses.

40.

4 41. 3 1 + ( 100 )

42. 3

−3

= 2/(3+5^(7-9)) Note that the entire exponent is in parentheses. 1+4 ( 100 ) = 3((1+4)/100)^(-3)

= 3*(1+4/100)^(-3)

43. 3 2𝑥−1 + 4 𝑥 − 1 = 3^(2*x-1)+4^x-1 2

45. 2 2𝑥 −𝑥+1 = 2^(2x^2-x+1) Note that the entire exponent is in parentheses.

47.

4𝑒 −2𝑥 2 − 3𝑒 −2𝑥 = 4*e^(-2*x)/(2-3e^(-2*x)) or 4(*e^(-2*x))/(2-3e^(-2*x)) or (4*e^(-2*x))/(2-3e^(-2*x)) 2 2

49. 3(1 − !− 12 " ) + 1

= 3(1-(-1/2)^2)^2+1

2 3+5 7−9

−3

2

44. 2 𝑥 − (2 2𝑥) 2 = 2^(x^2)-(2^(2*x))^2 2

46. 2 2𝑥 −𝑥 + 1 = 2^(2x^2-x)+1 Note that the entire exponent is in parentheses. 𝑒 2𝑥 + 𝑒 −2𝑥 𝑒 2𝑥 − 𝑒 −2𝑥 = (e^(2*x)+e^(-2*x))/(e^(2*x)e^(-2*x)) 48.

1 2 2 2 50. 3⎛ − ( ) ⎞ + 1 ⎜⎝ 9 3 ⎟⎠ = 3(1/9-(2/3)^2)^2+1


Solutions Section 0.2 Section 0.2 1. 3 3 = 27

2. (−2) 3 = −8

3. −(2 ⋅ 3) 2 = −(2 2 ⋅ 3 2) = −(4 ⋅ 9) = −36 or −(2 ⋅ 3) 2 = −6 2 = −36

4. (4 ⋅ 2) 2 = 4 2 ⋅ 2 2 = 16 ⋅ 4 = 64 or (4 ⋅ 2) 2 = 8 2 = 64

5. (

−2 2 (−2) 2 4 = = 3 ) 9 32

7. (−2) −3 =

3 3 3 3 27 6. ( ) = 3 = 2 8 2

1 1 1 = =− 8 (−2) 3 −8

1 1 8. −2 −3 = − 3 = − 8 2

1 −2 1 1 9. ( ) = = = 16 2 4 1∕16 (1∕4)

10. (

11. 2 ⋅ 3 0 = 2 ⋅ 1 = 2

12. 3 ⋅ (−2) 0 = 3 ⋅ 1 = 3

13. 2 32 2 = 2 3+2 = 2 5 = 32 or 2 32 2 = 8 ⋅ 4 = 32

14. 3 23 = 3 23 1 = 3 2+1 = 3 3 = 27or 3 23 = 9 ⋅ 3 = 27

15. 2 22 −12 42 −4 = 2 2−1+4−4 = 2 1 = 2

16. 5 25 −35 25 −2 = 5 2−3+2−2 = 5 −1 =

17. 𝑥 3𝑥 2 = 𝑥 3+2 = 𝑥 5

18. 𝑥 4𝑥 −1 = 𝑥 4−1 = 𝑥 3

19. −𝑥 2𝑥 −3𝑦 = −𝑥 2−3𝑦 = −𝑥 −1𝑦 = −

𝑦 𝑥

1 1 9 −2 −2 = = = 2 3 ) 4∕9 4 (−2∕3)

1 5

20. −𝑥𝑦 −1𝑥 −1 = −𝑥 1−1𝑦 −1 = −𝑥 0𝑦 −1 1 =− 𝑦

21.

𝑥3 1 = 𝑥 3−4 = 𝑥 −1 = 𝑥 𝑥4

22.

𝑦5 = 𝑦 5−3 = 𝑦 2 3 𝑦

23.

𝑥 2𝑦 2 = 𝑥 2−(−1)𝑦 2−1 = 𝑥 3𝑦 𝑥 −1𝑦

24.

𝑥 −1𝑦 1 = 𝑥 −1−2𝑦 1−2 = 𝑥 −3𝑦 −1 = 3 𝑥 2𝑦 2 𝑥 𝑦


25.

(𝑥𝑦 −1𝑧 3) 2 𝑥 2(𝑦 −1) 2(𝑧 3) 2 = 𝑥 2𝑦𝑧 2 𝑥 2𝑦𝑧 2 2 −2 6 𝑥 𝑦 𝑧 = 2 2 = 𝑥 2−2𝑦 −2−1𝑧 6−2 𝑥 𝑦𝑧 𝑧4 = 𝑥 0𝑦 −3𝑧 4 = 3 𝑦

27. !

𝑥𝑦 −2𝑧 3 (𝑥𝑦 −2𝑧) 3 " = 𝑥 −1𝑧 (𝑥 −1𝑧) 3 3 −6 3 𝑥 𝑦 𝑧 = −3 3 = 𝑥 3−(−3)𝑦 −6𝑧 3−3 𝑥 𝑧 𝑥6 = 𝑥 6𝑦 −6𝑧 0 = 𝑦6 𝑥 −1𝑦 −2𝑧 2 −1−1 −2−1 2 −2 𝑦 𝑧 ) ) = (𝑥 𝑥𝑦 = (𝑥 −2𝑦 −3𝑧 2) −2 = 𝑥 4𝑦 6𝑧 −4 𝑥 4𝑦 6 = 4 𝑧 −2

29. (

31. √4 = 2

𝑥 2𝑦𝑧 2 𝑥 2𝑦𝑧 2 = (𝑥𝑦𝑧 −1) −1 𝑥 −1𝑦 −1(𝑧 −1) −1 𝑥 2𝑦𝑧 2 = −1 −1 = 𝑥 2−(−1)𝑦 1−(−1)𝑧 2−1 𝑥 𝑦 𝑧 = 𝑥 3𝑦 2𝑧 4

Solutions Section 0.2 26.

𝑥 2𝑦 −1𝑧 0 (𝑥 2𝑦 −1𝑧 0) 2 = 𝑥𝑦𝑧 ) (𝑥𝑦𝑧) 2 𝑥 4𝑦 −2 = 2 2 2 = 𝑥 4−2𝑦 −2−2𝑧 −2 𝑥 𝑦 𝑧 𝑥2 = 𝑥 2𝑦 −4𝑧 −2 = 4 2 𝑦 𝑧 2

28. (

30.

𝑥𝑦 −2 = (𝑥 1−2𝑦 −2+1𝑧 −1) −3 ( 𝑥 2𝑦 −1𝑧 ) = (𝑥 −1𝑦 −1𝑧 −1) −3 = 𝑥 3𝑦 3𝑧 3 −3

32. √5 ≈ 2.236

33.

1 √1 1 = = √ 4 √4 2

34.

1 √1 1 = = √ 9 √9 3

35.

16 √16 4 = = √9 3 √9

36.

9 √9 3 = = √ 4 √4 2

37.

√4

5

=

2 5

38.

6

√25

=

6 5

39. √9 + √16 = 3 + 4 = 7

40. √25 − √16 = 5 − 4 = 1

41. √9 + 16 = √25 = 5

42. √25 − 16 = √9 = 3

3

3

3

3 √27

43. √8 − 27 = √−19 ≈ −2.668 45. √27∕8 =

3 √8

=

3 2

4

4

44. √81 − 16 = √65 ≈ 2.839 3

3

3

46. √8 × 64 = √8 ⋅ √64 = 2 ⋅ 4 = 8


Solutions Section 0.2

47. √(−2) 2 = √4 = 2

49.

48. √(−1) 2 = √1 = 1

1 16 √16 4 (1 + 15) = = = =2 √4 √4 √4 2

50.

1 36 √36 6 (3 + 33) = = = =2 √9 √9 3 √9

51. √𝑎 2𝑏 2 = √𝑎 2√𝑏 2 = 𝑎𝑏

𝑎 2 √𝑎 2 𝑎 52. √ 2 = = 𝑏 𝑏 √𝑏 2

53. √(𝑥 + 9) 2 = 𝑥 + 9 (𝑥 + 9 > 0 because 𝑥 is positive.)

54. (√𝑥 + 9) = 𝑥 + 9

3

3

3

55. √𝑥 3(𝑎 3 + 𝑏 3) = √𝑥 3 √(𝑎 3 + 𝑏 3) 3

= 𝑥 √(𝑎 3 + 𝑏 3)

57.

(Not 𝑥(𝑎 + 𝑏))

4𝑥𝑦 3 4𝑦 2 √4√𝑦 2 2𝑦 = = = √ 2 √𝑥 √𝑥 𝑥 √𝑥 𝑦

2

4

56. √

58.

4

√𝑥 4 𝑥4 𝑥 = = 4 4 𝑎𝑏 𝑎 4𝑏 4 √𝑎 4 √𝑏 4

4(𝑥 2 + 𝑦 2) √4√𝑥 2 + 𝑦 2 = 𝑐2 √𝑐 2 √ =

2√𝑥 2 + 𝑦 2 𝑐

(Not 2(𝑥 + 𝑦)∕𝑐)


Solutions Section 0.3 Section 0.3

3 𝑥4

2.

1 −4 1 𝑥 = 4 2 2𝑥

3 −2∕3 3 𝑥 = 4 4𝑥 2∕3

4.

4 −3∕4 4 𝑦 = 5 5𝑦 3∕4

6.

1 0.1𝑥 −2 𝑥 4 0.1 + = + 2 3 3 3𝑥 −4 3𝑥

1. 3𝑥 −4 =

3.

6 −1 6 5. 1 − 0.3 = 1 − 0.3𝑥 2 − 5𝑥 −2 − 5 𝑥 𝑥

3

5

7. 2 2∕3 = √2 2

8. 3 4∕5 = √3 4

3

3

3

3

9. 𝑥 4∕3 = √𝑥 4 = √𝑥 3 √𝑥 = 𝑥 √𝑥 11. (𝑥 1∕2𝑦 1∕3)

1∕5

5

= √√ 𝑥 √ 𝑦 3

3 3 3 13. − 𝑥 −1∕4 = − =− 4 1∕4 2 2𝑥 2 √𝑥 15. 0.2𝑥 −2∕3 +

3

7𝑥 −1∕2 0.2 3√ 𝑥 = 3 + 7 √𝑥 2

17.

=

0.2 3𝑥 1∕2 + 7 𝑥 2∕3

3 3 = 5∕2 4(1 − 𝑥) 4√(1 − 𝑥) 5 3 = 4√(1 − 𝑥) 4√1 − 𝑥 3 = 4(1 − 𝑥) 2√1 − 𝑥

4

4

4

4

10. 𝑦 7∕4 = √𝑦 7 = √𝑦 4 √𝑦 3 = |𝑦| √𝑦 3 12. 𝑥 −1∕3𝑦 3∕2 =

14.

16.

18.

𝑦 3∕2 √𝑦 3 |𝑦|√𝑦 = 3 = 3 𝑥 1∕3 √𝑥 √𝑥

4 3∕2 4√𝑥 3 4|𝑥|√𝑥 𝑥 = = 5 5 5 3.1 11 −1∕7 11 𝑥 − = 3.1𝑥 4∕3 − −4∕3 7 𝑥 7𝑥 1∕7 3 3 11 11 = 3.1 √𝑥 4 − 7 = 3.1𝑥 √𝑥 − 7 7 √𝑥 7 √𝑥 9(1 − 𝑥) 7∕3 9 = 4 4(1 − 𝑥) −7∕3 3

3

9 √(1 − 𝑥) 7 9 √(1 − 𝑥) 6 √1 − 𝑥 = = 4 4 3 9(1 − 𝑥) 2 √1 − 𝑥 = 4

19. √3 = 3 1∕2

20. √8 = 8 1∕2

21. √𝑥 3 = 𝑥 3∕2

22. √𝑥 2 = 𝑥 2∕3

3

3


3

23. √𝑥𝑦 2 = (𝑥𝑦 2) 1∕3 25.

Solutions Section 0.3

𝑥2 𝑥2 = = 𝑥 2−1∕2 = 𝑥 3∕2 √𝑥 𝑥 1∕2

24. √𝑥 2𝑦 = (𝑥 2𝑦) 1∕2 26.

𝑥

√𝑥

=

𝑥

𝑥 1∕2

= 𝑥 1−1∕2 = 𝑥 1∕2

27.

3 3 = 𝑥 −2 2 5 5𝑥

28.

2 2 = 𝑥3 −3 5 5𝑥

29.

3𝑥 −1.2 1 3 1 − 2.1 = 𝑥 −1.2 − 𝑥 −2.1 2 2 3 3𝑥

30.

2 𝑥 2.1 2 1.2 1 2.1 − = 𝑥 − 𝑥 3 3 3 3𝑥 −1.2

31.

33.

2𝑥 𝑥 0.1 4 − + 1.1 3 2 3𝑥 2 1 4 = 𝑥 − 𝑥 0.1 + 𝑥 −1.1 3 2 3 3√𝑥 5 4 − + 4 3√𝑥 3𝑥√𝑥 3𝑥 1∕2 5 4 = − + 1∕2 4 3𝑥 3𝑥 ⋅ 𝑥 1∕2 3 5 4 = 𝑥 1∕2 − 𝑥 −1∕2 + 𝑥 −3∕2 4 3 3 5

3 √𝑥 2 7 3𝑥 2∕5 7 35. − = − 4 4 2𝑥 3∕2 2√𝑥 3 3 7 = 𝑥 2∕5 − 𝑥 −3∕2 4 2

34.

36.

38.

39. 4 −1∕24 7∕2 = 4 −1∕2+7∕2 = 4 6∕2 = 4 3 = 64

40.

(𝑥 2 + 1) 3

3

4 √𝑥 2 + 1 1 3 = − (𝑥 2 + 1) 3 (𝑥 2 + 1) 1∕3 3 = (𝑥 2 + 1) −3 − (𝑥 2 + 1) −1∕3 4

41. 3 2∕33 −1∕6 = 3 2∕3−1∕6 = 3 1∕2 = √3

4𝑥 2 𝑥 3∕2 2 + − 2 3 6 3𝑥 4 1 2 = 𝑥 2 + 𝑥 3∕2 − 𝑥 −2 3 6 3 3

5√𝑥

5√𝑥 7 + 3 8 2 √𝑥

3 5𝑥 1∕2 7 − + 1∕2 8 5𝑥 2𝑥 1∕3 3 −1∕2 5 1∕2 7 −1∕3 = 𝑥 − 𝑥 + 𝑥 5 8 2 =

3

37.

1

32.

1 2 1 2 − = − 5 3∕2 √ 3𝑥 3∕5 8𝑥 𝑥 3 √𝑥 3 8𝑥 1 2 = 𝑥 −3∕2 − 𝑥 −3∕5 8 3 3

3 √(𝑥 2 + 1) 7 2 − 4 3(𝑥 2 + 1) −3 2 2 3 = (𝑥 + 1) 3 − (𝑥 2 + 1) 7∕3 3 4 2 1∕𝑎 1 = 2 1∕𝑎−2∕𝑎 = 2 −1∕𝑎 = 2∕𝑎 1∕𝑎 2 2

42. 2 1∕32 −12 2∕32 −1∕3 = 2 1∕3−1+2∕3−1∕3 1 = 2 −1∕3 = 1∕3 2


43.

𝑥 3∕2 1 = 𝑥 3∕2−5∕2 = 𝑥 −1 = 5∕2 𝑥 𝑥

45.

𝑥 1∕2𝑦 2 = 𝑥 1∕2−(−1∕2)𝑦 2−1 = 𝑥𝑦 𝑥 −1∕2𝑦

47.

44.

𝑦 5∕4 = 𝑦 5∕4−3∕4 = 𝑦 1∕2 = √𝑦 𝑦 3∕4

46.

𝑥 −1∕2𝑦 = 𝑥 −1∕2−2𝑦 1−3∕2 = 𝑥 −5∕2𝑦 −1∕2 𝑥 2𝑦 3∕2

Solutions Section 0.3

𝑥 1∕3 𝑦 2∕3 𝑥 1∕3 𝑦 2∕3 ⋅ = ( 𝑦 ) (𝑥) 𝑦 1∕3 𝑥 2∕3 = 𝑥 1∕3−2∕3𝑦 −1∕3+2∕3 = 𝑥 −1∕3𝑦 1∕3 or ( 𝑥𝑦 ) 1∕3

48.

49. 𝑥 2 − 16 = 0 ⇒ 𝑥 2 = 16 ⇒ 𝑥 = ±√16 ⇒ 𝑥 = ±4

𝑥 −1∕3 𝑦 1∕3 𝑥 −1∕3 𝑦 1∕3 ⋅ = (𝑥) (𝑦) 𝑦 −1∕3 𝑥 1∕3 𝑥 −1∕3−1∕3𝑦 1∕3+1∕3 = 𝑥 −2∕3𝑦 2∕3 or ( 𝑥𝑦 ) 2∕3

50. 𝑥 2 − 1 = 0 ⇒ 𝑥 2 = 1 ⇒ 𝑥 = ±√1 ⇒ 𝑥 = ±1

4 4 = 0 ⇒ 𝑥2 = 9 9 4 2 ⇒𝑥=± ⇒𝑥=± √9 3

1 1 = 0 ⇒ 𝑥2 = 10 10 1 1 ⇒𝑥=± ⇒𝑥=± √ 10 √10

51. 𝑥 2 −

52. 𝑥 2 −

53. 𝑥 2 − (1 + 2𝑥) 2 = 0 ⇒ 𝑥 2 = (1 + 2𝑥) 2 ⇒ 𝑥 = ±1 + 2𝑥 If 𝑥 = 1 + 2𝑥, then −𝑥 = 1 ⇒ 𝑥 = −1. 1 If 𝑥 = −(1 + 2𝑥), then = −1 ⇒ 𝑥 = − . 3 1 So, 𝑥 = 1 or − . 3

54. 𝑥 2 − (2 − 3𝑥) 2 = 0 ⇒ 𝑥 2 = (2 − 3𝑥) 2 ⇒ 𝑥 = ±2 − 3𝑥 1 If 𝑥 = 2 − 3𝑥, then 4𝑥 = 2 ⇒ 𝑥 = . 2 If 𝑥 = −(2 − 3𝑥), then −2𝑥 = −2 ⇒ 𝑥 = 1. 1 So, 𝑥 = 1 or . 2

55. 𝑥 5 + 32 = 0 ⇒ 𝑥 5 = −32

56. 𝑥 4 − 81 = 0 ⇒ 𝑥 4 = 81 4 ⇒ 𝑥 = ± √81 = ±3

57. 𝑥 1∕2 − 4 = 0 ⇒ 𝑥 1∕2 = 4 ⇒ 𝑥 = 4 2 = 16

58. 𝑥 1∕3 − 2 = 0 ⇒ 𝑥 1∕3 = 2 ⇒ 𝑥 = 23 = 8

⇒𝑥=

5 √−32 = −2

1 1 =0⇒1= 2 2 𝑥 𝑥 ⇒ 𝑥 2 = 1 ⇒ 𝑥 = ±√1 = ±1

59. 1 −

61. (𝑥 − 4) −1∕3 = 2 ⇒ 𝑥 − 4 = 2 −3 = ⇒𝑥=4+

1 33 = 8 8

60.

1 8

2 6 2 6 − 4=0⇒ 3= 4 3 𝑥 𝑥 𝑥 𝑥 ⇒ 2𝑥 4 = 6𝑥 3 ⇒ 2𝑥 = 6 ⇒ 𝑥 = 3

62. (𝑥 − 4) 2∕3 + 1 = 5 ⇒ (𝑥 − 4) 2∕3 = 4 ⇒ 𝑥 − 4 = ±4 3∕2 = ±8 ⇒ 𝑥 = 4 ± 8 = −4 or 12


Solutions Section 0.4 Section 0.4

1. 𝑥(4𝑥 + 6) = 4𝑥 2 + 6𝑥

2. (4𝑦 − 2)𝑦 = 4𝑦 2 − 2𝑦

3. (2𝑥 − 𝑦)𝑦 = 2𝑥𝑦 − 𝑦 2

4. 𝑥(3𝑥 + 𝑦) = 3𝑥 2 + 𝑥𝑦

5. (𝑥 + 1)(𝑥 − 3) = 𝑥 2 + 𝑥 − 3𝑥 − 3 = 𝑥 2 − 2𝑥 − 3

6. (𝑦 + 3)(𝑦 + 4) = 𝑦 2 + 3𝑦 + 4𝑦 + 12 = 𝑦 2 + 7𝑦 + 12

7. (2𝑦 + 3)(𝑦 + 5) = 2𝑦 2 + 3𝑦 + 10𝑦 + 15 = 2𝑦 2 + 13𝑦 + 15

8. (2𝑥 − 2)(3𝑥 − 4) = 6𝑥 2 − 6𝑥 − 8𝑥 + 8 = 6𝑥 2 − 14𝑥 + 8

9. (2𝑥 − 3) 2 = 4𝑥 2 − 12𝑥 + 9

10. (3𝑥 + 1) 2 = 9𝑥 2 + 6𝑥 + 1

2

2

11. !𝑥 + 𝑥1 " = 𝑥 2 + 2 + 12

12. !𝑦 − 1𝑦 " = 𝑦 2 − 2 + 12

13. (2𝑥 − 3)(2𝑥 + 3) = (2𝑥) 2 − 3 2 = 4𝑥 2 − 9

14. (4 + 2𝑥)(4 − 2𝑥) = 4 2 − (2𝑥) 2 = 16 − 4𝑥 2

𝑥

𝑦

15. !𝑦 − 1𝑦 "!𝑦 + 1𝑦 " = 𝑦 2 − ! 1𝑦 " = 𝑦 2 − 12

16. (𝑥 − 𝑥 2)(𝑥 + 𝑥 2) = 𝑥 2 − (𝑥 2) 2 = 𝑥 2 − 𝑥 4

17. (𝑥 2 + 𝑥 − 1)(2𝑥 + 4) = 𝑥 2(2𝑥 + 4) + 𝑥(2𝑥 + 4) − 1(2𝑥 + 4) = 2𝑥 3 + 4𝑥 2 + 2𝑥 2 + 4𝑥 − 2𝑥 − 4 = 2𝑥 3 + 6𝑥 2 + 2𝑥 − 4

18. (3𝑥 + 1)(2𝑥 2 − 𝑥 + 1) = 3𝑥(2𝑥 2 − 𝑥 + 1) + 1(2𝑥 2 − 𝑥 + 1) = 6𝑥 3 − 3𝑥 2 + 3𝑥 + 2𝑥 2 − 𝑥 + 1 = 6𝑥 3 − 𝑥 2 + 2𝑥 + 1

2

𝑦

19. (𝑥 2 − 2𝑥 + 1) 2 = (𝑥 2 − 2𝑥 + 1)(𝑥 2 − 2𝑥 + 1) = 𝑥 2(𝑥 2 − 2𝑥 + 1) − 2𝑥(𝑥 2 − 2𝑥 + 1) + 1(𝑥 2 − 2𝑥 + 1) = 𝑥 4 − 2𝑥 3 + 𝑥 2 − 2𝑥 3 + 4𝑥 2 − 2𝑥 + 𝑥 2 − 2𝑥 + 1 = 𝑥 4 − 4𝑥 3 + 6𝑥 2 − 4𝑥 + 1 20. (𝑥 + 𝑦 − 𝑥𝑦) 2 = (𝑥 + 𝑦 − 𝑥𝑦)(𝑥 + 𝑦 − 𝑥𝑦) = 𝑥(𝑥 + 𝑦 − 𝑥𝑦) + 𝑦(𝑥 + 𝑦 − 𝑥𝑦) − 𝑥𝑦(𝑥 + 𝑦 − 𝑥𝑦) = 𝑥 2 + 𝑥𝑦 − 𝑥 2𝑦 + 𝑥𝑦 + 𝑦 2 − 𝑥𝑦 2 − 𝑥 2𝑦 − 𝑥𝑦 2 + 𝑥 2𝑦 2 = 𝑥 2 + 2𝑥𝑦 + 𝑦 2 − 2𝑥 2𝑦 − 2𝑥𝑦 2 + 𝑥 2𝑦 2

21. (𝑦 3 + 2𝑦 2 + 𝑦)(𝑦 2 + 2𝑦 − 1) = 𝑦 3(𝑦 2 + 2𝑦 − 1) + 2𝑦 2(𝑦 2 + 2𝑦 − 1) + 𝑦(𝑦 2 + 2𝑦 − 1) = 𝑦 5 + 2𝑦 4 − 𝑦 3 + 2𝑦 4 + 4𝑦 3 − 2𝑦 2 + 𝑦 3 + 2𝑦 2 − 𝑦 = 𝑦 5 + 4𝑦 4 + 4𝑦 3 − 𝑦


22. (𝑥 − 2𝑥 + 4)(3𝑥 − 𝑥 + 2) = 𝑥 3(3𝑥 2 − 𝑥 + 2) − 2𝑥 2(3𝑥 2 − 𝑥 + 2) + 4(3𝑥 2 − 𝑥 + 2) = 3𝑥 5 − 𝑥 4 + 2𝑥 3 − 6𝑥 4 + 2𝑥 3 − 4𝑥 2 + 12𝑥 2 − 4𝑥 + 8 = 3𝑥 5 − 7𝑥 4 + 4𝑥 3 + 8𝑥 2 − 4𝑥 + 8 3

2

2

Solutions Section 0.4

23. (𝑥 + 1)(𝑥 + 2) + (𝑥 + 1)(𝑥 + 3) = (𝑥 + 1)(𝑥 + 2 + 𝑥 + 3) = (𝑥 + 1)(2𝑥 + 5)

25. (𝑥 2 + 1) 5(𝑥 + 3) 4 + (𝑥 2 + 1) 6(𝑥 + 3) 3 = (𝑥 2 + 1) 5(𝑥 + 3) 3(𝑥 + 3 + 𝑥 2 + 1) = (𝑥 2 + 1) 5(𝑥 + 3) 3(𝑥 2 + 𝑥 + 4)

24. (𝑥 + 1)(𝑥 + 2) 2 + (𝑥 + 1) 2(𝑥 + 2) = (𝑥 + 1)(𝑥 + 2)(𝑥 + 2 + 𝑥 + 1) = (𝑥 + 1)(𝑥 + 2)(2𝑥 + 3)

26. 10𝑥(𝑥 2 + 1) 4(𝑥 3 + 1) 5 + 15𝑥 2(𝑥 2 + 1) 5(𝑥 3 + 1) 4 = 5𝑥(𝑥 2 + 1) 4(𝑥 3 + 1) 4[2(𝑥 3 + 1) + 3𝑥(𝑥 2 + 1)] = 5𝑥(𝑥 2 + 1) 4(𝑥 3 + 1) 4(5𝑥 3 + 3𝑥 + 2)

27. (𝑥 3 + 1)√𝑥 + 1 − (𝑥 3 + 1) 2√𝑥 + 1 = (𝑥 3 + 1)√𝑥 + 1 ⋅ [1 − (𝑥 3 + 1)] = −𝑥 3(𝑥 3 + 1)√𝑥 + 1

28. (𝑥 2 + 1)√𝑥 + 1 − √(𝑥 + 1) 3 = √𝑥 + 1 ⋅ [𝑥 2 + 1 − √(𝑥 + 1) 2] = √𝑥 + 1 ⋅ [𝑥 2 + 1 − (𝑥 + 1)] = (𝑥 2 − 𝑥)√𝑥 + 1 = 𝑥(𝑥 − 1)√𝑥 + 1

29. √(𝑥 + 1) 3 + √(𝑥 + 1) 5 = √(𝑥 + 1) 3 ⋅ [1 + √(𝑥 + 1) 2] = √(𝑥 + 1) 3 ⋅ (1 + 𝑥 + 1) = (𝑥 + 2)√(𝑥 + 1) 3

30. (𝑥 2 + 1) √(𝑥 + 1) 4 − √(𝑥 + 1) 7 3

3

3

3

= √(𝑥 + 1) 4 ⋅ [𝑥 2 + 1 − √(𝑥 + 1) 3] 3

= √(𝑥 + 1) 4 ⋅ [𝑥 2 + 1 − (𝑥 + 1)] 3

= (𝑥 2 − 𝑥) √(𝑥 + 1) 4 3

= 𝑥(𝑥 − 1) √(𝑥 + 1) 4

31. 𝑎 = 2, 𝑏 = 6, 𝑐 = 5, so that the discriminant is 𝑏 2 − 4𝑎𝑐 = 6 2 − 4(2)(5) = 36 − 40 = −4 As the discriminant is negative, the expression does not factor at all.

32. 𝑎 = 4, 𝑏 = −6, 𝑐 = 2, so that the discriminant is 𝑏 2 − 4𝑎𝑐 = (−6) 2 − 4(4)(2) = 36 − 32 = 4 = 2 2 As the discriminant is a perfect square, the expression factors over the integers.

33. 𝑎 = −3, 𝑏 = −2, 𝑐 = 3, so that the discriminant is 𝑏 2 − 4𝑎𝑐 = (−2) 2 − 4(−3)(3) = 4 + 36 = 40 As the discriminant is positive but not a perfect square, the expression factors, but not over the integers. 34. 𝑎 = 1, 𝑏 = −4, 𝑐 = −7, so that the discriminant is 𝑏 2 − 4𝑎𝑐 = (−4) 2 − 4(1)(−7) = 16 + 28 = 44 As the discriminant is positive but not a perfect square, the expression factors, but not over the integers. 35. 𝑎 = 8, 𝑏 = 12, 𝑐 = 4, so that the discriminant is 𝑏 2 − 4𝑎𝑐 = 12 2 − 4(8)(4) = 144 − 128 = 16 = 4 2 As the discriminant is a perfect square, the expression factors over the integers.


Solutions Section 0.4 36. 𝑎 = 1, 𝑏 = 2, 𝑐 = 19, so that the discriminant is 𝑏 2 − 4𝑎𝑐 = 2 2 − 4(1)(19) = 4 − 76 = −72 As the discriminant is negative, the expression does not factor at all.

37. 𝑎 = 40, 𝑏 = −64, 𝑐 = 24, so that the discriminant is 𝑏 2 − 4𝑎𝑐 = (−64) 2 − 4(40)(24) = 4,096 − 3,840 = 256 = 16 2 As the discriminant is a perfect square, the expression factors over the integers. 38. 𝑎 = −10, 𝑏 = −32, 𝑐 = −32, so that the discriminant is 𝑏 2 − 4𝑎𝑐 = (−32) 2 − 4(−10)(−32) = 1,024 − 1,280 = −256 As the discriminant is negative, the expression does not factor at all.

39. 𝑎 = 6, 𝑏 = −22, 𝑐 = 16, so that the discriminant is 𝑏 2 − 4𝑎𝑐 = (−22) 2 − 4(6)(16) = 484 − 384 = 100 = 10 2 As the discriminant is a perfect square, the expression factors over the integers. 40. 𝑎 = 48, 𝑏 = 32, 𝑐 = 4, so that the discriminant is 𝑏 2 − 4𝑎𝑐 = 32 2 − 4(48)(4) = 1,024 − 768 = 256 = 16 2 As the discriminant is a perfect square, the expression factors over the integers. 41. a. 2𝑥 + 3𝑥 2 = 𝑥(2 + 3𝑥) b. 𝑥(2 + 3𝑥) = 0 𝑥 = 0 or 2 + 3𝑥 = 0 𝑥 = 0 or −2∕3

42. a. 𝑦 2 − 4𝑦 = 𝑦(𝑦 − 4) b. 𝑦(𝑦 − 4) = 0 𝑦 = 0 or 𝑦 − 4 = 0 𝑦 = 0 or 4

43. a. 6𝑥 3 − 2𝑥 2 = 2𝑥 2(3𝑥 − 1) b. 2𝑥 2(3𝑥 − 1) = 0 𝑥 2 = 0 or 3𝑥 − 1 = 0 𝑥 = 0 or 1∕3

44. a. 3𝑦 3 − 9𝑦 2 = 3𝑦 2(𝑦 − 3) b. 3𝑦 2(𝑦 − 3) = 0 𝑦 2 = 0 or 𝑦 − 3 = 0 𝑦 = 0 or 3

45. a. 𝑥 2 − 8𝑥 + 7 = (𝑥 − 1)(𝑥 − 7) b. (𝑥 − 1)(𝑥 − 7) = 0 𝑥 − 1 = 0 or 𝑥 − 7 = 0 𝑥 = 1 or 7

46. a. 𝑦 2 + 6𝑦 + 8 = (𝑦 + 2)(𝑦 + 4) b. (𝑦 + 2)(𝑦 + 4) = 0 𝑦 + 2 = 0 or 𝑦 + 4 = 0 𝑦 = −2 or −4

47. a. 𝑥 2 + 𝑥 − 12 = (𝑥 − 3)(𝑥 + 4) b. (𝑥 − 3)(𝑥 + 4) = 0 𝑥 − 3 = 0 or 𝑥 + 4 = 0 𝑥 = 3 or −4

48. a. 𝑦 2 + 𝑦 − 6 = (𝑦 − 2)(𝑦 + 3) b. (𝑦 − 2)(𝑦 + 3) = 0 𝑦 − 2 = 0 or 𝑦 + 3 = 0 𝑦 = 2 or −3

49. a. 2𝑥 2 − 3𝑥 − 2 = (2𝑥 + 1)(𝑥 − 2) b. (2𝑥 + 1)(𝑥 − 2) = 0 2𝑥 + 1 = 0 or 𝑥 − 2 = 0 𝑥 = −1∕2 or 2

50. a. 3𝑦 2 − 8𝑦 − 3 = (3𝑦 + 1)(𝑦 − 3) b. (3𝑦 + 1)(𝑦 − 3) = 0 3𝑦 + 1 = 0 or 𝑦 − 3 = 0 𝑦 = −1∕3 or 3


51. a. 6𝑥 2 + 13𝑥 + 6 = (2𝑥 + 3)(3𝑥 + 2) b. (2𝑥 + 3)(3𝑥 + 2) = 0 2𝑥 + 3 = 0 or 3𝑥 + 2 = 0 𝑥 = −3∕2 or −2∕3

52. a. 6𝑦 2 + 17𝑦 + 12 = (3𝑦 + 4)(2𝑦 + 3) b. (3𝑦 + 4)(2𝑦 + 3) = 0 3𝑦 + 4 = 0 or 2𝑦 + 3 = 0 𝑦 = −4∕3 or −3∕2

53. a. 12𝑥 2 + 𝑥 − 6 = (3𝑥 − 2)(4𝑥 + 3) b. (3𝑥 − 2)(4𝑥 + 3) = 0 3𝑥 − 2 = 0 or 4𝑥 + 3 = 0 𝑥 = 2∕3 or −3∕4

54. a. 20𝑦 2 + 7𝑦 − 3 = (4𝑦 − 1)(5𝑦 + 3) b. (4𝑦 − 1)(5𝑦 + 3) = 0 4𝑦 − 1 = 0 or 5𝑦 + 3 = 0 𝑦 = 1∕4 or −3∕5

55. a. 𝑥 2 + 4𝑥𝑦 + 4𝑦 2 = (𝑥 + 2𝑦) 2 b. (𝑥 + 2𝑦) 2 = 0 𝑥 + 2𝑦 = 0 𝑥 = −2𝑦

56. a. 4𝑦 2 − 4𝑥𝑦 + 𝑥 2 = (2𝑦 − 𝑥) 2 b. (2𝑦 − 𝑥) 2 = 0 2𝑦 − 𝑥 = 0 𝑦 = 𝑥∕2

57. a. 𝑥 4 − 5𝑥 2 + 4 = (𝑥 2 − 1)(𝑥 2 − 4) = (𝑥 + 1)(𝑥 − 1)(𝑥 + 2)(𝑥 − 2) b. (𝑥 + 1)(𝑥 − 1)(𝑥 + 2)(𝑥 − 2) = 0 𝑥 + 1 = 0 or 𝑥 − 1 = 0 or 𝑥 + 2 = 0 or 𝑥 − 2 = 0 𝑥 = ±1 or ±2

58. a. 𝑦 4 + 2𝑦 2 − 3 = (𝑦 2 − 1)(𝑦 2 + 3) = (𝑦 + 1)(𝑦 − 1)(𝑦 2 + 3) b. (𝑦 + 1)(𝑦 − 1)(𝑦 2 + 3) = 0 𝑦 + 1 = 0 or 𝑦 − 1 = 0 or 𝑦 2 + 3 = 0 𝑦 = ±1 (Notice that 𝑦 2 + 3 = 0 has no real solutions.)

59. a. 𝑥 2 − 3 = (𝑥 − √3)(𝑥 + √3) b. (𝑥 − √3)(𝑥 + √3) = 0 𝑥 − √3 = 0 or 𝑥 + √3 = 0 𝑥 = ±√3

60. a. 𝑦 2 − 7 = (𝑦 − √7)(𝑦 + √7) b. (𝑦 − √7)(𝑦 + √7) = 0 𝑦 − √7 = 0 or 𝑦 + √7 = 0 𝑦 = ±√7

Solutions Section 0.4


Solutions Section 0.5 Section 0.5 1.

2.

3.

4.

5.

6.

7.

8.

9.

𝑥 − 4 2𝑥 + 1 (𝑥 − 4)(2𝑥 + 1) 2𝑥 2 − 7𝑥 − 4 ⋅ = = 𝑥+1 𝑥−1 (𝑥 + 1)(𝑥 − 1) 𝑥2 − 1 2𝑥 − 3 𝑥 + 3 (2𝑥 − 3)(𝑥 + 3) 2𝑥 2 + 3𝑥 − 9 ⋅ = = 2 𝑥−2 𝑥+1 (𝑥 − 2)(𝑥 + 1) 𝑥 −𝑥−2

𝑥 − 4 2𝑥 + 1 (𝑥 − 4)(𝑥 − 1) + (𝑥 + 1)(2𝑥 + 1) 3𝑥 2 − 2𝑥 + 5 + = = 𝑥+1 𝑥−1 (𝑥 + 1)(𝑥 − 1) 𝑥2 − 1 2𝑥 − 3 𝑥 + 3 (2𝑥 − 3)(𝑥 + 1) + (𝑥 − 2)(𝑥 + 3) 3𝑥 2 − 9 + = = 2 𝑥−2 𝑥+1 (𝑥 − 2)(𝑥 + 1) 𝑥 −𝑥−2 𝑥2 𝑥 − 1 𝑥 2 − (𝑥 − 1) 𝑥 2 − 𝑥 + 1 − = = 𝑥+1 𝑥+1 𝑥+1 𝑥+1

(𝑥 2 − 1)(𝑥 − 1) − (𝑥 − 2) 𝑥 3 − 𝑥 2 − 2𝑥 + 3 𝑥2 − 1 1 − = = 𝑥−2 𝑥−1 (𝑥 − 2)(𝑥 − 1) 𝑥 2 − 3𝑥 + 2 1

+𝑥−1= 𝑥 ( 𝑥−1 ) 2

𝑥−2

! 𝑥2 "

𝑥 − 1 + 𝑥(𝑥 − 1) 𝑥 2 − 1 𝑥−1 +𝑥−1= = 𝑥 𝑥 𝑥

1 2𝑥 2 1 2𝑥 2 − 1 = − = 𝑥−2 𝑥−2 𝑥−2 𝑥−2

1 ⎡ 𝑥 − 3 1 ⎤ 1 ⎡ 𝑥 − 3 + 𝑥 ⎤ 2𝑥 − 3 + = = 𝑥 ⎢⎣ 𝑥𝑦 𝑦 ⎥⎦ 𝑥 ⎢⎣ 𝑥𝑦 ⎥⎦ 𝑥 2𝑦

10.

11.

12.

13.

14.

𝑦 2 ⎡ 2𝑥 − 3 𝑥 ⎤ 𝑦 2 ⎡ 2𝑥 − 3 + 𝑥 ⎤ 𝑦 2(3𝑥 − 3) 𝑦(3𝑥 − 3) 3𝑥𝑦 − 3𝑦 + = = = = 𝑥 ⎢⎣ 𝑦 𝑦 ⎥⎦ 𝑥 ⎢⎣ 𝑥𝑦 𝑥𝑦 𝑥 𝑥 ⎥⎦

(𝑥 + 1) 2(𝑥 + 2) 3 − (𝑥 + 1) 3(𝑥 + 2) 2 (𝑥 + 1) 2(𝑥 + 2) 2[(𝑥 + 2) − (𝑥 + 1)] (𝑥 + 1) 2 = = (𝑥 + 2) 4 (𝑥 + 2) 6 (𝑥 + 2) 6 6𝑥(𝑥 2 + 1) 2(𝑥 3 + 2) 3 − 9𝑥 2(𝑥 2 + 1) 3(𝑥 3 + 2) 2 (𝑥 3 + 2) 6 2 2 3 3𝑥(𝑥 + 1) (𝑥 + 2) 2[2(𝑥 3 + 2) − 3𝑥(𝑥 2 + 1)] 3𝑥(𝑥 2 + 1) 2(−𝑥 3 − 3𝑥 + 4) = = (𝑥 3 + 2) 4 (𝑥 3 + 2) 6 (𝑥 2 − 1)√𝑥 2 + 1 − 𝑥2 + 1

𝑥√𝑥 3 − 1 −

𝑥3 − 1

3𝑥 4

√𝑥 3−1

𝑥4 √𝑥 2+1

=

=

(𝑥 2 − 1)(𝑥 2 + 1) − 𝑥 4 (𝑥 2 + 1)√𝑥 2 + 1

𝑥(𝑥 3 − 1) − 3𝑥 4

(𝑥 3 − 1)√𝑥 3 − 1

=

=

−2𝑥 4 − 𝑥

−1

√(𝑥 2 + 1) 3

√(𝑥 3 − 1) 3

=

−𝑥(2𝑥 3 + 1) √(𝑥 3 − 1) 3


15.

1 − 12 𝑥 (𝑥+𝑦) 2

16.

1 − 13 𝑥 (𝑥+𝑦) 3

𝑦

=

Solutions Section 0.5

𝑥 2 − (𝑥 + 𝑦) 2 𝑥 2 − 𝑥 2 − 2𝑥𝑦 − 𝑦 2 −𝑦(2𝑥 + 𝑦) −(2𝑥 + 𝑦) = = = 2 2 2 2 2 2 𝑦𝑥 (𝑥 + 𝑦) 𝑦𝑥 (𝑥 + 𝑦) 𝑦𝑥 (𝑥 + 𝑦) 𝑥 2(𝑥 + 𝑦) 2

𝑥 2 − (𝑥 + 𝑦) 2 𝑥 3 − 𝑥 3 − 3𝑥 2𝑦 − 3𝑥𝑦 2 − 𝑦 3 = 𝑦 𝑦𝑥 2(𝑥 + 𝑦) 2 𝑦𝑥 3(𝑥 + 𝑦) 3 2 2 2 −𝑦(3𝑥 + 3𝑥𝑦 + 𝑦 ) −(3𝑥 + 3𝑥𝑦 + 𝑦 2) = = 𝑦𝑥 3(𝑥 + 𝑦) 3 𝑥 3(𝑥 + 𝑦) 3 =


Solutions Section 0.6 Section 0.6

1. 𝑥 + 1 = 0 ⇒ 𝑥 = 0 − 1 ⇒ 𝑥 = −1

2. 𝑥 − 3 = 1 ⇒ 𝑥 = 1 + 3 ⇒ 𝑥 = 4

3. −𝑥 + 5 = 0 ⇒ −𝑥 = −5 ⇒ 𝑥 = 5

4. 2𝑥 + 4 = 1 ⇒ 2𝑥 = −3 ⇒ 𝑥 = −

5. 4𝑥 − 5 = 8 ⇒ 4𝑥 = 13 ⇒ 𝑥 =

13 4 43 7

8. 3𝑥 + 1 = 𝑥 ⇒ 2𝑥 = −1 ⇒ 𝑥 = −

9. 𝑥 + 1 = 2𝑥 + 2 ⇒ −𝑥 = 1 ⇒ 𝑥 = −1 11. 𝑎𝑥 + 𝑏 = 𝑐 ⇒ 𝑎𝑥 = 𝑐 − 𝑏 ⇒ 𝑥 =

3 3 4 𝑥 + 1 = 0 ⇒ 𝑥 = −1 ⇒ 𝑥 = − 4 4 3

6.

7. 7𝑥 + 55 = 98 ⇒ 7𝑥 = 43 ⇒ 𝑥 = −

𝑐−𝑏 𝑎

12. 𝑥 − 1 = 𝑐𝑥 + 𝑑 ⇒ (1 − 𝑐)𝑥 = 𝑑 + 1 ⇒ 𝑥 =

𝑑+1 1−𝑐

1 2

14. 𝑥 2 + 𝑥 + 1 = 0 ⇒ Δ = 𝑏 2 − 4𝑎𝑐 = −3 < 0, so this equation has no real solutions. 15. 𝑥 2 − 𝑥 + 1 = 0 ⇒ Δ = 𝑏 2 − 4𝑎𝑐 = −3 < 0, so this equation has no real solutions. 16. 2𝑥 2 − 4𝑥 + 3 ⇒ Δ = 𝑏 2 − 4𝑎𝑐 = −8 < 0, so this equation has no real solutions.

18. 3𝑥 2 − 1 = 0 ⇒ 𝑥 2 =

5 5 ⇒𝑥=± √2 2 1 1 ⇒𝑥=± √3 3

19. −𝑥 2 − 2𝑥 − 1 = 0 ⇒ −(𝑥 + 1) 2 = 0 ⇒ 𝑥 = −1

3 20. 2𝑥 2 − 𝑥 − 3 = 0 ⇒ (2𝑥 − 3)(𝑥 + 1) = 0 ⇒ 𝑥 = , −1 2 21.

1 2

10. 𝑥 + 1 = 3𝑥 + 1 ⇒ −2𝑥 = 0 ⇒ 𝑥 = 0

13. 2𝑥 2 + 7𝑥 − 4 = 0 ⇒ (2𝑥 − 1)(𝑥 + 4) = 0 ⇒ 𝑥 = −4,

17. 2𝑥 2 − 5 = 0 ⇒ 𝑥 2 =

3 2

1 2 3 𝑥 − 𝑥 − = 0 ⇒ 𝑥 2 − 2𝑥 − 3 = 0 ⇒ (𝑥 + 1)(𝑥 − 3) = 0 ⇒ 𝑥 = −1, 3 2 2

1 1 22. − 𝑥 2 − 𝑥 + 1 = 0 ⇒ 𝑥 2 + 𝑥 − 2 = 0 ⇒ (𝑥 + 2)(𝑥 − 1) = 0 ⇒ 𝑥 = −2, 1 2 2


−𝑏 ± √𝑏 2 − 4𝑎𝑐 1 ± √5 = 2𝑎 2

Solutions Section 0.6

23. 𝑥 2 − 𝑥 = 1 ⇒ 𝑥 2 − 𝑥 − 1 = 0 ⇒ 𝑥 =

24. 16𝑥 2 = −24𝑥 − 9 ⇒ 16𝑥 2 + 24𝑥 + 9 = 0(4𝑥 + 3) 2 = 0 ⇒ 𝑥 = − 25. 𝑥 = 2 −

3 4

1 ⇒ 𝑥 2 = 2𝑥 − 1 ⇒ 𝑥 2 − 2𝑥 + 1 = 0 ⇒ (𝑥 − 1) 2 = 0 ⇒ 𝑥 = 1 𝑥

1 ⇒ (𝑥 + 4)(𝑥 − 2) = 1 ⇒ 𝑥 2 + 2𝑥 − 8 = 1 ⇒ 𝑥 2 + 2𝑥 − 9 − 0 𝑥−2 −𝑏 ± √𝑏 2 − 4𝑎𝑐 −2 ± √40 ⇒𝑥= = = −1 ± √10 2𝑎 2

26. 𝑥 + 4 =

27. 𝑥 4 − 10𝑥 2 + 9 = 0 ⇒ (𝑥 2 − 1)(𝑥 2 − 9) = 0 ⇒ 𝑥 2 = 1 or 𝑥 2 = 0 ⇒ 𝑥 = ±1, ±3 28. 𝑥 4 − 2𝑥 2 + 1 = 0 ⇒ (𝑥 2 − 1) 2 = 0 ⇒ 𝑥 = ±1 29. 𝑥 4 + 𝑥 2 − 1 = 0 ⇒ 𝑥 2 =

−𝑏 ± √𝑏 2 − 4𝑎𝑐 −1 ± √5 −1 ± √5 ⇒ 𝑥2 = ⇒ 𝑥 = ±√ 2𝑎 2 2

30. 𝑥 3 + 2𝑥 2 + 𝑥 = 0 ⇒ 𝑥(𝑥 2 + 2𝑥 + 1) = 0 ⇒ 𝑥(𝑥 + 1) 2 = 0 ⇒ 𝑥 = 0, −1

31. 𝑥 3 + 16𝑥 2 + 11𝑥 + 6 = 0 ⇒ (𝑥 + 1)(𝑥 + 2)(𝑥 + 3) = 0 ⇒ 𝑥 = −1, −2, −3 32. 𝑥 3 − 6𝑥 2 + 12𝑥 − 8 = 0 ⇒ (𝑥 − 2) 3 = 0 ⇒ 𝑥 = 2

33. 𝑥 3 + 4𝑥 2 + 4𝑥 + 3 = 0 ⇒ (𝑥 + 3)(𝑥 2 + 𝑥 + 1) = 0 ⇒ 𝑥 = −3 (For 𝑥 2 + 𝑥 + 1 = 0, Δ = 𝑏 2 − 4𝑎𝑐 = −3 < 0, so there are no real solutions to this quadratic equation.) 3

34. 𝑦 3 + 64 = 0 ⇒ 𝑦 3 = −64 ⇒ 𝑦 = √−64 = −4 3

35. 𝑥 3 − 1 = 0 ⇒ 𝑥 3 = 1 ⇒ 𝑥 = √1 = 1 36. 𝑥 3 − 27 = 0 3 ⇒ 𝑥 3 = 27 ⇒ 𝑥 = √27 = 3

37. 𝑦 3 + 3𝑦 2 + 3𝑦 + 2 = 0 ⇒ (𝑦 + 2)(𝑦 2 + 𝑦 + 1) = 0 ⇒ 𝑦 = −2 (For 𝑦 2 + 𝑦 + 1 = 0, Δ = 𝑏 2 − 4𝑎𝑐 = −3 < 0, so there are no real solutions to this quadratic equation.) 38. 𝑦 3 − 2𝑦 2 − 2𝑦 − 3 = 0 ⇒ (𝑦 − 3)(𝑦 2 + 𝑦 + 1) = 0 ⇒ 𝑦 = 3 (For 𝑦 2 + 𝑦 + 1 = 0, Δ = 𝑏 2 − 4𝑎𝑐 = −3 < 0, so there are no real solutions to this quadratic equation.) 39. 𝑥 3 − 𝑥 2 − 5𝑥 + 5 = 0 ⇒ (𝑥 − 1)(𝑥 2 − 5) = 0 ⇒ 𝑥 = 1, ± √5

40. 𝑥 3 − 𝑥 2 − 3𝑥 + 3 = 0 ⇒ (𝑥 − 1)(𝑥 2 − 3) = 0 ⇒ 𝑥 = 1, ± √3


Solutions Section 0.6 41. 2𝑥 − 𝑥 − 2𝑥 + 1 = 0 ⇒ (2𝑥 − 1)(𝑥 4 − 1) = 0 ⇒ (2𝑥 2 − 1)(𝑥 2 − 1)(𝑥 2 + 1) = 0 (Think of the cubic you get by substituting 𝑦 for 𝑥 2) 1 ⇒ 𝑥 = ±1, ± √2 6

4

2

2

42. 3𝑥 6 − 𝑥 4 − 12𝑥 2 + 4 = 0 ⇒ (3𝑥 2 − 1)(𝑥 4 − 4) = 0 ⇒ (3𝑥 2 − 1)(𝑥 2 − 2)(𝑥 2 + 2) = 0 (Think of the cubic you get by substituting 𝑦 for 𝑥 2) 1 ⇒ 𝑥 = ±√2, ± √3

43. (𝑥 2 + 3𝑥 + 2)(𝑥 2 − 5𝑥 + 6) = 0 ⇒ (𝑥 + 2)(𝑥 + 1)(𝑥 − 2)(𝑥 − 3) = 0 ⇒ 𝑥 = −2, −1, 2, 3 44. (𝑥 2 − 4𝑥 + 4) 2(𝑥 2 + 6𝑥 + 5) 3 = 0 ⇒ (𝑥 − 2) 4(𝑥 + 1) 3(𝑥 + 5) 3 = 0 ⇒ 𝑥 = −5, −1, 2


Solutions Section 0.7 Section 0.7

1. 𝑥 4 − 3𝑥 3 = 0, 𝑥 3(𝑥 − 3) = 0, 𝑥 = 0, 3

2. 𝑥 6 − 9𝑥 4 = 0, 𝑥 4(𝑥 2 − 9) = 0, 𝑥 = 0, ±3

3. 𝑥 4 − 4𝑥 2 = −4, 𝑥 4 − 4𝑥 2 + 4 = 0, (𝑥 2 − 2) 2 = 0, 𝑥 = ±√2

4. 𝑥 4 − 𝑥 2 = 6, 𝑥 4 − 𝑥 2 − 6 = 0, (𝑥 2 − 3)(𝑥 2 + 2) = 0, 𝑥 = ±√3

5. (𝑥 + 1)(𝑥 + 2) + (𝑥 + 1)(𝑥 + 3) = 0, (𝑥 + 1)(𝑥 + 2 + 𝑥 + 3) = 0, (𝑥 + 1)(2𝑥 + 5) = 0, 𝑥 = −1, −5∕2

6. (𝑥 + 1)(𝑥 + 2) 2 + (𝑥 + 1) 2(𝑥 + 2) = 0, (𝑥 + 1)(𝑥 + 2)(𝑥 + 2 + 𝑥 + 1) = 0, (𝑥 + 1)(𝑥 + 2)(2𝑥 + 3) = 0, 𝑥 = −1, −2, −3∕2 7. (𝑥 2 + 1) 5(𝑥 + 3) 4 + (𝑥 2 + 1) 6(𝑥 + 3) 3 = 0, (𝑥 2 + 1) 5(𝑥 + 3) 3(𝑥 + 3 + 𝑥 2 + 1) = 0, (𝑥 2 + 1) 5(𝑥 + 3) 3(𝑥 2 + 𝑥 + 4) = 0, 𝑥 = −3 (Neither 𝑥 2 + 1 = 0 nor 𝑥 2 + 𝑥 + 4 = 0 has a real solution.) 8. 10𝑥(𝑥 2 + 1) 4(𝑥 3 + 1) 5 − 10𝑥 2(𝑥 2 + 1) 5(𝑥 3 + 1) 4 = 0, 10𝑥(𝑥 2 + 1) 4(𝑥 3 + 1) 4[𝑥 3 + 1 − 𝑥(𝑥 2 + 1)] = 0, 10𝑥(𝑥 2 + 1) 4(𝑥 3 + 1) 4(1 − 𝑥) = 0, 𝑥 = −1, 0, 1 9. (𝑥 3 + 1)√𝑥 + 1 − (𝑥 3 + 1) 2√𝑥 + 1 = 0, (𝑥 3 + 1)√𝑥 + 1 [1 − (𝑥 3 + 1)] = 0, −𝑥 3(𝑥 3 + 1)√𝑥 + 1 = 0, 𝑥 = 0, −1

10. (𝑥 2 + 1)√𝑥 + 1 − √(𝑥 + 1) 3 = 0, √𝑥 + 1 [𝑥 2 + 1 − (𝑥 + 1)] = 0, (𝑥 2 − 𝑥)√𝑥 + 1 = 0, 𝑥(𝑥 − 1)√𝑥 + 1 = 0, 𝑥 = −1, 0, 1

11. √(𝑥 + 1) 3 + √(𝑥 + 1) 5 = 0, √(𝑥 + 1) 3 (1 + 𝑥 + 1) = 0, (𝑥 + 2)√(𝑥 + 1) 3 = 0, 𝑥 = −1 (𝑥 = −2 is not a solution because √(𝑥 + 1) 3 is not defined for 𝑥 = −2.) 3

3

12. (𝑥 2 + 1) √(𝑥 + 1) 4 − √(𝑥 + 1) 7 = 0, 3

√(𝑥 + 1) 4 [𝑥 2 + 1 − (𝑥 + 1)] = 0, 3

3

(𝑥 2 − 𝑥) √(𝑥 + 1) 4 = 0, 𝑥(𝑥 − 1) √(𝑥 + 1) 4 = 0, 𝑥 = −1, 0, 1

13. (𝑥 + 1) 2(2𝑥 + 3) − (𝑥 + 1)(2𝑥 + 3) 2 = 0, (𝑥 + 1)(2𝑥 + 3)(𝑥 + 1 − 2𝑥 − 3) = 0, (𝑥 + 1)(2𝑥 + 3)(−𝑥 − 2) = 0, 𝑥 = −2, −3∕2, −1


Solutions Section 0.7 14. (𝑥 − 1) (𝑥 + 2) − (𝑥 − 1) (𝑥 + 2) 2 = 0, (𝑥 2 − 1) 2(𝑥 + 2) 2(𝑥 + 2 − 𝑥 2 + 1) = 0, −(𝑥 2 − 1) 2(𝑥 + 2) 2(𝑥 2 − 𝑥 − 3) = 0, 𝑥 = −2, −1, 1, (1 ± √13)∕2 2

15.

16.

17.

2

3

6𝑥(𝑥 2 + 1) 2(𝑥 2 + 2) 4 − 8𝑥(𝑥 2 + 1) 3(𝑥 2 + 2) 3 = 0, (𝑥 2 + 2) 8 2𝑥(𝑥 2 + 1) 2(𝑥 2 + 2) 3[3(𝑥 2 + 2) − 4(𝑥 2 + 1)] = 0, (𝑥 2 + 2) 8 −2𝑥(𝑥 2 + 1) 2(𝑥 2 − 2) = 0, (𝑥 2 + 2) 5 −2𝑥(𝑥 2 + 1) 2(𝑥 2 − 2) = 0, 𝑥 = 0, ±√2 2(𝑥 2 − 1)√𝑥 2 + 1 − 𝑥2 + 1

𝑥4 − 2

4𝑥√𝑥 3 − 1 − 𝑥3 − 1

𝑥 4 − 4𝑥

19. 𝑥 − 20. 1 −

22.

23.

𝑥4 √𝑥 2+1

= 0,

2(𝑥 2 − 1)(𝑥 2 + 1) − 𝑥 4 (𝑥 2 + 1)√𝑥 2 + 1 4

= 0, 𝑥 4 − 2 = 0, 𝑥 = ± √2

3𝑥 4 √𝑥 3−1

(𝑥 3 − 1)√𝑥 3 − 1

21.

3

(𝑥 + 1) 2(𝑥 + 2) 3 − (𝑥 + 1) 3(𝑥 + 2) 2 = 0, (𝑥 + 2) 6 (𝑥 + 1) 2(𝑥 + 2) 2[(𝑥 + 2) − (𝑥 + 1)] = 0, (𝑥 + 2) 6 (𝑥 + 1) 2 = 0, (𝑥 + 1) 2 = 0, 𝑥 = −1 (𝑥 + 2) 4

(𝑥 2 + 1)√𝑥 2 + 1

18.

2

= 0,

4𝑥(𝑥 3 − 1) − 3𝑥 4 (𝑥 3 − 1)√𝑥 3 − 1

= 0,

= 0,

3

= 0, 𝑥 4 − 4𝑥 = 0, 𝑥(𝑥 3 − 4) = 0, 𝑥 = 0, ± √4

1 = 0, 𝑥 2 − 1 = 0, 𝑥 = ±1 𝑥

4 = 0, 𝑥 2 − 4 = 0, 𝑥 = ±2 𝑥2

1 9 − 3 = 0, 𝑥 2 − 9 = 0, 𝑥 = ±3 𝑥 𝑥

1 1 − = 0, 𝑥 + 1 − 𝑥 2 = 0, 𝑥 2 − 𝑥 − 1 = 0, 𝑥 = (1 ± √5)∕2 2 𝑥+1 𝑥 (𝑥 − 4)(𝑥 − 1) − 𝑥(𝑥 + 1) 𝑥−4 𝑥 − = 0, = 0, 𝑥+1 𝑥−1 (𝑥 + 1)(𝑥 − 1) −6𝑥 + 4 = 0, −6𝑥 + 4 = 0, 𝑥 = 2∕3 (𝑥 + 1)(𝑥 − 1)


Solutions Section 0.7 (2𝑥 − 3)(𝑥 + 1) − (2𝑥 + 3)(𝑥 − 1) 2𝑥 − 3 2𝑥 + 3 24. − = 0, = 0, 𝑥−1 𝑥+1 (𝑥 − 1)(𝑥 + 1) −2𝑥 = 0, −2𝑥 = 0, 𝑥 = 0 (𝑥 − 1)(𝑥 + 1) 25.

26.

3𝑥(𝑥 + 4) + (𝑥 + 1)(𝑥 + 4) 𝑥+4 𝑥+4 + = 0, = 0, 𝑥+1 3𝑥 3𝑥(𝑥 + 1) (𝑥 + 4)(3𝑥 + 𝑥 + 1) (𝑥 + 4)(4𝑥 + 1) = 0, = 0, 3𝑥(𝑥 + 1) 3𝑥(𝑥 + 1) (𝑥 + 4)(4𝑥 + 1) = 0, 𝑥 = −4, −1∕4

(2𝑥 − 3)(𝑥 + 1) − 𝑥(2𝑥 − 3) 2𝑥 − 3 2𝑥 − 3 − = 0, = 0, 𝑥 𝑥+1 𝑥(𝑥 + 1) (2𝑥 − 3)(𝑥 + 1 − 𝑥) (2𝑥 − 3) = 0, = 0, 2𝑥 − 3 = 0, 𝑥 = 3∕2 𝑥(𝑥 + 1) 𝑥(𝑥 + 1)


Solutions Section 0.8 Section 0.8

1. 𝑃 (0, 2), 𝑄(4, −2), 𝑅(−2, 3), 𝑆(−3.5, −1.5), 𝑇 (−2.5, 0), 𝑈(2, 2.5)

2. 𝑃 (−2, 2), 𝑄(3.5, 2), 𝑅(0, −3), 𝑆(−3.5, −1.5), 𝑇 (2.5, 0), 𝑈(−2, 2.5) 3.

4.

5. Solve the equation 𝑥 + 𝑦 = 1 for 𝑦 to get 𝑦 = 1 − 𝑥. Then plot some points: 𝑥

−2

𝑦=1−𝑥

3

−1 2

0

1

1

2

0

−1

Graph:

6. Solve the equation 𝑦 − 𝑥 = −1 for 𝑦 to get 𝑦 = −1 + 𝑥. Then plot some points: 𝑥

𝑦 = −1 + 𝑥

−2 −3

−1 −2

0

−1

1 0

2 1


Solutions Section 0.8 Graph:

7. Solve the equation 2𝑦 − 𝑥 2 = 1 for 𝑦 to get 𝑦 = 𝑦=

𝑥

−2

1 + 𝑥2 2

2.5

−1 1

0

0.5

1

2

1

1 + 𝑥2 . Then plot some points: 2

2.5

Graph:

8. Solve the equation 2𝑦 + √𝑥 = 1 for 𝑦 to get 𝑦 = 𝑦=

𝑥

1 − √𝑥 2

Graph:

0

0.5

1 0

4

−0.5

9

−1

16

−1.5

1 − √𝑥 . Then plot some points: 2


Solutions Section 0.8 4 9. Solve the equation 𝑥𝑦 = 4 for 𝑦 to get 𝑦 = . Then plot some points: 𝑥 𝑦=

𝑥

−3

4 𝑥

−2

− 43

1

−1

−2

2

4

−4

3 4 3

2

Graph:

1 10. Solve the equation 𝑥 2𝑦 = −1 for 𝑦 to get 𝑦 = − 2 . Then plot some points: 𝑥 𝑥

1 𝑦=− 2 𝑥

−3

− 19

−2

− 14

−1 −1

1

−1

2

− 14

3

− 19

Graph:

11. Solve the equation 𝑥𝑦 = 𝑥 2 + 1 for 𝑦 to get 𝑦 = 𝑥 + 𝑦=𝑥+

𝑥

1 𝑥

−2

− 52

−1 −2

− 12 − 52

1 2 5 2

1 2

2 5 2

1 . Then plot some points: 𝑥


Solutions Section 0.8 Graph:

12. Solve the equation 𝑥𝑦 = 2𝑥 3 + 1 for 𝑦 to get 𝑦 = 2𝑥 2 + 𝑦 = 2𝑥 2 +

𝑥

1 𝑥

−2 15 2

−1 1

− 12 − 32

1 2 5 2

1 3

2

17 2

Graph:

13. √(2 − 1) 2 + (−2 + 1) 2 = √2 14. √(6 − 1) 2 + (1 − 0) 2 = √26

15. √(0 − 𝑎) 2 + (𝑏 − 0) 2 = √𝑎 2 + 𝑏 2

16. √(𝑏 − 𝑎) 2 + (𝑏 − 0) 2 = √2(𝑏 − 𝑎) 2 or √2|𝑏 − 𝑎| 17. Set the two distances equal and solve: √(1 − 0) * 2 + (𝑘 − 0) 2 = √(1 − 2) 2 + (𝑘 − 1) 2 ⇒ √1 + 𝑘 2 = √2 − 2𝑘 + 𝑘 2 ⇒ 1 + 𝑘 2 = 2 − 2𝑘 + 𝑘 2 1 ⇒ 2𝑘 = 1 ⇒ 𝑘 = 2 18. Set the two distances equal and solve: √(𝑘 + 1) * 2 + (𝑘 − 0) 2 = √(𝑘 − 0) 2 + (𝑘 − 2) 2 ⇒ √2𝑘 2 + 2𝑘 + 1 = √2𝑘 2 − 4𝑘 + 4 ⇒ 2𝑘 2 + 2𝑘 + 1 = 2𝑘 2 − 4𝑘 + 4 1 ⇒ 6𝑘 = 3 ⇒ 𝑘 = 2

1 . Then plot some points: 𝑥


Solutions Section 0.8 19. Circle with center (0, 0) and radius 3. 20. The single point (0, 0).


Solutions Section 0.9 Section 0.9 1. Exponential Form

10 2 = 100

4 3 = 64

4 4 = 256

0.45 0 = 1

8 1∕2 = 2√2

1 Logarithmic Form log10 100 = 2 log4 64 = 3 log4 256 = 4 log0.45 1 = 0 log8 2√2 = 2

2. Exponential Form Logarithmic Form

10 1 = 10

5 5 = 3,125

log10 10 = 1

Exponential Form

0.5 3 = 0.125

Exponential Form

0.3 2 = 0.09

Logarithmic Form log0.5 0.125 = 3 3.

Logarithmic Form log0.3 0.09 = 2 Exponential Form Logarithmic Form 4. Exponential Form Logarithmic Form Exponential Form

log6 216 = 3

log4 √2 = 14

1 log3 ! 81 " = −4

0

10 −3 = 0.001

1 !2" = 1

1 3 −4 = 81

log1∕2 1 = 0

log10 0.001 = −3

1 log9 81 = −2

log2 1,024 = 10

log64 14 = − 13

2 −1 = 12

10 5 = 100,000

10 −5 = 0.00001

16 −1∕4 = 12

0.25 2 = 0.0625

1 9 −2 = 81

log2 12 = −1 2 11 = 2,048

Logarithmic Form log2 2,048 = 11

2 10 = 1,024

1 log4 ! 64 " = −3

6 3 = 216

log5 3,125 = 5 4 1∕4 = √2

1 4 −3 = 64

64 −1∕3 = 14

log10 100,000 = 5 log10 0.00001 = −5 log16 12 = − 14

log0.25 0.0625 = 2

5. log4 16 = the power to which we need to raise 4 in order to get 16. Because 4 boxed 2 = 16, this power is 2, so log4 16 = 2.

6. log5 125 = the power to which we need to raise 5 in order to get 125. Because 5 boxed 3 = 125, this power is 3, so log5 125 = 3.

1 1 1 7. log5 25 = the power to which we need to raise 5 in order to get 25 . Because 5 boxed −2 = 25 , this power is −2, so 1 log5 25 = −2.

1 1 1 8. log4 64 = the power to which we need to raise 4 in order to get 64 . Because 4 boxed −3 = 64 , this power is −3, so 1 log4 64 = −3.


Solutions Section 0.9 9. log 100,000 = log10 100,000 = the power to which we need to raise 10 in order to get 100,000. Because 10 boxed 5 = 100,000, this power is 5, so log 100,000 = 5. 10. log 1,000 = log10 1,000 = the power to which we need to raise 10 in order to get 1,000. Because 10 boxed 3 = 1,000, this power is 3, so log 1,000 = 3.

11. log16 16 = the power to which we need to raise 16 in order to get 16. Because 16 boxed 1 = 16, this power is 1, so log16 16 = 1. 1 1 1 1 boxed 1 1 = the power to which we need to raise in order to get . Because ! " = , this power is 1, 2 2 2 2 2 1 so log1∕2 = 1. 2 12. log1∕2

1 1 1 = the power to which we need to raise 4 in order to get . Because 4 boxed −2 = , this power is −2, 16 16 16 1 so log4 = −2. 16 13. log4

14. log2 log2

1 1 1 = the power to which we need to raise 2 in order to get . Because 2 boxed −3 = , this power is −3, so 8 8 8

1 = −3. 8

15. log2 √2 = the power to which we need to raise 2 in order to get √2. Because 2 boxed 1∕2 = √2, this power is 1 1 , so log2 √2 = . 2 2 16. log4 √2 = the power to which we need to raise 4 in order to get √2. Because 4 boxed 1∕4 = √2, this power is 1 1 , so log4 √2 = . 4 4 17. By Identity (1), log𝑏 3 + log𝑏 4 = log𝑏 (3 × 4) = log𝑏 boxed 12. 3 18. By Identity (2), log𝑏 3 − log𝑏 4 = log𝑏 boxed . 4

19. log𝑏 2 − log𝑏 5 − log𝑏 4 = log𝑏 2 − (log𝑏 5 + log𝑏 4) = log𝑏 2 − (log𝑏 (5 × 4)) (Identity 1) 2 1 = log𝑏 ! " (Identity 2) = log𝑏 boxed 5×4 10 20. log𝑏 3 + log𝑏 2 − log𝑏 7 = log𝑏 (3 × 2) − log𝑏 7 (Identity 1) = log𝑏 !

3×2 6 " (Identity 2) = log𝑏 boxed 7 7

3 3 21. log𝑏 3 − 3 log𝑏 2 = log𝑏 3 − log𝑏 2 3 (Identity 3) = log𝑏 ! 3 " (Identity 2) = log𝑏 boxed 8 2

22. 3 log𝑏 2 + 2 log𝑏 3 = log𝑏 2 3 + log𝑏 3 2 (Identity 3) = log𝑏 (2 3 × 3 2) (Identity 1) = log𝑏 boxed 72 23. 4 log𝑏 𝑥 + 5 log𝑏 𝑦 = log𝑏 𝑥 4 + log𝑏 𝑦 5 (Identity 3) = log𝑏 boxed 𝑥 4𝑦 5 (Identity 1)


Solutions Section 0.9 𝑝3 24. 3 log𝑏 𝑝 − 2 log𝑏 𝑞 = log𝑏 𝑝 3 − log𝑏 𝑞 2 (Identity 3) = log𝑏 boxed 2 (Identity 2) 𝑞 25. 2 log𝑏 𝑥 + 3 log𝑏 𝑦 − 4 log𝑏 𝑧 = log𝑏 𝑥 2 + log𝑏 𝑦 3 − log𝑏 𝑧 4 (Identity 3) 𝑥 2𝑦 3 = log𝑏 (𝑥 2𝑦 3) − log𝑏 𝑧 4 (Identity 1) = log𝑏 boxed 4 (Identity 2) 𝑧 26. 4 log𝑏 𝑝 − 3 log𝑏 𝑞 − 2 log𝑏 𝑟 = log𝑏 𝑝 4 − log𝑏 𝑞 3 − log𝑏 𝑟 2 (Identity 3)

= log𝑏 𝑝 4 − (log𝑏 𝑞 3 + log𝑏 𝑟 2) = log𝑏 𝑝 4 − log𝑏 (𝑞 3𝑟 2) (Identity 1) = log𝑏 boxed

27. 𝑥 log𝑏 2 − 2 log𝑏 𝑥 = log𝑏 2 𝑥 − log𝑏 𝑥 2 (Identity 3) = log𝑏 boxed

2𝑥 (Identity 2) 𝑥2

28. 𝑝 log𝑏 𝑞 + 𝑞 log𝑏 𝑝 = log𝑏 𝑞 𝑝 + log𝑏 𝑝 𝑞 (Identity 3) = log𝑏 boxed 𝑞 𝑝𝑝 𝑞 (Identity 1) 29. log 21 = log(3 × 7) = log 3 + log 7 = 𝑏 + 𝑐 (Identity 1)

30. log 14 = log(2 × 7) = log 2 + log 7 = 𝑎 + 𝑐 (Identity 1)

31. log 42 = log(2 × 3 × 7) = log 2 + log 3 + log 7 = 𝑎 + 𝑏 + 𝑐 (Identity 1) 32. log 28 = log(2 × 2 × 7) = log 2 + log 2 + log 7 = 2𝑎 + 𝑐 (Identity 1) 1 33. log! " = − log 7 = −𝑐 (Identity 5) 7

1 34. log! " = − log 3 = −𝑏 (Identity 5) 3

2 35. log! " = log 2 − log 3 = 𝑎 − 𝑏 (Identity 2) 3 7 36. log! " = log 7 − log 2 = 𝑐 − 𝑎 (Identity 2) 2

4 37. log! " = log 4 − log 7 (Identity 2) = log 2 + log 2 − log 7 (Identity 1) = 2𝑎 − 𝑐 7

2 38. log! " = log 2 − log 9 (Identity 2) = log 2 − (log 3 + log 3) (Identity 1) = 𝑎 − 2𝑏 9 39. log 16 = log 2 4 = 4 log 2 = 4𝑎 (Identity 3) 40. log 81 = log 3 4 = 4 log 3 = 4𝑏 (Identity 3) 41. log 0.03 = log!

3 " = log 3 − log 10 2 = log 3 − 2 log 10 = 𝑏 − 2 10 2

42. log 7,000 = log(7 × 10 3) = log 7 + log 10 3 = log 7 + 3 log 10 = 𝑐 + 3

𝑝4 (Identity 2) 𝑞 3𝑟 2


43. log 5 = log!

10 " = log 10 − log 2 = 1 − 𝑎 (Identity 2) 2

44. log 25 = log!

Solutions Section 0.9

100 " = log 100 − log 4 = log 10 2 − log 2 2 = 2 log 10 − 2 log 2 = 2 − 2𝑎 4

45. log √7 = log(7 1∕2) = 46. log!

1 𝑐 log 7 = 2 2

2 1 𝑏 " = log(2) − log √3 = log(2) − log(3 1∕2) = log 2 − log 3 = 𝑎 − √3 2 2

47. 4 = 2 𝑥 is exactly the equation we solve in order to calculate log2 4; answer: 2. Alternatively, write the equation 4 = 2 𝑥 in logarithmic form to get 𝑥 = log2 4 = 2.

48. 81 = 3 𝑥 is exactly the equation we solve in order to calculate log3 81; answer: 4. Alternatively, write the equation 81 = 3 𝑥 in logarithmic form to get 𝑥 = log3 81 = 4. 49. Take the base 3 logarithm of both sides of the given equation 27 = 3 2𝑥−1 to get log3 27 = log3 3 2𝑥−1 3 = (2𝑥 − 1) log3 3 3 = 2𝑥 − 1 3+1 𝑥= =2 2

By Identity 3 Solve for 𝑥.

By Identity 4

50. Take the base 4 logarithm of both sides of the given equation 4 2−3𝑥 = 256 to get log4 4 2−3𝑥 = log4 256 (2 − 3𝑥) log4 4 = 4 2 − 3𝑥 = 4 2−4 2 𝑥= =− 3 3

By Identity 3 Solve for 𝑥.

By Identity 4

1 51. Take the base 5 logarithm of both sides of the given equation 5 −𝑥+1 = to get 125 1 −𝑥+1 log5 5 = log5 ! " 125 (−𝑥 + 1) log5 5 = − log5 125 By Identities 3 and 5 −𝑥 + 1 = −3 𝑥=1+3=4

By Identity 4

𝑥 2 − 12 = −3 𝑥 = ±√9 = ±3

By Identity 4

Solve for 𝑥.

2 1 52. Take the base 3 logarithm of both sides of the given equation 3 𝑥 −12 = to get 27 2 1 log3 3 𝑥 −12 = log3 ! " 27 2 (𝑥 − 12) log3 3 = − log3 27 By Identities 3 and 5

Solve for 𝑥.

53. First divide both sides of the given equation by 50 to get 2.4 = 2 3𝑡. Take the common logarithm of both sides: log 2.4 = log(2 3𝑡)


log 2.4 = 3𝑡 log 2 log 2.4 3𝑡 = log 2 log 2.4 𝑡= ≈ 0.4210 3 log 2

Solutions Section 0.9 By Identity 3 Divide.

Solve for 𝑡.

If instead you take the base-2 logarithm, the answer is represented as

log2 (2.4) . 3

54. First divide both sides of the given equation by 500 to get 2 = 1.1 2𝑡. Take the common logarithm of both sides: log 2 = log(1.1 2𝑡) log 2 = 2𝑡 log 1.1 By Identity 3 log 2 2𝑡 = Divide. log 1.1 log 2 𝑡= ≈ 3.6363 Solve for 𝑡. 2 log 1.1 55. First divide both sides of the given equation by 300 to get sides:

10 " = log(1.3 4𝑡−1) 3 log 10 − log 3 = (4𝑡 − 1) log 1.3 log 10 − log 3 4𝑡 − 1 = log 1.3 log 10 − log 3 1 𝑡= ! + 1" ≈ 1.3972 4 log 1.3 log!

By Identities 2 and 3 Divide.

Solve for 𝑡.

56. First divide both sides of the given equation by 700 to get both sides:100 log! " = log(1.04 3𝑡+1) 7 log 100 − log 7 = (3𝑡 + 1) log 1.04 log 100 − log 7 3𝑡 + 1 = log 1.04 1 log 100 − log 7 𝑡= ! − 1" ≈ 22.2675 3 log 1.04

10 = 1.3 4𝑡−1. Take the common logarithm of both 3

100 = 1.04 3𝑡+1. Take the common logarithm of 7

By Identities 2 and 3 Divide.

Solve for 𝑡.


Solutions Section 1.1 Section 1.1 1. Using the table: 2. Using the table:

a. 𝑓(0) = 2

a. 𝑓(−1) = 4

b. 𝑓(2) = −0.5 b. 𝑓(1) = −1

3. Using the table: a. 𝑓(2) − 𝑓(−2) = −0.5 − 2 = −2.5 c. −2𝑓(−1) = −2(4) = −8 4. Using the table: a. 𝑓(1) − 𝑓(−1) = −1 − 4 = −5 c. 3𝑓(−2) = 3(2) = 6 5. From the graph, we estimate:

a. 𝑓(1) = 20

In a similar way, we find: c. 𝑓(3) = 30 f. 𝑓(3 − 2) = 𝑓(1) = 20 6. From the graph, we estimate:

7. From the graph, we estimate:

b. 𝑓(1)𝑓(−2) = (−1)(2) = −2

b. 𝑓(2) = 30

d. 𝑓(5) = 20\\e. 𝑓(3) − 𝑓(2) = 30 − 30 = 0

a. 𝑓(1) = 20

In a similar way, we find: c. 𝑓(3) = 10 f. 𝑓(3 − 2) = 𝑓(1) = 20

b. 𝑓(−1)𝑓(−2) = (4)(2) = 8

b. 𝑓(2) = 10

d. 𝑓(5) = 20 \\e. 𝑓(3) − 𝑓(2) = 10 − 10 = 0

a. 𝑓(−1) = 0

b. 𝑓(1) = −3 since the solid dot is on (1, −3).

In a similar way, we estimate c. 𝑓(3) = 3 d. Since 𝑓(3) = 3 and 𝑓(1) = −3,

𝑓(3) − 𝑓(1) 3 − (−3) = = 3. 3−1 3−1


8. From the graph, we estimate:

Solutions Section 1.1 a. 𝑓(−3) = 3 b. 𝑓(−1) = −2 since the solid dot is on (−1, −2).

In a similar way, we estimate c. 𝑓(1) = 0 𝑓(3) − 𝑓(1) 2 − 0 d. Since 𝑓(3) = 2 and 𝑓(1) = 0, = = 1. 3−1 3−1

1 , with its natural domain. 𝑥2 The natural domain consists of all 𝑥 for which 𝑓(𝑥) makes sense: all real numbers other than 0. 1 1 63 a. Since 4 is in the natural domain, 𝑓(4) is defined, and 𝑓(4) = 4 − 2 = 4 − . = 16 16 4 b. Since 0 is not in the natural domain, 𝑓(0) is not defined. 1 1 c. Since −1 is in the natural domain, 𝑓(−1) = −1 − = −1 − = −2. 2 1 (−1) 9. 𝑓(𝑥) = 𝑥 −

10. 𝑓(𝑥) =

2 − 𝑥 2, with domain [2, ∞) 𝑥

a. Since 4 is in [2, ∞), 𝑓(4) is defined, and 𝑓(4) = b. Since 0 is not in [2, ∞), 𝑓(0) is not defined.

2 1 31 − 4 2 = − 16 = − . 4 2 2 c. Since 1 is not in [2, ∞), 𝑓(1) is not defined.

11. 𝑓(𝑥) = √𝑥 + 10, with domain [−10, 0) a. Since 0 is not in [−10, 0), 𝑓(0) is not defined. b. Since 9 is not in [−10, 0), 𝑓(9) is not defined. c. Since −10 is in [−10, 0), 𝑓(−10) is defined, and 𝑓(−10) = √−10 + 10 = √0 = 0

12. 𝑓(𝑥) = √9 − 𝑥 2, with domain (−3, 3) a. Since 0 is in (−3, 3), 𝑓(0) is defined, and 𝑓(0) = √9 − 0 = 3. b. Since 3 is not in (−3, 3), 𝑓(3) is not defined. c. Since −3 is not in (−3, 3), 𝑓(−3) is not defined. 13. 𝑓(𝑥) = 4𝑥 − 3 a. 𝑓(−1) = 4(−1) − 3 = −4 − 3 = −7 b. 𝑓(0) = 4(0) − 3 = 0 − 3 = −3 c. 𝑓(1) = 4(1) − 3 = 4 − 3 = 1 d. Substitute 𝑦 for 𝑥 to obtain 𝑓(𝑦) = 4𝑦 − 3 e. Substitute (𝑎 + 𝑏) for 𝑥 to obtain 𝑓(𝑎 + 𝑏) = 4(𝑎 + 𝑏) − 3.

14. 𝑓(𝑥) = −3𝑥 + 4 a. 𝑓(−1) = −3(−1) + 4 = 3 + 4 = 7 b. 𝑓(0) = −3(0) + 4 = 0 + 4 = 4 c. 𝑓(1) = −3(1) + 4 = −3 + 4 = 1 d. Substitute 𝑦 for 𝑥 to obtain 𝑓(𝑦) = −3𝑦 + 4 e. Substitute (𝑎 + 𝑏) for 𝑥 to obtain 𝑓(𝑎 + 𝑏) = −3(𝑎 + 𝑏) + 4. 15. 𝑓(𝑥) = 𝑥 2 + 2𝑥 + 3 a. 𝑓(0) = (0) 2 + 2(0) + 3 = 0 + 0 + 3 = 3

b. 𝑓(1) = 1 2 + 2(1) + 3 = 1 + 2 + 3 = 6


Solutions Section 1.1 c. 𝑓(−1) = (−1) + 2(−1) + 3 = 1 − 2 + 3 = 2 d. 𝑓(−3) = (−3) 2 + 2(−3) + 3 = 9 − 6 + 3 = 6 e. Substitute 𝑎 for 𝑥 to obtain 𝑓(𝑎) = 𝑎 2 + 2𝑎 + 3. f. Substitute (𝑥 + ℎ) for 𝑥 to obtain 𝑓(𝑥 + ℎ) = (𝑥 + ℎ) 2 + 2(𝑥 + ℎ) + 3. 2

16. 𝑔(𝑥) = 2𝑥 2 − 𝑥 + 1 a. 𝑔(0) = 2(0) 2 − 0 + 1 = 0 − 0 + 1 = 1 b. 𝑔(−1) = 2(−1) 2 − (−1) + 1 = 2 + 1 + 1 = 4 c. Substitute 𝑟 for 𝑥 to obtain 𝑔(𝑟) = 2𝑟 2 − 𝑟 + 1. d. Substitute (𝑥 + ℎ) for 𝑥 to obtain 𝑔(𝑥 + ℎ) = 2(𝑥 + ℎ) 2 − (𝑥 + ℎ) + 1.

1 𝑠 1 1 a. 𝑔(1) = 1 2 + = 1 + 1 = 2 b. 𝑔(−1) = (−1) 2 + =1−1=0 1 −1 1 1 65 1 c. 𝑔(4) = 4 2 + = 16 + = or 16.25 d. Substitute 𝑥 for 𝑠 to obtain 𝑔(𝑥) = 𝑥 2 + 4 4 4 𝑥 1 2 e. Substitute (𝑠 + ℎ) for 𝑠 to obtain 𝑔(𝑠 + ℎ) = (𝑠 + ℎ) + 𝑠+ℎ 1 1 f. 𝑔(𝑠 + ℎ) − 𝑔(𝑠) = Answer to part (e) − Original function = !(𝑠 + ℎ) 2 + " − !𝑠 2 + " 𝑠+ℎ 𝑠 17. 𝑔(𝑠) = 𝑠 2 +

1 𝑟+4 1 1 1 1 a. ℎ(0) = b. ℎ(−3) = = = =1 0+4 4 (−3) + 4 1 1 1 1 c. ℎ(−5) = d. Substitute 𝑥 2 for 𝑟 to obtain ℎ(𝑥 2) = 2 . = = −1 (−5) + 4 (−1) 𝑥 +4 1 1 e. Substitute (𝑥 2 + 1) for 𝑟 to obtain ℎ(𝑥 2 + 1) = 2 . = 2 (𝑥 + 1) + 4 𝑥 + 5 1 f. ℎ(𝑥 2) + 1 = Answer to part (d) + 1 = 2 + 1\\ 𝑥 +4 18. ℎ(𝑟) =

19. 𝑓(𝑥) = −𝑥 3 (domain (−∞, ∞))

20. 𝑓(𝑥) = 𝑥 3 (domain [0, ∞))

Technology formula: -(x^3)\\

Technology formula: x^3


21. 𝑓(𝑥) = 𝑥 (domain (−∞, ∞)) 4

3

22. 𝑓(𝑥) = √𝑥 (domain (−∞, ∞))

23. 𝑓(𝑥) =

1 (𝑥 ≠ 0) 𝑥2

24. 𝑓(𝑥) = 𝑥 +

1 (𝑥 ≠ 0) 𝑥

Solutions Section 1.1 Technology formula: x^4

Technology formula: x^(1/3)

Technology formula: 1/(x^2)

Technology formula: x+1/x

25. a. 𝑓(𝑥) = 𝑥 (−1 ≤ 𝑥 ≤ 1) Since the graph of 𝑓(𝑥) = 𝑥 is a diagonal 45 ∘ line through the origin inclining up from left to right, the correct graph is (A). b. 𝑓(𝑥) = −𝑥 (−1 ≤ 𝑥 ≤ 1) Since the graph of 𝑓(𝑥) = −𝑥 is a diagonal 45 ∘ line through the origin inclining down from left to right, the correct graph is (D). c. 𝑓(𝑥) = √𝑥 (0 < 𝑥 < 4) Since the graph of 𝑓(𝑥) = √𝑥 is the top half of a sideways parabola, the correct graph is (E). 1 d. 𝑓(𝑥) = 𝑥 + − 2 (0 < 𝑥 < 4) 𝑥 If we plot a few points like 𝑥 = 1∕2, 1, 2, and 3, we find that the correct graph is (F). e. 𝑓(𝑥) = |𝑥| (−1 ≤ 𝑥 ≤ 1) Since the graph of 𝑓(𝑥) = |𝑥|is a "V"-shape with its vertex at the origin, the correct graph is (C). f. 𝑓(𝑥) = 𝑥 − 1 (−1 ≤ 𝑥 ≤ 1)


Solutions Section 1.1 Since the graph of 𝑓(𝑥) = 𝑥 − 1 is a straight line through (0, −1) and (1, 0), the correct graph is (B).

26. a. 𝑓(𝑥) = −𝑥 + 3 (0 < 𝑥 ≤ 3) Since the graph of 𝑓(𝑥) = −𝑥 + 3 is a straight line inclining down from left to right, the correct graph must be (D). b. 𝑓(𝑥) = 2 − |𝑥| (−2 < 𝑥 ≤ 2) Since 𝑓(𝑥) = 2 − |𝑥| is obtained from the graph of 𝑦 = |𝑥| by flipping it vertically (the minus sign in front of |𝑥|) and then moving it 2 units vertically up (adding 2 to all the values), the correct graph is (F). c. 𝑓(𝑥) = √𝑥 + 2 (−2 < 𝑥 ≤ 2) The graph of𝑓(𝑥) = √𝑥 + 2 is similar to that of 𝑦 = √𝑥, which is half a parabola on its side, and the correct graph is (A). d. 𝑓(𝑥) = −𝑥 2 + 2 (−2 < 𝑥 ≤ 2) The graph of 𝑓(𝑥) = −𝑥 2 + 2 is a parabola opening down, so the correct graph is (C). 1 e. 𝑓(𝑥) = − 1 𝑥 1 The graph of 𝑓(𝑥) = − 1 (0 < 𝑥 ≤ 3) is part of a hyperbola, and the correct graph is (E). 𝑥 f. 𝑓(𝑥) = 𝑥 2 − 1 (−2 < 𝑥 ≤ 2) The graph of 𝑓(𝑥) = 𝑥 2 − 1 is a parabola opening up, so the correct graph is (B). 27. Technology formula: 0.1*x^2 - 4*x+5 Table of values: 𝑥

𝑓(𝑥)

0 5

1

2

1.1

3

4

5

−5

𝑔(𝑥) 39.9

−4

−3

30.3

21.5

−2

13.5

−1

6.3

0

1

0.5

8

9

10

1.5

2.5

3.5

2

3

4

5

−0.1 −5.7 −10.5 −14.5 −17.7 −20.1

29. Technology formula: (x^2-1)/(x^2+1) Table of values: 𝑥

7

−2.6 −6.1 −9.4 −12.5 −15.4 −18.1 −20.6 −22.9 −25

28. Technology formula: 0.4*x^2-6*x-0.1 Table of values: 𝑥

6

4.5

5.5

6.5

7.5

8.5

9.5

10.5

4

5

6

7

8

9

ℎ(𝑥) −0.6000 0.3846 0.7241 0.8491 0.9059 0.9360 0.9538 0.9651 0.9727 0.9781 0.9820 30. Technology formula: (2*x^2+1)/(2*x^2-1) Table of values: 𝑥

−1

0

1

2

3

𝑟(𝑥) 3.0000 −1.0000 3.0000 1.2857 1.1176 1.0645 1.0408 1.0282 1.0206 1.0157 1.0124

𝑥 if − 4 ≤ 𝑥 < 0 {2 if 0 ≤ 𝑥 ≤ 4 Technology formula: x*(x\lt 0)+2*(x\gt =0) (For a graphing calculator, use ≥ instead of >=.) 31. 𝑓(𝑥) =


Solutions Section 1.1

a. 𝑓(−1) = −1. We used the first formula, since −1 is in [−4, 0). b. 𝑓(0) = 2. We used the second formula, since 0 is in [0, 4]. c. 𝑓(1) = 2. We used the second formula, since 1 is in [0, 4].

−1 if − 4 ≤ 𝑥 ≤ 0 {𝑥 if 0 < 𝑥 ≤ 4 Technology formula: (-1)*(x\lt =0)+x*(x\gt 0) (For a graphing calculator, use ≤ instead of <=.) 32. 𝑓(𝑥) =

a. 𝑓(−1) = −1. We used the first formula, since −1 is in [−4, 0]. b. 𝑓(0) = −1. We used the first formula, since 0 is in [−4, 0]. c. 𝑓(1) = 1. We used the second formula, since 1 is in (0, 4].

⎧𝑥 2 if − 2 < 𝑥 ≤ 0 33. 𝑓(𝑥) = ⎪ ⎨ 1 if 0 < 𝑥 ≤ 4 ⎪𝑥 ⎩ Technology formula: (x^2)*(x<=0)+(1/x)*(0<x) (For a graphing calculator, use ≤ instead of <=.)

a. 𝑓(−1) = 1 2 = 1. We used the first formula, since −1 is in (−2, 0]. b. 𝑓(0) = 0 2 = 0. We used the first formula, since 0 is in (−2, 0]. c. 𝑓(1) = 1∕1 = 1. We used the second formula, since 1 is in (0, 4].


if − 2 < 𝑥 ≤ 0 −𝑥 {√𝑥 if 0 < 𝑥 < 4 Technology formula: Excel: (-1\*x^2)\*(x\lt =0)+SQRT(ABS(x))\*(x\gt 0) TI-83/84 Plus: (-1\*x^2)\*(x≤0)+ √(x)\*(x\gt 0) 2

34. 𝑓(𝑥) =

Solutions Section 1.1

a. 𝑓(−1) = −(−1) 2 = −1. We used the first formula, since −1 is in (−2, 0]. b. 𝑓(0) = −0 2 = 0. We used the first formula, since 0 is in (−2, 0]. c. 𝑓(1) = √1 = 1. We used the second formula, since 1 is in (0, 4).

if − 1 < 𝑥 ≤ 0 ⎧𝑥 35. 𝑓(𝑥) = ⎪ 𝑥 1 if 0 < 𝑥 ≤ 2 + ⎨ ⎪ 𝑥 if 2 < 𝑥 ≤ 4 ⎩ Technology formula: x*(x<=0)+(x+1)*(0<x)*(x<=2)+x*(2<x) (For a graphing calculator, use ≤ instead of <=.)

a. 𝑓(0) = 0. We used the first formula, since 0 is in (−1, 0]. b. 𝑓(1) = 1 + 1 = 2. We used the second formula, since 1 is in (0, 2]. c. 𝑓(2) = 2 + 1 = 3. We used the second formula, since 2 is in (0, 2]. d. 𝑓(3) = 3. We used the third formula, since 3 is in (2, 4].

if − 1 < 𝑥 < 0 ⎧−𝑥 𝑥 2 if 0 ≤ 𝑥 ≤ 2 − 36. 𝑓(𝑥) = ⎪ ⎨ ⎪ if 2 < 𝑥 ≤ 4 ⎩−𝑥 Technology formula: x*(x\lt 0)+(x-2)*(0\lt =x)*(x\lt =2)+(-x)*(2\lt x) (For a graphing calculator, use ≤ instead of <=.) y

1 -1

2 -2 -4

4

x


Solutions Section 1.1

a. 𝑓(0) = 0 − 2 = −2. We used the second formula, since 0 is in [0, 2]. b. 𝑓(1) = 1 − 2 = −1. We used the second formula, since 1 is in [0, 2]. c. 𝑓(2) = 2 − 2 = 0. We used the second formula, since 2 is in [0, 2]. d. 𝑓(3) = −3. We used the third formula, since 3 is in (2, 4]. 37. 𝑓(𝑥) = 𝑥 2 a. 𝑓(𝑥 + ℎ) = (𝑥 + ℎ) 2 Therefore,

𝑓(𝑥 + ℎ) − 𝑓(𝑥) = (𝑥 + ℎ) 2 − 𝑥 2 = 𝑥 2 + 2𝑥ℎ + ℎ 2 − 𝑥 2 = 2𝑥ℎ + ℎ 2 = ℎ(2𝑥 + ℎ)

b. Using the answer to part (a), 𝑓(𝑥 + ℎ) − 𝑓(𝑥) ℎ(2𝑥 + ℎ) = = 2𝑥 + ℎ ℎ ℎ

38. 𝑓(𝑥) = 3𝑥 − 1 a. 𝑓(𝑥 + ℎ) = 3(𝑥 + ℎ) − 1 = 3𝑥 + 3ℎ − 1 Therefore, 𝑓(𝑥 + ℎ) − 𝑓(𝑥) = 3𝑥 + 3ℎ − 1 − (3𝑥 − 1) = 3𝑥 + 3ℎ − 1 − 3𝑥 + 1 = 3ℎ

b. Using the answer to part (a), 𝑓(𝑥 + ℎ) − 𝑓(𝑥) 3ℎ = =3 ℎ ℎ

39. 𝑓(𝑥) = 2 − 𝑥 2 a. 𝑓(𝑥 + ℎ) = 2 − (𝑥 + ℎ) 2 Therefore,

𝑓(𝑥 + ℎ) − 𝑓(𝑥) = 2 − (𝑥 + ℎ) 2 − (2 − 𝑥 2) = 2 − 𝑥 2 − 2𝑥ℎ − ℎ 2 − 2 + 𝑥 2 = −2𝑥ℎ − ℎ 2 = −ℎ(2𝑥 + ℎ)

b. Using the answer to part (a), 𝑓(𝑥 + ℎ) − 𝑓(𝑥) −ℎ(2𝑥 + ℎ) = = −(2𝑥 + ℎ) ℎ ℎ 40. 𝑓(𝑥) = 𝑥 2 + 𝑥 a. 𝑓(𝑥 + ℎ) = (𝑥 + ℎ) 2 + (𝑥 + ℎ)Therefore,

𝑓(𝑥 + ℎ) − 𝑓(𝑥) = (𝑥 + ℎ) 2 + (𝑥 + ℎ) − (𝑥 2 + 𝑥) = 𝑥 2 + 2𝑥ℎ + ℎ 2 + 𝑥 + ℎ − 𝑥 2 − 𝑥 = 2𝑥ℎ + ℎ 2 + ℎ = ℎ(2𝑥 + ℎ + 1)

b. Using the answer to part (a), 𝑓(𝑥 + ℎ) − 𝑓(𝑥) ℎ(2𝑥 + ℎ + 1) = = 2𝑥 + ℎ + 1 ℎ ℎ

41. From the table, a. 𝑝(2) = 0.67; Pemex produced 0.67 billion barrels of crude oil in 2017 (𝑡 = 2). 𝑝(3) = 0.61; Pemex produced 0.61 billion barrels of crude oil in 2018 (𝑡 = 3). 𝑝(6) = 0.62; Pemex produced 0.62 billion barrels of crude oil in 2021 (𝑡 = 6).


Solutions Section 1.1 b. 𝑝(6) − 𝑝(3) = 0.62 − 0.61 = 0.01; Annual crude oil production by Pemex increased by 0.01 billion barrels from 2018 (𝑡 = 3) to 2021 (𝑡 = 6). 42. From the table, a. 𝑠(0) = 0.69; Pemex produced 0.69 billion barrels of offshore crude oil in 2015 (𝑡 = 0). 𝑠(2) = 0.56; Pemex produced 0.56 billion barrels of offshore crude oil in 2017 (𝑡 = 2). 𝑠(4) = 0.51; Pemex produced 0.51 billion barrels of offshore crude oil in 2019 (𝑡 = 4). b. 𝑠(4) − 𝑠(0) = 0.51 − 0.69 = −0.18; Annual offshore crude oil production by Pemex decreased by 0.18 billion barrels from 2015 (𝑡 = 0) to 2019 (𝑡 = 4). 43. a. Graph of 𝑝:

The graph suggests a curve, so we exclude the linear models (A) and (C). Model (B) gives a parabola that curves up whereas the curve suggested by the graph curves down, leaving us with Model (D). Also, model (D) gives almost the exact values shown in the chart. (Use the technology formula -0.25x^2+3.5x+57.) b. Using Model (D), 𝑝(5) = −0.25(5) 2 + 3.5(5) + 57 = 68.25 Interpretation: 𝑡 = 5 represents 5 years since the start of 2013, or the start of 2018. Thus, we interpret the answer as follows: Approximately 68.25% of U.S. adults used Facebook at the start of 2018 c. The graph becomes less steep as time increases, indicating decelerating Facebook membership over the period 2013–2021. 44. a. Graph of 𝑝:

The graph suggests a curve, so we exclude the linear model (B). Model (D) gives a parabola that curves up whereas the curve suggested by the graph curves down, leaving us with Models (A) and (C). Model (A) gives values that round to the values shown in the chart wheeras Model (C) is way off. (Use the technology formula -0.32x^2+3.6x+13. b. Using Model (A), 𝑝(7.5) = −0.32(7.5) 2 + 3.6(7.5) + 13 = 22 Interpretation: 𝑡 = 7.5 represents 7.5 years since the start of 2013, or midway through 2020. Thus, we interpret the answer as follows: Approximately 22% of U.S. adults used Twitter midway through 2020. c. The graph becomes less steep as time increases, indicating decelerating Twitter membership over the period 2013–2021.

45. From the graph, 𝑓(11) ≈ 800. Because 𝑓 is the number of thousands of housing starts in year 11, we interpret the result as follows: Approximately 800,000 homes were started in 2016. Similarly, 𝑓(15) ≈ 1,000. Approximately 1,000,000 homes were started in 2020. Also, we estimate 𝑓(2.5) ≈ 800. Because 𝑡 = 2.5 is midway between 2007 and 2008, we interpret the result as follows: approximately 800,000 homes were started in the year beginning midway through 2007.


Solutions Section 1.1 46. From the graph, 𝑓(3) ≈ 600. Because 𝑓 is the number of thousands of housing starts in year 3, we interpret the result as follows: Approximately 600,000 homes were started in 2008. 𝑓(12) ≈ 900: Approximately 900,000 homes were started in 2016. 𝑓(14.5) ≈ 950: Approximately 950,000 homes were started in the year beginning midway through 2019.

47. 𝑓(14 − 10) = 𝑓(4) ≈ 400. Interpretation: Approximately 400,000 homes were started in 2009 (𝑡 = 4). 𝑓(14) − 𝑓(10) ≈ 900 − 700 = 200. Interpretation: 𝑓(14) − 𝑓(10) is the change in the number of housing starts (in thousands) from 2015 to 2019; there were approximately 200,000 more housing starts in 2019 than in 2015. 48. 𝑓(15 − 1) = 𝑓(14) ≈ 900. Interpretation: Approximately 900,000 homes were started in 2019 (𝑡 = 14). 𝑓(15) − 𝑓(1) ≈ 1,000 − 1,500 = −500. Interpretation: 𝑓(15) − 𝑓(1) is the change in the number of housing starts (in thousands) from 2006 to 2020; there were approximately 500,000 fewer housing starts in 2020 than in 2006.

49. 𝑓(𝑡 + 5) − 𝑓(𝑡) measures the change from year 𝑡 to the year five years later. It is greatest when the line segment from year 𝑡 to year 𝑡 + 5 is steepest upward-sloping. From the graph, or by computing differences of values estimated from the graph, this occurs when 𝑡 = 6 or 7: 𝑓(11) − 𝑓(6) = 800 − 400 = 400; 𝑓(12) − 𝑓(7) = 900 − 500 = 400. Interpretation: The greatest five-year increase in the number of housing starts occurred in 2011–2016 and again in 2012–2017. 50. 𝑓(𝑡) − 𝑓(𝑡 − 1) measures the change from year 𝑡 − 1 to the following year. It is least when the the line segment from year 𝑡 − 1 to year 𝑡 is steepest downward-sloping. From the graph, this occurs when 𝑡 = 2, for a change of 𝑓(2) − 𝑓(1) = 1,000 − 1,500 = −500. Interpretation: The greatest annual decrease in the number of housing starts occurred in 2006–2007.

51. a. From the graph, 𝑏(3) ≈ 50 and 𝑏(5) ≈ 35. The (50-day average) price of Bitcoin was approximately $50,000 on June 1, 2021 (𝑡 = 3) and $35,000 on August 1, 2021 (𝑡 = 5). 𝑏(7) − 𝑏(5) ≈ 50 − 35 = 15. The Bitcoin price increased by around $15,000 from August 1 to October 1, 2021. b. Increasing most rapidly when the graph is steepest upward from left to right on the interal [1, 3], which occurs at 𝑡 = 1 (integer answer required). Thus, during the period April 1–June 1, 2021, the price of Bitcoin was increasing most rapidly around April 1. c. Decreasing most rapidly when the graph is steepest down from left to right, which occurs at 𝑡 = 3 (integer answer required). Thus, during the period April 1–June 1, 2021, the price of Butcoin was decreasing most rapidly around June 1. 52. a. From the graph, 𝑒(2) ≈ 2.5 and 𝑒(4) ≈ 2.5. The price of Etherium was approximately $2,500 on May 1, 2021 (𝑡 = 2) and again on July 1, 2021 (𝑡 = 4) . 𝑒(3) − 𝑒(1) ≈ 2 − 3 = −1. The price of Etherium decreased by around $1,000 from April 1 to June 1, 2021. b. Increasing most rapidly when the graph is steepest upward from left to right on the interal [1, 5], which occurs at 𝑡 = 4 (integer answer required). Thus, during the period April 1–August 1, 2021, the price of Etherium was increasing most rapidly around July 1. c. Decreasing most rapidly when the graph is steepest down from left to right, which occurs at 𝑡 = 2 (integer answer required). Thus, during the period April 1–August 1, 2021, the price of Etherium was decreasing most rapidly around May 1. 53. 𝑟(𝑥) = 0.4𝑥 2 + 𝑥 + 26.5 (0 ≤ 𝑥 ≤ 5) a. The domain of 𝑟 is [0, 5].


𝑟(0) = 0.4(0) + 0 + 26.5 = 26.5; 𝑟(1) = 0.4(1) 2 + 1 + 26.5 = 27.9 𝑟(3) = 0.4(3) 2 + 3 + 26.5 = 33.1 𝑟(5) = 0.4(5) 2 + 5 + 26.5 = 41.5 Food delivery app revenues in the U.S. were projected to be $26.5 billion in 2020, $27.9 billion in 2021, $33.1 billion in 2023, and $41.5 billion in 2025. b. Graph: 2

Solutions Section 1.1

(As the curve is concave up the projected revenue was accelerating.)

54. 𝑟(𝑥) = −0.2𝑥 2 + 3𝑥 + 18 (0 ≤ 𝑥 ≤ 5) a. The domain of 𝑟 is [0, 5]. 𝑟(0) = −0.2(0) 2 + 3(0) + 18 = 18; 𝑟(1) = −0.2(1) 2 + 3(1) + 18 = 20.8 𝑟(3) = −0.2(3) 2 + 3(3) + 18 = 25.2 𝑟(5) = −0.2(5) 2 + 3(5) + 18 = 28 Food delivery app revenues in Europe were projected to be $18 billion in 2020, $20.8 billion in 2021, $25.2 billion in 2023, and $28 billion in 2025. b. Graph

(As the graph is concave down the projected revenue was decelerating). 55. a. The model is valid for the range 1958 (𝑡 = 0) through 1966 (𝑡 = 8). Thus, an appropriate domain is [0, 8]. 𝑡 ≥ 0 is not an appropriate domain because it would predict federal funding of NASA beyond 1966, whereas the model is based only on data up to 1966. 4.5 b. 𝑝(𝑡) = 2 1.07 (𝑡−8) 4.5 ⇒ 𝑝(5) = ≈ 2.4 Technology formula: 4.5/(1.07^((t-8)^2)) 2 1.07 (5−8) 𝑡 = 5 represents 1958 + 5 = 1963, and therefore we interpret the result as follows: In 1963, 2.4% of the U.S. federal budget was allocated to NASA. c. 𝑝(𝑡) is increasing most rapidly when the graph is steepest upward-sloping from left to right, and, among the given values of 𝑡, this occurs when 𝑡 = 5. Thus, the percentage of the budget allocated to NASA was


Solutions Section 1.1 increasing most rapidly in 1963.

56. a. [1, 55]; [0, 55] is not an appropriate domain because 𝑝 is undefined at 0. 5 b. 𝑝(𝑡) = 0.03 + 0.6 𝑡 5 ⇒ 𝑝(40) = 0.03 + ≈ 0.58 Technology formula: 0.03+5/t^0.6 40 0.6 𝑡 = 40 represents 1965 + 40 = 2005, and therefore we interpret the result as follows: In 2005, 0.58% of the US federal budget was allocated to NASA. c. If we evaluate 𝑝(𝑡) for 𝑡 = 100, 1,000, 100,000, 1,000,000 . . . , we find values of 𝑝(𝑡) decreasing toward 0.03. Thus, in the (very) long term, the percentage of the budget allocated to NASA is predicted to approach 0.03% 12, 200 57. a. 𝑝(𝑡) = 100⎛1 − 4.48 ⎞ (𝑡 ≥ 8.5) ⎝⎜ ⎠⎟ 𝑡 Technology formula: 100*(1-12200/t^4.48) b. Graph:

c. Table of values: 𝑡

9

𝑝(𝑡) 35.2

10

11

12

13

14

15

16

17

18

19

20

59.6

73.6

82.2

87.5

91.1

93.4

95.1

96.3

97.1

97.7

98.2

d. From the table, 𝑝(12) = 82.2, so that 82.2% of children are able to speak in at least single words by the age of 12 months. e. We seek the first value of 𝑡 such that 𝑝(𝑡) is at least 90. Since 𝑡 = 14 has this property (𝑝(14) = 91.1) we conclude that, at 14 months, 90% or more children are able to speak in at least single words. 5.27 × 10 17 (𝑡 ≥ 30) " 𝑡 12 Technology formula: 100*(1-5.27*10^17/t^12) b.Graph: 58. a. 𝑝(𝑡) = 100!1 −

c. Table of values: 𝑡

𝑝(𝑡)

30

31

32

33

34

35

36

37

38

39

40

0.8

33.1

54.3

68.4

77.9

84.4

88.9

92.0

94.2

95.7

96.9

d. From the table, 𝑝(36) = 88.9, so that 88.9% of children are able to speak in sentences of five or more words by the age of 36 months.


Solutions Section 1.1 e. We seek the first value of 𝑡 such that 𝑝(𝑡) is at least 75. Since 𝑡 = 34 has this property (𝑝(34) = 77.9) we conclude that, at 34 months, 75% or more children are able to speak in sentences of five or more words. if 0 ≤ 𝑡 < 16 ⎧8(1.22) 𝑡 ⎪ 59. 𝑣(𝑡) = ⎨400𝑡 − 6,200 if 16 ≤ 𝑡 < 25 ⎪0.45(𝑡 − 25) 3 + 3,800 if 25 ≤ 𝑡 ≤ 40 ⎩

a. 𝑣(10) = 8(1.22) 10 ≈ 58. We used the first formula, since 10 is in [0, 16). 𝑣(16) = 400(16) − 6,200 = 200. We used the second formula, since 16 is in [16, 25). 𝑣(40) = 0.45(40 − 25) 3 + 3,800 ≈ 5,319 We used the third formula, since 40 is in [25, 40]. Interpretation: Processor speeds were about 58 MHz in 1990, 200 MHz in 1996, and 5,319 MHz in 2020. b. Technology formula (using 𝑥 as the independent variable): (8*(1.22)^x)*(x<16)+(400*x-6200)*(x>=16)*(x<25) +(0.45*(x-25)^3+3800)*(x>=25) (For a graphing calculator, use ≤ instead of <=.) c. Using the above technology formula (for instance, on the Function Evaluator and Grapher on the Web site) we obtain the graph and table of values. Graph:

Table of values: 𝑡

𝑣(𝑡)

0

4

8

12

16

20

24

28

32

36

40

8

18

39

87

200 1,800 3,400 3,800 4,000 4,400 5,300

d. From either the graph or the table, we see that the speed reached 3,000 MHz between 𝑡 = 20 and 𝑡 = 24. We can obtain a more precise answer algebraically by using the formula for the corresponding portion of the graph: 3,000 = 400𝑡 − 6,200 giving 𝑡=

9,200 = 23 400

Since 𝑡 is time since 1980, 𝑡 = 23 corresponds to 2003.

⎧0.12𝑡 2 + 0.04𝑡 + 0.2 if 0 ≤ 𝑡 < 12 60. 𝑣(𝑡) = ⎪ if 12 ≤ 𝑡 < 26 ⎨1.1(1.22) 𝑡 ⎪400𝑡 − 10,200 if 26 ≤ 𝑡 ≤ 30 ⎩ 2 a. 𝑣(2) = 0.12(2) + 0.04(2) + 0.2 = 0.76. We used the first formula, since 2 is in [0, 12). 𝑣(12) = 1.1(1.22) 12 ≈ 12. We used the second formula, since 12 is in [12, 26). 𝑣(28) = 400(28) − 10, 200 = 1, 000. We used the third formula, since 28 is in [26, 30]. Interpretation: Processor speeds were about 0.76 MHz in 1972, 12 MHz in 1982, and 1,000 MHz in 1998.


Solutions Section 1.1

b. Technology formula (using 𝑥 as the independent variable): (0.12*x^2+0.04*x+0.2)*(x<12)+(1.1*(1.22)^x)*(x>=12)*(x<26) +(400*x-10200)*(x>=26) (For a graphing calculator, use ≤ instead of <=.) c. Using the above technology formula (for instance, on the Function Evaluator and Grapher on the Web site) we obtain the graph and table of values. Graph:

Table of values: 𝑡

0

2

𝑣(𝑡) 0.20 0.76

4

6

8

10

12

14

16

18

20

22

24

26

28

30

2.3

4.8

8.2

13

12

18

26

39

59

87

130

200 1,000 1,800

d. From either the graph or the table, we see that the speed reached 500 MHz around 𝑡 = 27. We can obtain a more precise answer algebraically by using the formula for the corresponding portion of the graph: 500 = 400𝑡 − 10,200

giving 𝑡=

10,700 = 26.75 ≈ 27 to the nearest year 400

Since 𝑡 is time since 1970, 𝑡 = 27 corresponds to 1997. 61. a. Each row of the table gives us a formula with a condition: First row in words: 10% of the amount over $0 if your income is over $0 and not over $9,950. Translation to formula: 0.10𝑥 if 0 < 𝑥 ≤ 9,950. Second row in words: $995.00 + 12% of the amount over $9,950 if your income is over $9,950 and not over $40,525. Translation to formula: 995.00 + 0.12(𝑥 − 9, 950) if 9,950 < 𝑥 ≤ 40,525. Continuing in this way leads to the following piecewise-defined function: ⎧0.10𝑥 ⎪995.00 + 0.12(𝑥 − 9,950) ⎪ ⎪4,664 + 0.22(𝑥 − 40,525) 𝑇 (𝑥) = ⎪ ⎨14,751 + 0.24(𝑥 − 86,375) ⎪33,603 + 0.32(𝑥 − 164,925) ⎪ ⎪47,843 + 0.35(𝑥 − 209,425) ⎪157,804.25 + 0.37(𝑥 − 523,600) ⎩

if 0 < 𝑥 ≤ 9,950 if 9,950 < 𝑥 ≤ 40,525 if 40,525 < 𝑥 ≤ 86,375 if 86,375 < 𝑥 ≤ 164,925 if 164,925 < 𝑥 ≤ 209,425 if 209,425 < 𝑥 ≤ 523,600 if 523,600 < 𝑥


Solutions Section 1.1

b. A taxable income of $45,000 falls in the bracket 40,525 < 𝑥 ≤ 86,375 and so we use the formula 4,664 + 0.22(𝑥 − 40,525) : 4,664 + 0.22(45,000 − 40,525) = 4,664 + 0.22(4,475) = $5,648.50 62. a. Each row of the table gives us a formula with a condition: First row in words: 10% of the amount over $0 if your income is over $0 and not over $8,700. Translation to formula: 0.10𝑥 if 0 < 𝑥 ≤ 8,700. Second row in words: $870.00 + 15% of the amount over $8,700 if your income is over $8,700 and not over $35,350. Translation to formula: 870.00 + 0.15(𝑥 − 8,700) if 8,700 < 𝑥 ≤ 35,350. Continuing in this way leads to the following piecewise-defined function: ⎧0.10𝑥 ⎪870.00 + 0.15(𝑥 − 8,700) ⎪ ⎪4,867.50 + 0.22(𝑥 − 35,350) 𝑇 (𝑥) = ⎨ 17,442.50 + 0.28(𝑥 − 85,650) ⎪ ⎪43,482.50 + 0.33(𝑥 − 178,650) ⎪112,683.50 + 0.35(𝑥 − 388,350) ⎩

if 0 < 𝑥 ≤ 8,700 if 8,700 < 𝑥 ≤ 35,350 if 35,350 < 𝑥 ≤ 85,650 if 85,650 < 𝑥 ≤ 178,650 if 178,650 < 𝑥 ≤ 388,350 if 388,350 < 𝑥

b. A taxable income of $45,000 falls in the bracket 35,350 < 𝑥 ≤ 85,650 and so we use the formula 4,867.50 + 0.22(𝑥 − 35,350) : 4,867.50 + 0.22(45,000 − 35,350) = 4,867.50 + 0.22(9,650) = $9,990.50.

63. The dependent variable is a function of the independent variable. Here, the market price of gold 𝑚 is a function of time 𝑡. Thus, the independent variable is 𝑡 and the dependent variable is 𝑚. 64. The dependent variable is a function of the independent variable. Here, the weekly profit 𝑃 is a function of the selling price 𝑠. Thus, the independent variable is 𝑠 and the dependent variable is 𝑃 .

65. To obtain the function notation, write the dependent variable as a function of the independent variable. Thus 𝑦 = 4𝑥 2 − 2 can be written as 𝑓(𝑥) = 4𝑥 2 − 2 or 𝑦(𝑥) = 4𝑥 2 − 2 66. To obtain the equation notation, introduce a dependent variable instead of the function notation. Thus 𝐶(𝑡) = −0.34𝑡 2 + 0.1𝑡 can be written as 𝑐 = −0.34𝑡 2 + 0.1𝑡 or 𝑦 = −0.34𝑡 2 + 0.1𝑡 67. False. A graph usually gives infinitely many values of the function while a numerical table will give only a finite number of values.

68. True. An algebraically specified function 𝑓 is specified by algebraic formulas for 𝑓(𝑥). Given such formulas, we can construct the graph of 𝑓 by plotting the points (𝑥, 𝑓(𝑥)) for values of 𝑥 in the domain of 𝑓. 69. False. In a numerically specified function, only certain values of the function are specified so we cannot know its value on every real number in [0, 10], whereas an algebraically specified function would give values for every real number in [0, 10].


Solutions Section 1.1 70. False. A graphically specified function is specified by a graph. However, we cannot always expect to find an algebraic formula whose graph is exactly the graph that is given. 71. Functions with infinitely many points in their domain (such as 𝑓(𝑥) = 𝑥 2) cannot be specified numerically. So, the assertion is false.

72. A numerical model supplies only the values of a function at specific values of the independent variable, whereas an algebraic model supplies the value of a function at every point in its domain. Thus, an algebraic model supplies more information.

73. (Answers may vary.) Take 𝑓(𝑥) = 𝑥 2. Then a. 𝑓(3 + 2) = 𝑓(5) = 5 2 = 25, whereas 𝑓(3) + 𝑓(2) = 3 2 + 2 2 = 9 + 4 = 13. Thus, 𝑓(3 + 2) ≠ 𝑓(3) + 𝑓(2). b. 𝑓(3 − 2) = 𝑓(1) = 1 2 = 1, whereas 𝑓(3) − 𝑓(2) = 3 2 − 2 2 = 9 − 4 = 5. Thus, 𝑓(3 − 2) ≠ 𝑓(3) − 𝑓(2). 74. (Answers may vary.) Take 𝑓(𝑥) = 𝑥 + 1. Then a. 𝑓(3 × 2) = 𝑓(6) = 6 + 1 = 7, whereas 3𝑓(2) = 3(2 + 1) = 3 × 3 = 9. Thus, 𝑓(3 × 2) ≠ 3𝑓(2). b. 𝑓(3 × 2) = 𝑓(6) = 6 + 1 = 7, whereas 𝑓(3)𝑓(2) = (3 + 1)(2 + 1) = 4 × 3 = 12. Thus, 𝑓(3 × 2) ≠ 𝑓(3)𝑓(2).

75. As the text reminds us: to evaluate 𝑓 of a quantity (such as 𝑥 + ℎ) replace 𝑥 everywhere by the whole quantity 𝑥 + ℎ : 𝑓(𝑥) = 𝑥2 − 1 𝑓(𝑥 + ℎ) = (𝑥 + ℎ) 2 − 1.

76. Knowing 𝑓(𝑥) for two values of 𝑥 does not convey any information about 𝑓(𝑥) at any other value of 𝑥. Interpolation is only a way of estimating 𝑓(𝑥) at values of 𝑥 not given.

77. If two functions are specified by the same formula 𝑓(𝑥), say, their graphs must follow the same curve 𝑦 = 𝑓(𝑥). However, it is the domain of the function that specifies what portion of the curve appears on the graph. Thus, if the functions have different domains, their graphs will be different portions of the curve 𝑦 = 𝑓(𝑥). 78. If we plot points of the graphs 𝑦 = 𝑓(𝑥) and 𝑦 = 𝑔(𝑥), we see that, since 𝑔(𝑥) = 𝑓(𝑥) + 10, we must add 10 to the 𝑦-coordinate of each point in the graph of 𝑓 to get a point on the graph of 𝑔. Thus, the graph of 𝑔 is 10 units higher up than the graph of 𝑓. 79. Suppose we already have the graph of 𝑓 and want to construct the graph of 𝑔. We can plot a point of the graph of 𝑔 as follows: Choose a value for 𝑥 (𝑥 = 7, say) and then "look back" 5 units to read off 𝑓(𝑥 − 5) (𝑓(2) in this instance). This value gives the 𝑦-coordinate we want. In other words, points on the graph of 𝑔 are obtained by "looking back 5 units" to the graph of 𝑓 and then copying that portion of the curve. Put another way, the graph of 𝑔 is the same as the graph of 𝑓, but shifted 5 units to the right:

80. Suppose we already have the graph of 𝑓 and want to construct the graph of 𝑔. We can plot a point of


Solutions Section 1.1 the graph of 𝑔 as follows: Choose a value for 𝑥 (𝑥 = 7, say) and then look on the other side of the 𝑦-axis to read off 𝑓(−𝑥) (𝑓(−7) in this instance). This value gives the 𝑦-coordinate we want. In other words, points on the graph of 𝑔 are obtained by "looking back" to the graph of 𝑓 on the opposite side of the 𝑦-axis and then copying that portion of the curve. Put another way, the graph of 𝑔(𝑥) is the mirror image of the graph of 𝑓(𝑥) in the 𝑦-axis:


Solutions Section 1.2 Section 1.2

1. 𝑓(𝑥) = 𝑥 2 + 1 with domain (−∞, ∞) 𝑔(𝑥) = 𝑥 − 1 with domain (−∞, ∞) a. 𝑠(𝑥) = 𝑓(𝑥) + 𝑔(𝑥) = (𝑥 2 + 1) + (𝑥 − 1) = 𝑥 2 + 𝑥 b. Since both functions are defined for every real number 𝑥, the domain of 𝑠 is the set of all real numbers: (−∞, ∞). c. 𝑠(−3) = (−3) 2 + (−3) = 9 − 3 = 6

2. 𝑓(𝑥) = 𝑥 2 + 1 with domain (−∞, ∞) 𝑔(𝑥) = 𝑥 − 1 with domain (−∞, ∞) a. 𝑑(𝑥) = 𝑔(𝑥) − 𝑓(𝑥) = (𝑥 − 1) − (𝑥 2 + 1) = −𝑥 2 + 𝑥 − 2 b. Since both functions are defined for every real number 𝑥, the domain of 𝑑 is the set of all real numbers: (−∞, ∞). c. 𝑑(−1) = −(−1) 2 + (−1) − 2 = −4 3. 𝑔(𝑥) = 𝑥 − 1 with domain (−∞, ∞) 𝑢(𝑥) = √𝑥 + 10 with domain [−10, 0) a. 𝑝(𝑥) = 𝑔(𝑥)𝑢(𝑥) = (𝑥 − 1)√𝑥 + 10 b. The domain of 𝑝 consists of all real numbers 𝑥 simultaneously in the domains of 𝑔 and 𝑢; that is, [−10, 0). c. 𝑝(−6) = (−6 − 1)√−6 + 10 = (−7)(2) = −14

4. ℎ(𝑥) = 𝑥 + 4 with domain [10, ∞) 𝑣(𝑥) = √10 − 𝑥 with domain [0, 10] a. 𝑝(𝑥) = ℎ(𝑥)𝑣(𝑥) = (𝑥 + 4)√10 − 𝑥 b. The domain of 𝑝 consists of all real numbers 𝑥 simultaneously in the domains of ℎ and 𝑣; that is, the single point 𝑥 = 10. c. As 1 is not in the domain of 𝑝, 𝑝(1) is not defined.

5. 𝑔(𝑥) = 𝑥 − 1 with domain (−∞, ∞) 𝑣(𝑥) = √10 − 𝑥 with domain [0, 10] 𝑣(𝑥) √10 − 𝑥 a. 𝑞(𝑥) = = 𝑔(𝑥) 𝑥−1 b. The domain of 𝑞 consists of all real numbers 𝑥 simultaneously in the domains of 𝑣 and 𝑔 such that 𝑔(𝑥) ≠ 0. Since 𝑔(𝑥) = 0 when 𝑥 − 1 = 0, or 𝑥 = 1 we exclude 𝑥 = 1 from the domain of the quotient. Thus, the domain consists of all 𝑥 in [0, 10] excluding 𝑥 = 1 (since 𝑔(1) = 0), or 0 ≤ 𝑥 ≤ 10; 𝑥 ≠ 1. c. As 1 is not in the domain of 𝑞, 𝑞(1) is not defined.

6. 𝑔(𝑥) = 𝑥 − 1 with domain (−∞, ∞) 𝑣(𝑥) = √10 − 𝑥 with domain [0, 10] 𝑔(𝑥) 𝑥−1 a. 𝑞(𝑥) = = 𝑣(𝑥) √10 − 𝑥 b. The domain of 𝑞 consists of all real numbers 𝑥 simultaneously in the domains of 𝑣 and 𝑔 such that 𝑣(𝑥) ≠ 0. Since 𝑣(𝑥) = 0 when √10 − 𝑥 = 0, or 𝑥 = 10 we exclude 𝑥 = 10 from the domain of the quotient. Thus, the domain consists of all 𝑥 in [0, 10] excluding 𝑥 = 10; that is, [0, 10).


Solutions Section 1.2 1−1 c. 1 is in the domain of 𝑞, and 𝑞(1) = =0 √10 − 1 7. 𝑓(𝑥) = 𝑥 2 + 1 with domain (−∞, ∞) a. 𝑚(𝑥) = 5𝑓(𝑥) = 5(𝑥 2 + 1) b. The domain of 𝑚 is the same as the domain of 𝑓 : (−∞, ∞). c. 𝑚(1) = 5𝑓(1) = 5(1 2 + 1) = 10 8. 𝑢(𝑥) = √𝑥 + 10 with domain [−10, 0) a. 𝑚(𝑥) = 3𝑢(𝑥) = 3√𝑥 + 10 b. The domain of 𝑚 is the same as the domain of 𝑢 : [−10, 0) c. 𝑚(−1) = 3𝑢(−1) = 3√−1 + 10 = 9

9. Number of music files = Starting number + New files = 200 + 10 × Number of days So, 𝑁(𝑡) = 200 + 10𝑡 (𝑁 = number of music files, 𝑡 = time in days) 10. Free space left = Current amount − Decrease = 50 − 5× Number of months So, 𝑆(𝑡) = 50 − 5𝑡 (𝑆 = space on your HD, 𝑡 = time in months)

11. The number of hours you study, ℎ(𝑛), equals 4 on Sunday through Thursday and equals 0 on the remaining days. Since Sunday corresponds to 𝑛 = 1 and Thursday to 𝑛 = 5, we get 4 if 1 ≤ 𝑛 ≤ 5 . {0 if 𝑛 > 5

12. The number of hours you watch movies, ℎ(𝑛), equals 5 on Saturday (𝑛 = 7) and Sunday (𝑛 = 1) and equals 2 on the remaining days. 5 if 𝑛 = 1or 𝑛 = 7 . {2 otherwise

13. For a linear cost function, 𝐶(𝑥) = 𝑚𝑥 + 𝑏. Here, 𝑚 = marginal cost = $1,500 per piano, 𝑏 = fixed cost = $1,000. Thus, the daily cost function is 𝐶(𝑥) = 1,500𝑥 + 1,000. a. The cost of manufacturing 3 pianos is 𝐶(3) = 1,500(3) + 1,000 = 4,500 + 1,000 = $5,500. b. The cost of manufacturing each additional piano (such as the third one or the 11th one) is the marginal cost, 𝑚 = $1,500. c. Same answer as (b). d. Variable cost = part of the cost function that depends on 𝑥 = $1,500𝑥 Fixed cost = constant summand of the cost function = $1,000 Marginal cost = slope of the cost function = $1,500 per piano


Solutions Section 1.2 e. Graph: C 8,000 7,000 6,000 5,000 4,000 3,000 2,000 x

1,000

0

1

2

3

4

14. For a linear cost function, 𝐶(𝑥) = 𝑚𝑥 + 𝑏. Here, 𝑚 = marginal cost = $88 per tuxedo, 𝑏 = fixed cost = $20. Thus, the cost function is 𝐶(𝑥) = 88𝑥 + 20. a. The cost of renting 2 tuxes is 𝐶(2) = 88(2) + 20 = $196 b. The cost of each additional tux is the marginal cost 𝑚 = $88. c. Same answer as (b). d. Variable cost = part of the cost function that depends on 𝑥 = $88𝑥 Fixed cost = constant summand of the cost function = $20 Marginal cost = slope of the cost function = $88 per tuxedo e. Graph: C 400 350 300 250 200 150 100 50 0

1

2

3

x

4

15. a. For a linear cost function, 𝐶(𝑥) = 𝑚𝑥 + 𝑏. Here, 𝑚 = marginal cost = $0.40 per copy, 𝑏 = fixed cost = $70. Thus, the cost function is 𝐶(𝑥) = 0.4𝑥 + 70. The revenue function is 𝑅(𝑥) = 0.50𝑥. (𝑥 copies at 50¢ per copy) The profit function is 𝑃 (𝑥) = 𝑅(𝑥) − 𝐶(𝑥) = 0.5𝑥 − (0.4𝑥 + 70) = 0.5𝑥 − 0.4𝑥 − 70 = 0.1𝑥 − 70

b. 𝑃 (500) = 0.1(500) − 70 = 50 − 70 = −20 Since 𝑃 is negative, this represents a loss of $20. c. For breakeven, 𝑃 (𝑥) = 0 :


0.1𝑥 − 70 = 0 0.1𝑥 = 70 70 𝑥= = 700 copies 0.1

Solutions Section 1.2

16. a. For a linear cost function, 𝐶(𝑥) = 𝑚𝑥 + 𝑏. Here, 𝑚 = marginal cost = $0.15 per serving, 𝑏 = fixed cost = $350. Thus, the cost function is 𝐶(𝑥) = 0.15𝑥 + 350. The revenue function is 𝑅(𝑥) = 0.50𝑥. The profit function is 𝑃 (𝑥) = 𝑅(𝑥) − 𝐶(𝑥) = 0.50𝑥 − (0.15𝑥 + 350) = 0.35𝑥 − 350

b. For break-even, 𝑃 (𝑥) = 0 : 0.35𝑥 − 350 = 0 0.35𝑥 = 350 𝑥 = 1,000 servings c. 𝑃 (1,500) = 0.35(1,500) − 350 = 525 − 350 = $175, representing a profit of $175. 17. The revenue per jersey is $100. Therefore, Revenue 𝑅(𝑥) = $100𝑥. Profit = Revenue − Cost 𝑃 (𝑥) = 𝑅(𝑥) − 𝐶(𝑥) = 100𝑥 − (2,000 + 10𝑥 + 0.2𝑥 2) = −2,000 + 90𝑥 − 0.2𝑥 2

To break even, 𝑃 (𝑥) = 0, so −2,000 + 90𝑥 − 0.2𝑥 2 = 0. This is a quadratic equation with 𝑎 = −0.2, 𝑏 = 90, 𝑐 = −2,000 and solution −𝑏 ± √𝑏 2 − 4𝑎𝑐 2𝑎 −90 ± √(90) 2 − 4(−2,000)(−0.2) = ≈ 23.44 or 426.56 jerseys. 2(−0.2)

𝑥=

Since the second value is outside the domain, we use the first: 𝑥 = 23.44 jerseys. To make a profit, 𝑥 should be larger than this value: at least 24 jerseys. 18. The revenue per pair is $120. Therefore, Revenue 𝑅(𝑥) = $120𝑥. Profit = Revenue − Cost 𝑃 (𝑥) = 𝑅(𝑥) − 𝐶(𝑥) = 120𝑥 − (3,000 + 8𝑥 + 0.1𝑥 2) = −3,000 + 112 − 0.1𝑥 2

To break even, 𝑃 (𝑥) = 0, so −3,000 + 112 − 0.1𝑥 2 = 0. This is a quadratic equation with 𝑎 = −0.1, 𝑏 = 112, 𝑐 = −3,000 and solution 𝑥=

−𝑏 ± √𝑏 2 − 4𝑎𝑐 2𝑎 −90 ± √(112) 2 − 4(−3,000)(−0.1) = ≈ 27.46 or 1092.54 jerseys. 2(−0.1)


Solutions Section 1.2 Since the second value is outside the domain, we use the first: 𝑥 = 27.46 jerseys. To make a profit, 𝑥 should be larger than this value: at least 28 pairs of cleats.

19. The revenue from one thousand square feet (𝑥 = 1) is $0.1 million. Therefore, Revenue 𝑅(𝑥) = $0.1𝑥. Profit = Revenue − Cost 𝑃 (𝑥) = 𝑅(𝑥) − 𝐶(𝑥) = 0.1𝑥 − (1.7 + 0.12𝑥 − 0.0001𝑥 2) = −1.7 − 0.02𝑥 + 0.0001𝑥 2

To break even, 𝑃 (𝑥) = 0, so −1.7 − 0.02𝑥 + 0.0001𝑥 2 = 0. This is a quadratic equation with 𝑎 = 0.0001, 𝑏 = −0.02, 𝑐 = −1.7 and solution\\

−𝑏 ± √𝑏 2 − 4𝑎𝑐 2𝑎 0.02 ± √(−0.02) 2 − 4(0.0001)(−1.7) 0.02 ± 0.03286 = ≈ 264 thousand square feet = 2(0.0001) 0.0002

𝑥=

20. The revenue from one thousand square feet (𝑥 = 1) is $0.2 million. Therefore, Revenue 𝑅(𝑥) = $0.2𝑥. Profit = Revenue − Cost 𝑃 (𝑥) = 𝑅(𝑥) − 𝐶(𝑥) = 0.2𝑥 − (1.7 + 0.14𝑥 − 0.0001𝑥 2) = −1.7 + 0.06𝑥 + 0.0001𝑥 2

To break even, 𝑃 (𝑥) = 0, so −1.7 + 0.06𝑥 + 0.0001𝑥 2 = 0. This is a quadratic equation with 𝑎 = 0.0001, 𝑏 = 0.06, 𝑐 = −1.7 and solution\\

−𝑏 ± √𝑏 2 − 4𝑎𝑐 2𝑎 −0.06 ± √(−0.06) 2 − 4(0.0001)(−1.7) −0.06 ± 0.0654 = ≈ 27 thousand square feet = 2(0.0001) 0.0002

𝑥=

21. The hourly profit function is given by Profit = Revenue − Cost 𝑃 (𝑥) = 𝑅(𝑥) − 𝐶(𝑥) (Hourly) cost function: This is a fixed cost of $10,200 only: 𝐶(𝑥) = 10,200 (Hourly) revenue function: This is a variable of $200 per passenger cost only: 𝑅(𝑥) = 200𝑥 Thus, the profit function is 𝑃 (𝑥) = 𝑅(𝑥) − 𝐶(𝑥) 𝑃 (𝑥) = 200𝑥 − 10,200 For the domain of 𝑃 (𝑥), the number of passengers 𝑥 cannot exceed the capacity: 381. Also, 𝑥 cannot be negative. Thus, the domain is given by 0 ≤ 𝑥 ≤ 381, or [0, 381]. For breakeven, 𝑃 (𝑥) = 0 200𝑥 − 10,200 = 0 10,200 200𝑥 = 10,200, or 𝑥 = = 51 200 If 𝑥 is larger than this, then the profit function is positive, and so there should be at least 52 passengers; 𝑥 ≥ 52, for a profit.


Solutions Section 1.2 22. The hourly profit function is given by Profit = Revenue − Cost 𝑃 (𝑥) = 𝑅(𝑥) − 𝐶(𝑥) (Hourly) cost function: This is a fixed cost of $6,500 only: 𝐶(𝑥) = 6,500 (Hourly) revenue function: This is a variable of $100 per passenger cost only: 𝑅(𝑥) = 200𝑥 Thus, the profit function is 𝑃 (𝑥) = 𝑅(𝑥) − 𝐶(𝑥) 𝑃 (𝑥) = 200𝑥 − 6,500 For the domain of 𝑃 (𝑥), the number of passengers 𝑥 cannot exceed the capacity: 150 Also, 𝑥 cannot be negative. Thus, the domain is given by 0 ≤ 𝑥 ≤ 150, or [0, 150]. For breakeven, 𝑃 (𝑥) = 0 200𝑥 − 5,600 = 0 5,600 200𝑥 = 5,600, or 𝑥 = = 28 200 If 𝑥 is larger than this, then the profit function is positive, and so there should be at least 29 passengers; 𝑥 ≥ 29, for a profit. 23. To compute the break-even point, we use the profit function: Profit = Revenue − Cost 𝑃 (𝑥) = 𝑅(𝑥) − 𝐶(𝑥) 𝑅(𝑥) = 2𝑥 $2 per unit 𝐶(𝑥) = Variable Cost + Fixed Cost = 40% of Revenue + 6,000 = 0.4(2𝑥) + 6,000 = 0.8𝑥 + 6,000 Thus, 𝑃 (𝑥) = 𝑅(𝑥) − 𝐶(𝑥) 𝑃 (𝑥) = 2𝑥 − (0.8𝑥 + 6,000) = 1.2𝑥 − 6,000 For breakeven, 𝑃 (𝑥) = 0 1.2𝑥 − 6,000 = 0 6,000 1.2𝑥 = 6,000, so 𝑥 = = 5,000 1.2𝑥 Therefore, 5,000 units should be made to break even. 24. To compute the break-even point, we use the profit function: Profit = Revenue − Cost 𝑃 (𝑥) = 𝑅(𝑥) − 𝐶(𝑥) 𝑅(𝑥) = 5𝑥 $5 per unit 𝐶(𝑥) = Variable Cost + Fixed Cost = 30% of Revenue + 7,000 = 0.3(5𝑥) + 7,000 = 1.5𝑥 + 7,000 Thus, 𝑃 (𝑥) = 𝑅(𝑥) − 𝐶(𝑥) 𝑃 (𝑥) = 5𝑥 − (1.5𝑥 + 7,000) = 3.5𝑥 − 7,000 For breakeven, 𝑃 (𝑥) = 0 3.5𝑥 = 7,000, so 𝑥 = 2,000 units. 25. To compute the break-even point, we use the revenue and cost functions: 𝑅(𝑥) = Selling price × Number of units = 𝑆𝑃 𝑥 𝐶(𝑥) = Variable Cost + Fixed Cost = 𝑉 𝐶𝑥 + 𝐹 𝐶 (Note that "variable cost per unit" is marginal cost.) For breakeven 𝑅(𝑥) = 𝐶(𝑥) 𝑆𝑃 𝑥 = 𝑉 𝐶𝑥 + 𝐹 𝐶 Solve for 𝑥 : 𝑆𝑃 𝑥 − 𝑉 𝐶𝑥 = 𝐹 𝐶 𝐹𝐶 𝑥(𝑆𝑃 − 𝑉 𝐶) = 𝐹 𝐶, so 𝑥 = . 𝑆𝑃 − 𝑉 𝐶


Solutions Section 1.2 26. To compute the break-even point, we use the revenue and cost functions: 𝑅(𝑥) = 𝑆𝑃 𝑥; 𝐶(𝑥) = 𝑉 𝐶𝑥 + 𝐹 𝐶 At breakeven 𝑅(𝐵𝐸) = 𝐶(𝐵𝐸) 𝑆𝑃 (𝐵𝐸) = 𝑉 𝐶(𝐵𝐸) + 𝐹 𝐶 Thus, 𝐹 𝐶 = 𝑆𝑃 (𝐵𝐸) − 𝑉 𝐶(𝐵𝐸) = 𝐵𝐸(𝑆𝑃 − 𝑉 𝐶).

27. Take 𝑥 to be the number of grams of perfume he buys and sells. The profit function is given by Profit = Revenue − Cost: that is, 𝑃 (𝑥) = 𝑅(𝑥) − 𝐶(𝑥) Cost function 𝐶(𝑥) : Fixed costs: 20,000 Cheap perfume @ $20 per g: 20𝑥 Transportation @ $30 per 100 g: 0.3𝑥 Thus the cost function is 𝐶(𝑥) = 20𝑥 + 0.3𝑥 + 20,000 = 20.3𝑥 + 20,000 Revenue function R(x) 𝑅(𝑥) = 600𝑥 $600 per gram Thus, the profit function is 𝑃 (𝑥) = 𝑅(𝑥) − 𝐶(𝑥) 𝑃 (𝑥) = 600𝑥 − (20.3𝑥 + 20,000) = 579.7𝑥 − 20,000, with domain 𝑥 ≥ 0. For breakeven, 𝑃 (𝑥) = 0 579.7𝑥 − 20,000 = 0 20,000 579.7𝑥 = 20,000, so 𝑥 = ≈ 34.50 579.7 Thus, he should buy and sell 34.50 grams of perfume per day to break even.

28. Take 𝑥 to be the number of grams of perfume he buys and sells. The profit function is given by Profit = Revenue − Cost Cost function: 𝐶(𝑥) = 400𝑥 + 30𝑥 = 430𝑥 Revenue: 𝑅(𝑥) = 420𝑥 Thus, the profit function is 𝑃 (𝑥) = 𝑅(𝑥) − 𝐶(𝑥) = 420𝑥 − 430𝑥 = −10𝑥, with domain 𝑥 ≥ 0. For breakeven, −10𝑥 = 0, so 𝑥 = 0 grams per day; he should shut down the operation. 29. a. To graph the demand function we use technology with the formula 760/x-1 and with xMin = 400 and xMax = 700. Graph:

760 760 − 1 = 0.9, 𝑞(500) = − 1 = 0.52 400 500 So, the change in demand is 0.52 − 0.9 = −0.38 billion units, which means that demand decreases by about 380 million units sold per year. b. 𝑞(400) =


Solutions Section 1.2 c. The value of 𝑞 on the graph decreases by smaller and smaller amounts as we move to the right on the graph, indicating that the demand decreases at a smaller and smaller rate (Choice (D)). 30. a. To graph the demand function we use technology with the formula 350/x+0.5 and with xMin = 100 and xMax = 300. Graph:

350 350 + 0.5 = 4, 𝑞(200) = + 0.5 = 2.25 100 200 So, the change in demand is 2.25 − 4 = −1.75 billion units, which means that demand decreases by about 1.75 billion units sold per year. c. The value of 𝑞 on the graph increases by larger and larger amounts as we move to the left on the graph, indicating that the demand increases at a greater and greater rate (Choice (A)). b. 𝑞(100) =

31. The price at which there is neither a shortage nor surplus is the equilibrium price, which occurs when demand = supply: −3𝑝 + 700 = 2𝑝 − 500 5𝑝 = 1200 𝑝 = $240 per skateboard 32. The price at which there is neither a shortage nor surplus is the equilibrium price, which occurs when demand = supply: −5𝑝 + 50 = 3𝑝 − 30 8𝑝 = 80 𝑝 = $10 per skateboard 33. a. The equilibrium price occurs when demand = supply: −𝑝 + 57 = 4𝑝 − 136 5𝑝 = 193 𝑝 = 38.6, or $38,600 per vehicle. b. Since $38,400 is below the equilibrium price, there would be a shortage at that price. To calculate it, compute demand and supply: Demand: 𝑞 = −38.4 + 57 = 18.6 million vehicles Supply: 𝑞 = 4(38.4) − 136 = 17.6 million vehicles Shortage = Demand − Supply = 18.6 − 17.6 = 1 million vehicles 34. a. The equilibrium price occurs when demand = supply: −0.15𝑝 + 23 = 3.85𝑝 − 131 4𝑝 = 154 𝑝 = 38.5, or $38,500 per vehicle. b. Since $38,600 is above the equilibrium price, there would be a surplus at that price. To calculate it, compute demand and supply: Demand: 𝑞 = −0.15(38.6) + 23 = 17.21 million vehicles


Solutions Section 1.2 Supply: 𝑞 = 3.85(38.6) − 131 = 17.61 million vehicles Surplus = Supply − Demand = 17.61 − 17.21 = 0.4 million vehicles 35. a. For equilibrium, Demand = Supply: 760 − 1 = 0.00304𝑝 − 1 𝑝 760 = 0.0.00304𝑝 𝑝 Cross-multiply: 760 0.00304𝑝 2 = 760 ⇒ 𝑝 2 = = 250,000 0.00304 So 𝑝 = √250,000 = $500. Thus, the equilibrium price is $500, and the equilibrium demand (or supply) is 760∕500 − 1 = 0.52 billion phones b. Graph:

Technology formulas: Demand: y = 760/x-1 Supply: y = 0.00304x-1 The graphs cross at (500, 0.52) confirming the calclation in part (a). c. Since $400 is below the equilibrium price, there would be a shortage at that price. To calculate it, compute demand and supply: 760 Demand: 𝑞 = − 1 = 0.9 billion phones 400 Supply: 0.00304(400) − 1 = 0.216 billion phones Shortage = Demand − Supply ≈ 0.9 − 0.216 = 0.684 billion phones, or 684 million phones. 36. a. For equilibrium, Demand = Supply: 350 + 0.5 = 0.0056𝑝 + 0.5 𝑝 350 = 0.0056𝑝 𝑝 Cross-multiply: 350 0.0056𝑝 2 = 350 ⇒ 𝑝 2 = = 62,500 0.0056 So 𝑝 = √62,500 = $250. Thus, the equilibrium price is $250, and the equilibrium demand (or supply) is 350∕250 + 0.5 = 1.9 billion phones


Solutions Section 1.2 b. Graph:

Technology formulas: Demand: y = 350/x+0.5 Supply: y = 0.0056x+0.5 The graphs cross at (250, 1.9) confirming the calclation in part (a). c. Since $200 is below the equilibrium price, there would be a shortage at that price. To calculate it, compute demand and supply: 350 Demand: 𝑞 = + 0.5 = 2.25 billion phones 200 Supply: 0.0056(200) + 0.5 = 1.62 billion phones Shortage = Demand − Supply ≈ 2.25 − 1.62 = 0.63 billion phones, or 630 million phones. 37. 𝐶(𝑞) = 2,000 + 100𝑞 2 a. 𝐶(10) = 2,000 + 100(10) 2 = 2,000 + 10,000 = $12,000 b. 𝑁 = 𝐶 − 𝑆, so 𝑁(𝑞) = 𝐶(𝑞) − 𝑆(𝑞) = 2,000 + 100𝑞 2 − 500𝑞

This is the cost of removing 𝑞 lb of PCPs per day after the subsidy is taken into account. c. 𝑁(20) = 2,000 + 100(20) 2 − 500(20) = 2,000 + 40,000 − 10,000 = $32,000

38. 𝐶(𝑞) = 1,000 + 100√𝑞 a. 𝐶(100) = 1,000 + 100√100 = 1,000 + 100(10) = $2,000 b. 𝑁 = 𝐶 − 𝑆, so 𝑁(𝑞) = 𝐶(𝑞) − 𝑆(𝑞) = 1,000 + 100√𝑞 − 200𝑞 This is the cost for dental coverage to the company if it has 𝑞 employees, after the subsidy is taken into account. c. 𝑁(100) = 1,000 + 100√100 − 200(100) = 1000 + 100(10) − 20,000 = −$18,000 The company makes $18,000 from the government for dental coverage if it employs 100 people. 39. The technology formulas are: (A) -0.2*t^2+t+16 (B) 0.2*t^2+t+16 (C) t+16 The following table shows the values predicted by the three models:


𝑡

Solutions Section 1.2 0

2

4

6

7

16

18

22

28

30

(A)

16

17.2

16.8

14.8

13.2

(B)

16

18.8

23.2

29.2

32.8

(C)

16

18

20

22

23

𝑆(𝑡)

As shown in the table, the values predicted by model (B) are much closer to the observed values 𝑆(𝑡) than those predicted by the other models. b. Since 1998 corresponds to 𝑡 = 8, 𝑆(𝑡) = 0.2𝑡 2 + 𝑡 + 16 𝑆(8) = 0.2(8) 2 + 8 + 16 = 36.8 So the spending on corrections in 1998 was predicted to be approximately $37 billion. 40. The technology formulas are: (A) 16+2*t (B) 16+t+0.5*t^2 (C) 16+t-0.5*t^2 The following table shows the values predicted by the three models: 𝑡

0

2

4

6

7

16

18

22

28

30

(A)

16

20

24

28

30

(B)

16

20

28

40

47.5

(C)

16

16

12

4

-1.5

𝑆(𝑡)

As shown in the table, the values predicted by model (A) are much closer to the observed values 𝑆(𝑡) than those predicted by the other models. b. Since 1998 corresponds to 𝑡 = 8, 𝑆(𝑡) = 16 + 2𝑡 𝑆(8) = 16 + 2(8) = 32 So the spending on corrections in 1998 was predicted to be $32 billion. 41. The technology formulas are: (A) 0.005*x+20.75 (B) 0.01*x+20+25/x (C) 0.0005*x^2-0.07*x+23.25 (The stars are optional for some technologies like graphing calculators and the Evaluator and Grapher App) Here is result from the Evaluator and Grapher App: Functions box (Cartesian mode):

Evaluator box (Set to 2 decimal places):

Model (C) fits the data perfectly to two decimal places—more closely than any of the other models.


Solutions Section 1.2 b. Graph of model (C):

0.0005*x^2-0.07*x+23.25

The lowest point on the graph occurs at 𝑥 = 70 with a 𝑦-coordinate of 20.8. Thus, the lowest cost per shirt is $20.80, which the team can obtain by buying 70 shirts. 42. The technology formulas are: (A) 0.05*x+20.75 (B) 0.1*x+20+25/x (C) 0.0008*x^2-0.07*x+23.25 (The stars are optional for some technologies like graphing calculators and the Evaluator and Grapher App) Here is result from the Evaluator and Grapher App: Functions box (Cartesian mode):

Evaluator box (Set to 2 decimal places):

Model (B) fits the data perfectly to two decimal places—more closely than any of the other models. b. Graph of model (B):

0.1*x+20+25/x

The lowest point on the graph occurs at 𝑥 = 16 with a y-coordinate of 23.1625. Thus, the lowest cost per hat is $23.16, which the team can obtain by buying 16 hats. 43. Here are the technology formulas as entered in the online Function Evaluator and Grapher at the Web Site: (A) 0.075*0.83^x (B) 0.24/(x+3) (C) 0.00054*x^2-0.012*x+0.075 (D) -0.006*x+0.065 The following graph shows all four curves together with the plotted points (entered as shown in the margin technology note with Example 5).


Solutions Section 1.2

As shown in the graph, the values predicted by models (A) and (C) are much closer to the observed values than those predicted by the other models. b. Since 2030 corresponds to 𝑡 = 20, Model (A): 𝑐(20) = 0.075 * 0.83 20 ≈ $0.00181 Model (C): 𝑐(20) = 0.00054(20 2) − 0.012(20) + 0.075 ≈ $0.0510 So model (A) gives the lower price: approximately $0.0018 per GB. 44. Here are the technology formulas as entered in the online Function Evaluator and Grapher at the Web Site: (A) -0.038+0.8/(x+7) (B) 0.075*1.21^(-x) (C) 0.00034*(x-14.5)^2 (D) -0.022+0.8/(x+9.32) The following graph shows all three curves together with the plotted points (entered as shown in the margin technology note with Example 5).

As shown in the graph, the values predicted by models (A), (B), and (C) are much closer to the observed values than those predicted by (D). b. Since 2030 corresponds to 𝑡 = 20, 0.8 Model (A): 𝑐(20) = −0.038 + ≈ −$0.084 (unreasonable as prices cannot be negative) 20 + 7 Model (B): 𝑐(20) = 0.075(1.21 −20) ≈ $0.0017 (reasonable given the current trends) Model (C): 𝑐(20) = 0.00034(20 − 14.5) 2 ≈ $0.010 (unreasonable that the price would be the same again in 10 years) 45. A plot of the given points gives a straight line (Option (A)). Options (B) and (C) give curves, so (A) is the best choice.


Solutions Section 1.2 46. Plotting the three data points suggests a concave down curve, suggesting that a linear model may not be the best fit. An exponential model 𝑝(𝑡) = 𝐴𝑏 𝑡 would result in a concave up curve, regardless of whether 𝑏 is larger than 1 or less than 1. This leaves a quadratic model as the only possible choice. In fact, a quadratic can always be found that passes through any three points not on the same straight line with different 𝑥-coordinates. Therefore, a quadratic model would give an exact fit. 47. A plot of the given data suggests a concave-down curve that becomes steeper downward as the price 𝑝 increases, suggesting Model (D). Model (A) would predict increasing demand with increasing price, Model (B) would correspond to a descending curve that becomes less steep as 𝑝 increases (a concave up curve), and Model (C) would give a concave-up parabola. 48. Model (B) is the best choice; Model (A) would predict increasing demand with increasing price, Model (D) would correspond to a concave down parabola, and Model (C), would predict demand that, rather than flattening out as the price increases, would begin to climb again. 49. Apply the formula 𝑟 𝑛𝑡 𝐴(𝑡) = 𝑃 !1 + " 𝑛 with 𝑃 = $1,000, 𝑟 = 6∕100 = 0.06, 𝑛 = 4, 𝑡 = 4. 0.06 4×4 4 ) = 1,000(1.015) 16 ≈ $1,268.99

𝐴(4) = 1,000(1 +

50. Apply the formula 𝑟 𝑛𝑡 𝐴(𝑡) = 𝑃 !1 + " 𝑛 with 𝑃 = $10,000, 𝑟 = 2∕100 = 0.02, 𝑛 = 4, 𝑡 = 5 0.02 4×5 4 ) = 1,000(1.005) 20 ≈ $11,048.96

𝐴(5) = 1,000(1 +

51. Apply the formula 𝑟 𝑛𝑡 𝐴(𝑡) = 𝑃 !1 + " 𝑛 with 𝑃 = 5,000, 𝑟 = 0.15∕100 = 0.0015, and 𝑛 = 12. We get the model 𝐴(𝑡) = 5,000(1 + 0.0015∕12) 12𝑡 In June 2028 (𝑡 = 7), the deposit would be worth 5,000(1 + 0.0015∕12) 12(7) ≈ $5,053. 52. Apply the formula 𝑟 𝑛𝑡 𝐴(𝑡) = 𝑃 !1 + " 𝑛 with 𝑃 = 4,000, 𝑟 = 0.0120, and 𝑛 = 365. We get the model 𝐴(𝑡) = 4,000(1 + 0.0120∕365) 365𝑡 In March 2029 (𝑡 = 8), the deposit would be worth 4,000(1 + 0.0120∕365) 365(8) ≈ $4,403.

53. 𝑃 (0) = 200, 𝑃 (1) = 230, 𝑃 (2) = 260, ... and so on. Thus, the population is increasing by 30 per year. 54. 𝐵(0) = 5000, 𝐵(1) = 4800, 𝐵(2) = 4600, ... and so on. Thus, the balance is decreasing by $200 per day.


Solutions Section 1.2 55. Curve fitting. The model is based on fitting a curve to a given set of observed data. 56. Analytical. The model is obtained by analyzing the situation being modeled.

57. The given model is 𝑐(𝑡) = 4 − 0.2𝑡. This tells us that 𝑐 is $4 at time 𝑡 = 0 (January) and is decreasing by $0.20 per month. So, the cost of downloading a movie was $4 in January and is decreasing by 20¢ per month. 58. The given model is 𝑐(𝑡) = 4 − 0.2𝑡 and therefore passes through the points (𝑡, 𝑐) = (0, 4) and (1, 3.8). So, the cost of downloading a movie was $4 in January and $3.80 in February. 59. In a linear cost function, the variable cost is 𝑥 times the marginal cost.

60. In a linear cost function, the marginal cost is the additional (or incremental) cost per item. 61. Yes, as long as the supply is going up at a faster rate, as illustrated by the following graph:

62. There would be a shortage at any given price. Therefore, consumers would be willing to pay more for a scarce commodity and sellers would naturally oblige by charging more, resulting in an upward spiral of prices for the commodity.

63. Extrapolate both models and choose the one that gives the most reasonable predictions.

64. No; as long as 𝑎 is negative, the value of 𝑠(𝑡) for large 𝑡 will be negative, making the model unreasonable for large value of 𝑡.

65. The value of 𝑓 − 𝑔 at 𝑥 is 𝑓(𝑥) − 𝑔(𝑥). Since 𝑓(𝑥) ≥ 𝑔(𝑥) for every 𝑥, it follows that 𝑓(𝑥) − 𝑔(𝑥) ≥ 0 for every 𝑥. 66. The value of

𝑓 𝑓(𝑥) 𝑓(𝑥) at 𝑥 is . Since 𝑓(𝑥) > 𝑔(𝑥) > 0 for every 𝑥, it follows that > 1 for every 𝑥. 𝑔 𝑔(𝑥) 𝑔(𝑥)

𝑓 𝑓(𝑥) 𝑓 at 𝑥 are ratios , it follows that the units of measurement of are units of 𝑓 𝑔 𝑔(𝑥) 𝑔 per unit of 𝑔; that is, books per person. 67. Since the values of

68. Write 𝑓(𝑥) = 𝑚𝑥 + 𝑏 and 𝑔(𝑥) = 𝑛𝑥 + 𝑐. Then 𝑓(𝑥) − 𝑔(𝑥) = 𝑚𝑥 + 𝑏 − (𝑛𝑥 + 𝑐) = (𝑚 − 𝑛)𝑥 + (𝑏 − 𝑐), also a linear function.


Solutions Section 1.3 Section 1.3 1.

𝑥 𝑦

−1 5

0 8

1

We calculate the slope 𝑚 first. The first two points shown give changes in 𝑥 and 𝑦 of Δ𝑥 = 0 − (−1) = 1 Δ𝑦 = 8 − 5 = 3 This gives a slope of Δ𝑦 3 𝑚= = = 3. Δ𝑥 1 Now look at the second and third points: The change in 𝑥 is again Δ𝑥 = 1 − 0 = 1 and so Δ𝑦 must be given by the formula Δ𝑦 = 𝑚Δ𝑥 Δ𝑦 = 3(1) = 3 This means that the missing value of 𝑦 is 8 + Δ𝑦 = 8 + 3 = 11. 2.

𝑥 𝑦

−1

0

−1

−3

2

3

1

We calculate the slope 𝑚 first. The first two points shown give changes in 𝑥 and 𝑦 of Δ𝑥 = 0 − (−1) = 1 Δ𝑦 = −3 − (−1) = −2 This gives a slope of Δ𝑦 −2 𝑚= = = −2. Δ𝑥 1 Now look at the second and third points: The change in 𝑥 is again Δ𝑥 = 1 − 0 = 1 and so Δ𝑦 must be given by the formula Δ𝑦 = 𝑚Δ𝑥 Δ𝑦 = −2(1) = −2 This means that the missing value of 𝑦 is −3 + Δ𝑦 = −3 + (−2) = −5. 3.

𝑥 𝑦

−1

−2

5

We calculate the slope 𝑚 first. The first two points shown give changes in 𝑥 and 𝑦 of Δ𝑥 = 3 − 2 = 1 Δ𝑦 = −2 − (−1) = −1 This gives a slope of Δ𝑦 −1 𝑚= = = −1. Δ𝑥 1 Now look at the second and third points: The change in 𝑥 is Δ𝑥 = 5 − 3 = 2 and so Δ𝑦 must be given by the formula Δ𝑦 = 𝑚Δ𝑥 Δ𝑦 = (−1)(2) = −2 This means that the missing value of 𝑦 is −2 + Δ𝑦 = −2 + (−2) = −4.


4.

𝑥

2

4

𝑦

−1

−2

𝑥

−2

0

5

Solutions Section 1.3

We calculate the slope 𝑚 first. The first two points shown give changes in 𝑥 and 𝑦 of Δ𝑥 = 4 − 2 = 2 Δ𝑦 = −2 − (−1) = −1 This gives a slope of Δ𝑦 −1 1 𝑚= = =− . Δ𝑥 2 2 Now look at the second and third points: The change in 𝑥 is Δ𝑥 = 5 − 4 = 1 and so Δ𝑦 must be given by the formula Δ𝑦 = 𝑚Δ𝑥 1 1 Δ𝑦 = !− "(1) = − 2 2 This means that the missing value of 𝑦 is 1 5 −2 + Δ𝑦 = −2 + !− " = − or −2.5. 2 2 5.

𝑦

4

2

10

We calculate the slope 𝑚 first. The first and third points shown give changes in 𝑥 and 𝑦 of Δ𝑥 = 2 − (−2) = 4 Δ𝑦 = 10 − 4 = 6 This gives a slope of Δ𝑦 6 3 𝑚= = = . Δ𝑥 4 2 Now look at the first and second points: The change in 𝑥 is Δ𝑥 = 0 − (−2) = 2 and so Δ𝑦 must be given by the formula Δ𝑦 = 𝑚Δ𝑥 3 Δ𝑦 = !− "(2) = 3 2 This means that the missing value of 𝑦 is 4 + Δ𝑦 = 4 + 3 = 7. 6.

𝑥 𝑦

0

−1

3

6

−5

We calculate the slope 𝑚 first. The first and third points shown give changes in 𝑥 and 𝑦 of Δ𝑥 = 6 − 0 = 6 Δ𝑦 = −5 − (−1) = −4 This gives a slope of Δ𝑦 −4 2 𝑚= = =− . Δ𝑥 6 3 Now look at the first and second points: The change in 𝑥 is Δ𝑥 = 3 − 0 = 3 and so Δ𝑦 must be given by the formula Δ𝑦 = 𝑚Δ𝑥 2 Δ𝑦 = !− "(3) = −2 3


Solutions Section 1.3 This means that the missing value of 𝑦 is −1 + Δ𝑦 = −1 + (−2) = −3 7. From the table, 𝑏 = 𝑓(0) = −2. The slope (using the first two points) is 𝑦 − 𝑦1 −2 − (−1) −1 1 𝑚= 2 = = =− . 𝑥2 − 𝑥1 0 − (−2) 2 2 Thus, the linear equation is 1 𝑥 𝑓(𝑥) = 𝑚𝑥 + 𝑏 = − 𝑥 − 2, or 𝑓(𝑥) = − − 2. 2 2 8. From the table, 𝑏 = 𝑓(0) = 3. The slope (using the first two points) is 𝑦 − 𝑦1 2−1 1 𝑚= 2 = = 𝑥2 − 𝑥1 (−3) − (−6) 3 Thus, the linear equation is 1 𝑥 𝑓(𝑥) = 𝑚𝑥 + 𝑏 = 𝑥 + 3, or 𝑓(𝑥) = + 3. 3 3

9. The slope (using the first two points) is 𝑦 − 𝑦1 −2 − (−1) −1 𝑚= 2 = = = −1. 𝑥2 − 𝑥1 −3 − (−4) 1 To obtain 𝑓(0) = 𝑏, use the formula for 𝑏 : 𝑓(0) = 𝑏 = 𝑦1 − 𝑚𝑥1 = −1 − (−1)(−4) = −5 This gives 𝑓(𝑥) = 𝑚𝑥 + 𝑏 = −𝑥 − 5. 10. The slope (using the first two points) is 𝑦 − 𝑦1 6−4 2 𝑚= 2 = = =2 𝑥2 − 𝑥1 2 − 1 1 To obtain 𝑓(0) = 𝑏, use the formula for 𝑏 : 𝑓(0) = 𝑏 = 𝑦1 − 𝑚𝑥1 = 4 − (2)(1) = 2 This gives 𝑓(𝑥) = 𝑚𝑥 + 𝑏 = 2𝑥 + 2.

Using the point (𝑥1 , 𝑦1 ) = (−4, −1)

Using the point (𝑥1 , 𝑦1 ) = (1, 4)

11. In the table, 𝑥 increases in steps of 1 and 𝑓 increases in steps of 4, showing that 𝑓 is linear with slope Δ𝑦 4 𝑚= = =4 Δ𝑥 1 and intercept 𝑏 = 𝑓(0) = 6 giving 𝑓(𝑥) = 𝑚𝑥 + 𝑏 = 4𝑥 + 6. The function 𝑔 does not increase in equal steps, so 𝑔 is not linear.

12. In the table, 𝑥 increases in steps of 10 and 𝑔 increases in steps of 5, showing that 𝑔 is linear with slope Δ𝑦 5 1 𝑚= = = Δ𝑥 10 2 and intercept 𝑏 = 𝑔(0) = −4 giving 1 𝑔(𝑥) = 𝑚𝑥 + 𝑏 = 𝑥 − 4. 2


Solutions Section 1.3 The function 𝑓 does not increase in equal steps, so 𝑓 is not linear.

13. In the first three points listed in the table, 𝑥 increases in steps of 3, but 𝑓 does not increase in equal steps, whereas 𝑔 increases in steps of 6. Thus, based on the first three points, only 𝑔 could possibly be linear, with slope Δ𝑦 6 𝑚= = =2 Δ𝑥 3 and intercept 𝑏 = 𝑔(0) = −1 giving 𝑔(𝑥) = 𝑚𝑥 + 𝑏 = 2𝑥 − 1. We can now check that the remaining points in the table fit the formula 𝑔(𝑥) = 2𝑥 − 1, showing that 𝑔 is indeed linear.

14. In the first and last pairs of points listed in the table, 𝑥 increases in steps of 3, but 𝑓 does not increase in equal steps, whereas 𝑔 increases in steps of 9. Thus, based on those points, only 𝑔 could possibly be linear, with slope Δ𝑦 9 𝑚= = =3 Δ𝑥 3 and intercept 𝑏 = 𝑔(0) = −1 giving 𝑔(𝑥) = 𝑚𝑥 + 𝑏 = 3𝑥 − 1. We can now check that the remaining points in the table fit the formula 𝑔(𝑥) = 3𝑥 − 1, showing that 𝑔 is indeed linear. 15. Slope = coefficient of 𝑥 = − 16. Slope = coefficient of 𝑥 = 17. Slope = coefficient of 𝑥 =

2 3 1 6

18. Write the equation as 𝑦 = − Slope = coefficient of 𝑥 = −

2 3

3 2

2𝑥 1 + 3 3

19. If we solve for 𝑥 we find that the given equation represents the vertical line 𝑥 = −1∕3, and so its slope is infinite (undefined). 20. 8𝑥 − 2𝑦 = 1. Solving for 𝑦 : 2𝑦 = 8𝑥 − 1 1 𝑦 = 4𝑥 − 2 Slope = coefficient of 𝑥 = 4 21. 3𝑦 + 1 = 0. Solving for 𝑦 : 3𝑦 = −1 1 𝑦=− 3


Slope = coefficient of 𝑥 = 0

Solutions Section 1.3

22. If we solve for 𝑥 we find that the given equation represents the vertical line 𝑥 = −3∕2, and so its slope is infinite (undefined). 23. 4𝑥 + 3𝑦 = 7. Solve for 𝑦 : 3𝑦 = −4𝑥 + 7 4 7 𝑦=− 𝑥+ 3 3 4 Slope = coefficient of 𝑥 = − 3 24. 2𝑦 + 3 = 0. Solve for 𝑦 : 2𝑦 = −3 3 𝑦=− 2 Slope = coefficient of 𝑥 = 0 25. 𝑦 = 2𝑥 − 1 𝑦-intercept = −1, slope = 2

26. 𝑦 = 𝑥 − 3 𝑦-intercept = −3, slope = 1

27. 𝑦-intercept = 2, slope = −

1 28. 𝑦 = − 𝑥 + 3 2

𝑦-intercept = 3, slope = −

29. 𝑦 +

2 3

1 2

1 1 𝑥 = −4. Solve for 𝑦 to obtain 𝑦 = − 𝑥 − 4 4 4


y-intercept = −4, slope = −

1 4

Solutions Section 1.3

1 1 𝑥 = −2. Solve for 𝑦 to obtain 𝑦 = 𝑥 − 2 4 4 1 𝑦-intercept = −2, slope = 4 30. 𝑦 −

31. 7𝑥 − 2𝑦 = 7. Solve for 𝑦 : 7 7 −2𝑦 = −7𝑥 + 7, so 𝑦 = 𝑥 − 2 2 7 7 𝑦-intercept = − = −3.5, slope = = 3.5 2 2

32. 2𝑥 − 3𝑦 = 1. Solve for 𝑦 : 2 1 −3𝑦 = −2𝑥 + 1, so 𝑦 = 𝑥 − 3 3 1 2 𝑦-intercept = − , slope = 3 3

33. 3𝑥 = 8. Solve for 𝑥 to obtain 𝑥 = The graph is a vertical line:

8 . 3


Solutions Section 1.3 7 34. 2𝑥 = −7. Solve for 𝑥 to obtain 𝑥 = − = −3.5. 2 The graph is a vertical line:

35. 6𝑦 = 9. Solve for 𝑦 to obtain 𝑦 =

9 3 = = 1.5 6 2

36. 3𝑦 = 4. Solve for 𝑦 to obtain 𝑦 =

4 3

𝑦-intercept =

3 = 1.5, slope = 0. The graph is a horizontal line: 2

4 𝑦-intercept = , slope = 0. The graph is a horizontal line: 3

37. 2𝑥 = 3𝑦. Solve for 𝑦 to obtain 𝑦 = 𝑦-intercept = 0, slope =

2 3

2 𝑥 3

3 38. 3𝑥 = −2𝑦. Solve for 𝑦 to obtain 𝑦 = − 𝑥 2 3 𝑦-intercept = 0, slope = − 2

39. (0, 0) and (1, 2) 𝑦 − 𝑦1 2−0 𝑚= 2 = =2 𝑥2 − 𝑥1 1 − 0

40. (0, 0) and (−1, 2) 𝑦 − 𝑦1 2−0 𝑚= 2 = = −2 𝑥2 − 𝑥1 −1 − 0


41. (−1, −2) and (0, 0) 𝑦 − 𝑦1 0 − (−2) 𝑚= 2 = =2 𝑥2 − 𝑥1 0 − (−1)

42. (2, 1) and (0, 0) 𝑦 − 𝑦1 0−1 1 𝑚= 2 = = 𝑥2 − 𝑥1 0 − 2 2

Solutions Section 1.3

43. (4, 3) and (5, 1) 𝑦 − 𝑦1 1 − 3 −2 𝑚= 2 = = = −2 𝑥2 − 𝑥1 5 − 4 1

44. (4, 3) and (4, 1) 𝑦 − 𝑦1 1−3 𝑚= 2 Undefined = 𝑥2 − 𝑥1 4 − 4

45. (1, −1) and (1, −2) 𝑦 − 𝑦1 −2 − (−1) 𝑚= 2 Undefined = 𝑥2 − 𝑥1 1−1

46. (−2, 2) and (−1, −1) 𝑦 − 𝑦1 −1 − 2 −3 𝑚= 2 = = = −3 𝑥2 − 𝑥1 −1 − (−2) 1

47. (2, 3.5) and (4, 6.5) 𝑦 − 𝑦1 6.5 − 3.5 3 𝑚= 2 = = = 1.5 𝑥2 − 𝑥1 4−2 2

48. (10, −3.5) and (0, −1.5) 𝑦 − 𝑦1 −1.5 − (−3.5) 2 𝑚= 2 = = = −0.2 𝑥2 − 𝑥1 0 − 10 −10

49. (300, 20.2) and (400, 11.2) 𝑦 − 𝑦1 11.2 − 20.2 −9 𝑚= 2 = = = −0.09 𝑥2 − 𝑥1 400 − 300 100

50. (1, −20.2) and (2, 3.2) 𝑦 − 𝑦1 3.2 − (−20.2) 23.4 𝑚= 2 = = = 23.4 𝑥2 − 𝑥1 2−1 1

1 3 51. (0, 1) and !− , " 2 4 𝑦2 − 𝑦1 3∕4 − 1 −1∕4 2 1 𝑚= = = = = 𝑥2 − 𝑥1 −1∕2 − 0 −1∕2 4 2

1 1 3 52. ! , 1" and !− , " 2 2 4 𝑦2 − 𝑦1 3∕4 − 1 −1∕4 1 𝑚= = = = 𝑥2 − 𝑥1 −1∕2 − 1∕2 4 −1

53. (𝑎, 𝑏) and (𝑐, 𝑑) (𝑎 ≠ 𝑐) 𝑦 − 𝑦1 𝑑−𝑏 𝑚= 2 = 𝑥2 − 𝑥1 𝑐 − 𝑎

54. (𝑎, 𝑏) and (𝑐, 𝑏) (𝑎 ≠ 𝑐) 𝑦 − 𝑦1 𝑏−𝑏 𝑚= 2 = =0 𝑥2 − 𝑥1 𝑐 − 𝑎

55. (𝑎, 𝑏) and (𝑎, 𝑑) (𝑏 ≠ 𝑑) 𝑦 − 𝑦1 𝑑−𝑏 𝑚= 2 \ \ Undefined = 𝑥2 − 𝑥1 𝑎 − 𝑎

56. (𝑎, 𝑏) and (−𝑎, −𝑏) (𝑎 ≠ 0) 𝑦 − 𝑦1 −𝑏 − 𝑏 −2𝑏 𝑏 𝑚= 2 = = = 𝑥2 − 𝑥1 −𝑎 − 𝑎 −2𝑎 𝑎

57. (−𝑎, 𝑏) and (𝑎, −𝑏) (𝑎 ≠ 0) 𝑦 − 𝑦1 𝑏 −𝑏 − 𝑏 −2𝑏 𝑚= 2 = = =− 𝑥2 − 𝑥1 𝑎 − (−𝑎) 2𝑎 𝑎

58. (𝑎, 𝑏) and (𝑏, 𝑎) (𝑎 ≠ 𝑏) 𝑦 − 𝑦1 𝑎−𝑏 𝑚= 2 = = −1 𝑥2 − 𝑥1 𝑏 − 𝑎


59. a. 𝑚 =

Δ𝑦 1 = =1 Δ𝑥 1

Solutions Section 1.3

60. a. 𝑚 =

Δ𝑦 −1 = = −1 Δ𝑥 1

b. 𝑚 =

Δ𝑦 1 = Δ𝑥 2

b. 𝑚 =

Δ𝑦 2 = =2 Δ𝑥 1

c. 𝑚 =

Δ𝑦 0 = =0 Δ𝑥 1

c. 𝑚 =

Δ𝑦 0 = =0 Δ𝑥 1

d. 𝑚 =

Δ𝑦 3 = =3 Δ𝑥 1

d. 𝑚 =

Δ𝑦 1 = =1 Δ𝑥 1

e. 𝑚 =

Δ𝑦 −2 1 = =− Δ𝑥 4 2

e. 𝑚 =

f. 𝑚 =

Δ𝑦 −1 1 = =− Δ𝑥 3 3 Δ𝑦 −1 = = −1 Δ𝑥 1

f. Vertical line; undefined slope

g. 𝑚 =

Δ𝑦 3 = =3 Δ𝑥 1

g. Vertical line; undefined slope

h. 𝑚 =

Δ𝑦 −1 1 = =− Δ𝑥 4 4

h. 𝑚 =

Δ𝑦 −1 1 = =− Δ𝑥 3 3

i. 𝑚 =

Δ𝑦 −2 = = −2 Δ𝑥 1

i. 𝑚 =

Δ𝑦 −2 = = −2 Δ𝑥 1


61. Through (1, 3) with slope 3 Point: (1, 3) Slope: 𝑚 = 3 Intercept: 𝑏 = 𝑦1 − 𝑚𝑥1 = 3 − 3(1) = 0 Thus, the equation is 𝑦 = 𝑚𝑥 + 𝑏 = 3𝑥 + 0, or 𝑦 = 3𝑥 Solutions Section 1.3

62. Through (2, 1) with slope 2 Point: (2, 1) Slope: 𝑚 = 2 Intercept: 𝑏 = 𝑦1 − 𝑚𝑥1 = 1 − 2(2) = −3 Thus, the equation is 𝑦 = 𝑚𝑥 + 𝑏 = 2𝑥 − 3 3 1 63. Through !1, − " with slope 4 4 3 1 Point: !1, − " Slope: 𝑚 = 4 4

Thus, the equation is 𝑦 = 𝑚𝑥 + 𝑏 = 1 1 64. Through !0, − " with slope 3 3 1 1 Point: !0, − " Slope: 𝑚 = 3 3

3 1 Intercept: 𝑏 = 𝑦1 − 𝑚𝑥1 = − − (1) = −1 4 4

1 𝑥−1 4

1 1 1 Intercept: 𝑏 = 𝑦1 − 𝑚𝑥1 = − − (0) = − 3 3 3 1 1 Thus, the equation is 𝑦 = 𝑚𝑥 + 𝑏 = 𝑥 − 3 3 65. Through (20, −3.5) and increasing at a rate of 10 units of 𝑦 per unit of 𝑥 Δ𝑦 10 Point: (20, −3.5) Slope: 𝑚 = = = 10 Δ𝑥 1 Intercept: 𝑏 = 𝑦1 − 𝑚𝑥1 = −3.5 − (10)(20) = −3.5 − 200 = −203.5 Thus, the equation is 𝑦 = 𝑚𝑥 + 𝑏 = 10𝑥 − 203.5

66. Through (3.5, −10) and increasing at a rate of 1 unit of 𝑦 per 2 units of 𝑥 Δ𝑦 1 Point: (3.5, −10) Slope: 𝑚 = = = 0.5 Δ𝑥 2 Intercept: 𝑏 = 𝑦1 − 𝑚𝑥1 = −10 − (0.5)(3.5) = −11.75 Thus, the equation is 𝑦 = 𝑚𝑥 + 𝑏 = 0.5𝑥 − 11.75 67. Through (2, −4) and (1, 1)

𝑦2 − 𝑦1 5 = = −5 𝑥2 − 𝑥1 −1 Intercept: 𝑏 = 𝑦1 − 𝑚𝑥1 = −4 − (−5)(2) = 6 Thus, the equation is 𝑦 = 𝑚𝑥 + 𝑏 = −5𝑥 + 6 Point:(2, −4)

Slope: 𝑚 =

68. Through (1, −4) and (−1, −1) 𝑦 − 𝑦1 3 Point: (1, −4) Slope: 𝑚 = 2 = = −1.5 𝑥2 − 𝑥1 −2 Intercept: 𝑏 = 𝑦1 − 𝑚𝑥1 = −4 − (−1.5)(1) = −2.5 Thus, the equation is 𝑦 = 𝑚𝑥 + 𝑏 = −1.5𝑥 − 2.5

69. Through (1, −0.75) and (0.5, 0.75) 𝑦 − 𝑦1 0.75 − (−0.75) 1.5 Point: (1, −0.75) Slope: 𝑚 = 2 = = = −3 𝑥2 − 𝑥1 0.5 − 1 −0.5 Intercept: 𝑏 = 𝑦1 − 𝑚𝑥1 = −0.75 − (−3)(1) = −0.75 + 3 = 2.25 Thus, the equation is 𝑦 = 𝑚𝑥 + 𝑏 = −3𝑥 + 2.25


70. Through (0.5, −0.75) and (1, −3.75) 𝑦 − 𝑦1 −3.75 − (−0.75) −3 Point: (0.5, −0.75) Slope: 𝑚 = 2 = = = −6 𝑥2 − 𝑥1 1 − 0.5 0.5 Intercept: 𝑏 = 𝑦1 − 𝑚𝑥1 = −0.75 − (−6)(0.5) = −0.75 + 3 = 2.25 Thus, the equation is 𝑦 = 𝑚𝑥 + 𝑏 = −6𝑥 + 2.25 Solutions Section 1.3

71. Through (6, 6) and parallel to the line 𝑥 + 𝑦 = 4 Point: (6, 6) Slope: Same as slope of 𝑥 + 𝑦 = 4. To find the slope, solve for 𝑦, getting 𝑦 = −𝑥 + 4. Thus, 𝑚 = −1. Intercept: 𝑏 = 𝑦1 − 𝑚𝑥1 = 6 − (−1)(6) = 6 + 6 = 12 Thus, the equation is 𝑦 = 𝑚𝑥 + 𝑏 = −𝑥 + 12 72. Through (1∕3, −1) and parallel to the line 3𝑥 − 4𝑦 = 8 Point: (1∕3, −1)

Slope: Same as slope of 3𝑥 − 4𝑦 = 8. To find the slope, solve for 𝑦, getting 𝑦 = 3 1 1 5 Intercept: 𝑏 = 𝑦1 − 𝑚𝑥1 = (−1) − ! "! " = −1 − = − 4 3 4 4 3 5 Thus, the equation is 𝑦 = 𝑚𝑥 + 𝑏 = 𝑥 − 4 4

3 3 𝑥 − 2. Thus, 𝑚 = . 4 4

73. Through (0.5, 5) and parallel to the line 4𝑥 − 2𝑦 = 11 Point: (0.5, 5)

Slope: Same as slope of 4𝑥 − 2𝑦 = 11. To find the slope, solve for 𝑦, getting 𝑦 = 2𝑥 − Intercept: 𝑏 = 𝑦1 − 𝑚𝑥1 = 5 − (2)(0.5) = 5 − 1 = 4 Thus, the equation is 𝑦 = 𝑚𝑥 + 𝑏 = 2𝑥 + 4

74. Through (1/3, 0) and parallel to the line 6𝑥 − 2𝑦 = 11 Point: (1∕3, 0)

Slope: Same as slope of 6𝑥 − 2𝑦 = 11. To find the slope, solve for 𝑦, getting 𝑦 = 3𝑥 − 1 Intercept: 𝑏 = 𝑦1 − 𝑚𝑥1 = 0 − (3)! " = −1 3 Thus, the equation is 𝑦 = 𝑚𝑥 + 𝑏 = 3𝑥 − 1 75. Through (0, 0) and (𝑝, 𝑞)

𝑦2 − 𝑦1 𝑞−0 𝑞 = = 𝑥2 − 𝑥1 𝑝 − 0 𝑝 𝑞 Intercept: 𝑏 = 𝑦1 − 𝑚𝑥1 = 0 − (0) = 0 𝑝 𝑞 Thus, the equation is 𝑦 = 𝑚𝑥 + 𝑏 = 𝑥 𝑝 Point: (0, 0)

Slope: 𝑚 =

76. Through (𝑝, 𝑞) parallel to 𝑦 = 𝑟𝑥 + 𝑠 Point: (𝑝, 𝑞) Slope: Since the line has the same slope as 𝑦 = 𝑟𝑥 + 𝑠, 𝑚 = 𝑟 Intercept: 𝑏 = 𝑦1 − 𝑚𝑥1 = 𝑞 − 𝑟𝑝 Thus, the equation is 𝑦 = 𝑚𝑥 + 𝑏 = 𝑟𝑥 + 𝑞 − 𝑟𝑝 or 𝑦 = 𝑟(𝑥 − 𝑝) + 𝑞 77. Through (𝑝, 𝑞) and (𝑟, 𝑞) (𝑝 ≠ 𝑟) 𝑦 − 𝑦1 𝑞−𝑞 Point: (𝑝, 𝑞) Slope: 𝑚 = 2 = =0 𝑥2 − 𝑥1 𝑟 − 𝑝

11 . Thus, 𝑚 = 2. 2

11 . Thus, 𝑚 = 2. 2


Solutions Section 1.3 Intercept: 𝑏 = 𝑦1 − 𝑚𝑥1 = 𝑞 − (0)𝑝 = 𝑞 Thus, the equation is 𝑦 = 𝑚𝑥 + 𝑏; that is, 𝑦 = 𝑞 78. Through (𝑝, 𝑞) and (−𝑝, −𝑞) (𝑝 ≠ 0) 𝑦 − 𝑦1 −𝑞 − 𝑞 −2𝑞 𝑞 Point: (𝑝, 𝑞) Slope: 𝑚 = 2 = = = 𝑥2 − 𝑥1 −𝑝 − 𝑝 −2𝑝 𝑝 𝑞 Intercept: 𝑏 = 𝑦1 − 𝑚𝑥1 = 𝑞 − 𝑝 = 𝑞 − 𝑞 = 0 𝑝 𝑞 Thus, the equation is 𝑦 = 𝑚𝑥 + 𝑏 = 𝑥 𝑝

79. Through (−𝑝, 𝑞) and (𝑝, −𝑞) (𝑝 ≠ 0) 𝑦 − 𝑦1 𝑞 −𝑞 − 𝑞 −2𝑞 Point: (−𝑝, 𝑞) Slope: 𝑚 = 2 = = =− 𝑥2 − 𝑥1 𝑝 − (−𝑝) 2𝑝 𝑝 𝑞 Intercept: 𝑏 = 𝑦1 − 𝑚𝑥1 = 𝑞 − !− "(−𝑝) = 𝑞 − 𝑞 = 0 𝑝 𝑞 Thus, the equation is 𝑦 = 𝑚𝑥 + 𝑏 = − 𝑥 𝑝

80. Through (𝑝, 𝑞) and (𝑟, 𝑠) (𝑝 ≠ 𝑟) 𝑦 − 𝑦1 𝑠−𝑞 Point: (𝑝, 𝑞) Slope: 𝑚 = 2 = 𝑥2 − 𝑥1 𝑟 − 𝑝 𝑠−𝑞 Intercept: 𝑏 = 𝑦1 − 𝑚𝑥1 = 𝑞 − ! "𝑝 𝑟−𝑝 𝑠−𝑞 𝑠−𝑞 𝑠−𝑞 Thus, the equation is 𝑦 = 𝑚𝑥 + 𝑏 = ! "𝑥 + 𝑞 − ! "𝑝, or 𝑦 = ! "(𝑥 − 𝑝) + 𝑞 𝑟−𝑝 𝑟−𝑝 𝑟−𝑝 81. We are given two points on the graph of the linear cost function: (100, 10,500) and (120, 11,000) (𝑥 is the number of bicycles, and the second coordinate is the cost 𝐶). Marginal cost: 𝐶 − 𝐶1 11,000 − 10,500 500 𝑚= 2 = $25 per bicycle = = 𝑥2 − 𝑥1 120 − 100 20 Fixed cost: 𝑏 = 𝐶1 − 𝑚𝑥1 = 10,500 − (25)(100) = 10,500 − 2,500 = $8,000 82. We are given two points on the graph of the linear cost function: (1,000, 6,000) and (1,500, 8,500) (𝑥 is the number of cases, and the second coordinate is the cost 𝐶). Marginal cost: 𝐶 − 𝐶1 8,500 − 6,000 2,500 𝑚= 2 = $5 per case. = = 𝑥2 − 𝑥1 1,500 − 1,000 500 Fixed cost: 𝑏 = 𝐶1 − 𝑚𝑥1 = 6,000 − (5)(1,000) = $1,000 83. We are given two points on the graph of the linear cost function: (10, 5,920) and (20, 11,820) (𝑥 is the number of iPhones made in an hour, and the second coordinate is the cost 𝐶). Marginal cost: 𝐶 − 𝐶1 11,820 − 5,920 5,900 𝑚= 2 = $590 per iPhone = = 𝑥2 − 𝑥1 20 − 10 10 Fixed cost: 𝑏 = 𝐶1 − 𝑚𝑥1 = 5,920 − (590)(10) = $20 Thus, the cost equation is 𝐶 = 𝑚𝑥 + 𝑏 = 590𝑥 + 20. The cost to manufacture each additional iPhone is the marginal cost: $590.


Solutions Section 1.3 The cost to manufacture 40 iPhones in an hour is obtained by setting 𝑥 = 40 in the cost equation: 𝐶(40) = 590(40) + 20 = $23,620 84. We are given two points on the graph of the linear cost function: (8, 4,110) and (16, 8,190) (𝑥 is the number of consoles made in an hour, and the second coordinate is the cost 𝐶). Marginal cost: 𝐶 − 𝐶1 8,190 − 4,110 4,080 𝑚= 2 = $510 per unit = = 𝑥2 − 𝑥1 16 − 8 8 Fixed cost: 𝑏 = 𝐶1 − 𝑚𝑥1 = 4,110 − (510)(8) = $30 Thus, the cost equation is 𝐶 = 𝑚𝑥 + 𝑏 = 510𝑥 + 30. The cost to manufacture each additional Kinect is the marginal cost: $510. The cost to manufacture 30 Kinects in an hour is obtained by setting 𝑥 = 30 in the cost equation: 85. A linear demand function has the form 𝑞 = 𝑚𝑝 + 𝑏 (𝑝 is the price, and 𝑞 is the demand). We are given two points on its graph: (1, 1,960) and (5, 1,800). Slope: 𝑞 − 𝑞1 1,800 − 1,960 −160 𝑚= 2 = = = −40 𝑝2 − 𝑝1 5−1 4 Intercept: 𝑏 = 𝑞1 − 𝑚𝑝1 = 1,960 − (−40)(1) = 1,960 + 40 = 2,000 Thus, the demand equation is 𝑞 = 𝑚𝑝 + 𝑏 = −40𝑝 + 2,000 86. A linear demand function has the form 𝑞 = 𝑚𝑝 + 𝑏. (𝑝 is the price, and 𝑞 is the demand). We are given two points on its graph: (5, 3,950) and (10, 3,700). Slope: 𝑞 − 𝑞1 3,700 − 3,950 −250 𝑚= 2 = = = −50 𝑝2 − 𝑝1 10 − 5 5 Intercept: 𝑏 = 𝑞1 − 𝑚𝑝1 = 3,950 − (−50)5 = 4,200 Thus, the demand equation is 𝑞 = 𝑚𝑝 + 𝑏 = −50𝑝 + 4,200

87. a. A linear demand function has the form 𝑞 = 𝑚𝑝 + 𝑏. (𝑝 is the price, and 𝑞 is the demand). We are given two points on its graph: 2020: (𝑝, 𝑞) = (650, 220) 2024: (𝑝, 𝑞) = (450, 700) 𝑞 − 𝑞1 700 − 220 480 Slope: 𝑚 = 2 = = = −2.4 𝑝2 − 𝑝1 450 − 650 −200 Intercept: 𝑏 = 𝑞1 − 𝑚𝑝1 = 220 − (−2.4)650 = 1,780 Thus, the demand equation is 𝑞 = 𝑚𝑝 + 𝑏 = −2.4𝑝 + 1,780. If 𝑝 = $400, then 𝑞 = −2.4(400) + 1,780 = 820 million phones. b. Since the slope is −2.4 million phones per unit increase in price, we interpret the slope as follows: For every $1 increase in price, sales of smartphones decrease by 2.4 million units. 88. a. A linear demand function has the form 𝑞 = 𝑚𝑝 + 𝑏. (𝑝 is the price, and 𝑞 is the demand). We are given two points on its graph: 2013: (335, 1,010) 2017: (265, 1,710)


Solutions Section 1.3 𝑞 − 𝑞1 1,710 − 1,010 700 Slope: 𝑚 = 2 = = = −10 𝑝2 − 𝑝1 265 − 335 −70 Intercept: 𝑏 = 𝑞1 − 𝑚𝑝1 = 1,010 − (−10)335 = 4,360 Thus, the demand equation is 𝑞 = 𝑚𝑝 + 𝑏 = −10𝑝 + 4,360. If 𝑝 = $385, then 𝑞 = −10(385) + 4,360 = 510 million phones. b. Since the slope is −10 million phones per unit increase in price, we interpret the slope as follows: For every $1 increase in price, sales of smartphones decrease by 10 million units.

89. a. A linear demand function has the form 𝑞 = 𝑚𝑝 + 𝑏. (𝑝 is the price, and 𝑞 is the demand). We are given two points on its graph: (3, 28,000) and (5, 19,000). 𝑞 − 𝑞1 19,000 − 28,000 −9,000 Slope: 𝑚 = 2 = = = −4,500 𝑝2 − 𝑝1 5−3 2 Intercept: 𝑏 = 𝑞1 − 𝑚𝑝1 = 28,000 − (−4,500)3 = 28,000 + 13,500 = 41,500 Thus, the demand equation is 𝑞 = 𝑚𝑝 + 𝑏 = −4,500𝑝 + 41,500. b. The units of measurement of the slope are generally units of 𝑦 per unit of 𝑥. In this case: Units of 𝑞 per unit of 𝑝. That is, Rides per day per $1 increase in the fare. Since the slope is −4,500 rides/day per $1 increase in the price, we interpret it as saying that ridership decreases by 4,500 rides per day for every $1 increase in the fare. c. From part (a), the demand equation is 𝑞 = −4,500𝑝 + 41,500 If the fare is $6, we have 𝑝 = 6, so 𝑞 = −4500(6) + 41,500 = −27,000 + 41,500 = 14,500 rides/day.

90. a. A linear demand function has the form 𝑞 = 𝑚𝑝 + 𝑏. (𝑝 is the price, and 𝑞 is the demand). We are given two points on its graph: (5, 14) and (3, 18). 𝑞 − 𝑞1 18 − 14 4 Slope: 𝑚 = 2 = = = −2 𝑝2 − 𝑝1 3−5 −2 Intercept: 𝑏 = 𝑞1 − 𝑚𝑝1 = 14 − (−2)(5) = 24 Thus, the demand equation is 𝑞 = 𝑚𝑝 + 𝑏 = −2𝑝 + 24. b. The units of measurement of the slope are generally units of 𝑦 per unit of 𝑥. In this case: Units of 𝑞 per unit of 𝑝. That is, Millions of rides/day per Z1 increase in the fare. Since the slope is −2 million rides/day per Z1 increase in the price, we interpret it as saying that ridership decreases by 2 million rides per day for every Z1 increase in the fare. c. From part (a), the demand equation is 𝑞 = −2𝑝 + 24 If the fare is Z10, we have 𝑝 = 10, so 𝑞 = −2(10) + 24 = 4 million rides/day.

91. a. In a linear model of 𝑦 versus time 𝑡, the slope is the number of units of 𝑦 per unit time, and we are given this quantity: 40 million pounds/year. Thus, working in millions of pounds, we can take 𝑚 = 40. We are also given the 𝑦-intercept (the value of 𝑦 at 𝑡 = 0) as 𝑏 = 290. Thus, the model is 𝑦 = 40𝑡 + 290 million pounds of pasta b. In 2005, 𝑡 = 15, and so 𝑦(15) = 40(15) + 290 = 890 million pounds. 92. a. In a linear model of 𝑦 versus time 𝑡, the slope is the number of units of 𝑦 per unit time, and we are given this quantity: 60 million kg/year. Thus, working in millions of kilograms, we can take 𝑚 = 60. We are also given the 𝑦-intercept (the value of 𝑦 at 𝑡 = 0) as 𝑏 = 550. Thus, the model is 𝑦 = 60𝑡 + 550 million kg of mercury b. The year 2230 corresponds to 𝑡 = 20, and so 𝑦 = 60(20) + 550 = 1,750 million kg.


Solutions Section 1.3 93. a. The desired linear model has the form 𝑛 = 𝑚𝑡 + 𝑏, where 𝑡 is time in years since 2013. We are given two points on its graph: 2015 data: (2, 90); 2019 data: (6, 270) 𝑛 − 𝑛1 270 − 90 180 Slope: 𝑚 = 2 = = = 45 𝑡2 − 𝑡1 6−2 4 Intercept: 𝑏 = 𝑛1 − 𝑚𝑡1 = 90 − (45)(2) = 0 Thus, the linear model is 𝑛 = 𝑚𝑝 + 𝑏 = 45𝑡. b. The units of measurement of the slope are units of 𝑛 per unit of 𝑡; that is, thousands of (daily) transactions per year. The number of daily Bitcoin transactions increased at a rate of 45,000 per year. c. The year 2021 corresponds to 𝑡 = 8, so 𝑛 = 45(8) = 360 thousand, somewhat higher than the actual number of transactions. 94. a. The desired linear model has the form 𝐸 = 𝑚𝑡 + 𝑏, where 𝑡 is time in years since 2010. We are given two points on its graph: 2016 data: (6, 213); 2021 data: (11, 210) 𝐸 − 𝐸1 210 − 213 −3 Slope: 𝑚 = 2 = = = −0.6 𝑡2 − 𝑡1 11 − 6 5 Intercept: 𝑏 = 𝐸1 − 𝑚𝑡1 = 213 − (−0.6)(6) = 216.6 Thus, the linear model is 𝐸 = 𝑚𝑝 + 𝑏 = −0.6𝑡 + 216.6 b. The units of measurement of the slope are units of 𝐸 per unit of 𝑡; that is, billions of dollars per year. Exxon Mobil's operating expenses decreased at a rate of $0.6 billion per year. c. The year 2018 corresponds to 𝑡 = 8, and so 𝐸 = −0.6(8) + 216.6 = $211.8 billion, which differs quite significantly from the actual operating expenses. 95. 𝑠(𝑡) = 2.5𝑡 + 10 a. Velocity = slope = 2.5 feet/sec. b. After 4 seconds, 𝑡 = 4, so 𝑠(4) = 2.5(4) + 10 = 10 + 10 = 20 Thus the model train has moved 20 feet along the track. c. The train will be 25 feet along the track when 𝑠 = 25. Substituting gives 25 = 2.5𝑡 + 10 Solving for time t gives 2.5𝑡 = 25 − 10 = 15 15 𝑡= = 6 seconds 2.5 96. 𝑠(𝑡) = −1.8𝑡 + 9 a. Velocity = slope = −1.8 feet/sec. b. 𝑠(4) = −1.8(4) + 9 = 1.8 feet from the ground. c. 0 = −1.8𝑡 + 9, giving 𝑡 = 5 seconds

97. a. Take 𝑠 to be displacement from Jones Beach, and 𝑡 to be time in hours. We are given two points: (𝑡, 𝑠) = (10, 0) (𝑡, 𝑠) = (10.1, 13)

𝑠 = 0 for Jones Beach. 6 minutes = 0.1 hours

We are asked for the speed, which equals the magnitude of the slope. 𝑠 − 𝑠1 13 − 0 13 𝑚= 2 = = = 130 𝑡2 − 𝑡1 10.1 − 10 0.1 Units of slope = units of 𝑠 per unit of 𝑡 = miles per hour Thus, the police car was traveling at 130 mph. b. For the displacement from Jones Beach at time 𝑡, we want to express 𝑠 as a linear function of 𝑡; namely, 𝑠 = 𝑚𝑡 + 𝑏. We already know 𝑚 = 130 from part (a). For the intercept, use


Solutions Section 1.3 𝑏 = 𝑠1 − 𝑚𝑡1 = 0 − 130(10) = −1,300 Therefore, the displacement at time 𝑡 is 𝑠 = 𝑚𝑡 + 𝑏 = 130𝑡 − 1,300

98. a. Take 𝑠 to be displacement from Jones Beach, and 𝑡 to be time in hours. We are given two points: (𝑡, 𝑠) = (9.9, 0) and (10.1, 13) We are asked for the speed, which equals the magnitude of the slope. 𝑠 − 𝑠1 13 − 0 𝑚= 2 = = 65 miles per hour 𝑡2 − 𝑡1 10.1 − 9.9 Thus, the perp was traveling at 65 mph. b. 𝑠 = 𝑚𝑡 + 𝑏, where 𝑚 = 65 from part (a), and 𝑏 = 𝑠1 − 𝑚𝑡1 = 0 − 65(9.9) = −643.5 Therefore, 𝑠 = 𝑚𝑡 + 𝑏 = 65𝑡 − 643.5

99. a. The desired linear model has the form 𝐿 = 𝑚𝑛 + 𝑏. We are given two points on its graph: Second edition data: (2, 585); Seventh edition data: (7, 782) 𝐿 − 𝐿1 782 − 585 197 Slope: 𝑚 = 2 = = = 39.4 𝑛2 − 𝑛1 7−2 5 Intercept: 𝑏 = 𝐿1 − 𝑚𝑛1 = 585 − (39.4)(2) = 506.2 Thus, the linear model is 𝐿 = 𝑚𝑛 + 𝑏 = 39.4𝑛 + 506.2 b. The units of measurement of the slope are units of 𝐿 per unit of 𝑛; that is, pages per edition; Applied Calculus is growing at a rate of 39.4 pages per edition. c. The length 𝐿 will equal 1,500 when 39.4𝑛 + 506.2 = 1,500. Solving for 𝑛 gives 39.4𝑛 = 1,500 − 506.2 = 993.8 993.8 𝑛= ≈ 25.2 39.4 Thus, by the 26th edition, the book will be over 1,500 pages long.

100. a. The desired linear model has the form 𝐿 = 𝑚𝑛 + 𝑏. We are given two points on its graph: Second edition data: (2, 603); Fifth edition data: (5, 690) 𝐿 − 𝐿1 690 − 603 87 Slope: 𝑚 = 2 = = = 29 𝑛2 − 𝑛1 5−2 3 Intercept: 𝑏 = 𝐿1 − 𝑚𝑛1 = 603 − (29)(2) = 545 Thus, the linear model is 𝐿 = 𝑚𝑛 + 𝑏 = 29𝑛 + 545. b. The units of measurement of the slope are units of 𝐿 per unit of 𝑛; that is, pages per edition; Finite Mathematics is growing at a rate of 29 pages per edition. 1,000 − 545 c. 𝐿 = 29𝑛 + 545 = 1,000 when 𝑛 = ≈ 15.7. Thus, by the 16th edition, the book will be over 29 1,000 pages long. 101. 𝐹 = Fahrenheit temperature, 𝐶 = Celsius temperature, and we want 𝐹 as a linear function of 𝐶. That is, 𝐹 = 𝑚𝐶 + 𝑏. (𝐹 plays the role of 𝑦 and 𝐶 plays the role of 𝑥.) We are given two points: (𝐶, 𝐹 ) = (0, 32) (𝐶, 𝐹 ) = (100, 212)

Slope: 𝑚 =

Freezing point Boiling point

𝐹2 − 𝐹1 212 − 32 180 = = = 1.8 𝐶2 − 𝐶1 100 − 0 100


Solutions Section 1.3 Intercept: 𝑏 = 𝐹1 − 𝑚𝐶1 = 32 − 1.8(0) = 32 Thus, the linear relation is 𝐹 = 𝑚𝐶 + 𝑏 = 1.8𝐶 + 32 When 𝐶 = 30 ∘ 𝐹 = 1.8(30) + 32 = 54 + 32 = 86 ∘ When 𝐶 = 22 ∘ 𝐹 = 1.8(22) + 32 = 39.6 + 32 = 71.6 ∘. Rounding to the nearest degree gives 72 ∘F. When 𝐶 = −10 ∘, 𝐹 = 1.8(−10) + 32 = −18 + 32 = 14 ∘. When 𝐶 = −14 ∘, 𝐹 = 1.8(−14) + 32 = −25.2 + 32 = 6.8 ∘. Rounding to the nearest degree gives 7°F.

102. 𝐹 = Fahrenheit temperature, 𝐶 = Celsius temperature, and we want 𝐹 as a linear function of 𝐶. That is, 𝐹 = 𝑚𝐶 + 𝑏. (𝐹 plays the role of 𝑦 and 𝐶 plays the role of 𝑥.) We are given two points: (𝐹 , 𝐶) = (32, 0) and (212, 100) 𝐶 − 𝐶1 100 − 0 100 5 Slope: 𝑚 = 2 = = = 𝐹2 − 𝐹1 212 − 32 180 9 5 160 Intercept: 𝑏 = 𝐶1 − 𝑚𝐹1 = 0 − (32) = − 9 9 Thus, the linear relation is 5 160 𝐶 = 𝑚𝐹 + 𝑏 = 𝐹 − 9 9 5 160 360 𝐶(104) = (104) − = = 40 ∘ 9 9 9 5 160 225 𝐶(77) = (77) − = = 25 ∘ 9 9 9 5 160 −90 𝐶(14) = (14) − = = −10 ∘ 9 9 9 5 160 −360 𝐶(−40) = (−40) − = = −40 ∘ 9 9 9 103. a. 𝑆 = Southwest Airlines net income (in $ millions), 𝐽 = JetBlue Airways net income (in $ millions), and we want 𝐽 as a linear function of 𝑆. That is, 𝐽 = 𝑚𝑆 + 𝑏 𝐽 plays the role of 𝑦 and 𝑆 plays the role of 𝑥. We are given two points: (𝑆, 𝐽) = (2,000, 1,100) (𝑆, 𝐽) = (−3,000, −1,400)

2017 data 2020 data

𝐽2 − 𝐽1 −1,400 − 1,100 −2,500 = = = 0.5 𝑆2 − 𝑆1 −3,000 − 2,000 −5,000 Intercept: 𝑏 = 𝐽1 − 𝑚𝑆1 = 1,100 − (0.5)(2,000) = 100 Thus, the linear relation is 𝐽 = 𝑚𝑆 + 𝑏 = 0.5𝑆 + 100. b. In 2019, Southwest Airlines' net income was 𝑆 = 2,300, so 𝐽 = 0.5(2,300) + 100 = 1,250, predicting a $1,250 million net income for JetBlue, $650 million higher than the actual $600 million net income JetBlue earned in 2019. c. The units of measurement of the slope are units of 𝐽 per unit of 𝑆; that is, millions of dollars of JetBlue Airways net income per million dollars of Southwest Airlines net income; JetBlue Airways earned an additional net income of $0.50 per $1 additional net income earned by Southwest Airlines. Slope: 𝑚 =


Solutions Section 1.3 104. a. 𝐴 = Alaska Air Group net income (in $ millions), 𝐽 = JetBlue Airways net income (in $ millions), and we want 𝐽 as a linear function of 𝐴. That is, 𝐽 = 𝑚𝐴 + 𝑏 𝐽 plays the role of 𝑦 and 𝐴 plays the role of 𝑥. We are given two points: (𝐴, 𝐽) = (800, 600) (𝐴, 𝐽) = (−1,200, −1,400)

2016 data 2020 data

𝐽2 − 𝐽1 −1,400 − 600 −2,000 = = =1 𝐴2 − 𝐴1 −1,200 − 800 −2,000 Intercept: 𝑏 = 𝐽1 − 𝑚𝐴1 = 600 − (1)(800) = −200 Thus, the linear relation is 𝐽 = 𝑚𝐴 + 𝑏 = 𝐴 − 200. b. 2017: 𝐴 = 1,000; 𝐽 = 1,000 − 200 = $800 million 2018: 𝐴 = 400; 𝐽 = 400 − 200 = $200 million 2019: 𝐴 = 800; 𝐽 = 800 − 200 = $600 million comparing these values with those in the table shows that the model gives a perfect prediction for every year except 2017 ($300 million lower than the actual JetBlue income). c. The units of measurement of the slope are units of 𝐽 per unit of 𝐴; that is, millions of dollars of JetBlue Airways net income per million dollars of Alaska Air Group net income; JetBlue Airways earned an additional net income of $1 per $1 additional net income earned by Alaska Air Group. Slope: 𝑚 =

105. Income = royalties + screen rights

𝐼 = 5\% of net profits + 50,000 𝐼 = 0.05𝑁 + 50,000 Equation notation 𝐼(𝑁) = 0.05𝑁 + 50,000 Function notation

For an income of $100,000,

100,00 = 0.05𝑁 + 50,000 0.05𝑁 = 50,000 50,000 𝑁= = 1,000,000 0.05

Her marginal income is her increase in income per $1 increase in net profit. This is the slope, 𝑚 = 0.05 dollars of income per dollar of net profit, or 5\hbox{\rm\rlap/c} per dollar of net profit. 106. 𝐼 = 𝑚𝑁 + 𝑏 𝑏 = 100,000, 𝑚 = 0.08, and so 𝐼 = 0.08𝑁 + 100,000 For an income of $1,000,000, 1,000,000 = 0.08𝑁 + 100,000 0.08𝑁 = 900,000 𝑁 = 11,250,000 Marginal income is 𝑚 = 8 hbox rlap ∕c per dollar of net profit.

107. The year 2000 corresponds to 𝑡 = 10, which is in the range 6 ≤ 𝑡 < 15, so we use the first equation: 𝑣(𝑡) = 400𝑡 − 2,200. The slope is 400 MHz/year, telling us that the speed of a processor was increasing by 400 MHz/year. 108. The year 2000 corresponds to 𝑡 = 25, which is in the range 20 ≤ 𝑡 ≤ 30, so we use the second equation: 𝑣(𝑡) = 174𝑡 − 3,420. The slope is 174 MHz/year, telling us that the speed of a processor was increasing by 174 MHz/year.


Solutions Section 1.3 109. a. The data are 𝑡

𝑦

0

20

40

78

2,100 2,950

1970–1990 (first two data points): 𝑦 − 𝑦1 700 − 78 Slope: 𝑚 = 2 = = 31.1 𝑡2 − 𝑡1 20 − 0 Intercept: 𝑏 = 78, specified in first data point Thus, the linear model is 𝑦 = 𝑚𝑡 + 𝑏 = 31.1𝑡 + 78. b. 1990–2010 (second and third data points): 𝑦 − 𝑦1 2,950 − 700 Slope: 𝑚 = 2 = = 112.5 𝑡2 − 𝑡1 40 − 20 Intercept: 𝑏 = 𝑦1 − 𝑚𝑡1 = 700 − (112.5)20 = −1,550 Thus, the linear model is 𝑦 = 𝑚𝑡 + 𝑏 = 112.5𝑡 − 1,550. c. Since the first model is valid for 0 ≤ 𝑡 ≤ 20 and the second one for 20 ≤ 𝑡 ≤ 40, we put them together as 31.1𝑡 + 78 if 0 ≤ 𝑡 < 20 𝑦= {112.5𝑡 − 1,550 if 20 ≤ 𝑡 ≤ 40. Notice that, since both formulas agree at 𝑡 = 20, we can also say 31.1𝑡 + 78 if 0 ≤ 𝑡 ≤ 20 𝑦= {112.5𝑡 − 1,550 if 20 < 𝑡 ≤ 40. d. Since 2004 is represented by 𝑡 = 34, we use the second formula to obtain 𝑦 = 112.5(34) − 1,550 = 2,275, or $2,275,000, in good agreement with the actual value shown in the graph. 110. a. The data are 𝑡

0

20

40

𝑦 222 2,100 5,600

1980–2000 (first two data points): 𝑦 − 𝑦1 2,100 − 222 Slope: 𝑚 = 2 = = 93.9 𝑡2 − 𝑡1 20 − 0 Intercept: 𝑏 = 222, specified in first data point Thus, the linear model is 𝑦 = 𝑚𝑡 + 𝑏 = 93.9𝑡 + 222. b. 2000–2020 (second and third data points): 𝑦 − 𝑦1 5,600 − 2,100 Slope: 𝑚 = 2 = = 175 𝑡2 − 𝑡1 40 − 20 Intercept: 𝑏 = 𝑦1 − 𝑚𝑡1 = 2,100 − (175)20 = −1,400 Thus, the linear model is 𝑦 = 𝑚𝑡 + 𝑏 = 175𝑡 − 1,400. c. Since the first model is valid for 0 ≤ 𝑡 ≤ 20 and the second one for 20 ≤ 𝑡 ≤ 40, we put them together as 93.9𝑡 + 222 if 0 ≤ 𝑡 < 20 𝑦= {175𝑡 − 1,400 if 20 ≤ 𝑡 ≤ 40. Notice that, since both formulas agree at 𝑡 = 20, we can also say


93.9𝑡 + 222 if 0 ≤ 𝑡 ≤ 20 {175𝑡 − 1,400 if 20 < 𝑡 ≤ 40. d. Since 1992 is represented by 𝑡 = 12, we use the first formula to obtain 𝑦 = 93.9(12) + 222 = 1,348.8, or $1,348,800, considerably higher than the actual value shown on the graph. The actual cost is not linear in the range 1980–2000. Solutions Section 1.3

𝑦=

111. 2006–2012: Points: 2006 data: (𝑡, 𝑆) = (0, 531) 2012 data: (𝑡, 𝑆) = (6, 547) 𝑆 − 𝑆1 547 − 531 8 Slope: 𝑚 = 2 = = 𝑡2 − 𝑡1 6−0 3 Intercept: We are given the 𝑆-intercept as 531. 8 Thus, the linear model is 𝑆 = 𝑚𝑡 + 𝑏 = 𝑡 + 531. 3 2012–2018: Points: 2000 data: (𝑡, 𝑆) = (6, 547) 2004 data: (𝑡, 𝑆) = (12, 529) 𝑆 − 𝑆1 529 − 547 Slope: 𝑚 = 2 = = −3 𝑡2 − 𝑡1 12 − 6 Intercept: 𝑏 = 𝑆1 − 𝑚𝑡1 = 547 − (−3)6 = 565 Thus, the linear model is 𝑆 = 𝑚𝑡 + 𝑏 = −3𝑡 + 565 Putting them together gives 𝑆=

8 𝑡 + 531 3

{−3𝑡 + 565

if 0 ≤ 𝑡 < 6

if 6 ≤ 𝑡 ≤ 12. The score in 2009 is 𝑆(3), so we use the first formula to obtain 8 𝑆(3) = (3) + 531 = 539, in exact agreement with the actual score. 3 112. 2001–2004: Points: 2001 data: (𝑡, 𝑃 ) = (0, 9.7) 2004 data: (𝑡, 𝑃 ) = (3, 4.3) 𝑃 − 𝑃1 4.3 − 9.7 Slope: 𝑚 = 2 = = −1.8 𝑡2 − 𝑡1 3−0 Intercept: 𝑏 = 9.7, specified in first data point Thus, the linear model is 𝑁 = 𝑚𝑡 + 𝑏 = −1.8𝑡 + 9.7. 2004–2007: Points: 2004 data: (𝑡, 𝑃 ) = (3, 4.3) 2007 data: (𝑡, 𝑃 ) = (6, 10.3) 𝑃 − 𝑃1 10.3 − 4.3 Slope: 𝑚 = 2 = =2 𝑡2 − 𝑡1 6−3 Intercept: 𝑏 = 𝑃1 − 𝑚𝑡1 = 4.3 − (2)(3) = −1.7 Thus, the linear model is 𝑁 = 𝑚𝑡 + 𝑏 = 2𝑡 − 1.7. Putting them together gives −1.8𝑡 + 9.7 if 0 ≤ 𝑡 ≤ 3 𝑃= {2𝑡 − 1.7 if 3 < 𝑡 ≤ 6. The percentage of delinquent borrowers in 2006 is 𝑃 (5), so we use the second formula to obtain 𝑃 (5) = 2(5) − 1.7 = 8.3%.


Solutions Section 1.3 113. Compute the corresponding successive changes Δ𝑥 in 𝑥 and Δ𝑦 in 𝑦, and compute the ratios Δ𝑦∕Δ𝑥. If the answer is always the same number, then the values in the table come from a linear function. 114. The desired equation has the form 𝑦 = 𝑚𝑥 + 𝑏. The slope 𝑚 is given by 𝑚 = Δ𝑦∕Δ𝑥, where Δ𝑥 and Δ𝑦 are corresponding changes in 𝑥 and 𝑦. The intercept 𝑏 is given by the 𝑦-value corresponding to 𝑥 = 0, if supplied. If it is not supplied, choose any point (𝑥1 , 𝑦1 ) and use the formula 𝑏 = 𝑦1 − 𝑚𝑥1 . 115. To find the linear function, solve the equation 𝑎𝑥 + 𝑏𝑦 = 𝑐 for 𝑦 : 𝑏𝑦 = −𝑎𝑥 + 𝑐 𝑎 𝑐 𝑦 =− 𝑥+ 𝑏 𝑏

𝑎 𝑐 Thus, the desired function is 𝑓(𝑥) = − 𝑥 + . 𝑏 𝑏 𝑎 𝑐 If 𝑏 = 0, then and are undefined, and 𝑦 cannot be specified as a function of 𝑥. (The graph of the 𝑏 𝑏 resulting equation would be a vertical line.)

116. The slope of the line with equation 𝑦 = 𝑚𝑥 + 𝑏 is the number of units that 𝑦 increases per unit increase in 𝑥 . 117. The slope of the line is 𝑚 = as fast as 𝑥, then its slope is 3 .

Δ𝑦 3 = = 3. Therefore, if, in a straight line, 𝑦 is increasing three times Δ𝑥 1

118. The slope is 𝑚 = −4∕3 units of 𝑦 per unit of 𝑥. We do not have enough information to compute the intercept. 119. If 𝑚 is positive, then 𝑦 will increase as 𝑥 increases; if 𝑚 is negative, then 𝑦 will decrease as 𝑥 increases; if 𝑚 is zero, then 𝑦 will not change as 𝑥 changes. 120. Since Δ𝑦 = −Δ𝑥, the function is linear with slope Δ𝑦 −Δ𝑥 𝑚= = = −1. Δ𝑥 Δ𝑥 121. The slope computed in cell C2 is given by 𝑦 − 𝑦1 −1 − 2 𝑚= 2 = = −1.5 𝑥2 − 𝑥1 3−1 If we increase the 𝑦-coordinate in cell B3, this increases 𝑦2 , and thus increases the numerator Δ𝑦 = 𝑦2 − 𝑦1 without affecting the denominator Δ𝑥. Thus the slope will increase. 122. The slope increases: Δ𝑦 is negative, and stays the same, while Δ𝑥 becomes a larger positive number, so the (negative) slope decreases in absolute value, meaning that it actually increases. 123. The units of the slope 𝑚 are units of 𝑦 (bootlags) per unit of 𝑥 (zonars). The intercept 𝑏 is on the


Solutions Section 1.3 𝑦-axis, and is thus measured in units of 𝑦 (bootlags). Thus, 𝑚 is measured in bootlags per zonar and 𝑏 is measured in bootlags. 124. Units of slope = units of 𝑦 per unit of 𝑥 = miles per dollar. Thus, the independent variable is measured in dollars and the dependent variable is measured in miles.

125. If a quantity changes linearly with time, it must change by the same amount for every unit change in time. Thus, since it increases by 10 units in the first day, it must increase by 10 units each day, including the third. 126. Since 𝑄(0) is positive, 𝑏 is positive. Since 𝑄 decreases with increasing 𝑇 , 𝑚 is negative.

127. Since the slope is 0.1, the velocity is increasing at a rate of 0.1 m/sec every second. Since the velocity is increasing, the object is accelerating (choice B). 128. Velocity = slope = 0.2 units of position per unit time. Thus, the object is moving with constant speed (choice A). 129. Write 𝑓(𝑥) = 𝑚𝑥 + 𝑏 and 𝑔(𝑥) = 𝑛𝑥 + 𝑐. Then 𝑓(𝑥) + 𝑔(𝑥) = 𝑚𝑥 + 𝑏 + (𝑛𝑥 + 𝑐) = (𝑚 + 𝑛)𝑥 + (𝑏 + 𝑐), also a linear function with slope 𝑚 + 𝑛.

130. Not necessarily; for instance, if 𝑓(𝑥) = 2𝑥 + 1 and 𝑔(𝑥) = 𝑥, then their ratio has values 2𝑥 + 1 1 = 2 + , not a linear function of 𝑥. (Only if 𝑔 is a nonzero constant will the ratio will be linear.) 𝑥 𝑥 131. Answers may vary. For example, 𝑓(𝑥) = 𝑥 1∕3, 𝑔(𝑥) = 𝑥 2∕3 gives 𝑓(𝑥)𝑔(𝑥) = 𝑥 1∕3𝑥 2∕3 = 𝑥.

132. Answers may vary. For example, 𝑓(𝑥) = 𝑥 3 + 2𝑥 2, 𝑔(𝑥) = 𝑥 2 gives 𝑓(𝑥) 𝑥 3 + 2𝑥 2 = = 𝑥 + 2 (with domain all real numbers other than 0). 𝑔(𝑥) 𝑥2 133. Increasing the number of items from the break-even number results in a profit: Because the slope of the revenue graph is larger than the slope of the cost graph, it is higher than the cost graph to the right of the point of intersection, and hence corresponds to a profit.

134. You should solve your equation for 𝑞 to obtain 𝑞 as a function of 𝑝. Simply switching 𝑝 and 𝑞 will result in the wrong equation, and starting from scratch would be a less efficient way of obtaining the result you want.


Solutions Section 1.4 Section 1.4

1. (1, 1), (2, 2), (3, 4); 𝑦 = 𝑥 − 1

𝒚̂ =𝒙 − 𝟏 𝒚 − 𝒚̂ (𝒚 − 𝒚̂ ) 𝟐

𝒙

𝒚

1

1

0

1

1

2

2

1

1

1

3

4

2

2

4

SSE = Sum of squares of residuals (last column) = 1 + 1 + 4 = 6 2. (0, 1), (1, 1), (2, 2); 𝑦 = 𝑥 + 1

𝒚̂ =𝒙 + 𝟏 𝒚 − 𝒚̂ (𝒚 − 𝒚̂ ) 𝟐

𝒙

𝒚

0

1

1

1

1

2

2

2

3

0

0

−1

1

−1

1

SSE = Sum of squares of residuals (last column) = 0 + 1 + 1 = 2 3. (0, −1), (1, 3), (4, 6), (5, 0); 𝑦 = −𝑥 + 2 𝒙 0

𝒚

−1

1

3

4

6

5

0

𝒚̂ = − 𝒙 + 𝟐 𝒚 − 𝒚̂ (𝒚 − 𝒚̂ ) 𝟐 2

−3

9

1

2

4

−3

8

64

3

9

−2

SSE = Sum of squares of residuals (last column) = 9 + 4 + 64 + 9 = 86 4. (2, 4), (6, 8), (8, 12), (10, 0); 𝑦 = 2𝑥 − 8 𝒙

𝒚

𝒚̂ =𝟐𝒙 − 𝟖 𝒚 − 𝒚̂ (𝒚 − 𝒚̂ ) 𝟐

2

4

8

64

6

8

4

4

16

8

12

8

4

16

10

0

12

−4

−12

144

SSE = Sum of squares of residuals (last column) = 240 5. (1, 1), (2, 2), (3, 4) a. 𝑦 = 1.5𝑥 − 1


Solutions Section 1.4

𝒚̂ =𝟏.𝟓𝒙 − 𝟏 𝒚 − 𝒚̂ (𝒚 − 𝒚̂ ) 𝟐

𝒙

𝒚

1

1

0.5

0.5

0.25

2

2

2

0

0

3

4

3.5

0.5

0.25

SSE = Sum of squares of residuals = 0.5 b. 𝑦 = 2𝑥 − 1.5

𝒚̂ =𝟐𝒙 − 𝟏.𝟓 𝒚 − 𝒚̂ (𝒚 − 𝒚̂ ) 𝟐

𝒙

𝒚

1

1

0.5

2

2

2.5

3

4

4.5

0.5

0.25

−0.5

0.25

−0.5

0.25

SSE = Sum of squares of residuals = 0.75 The model that gives the better fit is (a) because it gives the smaller value of SSE. 6. (0, 1), (1, 1), (2, 2) a. 𝑦 = 0.4𝑥 + 1.1

𝒚̂ =𝟎.𝟒𝒙 + 𝟏.𝟏 𝒚 − 𝒚̂ (𝒚 − 𝒚̂ ) 𝟐

𝒙

𝒚

0

1

1.1

1

1

1.5

2

2

1.9

−0.1 −0.5

0.01

0.1

0.01

0.25

SSE = Sum of squares of residuals = 0.27 b. 𝑦 = 0.5𝑥 + 0.9

𝒚̂ =𝟎.𝟓𝒙 + 𝟎.𝟗 𝒚 − 𝒚̂ (𝒚 − 𝒚̂ ) 𝟐

𝒙

𝒚

0

1

0.9

1

1

1.4

2

2

1.9

0.1

−0.4

0.01

0.1

0.01

0.16

SSE = Sum of squares of residuals = 0.18 The model that gives the better fit is (b) because it gives the smaller value of SSE. 7. (0, −1), (1, 3), (4, 6), (5, 0) a. 𝑦 = 0.3𝑥 + 1.1 𝒙 0

𝒚

−1

𝒚̂ =𝟎.𝟑𝒙 + 𝟏.𝟏 𝒚 − 𝒚̂ (𝒚 − 𝒚̂ ) 𝟐 1.1

−2.1

4.41

1

3

1.4

1.6

2.56

4

6

2.3

3.7

13.69

5

0

2.6

−2.6

6.76

SSE = Sum of squares of residuals = 27.42


b. 𝑦 = 0.4𝑥 + 0.9 𝒙

𝒚

−1

0

Solutions Section 1.4

𝒚̂ =𝟎.𝟒𝒙 + 𝟎.𝟗 𝒚 − 𝒚̂ (𝒚 − 𝒚̂ ) 𝟐 −1.9

0.9

3.61

1

3

1.3

1.7

2.89

4

6

2.5

3.5

12.25

5

0

2.9

−2.9

8.41

SSE = Sum of squares of residuals = 27.16 The model that gives the better fit is (b) because it gives the smaller value of SSE. 8. (2, 4), (6, 8), (8, 12), (10, 0) a. 𝑦 = −0.1𝑥 + 7

𝒚̂ = − 𝟎.𝟏𝒙 + 𝟕 𝒚 − 𝒚̂ (𝒚 − 𝒚̂ ) 𝟐

𝒙

𝒚

2

4

6.8

6

8

6.4

1.6

2.56

8

12

6.2

5.8

33.64

10

0

6

−2.8 −6

7.84

36

SSE = Sum of squares of residuals = 80.04 b. 𝑦 = −0.2𝑥 + 6

𝒚̂ = − 𝟎.𝟐𝒙 + 𝟔 𝒚 − 𝒚̂ (𝒚 − 𝒚̂ ) 𝟐

𝒙

𝒚

2

4

5.6

6

8

4.8

3.2

10.24

8

12

4.4

7.6

57.76

10

0

4

−1.6 −4

2.56

16

SSE = Sum of squares of residuals = 86.56 The model that gives the better fit is (a) because it gives the smaller value of SSE. 9. Data points (𝑥, 𝑦) : (1, 1), (2, 2), (3, 4) 𝒙

𝒚

𝒙𝒚

𝒙𝟐

2

2

4

4

1 3 6

1 4 7

1

12 17

1 9

14

(The bottom row contains the column sums.)

𝑛 = 3 (number of data points) 𝑛(∑ 𝑥𝑦) − (∑ 𝑥)(∑ 𝑦) 3(17) − (6)(7) 9 𝑚= = = = 1.5 6 𝑛(∑ 𝑥 2) − (∑ 𝑥) 2 3(14) − 6 2


Intercept: 𝑏 =

∑ 𝑦 − 𝑚(∑ 𝑥) 7 − 1.5(6) −2 ≈ −0.6667 = = 𝑛 3 3

Solutions Section 1.4

Thus, the regression line is 𝑦 = 𝑚𝑥 + 𝑏 ≈ 1.5𝑥 − 0.6667. Graph:

6 5 4 3 2 1

y

x -1

0

1

2

𝒙

𝒚

𝒙𝒚

𝒙𝟐

1

1

1

1

3

10. Data points (𝑥, 𝑦) : (0, 1), (1, 1), (2, 2) 0 2 3

1 2 4

0

4

0

4

4

5

5

(The bottom row contains the column sums.)

𝑛 = 3 (number of data points) 𝑛(∑ 𝑥𝑦) − (∑ 𝑥)(∑ 𝑦) 3(5) − (3)(4) 3 𝑚= = = = 0.5 6 𝑛(∑ 𝑥 2) − (∑ 𝑥) 2 3(5) − 3 2 ∑ 𝑦 − 𝑚(∑ 𝑥) 4 − 0.5(3) 2.5 Intercept: 𝑏 = ≈ 0.8333 = = 𝑛 3 3

Thus, the regression line is 𝑦 = 𝑚𝑥 + 𝑏 ≈ 0.5𝑥 + 0.8333. Graph:

2.5

y

2 1.5 1 0.5 -1

0

1

2

x

3


Solutions Section 1.4 11. Data points (𝑥, 𝑦) : (0, −1), (1, 3), (3, 6), (4, 1) 𝒙

𝒚

𝒙𝒚

𝒙𝟐

3

3

1

0

−1

3

6

18

9

25

1 4

1

8

0 4

0 9

16 26

(The bottom row contains the column sums.)

𝑛 = 4 (number of data points) 𝑛(∑ 𝑥𝑦) − (∑ 𝑥)(∑ 𝑦) 4(25) − (8)(9) 28 𝑚= = = = 0.7 40 𝑛(∑ 𝑥 2) − (∑ 𝑥) 2 4(26) − 8 2 ∑ 𝑦 − 𝑚(∑ 𝑥) 9 − 0.7(8) 3.4 Intercept: 𝑏 = = = = 0.85 𝑛 4 4

Thus, the regression line is 𝑦 = 𝑚𝑥 + 𝑏 = 0.7𝑥 + 0.85. Graph:

6 5 4 3 2 1

y

x -1

0

1

2

𝒙

𝒚

𝒙𝒚

𝒙𝟐

4

8

32

16

0

0

100

3

4

5

12. Data points (𝑥, 𝑦) : (2, 4), (4, 8), (8, 12), (10, 0) 2

4

8

12

24

24

10

8

96 136

4

64

184

(The bottom row contains the column sums.)

𝑛 = 4 (number of data points) 𝑛(∑ 𝑥𝑦) − (∑ 𝑥)(∑ 𝑦) 4(136) − (24)(24) −32 𝑚= = = = −0.2 160 𝑛(∑ 𝑥 2) − (∑ 𝑥) 2 4(184) − 24 2 ∑ 𝑦 − 𝑚(∑ 𝑥) 24 − (−0.2)(24) 28.8 Intercept: 𝑏 = = = = 7.2 𝑛 4 4

Thus, the regression line is 𝑦 = 𝑚𝑥 + 𝑏 = −0.2𝑥 + 7.2.


Solutions Section 1.4 Graph:

12 10 8 6 4 2

y

0

2

4

6

𝒙

𝒚

𝒙𝒚

𝒙𝟐

𝒚𝟐

1

3

3

1

9

2

4

8

4

16

5

6

30

25

36

8

13

41

30

61

13. a. (1, 3), (2, 4), (5, 6)

8

10

x

12

(The bottom row contains the column sums.)

𝑛 = 3 (number of data points)} 𝑛(∑ 𝑥𝑦) − (∑ 𝑥)(∑ 𝑦) 3(41) − (8)(13) 𝑟= = 2 2 2 2 √3(30) − 8 2√3(61) − 13 2 𝑛(∑ 𝑥 ) − (∑ 𝑥) ⋅ 𝑛(∑ 𝑦 ) − (∑ 𝑦) √ √ 19 ≈ ≈ 0.9959 19.078784

b. (0, −1), (2, 1), (3, 4) 𝒙 0

𝒚

−1

𝒙𝒚

𝒙𝟐

𝒚𝟐

0

0

1

2

1

2

4

1

3

4

12

9

16

5

4

14

13

18

(The bottom row contains the column sums.)

𝑛 = 3 (number of data points)} 𝑛(∑ 𝑥𝑦) − (∑ 𝑥)(∑ 𝑦) 3(14) − (5)(4) 𝑟= = 𝑛(∑ 𝑥 2) − (∑ 𝑥) 2 ⋅ 𝑛(∑ 𝑦 2) − (∑ 𝑦) 2 √3(13) − 5 2√3(18) − 4 2 √ √ 22 ≈ ≈ 0.9538 23.0651252


c. (4, −3), (5, 5), (0, 0) 𝒙

𝒚

𝒙𝒚

Solutions Section 1.4 𝒙𝟐

𝒚𝟐

16

9

−3

−12

5

5

25

25

25

0

0

0

0

0

9

2

13

41

34

4

(The bottom row contains the column sums.)

𝑛 = 3 (number of data points)} 𝑛(∑ 𝑥𝑦) − (∑ 𝑥)(∑ 𝑦) 3(13) − (9)(2) 𝑟= = 2 2 2 2 √3(41) − 9 2√3(34) − 2 2 𝑛(∑ 𝑥 ) − (∑ 𝑥) ⋅ 𝑛(∑ 𝑦 ) − (∑ 𝑦) √ √ 21 ≈ ≈ 0.3273 64.1560597

The value of 𝑟 in part (a) has the largest absolute value. Therefore, the regression line for the data in part (a) is the best fit. The value of 𝑟 in part (c) has the smallest absolute value. Therefore, the regression line for the data in part (c) is the worst fit. Since 𝑟 is not ±1 for any of these lines, none of them is a perfect fit. 14. a. (1, 3), (−2, 9), (2, 1) 𝒙

𝒚

𝒙𝒚

𝒙𝟐

𝒚𝟐

1

3

3

9

-2

9

−18

1 4

81

2

1

2

4

1

1

13

9

91

−13

(The bottom row contains the column sums.)

𝑛 = 3 (number of data points)} 𝑛(∑ 𝑥𝑦) − (∑ 𝑥)(∑ 𝑦) 3(−13) − (1)(13) 𝑟= = 2 2 2 2 √3(9) − 1 2√3(91) − 13 2 𝑛(∑ 𝑥 ) − (∑ 𝑥) ⋅ 𝑛(∑ 𝑦 ) − (∑ 𝑦) √ √ −52 = = −1 (Best, perfect fit) 52

b. (0, 1), (1, 0), (2, 1) 𝒙

𝒚

𝒙𝒚

𝒙𝟐

𝒚𝟐

0

1

0

0

1

1

0

0

1

0

2

1

2

4

1

3

2

2

5

2

(The bottom row contains the column sums.)


𝑛 = 3 (number of data points)} 𝑛(∑ 𝑥𝑦) − (∑ 𝑥)(∑ 𝑦) 3(2) − (3)(2) 𝑟= = = 0 (Worst) 𝑛(∑ 𝑥 2) − (∑ 𝑥) 2 ⋅ 𝑛(∑ 𝑦 2) − (∑ 𝑦) 2 √3(5) − 3 2√3(2) − 2 2 √ √ Solutions Section 1.4

c. (0, 0), (5, −5), (2, −2.1) 𝒙

𝒚

𝒙𝒚

𝒙𝟐

𝒚𝟐

0

0

0

0

0

25

25

4

4.41

29

29.41

−5

5

−25

−2.1 −4.2

2

−7.1 −29.2

7

(The bottom row contains the column sums.)

𝑛 = 3 (number of data points)} 𝑛(∑ 𝑥𝑦) − (∑ 𝑥)(∑ 𝑦) 3(−29.2) − (7)(−7.1) 𝑟= = 𝑛(∑ 𝑥 2) − (∑ 𝑥) 2 ⋅ 𝑛(∑ 𝑦 2) − (∑ 𝑦) 2 √3(29) − 7 2√3(29.41) − (−7.1) 2 √ √ −37.9 ≈ ≈ −0.9997 37.9098932

15. Data points (𝑥, 𝑦) : (0, 3.9), (4, 4.8), (8, 5.7) 𝒙

𝒚

𝒙𝒚

𝒙𝟐

4.8

19.2

16

14.4

64.8

80

0

3.9

8

5.7

4 12

0

45.6

0

64

(The bottom row contains the column sums.)

𝑛 = 3 (number of data points) 𝑛(∑ 𝑥𝑦) − (∑ 𝑥)(∑ 𝑦) 3(64.8) − (12)(14.4) 21.6 𝑚= ≈ 0.23 = = 96 𝑛(∑ 𝑥 2) − (∑ 𝑥) 2 3(80) − 12 2 ∑ 𝑦 − 𝑚(∑ 𝑥) 14.4 − 0.225(12) 11.7 Intercept: 𝑏 = = = = 3.9 𝑛 3 3

Thus, the regression line is 𝑦 = 𝑚𝑥 + 𝑏 ≈ 0.23𝑥 + 3.9. Since 2018 corresponds to 𝑥 = 6, the prediction for 2018 is 𝑦 = 0.23(6) + 3.9 ≈ 5.3 million subscribers.


Solutions Section 1.4 16. Data points (𝑥, 𝑦) : (0, 35), (10, 33), (20, 32) 𝒙

𝒚

𝒙𝒚

𝒙𝟐

33

330

100

100

970

500

0

35

20

32

10 30

0

640

0

400

(The bottom row contains the column sums.)

𝑛 = 3 (number of data points) 𝑛(∑ 𝑥𝑦) − (∑ 𝑥)(∑ 𝑦) 3(970) − (30)(100) −90 𝑚= = = = −0.15 600 𝑛(∑ 𝑥 2) − (∑ 𝑥) 2 3(500) − 30 2 ∑ 𝑦 − 𝑚(∑ 𝑥) 100 − (−0.15)(30) 104.5 Intercept: 𝑏 = ≈ 34.83 = = 𝑛 3 3

Thus, the regression line is 𝑦 = 𝑚𝑥 + 𝑏 ≈ −0.15𝑥 + 34.83. Since 2022 corresponds to 𝑥 = 22, the prediction for 2022 is 𝑦 = −0.15(22) + 34.83. ≈ 31.5 million subscribers.

17. A linear demand function has the form 𝑞 = 𝑚𝑝 + 𝑏. (𝑝 is the price, and 𝑞 is the demand). Data points (𝑝, 𝑞) : (6, 1.2), (5, 1.4), (4.5, 1.5) 𝒑

𝒒

𝒑𝒒

𝒑𝟐

6

1.2

7.2

4.5

1.5

6.75 20.25

5

15.5

1.4 4.1

7

36 25

20.95 81.25

(The bottom row contains the column sums.)

𝑛 = 3 (number of data points) 𝑛(∑ 𝑝𝑞) − (∑ 𝑝)(∑ 𝑞) 3(20.95) − (15.5)(4.1) −0.7 𝑚= = = = −0.2 3.5 𝑛(∑ 𝑝 2) − (∑ 𝑝) 2 3(81.25) − 15.5 2 ∑ 𝑞 − 𝑚(∑ 𝑝) 4.1 − (−0.2)(15.5) 7.2 Intercept: 𝑏 = = = = 2.4 𝑛 3 3

Thus, the regression line is 𝑞 = 𝑚𝑝 + 𝑏 = −0.2𝑝 + 2.4. When the selling price is $550, 𝑝 = 5.5, and so 𝑞 ≈ −0.2(5.5) + 2.4 = 1.3 billion smartphones.


Solutions Section 1.4 18. A linear demand function has the form 𝑞 = 𝑚𝑝 + 𝑏. (𝑝 is the price, and 𝑞 is the demand). Data points (𝑝, 𝑞) : (4, 0.7), (3, 1), (2.5, 1.5) 𝒑

𝒒

𝒑𝒒

𝒑𝟐

4

0.7

2.8

16

2.5

1.5

3.75

6.25

3

9.5

1

3.2

3

9

9.55 31.25

(The bottom row contains the column sums.)

𝑛 = 3 (number of data points) 𝑛(∑ 𝑝𝑞) − (∑ 𝑝)(∑ 𝑞) 3(9.55) − (9.5)(3.2) −1.75 𝑚= = = = −0.5 3.5 𝑛(∑ 𝑝 2) − (∑ 𝑝) 2 3(31.25) − 9.5 2 ∑ 𝑞 − 𝑚(∑ 𝑝) 3.2 − (−0.5)(9.5) 7.95 Intercept: 𝑏 = ≈ 2.7 = = 𝑛 3 3

Thus, the regression line is 𝑞 = 𝑚𝑝 + 𝑏 ≈ −0.5𝑝 + 2.7. When the selling price is $350, 𝑝 = 3.5, and so 𝑞 ≈ −0.5(3.5) + 2.7 = 0.95 billion, or 950 million smartphones. 19. Following is the table we use to compute the regression line: 𝑥

𝑦

𝑥𝑦

𝑥2

20

3

60

400

40

6

240

1,600

80

9

720

6,400

100

15

1,500 10,000

240

33

2,520 18,400

(The bottom row contains the column sums.)

𝑛 = 3 (number of data points)} 𝑛(∑ 𝑥𝑦) − (∑ 𝑥)(∑ 𝑦) 4(2,520) − (240)(33) 2,160 Slope: 𝑚 = = = = 0.135 16,000 𝑛(∑ 𝑥 2) − (∑ 𝑥) 2 4(18,400) − 240 2 Intercept: 𝑏 =

∑ 𝑦 − 𝑚(∑ 𝑥) 33 − (0.135)(240) 0.6 = = = 0.15 𝑛 4 4

The regression model is therefore 𝑦 = 𝑚𝑥 + 𝑏 = 0.135𝑥 + 0.15. 𝑦(50) = 0.135(50) + 0.15 = 6.9 million jobs


Solutions Section 1.4 20. Following is the table we use to compute the regression line: 𝑥

𝑦

𝑥𝑦

𝑥2

10

200

2,000

100

40

900

36,000

1,600

50

1,000 50,000

2,500

80

2,000 160,000 6,400

180 4,100 248,000 10,600 (The bottom row contains the column sums.)

𝑛 = 3 (number of data points)} 𝑛(∑ 𝑥𝑦) − (∑ 𝑥)(∑ 𝑦) 4(248,000) − (180)(4,100) 254,000 Slope: 𝑚 = = = = 25.4 10,000 𝑛(∑ 𝑥 2) − (∑ 𝑥) 2 4(10,600) − 180 2 Intercept: 𝑏 =

∑ 𝑦 − 𝑚(∑ 𝑥) 4100 − (25.4)(180) −472 = = = −118 𝑛 4 4

The regression model is therefore 𝑦 = 𝑚𝑥 + 𝑏 = 25.4𝑥 − 118. 𝑦(70) = 25.4(70) − 118 = $1,660 billion

21. a. Data points (𝑅, 𝑃 ) : (190, 4), (240, 12), (300, 12), (420, 27) 𝑹

𝑷

𝑹𝑷

240

12

2,880

420

27

11,340 176,400

190 300 1,150

4

12 55

𝑹𝟐

760

36,100

3,600

90,000

57,600

18,580 360,100

(The bottom row contains the column sums.)

𝑛 = 4 (number of data points) 𝑛(∑ 𝑅𝑃 ) − (∑ 𝑅)(∑ 𝑃 ) 4(18,580) − (1,150)(55) 11,070 𝑚= ≈ 0.094 = = 2 2 2 117,900 𝑛(∑ 𝑅 ) − (∑ 𝑅) 4(360,100) − 1,150 ∑ 𝑃 − 𝑚(∑ 𝑅) 55 − 0.09389(1,150) −52.9771 Intercept: 𝑏 = ≈ ≈ −13.244 = 𝑛 4 4

Thus, the regression line is 𝑃 = 𝑚𝑅 + 𝑏 ≈ 0.094𝑅 − 13.244.


Solutions Section 1.4 Graph:

P

30 25 20 15 10 5 0 R 100 200 300 400 500 (Independent variable is 𝑅 and dependent variable is 𝑃 ) b. The units of measurement of the slope are units of profit per unit of revenue: billions of dollars of profit per billion dollars of revenue, or just dollars of profit per dollar of revenue. Thus, Amazon earned $0.094 in profit per additional $1 in revenue. c. 𝑃 = 0.094𝑅 − 13.244, and we are given 𝑃 = 15. Substituting gives 15 = 0.094𝑅 − 13.244 Solving for 𝑅 gives 28.244 𝑅= ≈ 300 0.094 Thus, the company would need to earn about $300 billion in revenue. d. The graph shows a good fit, so the linear model seems reasonable. 22. a. Data points (𝑅, 𝐼) : (190, 5), (240, 15), (300, 14), (420, 28) 𝑹

𝑰

240

15

3,600

420

28

11,760 176,400

190 300 1,150

5

14 62

𝑹𝑰

𝑹𝟐

950

36,100

4,200

90,000

57,600

20,510 360,100

(The bottom row contains the column sums.)

𝑛 = 4 (number of data points) 𝑛(∑ 𝑅𝐼) − (∑ 𝑅)(∑ 𝐼) 4(20,510) − (1,150)(62) 10,740 𝑚= ≈ 0.091 = = 117,900 𝑛(∑ 𝑅 2) − (∑ 𝑅) 2 4(360,100) − 1,150 2 ∑ 𝐼 − 𝑚(∑ 𝑅) 62 − 0.09109(1,150) −42.75827 Intercept: 𝑏 = ≈ ≈ −10.69 = 𝑛 4 4

Thus, the regression line is 𝐼 = 𝑚𝑅 + 𝑏 ≈ 0.091𝑅 − 10.69.


Solutions Section 1.4 Graph:

I

30 25 20 15 10 5 0 R 180 230 280 330 380 430 (Independent variable is 𝑅 and dependent variable is 𝐼) b. The units of measurement of the slope are units of operating income per unit of revenue: billions of dollars of operating operating income per billion dollars of revenue, or just dollars of operating income per dollar of revenue. Thus, Amazon had operating income of $0.091 per $1 in revenue. c. 𝐼 = 0.091𝑅 − 10.69, and we are given 𝐼 = 10. Substituting gives 10 = 0.091𝑅 − 10.69 Solving for 𝑅 gives 20.69 𝑅= ≈ 227 0.091 Thus, the company would need to earn approximately $227 billion in revenue. d. The graph shows a good fit, so the linear model seems reasonable. 23. a. The following result and plot were obtained using the Function Evaluator and Grapher on the Web site with the setup shown. Regression equation: 𝐿 = 39.29𝑛 + 528.71

b. The units of measurement of the slope are units of 𝐿 per unit of 𝑛; that is, pages per edition; Applied Calculus is growing at a rate of 39.29 pages per edition. 24. a. The following result and plot were obtained using the Function Evaluator and Grapher on the Web site with the setup shown. Regression equation: 𝐿 = 25.43𝑛 + 552.24


Solutions Section 1.4

b. The units of measurement of the slope are units of 𝐿 per unit of 𝑛; that is, pages per edition; Finite Mathematics is growing at a rate of 25.43 pages per edition.

25. a. Since production is a function of cultivated area, we take 𝑥 as cultivated area, and 𝑦 as production: 𝑥 𝑦

25

30

32

40

52

15

25

30

40

60

See the technology note accompanying Example 2 for the use of technology to obtain regression lines. We obtained the following regression line and plot in Excel. (coefficients rounded to two decimal places):𝑦 = 1.62𝑥 − 23.87

b. To interpret the slope 𝑚 = 1.62, recall that units of 𝑚 are units of 𝑦 per unit of 𝑥; that is, millions of tons of production of soybeans per million acres of cultivated land. Thus, production increases by 1.62 million tons of soybeans per million acres of cultivated land. More simply, each acre of cultivated land produces about 1.62 tons of soybeans.

26. a. Since production is a function of cultivated area, we take 𝑥 as cultivated area, and 𝑦 as production: 𝑥 𝑦

30

42

69

59

74

74

20

33

55

57

83

88

See the technology note accompanying Example 2 for the use of technology to obtain regression lines. We obtained the following regression line and plot in Excel. (coefficients rounded to two decimal places):𝑦 = 1.38𝑥 − 24.04

b. To interpret the slope 𝑚 = 1.38, recall that units of 𝑚 are units of 𝑦 per unit of 𝑥; That is, millions of tons of production of soybeans per million acres of cultivated land. Thus, production increases by 1.38 million tons of soybeans per million acres of cultivated land. More simply, each acre of cultivated land produces about 1.38 tons of soybeans. 27. a. 𝑦 = Continental net income as a function of 𝑥 = Price of oil. See the technology notes


Solutions Section 1.4 accompanying Example 2 and 3 for the use of technology to obtain regression lines and correlation coefficients. The following result and plot were obtained using the Function Evaluator and Grapher on the Web site with the setup shown:

Regression equation: 𝑦 = −11.85𝑥 + 797.71 Correlation coefficient: 𝑟 ≈ −0.414b. As |𝑟| ≈ 0.414 is significantly less than 0.8, the values of 𝑥 and 𝑦 are not strongly correlated, so that Continental's net income does not appear correlated to the price of oil.c. The points in the graph are nowhere near the regression line, confirming the conclusion in (b).

28. a. 𝑦 = Continental net income as a function of 𝑥 = Price of oil. See the technology notes accompanying Examples 2 and 3 for the use of technology to obtain regression lines and correlation coefficients. The following result and plot were obtained using the Function Evaluator and Grapher on the Web site with the setup shown:

Regression equation: 𝑦 = −28.90𝑥 + 1208.01 Correlation coefficient: 𝑟 ≈ −0.408b. As |𝑟| ≈ 0.408 is significantly less than 0.8, the values of 𝑥 and 𝑦 are not strongly correlated, so that American's net income does not appear correlated to the price of oil.c. The points in the graph are nowhere near the regression line, confirming the conclusion in (b).

29. a. Using 𝑥 = Number of natural science doctorates and 𝑦 = Number of engineering doctorates gives us the following table of values: 𝑥 5,000 5,400 5,900 6,200 6,300

6,700

𝑦 7,600 8,400 9,600 9,500 10,200 10,800

Using one of the technology methods of Example 2 (see the marginal note on using technology), we obtain the following regression line and plot (coefficients rounded to three significant digits): 𝑦 = 1.85𝑥 − 1570


Solutions Section 1.4 Graph:

y

12000 10000 8000 6000 4000 2000 0 x 4500 5500 6500 b. To interpret the slope, recall that units of the slope are units of 𝑦 (engineering doctorates) per unit of 𝑥 (natural science doctorates). Thus, 𝑚 = 1.85 engineering doctorates per natural science doctorate, indicating that there are around 1.85 additional doctorates in engineering per additional doctorate in the natural sciences. c. Using the technology method of Example 3, we can use technology to show the value of 𝑟 2 : 𝑟 2 ≈ 0.9649 𝑟 = √𝑟 2 ≈ √0.9649 ≈ 0.982 Since 𝑟 is close to 1, the correlation between 𝑥 and 𝑦 is a strong one. d. Yes; the data points are close to and randomly scattered about the regression line.

30. a. Using 𝑥 = Number of social science doctorates and 𝑦 = Number of education doctorates gives us the following table of values: 𝑥 7,900 8,500 8,700 9,000 8,900 8,900 𝑦 5,300 4,800 4,800 5,100 4,800 4,700

Using one of the technology methods of Example 2 (see the marginal note on using technology), we obtain the following regression line and plot (coefficients rounded to three significant digits): 𝑦 = −0.353𝑥 + 7970 Graph:

5500

y

5000

4500 x 7500 8000 8500 9000 9500 b. To interpret the slope, recall that units of the slope are units of 𝑦 (education doctorates) per unit of 𝑥 (social science doctorates). Thus, 𝑚 ≈ −0.35 education doctorates per social science doctorate, indicating that there are about 0.35 fewer doctorates in education per additional doctorate in the social sciences. c. Using the technology method of Example 3, we can use technology to show the value of 𝑟 2 : 𝑟 2 ≈ 0.3914 𝑟 = √𝑟 2 ≈ √0.3914 ≈ 0.626 Since 𝑟 is not close to 1, the correlation between 𝑥 and 𝑦 is a poor one. d. No; the data points suggest no relationship in particular; they are far from the regression line and haphazardly scattered.


Solutions Section 1.4 31. a. As 𝑡 is time in years since 2010, we use the following set of data for the regression: 𝑡

0

2

4

6

8

10

𝑦 5,000 5,400 5,900 6,200 6,300 6,700 Using one of the technology methods of Example 2 (see the marginal note on using technology), we obtain the following regression line and plot (coefficients rounded to three significant digits): 𝑦 = 164𝑡 + 5100 Graph:

7000

y

6000 5000 4000

0

2

4

6

t

8

10

𝑟 ≈ 0.985 b. Units of the slope are units of 𝑦 (natural science doctorates) per unit of 𝑡 (years); thus, doctorates per year. So, the number of natural science doctorates has been increasing at a rate of about 164 per year. c. The slopes of successive pairs of points do not show an increasing nor decreasing trend as we go from left to right, so the number of natural science doctorates is increasing at a more-or-less constant rate. d. Yes: If 𝑟 had been equal to 1, then the points would lie exactly on the regression line, which would indicate that the number of doctorates is growing at a constant rate. 32. a. As 𝑡 is time in years since 2010, we use the following set of data for the regression: 𝑡

0

2

4

6

8

10

𝑦 7,900 8,500 8,700 9,000 8,900 8,900 Using one of the technology methods of Example 2 (see the marginal note on using technology), we obtain the following regression line and plot (coefficients rounded to three significant digits): 𝑦 = 92.9𝑡 + 8190 Graph:

10000

y

9000 8000 7000

t

0 2 4 6 8 10 𝑟 ≈ 0.916, indicating a reasonably good fit. b. Units of the slope are units of 𝑦 (social science doctorates) per unit of 𝑡 (years); thus, doctorates per year. Thus, the number of social science doctorates has been increasing at a rate of about 93 per year. c. The data points suggest a concave-down curve rather than a straight line, indicating that the number of


Solutions Section 1.4 doctorates has been growing at a slower and slower rate (the slopes of successive pairs of points increase as we go from left to right). d. No: If 𝑟 had been equal to 1, then the points would lie exactly on the regression line, which would indicate that the number of doctorates is growing at a constant rate. 33. a. More-or-less constant rate; Exercise 29 suggests a roughly linear relationship between the number of natural science doctorates and the number of engineering doctorates, and Exercise 31 suggests that the number of natural science doctorates has been increasing at a more-or less constant rate. Therefore, the number of engineering doctorates is also increasing at a more-or-less constant rate. b. No; 𝑟 = 1 in Exercise 29 would indicate an exactly linear relationship between the number of natural science doctorates and the number of engineering doctorates, and so the conclusion would be the same. c. No; 𝑟 = 1 in Exercise 31 would indicate that the number of natural science doctorates has been increasing at a constant rate, and so the conclusion would be the same. 34. a. Haphazard; The graph in Exercise 30 suggests that the number of education doctorates has no particular relationship to the number of social science doctorates, and Exercise 32 suggests that the number of social science doctorates has been increasing with time. Therefore, the number of education doctorates would be expected to have no particular relationship to time either; that is, to behave haphazardly with respect to time as well. b. Yes; 𝑟 = 1 in Exercise 30 would indicate an exactly linear relationship between the number of social science doctorates and the number of education doctorates, and so the conclusion would be that the number of education doctorates increased at a faster and faster rate. c. No; 𝑟 = 1 in Exercise 32 would indicate that the number of social science doctorates has been increasing at a constant rate, and so the conclusion would be the same, as the number of education doctorates would still be haphazard. 35. a. Using the method of Example 3, we obtain the following regression line and plot (coefficients rounded to two decimal places): 𝑝 = 0.13𝑡 + 0.22; 𝑟 ≈ 0.97 Graph:

b. The first and last points lie above the regression line, while the central points lie below it, suggesting a curve. c. Here is a worksheet showing the computation of the residuals (based on Example 1 in the text):

Notice that the residuals are positive at first, become negative, and then become positive, confirming the impression from the graph.


Solutions Section 1.4 36. a. Using the method of Example 3, we obtain the following regression line and plot (coefficients rounded to two decimal places): 𝑐 = 35.5𝑡 − 21; 𝑟 ≈ 0.92 Graph:

b. The first and last points lie above the regression line, while the central points lie below it, suggesting a curve. c. Here is a worksheet showing the computation of the residuals (based on Example 1 in the text):

Notice that the residuals are positive at first, become negative, and then become positive, confirming the impression from the graph. 37. The regression line is defined to be the line that gives the lowest sum-of-squares error, SSE. If we are given two points, (𝑎, 𝑏) and (𝑐, 𝑑) with 𝑎 ≠ 𝑐, then there is a line that passes through these two points, giving SSE = 0. Since 0 is the smallest value possible, this line must be the regression line. 38. SSE = 0; the straight line passing through the given points has predicted values equal to the observed values. Hence, the residuals are zero, giving SSE = 0.

39. If the points (𝑥1 , 𝑦1 ), (𝑥2 , 𝑦2 ), ..., (𝑥𝑛 , 𝑦𝑛 ) lie on a straight line, then the sum-of-squares error, SSE, for this line is zero. Since 0 is the smallest value possible, this line must be the regression line. 40. No. The regression line may pass through none of the given points. 41. Calculation of the regression line: 𝑥

𝑦

𝑥𝑦

𝑥2

0

0

0

0

−𝑎 𝑎 0

𝑎 𝑎

2𝑎

−𝑎 2

𝑎2

0

2𝑎 2

𝑎2

𝑎2

(The bottom row contains the column sums.) 𝑛 = 3 (number of data points)}


Slope: 𝑚 =

𝑛(∑ 𝑥𝑦) − (∑ 𝑥)(∑ 𝑦)

3(0) − (0)(2𝑎) =0 3(2𝑎 2) − 0 2

Solutions Section 1.4

𝑛(∑ 𝑥 2) − (∑ 𝑥) 2

Correlation coefficient 𝑟 =

=

𝑛(∑ 𝑥𝑦) − (∑ 𝑥)(∑ 𝑦)

𝑛(∑ 𝑥 2) − (∑ 𝑥) 2 ⋅

𝑛(∑ 𝑦 2) − (∑ 𝑦) 2

√ √ we have just seen that this numerator is zero. Hence, 𝑟 = 0.

has the same numerator as 𝑚, and

42. Calculation of the regression line: 𝑥 0 0

𝑎

𝑎

𝑦

𝑥𝑦

𝑥2

−𝑎

0

0

0

0

0

0

0

0

𝑎

𝑎2 𝑎2

(The bottom row contains the column sums.)

𝑛 = 3 (number of data points)} 𝑛(∑ 𝑥𝑦) − (∑ 𝑥)(∑ 𝑦) 3(0) − (𝑎)(0) Slope: 𝑚 = = =0 𝑛(∑ 𝑥 2) − (∑ 𝑥) 2 3𝑎 2 − 0 2

Correlation coefficient 𝑟 =

𝑛(∑ 𝑥𝑦) − (∑ 𝑥)(∑ 𝑦)

𝑛(∑ 𝑥 2) − (∑ 𝑥) 2 ⋅

𝑛(∑ 𝑦 2) − (∑ 𝑦) 2

√ √ we have just seen that this numerator is zero. Hence, 𝑟 = 0.

has the same numerator as 𝑚, and

43. No. The regression line through (−1, 1), (0, 0), and (1, 1) passes through none of these points. 44. A mathematical model may only be valid for a limited range of values of the variables concerned, and extrapolation can lead to absurd results. 45. (Answers may vary.) The data in Exercise 35 give 𝑟 ≈ 0.97, yet the plotted points suggest a curve, not a straight line.

46. (Answers may vary.) If 𝑟 is not close to 1, then the points are not close to the regression line; they may be scattered randomly above and below the line in a manner not suggesting a parabola.


Solutions Chapter 1 Review Chapter 1 Review

1. a. 1 b. −2 c. 0 d. 𝑓(2) − 𝑓(−2) = 0 − 1 = −1

2. a. −1 b. −3 c. 0 d. 𝑓(2) − 𝑓(−2) = 0 − (−1) = 1

3. a. 1 b. 0 c. 0 d. 𝑓(1) − 𝑓(−1) = 0 − 1 = −1

4. a. 2 b. −1 c. 0 d. 𝑓(1) − 𝑓(−1) = 0 − 2 = −2

5. 𝑦 = −2𝑥 + 5 𝑦-intercept = 5, slope = −2

6. 2𝑥 − 3𝑦 = 12 Solving for 𝑦 gives

−3𝑦 = −2𝑥 + 12 𝑦 = 23 𝑥 − 4 : 𝑦-intercept = −4, slope = 23


7. 𝑦 =

1 𝑥 2

{𝑥 − 1

if − 1 ≤ 𝑥 ≤ 1

Solutions Chapter 1 Review

if 1 < 𝑥 ≤ 3

8. 𝑓(𝑥) = 4𝑥 − 𝑥 2 with domain [0, 4] Technology formula: 4*x-x^2

9. The graph of the function has a V-shape, indicating an absolute value function.

10. The graph of the function is a straight line, indicating a linear function.

11. The graph of the function is a straight line, indicating a linear function.

12. In the graph, 𝑦 doubles for each 1-unit increase in 𝑥, indicating an exponential model.


14. In the graph, 𝑦 is halved for each 1-unit increase in 𝑥, indicating an exponential model.

Solutions Chapter 1 Review 13. The parabolic shape of the graph indicates a quadratic model.

15. Through (3, 2) with slope −3 Point: (3, 2) Slope: 𝑚 = −3 Intercept: 𝑏 = 𝑦1 − 𝑚𝑥1 = 2 − (−3)(3) = 2 + 9 = 11 Thus, the equation is 𝑦 = 𝑚𝑥 + 𝑏 = −3𝑥 + 11. 16. Through (−2, 4) with slope −1 Point: (−2, 4) Slope: 𝑚 = −2 Intercept: 𝑏 = 𝑦1 − 𝑚𝑥1 = 4 − (−1)(−2) = 2 Thus, the equation is 𝑦 = 𝑚𝑥 + 𝑏 = −𝑥 + 2. 17. Through (1, −3) and (5, 2)

𝑦2 − 𝑦1 2 − (−3) 5 = = = 1.25 𝑥2 − 𝑥1 5−1 4 Intercept: 𝑏 = 𝑦1 − 𝑚𝑥1 = −3 − (1.25)(1) = −4.25 Thus, the equation is 𝑦 = 𝑚𝑥 + 𝑏 = 1.25𝑥 − 4.25. Point: (1, −3)

Slope: 𝑚 =

18. Through (−1, 2) and (1, 0)

𝑦2 − 𝑦1 0−2 −2 = = = −1 𝑥2 − 𝑥1 1 − (−1) 2 𝑏 = 𝑦1 − 𝑚𝑥1 = 2 − (−1)(−1) = 1 Thus, the equation is 𝑦 = 𝑚𝑥 + 𝑏 = −𝑥 + 1. Point: (−1, 2)

Slope: 𝑚 =

Intercept:

19. Through (1, 2) parallel to 𝑥 − 2𝑦 = 2 Point: (1, 2) Slope: Same as slope of 𝑥 − 2𝑦 = 2. To find the slope, solve for 𝑦 : −2𝑦 = −𝑥 − 2 1 1 𝑦 = 𝑥 + 1, so that 𝑚 = . 2 2 1 3 Intercept: 𝑏 = 𝑦1 − 𝑚𝑥1 = 2 − (1) = 2 2 1 3 Thus, the equation is 𝑦 = 𝑚𝑥 + 𝑏 = 𝑥 + . 2 2 20. Through (−3, 1) parallel to −2𝑥 − 4𝑦 = 5 Point: (−3, 1)

1 5 Slope: Same as slope of −2𝑥 − 4𝑦 = 5. To find the slope, solve for 𝑦, getting 𝑦 = − 𝑥 − . 2 4 1 Thus, 𝑚 = − . 2 1 1 Intercept: 𝑏 = 𝑦1 − 𝑚𝑥1 = 1 + (−3) = − 2 2 1 1 Thus, the equation is 𝑦 = 𝑚𝑥 + 𝑏 = − 𝑥 − . 2 2


Solutions Chapter 1 Review 21. With slope 4 crossing 2𝑥 − 3𝑦 = 6 at its 𝑥-intercept We need the 𝑥-intercept of 2𝑥 − 3𝑦 = 6. This is given by setting 𝑦 = 0 and solving for 𝑥 : 2𝑥 − 0 = 6 𝑥=3 Thus, the point is (3, 0) because 𝑦 = 0 on the 𝑥-axis. Slope: 𝑚 = 4 Intercept: 𝑏 = 𝑦1 − 𝑚𝑥1 = 0 − 4(3) = −12 Thus, the equation is 𝑦 = 𝑚𝑥 + 𝑏 = 4𝑥 − 12 22. With slope 1/2 crossing 3𝑥 + 𝑦 = 6 at its 𝑥-intercept We need the x-intercept of 3𝑥 + 𝑦 = 6. This is given by setting 𝑦 = 0 and solving for 𝑥 : 3𝑥 + 0 = 6 𝑥=2 Thus, the point is (2, 0) because 𝑦 = 0 on the 𝑥-axis. 1 Slope: 𝑚 = 2 1 Intercept: 𝑏 = 𝑦1 − 𝑚𝑥1 = 0 − (2) = −1 2 Thus, the equation is 1 𝑦 = 𝑚𝑥 + 𝑏 = 𝑥 − 1 2 23. 𝑦 = −𝑥∕2 + 1 : 𝑥

−1

Observed 𝑦 Predicted 𝑦 Residual2 1

1.5

0.25

1

2

0.5

2.25

2

0

0

0

SSE:

2.5

𝑦 = −𝑥∕4 + 1 : 𝑥

−1

Observed 𝑦 Predicted 𝑦 Residual2 1

1.25

0.0625

1

2

0.75

1.5625

2

0

0.5

0.25

SSE:

1.875

The second line, 𝑦 = −𝑥∕4 + 1, is a better fit.


24. 𝑦 = 𝑥 + 1 : 𝑥

−2

Solutions Chapter 1 Review

Observed 𝑦 Predicted 𝑦 Residual2 −1

−1

1

0

1

0

1

1

0

1

2

2

0

2

4

3

1

3

3

4

1

SSE:

3

−1

𝑦 = 𝑥∕2 + 1 :

0

𝑥

Observed 𝑦 Predicted 𝑦 Residual2

−1

0

1

1

0.5

0.25

0

1

1

0

1

2

1.5

0.25

2

4

2

4

3

3

2.5

0.25

SSE:

5.75

−2

−1

The first line, 𝑦 = 𝑥 + 1, is the better fit. 25.

𝑥

𝑦

−1

𝑥𝑦

−1

1

𝑥2

𝑦2

1

1

1

2

2

1

4

2

0

0

4

0

2

3

1

6

5

(The bottom row contains the column sums.)

𝑛 = 3 (number of data points) 𝑛(∑ 𝑥𝑦) − (∑ 𝑥)(∑ 𝑦) 3(1) − (2)(3) −3 Slope: 𝑚 = ≈ −0.214 = = 14 𝑛(∑ 𝑥 2) − (∑ 𝑥) 2 3(6) − 2 2 Intercept: 𝑏 =

∑ 𝑦 − 𝑚(∑ 𝑥) 3 − (−0.214)(2) ≈ 1.14 = 𝑛 3

Thus, the regression line is 𝑦 = 𝑚𝑥 + 𝑏 = −0.214𝑥 + 1.14. 𝑟=

𝑛(∑ 𝑥𝑦) − (∑ 𝑥)(∑ 𝑦)

𝑛(∑ 𝑥 2) − (∑ 𝑥) 2 ⋅

𝑛(∑ 𝑦 2) − (∑ 𝑦) 2

=

3(1) − (2)(3)

√3(6) − 2 2√3(5) − 3 2

≈ −0.33


Solutions Chapter 1 Review 26.

𝑥

𝑦

−2

−1

−1

1

𝑥𝑦

𝑥2

𝑦2

2

−1

4

1

1

1

0

1

0

0

1

1

2

2

1

4

2

4

8

4

16

3

3

9

9

9

3

10

20

19

32

(The bottom row contains the column sums.)

𝑛 = 6 (number of data points) 𝑛(∑ 𝑥𝑦) − (∑ 𝑥)(∑ 𝑦) 6(20) − (3)(10) 90 Slope: 𝑚 = ≈ 0.857 = = 2 2 2 105 𝑛(∑ 𝑥 ) − (∑ 𝑥) 6(19) − 3 Intercept: 𝑏 =

∑ 𝑦 − 𝑚(∑ 𝑥) 10 − (0.857)(3) ≈ 1.24 = 𝑛 6

Thus, the regression line is 𝑦 = 𝑚𝑥 + 𝑏 = 0.857𝑥 + 1.24. 𝑟=

𝑛(∑ 𝑥𝑦) − (∑ 𝑥)(∑ 𝑦)

𝑛(∑ 𝑥 2) − (∑ 𝑥) 2 ⋅

𝑛(∑ 𝑦 2) − (∑ 𝑦) 2

=

6(20) − (3)(10)

√6(19) − 3 2√6(32) − 10 2

≈ 0.92

27. a. Graph:

Since the data definitely suggests a curve, we rule out a linear function, leaving us with a choice of quadratic or exponential. Of the two, an exponential function would fit best, given the leveling off we see on the left; the graph of a quadratic function would not flatten out, but instead form a low point and begin rising again toward the left.b. The ratios (rounded to 1 decimal place) are: 𝑉 (1)∕𝑉 (0) 𝑉 (2)∕𝑉 (1) 300 =3 100

𝑉 (3)∕𝑉 (2)

𝑉 (4)∕𝑉 (3)

𝑉 (5)∕𝑉 (4)

𝑉 (6)∕𝑉 (5)

1,000 3,300 10,500 33,600 107,400 ≈ 3.3 ≈ 3.2 ≈ 3.2 ≈ 3.2 = 3.3 300 1,000 3,300 10,500 33,600

They are close to 3.2. c. The data suggest that website traffic is increasing by a factor of around 3.2 per year, so the prediction for next year (year 6) would be around 3.2×107,400 ≈ 343,700 visits per day.


Solutions Chapter 1 Review 28. a. Graph:

Since the data definitely suggest a curve, we rule out a linear function, leaving us with a choice of quadratic or exponential. Of the two, a quadratic function would fit best, given the parabolic shape of the graph. b. The differences (rounded to 1 decimal place) are: 𝐶(1) − 𝐶(0)

−0.32 ≈ −0.3

𝐶(2) − 𝐶(1) 𝐶(3) − 𝐶(2) 𝐶(4) − 𝐶(3) 𝐶(5) − 𝐶(4) −0.1

0.12 ≈ 0.1

0.28 ≈ 0.3

0.48 ≈ 0.5

The rounded differences increase linearly with slope 0.2. c. Assuming the linear trend of differences continue, the next difference 𝐶(6) − 𝐶(5) will be around 0.7, so that the cost of a paperback will be about $5.88 + 0.70 = $6.58.

0.03𝑥 + 2 if 0 ≤ 𝑥 ≤ 50 {0.05𝑥 + 1 if 𝑥 > 50 Notice that 𝑥 is thousands of visit per day, so 10,000 visits corresponds to 𝑥 = 10, and the servers will crash an average of 𝑐(10) = 0.03(10) + 2 = 2.3 times per day. (We used the first formula because 10 is in the interval [0, 50].) For 50,000 visitors, 𝑐(50) = 0.03(50) + 2 = 3.5 crashes per day. (We again used the first formula because 50 is still in the interval [0, 50].) For 100,000 visitors, 𝑐(100) = 0.05(100) + 1 = 6 crashes per day. (We used the second formula because 100 is in the interval (50, +∞).) b. The coefficient 0.03 is the slope of the first formula, indicating that, for website traffic of up to 50,000 visits per day (0 ≤ 𝑥 ≤ 50), the number of crashes is increasing by 0.03 per additional thousand visits. c. To experience 8 crashes in a day, we desire 𝑐(𝑥) = 8. If we try the first formula, we get 0.03𝑥 + 2 = 8 giving 𝑥 = (8 − 2)∕0.03 = 200 , which is not in the domain of the first formula. So, we try the second formula: 0.05𝑥 + 1 = 8 7 0.05𝑥 = 7, so 𝑥 = = 140, 0.05 which is in the domain of the second formula. Thus, we estimate that there were 140,000 visitors that day. 29. a. 𝑐(𝑥) =

1.55𝑥 if 0 ≤ 𝑥 ≤ 100 {1.75𝑥 − 20 if 100 < 𝑥 ≤ 250 𝑠(60) = 1.55(60) = 93 books per day (We used the first formula, since 60 is in [0, 100].) 𝑠(100) = 1.55(100) = 155 books per day (We used the first formula, since 100 is in [0, 100].) 𝑠(160) = 1.75(160) − 20 = 260 books per day (We used the second formula, since 160 is in (100, 250].) b. The coefficient 1.75 is the slope of the second formula, measured in books sold per thousand visitors. Thus, book sales are increasing at a rate of 1.75 books per thousand new visitors when the number of visitors is between 100,000 and 250,000 per day. c. To sell an average of 300 books per day, we desire 𝑛(𝑥) = 300. If we try the first formula, we get 1.55𝑥 = 300, giving 𝑥 ≈ 194, which is not in the domain of the first formula. So, we try the second formula: 30. a.

𝑠(𝑥) =


1.75𝑥 − 20 = 300

Solutions Chapter 1 Review

320 ≈ 182.9 thousand visitors, 1.75 which is in the domain of the second formula. Thus, about 182,900 visitors per day will result in average sales of 300 books per day. 1.75𝑥 = 320, so 𝑥 =

31.

𝑡

1

𝑛(𝑡) 12.5

2

3

4

5

6

37.5

62.5

72.0

74.5

75.0

(a) Technology formulas: (A): 300/(4+100*5^(-t)) (C): -2.3*t^2+30.0*t-3.3

(B): 13.3*t+8.0 (D): 7*3^(0.5*t)

Here are the values for the four given models (rounded to 1 decimal place): 𝑡

1

2

3

4

5

6

(A) 12.5

37.5

62.5

72.1

74.4

74.9

(B) 21.3

34.6

47.9

61.2

74.5

87.8

(C) 24.4

47.5

66.0

79.9

89.2

93.9

(D) 12.1

21.0

36.4

63.0 109.1 189.0

Model (A) gives an almost perfect fit, whereas the other models are not even close. b. Looking at the table, we see the following behavior as 𝑡 increases: (A) Leveling off (B) Rising (C) Rising (begins to fall after 7 months, however) (D) Rising 32.

𝑡

1

2

𝑛(𝑡) 1,330 520

3

4

5

520 1,340 2,980

(a) Technology formulas: (A): 3000/(1+12*2^(-t)) (C): 300*1.6^t

(B): 2000/(4.2-0.7*t) (D): 100*(4.1*t^2-20.4*t+29.5)

Here are the values for the three given models (rounded to the nearest integer): 𝑡

1

2

(A)

429

750 1,200 1,714 2,182

(B)

571

714

(C)

480

768 1,229 1,966 3,146

(D) 1,320 510

3

952

520

4

5

1,429 2,857

1,350 3,000

Model (D) gives a close fit, whereas the other models are not even close. b. If you extrapolate the models, you find the following behavior: (A) Leveling off (B) Becomes undefined and then negative (C) Rising (D) Rising


Solutions Chapter 1 Review 33. a. Using 𝑣(𝑐) = −0.000005𝑐 + 0.085𝑐 + 1,750, we get 2

𝑣(5,000) = −0.000005(5,000) 2 + 0.085(5,000) + 1,750 = −125 + 425 + 1,750 = 2,050 𝑣(6,000) = −0.000005(6,000) 2 + 0.085(6,000) + 1,750 = −180 + 510 + 1,750 = 2,080

Thus, increasing monthly advertising from $5,000 to $6,000 per month would result in 2,080 − 2,050 = 30 more visits per day. b. The following table shows the result of increasing expenditure by steps of $1,000: Tech formula: -0.000005*x^2+0.085*x+1750 𝑐 5,000 6,000 7,000 8,000 9,000 10,000

𝑣(𝑐) 2,050 2,080 2,100 2,110 2,110 2,100

The successive changes in the numbers of visits are: 2,080 − 2,050 = 30; 2,100 − 2,080 = 20; 2,110 − 2,100 = 10; 2,110 − 2,110 = 0; 2,100 − 2,110 = −10, showing that the numbers of visits would increase at a slower and slower rate and then begin to decrease. c. Here is a portion of the graph of 𝑣 :

For 𝑐 = 8,500 or larger, we see that website traffic is projected to decrease as advertising increases, and then drop toward zero. Thus, the model does not appear to give a reasonable prediction of traffic at expenditures larger than $8,500 per month. 34. a. Using 𝑐(𝑛) = 0.0008𝑛 2 − 72𝑛 + 2,000,000, we get

𝑐(20,000) = 0.0008(20,000) 2 − 72(20,000) + 2,000,000 = 880,000 𝑐(30,000) = 0.0008(30,000) 2 − 72(30,000) + 2,000,000 = 560,000

Thus, increasing the run size from 20,000 to 30,000 per month would result in a savings of 880, 000 − 560, 000 = 320,000 dollars. b. The following table shows the result of increasing run size in steps of 10,000: Tech formula: 0.0008*x^2-72*x+2000000 𝑛 20,000

30,000

40,000

50,000

60,000

70,000

𝑐(𝑛) 880,000 560,000 400,000 400,000 560,000 880,000 Change

-320,000 -160,000

0

160,000 320,000

The table shows that the cost decreases at a slower and slower rate and then begins to increase. Going from 30,000 to 40,000 decreases the cost by $160,000—considerably less than going from 20,000 to 30,000.


Solutions Chapter 1 Review

c. Here is a portion of the graph of 𝑣 :

The graph shows that the cost is a minimum for a print run size of around 45,000.

𝑣2 − 𝑣1 2,100 − 2,050 50 = = = 0.05 𝑐2 − 𝑐1 6,000 − 5,000 1,000 Intercept: 𝑏 = 𝑣1 − 𝑚𝑐1 = 2,050 − (0.05)(5,000) = 1,800 Thus, the equation is 𝑣 = 𝑚𝑐 + 𝑏 = 0.05𝑐 + 1,800. b. A budget of $7,000 per month for banner ads corresponds to 𝑣 = 7,000. 𝑣(7,000) = 0.05(7,000) + 1,800 = 2,150 new visitors per day c. We are given 𝑣 = 2,500 and want 𝑐. 2,500 = 0.05𝑐 + 1,800 0.05𝑐 = 2,500 − 1,800 = 700 700 Thus, 𝑐 = = $14,000 per month. 0.05 35. a. Point: (5,000, 2,050)

Slope: 𝑚 =

𝑐2 − 𝑐1 550,000 − 880,000 330,000 = =− = −16.5 𝑛2 − 𝑛1 40,000 − 20,000 20,000 Intercept: 𝑏 = 𝑐1 − 𝑚𝑛1 = 880,000 − (−16.5)(20,000) = 1,210,000 Thus, the equation is 𝑐 = 𝑚𝑛 + 𝑏 = −16.5𝑛 + 1,210,000. b. 𝑐(25,000) = −16.5(25,000) + 1,210,000 = $797,500 c. We are given 𝑐 = 418,000 and want 𝑛. 418,000 = −16.5𝑛 + 1,210,000 −16.5𝑛 = 418,000 − 1,210,000 = −792,000 −792,000 Thus, 𝑛 = = 48,000. −16.5 36. a. Point: (20,000, 880,000)

Slope: 𝑚 =

𝑑2 − 𝑑1 93.5 − 74.5 19 = = = 0.95 𝑤2 − 𝑤1 90 − 70 20 Intercept: 𝑏 = 𝑑1 − 𝑚𝑤1 = 74.5 − (0.95)(70) = 8 Thus, the equation is 𝑑 = 𝑚𝑤 + 𝑏 = 0.95𝑤 + 8. OHagan dropped 90 m, so 𝑑 = 90, and we want 𝑤. 90 = 0.95𝑤 + 8 0.95𝑤 = 90 − 8 = 82 82 Thus, 𝑤 = ≈ 86 kg. .95 37. Point: (𝑤, 𝑑) = (70, 74.5)

Slope: 𝑚 =

𝑇2 − 𝑇1 75 − 80 −5 = = = 0.25 𝑟2 − 𝑟1 120 − 140 −20 Intercept: 𝑏 = 𝑇1 − 𝑚𝑟1 = 80 − (0.25)(140) = 45 Thus, the equation is 𝑇 = 𝑚𝑟 + 𝑏 = 0.25𝑟 + 45. 𝑇 = 65, and we want 𝑟. 65 = 0.25𝑟 + 45 0.25𝑟 = 65 − 45 = 20 38. Point: (𝑟, 𝑇 ) = (140, 80)

Slope: 𝑚 =


Thus, 𝑟 =

20 = 80 chirps/min. 0.25

Solutions Chapter 1 Review

39. a. Cost function: 𝐶 = 𝑚𝑥 + 𝑏, where 𝑏 = fixed cost = $500 per week, and 𝑚 = marginal cost = $5.50 per album Thus, the linear cost function is 𝐶 = 5.5𝑥 + 500. Revenue function: 𝑅 = 𝑚𝑥 + 𝑏, where 𝑏 = fixed revenue = 0, and 𝑚 = marginal revenue = $9.50 per album Thus, the linear revenue function is 𝑅 = 9.5𝑥. Profit function: 𝑃 = 𝑅 − 𝐶 𝑃 = 9.5𝑥 − (5.5𝑥 + 500) = 4𝑥 − 500 b. For breakeven, 𝑃 = 0 4𝑥 − 500 = 0 4𝑥 = 500 500 𝑥= = 125 albums per weekTo make a profit, the company should sell more than this number. 4 c. 𝑅 = 8.00𝑥 𝑃 = 8𝑥 − (5.5𝑥 + 500) = 2.5𝑥 − 500 For breakeven, 500 2.5𝑥 − 500 = 0, so 𝑥 = = 200 2.5 To make a profit, the company should sell more than this number.

40. a. Cost function: 𝐶 = 𝑚𝑥 + 𝑏, where 𝑏 = fixed cost = $900 per month, and 𝑚 = marginal cost = $4 per novel Thus, the linear cost function is 𝐶 = 4𝑥 + 900. Revenue function: 𝑅 = 𝑚𝑥 + 𝑏, where 𝑏 = fixed revenue = 0, and 𝑚 = marginal revenue = $5.50 per novel Thus, the linear revenue function is 𝑅 = 5.50𝑥. Profit function: 𝑃 = 𝑅 − 𝐶 𝑃 = 5.50𝑥 − (4𝑥 + 900) = 5.50𝑥 − 4𝑥 − 900 = 1.50𝑥 − 900b. For breakeven, 𝑃 = 0 1.50𝑥 − 900 = 0 1.50𝑥 = 900 900 𝑥= = 600 novels per monthc. 𝑅 = 5.00𝑥 1.50 𝑃 = 5.00𝑥 − (4𝑥 + 900) = 5𝑥 − 4𝑥 − 900 = 𝑥 − 900 For breakeven, 𝑥 − 900 = 0, so 𝑥 = 900 novels per month 41. a. Demand: We are given two points: (𝑝, 𝑞) = (7, 500) and (9.5, 300) 𝑞 − 𝑞1 300 − 500 −200 Slope: 𝑚 = 2 = = = −80 𝑝2 − 𝑝1 9.5 − 7 2.5 Intercept: 𝑏 = 𝑞1 − 𝑚𝑝1 = 500 − (−80)(7) = 500 + 560 = 1,060 Thus, the demand equation is 𝑞 = 𝑚𝑝 + 𝑏 = −80𝑝 + 1,060. b. When 𝑝 = $12, the demand is 𝑞 = −80(12) + 1,060 = 100 albums per week c. From Exercise 39, the cost function is 𝐶 = 5.5𝑞 + 500 = 5.5(−80𝑝 + 1,060) + 500 = −440𝑝 + 5,830 + 500 𝐶 = −440𝑝 + 6,330

We use 𝑞 for the monthly sales rather than 𝑥.

We want everything expressed in terms of 𝑝, so we used the demand equation.

To compute the profit in terms of price, we need the revenue as well:


Solutions Chapter 1 Review 𝑅 = 𝑝𝑞 = 𝑝(−80𝑝 + 1,060) = −80𝑝 2 + 1,060𝑝Profit: 𝑃 = 𝑅 − 𝐶𝑃 = −80𝑝 2 + 1,060𝑝 − (−440𝑝 + 6,330) = −80𝑝 2 + 1,500𝑝 − 6,330 Now we compare profits for the three prices: 𝑃 (7.00) = −80(7) 2 + 1,500(7) − 6,330 = $250 𝑃 (9.50) = −80(9.5) 2 + 1,500(9.5) − 6,330 = $700 𝑃 (12) = −80(12) 2 + 1,500(12) − 6,330 = $150 Thus, charging $9.50 will result in the largest weekly profit of $700. 42. a. Demand: We are given two points: (𝑝, 𝑞) = (10, 350) and (5.5, 620) 𝑞 − 𝑞1 620 − 350 270 Slope: 𝑚 = 2 = = = −60 𝑝2 − 𝑝1 5.5 − 10 −4.5 Intercept: 𝑏 = 𝑞1 − 𝑚𝑝1 = 350 − (−60)(10) = 350 + 600 = 950 Thus, the demand equation is 𝑞 = 𝑚𝑝 + 𝑏 = −60𝑝 + 950. b. When 𝑝 = $15, the demand is 𝑞 = −60(15) + 950 = −900 + 950 = 50 novels per month c. From Exercise 40, the cost function is 𝐶 = 4𝑞 + 900 = 4(−60𝑝 + 950) + 900

= −240𝑝 + 3,800 + 900 𝐶 = −240𝑝 + 4,700

We use 𝑞 for the monthly sales rather than 𝑥.

We want everything expressed in terms of 𝑝, so we used the demand equation.

To compute the profit in terms of price, we need the revenue as well: 𝑅 = 𝑝𝑞 = 𝑝(−60𝑝 + 950) = −60𝑝 2 + 950𝑝 Profit: 𝑃 = 𝑅 − 𝐶 𝑃 = −60𝑝 2 + 950𝑝 − (−240𝑝 + 4,700) = −60𝑝 2 + 1,190𝑝 − 4,700 Now we compare profits for the three prices: 𝑃 (5.50) = −60(5.5) 2 + 1,190(5.5) − 4,700 = $30 𝑃 (10) = −60(10) 2 + 1,190(10) − 4,700 = $1,200 𝑃 (15) = −60(15) 2 + 1,190(15) − 4,700 = −$350 (loss) Thus, charging $10 will result in the largest monthly profit of $1,200. 43. a. Calculation of the regression line: 𝑥

𝑦

𝑥𝑦

𝑥2

8

440

3,520

64

8.5

380

3,230

72.25

10

250

2,500

100

11.5

180

2,070 132.25

38

1,250 11,320 368.5

(The bottom row contains the column sums.)

𝑛 = 4 (number of data points) 𝑛(∑ 𝑥𝑦) − (∑ 𝑥)(∑ 𝑦) 4(11,320) − (38)(1,250) −2,220 Slope: 𝑚 = = = = −74 30 𝑛(∑ 𝑥 2) − (∑ 𝑥) 2 4(368.5) − 38 2 Intercept: 𝑏 =

∑ 𝑦 − 𝑚(∑ 𝑥) 1,250 − (−74)(38) = = 1,015.5 𝑛 4


Solutions Chapter 1 Review Thus, the regression line is 𝑦 = 𝑚𝑥 + 𝑏 = −74𝑥 + 1,015.5. Using the variable names 𝑝 and 𝑞 makes this equation 𝑞 = −74𝑝 + 1,015.5. b. 𝑞(10.50) = −74(10.50) + 1,015.5 = 238.5 ≈ 239 albums per week 44. a. Calculation of the regression line: 𝑥

𝑦

𝑥𝑦

𝑥2

5.5

620

3,410

30.25

10

350

3,500

100

11.5

350

4,025 132.25

12

300

3,600

39

1,620 14,535 406.5

144

(The bottom row contains the column sums.)

𝑛 = 4 (number of data points) 𝑛(∑ 𝑥𝑦) − (∑ 𝑥)(∑ 𝑦) 4(14,535) − (39)(1,620) −5,040 Slope: 𝑚 = = = = −48 105 𝑛(∑ 𝑥 2) − (∑ 𝑥) 2 4(406.5) − 39 2 Intercept: 𝑏 =

∑ 𝑦 − 𝑚(∑ 𝑥) 1,620 − (−48)(39) = = 873 𝑛 4

Thus, the regression line is 𝑦 = 𝑚𝑥 + 𝑏 = −48𝑥 + 873. Using the variable names 𝑝 and 𝑞 makes this equation 𝑞 = −48𝑝 + 873. b. 𝑞(8) = −48(8) + 873 = 489 novels per month


Solutions Chapter 1 Case Study Chapter 1 Case Study

1. Here is the given data, with 𝑡 = 0 representing 2020: 𝑡

𝑦

0

1

2

3

4

5

6

0

0.3

1.5

2.6

3.4

4.3

5.0

Using technology, we obtain the following linear regression model: 𝑦 = 0.8893𝑡 − 0.225; 𝑟 ≈ 0.9948 Since 𝑚 ≈ 0.889, MeTube viewership is increasing at a rate of about 0.889 billion views per year. 2. Graph:

The graph does not suggest a quadratic model because the plotted points do not suggest a parabola. 3. Graph with regression parabola:

Regression equation: 𝑦 = −0.001190𝑡 2 + 0.8964𝑡 − 0.2310 The parabola appears to fit no better than the regression line, suggesting that the quadratic model is not appropriate. 4. Here is the tabulated data together with the result of =LINEST(A2:A8,B2:C8,,TRUE):

𝑝 is computed using =TDIST(ABS(E1/E2),F4,2), and returns the value 𝑝 ≈ 0.9662. There is a very low degree of confidence, 1 − 𝑝 ≈ 0.0338, or 3.38%, we can have in asserting that the coefficient of 𝑥 2 is not zero (or that a quadratic model is needed) and so a quadratic model is not appropriate.


Solutions Section 2.1 Section 2.1

1. 𝑓(𝑥) = 2𝑥 2 − 𝑥 − 2 a. 𝑎 = 2, 𝑏 = −1, 𝑐 = −2 b. Table of values: 𝑥

𝑓(𝑥)

−3 19

−2 8

−1 1

0

1

−2

−1

0

1

2

3

4

13

2

3

c. 𝑓(𝑎 + ℎ) = 2(𝑎 + ℎ) 2 − (𝑎 + ℎ) − 2 = 2(𝑎 2 + 2𝑎ℎ + ℎ 2) − (𝑎 + ℎ) − 2 = 2𝑎 2 + 4𝑎ℎ + 2ℎ 2 − 𝑎 − ℎ − 2 d. 2x^2-x-2 2. 𝑓(𝑥) = −2𝑥 2 + 𝑥 + 2 a. 𝑎 = −2, 𝑏 = −1, 𝑐 = 2 b. Table of values: 𝑥

−3

𝑓(𝑥) −19

−2 −8

−1 −1

2

1

−4

−13

1

2

3

c. 𝑓(𝑎 + ℎ) = −2(𝑎 + ℎ) 2 + (𝑎 + ℎ) + 2 = −2(𝑎 2 + 2𝑎ℎ + ℎ 2) + (𝑎 + ℎ) + 2 = −2𝑎 2 − 4𝑎ℎ − 2ℎ 2 + 𝑎 + ℎ + 2 d. -2x^2+x+2 3. 𝑓(𝑥) = 10𝑥 2 − 5𝑥 a. 𝑎 = 10, 𝑏 = −5, 𝑐 = 0 b. Table of values: 𝑥

−3

𝑓(𝑥) 105

−2 50

−1 15

0 0

5

30

75

1

2

3

c. 𝑓(𝑎 + ℎ) = 10(𝑎 + ℎ) 2 − 5(𝑎 + ℎ) = 10(𝑎 2 + 2𝑎ℎ + ℎ 2) − 5(𝑎 + ℎ) = 10𝑎 2 + 20𝑎ℎ + 10ℎ 2 − 5𝑎 − 5ℎ d. 10x^2-5x 4. 𝑓(𝑥) = −𝑥 2 − 50 a. 𝑎 = −1, 𝑏 = 0, 𝑐 = −50 b. Table of values: 𝑥

−3

𝑓(𝑥) −59

−2

−54

−1

−51

0

−50

−51

−54

−59

c. 𝑓(𝑎 + ℎ) = −(𝑎 + ℎ) 2 − 50 = −(𝑎 2 + 2𝑎ℎ + ℎ 2) − 50 = −𝑎 2 − 2𝑎ℎ − ℎ 2 − 50 d. -(x^2)-50 (See the margin note next to Quick Example 2 in the text as to the reason for the parentheses.) 5. 𝑓(𝑥) = −𝑥 2 − 𝑥 − 1 a. 𝑎 = −1, 𝑏 = −1, 𝑐 = −1


Solutions Section 2.1 b. Table of values: 𝑥

𝑓(𝑥)

−3 −7

−2 −3

−1 −1

0

1

2

3

−1

−3

−7

−13

0

1

2

3

c. 𝑓(𝑎 + ℎ) = −(𝑎 + ℎ) 2 − (𝑎 + ℎ) − 1 = −(𝑎 2 + 2𝑎ℎ + ℎ 2) − (𝑎 + ℎ) − 1 = −𝑎 2 − 2𝑎ℎ − ℎ 2 − 𝑎 − ℎ − 1 d. -(x^2)-x-1 (See margin note next to Quick Example 2 in the textbook as to the reason for the parentheses.) 6. 𝑓(𝑥) = −3𝑥 2 + 3𝑥 − 1 a. 𝑎 = −3, 𝑏 = 3, 𝑐 = −1 b. Table of values: 𝑥

−3

𝑓(𝑥) −37

−2

−19

−1 −7

−1

−1

−7

−19

c. 𝑓(𝑎 + ℎ) = −3(𝑎 + ℎ) 2 + 3(𝑎 + ℎ) − 1 = −3(𝑎 2 + 2𝑎ℎ + ℎ 2) + 3(𝑎 + ℎ) − 1 = −3𝑎 2 − 6𝑎ℎ − 3ℎ 2 + 3𝑎 + 3ℎ − 1 d. -3x^2+3x-1

7. 𝑓(𝑥) = 𝑥 2 + 3𝑥 + 2. 𝑎 = 1, 𝑏 = 3, 𝑐 = 2; −𝑏∕(2𝑎) = −3∕2, 𝑓(−3∕2) = −1∕4, so: vertex: (−3∕2, −1∕4), 𝑦-intercept = 𝑐 = 2. 𝑥 2 + 3𝑥 + 2 = (𝑥 + 1)(𝑥 + 2), so: 𝑥-intercepts: −2, −1. 𝑎 > 0 so the parabola opens upward.

8. 𝑓(𝑥) = −𝑥 2 − 𝑥. 𝑎 = −1, 𝑏 = −1, 𝑐 = 0; −𝑏∕(2𝑎) = −1∕2, 𝑓(−1∕2) = 1∕4, so: vertex: (−1∕2, 1∕4), 𝑦-intercept = 𝑐 = 0. −𝑥 2 − 𝑥 = −𝑥(𝑥 + 1), so: 𝑥-intercepts: −1, 0. 𝑎 < 0 so the parabola opens downward.

9. 𝑓(𝑥) = −𝑥 2 + 4𝑥 − 4. 𝑎 = −1, 𝑏 = 4, 𝑐 = −4; −𝑏∕(2𝑎) = 2, 𝑓(2) = 0, so: vertex: (2, 0), 𝑦-intercept = 𝑐 = −4. −𝑥 2 + 4𝑥 − 4 = −(𝑥 − 2) 2, so: 𝑥-intercept: 2. 𝑎 < 0 so the parabola opens downward.


Solutions Section 2.1

10. 𝑓(𝑥) = 𝑥 2 + 2𝑥 + 1. 𝑎 = 1, 𝑏 = 2, 𝑐 = 1; −𝑏∕(2𝑎) = −1, 𝑓(−1) = 0, so: vertex: (−1, 0), 𝑦-intercept = 𝑐 = 1. 𝑥 2 + 2𝑥 + 1 = (𝑥 + 1) 2, so: 𝑥-intercept: −1. 𝑎 > 0 so the parabola opens upward.

11. 𝑓(𝑥) = −𝑥 2 − 40𝑥 + 500. 𝑎 = −1, 𝑏 = −40, 𝑐 = 500; −𝑏∕(2𝑎) = −20, 𝑓(−20) = 900, so: vertex: (−20, 900), 𝑦-intercept = 𝑐 = 500. −𝑥 2 − 40𝑥 + 500 = −(𝑥 + 50)(𝑥 − 10), so: 𝑥-intercepts: −50, 10. 𝑎 < 0 so the parabola opens downward.

12. 𝑓(𝑥) = 𝑥 2 − 10𝑥 − 600. 𝑎 = 1, 𝑏 = −10, 𝑐 = −600; −𝑏∕(2𝑎) = 5; 𝑓(5) = −625, so: vertex: (5, −625), 𝑦-intercept = 𝑐 = −600. 𝑥 2 − 10𝑥 − 600 = (𝑥 + 20)(𝑥 − 30), so: 𝑥-intercepts: −20, 30. 𝑎 > 0 so the parabola opens upward.

13. 𝑓(𝑥) = 𝑥 2 + 𝑥 − 1. 𝑎 = 1, 𝑏 = 1, 𝑐 = −1; −𝑏∕(2𝑎) = −1∕2; 𝑓(−1∕2) = −5∕4, so: vertex: (−1∕2, −5∕4), 𝑦-intercept = 𝑐 = −1. From the quadratic formula: 𝑥-intercepts: −1∕2 ± √5∕2. 𝑎 > 0 so the parabola opens upward.


Solutions Section 2.1

14. 𝑓(𝑥) = 𝑥 2 + √2𝑥 + 1. 𝑎 = 1, 𝑏 = √2, 𝑐 = 1; −𝑏∕(2𝑎) = −√2∕2; 𝑓(−√2∕2) = 1∕2, so: vertex: (−√2∕2, 1∕2), 𝑦-intercept = 𝑐 = 1. 𝑏 2 − 4𝑎𝑐 = −2 < 0, so no 𝑥-intercept. 𝑎 > 0 so the parabola opens upward.

15. 𝑓(𝑥) = 𝑥 2 + 1. 𝑎 = 1, 𝑏 = 0, 𝑐 = 1; −𝑏∕(2𝑎) = 0, 𝑓(0) = 1, so: vertex: (0, 1), 𝑦-intercept = 𝑐 = 1. 𝑏 2 − 4𝑎𝑐 = −4 < 0, so no 𝑥-intercept. 𝑎 > 0 so the parabola opens upward.

16. 𝑓(𝑥) = −𝑥 2 + 5. 𝑎 = −1, 𝑏 = 0, 𝑐 = 5; −𝑏∕(2𝑎) = 0, 𝑓(0) = 5, so vertex: (0, 5), 𝑦-intercept = 𝑐 = 1. From the quadratic formula: 𝑥-intercepts: −√5, √5. 𝑎 < 0 so the parabola opens downward.

17. 𝑞 = −4𝑝 + 100, 𝑅 = 𝑝𝑞 = 𝑝(−4𝑝 + 100) = −4𝑝 2 + 100𝑝; maximum revenue when 𝑝 = −𝑏∕(2𝑎) = $12.50


Solutions Section 2.1

18. 𝑞 = −3𝑝 + 300, 𝑅 = 𝑝𝑞 = 𝑝(−3𝑝 + 300) = −3𝑝 2 + 300𝑝, maximum revenue when 𝑝 = −𝑏∕(2𝑎) = $50

19. 𝑞 = −2𝑝 + 400, 𝑅 = 𝑝𝑞 = 𝑝(−2𝑝 + 400) = −2𝑝 2 + 400𝑝, maximum revenue when 𝑝 = −𝑏∕(2𝑎) = $100

20. 𝑞 = −5𝑝 + 1200, 𝑅 = 𝑝𝑞 = 𝑝(−5𝑝 + 1200) = −5𝑝 2 + 1200𝑝, maximum revenue when 𝑝 = −𝑏∕(2𝑎) = $120

21. 𝑦 = −0.7955𝑥 2 + 4.4591𝑥 − 1.6000 22. 𝑦 = −0.7955𝑥 2 − 4.4591𝑥 − 1.6000


23. 𝑦 = −1.1667𝑥 − 6.1667𝑥 − 3.0000 2

Solutions Section 2.1

24. 𝑦 = −0.3333𝑥 2 + 1.6667𝑥 + 3.0000 25. a. Positive because the data suggest a curve that is concave up. b. The data suggest a parabola that is concave up (𝑎 positive) and with 𝑦-intercept around 1,840. Only choice (C) has both properties. c. −𝑏∕(2𝑎) = 44∕12 ≈ 4, which is 2014. Extrapolating in the positive direction leads one to predict more and more steeply rising military expenditure, which may or may not occur; extrapolating in the negative direction predicts more and more steeply increasing military expenditure as we go back in time, contradicting history — military expenditures have risen and fallen many times. 26. a. Negative because the data suggest a curve that is concave down. b. The data suggest a parabola that is concave down (𝑎 negative) and with 𝑦-intercept around 72. Only choice (B) has both properties. c. −𝑏∕(2𝑎) = 0.7∕0.16 = 4.375, which is closest to 2014. Extrapolating in either direction leads one to predict eventually negative values for the funding, which doesn't make sense. 27. The given function 𝐼(𝑡) = 11𝑡 2 − 170𝑡 + 1,300 is quadratic with graph a concave-up parabola (as 𝑎 = 11 is positive). Thus, its vertex is the lowest point on the graph and occurs when 𝑡 = −𝑏∕(2𝑎) = 170∕22 ≈ 7.73 ≈ 8, corresponding to 2018. At that time, imports were about

𝐼(10.26) = 11(7.73) 2 − 170(7.73) + 1,300 ≈ 640 thousand barrels/day (to two significant digits).

28. The given function 𝑃 (𝑡) = −0.01𝑡 2 + 0.01𝑡 + 2.5 is quadratic with graph a concave-down parabola (as 𝑎 = −0.01 is negative). Thus, its vertex is the highest point on the graph and occurs when 𝑡 = −𝑏∕(2𝑎) = 0.01∕0.02 = 0.5, midway through 2010. At that time, production was approximately 𝑃 (0.5) = −0.01(0.5) 2 + 0.01(0.5) + 2.5 ≈ 2.5 million barrels/day (to two significant digits).

29. a. The vertex of 𝑦 = 0.4𝑥 2 + 𝑥 + 26.5 occurs when 𝑥 = −𝑏∕(2𝑎) = −1∕0.8 = −1.25 ≈ −1, which would correspond to 2019. b. 𝑟(𝑥) = 0.4𝑥 2 + 𝑥 + 26.5 has domain [0, 5], and its value increases as 𝑥 increases from 0 (all the coefficients are positive). So, its lowest value occurs at 𝑥 = 0, corresponding to 2020. (One can also graph the given function to verify that its lowest value occurs at 𝑥 = 0.) c. The 𝑥-coordinate of the vertex in part (a) is outside the domain [0, 5] of 𝑟(𝑥) and so the model does not apply to that value of 𝑥. d. From the answer to part (b), the lowest revenue is 𝑟(0) = 0.4(0) 2 + 0 + 26.5 = 26.5 billion dollars. 30. a. The vertex of 𝑦 = −0.2𝑥 2 + 3𝑥 + 18 occurs when 𝑥 = −𝑏∕(2𝑎) = 3∕0.4 = 7.5 ≈ 8, which would correspond to 2028. b. 𝑟(𝑥) = −0.2𝑥 2 + 3𝑥 + 18 has domain [0, 5], and its graph is seen to increase as 𝑥 increases from 0 to 5. So, its highest value occurs at 𝑥 = 5, corresponding to 2025. c. The 𝑥-coordinate of the vertex in part (a) is outside the domain [0, 5] of 𝑟(𝑥) and so the model does not apply to that value of 𝑥. d. From the answer to part (b), the highest revenue is 𝑟(5) = −0.2(5) 2 + 3(5) + 18 = 28 billion dollars. 31. 𝑞 = −0.5𝑝 + 140. Revenue is 𝑅 = 𝑝𝑞 = −0.5𝑝 2 + 140𝑝. Maximum revenue occurs when 𝑝 = −𝑏∕(2𝑎) = $140; the corresponding revenue is 𝑅 = $9, 800. 32. 𝑞 = −2𝑝 + 320. Revenue is 𝑅 = 𝑝𝑞 = −2𝑝 2 + 320𝑝. Maximum revenue occurs when


Solutions Section 2.1 𝑝 = −𝑏∕(2𝑎) = $80; the corresponding revenue is 𝑅 = $12, 800.

33. The given data points are (𝑝, 𝑞) = (40, 200, 000) and (60, 160, 000). The line passing through these points is 𝑞 = −2000𝑝 + 280, 000. Revenue is 𝑅 = 𝑝𝑞 = −2000𝑝 2 + 280, 000𝑝. Maximum revenue occurs when 𝑝 = −𝑏∕(2𝑎) = 70 houses; the corresponding revenue is 𝑅 = $9, 800, 000. 34. The given data points are (𝑝, 𝑞) = (50, 190, 000) and (70, 170, 000). The line passing through these points is 𝑞 = −1000𝑝 + 240, 000. Revenue is 𝑅 = 𝑝𝑞 = −1000𝑝 2 + 240, 000𝑝. Maximum revenue occurs when 𝑝 = −𝑏∕(2𝑎) = 120 houses; the corresponding revenue is 𝑅 = $14, 400, 000. 35. a. A linear demand function has the form 𝑞 = 𝑚𝑝 + 𝑏 (𝑥 is the price 𝑝 and 𝑦 is the demand 𝑞). We are given two points on its graph: (3, 28, 000) and (5, 19, 000). Slope: 𝑚 = 𝑝2 −𝑝1 = 19,000−28,000 = −9,000 = −4, 500 5−3 2 𝑞 −𝑞 2

1

Intercept:

𝑏 = 𝑞1 − 𝑚𝑝1 = 28, 000 − (−4, 500)(3) = 28, 000 + 13, 500 = 41, 500

Thus, the demand equation is 𝑞 = 𝑚𝑝 + 𝑏 = −4, 500𝑝 + 41, 500. b. Revenue: 𝑅= 𝑝𝑞 = 𝑝(−4, 500𝑝 + 41, 500)= −4, 500𝑝 2 + 41, 500𝑝

For maximum revenue,

𝑝 = −𝑏 ≈$4.61. = −41,500 2𝑎 −900

The corresponding daily revenue is

𝑅 = −4500(4.61) 2 + 41, 500(4.61) = $95, 680.55.

c. The maximum annual revenue the company could have earned was 95, 680.55 × 365≈$34, 923, 400, which is about $10 million short of what it needed to break even. Therefore, it would not have been possible to break even.

36. a. A linear demand function has the form 𝑞 = 𝑚𝑝 + 𝑏 (𝑥 is the price 𝑝 and 𝑦 is the demand 𝑞). We are given two points on its graph: (5, 14) and (3, 18). Slope: 4 𝑚 = 𝑝2 −𝑝1 = 18−14 = −2 = −2 3−5 𝑞 −𝑞 2

1

Intercept:

𝑏 = 𝑞1 − 𝑚𝑝1 = 14 − (−2)(5) = 24

Thus, the demand equation is 𝑞 = 𝑚𝑝 + 𝑏 = −2𝑝 + 24. b. Revenue: 𝑅= 𝑝𝑞 = 𝑝(−2𝑝 + 24)= −2𝑝 2 + 24𝑝

For maximum revenue,

𝑝 = −𝑏 = −24 =Ż6. 2𝑎 −4

The corresponding daily revenue is


𝑅 = −2(6) 2 + 24(6) =Ż72 million.

Solutions Section 2.1

c. The maximum annual revenue the company could have earned was 72, 000, 000 × 670 =Ż48,240,000,000, which is about Ż240 million more than it needed to finance the lab. Therefore, it would have been possible to finance the lab.

37. a. The data points are (𝑥, 𝑞) = (2, 280) and (1.5, 560). Thus, 𝑞 = −560𝑥 + 1, 400 and 𝑅 = 𝑥𝑞 = −560𝑥 2 + 1, 400𝑥. b. 𝑃 = 𝑅 − 𝐶 = −560𝑥 2 + 1400𝑥 − 30. The largest monthly profit occurs when 𝑥 = −𝑏∕(2𝑎) = $1.25 and then 𝑃 = $845 per month.

38. a. The data points are (𝑥, 𝑞) = (8, 400) and (4, 600). Thus, 𝑞 = −50𝑥 + 800 and 𝑅 = −50𝑥 2 + 800𝑥. b. 𝑃 = 𝑅 − 𝐶 = −50𝑥 2 + 800𝑥 − 500. The largest weekly profit occurs when 𝑥 = −𝑏∕(2𝑎) = $8 per T-shirt and then 𝑃 = $2700 per week. 39. As a function of 𝑞, 𝐶 = 0.5𝑞 + 20. Substituting 𝑞 = −400𝑥 + 1, 200, we get 𝐶 = 0.5(−400𝑥 + 1, 200) + 20 = −200𝑥 + 620.

The profit is 𝑃 = 𝑅 − 𝐶 = 𝑥𝑞 − 𝐶 = −400𝑥 2 + 1400𝑥 − 620. The profit is largest when 𝑥 = −𝑏∕(2𝑎) =$1.75 per log-on; the corresponding profit is 𝑃 =$605 per month. 40. As a function of 𝑞, 𝐶 = 5𝑞 + 400. Substituting for 𝑞, we get 𝐶 = 5(−40𝑥 + 600) + 400 = −200𝑥 + 3400.

The profit is 𝑃 = 𝑅 − 𝐶 = 𝑥𝑞 − 𝐶 = −40𝑥 2 + 800𝑥 − 3400. The profit is largest when 𝑥 = −𝑏∕(2𝑎) =$10 per T-shirt; the corresponding profit is 𝑃 =$600 per week.

41. a. The data points are (𝑝, 𝑞) = (10, 300) and (15, 250), so 𝑞 = −10𝑝 + 400. b. 𝑅 = 𝑝𝑞 = −10𝑝 2 + 400𝑝 c. 𝐶 = 3𝑞 + 3000 = 3(−10𝑝 + 400) + 3, 000 = −30𝑝 + 4, 200 d. 𝑃 = 𝑅 − 𝐶 = −10𝑝 2 + 430𝑝 − 4, 200. The maximum profit occurs when 𝑝 = −𝑏∕(2𝑎) =$21.50.

42. a. The data points are (𝑝, 𝑞) = (2500, 15) and (2000, 20), so 𝑞 = −0.01𝑝 + 40. b. 𝑅 = 50𝑝𝑞 = 50𝑝(−0.01𝑝 + 40) = −0.5𝑝2 + 2000𝑝 c. (i) 𝐶 = 120, 000 + 80, 000𝑞, so (ii) 𝐶 = 120, 000 + 80, 000(−0.01𝑝 + 40) = −800𝑝 + 3, 320, 000. d. 𝑃 = 𝑅 − 𝐶 = −0.5𝑝 2 + 2800𝑝 − 3, 320, 000. The maximum profit occurs when 𝑝 = −𝑏∕(2𝑎) =$2800 per hour.


Solutions Section 2.1 43. Here is the Excel tabulation of the data, together with the scatter plot and the quadratic (polynomial order 2) trendline (with the option "Display equation on chart" checked):

From the trendline, the quadratic model is 𝑓(𝑡) = 6𝑡 2 − 46𝑡 + 1,840. To estimate world military expenditure in 2017, substitute the corresponding value 𝑡 = 7, to obtain 𝑓(7) = 6(7) 2 − 46(7) + 1,840 = 1,812 representing $1,812 billion. The actual figure (from Exercise 25) is $1,850 billion, so the predicted value is $38 billion lower than the actual value. 44. Here is the Excel tabulation of the data, together with the scatter plot and the quadratic (polynomial order 2) trendline (with the option "Display equation on chart" checked):

From the trendline, the quadratic model is 𝑓(𝑡) = −0.0731𝑡 2 + 0.616𝑡 + 72.08. To estimate the amount of funding in 2015, substitute the corresponding value 𝑡 = 5, to obtain 𝑓(5) = −0.0731(5) 2 + 0.616(5) + 72.08 ≈ 73.33, or $73.33 billion. The actual amount of funding (from Exercise 26) was $73.33 billion which agrees with the predicted figure to two decimal places.


Solutions Section 2.1 45. a. Here is the Excel tabulation of the data, together with a scatter plot and quadratic (polynomial order 2) trendline. The option "Display equation on chart" has been checked.

We round the coefficients to two significant digits to get 𝑆(𝑡) = −0.70𝑡 2 − 6.0𝑡 + 50. b. To predict the sales in 2015 we substitute 𝑡 = 5 and compute 𝑆(5) = −0.70(5) 2 − 6.0(5) + 50 = 2.5 ≈ 3 million units. \\Likewise, the 2016 prediction is 𝑆(6) = −0.70(6) 2 − 6.0(6) + 50 = −11.2 ≈ −11 million units. Even though we expected 2015 sales to be much lower than the 2014 sales, the 2016 prediction, being negative, shows the danger of extrapolating curve-fitting models. 46. a. Here is the Excel tabulation of the data, together with a scatter plot and quadratic (polynomial order 2) trendline. The option "Display equation on chart" has been checked.

We round the coefficients to two significant digits to get 𝑆(𝑡) = −3.2𝑡 2 + 20𝑡 + 22. b. To predict the sales in 2010 we substitute 𝑡 = 5 and compute 𝑆(5) = −3.2(5) 2 + 20(5) + 22 = 42 million units. \\Likewise, the 2011 prediction is 𝑆(6) = −3.2(6) 2 + 20(6) + 22 = 26.8 ≈ 27 million units. These values are far from the figures of 50.4 and 42.6 million units given in the table in the preceding exercise. This shows the danger of shows the danger of extrapolating curve-fitting models.


Solutions Section 2.1 47. If 𝑎 = 0, then 𝑓(𝑥) = 𝑏𝑥 + 𝑐, a linear function, so its graph is a straight line.

48. Since 𝑐 is the 𝑦-intercept of the graph, 𝑐 = 0 implies that the graph has a 𝑦-intercept of zero, meaning that it passes through the origin.

49. Since the curve is concave up, 𝑎 is positive. Since the 𝑦-intercept is negative, 𝑐 is negative. Hence the correct choice is (C). 50. Since the curve is concave down, 𝑎 is negative. Since the 𝑦-intercept is positive, 𝑐 is positive. Hence the correct choice is (B).

51. Positive; The 𝑥-coordinate of the vertex is negative, so −𝑏∕(2𝑎) must be negative. Since 𝑎 is positive (the parabola is concave up), this means that 𝑏 must also be positive to make −𝑏∕(2𝑎) negative. 52. Positive; The 𝑥-coordinate of the vertex is positive, so −𝑏∕2𝑎 must be positive. Since 𝑎 is negative (the parabola is concave down), this means that 𝑏 must be positive to make −𝑏∕2𝑎 positive.

53. The 𝑥-coordinate of the vertex represents the unit price that leads to the maximum revenue, the 𝑦-coordinate of the vertex gives the maximum possible revenue, the 𝑥-intercepts give the unit prices that result in zero revenue, and the 𝑦-intercept gives the revenue resulting from zero unit price (which is obviously zero). 54. The 𝑥-coordinate of the vertex gives the time at which the stone reaches its highest point, the 𝑦-coordinate of the vertex gives the maximum height, the 𝑥-intercepts give the times that the stone is at zero height, while the 𝑦-intercept gives the height of the stone at time zero. 55. Graph the data to see whether the points suggest a curve rather than a straight line. If the curve suggested by the graph is concave up or concave down, then a quadratic model would be a likely candidate. 56. Graphing the data shows that neither a linear model nor a quadratic model is appropriate. The curve suggested by the graph is concave down on [0, 4] and concave up on [4, 8]. This is unlike a parabola, which is either always concave up or concave down. 57. No; The graph of a quadratic function is a parabola. In the case of a concave-up parabola, the curve would unrealistically predict sales increasing extremely fast and becoming unrealistically large in the future. In the case of a concave-down parabola, the curve would predict "negative" sales from some point on. 58. Answers may vary. Some uses are: using the model to interpolate; that is, predict missing values inside the domain; using the model for short-term prediction; using a quadratic model to determine maximum possible revenue or profit.

59. If 𝑞 = 𝑚𝑝 + 𝑏 (with 𝑚 < 0), then the revenue is given by 𝑅 = 𝑝𝑞 = 𝑚𝑝 2 + 𝑏𝑝. This is the equation of a parabola with 𝑎 = 𝑚 < 0 and so is concave down. Thus, the vertex is the highest point on the parabola, showing that there is a single highest value for 𝑅, namely, the 𝑦-coordinate of the vertex. 60. The given equation is the equation of a concave-up parabola, and thus has the vertex as its lowest point. The 𝑦-coordinate of the vertex therefore gives the lowest possible average cost, which occurs when 𝑥 is assigned the value of the 𝑥-coordinate of the vertex.


2

Solutions Section 2.1

61. Since 𝑅 = 𝑝𝑞, the demand must be given by 𝑞 = 𝑅𝑝 = −50𝑝 𝑝+60𝑝 = −50𝑝 + 60. 2

62. Since 𝑅 = 𝑝𝑞, the demand is given by 𝑞 = 𝑅𝑝 = −50𝑝 +60𝑝+50 . This is not linear. = −50𝑝 + 60 + 50 𝑝 𝑝


Solutions Section 2.2 Section 2.2 1. 4^x 𝑥

𝑓(𝑥)

2. 3^x 𝑥

𝑓(𝑥)

−3

−2

−1

0

1 4

16

64

−3

−2

−1

0

1

2 9

3

27

−2

−1

0

1

2 1 9

3

1 27

−2

−1

0

1 1 4

2

1 16

3

1 64

0

1

2 8

3

16

2

3

1 64

1 16

1 27

1 9

3. 3^(-x) 𝑥

𝑓(𝑥)

−3 27

9

4. 4^(-x) 𝑥

𝑓(𝑥)

−3 64

16

1 4

1 3

3

4

5. 2*2^x or 2*(2^x) 𝑥

𝑔(𝑥)

−3

−2

1 4

1 2

−1 1

6. 2*3^x or 2*(3^x) 𝑥

𝑔(𝑥)

−3 2 27

𝑥

−3

ℎ(𝑥) −24

1

1

1

2

3

1 3

4

3

−2

−1

0

1 6

18

54

−2

−1

0

1

2

3

2 9

7. -3*2^(-x)

1

2

−12

2 3

−6

2

−3

3 2

3 4

3 8


Solutions Section 2.2 8. -2*3^(-x) 𝑥

−3

ℎ(𝑥) −54 9. 2^x-1 𝑥

𝑟(𝑥)

−3

7 8

10. 2^(-x)+1 𝑥

𝑟(𝑥)

−3 9

11. 2^(x-1) 𝑥

𝑠(𝑥)

−3 1 16

12. 2^(1-x) 𝑥

𝑠(𝑥) 13.

−3 16

0

1

2

3

−2

−1 −6

−2

2 3

2 9

−2 3 4

−1

0

1

2

3

−2

−1

0

1

2

3

−2

−1

0

1

2

3

−2

−1

0

1

2

3

−18

5

1 8

8

1 2

3

1 4

4

0

2

1 2

2

1

3 2

1

1

3

5 4

2

1 2

2 27

7

9 8

4

1 4 14.


Solutions Section 2.2 15.

16.

17.

18.

19.

𝑥

𝑓(𝑥) 𝑔(𝑥)

−2

0.5 8

−1

1.5 4

0

4.5 2

1

13.5 1

2

40.5 1 2

For every increase in 𝑥 by one unit, the value of 𝑓 is multiplied by 3, so 𝑓 is exponential. Since 𝑓(0) = 4.5, the exponential model is 𝑓(𝑥) = 4.5(3 𝑥). For every increase in 𝑥 by one unit, the value of 𝑔 is multiplied by 1/2, so 𝑔 is exponential. Since 𝑔(0) = 2, the exponential model is 𝑔(𝑥) = 2(1∕2) 𝑥, or 2(2 −𝑥). 20.

𝑥

−2

−1

0

1

2

𝑔(𝑥)

3

0

−1

0

3

𝑓(𝑥)

1 2

1

2

4

8

For every increase in 𝑥 by one unit, the value of 𝑓 is multiplied by 2, so 𝑓 is exponential. The values of 𝑔 decrease and then increase, so 𝑔 is not exponential. Since 𝑓(0) = 2, the exponential model is 𝑓(𝑥) = 2(2 𝑥).


𝑥

−2

𝑔(𝑥)

0.3

𝑥

−2

21.

−1

𝑓(𝑥) 22.5

7.5 0.9

0

Solutions Section 2.2

1

2.5

7.5

2.7

8.1

2

22.5 16.2

When 𝑥 increases from −1 to 0, the value of 𝑓 is multiplied by 1/3, but when 𝑥 is increased from 0 to 1, the value of 𝑓 is multiplied by 3. So 𝑓 is not exponential. When 𝑥 increases from −1 to 0, the value of 𝑔 is multiplied by 3, but when 𝑥 is increased from 1 to 2, the value of 𝑔 is multiplied by 2. So 𝑔 is not exponential. 22.

𝑓(𝑥) 𝑔(𝑥)

−1

0.3

0.9

3

0

1

2.7

2

8.1

24.3

1.5

0.75 0.375 0.1875

−1

0

For every increase in 𝑥 by one unit, the value of 𝑓 is multiplied by 3, so 𝑓 is exponential. Since 𝑓(0) = 2.7, the exponential model is 𝑓(𝑥) = 2.7(3 𝑥). For every increase in 𝑥 by one unit, the value of 𝑔 is multiplied by 0.5, so 𝑔 is exponential. Since 𝑔(0) = 0.75, the exponential model is 𝑔(𝑥) = 0.75(0.5) 𝑥, or 0.75(2 −𝑥). 𝑥

23.

−2

𝑓(𝑥) 100 𝑔(𝑥) 100

200 20

1

400

600

4

2

800

0.8

0.16

1

2

10

2

The values of 𝑓(𝑥) double for every one-unit increase in 𝑥 except when 𝑥 increases from 1 to 2, when the value of 𝑓 is multiplied by 4/3. Hence 𝑓 is not exponential. On the other hand, 𝑔 is multiplied by 0.2 for every increase by one unit in 𝑥, so 𝑔 is exponential. Since 𝑔(0) = 4, the exponential model is 𝑔(𝑥) = 4(0.2) 𝑥. 𝑥

24.

𝑓(𝑥) 𝑔(𝑥)

−2

−1

0.8

0.2

80

40

0

0.1 20

0.05 0.025

The value of 𝑓(𝑥) is multiplied by 1/4 when 𝑥 increases from −2 to −1 but halved 𝑥 when increases from −1 to 0. Hence 𝑓 is not exponential. The value of 𝑔(𝑥) is multiplied by 1/2 when 𝑥 increases from −2 to −1 but multiplied by 0.2 when 𝑥 increases from 1 to 2. Hence 𝑔 is not exponential. 25. 2.718^(-2*x) 𝑥

−3

−2

−1

𝑓(𝑥) 403.2 54.58 7.388

0 1

1

2

3

0.1354 0.01832 0.002480


Solutions Section 2.2 26. 2.718^(x/5) 𝑥

−3

−2

0

−1

𝑔(𝑥) 0.5488 0.6703 0.8187 27. 1.01*2.02^(-4*x) 𝑥

−3

−2

1

1

2

3

1.221 1.492 1.822

−1

0

1

2

3

−1

0

1

2

3

0

1

2

3

ℎ(𝑥) 4 662 280.0 16.82 1.01 0.06066 0.003643 0.0002188 28. 3.42*3^(-x/5) 𝑥

−3

−2

ℎ(𝑥) 6.612 5.308 4.261 3.42 2.745 2.204 1.769 29. 50*(1+1/3.2)^(2*x) 𝑥

−3

−2

−1

𝑟(𝑥) 9.781 16.85 29.02

50

30. 0.043*(4.5-5/1.2)^(-x) 𝑥

−3

−2

−1

86.13 148.4 255.6 0

1

2

3

𝑟(𝑥) 0.001592 0.004778 0.01433 0.043 0.129 0.387 1.161 31. The following solutions also show some common errors you should avoid. 2^(x-1) not 2^x-1 32. 2^(-4*x) not 2^-4*x 33. 2/(1-2^(-4*x)) not 2/1-2^-4*x and not 2/1-2^(-4*x) 34. 2^(3-x)/(1-2^x) or (2^(3-x))/(1-2^x) not 2^3-x/1-2^x and not 2^3-x/(1-2^x) 35. (3+x)^(3*x)/(x+1) or ((3+x)^(3*x))/(x+1)not (3+x)^(3*x)/x+1 and not (3+x^(3*x))/(x+1) 36. 20.3^(3*x)/(1+20.3^(2*x)) or (20.3^(3*x))/(1+20.3^(2*x))not 3^(3*x)/1+20.3^(2*x) and not (20.3^3*x)/(1+20.3^2*x) 37. 2*e^((1+x)/x) or 2*EXP((1+x)/x)not 2*e^1+x/x and not 2*e^(1+x)/x and not 2*EXP(1+x)/x 38. 2*e^(2/x)/x or (2*e^(2/x))/x or 2*EXP(2/x)/x or (2*EXP(2/x))/xnot 2*e^((2/x)/x) and not 2*e^2/x/x and not 2*EXP((2/x)/x)


39. In these solutions, 𝑓1 is black and 𝑓2 is gray.

40.

y1 = 1.6^x y2 = 1.8^x

y1 = 2.2^x y2 = 2.5^x

41. (Note that the 𝑥-axis shown here crosses at 200, not 0.)

42. (Note that the 𝑥-axis shown here crosses at 95.)

y1 = 300*1.1^x y2 = 300*1.1^(2*x)

y1 = 100*1.01^(2*x) y2 = 100*1.01^(3*x)

43.

44.

y1 = 2.5^(1.02*x) y2 = e^(1.02*x) or exp(1.02*x)

y1 = 2.5^(-1.02*x) y2 = e^(-1.02*x) or exp(-1.02*x)

45. (Note that the 𝑥-axis shown here crosses at 900.)

46. (Note that the 𝑥-axis here crosses at 1,100.)

Solutions Section 2.2

y1 = 1000*1.045^(-3*x) y2 = 1000*1.045^(3*x)

y1 = 1202*1.034^(-3*x) y2 = 1202*1.034^(3*x)

47. Each time 𝑥 increases by 1, 𝑓(𝑥) is multiplied by 0.5. Also, 𝑓(0) = 500. So, 𝑓(𝑥) = 500(0.5) 𝑥.


Solutions Section 2.2 48. Each time 𝑥 increases by 1, 𝑓(𝑥) is multiplied by 2. Also, 𝑓(0) = 500. So, 𝑓(𝑥) = 500(2) 𝑥. 49. Each time 𝑥 increases by 1, 𝑓(𝑥) is multiplied by 3. Also, 𝑓(0) = 10. So, 𝑓(𝑥) = 10(3) 𝑥.

50. Each time 𝑥 increases by 1, 𝑓(𝑥) is multiplied by 1/3. Also, 𝑓(0) = 90. So, 𝑓(𝑥) = 90(1∕3) 𝑥. 51. Each time 𝑥 increases by 1, 𝑓(𝑥) is multiplied by 225∕500 = 0.45. Also, 𝑓(0) = 500. So, 𝑓(𝑥) = 500(0.45) 𝑥.

52. Each time 𝑥 increases by 1, 𝑓(𝑥) is multiplied by 3∕5 = 0.6. Also, 𝑓(0) = 5. So, 𝑓(𝑥) = 5(0.6) 𝑥. 53. Write 𝑓(𝑥) = 𝐴𝑏 𝑥. We have 𝐴𝑏 1 = −110 and 𝐴𝑏 2 = −121. Dividing, 𝑏 = −121∕(−110) = 1.1. Substituting, 𝐴(1.1) = −110, so 𝐴 = −100. Thus, 𝑓(𝑥) = −100(1.1) 𝑥.

54. Write 𝑓(𝑥) = 𝐴𝑏 𝑥. We have 𝐴𝑏 1 = −41 and 𝐴𝑏 2 = −42.025. Dividing, 𝑏 = −42.025∕(−41) = 1.025. Substituting, 𝐴(1.025) = −41, so 𝐴 = −41∕1.025 = −40. Thus, 𝑓(𝑥) = −40(1.025) 𝑥. 55. We want an equation of the form 𝑦 = 𝐴𝑏 𝑥. Substituting the coordinates of the given points gives 36 = 𝐴𝑏 2 324 = 𝐴𝑏 4

Dividing the second equation by the first gives 324∕36 = 9 = 𝑏 2, so 𝑏 = 3. Substituting into the first equation now gives 36 = 𝐴(3) 2 = 9𝐴, so 𝐴 = 36∕9 = 4. Hence the model is 𝑦 = 𝐴𝑏 𝑥 = 4(3 𝑥). 56. We want an equation of the form 𝑦 = 𝐴𝑏 𝑥. Substituting the coordinates of the given points gives −4 = 𝐴𝑏 2 −16 = 𝐴𝑏 4.

Dividing the second equation by the first gives −16∕(−4) = 4 = 𝑏 2, so 𝑏 = 2. Substituting into the first equation now gives −4 = 𝐴(2) 2 = 4𝐴, so 4𝐴 = −14. Hence the model is 𝑦 = 𝐴𝑏 𝑥 = −1(2 𝑥). 57. We want an equation of the form 𝑦 = 𝐴𝑏 𝑥. Substituting the coordinates of the given points gives −25 = 𝐴𝑏 −2 −0.2 = 𝐴𝑏.

Dividing the second equation by the first gives −0.2∕(−25) = 0.008 = 𝑏 3, so 𝑏 = 0.2. Substituting into the second equation now gives −0.2 = 0.2𝐴, so 𝐴 = −1. Hence the model is 𝑦 = 𝐴𝑏 𝑥 = −1(0.2 𝑥). 58. We want an equation of the form 𝑦 = 𝐴𝑏 𝑥. Substituting the coordinates of the given points gives 1.2 = 𝐴𝑏 0.108 = 𝐴𝑏 3.

Dividing the second equation by the first gives 0.108∕1.2 = 0.09 = 𝑏 2, so 𝑏 = 0.3. Substituting into the first equation now gives 1.2 = 0.3𝐴, so 𝐴 = 4. Hence the model is 𝑦 = 𝐴𝑏 𝑥 = 4(0.3 𝑥). 59. Write 𝑓(𝑥) = 𝐴𝑏 𝑥. We have 𝐴𝑏 1 = 3 and 𝐴𝑏 3 = 6. Dividing, 𝑏 2 = 6∕3 = 2, so 𝑏 = √2 ≈ 1.4142. Substituting, 𝐴√2 = 3, so 𝐴 = 3∕√2 ≈ 2.1213. Thus, 𝑦 = 2.1213(1.4142 𝑥).


3

60. Write 𝑓(𝑥) = 𝐴𝑏 𝑥. We have 𝐴𝑏 1 = 2 and 𝐴𝑏 4 = 6. Dividing, 𝑏 3 = 6∕2 = 3, so 𝑏 = √3 ≈ 1.4422. Solutions Section 2.2

3

3

Substituting, 𝐴 √3 = 2, so 𝐴 = 2∕ √3 ≈ 1.3867. Thus, 𝑦 = 1.3867(1.4422 𝑥).

61. Write 𝑓(𝑥) = 𝐴𝑏 𝑥. We have 𝐴𝑏 2 = 3 and 𝐴𝑏 6 = 2. Dividing, 𝑏 4 = 2∕3, so 𝑏 = √2∕3 ≈ 0.9036. 4

Substituting, 𝐴( √2∕3) 2 = 3, so 𝐴 = 3∕( √2∕3) 2 ≈ 3.6742. Thus, 𝑦 = 3.6742(0.9036 𝑥). 4

4

62. Write 𝑓(𝑥) = 𝐴𝑏 𝑥. We have 𝐴𝑏 −1 = 2 and 𝐴𝑏 3 = 1. Dividing, 𝑏 4 = 1∕2, so 𝑏 = √1∕2 ≈ 0.8409. Substituting, 𝐴∕ √1∕2 = 2, so 𝐴 = 2 √1∕2 ≈ 1.6818. Thus, 𝑦 = 1.6818(0.8409 𝑥). 4

4

4

63. 𝑦 = 1.0442(1.7564) 𝑥 64. 𝑦 = 1.0442(0.5694) 𝑥

65. 𝑦 = 15.1735(1.4822) 𝑥 66. 𝑦 = 0.4782(1.8257) 𝑥

67. We want a model of the form 𝑓(𝑡) = 𝐴𝑏 𝑡. We are given two points on the graph: (0, 300) and (2, 75). Substituting the coordinates, we get 300 = 𝐴𝑏 0 = 𝐴 75 = 𝐴𝑏 . 2

Substitute (0, 300). Substitute (2, 75).

This gives 𝐴 = 300, 75 = 300𝑏 2, so 𝑏 2 = 75∕300 = 0.25, giving 𝑏 = 0.25 1∕2 = 0.5, and the model is 𝑓(𝑡) = 300(0.5) 𝑡. After 5 hours, 𝑓(5) = 300(0.5) 5 = 9.375 mg. 68. We want a model of the form 𝑓(𝑡) = 𝐴𝑏 𝑡. We are given two points on the graph: (0, 200) and (2, 112.5). Substituting the coordinates, we get 200 = 𝐴𝑏 0 = 𝐴 112.5 = 𝐴𝑏 2.

Substitute (0, 200).

Substitute (2, 112.5).

This gives 𝐴 = 200, 112.5 = 200𝑏 2, so 𝑏 2 = 112.5∕200 = 0.5625, giving 𝑏 = 0.5625 1∕2 = 0.75, and the model is 𝑓(𝑡) = 200(0.75) 𝑡. After 4 hours, 𝑓(4) = 200(0.75) 4 ≈ 63 mg/dL. 69. a. Linear model: 𝑆 = 𝑚𝑡 + 𝑏 Points: (0, 320) and (5, 950) 𝑆 − 𝑆1 950 − 320 Slope: 𝑚 = 2 = = 126 𝑡2 − 𝑡1 5−0 Intercept: 𝑏 = 𝑆1 − 𝑚𝑡1 = 320 − (126)(0) = 320 Model: 𝑆 = 𝑚𝑡 + 𝑏 = 126𝑡 + 320 Exponential model: 𝑆 = 𝐴𝑏 𝑡 Substitute (0, 320): 320 = 𝐴𝑏 0 = 𝐴; substitute (5, 950): 950 = 𝐴𝑏 5 = 320𝑏 5. Thus 950 = 𝑏5 320


𝑏 =(

950 1∕5 ≈ 1.24. 320 )

Solutions Section 2.2

Model: 𝑆 = 𝐴𝑏 𝑡 = 320(1.24) 𝑡 The exponential model is applicable: The successive ratios of the values of 𝑆 are not too far from 𝑏 = 1.24. The successive differences climb steadily, making a linear model not appropriate. b. 2020 corresponds to 𝑡 = 4, and so 𝑆 = 𝐴𝑏 𝑡 = 320(1.24) 4 ≈ 757 GW, in reasonable agreement with the actual figure of 770 GW. 70. a. Linear model: 𝑊 = 𝑚𝑡 + 𝑏 Points: (0, 200) and (10, 710) 𝑊 − 𝑊1 710 − 200 Slope: 𝑚 = 2 = = 51 𝑡2 − 𝑡1 10 − 0 Intercept: 𝑏 = 𝑊1 − 𝑚𝑡1 = 200 − (51)(0) = 200 Model: 𝑊 = 𝑚𝑡 + 𝑏 = 51𝑡 + 200 Exponential model: 𝑊 = 𝐴𝑏 𝑡 Substitute (0, 200): 200 = 𝐴𝑏 0 = 𝐴; substitute (10, 710): 710 = 𝐴𝑏 10 = 200𝑏 10. Thus 710 = 3.55 = 𝑏 10 200 𝑏 = 3.55 1∕10 ≈ 1.14.

Model: 𝑊 = 𝐴𝑏 𝑡 = 200(1.14) 𝑡 The exponential model is applicable: The successive ratios of the values of 𝑊 are not too far from 𝑏 2 ≈ 1.30. (Note that 𝑏 2 is the ratio we expect to see every two years, as in the values given.) The successive differences climb fairly steadily, making a linear model not appropriate. b. 2018 corresponds to 𝑡 = 8, and so 𝑊 = 𝐴𝑏 𝑡 = 200(1.14) 8 ≈ 571 GW, in reasonable agreement with the actual figure of 590 GW. 71. a. The successive ratios are 𝑡

Ratios

10

20

30

40

50

60

210 229 252 282 309 331 ≈ 1.12 ≈ 1.09 ≈ 1.10 ≈ 1.12 ≈ 1.10 ≈ 1.07 187 210 229 252 282 309

As the ratios are all close to 1.1 with no systematic change, we conclude that the growth is approximately exponential. b. The 1970 and 1980 data together with the answer to part (a) give us: (10, 210): 𝐴𝑏 10 = 210 (20, 229): 𝐴𝑏 20 = 229 229 𝑏 10 = ≈ 1.09047619 210 𝑏 ≈ 1.09047619 1∕10 ≈ 1.00870 to 6 significant digits 210 210 2 𝐴𝑏 10 = 210 gives 𝐴 = 10 = ≈ 192.576, 229 𝑏

So the model is 𝑈(𝑡) = 192.576(1.00870) 𝑡, and predicts 𝑈(60) = 192.576(1.00870) 60 ≈ 324 million for 2020, reasonably close to the actual figure of 331 million.


Solutions Section 2.2 72. a. The successive ratios are 𝑡

Ratios

10

20

30

40

50

60

3.70 4.46 5.33 6.14 6.96 7.79 ≈ 1.22 ≈ 1.21 ≈ 1.20 ≈ 1.15 ≈ 1.13 ≈ 1.12 3.03 3.70 4.46 5.33 6.14 6.96

As the ratios show a definite downward trend from 1.22 to 1.12, we conclude that the growth is not exponential, as exponential growth would require the ratios to be approximately constant. b. The 1970 and 1980 data together with the answer to part (a) give us: (10, 3.70): 𝐴𝑏 10 = 3.70 (20, 4.46): 𝐴𝑏 20 = 4.46 4.46 𝑏 10 = ≈ 1.205405405 3.70 𝑏 ≈ 1.205405405 1∕10 ≈ 1.01886 to 6 significant digits 3.70 3.70 2 𝐴𝑏 10 = 3.70 gives 𝐴 = 10 = ≈ 3.06951, 4.46 𝑏

So the model is 𝑊 (𝑡) = 3.06951(1.01886) 𝑡, and predicts 𝑊 (60) = 3.06951(1.01886) 60 ≈ 9.42 billion in 2020, more than 20% higher than the actual figure of 7.79 billion. 73. If 𝑦 represents the size of the culture at time 𝑡, then 𝑦 = 𝐴𝑏 𝑡. We are told that the initial size is 1,000, so 𝐴 = 1, 000. We are told that the size doubles every 3 hours, so 𝑏 3 = 2, or 𝑏 = 2 1∕3. Thus, 𝑦 = 1, 000(2 1∕3) 𝑡 = 1, 000(2 𝑡∕3). There will be 1, 000(2 48∕3) = 65, 536, 000 bacteria after 2 days. 74. If 𝑦 represents the size of the culture at time 𝑡, then 𝑦 = 𝐴𝑏 𝑡. We are told that the initial size is 1,000, so 𝐴 = 1, 000. We are told that 𝑦 = 1, 500 when 𝑡 = 2, so 1, 500 = 1, 000𝑏 2, giving 𝑏 = 1.5 1∕2. Thus, 𝑦 = 1, 000(1.5 1∕2) 𝑡 = 1, 000(1.5 𝑡∕2). There will be 1, 000(1.5 48∕2) ≈ 16, 800, 000 bacteria after 2 days.

75. The desired model is 𝐶(𝑡) = 𝐴𝑏 𝑡. At time 𝑡 = 0 (April 1, 2021) the number of active cases was 615,000, so 𝐴 = 615,000. Since the number was increasing by 6.9% each day, that number is multiplied by 1.069 each day, so 𝑏 = 1.069. Hence, the model is 𝐶(𝑡) = 615,000(1.069) 𝑡. As April 21, 2021 corresponds to 𝑡 = 20, the estimated number of cases was about 𝐶(20) = 615,000(1.069) 20 ≈ 2,335,765. 76. The desired model is 𝑆(𝑡) = 𝐴𝑏 𝑡. At time 𝑡 = 0 (April 1, 2003) the number of cases was 1,804, so 𝐴 = 1,804. Since the number was increasing by 4% each day, the number of cases is multiplied by 1.04 each day, so 𝑏 = 1.04. Hence, the model is 𝑆(𝑡) = 1,804(1.04) 𝑡. Since April 30, 2003 corresponds to 𝑡 = 29, the number of cases was about 𝑆(14) = 1,804(1.04) 29 ≈ 5,626. 77. Apply the formula 𝑟 𝑛𝑡 𝐴(𝑡) = 𝑃 !1 + " 𝑛 with 𝑃 = 5,000, 𝑟 = 0.05∕100 = 0.0005, and 𝑛 = 12. We get the model 𝐴(𝑡) = 5,000(1 + 0.0005∕12) 12𝑡 In July 2028 (𝑡 = 7), the deposit would be worth 5,000(1 + 0.0005∕12) 12(7) ≈ $5,018. 78. Apply the formula 𝑟 𝑛𝑡 𝐴(𝑡) = 𝑃 !1 + " 𝑛 with 𝑃 = 4,000, 𝑟 = 0.0061, and 𝑛 = 365. We get the model 𝐴(𝑡) = 4,000(1 + 0.0061∕365) 365𝑡


In July 2029 (𝑡 = 8), the deposit would be worth 4,000(1 + 0.0061∕365) 365(8) ≈ $4,200. Solutions Section 2.2

79. Substitute 𝐴 = 15,000, 𝑃 = 10,000, 𝑟 = 0.025, and 𝑛 = 1 into the compound interest formula: 15,000 = 10,000(1 + 0.025) 𝑡, (1.025) 𝑡 = 1.5, 𝑡 = log(1.5)∕ log(1.025) ≈ 16 years.

80. Substitute 𝐴 = 11,000, 𝑃 = 10,000, 𝑟 = 0.00025, and 𝑛 = 1 into the compound interest formula: 11,000 = 10,000(1 + 0.00025) 𝑡, (1.00025) 𝑡 = 1.1, 𝑡 = log(1.1)∕ log(1.00025) ≈ 381 years. 81. Substitute 𝐴 = 20,000, 𝑃 = 10,400, 𝑟 = 0.010, and 𝑛 = 12 into the compound interest formula: 20,000 = 10,400(1 + 0.010∕12) 12𝑡, (1 + 0.010∕12) 12𝑡 = 200∕104, 12𝑡 = log(200∕104)∕ log(1 + 0.010∕12) ≈ 785 months. 82. Substitute 𝐴 = 20,000, 𝑃 = 10,400, 𝑟 = 0.025, and 𝑛 = 12 into the compound interest formula: 20,000 = 10,400(1 + 0.025∕12) 12𝑡, (1 + 0.025∕12) 12𝑡 = 200∕104, 12𝑡 = log(200∕104)∕ log(1 + 0.025∕12) ≈ 314 months.

83. Substitute 𝐴 = 2𝑃 , 𝑟 = 0.035, and 𝑛 = 2 into the compound interest formula: 2𝑃 = 𝑃 (1 + 0.035∕2) 2𝑡, (1.0175) 2𝑡 = 2, 2𝑡 = log 2∕ log 1.0175, 𝑡 = (log 2∕ log 1.0175)∕2 ≈ 20 years.

84. Substitute 𝐴 = 2𝑃 , 𝑟 = 0.0425, and 𝑛 = 2 into the compound interest formula: 2𝑃 = 𝑃 (1 + 0.0425∕2) 2𝑡, (1.02125) 2𝑡 = 2, 2𝑡 = log 2∕ log 1.02125, 𝑡 = (log 2∕ log 1.02125)∕2 ≈ 16 years. 85. If 99.95% has decayed, then 0.05% remains, so 𝐶(𝑡) = 0.0005𝐴. Therefore, 0.0005𝐴 = 𝐴(0.999879) 𝑡, (0.999879) 𝑡 = 0.0005, so 𝑡 = log(0.0005)∕ log(0.999879) ≈ 63,000 years old. 86. If 30% has decayed, then 70% remains, so 𝐶(𝑡) = 0.70𝐴. Therefore, 0.70𝐴 = 𝐴(0.999879) 𝑡, (0.999879) 𝑡 = 0.70, 𝑡 = log(0.70)∕ log(0.999879) ≈ 3,000 years old.

87. a. Let 𝑦 be the number of frogs in year 𝑡, with 𝑡 = 0 representing 2 years ago; we seek a model of the form 𝑦 = 𝐴𝑏 𝑡. We are given the initial value of 𝐴 = 50,000 and are told that 50,000𝑏 2 = 32,000. This gives 𝑏 = (32,000∕50,000) 1∕2 = 0.64 1∕2 = 0.8. Thus, 𝑦 = 50,000(0.8) 𝑡. b. When 𝑡 = 3, 𝑦 = 50,000(0.8) 3 = 25,600 tags. 1,000 c. We want 𝑡 so that 50,000(0.8) 𝑡 = 1,000 so 0.8 𝑡 = = 0.02. Taking the logarithm of both sides 50,000 gives log(0.8 𝑡) = log 0.02 ⟹ 𝑡 log 0.8 = log 0.02, so log 0.02 𝑡= ≈ 17.53 years, log 0.8 so by 𝑡 = 18 (16 years from now) the number will have dropped to less than 1,000.

88. a. Let 𝑦 be the number of flies in year 𝑡, with 𝑡 = 0 representing 3 years ago; we seek a model of the form 𝑦 = 𝐴𝑏 𝑡. We are given the initial value of 𝐴 = 4,000 and we are told that 4,000𝑏 3 = 1,372. This gives 𝑏 = (1,372∕4,000) 1∕3 = 0.343 1∕3 = 0.7. Thus, 𝑦 = 4,000(0.7) 𝑡. b. When 𝑡 = 4, 𝑦 = 4,000(0.7) 4 = 960.4 ≈ 960 flies. 500 c. We want 𝑡 so that 4,000(0.7) 𝑡 = 500 so 0.7 𝑡 = = 0.125. Taking the logarithm of both sides gives 4,000 log(0.7 𝑡) = log 0.125 ⟹ 𝑡 log 0.7 = log 0.125, so log 0.125 𝑡= ≈ 5.8 years, log 0.7 so by 𝑡 = 6 (three years from now) the number will have dropped to less than 500.


Solutions Section 2.2 89. a. Here is the Excel tabulation of the data, together with the scatter plot and the exponential trendline (with the option "Display equation on chart" checked):

Note that the displayed model has the form 𝑃 (𝑡) = 0.3394𝑒 0.1563𝑡. 𝑒 is the number 2.71828183...... discussed in the next section, and built in to all calculators. To express this in the form 𝐴(𝑏) 𝑡 we write 0.153𝑒 0.0863𝑡 = 0.3394(𝑒 0.1563) 𝑡 ≈ 0.3394(1.169) 𝑡

𝑒 0.1563 is EXP(0.1563) in Excel

so the model is

𝑃 (𝑡) = 0.339(1.169) 𝑡.

b. In 2005, 𝑡 = 11, so the predicted cost is 𝑃 (15 = 0.339(1.169) 11 ≈ $1.9 million.

90. a. Here is the Excel tabulation of the data, together with the scatter plot and the exponential trendline (with the option "Display equation on chart" checked):

Note that the displayed model has the form 𝑃 (𝑡) = 0.1533𝑒 0.0863𝑡. 𝑒 is the number 2.71828183...... discussed in the next section, and built in to all calculators. To express this in the form 𝐴(𝑏) 𝑡 we write 0.153𝑒 0.0863𝑡 = 0.153(𝑒 0.0863𝑡) 𝑡 ≈ 0.153(1.090) 𝑡

so the model is

𝑒 0.0863 is EXP(0.0863) in Excel


𝑃 (𝑡) = 0.153(1.090) 𝑡.

Solutions Section 2.2

b. In 2005, 𝑡 = 15, so the predicted cost is 𝑃 (15) = 0.153(1.090) 15 ≈ $0.56 million.

91. a. Here is the Excel tabulation of the data, together with the scatter plot and the exponential trendline (with the option "Display equation on chart" checked; we have enlarged the graph in order to see more clearly how the points are scattered around the regression curve):

Note that the displayed model has the form 𝑈(𝑡) = 189.17𝑒 0.0096𝑡. 𝑒 is the number 2.71828183 . . . discussed in the next section, and built in to all calculators. To express this in the form 𝐴(𝑏) 𝑡 we write 189.17𝑒 0.0096𝑡 = 189.17(𝑒 0.0096) 𝑡 ≈ 189.2(1.010) 𝑡

so the model is

𝑈(𝑡) = 189.2(1.010) 𝑡.

𝑒 0.0096 is EXP(0.0096) in Excel


Solutions Section 2.2 b. Yes; the points are very close to, and scattered randomly around, the regression curve. 92. a. Here is the Excel tabulation of the data, together with the scatter plot and the exponential trendline (with the option "Display equation on chart" checked; we have enlarged the graph in order to see more clearly how the points are scattered around the regression curve):

Note that the displayed model has the form 𝑊 (𝑡) = 3.1722𝑒 0.0158𝑡 𝑒 is the number 2.71828183 . . . discussed in the next section, and built in to all calculators. To express this in the form 𝐴(𝑏) 𝑡 we write 3.1722𝑒 0.0158𝑡 = 3.1722(𝑒 0.0158) 𝑡 ≈ 3.172(1.016) 𝑡

𝑒 0.0158 is EXP(0.0158) in Excel

so the model is

𝑊 (𝑡) = 3.172(1.016) 𝑡.

b. No; the points suggest a graph that is less curved than the regression curve. 93. (B) An exponential function eventually becomes larger than any polynomial. 94. (B) An exponential decay function eventually becomes smaller than the reciprocal of any polynomial (see Exercise 99).


Solutions Section 2.2 95. Exponential functions of the form 𝑓(𝑥) = 𝐴𝑏 𝑥 (𝑏 > 1) increase rapidly for large values of 𝑥. In reallife situations, such as population growth, this model is reliable only for relatively short periods of growth. Eventually, population growth tapers off because of pressures such as limited resources and overcrowding. 96. If an investment earns 5% compounded continuously, it is as if interest is being added every moment. Thus, the interest for 1 month will have been calculated on a growing amount, rather than on the original fixed (smaller) amount used for interest compounded monthly. 97. Linear functions are better for cost models where there is a fixed cost and a variable cost and for simple interest, where interest is paid only on the original amount invested. Exponential models are better for compound interest and population growth. In both of these latter examples, the rate of growth depends on the current number of items, rather than on a fixed initial quantity. 98. Quadratic models are better for revenue and profit functions where demand depends linearly on the price. Exponential models are better for compound interest and population growth. 99. Take the ratios 𝑦2 ∕𝑦1 and 𝑦3 ∕𝑦2 . If they are the same, the points fit on an exponential curve.

100. For the points to fit on an exponential curve we must have 𝑦2 ∕𝑦1 = 𝑦3 ∕𝑦2 , hence 𝑦3 = 𝑦22∕𝑦1 .


Solutions Section 2.3 Section 2.3

1. Use the formula 𝑓(𝑡) = 𝑃 𝑒 𝑟𝑡 with 𝑃 = 5,000 and 𝑟 = 0.10, giving 𝑓(𝑡) = 5,000𝑒 0.10𝑡.

2. Use the formula 𝑓(𝑡) = 𝑃 𝑒 𝑟𝑡 with 𝑃 = 2,000 and 𝑟 = 0.053, giving 𝑓(𝑡) = 2,000𝑒 0.053𝑡.

3. Use the formula 𝑓(𝑡) = 𝑃 𝑒 𝑟𝑡 with 𝑃 = 1,000 and 𝑟 = −0.063, giving 𝑓(𝑡) = 1,000𝑒 −0.063𝑡.

4. Use the formula 𝑓(𝑡) = 𝑃 𝑒 𝑟𝑡 with 𝑃 = 10,000 and 𝑟 = −0.60, giving 𝑓(𝑡) = 10,000𝑒 −0.60𝑡. 5. 𝑓(𝑥) = 4𝑒 2𝑥 = 4(𝑒 2) 𝑥 ≈ 4(7.389) 𝑥

6. 𝑓(𝑥) = 2.1𝑒 −0.1𝑥 = 2.1(𝑒 −0.1) 𝑥 ≈ 2.1(0.9048) 𝑥

7. 𝑓(𝑡) = 2.1(1.001) 𝑡 = 2.1𝑒 (ln 1.001)𝑡 ≈ 2.1𝑒 0.0009995𝑡

8. 𝑓(𝑡) = 2.1(0.991) 𝑡 = 23.4𝑒 (ln 0.991)𝑡 ≈ 23.4𝑒 −0.009041𝑡 9. 𝑓(𝑡) = 10(0.987) 𝑡 = 10𝑒 (ln 0.987)𝑡 ≈ 10𝑒 −0.01309𝑡 10. 𝑓(𝑡) = 2.3(2.2) 𝑡 = 2.3𝑒 (ln 2.2)𝑡 ≈ 2.3𝑒 0.7885𝑡

11. 𝑄 = 1,000 when 𝑡 = 0; half-life = 1. We want a model of the form 𝑄 = 𝑄0 𝑒 −𝑘𝑡 for suitable 𝑄0 and 𝑘. We are given 𝑄0 = 1,000. For 𝑘, we use the formula 𝑡ℎ 𝑘 = ln 2 with 𝑡ℎ = half-life = 1, so 𝑘 = ln 2, and the model is 𝑄 = 𝑄0 𝑒 −𝑘𝑡 = 1,000𝑒 −𝑡 ln 2.

12. 𝑄 = 2,000 when 𝑡 = 0; half-life = 5. We want a model of the form 𝑄 = 𝑄0 𝑒 −𝑘𝑡 for suitable 𝑄0 and 𝑘. We are given 𝑄0 = 2,000. For 𝑘, we use the formula 𝑡ℎ 𝑘 = ln 2 with 𝑡ℎ = half-life = 5, so 2𝑘 = ln 2, giving 𝑘 = (ln 2)∕5 and the model is 𝑄 = 𝑄0 𝑒 −𝑘𝑡 = 2,000𝑒 −𝑡(ln 2)∕5.

13. 𝑄 = 500 when 𝑡 = 0; half-life = 50. We want a model of the form 𝑄 = 𝑄0 𝑒 −𝑘𝑡 for suitable 𝑄0 and 𝑘. We are given 𝑄0 = 500. For 𝑘, we use the formula 𝑡ℎ 𝑘 = ln 2 with 𝑡ℎ = half-life = 50, so 𝑘 = ln 2∕50, and the model is 𝑄 = 𝑄0 𝑒 −𝑘𝑡 = 500𝑒 −𝑡 ln 2∕50. 14. 𝑄 = 50 when 𝑡 = 0; half-life = 500. We want a model of the form 𝑄 = 𝑄0 𝑒 −𝑘𝑡 for suitable 𝑄0 and 𝑘. We are given 𝑄0 = 50. For 𝑘, we use the formula 𝑡ℎ 𝑘 = ln 2 with 𝑡ℎ = half-life = 500, so 𝑘 = ln 2∕500, and the model is 𝑄 = 𝑄0 𝑒 −𝑘𝑡 = 50𝑒 −𝑡 ln 2∕500. 15. 𝑄 = 1,000 when 𝑡 = 0; doubling time = 2. We want a model of the form 𝑄 = 𝑄0 𝑒 𝑘𝑡 for suitable 𝑄0 and 𝑘. We are given 𝑄0 = 1,000. For 𝑘, we use the formula 𝑡𝑑 𝑘 = ln 2 with 𝑡𝑑 = doubling time = 2, so 2𝑘 = ln 2, giving 𝑘 = (ln 2)∕2 and the model is 𝑄 = 𝑄0 𝑒 𝑘𝑡 = 1,000𝑒 𝑡(ln 2)∕2.


Solutions Section 2.3 16. 𝑄 = 2,000 when 𝑡 = 0; doubling time = 5. We want a model of the form 𝑄 = 𝑄0 𝑒 𝑘𝑡 for suitable 𝑄0 and 𝑘. We are given 𝑄0 = 2,000. For 𝑘, we use the formula 𝑡𝑑 𝑘 = ln 2 with 𝑡𝑑 = doubling time = 5, so 5𝑘 = ln 2, giving 𝑘 = (ln 2)∕5 and the model is 𝑄 = 𝑄0 𝑒 𝑘𝑡 = 2,000𝑒 𝑡(ln 2)∕5.

17. 𝑄 = 1,000𝑒 0.5𝑡. Since the exponent is positive, the model represents exponential growth. We use the formula 𝑡𝑑 𝑘 = ln 2, giving 𝑡𝑑 (0.5) = ln 2, giving doubling time = 𝑡𝑑 = (ln 2)∕0.5 = 2 ln 2.

18. 𝑄 = 1,000𝑒 −0.025𝑡. Since the exponent is negative, the model represents exponential decay. We use the formula 𝑡ℎ 𝑘 = ln 2, giving 𝑡ℎ (0.025) = ln 2, giving half-life = 𝑡ℎ = (ln 2)∕0.025 = 40 ln 2. 19. 𝑄 = 100𝑒 −𝑡. Since the exponent is negative, the model represents exponential decay. We use the formula 𝑡ℎ 𝑘 = ln 2, giving 𝑡ℎ (1) = ln 2, giving half-life = 𝑡ℎ = ln 2.

20. 𝑄 = 5 000𝑒 𝑡∕3. Since the exponent is positive, the model represents exponential growth. We use the formula 𝑡𝑑 𝑘 = ln 2, giving 𝑡𝑑 (1∕3) = ln 2, giving doubling time = 𝑡𝑑 = 3 ln 2. 21. 𝑄 = 𝑄0 𝑒 −4𝑡. Since the exponent is negative, the model represents exponential decay. We use the formula 𝑡ℎ 𝑘 = ln 2, giving 𝑡ℎ (4) = ln 2, giving half-life = 𝑡ℎ = (ln 2)∕4. 22. 𝑄 = 𝑄0 𝑒 𝑡. Since the exponent is positive, the model represents exponential growth. We use the formula 𝑡𝑑 𝑘 = ln 2, giving 𝑡𝑑 (1) = ln 2, giving doubling time = 𝑡𝑑 = ln 2.

23. a. From the continuous compounding formula, 1,000𝑒 0.04×10 = $1,491.82, of which $491.82 is interest. b. 𝑃 = 1,000𝑒 0.04𝑡 = 1,000(𝑒 0.04) 𝑡 ≈ 1,000(1.040811) 𝑡, which corresponds to an annually compounded interest rate of 4.08% 24. a. From the continuous compounding formula, 2,000𝑒 0.31×10 = $44,395.90, of which $42,395.90 is interest. b. 𝑃 = 2,000𝑒 0.31𝑡 = 2,000(𝑒 0.31) 𝑡 ≈ 2,000(1.3634) 𝑡, which corresponds to an annually compounded interest rate of 36.34%

25. Substitute 𝐴 = 700, 𝑃 = 500, and 𝑟 = 0.10 in the continuous compounding formula: 700 = 500𝑒 0.10𝑡. Solve for 𝑡: 𝑒 0.10𝑡 = 700∕500 = 7∕5, 0.10𝑡 = ln(7∕5), 𝑡 = 10 ln(7∕5) ≈ 3.36 years. 26. Substitute 𝐴 = 700, 𝑃 = 500, and 𝑟 = 0.15 in the continuous compounding formula: 700 = 500𝑒 0.15𝑡. Solve for 𝑡: 𝑒 0.15𝑡 = 700∕500 = 7∕5, 0.15𝑡 = ln(7∕5), 𝑡 = ln(7∕5)∕0.15 ≈ 2.24 years. 27. Substitute 𝐴 = 3𝑃 and 𝑟 = 0.10 in the continuous compounding formula: 3𝑃 = 𝑃 𝑒 0.10𝑡. Solve for 𝑡: 𝑒 0.10𝑡 = (3𝑃 )∕𝑃 = 3, 0.10𝑡 = ln 3, 𝑡 = 10 ln 3 ≈ 11 years. 28. Substitute 𝐴 = 1,000,000, 𝑃 = 1,000 and 𝑟 = 0.10 in the continuous compounding formula: 1,000,000 = 1,000𝑒 0.10𝑡. Solve for 𝑡: 𝑒 0.10𝑡 = 1,000,000∕1,000 = 1,000, 0.10𝑡 = ln 1,000, 𝑡 = 10 ln 1,000 ≈ 69 years. 29. The bank can afford to pay its customers 𝑟 𝑛𝑡 𝑃 (1 + ) = 𝑃 (1.036) 𝑡 𝑛


= 𝑃 #𝑒 ln 1.036$

Solutions Section 2.3

𝑡

≈ 𝑃 (𝑒 0.035367) 𝑡, corresponding to about 3.54% compounded continuously. 30. The bank can afford to pay its customers 𝑟 𝑛𝑡 𝑃 (1 + ) = 𝑃 (1.0456) 𝑡 𝑛

= 𝑃 #𝑒 ln 1.0456$ ≈ 𝑃 (𝑒 0.04459) 𝑡,

𝑡

corresponding to about 4.46% compounded continuously.

31. The desired model is 𝐶(𝑡) = 𝐴𝑒 𝑘𝑡. At time 𝑡 = 0 (April 1, 2014) the number of cases was 100, so 𝐴 = 100. Because the number was increasing exponentially with a growth constant of 72% per month, 𝑘 = 0.72 per month. Hence, the model is 𝐶(𝑡) = 100𝑒 0.72𝑡. As August 1, 2014 corresponds to 𝑡 = 4, the number of cases was about 𝐶(4) = 100𝑒 0.72×4 ≈ 1,781. 32. The desired model is 𝐷(𝑡) = 𝐴𝑒 𝑘𝑡. At time 𝑡 = 0 (April 1, 2014) the number of deaths was 90, so 𝐴 = 90. Because the number was increasing exponentially with a growth constant of 60% per month, 𝑘 = 0.60. Hence, the model is 𝐷(𝑡) = 90𝑒 0.60𝑡. As October 1, 2014 corresponds to 𝑡 = 6, the number of deaths was about 𝐷(6) = 90𝑒 0.60×6 ≈ 3,294.

33. At time 𝑡 = 0 (April 1, 2021) the number of active cases was 614,649, so 𝐴 = 614,649. Since the number was increasing by 6.9% each day, that number is multiplied by 1.069 each day, so an exponential model for the number of active cases is 𝐶(𝑡) = 614,649(1.069) 𝑡. To convert this to the desired form 𝐴𝑒 𝑘𝑡 we follow the hint and write 𝐶(𝑡) = 614,649(1.069) 𝑡 = 614,649#𝑒 ln 1.069$

𝑡

≈ 614,649(𝑒 0.06672363) 𝑡 = 614,649𝑒 0.06672363𝑡

Thus, 𝑘 ≈ 0.06672 per day to 4 significant digits, and the model is 𝐶(𝑡) = 614,649𝑒 0.06672𝑡. As April 21, 2021 corresponds to 𝑡 = 20, the estimated number of cases was about 𝐶(20) = 614,649𝑒 0.06672(20) ≈ 2,334,000. The figure with the fewest significant digits we were given was the rate of increase of 6.9% per day, so we should not trust the answer to more than two significant digits. 34. At time 𝑡 = 0 (April 1, 2003) the number of cases was 1,804, so 𝐴 = 1,804. Since the number was increasing by 4% each day, the number of cases is multiplied by 1.04 each day, so an exponenial model for the number of cases is 𝑆(𝑡) = 1,804(1.04) 𝑡. To convert this to the desired form 𝐴𝑒 𝑘𝑡 we follow the hint and write 𝑆(𝑡) = 1,804(1.04) 𝑡 = 1,804#𝑒 ln 1.04$

𝑡

≈ 1,804(𝑒 0.0392207) 𝑡 = 1,804𝑒 0.0392207𝑡

Thus, 𝑘 ≈ 0.03922 per day to 4 significant digits, and the model is 𝑆(𝑡) = 1,804𝑒 0.03922𝑡. As April 30, 2003 corresponds to 𝑡 = 29, the number of cases was about 𝑆(29) = 1,804𝑒 0.03922(29) ≈ 5,626.

35. We want a model of the form 𝐴 = 𝑃 𝑒 𝑟𝑡. For the rate of interest 𝑟, we use the formula 𝑡𝑑 𝑟 = ln 2 with


Solutions Section 2.3 𝑡𝑑 = doubling time = 3, so 3𝑟 = ln 2, giving 𝑟 = (ln 2)∕3 ≈ 0.231, or 23.1%.

36. We want a model of the form 𝐴 = 𝑃 𝑒 −𝑟𝑡. For the rate of interest 𝑟, we use the formula 𝑡ℎ 𝑟 = ln 2 with 𝑡ℎ = half-life = 2, so 2𝑟 = ln 2, giving 𝑟 = (ln 2)∕2 ≈ 0.347, or 34.7%. The investment is depreciating continuously at a rate of 34.7% per year. 37. The growth constant is 𝑘 = 0.025 per year, so the doubling time is 𝑡𝐷 = (ln 2)∕0.025 ≈ 27.73 years. 38. The growth constant is 𝑘 = 0.035 per year, so the doubling time is 𝑡𝐷 = (ln 2)∕0.035 ≈ 19.80 years. 39. For interest compounded twice per year, 0.00025 2𝑡 𝐴(𝑡) = 𝑃 (1 + = 𝑃 (1.000125) 2𝑡 = 𝑃 𝑒 (ln 1.000125)⋅2𝑡 2 ) ≈ 𝑃 𝑒 0.000249984𝑡, which would be 0.0249984% per year. 40. For interest compounded quarterly, 0.01 4𝑡 𝐴(𝑡) = 𝑃 (1 + = 𝑃 (1.0025) 4𝑡 = 𝑃 𝑒 (ln 1.0025)⋅4𝑡 4 ) ≈ 𝑃 𝑒 0.0099875𝑡, which would be 0.99875% per year.

41. a. The years 1950, 2000, 2050, and 2100 correspond to 𝑡 = 200, 250, 300, and 350 respectively. Technology formula: 280*e^(0.00146x) year 1950 2000 2050 2100 𝑡 200

𝐶(𝑡) parts per million 375

250

300

350

403

434

467

b. The level surpasses 450 when

450 = 280𝑒 0.00146𝑡 450 𝑒 0.00146𝑡 = 280 450 0.00146𝑡 = ln ( = ln 450 − ln 280 280 ) 1 𝑡= (ln 450 − ln 280) ≈ 324.97. 0.00146

Thus, the level passes 450 shortly before the end of 2074 (𝑡 = 324.97; we should not round, as the 450 level is predicted to be passed shortly before 2075 (𝑡 = 325)). 42. a. The years 1950, 2000, 2050, and 2100 correspond to 𝑡 = 200, 250, 300, and 350 respectively. Technology formula: 720*e^(0.00355x) year 1950 2000 2050 2100 𝑡

200

250

300

350

𝐶(𝑡) parts per billion 1,460 1,750 2,090 2,490 b. The level surpasses 2,000 when


2,000 = 720𝑒 0.00355𝑡 2,000 𝑒 0.00355𝑡 = 720 2,000 0.00355𝑡 = ln ( = ln 2,000 − ln 720 720 ) 1 𝑡= (ln 2,000 − ln 720) ≈ 287.79. 0.00355

Solutions Section 2.3

Thus, the level passes 2,000 around three-quarters way through 2037 (𝑡 = 287.79; we should not round, as the 2,000 level is predicted to be passed before 2038 (𝑡 = 288)). 43. The decay constant is 𝑘 = 0.000433. Thus, ln 2 ln 2 𝑡ℎ = ≈ 1,600 years (to the nearest 100 years). = 𝑘 0.000433 44. The decay constant is 𝑘 = 0.0861. Thus, ln 2 ln 2 𝑡ℎ = ≈ 8.05 days (to two decimal places). = 𝑘 0.0861 45. a. 𝑡ℎ 𝑘 = ln 2 5𝑘 = ln 2 𝑘 = (ln 2)∕5 ≈ 0.139 𝑄(𝑡) = 𝑄0 𝑒 −𝑘𝑡 ≈ 𝑄0 𝑒 −0.139𝑡 b. One third has decayed when 2/3 is left: 2 𝑄 = 𝑄0 𝑒 −0.139𝑡 3 0 2 = 𝑒 −0.139𝑡 3

ln(2∕3) = −0.139𝑡 𝑡 = ln(2∕3)∕(−0.139) ≈ 3 years.

46. a. 𝑡ℎ 𝑘 = ln 2 28𝑘 = ln 2 𝑘 = (ln 2)∕28 ≈ 0.0248 𝑄(𝑡) = 𝑄0 𝑒 −𝑘𝑡 ≈ 𝑄0 𝑒 −0.0248𝑡 b. Three fifths has decayed when 2/5 is left: 2 𝑄 = 𝑄0 𝑒 −0.0248𝑡 5 0 2 = 𝑒 −0.0248𝑡 5

ln(2∕5) = −0.0248𝑡 𝑡 = ln(2∕5)∕(−0.0248) ≈ 37 years.

47. Let 𝑄(𝑡) be the amount left after 𝑡 million years. We first find a model of the form 𝑄(𝑡) = 𝑄0 𝑒 −𝑘𝑡. To find 𝑘 use 𝑡ℎ 𝑘 = ln 2: 710𝑘 = ln 2 𝑘 = (ln 2)∕710 ≈ 0.0009763 𝑄(𝑡) = 𝑄0 𝑒 −𝑘𝑡 ≈ 𝑄0 𝑒 −0.0009763𝑡. We now answer the question: 𝑄0 = 10g, 𝑄(𝑡) = 1g. Substituting in the model gives 1 = 10𝑒 −0.0009763𝑡 𝑒 −0.0009763𝑡 = 0.1 −0.0009763𝑡 = ln(0.1) 𝑡 = − ln(0.1)∕0.0009763 ≈ 2,360 million years (rounded to three significant digits).


Solutions Section 2.3 48. Let 𝑄(𝑡) be the amount left after 𝑡 years. We first find a model of the form 𝑄(𝑡) = 𝑄0 𝑒 −𝑘𝑡. To find 𝑘 use 𝑡ℎ 𝑘 = ln 2: 24,400𝑘 = ln 2 𝑘 = (ln 2)∕24,400 ≈ 0.000028408 𝑄(𝑡) = 𝑄0 𝑒 −𝑘𝑡(𝑄0 𝑒 −0.000028408𝑡. We now answer the question: 𝑄0 = 10g, 𝑄(𝑡) = 1g. Substituting in the model gives 1 = 10𝑒 −0.000028408𝑡 𝑒 −0.000028408𝑡 = 0.1 −0.000028408𝑡 = ln(0.1) 𝑡 = − ln(0.1)∕0.000028408 ≈ 81,100 years (rounded to three significant digits). 49. a. By the discussion at the end of the section, Tripling time × Growth constant = ln 3 6𝑘 = ln 3, so ln 3 𝑘= ≈ 0.183. 6 ln 2 b. The doubling time is ≈ 3.8 months. 𝑘 50. a. By the discussion at the end of the section, Quadrupling time × Growth constant = ln 4 8𝑘 = ln 4, so ln 4 𝑘= ≈ 0.1733. 8 ln 3 b. The tripling time is ≈ 6.3 months. 𝑘

51. Let 𝑄(𝑡) be the amount left after 𝑡 hours. We first find a model of the form 𝑄(𝑡) = 𝑄0 𝑒 −𝑘𝑡. To find 𝑘 use 𝑡ℎ 𝑘 = ln 2: 2𝑘 = ln 2 𝑘 = (ln 2)∕2 ≈ 0.3466 𝑄(𝑡) = 𝑄0 𝑒 −𝑘𝑡 ≈ 𝑄0 𝑒 −0.3466𝑡. We now answer the question: 𝑄0 = 300mg, 𝑄(𝑡) = 100mg. Substituting in the model gives 100 = 300𝑒 −0.3466𝑡 𝑒 −0.3466𝑡 = 1∕3 −0.3466𝑡 = ln(1∕3) 𝑡 = − ln(1∕3)∕0.3466 ≈ 3.2 hours.

52. Let 𝑄 = 𝑄0 𝑒 −𝑘𝑡 represent the blood alcohol level at time 𝑡 hours. We are told that 𝑄0 = 200 and that one fourth of the alcohol is removed every hour; so, after 1 hour, three fourths remains, so 0.75𝑄0 = 𝑄0 𝑒 −𝑘. Thus, 𝑒 −𝑘 = 0.75, and so 𝑘 = − ln(0.75) ≈ 0.28768. We wish to find 𝑡 when 𝑄 = 80: 80 = 200𝑒 −0.28768𝑡 80 ln 0.4 𝑒 −0.28768𝑡 = ≈ 3.2 hours. = 0.4, so 𝑡 = 200 −0.28768

53. We first find a model of the form 𝐴(𝑡) = 𝑃 𝑏 𝑡. We are given the data points (0, 1) and (2, 0.7), so 𝑃 = 1 and 𝑏 2 = 0.7, so 𝑏 = (0.7) 1∕2; the model is 𝐴(𝑡) = (0.7) 𝑡∕2 = (√0.7) 𝑡. This model can also be written as 𝑒 𝑟𝑡, where 𝑟 = ln √0.7 ≈ −0.1783, so 𝑘 ≈ 0.1783: ln 2 ln 2 The half life is then ≈ ≈ 3.89 days. 𝑘 0.1783


Solutions Section 2.3 54. We first find a model of the form 𝐴(𝑡) = 𝑃 𝑏 𝑡. We are given the data points (0, 1) (half of the original 2 gm was stolen) and (3, 0.1), so 𝑃 = 1 and 𝑏 3 = 0.1, so 𝑏 = (0.1) 1∕3, and the model is 𝐴(𝑡) = (0.1) 𝑡∕3 = ((0.1) 1∕3) 𝑡, so 𝑏 = (0.1) 1∕3. This model can also be written as 𝑒 𝑟𝑡, where 𝑟 = ln(0.1) 1∕3 ≈ −0.76753, so 𝑘 ≈ 0.76753: ln 2 ln 2 The half life is then ≈ ≈ 0.903 days. 𝑘 0.76753 55. Answers may vary. One example is in the case of continuous compounding; the use of 𝐴𝑏 𝑡 would require us to write the continuous compounding formula 𝐴𝑒 𝑟𝑡 as 𝐴(𝑒 𝑟) 𝑡.

56. Answers may vary. One example is in the case of compound interest, where the use of 𝐴𝑒 𝑘𝑡 would 𝑛

require us to write the compound interest formula as 𝐴𝑒 (ln(1+𝑟∕𝑛) )𝑡.

57. This reasoning is suspect. The bank need not use its computer resources to update all the accounts every minute but can instead use the continuous compounding formula to calculate the balance in any account at any time as needed. 58. In the first minute your balance has grown to around $10,000.00228, which our spreadsheet rounds to $10,000, and so that repeats evey minute, leaving you wih no interest. 59. The article was published about a year before the "housing bubble" burst in 2006, whereupon, contrary to the prediction of the graph, house prices started to fall and continued to drop for several years. This shows the danger of using any mathematical model to extrapolate. However, the blogger was cautious in the choice of words, claiming only to be estimating what the future U.S. median house price "might be."

60. Suppose for the sake of argument that the model had been constructed around 𝑡 = 20 (1993). Then the resulting exponential model would have predicted house prices reasonably accurately for around 12 more years. It is just coincidence that the particular model was created a year before prices started to fall. In general, exponential models are very good at extrapolating current trends; it is only when things go awry that they become unreliable.


Solutions Section 2.4 Section 2.4

1. 𝑁 = 7, 𝐴 = 6, 𝑏 = 2; 7/(1+6*2^-x)

2. 𝑁 = 4, 𝐴 = 0.333, 𝑏 = 4; 4/(1+0.333*4^-x)

3. 𝑁 = 10, 𝐴 = 4, 𝑏 = 0.3; 10/(1+4*0.3^-x)

4. 𝑁 = 100, 𝐴 = 5, 𝑏 = 0.5; 100/(1+5*0.5^-x)

5. 𝑁 = 4, 𝐴 = 7, 𝑏 = 1.5; 4/(1+7*1.5^-x)


Solutions Section 2.4 6. 𝑁 = 8.5, 𝐴 = 3.25, 𝑏 = 1.05; 8.5/(1+3.25*1.05^-x)

7. 𝑁 = 200, the limiting value. 10 = 𝑓(0) = 𝑁∕(1 + 𝐴) = 200∕(1 + 𝐴), so 𝐴 = 19. For small values of 𝑥, 𝑓(𝑥) ≈ 10𝑏 𝑥; to double with every increase of 1 in 𝑥 we must therefore have 𝑏 = 2. This gives 200 𝑓(𝑥) = . 1 + 19(2 −𝑥) 8. 𝑁 = 10, the limiting value. 1 = 𝑓(0) = 𝑁∕(1 + 𝐴) = 10∕(1 + 𝐴), so 𝐴 = 9. For small values of 𝑥, 𝑓(𝑥) ≈ 𝑏 𝑥; to grow by 50% with every increase of 1 in 𝑥 we must therefore have 𝑏 = 1.5. This gives 10 𝑓(𝑥) = . 1 + 9(1.5) −𝑥

9. 𝑁 = 6, the limiting value. 3 = 𝑓(0) = 𝑁∕(1 + 𝐴) = 6∕(1 + 𝐴), so 𝐴 = 1. 4 = 𝑓(1) = 6∕(1 + 𝑏 −1), 6 1 + 𝑏 −1 = 6∕4 = 3∕2, 𝑏 −1 = 1∕2, 𝑏 = 2. So, 𝑓(𝑥) = . 1 + 2 −𝑥

10. 𝑁 = 4, the limiting value. 1 = 𝑓(0) = 𝑁∕(1 + 𝐴) = 4∕(1 + 𝐴), so 𝐴 = 3. 2 = 𝑓(1) = 4∕(1 + 3𝑏 −1), 4 1 + 3𝑏 −1 = 4∕2 = 2, 𝑏 −1 = 1∕3, 𝑏 = 3. So, 𝑓(𝑥) = . 1 + 3(3 −𝑥)

11. B: The limiting value is 9, eliminating (A). The initial value is 2 = 9∕(1 + 3.5), not 9∕(1 + 0.5), so the answer is (B), not (C). 12. A: The limiting value is 8, eliminating (C). The initial value is 1 = 8∕(1 + 7) as in (A).

13. B: The graph decreases, so 𝑏 < 1, eliminating (C). The 𝑦-intercept is 2 = 8∕(1 + 3) as in (B).

14. C: The graph decreases, so 𝑏 < 1, eliminating (A). The 𝑦-intercept is 2.5 = 10∕(1 + 3) as in (C).

15. C: The initial value is 2, as in (A) or (C). If 𝑏 were 5, an increase in 1 in 𝑥 would multiply the value of 𝑓(𝑥) by approximately 5 when 𝑥 is small. However, increasing 𝑥 from 0 to 10 does not quite double the value. Hence 𝑏 must be smaller, as in (C). 16. C: The initial value is 2, as in (A) or (C). If 𝑏 were 15, an increase in 1 in 𝑥 would multiply the value of 𝑓(𝑥) by approximately 15 when 𝑥 is small. However, increasing 𝑥 from 0 to 20 does not quite double the value. Hence 𝑏 must be smaller, as in (C).


17. 𝑦 =

7.2 1 + 2.4(1.04) −𝑥

19. 𝑦 =

97 1 + 2.2(0.942) −𝑥

21. 𝑓(𝑥) = log4 𝑥. We plot some points and draw the graph: 𝑥 1/16

𝑓(𝑥)

−2

1/4 −1

1/2

−1∕2

18. 𝑦 =

10 1 + 2.5(1.04) −𝑥

20. 𝑦 =

75 1 + 1.5(0.980) −𝑥

Solutions Section 2.4

1

2

4

0

1/2

1

22. 𝑓(𝑥) = log5 𝑥. We plot some points and draw the graph: 𝑥 1/25

𝑓(𝑥)

−2

1/5 −1

1

5

0

1


23. 𝑓(𝑥) = log1∕4 𝑥. We plot some points and draw the graph:

24. 𝑓(𝑥) = log1∕5 𝑥. We plot some points and draw the graph:

Solutions Section 2.4

𝑥 1/16

𝑓(𝑥)

2

1/4

1/2

1

1

1/2

0

2

−1∕2

4

−1

25. 𝑓(𝑥) = log4 𝑥 + 1. We plot some points and draw the graph: 𝑥 1/16

𝑓(𝑥)

−1

1/4 0

1/2

1∕2

1

2

4

1

3/2

2

27. 𝑓(𝑥) = log4 (𝑥 − 1). We plot some points and draw the graph: 𝑥 17/16

𝑓(𝑥)

−2

5/4 −1

3/2

−1∕2

2

3

5

0

1/2

1

𝑥 1/25

𝑓(𝑥)

2

1/5

1

1

0

5

−1

26. 𝑓(𝑥) = log5 𝑥 − 1. We plot some points and draw the graph: 𝑥 1/25

𝑓(𝑥)

−3

1/5 −2

1

5 0

−1

28. 𝑓(𝑥) = log5 (𝑥 + 1). We plot some points and draw the graph: 𝑥 −24∕25 −4∕5

𝑓(𝑥)

−2

−1

0

4

0

1

29. a. We can eliminate (C) because 𝑏 = 0.8 is less than 1, giving a decreasing function of 𝑡. We can eliminate (D) because 𝑏 = 13.313, indicating an initial rate of growth of 1,331.3% per month—far faster than suggested by the data. (Alternatively, we can calculate ln 𝐴∕ ln 𝑏 ≈ 2 and notice that the point of inflection is further to the right than that.) The value 𝑁 = 44.6 in (B) predicts a leveling-off value of 44.6%, which is clearly too small. Thus, we are left with choice (A). b. Mid-January is in the initial period of the model, when the rate of growth is governed by exponential


Solutions Section 2.4 growth with base 𝑏 (𝐴 = 142 is much larger than 1). Since 𝑏 = 3.066, the number was being multiplied by 3.066 = 1 + 2.066 each month, so growing by around 207% per month at that time.

30. a. We can eliminate (B) because 𝑏 = 0.931 < 1, giving a decreasing function of 𝑡. We can eliminate (A) because 𝑏 = 3.83, indicating that the curve initially behaves like an exponential function with base 3.8, almost quadrupling every month. This is not what is suggested in the data. (Alternatively, we can calculate ln 𝐴∕ ln 𝑏 ≈ 3 and notice that the point of inflection is significantly further to the right than that.) The value 𝑁 = 60.32 in (C) predicts a horizontal asymptote of 60.32, which is clearly too small. Thus, we are left with choice (D). b. Mid-February is in the initial period of the model, when the rate of growth is governed by exponential growth with base 𝑏 (𝐴 = 104 is much larger than 1). Since 𝑏 = 2.88, the number was being multiplied by 2.88 = 1 + 1.88 each month, so growing by around 188% per month at that time.

31. a. We can eliminate (B) and (D) because 𝑏 = 0.8 is less than 1, giving a decreasing function of 𝑡. The value 𝑁 = 2.0 in (C) predicts a leveling-off value of around 2%, which is clearly too small. Thus, we are left with choice (A). ln 𝐴 ln 8.6 b. The logistic curve is steepest when 𝑡 = . This occurs when 𝑡 = ≈ 3.7 years since 2000; during ln 𝑏 ln 1.8 2003. 32. a. We can eliminate (D) because 𝑏 = 0.8 is less than 1, giving a decreasing function of 𝑡. The leveling-off value in the graph is around 𝑁 = 1, 300, so we can eliminate (A) and (C) Thus, we are left with choice (B). ln 𝐴 ln 4.2 b. The logistic curve is steepest when 𝑡 = . This occurs when 𝑡 = ≈ 2.7 years since 2000; during ln 𝑏 ln 1.7 2002. 33. a. The limiting value of a logistic function with 𝑏 < 1 is 0, so the long-term projected emission would be zero. 𝑁 49.2 b. For large 𝑡, 𝐶(𝑡) ≈ 𝑏 𝑡 = (0.957) 𝑡 ≈ 223.6(0.957) 𝑡. 𝐴 0.22 c. Set 𝐶(𝑡) = 20 and solve for 𝑡: 49.2 20 = 1 + 0.22(0.957) −𝑡 49.2 1 + 0.22(0.957) −𝑡 = = 2.46 20 log((2.46 − 1)∕0.22) 𝑡=− ≈ 43, log(0.957) representing 2063. 34. a. For 𝑏 < 1 the parameter 𝑁 is the value of the logistic function for large negative values of 𝑡, so, if the model were to be extrapolated, it would indicate that sulfur emissions in the distant past were approximately 90 metric tons above the 47-ton baseline. 𝑁 89.79 b. For large 𝑡, 𝑆(𝑡) ≈ 𝑏 𝑡 = (0.91) 𝑡 ≈ 498.83(0.91) 𝑡. 𝐴 0.18 c. Set 𝑆(𝑡) = 25 and solve for 𝑡: 89.79 25 = 1 + 0.18(0.91) −𝑡 89.79 1 + 0.18(0.91) −𝑡 = = 3.5916 25 log((3.5916 − 1)∕0.18) 𝑡=− ≈ 28, log(0.91) representing 2048.


Solutions Section 2.4 35. 𝑁 = 10, 000, the susceptible population. 1, 000 = 10, 000∕(1 + 𝐴), so 𝐴 = 9. In the initial stages, the rate 10, 000 of increase was 25% per day, so 𝑏 = 1.25. Thus, 𝑁(𝑡) = . 𝑁(7) ≈ 3, 463 cases. 1 + 9(1.25) −𝑡 36. 𝑁 = 10, 000, the susceptible population. 1 = 10, 000∕(1 + 𝐴), so 𝐴 = 9, 999. In the initial stages, the rate 10, 000 of increase was 40% per day, so 𝑏 = 1.4. Thus, 𝑁(𝑡) = ; 𝑁(21) ≈ 1, 049 cases. 1 + 9, 999(1.4) −𝑡

37. 𝑁 = 3, 000, the total available market. 100 = 3, 000∕(1 + 𝐴), so 𝐴 = 29. Sales are initially doubling every 3, 000 5 days, so 𝑏 5 = 2, or 𝑏 = 2 1∕5. Thus, 𝑁(𝑡) = . Set 𝑁(𝑡) = 700 and solve for 𝑡: 1 + 29(2 1∕5) −𝑡 700 = 3, 000∕[1 + 29(2 1∕5) −𝑡], 1 + 29(2 1∕5) −𝑡 = 30∕7, (2 1∕5) −𝑡 = 23∕(29 × 7), 𝑡 = − log[23∕(29 × 7)]∕ log(2 1∕5) ≈ 16 days. 38. 𝑁 = 100, the saturation level. 2 = 100∕(1 + 𝐴), so 𝐴 = 49. Sales are initially doubling every 2 years, so 100 𝑏 2 = 2, or 𝑏 = √2. Thus, 𝑁(𝑡) = . Set 𝑁(𝑡) = 50 and solve for 𝑡: 1 + 49(√2) −𝑡 50 = 100∕[1 + 49(√2) −𝑡], 1 + 49(√2) −𝑡 = 2, (√2) −𝑡 = 1∕49, 𝑡 = − log(1∕49)∕ log(√2) ≈ 11 years, or sometime in 2004. 39. a. You can use the Evaluator and Grapher app (or the version online at the Website) as follows: 1. In the "Points and curve-fit" window (the curve with plotted points icon in the app) enter the data and choose the logistic model either by pressing "Examples: a few times or typing it in directly: $1/(1+$2*$3^(-x)) 2. Press "Fit curve." 3. To see the graph and points in a convenient window, in "Graph settings" window (the "f(x)" icon in the app) put XMin = 0 and xMax = 100, and then press "Plot graphs" (or the graph icon in the app)

The resulting regression equation, as a function of 𝑥, shown in the "Enter function of x" window (f(x) button in the app) is, with coefficients rounded to four significant digits: 𝐶(𝑡) =

15.71 1 + 82.88(1.079) −𝑡

b. To predict the 2018 value, use 𝑡 = 48 (or enter 48 as 𝑥 in the "Evaluator" window (table icon in the app) and press "Evaluate"): 𝐶(45) =

15.71 ≈ 5.0 million dollars, slightly lower than the observed $5.2 million value. 1 + 82.88(1.079) −48

c. The long-term prediction of the price is 𝑁 ≈ $15.7 million. d. The answer to part (c) of this exercise is significantly higher than the prediction based on the data in the next exercise. The given data does not extend beyond the apparent point of inflection of the logistic curve, which is a significant distance from the point where the curve levels off, so we should not expect a reliable prediction of the long-term value. 40. a. You can use the Evaluator and Grapher app (or the version online at the Website) as follows: 1. In the "Points and curve-fit" window (the curve with plotted points icon in the app) enter the data and


Solutions Section 2.4 choose the logistic model either by pressing "Examples: a few times or typing it in directly: $1/(1+$2*$3^(-x)) 2. Press "Fit curve." 3. To see the graph and points in a convenient window, in "Graph settings" window (the "f(x)" icon in the app) put XMin = 0 and xMax = 100, and then press "Plot graphs" (or the graph icon in the app)

The resulting regression equation, as a function of 𝑥, shown in the "Enter function of x" window (f(x) button in the app) is, with coefficients rounded to four significant digits: 𝐶(𝑡) =

11.34 1 + 54.25(1.094) −𝑡

b. To predict the 2018 value, use 𝑡 = 43 (or enter 43 as 𝑥 in the "Evaluator" window (table icon in the app) and press "Evaluate"): 𝐶(43) =

11.34 ≈ 5.3 million dollars, slightly higher than the observed $5.2 million value. 1 + 54.25(1.094) −43

c. The long-term prediction of the price is 𝑁 ≈ $11.3 million. d. The answer to part (c) of this exercise is significantly lower than the $15.7 million prediction based on the data in the preceding exercise. The given data does not extend beyond the apparent point of inflection of the logistic curve, which is a significant distance from the point where the curve levels off, so we should not expect a reliable prediction of the long-term value.

41. a. Following Example 2, if using Excel, start with initial rough estimates of 𝑁, 𝐴, and 𝑏. (Their exact value is not important.) 𝑁 = leveling-off value ≈ 1, 100. 𝑏 = 1.1 (slightly greater than 1, since 𝑁 is increasing with 𝑡). 𝐴: Use 𝑦-intercept = 𝑁∕(1 + 𝐴): 1, 100 767 = 1+𝐴 1 + 𝐴 = 1, 100∕767 ≈ 1.4. So, 𝐴 ≈ 0.4. Solver then gives the following solution: 𝑁 ≈ 1, 090, 𝐴 ≈ 0.410, 𝑏 ≈ 1.09. Thus, the regression model is 1, 090 𝐵(𝑡) = . 1 + 0.410(1.09) −𝑡 The model predicts that the number will level off around 𝑁 = 1, 090 teams. ln 𝐴 ln 0.410 b. The logistic curve is steepest when 𝑡 = . This occurs when 𝑡 = ≈ −10.3. This means that, ln 𝑏 ln 1.09 according to the model, the number of teams was rising fastest about 10.3 years prior to 1990; that is, sometime during 1979. c. Since the coefficient of 𝑏 is 1.09, the number of men's basketball teams was growing by about 9% per year in the past, well before 1979. 42. a. Following Example 2, if using Excel, start with initial rough estimates of 𝑁, 𝐴, and 𝑏. (Their exact value is not important.) 𝑁 = leveling-off value ≈ 1, 050. 𝑏 = 1.1 (slightly greater than 1, since 𝑁 is increasing with 𝑡). 𝐴: Use 𝑦-intercept = 𝑁∕(1 + 𝐴): 1, 050 782 = 1+𝐴 1 + 𝐴 = 1, 050∕782 ≈ 1.3. So, 𝐴 ≈ 0.3. Solver then gives the following solution: 𝑁 ≈ 1, 080, 𝐴 ≈ 0.401, 𝑏 ≈ 1.12. Thus, the regression model is 1, 080 𝐵(𝑡) = . 1 + 0.401(1.12) −𝑡


Solutions Section 2.4 The model predicts that the number will level off around 𝑁 = 1, 080 teams. ln 𝐴 ln 0.401 b. The logistic curve is steepest when 𝑡 = . This occurs when 𝑡 = ≈ −8.06. This means that, ln 𝑏 ln 1.12 according to the model, the number of teams was rising fastest about 8.06 years prior to 1990; that is, sometime during 1981. c. Since the coefficient of 𝑏 is 1.12, the number of women's basketball teams was growing by about 12% per year in the past, well before 1981. 43. For a limiting value of 4.5 million, we take 𝑁 = 4, 500. Here is the set of values (𝑥, 𝑁∕𝑦 − 1) with 𝑁 = 4, 500: 𝑥

0

10

20

30

40

55

60

65

70

75

50

𝑁∕𝑦 − 1 1.02703 0.90678 0.65441 0.40187 0.27119 0.17801 𝑥

𝑁∕𝑦 − 1 0.15090 0.12782 0.11111 0.10294 0.09756

The exponential regression equation is 𝑦 ≈ 1.1466(0.96551) 𝑥 so that 𝐴 = 1.1466. 𝑏 −1 = 0.96551, giving 𝑏 = 1∕0.96551 ≈ 1.0357. Thus the logistic model is 4, 500 𝑦= , 1 + 1.1466(1.0357) −𝑡 To predict when 𝑦 = 4, 000, we set 4, 500 4, 000 = 1 + 1.1466(1.0357) −𝑡 and solve for 𝑡: 4, 500 1 + 1.1466(1.0357) −𝑡 = = 1.125 4, 000 so (1.0357) −𝑡 = (1.125 − 1)∕1.1466 ≈ 0.1090, giving ln(0.1090) 𝑡=− ≈ 63, or 2013. ln(1.0357)

44. For a limiting value of 110,000 million, we take 𝑁 = 110. Here is the set of values (𝑥, 𝑁∕𝑦 − 1) with 𝑁 = 110: 𝑥

0

10

20

30

40

55

60

65

70

75

50

𝑁∕𝑦 − 1 3.07407 2.33333 0.74603 0.12245 0.05769 0.03774 𝑥

𝑁∕𝑦 − 1 0.01852 0.01852 0.02804 0.02804 0.01852

The exponential regression equation is 𝑦 ≈ 2.3596(0.92877) 𝑥 so that 𝐴 = 2.3596. 𝑏 −1 = 0.92877, giving 𝑏 ≈ 1.07670. Thus the logistic model is 110 𝑦= . 1 + 2.3596(1.0767) −𝑡 To predict when 𝑦 = 80, we set 110 80 = 1 + 2.3596(1.0767) −𝑡 and solve for 𝑡: 110 1 + 2.3596(1.0767) −𝑡 = = 1.12 80 so (1.0767) −𝑡 = (1.125 − 1)∕2.3596 ≈ 0.0530, giving ln(0.0530) 𝑡=− ≈ 40, or 1990. ln(1.0767)


Solutions Section 2.4

45. a. Substitute: 2 7.9 = log 𝑀0 − 10.7. 3 Solve for log 𝑀0 : 3 log 𝑀0 = (7.9 + 10.7) = 27.9, 2 𝑀0 = 10 27.9 ≈ 7.943 × 10 27 ergs. b. Compute the energy released as in (a): 2 9.0 = log 𝑀0 − 10.7 3 3 log 𝑀0 = (9.0 + 10.7) = 29.55 2 𝑀 = 10 29.55 ≈ 3.548 × 10 29 ergs. Comparing, the energy released in 1906 was 7.943 × 10 27 ≈ 2.24% of the seismic moment in 2011. 3.548 × 10 29 c. Start with the given equation: 2 𝑅 = log 𝑀0 − 10.7 3 log 𝑀0 = 1.5(𝑅 + 10.7). In exponent form, this is 𝑀0 = 10 1.5(𝑅+10.7). d. Applying part (c) to each of the two earthquakes gives 𝑀1 = 10 1.5(𝑅1 +10.7) 𝑀2 = 10 1.5(𝑅2 +10.7). Dividing gives 𝑀2 10 1.5(𝑅2 +10.7) = = 10 1.5(𝑅2 −𝑅1 ). 𝑀1 10 1.5(𝑅1 +10.7) e. If 𝑅2 − 𝑅1 = 2, then 𝑀2 ∕𝑀1 = 10 1.5(2) = 10 3 = 1,000.

46. a. By substitution we find: Whisper: 21 dB, TV: 75 dB, Loud music: 120 dB, Jet aircraft: 140 dB b. Loud music and jet aircraft: These are the ones greater than 90 dB. 𝐼 c. 𝐷 = 10 log 𝐼0 𝐼 log = 0.1𝐷 𝐼0 𝐼 = 10 0.1𝐷 𝐼0 𝐼 = 𝐼0 10 0.1𝐷 d. From part (c) 𝐼1 = 𝐼0 10 0.1𝐷1 and 𝐼2 = 𝐼0 10 0.1𝐷2 . This gives 𝐼2 ∕𝐼1 = 10 0.1𝐷2 ∕10 0.1𝐷1 = 10 0.1(𝐷2 −𝐷1 ). e. If 𝐷2 − 𝐷1 = 1, then 𝐼2 ∕𝐼1 = 10 0.1 ≈ 1.259.

47. a. By substitution: 75 dB, 69 dB, 61 dB b. 𝐷 = 10 log(320 × 10 7) − 10 log 𝑟 2 ≈ 95 − 20 log 𝑟 c. Solve 0 = 95 − 20 log 𝑟: log 𝑟 = 95∕20, 𝑟 = 10 95∕20 ≈ 57, 000 feet (rounding up so that we're beyond the point where the decibel level is 0 dB). 48. a. By substitution: Blood: 7.4; Milk: 6.3; Soap solution: 11; Black coffee: 6.9 b. Substitute, then solve for 𝐻 +: 5.0 = − log(𝐻 +), 𝐻 + = 10 −5 moles. c. Solve for 𝐻 + in terms of the pH: 𝐻 + = 10 −𝑝𝐻 . So, if the pH increases by 1, 𝐻 + decreases to 10 −1 = 1∕10


Solutions Section 2.4 of the original amount. 49. a. To obtain the logarithmic regression equation for the data in Excel, do a scatter plot and add a Logarithmic trendline with the option "Display equation on chart" checked. On the TI-83/84 Plus, use [STAT], select CALC, and choose the option LnReg. The resulting regression equation with coefficients rounded to 3 digits is 𝑃 (𝑡) = 32.1 ln 𝑡 − 36.0. b. The over-65 population reaches 80 million when 32.1 ln 𝑡 − 36.0 = 80 116 ln 𝑡 = 32.1 𝑡 = 𝑒 116∕32.1 ≈ 37.1,

which is during 2037. c. The logarithm increases without bound (choice (A)). 50. a. To obtain the logarithmic regression equation for the data in Excel, do a scatter plot and add a Logarithmic trendline with the option "Display equation on chart" checked. On he TI-83/84 Plus, use [STAT], select CALC, and choose the option LnReg. The resulting regression equation with coefficients rounded to 3 digits is 𝑃 (𝑡) = 8.06 ln 𝑡 − 15.7. b. The over-85 population reaches 16 million when 8.06 ln 𝑡 − 15.7 = 16 31.7 ln 𝑡 = 8.06 𝑡 = 𝑒 31.7∕8.06 ≈ 51.1,

which is during 2051. c. The logarithm term eventually becomes much larger than the constant term, so the larger coefficient on the logarithm will eventually give a higher predicted percentage (choice (C)). 51. Following is the Excel tabulation of the data, together with the scatter plot and the logarithmic trendline (with the option "Display equation on chart" checked):

From the regression output, the model is 𝑆(𝑡) = 42.2 ln 𝑡 + 84.3 (coefficients rounded to three significant digits). Extrapolating in the positive direction results in a prediction of ever-increasing R&D expenditures by industry. This is reasonable to a point, as expenditures cannot reasonably be expected to increase without bound. Extrapolating in the negative direction eventually leads to negative values, which do not model reality. 52. Following is the Excel tabulation of the data, together with the scatter plot and the logarithmic trendline (with the option "Display equation on chart" checked):


Solutions Section 2.4

From the regression output, the model is 𝑆(𝑡) = 5.76 ln 𝑡 + 3.46 (coefficients rounded to three significant digits). Extrapolating in the positive direction results in a prediction of ever-increasing R&D expenditures by the government. This is reasonable to a point, as expenditures cannot reasonably be expected to increase without bound. Extrapolating in the negative direction eventually leads to negative values, which do not model reality. 53. a. To obtain the logarithmic regression equation for the data in a spreadsheet like Excel, do a scatter plot and add a Logarithmic trendline with the option "Display equation on chart" checked. On the TI-83/84 Plus, use [STAT], select CALC, and choose the option LnReg. The resulting regression equation with coefficients rounded to 3 digits is 𝑈(𝑡) = −8.25 ln 𝑡 + 50.9. 2030 corresponds to 𝑡 = 30, and 𝑈(30) = −8.25 ln 30 + 50.9 ≈ 22.8% b. For the logistic curve, 184 𝑉 (30) = 26.9 + ≈ 26.9%. 1 + 2.09(0.746) −30 The graph of the U.S. share appears to level off between 25% and 30%, which agrees with the logistic curve projection for 2030. The logarithmic model projects a significantly lower value, and so appears less reasonable. c. As the logarithm curve increases without bound as 𝑡 gets large, the negative sign causes it to decrease without bound, so the values of 𝑈(𝑡) will eventually become larger and larger negative. On the other hand, 𝑉 (𝑡) approaches 26.9 + 0 = 26.9% (using the propery for the long-term behavior of a logistic function with 𝑏 < 1), confirming the conclusion in part (b). 54. a. To obtain the logarithmic regression equation for the data in a spreadsheet like Excel, do a scatter plot and add a Logarithmic trendline with the option "Display equation on chart" checked. On the TI-83/84 Plus, use [STAT], select CALC, and choose the option LnReg. The resulting regression equation with coefficients rounded to 3 digits is 𝐶(𝑡) = 16.3 ln 𝑡 − 20.7. 2030 corresponds to 𝑡 = 30, and 𝐶(30) = 16.3 ln 30 − 20.7 ≈ 34.7% b. For the logistic curve, 28.6 𝐷(30) = ≈ 28.5%. 1 + 10.4(1.30) −30 The graph of the China share appears to level off between 25% and 30%, which agrees with logistic curve projection. The logarithmic model projects a considerably higher value, and so appears less reasonable. c. The logarithm curve increases without bound as 𝑡 gets large, so the values of 𝐶(𝑡) will eventually become larger and larger without bound. On the other hand, 𝐷(𝑡) approaches 28.6% (using the propery for the longterm behavior of a logistic function with 𝑏 > 1), confirming the conclusion in part (b). 55. Just as diseases are communicated via the spread of a pathogen (such as a virus), new technology is communicated via the spread of information (such as advertising and publicity). Further, just as the spread of a disease is ultimately limited by the number of susceptible individuals, so the spread of a new technology is


Solutions Section 2.4 ultimately limited by the size of the potential market. 56. Exponential functions grow large without bound and fail to take into account the limited size of the susceptible population. Logistic functions correctly predict a slowing in the spread of an epidemic as the number of cases grows toward the total susceptible population. 57. It can be used to predict where the sales of a new commodity might level off. 58. Answers may vary. Life expectancy figures (Example 5 should (optimistically) continue to increase without a definite bound as predicted by a logistic model. Similarly, the total mount of funding on R&D (Exercises 51 and 52) could conceivably continue to increase without bound as does a logarithmic model. 59. The curve is still a logistic curve, but decreases when 𝑏 > 1 and increases when 𝑏 < 1.

60. If 𝐴 = 0, then the function is constant: 𝑃 (𝑡) = 𝑁. If 𝐴 < 0, then the function will have a vertical asymptote at 𝑡 = ln 𝐴∕ ln 𝑏; to the left of the asymptote it approaches 0; to the right it decreases to 𝑁. ln 𝐴 in the formula for 𝑦 gives ln 𝑏 𝑁 𝑁 𝑦= . = ln 𝐴∕ ln 𝑏 − 1 + 𝐴𝑏 1 + 𝐴𝑏 − ln𝑏 𝐴 1 1 But 𝑏 − ln𝑏 𝐴 = = , so 𝑏 ln𝑏 𝐴 𝐴 𝑁 𝑁 𝑦= = = 𝑁∕2. 1 + 𝐴(1∕𝐴) 1 + 1 61. Substituting 𝑡 =

62. For 𝑡 = ln 𝐴∕ ln 𝑏 to be positive, the numerator and denominator must have the same sign. Case 1: Both positive: In this case, both 𝐴 and 𝑏 must be > 1. Case 2: Both negative: In this case, both 𝐴 and 𝑏 must be < 1. For 𝑡 = ln 𝐴∕ ln 𝑏 to be negative, the numerator and denominator must have opposite sign. That is, either 𝐴 > 1 and 𝑏 < 1, or 𝐴 < 1 and 𝑏 > 1. For 𝑡 = ln 𝐴∕ ln 𝑏 to be zero, ln 𝐴 must be zero; that is, 𝐴 = 1. 63. Graph:

The lowest graph on the right is 𝑦 = ln 𝑥. The middle graph is 𝑦 = 2 ln 𝑥, and the uppermost graph is 𝑦 = 2 ln 𝑥 + 0.5. Multiplying by 𝐴 stretches the graph in the 𝑦-direction by a factor of 𝐴. Adding 𝐶 moves the graph 𝐶 units vertically up.


Solutions Section 2.4 64. Graph:

The top graph on the right is 𝑦 = ln 𝑥, the one ending below that is − ln 𝑥. The second lowest graph is 𝑦 = −2 ln 𝑥, and the lowest graph is 𝑦 = −2 ln 𝑥 − 0.5. Multiplying by a negative 𝐴 flips the graph vertically and stretches the graph in the 𝑦-direction by a factor of |𝐴|. Adding a negative 𝐶 moves the graph |𝐶| units vertically down. 65. The logarithm of a negative number, were it defined, would be the power to which a base must be raised to give that negative number. But raising a base to a power never results in a negative number, so there can be no such number as the logarithm of a negative number. 66. They can be used to solve equations analytically for an exponent, as in the calculation of the time an investment should be held to yield a given return.

67. Any logarithmic curve 𝑦 = log𝑏 𝑡 + 𝐶 will eventually surpass 100% and hence will not be suitable as a long-term predictor of market share. 68. Any logarithmic curve 𝑦 = log𝑏 𝑡 + 𝐶 will become negative as 𝑡 gets close to zero, and hence not be suitable for indefinite backward extrapolation. 69. Time is increasing logarithmically with population: Solving 𝑃 = 𝐴𝑏 𝑡 for 𝑡 gives 𝑡 = log𝑏 (𝑃 ∕𝐴) = log𝑏 𝑃 − log𝑏 𝐴, which is of the form 𝑡 = log𝑏 𝑃 + 𝐶.

70. Time is increasing exponentially with population: Solving 𝑃 = log𝑏 𝑡 + 𝐶 for 𝑡 gives 𝑡 = 𝑏 (𝑃 −𝐶) = 𝑏 −𝐶 𝑏 𝑃 , which is of the form 𝑡 = 𝐴𝑏 𝑃 .

71. For the two function, write 𝑄1 = 𝐴1 ln 𝑡 + 𝐵1 𝑄2 = 𝐴2 ln 𝑡 + 𝐵2 . Adding gives 𝑄1 + 𝑄2 = 𝐴1 ln 𝑡 + 𝐵1 + 𝐴2 ln 𝑡 + 𝐵2 = (𝐴1 + 𝐴2 ) ln 𝑡 + (𝐵1 + 𝐵2 ), which is of the form Constant× ln 𝑡+Constant, and is thus a logarithmic function. 72. The units in the first model are millions of dollars and in the second billions of dollars, so we multiply the second by 1,000 to convert it to millions of dollars as well. Using more significant digits in the two regression models, their sum is 42.181 ln 𝑡 + 84.304 + 5,757.6 ln 𝑡 + 3,457.4 ≈ 5,799.8 ln 𝑡 + 3,541.7. The regression model of the sum of the data (again, multiplying the second set of data by 1,000) is 5,799.8 ln 𝑡 + 3,541.7, which agrees with the sum to five significant digits. This suggests that the sum of logarithmic regression


Solutions Section 2.4 models is the regression model of the sum.


Solutions Chapter 2 Review Chapter 2 Review

1. 𝑎 = 1, 𝑏 = 2, 𝑐 = −3, so −𝑏∕(2𝑎) = −1; 𝑓(−1) = −4, so the vertex is (−1, −4). 𝑦-intercept: 𝑐 = −3. 𝑥 2 + 2𝑥 − 3 = (𝑥 + 3)(𝑥 − 1), so the 𝑥-intercepts are −3 and 1. 𝑎 > 0, so the parabola opens upward.

2. 𝑎 = −1, 𝑏 = −1, 𝑐 = −1, so −𝑏∕(2𝑎) = −1∕2; 𝑓(−1∕2) = −3∕4, so the vertex is (−1∕2, −3∕4). 𝑦-intercept: 𝑐 = −1. 𝑏 2 − 4𝑎𝑐 = −3 < 0, so there are no 𝑥-intercepts. 𝑎 < 0, so the parabola opens downward.

3. For every increase in 𝑥 by 1 unit, the value of 𝑓 is multiplied by 1∕2, so 𝑓 is exponential. Since 𝑓(0) = 5, the exponential model is 𝑓(𝑥) = 5(1∕2) 𝑥, or 5(2 −𝑥). 𝑔(2) = 0, whereas exponential functions are never zero, so 𝑔 is not exponential. 4. The values of 𝑓 decrease by 2 for every increase in 𝑥 by 1 unit, so 𝑓 is linear (not exponential). For every increase in 𝑥 by 1 unit, the value of 𝑔 is multiplied by 2, so 𝑔 is exponential. Since 𝑔(0) = 3, the exponential model is 𝑔(𝑥) = 3(2 𝑥). 5. We use the table of values shown on the left to obtain the graph shown on the right:


Solutions Chapter 2 Review

𝑥

-3

-2

-1

0

1

2

3

𝑔(𝑥) 27/2

1/18

1/6

1/2

3/2

9/2

27/2

9/2

3/2

1/2

1/6

1/18

1/54

𝑓(𝑥) 1/54

6. We use the table of values shown on the left to obtain the graph shown on the right:

𝑥

-3

-2

-1

0

1

2

3

𝑔(𝑥) 128

1/8

1/2

2

8

32

128

32

8

2

1/2

1/8

1/32

𝑓(𝑥) 1/32

7. The technology formulas for the two functions are TI-83/84 Plus:\quad e^x; e^(0.8*x) Excel:\quad =EXP(x); =EXP(0.8*x)

8. The technology formulas for the two functions are TI-83/84 Plus and Excel:\quad 2*1.01^x ; 2*0.99^x Technology gives us the following graphs:

Technology gives us the following graphs:

9. Use 𝐴 = 𝑃 !1 +

𝑟 𝑚𝑡 0.03 60 " ; 𝑃 = 3, 000, 𝑟 = 0.034; 𝑚 = 12; 𝑡 = 5. So, 𝐴 = 3, 000!1 + " ≈ $3, 484.85. 𝑚 12


𝑟 " ; 𝑃 = 10, 000, 𝑟 = 0.025; 𝑚 = 4; 𝑡 = 10. So, 𝑚 0.025 40 𝐴 = 10, 000!1 + " ≈ $12, 830.27. 4 10. Use 𝐴 = 𝑃 !1 +

𝑚𝑡

Solutions Chapter 2 Review

𝑟 𝑚𝑡 " ; 𝐴 = 5, 000, 𝑟 = 0.03; 𝑚 = 12; 𝑡 = 10. Substituting gives 𝑚 0.03 120 5, 000 = 𝑃 !1 + " ⇒𝑃 = 5, 000∕(1 + 0.03∕12) 120 ≈ $3, 705.48. 12

11. Use 𝐴 = 𝑃 !1 +

𝑟 𝑚𝑡 " ; 𝐴 = 10, 000, 𝑟 = 0.025; 𝑚 = 4; 𝑡 = 10. Substituting gives 𝑚 0.025 40 10, 000 = 𝑃 !1 + " ⇒𝑃 = 10, 000∕(1 + 0.025∕4) 40 ≈ $7, 794.07. 4

12. Use 𝐴 = 𝑃 !1 +

13. Use 𝐴 = 𝑃 𝑒 𝑟𝑡; 𝑃 = 3, 000, 𝑟 = 0.03; 𝑡 = 5. So, 𝐴 = 3, 000𝑒 0.15 ≈ $3, 485.50.

14. Use 𝐴 = 𝑃 𝑒 𝑟𝑡; 𝑃 = 10, 000, 𝑟 = 0.025; 𝑡 = 10. So, 𝐴 = 10, 000𝑒 0.25 ≈ $12, 840.25.

15. Increasing 𝑥 by one half unit triples the value of 𝑓. Therefore, increasing 𝑥 by 1 unit multiplies the value of 𝑓 by 9, giving 𝑏 = 9. 𝐴 = 𝑓(0) = 4.5. Therefore, 𝑓(𝑥) = 𝐴𝑏 𝑥 = 4.5(9 𝑥).

16. Increasing 𝑥 by 1 unit decreases the value of 𝑓 by 75%. That is, it reduces the value of 𝑓 to 25% of its original value. Therefore, 𝑏 = 0.25. 𝐴 = 𝑓(0) = 5. Therefore, 𝑓(𝑥) = 𝐴𝑏 𝑥 = 5(0.25 𝑥). 17. 2 = 𝐴𝑏 1 and 18 = 𝐴𝑏 3. Dividing, 𝑏 2 = 9, so 𝑏 = 3. Then 2 = 3𝐴, so 𝐴 = 2∕3: 𝑓(𝑥) =

2 𝑥 3 . 3

18. 10 = 𝐴𝑏 1 and 5 = 𝐴𝑏 3. Dividing, 𝑏 2 = 1∕2, so 𝑏 = 1∕√2. Then 10 = 𝐴∕√2, so 𝐴 = 10√2: 1 𝑥 𝑓(𝑥) = 10√2! " . √2 19. We use the following table of values: 𝑥 1/27

𝑓(𝑥) 𝑔(𝑥)

1/9

1/3

1

3

9

27

-3

-2

-1

0

1

2

3

3

2

1

0

-1

-2

-3

Graphing these gives the following curves:


Solutions Chapter 2 Review 20. We use the following table of values: 𝑥 1/1,000 1/100 1/10

𝑓(𝑥) 𝑔(𝑥)

1

10

100 1,000

-3

-2

-1

0

1

2

3

3

2

1

0

-1

-2

-3

Graphing these gives the following curves:

21. 𝑄0 = 5 (given) 𝑡ℎ 𝑘 = ln 2 100𝑘 = ln 2 𝑘 = (ln 2)∕100 ≈ 0.00693 𝑄 = 𝑄0 𝑒 −𝑘𝑡 = 5𝑒 −0.00693𝑡

22. 𝑄0 = 10, 000 (given) 𝑡ℎ 𝑘 = ln 2 5𝑘 = ln 2 𝑘 = (ln 2)∕5 ≈ 0.139 𝑄 = 𝑄0 𝑒 −𝑘𝑡 = 10, 000𝑒 −0.139𝑡

23. 𝑄0 = 2.5 (given) 𝑡𝑑 𝑘 = ln 2 2𝑘 = ln 2 𝑘 = (ln 2)∕2 ≈ 0.347 𝑄 = 𝑄0 𝑒 𝑘𝑡 = 2.5𝑒 0.347𝑡

24. 𝑄0 = 10, 000 (given) 𝑡𝑑 𝑘 = ln 2 15𝑘 = ln 2 𝑘 = (ln 2)∕15 ≈ 0.0462 𝑄 = 𝑄0 𝑒 𝑘𝑡 = 10, 000𝑒 0.0462𝑡

25. 3, 000 = 2, 000(1 + 0.04∕12) 12𝑡 𝑡 = [log(3∕2)∕ log(1 + 0.04∕12)]∕12 ≈ 10.2 years

26. 3, 000 = 2, 000(1 + 0.0675∕365) 365𝑡 𝑡 = [log(3∕2)∕ log(1 + 0.0675∕365)]∕365 ≈ 6 years 27. 3, 000 = 2, 000𝑒 0.0375𝑡 𝑡 = ln(3∕2)∕0.0375 ≈ 10.8 years

28. 1, 200 = 1, 000(1 + 1∕4) 4𝑡 = 1, 000(1.25 4𝑡) 𝑡 = [log(1, 200∕1, 000)∕ log(1.25)]∕4 ≈ 0.2 years

29. 𝑁 = 900 is given, 100 = 900∕(1 + 𝐴) gives 𝐴 = 8, initially increasing 50% per unit increase in 𝑥 gives 𝑏 = 1.5: 900 𝑓(𝑥) = . 1 + 8(1.5) −𝑥 30. 𝑁 = 25 is given, 5 = 25∕(1 + 𝐴) gives 𝐴 = 4, 𝑏 = 1.1 is given in the initial exponential form: 25 𝑓(𝑥) = . 1 + 4(1.1) −𝑥


Solutions Chapter 2 Review 31. 𝑁 = 20 is given, 20∕(1 + 𝐴) = 5, so 𝐴 = 3, decreasing at a rate of 20% per unit of 𝑥 near 0 means 𝑏 = 1 − 0.2 = 0.8: 20 𝑓(𝑥) = . 1 + 3(0.8) −𝑥 32. 𝑁 ≈ 10 and 𝑏 = 0.8 are given, and for small 𝑥, 𝑁∕(1 + 𝐴) = 10∕(1 + 𝐴) = 2, so 𝐴 = 4, giving 10 𝑓(𝑥) = . 1 + 4(0.8) −𝑥

33. a. The largest volume will occur at the vertex: −𝑏∕(2𝑎) = 0.085∕(2 × 0.000005) ≈ $8, 500 per month; substituting 𝑐 = 8, 500 gives ℎ = an average of approximately 2,100 hits per day. b. Solve ℎ = 0 using the quadratic formula: 𝑐 ≈ $29, 049 per month (the other solution given by the quadratic formula is negative). c. The fact that −0.000005, the coefficient of 𝑐 2, is negative. 34. a. Since 𝑡 is time in years since the start of 2000 and the data are from the start of 2001 to the end of 2003, the domain is 1 ≤ 𝑡 ≤ 4, or [1, 4]. b. The growth rate is a minimum at the vertex: 𝑏 −6 𝑡=− =− = 1.5, 2𝑎 4 midway through 2001. c. A zero rate of growth would correspond to an intercept of the 𝑡-axis. However, the discriminant 𝑏 2 − 4𝑎𝑐 = 36 − 4(2)(12) = −60 is negative, so there are no 𝑡-intercepts, and hence no zero rate of growth. d. The fact that the coefficient of 𝑡 2 is positive. e. No. What was decreasing was the number of new broadband users. Put another way, the number of broadband users was growing at a declining rate in the first half of 2001. f. The beginning of 2013 is 𝑡 = 13; 𝑛(13) = 2(13) 2 − 6(13) + 12 = 272 million new users per year at the start of 2013 𝑛(14) = 2(14) 2 − 6(14) + 12 = 320 million new users per year at the start of 2014. These numbers are unreasonably large (recall that they represent new users per year). In fact, their sum exceeds the entire U.S. population.

35. a. 𝑅 = 𝑝𝑞 = −60𝑝 2 + 950𝑝. The maximum revenue occurs at the vertex: 𝑝 = −𝑏∕(2𝑎) = 950∕(2 × 60) = $7.92 per novel. At that price the monthly revenue is 𝑅 = $3, 760.42. b. 𝐶 = 900 + 4𝑞 = 900 + 4(−60𝑝 + 950) = −240𝑝 + 4, 700, so 𝑃 = 𝑅 − 𝐶 = −60𝑝 2 + 950𝑝 − (−240𝑝 + 4, 700) = −60𝑝 2 + 1, 190𝑝 − 4, 700. The maximum monthly profit occurs at the vertex: 𝑝 = −𝑏∕(2𝑎) = 1, 190∕(2 × 60) = $9.92 per novel. At that price, the monthly profit is 𝑃 = $1, 200.42. 36. a. Demand Function: The given points are (𝑝, 𝑞) = (0, 2, 000) and (0.10, 1, 000). 𝑞 − 𝑞1 1, 000 − 2, 000 Slope: 𝑚 = 2 = = −10, 000 𝑝2 − 𝑝1 0.10 − 0 Intercept: 𝑏 = 2, 000 Thus, the demand equation is 𝑞 = 𝑚𝑝 + 𝑏 = −10, 000𝑝 + 2, 000. b. Revenue = 𝑝𝑞 = 𝑝(−10, 000𝑝 + 2, 000) = −10, 000𝑝 2 + 2, 000𝑝. The maximum revenue occurs at the vertex: 𝑝 = −𝑏∕(2𝑎) = 2, 000∕(20, 000) = $0.1, or 10¢ per newspaper. At that price the monthly revenue is 𝑅 = −10, 000(0.1) 2 + 2, 000(0.1) = $100. c. 𝐶 = 200 + 0.04𝑞 = 200 + 0.04(−10, 000𝑝 + 2, 000) = −400𝑝 + 280, so 𝑃 = 𝑅 − 𝐶 = −10, 000𝑝 2 + 2, 000𝑝 − (−400𝑝 + 280) = −10, 000𝑝 2 + 2, 400𝑝 − 280.


Solutions Chapter 2 Review The maximum monthly profit occurs at the vertex: 𝑝 = −𝑏∕(2𝑎) = 2, 400∕20, 000 = $0.12 per newspaper. At that price, the monthly profit is 𝑃 = −10, 000(0.12) 2 + 2, 400(0.12) − 280 = −136, a loss of $136. 37. a. 1997 corresponds to 𝑡 = 0, and so the harvest was 𝑛 = 9.1(0.81 0) = 9.1 million pounds. The value of the base 𝑏 = 0.81 of the exponential model tells us that the value each year is 0.81 times, or 81% of the value the previous year. In other words, the value decreases by 100 − 81 = 19 each year. b. 2013 corresponds to 𝑡 = 16, and 𝑛(16) = 9.1(0.81 16) ≈ 0.31, or about 310,000 pounds. 38. Let 𝑆 be the stock price and 𝑡 be the time in hours since the IPO. Model the stock price by 𝑆 = 𝐴𝑏 𝑡. Then 𝐴 = 10, 000 and 𝑏 3 = 2 (since the stock is doubling in price every 3 hours), so 𝑏 = 2 1∕3. After 8 hours, 𝑆 = 10, 000(2 1∕3) 8 = $63, 496.04. 39. The model for the lobster harvest from Exercise 37 is 𝑛(𝑡) = 9.1(0.81 𝑡) million pounds. We need to find the value of 𝑡 when it first dips below 200,000, which is 0.2 million pounds: 0.2 = 9.1(0.81 𝑡) 0.2∕9.1 = 0.81 𝑡 log(0.2∕9.1) = 𝑡 log(0.81) 𝑡 = log(0.2∕9.1)∕ log(0.81) ≈ 18.1. Thus, the harvest is still above 200,000 lobsters at 𝑡 = 18 (2015), but will be below 200,000 at 𝑡 = 19 (2016). 40. Solve 50, 000 = 10, 000(2 𝑡∕3): 𝑡 = 3 log 5∕ log 2 ≈ 7.0 hours.

41. In 2010 (𝑡 = 13), the harvest was 9.1(0.81 13) ≈ 0.5880. From that point on, the model is 0.5880(1.24) 𝑡 where 𝑡 is now time in years since 2010. In 2013, the size of the harvest is 0.5880(1.24) 3 ≈ 1.12 million pounds.

42. After 10 hours the stock is worth 10, 000(2 10∕3) = $100, 793.68. From this level it follows a new exponential curve with 𝐴 = 100, 793.68 and 𝑏 4 = 2∕3 (it loses 1/3 of its value every 4 hours), hence 𝑏 = (2∕3) 1∕4. Now solve 10, 000 = 100, 793.68(2∕3) 𝑡∕4 for 𝑡 = 4 log(10, 000∕100, 793.68)∕ log(2∕3) ≈ 22.8. Adding the first 10 hours, the stock will be worth $10,000 again 32.8 hours after the IPO. 43. We use the data shown in the following table: 𝑡

0

𝑛 11.6

2

4

6

8

10

12

8.3

3.0

1.4

1.6

1.2

0.8

The regression model is Excel: 𝑛(𝑡) = 9.5841𝑒 −0.224𝑡 = 9.5841(𝑒 −0.224) 𝑡 ≈ 9.5841(0.7993) 𝑡 TI-83/84 Plus and Website: 9.5841(0.79967) 𝑡 Rounding to 2 digits gives 𝑛(𝑡) = 9.6(0.80 𝑡) million pounds of lobster.


Solutions Chapter 2 Review 44. We use the data shown in the following table: 𝑡

0

𝑃 1.00

1

2

3

4

5

6

7

8

1.30

1.75

2.05

2.60

3.00

3.95

5.00

6.70

The regression model is Excel: 𝑃 (𝑡) = 1.0321𝑒 0.2276𝑡 = 1.0321(𝑒 0.2276) 𝑡 ≈ 1.03(1.26) 𝑡 TI-83/84 Plus and Web Site: 𝑦 = 1.03206(1.25564 𝑡) Rounding to 2 digits gives 𝑃 (𝑡) = 1.03(1.26) 𝑡. To predict numerically when the price passes $10 we can use a table of values in the TI-83/84 Plus, Excel, or the Website with y = 1.03*(1.26^x) 𝑥

1.03*(1.26^x)

0

1.03

1

1.2978

2

1.6352

3

2.0604

4

2.5961

5

3.2711

6

4.1215

7

5.1932

8

6.5434

9

8.2446

10

10.3883

Thus, the stock price first passes $10 at the end of the 10th hour.

45. C: (A) is true because 𝐿1 = 𝐿0 𝑒 −1∕𝑡 < 𝐿0 . (B) is true because 𝐿3𝑡 = 𝐿0 𝑒 −3 > 0. (D) is true because increasing 𝑡 decreases 𝑥∕𝑡, which increases 𝑒 −𝑥∕𝑡 (makes it closer to 1), hence increases 𝐿𝑥 for every 𝑥. (E) is true because 𝑒 −𝑥∕𝑡 is never 0. On the other hand, (C) is false because 𝐿𝑡 = 𝐿0 𝑒 −1 ≈ 0.37𝐿0 < 𝐿0 ∕2. 46. a. The given function has the form Constant + logistic function. The logistic function part levels off at 𝑁 = 4, 470. Therefore, adding the constant gives a leveling of at 4, 470 + 6, 050 = 10, 520. b. We solve 4, 470 10, 000 = 6, 050 + 1 + 14(1.73 −𝑡) 4, 470 3, 950 = 1 + 14(1.73 −𝑡) 1 + 14(1.73 −𝑡) = 4, 470∕3, 950 14(1.73 −𝑡) = 4, 470∕3, 950 − 1 1.73 −𝑡 = (4, 470∕3, 950 − 1)∕14 −𝑡 log(1.73) = log((4, 470∕3, 950 − 1)∕14) 𝑡 = − log((4, 470∕3, 950 − 1)∕14)∕ log(1.73) ≈ 8.5, about 8.5 weeks. c. Sales are rising most rapidly when 𝑡 = ln 𝐴∕ ln 𝑏 = ln 14∕ ln 1.73 ≈ 5 to the nearest week.


Solutions Chapter 2 Case Study Chapter 2 Case Study 1. Using, for example, a spreadsheet logarithmic regression, we obtain 𝑁(𝑡) ≈ 0.1895 ln 𝑡 + 19.665 2029: 𝑁(10) ≈ 0.1895 ln 10 + 19.665 ≈ 20.1013 2030: 𝑁(11) ≈ 0.1895 ln 11 + 19.665 ≈ 20.1194 2031: 𝑁(12) ≈ 0.1895 ln 12 + 19.665 ≈ 20.1359 2032: 𝑁(13) ≈ 0.1895 ln 13 + 19.665 ≈ 20.1511 2029–2030: 𝑁(11) − 𝑁(10) ≈ 0.0181: 18,100 students 2030–2031: 𝑁(12) − 𝑁(11) ≈ 0.0165: 16,500 students 2032–2031: 𝑁(13) − 𝑁(12) ≈ 0.0152: 15,200 students 2. The value of a logistic function 𝑁(𝑡) = shifted logistic function 𝑁(𝑡) = model is

𝑁(𝑡) =

𝑀 for large values of 𝑡 is approximately 𝑀. Thus, for a 1 + 𝐴𝑏 −𝑡

𝑀 + 𝐶, its value for large 𝑡 is approximately 𝑀 + 𝐶. Here, the 1 + 𝐴𝑏 −𝑡

0.407 + 19.7, 1 + 37.0(2.058 −𝑡)

so the value for large 𝑡 is approximately

𝑀 + 𝐶 = 0.407 + 19.7 ≈ 20.1 million students

3. The values of a logarithmic function 𝑁(𝑡) = 𝐴 ln 𝑥 + 𝐶 grow without bound for larger and larger values of 𝑡, so the logarithmic model 𝑁(𝑡) ≈ 0.1895 ln 𝑡 + 19.665 grows larger and larger without bound. 4. Using the current parameters as the guess, we obtain 𝑁(𝑡) =

0.964 + 19.65 1 + 13.4(1.394 −𝑡)

The new estimates for 2027–2029 are (to three significant digit): 0.964 2027: 𝑁(8) = + 19.65 ≈ 20.1 1 + 13.4(1.394 −8) 0.964 2028: 𝑁(9) = + 19.65 ≈ 20.2 1 + 13.4(1.394 −9) 0.964 2029: 𝑁(10) = + 19.65 ≈ 20.3 1 + 13.4(1.394 −10) Long term projection: 𝑀 + 𝐶 ≈ 0.964 + 19.65 ≈ 20.6 million students. 5. Using a spreadsheet with the smaller set of data, we obtain 𝑁(𝑡) = 0.1596 ln 𝑡 + 19.689

The new estimates for 2027–2029 to three significant digits: 2027: 𝑁(8) = 0.1596 ln 8 + 19.689 ≈ 20.0 2028: 𝑁(9) = 0.1596 ln 9 + 19.689 ≈ 20.0 2029: 𝑁(10) = 0.1596 ln 10 + 19.689 ≈ 20.1 Long term projection: Enrollment increases without bound. 6. Answers may vary: If, for example, the college-age population in the U.S. were to eventually level off without significant increases, then a (shifted) logistic model would be more realistic, as a logarithmic


Solutions Chapter 2 Case Study model predicts ever-increasing college populations. On the other hand, if the college-age population in the U.S. continues to grow, but at a smaller and smaller rate, then a logarithmic model would be more realistic for fairly long-term projections (even though the logarithmic model eventually increases without bound the time it needs to make measureable increases grows exponentially).


Solutions Section 3.1 Section 3.1

1. = 2,000, = 0.06, = 1 = = 2,000(0.06)(1) = $120 = + = 2,000 + 120 = $2,120

2. = 1,000, = 0.04, = 10 = = 1,000(0.04)(10) = $400 = + = 1,000 + 400 = $1,400

3. = 4,000, = 0.005, = 8 = = 4,000(0.005)(8) = $160 = + = 4,000 + 160 = $4,160

4. = 2,000, = 0.001, = 12 = = 2,000(0.001)(12) = $24 = + = 2,000 + 24 = $2,024

5. = 20,200, = 0.05, = 1 2 = = 20,200(0.05)(1 2) = $505 = + = 20,200 + 505 = $20,705

6. = 10,100, = 0.11, = 1 4 = = 10,100(0.11)(1 4) = $277.75 = + = 10,100 + 277.75 = $10,377.75

7. = 10,000, = 0.03, = 10 12 = = 10,000(0.03)(10 12) = $250 = + = 10,000 + 250 = $10,250

8. = 6,000, = 0.09, = 5 12 = = 6,000(0.09)(5 12) = $225 = + = 6,000 + 225 = $6,225

9. = 12,000, = 0.0005, = 10 = = 12,000(0.0005)(10) = $60 = + = 12,000 + 60 = $12,060

10. = 8,000, = 0.0003, = 5 = = 8,000(0.0003)(5) = $12 = + = 8,000 + 12 = $8,012

11. = (1 + ) = 10,000 (1 + 0.02 × 5) = $9,090.91

12. = (1 + ) = 20,000 (1 + 0.05 × 2) = $18,181.82

13. = (1 + ) = 1,000 (1 + 0.07 × 0.5) = $966.18

14. = (1 + ) = 5,000 (1 + 0.01 × 3) = $4,854.37

15. = (1 + ) = 15,000 (1 + 0.0003 × 15) = $14,932.80

16. = (1 + ) = 30,000 (1 + 0.06 × 20 12) = $27,272.73

17. = (1 + ) = 5,000(1 + 0.08 × 0.5) = $5,200

18. = (1 + ) = 10,000(1 + 0.01 × 15) = $11,500

19. = (1 + ) = 1,000 (1 + 0.0002 × 12) = $997.61

20. = (1 + ) = 30,360 (1 + 0.095 × 4) = $22,000


Solutions Section 3.1 21. We are given the interest and asked to compute . = 250, = 1,000, = 5 = 250 = 1,000 × × 5 = 5,000 250 = = 0.05, or 5% 5,000

22. We are given the interest and asked to compute . = 2,800, = 10,000, = 4 = 2,800 = 10,000 × × 4 = 40,000 2,800 = = 0.07, or 7% 40,000

23. Interest every six months: = = (10,000)(0.08750)(0.5) = $437.50 Total interest over the 10-year life of the bond: = = (10,000)(0.08750)(10) = $8,750

24. Interest every six months: = = (5,000)(0.03250)(0.5) = $81.25 Total interest over the 7-year life of the bond: = = (5,000)(0.03250)(7) = $1,137.50

25. =

26. =

4,083 = = $3,000 1 + 1 + (.019)(10) 2,875 = = $2,500 1 + 1 + (.0375)(4)

27. Goldman Sachs: = = (5,000)(0.0375)(4) = $750 Wells Fargo: = = (5,000)(0.032)(5) = $800 Wells Fargo would pay the most total interest, $800. 28. Ford: = = (2,000)(0.03250)(7) = $455 Verizon: = = (2,000)(0.04125)(6) = $495 Verizon would pay the most total interest, $495.

29. We are given present and future values, and want to compute . = 4,640, = 4,000, = 0.08 = (1 + ) 4,640 = 4,000(1 + 0.08 ) 4,640 1 + 0.08 = = 1.160 4,000 0.08 = 0.160 0.160 = = 2 years 0.08

30. We are given and , and want to compute . = 1,000, = 640, = 0.08 = 640 = 1,000 × 0.08 = 80 640 = = 8 years 80

31. = 50 = 1,000 × 4 = 4,000 ; = 50 4,000 = 0.0125, which is 1.25% weekly interest. Annual rate = 52 × 0.0125 = 0.65, or 65%

32. = 60 = 1,500 × 3 = 4,500 ; = 60 4,500 = 0.0133, which is 1.33% weekly interest. Annual rate = 52 × 60 4,500 0.6933, or 69.33%


Solutions Section 3.1 33. You will pay 5,000 × 0.09 × 2 = $900 in interest on the loan. Adding the $100 fee, you pay the bank a total of $1,000 over the two years. To find the effective rate, we solve 1,000 = 5,000 × 2 = 10,000 ; = 1,000 10,000 = 0.1, so the rate is 10%.

34. You will pay 7,000 × 0.08 × 3 = $1,680 in interest on the loan. Adding the $100 fee, you pay the bank a total of $1,780 over the two years. To find the effective rate, we solve 1,780 = 7,000 × 3 = 21,000 ; = 1,780 21,000 0.08476, so the rate is 8.476%. 35. 5 = 69 12; = 12 × 5 69 36. 10 = 99 6; = 6 × 10 99

0.86957 or 86.957% 0.60606 or 60.606%

37. Use = (1 + ) with = 13,800, = 61,400, = 12 (months) 61,400 = 13,800(1 + 12 ) = 13,800 + 165,600 = (61,400 13,800) 165,600 0.2874, or 28.74%

38. Use = (1 + ) with = 13,800, = 32,600, = 6 (months; selling in April 2021) 32,600 = 13,800(1 + 6 ) = 13,800 + 82,800 = (32,600 13,800) 82,800 0.2271, or 22.71% 39. Selling in any month subsequent to April 1 would have given you a profit. We could calculate the monthly returns for each of those six possible months. However, notice from Quick Example 6 that the interest per month for each of these six alternatives would be the slope of the line joining the April 1 point and the corresponding point on the graph. Of these six lines from the April 1 point, the line to the May 1 point is the steepest, so gives the largest monthly interest, and hence the largest monthly interest rate. Monthly interest (April 1 to May 1) = Slope = Monthly interest rate =

52,100

Monthly interest 19,500 = 32,600

1

32,600

0.5982,

= 19,500.

or 59.82% 40. Selling on July 1, August 1, or September 1 would have resulted in a loss. We could calculate the monthly (negative) returns for each of those three possible months. However, notice from Quick Example 6 that the loss per month for each of these six alternatives would be the slope of the line joining the May 1 point and the corresponding point on the graph. Of the three lines from the May 1 point, the line to the July 1 point is the steepest negative, so gives the largest monthly loss, and hence the largest monthly percentage loss. Monthly loss (May 1 to July 1) = Slope = Monthly (negative) interest rate =

41,500

2

52,100

Monthly loss 5,300 = 52,100

=

5,300.

0.1017,

or a 10.17% loss. 41. Yes. Simple interest increase is linear. The slope of the line through the points for January 1 and February 1 and that of the line through the points for February 1 and March 1 are equal, (13,200 in each case) and therefore the three points are on the same straight line. 42. No. Simple interest increase is linear. The graph is visibly not linear in that time period; the slopes of the


Solutions Section 3.1 lines through the successive pairs of marked points are quite different.

43. 1950 population: = 500,000; 2020 population: = 3,300,000 = = 3,300,000 500,000 = 2,800,000 = 2,800,000 = 500,000 (70) = 35,000,000 2,800,000 = = 0.08 or 8% 35,000,000 44. 1950 population: = 500,000; 1990 population: = 2,500,000 = = 2,500,000 500,000 = 2,000,000 = 2,000,000 = 500,000 (40) = 20,000,000 2,000,000 = 0.10 or 10% 20,000,000

45. We are given: = 1950 population = 500,000; = 0.08 (from Exercise 43); = 80 (years to 2030) = (1 + ) = 500,000(1 + 0.08 × 80) = 3,700,000

46. We are given: = 1950 population = 500,000; = 0.10 (from Exercise 44); = 80 (years to 2030) = (1 + ) = 500,000(1 + 0.10 × 80) = 4,500,000

47. = 1950 population = 500,000; = 0.08 (from Exercise 43). After years, the population will be = (1 + ) = 500,000(1 + 0.08 ) = 500,000 + 40,000 or 500 + 40 thousand ( = time in years since 1950). Graph:

48. = 1950 population = 500,000; = 0.10 (from Exercise 44). After years, the population will be = (1 + ) = 500,000(1 + 0.10 ) = 500,000 + 50,000 or 500 + 50 thousand ( = time in years since 1950).


Solutions Section 3.1 Graph:

49. Actual discount = 0.0025 2 = 0.00125. Selling price = 5,000 = $4,993.75, = $5,000, = 0.5 = (1 + ) 5,000 = 4,993.75(1 + 0.5 ) 1 + 0.5 = 5,000 4,993.75 0.5 = 5,000 4,993.75 1 = 2(5,000 4,993.75 1) 0.002503, or 0.2503%

50. Actual discount = 0.0006 4 = 0.00015. Selling price = 15,000 = $14,997.75, = $15,000, = 0.25 = (1 + ) 15,000 = 14,997.75(1 + 0.25 ) 1 + 0.25 = 15,000 14,997.75 0.25 = 15,000 14,997.75 1 = 4(15,000 14,997.75 1) 0.00060009, or 0.060009%

(0.00125)(5,000) = $4,993.75.

(0.00015)(15,000) = $14,997.75.

51. To say that the discount rate is 3.705% means that its selling price ( ) is 3.705% lower than its maturity value ( ). To simplify the calculation, let us use a T-bill with a maturity value of $10,000: = 10,000 10,000(0.03705 2) = 9814.75 10,000 = 9,814.75(1 + 0.5 ) = 9,814.75 + 4,907.375 = (10,000 9,814.75) 4,907.375 = 0.03775, or 3.775%

52. To say that the discount rate is 3.470% means that its selling price ( ) is 3.470% lower than its maturity value ( ). To simplify the calculation, let us use a T-bill with a maturity value of $10,000: = 10,000 10,000(0.03470 4) = 9,913.25 10,000 = 9,913.25(1 + 0.25 ) = 9,913.25 + 2,478.3125 = (10,000 9,913.25) 2,478.3125 = 0.03500, or 3.500%


Solutions Section 3.1 53. Graph (A) is the only possible choice, because the equation = (1 + ) = + gives the future value as a linear function of time. 54. = (1 + ) = + ( ) , which is a linear equation with slope . Thus, the slope is Slope = Interest rate × Present value. 55. = (1 + ) = + . Since = 1,000 + 0.5 , we have = 1,000 = 1,000 = 0.5 so 0.5 = = 0.0005, or 0.05% 1,000 56. = (1 + ) = + . Since = 400 + 5 , we have = 400 = 400 = 5 so 5 = = 0.0125, or 1.25% 400

57. Wrong. In simple interest growth, the change each year is a fixed percentage of the starting value, and not the preceding year's value. (Also see the next exercise.) 58. The economy last year was larger than the year before; 1% of last year's economy is larger than 1% of the year before's.

59. Simple interest is always calculated on a constant amount, . If interest is paid into your account, then the amount on which interest is calculated does not remain constant. 60. To say that the discount rate is after one year, or = (1

)

One has, for a single year

= (1 + ) = (1 ) (1 + ) 1 = (1 )(1 + ) 1 1+ = 1 giving 1 = 1 1

means that its selling price ( ) is (1

) times its maturity value ( )


Solutions Section 3.2 Section 3.2

1. We use = (1 + ) . = $10,000, = 0.002, = 1, = 15

2. We use = (1 + ) . = $10,000, = 0.0005, = 1, = 6

Technology: 10000*(1+0.002)^15

Technology: 10000*(1+0.0005)^6

3. We use = (1 + ) . = $10,000, = 0.002, = 1, = 10 × 12 = 120

4. We use = (1 + ) . = $10,000, = 0.0045, = 1, = 20 × 12 = 240

Technology: 10000*(1+0.002)^120

Technology: 10000*(1+0.0045)^240

5. We use = (1 + ) . = $10,000, = 0.03, = 1, = 10

6. We use = (1 + ) . = $10,000, = 0.04, = 1, = 8

Technology: 10000*(1+0.03)^10

Technology: 10000*(1+0.04)^8

7. We use = (1 + ) . = $10,000, = 0.025, = 4, = 5

8. We use = (1 + ) . = $10,000, = 0.015, = 52, = 5

= (1 + ) = 10,000(1 + 0.002) 15 = 10,000(1.002) 15 $10,304.24

= (1 + ) = 10,000(1 + 0.002) 120 = 10,000(1.002) 120 $12,709.44

= (1 + ) = 10,000(1 + 0.03) 10 = 10,000(1.03) 10 $13,439.16

= (1 + ) = 10,000(1 + 0.025 4) 4×5 = 10,000(1.00625) 20 $11,327.08

Technology: 10000*(1+0.025/4)^(4*5)

= (1 + ) = 10,000(1 + 0.0005) 6 = 10,000(1.0005) 6 $10,030.04

= (1 + ) = 10,000(1 + 0.0045) 240 = 10,000(1.0045) 240 $29,375.54

= (1 + ) = 10,000(1 + 0.04) 8 = 10,000(1.04) 8 $13,685.69

= (1 + ) 0.015 52×5 = 10,000 1 + 52

$10,778.72

Technology: 10000*(1+0.015/52)^(52*5)


Solutions Section 3.2

9. We use = (1 + ) . = $10,000, = 0.065, = 365, = 10

10. We use = (1 + ) . = $10,000, = 0.112, = 12, = 12

Technology: 10000* (1+0.065/365)^(365*10)

Technology: 10000* (1+0.112/12)^(12*12)

11. We use = (1 + ) = $1,000, = 10, = 0.05

12. We use = (1 + ) = $1,000, = 5, = 0.06

Technology: 1000*(1+0.05)^(-10)

Technology: 1000*(1+0.06)^(-5)

13. We use = (1 + ) = $1,000, = 5, = 0.042, = 52

14. We use = (1 + ) = $1,000, = 10, = 0.053, = 4

Technology: 1000*(1+0.042/52)^(-52*5)

Technology: 1000*(1+0.053/4)^(-4*10)

15. We use = (1 + ) = $1,000, = 4, = 0.05

16. We use = (1 + ) = $1,000, = 5, = 0.04

Technology: 1000*(1-0.05)^(-4)

Technology: 1000*(1-0.04)^(-5)

= (1 + ) 0.065 365×10 = 10,000 1 + 365 0.065 3,650 $19,154.30 = 10,000 1 + 365

= (1 + ) = 1,000(1 + 0.05) 10 = 1,000(1.05) 10 $613.91

= (1 + ) 0.042 52×5 = 1,000 1 + 52 0.042 260 $810.65 = 1,000 1 + 52

= (1 + ) = 1,000(1 0.05) 4 = 1,000(0.95 4) $1,227.74

= (1 + ) 0.112 12×12 = 10,000 1 + 12 0.112 144 $38,105.24 = 10,000 1 + 12

= (1 + ) = 1,000(1 + 0.06) 5 = 1,000(1.06) 5 $747.26

= (1 + ) 0.053 4×10 = 1,000 1 + 4 0.053 40 $590.66 = 1,000 1 + 4

= (1 + ) = 1,000(1 0.04) 5 = 1,000(0.96 5) $1,226.43


Solutions Section 3.2

17. nom = 0.05, = 4

18. nom = 0.05, = 12

e = (1 + nom ) 1 0.05 4 1 = 1 + 4 0.0509, or 5.09%

e = (1 + nom ) 1 0.05 12 1 = 1 + 12 0.0512, or 5.12%

Technology: (1+0.05/4)^4-1

Technology: (1+0.05/12)^12-1

19. nom = 0.10, = 12

20. nom = 0.10, = 365

Technology: (1+0.10/12)^12-1

Technology: (1+0.10/365)^365-1

21. nom = 0.10, = 365 × 24 = 8,760

22. nom = 0.10, = 365 × 24 × 60 = 525,600

e = (1 + nom ) 1 0.10 12 1 = 1 + 12 0.1047, or 10.47%

e = (1 + nom ) 1 0.10 365 1 = 1 + 365 0.1052, or 10.52%

e = (1 + nom ) 1 0.10 8,760 1 = 1 + 8,760 0.1052, or 10.52%

e = (1 + nom ) 1 0.10 525,600 = 1 + 525,600 0.1052, or 10.52%

1

Technology: (1+0.10/8760)^8760-1

Technology: (1+0.10/525600)^525600-1

23. = $1,000, = 0.06, = 4, = 4

24. = $10,000, = 0.02, = 4, = 5

The deposit will have grown by $1,268.99 $1,000 = $268.99. Technology: 1000*(1+0.06/4)^(16)

The deposit will have grown by $11,048.96 $10,000 = $1,048.96 Technology: 10000*(1+0.02/4)^(20)

25. = $3,000, =

26. = $5,000, =

= (1 + ) = 1,000(1 + 0.06 4) 4×4 = 1,000(1.015) 16 $1,268.99

0.376, = 1, = 3

= (1 + ) = 3,000(1 0.376) 3

$728.91

Technology: 3000*(1-0.376)^3

= (1 + ) = 1,000(1 + 0.02 4) 4×5 = 1,000(1.005) 20 $11,048.96

0.42, = 1, = 4

= (1 + ) = 5,000(1 0.42) 4

$565.82

Technology: 5000*(1-0.42)^4


Solutions Section 3.2

27. = $5,000, = 0.055, = 1, = 10

28. = $10,000, = 0.0625, = 1, = 15

Technology: 5000*(1+0.055)^-10

Technology: 10000*(1+0.0625)^-15

29. Gold: = $5,000, = 0.10, = 1, = 10

30. Munis: = $5,000, = 0.06, = 1, = 10

CDs: = $5,000, = 0.05, = 2, = 10

CDs: = $5,000, = 0.03, = 6, = 10

Combined value after 10 years = $12,968.71 + $8,193.08 = $21,161.79 Technology: 5000*(1+0.10)^10+5000* (1+0.05/2)^(2*10)

Combined value after 10 years = $8,954.24 + $6,774.24 = $15,698.49 Technology: 5000*(1+0.06)^10+5000* (1+0.03/6)^(6*10)

31. = $200,000, =

32. = $40,000, =

= (1 + ) = 5,000(1 + 0.055) 10 = 5,0001.055 10 $2,927.15

= (1 + ) = 5,000(1 + 0.10) 10 = 5,000(1.10) 10 $12,968.71

= (1 + ) = 5,000(1 + 0.05 2) 2×10 = 5,000(1.025) 20 $8,193.08

0.02, = 1, = 10

= (1 + ) = 200,000(1 0.02) 10 = 200,000(0.98) 10 $163,414.56

= (1 + ) = 10,000(1 + 0.0625) 15 = 10,0001.0625 15 $4,027.78

= (1 + ) = 5,000(1 + 0.06) 10 = 5,000(1.06) 10 $8,954.24

= (1 + ) = 5,000(1 + 0.03 6) 6×10 = 5,000(1.005) 60 $6,774.24

0.051, = 1, = 12

= (1 + ) = 40,000(1 0.051) 12 = 40,000(0.949) 12 $21,342.95

Technology: 200000*(1-0.02)^10

Technology: 40000*(1-0.051)^12

33. = $1,000,000, = 0.06, = 1, = 30

34. = $1,000,000, = 0.07, = 1, = 40

Technology: 1000000*(1+0.06)^-30

Technology: 1000000*(1+0.07)^-40

= (1 + ) = 1,000,000(1 + 0.06) 30 = 1,000,000(1.06) 30 $174,110

= (1 + ) = 1,000,000(1 + 0.07) 40 = 1,000,000(1.07) 40 $66,780


35. = $297.91, = = 6 × 3 = 18,

0.05, = 1,

Solutions Section 3.2

= (1 + ) = 297.91(1 0.05) 18 = 297.91(0.95) 18 $750.00

36. = 100, = 0.06, = 1, = 2 × 5 = 10 = (1 + ) = 100(1 + 0.06) 10 = 100(1.06) 10 179 kits per month

Technology: 100*(1+0.06)^10

Technology: 297.91*(1-0.05)^-18 37. = (1 + ) = 7,445.37(1 + 0.02544) 19 38. = (1 + ) = 8,368.30(1 + 0.01836) 4

$12,000

$9,000

39. = (1 + ) = 10,000(1 + 0.03045) 10 = 10,000(1.03045) 10 40. = (1 + ) = 5,000(1 + 0.03258) 7 = 5,000(1.03258) 7

$7,408.51

$3,994.88

41. Total interest on $1 during the 6-year life of the bonds is = = (1 + 0.01551) 6 1 $0.09674 Thus, the monthly simple interest rate would be 0.09674 (6 × 12)

0.0013, or 0.13%.

42. Total interest on $1 during the 5-year life of the bonds is = = (1 + 0.02486) 5 1 $0.13064 Thus, the biannual simple interest rate would be 0.13064 (5 × 2)

0.0131, or 1.31%.

43. = $100,000, = 0.04, = 1, = 15

44. = $80,000, = 0.05, = 1, = 10

Technology: 100000*(1+0.04)^-15

Technology: 80000*(1+0.05)^-10

45. = $30,000, = 0.02, = 1, = 5

46. = $30,000, = 0.01, = 1, = 5 × 2 = 10

Technology: 30000*(1+0.02)^-5

Technology: 30000*(1+0.01)^-10

= (1 + ) = 100,000(1 + 0.04) 15 = 100,000(1.04) 15 $55,526.45 per year

= (1 + ) = 30,000(1 + 0.02) 5 = 30,000(1.02) 5 $27,171.92

= (1 + ) = 80,000(1 + 0.05 1) 10 = 80,000(1.05) 10 $49,113.06 per year

= (1 + ) = 30,000(1 + 0.01) 10 = 30,000(1.01) 10 $27,158.61


Solutions Section 3.2

47. = $200,000, = 0.06, = 1, = 10 = (1 + ) = 200,000(1 + 0.06) 10 = 200,000(1.06) 10 $111,678.96

Technology: 200000*(1+0.06)^-10

48. = $200,000, = 0.005, = 1, = 10 × 12 = 120 = (1 + ) = 200,000(1 + 0.005) 120 = 200,000(1.005) 120 $109,926.55

Technology: 200000*(1+0.005)^-120

49. Step 1: Calculate the future value of the investment: = $1,000, = 0.05, = 1, = 2 = (1 + ) = 1,000(1 + 0.05) 2 = 1,000(1.05) 2 $1,102.50

Technology: 1000*(1+0.05)^2 Step 2: Discount this value using inflation: = $1,102.50, = 0.03, = 1, = 2 = (1 + ) = 1,102.50(1 + 0.03) 2 = 1,102.50(1.03) 2 $1,039.21

Technology: 1102.50*(1+0.03)^-2

50. Step 1: Calculate the future value of the investment: = $10,000, = 0.08, = 12, = 2 = (1 + ) = 10,000(1 + 0.08 12) 12×2

$11,728.88

Technology: 10000*(1+0.08/12)^24 Step 2: Discount this value using inflation: = $11,728.88, = 0.01, = 1, = 24 = (1 + ) = 11,728.88(1 + 0.01) 24 = 11,728.88(1.01) 24 $9,237.27

Technology: 11728.88*(1+0.01)^-24


Solutions Section 3.2 51. Compare the effective yields of the two investments: First Investment: nom = 0.12, = 1 e = nom = 0.12, or 12%

52. Compare the effective yields of the three investments: First Investment: nom = 0.15, = 1 e = nom = 0.15, or 15%

Second Investment: nom = 0.119, = 12

Second Investment: nom = 0.145, = 4

The better investment is the second.

Technology: (1+0.145/4)^4-1

e = (1 + nom ) 1 0.119 12 1 = 1 + 12 0.1257, or 12.57%

e = (1 + nom ) 1 0.145 4 1 = 1 + 4 0.1531, or 15.31%

Third Investment: nom = 0.14, = 12

e = (1 + nom ) 1 0.14 12 1 = 1 + 12 0.1493, or 14.93%

Technology: (1+0.14/12)^12-1 The best investment is the second: 14.5% compounded quarterly. 53. = $24, = 0.063, = 1, = 2021 1626 = 395

54. = $24, = 0.062, = 1, = 2021 1626 = 395

This is more than the 2021 estimated market value of $508,176 million. Thus, the Lenape could have bought the island back in 2021. Technology: 24*(1+0.063)^395

This is less than the 2021 estimated market value of $508,176 million. Thus, the Lenape could not have bought the island back in 2021. Technology: 24*(1+0.062)^395

55. = 100 reals, = 0.0968, = 1, = 5

56. = 1,000 pesos, = 0.5140, = 1, = 5

Technology: 100*(1+0.0968)^5

Technology: 1000*(1+0.5140)^5

= (1 + ) = 24(1 + 0.063) 395 = 24(1.063) 395 $725,856 million

= (1 + ) = 100(1 + 0.0968) 5 = 100(1.0968) 5 159 reals

= (1 + ) = 24(1 + 0.062) 395 = 24(1.062) 395 $500,490 million

= (1 + ) = 1,000(1 + 0.5140) 5 = 1,000(1.5140) 5 7,955 pesos


Solutions Section 3.2

57. = 1,000 bolivianos, = 0.0020, = 1, = 10 = (1 + ) = 1,000(1 + 0.0020) 10 = 1,000(1.0020) 10 980 bolivianos

58. = 20,000 pesos, = 0.0559, = 1, = 10 = (1 + ) = 20,000(1 + 0.0559) 10 = 20,000(1.0559) 10 11,609 pesos

Technology: 20000*(1+0.0559)^-10

Technology: 1000*(1+0.0020)^-10 59. Step 1: Calculate the future value of the investment: = 1,000 pesos, = 0.05, = 2, = 10

60. Step 1: Calculate the future value of the investment: = 1,000 pesos, = 0.05, = 2, = 10

Technology: 1000*(1+0.05/2)^20

Technology: 1000*(1+0.05/2)^20

Step 2: Discount this amount using the inflation rate: = 1,638.62, = 0.0480, = 1, = 10

Step 2: Discount this amount using the inflation rate: = 1,638.62, = 0.0759, = 1, = 10

Technology: 1638.62*(1+0.0480)^-10

Technology: 1638.62*(1+0.0759)^-10

= (1 + ) 0.05 2×10 = 1,000 1 + 2 = 1,000(1.025) 20 1,638.62 pesos

= (1 + ) = 1,638.62(1 + 0.0480) 10 = 1,638.62(1.0480) 10 1,025 pesos

= (1 + ) 0.05 2×10 = 1,000 1 + 2 = 1,000(1.025) 20 1,638.62 pesos

= (1 + ) = 1,638.62(1 + 0.0759) 10 = 1,638.62(1.0759) 10 788 pesos


Solutions Section 3.2 61. We compare the future values of 1 unit of the currency for a 1-year period: Mexico: = 1 peso, = 0.06, = 1, = 1

62. We compare the future values of 1 unit of the currency for a 1-year period: Brazil: = 1 real, = 0.10, = 1, = 1

Now discount this using inflation: = 1.06, = 0.0559, = 1, = 1

Now discount this using inflation: = 1.10, = 0.0968, = 1, = 1

= (1 + ) = 1(1 + 0.06) 1 = 1.06 pesos

= (1 + ) = 1.06(1 + 0.0559) 1

= (1 + ) = 1(1 + 0.10) 1 = 1.10 reals

= (1 + ) = 1.10(1 + 0.0968) 1 1.0029 reals

1.0039 pesos

Nicaragua: = 1 córdoba, = 0.045, = 2, = 1

Uruguay: = 1 peso, = 0.075, = 2, = 1

Now discount this using inflation: = 1.0455, = 0.0412, = 1, = 1

Now discount this using inflation: = 1.0764, = 0.0759, = 1, = 1

The investment in Nicaragua is better.

The investment in Brazil is better.

= (1 + ) 0.045 2 = 1 1 + 2 = 1.0455 córdobas

= (1 + ) = 1.0455(1 + 0.0412) 1 = 1.0041 córdobas

= (1 + ) 0.075 2×1 = 1 1 + 2 1.0764 pesos

= (1 + ) 1.0764(1 + 0.0759) 1 1.0005 pesos

63. = 58,900, = 61,400, = 12, = 4 months = 1 3 year, so = 12 3 = 4 61,400 = 58,900(1 + 12) 4, so solving for gives = 12[(61,400 58,900) 1 4 1] 12.54% 64. = 32,600, = 61,400, = 12, = 6 months = 1 2 year, so = 12 2 = 6 61,400 = 32,600(1 + 12) 6, so solving for gives = 12[(61,400 32,600) 1 6 1] 133.54%

0.1254 or 1.3354 or

65. Calculating the annual returns as in the preceding exercises (or by using one of the TVM utilities), we get the following figures: Aug

Sep

Oct

159.04% 27.16% 167.38% The largest annual return would have been 167.38% if you had sold on October 1, 2021.


Solutions Section 3.2 66. Calculating the annual returns as in the preceding exercises (or by using one of the TVM utilities), we get the following figures: Jan

Feb

Mar

144.83% 306.40% 319.69% The largest annual return would have been 319.69% if you had sold on March 1, 2021. 67. No. Compound interest increase is exponential, and hence a curve that is concave upwards. However, the three points coresponing to January 1, February 1 and March 1 2021 are on a straight line (see Exercise Exercise ?? in Section __._) and therefore not a curve. 68. No. Compound interest increase is exponential, and hence a curve that is concave upwards.However, the three points coresponing to April 1, May 1, and June 1 2021 define a curve that is concave down and therefore not exponentially increasing. 69. My investment: = $5,000, = 0.054, = 2 0.054 2 = (1 + ) = 5,000 1 + = 5,000(1.027) 2 2 Friend's investment: = $6,000, = 0.048, = 2 0.048 2 = (1 + ) = 6,000 1 + = 6,000(1.024) 2 2 Solution via logarithms: 5,000(1.027) 2 = 6,000(1.024) 2 (1.027) 2 = 1.2(1.024) 2 2 log 1.027 = log[1.2(1.024) 2 ] = log 1.2 + 2 log 1.024 2 (log 1.027 log 1.024) = log 1.2 log 1.2 = 31 2(log 1.027 log 1.024) Solution via graphing: Technology Formulas: Y1 = 5000*1.027^(2*x) Graph and zoomed-in view:

Y2 = 6000*1.024^(2*x)

The graphs cross around 31 years. The value of the investment then is 5,000(1.027) 2×31 $26,100 (rounded to 3 significant digits) 70. = $3,000, = 0.05, = 365

= (1 + ) = 3,000 1 +

Logarithms:

0.05 365 365


0.05 365 3,000 1 + = 10,000 365 0.05 365 10 = 1 + 365 3 0.05 10 365 log 1 + = log 365 3 log(10 3) = 24 365 log(1 + 0.05 365)

Solutions Section 3.2

Technology: Y1 = 3000*(1+0.05/365)^(365*x) Graph and zoomed-in view:

Y2 = 10000

24 years

71. = 40,000, = 1.0 (a 100% increase per period), = 1, in half-years = (1 + ) = 40,000(1 + 1.0) = 40,000(2) Logarithms: 40,000(2) = 1,000,000 2 = 25 log 2 = log 25 log 25 = 4.65 half-years log 2 Technology: Y1 = 40000*2^x Graph and zoomed-in view:

Y2 = 1000000

4.65. Since measures 6-month periods, this corresponds to 4.65 2

72. = $4,354, = 0.05, = 1, in half-years = (1 + ) = 4,354(1 0.05) = 4,354(0.95) Logarithms: 4,354(0.95) = 50

2.3 years.


0.95 = 50 4,354 log 0.95 = log(50 4,354) log(50 4,354) 87.1 = log 0.95 Technology: Y1 = 4354*0.95^x Graph and zoomed-in view:

Solutions Section 3.2

Y2 = 50

87.1. Since measures 6-month periods, this corresponds to 87.1 2

44 years.

73. a. The amount you pay for the bond is its present value. = $100,000, = 0.15, = 1, = 30 = (1 + ) = 100,000(1 + 0.15) 30 $1,510.31 b. Because the future value remains fixed at $100,000, the value of the bond at any time is its present value at that time, given the prevailing interest rate. Since it will pay $100,000 in 13 years' time, we have: = $100,000, = 0.0475, = 1, = 13 = (1 + ) = 100,000(1 + 0.0.475) 13 $54,701.29 c. By part (a) the bond cost you $1,510.31 and, by part (b), was worth $54,701.29 after 17 years. = $1,510.31, = $54,701.29, = 1, = 17 = (1 + ) 54,701.29 = 1,510.31(1 + ) 17 54,701.29 = (1 + ) 17 1,510.31 54,701.29 1 17 1+ = � � 1,510.31 54,701.29 1 17 = � 1 0.2351, or 23.51% � 1,510.31 Technology: (54701.29/1510.31)^(1/17)-1 74. a. The amount you pay for the bond is its present value. = $100,000, = 0.05, = 1, = 30 = (1 + ) = 100,000(1 + 0.05) 30 $23,137.74 b. The value of the bond at any time is its present value at that time, given the prevailing interest rate. Since it will pay $100,000 in 15 years' time, we have: = $100,000, = 0.12, = 1, = 15 = (1 + ) = 100,000(1 + 0.12) 15 $18,269.63 c. By part (a), the bond cost you $23,137.74 and, by part (b), was worth $18,269.63 after 15 years. = $23,137.74, = $18,269.63, = 1, = 15 = (1 + )


Solutions Section 3.2 18,269.63 = 23,137.74(1 + ) 15 18,269.63 = (1 + ) 15 23,137.74 18,269.63 1 15 1+ = � � 23,137.74 18,269.63 1 15 = � 1 0.0156, or 1.56 � 23,137.74 You will have lost 1.56% per year. Technology: (18269.63/23137.74)^(1/15)-1

75. The function = (1 + ) is not a linear function of , but an exponential function. Thus, its graph is not a straight line. 76. Wrong. The second investment earns more, because the second investment pays interest on a larger amount after 6 months.

77. Wrong. Its growth can be modeled by 0.01(1 + 0.10) = 0.01(1.10) . This is an exponential function of , not a linear one. 78. After one interest period: The calculations are the same for a single interest period. 79. A compound interest investment behaves as though it were being compounded once a year at the effective rate. Thus, if two equal investments have the same effective rates, they grow at the same rate and the graphs of future value will be the same. 80. The effective rate equals the nominal rate when the interest is compounded once a year because, by definition, the effective rate is the annually compounded rate that would result in the same future value. 81. The effective rate exceeds the nominal rate when the interest is compounded more than once a year because then interest is being paid on interest accumulated during each year, resulting in a larger effective rate. Conversely, if the interest is compounded less often than once a year, the effective rate is less than the nominal rate. 82. No, it will not return to the same value it had originally. The simplest way to see this is to note the following lack of symmetry: The interest earned in year 5 is slightly less than the depreciation in year 6, since the depreciation is calculated on a larger value. A direct (two-step) calculation of the future value after 10 years gives (1.1 5)(0.9 5) 0.95099 , slightly less than . 83. Compare their future values in constant dollars. That is, compute their future values, and then discount each for inflation. The investment with the larger future value is the better investment. 84. First compute the value in 2011 dollars, using the 2011 inflation rate, then use the 2010 inflation rate to convert the answer to 2010 dollars, and so on, down to 2000 dollars. 85. = 100, = 0.10, = 1, 10, 100, … Y1 = 100*(1.10)^x Y2 = 100*(1+ 0.10/10)^(10*x) Y3 = 100*(1+ 0.10/100)^(100*x) …


Solutions Section 3.2

The graphs are approaching a particular curve (shown darker) as gets larger, approximately the curve given by the largest value of . 86. Since the graph gets arbitrarily close to the -axis as gets larger, the future value gets arbitrarily close to zero as time goes on, so it will eventually dip below any value that is set, like $1.


Solutions Section 3.3 Section 3.3

1. = $100, = interest paid each period = 0.05 12, = total number of periods = 12 × 10 = 120 (1 + 0.05 12) 120 1 (1 + ) 1 = $15,528.23 = 100 0.05 12 Technology: 100*((1+0.05/12)^120-1)/(0.05/12) 2. = $150, = 0.03 12, = 12 × 20 = 240 (1 + 0.03 12) 240 1 (1 + ) 1 = $49,245.30 = 150 0.03 12 Technology: 150*((1+0.03/12)^240-1)/(0.03/12)

3. = $1,000, = 0.07 4, = 4 × 20 = 80 (1 + 0.07 4) 80 1 (1 + ) 1 = $171,793.82 = 1,000 0.07 4 Technology: 1000*((1+0.07/4)^80-1)/(0.07/4) 4. = $2,000, = 0.07 4, = 4 × 10 = 40 (1 + 0.07 4) 40 1 (1 + ) 1 = $114,468.27 = 2,000 0.07 4 Technology: 2000*((1+0.07/4)^40-1)/(0.07/4) 5. = $5,000, = $100, = 0.05 12, = 12 × 10 = 120

We need to calculate the sum of = (1 + ) and =

(1 + )

1

(1 + )

1

.

(1 + 0.05 12) 120 1 $23,763.28 0.05 12 Technology: 5000*(1+0.05/12)^120+100*((1+0.05/12)^120-1)/(0.05/12) = 5,000(1 + 0.05 12) 120 + 100

6. = $10,000, = $150, = 0.03 12, = 12 × 20 = 240

We need to calculate the sum of = (1 + ) and =

.

(1 + 0.03 12) 240 1 $67,452.85 0.03 12 Technology: 10000*(1+0.03/12)^240+150*((1+0.03/12)^240-1)/(0.03/12) = 10,000(1 + 0.03 12) 240 + 150

7. = $10,000, = 0.05 12, = 12 × 5 = 60 0.05 12 = = 10,000 (1 + ) 1 (1 + 0.05 12) 60 1 Technology: 10000*0.05/12/((1+0.05/12)^60-1)

$147.05

9. = $75,000, = 0.06 4, = 4 × 20 = 80 0.06 4 = = 75,000 (1 + ) 1 (1 + 0.06 4) 80 1 Technology: 75000*0.06/4/((1+0.06/4)^80-1)

$491.12

8. = $20,000, = 0.03 12, = 12 × 10 = 120 0.03 12 = $143.12 = 20,000 (1 + ) 1 (1 + 0.03 12) 120 1 Technology: 20000*0.03/12/((1+0.03/12)^120-1)


Solutions Section 3.3 10. = $100,000, = 0.07 4, = 4 × 20 = 80 0.07 4 $582.09 = = 100,000 (1 + ) 1 (1 + 0.07 4) 80 1 Technology: 100000*0.07/4/((1+0.07/4)^80-1)

11. We first account for the future value of the $10,000 already in the account: = $10,000, = 0.05 12, = 12 × 5 = 60 = (1 + ) = 10,000(1 + 0.05 12) 60 We subtract this from $20,000 to get the future value of the payments, so: = (1 + ) 1 where = $20,000 10,000(1 + 0.05 12) 60 =

[20,000

10,000(1 + 0.05 12) 60](0.05 12)

$105.38 (1 + 0.05 12) 60 1 Technology: (20000-10000*(1+0.05/12)^60)*0.05/12/((1+0.05/12)^60-1) 12. We first account for the future value of the $10,000 already in the account: = $10,000, = 0.03 12, = 12 × 10 = 120 = (1 + ) = 10,000(1 + 0.03 12) 120 We subtract this from $30,000 to get the future value of the payments, so: = (1 + ) 1 where = $30,000 10,000(1 + 0.03 12) 120 =

[30,000

10,000(1 + 0.03 12) 120](0.03 12)

$118.12 (1 + 0.03 12) 120 1 Technology: (30000-10000*(1+0.03/12)^120)*0.03/12/((1+0.03/12)^120-1) 13. = $500, = 0.03 12, = 12 × 20 = 240 1 (1 + 0.03 12) 240 1 (1 + ) = $90,155.46 = 500 0.03 12 Technology: 500*(1-(1+0.03/12)^(-240))/(0.03/12)

14. = $1,000, = 0.05 12, = 12 × 15 = 180 1 (1 + 0.05 12) 180 1 (1 + ) = $126,455.24 = 1000 0.05 12 Technology: 1000*(1-(1+0.05/12)^(-180))/(0.05/12) 15. = $1,500, = 0.06 4, = 4 × 20 = 80 1 (1 + 0.06 4) 80 1 (1 + ) = $69,610.99 = 1,500 0.06 4 Technology: 1500*(1-(1+0.06/4)^(80))/(0.06/4) 16. = $2,000, = 0.04 4 = 0.01, = 4 × 20 = 80 1 (1 + ) 1 (1 + 0.01) 80 = = 2,000 0.01 Technology: 2000*(1-(1+0.01)^(-80))/0.01

$109,776.41


Solutions Section 3.3 17. = $10,000, = $500, = 0.03 12, = 12 × 20 = 240 We need to fund both the future value and the payments, so the present value is the sum (1 + ) 1 (1 + 0.03 12) 240 = 10,000(1 + 0.03 12) 240 + 500 0.03 12

= (1 + ) +

1

$95,647.68

Technology: 10000*(1+0.03/12)^(-240)+500*(1-(1+0.03/12)^(-240))/(0.03/12)

18. = $10,000, = $1,000, = 0.05 12, = 12 × 15 = 180 We need to fund both the future value and the payments, so the present value is the sum (1 + ) 1 = 10,000(1 + 0.05 12) 180 + 1,000

= (1 + ) +

1

(1 + 0.05 12) 180 0.05 12

$135,917.31

Technology: 20000*(1+0.05/12)^(-180)+1000*(1-(1+0.05/12)^(-180))/(0.05/12) 19. = $100,000, = 0.03 12, = 12 × 20 = 240 0.03 12 = $554.60 = 100,000 1 (1 + ) 1 (1 + 0.03 12) 240 Technology: 100000*(0.03/12)/(1-(1+0.03/12)^(-240))

20. = $150,000, = 0.05 12, = 12 × 15 = 180 0.05 12 = $1,186.19 = 150,000 1 (1 + ) 1 (1 + 0.05 12) 180 Technology: 150000*(0.05/12)/(1-(1+0.05/12)^(-180)) 21. = $75,000, = 0.04 4 = 0.01, = 4 × 20 = 80 0.01 = = 75,000 1 (1 + ) 1 (1 + 0.01) 80 Technology: 75000*0.01/(1-(1+0.01)^(-80))

22. = $200,000, = 0.06 4 = 0.015, = 4 × 15 = 60 0.015 = = 200,000 1 (1 + ) 1 (1 + 0.015) 60 Technology: 200000*0.015/(1-(1+0.015)^(-60))

$1,366.41

$5,078.69

23. = $100,000, = $10,000, = 0.03 12, = 12 × 20 = 240 Part of the present value has to fund the future value of $10,000: (1 + ) = 10,000(1 + 0.03 12) 240 is the amount required; we subtract this from the present value and use = 100,000 10,000(1 + 0.03 12) 240 in the payment formula. (0.03 12)[100,000 10,000(1 + 0.03 12) 240] = $524.14 = 1 (1 + ) 1 (1 + 0.03 12) 240 Technology: (0.03/12)*(100000-10000*(1+0.03/12)^(-240))/(1-(1+0.03/12)^(-240)) 24. = $150,000, = $20,000, = 0.05 12, = 12 × 15 = 180


Solutions Section 3.3 Part of the present value has to fund the future value of $20,000: (1 + ) = 20,000(1 + 0.05 12) 180 is the amount required; we subtract this from the present value and use = 150,000 20,000(1 + 0.05 12) 180 in the payment formula. (0.05 12)[150,000 20,000(1 + 0.05 12) 180] = $1,111.37 = 1 (1 + ) 1 (1 + 0.05 12) 180 Technology: (0.05/12)*(150000-20000*(1+0.05/12)^(-180))/(1-(1+0.05/12)^(-180)) 25. = $10,000, = 0.09 12 = 0.0075, = 4 × 12 = 48 0.0075 = $248.85 = 10,000 1 (1 + ) 1 (1 + 0.0075) 48 Technology: 10000*0.0075/(1-(1+0.0075)^(-48))

26. = $20,000, = 0.08 12, = 12 × 5 = 60 0.08 12 = $405.53 = 20,000 1 (1 + ) 1 (1 + 0.08 12) 60 Technology: 20000*(0.08/12)/(1-(1+0.08/12)^(-60))

27. = $100,000, = 0.05 4, = 4 × 20 = 80 0.05 4 = $1,984.65 = 100,000 1 (1 + ) 1 (1 + 0.05 4) 80 Technology: 100000*(0.05/4)/(1-(1+0.05/4)^(-80))

28. = $1,000,000, = 0.04 4, = 4 × 10 = 40 0.04 4 = $30,455.60 = 1,000,000 1 (1 + ) 1 (1 + 0.04 4) 40 Technology: 1000000*(0.04/4)/(1-(1+0.04/4)^(-40)) 29. = 100,000, = 0.043 12, = 12 × 30 = 360 0.043 12 = = 100,000 = $494.87 1 (1 + ) 1 (1 + 0.043 12) 360 Technology: 100000*(0.043/12)/(1-(1+0.043/12)^(-360))

30. = 250,000, = 0.062 12, = 12 × 15 = 180 0.062 12 = = 250,000 = $2,136.75 1 (1 + ) 1 (1 + 0.062 12) 180 Technology: 250000*(0.062/12)/(1-(1+0.062/12)^(-180))

31. = 1,000,000, = 0.054 12, = 12 × 30 = 360 0.054 12 = = 1,000,000 = $5,615.31 1 (1 + ) 1 (1 + 0.054 12) 360 Technology: 1000000*(0.054/12)/(1-(1+0.054/12)^(-360))

32. = 2,000,000, = 0.045 12, = 12 × 15 = 180 0.045 12 = = 2,000,000 = $15,299.87 1 (1 + ) 1 (1 + 0.045 12) 180 Technology: 2000000*(0.045/12)/(1-(1+0.045/12)^(-180))


Solutions Section 3.3 33. We calculated = $494.87 in Exercise 29. = 0.043 12, = 12 × 30 = 360, = 12 × 10 = 120, so = 240 1 (1 + ) ( ) = 1 (1 + 0.043 12) 240 = 494.87 = $79,573.29 0.043 12 Technology: 494.87*(1-(1+0.043/12)^(-240))/(0.043/12) 34. We calculated = $2,136.75 in Exercise 30. = 0.062 12, = 12 × 15 = 180, = 12 × 10 = 120, so = 60 1 (1 + ) ( ) = 1 (1 + 0.062 12) 60 = 2,136.75 = $109,994.70 0.062 12 Technology: 2136.75*(1-(1+0.062/12)^(-60))/(0.062/12)

35. We calculated = $5,615.31 in Exercise 31. = 0.054 12, = 12 × 30 = 360, = 12 × 5 = 60, so = 300 1 (1 + ) ( ) = 1 (1 + 0.054 12) 300 = 5,615.31 = $923,373.42 0.054 12 Technology: 5615.31*(1-(1+0.054/12)^(-300))/(0.054/12) 36. We calculated = $15,299.87 in Exercise 32. = 0.045 12, = 12 × 15 = 180, = 12 × 8 = 96, so = 84 1 (1 + ) ( ) = 1 (1 + 0.045 12) 84 = 15,299.87 = $1,100,697.30 0.045 12 Technology: 15299.87*(1-(1+0.045/12)^(-84))/(0.045/12) 37. First, we calculate the monthly payments: = 50,000, = 0.085 12, = 12 × 200 = 2,400 0.085 12 = = 50,000 = $354.17 1 (1 + ) 1 (1 + 0.085 12) 2,400 Technology: 50000*(0.085/12)/(1-(1+0.085/12)^(-2400)) Now calculate the outstanding balance based on the above payments: = 12 × 20 = 240, so = 2,160 1 (1 + ) ( ) = 1 (1 + 0.085 12) 2,160 = 354.17 = $50,000.46 0.085 12 Technology: 354.17*(1-(1+0.085/12)^(-2160))/(0.085/12) This is more than the original value of the loan. In a 200-year mortgage, the fraction of the initial payments going toward reducing the principal is so small that the rounding upward of the payment makes it appear that, for many years at the start of the mortgage, more is owed than the original amount borrowed. 38. First, we calculate the monthly payments: = 100,000, = 0.096 12, = 12 × 200 = 2,400


Solutions Section 3.3 0.096 12 = = 100,000 = $800 1 (1 + ) 1 (1 + 0.096 12) 2,400 Technology: 100000*(0.096/12)/(1-(1+0.096/12)^(-2400)) Now calculate the outstanding balance based on the above payments: = 12 × 20 = 240, so = 2,160 1 (1 + ) ( ) = 1 (1 + 0.096 12) 2,160 = 800 = $100,000.00 0.096 12 Technology: 800*(1-(1+0.096/12)^(-2160))/(0.096/12) The outstanding principal has not changed from the original amount. In a 200-year mortgage, the fraction of the initial payments going toward reducing the principal is so small that rounding the payment amount to the nearest cent erases that contribution in this case. 39. The periodic payments are based on a 4.875% annual payment. For payments twice a year, this is = 1,000(0.04875 2) = $24.375 Since the bond yield is 4.880%, = 0.0488 2 = 0.0244; = 2 × 10 = 20. The present value comes from the future value of $1,000 and the payments, which we treat like an annuity: (1 + ) 1 (1 + 0.0244) 20 = 1,000(1 + 0.0244) 20 + 24.375 0.0244 1 1.0244 20 $999.61 = 1,000(1.0244) 20 + 24.375 0.0244

= (1 + ) +

1

Technology: 1000*1.0244^(-20)+24.375*(1-1.0244^(-20))/0.0244 Online Time Value of Money Utility:

40. The periodic payments are based on a 5.375% annual payment. For payments twice a year, this is = 1,000(0.05375 2) = $26.875 Since the bond yield is 5.460%, = 0.0546 2 = 0.0273; = 2 × 30 = 60. The present value comes from the future value of $1,000 and the payments, which we treat like an annuity: (1 + ) 1 (1 + 0.0273) 60 = 1,000(1 + 0.0273) 60 + 26.875 0.0273 60 1 1.0273 $987.53 = 1,000(1.0273) 60 + 26.875 0.0273

= (1 + ) +

1

Technology: 1000*1.0273^(-60)+26.875*(1-1.0273^(-60))/0.0273


Solutions Section 3.3 Online Time Value of Money Utility:

41. The periodic payments are based on a 3.625% annual payment. For payments twice a year, this is = 1,000(0.03625 2) = $18.125 Since the bond yield is 3.705%, = 0.03705 2 = 0.018525; = 2 × 2 = 4. The present value comes from the future value of $1,000 and the payments, which we treat like an annuity: (1 + ) 1 = 1,000(1 + 0.018525) 4 + 18.125

= (1 + ) +

1

(1 + 0.018525) 4 0.018525 1 1.018525 4 $998.47 = 1,000(1.018525) 4 + 18.125 0.018525

Technology: 1000*1.018525^(-4)+18.125*(1-1.018525^(-4))/0.018525 Online Time Value of Money Utility:

42. The periodic payments are based on a 4.375% annual payment. For payments twice a year, this is = 1,000(0.04375 2) = $21.875 Since the bond yield is 4.475%, = 0.04475 2 = 0.022375; = 2 × 5 = 10. The present value comes from the future value of $1,000 and the payments, which we treat like an annuity: (1 + ) 1 = 1,000(1 + 0.022375) 10 + 21.875

= (1 + ) +

1

(1 + 0.022375) 10 0.022375 10 1 1.022375 $995.56 = 1,000(1.022375) 10 + 21.875 0.022375


Solutions Section 3.3 Technology: 1000*1.022375^(-10)+21.875*(1-1.022375^(-10))/0.022375 Online Time Value of Money Utility:

43. The periodic payments are based on a 5.5% annual payment. For payments twice a year, this is = 1,000(0.055 2) = $27.5 Since the bond yield is 6.643%, = 0.06643 2 = 0.033215; = 2 × 10 = 20. The present value comes from the future value of $1,000 and the payments, which we treat like an annuity: (1 + ) 1 (1 + 0.033215) 20 = 1,000(1 + 0.033215) 20 + 27.5 0.033215 20 1 1.033215 $917.45 = 1,000(1.033215) 20 + 27.5 0.033215

= (1 + ) +

1

Technology: 1000*1.033215^(-20)+27.5*(1-1.033215^(-20))/0.033215 Online Time Value of Money Utility:

44. The periodic payments are based on a 6.25% annual payment. For payments twice a year, this is = 1,000(0.0625 2) = $31.25 Since the bond yield is 33.409%, = 0.33409 2 = 0.167045; = 2 × 10 = 20. The present value comes from the future value of $1,000 and the payments, which we treat like an annuity: (1 + ) 1 (1 + 0.167045) 20 = 1,000(1 + 0.167045) 20 + 31.25 0.167045 20 1 1.167045 $224.08 = 1,000(1.167045) 20 + 31.25 0.167045

= (1 + ) +

1


Solutions Section 3.3 Technology: 1000*1.167045^(-20)+31.25*(1-1.167045^(-20))/0.167045 Online Time Value of Money Utility:

45. = $400, = 0.3031 12, = 12 × 20 = 240 (1 + 0.3031 12) 240 1 (1 + ) 1 = $6,288,642.68 = 400 0.3031 12 Technology: 400*((1+0.3031/12)^240-1)/(0.3031/12) 46. = $380, = 0.1798 12, = 12 × 25 = 300 (1 + 0.1798 12) 300 1 (1 + ) 1 = $2,171,731.44 = 380 0.1798 12 Technology: 380*((1+0.1798/12)^300-1)/(0.1798/12) 47. Current value: = $500, = 0.0363 12, = 12 × 15 = 180 (1 + 0.0363 12) 180 1 (1 + ) 1 = $119,393.41 = 500 0.0363 12 Technology: 500*((1+0.0363/12)^180-1)/(0.0363/12) Value in 20 years: = $500, = 0.0310 12, = 12 × 20 = 240 (1 + ) 1 = (1 + ) + (1 + 0.0310 12) 240 1 = 119,393.41(1 + 0.0310 12) 240 + 500 0.0310 12 $387,723.01 Technology: 119393.41*(1+0.0310/12)^240 + 500*((1+0.0310/12)^240-1)/(0.0310/12) 48. Current value: = $500, = 0.0310 12, = 12 × 15 = 180 (1 + 0.0310 12) 180 1 (1 + ) 1 = $114,398.69 = 500 0.0310 12 Technology: 500*((1+0.0310/12)^180-1)/(0.0310/12) Value in 20 years: = $500, = 0.0363 12, = 12 × 20 = 240 (1 + ) 1 = (1 + ) + (1 + 0.0363 12) 240 1 = 114,398.69(1 + 0.0363 12) 240 + 500 0.0363 12 $412,135.79 Technology: 114398.69*(1 + 0.0363/12)^240 + 500*((1+0.0363/12)^240-1)/(0.0363/12) 49. = $1,500,000, = 0.0363 12, = 12 × 30 = 360


Solutions Section 3.3 0.0363 12 = $2,307.49 = 1,500,000 (1 + ) 1 (1 + 0.0363 12) 360 1 Technology: 1500000*0.0363/12/((1+0.0363/12)^360-1)

50. = $1,000,000, = 0.0310 12, = 12 × 25 = 300 0.0310 12 = $2,210.96 = 1,000,000 (1 + ) 1 (1 + 0.0310 12) 300 1 Technology: 1000000*0.0310/12/((1+0.0310/12)^300-1)

51. Take to be the amount you deposit each month into the stock fund. Then the deposit into the bond fund is four times that amount. The future value of the account is the sum of the future values of two investments: $ per month at 30.31% for 12 × 25 = 300 months, and $4 per month at 3.10% for 300 months: (1 + 0.3031 12) 300 1 (1 + 0.0310 12) 300 1 = + 4 0.3031 12 0.0310 12 70,362.85 + 1,809.17 = 72,172.02 2,000,000 We need this amount to equal two million: 72,172.02 = 2,000,000, so = 27.712. 72,172.02 Thus, $27.72 should be invested in the stock fund each month (rounding up to ensure you save enough). The amount in the bond fund is four times that amount: 4 × 27.72 = $110.88 per month.

52. Take to be the amount deposited each month into the stock fund. Then the deposit into the bond fund is three times that amount. The future value of the account is the sum of the future values of two investments: $ per month at 17.98% for 12 × 25 = 300 months, and $3 per month at 3.63% for 300 months: (1 + 0.1798 12) 300 1 (1 + 0.0363 12) 300 1 = + 3 0.1798 12 0.0363 12 5,715.08 + 1,462.54 = 7,177.62 We need this amount to equal one million: 7,177.62 = 1,000,000, so 1,000,000 = 139.322 139.33 (rounding up to ensure sufficient savings). This is the amount 7,177.62 that should be in the stock fund. The total investment per month is four times that amount: 4 × 139.33 = $557.32 per month. Note that rounding up to three decimal places instead of two results in a slightly smaller value, $557.29, for 4 , but in that case your deposits in the stock fund would be slighly more than 25% of the total. 53. Funding the $5,000 withdrawals: = 5,000, = 0.3031 12, = 12 × 10 = 120 1 (1 + 0.3031 12) 120 1 (1 + ) = $188,033.18 = 5,000 0.3031 12 Technology: 5000*(1-(1 + 0.3031/12)^(-120))/(0.3031/12) To fund the lump sum of $30,000 after 10 years, we need the present value of $30,000 under compound interest: = 30,000(1 + 0.3031 12) 120 = $1,503.58. (We rounded up to ensure that we won't fall a little short.)

So, the total in the trust should be $188,033.18 + $1,503.58 = $189,536.76. However, you can check on a TVM calculator that $189,536.75 would also be sufficient to fund the 54. Funding the $2,000 withdrawals: = 2,000, = 0.1798 12, = 12 × 20 = 240


Solutions Section 3.3 1 (1 + 0.1798 12) 240 (1 + ) = $129,720.83 (We rounded up to = 2,000 0.1798 12 ensure that we won't fall a little short.) Technology: 2000*(1-(1 + 0.1798/12)^(-240))/(0.1798/12) To fund the lump sum of $100,000 after 20 years, we need the present value of $100,000 under compound interest: 1

= 100,000(1 + 0.1798 12) 240 = $2,817.49. (We are rounding up to ensure that won't fall a little short.)

So, the total in the trust should be $129,720.83 + $2,817.49 = $132,538.32. Notice that we rounded up twice to get the total. However, using the TVM solver will show that payments of one penny less, $132,538.31, are also sufficient to fund the trust. 55. This is a two-stage process: (1) An accumulation stage to build the retirement fund (2) An amortization stage depleting the fund during retirement Stage 1: Building the retirement fund. = $1,200, = 0.04 4 = 0.01, = 4 × 40 = 160. (1 + ) 1 160 1.01 1 = 1,200 0.01 $469,659.16

=

Technology: 1200*(1.01^160-1)/0.01 Stage 2: Depleting the fund. = $469,659.16, = 0.04 4 = 0.01, = 4 × 25 = 100. 1 (1 + ) 469,659.16 × 0.01 = 1 1.01 100 $7,451.49

=

Technology: 469659.16*0.01/(1-1.01^(-100)) 56. This is a two-stage process: (1) An accumulation stage to build the retirement fund (2) An amortization stage depleting the fund during retirement Stage 1: Building the retirement fund. = $300, = 0.05 12, = 12 × 45 = 540. (1 + ) 1 (1 + 0.05 12) 540 = 300 0.05 12 $181,630.10

=

1

Technology: 300*((1+0.05/12)^540-1)/(0.05/12) Stage 2: Depleting the fund. = $607,931.19, = 0.05 12, = 12 × 20 = 240. =

1

(1 + )


=

607,931.19 × 0.05 12

Solutions Section 3.3

1 (1 + 0.05 12) 240 $4,012.08

Technology: 607931.19*(0.05/12)/(1-(1+0.05/12)^(-240)) 57. This is a two-stage process: (1) An accumulation stage to build the retirement fund (2) An amortization stage depleting the fund during retirement As in the text, we work backward, starting with Stage 2, where we have = $5,000, = 0.03 12 = 0.0025, and = 20 × 12 = 240, and we need to calculate the starting value . (1 + ) 1 (1 + 0.0025) 240 = 5,000 0.0025 1 (1.0025) 240 = 5,000 0.0025 $901,554.57

=

1

Technology: 5000*(1-1.0025^(-240))/0.0025 Thus, in Stage 1, you need to accumulate $901,554.57 in the annuity: = $901,554.57, = 0.03 12 = 0.0025, = 40 × 12 = 480. (1 + ) 1 901,554.57 × 0.0025 = 1.0025 480 1 $973.54 per month

=

Technology: 901554.57*0.0025/(1.0025^480-1) 58. This is a two-stage process: (1) An accumulation stage to build the retirement fund (2) An amortization stage depleting the fund during retirement (1 + ) 1 (1 + 0.0125) 100 = 12,000 0.0125 1 (1.0125) 100 = 12,000 0.0125 $682,816.07

=

1

Technology: 12000*(1-1.0125^(-100))/0.0125 Thus, in Stage 1, Meg needs to accumulate $682,816.07 in the annuity: = $682,816.07, = 0.05 4 = 0.0125, = 4 × 45 = 180. (1 + ) 1 682,816.07 × 0.0125 = 1.0125 180 1 $1,021.40 per quarter

=


Solutions Section 3.3 Technology: 682816.07*0.0125/(1.0125^180-1)

59. = $1,000,000, = 0.048 12 = 0.004, = 12 × (87 30) = 684 0.004 = $278.92 = 1,000,000 (1 + ) 1 1.004 684 1 Technology: 1000000*0.004/(1.004^684-1) 60. = $1,000,000, = 0.048 12 = 0.004, = 12 × (76 30) = 552 0.004 = $496.43 = 1,000,000 (1 + ) 1 1.004 552 1 Technology: 1000000*0.004/(1.004^552-1) 61. Take to be the current age of an insured person. = $500,000, = 0.048 12 = 0.004; 0.004 = = 500,000 (1 + ) 1 1.004 1 0.004 Men: = = 500,000 12(75 ) (1 + ) 1 1.004 1 0.004 Women: = = 500,000 (1 + ) 1 1.004 12(80 ) 1 Technology: 500000*0.004/(1.004^(12*(75-x))-1) 500000*0.004/(1.004^(12*(80-x))-1) Result: Age

Men

Women

30

$261.99

$200.59

50

$864.98

$623.33

70 $7,389.87 $3,254.53 62. Take to be the current age of an insured person. = $800,000, = 0.048 12 = 0.004; 0.004 = = 800,000 (1 + ) 1 1.004 1 0.004 Men: = = 800,000 12(73 ) (1 + ) 1 1.004 1 0.004 Women: = = 800,000 (1 + ) 1 1.004 12(79 ) 1 Technology: 800000*0.004/(1.004^(12*(73-x))-1) 800000*0.004/(1.004^(12*(79-x))-1) Result: Age

Men

Women

20

$274.30

$201.46

40

$829.22

$584.26

60 $3,703.46 $2,155.21


Solutions Section 3.3 63. While in Mexico: = $750,000, = 0.048 12 = 0.004, = 12 × (73 22) = 612 0.004 = $285.47 = 750,000 (1 + ) 1 1.004 612 1 When he moved to Canada eight years later, = 12 × 8 = 96, so the value of the policy was (1 + ) 1 1.004 96 1 = $33,330.15. = 285.47 0.004 To calculate the required payments in Canada, we use an annuity with a present value of $33,330.15 and want the payments necessary to increase this amount to $750,000 in (80 30) = 50 years ( = 12 × 50 = 600), so we use the formula in the "Before we go on" discussion in Example 1: (1 + ) 1 = (1 + ) + and solve for to obtain = ( (1 + ) ) (1 + ) 1 0.004 = �750,000 33,330.15(1.004) 600� 1.004 600 1 $154.19. So, the premiums decreased by 285.47 154.19 = $131.28. 64. While in the U.K.: = $800,000, = 0.048 12 = 0.004, = 12 × (83 25) = 696 0.004 = $212.00 = 800,000 (1 + ) 1 1.004 696 1 When she moved to India ten years later, = 12 × 10 = 120, so the value of the policy was (1 + ) 1 1.004 120 1 = $32,569.98. = 212.00 0.004 To calculate the required payments in India, we use an annuity with a present value of $32,569.98 and want the payments necessary to increase this amount to $800,000 in (72 35) = 37 years ( = 12 × 37 = 444), so we use the formula in the "Before we go on" discussion in Example 1: (1 + ) 1 = (1 + ) + and solve for to obtain = ( (1 + ) ) (1 + ) 1 0.004 = �800,000 32,569.98(1.004) 444� 1.004 444 1 $498.08. So, the premiums increased by 498.08 212.00 = $286.08. 65. We know the payments and need to calculate the present value: = 0.0303 12, = 12 × 30 = 360, = 600 = 1 (1 + ) 0.0303 12 600 = 1 (1 + 0.0303 12) 360 1 (1 + 0.0303 12) 360 so = 600 $141,768.99. 0.0303 12 This is the amount you can afford to finance. Including the down payment gives a total of $141,768.99 + 20,000 = $161,768.99. 66. We know the payments and need to calculate the present value:


Solutions Section 3.3 = 0.0230 12, = 12 × 15 = 180, = 900 = 1 (1 + ) 0.0230 12 900 = 1 (1 + 0.0230 12) 180 1 (1 + 0.0230 12) 180 so = 900 $136,899.74. 0.0230 12 This is the amount you can afford to finance. Including the down payment gives a total of $136,899.74 + 50,000 = $186,899.74

67. Your hunch: Wait a month for the price to go down to $140,000: = $140,000, = 0.0310 12, = 12 × 30 = 360 0.0310 12 = $597.82 = 140,000 1 (1 + ) 1 (1 + 0.0310 12) 360 Technology: 140000*(0.0310/12)/(1-(1+0.0310/12)^(-360)) Broker's suggestion: Buy now for $150,000: = $150,000, = 0.0303 12, = 12 × 30 = 360 0.0303 12 = $634.84 = 150,000 1 (1 + ) 1 (1 + 0.0303 12) 360 Technology: 150000*(0.0303/12)/(1-(1+0.0303/12)^(-360)) Conclusion: Wait a month and pay $634.84 597.82 = $37.02 less per month. 68. Your friends' hunch: Wait a month for the price to go down to $297,000: = $297,000, = 0.0250 12, = 12 × 15 = 180 0.0250 12 = $1,980.36 = 297,000 1 (1 + ) 1 (1 + 0.0250 12) 180 Technology: 297000*(0.0250/12)/(1-(1+0.0250/12)^(-180)) Broker's suggestion: Buy now for $300,000: = $300,000, = 0.0230 12, = 12 × 15 = 180 0.0230 12 = $1,972.25 = 300,000 1 (1 + ) 1 (1 + 0.0230 12) 180 Technology: 300000*(0.0230/12)/(1-(1+0.0230/12)^(-180)) Conclusion: The agent was right; buying now would have saved your friends $1,980.36 1,972.25 = $8.11 per month.

69. We first calculate the payments: = $150,000, = 0.0303 12, = 12 × 30 = 360 0.0303 12 = $634.84 = 150,000 1 (1 + ) 1 (1 + 0.0303 12) 360 Technology: 150000*(0.0303/12)/(1-(1+0.0303/12)^(-360)) Now calculate the outstanding principal: = 12 × 15 = 180, = 360 180 = 180, = 634.84 1 (1 + ) ( ) Outstanding principal = 1 (1 + 0.0303 12) 180 $91,736.52 = 634.84 0.0303 12 Technology: 634.84*(1-(1+0.0303/12)^-180)/(0.0303/12) 70. We first calculate the payments: = $300,000, = 0.0230 12, = 12 × 15 = 180 0.0230 12 = $1,972.25 = 300,000 1 (1 + ) 1 (1 + 0.0230 12) 180 Technology: 300000*(0.0230/12)/(1-(1+0.0230/12)^(-180))


Solutions Section 3.3 Now calculate the outstanding principal: = 12 × 8 = 96, = 180 96 = 84, = 1,972.25 1 (1 + ) ( ) Outstanding principal = 1 (1 + 0.0230 12) 84 $152,885.46 = 1,972.25 0.0230 12 Technology: 1972.25*(1-(1+0.0230/12)^-84)/(0.0230/12) 71. We first calculate the payments on the original mortgage: = $250,000, = 0.0303 12, = 12 × 30 = 360 0.0303 12 = $1,058.06 = 250,000 1 (1 + ) 1 (1 + 0.0303 12) 360 Technology: 250000*(0.0303/12)/(1-(1+0.0303/12)^(-360)) Now calculate the outstanding principal: = 12 × 10 = 120, = 360 120 = 240, = 1,058.06 1 (1 + ) ( ) Outstanding principal = 240 1 (1 + 0.0303 12) $190,264.14 = 1,058.06 0.0303 12 Technology: 1058.06*(1-(1+0.0303/12)^-240)/(0.0303/12) Now calculate the mortgage payments at the new rate on the the outstanding principal plus prepayment fee: = $190,264.14 × 1.04 $197,874.71, = 0.0200 12, = 12 × 20 = 240 0.0200 12 = $1,001.02 = 197,874.71 1 (1 + ) 1 (1 + 0.0200 12) 240 Technology: 197874.71*(0.0200/24)/(1-(1+0.0200/24)^(-240)) Saving = $1,058.06 1,001.02 = $57.04 72. We first calculate the payments on the original mortgage: = $500,000, = 0.0230 12, = 12 × 15 = 180 0.0230 12 = $3,287.08 = 500,000 1 (1 + ) 1 (1 + 0.0230 12) 180 Technology: 500000*(0.0230/12)/(1-(1+0.0230/12)^(-180)) Now calculate the outstanding principal: = 12 × 5 = 60, = 180 60 = 120, = 3,287.08 1 (1 + ) ( ) Outstanding principal = 1 (1 + 0.0230 12) 120 $352,074.52 = 3,287.08 0.0230 12 Technology: 3287.08*(1-(1+0.0230/12)^-120)/(0.0230/12) Now calculate the mortgage payments at the new rate on the the outstanding principal plus prepayment fee: = $352,074.52 × 1.03 $362,636.76, = 0.0150 12, = 12 × 10 = 120 0.0150 12 = $3,256.17 = 362,636.76 1 (1 + ) 1 (1 + 0.0150 12) 120 Technology: 362636.76*(0.0150/12)/(1-(1+0.0150/12)^(-120)) Saving = $3,287.08 3,256.17 = $30.91 73. We first calculate the payments: = $40,000, = 0.0920 12, = 12 × 5 = 60 0.0920 12 = = 40,000 1 (1 + ) 1 (1 + 0.0920 12) 60

$834.22


Solutions Section 3.3 Technology: 40000*(0.0920/12)/(1-(1+0.0920/12)^(-60)) Now calculate the amounts you could have financed at the other rates: At 9.00%: = 0.0900 12, = 60, = 834.22 = 1 (1 + ) 0.0900 12 834.22 = 1 (1 + 0.0900 12) 60 1 (1 + 0.0900 12) 60 so = 834.22 $40,187.19. 0.0900 12 At 9.50%: = 0.0950 12, = 60, = 834.22 = 1 (1 + ) 0.0950 12 834.22 = 1 (1 + 0.0950 12) 60 1 (1 + 0.0950 12) 60 so = 834.22 $39,721.24. 0.0950 12 74. We first calculate the payments: = $20,000, = 0.0945 12, = 12 × 3 = 36 0.0945 12 = $640.19 = 20,000 1 (1 + ) 1 (1 + 0.0945 12) 36 Technology: 20000*(0.0945/12)/(1-(1+0.0945/12)^(-36)) Now calculate the amounts you could have financed at the other rates: At 9.20%: = 0.0920 12, = 36, = 640.19 = 1 (1 + ) 0.0920 12 640.19 = 1 (1 + 0.0920 12) 36 1 (1 + 0.0920 12) 36 so = 640.19 $20,073.12. 0.0920 12 At 9.75%: = 0.0975 12, = 36, = 640.19 = 1 (1 + ) 0.0975 12 640.19 = 1 (1 + 0.0975 12) 36 1 (1 + 0.0975 12) 36 so = 640.19 $19,912.63. 0.0975 12 75. Treat the card as a loan: = 5,000, = 0.1399 12, = 12 × 10 = 120 = 1 (1 + ) 0.1399 12 $77.60. = 5,000 1 (1 + 0.1399 12) 120 Technology: 5000*(0.1399/12)/(1-(1+0.1399/12)^(-120)) 76. Treat the card as a loan: = 8,000, = 0.1399 12, = 12 = 1 (1 + ) 0.1399 12 = 8,000 1 (1 + 0.1399 12) 12

$718.26.


Solutions Section 3.3 Technology: 8000*(0.1399/12)/(1-(1+0.1399/12)^(-12))

77. Solid Savings & Loan: = $10,000, = 0.09 12 = 0.0075, = 12 × 4 = 48. 10,000 × 0.0075 = $248.85 = 1 (1 + ) 1 1.0075 48 Technology: 10000*0.0075/(1-1.0075^(-48)) Fifth Federal Bank & Trust: = $10,000, = 0.07 12, = 12 × 3 = 36. 10,000 × 0.07 12 = $308.77 = 1 (1 + ) 1 (1 + 0.07 12) 36 Technology: 10000*(0.07/12)/(1-(1+0.07/12)^(-36)) Answer: You should take the loan from Solid Savings & Loan: It will have payments of $248.85 per month. The payments on the other loan would be more than $300 per month.

78. 10% Loan: = $20,000, = 0.10 12, = 12 × 5 = 60 20,000 × 0.10 12 = $424.94 = 1 (1 + ) 1 (1 + 0.10 12) 60 Technology: 20000*(0.10/12)/(1-(1+0.10/12)^(-60)) 9% Loan: = $20,000, = 0.09 12 = 0.0075, = 12 × 4 = 48 20,000 × 0.0075 = $497.70 = 1 (1 + ) 1 1.0075 48 Technology: 20000*0.0075/(1-1.0075^(-48)) The first loan will have lower monthly payments. Total Interest Payments: 10% Loan: You pay a total of 60 × $424.94 = $25,496.40. Of this, $25,496.40 20,000 = $5,496.40 is interest. 9% Loan: You pay a total of 48 × $497.70 = $23,889.60. Of this, $23,889.60 20,000 = $3,889.60 is interest. Thus, the first loan will have a larger total interest payment. 79. We first calculate the original payments: = $96,000, = 0.0975 12, = 12 × 30 = 360 0.0975 12 = $824.79 = 96,000 1 (1 + ) 1 (1 + 0.0975 12) 360 Technology: 96000*(0.0975/12)/(1-(1+0.0975/12)^(-360)) Now calculate the outstanding principal: = 12 × 4 = 48, = 360 48 = 312, = 824.79 1 (1 + ) ( ) Outstanding principal = 1 (1 + 0.0975 12) 312 $93,383.71 = 824.79 0.0975 12 Technology: 824.79*(1-(1+0.0975/12)^(-312))/(0.0975/12) Total Interest Paid During the First Loat = Sum of payments − Reduction in principal = 48 × 824.79 (96,000 93,383.71) = $36,973.63 Now calculate the mortgage payments at the new rate on the outstanding principal: = $93,383.71, = 0.06875 12, = 12 × 30 = 360 0.06875 12 = $613.46 = 93,383.71 1 (1 + ) 1 (1 + 0.06875 12) 360 Technology: 93383.71*(0.06875/12)/(1-(1+0.06875/12)^(-360)) Total Interest Paid During the New Loat = Sum of payments − Reduction in principal = 360 × 613.46 93,383.71 = $127,461.89 Thus, the total interest paid over the duration of the two loans is $36,973.63 + 127,461.89 = $164,435.52. Had the mortgage not been refinanced, the total interest would have been


Solutions Section 3.3 Total Interest Paid = Sum of payments − Reduction in principal = 360 × 824.79 96,000 = $20,0924.40.Thus, the saving on interest is $200,924.40 164, 435.52 = $36, 488.88 80. We first calculate the original payments: = $120,000, = 0.10 12, = 12 × 30 = 360 0.10 12 = $1,053.09 = 120,000 1 (1 + ) 1 (1 + 0.10 12) 360 Technology: 120000*(0.10/12)/(1-(1+0.10/12)^(-360)) Now calculate the outstanding principal: = 12 × 3 = 36, = 360 36 = 324, = 1,053.09 1 (1 + ) ( ) Outstanding principal = 1 (1 + 0.10 12) 324 $117,782.44 = 1,053.09 0.10 12 Technology: 1053.09*(1-(1+0.10/12)^(-324))/(0.10/12) Total Interest Paid = Sum of payments − Reduction in principal = 36 × 1,053.09 (120,000 117,782.44) = $35,693.68 Now calculate the mortgage payments at the new rate on the outstanding principal: = $117,782.44, = 0.065 12, = 12 × 15 = 180 0.065 12 = $1,026.01 = 117,782.44 1 (1 + ) 1 (1 + 0.065 12) 180 Technology: 117,782.44*(0.065/12)/(1-(1+0.065/12)^(-180)) Total Interest Paid During the New Loat = Sum of payments − Reduction in principal = 180 × 1,026.01 117,782.44 = $66,899.36 Thus, the total interest paid over the duration of the loan is $35,693.68 + 66,899.36 = $102,593.04. Had the mortgage not been refinanced, the total interest would have been Total Interest Paid = Sum of payments − Reduction in principal = 360 × 1,053.09 120,000 = $259,112.40.Thus, the saving on interest is $259,112.40 102,593.04 = $156,519.36


Solutions Section 3.3 81. We can construct an amortization table using the technique outlined in the Technology Guides. For example, using Excel, we might set it up as follows:

Adding the payments on principal (Column C) and interest payments (Column D) for each year will give the following table: Year

Interest

Payment on Principal

1

$3,934.98

$1,798.98

2

$3,785.69

$1,948.27

3

$3,623.97

$2,109.99

4

$3,448.84

$2,285.12

5

$3,259.19

$2,474.77

6

$3,053.77

$2,680.19

7

$2,831.32

$2,902.64

8

$2,590.39

$3,143.57

9

$2,329.48

$3,404.48

10

$2,046.91

$3,687.05

11

$1,740.88

$3,993.08

12

$1,409.47

$4,324.49

13

$1,050.54

$4,683.42

14

$661.81

$5,072.15

15

$240.84

$5,491.80


Solutions Section 3.3 82. We can create an amortization table as in Exercise 81. We will get the following: Year

Interest

Payment on Principal

1

$9,238.08

$556.32

2

$9,181.34

$613.06

3

$9,118.83

$675.57

4

$9,049.93

$744.47

5

$8,974.02

$820.38

6

$8,890.36

$904.04

7

$8,798.17

$996.23

8

$8,696.56

$1,097.84

9

$8,584.62

$1,209.78

10

$8,461.26

$1,333.14

11

$8,325.30

$1,469.10

12

$8,175.50

$1,618.90

13

$8,010.39

$1,784.01

14

$7,828.46

$1,965.94

15

$7,627.96

$2,166.44

Adding up the payments on principal gives a total of $17,955.22 paid on principal over the first 15 years, so $95,000 17,955.22 = $77, 044.78 is still owed on the mortgage. 83. The payments on the loan, ignoring the fee, are 0.09 12 = 5,000 = $228.42. 1 (1 + 0.09 12) 24 Add to each payment 100/24 = $4.17 to get a new payment of $232.59. Now use technology to find the interest rate being charged on a 2-year $5,000 loan with this payment. For example, we can use the Online Time Value of Money Utility:

We see that the interest rate is 10.81%. 84. The payments on the loan, ignoring the fee, are 0.08 12 = 7,000 = $219.35. 1 (1 + 0.08 12) 36 Add to each payment 100/36 = $2.78 to get a new payment of $222.13. Now use technology to find the


Solutions Section 3.3 interest rate being charged on a 3-year $7,000 loan with this payment. For example, we can use the Online Time Value of Money Utility:

We see that the interest rate is 8.86%. 85. TI-83/84 Plus:

This gives 153.5 12 13 years to retirement. Online Time Value of Money Utility:

86. TI-83/84 Plus:

This gives 63.72 4 16 years to retirement. Online Time Value of Money Utility:


Solutions Section 3.3 87. TI-83/84 Plus:

This gives 55.798 12 4.5 years to repay the debt. Online Time Value of Money Utility:

88. Since we are not told what "eventually" means, let us try a 50-year schedule. We get TI-83/84 Plus:

In other words, if we pay $25.01 it will take us 50 years to pay off the debt. Larger values of give amounts fractionally larger than $25.00. To be assured of eventually paying off the debt, you should therefore pay $25.01. Online Time Value of Money Utility:

Another way of seeing this: The monthly interest on $2,000 at 15% is $2000 × 0.15 12 = $25.00. So, if you pay only $25.00, all you are paying is interest and you are not reducing the $2,000 principal. 89. Graph the future value of both accounts using the formula for the future value of a sinking fund. Your account: = 0.045 12 = 0.00375, = $100. (1 + ) 1 1.00375 1 = = 100 0.00375 To graph this, use the technology formula 100*(1.00375^x-1)/0.00375. Lucinda's account: = 0.065 12, = $75.


Solutions Section 3.3 (1 + 0.065 12) 1 (1 + ) 1 = = 75 (0.065 12) To graph this, use the technology formula 75*((1+0.065/12)^x-1)/(0.065/12) Graph:

The graphs appear to cross around = 300, so we zoom in there:

The graphs cross around = 287, so the number of years is approximately = 287.5 12

24 years.

90. If you purchased the car with an 8% per year loan, the length of the loan would be given as follows: TI-83/84 Plus:

To the nearest year, this is 61 5 so the answer is 5 years or less.

5 years. Leasing the car for less time would result in lower payments,

91. He is wrong because his estimate ignores the interest that will be earned by your annuity—both while it is increasing and while it is decreasing. Your payments will be considerably smaller (depending on the interest earned). 92. She is incorrect. For example, compare Option 1, making a single payment of $10,000 and letting it earn interest for the next 10 years, with Option 2, making yearly payments of $1,000 for 10 years. All $10,000 earns interest for all 10 years in Option 1, while most of your money earns interest for a shorter period of time (and hence earns less interest) in Option 2. 93. Wrong; the split investment earns more. For instance, after ten years it earns $31,056.46 + $32,775.87 = $63,832.33, which is more than the $63,803.03 earned by the single investment. (Mathematically, the future value does not depend linearly on the interest rate.) 94. Wrong; the combined investments earn net interest. For instance, after 10 years the value is: $62,112.91 + $37,833.85 = $99,946.76, which is more than the accumulated payments: $96,000.


Solutions Section 3.3 95. He is not correct. For instance, the payments on a $100,000 10-year mortgage at 12% are $1,434.71, while for a 20-year mortgage at the same rate, they are $1,101.09, which is a lot more than half the 10year mortgage payment. 96. He is correct. For instance, the payments on a $100,000 10-year mortgage at 12% are $1,434.71, while for a $200,000 10-year mortgage at the same rate, they are twice that amount: $2,869.42.

97. = maturity value = $1,000, = 1,000(0.035 2) = $17.50, = selling price = $994.69, = 2 × 5 = 10. Using technology, we compute the interest rate to be approximately 3.617%. (The online utility rounds this to two decimal places.) TI-83/84 Plus:

Online Time Value of Money Utility ("Years" mode):

98. = $1,000, = 1,000(0.03375 2) = $16.875, = $991.20, = 2 × 10 = 20. Using technology, we compute the interest rate to be approximately 3.480%. (The online utility rounds this to two decimal places.) TI-83/84 Plus:

Online Time Value of Money Utility ("Years" mode):

99. = (1 + ) =

(1 + )

1

(1 + ) =

1

(1 + )

100. There are several possible explanations, including the following: Consider two funds with identical interest rates and compounding: The accumulating fund (beginning with no money) and the payment fund, beginning with the present value necessary to fund the payments. Move each payment from the payment fund to the accumulating fund. Since the funds pay the same interest, the net effect is a single account paying compound interest, with no payments or withdrawals. Therefore, the present value of the payment fund must give the future value of the accumulating fund using the future value formula for compound interest.


Solutions Chapter 3 Review Chapter 3 Review

1. = 6,000(1 + 0.0475 × 5) = $7,425.00

2. = 10,000(1 + 0.0525 × 2.5) = $11,312.50

3. = 6,000(1 + 0.0475 12) 60 = $7,604.88

4. = 10,000(1 + 0.0525 2) 5 = $11,383.24

5. = 100

(1 + 0.0475 12) 60 0.0475 12

1

= $6,757.41

6. = 2,000

(1 + 0.0525 2) 5 0.0525 2

1

= $10,538.96

7. = 6,000 (1 + 0.0475 × 5) = $4,848.48

8. = 10,000 (1 + 0.0525 × 2.5) = $8,839.78

9. = 6,000(1 + 0.0475 12) 60 = $4,733.80

10. = 10,000(1 + 0.0525 2) 5 = $8,784.85

11.

= 100

1

(1 + 0.0475 12) 0.0475 12

13.

= 12,000

15.

= 6,000

17.

0.0475 12

(1 + 0.0475 12) 60

1

= 10,000

60

1

0.0475 12

= $5,331.37

1

(1 + 0.0475 12) 60 0.0475 12

= $177.58

= $112.54

(1 + 0.0475 12) 60

= $187.57

12. = 2,000

1

14. = 20,000

16. = 10,000

(1 + 0.0525 2) 5 = $9,258.32 0.0525 2 0.0525 2

(1 + 0.0525 2) 5

18. = 15,000

19. 10,000 = 6,000(1 + 0.0475 ) = 6,000 + 285 . = (10,000

20. 15,000 = 10,000(1 + 0.0525 ) = 10,000 + 525 . = (15,000

0.0525 2

1

1

= $3,795.44

(1 + 0.0525 2) 5

1

0.0525 2

(1 + 0.0525 2) 5

6,000) 285 = 14.0 years.

10,000) 525 = 9.5 years

21. 10,000 = 6,000(1 + 0.0475 12) 12 . To solve algebraically requires logarithms: log(10,000 6,000) = 10.8 years 12 log(1 + 0.0475 12) We could also find this using, for example, the TI-83/84 Plus TVM Solver. 22. 15,000 = 10,000(1 + 0.0525 2) 2 . To solve algebraically requires logarithms: log(15,000 10,000) = 7.8 years 2 log(1 + 0.0525 2) We could also find this using, for example, the TI-83/84 Plus TVM Solver.

= $2,160.22

= $3,240.33


Solutions Chapter 3 Review (1 + 0.0475 12) 12 1 23. 10,000 = 100 . To solve algebraically requires logarithms: 0.0475 12 log[0.0475 × 10,000 (100 × 12) + 1] = 7.0 years 12 log(1 + 0.0475 12) We could also find this using, for example, the TI-83/84 Plus TVM Solver. (1 + 0.0525 2) 2 1 . To solve algebraically requires logarithms: 0.0525 2 log[0.0525 × 15,000 (2,000 × 2) + 1] = 3.5 years 12 log(1 + 0.0525 2) We could also find this using, for example, the TI-83/84 Plus TVM Solver. 24. 15,000 = 2,000

25. Each interest payment is 10,000 × 0.06 2 = $300. For an annuity earning 7% and paying $300 every 6 months for 5 years, the present value is 1 (1 + 0.07 2) 10 = 300 = $2,494.98. 0.07 2 The present value of the $10,000 maturity value is = 10,000(1 + 0.07 2) 10 = $7,089.19. The total price is $2,494.98 + 7,089.19 = $9,584.17. 26. Each interest payment is 10,000 × 0.06 2 = $300. For an annuity earning 5% and paying $300 every 6 months for 5 years, the present value is 1 (1 + 0.05 2) 10 = 300 = $2,625.62. 0.05 2 The present value of the $10,000 maturity value is = 10,000(1 + 0.05 2) 10 = $7,811.98. The total price is $2,625.62 + 7,811.98 = $10,437.60. 27. 5.346% (using, for example, the TI-83/84 Plus TVM Solver) 28. 4.662% (using, for example, the TI-83/84 Plus TVM Solver) 29. = 3.28, = 45.74, = 9 12 years 45.74 = 3.28(1 + 9 12) 1 + 9 12 = 45.74 3.28 = (45.74 3.28 1) (9 12) 17.2602 = 1,726.02%

30. = 33.95, = 12.36, = 22 12 years 12.36 = 33.95(1 + 22 12) 1 + 22 12 = 12.36 33.95 = (12.36 33.95 1) (22 12) 0.3469 =

34.69%

31. The only dates on which she would have gotten an increase rather than a decrease were November 2019, February 2020, and August 2020. Calculating the annual returns as in the preceding exercises, we get the following figures: Jan. 2017–Nov. 2019: 60.45% Jan. 2017–Feb. 2020: 85.28% Jan. 2017–Aug. 2020: 75.37% The largest annual return was 85.28% if she sold in February 2020. 32. The only dates on which he would have gotten a loss rather than an increase were December 2015, January 2017, March 2018, and October 2018. Calculating the annual returns as in the preceding exercises, we get the following


Solutions Chapter 3 Review figures: Aug. 2014–Dec. 2015: 40.79% Aug. 2014–Jan. 2017: 10.02% Aug. 2014–Mar. 2018: 15.81% Aug. 2014–Oct. 2018: 7.17% The largest annual loss was 40.79% if he sold in December 2015. 33. No. Simple interest increase is linear. We can compare slopes between successive points to see whether the slope remained roughly constant: From December 2012 to August 2014 the slope was (16.31 3.28) (20 12) = 7.818, while from August 2014 to March 2015 the slope was (33.95 16.31) (7 12) = 30.24. These slopes are quite different. 34. First calculate the annual interest rate from February 2020 through August 2020: = (45.74 44.86 1) (6 12) 0.03923 Now use the simple interest formula to get the price in December 2021: 44.86(1 + 0.03923 × 22 12) 48.09

35. Use the compound interest formula: = (1 + ) , where = $150,000, = 0.20, = 1, = 1, 2, 3, 4, 5. 2010: = 150,000(1.20) = $180,000 2011: = 150,000(1.20) 2 = $216,000 2012: = 150,000(1.20) 3 = $259,200 2013: = 150,000(1.20) 4 = $311,040 2014: = 150,000(1.20) 5 = $373,248 Revenues first surpass $300,000 in 2013. 36. = $20,000, = 0.0375, = 1, = 7 (quarters) = (1 + ) = 20,000(1 0.0375) 7 $15,305.06

37. After the first day of trading, the value of each share will be $6. Thereafter, the shares appreciate in value by 8% per month for 6 months, and O'Hagan desires a future value of at least $500,000. = 500,000, = 0.08, = 1, = 6 = (1 + ) = 500,000 × 1.08 6 $315,084.81 Therefore, since each share will be worth $6 after the first day, the number of shares they must sell is at least 315,084.81 52,514.14. 6 Since they can offer only a whole number of shares, we must round this up to get the minimum desired future value. Thus, they should offer at least 52,515 shares. 38. After the first day of trading, the value of each share will be $3(1 in value by 10% per week for 5 weeks, and so the future value is = (1 + ) = 1.20(1 0.10) 5 $0.71 per share, so the approximate market value is 600,000 × 0.71 = $426,000.

39. = 250,000, = 0.095 12, = 12 × 10 = 120 250,000 × 0.095 12 = = 1 (1 + ) 1 (1 + 0.095 12) 120 40. = 250,000, = 0.065 12, = 12 × 8 = 96 250,000 × 0.065 12 = = 1 (1 + ) 1 (1 + 0.065 12) 96

$3,234.94 $3,346.56

0.6) = $1.20. Thereafter, the shares depreciate


Solutions Chapter 3 Review 41. = 0.095 12, = 3,000, = 12 × 10 = 120 1 (1 + 0.095 12) 120 1 (1 + ) = $231,844 (to the nearest dollar) = 3,000 (0.095 12) 42. = 0.065 12, = 3,000, = 12 × 8 = 96 1 (1 + 0.065 12) 96 1 (1 + ) = = 3,000 (0.065 12)

$224,111 (to the nearest dollar)

43. = 250,000, = 0, = 3,000, = 12 × 10 = 120, and we are seeking the interest rate. Online Time Value of Money Utility:

The interest rate would be 7.75%.

44. = 250,000, = 0, = 3,000, = 12 × 8 = 96, and we are seeking the interest rate. Online Time Value of Money Utility:

The interest rate would be 3.59%.

45. = 50,000, = 1,000 + 800 (company contribution) = 1,800, = 0.073 12, = 1, = 12 × 10 = 120. Considering the contribution of the present $50,000 as well as the payments, we get (1 + 0.073 12) 120 1 (1 + ) 1 = (1 + ) + $420,275 = 50,000(1 + 0.073 12) 120 + 1,800 0.073 12 (to the nearest dollar). Technology: 50000*(1+0.073/12)^120+1800*((1+0.073/12)^120-1)/(0.073/12) 46. = 60,000, = 950 + 800(company contribution) = 1,750, = 0.073 12, = 1, = 12 × 8 = 96. Considering the contribution of the present $60,000 as well as the payments, we get (1 + 0.073 12) 96 1 (1 + ) 1 = (1 + ) + $334,670 = 60,000(1 + 0.073 12) 96 + 1,750 0.073 12 (to the nearest dollar). Technology: 60000*(1+0.073/12)^96+1750*((1+0.073/12)^96-1)/(0.073/12)


Solutions Chapter 3 Review 47. For the company's contribution, take = 800, = 0.073 12, = 12 × 10 = 120. (1 + 0.073 12) 120 1 (1 + ) 1 = $140,778 = 800 0.073 12 (to the nearest dollar). Technology: 800*((1+0.073/12)^120-1)/(0.073/12)

48. For the company's contribution, take = 800, = 0.073 12, = 12 × (8 + 5) = 156. (1 + 0.073 12) 156 1 (1 + ) 1 = $207,217 = 800 0.073 12 (to the nearest dollar). Technology: 800*((1+0.073/12)^156-1)/(0.073/12)

49. We first take out the effect of the initial $50,000: = 50,000, = 0.073 12, = 1, = 12 × 10 = 120. = (1 + ) = 50,000(1 + 0.073 12) 120 Thus, the payments have to result in a future value of = 500,000 50,000(1 + 0.073 12) 120 396,475.163 We can now use the payment formula to determine the necessary payments: 396,475.163 × 0.073 12 = $2,253.06 (1 + ) 1 (1 + 0.073 12) 120 1 Since $800 of this is contributed by the company, Callahan's payments should be $2,253.06 800 = $1,453.06.

50. We first take out the effect of the initial $60,000: = 50,000, = 0.073 12, = 1, = 12 × 8 = 96. = (1 + ) = 60,000(1 + 0.073 12) 96 Thus, the payments have to result in a future value of = 600,000 60,000(1 + 0.073 12) 96 $492,598.3656 We can now use the payment formula to determine the necessary payments: 492,598.3656 × 0.073 12 = $3,793.08 (1 + ) 1 (1 + 0.073 12) 96 1 Since $800 of this is contributed by the company, Egan's payments should be $3,793.08 800 = $2,993.08.

51. First compute the amount Callahan needs at the start of retirement: = 5,000, = 0.087 12, = 12 × 30 = 360 1 (1 + 0.087 12) 360 1 (1 + ) = $638,461.93 = 5,000 0.087 12 Technology: 5000*(1-(1+0.087/12)^(-360))/(0.087/12) In order to accumulate this amount, using the information from Exercise 45, we first discount the effect of the current $50,000 in the account: = 50,000, = 638,461.93, = 0.073 12, = 12 × 10 = 120. The initial $50,000 will grow to 50,000(1 + 0.073 12) 120 so the payments need to result in a future value of only 638,461.93 50,000(1 + 0.073 12) 120 534,937.093. Now use the payment formula: 534,937.093 × 0.073 12 = $3,039.90 = (1 + ) 1 (1 + 0.073 12) 120 1 Since $800 of this is contributed by the company, Callahan's payments should be $3,039.90 800 = $2,239.90. 52. First compute the amount Egan needs at the start of retirement: = 6,000, = 0.078 12, = 12 × 25 = 300


Solutions Chapter 3 Review 1 (1 + 0.078 12) 300 (1 + ) = $790,915.68 = 6,000 0.078 12 Technology: 6000*(1-(1+0.078/12)^(-300))/(0.078/12) In order to accumulate this amount, we first discount the effect of the current $60,000 in the account: = 60,000, = 0.073 12, = 12 × 8 = 96. = (1 + ) = 60,000(1 + 0.073 12) 96 Thus, the payments have to result in a future value of = 790,915.68 60,000(1 + 0.073 12) 96 $683,514.04. Now use the payment formula: 683,514.04 × 0.073 12 = $5,263.17 (1 + ) 1 (1 + 0.073 12) 96 1 Since $800 of this is contributed by the company, Egan's payments should be $5,263.17 800 = $4,463.17. 1

53. The bond will pay interest every 6 months amounting to 50,000(0.072 2) = $1,800. For someone purchasing this bond after one year, there will be 9 years to maturity. Think of the bond as an investment that will pay the owner $1,800 every 6 months for 9 years, at which time it will pay $50,000. This is exactly the behavior of an annuity paired with an investment with future value $50,000 = 50,000, = 1,800, = 0.063 2 = 0.0315, = 1, = 2 × 9 = 18 The present value has contributions both from the investment and the annuity: 1 (1 + ) 1 1.0315 180 = (1 + ) + $53,055.66 = 50,000(1.0315) 18 + 1,800 0.0315 Technology: 50000*1.0315^(-18)+1800*(1-1.0315^(-18))/0.0315 54. This is similar to Exercise 53, but with = 50,000, = 1,800, = 0.06 2 = 0.03, = 2 × 8.5 = 17 1 (1 + ) 1 1.03 170 = (1 + ) + $53,949.84 = 50,000(1.03) 17 + 1,800 0.03 Technology: 50000*1.03^(-17)+1800*(1-1.03^(-17))/0.03

55. Here, = 50,000, = 54,000, = 1,800, = 2 × 8.5 = 17, and we are seeking the interest rate. Online Time Value of Money Utility:

The interest rate would have to be 5.99%.

56. Here, = 50,000, = 52,000, = 1,800, = 2 × 8.5 = 17, and we are seeking the interest rate.


Solutions Chapter 3 Review Online Time Value of Money Utility:

The interest rate would have to be 6.58%.


Solutions Chapter 3 Case Study Chapter 3 Case Study 1. The start of the sixth year is the critical time; if the Wongs can afford payments then, they will continue to be able to afford them throughout the loan as one can verify by glancing at the worksheet. To solve with Excel, use one of the three TVM utilities discussed in the book or, in Excel, use =RATE(12*25,-2271.09,343700.55)*12 This will return approximately 0.06267892, or 6.27% per year. Since this is 5% above the Fed discount rate, the latter would have to be 1.27%. 2. This is similar to Exercise 1, and we use =RATE(12*25,-2271.09,380000)*12 which returns approximately 0.0522, or 5.22%, meaning that the Fed rate would have to be 0.22%. 3. This is similar to Exercise 1, and we use =RATE(12*25,-2271.09,400640.49)*12 which returns approximately 0.0469, or 4.69%, meaning that the Fed rate would have to be negative, which is impossible; the Wongs could never make the payments regardless of the Fed rate in this case. 4. Adjusting the price in cell K3 gives affordable payments in the most expensive scenario for a $195,000 mortgage. Adding the $20,000 down payment, the Wongs could afford a $215,000 home. 5. Adjusting the price in cell K3 gives affordable payments in the most expensive scenario for a $170,000 mortgage (to the nearest $5,000). Adding the $20,000 down payment, the Wongs could afford a $190,000 home. 6. Of the three types of mortgage, the hybrid is the most affordable. and the first steep payment (in worstcase scenario 3) is the $4,402.22. 28% of their income first passes that level in year 23 (see the spreadsheet), meaning that they would have to wait until year 23 5 = 18, or 17 more years, before being able to afford such a home.


Solutions Section 4.1 Section 4.1

1. 2𝑥 − 𝑦 = 1 Solving for 𝑦 gives 𝑦 = 2𝑥 − 1, so the general solution is (𝑥, 𝑦) = (𝑥, 2𝑥 − 1); 𝑥 arbitrary. This is the solution parameterized by 𝑥. We get particular solutions by choosing values for 𝑥, such as 𝑥 = −1 ⇒ (𝑥, 𝑦) = (𝑥, 2𝑥 − 1) = (𝑥, 2(−1) − 1) = (−1, −3) 𝑥 = 0 ⇒ (𝑥, 𝑦) = (𝑥, 2𝑥 − 1) = (0, 2(0) − 1) = (0, −1) 𝑥 = 1 ⇒ (𝑥, 𝑦) = (𝑥, 2𝑥 − 1) = (1, 2(1) − 1) = (1, 1) 1 For the solution parameterized by 𝑦, solve the original equation for 𝑥 to get 𝑥 = (𝑦 + 1), so the general 2 solution parameterized by 𝑦 is (𝑥, 𝑦) = ! 12 (𝑦 + 1), 𝑦"; 𝑦 arbitrary

2. 𝑥 + 3𝑦 = 3

1 Solving for 𝑦 gives 𝑦 = − 𝑥 + 1, so the general solution is 3 1 (𝑥, 𝑦) = (𝑥, − 𝑥 + 1); 𝑥 arbitrary. 3 This is the solution parameterized by 𝑥. We get particular solutions by choosing values for 𝑥, such as 1 1 𝑥 = −3 ⇒ (𝑥, 𝑦) = (𝑥, − 𝑥 + 1) = (−3, − (−3) + 1) = (−3, 2) 3 3 1 1 𝑥 = 0 ⇒ (𝑥, 𝑦) = (𝑥, − 𝑥 + 1) = (0, − (0) + 1) = (0, 1) 3 3 1 1 𝑥 = 3 ⇒ (𝑥, 𝑦) = (𝑥, − 𝑥 + 1) = (3, − (3) + 1) = (3, 0) 3 3 For the solution parameterized by 𝑦, solve the original equation for 𝑥 to get 𝑥 = −3𝑦 + 3, so the general solution parameterized by 𝑦 is (𝑥, 𝑦) = (−3𝑦 + 3, 𝑦); 𝑦 arbitrary. 3. 3𝑥 + 4𝑦 = 2

3 1 Solving for 𝑦 gives 𝑦 = − 𝑥 + , so the general solution is 4 2 3 1 (𝑥, 𝑦) = (𝑥, − 𝑥 + ); 𝑥 arbitrary. 4 2 This is the solution parameterized by 𝑥. We get particular solutions by choosing values for 𝑥, such as 3 1 3 1 𝑥 = −2 ⇒ (𝑥, 𝑦) = (𝑥, − 𝑥 + ) = (−2, − (−2) + ) = (−3, 2) 4 2 4 2 3 1 3 1 1 𝑥 = 0 ⇒ (𝑥, 𝑦) = (𝑥, − 𝑥 + ) = (0, − (0) + ) = (0, ) 4 2 4 2 2 3 1 3 1 𝑥 = 2 ⇒ (𝑥, 𝑦) = (𝑥, − 𝑥 + ) = (2, − (2) + ) = (2, −1) 4 2 4 2 1 For the solution parameterized by 𝑦, solve the original equation for 𝑥 to get 𝑥 = (−4𝑦 + 2), so the 3 general solution parameterized by 𝑦 is 1 (𝑥, 𝑦) = ( (−4𝑦 + 2), 𝑦); 𝑦 arbitrary. 3 4. 4𝑥 − 3𝑦 = 6

1 Solving for 𝑦 gives 𝑦 = (4𝑥 − 6), so the general solution is 3 1 (𝑥, 𝑦) = (𝑥, (4𝑥 − 6)); 𝑥 arbitrary. 3


Solutions Section 4.1 This is the solution parameterized by 𝑥. We get particular solutions by choosing values for 𝑥, such as 1 1 𝑥 = −3 ⇒ (𝑥, 𝑦) = (𝑥, (4𝑥 − 6)) = (−3, (4(−3) − 6)) = (−3, −6) 3 3 1 1 𝑥 = 0 ⇒ (𝑥, 𝑦) = (𝑥, (4𝑥 − 6)) = (0, (4(0) − 6)) = (0, −2) 3 3 1 1 𝑥 = 3 ⇒ (𝑥, 𝑦) = (𝑥, (4𝑥 − 6)) = (3, (4(3) − 6)) = (3, 2) 3 3 1 For the solution parameterized by 𝑦, solve the original equation for 𝑥 to get 𝑥 = (3𝑦 + 6), so the general 4 solution parameterized by 𝑦 is 1 (𝑥, 𝑦) = ( (3𝑦 + 6), 𝑦); 𝑦 arbitrary. 4 5. 4𝑥 = −5 We cannot solve for 𝑦, so the general solution cannot be parameterized by 𝑥.

5 For the solution parameterized by 𝑦, solve the original equation for 𝑥 to get 𝑥 = − , so the general 4 solution parameterized by 𝑦 is 5 (𝑥, 𝑦) = (− , 𝑦); 𝑦 arbitrary. 4 We get particular solutions by choosing values for 𝑦 : (−5∕4, −1), (−5∕4, 0), (−5∕4, 1). 6. −3𝑦 = 2

2 For the solution parameterized by 𝑥, solve the original equation for 𝑦 to get 𝑦 = − , so the general 3 solution parameterized by 𝑥 is 2 (𝑥, 𝑦) = (𝑥, − ); 𝑥 arbitrary. 3 We get particular solutions by choosing values for 𝑥 : (−1, −2∕3), (0, −2∕3), (1, −2∕3); We cannot solve for 𝑥, so the general solution cannot be parameterized by 𝑦. 7. 𝑥 − 𝑦 = 0 𝑥+𝑦=4 Adding gives 2𝑥 = 4 ⇒ 𝑥 = 2. Substituting 𝑥 = 2 in the first equation: 2 − 𝑦 = 0 ⇒ 𝑦 = 2. Solution: (2, 2) Graph: 𝑦 = 𝑥; 𝑦 = 4 − 𝑥

8. 𝑥 − 𝑦 = 0 𝑥 + 𝑦 = −6 Adding gives 2𝑥 = −6 ⇒ 𝑥 = −3. Substituting 𝑥 = −3 in the first equation: −3 − 𝑦 = 0 ⇒ 𝑦 = −3. Solution: (−3, −3) Graph: 𝑦 = 𝑥; 𝑦 = −6 − 𝑥


9. 𝑥 + 𝑦 = 4 𝑥−𝑦=2 Adding gives 2𝑥 = 6 ⇒ 𝑥 = 3. Substituting 𝑥 = 3 in the first equation: 3 + 𝑦 = 4 ⇒ 𝑦 = 4 − 3 = 1. Solution: (3, 1) Graph: 𝑦 = 4 − 𝑥; 𝑦 = 𝑥 − 2

10. 2𝑥 + 𝑦 = 2 −2𝑥 + 𝑦 = 2 Adding gives 2𝑦 = 4 ⇒ 𝑦 = 2. Substituting 𝑦 = 2 in the first equation: 2𝑥 + 2 = 2 ⇒ 𝑥 = 0. Solution: (0, 2) Graph: 𝑦 = 2 − 2𝑥; 𝑦 = 2𝑥 + 2

Solutions Section 4.1

11. 3𝑥 − 2𝑦 = 6 2𝑥 − 3𝑦 = −6 Multiply the first equation by 2 and the second by −3 : 6𝑥 − 4𝑦 = 12 −6𝑥 + 9𝑦 = 18. Adding gives 5𝑦 = 30 ⇒ 𝑦 = 6. Substituting 𝑦 = 6 in the first equation: 3𝑥 − 12 = 6 ⇒ 3𝑥 = 18 ⇒ 𝑥 = 6. Solution: (6, 6) 3 2 Graph: 𝑦 = 𝑥 − 3; 𝑦 = 𝑥 + 2 2 3

12. 2𝑥 + 3𝑦 = 5 3𝑥 + 2𝑦 = 5 Multiply the first equation by 2 and the second by −3 : 4𝑥 + 6𝑦 = 10 −9𝑥 − 6𝑦 = −15. Adding gives −5𝑥 = −5 ⇒ 𝑥 = 1. Substituting 𝑥 = 1 in the first equation gives 2 + 3𝑦 = 5 ⇒ 𝑦 = 1. Solution: (1, 1) 2 5 3 5 Graph: 𝑦 = − 𝑥 + ; 𝑦 = − 𝑥 + 3 3 2 2


13. 0.5𝑥 + 0.1𝑦 = 0.7 0.2𝑥 − 0.2𝑦 = 0.6 Multiply both equations by 10: 5𝑥 + 𝑦 = 7 2𝑥 − 2𝑦 = 6. Divide the second equation by 2: 5𝑥 + 𝑦 = 7 𝑥 − 𝑦 = 3. Adding gives 10 5 6𝑥 = 10 ⇒ 𝑥 = = . 6 3 5 Substituting 𝑥 = in 𝑥 − 𝑦 = 3 gives 3 5 5 4 −𝑦=3⇒𝑦= −3=− . 3 3 3 5 4 Solution: ( , − ) 3 3 Graph: 𝑦 = −5𝑥 + 7; 𝑦 = 𝑥 − 3

14. −0.3𝑥 + 0.5𝑦 = 0.1 0.1𝑥 − 0.1𝑦 = 0.4 Multiply both equations by 10 : 𝑥 − 𝑦 = 4. −3𝑥 + 5𝑦 = 1 Multiply the second equation by 3: 3𝑥 − 3𝑦 = 12. −3𝑥 + 5𝑦 = 1 Adding gives 13 2𝑦 = 13 ⇒ 𝑦 = = 6.5. 2 Substituting 𝑦 = 6.5 in 𝑥 − 𝑦 = 4 gives 𝑥 − 6.5 = 4 ⇒ 𝑥 = 10.5. Solution: (10.5, 6.5) Graph: 𝑦 = 0.6𝑥 + 0.2; 𝑦 = 𝑥 − 4

Solutions Section 4.1

𝑥 𝑦 𝑥 − =1 + 𝑦 = −2 3 2 4 Multiply the first equation by 6 and the second by 4: 2𝑥 − 3𝑦 = 6 𝑥 + 4𝑦 = −8. Multiply the second equation by −2 : 2𝑥 − 3𝑦 = 6 −2𝑥 − 8𝑦 = 16. Adding gives 22 −11𝑦 = 22 ⇒ 𝑦 = = −2. −11 Substituting 𝑦 = −2 into 𝑥 + 4𝑦 = −8 gives 𝑥 + 4(−2) = −8 ⇒ 𝑥 − 8 = −8 ⇒ 𝑥 = 0. Solution: (0, −2) Graph: 𝑦 = (2∕3)𝑥 − 2; 𝑦 = −𝑥∕4 − 2 15.

2𝑥 𝑦 1 𝑥 3 + =− −𝑦=− 3 2 6 4 4 Multiply the first equation by 6 and the second by 4: 𝑥 − 4𝑦 = −3. −4𝑥 + 3𝑦 = −1 Multiply the second equation by 4: −4𝑥 + 3𝑦 = −1 ⇒ 4𝑥 − 16𝑦 = −12. Adding gives −13𝑦 = −13 ⇒ 𝑦 = 1. Substituting into 𝑥 − 4𝑦 = −3 gives 𝑥 − 4 = −3 ⇒ 𝑥 = 1. Solution: (1, 1) Graph: 𝑦 = (4∕3)𝑥 − 1∕3; 𝑦 = (1∕4)𝑥 + 3∕4 16. −


3𝑦 1 =− 2 2 Multiply the second equation by 2: 2𝑥 + 3𝑦 = 1 −2𝑥 − 3𝑦 = −1. Adding gives 0 = 0, indicating that the system is redundant: The graphs are the same, so there are infinitely many solutions. We obtain the solutions by solving either equation for 𝑦 (or 𝑥). 2𝑥 + 3𝑦 = 1 ⇒ 3𝑦 = −2𝑥 + 1 ⇒ 𝑦 = (−2𝑥 + 1)∕3 Solution: (𝑥, (−2𝑥 + 1)∕3); 𝑥 arbitrary Graph: 𝑦 = (−2𝑥 + 1)∕3

18. 2𝑥 − 3𝑦 = 1 6𝑥 − 9𝑦 = 3 Divide the second equation by −3 : 2𝑥 − 3𝑦 = 1 −2𝑥 + 3𝑦 = −1. Adding gives 0 = 0, indicating that the system is redundant: There are infinitely many solutions, which we can obtain by solving either equation for 𝑦 (or 𝑥): 2𝑥 − 3𝑦 = 1 ⇒ 3𝑦 = 2𝑥 − 1 ⇒ 𝑦 = (2𝑥 − 1)∕3. Solution: (𝑥, (2𝑥 − 1)∕3); 𝑥 arbitrary Graph: 𝑦 = (2𝑥 − 1)∕3

3𝑦 1 =− 2 2 Multiply the second equation by 2: 2𝑥 + 3𝑦 = 2 −2𝑥 − 3𝑦 = −1. Adding gives 0 = 1, indicating that the system is inconsistent. There is no solution; the lines are parallel. Graph: 𝑦 = (−2𝑥 + 2)∕3; 𝑦 = (−2𝑥 + 1)∕3

20. 2𝑥 − 3𝑦 = 2 6𝑥 − 9𝑦 = 3 Divide the second equation by −3 : 2𝑥 − 3𝑦 = 2 −2𝑥 + 3𝑦 = −1. Adding gives 0 = 1, indicating that the system is inconsistent. There is no solution; the lines are parallel. Graph: 𝑦 = (2𝑥 − 2)∕3; 𝑦 = (2𝑥 − 1)∕3

17. 2𝑥 + 3𝑦 = 1

19. 2𝑥 + 3𝑦 = 2

−𝑥 −

−𝑥 −

Solutions Section 4.1

Parallel lines; no solution Parallel lines; no solution


21. 2𝑥 + 8𝑦 = 10 𝑥+𝑦=5 To graph these, solve for 𝑦 : 𝑦 = (−2𝑥 + 10)∕8 𝑦 = −𝑥 + 5. Graph, with vertical and horizontal scales of 0.1:

22. 2𝑥 − 𝑦 = 3 𝑥 + 3𝑦 = 5 To graph these, solve for 𝑦 : 𝑦 = 2𝑥 − 3 𝑦 = (−𝑥 + 5)∕3. Graph, with vertical and horizontal scales of 0.1:

The grid point closest to the intersection of the lines gives the approximate solution (in this case the exact solution). Solution: (5, 0)

The grid point closest to the intersection of the lines gives the approximate solution (in this case the exact solution). Solution: (2, 1)

23. 3.1𝑥 − 4.5𝑦 = 6 4.5𝑥 + 1.1𝑦 = 0 To graph these, solve for 𝑦 : 𝑦 = (3.1𝑥 − 6)∕4.5 𝑦 = −4.5𝑥∕1.1. Graph, with vertical and horizontal scales of 0.1:

24. 0.2𝑥 + 4.5𝑦 = 1 1.5𝑥 + 1.1𝑦 = 2 To graph these, solve for 𝑦 : 𝑦 = (−0.2𝑥 + 1)∕4.5 𝑦 = (−1.5𝑥 + 2)∕1.1. Graph, with vertical and horizontal scales of 0.1:

Solutions Section 4.1

The grid point closest to the intersection of the lines gives the approximate solution. Approximate solution: (0.3, −1.1) 25. 10.2𝑥 + 14𝑦 = 213 4.5𝑥 + 1.1𝑦 = 448 To graph these, solve for 𝑦 : 𝑦 = (−10.2𝑥 + 213)∕14 𝑦 = (−4.5𝑥 + 448)∕1.1. Graph, with vertical and horizontal scales of 0.1:

The grid point closest to the intersection of the lines gives the approximate solution. Approximate solution: (116.6, −69.7)

The grid point closest to the intersection of the lines gives the approximate solution. Approximate solution: (1.2, 0.2) 26. 100𝑥 + 4.5𝑦 = 540 1.05𝑥 + 1.1𝑦 = 0 To graph these, solve for 𝑦 : 𝑦 = −100𝑥 + 540)∕4.5 𝑦 = −1.05𝑥∕1.1. Graph, with vertical and horizontal scales of 0.1:

The grid point closest to the intersection of the lines gives the approximate solution. Approximate solution: (5.6, −5.4)


27. Line through (0, 1) and (4.2, 2) : 𝑦 − 𝑦1 2−1 1 Point: (0, 1) Slope: 𝑚 = 2 Intercept: 𝑏 = 1 = = 𝑥2 − 𝑥1 4.2 − 0 4.2 1 Thus, the equation is 𝑦 = 𝑚𝑥 + 𝑏 = 𝑥 + 1. 4.2 Line through (2.1, 3) and (5.2, 0) : 𝑦 − 𝑦1 0−3 3 Point: (5.2, 0) Slope: 𝑚 = 2 Intercept: = =− 𝑥2 − 𝑥1 5.2 − 2.1 3.1 3 15.6 𝑏 = 𝑦1 − 𝑚𝑥1 = 0 − (− )5.2 = 3.1 3.1 3 15.6 Thus, the equation is 𝑦 = 𝑚𝑥 + 𝑏 = − 𝑥+ . 3.1 3.1 Solutions Section 4.1

(Given)

The grid point closest to the intersection of the lines gives the approximate solution. Approximate solution: (3.3, 1.8) 28. Line through (2.1, 3) and (4, 2) : 𝑦 − 𝑦1 2−3 1 Point: (2.1, 3) Slope: 𝑚 = 2 = =− 𝑥2 − 𝑥1 4 − 2.1 1.9 1 7.8 𝑏 = 𝑦1 − 𝑚𝑥1 = 3 − (− )2.1 = 1.9 1.9 1 7.8 Thus, the equation is 𝑦 = 𝑚𝑥 + 𝑏 = − 𝑥+ . 1.9 1.9 Line through (3.2, 2) and (5.1, 3) : 𝑦 − 𝑦1 3−2 1 Point: (3.2, 2) Slope: 𝑚 = 2 = = 𝑥2 − 𝑥1 5.1 − 3.2 1.9 1 0.6 𝑏 = 𝑦1 − 𝑚𝑥1 = 2 − ( )3.2 = 1.9 1.9 1 0.6 Thus, the equation is 𝑦 = 𝑚𝑥 + 𝑏 = 𝑥+ . 1.9 1.9

Intercept:

Intercept:

The grid point closest to the intersection of the lines gives the approximate solution. Approximate solution: (3.6, 2.2) 29. Line through (0, 0) and (5.5, 3) : 𝑦 − 𝑦1 3−0 3 Point: (0, 0) Slope: 𝑚 = 2 = = 𝑥2 − 𝑥1 5.5 − 0 5.5 3 Thus, the equation is 𝑦 = 𝑚𝑥 + 𝑏 = 𝑥. 5.5

Intercept: 0 (Given)


Line through (5, 0) and (0, 6) : 𝑦 − 𝑦1 6−0 6 Point: (0, 6) Slope: 𝑚 = 2 = =− 𝑥2 − 𝑥1 0 − 5 5 6 Thus, the equation is 𝑦 = 𝑚𝑥 + 𝑏 = − 𝑥 + 6. 5

Solutions Section 4.1 Intercept: 6 (Given)

The grid point closest to the intersection of the lines gives the approximate solution. Approximate solution: (3.4, 1.9) 30. Line through (4.3, 0) and (0, 5) : 𝑦 − 𝑦1 5−0 5 Point: (0, 5) Slope: 𝑚 = 2 Intercept: 5 (Given) = =− 𝑥2 − 𝑥1 0 − 4.3 4.3 5 Thus, the equation is 𝑦 = 𝑚𝑥 + 𝑏 = − 𝑥 + 5. 4.3 Line through (2.1, 2.2) and (5.2, 1) : 𝑦 − 𝑦1 1 − 2.2 1.2 Point: (5.2, 1) Slope: 𝑚 = 2 = =− 𝑥2 − 𝑥1 5.2 − 2.1 3.1 1.2 6.24 9.34 Intercept: 𝑏 = 𝑦1 − 𝑚𝑥1 = 1 − (− )5.2 = 1 + = 3.1 3.1 3.1 1.2 9.34 Thus, the equation is 𝑦 = 𝑚𝑥 + 𝑏 = − 𝑥+ . 3.1 3.1

The grid point closest to the intersection of the lines gives the approximate solution. Approximate solution: (2.6, 2.0)

31. Let 𝑥 = the number of soccer fans, and let 𝑦 = the number of football fans. Reword the given statement using the phrases "the number of soccer fans," which is 𝑥, and "the number of football fans," which is 𝑦 : The number of soccer fans is twice the number of football fans. Translate the phrases into symbols: 𝑥 = 2𝑦. In standard form, this becomes 𝑥 − 2𝑦 = 0. 32. Let 𝑥 = the number of hockey players, and let 𝑦 = the number of lacrosse players. Reword the given statement using the phrases "the number of hockey players," which is 𝑥, and "the number of lacrosse players," which is 𝑦 : The number of hockey players is 90% of the number of lacrosse players. (No rewording necessary in this case.) Translate the phrases into symbols:


90 𝑦 = 0.90𝑦. 100 In standard form, this becomes 𝑥 − 0.90𝑦 = 0. 𝑥=

Solutions Section 4.1

33. Let 𝑥 = the number of new clients, and let 𝑦 = the number of old clients. Reword the given statement using the phrases "the number of new clients," which is 𝑥, and "the number of old clients," which is 𝑦 : The number of new clients is 110% the number of old clients. (No rewording necessary in this case.) Translate the phrases into symbols: 110 𝑥= 𝑦 = 1.10𝑦. 100 In standard form, this becomes 𝑥 − 1.10𝑦 = 0.

34. Let 𝑥 = the number of bondholders, and let 𝑦 = the number of stockholders. Reword the given statement using the phrases "the number of bondholders," which is 𝑥, and "the number of stockholders," which is 𝑦 : The number of bondholders is half the number of stockholders. Translate the phrases into symbols: 1 𝑥 = 𝑦. 2 We can rewrite this equation in standard form in various ways, such as 𝑥 − 12 𝑦 = 0 or 2𝑥 − 𝑦 = 0. 35. Let 𝑥 = the number of gas giants, and let 𝑦 = the number of rocky planets. We are given two pieces of information: 1. There are three times as many gas giants as rocky planets. 2. Their total is 12. Thus, we expect to obtain two equations. Reword the given information using the phrases "the number of gas giants," which is 𝑥, and "the number of rocky planets," which is 𝑦 : 1. The number of gas giants is three times the number of rocky planets. 2. The number of gas giants and the number of rocky planets add up to 12. Translate the phrases into symbols: 1. 𝑥 = 3𝑦, or 𝑥 − 3𝑦 = 0 2. 𝑥 + 𝑦 = 12

36. Let 𝑥 = the number of planets that support life, and let 𝑦 = the number of lifeless planets. We are given two pieces of information: 1. Their total is 25. 2. There are four times as many planets that support life as rocky planets. Thus, we expect to obtain two equations. Reword the given information using the phrases "the number of planets that support life," which is 𝑥, and "the number of lifeless planets," which is 𝑦 : 1. The number of planets that support life and the number of lifeless planets add up to 25 2. The number of planets that support life is four times the number of lifeless planets. Translate the phrases into symbols: 1. 𝑥 + 𝑦 = 25 2. 𝑥 = 4𝑦, or 𝑥 − 4𝑦 = 0 37. Let 𝑥 = the number of ordinary shares, and let 𝑦 = the number of preferred shares. We are given two pieces of information: 1. 20% of the total are preferred shares. 2. There are 15 more ordinary shares than preferred shares. Reword the given information using the phrases "the number of ordinary shares," which is 𝑥, and "the


Solutions Section 4.1 number of preferred shares," which is 𝑦 : 1. The number of preferred shares is 20% of the total number (of ordinary and preferred shares). 2. The number of ordinary shares is 15 more than the number of preferred shares. Translate the phrases into symbols: 20 1 1. 𝑦 = (𝑥 + 𝑦) = (𝑥 + 𝑦) 100 5 Simplify by multiplying both sides by 5: 5𝑦 = 𝑥 + 𝑦 ⇒ 𝑥 = 4𝑦, or 𝑥 − 4𝑦 = 0 2. 𝑥 = 15 + 𝑦 or 𝑥 − 𝑦 = 15

38. Let 𝑥 = the number of paid-up customers, and let 𝑦 = the number customers who owe money. We are given two pieces of information: 1. 75% of the total customer base is paid up. 2. There are 14,000 more paid-up customers than customers who owe money. Reword the given information using the phrases "the number of paid-up customers," which is 𝑥, and "the number of customers who still owe money," which is 𝑦 : 1. The number of paid-up customers is 75% of the total number (of paid-up customers and customers who owe money). 2. The number of paid-up customers is 14,000 more than the number of customers who owe money. Translate the phrases into symbols: 75 3 1. 𝑥 = (𝑥 + 𝑦) = (𝑥 + 𝑦) 100 4 Simplify by multiplying both sides by 4: 4𝑥 = 3(𝑥 + 𝑦) ⇒ 𝑥 = 3𝑦, or 𝑥 − 3𝑦 = 0 2. 𝑥 = 14,000 + 𝑦, or 𝑥 − 𝑦 = 14,000 39. Unknowns: 𝑥 = the number of quarts of Creamy Vanilla; 𝑦 = the number of quarts of Continental Mocha Arrange the given information in a table with unknowns across the top: Vanilla (𝑥) Mocha (𝑦) Available

Eggs

2

1

500

Cream

3

3

900

We can now set up an equation for each of the items listed on the left: Eggs: Cream:

2𝑥 + 𝑦 = 500 3𝑥 + 3𝑦 = 900.

Multiply the first equation by −1 and divide the second by 3: 𝑥 + 𝑦 = 300. −2𝑥 − 𝑦 = −500 Adding gives −𝑥 = −200 ⇒ 𝑥 = 200 quarts of vanilla. Substituting 𝑥 = 200 in the first equation gives 2(200) + 𝑦 = 500 ⇒ 400 + 𝑦 = 500 ⇒ 𝑦 = 500 − 400 = 100 quarts of mocha. Solution: Make 200 quarts of vanilla and 100 quarts of mocha.

40. Unknowns: 𝑥 = the number of sections of Finite Math; 𝑦 = the number of sections of Applied Calculus Since there are a total of 110 sections, 𝑥 + 𝑦 = 110. Information about enrollment gives 60𝑥 + 50𝑦 = 6, 000 . Multiply the first equation by 5 and divide the second equation by −10 : 5𝑥 + 5𝑦 = 550 −6𝑥 − 5𝑦 = −600.


Solutions Section 4.1

Adding gives −𝑥 = −50 ⇒ 𝑥 = 50 sections of Finite Math. Substituting 𝑥 = 50 in the first equation gives 50 + 𝑦 = 110 ⇒ 𝑦 = 60 sections of Applied Calculus. Solution: Offer 50 sections of Finite Math, 60 sections of Applied Calculus.

41. Unknowns: 𝑥 = the number of servings of Multigrain Cereal; 𝑦 = the number of servings of 2nd Foods Mango Arrange the given information in a table with unknowns across the top: Cereal (𝑥) Mango (𝑦) Desired

Calories

60

80

220

Carbs (g)

11

19

49

We can now set up an equation for each of the items listed on the left: 60𝑥 + 80𝑦 = 220 11𝑥 + 19𝑦 = 49.

Calories: Carbs:

Divide the first equation by 20: 3𝑥 + 4𝑦 = 11 11𝑥 + 19𝑦 = 49. Multiply the first equation by −11 and the second by 3: 33𝑥 + 57𝑦 = 147. −33𝑥 − 44𝑦 = −121 Adding gives 13𝑦 = 26 ⇒ 𝑦 = 2 servings of 2nd Foods Mango. Substituting 𝑦 = 2 in the equation 3𝑥 + 4𝑦 = 11 gives 3𝑥 + 4(2) = 11 ⇒ 3𝑥 = 3 ⇒ 𝑥 = 1 serving of Multigrain Cereal. Solution: Use 1 serving of Multigrain Cereal and 2 servings of 2nd Foods Mango

42. Unknowns: 𝑥 = the number of ounces of cereal; 𝑦 = the number of ounces of fruit Arrange the given information in a table with unknowns across the top: Cereal (𝑥) Fruit (𝑦) Desired

Protein

0.5

0.2

1

Iron

1

2

5

We can now set up an equation for each of the items listed on the left: Cereal: Fruit:

0.5𝑥 + 0.2𝑦 = 1 𝑥 + 2𝑦 = 5.

Multiply the first equation by 10: 5𝑥 + 2𝑦 = 10 𝑥 + 2𝑦 = 5. Subtracting gives 4𝑥 = 5 ⇒ 𝑥 = 1.25 oz of cereal. Substituting this value of x into the second equation gives 1.25 + 2𝑦 = 5 ⇒ 2𝑦 = 3.75 ⇒ 𝑦 = 1.875 oz of fruit. Solution: Mix 1.25 oz of cereal and 1.875 oz of fruit. 43. a. One-third of the U.S. RDA for protein is 20 g. Unknowns: 𝑥 = the number of servings of Vegetarian Baked Beans; 𝑦 = the number of slices of whole wheat bread Arrange the given information in a table with unknowns across the top:


Beans (𝑥) Bread (𝑦) Desired

Solutions Section 4.1

Protein (g)

7

3

20

Carbs (g)

30

12

84

We can now set up an equation for each of the items listed on the left: Protein: Carbs:

7𝑥 + 3𝑦 = 20 30𝑥 + 12𝑦 = 84.

Protein: Carbs:

7𝑥 + 3𝑦 = 30 30𝑥 + 12𝑦 = 84.

Multiply the first equation by −4: 30𝑥 + 12𝑦 = 84. −28𝑥 − 12𝑦 = −80 Adding gives 2𝑥 = 4 ⇒ 𝑥 = 2 servings of beans. Substituting 𝑥 = 2 in the equation 7𝑥 + 3𝑦 = 20 gives 7(2) + 3𝑦 = 20 ⇒ 14 + 3𝑦 = 20 ⇒ 3𝑦 = 6 ⇒ 𝑦 = 2 slices of bread. Solution: Prepare 2 servings of Vegetarian Baked Beans and 2 slices of whole wheat bread. b. Increasing the protein level to half the RDA gives:

Multiply the first equation by −4: 30𝑥 + 12𝑦 = 84. −28𝑥 − 12𝑦 = −120 Adding gives 2𝑥 = −36 ⇒ 𝑥 = −18 servings of beans. Since it is impossible to prepare a negative number of servings of beans, we conclude that it is not possible to prepare such a meal.

44. a. 𝑥 = the number of servings of Vegetarian Baked Beans; 𝑦 = the number of slices of sourdough bread Arrange the given information in a table with unknowns across the top: Pork & Beans (𝑥) Bread (𝑦) Desired

Protein

7

2

12

Carbs.

30

8

50

We can now set up an equation for each of the items listed on the left: Protein: Carbs:

7𝑥 + 2𝑦 = 12 30𝑥 + 8𝑦 = 50.

Multiply the first equation by −4 : 30𝑥 + 8𝑦 = 50. −28𝑥 − 8𝑦 = −48 Adding gives 2𝑥 = 2 ⇒ 𝑥 = 1 serving of beans. Substituting 𝑥 = 1 in the equation 7𝑥 + 2𝑦 = 12 gives 7(1) + 2𝑦 = 12 ⇒ 2𝑦 = 5 ⇒ 𝑦 = 52 slices of bread.

Solution: Prepare 1 serving of beans and 5/2 slices of bread. b. The new equations are Protein: Carbs:

7𝑥 + 2𝑦 = 12 30𝑥 + 8𝑦 = 48.

Multiply the first equation by −4 :


Solutions Section 4.1 30𝑥 + 8𝑦 = 48.

−28𝑥 − 8𝑦 = −48 Adding gives 𝑥 = 0. Substituting in the first equation gives 𝑦 = 6. Solution: Use no beans and 6 slices of bread.

45. Unknowns: 𝑥 = number of servings of Designer Whey; 𝑦 = number of servings of Muscle Milk Arrange the given information in a table with unknowns across the top: Designer Whey Muscle Milk Desired Protein

20

16

152

Carbs

6

10

56

Cost

$0.80

$0.90

We can now set up an equation for each nutrient listed on the left: 20𝑥 + 16𝑦 = 152 6𝑥 + 10𝑦 = 56.

Protein: Carbs:

Multiply the first equation by 5, the second by −8, and add: 100𝑥 + 80𝑦 = 760 −48𝑥 − 80𝑦 = −448 52𝑥 = 312 ⇒ 𝑥 = 6 servings of Designer Whey. Substitute into the second equation to obtain 36 + 10𝑦 = 56 ⇒ 10𝑦 = 20 ⇒ 𝑦 = 2 servings of Muscle Milk. Cost = 6(0.80) + 2(0.90) = $6.60. Solution: Mix 6 servings of Designer Whey and 2 servings of Muscle Milk for a cost of $6.60.

46. Unknowns: 𝑥 = number of servings of Muscle Milk; 𝑦 = number of servings of 100% Whey Gold Standard Arrange the given information in a table with unknowns across the top: Muscle Milk 100% Whey Gold Standard Desired Protein

16

24

320

Sodium

80

210

2,500

Cost

$0.90

$1.00

We can now set up an equation for each nutrient listed on the left: Protein: Sodium:

16𝑥 + 24𝑦 = 320 80𝑥 + 210𝑦 = 2, 500.

Divide the first equation by 8 and the second equation by 10 to obtain 2𝑥 + 3𝑦 = 40 8𝑥 + 21𝑦 = 250. Take 7 times the first − second: 6𝑥 = 30 ⇒ 𝑥 = 5 servings of Muscle Milk. Substitute into the first equation to obtain 80 + 24𝑦 = 320 ⇒ 𝑦 = 10 servings of 100% Whey Gold Standard. Cost = 5(0.90) + 10(1.00) = $14.50. Solution: Mix 5 servings of Muscle Milk and 10 servings of 100% Whey Gold Standard for a cost of $14.50. 47. We first determine how many servings of each kind of supplement it contains: Unknowns: 𝑥 = number of servings of Designer Whey; 𝑦 = number of servings of 100% Whey Gold


Solutions Section 4.1 Standard. Arrange the given information in a table with unknowns across the top: Designer Whey 100% Whey Gold Standard Total Protein

20

24

488

Cost

0.80

1.00

20

Carbs

6

3

We can now set up an equation for protein content and one for cost: 20𝑥 + 24𝑦 = 488 0.80𝑥 + 1.00𝑦 = 20.

Protein: Cost:

Divide the first equation by 4 and multiply the second by 5: 5𝑥 + 6𝑦 = 122 4𝑥 + 5𝑦 = 100. 5 times the first minus 6 times the second gives 𝑥 = 10 servings of Designer Whey. Substituting in the first equation gives 24𝑦 = 288 ⇒ 𝑦 = 12 servings of 100% Whey Gold Standard. From the table, Total amount of carbohydrates = 10(6) + 12(3) = 96 g. 48. We first determine how many servings of each kind of supplement it contains: Unknowns: 𝑥 = number of servings of Designer Whey; 𝑦 = number of servings of Muscle Milk. Arrange the given information in a table with unknowns across the top: Designer Whey Muscle Milk Total Carbs

6

10

190

Cost

0.80

0.90

21

Protein

20

16

We can now set up an equation for carbohydrate content and for cost: 6𝑥 + 10𝑦 = 190 0.80𝑥 + 0.90𝑦 = 21.

Carbs: Cost:

Divide the first equation by 2 and multiply the second by 10: 3𝑥 + 5𝑦 = 95 8𝑥 + 9𝑦 = 210. 9 times the first minus 5 times the second gives −13𝑥 = −195 ⇒ 𝑥 = 15 servings of Designer Whey. Substituting in the first gives 𝑦 = 10 servings of Muscle Milk. From the table, Total supply of protein = 15(20) + 10(16) = 460 g.

49. Unknowns: 𝑥 = number of shares of TWTR; 𝑦 = number of shares of MSFT Arrange the given information in a table with unknowns across the top: TWTR (𝑥) MSFT (𝑦) Total

Sep 1

66

302

43,400

Sep 30

60

282

40,200

We can now set up equations for the start and end of September: Sep 1:

66𝑥 + 302𝑦 = 43,400


60𝑥 + 282𝑦 = 40,200.

Solutions Section 4.1

Sep 30:

Divide the first equation by 2 and the second by 6: 33𝑥 + 151𝑦 = 21,700 10𝑥 + 47𝑦 = 6,700. Multiply the first by −10 and the second by 33: 330𝑥 + 1,551𝑦 = 221,100. −330𝑥 − 1,510𝑦 = −217,000 Adding gives 41𝑦 = 4,100 ⇒ 𝑦 = 100 shares of MSFT. Substituting 𝑦 = 100 into 10𝑥 + 47𝑦 = 6,700 gives 10𝑥 + 47(100) = 6,700 ⇒ 10𝑥 = 2,000 ⇒ 𝑥 = 200 shares of TWTR. Solution: You bought 200 shares of TWTR and 100 shares of MSFT.

50. Unknowns: 𝑥 = number of shares of HES; 𝑦 = number of shares of XOM Arrange the given information in a table with unknowns across the top: HES (𝑥) XOM (𝑦) Total

Jul 1

88

64

18,400

Sep 30

78

60

16,800

We can now set up equations for the start of November and end of January: Jul 1: Sep 30:

88𝑥 + 64𝑦 = 18,400 78𝑥 + 60𝑦 = 16,800.

Divide the first equation by 8 and the second by 6 : 11𝑥 + 8𝑦 = 2,300 13𝑥 + 10𝑦 = 2,800. Multiply the first by 5 and the second by −4: 55𝑥 + 40𝑦 = 11,500 −52𝑥 − 40𝑦 = −11,200. Adding gives 3𝑥 = 300 ⇒ 𝑥 = 100 shares of HES. Substituting 𝑥 = 100 into 78𝑥 + 60𝑦 = 16,800 gives 78(100) + 60𝑦 = 16,800 ⇒ 60𝑦 = 9,000 ⇒ 𝑦 = 150 shares of XOM. Solution: You bought 100 shares of HES and 150 shares of XOM.

51. Unknowns: 𝑥 = number of shares of TD; 𝑦 = number of shares of CNA The total investment was $25,800 ⇒ 66𝑥 + 42𝑦 = 25,800. You earned $969 in dividends: TD dividend = 4% of 66𝑥 invested = 0.04(66𝑥) = 2.64𝑥 CNA dividend = 3.5% of 42𝑦 invested = 0.035(42𝑦) = 1.47𝑦. Thus, 2.64𝑥 + 1.47𝑦 = 969. We therefore have the following system: 66𝑥 + 42𝑦 = 25,800 2.64𝑥 + 1.47𝑦 = 969. Divide the first equation by 6 and multiply the second equation by 100/3: 11𝑥 + 7𝑦 = 4,300 88𝑥 + 49𝑦 = 32,300. Multiply the first equation by 8 and subtract: 7𝑦 = 2,100 ⇒ 𝑦 = 300 shares of CNA. Substituting 𝑦 = 300 in 11𝑥 + 7𝑦 = 4,300 gives 11𝑥 + 7(300) = 4,300 ⇒ 11𝑥 = 2,200 ⇒ 𝑥 = 200 shares of TD. Solution: You purchased 200 shares of TD and 300 shares of CNA.

52. Unknowns: 𝑥 = number of shares of PAA; 𝑦 = number of shares of CVX The total investment was $30,000 ⇒ 10𝑥 + 100𝑦 = 30,000. You earned $1,700 in dividends:


Solutions Section 4.1 PAA dividend = 7% of 10𝑥 invested = 0.07(10𝑥) = 0.7𝑥 CVX dividend = 5% of 100𝑦 invested = 0.05(100𝑦) = 5𝑦. Thus, 0.7𝑥 + 5𝑦 = 1,700. We therefore have the following system: 10𝑥 + 100𝑦 = 30,000 0.7𝑥 + 5𝑦 = 1,700. Divide the first equation by 2 and multiply the second by 10: 5𝑥 + 50𝑦 = 15,000 7𝑥 + 50𝑦 = 17,000. Subtracting the first from the second gives 2𝑥 = 2,000 ⇒ 𝑥 = 1,000 shares of PAA. Substituting 𝑥 = 1,000 in 10𝑥 + 100𝑦 = 30,000 gives 10,000 + 100𝑦 = 30,000 ⇒ 100𝑦 = 20,000 ⇒ 𝑦 = 200 shares of CVX. Solution: You purchased 1,000 shares of PAA and 200 shares of CVX.

53. Unknowns: 𝑥 = number of members voting in favor; 𝑦 = number of members voting against We are given two pieces of information: (1) Total number of votes is 435 𝑥 + 𝑦 = 435 (2) 49 more members voted in favor than against. Rephrasing this gives: The number of members voting in favor exceeded the number of members voting against by 49. 𝑥 − 𝑦 = 49 Thus we have two equations: 𝑥 + 𝑦 = 435 𝑥 − 𝑦 = 49. Adding gives 2𝑥 = 484 ⇒ 𝑥 = 242. Substituting 𝑥 = 242 in the first equation gives 242 + 𝑦 = 435 ⇒ 𝑦 = 435 − 242 = 193. Solution: 242 voted in favor and 193 voted against. 54. Unknowns: 𝑥 = number of senators voting in favor; 𝑦 = number of senators voting against We are given two pieces of information: (1) Total number of votes is 100. 𝑥 + 𝑦 = 100 (2) For a supermajority, twice as many senators must vote in favor of the bill as vote against it. Rephrasing: The number voting in favor is twice the number voting against. 𝑥 = 2𝑦 Thus we have two equations: 𝑥 + 𝑦 = 100 𝑥 − 2𝑦 = 0. Subtracting gives 3𝑦 = 100 ⇒ 𝑦 = 33⅓. Substituting this value in the first equation gives 𝑥 = 66⅔. For a supermajority, at least 66⅔ senators must vote in favor. Since there are no fractional votes, the solution is: At least 67 votes in favor. 55. Unknowns: 𝑥 = number of soccer games won; 𝑦 = number of football games won We are given two pieces of information: (1) The total number of games was 12 ⇒ 𝑥 + 𝑦 = 12 (2) Total number of points earned was 38 ⇒ 2𝑥 + 4𝑦 = 38 (Two points per soccer game and 4 per football game) Thus we have two equations: 𝑥 + 𝑦 = 12 2𝑥 + 4𝑦 = 38. Dividing the second by −2 : 𝑥 + 𝑦 = 12 ⇒ −𝑥 − 2𝑦 = −19. Adding gives


−𝑦 = −7 ⇒ 𝑦 = 7 football games. Substituting 𝑦 = 7 in the first equation gives 𝑥 + 7 = 12 ⇒ 𝑥 = 5 soccer games. Solution: Lombardi House won 5 soccer games and 7 football games. Solutions Section 4.1

56. Unknowns: 𝑥 = number of exposés on the football team; 𝑦 = number of exposés on the film society treasurer We are given two pieces of information: (1) The total number of exposés was 10 ⇒ 𝑥 + 𝑦 = 10. (2) Lawsuit damages amounted to $37 million ⇒ 4𝑥 + 3𝑦 = 37 ($4 million per football team lawsuit, $3 million per film society lawsuit). Thus we have two equations: 𝑥 + 𝑦 = 10 4𝑥 + 3𝑦 = 37. Multiplying the first by −4 : −4𝑥 − 4𝑦 = −40 ⇒ 4𝑥 + 3𝑦 = 37. Adding gives −𝑦 = −3 ⇒ 𝑦 = 3 exposés on the film society treasurer. Substituting in the first equation gives 𝑥 + 3 = 10 ⇒ 𝑥 = 7 exposés on the football team. Solution: There were 7 exposés on the football team, 3 exposés on the film society treasurer. 57. Unknowns: 𝑥 = number of brand 𝑋 pens; 𝑦 = number of brand 𝑌 pens We are given two pieces of information: (1) The total number of pens is 12 ⇒ 𝑥 + 𝑦 = 12. (2) The total amount spent was $42 ⇒ 4𝑥 + 2.8𝑦 = 42 ($4 per band 𝑋 pen and $2.80 per brand 𝑌 pen). Thus we have two equations: 𝑥 + 𝑦 = 12 4𝑥 + 2.8𝑦 = 42. Multiply the second by 10: 𝑥 + 𝑦 = 12 ⇒ 40𝑥 + 28𝑦 = 420. Multiply the first by 10 and divide the second by −4 : 10𝑥 + 10𝑦 = 120 ⇒ −10𝑥 − 7𝑦 = −105. Adding, 3𝑦 = 15 ⇒ 𝑦 = 5 brand 𝑌 pens. Substituting this value in the first equation 𝑥 + 𝑦 = 12 gives 𝑥 + 5 = 12 ⇒ 𝑥 = 7 brand 𝑋 pens. Solution: Elena purchased 7 brand 𝑋 pens and 5 brand 𝑌 pens.

58. Unknowns: 𝑥 = number of reams of yellow paper; 𝑦 = number of reams of white paper We are given two pieces of information: (1) A total of 100 reams are purchased ⇒ 𝑥 + 𝑦 = 100 (2) The total budget is $560 ⇒ 5𝑥 + 6.5𝑦 = 560. Multiply the first equation by 10 and the second by −2 : 10𝑥 + 10𝑦 = 1, 000 ⇒ −10𝑥 − 13𝑦 = −1, 120. Adding gives −3𝑦 = −120 ⇒ 𝑦 = 40 reams of white paper. Substituting in the first equation gives 𝑥 + 40 = 100 ⇒ 𝑥 = 60 reams of yellow paper. Solution: Earl should order 60 reams of yellow paper and 40 reams of white paper. 59. If we want demand to equal supply, we can simply set the two functions equal: −60𝑝 + 150 = 80𝑝 − 60 140𝑝 = 210


210 = $1.50 per pet chia. 140 (We do not need to solve for 𝑞.) 𝑝=

Solutions Section 4.1

60. Set demand equal to supply: −10,000𝑝 + 2,000 = 4,000𝑝 + 600 14,000𝑝 = 1,400 𝑝 = $0.10, or 10¢.

61. Demand: 𝐷 = 85 − 5𝑃 Supply: 𝑆 = 25 + 5𝑃 For the equilibrium, we can equate the supply and demand: Demand = Supply ⇒ 85 − 5𝑃 = 25 + 5𝑃 ⇒ −10𝑃 = −60 ⇒ 𝑃 = $6 per widget. Substituting 𝑃 = $6 in the demand curve gives 𝐷 = 85 − 5(6) = 85 − 30 = 55 widgets. 62. Demand: 𝑄𝐷 = 100 − 10𝑃 Supply: 𝑄𝑆 = 20 + 5𝑃 Since the government sets 𝑃 = 7, we compute 𝑄𝐷 = 100 − 10(7) = 30 𝑄𝑆 = 20 + 5(7) = 55, giving a surplus of 55 − 30 = 25.

63. Demand: The given points are (𝑝, 𝑞) = (3, 16) and (4.5, 10). 𝑞 − 𝑞1 10 − 16 −6 𝑚= 2 = = = −4 𝑝2 − 𝑝1 4.5 − 3 1.5 𝑏 = 𝑞1 − 𝑚𝑝1 = 16 − (−4)(3) = 16 + 12 = 28 Thus, the demand equation is 𝑞 = 𝑚𝑝 + 𝑏 = −4𝑝 + 28. Supply: The given points are (𝑝, 𝑞) = (3, 10) and (4.5, 16). 𝑞 − 𝑞1 16 − 10 6 𝑚= 2 = = =4 𝑝2 − 𝑝1 4.5 − 3 1.5 𝑏 = 𝑞1 − 𝑚𝑝1 = 10 − (4)(3) = 10 − 12 = −2 Thus, the supply equation is 𝑞 = 𝑚𝑝 + 𝑏 = 4𝑝 − 2. For the equilibrium price, we can equate the supply and demand:

Supply = Demand ⇒ −4𝑝 + 28 = 4𝑝 − 2 ⇒ −8𝑝 = −30 ⇒ 𝑝 =

30 = 3.75 KMW/kg. 8

64. Demand: The given points are (𝑝, 𝑞) = (4.5, 12) and (2.5, 20). 𝑞 − 𝑞1 20 − 12 8 𝑚= 2 = = = −4 𝑝2 − 𝑝1 2.5 − 4.5 −2 𝑏 = 𝑞1 − 𝑚𝑝1 = 12 − (−4)(4.5) = 30 Thus, the demand equation is 𝑞 = 𝑚𝑝 + 𝑏 = −4𝑝 + 30. Supply: The given points are (𝑝, 𝑞) = (4.5, 15) and (2.5, 11). 𝑞 − 𝑞1 11 − 15 −4 𝑚= 2 = = =2 𝑝2 − 𝑝1 2.5 − 4.5 −2 𝑏 = 𝑞1 − 𝑚𝑝1 = 15 − (2)(4.5) = 6 Thus, the supply equation is 𝑞 = 𝑚𝑝 + 𝑏 = 2𝑝 + 6. For the equilibrium price, we can equate the supply and demand: Supply = Demand ⇒ −4𝑝 + 30 = 2𝑝 + 6 ⇒ 6𝑝 = 24 ⇒ 𝑝 = 4 KMW/kg. 65. Unknowns: 𝑥 = number of pairs of dirty socks; 𝑦 = number of T-shirts


Solutions Section 4.1 We are given two pieces of information: (1) A total of 44 items were washed: ⇒ 𝑥 + 𝑦 = 44. (2) There were three times as many pairs of dirty socks as T-shirts. Rephrasing this gives: The number of pairs of dirty socks was three times the number of T-shirts, or 𝑥 = 3𝑦. Thus we have two equations: 𝑥 + 𝑦 = 44 𝑥 − 3𝑦 = 0. Subtracting (or multiplying the second by −1 and adding) gives 𝑦 = 44 ⇒ 𝑦 = 11 T-shirts. Substituting 𝑦 = 11 in the first equation gives 𝑥 + 11 = 44 ⇒ 𝑥 = 44 − 11 = 33 pairs of dirty socks. Solution: Joe's roommate threw out 33 pairs of dirty socks and 11 T-shirts.

66. Unknowns: 𝑥 = number of pieces of raw salmon. 𝑦 = number of pieces of raw tuna Total number of pieces consumed: ⇒ 𝑥 + 𝑦 = 3 Protein intake: Salmon: 50% of 1 oz = 0.5 oz per piece Tuna: 40% of 1.25 = 0.5 oz per piece 0.5𝑥 + 0.5𝑦 = 1.5 Multiplying by 2 gives 𝑥 + 𝑦 = 3, the same equation as the first one. Thus, the system is redundant and has infinitely many solutions. The customer could have consumed any combination of tidbits totaling three.

67. Unknowns: 𝑥 = size of raise for each full-time employee; 𝑦 = size of raise for each part-time employee We are given two pieces of information: (1) Total budget = $6,000. There are 4 full-time employees each getting a raise of 𝑥 and 2 part-time employees each getting a raise of 𝑦. Thus, 4𝑥 + 2𝑦 = 6,000. (2) The raise received by each full-time employee is twice the raise that each of the part-time employees receives. 𝑥 = 2𝑦 Thus we have two equations: 4𝑥 + 2𝑦 = 6,000 𝑥 − 2𝑦 = 0. Adding gives 5𝑥 = 6,000 ⇒ 𝑥 = $1,200 per full-time employee. (We are not asked for the value of 𝑦.) 68. Unknowns: 𝑥 = number of paperback copies sold; 𝑦 = number of hardback copies sold after the paperback appeared We are given two pieces of information: (1) A total of 441,000 − 36,000 = 405,000 copies were sold: ⇒ 𝑥 + 𝑦 = 405,000 (2) The number of paperback copies is 9 times the number of hardbacks sold since the paperback appeared. ⇒ 𝑥 = 9𝑦 ⇒ 𝑥 − 9𝑦 = 0 Subtracting this from the first equation gives 10𝑦 = 405,000 ⇒ 𝑦 = 40,500 hardback copies. Substituting in the first equation gives 𝑥 + 40,500 = 405,000 ⇒ 𝑥 = 364,500 paperback copies.

69. The three lines in a plane must intersect in a single point for there to be a unique solution. This can happen in two ways: (1) The three lines intersect in a single point, or (2) two of the lines are the same, and the third line intersects it in a single point. 70. There are three ways this can happen: (1) The lines are all parallel; (2) two of the lines are parallel; or (3) the three lines intersect in three different points (forming a triangle).


Solutions Section 4.1 71. The equilibrium price occurs at the point where the demand and supply lines cross. Even if two lines have negative slope, they will still intersect if the slopes differ. Therefore, there can be an equilibrium price. 72. The given system of equations cannot be satisfied by any actual number of rocks and pebbles. 73. You cannot round both of them up, since there will not be sufficient eggs and cream. Rounding both answers down will ensure that you will not run out of ingredients. It may be possible to round one answer down and the other up and still have sufficient eggs and cream, and this should be tried. 74. Wrong. Since your factory can only produce whole numbers of gallons, 𝑦 must be a whole number between 0 and 200, giving 201 possible solutions.

75. Since multiplying both sides of an equation by a nonzero number has no effect on its solutions, the graph (which represents the set of all solutions) is unchanged: (B). 76. The graphs are the same: (A). 77. The associated system has no solutions, and so the lines do not intersect. Thus, they must be parallel: (B). 78. Since we obtain a unique solution, the graphs are not parallel: (D). 79. Answers will vary. 80. Answers will vary. 81. Choosing two lines at random gives two random slopes (which are numbers). Since two randomly selected numbers are unlikely to be the same, it follows that two randomly chosen straight lines are very unlikely to be parallel (or the same line). Thus, the two lines are very likely to intersect in a point, giving a unique solution. 82. It is very unlikely. Three randomly chosen lines are unlikely to meet in a single point.


Solutions Section 4.2 Section 4.2 1.

1

1

1

4

1 2 2 1

1

0

3

0 1 1 2

1

1

4

0 2 2 (1 2) 2

1 0 3

1

1

0

0 1 1

1

Translating back to equations gives the solution: = 3, = 1. 2.

2

1 2

2 1 2 2 + 1

2 0 0 (1 2) 1

2 1 2

0 2 4 (1 2) 2

1 0 0

2

1

0

1

0 1 2 0 1 2 Translating back to equations gives the solution: = 0, = 2. 3.

3

3

6

2

3 6 3 2 2 1

2

18 (1 3) 1

0

1

0

3 2

6

0 5 30 (1 5) 2 6

4.

2

2

3 5

2 5 2 2 3 1

3

0

2

(1 2) 1

3

1

2

3

2

1 0 6

5

0 5 5 (1 5) 2

1

0

1

2

5.

2

1 3 2 1 2 2 2

1 3 2 1 2 0

0

0

2

3

2 3 1 2 + 1

2

3

5

0

1

6

6

1 2 2

1 + 3 2

1

2 3 1 (1 2) 1

1 0 1

1

3

1

0

0 1 1 2 0 1 1 0 1 1 Translating back to equations gives the solution: = 1, = 1. →

1

1 + 2

2 1 2

0 1 6 2 0 1 6 0 1 6 Translating back to equations gives the solution: = 6, = 6. →

4

0 0 0

Translating back to equations gives: + 32 = 12 . Solve for : = 12 32

General Solution: = 12 32 ; is arbitrary or, in coordinate form, � 12 (1 3 ), �; arbitrary 6.

2 3 1

0

0

6 9 3 (1 3) 2

1 3 2 1 2 0

2 2

3 1

3 1 2 1

Translating back to equations gives: 32 = 12 . Solve for : = 12 + 32

2 3 1 (1 2) 1 0

0

0

General Solution: = 12 + 32 = 12 (1 + 3 ); is arbitrary or, in coordinate form, � 12 (1 + 3 ), �; arbitrary


7.

2

3

2

2

3

2

Solutions Section 4.2 →

2 3 2

2 3

2

1 3 2 1 2 2 2

2 3 1 2 + 1

2 3 2

2

3 2

0 0 1

Since Row 2 translates to the false statement 0 = 1, there is no solution. 8.

6 9 3 (1 3) 2

3 1 2 1

2

0

0

1

Since Row 2 translates to the false statement 0 = 1, there is no solution.

9.

1

1

3

1

3 2 3 1

1

1 0 1 4 0

1

1

4

0

0 1 3 4 0 0

1

2 3 1 → 0

0

1

1

4

1

3

4 1 + 2

3 3 2

4

10. 3 2 1 0 0

1

5 1 1 5 5 3 1

1

5

4

1

1

5 1 + 2

4 3 2

5

1 3

1

2 1 2 3 1

25 5 5

1

0

1

1

3 25 1 (1 5) 1

0

1

→ 0

11.

3

1

1

1

2

1

0

1

0.5

0.1

1

0

0

1

1

5

4

0 30 24 (1 6) 3

→ 0 5 4 (1 5) 2 → 0 1 4 5 0 0 0 0 0 0 1

2 3 1 → 0

1 1 3 1

0

1

4 2

0

1

4 1 + 2

1 2 3 2

4

0

1

→ 0 4 0

0

Since Row 3 translates to the false statement 0 = 3, there is no solution. 12. 3 2 4

2

5 1 1 5 5 3

0 8 5

1 3

2

1

2 2 2 3 1

25 5

1

3 25 1

1

→ 0

2

8

1

13. 0.1 0.1 1

1

0.3

10 1

10 2 →

11 3 3 3

5 1 3

1

1 3

17

3 5 2 1

11 5 3 3 1

1

3

5

4 1 + 2

0 55 24 8 3 55 2

0 0 83 Since Row 3 translates to the false statement 0 = 83, there is no solution. 1.7

1 0 1 5

Translating back to equations gives the solution: = 15 , = 45 . 0

(1 4) 1

1

→ 0 4 3 (1 4) 2 →

Translating back to equations gives the solution: = 14 , = 34 . 1

0

5

1

17

→ 0 6 2 (1 2) 2 → 0 12

4

(1 4) 3


5 0 0

17 3 1 + 2

1

3

1

3

3 + 2

1

15

0

1 0 10 3

3 1 (1 3) 2 → 0 1 0 0 0 0

0

50 (1 15) 1

Solutions Section 4.2

0

Translating back to equations gives the solution: = 10 , = 13 . 3

14. 3 0

0.1 10 1

0.5

0.3 1

4

1

1 17 1 17

1

17 3 →

0

2

13

1

2

1 0

1

0 1 3

0

13 3 2

2

1

1 2 1 5 2

5

5

3

1

4 3 2 + 1 →

1

1

17 3 3 + 1

1

6 0 63 (1 6) 1 0

13

(1 2) 2

1

2

2 0

0

1 3 0

3 5 0

2 13

8 52 (1 4) 3

0

1

1

1

2

0

0

0

1 4 3 + 2 1

0 2 3 →

1

1

0 2 1

2

1

0

0

0 0

0

0

1 0

4

3 4

0 4 1

0

16.

0 1

1 0 0 1

2

0

1 0 4 2 → 0 1 0 4 0 0 1 4 0 1 4

1

2

0

6

1 1 0

1 0 0

6

3 2 5 3 1

0

1

0 0 1

1

3

1

6

→ 0 0

1 1

→ 0 1 0 1

0

4 1 2 2

1 0

2 1 3 2

1 0

17. 1 3 1 3 2 3 1 0

1 2 1

6

0

1

2

0 1 0

3

0

1

1 0 0

4

1

0 0 1 1 2

1

1

1

0

3 4 3 + 4 2

4 1 + 2 3

0 2 3

1

1

6

1

1

0

0

1

0 0

1

1

0 0 1 1

1 1

0

1 1 + 2 2

2

1 0 1 + 2 2

1

→ 0 1

Translating back to equations gives the solution: = 6, = 1, = 1. 1

4

1 0 0 4

0

4

1

0

0 0 1 4 0 Translating back to equations gives the solution: = 4, = 4, = 4. 1

2

1

0 (1 3) 2 →

3

3

1 0 21 2 → 0 1 13 2

Translating back to equations gives the solution: = 21 , = 13 . 2 2

15.

1

3 2

3 2 → 2 3

1

1 1 1

0

→ 0 1 0

0

1

6 1

1 2 0

4

2 2

2

1

2

1

3

0

2 1 → 0 2 4 3 1

1 + 2 3 2 3

0 1 4

→ 0 1 0

Translating back to equations gives the solution: = 1, = 3, = 12 .

0

0

4

(1 2) 2 →

3

2

1

2 1 (1 2) 3


1

18. 1 3 1 2

6 0

2

1 0

0

0 3

0

1 2 1 3

1 3

(1 2) 1

12 12 1

0

2

3 0

→ 0 3

0 0 8 24 (1 8) 3 0 1

3 2 →

2

1 2 1 2 3

0

2

2 1

0

Solutions Section 4.2

0 0

1 0 0 1

→ 0 1 0 2

0 0 1 3 3

2

1

1

6

1 1

2

1

0

1

0

1

6

2 1

2 2 1 → 0

1 2 2 3 1

0

1 + 3

3 0

0

12 2 + 2 3 → 0 3

3

1 2 1 2 0 2 3

1

1 0 1 + 2 2

2

1 0

1

1

0

0 2 2 →

1

19. 1 2 1 2

1

0

1 0 3 + 2

0

2 1

1 2

20. 1 2

0

3

1

1 0

0

1 2 1 2 2 → 1

2 0 2 4 (1 2) 1 0 1 2 4

2

12

3 1 + 2

2 4 3 3 2

(1 3) 2 →

0 0 1 3

2

1 1

1 0 2

1

0 2 1

1 1 0 3 + 2 1

2

0

1 0 1

2

1

0

1 0 1 0

1 1 0 2 → 0 1 1 0 0 0 0 0 0 0 0

2

1 3

0

0

1 2 2 2 1

0

1 1 2 2 3 3 1

1 0 1 2 → 0 1 2 4

2

1

3

0

3

2 1

→ 0

1

0

1

1 0 3

0 (1 3) 2 →

0

0

3 0 (1 3) 3

1 + 2

2 4

2 4 3 2

0 0 0 0 0 0 0 0 Translating back to equations gives: = 2, 2 = 4. Thus, the general solution is = 2, = 2 4, arbitrary, or ( 2, 2 4, ); arbitrary.

21.

1

1

2

2

2 2

1 2

3 5 3 5 3 5 2 5 5 3

1 1 0 0

2

0 0

1 1 + 2 2 2

1

5

3

3 3 2

1 1 2 1

→ 2 2 2 3 3 3

1 1

0

→ 0 0 1 0 0

3

2

2 2

(1 2) 2 →

0

1

1

1 2

1 1 3

1 2 1

1 1 3 3

Since Row 3 translates to the false statement 0 = 1, there is no solution.

22.

1

1

2 7

1

1 0

1 7

(1 3) 1

3 6

0 Translating back to equations gives: = 0, = 0. Thus, the general solution is = , = , arbitrary, or ( , , ); arbitrary. 1

1

0

0 0 1 3

1 2 0 2 1

1

0

0

Translating back to equations gives the solution: = 1, = 2, = 3. 1 2

3

0

2 0

8 7 14 7 3

1

→ 1 1 7 2

0

0

8 98 (1 2) 3

1 1 1

1

1 2

2 1

3 3 1

1 2

1 7 0

0

2 1 →

4 49 3 1


1

1

1

1 2

0 2 6

1

(1 2) 2 → 0

2

0 1 3 51

1

0

1 2 1 + 2

Solutions Section 4.2

1

3

1

1

1

1

0

4 1

→ 0 1 3

3 51 3 2

0

0

0

Since Row 3 translates to the false statement 0 = 50, there is no solution.

23.

0.5 0.5 0.5 1.5 2 1 4.2

2.1 2.1

0.2

0

1 1 1 3 0

5 3

0

0

1 2 3

1 0 0 1 1 0

1 0 1

0

1

0

1

1 0 0 1

→ 0 1 0

0

1

0

1 3 1 2

1

1

3 3 6 (1 3) 2 →

1

1

1 2

1

3

10 2 → 42 21 21 0 (1 21) 2 →

0

0.2

1

2 3 3 2

1 0 0 0

1 0

2

0

1

1

1

1

1

50

1 1 3

1 1 0 2 + 2 1 → 0 1 0 3 + 1

2 2 3 →

1

0 0 1 1 0 0 1 1 Translating back to equations gives the solution: = 1, = 1, = 1.

24.

0.25 0.5 0.2 0.5

1 2 0

3

0 1

0.2

1.5

0

0

0

1

1

0.5

3 1 + 2 2 3 3 + 2

3 0 2 6 1 + 2 3 0 3 1 3 2 + 3 0 0

1

4 1

0.2 0.6 5 2 →

1 3 2

0

0

1 0 0 2

2 3

1

2

1

1

1

3

3 0 2 6

→ 0 3 1 3 0 0

5

0 0 1

0

0

0

0

1 3 2 1 → 2

1

3 1

(1 5) 3

3 0 0 6 (1 3) 1

→ 0 3 0 3 (1 3) 2 →

0 1 0 1

0 0 1 0 Translating back to equations gives the solution: = 2, = 1, = 0. 25.

2

3

1 0

1 1 4

1 1 5 2 2 3 1

0

1

2 1 0

1

1

4

1 2

1 + 2

(1 2) 1

3 1 1 0 1 + 2

2 0

0

2

0 1 1 2

0 1 1 2 Translating back to equations gives: = 1, = 2. Thus, the general solution is = 1, = 2, arbitrary, or (1, 2, ); arbitrary. 26.

3

1

1 1 0 1

1

4 3 2 1

3 0 0 3 (1 3) 1 0 1 1 3

3 1 1 0

1 0 0 1 0 1 1 3

4

4

0

12 (1 4) 2

0

1

1

3


Solutions Section 4.2 Translating back to equations gives: = 1, + = 3. Thus, the general solution is = 1, = + 3, arbitrary, or (1, + 3, ); arbitrary. 27.

0.75 0.75 1 4 4 1

0

0

1

3 3 4

4

1

16

0

16 16 (1 16) 2

3 3 0 12 (1 3) 1

3 1

1

4

0 3 2 1

3 3 4 16 1 + 4 2 0

3 4 16

0

1 1 0

1

1

4

0 0 1 1 0 0 1 1 Translating back to equations gives: = 4, = 1. Thus, the general solution is = + 4, arbitrary, = 1, or ( + 4, , 1); arbitrary. 28.

2

1

1

4

1 0.5 0.5 1.5 2 2

2

1

1

4

1 3 2 + 1

1

2

2 1 1 4 0

0

Since Row 2 translates to the false statement 0 = 7, there is no solution.

0 7

29. 3 1 1 12 (1 3) 1 → 1 1 3 1 3 4

Translating back to equations gives: + 13 13 = 4.

Thus, the general solution is = 4 3 + 3, arbitrary, arbitrary, or (4 3 + 3, , ); , arbitrary.

30. The augmented matrix 1 1 2 21 is already reduced. Translating to equations gives + 3 = 21. General solution: (21 + 3 , , ); , arbitrary 1

31.

1

2

2

0.75 0.75 1

1 1

2

0 0 1 0 0 2 0 1

0

0 0

0

1 0

0

0 0 1

2

2

1

2

0.25 4 3

0

2

4

(1 2) 3

2

2

1

2

1 2

20

17 0

1

1 21

1 1

0 0 0 0 0 1

2 3

0 0 0

2

2

1

2

3

2

4

1

1 0 2 21 2

1 1 0

1 0 0 17 0 0 1

1

2 0

(1 2) 2

1 1 + 2 2 2

2

20

3 2

1

1

0 0 0

1 0

32.

1

1

1

7

0

0.75 0.5 0.25 14 4 3 1

1

1

4

1

1 3 1

3

1 4

0 1

0

1 2

1 7 2 1

1

1

0

2 1

56 3 3 1 4

4 1

1 1

2 1

3 3 1

1 0

0

3 1 4

2

0

20

4 → 2 → 3 → 4 \quad → \quad

1

1

1 0 2 21 4 + 1

0 1 0 20 0 1 0 20 Translating back to equations gives the solution: = 17, = 20, = 2. 1

2

1

3

1

1

→ 1 0 0 17 0 1 0 0 0 1 0 0 0

20

2 0


1

1

1 2

0 2 6

0 5

2

4

62

0 1 3

1

0

0

0

0

1

0

0

0

0

0

1

0

2

6

4 1

(1 2) 2 (1 2) 4

1

0 11

3

0

1

1

0

3

62 3 5 2

4

0

1

3

0

0

0

11

0 1 0 10

2 + 3 3

1

1

0

0 0 1

4 1 1 + 4 3

0 1 3

1 0 0

0 1 0 10 2

1

3

5

0

1 2 1 + 2

1

0

57 (1 19) 3

19

1

1

Solutions Section 4.2

3

1

4 3

3

3

0 0 0 0 0 0 0 0 Translating back to equations gives the solution: = 11, = 10, = 3. 1

33.

0 1 1

1 0 0 1

1 5 0 1

1 2 1 1

3 7 2 2 3 1 1 5 1 1 4 1

3 2

0 0

2

0 1

0

0 0

2 0

0

0

0

1 1

1

0

1

1 0

0

2 1 + 3 3 2 + 3

0 3 (1 2) 1 0

0

0 0 2 0 1 (1 2) 3 0 0

1

1

0

5 0 1 1 2

1

0

1

0

2 1 1

2 2 1 3 2 2

2

0

0

0 1 0

2 0

0

0 1

0

0 0 2

0 0

0

2 3 1 + 2 4 1

0 1 0 0

0

1

1 0

0

0 0 1 0

1 2

1

0

0 0 0 1

2 4

0

1 0 0 0 3 2 →

0

Translating back to equations gives the solution: = 32 , = 0, = 12 , = 0. 1

1

0

4

1

34.

2

2 3 2 1 2 2 1

4

0

3

0 2

4 4

6 9

10

1 0

1

4 6

4 2 1

0 4 3

6

3 2 + 3

4

5

2

0 0

0 0 0

3

5

15 22

0

1 5

4 5 3

0 0

0 0

3 0

5 1

1

0

1 5 4

0 4 0 11 2 2 + 11 4 3 + 5 4

0 4

0

4

0

2

0

4

1

3 6 3 6

1

4

9

8

5

1

5

2

0 4 0 11 2 0 0

4 →

4

0

1 + 3

1

0 0

0

3 0

0 0

3

2

0

4

4 1 + 2 3 + 2

2 4 + 2

(1 3) 4

(1 2) 1

0 4 0 0 2 (1 2) 2 0 0

0 0

3 0 0 1

1 0


2

0

0 0

(1 2) 1

1

0 2 0 0 1 (1 2) 2 0 0

0

3 0

0

(1 3) 3

1

0 1

0

Solutions Section 4.2

1 0 0 0 1 2 0 1 0 0 1 2 0 0 1 0 1 3

0 0 0 1

0

Translating back to equations gives the solution: = 12 , = 12 , = 13 , = 0. 1

35.

1 5 0 1

0

1 2 1 1

1 5 1 1 3 1

1 1

2 7 2 2 4 1

1

1 0 3 1 0 1 + 3 0 1 2

1 2 3

1

0 0 0

1

0 0 0

0

0 4 3

1

0

1

0

5 0 1 1 2

1

2 1 1

0

0

0 1 0

2 2 1 4 2

1

1 0 3 0 0 0 1 2 0 1 0 0 0 1 0 0 0 0 0 0

Translating back to equations gives: + 3 = 0, + 2 = 1, = 0. General solution: = 3 , = 1 2 , arbitrary, = 0 or ( 3 , 1 2 , , 0); arbitrary 1

1

0

4

1

36.

2

2 3 2 1 2 2 1

4

0

3 10

0

4

3

6

3

1

3

7

1

4 4

4 3 1

1 + 3

0 4 3 6 3 2 + 3 0 0

0

3

0

3

1 0 0

1

5

4 3

1

5

5 4

1 2

0

0

0 1 0 11 4 1 2

1

1

0

4

1

0

4

3 6 3

4

0

0

5

0

4

0

0

6

1

3

5

5

1

2

4

1

4 1 + 2 3 + 2

(1 4) 1

0 4 0 11 2 (1 4) 2 0

0

0

3

0

0

0

0

(1 3) 3

0 0 1 5 3 1 3 0 0 0

Translating back to equations gives: + 54 = 12 , + 11 = 12 , 53 = 13 . 4

General solution: � 12 54 , 12 11 , 13 + 53 , �, arbitrary 4

37.

1

1

2

2 1 4 1 3

1

7

2

2 2 1

8 2 3 3 2 1

5 0 17 8 7 (1 5) 1 0 5 0 0

6 0

6 0

→ 0

5

0

5

6

6

6

6

1

1 3 2

1 0 17 5 8 5 7 5

1 (1 5) 2 → 0 1 0

1 2 1 4 1 5 1 + 2 2

0 0

6 5 0

6 5 0

1 5 0

Translating back to equations gives: + 17 85 = 75 , + 65 + 65 = 15 . 5

General solution: = 7 5 17 5 + 8 5, = 1 5 6 5 6 5, , arbitrary, or (7 5 17 5 + 8 5, 1 5 6 5 6 5, , ); , arbitrary,


Solutions Section 4.2

or � 15 (7 17 + 8 ), 15 (1 6 6 ), , �; , arbitrary

38.

1

3 2 1 1

1

2

3

0

1

2 2 1

2

2 0 1 1 3 (1 2) 1 0 6 0 0

3 0

→ 0

3 3 2 1

1

1

1 3 2 1 1 2 1 + 2 0

3

6

3

1 0 1 2 1 2 3 2

3 1 (1 6) 2 → 0 1 0 0

6

1 2

0 0

1 2 1 6

0

0

3

1

1 3 2

3

0

Translating back to equations gives: 12 + 12 = 32 , + 12 + 12 = 16 .

General solution: = 3 2 + 2 2, = 1 6 2 2, , arbitrary or � 12 (3 + ), 16 (1 3 3 ), , �; , arbitrary 1 0

39. 0 0

0

1

1

1

1

15 1 2

1

1 1 1 2

0

0

0 0

1 0

1

1

1 1 1 0

1

0

0 0 0

1 1

2

0 0 0

0

0 0 1

1 1

0 1 1

1 0 0 0 0 1 0 1 0 0 0 2

5

7 1 + 2 4

1 0 0 2 0 1 0

12

10 2 2 4 12 3 4 5

1 0

2

0

2

17 1 2 3

0 1 1 1 1 2 2 + 3

→ 0 0 0 0 0 0

1

1

0

1

12

1 1 1

0

0

1

5

1 0 0 0 2 9 1 + 2 5 0 1 0 0

2

12 2 2 5

0 0 0 0

1

5

→ 0 0 1 0

13 3 2 5 →

2

0 0 0 1 1 1 4 + 5

0 0 1 0 0 3 0 0 0 1 0 4

0 0 0 0 1 5 Translating back to equations gives the solution: = 1, = 2, = 3, = 4, = 5. 1 1 1 1 1 1 1 + 2 0

40. 0 0 0

1

1

1

1 2

0

0

0

1 1

0 0

1 1 1 1 0

1

1 1

1 0 0

2

0 1 1 2 4

0 0 0

1

1 1

0 1 0

2

0 1 2 2 4

0 0 1 1 1 1 3 + 4 0 0 0

0

1 1

1 0 0 1

→ 0 0 0 0 0 0

2 1 1 0 0

0 1

2 3 1 2 3 1 2 2 3

1 1 1 1 0

1 1 1 1

1 0 0 0 2 1 1 + 2 5 0 1 0 0 2 1 2 + 2 5

→ 0 0 1 0 0 0 0 1 0 0 0 0

2 1 1

2 1

1

3 2 5 → 4 5


1 0 0 0 0 1

Solutions Section 4.2

0 1 0 0 0 1 0 0 1 0 0 0 0 0 0 1 0 0

0 0 0 0 1 1 Translating back to equations gives the solution: = 1, = 1, = 0, = 0, = 1.

41.

1

1

2

1

0

1

4

1

1

1

1 3 2 4 2 1

1

1

1

1

0

0

1 0

0

0

0

0 0

0

0 4

1

0

0 0 1 0

0

0 2

0

0 8

0

0

0 1 1 1

2

0 0 0 0

0

0

0 0

2

0

0

0 1

0

0

0

0

0

1 1

1 2

2 2 3 1

2 0

1 0

0

(1 4) 4 (1 8) 5

1

1

0

3

3

0 0

0

0

1

0

0 1 1 1

(1 2) 3 → 0 0 0 0

1 0

0

0 0

0

0 0

1

0

→ 0

1 7 6 5 4 1

0 1 1 1 1 2 0 0

1

0 0 0

1

0

1 1 1

0

0 0

11 6 5 3 2

42. 1 0 0

0 1 1 1

1 1 1

1

1 2

1

1 2

1

1

2

1

1

2

1 1 1 1 2 2 2 2 1

1

1

1

1

1

1

1

1

3

2 4 2

5

0 0

1

4 3

5 3

0 0

15

1

9 4 1

0 2 2 2 2 6 (1 2) 4

3

0

18 3 1 → 0 6 5 1

15 1 2 3

2 3 2

3

0 0 0 0 0 0 0 0 0 0 0 0 Translating back to equations gives: = 2, + = 2, = 0. General solution: = 2, = 2 + , arbitrary, arbitrary, = 0 or ( 2, 2 + , , .0); , arbitrary 1

2

2 2 3

0 0 2 0 1

3

2 0

1 + 2

1

1

0 1 1 1 0 2

3 → 0 0

0

1 1

1 1

1

1

1 1 1

1 1

0 1 1 1 1 3 5 + 2

1

1

1

1 1 1

15 3 3

0 3 3 3 3 9 (1 3) 5

1 0 0 0 0 12 0 1 1 1 1

3

0 0 0 0 0

0

3 2 → 0 0 0 0 0

0 1 1 1 1 3 4 + 2

1

0 0 0 0 0

0 0

Translating back to equations gives: = 12, + + + = 3. General solution: = 12, = 3 , , , arbitrary, or (12, 3 , , , ); , , arbitrary


43. + 2 + = 30 2 + 2 = 30 + 3 + 3 4 = 2 2 9 + = 4 Using technology:

Solutions Section 4.2

Matrix #1 x y 1 2 2 0 1 3 2 -9

z -1 -1 3 0

w 1 2 -4 1

30 30 2 4

Matrix #2 x y 1 2 0 -4 0 1 0 -13

z -1 1 4 2

w 1 0 -5 -1

30 -30 -28 -56

Matrix #3 x y 2 0 0 -4 0 0 0 0

z -1 1 17 -5

w 2 0 -20 -4

30 -30 -142 166

Matrix #4 x y 34 0 0 -68 0 0 0 0

z 0 0 17 0

w 14 368 20 -368 -20 -142 -168 2112

Matrix #5 x y 17 0 0 -17 0 0 0 0

z 0 0 17 0

w 7 5 -20 -7

184 -92 -142 88

Matrix #6 x y 17 0 0 -119 0 0 0 0

z 0 0 119 0

w 0 0 0 -7

272 -204 -2754 88

Matrix #7 x y 1 0 0 1 0 0 0 0

z 0 0 1 0

w 0 0 0 1

16 12/7 -162/7 -88/7

Solution: (16, 12 7, 162 7, 88 7)


44. 4 2 + + = 20 3 + 3 4 = 2 2 + 4 = 4 + 3 + 3 = 2 Using technology:

Solutions Section 4.2

Matrix #1 x y 4 -2 0 3 2 4 1 3

z 1 3 0 3

w 1 -4 -1 0

20 2 4 2

Matrix #2 x y 4 -2 0 3 0 10 0 14

z 1 3 -1 11

w 1 -4 -3 -1

20 2 -12 -12

Matrix #3 x y 12 0 0 3 0 0 0 0

z 9 3 -33 -9

w -5 -4 31 53

64 2 -56 -64

Matrix #4 x y 396 0 0 33 0 0 0 0

z 0 0 -33 0

w 114 -13 31 1470

1608 -34 -56 -1608

Matrix #5 x y 66 0 0 33 0 0 0 0

z 0 0 -33 0

w 19 -13 31 245

268 -34 -56 -268

Matrix #6 x y 16170 0 0 8085 0 0 0 0

z 0 0 -8085 0

w 0 0 0 245

70752 -11814 -5412 -268

Matrix #7 x y 1 0 0 1 0 0 0 0

z 0 0 1 0

w 0 0 0 1

1072/245 -358/245 164/245 -268/245

Solution: � 1,072 , 358 , 164 , 268 � 245 245 245 245


45. + 2 + 3 + 4 + 5 = 6 2 + 3 + 4 + 5 + = 5 3 + 4 + 5 + + 2 = 4 4 + 5 + + 2 + 3 = 3 5 + + 2 + 3 + 4 = 2 Using technology:

Solutions Section 4.2

Matrix #1 x y 1 2 2 3 3 4 4 5 5 1

z 3 4 5 1 2

w 4 5 1 2 3

t 5 1 2 3 4

6 5 4 3 2

Matrix #2 x y 1 2 0 -1 0 -2 0 -3 0 -9

z 3 -2 -4 -11 -13

w 4 -3 -11 -14 -17

t 5 -9 -13 -17 -21

6 -7 -14 -21 -28

Matrix #3 x y 1 0 0 -1 0 0 0 0 0 0

z -1 -2 0 -5 5

w -2 -3 -5 -5 10

t -13 -9 5 10 60

-8 -7 0 0 35

Matrix #4 x y 1 0 0 -1 0 0 0 0 0 0

z -1 -2 0 -1 1

w -2 -3 -1 -1 2

t -13 -9 1 2 12

-8 -7 0 0 7

Matrix #5 x y 1 0 0 -1 0 0 0 0 0 0

z -1 -2 0 -1 1

w 0 0 -1 0 0

t -15 -12 1 1 14

-8 -7 0 0 7

Matrix #6 x y 1 0 0 -1 0 0 0 0 0 0

z 0 0 0 -1 0

w 0 0 -1 0 0

t -16 -14 1 1 15

-8 -7 0 0 7


Solutions Section 4.2 Matrix #7 x y 15 0 0 -15 0 0 0 0 0 0

z 0 0 0 -15 0

w 0 0 -15 0 0

t 0 0 0 0 15

Matrix #8 x y z 1 0 0 0 1 0 0 0 0 0 0 1 0 0 0

w 0 0 1 0 0

t 0 0 0 0 1

-8/15 7/15 7/15 7/15 7/15

-8 -7 -7 -7 7

Solution: ( 8 15, 7 15, 7 15, 7 15, 7 15)

46. 2 + 3 4 = 0 2 + 3 4 + = 0 3 4 + 2 = 0 4 + 2 + 3 = 0 2 + 3 4 = 1 Using technology: Matrix #1 x y 1 -2 -2 3 3 -4 -4 0 0 1

z 3 -4 0 1 -2

w -4 0 1 -2 3

t 0 1 -2 3 -4

0 0 0 0 1

Matrix #2 x y 1 -2 0 -1 0 2 0 -8 0 1

z 3 2 -9 13 -2

w -4 -8 13 -18 3

t 0 1 -2 3 -4

0 0 0 0 1

Matrix #3 x y 1 0 0 -1 0 0 0 0 0 0

z -1 2 -5 -3 0

w 12 -8 -3 46 -5

t -2 1 0 -5 -3

0 0 0 0 1

Matrix #4 x y 5 0 0 -5 0 0 0 0 0 0

z 0 0 -5 0 0

w 63 -46 -3 239 -5

t -10 5 0 -25 -3

0 0 0 0 1


Solutions Section 4.2 Matrix #5 x y 1195 0 0 -1195 0 0 0 0 0 0

z 0 0 -1195 0 0

w 0 0 0 239 0

t -815 45 -75 -25 -842

0 0 0 0 239

Matrix #6 x y 239 0 0 -239 0 0 0 0 0 0

z 0 0 -239 0 0

w 0 0 0 239 0

t -163 9 -15 -25 -842

0 0 0 0 239

Matrix #7 x y z w t 201238 0 0 0 0 -38957 0 -2012380 0 0 2151 0 0 -2012380 0 -3585 0 0 0 201238 0 -5975 0 0 0 0 -842 239 Matrix #8 x y 1 0 0 1 0 0 0 0 0 0

z 0 0 1 0 0

w 0 0 0 1 0

t 0 0 0 0 1

-163/842 -9/842 15/842 -25/842 -239/842

9 15 25 Solution: � 163 , 842 , 842 , 842 , 239 � 842 842

47. 1.6 + 2.4 3.2 = 4.4 5.1 6.3 + 0.6 = 3.2 4.2 + 3.5 + 4.9 = 10.1 We use the Excel Matrix Pivot Tool (on the Website):

→ Solution (rounded to 1 decimal place): (1.0, 1.4, 0.2)

48. 2.1 + 0.7 1.4 = 2.3 3.5 4.2 4.9 = 3.3 1.1 + 2.2 3.3 = 10.2 We use the Excel Matrix Pivot Tool (on the Website):


Solutions Section 4.2

→ Solution (rounded to 1 decimal place): (0.9, 2.2, 1.9)

49. 0.2 + 0.3 + 0.4 = 4.5 2.2 + 1.1 4.7 + 2 = 8.3 9.2 1.3 = 0 3.4 + 0.5 3.4 = 0.1 We use the Excel Matrix Pivot Tool (on the Website):

Solution (rounded to 1 decimal place): ( 5.5, 0.9, 7.4, 6.6) 50. 1.2 0.3 + 0.4 2 = 4.5 1.9 0.5 3.4 = 0.2 12.1 1.3 = 0 3 + 2 1.1 = 9 We use the Excel Matrix Pivot Tool (on the Website):

Solution (rounded to 1 decimal place): (5.0, 0.2, 5.8, 1.9) 51. A pivot is an entry in a matrix that is selected to "clear a column"; that is, use the row operations of a certain type to obtain zeros everywhere above and below it. "Pivoting" is the procedure of clearing a column


Solutions Section 4.2 using a designated pivot. 52. Locate the leading entry of each row, and check to see if it is a 1 and its column is cleared. If there is a leading entry that is not a 1 or whose column is not cleared, the matrix is not reduced. Finally, check to see that the rows are arranged so that rows of zeros are at the bottom, and the leading entry in each row is to the right of the leading entry in the row above. 53. 2 1 + 5 4 , or 6 1 + 15 4 (which is less desirable, since it will produce a row in which every entry is divisible by 3) 54. 2 4 3 2 , or 4 4 6 2 (which is less desirable)

55. It will include a row of zeros. (Subtracting the two rows produces a row of zeros.) 56. It will include a row of zeros. 57. The claim is wrong. If there are more equations than unknowns, there can be a unique solution as well as row(s) of zeros in the reduced matrix, as in Example 6. 58. He is correct: There is at most one pivot per row, and since there are only 5 rows, there cannot be more than 5 pivots. So, at least one of the unknowns must be arbitrary. 59. Since there are 5 columns, there are 4 unknowns. (The last column is for the right-hand sides.) Since there are 5 rows of which 3 are zero, that leaves 2 rows with pivots. Thus, there are 2 unknowns that are not parameters. The remaining 2 unknowns are arbitrary (parameters). 60. Three: 6 unknowns minus 3 pivots 61. The number of pivots must equal the number of variables, since no variable will be used as a parameter. 62. The number of pivots must be less than the number of variables, since at least one variable will be used as a parameter. 63. A simple example is = 1; = 1; + = 2.

64. A simple example is: = 1; = 0; = 1. 65. It has to be the zero solution (each unknown is equal to zero): Putting each unknown equal to zero causes each equation to be satisfied because the right-hand sides are zero. Thus, the zero solution is in fact a solution. Because the solution is unique, this solution is the only solution. 66. No; By the preceding exercise, if the solution were unique, then that solution would have to be (0, 0, 0), which it is not.

67. No: As was pointed out in Exercise 65, every homogeneous system has at least one solution (namely, the zero solution) and hence cannot be inconsistent. 68. No: If a system has the zero solution, then substituting zero for each unknown satisfies every equation in the system. But substituting zero for each unknown causes the left-hand sides to be zero, and hence the righthand sides must also be zero, meaning that the system was homogeneous to begin with.


Solutions Section 4.3 Section 4.3

1. Unknowns: = the number of batches of vanilla, = the number of batches of mocha, = the number of batches of strawberryArrange the given information in a table with unknowns across the top: Vanilla Mocha Strawberry Avail. ( ) ( ) ( ) Eggs

2

1

1

350

Milk

1

1

2

350

Cream

2

2

1

400

We can now set up an equation for each of the items listed on the left: 2 + + = 350 + + 2 = 350 2x + 2y + z = 400.

Eggs: Milk: Cream: 2 1 2

1 1 350

2

1 2 350 2 2 1 → 0 2 1 400 3 1

1 0 0 1

0

1 3

350

0

1 3

1 1 1

1 350 1 2

3 350 0

1 0

2 + 3 3 → 0 1

2 0 2

→ 0 1

50 3 2

0 0

0

3

350

→ 0 1 0

50

(1 2) 1

0 0 3 300 (1 3) 3

100

1 0 0 100

50

0 0 1 100 3 0 0 1 100 0 0 1 100 = 100, = 50, = 100 Solution: Make 100 batches of vanilla, 50 batches of mocha, and 100 batches of strawberry.

2. Unknowns: = the number of plain burgers, = the number of double cheeseburgers, = the number of regular cheeseburgers Plain Double Cheese Regular Cheese Avail. ( ) ( ) ( ) Beef patties

1

2

1

19

Buns

1

1

1

13

Cheese slices

0

2

1

15

Beef patties: + 2 + = 19 Buns + + = 13 Cheese slices: 2 + = 15 1 1 0

1

2 1 19

1

1 1 13 2 1 → 0 2 1 15

0

0

4

0

2

1 2

1 0 0 4

1 19 1 + 2 2 0 6

1 15 3 + 2 2

1

0

→ 0 1

0 1 0 6 2 → 0 1 0 6 = 4, = 6, = 3

0

0

1

0

1

7

6 3

1 3

0 0 1 3 0 0 1 3 Solution: Make 4 plain burgers, 6 double cheeseburgers, and 3 regular cheeseburgers.


Solutions Section 4.3 3. Unknowns: = the number of sections of Finite Math, = the number of sections of Applied Calculus, = the number of sections of Computer Methods We are given three pieces of information: + + = 6. 40 + 40 + 10 = 210. 40,000 + 60,000 + 20,000 = 260,000 40 + 60 + 20 = 260.

(1) There are a total of 6 sections: (2) The total number of students is 210: (3) The total revenue is $260,000: or, working in thousands of dollars,

Thus, we have a system of 3 equations in 3 unknowns: + + =6 40 + 40 + 10 = 210 40 + 60 + 20 = 260. 1 1 1 6 1 1 1 6 40 40 10 210 (1 10) 2 →

4

1

1

40 60 20 260 (1 20) 3 1 1 0 0 0 1

1

1

1

1 0 0 3 0 0 1 1

6

1 1

1 + 2 3 2

→ 0 0

2

0

1 1

1

6

4 1 21 2 4 1 → 0 0 3 3 (1 3) 2 → 3 1 13 3 2 1

1

= 3, = 2, = 1

0

5

1 1 0

2

1 3

0 1 1

1 0

0

3

1

→ 0 0 1 1 2 → 0 1

0

2

0 1 0 2 Solution: Offer 3 sections of Finite Math, 2 sections of Applied Calculus, and 1 section of Computer Methods.

4. Unknowns: = the number of sections of Ancient History, = the number of sections of Medieval History, = the number of sections of Modern History Sections: Students: Professors: 1 2 1

1

+ + = 45 100 + 50 + 200 = 5,000 2 + + 4 = 100 + + 2 = 60

1 1

45

0

10

1

1 4 100 2 2 1 → 0 1 2

0

60 3 1

0

1

1 0

1 0 0 10 → 0 1 0 20 2 0 1 0 20

1 45 1 + 2 2 10

1 15

1

0

→ 0 1 0

0

= 10, = 20, = 15

3

2

1

55 1 3 3

10 2 2 3 →

15

0 0 1 15 0 0 1 15 Solution: Offer 10 sections of Ancient History, 20 of Medieval, and 15 of Modern.

5. Unknowns: = revenue (in millions) earned though subscription streaming, = revenue (in millions) earned through non-subscription streaming, = revenue (in millions) earned through non-streaming sales. Total revenues were $540 million: + + = 540 Subscription streaming brought in $160 million more than non-subscription streaming. Reword this as follows: The revenue earned from subscription streaming was $160 million more than the revenue earned from non-subscription streaming: = + 160, or = 160. Non-subscription streaming brought in nine times as much as non-streaming sales. Reword this as follows: The revenue earned from non-subscription streaming was nine times the revenue earned from


non-streaming sales: = 9 , or 9 = 0. We thus solve the system + + = 540 = 160 9 = 0. 1

1

1

1

→ 2

0

0

1

160 2 1 → 0

1

700

9

0 2

1

0

1

0

1

0

540

Solutions Section 4.3

1

1 0 0 340 0 1 0 180

0

1 + 3

0

1

2 2

1

0

380 2 3 → 0 2 20

0

0

1

540

2 1 + 2

0

680

(1 2) 1

1 380 9 0

1

= 340, = 180, = 20

0

2 3 + 2

2

0

1

0

→ 0 2 0

0

360 (1 2) 2 → 0 1 20

0

0

1

700

1

380

0

340

19 380 (1 19) 3

0

1

180 2 → 20 3

0 0 1 20 Solution: Revenues were $340 million from subscription streaming, $180 million from non-subscription streaming, and $20 million from non-streaming sales.

6. Unknowns: = revenue (in millions) earned though subscription streaming, = revenue (in millions) earned through non-subscription streaming, = revenue (in millions) earned through non-streaming sales. Total revenues were $660 million: + + = 660. Subscription streaming brought in $240 million more than non-subscription streaming. Reword this as follows: The revenue earned from subscription streaming was $240 million more than the revenue earned from non-subscription streaming: = + 240, or = 240. Subscription streaming brought in twice as much as the other two sales avenues combined. Reword this as follows: The revenue earned from subscription streaming was twice the sum of the sales from the other two avenues: = 2( + ), or 2 2 = 0. We thus solve the system + + = 660 = 240 2 2 = 0. 1 1 1 660 1 1 1 660 1 1 1 660 2 1 + 2 1 1

→ 2

1

0

240 2 1 → 0 2 1 420

0

1

900

2 2

0 2 0

0

1

1

1 0 0 440 0 1 0 200

0

3 1

1 + 3

0 3 3 660 (1 3) 3 2

0

420 2 3 → 0 2 20

0

0

0

0

1

= 440, = 200, = 20

880

→ 0

(1 2) 1

0

2

1

1

0

400 (1 2) 2 → 0 1 20

0

0

1 420

1 220 2 3 2 0

0

1

440

200 2 → 20 3

0 0 1 20 Solution: Revenues were $440 million from subscription streaming, $200 million from non-subscription streaming, and $20 million from non-streaming sales.


Solutions Section 4.3 7. Unknowns: = the number of Airbus A330-300s, = the number of Boeing 767-300ERs, = the number of Boeing Dreamliner 787-9s

Passengers: 330 + 270 + 240 = 4,980 Cost: 260 + 220 + 290 = 4,760 The number of Boeings is twice the number of Airbuses: + = 2 , or 2 = 0. Solving: 2 0 2 1 1 0 1 1 330 270 240 4980 (1 30) 2 → 11 260 220 290 4760 (1 10) 3 2 1 1 0 29

0

26 22

2 1 1

27 332

→ 0

0

29 27 332

0

29 0 116 (1 29) 2 → 0 1 0 4

0 35

58 0

29

0 0 0

42 476 (1 7) 3

2 332 (1 2) 1

39 312 (1 39) 3

0

29

29 0

5

0

0 174 (1 29) 1

9

0

8

29 476 3 13 1

0

27 332 6

166 2 2 11 1 →

29 1 + 2

68 29 3 5 2

1 166 1 + 3

29 27 332 2 27 3 → 0

1 0 0 6

1

8

= 6, = 4, = 8

0 0 1 8 0 0 1 8 Solution: Order 6 Airbus A330-300s, 4 Boeing 767-300ERs, and 8 Dreamliners. 8. Unknowns: = the number of Airbus A330-300s, = the number of Boeing 767-300ERs, = the number of Boeing Dreamliner 787-9s

Passengers: 330 + 270 + 240 = 2,940 Cost: 260 + 220 + 290 = 2,540 The number of Airbuses is equal to the number of Boeings: = + or = 0. 1 0 1 1 1 0 1 1

Solving: 330 270 240 2940 (1 30) 2 → 11 260 220 290 2540 (1 10) 3

1 1 1 20

0

20 19 98 2 19 3 →

20 0

48

0 0

98

20 1 + 2

0 0

19

0

55 254 5 3 12 2

1 98 1 + 3 1

1 0 0 5 0 1 0 3

2

20

20 0 0

= 5, = 3, = 2

0 0

0

26 22

0

1 98

20 19 98 0

8

98 2 11 1 →

29 254 3 26 1

47 94 (1 47) 3

0 100 (1 20) 1

20 0 0

9

1

60 (1 20) 2 → 2

0 0 1 2 Solution: Order 5 Airbus A330-300s, 3 Boeing 767-300ERs, and 2 Dreamliners.

9. Unknowns: = the number of tons from CCC, = the number of tons from SSS, = the number of tons from BBF


Solutions Section 4.3

Total order of cheese is 100 tons: + + = 100. Total cost = $5,990: 80 + 50 + 65 = 5,990 Same amount from CCC and BBF: = , or = 0 Solving: 1 1

1

100

1

1

100

1

1

66

2

1

1

100

1

100

1

66

80 50 65 5990 (1 5) 2 → 16 10 13 1198 2 16 1 → 1

0

1

1

2

0

2

0

1

0

1

0 6 3 402 (1 3) 2 → 0 0 1 2 100 0 2 1 134 0

0

3

0 2

0

0

0

0

1

1 0 0 22 0 1 0 56

0

0

1

1

2 1 0

1 134 1

112 (1 2) 2 → 0 1

0

44

22

(1 2) 1

0

0

1

0

0

0

= 22, = 56, = 22

3 1

2 1 + 2

2 100 2 3 2

→ 0 2

66 (1 3) 3

0

1 0

1 + 3

134 2 3 → 22

22

56 2 →

1 22 3

0 0 1 22 Solution: The store ordered 22 tons from Cheesy Cream, 56 tons from Super Smooth & Sons, and 22 tons from Bagel's Best Friend.

10. Unknowns: = the number of tons from CCC, = the number of tons from SSS, = the number of tons from BBF Total order of cheese is 36 tons: + + = 36. Total cost = $2,310: 80 + 50 + 65 = 2,310 Two more tons of cream cheese came from BBF. than from SSS: = + 2 or + = 2 Solving: 1 1 1 36 1 1 1 36 80 50 65 2310 (1 5) 2 → 16 10 13 462 2 16 1 → 0

1

1

2

1

1

1

36

2

0

1

34

2

0

0

20

0

1

0 6 3 114 (1 3) 2 → 0 0 1

1

2

0 2 1 38 0

0

3

42

(1 3) 3

(1 2) 1

2

0

1

1

1

2 1

0

1

1

2

36

1 38 1

34

2

2 1 + 2 2 3 2

1 3

→ 0 2 1 38 2 + 3 → 1

0

0

0

0

1

10

14

0 2 0 24 (1 2) 2 → 0 1 0 12 2 → 0

0

1

14

0

0

1

14


1 0 0 10

Solutions Section 4.3 = 10, = 12, = 14

0 1 0 12

0 0 1 14 Solution: The store ordered 10 tons from Cheesy Cream, 12 tons from Super Smooth & Sons, and 14 tons from Bagel's Best Friend.

11. Unknowns: = the number of evil sorcerers slain, = the number of trolls slain, = the number of orcs slain Total number slain was 560: + + = 560. Total number of sword thrusts was 620: 2 + 2 + = 620. The number of trolls slain is five times the number of evil sorcerers slain: = 5 , or 5 + = 0. Solving: 1 1 1 560 1 1 1 560 1 + 2 2

2 1 620 2 2 1 → 0 0

5 1 0

0

3 + 5 1

0 6

300

(1 6) 3

1 1

0

60

0 0 1 500 0

1 0

0

10

1

0 6 1

5

1

→ 0

0

0

0

500

2800 3 + 5 2

50

60

1 500

1

0

1 0 0

10

0 0 1 500 2 → 0 0 1 500

1 3

= 10, = 50, = 500

0 1 0 50 0 1 0 50 Solution: Halmar has slain 10 evil sorcerers, 50 trolls, and 500 orcs.

12. Unknowns: = the number of ounces of rose oil, = the number of ounces of fermented prune oil, = the number of ounces of alcohol Total amount in a bottle is 22 ounces: + + = 22. Each bottle contained 4 more ounces of alcohol than oil of fermented prunes: = + 4, or + = 4. The amount of alcohol was equal to the combined volume of the other two ingredients: = + , or + = 0. Solving: 1 1 1 22 1 1 1 22 0

1

0

1

1

1 0 1

1

1 0

0

0 1 0

0

1

1 1

1

4

0 3 1 22 4

1 11

0

0

4

→ 0 1

1 + 2

0

1

0

→ 0 1 0

1 0 0

7 2 → 0 1 0

1 11 3

0

1

0

4

2 22 (1 2) 3

4

2

1

1

26 4

11

1 + 2 3 2 + 3

7 = 4, = 7, = 11

0 0 1 11

Solution: Each bottle contained 4 oz. rose petal extract, 7 oz. oil of fermented prunes, and 11 oz. alcohol. 13. Unknowns: = amount of money donated to the MPBF, = amount of money donated to the SCN, = amount of money donated to the NY Jets Given information: (1) The society donated twice as much to the NY Jets as to the MPBF. Rephrase this as follows: The


Solutions Section 4.3 amount of money donated to the NY Jets was equal to twice the amount of money donated to the MPBF: = 2 , or 2 = 0. (2) The society donated equal amounts to the first two funds: = , or = 0. (3) Money donated back to the society: + 2 + 2 = 4,200 Solving: 2 0 1 0 2 0 1 0 1 1

1 2

0

2

4200 2 3 1

7

8400 (1 7) 3

2

0

1

2

0

0

0 2 0

0

2 2 1 → 0

0

1

0

0

(1 2) 1

1200

2

0

0

→ 0 2 0

0

1

0

2 4

1 1

1

0

1

5

0

0

8400 3 + 2 2

1 + 3

2 3 →

0

1200

600

0 2 0 1200 (1 2) 2 → 0 1 0 600 2 → 0

0

1 0 0 0 1 0

1

1200

0

600 600

0

1 1200

0 0 1 1200 Solution: It donated $600 to each of the MPBF and the SCN, and $1,200 to the Jets.

14. Unknowns: = number of good reviews, = number of bad reviews, = number of mediocre reviews, = number of reviews left blank Given information: There are 280 more bad reviews than good ones: = 280. There are half as many blank reviews as bad ones: = 1 2 , or 2 = 0. The good reviews and blank reviews together total 170: + = 170. Removing 280 of the bad reviews leaves him with a total of 400 reviews of all types: + ( 280) + + = 400, or + + + = 680. 1 0

1 0

0

1 0 2

280 0

0 0

1

170 3 + 1

1 0 0

2

280

1 1

0

0

0

1 1

1

1 0 2 0 0

0 1

3

5

680 4 + 1 0

450 (1 3) 3

960

1 0 0 0 20 1 0 0

1 0 0 300 0 0 1 150

1

1 0

1

0

0

0

0 2

1 2

1 0 0 0 0 0

0

0 1

2

1 0 2 0 0 0 1

1 0 0 0

20

0 1 0 0 300 0 0 0 1 150

1 5

1 1

280 1 2 0

450 3 2

960 4 2 2

280 1 2 3 0

150

2 + 2 3

960 4 5 3

3 4 →

1 0 0 0

20

0 1 0 0 300 0 0 1 0 210

0 0 1 0 210 0 0 0 1 150 0 0 1 0 210 Solution: There were originally 20 good reviews, 300 bad ones, 210 mediocre ones, and 150 blank ones. 15. Unknowns: = the number of empty seats on United, = the number of empty seats on American, = the number of empty seats on Southwest We are given the following information: (1) United, American, and Southwest flew a total of 210 empty seats: + + = 210.


Solutions Section 4.3 (2) The total cost of these seats was $108,900. (Note that the figures in the table are for 1,000 miles, but the trip was 3,000 miles): 3(181) + 3(197) + 3(134) = 108,900. That is, 543 + 591 + 402 = 108,900. (3) United had three times as many empty seats as American: = 3 , or 3 = 0 We use the Pivot and Gauss-Jordan Tool on the Website: →

→ Solution: United: 120; American: 40; Southwest: 50

16. Unknowns: = the number of empty seats on United, = the number of empty seats on JetBlue, = the number of empty seats on Southwest We are given the following information: (1) United, JetBlue, and Southwest flew a total of 300 empty seats: + + = 300. (2) The total cost of these seats was $86,500. (Note that the figures in the table are for one mile, but the trip was 2,000 miles): 2(181) + 2(197) + 2(134) = 86,500. That is, 362 + 246 + 268 = 86,500. (3) JetBlue had three times as many empty seats as Southwest: = 3 , or 3 = 0. We use the Pivot and Gauss-Jordan Tool on the Website: →

→ Solution: United: 100; JetBlue: 150; Southwest: 50

17. Unknowns: = amount invested in SHPIX, = amount invested in RYURX, = amount invested in RYCWX + + = 9,000. = , or = 0. 0.17 + 0.16 = 1,170.

The total investment was $9,000: You invested an equal amount in RYURX and RYCWX: YTD loss from the first two funds was $1,170: Solving: 1 1 0

1

0.17 0.16 1 0

1 1

1

1

1

1 0

9000 0

1170 100 3

9000 0

0 1 17 36000 3 + 2

0

1

1

1

1

→ 0 1

1

1 2

1

9000 0

117000 3 17 1

0

0 0 18 36000 (1 18) 3

17 16 1 0

0

2

9000


1 0

2

0 1

9000

1

0 0

1

0

2000

1 0 0 5000 0 1 0 2000

1 + 2 3 2 3

Solutions Section 4.3

1 0

0

→ 0 1

5000

0

2000

0 0 1 2000 3

= 5,000, = 2,000, = 2,000

0 0 1 2000 Solution: You invested $5,000 in SHPIX, $2,000 in RYURX, $2,000 in RYCWX.

18. Unknowns: = amount invested in SHPIX, = amount invested in RYURX, = amount invested in RYCWX The total investment was $7,000: You invested an equal amount in SHPIX and RYURX: Total loss for the year was $1,410:

Solving: 1 0

0

2

1

2

1

1 1

1

1

1

0

7000 0

0.17 0.16 0.25 1410 100 3

22000 2 3 2

0

1

7000

1 7000 8

0

1

7000

0

0

2000

2 1 + 2

1 3

2

1

1

1

1

+ + = 7,000. = , or = 0. 0.17 + 0.16 + 0.25 = 1,410. 1

0

7000 0

17 16 25 141000 3 17 1

0

1

7000

→ 0 2 1 7000 2

0

0

2 1

0

17

51000 (1 17) 3

4000

(1 2) 1

0 2 1 7000 2 + 3 → 0 2 0 4000 (1 2) 2 → 0 1

0

1

3000

0

0

1

1 0 0 2000 → 0 1 0 2000 2 0 1 0 2000

3000

= 2,000, = 2,000, = 3,000

0 0 1 3000 0 0 1 3000 Solution: You invested $2,000 in each of SHPIX and RYURX and $3,000 in RYCWX.

19. Unknowns: = the number of shares of WSR, = the number of shares of TKOMY, = the number of shares of STX The total investment was $7,850. Investment in WSR = shares @ $10 = 10 Investment in TKOMY = shares @ $53 = 53 Investment in STX = shares @ $84 = 84 Thus, 10 + 53 + 84 = 7,850. You expected to earn $272 in dividends: WSR dividend = 4% of 10 invested = 0.04(10 ) = 0.4 TKOMY dividend = 4% of 53 invested = 0.04(53 ) = 2.12 STX dividend = 3% of 84 invested = 0.03(84 ) = 2.52 Thus, 0.4 + 2.12 + 2.52 = 272. You purchased a total of 200 shares: + + = 200. We therefore have the following system: + + = 200 10 + 53 + 84 = 7,850 0.4 + 2.12 + 2.52 = 272. Solving:


1

10

1

53

1

84

0.4 2.12 2.52 1

1

200

7850

200 43 1 2

1

0

43

74 5850

0

43

74

0

43

1 0

0

0 1

0 0

5850 2 + 74 3 →

1

100 50

50

0 0 1 50 3

1

200

10 53 63 6800 3 10 1

31 2750 1 31 3

0

1

→ 10 53 84 7850 2 10 1 →

53 4800 3 2

43

0

272 25 3

1

Solutions Section 4.3

43 0 0

0

43

43 0

0 0

1 0 0 100

→ 0 1 0

50

0 0 1

31 74

2750

5850

0

21 1050 (1 21) 3

43 0

0 0

1

4300 (1 43) 1

2150 (1 43) 2 → 50

50

Solution: You purchased 100 shares of WSR, 50 shares of TKOMY, and 50 shares of STX.

20. Unknowns: = the number of shares of WSR, = the number of shares of TKOMY, = the number of shares of STX The total investment was $13,150. Investment in WSR = shares @ $10 = 10 Investment in TKOMY = shares @ $53 = 53 Investment in STX = shares @ $84 = 84 Thus, 10 + 53 + 84 = 13,150. You expected to earn $484 in dividends: WSR dividend = 4% of 10 invested = 0.04(10 ) = 0.4 TKOMY dividend = 4% of 53 invested = 0.04(53 ) = 2.12 STX dividend = 3% of 84 invested = 0.03(84 ) = 2.52 Thus, 0.4 + 2.12 + 2.52 = 484. You purchased a total of 300 shares: + + = 300. We therefore have the following system: + + = 300 10 + 53 + 84 = 13,150 0.4 + 2.12 + 2.52 = 484. Solving: 1 1 1 300 1 1 1 300 10

53

84

0.4 2.12 2.52 1

1

1

13150

300

484

0

43

74 10150

0

43

74

0

43 0

43

1 0 0 1

53

0 0

31

0 0

1

25 3

43 1 2

9100 3 2

→ 10 53 84 13150 2 10 1 → 10 53 63 12100 3 10 1

2750 1 31 3

10150 2 + 74 3 →

100 150

50

0 0 1 50 3

43 0 0

0

43 0 0

1 0 0 100

→ 0 1 0 150 0 0 1

50

43 0

31

0

43 0

74

2750

10150

21 1050 (1 21) 3 0

0

1

4300 (1 43) 1

6450 (1 43) 2 → 50

Solution: You purchased 100 shares of WSR, 150 shares of TKOMY, 50 shares of STX.


Solutions Section 4.3 21. With , , , and as indicated, the first piece of information we have is that + + + = 95. The remaining equations must be written in standard form: 3 + 3 = 15 + = 75 + = 23 1 1 1 1 95 1 1 3 1

The augmented matrix of the system is

1

3

15

1 1 75

.

1 1 1 1 23 Using pivoting technology (for instance, press "reduce completely" in the Pivot and Guass-Jordan Tool on the Website), we obtain the solution = 53, = 32, = 6, = 4. Chrome: 53%, Safari: 32%, Edge: 6%, Firefox: 4%

22. With , , , and as indicated, the first piece of information we have is that + + + = 99. The remaining equations must be written in standard form: =0 + =9 5 5 = 14. 1 1 1 1 99 1 1 1 1 1

The augmented matrix of the system is

1

0

0

9

1

.

0 1 5 5 14 Using pivoting technology (for instance, press "reduce completely" in the Pivot and Guass-Jordan Tool on the Website), we obtain the solution: = 49, = 44, = 5, = 1. Safari: 49%, Chrome: 44%, Samsung Internet: 5%, Firefox: 1%

23. Since the percentage market shares add up to 100, the third equation is + + + = 100. If we rewrite the given equations in standard form, we get the second and third equations: =0 0.1 = 5. Row reduction: 1 1 1 1 100 1 1 1 1 100 1 1 1 0

1 0

0

2

0

1

0

10 1

1

2 0

0 10 0

0

0

1 0 0 0 1 0

1

0.1

0

5

1

100

1

100

2 1 100 0

0

50

10 3

2 1 + 2

1 0

0

2

1 2 3 5

50

50 45

0

10 1

50

0

1

0

0

1

2 1 → 100

→ 0 2 2 1 100 5 2 + 3 → 2

0

0

6 450 (1 2) 2 → 0 5

10 1

0

1 1

0

0

0

0

10

1

50

100

(1 2) 1

3 225 (1 5) 2 →

10 1

50

0 0 1 1 10 5 Translating back to equations (and converting to decimals) gives + 0.5 = 50 + 0.6 = 45 0.1 = 5. Solving for , , and in terms of gives the general solution: = 50 0.5 = 45 0.6

(1 10) 3


Solutions Section 4.3 = 5 + 0.1 arbitrary. We now answer the question: Which of the three companies' market share is most impacted by the share held by other companies? Since represents other companies, we look for which of , , or has the coefficient of with the greatest absolute value—this is , representing American Family. Thus, American Family is most affected by other companies.

24. Since the percentage market shares add up to 100, the third equation is + + + = 100. If we rewrite the given equations in standard form, we get the second and third equations: 4 = 35 2 + = 15. Row reduction: 1 1 1 1 100 1 1 1 1 100 2 1 + 2 1 0

1 4 2

1

0

1

2

0

3

1

4

0

0

5

35 2 1 → 0 15

0

65

0 2 5 1 135 0

0

0 4 0

0

4 2 120 (1 2) 3 0

3

310 (1 4) 1 30

2

2

5 1 135

2

1

0

3

→ 0 2 0

0

1 0 0

5 2

1 1

15

65

3 + 2

2 1 3 3

1 135 2 2 5 3 → 1

5 4

60

155 2

(1 4) 2 → 0 1 0 3 4 15 2

2 1 60 (1 2) 3

0 0 1

1 2

30

Translating back to equations (and converting to decimals) gives + 1.25 = 77.5 0.75 = 7.5 + 0.5 = 30. Solving for , , and in terms of gives the general solution: = 77.5 1.25 = 7.5 + 0.75 = 30 0.5 arbitrary. We now answer the question: Which of the three companies' market share is most impacted by the share held by other companies? Since represents other companies, we determine which of the four unknowns has the coefficient of with the greatest absolute value—this is , representing State Farm. Thus, State Farm is most impacted by other companies. 25. Unknowns: = the number of books sent from Brooklyn to Long Island, = the number of books sent from Brooklyn to Manhattan, = the number of books sent from Queens to Long Island, = the number of books sent from Queens to Manhattan We represent the given information in a diagram:

Note that, since a total of 3,000 books are ordered and there are a total of 3,000 in stock, both warehouses need to clear all their stocks. Books from Brooklyn: + = 1,000

Books from Queens: + = 2,000


Books to Long Island: + = 1,500

Books to Manhattan Order: + = 1,500

Solutions Section 4.3

a. Transportation budget: 5 + + 4 + 2 = 9,000 We have five equations in four unknowns. Solving: 1 0 1 0 1500 1 0 1 0 1500 0

1 0 1 1500

5

1 4 2 9000 5 5 1

1 0

1 0 0 1000 3 1 0 1 1 2000

1 0 0 1

1 0

1500

1 + 3

1

2000

4 + 3

1500

0 0

1

1 2000

2 0

0

0

0

2

0 0

0 0 0 2

1

1 0

0

1

0

1

0

1

→ 0

0 1

0

5 3

0

1 0

0

1 1500

1 0 500 3 2 → 1

1 2000

1 2 1500 5 2

1 0 0 1

0 0

1 1

1500

2 2 5

→ 0 0 1 1 2000 2 3 + 5 → 0 0 0 0

0 0

0 2

1000

(1 2) 1

1 0 0 0

2000

(1 2) 5

0 0 0 1 1000

1000

500 2 1 + 5

(1 2) 2

0 1 0 0

500 500

0

2000

0 0 2 0 2000 (1 2) 3 → 0 0 1 0 1000 Rearrange rows → 0 0 0 0 0 0 0 0 0 0 0 0

1 0 0 0 0 1 0 0

500 500

0 0 1 0 1000 0 0 0 1 1000

0 0 0 0 0 Solution: Brooklyn to Long Island: 500 books; Brooklyn to Manhattan: 500 books; Queens to Long Island: 1,000 books; Queens to Manhattan: 1,000 books. b. If we remove the equation that says that the total cost is $9,000, and solve, we get 1 0 1 0 1500 1 0 1 0 1500 0 1 0

1 0 1 1500

1 0 0 1000 3 1 0 1 1 2000

1 0 0 1 0 0

0 0

1

0

0

1

1 1

1500 1500

1 2000 1

2000

1 0 0 1 500 0 1 0 0 0 1

1 1

1500 2000

1 + 3 4 + 3

0 0 0

1 1 0

0

1 1500

1 0 500 3 2 1

1 0 0 1

1 2000

0 0

1 1

500 1500

0 0 1 1 2000 3 0 0

0

0

0

0 0 0 0 0 Translating back to equations gives: = 500, + = 1500, + = 2000. The general solution is = 500, = 1,500 , = 2,000 . The total cost is then


Solutions Section 4.3 = 5 + + 4 + 2 = 5( 500) + 1,500 + 4(2,000 ) + 2 = 7,000 + 2 . To decrease the cost, we should therefore make as small as possible. We cannot set = 0, as that would result in = 500 being negative; the smallest we can make is 500. Using this value for gives the following solution: Brooklyn to Long Island: 0 books; Brooklyn to Manhattan: 1,000 books; Queens to Long Island: 1,500 books; Queens to Manhattan: 500 books for a total cost of 7,000 + 2(500) = $8,000.

26. Unknowns: = number of boards sent from Tucson to Honolulu, = number of boards sent from Tucson to Venice Beach, = number of boards sent from Toronto to Honolulu, = number of boards sent from Toronto to Venice Beach We represent the given information in a diagram:

a. Tucson: + = 620 Toronto: + = 410 + = 530 Budget: 10 + 5 + 20 + 10 = 10,200 1 1 0 0 1 1 0 0 620 0 1 0

0 0 1

1 1 0

1

410

0

500

1

530

10 5 20 10 10200 (1 5) 5 1

1

0

1

0

0

0

0

620

1

530

1

410

1

1

4

2

410

0

0

0

0 1 0 0 0

0

0

0

0

1

0

1

0

0

1

1

0

800

1 1

1 0 0 0 400 0 0 1 0 100

90

5 4 2

2 + 5

0 0

0

1 0

1

0

0

530

1

1 + 3

530

4 + 3

410

0

0 2 840 5 3

0 1

0 0

0

0

400 100

220 3 → 0

0 1 310 5

1 0 0 0 400 0 1 0 0 220

0 1 0 0 220 Rearrange rows → 0 0 1 0 100 0 0 0 0

620

0 1 530

530 3 5 → 0 1 0 0 0 0 0 310

500 3 1

1

1

0

0

1

1 5

0

0

1 4 2 2040 5 2 1

1 0 1

1

1

0 1

2

0

410

0 1 0

→ 0

1

0

0 1 1

1

Venice Beach:

620

0

0 120 3 2

0 1

Honolulu: + = 500

0 0 0 1 310

0 0 0 1 310 0 0 0 0 0 Solution: Tucson to Honolulu: 400 boards per week; Tucson to Venice Beach: 220 boards per week; Toronto to Honolulu: 100 boards per week; Toronto to Venice Beach: 310 boards per week. b. If we remove the budget equation and row-reduce, we have:


1

1 0 0 620

0

0 1 1 410

1

1

1 0

0 0

0

0 1 0 500 3 1 1 0 1 530 0

1 1

0

0

1

1

410

0 1 530

0

1 0 0 1 0 0 1

620

0 1 0

1

1

1

90

530

410

1 →

1 + 3

1

0

0

0

0

1

1

0

0 1

1

0

0

0

1

0

1

410

0 120 3 2

0 1

4 + 3

620

Solutions Section 4.3

1

530

1

1

90

410

0 1 0 1 530 3 0

0

0

0

0

530

0 0 0 0 0 Translating back to equations gives: = 90, + = 410, + = 530. General solution: = 90 + , = 530 , = 410 , arbitrary. Thus, the cost is = 10(90 + ) + 5(530 ) + 20(410 ) + 10 = 11,750 5 . The largest can be so as to have nonnegative answers for the other unknowns is 410, giving: Tucson to Honolulu: 500 boards per week; Tucson to Venice Beach: 120 boards per week; Little Rock to Honolulu: 0 boards per week; Little Rock to Venice Beach: 410 boards per week. Minimum cost is = 11,750 5(410) = $9,700.

27. a. Unknowns: = the number of tourists from North America to Australia, = the number of tourists from North America to South Africa, = the number of tourists from Europe to Australia, = the number of tourists from Europe to South Africa We represent the given information in a diagram:

To avoid all the zeros, let us measure all these quantities in thousands.

1

North America: + = 1,460 Australia: + = 2,670 1 0 0 1460

0

0 1 1 3210

1

1

1 0

0 0

0

0 1 0 2670 3 1 1 0 1 2000 0

1 1

0 1

0 1

1460 3210

0 1 2000

0

1

2000

1 + 3 4 + 3

1 0

Europe: + = 3,210 South Africa: + = 2,000

1 0

0

1

1 3210

0

0 1

0 1

1

0

0

0

1

1

0 1460

0

0 1210 3 2 1 2000 1

1

540 3210

0 1 0 1 2000 3 0

0

0

0

0


1 0 0 1 540

Solutions Section 4.3

0 0 1

1

3210

0

0 1

1

3210

0

0

0

0 1 1

90

0

0

0 1 0

1

2000

0 0 0 0 0 This system has infinitely many solutions, which is why the given information is not sufficient to determine the number of tourists from each region to each destination. b. We are told that + + + = 4,670. However, this equation can be obtained by adding the first two equations in part (a). Thus, the new equation gives us no additional information, and so the associated system of linear equations will have the same infinite solution set as part (a). c. The additional information is, rephrased: The number of tourists from Europe to Australia = 90,000 more than the number of tourists from Europe to South Africa: = + 90, or = 90. Adding this to our list of equations and solving: 1 1 0 0 1460 1 1 0 0 1460 1 0

0 1 1 0

1

1

0

0

1

0

0

0

1

0 1

0

2670 3 1 → 0 1

0

1460

1

3210

1

0

0

1

0

0

0

0

0

1

0 1 0 0

0

2000

0 2 3120 (1 2) 5

1 0

0

2000

0 1 0 1 2000 0

0

1

3210

2 + 5

0 0

1

1

1 0

1 1 0

1

1 0

0 0

0 1

1

1560

0

0

3210

1210 3 2 →

0

0 1

1

1

0 1

2000

5 2

90

1460

1 + 3

2000

4 + 3

3210

0 1 2000

0

1

0 1 1560 0 1

2000 3 5 → 0 1 0 0 0 0 0

1

0 0 1 0 1650 0 0 0 0

1

0 → 0

540 1 5

1 0 0 0 1020 0 1 0 0

1

0 0

0

0

1020

1650

440 3 → 0

0 1 1560 5

1 0 0 0 1020 0 1 0 0

440

440 Rearrange rows → 0 0 1 0 1650 0

0 0 0 1 1560

0 0 0 1 1560 0 0 0 0 0 Since this is a unique solution, we can determine the numbers from each country to each destination: North America to Australia: 1,020,000, North America to South Africa: 440,000, Europe to Australia: 1,650,000, Europe to South Africa: 1,560,000.

28. a. Unknowns: = the number of tourists from North America to Australia, = the number of tourists from North America to South Africa, = the number of tourists from Europe to Australia, = the number of tourists from Europe to South Africa We represent the given information in a diagram:


Solutions Section 4.3

To avoid all the zeros, let us measure all these quantities in thousands.

1

Total: + + + = 1,220 Australia: + = 680 1 1 1 1220

1

1 0 0

330 2 1

1

1

0

1 0

0

0 0

0 1 0 1 0 1 0

1 1

0

680 3 1 540

330

1 1 890 0

1 540

1

890

0

1

540

1 0 0 1 210 0 0 1 0 1 0

1

North America: + = 330 South Africa: + = 540

1

1

0

1 + 3 4 + 3

0

0 1 0

1

1 0

1

1220 1 + 2

1

1 890

0

0

0

0

0

0 1 0

1

0

1 540 1

540

1 210

1 1 890 2 0

0

1 540 3 0

0

540

0 0 0 0 0 This system has infinitely many solutions, which is why the given information is not sufficient to determine the number of tourists from each region to each destination. b. Including the additional equation + = 890 adds no new information, since this equation can be obtained by subtracting the second equation from the first in part (a). (Also notice that it is the equation represented by the second row of the reduced matrix above.) c. Given that = 450, we can take the general solution from part (a) and compute , , and : = 210 = 450 210 = 240 = 540 = 540 450 = 90 = 890 = 890 450 = 440. Solution: North America to Australia: 240,000, North America to South Africa: 90,000, Europe to Australia: 440,000, Europe to South Africa: 450,000 29. Used Alcohol Alcohol-Free Totals 10th grade 12th grade Totals

30,150

79,850

50,000 60,000

a. Using the row and column totals, we get four equations: 10th grade: + = 50,000

12th grade: + = 60,000


Used Alcohol: + = 30,150

Alcohol-free: + = 79,850. Solutions Section 4.3

Row-reducing the augmented matrix gives: 1 1 0 0 50000 1 1 0

0 1 1 60000

1

1

1 0

0 0

0

0 1 0 30150 3 1 1 0 1 79850 0

1 1

0

0

1

1

50000 60000

0 1 79850

0

1

79850

1 0 0 1 29850 0 0 1 0 1 0

1 1

60000

0

1 + 3 4 + 3

0

0

1

0

0 1 29850

1

0

0

1

60000

0 19850 3 2

0

1

50000

1

0 1 0

0 1

79850

1

1

60000

0 1 0 1 79850 3

0

0

0

0

0

79850

0 0 0 0 0 The row-reduced matrix shows that there are infinitely many solutions, and hence no unique solution. Thus, the given data are insufficient to obtain the missing data ( , , and ). b. Twice as many 12th graders were alcohol-free than used alcohol. Rephrase as: The number of 12th graders who were alcohol free was twice the number of 12th graders who used alcohol: = 2 , or 2 + = 0. If we include this additional equation, we get the system + = 50,000 + = 60,000 + = 30,150 + = 79,850 2 + = 0. 1 1 0 0 50000 1 1 0 0 50000 0

0

0

0 2 1

1 0

0 1

1 1 0

1 60000 0

0

0

50000

0

1

0

1

79850

1

1

60000

0 1 0 1 79850 0

0

0

3

120000 (1 3) 5

1

0

0 1 29850 1 + 5 1

1

60000

0

0

0

1

40000

0

0

1

0

0

2 1

0

1 79850

1 0

0

0 30150 3 1 → 0 1

1 0

0

2 5

1 0

1

1

0 19850 3 2

1

1 0

0

0

1

1

→ 0

1

1

0

0 0

0

0

0 1

0 0

0

1 0

0

60000

0 1

79850 0

5 + 2 2

50000

1 + 3

79850

4 + 3

60000

0 1 79850

0 0

1 0

1 1

40000

10150 20000

0 1 0 1 79850 3 + 5 → 0 1 0 0 39850 3 → 0 0 0 0 0 0 0 0 0 0 40000


1 0 0 0 10150

1 0 0 0 10150

Solutions Section 4.3

0 0 1 0 20000

0 1 0 0 39850

0 1 0 0 39850 Rearrange rows → 0 0 1 0 20000 0 0 0 0

0

0 0 0 1 40000

0 0 0 1 40000 0 0 0 0 0 Thus, the missing data is: = 10,150, = 39,850, = 20,000, = 40,000. 30. Vaped Vaping-Free Totals

8th grade

12th grade Totals

40,000

78,080

100,000

60,000

a. Using the row and column totals, we get four equations: 8th grade: + = 40,000 Vaping-free: + = 78,080

12th grade: + = 60,000 Overall Total: + + + = 100,000

Row-reducing the augmented matrix gives: 1 1 0 0 40000 1 1 0

0 1 1

1

1

0 1

0 0

1 0 1

60000 78080

1 1 1 100000 4 1

0 1

0 0 40000 1 3 1 1 60000

0 1 78080

0 0 0 1 0 0

0

0 40000

1

1 60000

0 1

1 78080

1 60000 4 2

1 0 0 1 38080 →

0 0 1 0 1 0

1

60000

1

78080

0 0 0 0 0 0 0 0 0 0 The row-reduced matrix shows that there are infinitely many solutions, and hence no unique solution. Thus, the given data are insufficient to obtain the missing data. b. 11,920 more 12th graders vaped than 8th graders: = + 11,920, or + = 11,920. Adding this extra equation to the system and solving gives: 1 1 0 0 40000 1 1 0 0 40000 0 0 1

0 1 1 1 0 1

1

1

0

0

0 0

78080

1 1 1 100000 4 1

1 0 1 0 0

60000

0 1 1

0 1

0

0

0 1

1

0

11920 5 + 1

40000 1 3 60000

78080 0

0 1 8080 5 3

0 0

1

1 60000

0 1

1

0 51920 5 2

→ 0 1 0 0

0 1

1 78080

1 60000 4 2

1 0 0

1

0 0 0

0

0 0 1

→ 0 1 0 0 0 0

1 1

2

38080 2 1 5 60000 78080 0

86160

2 2 + 5

2 3 + 5 →


2 0 0

0

10000

0 0 0

0

0

0 0 2 0 2 0

0 0

33840 70000

(1 2) 1

Solutions Section 4.3

(1 2) 2

1 0 0 0

5000

0 0 0 0

0

1 0 0 0

0 0 1 0 16920

(1 2) 3

0 0 0 2 86160 (1 2) 5

5000

0 1 0 0 35000

→ 0 1 0 0 35000 Rearrange rows → 0 0 1 0 16920 0 0 0 1 43080

0 0 0 1 43080

Solution: ( , , , ) = (5,000, 35,000, 16,920, 43,080)

0 0 0 0

0

31. = daily traffic flow along Eastward Blvd., = daily traffic flow along Northwest La., = daily traffic flow along Southwest La.

a. Intersection A: Traffic in = Traffic out: 150 + = , or = 150 Intersection B: Traffic in = Traffic out: = 200 + , or = 200 Intersection C: Traffic in = Traffic out: 50 + = , or = 50

This gives us a system of three linear equations: = 150 = 200 = 50. Solving: 1 1 0 150 1 1 0 150 1 + 2 1

0

0

1

1 200 2 1 → 0 1

50

0

1 1

1 1

50

50 3 2

1 0 1 200

→ 0 1 1 0 0

0

50 0

Since there are infinitely many solutions, it is not possible to determine the daily flow of traffic along each of the three streets from the information given. Translating the row-reduced matrix back into equations gives = 200, or = 200 + , = 50, or = 50 + . Thus, the general solution is: = 200 + , = 50 + , 0 arbitrary, where is traffic along Southwest Lane. Thus, it would suffice to know the traffic along Southwest to obtain the other traffic flows. b. If we set = 60, we obtain, from the general solution in part (a), = 200 + 60 = 260 vehicles along Eastward Blvd., = 50 + 60 = 110 vehicles along Northwest La., = 60 vehicles along Southwest La. c. From the general solution in part (a), the traffic flow along Northwest La. is given by = 50 + . Since 0, the value of must be at least 50 vehicles per day.


Solutions Section 4.3 32. = traffic flow along April, = traffic flow along Division, = traffic flow along Broadway, = traffic flow along Embankment

a. Intersection A: Traffic in = Traffic out: 300 = + , or + = 300 Intersection B: Traffic in = Traffic out: + = 300 Intersection C: Traffic in = Traffic out: = 100 + , or = 100 Intersection D: Traffic in = Traffic out: 100 + = , or + = 100 Solving: 1 0 0 1 0

1

1

1 0 0 1 0 0 0 0

1

0

0

1 0

1

0 1 0

1 300

0 100 3 1

1 1 100

1

1 0 0

0

0 300

1

0 300 1 + 3 1 300

1

1

1 100

1 100 4 3 400 300

0 0 1 1 100

0

0

1

1

0

0

1

300

300

0 1 1 0 200 3 + 2 0

1 0 0 1

0

0 0

1 1

1 400 1 300

100

0 0 1 1 100 3 0 0

0

0

0

0 0 0 0 0 General Solution: = 400 , = 300 , = 100 + , arbitrary, but 100 300. Since there are infinitely many solutions, it is not possible to determine the number of vehicles on each street per hour. b. Measuring the traffic flow on any of April, Broadway, Division, or Embankment will determine the answer. c. From the general solution in part (a), the traffic flow along April St. is given by = 400 . Since the largest possible value of is 300, the smallest value of is 400 300 = 100 vehicles per hour.


Solutions Section 4.3 33. = traffic on middle section of Bree, =traffic on middle section of Jeppe, =traffic on middle section of Simmons, = traffic on middle section of Harrison

a. Intersection A: Traffic in = Traffic out: 150 + = 50 + , or = 100 Intersection B: Traffic in = Traffic out: 100 + = 200 + , or = 100 Intersection C: Traffic in = Traffic out: 100 + = 100 + , or = 0 Intersection D: Traffic in = Traffic out: 50 + = 50 + , or = 0

This gives us a system of three linear equations: = 100 = 100 =0 = 0. Solving: 1 0 1 0 100 1 0 1 1

0 0

1

0 0

0

0

0

1 1 1

0

0 1

1

0

1 100 2 1 0

1

0

0

0 1 100 1 1

0 1 0 1

0 0 0

0 0

1

0 1 1 0 1

0

100 1 + 2

0

0

1 0

1

1 0 0 1 100 4 3

0 0 1 1 0 1 0 1 0 0 0

0

0 0

0

0

0

3 + 2

Translating back to equations gives: = 100, = 0, = 0. Thus, the general solution is: = 100, = , = , arbitrary, where is traffic along the middle section of Harrison. b. Because = has more than one possible value, it is not possible to determine the traffic flow along the middle section of Jeppe. c. Given = 400, the first equation of the general solution says that 400 = 100, so = 500. We need Simmons ( ). But the third equation says = . So = 500 cars down the middle section of Simmons. d. If is less than 100, would become negative, by the first equation of the solution. Thus, 100. e. Simmons is = , which can be as large as we like without making any of the variables negative (see the solution). Thus, there is no upper limit to the traffic on the middle section of Simmons. We can visualize this by imagining thousands of cars going around and around the center block without affecting the recorded numbers in the diagram.


Solutions Section 4.3 34. a.

Intersection A: Traffic in = Traffic out: = 50 + = 50 Intersection B: Traffic in = Traffic out: + = 50 + + + = 50 Intersection C: Traffic in = Traffic out: 100 + = + + = 100

This gives us a system of three linear equations: = 50 + + = 50 + = 100. Solving: 1 0 0 1 50 1 0 1 0

1

1

1 1

1 0 0 1

0

1

50

0 1 1 1 100

50

100

2 + 1 → 0

1

0

1

0 1 1

1 1 1

50

100

100 3 + 2

0 0 0 0 0 Translating back to equations gives: = 50, + = 100. Thus, the general solution is: = 50 + , = 100 + , and arbitrary. b. We are given = 20, and we want , which is = 50 + = 50 + 20 = 70 cars/min. c. We are given = 20 and = 90 and asked for . Substituting the given information into the second equation of the solution gives 90 = 100 20 + , so = 10. Thus, = 50 + = 50 + 10 = 60 cars/min. d. = 0, and we are asked for the minimum value of = 100 + = 100 + , which is 100 cars/min (when = 0). 35. a. Let us take the unknowns to be the net traffic flow going east on the three stretches of Broadway as shown in the figure. (If any of the unknowns is negative, it indicates a net positive flow in the opposite direction.)

Intersection A: Traffic in = Traffic out: 180 + = 50 + = 130 Intersection B: Traffic in = Traffic out: 200 + = 40 + = 160 This gives us a system of two linear equations: = 130 = 160.Solving: 1 1 0 130 1 + 2 1 0 1 290 → 0 1 1 160 0 1 1 160


Solutions Section 4.3 Translating back to equations gives: = 290, = 160. Thus, the general solution is: = 290, = 160, arbitrary. As there are infinitely many solutions, we cannot determine the traffic flow along each stretch of Broadway. Knowing any one of , , or would enable us to solve for the other two unknowns uniquely. b. The general solution from part (a) is = 290, = 160, arbitrary. East of Fleet Street, the traffic flow is . If is smaller than 160, the above solution shows that the values of and are negative, indicating a net flow to the west on those stretches. 36. a. The unknowns are the currents labeled in the diagram, and we equate current in and current out at each junction:

Junction A: 10 = 5 + = 5 Junction B: 5 + = = 5 Junction C: = + = 0 Junction D: + = 10

Solving: 1 0 0

1

0

0

1

0

1 1

1 0 0 1

0 0 0 0

0

0 0

1

1 0 0

1

1

0

0

1 5 0

0

1

1

1

0

10

5

5

1 10 1

5

0 1 0 1 5 0 0 1

5

10

10

3 1

4 + 3

1 →

0

0

1

0

0 1

0

0

0

0 1 1

1 0 →

0

0 0

1

5

1 5 0

1

0

1

5 3 + 2 10

5

5

0 0 1 1 10 3 0 0

0

0

0

General Solution: = 5, = 5, = + 10, and 5 10.

0 0 0 0 0 b. Measure the current on any wire except the one labeled above.

37. We rewrite each given equation in standard form, using the given information that dollars: 120 = + + = 120 = 0.2 0.2 = 0 = 0.1 0.1 = 0 ! = + ! = 0.

The matrices below are set up with the unknowns in the order: , , , !.

= 120 billion


1

0

1 0

1

0

1 0

0

0.2 0

1

1 1

0 120

0.1 0

1

0

1

0

120

0

10 3

6 0 600 (1 6) 2

0

0

0 1

0

0

5 2

0

0 10 1 0 1

0

0

0 10

0 1 1 0

1

0

0

1

120

0

100

0

20

1

0

20

20

0 0

0 0

1 0 0 0 0 1 0 0

0 1

20 10

0

10 30

5

1

0 120

0

1 0

0

2 5 1

0

1

120

1 + 2

0

10 1 0

0

0

1 1 1

(1 10) 3

0 0 1 0 100 2 0 1

0

1 0 100 0

1

Solutions Section 4.3

0 1

0

0

0

4 + 1

0 100

0

0

1

120

0

10

0

20

1 0 100

1

0

0 1

0

1 0 0 0

1

0

0

1

0

0

1

1

0 10 1

0

0

1

20

0 0 1 0 100 0 1 0 0

10

0 0 0 1

20

3 2 4 + 2

4 + 3

Rearrange rows →

30

= 20, = 10, = 100, ! = 30

0 0 1 0 100

0 0 0 1 30 We are asked for bank reserves , which are therefore $10 billion.

38. We rewrite each given equation in standard form, using the given information that ! = 42 billion dollars: = + = 0 = 0.2 0.2 = 0 = 0.1 0.1 = 0 42 = + + = 42.

The matrices below are set up with the unknowns in the order: , , , . 1 1 0 1 0 1 1 0 1 0 5 1 + 2 0

0 0

1

0 1

5 0 0 5

0 0.2

1 0.1 1

0 0

0 0

10

0 5

0

0 0 5 0

5

0

0

6 1 1 1

6 1

0 0 10 1 0 0

0

1

0 5 2

0 10 3

42

0 0 0

210 2 4 3 0 0 0

140

0 0 0

1 + 6 4 2 + 4 3 + 4

5 0 1

0

1

10 1 1

5 0 0 5

0 0

0

0

42 5 4 2

1

0 0

0 5

0

0 0

3

0 0

420 (1 3) 4

0

0 840 (1 5) 1 0 140 (1 5) 2

0 0 10 0 140 (1 10) 3 0 0

0

6

0 0 10 1

5 0 →

0

1 140


1 0 0 0 168 0 1 0 0 0 0 1 0

28 14

Solutions Section 4.3 = 168, = 28, = 14, = 140

0 0 0 1 140 We are asked for money stock

, which is therefore $168 billion.

39.

Adding the values along each beam gives + + = 3,050 + + 1,000 = 3,030, or + = 2,030 + + 1,000 = 3,020, or + = 2,020. 1 1 1 3050 1 1 1 3050 1 + 2 1 1

1

0 1 2030 2 1 → 0 1 0 2020 3 1

0

0 1 0

0

1

0

1

2030

1020 1030

1 0 0 1000

1 + 3

0

1 0

1

0

1 1030

0

→ 0 1 0

1020

0

0

0

1000

1020 2 →

1 1030 3

0 1 0 1020 = 1,000, = 1,020, = 1,030

0 0 1 1030 From the table, we find the corresponding components: = water, = gray matter, = tumor. 40.

Adding the values along each beam gives + + 2,000 = 4,020, or + = 2,020 + + = 4,020 + 2,000 + = 5,020, or + = 3,020.


1 1 0

0 1 2020

1

0

1 1 4020 2 1 → 0 1 1 3020

1 0

2020 1 3

1

0 1

0

2000

1 2020

Solutions Section 4.3

1

0

0 2000

1 3020 3 2

1

1 0 0 1000

→ 0 1 0 2000 = 1,000, = 2,000, = 1,020

0 0 1 1020 0 0 1 1020 From the table, we find the corresponding components: = water, = bone, = gray matter. 41.

Adding the values along each beam gives + + + = 4,030 + + = 3,000 3 + 2 = 3,000 2 + = 2,060. Solving: 1 1 1 1 4030 1 1 1

1 0 1 3000 2 1

1

1

3 0

0

0 0 2 3000 3 3 1 0 2 1 2060 0

1

0

0

0

3

0

0

0

3

0

0 3

0

0

0

0

0

1

0

3000

1030

1 6000

0

1

0

3000

1 0 1030 0

0

1 0 0 0 1000 0 0 1 0 1030 0 1 0 0 2000

0

0

0 3 0

0

3 1 + 3

1 3

0

1

0

(1 3) 1

0 6000 (1 3) 3

1

0

1

Rearrange rows →

0

2

0

0

0 0

1030

0

2

1 + 2

1 9090 3 3 2 1

1

0

0

0

4030

0

0

0 3

0 1 0

3

1

0

0

2060 0

4 + 2 2

3000

1030

0

1

1000

0

0 2000 3

1

1 2 4

1 6000 3 + 4

1 0 1030 2 0

0

1 0 0 0 1000 0 1 0 0 2000 0 0 1 0 1030

0 0 0 1 0 0 0 0 1 0 = 1,000, = 2,000, = 1,030, = 0 From the table, we find the corresponding components: = water, = bone, = tumor, = air.


Solutions Section 4.3 42.

Adding the values along each beam gives + + = 4,020 3 + 2 = 8,040 3 + 2 = 3,000 2 + = 1,020. Solving: 1 1 0 1 4020 1 1 3

0 0 2 8040 2 3 1

3

0

0 0

3 2 0 3000 0 2 1 1020 0

2

8040

0 3

0

1 4020

3

0

2

0 0

0 0

0

2 2

1 1020 1

1020

8040

4 3

0 1

0 0 0

2 1 1020 3 + 4 0

1

0 0

1020

2000

0 1 0 0 1000 2 0

0

1 0

0

0 0

1 2 4

0 3 0 1 4020 2 + 4 0

0

3 0

3

1

4020

0 1 4020

3 0

2 2

0

0 1

2

3000 1020

3 1 + 2 3 + 2

0 0

0 0

0

2 1 1020 0

2

0 0

2040

6000

(1 2) 4

0

0 0

2 0

0 1

1 0 0 0 2000 0 1 0 0 1000 0 0 1 0

0

0

1020

(1 3) 1

0 3 0 0 3000 (1 3) 2 0

8040

0 3 0 1 4020 3

0

(1 2) 3

0 0 0 1 1020 0 0 0 1 1020 = 2,000, = 1,000, = 0, = 1,020 From the table, we find the corresponding components: = bone, = water, = air, = gray matter.


Solutions Section 4.3 43. Since the three beams extending from left to right pass though the same four squares on the left, let us label those squares . The vertical beam passes through an additional two squares, which we will label as and as shown:

Adding the values along each beam gives + + = 3,080 + = 6,130 + = 6,110 + = 6,080. Solving: 1 0 1 0 1 1 3080 0 1 0

1 1 0 6130

1 0 0 6110 3 1 1 0 1 6080

1 0 0 1

1 1

1 0

0 0

2

0 2

0

1

0

0

0 0

2 0

1 0

3080 6130

1 3100 1

1

50

3060

9160

1 0 0 1

0 0

1

0

1000

0

1

0

2 2 + 3 2 4 3

1 →

1 4 2 + 4

1030 5080

1

2 1 + 3

0 0 2 1 3100 3 + 4 0 0

0

0 0 1 0 1050 3

1

1

1

3080

0

6130

1 1 3030 3 2 0

6080 4 2

1

2 0

0

0 2

1

0

1

3060

9160

0 0 2 1 3100 0 0

2 0 0 2

0 0

0

0

3

3000

2060

(1 3) 4

(1 2) 1

0 10160 (1 2) 2

0 0 2 0 2100 (1 2) 3 0 0

0

1

1 0 0 0 1030 0 1 0 0 5080

1000

0 0 1 0 1050

0 0 0 1 1000 0 0 0 1 1000 All we need is the value of : 1,030, which corresponds to tumor. 44. Since the three beams extending from left to right pass though the same four squares on the left, let us label those squares . The vertical beam passes through an additional two squares, which we will label as and as shown:


Solutions Section 4.3

Adding the values along each beam gives + + = 2,050 + = 6,130 + = 5,080 + = 6,080. Solving: 1 0 1 0 1 1 2050 0 1 0

1 1 0 6130

1 0 0 5080 3 1 1 0 1 6080

1 0 0 1

1 1

1 0

0 0

2

0 2

0

1

0

0

0 0

2 0

1 0

2050 6130

1 3100 1

1

50

1000

9160

1 0 0 1

0 0

1

0

1000

0

1

0

2 2 + 3 2 4 3

1 →

1 4 2 + 4

0

5080

1

2 1 + 3

0 0 2 1 3100 3 + 4 0 0

0

0 0 1 0 1050 3

1

1

1

2050

0

6130

1 1 3030 3 2 0

6080 4 2

1

2 0

0

0 2

1

0

1000

9160

1

0 0 2 1 3100 0 0

2 0 0 2

0

0

0

3

0

3000

(1 3) 4

(1 2) 1

0 10160 (1 2) 2

0

0 0 2 0 2100 (1 2) 3 0 0

1 0 0 0

0

0

1

0 1 0 0 5080

1000

0 0 1 0 1050

0 0 0 1 1000 0 0 0 1 1000 All we need is the value of : 0, which corresponds to air.

45. Unknowns: = the number of Democrats who voted in favor, = the number of Republicans who voted in favor, = the number of Others who voted in favor Note: 333 Democrats voted against, 89 Republicans voted against, and 13 Others voted against. Given information: (1) There were 31 more votes in favor than against: The total number of votes in favor = + + Total number of votes against = (333 ) + (89 ) + (13 ) = 435 Thus, + + (435 ) = 31, or 2 + 2 + 2 = 466, or + + = 233. (2) 10 times as many Democrats voted for the bill as Republicans. Rephrasing this: The number of Democrats voting for the bill was 10 times the number of Republicans voting for the bill: = 10 , or 10 = 0. (3) 36 more non-Democrats voted against the bill than for it. Rephrasing this: The number of non-


Solutions Section 4.3 Democrats voting against the bill exceeded the number of non-Democrats voting for by 36: (89 ) + (13 ) ( + ) = 36, or 2 2 = 66, or + = 33. Thus, we have three equations with three unknowns: + + = 233 10 = 0 + = 33. 1 1 1 233 1 1 1 233 11 1 + 2 10 0

0

11 1 233

0

11 0 220 (1 11) 2 → 0 1 0 20 2 →

0

11 0

11 0

1

1

0

10

0

10

0

2 1 → 0

1

33

2330 130

0

(1 10) 3

0 2200 (1 11) 1

0 0

1

13

1 0 0 200 0 1 0

20

11 1

11

→ 1

0 0

0

0

0

1 233 1

0

33

10

11 3 + 2

2330 1 10 3

11 1 233 2 + 3 0

1

0 200 1

13

13

0 0 1 13 Solution: 200 Democrats, 20 Republicans, 13 of other parties voted for the bill.

46. Unknowns: = the number of Democrats who voted in favor, = the number of Republicans who voted in favor, = the number of Others who voted in favor Note: 75 Democrats voted against, 17 Republicans voted against, and 4 Others voted against. Total number against = 96 Given information: (1) 16 more votes in favor than against: ( + + ) (96 ) = 16, or 2 + 2 + 2 = 112, or + + = 56 (2) Three times as many Democrats voting in favor as non-Democrats voting in favor: = 3( + ), or 3 3 = 0 (3) 32 more Democrats voting in favor than Republicans: = 32 Solving: 1 1 1 56 1 1 1 56 1 1

1 0

0

1

3 3

1 1

1

2 0

0

1

0 2 1 → 0 4 4 56 (1 4) 2 →

32 3 1 56

1 14

1 + 2

1 24 3 2 2

0

42

0 2 1 24 1

0

0

42

→ 0 1 1 14 2 + 3 → 0

0

1 0 0 42

0 1 0 10 2 → 0 1 0 10

1

4

0 0 1 4 0 0 1 4 Solution: 42 Democrats, 10 Republicans, 4 of other parties voted for the bill.

47. = amount invested in company X, = amount invested in company Y, = amount invested in company Z, = amount invested in company W Investments totaled $65 million: + + + = 65.


Solutions Section 4.3 Total return on investments was $8 million: 0.15 0.20 + 0.20 = 8. Colossal invested twice as much in company X as in company Z. Rewording this: The amount invested in company X was twice the amount invested in company Z: = 2 , or 2 = 0. The amount invested in company W was 3 times the amount invested in company Z: = 3 , or 3 + = 0. 1 1 1 1 65 1 1 1 1 65 0.15 0.20 1 0

1

0

0

1

7

1

0

0

7

0

0 7 0

0

0

1

3

0

0.20

1

65

1

35

8

420

1

3 4

3

0

0

1

0

0 1 0

0

9 1 + 4 3

63

0

0 0

9

1

0

15

1

0

30

0

0 7

1 8 4

4 210

0

3 4 3

2 4

9 4 210 3 + 4 4 0

0

420

0

0

7 1 + 2

0

8

0

0

0

1

0

7

1

35 3 2 3

0

0

3

0

4

1

0 3

9

0

3

3 1 65 7 3 2

9

0

1

3

0

0

0

2

8 20 2

20

5 2

1 0 10 3

4 160 2 3 1

4

8

2 0 3 1

1

3

3 1

0 0

420

35

0

18 8 420 (1 2) 3

0

21

0

0 0

0

0 0 0

1 0 0 0 20 0 1 0 0

3

0

0 3 0

0

5

0

0

1

56 7

0

2940 (1 7) 1 105

(1 7) 2

0

9 4 210

0

0 15 (1 3) 2

0

0

0

7

210

(1 7) 4

0 180 (1 9) 1

9 0 90 (1 9) 3 0

1

30

0 0 1 0 10

0 0 0 1 30 0 0 0 1 30 = 20, = 5, = 10, = 30 As this is a unique solution, Smiley has sufficient information to piece together Colossal's investment portfolio: Colossal invested $20m in company X; $5m in company Y, $10m in company Z, and $30m in company W. 48. = amount invested in company X, = amount invested in company Y, = amount invested in company Z Investments totaled $65 million: + + = 65. Total return on investments was $8 million: 0.15 0.20 = 8. The investment in X amounted to $30 million: = 30. Twice as much was invested in company X as in company Z: 2 = 0. 1 1 1 65 1 1 1 65 0.15 0.20 1 0

0

0

1 1

1

7

1

1

0

0

30

1 35 7 3 2

0

1

0

8 20 2

2

65

3 35

0

7 1 + 2

3 65 7 4 2

3

4

7

0

1 1

0

0

0

0

2

0

4

0 7 0

0

0

4

3

160 2 3 1 30 3 1 0

4 1

420

35

210 (1 2) 3

18 420 (1 6) 4


7

0

0 7 0

0

0

0

4

3

2 3

420

1 + 2 3

35 2 2 3 3

105

70 2 4 3 3

Solutions Section 4.3

7

0

0

0

0 14 0

0

0

0

210

245

2 105 0

175

Smiley should take the next flight out of the country. (There is no solution.) 49. It is not realistic to expect to use exactly all of the ingredients. Solutions of the associated system may involve negative numbers or not exist. Only solutions with nonnegative values for all the unknowns correspond to being able to use up all of the ingredients. 50. The answer would indicate that there is no way to satisfy all the requirements stated in the exercise, since fractions of sections are not allowed for. Rounding the solutions will lead to values not satisfying the exact requirements in the exercise. 51. The blend consists of 100 pounds of ingredient 52. Yes; = 0

. This says that = 100, which is a linear equation.

53. The blend contains 30% ingredient by weight. Rephrasing: The weight of ingredient is 30% of the combined weights of , , and " : = 0.30( + + ) = 0.30 + 0.30 + 0.30 0.3 0.7 + 0.3 = 0,which is a linear equation. 54. = ; not linear

55. There is at least 30% ingredient by weight. Rephrasing: Tthe weight of ingredient is at least 30% of the combined weights of , , and " : 0.30( + + ).This gives a linear inequality, not an equation. 56. There is twice as much ingredient 2 2 = 0,which is linear.

by weight as and " combined: = 2( + ), or

57. Answers will vary. 58. Answers will vary, but these equations lend themselves particularly to some kind of transportation problem.

59. Labeling the numbers in the top row as and and those in the bottom row as and , the indicated equalities give us the following system of equations: + =8 =6 + = 13 + =8 The solution to this system of equations is = 3.5, = 4.5, = 9.5, and = 3.5.

60. Labeling the numbers in the top row as and and those in the bottom row as and , the indicated equalities give us the following system of equations: + =8 + =6 + = 13


Solutions Section 4.3 + =8 This system of equations has no solution, so the puzzle is impossible to solve. 61. Answers will vary, but need to be based on the game in Exercise 59, because all versions of Exercise 60 lead to either an inconsistent a redundant system. For instance, take the version of the game in Exercise 59 with the sums across both equal to 1 and the vertical sums both equal to 0. 62. Answers will vary, but they need to be based on the game in Exercise 60 because all vesions of Exercise 59 lead to a unique solution. For instance, take the version of the game in Exercise 59 with the sums across both equal to 1 and the vertical sums also both equal to 1.


Solutions Chapter 4 Review Chapter 4 Review

1. To graph with technology, solve for : First line: = 2 + 2 Second line: = 2 1.

2. To graph with technology, solve for : First line: = 2 3 Second line: = + 2.

As the graphs intersect in a single point, there is a single (unique) solution.

As the graphs intersect in a single point, there is a single (unique) solution.

3. To graph with technology, solve for : First line: = 2 3 Second line: = 2 3.

4. To graph with technology, solve for : First line: = 2 3 + 2 3 Second line: = 2 3 1 3.

Infinitely many solutions

No solution (the lines are parallel and hence do not intersect).

5. To graph with technology, solve for : First line: = + 1 Second line: = 2 + 0.3 Third line: = 3 2 + 13 20.

6. To graph with technology, solve for : First line: = 6 + 0.2 Second line: = 6 + 0.2 Third line: = 6 0.2.

As the intersection of all three graphs is a single point, there is a single (unique) solution.

As the intersection of all three graphs is a single point, there is a single (unique) solution.

7.

+ 2 = 4 2 = 1

Multiply the second equation by 2: + 2 = 4 4 2 = 2.


Adding gives 5 = 6 = 65 .

Solutions Chapter 4 Review

Substituting = 6 5 in the first equation gives 65 + 2 = 4 2 = 4 65 = 14 = 75 . 5 Solution: = 65 , = 75 8.

0.2 0.1 = 0.3 0.2 + 0.2 = 0.4

Multiply the first equation by 10 and the second by 10 : 2 = 3 2 2 = 4. Adding gives 3 = 1 = 13 .

Substituting = 1 3 in 2 = 3 gives 2 13 = 3 2 = 3 + 13 = 10 = 53 . 3 Solution: = 53 , = 13 9.

1 34 = 0 2

6 9 = 0

Multiply the first equation by 4 and divide the second by 3 : 2 3 = 0 2 + 3 = 0. Adding gives 0 = 0, so the system is redundant (the equations give the same lines). To get the general solution, we solve for : 2 = 3 = 3 . 2 General solution: = 3 , arbitrary, or � 3 , �; arbitrary 2 2 10.

2 + 3 = 2 3 2 = 1 2

Multiply the second equation by 2: 2 + 3 = 2 2 3 = 1. Adding gives 0 = 3, showing that the given system is inconsistent, and therefore has no solution. 11.

+ =1 2 + = 0.3

3 + 2 = 13 10 1 1 2 1

1

0.3

1

1

10 2 → 20 10

3 2 13 10 10 3

1

1

3 2 20 1 → 0

30 20 13 3 30 1

0

1

10 10

1

17

10 1 + 2

17 3 2


10 0 0

7 (1 10) 1

0

Solutions Chapter 4 Review

1 0 7 10

10 17 (1 10) 2 → 0 1 17 10 0 0 0 0 0

7 Translating back to equations gives the solution: = 10 , = 17 . 10

12.

3 + 0.5 = 0.1 6 + = 0.2

3 0.05 = 0.01 10

3

0.1 10 1

0.5

6

1

0.2 5 2

3 10 0.05 0.01 100 3 60 0 0

0 0

2 (1 60) 1 0

10 0 (1 10) 3

30 30

30

5

1

1 2 1 →

5

5 1 3 1

1 0 1 30

→ 0 0 0 1

0

30 0 0

5

0

10

1 2 1 + 3 0

0

0

1 Translating back to equations gives: = 30 , = 0.

13.

+ 2 = 3 =0 + 3 2 = 2 1

1

1

1

2

0

3

0

1

0 1 3

0 1

2 1 → 0

3 2 2 3 1

0 2 0

0

0

1

1

1 0 0 1 0 1 0 1

1 0 1

2

2 1

0

3 2 3 → 0 2 1

0

0

0

3 1 + 2

0

1

1 2

0

1

3 1

2

1

2 3 + 2

1

0

→ 0 2 1 3 1

0

0

(1 2) 2 → 0 1 0

0

0

0 0 1 1 Translating back to equations gives the solution: = 1, = 1, = 1. 14.

+ =2 7 + = 6

12 + 13 = 1

+ + =6

1 0

0

0

5 5 (1 5) 3

1

1 1

1

2 → 3


1

1

1

7

1

6

1

1 1 2 1 3 1 6 3 1

1

1 1 0

1

0

3

0

1

1

1

1

2

2

1 1

6

2

1 0 0 1 0 1 0 2

4 2

2

1 6 2 7 1

1

6

3

1

1 + 2

4 6 3 3 2 0

7

1

1

Solutions Chapter 4 Review

1

1 0 0 1 0 0

0 0

2

1

0

6 3 6 1

1

6 4 1

1 1

1

1 2 3

1

3

1

3

4 + 3

0

8

0 0 →

3 2

1 0 0 1

1

2

8 8 (1 8) 2 4 6 0

4

0

1

0

2

(1 2) 4

0 0 1 3 3 0 0

0

0

0 0 1 3

0 0 0 0 Translating back to equations gives the solution: = 1, = 2, = 3. 15.

12 + = 0

1 12 = 1 2 3 12 + 12 = 1 2

1

1 2

1 2 0

3 2 1 2

1

0

2 1

1 2 1 2 2 → 1 2

1 2 3

2 0 2 4 (1 2) 1 0 1 4 4

2

2

1

1

1 2 2 2 1

0

3

0

1

1 0 1 2

→ 0 1 4 4

2 2 3 3 1

1

2 1

→ 0 0

1 1

2

0

4 4

1 + 2

4 4 3 2

0 0 0 0 0 0 0 0 Translating back to equations gives: = 2, 4 = 4. General solution: = 2, = 4 4 = 4( 1), arbitrary, or ( 2, 4( 1), ); arbitrary 16.

+ 2 = 1 2 2 + 4 = 2 0.75 + 0.75 1.5 = 0.75 1

1

2

2

0 0

0

2 4

1 2

0.75 0.75 1.5 0.75 4 3

1 1 2 1 0

1

1

→ 2 2 3

3

2 1 4

2

1

1

(1 2) 2 → 1 1

6 3 (1 3) 3

1

1

2 1 2

1

2 + 1 →

2 1 3 1

0 0 0 0 Translating back to equations gives: + 2 = 1. General solution: = 2 + 2 , arbitrary, arbitrary, or ( 1 + 2 , , ); , arbitrary


Solutions Chapter 4 Review 17. Rewrite the given equations in standard form: 12 = 0

1 + 12 = 2 2

3 + = 0. 1

1 2

2

1

2 0 2 8 0 1 2 8

0

1

3

0

1

1

1

1 2 2 2 2 →

0

3

0 2 1

0

1 2

0

1

0

4 2 2 1

1 0 2 3 + 3 1

2 1

→ 0

1

0

2

0 1 + 2

8

0 1 2 0 3 + 2

0 0 0 8 Since Row 3 translates to the false statement 0 = 8, there is no solution. 18.

+ =1 + =1 + =1 2 + = 3 1

0 1

1

1 1

2

1 0

0

0

0

0 1 1 0 0 0 0

1 1

1

1

1

1 1

0

1

1

1

1 1 3 1 0

2

1

1 1

3 4 2 1

2 1

2 + 3

2 1 4 3

0

1 0

1 0 3 2

1 0 0

1

2

0 0

1 2

1

0 1 0 1

1

1 1 + 2

1

0

1

0

0

1

1 4 2 2

0 0 1 2 1 0 0 0

0

0

Translating back to equations gives: + = 2, = 0, 2 = 1. General solution: = 2 , = , = 1 + 2 , arbitrary, or (2 , , 1 + 2 , ); arbitrary

19. 5# 9 = 160 We are given the additional information: # = , or # = 0, giving us a second linear equation. We can solve the resulting system of two equations in two unknowns by multiplying the second equation by 5 : 5# 9 = 160, 5# + 5 = 0. Adding gives 4 = 160 = 40°. Since = # , # = 40° also.

20. 5# 9 = 160 We are told that the Celsius temperature is half the Fahrenheit temperature: = 12 # , giving us the system 5# 9 = 160 1 # = 0. 2

Multiply the first equation by 2 and the second by 20 : 10# 18 = 320 10# + 20 = 0.


Solutions Chapter 4 Review Adding: 2 = 320 = 160 Substituting = 160 in the equation = 12 # gives # = 2 = 320.

Thus, the temperature is 160°C, or 320°F.

21. 5# 9 = 160 We are told that the Fahrenheit temperature is 1.8 times the Celsius temperature: # = 1.8 , or # 1.8 = 0, giving us the system 5# 9 = 160 # 1.8 = 0. Multiplying the second equation by 5 gives 5# 9 = 160 5# + 9 = 0. Adding gives the false statement 0 = 160, showing that the given system is inconsistent (has no solution). Thus, it is not possible for the Fahrenheit temperature of an object to be 1.8 times its Celsius temperature. 22. 5# 9 = 160 We are told that the Fahrenheit temperature is 30° more than 1.8 times its Celsius temperature: # = 1.8 + 30, or # 1.8 = 30, giving us the system 5# 9 = 160 # 1.8 = 30. Multiplying the second equation by 5 gives 5# 9 = 160 5# + 9 = 150. Adding gives the false statement 0 = 10, showing that the given system is inconsistent (has no solution). Thus, it is not possible for the Fahrenheit temperature of an object to be 30° more than 1.8 times its Celsius temperature. 23. The total population of the four cities is 10 million people: + + + = 10.This is a linear equation.

24. City $ has three times as many people as cities % and combined. Rephrasing: The population of City $ equals three times the sum of the populations of cities % and : = 3( + ), or 3 3 = 0. This is a linear equation. 25. There are no people living in city : = 0. This is a linear equation.

26. The population of city $ is the sum of the squares of the populations of the other three cities: = 2 + 2 + 2, or 2 2 2 = 0. This is not a linear equation because of the squared terms. 27. City has 30% more people than City %. Rephrasing: The population of City is 130% of the population of City % : = 1.30 , or 1.30 + = 0.This is a linear equation.

28. City has 30% fewer people than City %. Rephrasing: The population of City is 70% of the population of City % : = 0.7 , or 0.7 + = 0. This is a linear equation. 29. Unknowns: = the number of packages from Duffin House, = the number of packages from Higgins Press Arrange the given information in a table:


Solutions Chapter 4 Review Duffin Higgins Desired totals ( ) ( ) Horror

5

5

4,500

Romance

5

11

6,600

Horror: 5 + 5 = 4,500 Romance: 5 + 11 = 6,600 To solve, multiply the first equation by 1 and add, to obtain 6 = 2,100 = 350. The first equation can be divided by 5 and rewritten as + = 900. Substituting = 350 gives us + 350 = 900 = 550. Solution: Purchase 550 packages from Duffin House, 350 from Higgins Press.

30. Unknowns: = the number of packages from Duffin House, = the number of packages from Higgins Press Cost: 50 + 150 = 50,000 Also, you have promised to buy twice as many packages from Duffin as from Higgins. Rephrasing: The number of packages from Duffin equals twice the number from Higgins: = 2 , or 2 = 0. Dividing the first equation by 50 gives: + 3 = 1,000 2 = 0. Subtracting the second from the first gives 5 = 1,000 = 200. Substituting = 200 in the second equation gives 2(200) = 0 = 400. Solution: Purchase 400 packages from Duffin House, 200 from Higgins Press.

31. Unknowns: = the number of packages from Duffin House, = the number of packages from Higgins Press Cost: 50 + 150 = 90,000 Also, you have promised to spend twice as much money for books from Duffin as from Higgins: Amount spent on Duffin = 2× amount spent on Higgins: 50 = 2(150) , or 50 300 = 0. Dividing each of these equations by 50 gives: + 3 = 1,800 6 = 0. Multiplying the first equation by 2 and adding gives: 3 = 3,600 = 1,200. Substituting = 1,200 in the second equation gives 1,200 6 = 0 6 = 1,200 = 200. Solution: Purchase 1,200 packages from Duffin House, 200 from Higgins Press. 32. Unknowns: = the number of packages from Duffin House, = the number of packages from Higgins Press Cost: 50 + 150 = 60,000 Also, you have promised to spend the same amount on both publishers: Amount spent on Duffin = amount spent on Higgins: 50 = 150 , or 50 150 = 0. Dividing each of these equations by 50 gives + 3 = 1,200 3 = 0. Adding gives 2 = 1,200 = 600. Substituting = 600 in the second equation gives 600 3 = 0 3 = 600 = 200. Solution: Purchase 600 packages from Duffin House, 200 from Higgins Press.

33. Demand: = 1,000 + 140,000 Supply: = 2,000 + 20,000 For the equilibrium price, we can equate the supply and demand: Demand = Supply 1,000 + 140,000 = 2,000 + 20,000 3,000 = 120,000 = 120,000 = $40 per book. 3,000 34. Demand: = 2 + 18 Supply: = 3 + 3 For the equilibrium price, we can equate the supply and demand:


Solutions Chapter 4 Review Demand = Supply 2 + 18 = 3 + 3 5 = 15 = $3 million.

35. Unknowns: = the number of baby sharks, = the number of piranhas, = the number of squids Arrange the given data in a table with the unknowns across the top: Sharks Piranha Squid Total Consumed ( ) ( ) ( ) Goldfish

1

1

1

21

Angelfish

2

0

1

21

Butterfly fish

2

3

0

35

Goldfish: + + = 21 Angelfish: 2 + = 21 Butterfly fish: 2 + 3 = 35 1 2 2

2

1 1 21

1

0 1 21 2 2 1 → 0 3 0 35 3 2 1

0

1

0 2 0

1

1

0

1

1 0 0 7 0 1 0 7

21

1 + 3

2

0

1

2

1

0

1 21 2

21 2 3 → 0 2 7

0

21

0

0

0

2 1 + 2

7 2 3 + 2

1

(1 2) 1

14

2

0

1

0

21

0

7

→ 0 2 1 21 0

0

5 35 (1 5) 3

0

1 7 3

14 (1 2) 2 → 0 1 7

1

0

0

7 2 →

0 0 1 7 Solution: He has 7 of each type of carnivorous creature.

36. Unknowns: = the number of servings of granola, = the number of servings of nutty granola, = the number of servings of nuttiest granola From the data in the given table: + + 5 = 1,500 4 + 8 + 8 = 10,000 2 + 4 + 8 = 4,000 1 1 5 1500 1 1 5 1500 1 1 5 1500 1 2 4 8 8 10000 (1 4) 2 → 2 4 8 1 0

8

4000 (1 2) 3 500

2

1

2 2 2500 2 1 → 0 2 4 2000 3 1

1 0

8

500

1 8 3

0

1 1

3 1000 1

500 3 2

1 0 0 2500 → 0 1 3 1000 2 + 3 3 → 0 1 0 250 0 0 1 250 500 (1 2) 3 0 0 1 250

0 1 3 1000 0 0

1

The solution of the system is: 2,500 granola treats, 250 nutty granola treats, and 250 nuttiest granola treats. Since the last number is negative, it is impossible for them to use up all the ingredients.

37. Unknowns: = the number of hits at OHaganBooks.com, = the number of hits at JungleBooks.com, = the number of hits at FarmerBooks.com We are given the following information: (1) Combined Web site traffic at the three sites is estimated at 10,000 hits per day: + + = 10,000.


Solutions Chapter 4 Review (2) The total number of orders is 1,500 per day: 0.10 + 0.20 + 0.20 = 1,500. (3) FarmerBooks.com gets as many book orders as the other two combined: 0.20 = 0.10 + 0.20 , or 0.10 + 0.20 0.20 = 0. Solving this system of three linear equations in two unknowns: 1 1 1 10000 1 1 1 10000 0.10 0.20

0.20

0.10 0.20 0.20 1 0 0

1 1

1

10000

0

5000

1

5000

1500 10 2 → 0

10 3

1 2

3 10000 3 2

1

1 0 0 1

1

0 0

5000

1

3750

1 0 0 5000

1

2

1

15000 2 1 →

2 2

1 0

0

1 0

0

→ 0 1

2

1

5000 5000

3 1

0

0 0 4 15000 (1 4) 3

2 + 3 → 0 1

0

5000 1250

0 0 1 3750 3

0 1 0 1250

0 0 1 3750 Solution: 5,000 hits per day at OHaganBooks.com, 1,250 at JungleBooks.com, 3,750 at FarmerBooks.com

38. Unknowns: = revenue (in billions) earned from books on rock music, = revenue (in billions) earned from books on rap music, = revenue (in billions) earned from books on classical music Total revenues were 5.8 billion: + + = 5.8 Books on rock music brought in twice as much revenue as books on rap music. Reword this as: The revenue earned from books on rock music was twice the revenue earned from books on rap music: = 2 , or 2 = 0. Books on rock music brought in 900% the revenue of books on classical music. Reword this as: The revenue earned from books on rock music was 9 times the revenue earned from books on classical music: = 9 , or 9 = 0. We thus solve the system: + + = 5.8 2 = 0 9 = 0 5 5 5 29 1 1 1 5.8 5 1 1 2 1 5

0

5

0

15

0

15

0

15 0

5

0

0

0

0 5 0

0

50 29 3 3 2

0

5

0

29

3 1 + 2

58

1 + 2 3

15

0 5 2 1 →

0 5 3 1 0

10

58

0

15 0

145 58 (1 29) 3

0

15

0

9 (1 5) 2 → 0 1 0

9 5

10

0

0

5

5

1

2

9

5

0

1

0

9

0

0

5

29

29 2 3 2

18 (1 5) 1

5 2 (1 5) 3

15 0

0

0

1 0 0 18 5 0 0 1

2 5

= 18 5 = 3.6, = 9 5 = 1.8, = 2 5 = 0.4.

5

0

5

29

54

(1 3) 1

27 (1 3) 2 → 2


Solutions Chapter 4 Review Solution: Revenues were $3.6 billion for books on rock music, $1.8 billion for books on rap music, and $0.4 billion for books on classical music.

39. Unknowns: = the number of shares of HAL, = the number of shares of POM, = the number of shares of WELL The total investment was $12,400: Investment in HAL = shares @ $100 = 100 Investment in POM = shares @ $20 = 20 Investment in WELL = shares @ $25 = 25 . Thus, 100 + 20 + 25 = 12,400. He earned $56 in dividends: HAL dividend = 0.5% of 100 invested = 0.005(100 ) = 0.5 POM dividend = 1.5% of 20 invested = 0.015(20 ) = 0.3 WELL dividend = 0 Thus, 0.5 + 0.3 = 56. He purchased a total of 200 shares: + + = 200. We therefore have the following system: 100 + 20 + 25 = 12,400 0.5 + 0.3 = 56 + + = 200 100 20 25 12400 100 20 25 12400 (1 5) 1 0.5 0.3 1

0

1

20

1

56

200

4 5 2480

560 4 2 1

5

3 0

0

8 5 240

1

1 1

40 0 15 0

10 2 →

200 20 3 1 5200 (1 5) 1

2000 (1 25) 3

0 25

8 0 0 800 (1 8) 1

5

3

0

560

0

8

200

5 240

1520 3 2 2

1

20

1

1

4

0

16 15

0 0

1

8 0

2480 2 1 2

5

3

1040 1 3 3

→ 0 8 5 240 2 + 5 3 → 80

1 0 0 100

0 8 0 160 (1 8) 2 → 0 1 0

20

0 0 1 80 0 0 1 80 Solution: He purchased 100 shares of HAL, 20 shares of POM, and 80 shares of WELL.

40. Unknowns: = the number of shares of DHS, = the number of shares of HPR, = the number of shares of SPUB He purchased a total of 2,000 shares: + + = 2,000. The total investment was $20,000 8 + 10 + 15 = 20,000. The total profit was $3,400: 0.20(8 ) + 0.15(10 ) + 0.15(15 ) = 3,400 1.6 + 1.5 + 2.25 = 3,400. 1 8

1

10

1

15

1.6 1.5 2.25 1 0

1 2

1

7

2000

20000

3400 20 3

2000 2 1 2

4000

0 2 13 4000 3 + 2

1 8

1

1

2000

10 15 20000 2 8 1

32 30 45 68000 3 32 1

2 0 5

→ 0 2

7

0

4000

0 0 20 8000 (1 20) 3


2 0 5 0 2 0 0

2 0 0 2000 (1 2) 1

4000 2 7 3 → 0 2 0 1200 (1 2) 2 →

7 1

1 + 5 3

0

Solutions Chapter 4 Review

400

0 0 1

1 0 0 1000 0 1 0

600

400

0 0 1 400 Solution: He purchased 1,000 DHS shares, 600 HPR shares, and 400 SPUB shares.

41. Unknowns: = the number of credits of Liberal Arts, = the number of credits of Sciences, = the number of credits of Fine Arts, = the number of credits of Mathematics Given information: (1) The total number of credits is 124: + + + = 124. (2) An equal number of Science and Fine Arts credits: = , or = 0. (3) Twice as many Mathematics credits as Science credits and Fine Arts credits combined. Rephrasing: The number of Mathematics credits is twice the sum of the numbers of Science and Fine Arts credits: = 2( + ), or 2 + 2 = 0. (4) Liberal Arts credits exceed Mathematics credits by one third of the number of Fine Arts credits. Rephrasing: The number of Liberal Arts credits minus the number of Mathematics credits is one third of the number of Fine Arts credits: = 13 , or 13 = 0 1 1

1

0 1

1

1

1

0 2

1

124

0

2

1

1

0

0

0

1 0 1 3 1 0

0

1 1

2

1

2

0

3 2 2

0

2 0 0

3

0 4 0

1

2 0 0

0

0 0 4

1

0 0

0 0 4

0 0

104 (1 2) 1 48 48

0 0 0 1 48 1 0 0 0 52 0 1 0 0 12

(1 4) 2 (1 4) 3

1

124

0

2

0

0

1

0 1 3 →

1 0

0

2

1

0 1 1 0 0

4

0

1

4 3 1 124 0

0

2 1 3 4 2 + 3

0 0 7 6 372 4 4 + 7 3

2 0 0

248

0 0 0 31 1488 (1 31) 4

0 4 0

2

3

0 3 4 6 372 4 + 3 2

1

1 1

0

3 4

1 2

1

0

0

124

1

1

0 4 0 0 0 4 0 0 0

1 0 0 0 1 0 0 0 1

0 0 0

3

1 1 1 52 12 12

248 1 + 3 4 0 0

48

0 0 0 1 48 4

2 4 3 4

0 0 1 0 12

0 0 0 1 48 Solution: Billy-Sean is forced to take exactly the following combination: Liberal Arts: 52 credits, Sciences: 12 credits, Fine Arts: 12 credits, Mathematics: 48 credits.

42. Unknowns: = the number of credits of Liberal Arts, = the number of credits of Sciences, = the


Solutions Chapter 4 Review number of credits of Verbal Expression, = the number of credits of Mathematics Given information: (1) The total number of credits is 120: + + + = 120. (2) An equal number of Science and Liberal Arts credits: = , or = 0. (3) Twice as many Verbal Expression credits as Science credits and Fine Arts credits combined. Rephrasing: The number of Verbal Expression credits is twice the sum of the numbers of Science and Liberal Arts credits: = 2( + ), or 2 + 2 = 0. (4) Liberal Arts credits exceed Mathematics credits by one quarter of the number of Verbal Expression Credits. Rephrasing: The number of Liberal Arts credits minus the number of Mathematics credits is one quarter of the number of Verbal Expression credits: = 14 , or 14 = 0. 1

1

1 1 2 1

2

1

1

0

0

0

1

0

0

4 4

0

1 4 1

0

2

1 1 120

2

0

1

1

0

0

1

0

4

0 2 0

1

1

1

120 0

120

3 2 240

1

2 1 + 2

5 8 480 4 2 2 1

120

1 120 3 2 3

0

1

0

3

0 6

0

1

120 2 4

0

0

20

0

6 0

0

0

1

0

3

0 1

0

0 1 0 0

0 0 0 0

0

1 0

2

2 1 0

0

1

80 3 4 3 120

0

20 2 80 3 4

1 2 0

2

1

0

0

0

1

0

1 4

0

0

2 1

1

120

0

0

3 2 1 4 4 1

0

0

3 2 240

0

0

3 6 240 (1 3) 4

0

0 6 0 0

0

3 2 240

0

0

0

0

0

0

0

0 0

0 1 0 0 20

3 0

4 0

0

1

120

1 120

0 6 0

1

0

1 0 0 0 20

120

0 2 1 1 120

6 →

1

1

6

1 + 4

240 3 2 4

0

4

3 1 + 3

1

2 240

2

1

0

120

(1 4) 4 (1 6) 1

120 (1 6) 2 240 (1 3) 3 0

0 0 1 0 80 0 0 0 1

0

Solution: Billy-Sean is now forced to take exactly the following combination: Liberal Arts: 20 credits, Sciences: 20 credits, Verbal Expression: 80 credits, Mathematics: 0 credits!


Solutions Chapter 4 Review 43. a. Order Department: Traffic in = Traffic out 500 = + + Top right router: Traffic in = Traffic out 100 + = 200, or = 100 Bottom left router: Traffic in = Traffic out = 100 + , or = 100 Shipping Department: Traffic in = Traffic out + + 200 = 500, or + = 300

1

1 1

1

0 0

1

0

0 0

0

0

500

100 2 1

1 0 1 100

0 1

0 1 0

0

0

0

1 0 0

0

1

1

1 1

0

300

0

0

100

0 1 0 1 100 0 0 1

1

100

400 2 3

1 300 1

1

300

4 + 3

0 0 0 →

1

1

1

1

1

0

1 0

0 1

0 1 0 0

0 0

0

500

0

400

0

0

1 1

0

100

300

1

1 + 2 3 + 2

100

100 2

1 1 300 3 0

0

0

300

0 0 0 0 0 Translating back to equations gives: = 100, = 100, + = 300. General solution: = 100, = 100 + , = 300 , arbitrary b. Because = 100 + and can be any number 0 ( cannot be negative because it represents a number of book orders), the smallest possible value of is 100 books per day. c. The equation = 300 tells us that cannot exceed 300 books per day, or else would become negative. d. If there is no traffic along , then = 0, giving: = 100 = 100 + 0 = 300 . Thus, from the third equation, = 300, giving us the particular solution = 100, = 100 + 300 = 400, = 0, = 300. e. If there is the same volume of traffic along and , then = , and so 100 + = 300 2 = 200 = 100 books per day. 44. a. At each intersection, equate traffic entering with traffic leaving. Lagoon Drive/North Beach Way Intersection: Intersection of Smith St., Beach View St., and Lagoon Drive: Beach View St./North Beach Way Intersection: South Beach Way, Dock Rd. and Smith St Intersection: Rewriting these equations in standard form gives: = 200

200 + = 50 = + + = = 50 +


+ = 50 + =0 = 50. 1 0 0 1 0

0

1

1

0

0 1

1

1 1

0

0

0

1

0

1 0 0

1

1 1

0

1

0

0

Solutions Chapter 4 Review

0

0

0

0 1 0

1 0

50 0

0

1

50

0

0

250

0

0

0

1

1 0 0 1 0 200 0 0 1

200

0

50

200 50

50

3 1

1 0

4 + 3

0 0

0 1

0 1

1

0

1

0

1

1

0

0

0

0

0

1

0 1 0

0

0

1 0

1

1

0 0 0

0 1 0

1

0

0

0

200 50

200 3 2 50

200 50

250 3

1 200 4

0 250

0 0 0 0 1 200 Translating back to equations gives: = 200, + = 50, = 250, = 200. General solution: = 200 + , = 250, = 50 , is arbitrary, = 200 b. The Lagoon Drive traffic is represented by , which is arbitrary in the general solution. However, the equation for , namely = 50 , tells us that cannot exceed 50 (or else becomes negative). Thus, the maximum traffic along Lagoon Drive is 50 cars every 5 minutes. c. This time, is allowed to be negative (corresponding to net traffic flow in the opposite direction) and so there is no upper limit to the value of , since none of the other variables becomes negative for large . Hence, there is no limit to the possible traffic along Lagoon Drive. 45. Unknowns: = the number of packages from New York to Texas, = the number of packages from New York to California, = the number of packages from Illinois to Texas, = the number of packages from Illinois to California

Books to Texas: + = 600 Books to California: + = 200 Books from Illinois: + = 300 Budget: 20 + 50 + 30 + 40 = 22,000 1 0 1 0 600 1 0 0

1 0

0 1

1 1

200 300

20 50 30 40 22000 (1 10) 4

0 0 2

0 1 0

1 0 1 0 1 1

600

200 300

5 3 4 2200 4 2 1


1 0

0

600

0 1

200

1 0 0 1

300

0 0

1

1 0

0 5

1 1

1 0

300

1 4 1000 4 5 2

0 1 0

1

1 0 0

0

0 0 1

1

0

0 0 1

0 1 0 0 0 0

0

0 1 1

1 0 0

1

300

1

200

1

1

450

1 0 0 0 450

300

150

0 0 0 1 150 4

0 1 0 0 0 1 0 0 0

0 1 0 0

200

1

50

0

1

200

0 0 0 2 300 (1 2) 4

0 1 0

600 1 3

Solutions Chapter 4 Review

1

1

0

4 3

300

3 + 4

300

150

50

1 4 2 + 4

0 0 1 0 150 0 0 0 1 150

Solution: New York to Texas: 450 packages, New York to California: 50 packages, Illinois to Texas: 150 packages, Illinois to California: 150 packages 46. Unknowns: = Number of people flown from Chicago to LA, = Number flown from Chicago to New York, = Number flown from Denver to LA, = Number flown from Denver to New York

Salespeople available in Chicago: + = 20 Salespeople available in Denver: + = 10 Number wanted at LA: + = 15 Number wanted at NY: + = 15 Cost: 200 + 150 + 400 + 200 = 6,500 1 1 0 0 1 1 0 0 20 0

0

1 0

1

0 1

1 0

1 0 1

10 15 15

200 150 400 200 6500 (1 50) 5 1

1

0

1

0

0

0 1 0 1

0

0 20

1

1 10

8

4 50 5 8 2

1

0

0 5 3 2 1 15

1 0

→ 0 0 0

1 0

1 1

1

20

0

0 1 1

4

3 8 4 130 5 4 1

1 0

0 1

0 1 0 1 0 1

15 3 1 15

0

20

1 + 3

1

15

4 + 3

1

10

0 1 15

0

10

0 4 30 5 3


1

0

0

1

0

0

0

0

0

0

1

0 1 0 0

0

0

1

1 3

1 0 0 0 10 0 0 1 0

5

5

10

3 1 5 3 2 + 5

3

0

0

0

0

0

0

0

3

15 3 3 5 → 0 3 0 0

15

0

0

0

0 0

0

0

30 15

(1 3) 2

30 (1 3) 3 → 0

0 3 15 (1 3) 5

1 0 0 0 10 0 1 0 0 10

0 1 0 0 10 Rearrange rows → 0 0 1 0 0 0 0 0

(1 3) 1

Solutions Chapter 4 Review

0 0 0 1

5 5

0 0 0 1 5 0 0 0 0 0 = 10, = 10, = 5, = 5 Send 10 people from Chicago to LA, 10 from Chicago to NY, and 5 each from Denver to LA and Denver to NY.


Solutions Chapter 4 Case Study Chapter 4 Case Study 1. For the Earth Suburban: = 10, = 240 : = 20, = 330 : = 50, = 300 :

240 = 100& + 10 + ' 330 = 400& + 20 + ' 300 = 2,500& + 50 + '

& = 0.25, = 16.5, ' = 100 and so = 0.25 2 + 16.5 + 100 16.5 Optimal DOH = = 33% 2( 0.25) Reduction in consumption = 0.25(33) 2 + 16.5(33) + 100 ( 373 gallons per year Cost increment = 9,000 + 50(33 10) = $10,150 2. Green Town Hopper:

= 5, = 180 : = 5.5, = 230 : = 7, = 200 :

180 = 25& + 5 + ' 230 = 30.25& + 5.5 + ' 200 = 49& + 7 + '

The solution is & = 60, = 730, ' = 1,970, so = 60 2 + 730 1,970. The optimal cost = (2&) = 730 [2( 60)] = 6.083), or about $6,083.33. To get the corresponding DOH, we use the equation Cost Increment = 5,000 + 50( *! 10) 6,083.33 = 5,000 + 50( *! 10) *! = (6,083.33 5,000) 50 + 10 ( 31.67%, the same results as in the text. We should expect the same optimal value, as there is a one-to-one relationship between costs and values of DOH, so the greatest reduction in gasoline consumption corresponds to exactly one value of DOH and exactly one value of the cost. Calculating them in two ways should yield the same result. 3. Electra Supreme:

= 10, = 220 : = 33.75, = 295.2 : = 50, = 260 :

220 = 100& + 10 + ' 295.2 = 1,139.0625& + 33.75 + ' 260 = 2,500& + 50 + '

Solution: & ( 0.133, ( 9.00, ' ( 143, the same results as in the text (though not exactly, due to rounding in the reduction in consumption at the optimal DOH). 4. We substitute the given data in the equation of a parabola = & 2 + + ' to obtain 2=&+ +' 9 = 4& + 2 + ' 19 = 9& + 3 + '. Solving the system gives & = 32 , = 52 , ' = 2, so the equation is = 32 2 + 52 2.

5. No. The given points all lie on a straight line (with slope 7). When we try to find the equation of a parabola through these three points, we will get & = 0. 6. No. There is only one parabola passing through the first three points (the one in Exercise 4) and, when = 1, this gives


Solutions Chapter 4 Case Study 3 5 = 2 = 3 + 2, 2 2 so it does not pass through ( 1, 2). 7. Model: = & 3 + 2 + ' + , = 10, = 180 : = 20, = 230 : = 30, = 255 : = 50, = 200 :

180 = 1,000& + 100 + 10' + , 230 = 8,000& + 400 + 20' + , 255 = 27,000& + 900 + 30' + , 200 = 125,000& + 2,500 + 50' + ,

Solution: & = 0.00125, = 0.05, ' = 7.375, , = 112.5 So, = 0.00125 3 0.05 2 + 7.375 + 112.5 Graph:

For the maximum value, ( 256.5, DOH = ( 33%, Cost ( 5,000 + 50(33 10) = $6,150.


Solutions Section 5.1 Section 5.1 1. =

1 5 0

1 4

;

has 1 row and 4 columns. Therefore, it is a 1 × 4 matrix. 13 is the entry in row 1 and column 3, so 13 = 0. 2. =

44 55 has 1 row and 2 columns. Therefore, it is a 1 × 2 matrix. 12 is the entry in row 1 and column 2, so 12 = 55. 3. =

5 2

1

2 8

has 4 rows and 1 column. Therefore, it is a 4 × 1 matrix.

11 is the entry in row 1 and column 1, so 11 = 4. =

15 18 6

0

6

5

11

12

5 . 2

has 4 rows and 3 columns. Therefore, it is a 4 × 2 matrix.

48 18 31 is the entry in row 3 and column 1, so 31 = 6.

5.

=

21

13

22

1

23

2

3

... ... ...

1 2

has

rows and columns. Therefore, it is a × matrix.

22 is the entry in row 2 and column 2, so

6. =

2 1

0

22 =

22 .

has 2 rows and 3 columns. Therefore, it is a 2 × 3 matrix. 3 5 3 21 is the entry in row 2 and column 1, so 21 = 3. 7. =

1

3

has 2 rows and 2 columns. Therefore, it is a 2 × 2 matrix. 5 6 12 is the entry in row 1 and column 2, so 12 = 3. 8. =

has 2 rows and 4 columns. Therefore, it is a 2 × 4 matrix. +1 3 0 23 is the entry in row 1 and column 2, so 23 = 3. 9. =

1 2

has 1 row and columns. Therefore, it is a 1 × matrix.

1 is the entry in row 1 and column , so 1 = .


has 1 row and 4 colums. Therefore, it is a 1 × 4 matrix. 1 is the entry in row 1 and column , so 1 = .

10.

=

Solutions Section 5.1

+ +

=

3 4

=

+ 5 4 Equating corresponding entries gives + = 3 + = 5 + = 7 = 4 Substitute = 4 in the next-to-last equation to get = 3. Then substitute into the second to get = 2, then into the first to get = 1. Thus, = 1, = 2, = 3, = 4 11.

0 0

0 6 = 0 = = 0 = = 0 = = 6 Thus, = = = = 6. 12.

13. We obtain + by adding corresponding entries: 0 1 0.25 1 0.25 2 + = 1 0 + 0 0.5 = 1 0.5 1

14. =

0 1

2

1

1

0

2

1

1 1

1

3

1

1 1

=

1 0 0

5

2

0

1 3

15. To obtain + , add corresponding entries of and and subtract those of : 0 1 0.25 1 1 1 0.75 1 + = 1 0 + 0 0.5 1 1 = 0 0.5 1

16. 12 = 12

0.25 1 0

1

0

17. 2 = 2 1

1

2

0.5 = 3

1 0

2

3 0

1

12 1 1

12

3

1 1

6

36

1 1

1 1

=

1 1 1

1

1 5

1

6


0

1

1

18. 2 + 0.5 = 2 1

0

+ 0.5 1

2

1

1

0.5

2.5

Solutions Section 5.1 1

1 1

2.5

2.5

=

0.5

3.5

19. The transpose, , of is obtained by writing the rows of as columns: 0 1 0 1 1 = 1 0 ; = 1 0 2 1 2

Thus, 2 = 2

0

3

2

26. 2 4 = 2 3

27. =

0

0

0

5 1

2

1 1

1 1 0

2

1 1 0

2

1 1

1

1

28. 2 = 2 1

1

0

0

5

1 = 1

2

1

3

0

3 0

1

0

0

1 2

0

1

4 3 1

1

0

1

1 2

1

4

= =

1 2 5

5

2 4 4

3

=

4

2 3

3

1

15 3

1

1

1

3

1

+

1

5

4 4

5 1

1

9

5

1

3

4 1 1

3

=

5 1

4

1

0

4

1

5 2 1 2

0

=

=

5 1

3 2

=

=

3

1

3

Therefore, 3 = 3 0

0

1

1 1

1 1 1

1

0

4

1

5 1

1

1 1

5 1

25. 2 = 2

3

+

1

5 1

3

0

2

2 2

2 0

+3

1

0

0

=

1 1

1 0

23. + = 1 2

2

0

1 1

21. + =

24. 12 =

1 1

1 0

20. + 3 =

22. =

0

4 6

=

4

2 +

5 + 3 +

1

3

6

0

4

4 4 18

1+ 2


Solutions Section 5.1 29. TI-83/84 Plus format: [A]-[C] Online Matrix Algebra Tool format: A-C 1.5 2.35 5.6 10 20 30 8.5 22.35 24.4 = = 44.2 0 12.2 54.2 20 42.2 10 20 30 30. TI-83/84 Plus format: [C]-[A] Online Matrix Algebra Tool format: C-A 10 20 30 1.5 2.35 5.6 8.5 22.35 24.4 = = 44.2 0 12.2 10 20 30 54.2 20 42.2 31. TI-83/84 Plus format: 1.1[B] Online Matrix Algebra Tool format: 1.1*B 1.4 7.8 1.54 8.58 1.1 = 1.1 5.4 0 = 5.94 0 5.6 6.6

6.16 7.26

32. TI-83/84 Plus format: -0.2[B] Online Matrix Algebra Tool format: -0.2*B 1.4 7.8 0.28 1.56 0.2 = 0.2 5.4 0 = 1.08 0 5.6 6.6

1.12 1.32

33. TI-83/84 Plus format: [A]T+4.2[B] Online Matrix Algebra Tool format: A^T+4.2*B 1.5 44.2 1.4 7.8 7.38 76.96 + 4.2 = 2.35 0 +4.2 5.4 0 = 20.33 0 5.6

12.2

5.6 6.6

34. TI-83/84 Plus format: ([A]+2.3[C])T 24.5 21.2 ( + 2.3 ) = 43.65 46 74.6

29.12 39.92

Online Matrix Algebra Tool format: (A+2.3*C)^T

56.8

35. TI-83/84 Plus format: (2.1[A]-2.3[C])T 2.3*C)^T 19.85 115.82 (2.1 2.3 ) = 50.935 46

Online Matrix Algebra Tool format: (2.1*A-

36. TI-83/84 Plus format: ([A]-[C])T-[B] 9.9 46.4 ( ) = 27.75 20

Online Matrix Algebra Tool format: (A-C)^T-B

57.24

30

94.62

35.6

37. Regard each row of the given table as a matrix: Available in 2018 = 2,400 810 2,600 610 Change in 2019 = 600 90 300 270

Change in 2020 = 400 30 200 30 Available in 2019 = Available in 2018 + Change in 2019


= 2,400 810 2,600 610 + 600 90 300 270 = 3,000 900 2,300 340 Available in 2020 = Available in 2019 + Change in 2020 = 3,000 900 2,300 340 + 400 30 200 30 = 3,400 930 2,500 370 Solutions Section 5.1

38. Regard each row of the given table as a matrix: Prices in 2018 = 289 200 226 385 Change in 2019 = 12 13 10 16

Change in 2020 = 37 20 23 48 Prices in 2019 = Prices in 2018 + Change in 2019 = 289 200 226 385 + 12 13 10 16 = 301 213 236 401 Prices in 2020 = Prices in 2019 + Change in 2020 = 301 213 236 401 + 37 20 23 48 = 338 233 259 449 39. Write the given inventory table as a 2 × 3 matrix: 1,000 2,000 5,000 Inventory = 1,000 5,000 2,000 Write the given sales figures as a similar matrix: 700 1,300 2,000 Sales = 400 300 500

We then compute the remaining inventory by subtracting the sales: Remaining Inventory = Inventory Sales 1,000 2,000 5,000 700 1,300 2,000 300 700 3,000 = = 1,000 5,000 2,000 400 300 500 600 4,700 1,500 40. The inventory at the start of the year is obtained from the table in Exercise 39: 1,000 2,000 5,000 Inventory = 1,000 5,000 2,000 The sales each month are also given in Exercise 39: 700 1,300 2,000 Sales = 400 300 500

700 1,300 2,000 4,200 7,800 12,000 = 400 300 500 2,400 1,800 3,000

a. Total sales after 6 months = 6 Sales = 6

b. Each month, we subtract sales and add restock: Restock =

600 1,500 1,500 500 500 500

After 6 months: Remaining Inventory = Inventory 6 Sales + 6 Restock 1,000 2,000 5,000 700 1,300 2,000 600 1,500 1,500 = 6 +6 1,000 5,000 2,000 400 300 500 500 500 500 = 41.

400 3,200 2,000 1,600 7,200 2,000


Solutions Section 5.1

Arrange the revenue and costs in two 3 × 3 matrices: 10,000 9,000 11,000 2,000 1,800 2,200 Revenue = 8,000 7,200 8,800 Cost = 2,400 1,440 1,760 4,000 5,000 6,000 1,200 1,500 2,000 Profit = Revenue Cost 10,000 9,000 11,000 2,000 1,800 2,200 8,000 7,200 8,800 = 8,000 7,200 8,800 2,400 1,440 1,760 = 5,600 5,760 7,040 4,000 5,000 6,000 1,200 1,500 2,000 2,800 3,500 4,000 42.

Arrange the costs and profits in two 3 × 3 matrices: 1,800 2,200 2,400 10,000 14,000 16,000 Cost = 1,400 1,700 1,200 Profit = 8,000 12,000 14,000 1,500 2,000 1,300 9,000 14,000 12,000 Revenue = Profit + Cost 10,000 14,000 16,000 1,800 2,200 2,400 11,800 16,200 18,400 = 8,000 12,000 14,000 + 1,400 1,700 1,200 = 9,400 13,700 15,200 9,000 14,000 12,000 1,500 2,000 1,300 10,500 16,000 13,300 43. Row vectors for the populations, in millions: 2010 distribution = = [55.4 67.0 72.1 114.9]; 2020 distribution = = [57.6 69.0 78.6 126.3]; Net change 2010 to 2020 = = = [2.2 2.0 6.5 11.4] As all the net changes are positive, they represent net increases in the population. Population in 2030 = + = [57.6 69.0 78.6 126.3] + [2.2 2.0 6.5 11.4] = [59.8 71.0 85.1 44. Row vectors for the populations, in millions:

137.7]


Solutions Section 5.1 2000 distribution = = [53.6 64.4 63.2 100.2] Net change 2000 to 2010 = = [1.8 2.6 8.9 14.7] Population in 2020 = + 2 = [57.2 69.6 81.0 129.6] The actual population in the South in 2020 was 126.3 million (preceding exercise), which is 3.3 million less than the predicted population of 129.6 million. 45. Regard each row in the table as a matrix. Foreclosures in California = [55,900 51,900 54,100 56,200 59,400] Foreclosures in Florida = [19,600 19,200 23,800 22,400 23,600] Foreclosures in Texas = [8,800 9,100 9,300 10,600 10,100] Total Foreclosures = Foreclosures in California\quad+ Foreclosures in Florida + Foreclosures in Texas = [55,900 51,900 54,100 56,200 59,400] + [19,600 19,200 23,800 22,400 23,600] +[8,800 9,100 9,300 10,600 10,100] = [84,300 80,200 87,200 89,200 93,100] 46. Regard each column in the table as a matrix. 55,900 51,900 Foreclosures in April = 19,600 Foreclosures in May = 19,200 8,800 9,100 54,100 Foreclosures in June = 23,800 9,300 59,400 Foreclosures in Aug = 23,600 10,100

56,200 Foreclosures in July = 22,400 10,600

Total Foreclosures = Foreclosures in April + Foreclosures in May + Foreclosures in June + Foreclosures in July + Foreclosures in Aug 55,900 51,900 54,100 56,200 59,400 277,500 = 19,600 + 19,200 + 23,800 + 22,400 + 23,600 = 108,600 8,800 9,100 9,300 10,600 10,100 47,900 47. (Refer to the solution to Exercise 45.) The difference between the number of foreclosures in California and in Florida is given by Difference = Foreclosures in California Foreclosures in Florida = [55,900 51,900 54,100 56,200 59,400] [19,600 19,200 23,800 22,400 23,600] = [36,300 32,700 30,300 33,800 35,800] The difference was greatest in April. 48. (Refer to the solution to Exercise 46.) Foreclosures in August Foreclosures in April 59,400 55,900 3,500 = 23,600 19,600 = 4,000 10,100 8,800 1,300

The difference was greatest in Florida.

49. First, organize the starting inventory as a 2 × 3 matrix:


Solutions Section 5.1 Proc Mem Pom II

500

Pom Classic 200

Tubes

5,000 10,000

2,000 20,000

500 5,000 10,000

200 2,000 20,000 The parts used in making one of each computer can also be arranged in a matrix: 2 16 20 Use = 1 4 40 a. After 2 months, the company has made 100 of each computer, so the inventory remaining is Inventory Remaining = Inventory 100 Use 500 5,000 10,000 2 16 20 300 3,400 8,000 = = 100 200 2,000 20,000 1 4 40 100 1,600 16,000 b. After months, the company has made 50 of each computer. Therefore, the inventory remaining is 500 5,000 10,000 2 16 20 500 100 5,000 800 10,000 1,000 = 50 200 2,000 20,000 1 4 40 200 50 2,000 200 20,000 2,000 Thus, Inventory =

The 1,1 entry is zero after = 5 months The 1,3 entry is zero after = 10 months The 2,2 entry is zero after = 10 months

The 1,2 entry is zero after = 6.25 months The 2,1 entry is zero after = 4 months The 2,3 entry is zero after = 10 months

Thus, after 4 months, the first entry becomes zero, meaning that the company will run out of Pom Classic processor chips after 4 months. 50. As in Exercise 49, organize the starting inventory as a 2 × 3 matrix: Proc Mem Tubes Pom II

500

Pom Classic 200

5,000 10,000

2,000 20,000

500 5,000 10,000

200 2,000 20,000 The parts used in making one of each computer can also be arranged in a matrix: 2 16 20 Use = 1 4 40 We also have monthly restocking of parts: 100 1,000 3,000 Restock = 50 1,000 2,000 a. In 6 months, the company builds 300 of each type of computer, so the inventory remaining is Inventory Remaining = Inventory 300 Use +6 Restock 500 5,000 10,000 2 16 20 100 1,000 3,000 = +6 300 200 2,000 20,000 1 4 40 50 1,000 2,000 Thus, Inventory =

500 6,200 22,000

200 6,800 20,000 b. In months, the inventory remaining is: Inventory Remaining = Inventory 50 Use + Restock =


=

500 5,000 10,000

2 16 20

50

200 2,000 20,000

100 1,000 3,000

Solutions Section 5.1

1

4

+

40

500 5,000 + 200 10,000 + 2,000

50

1,000 2,000

200 2,000 + 800 20,000 Since the stock is always increasing, the company will never run out of parts. =

51. a. Arrange the 2019 tourism figures in a matrix: =

1,020 150

440 120

1,650 1,560

Then, arrange the predicted changes from 2019 to 2020 in a new matrix: = Tourism in 2020 = Tourism in 2019 + Change = + =

1,000 160

480 120

20 40 10 50

0

50

1,700 1,610 b. The average of the tourism figures is obtained by taking the average of the entries in and . To obtain the average, add and divide by 2: Average = 12 ( + ) = 2019 figures =

1,020 150

52. a. = 2019 Tourism = =

40

120

1,650 1,560

1 1 Average = ( + ) = 2 2

240

440

2,020 310

920 240

3,350 3,170

1,020

440

90

1,020

150

120

1,650 1,560

30

150

= 2020 figures = 1,010 155

460 120

1,000 160

= 2020 Tourism =

440 120

=

780 110

120

1,700 1,610

1,675 1,585

=

480

240

90

30

440 450

350 90

40

(From part (a))

440 450 1,650 1,560 1,210 1,110 represents the change in visitors from 2019 to 2020 (that they are all negative indicates lower 2020 tourism numbers in each category). b. 2021 tourism = 2020 tourism + ( 1) 12 × Change from 2019 to 2020 = 12 ( ) = 12 ( + ) 53. No; for two matrices to be equal, they must have the same dimensions.

54. = 0, the 2 × 3 zero matrix. For two matrices to be equal, they must have the same corresponding entries, and so their difference has each entry zero.

55. ( + ) = + The left-hand side is the !"th entry of + . The right-hand side is the sum of the !"th entries of and . Thus, the equation tells us that the !" th entry of the sum + is obtained by adding the !"th entries of and .


Solutions Section 5.1 56. (# ) = #( ) The !"th entry of the constant multiple # is obtained by multiplying the !"th entry of by #.

57. If is any matrix, then 11 , 22 , ..., , ... denote the entries going down the main diagonal (top left to bottom right). Thus, if = 0 for every !, then would have zeros down the diagonal: 0 # # # # # 0 # # # = # # 0 # # # # # 0 # # # # # 0 (The symbols # indicate arbitrary numbers.)

58. It would have zeros everywhere except possibly down the main diagonal, such as in =

1 0

0 0

0 0

0 1 0 0

0

3

.

59. The transpose of an $ × matrix is the × $ matrix obtained by writing its rows as columns. Thus, the entry originally in Row " and Column ! winds up in Row ! and Column " when a matrix is transposed. In other words,!"th entry of the transpose = "!th entry of the original matrix: ( ) =

60. Answers will vary.

a.

1 4 4 0

b.

0

4 5

4 6

5

1

1 11

61. In a skew-symmetric matrix, the entry in the !" position is the negative of that in the "! position. In other words, each entry is the negative of its mirror image in the diagonal. What about the diagonal entries? Each diagonal entry stays the same when the matrix is transposed. So, in a skew-symmetric matrix, each diagonal entry must equal its own negative. The only way this can happen is if the diagonal entries are zero. 0 4 5 0 4 Answers will vary. a. b. 4 0 1 4 0 5 1 0

62. A matrix that is both symmetric and skew symmetric must be zero, since the !"th entry must equal both the "!th entry and its negative.

63. The associativity of matrix addition is a consequence of the associativity of addition of numbers, since we add matrices by adding the corresponding entries (which are real numbers). 64. No; Subtraction of real numbers is not associative, and therefore matrix subtraction cannot be associative either (1×1 matrices are just real numbers). For instance, [4] ([2] [1]) = [4] [1] = [3], whereas ([4] [2]) [1] = [2] [1] = [1]. 65. Answers will vary. 66. Answers will vary.


Solutions Section 5.2 Section 5.2

9

1. 1 3 1

3. 1

1

1

13

1 2

1

4. 1 1

5 6

=

3 4 1 4

6. 4 1 1 5 +

1 8

= 16 + 0 8 = 24

3 1 1 = 4 + 4 = 2

= 4 + =

is undefined. (The dimensions are 1 × 3 and 2 × 1; the 3 and the 2 do not match.)

1

1 2 is undefined. 3 1

9. 1 1 10. 2

1 + 12 3

= 2 +

1

7. 1 3 2

2. 4 0 1

5. 0 2 1

8. 3 2

=

= 9 + 3 + 1 = 13

4

0

1

3

1 2 1

= (3 + 0) ( 1 + 1) ( 4 2) ( 3 + 1) = 3 0 6 2

3 1 4 3 4

11. 1 1 2 3 6 37 7 12. 0 1 1 2 8 3 23

4

0 1 3

1

= ( 6 4) (2 + 0) (8 1) (6 3) = 10 2 7 3

2

0

2

1 0 8

1

1

2

1

0

1

0 6

5

1 0

2

3 2

1 2 11

= ( 1 2 + 0 3) (2 + 1 + 10 + 24) (0 + 0 + 4 + 3) =

= (0 + 0 6 2) (0 + 1 + 0 4) (0 + 3 2 + 22) =


0 1 1

13.

1 0 1

14.

0 1 1

15.

1

0

0 1

16.

1 1

3 3

17.

0 1

18.

3 3

19.

1 1

20.

0 1

1 1

1 0

2

1 1 1 1 0 1

4 2 =

1 1 1 0

0 1

5 7 0

=

1 1

(0 + 4 + 0) (0 + 2 1) (3 + 4 + 0) (3 + 2 1)

(3 5) ( 3 + 7)

(0 + 1) (0 1) (0 + 1) (0 1)

1 1 1 1

(2 2) (3 3)

0 0 0 0

2 3

3 3 2 1

=

(0 + 2)

(0 1)

(3 + 0) ( 3 + 0)

1

0

1

1

3 3

1

0

1

1 1 4

1

2

0

1

=

(5 7) ( 5 + 7)

(2 2) (3 3)

9

17

0

4 1 7 4

2 4

=

=

4 7 1

2 4

(3 3) ( 3 + 3)

2 3

=

=

0 0 =

1

( 1 + 0 + 0)

0 1

=

(3 5) ( 3 + 7)

=

2 3

(1 + 0 8)

(0 + 1 + 8) (1 + 0 + 16) ( 1 + 1 + 0)

(0 + 0) (1 1) =

1 1

(0 + 0 4)

=

(0 + 0) (1 + 0)

=

1 1

5 7

0

1 1

0 1

1

1

4 8

3 1 1

Solutions Section 5.2

=

=

0

0

2 2

2 1 3 3

2 3 is undefined.

1

1 1 (The dimensions are 2 × 2 and 3 × 2; the 2 and the 3 do not match.) 21.

is undefined. 0 1 1 2 1 (The dimensions are 2 × 3 and 2 × 2; the 3 and the 2 do not match.) 22.

23. 2 2 0

0

1

1

24. 4 1 1 1

0

1

1 1 4

0

1

2 1

4

0 2

(1 + 0 + 0) ( 1 + 0 4) (4 + 0 1)

1 5 3 0

9

(1 + 8 + 0) (2 + 2 + 0)

9 4

4

0 = (2 2 + 0) ( 2 2 + 4) (8 + 0 + 1) = 0 1

4 0

1

(0 + 0 + 0)

(0 + 0 + 4)

(0 + 0 + 1)

( 4 + 0 + 0)

0

4

1

= (4 4 + 0) (8 1 2) ( 16 + 0 + 1) = 0 5 15 (1 + 0 + 0) (2 + 0 2)

( 4 + 0 + 1)

1 0

3


1

25.

0

1

0 1

3

1

7 0

1

1 4

0

1 0 1

1

0 0

1 1

(1 + 0 + 2 + 0)

1

1 0

2 1

26.

1 0

Solutions Section 5.2

1

2

2

4

2 1 1

1

0

( 1 3 + 0 + 0) ( 2 + 0 + 2 + 0)

=

3 2 1

(0 + 3 + 0 + 0)

3

4 0

=

3

(1 3 14 + 0)

=

16

( 1 + 0 + 4 + 4) ( 1 + 0 4 + 1)

=

(1 + 3 + 2 + 1)

7

4 7

27. An on-line matrix algebra utility is available on the Website at 1.1 2.3

3.4

1.2

2.1

0

0.23

3.3

Student Web Site → Online Utilities → Matrix Algebra Tool

3.4 4.4 2.3

0

2.3

2.2

1.1

1.1

3.4 4.8 4.2 3.4

5.6

1.2 1.3 1.1 1.1 1 2.2 TI-83/84 Plus format: [A]*[B]

1

=

5.36

21.65

13.18 5.82 16.62 11.21

9.9

0.99

9.8 2.34 2.46 2.1 Online Matrix Algebra Tool format: A*B

28. An on-line matrix algebra utility is available on the Website at 1.2 3.3

2.3

3.4

4.5

9.8 1 1.1

4.4

5.5

6.6

2.3 4.3 2.2 1.1

8.8 2 2.2 7.7 3 3.3

2.2 1.2 1 1.1 6.6 4 4.4 TI-83/84 Plus format: [A]*[B]

29. =

87.88

34

Student Web Site → Online Utilities → Matrix Algebra Tool

=

156.97

55

24.98 8.5

37.4 60.5 9.35

10.56 1.2 1.32 Online Matrix Algebra Tool format: A*B

0 1 1 1 0 0 1 1 0 0 0 1 0 0 0 0

2 = =

0 1 1 1

0 1 1 1

0 0 0 0

0 0 0 0

0 0 1 1 0 0 0 1

0 0 1 1 0 0 0 1

(0 + 0 + 0 + 0) (0 + 0 + 0 + 0) (0 + 1 + 0 + 0) (0 + 1 + 1 + 0)

(0 + 0 + 0 + 0) (0 + 0 + 0 + 0) (0 + 0 + 0 + 0) (0 + 0 + 1 + 0)

= (0 + 0 + 0 + 0) (0 + 0 + 0 + 0) (0 + 0 + 0 + 0) (0 + 0 + 0 + 0) = (0 + 0 + 0 + 0) (0 + 0 + 0 + 0) (0 + 0 + 0 + 0) (0 + 0 + 0 + 0)

(0 + 0 + 0 + 0) (0 + 0 + 0 + 0) (0 + 0 + 0 + 0) (0 + 0 + 0 + 0)

0 0 1 2 0 0 0 1 0 0 0 0 0 0 0 0


3 = 2 =

0 1 1 1

0 0 1 2

0 0 0 0

0 0 0 0

0 0 1 1 0 0 0 1

Solutions Section 5.2

0 0 0 1 0 0 0 0

(0 + 0 + 0 + 0) (0 + 0 + 0 + 0) (0 + 0 + 0 + 0) (0 + 1 + 0 + 0)

(0 + 0 + 0 + 0) (0 + 0 + 0 + 0) (0 + 0 + 0 + 0) (0 + 0 + 0 + 0)

= (0 + 0 + 0 + 0) (0 + 0 + 0 + 0) (0 + 0 + 0 + 0) (0 + 0 + 0 + 0) = (0 + 0 + 0 + 0) (0 + 0 + 0 + 0) (0 + 0 + 0 + 0) (0 + 0 + 0 + 0)

(0 + 0 + 0 + 0) (0 + 0 + 0 + 0) (0 + 0 + 0 + 0) (0 + 0 + 0 + 0)

4 = 3 =

0 1 1 1 0 0 1 1 0 0 0 1 0 0 0 0

0 0 0 1

0 0 0 0

0 0 0 0 0 0 0 0

=

0 0 0 0

0 0 0 0 0 0 0 0

0 2 0 1 0 0 2 0 0 0 0 0 0

2 = =

0 0 0 0 0 0 0 0

0 0 0 0 0 0 0 0 0 0 0 0 0 0 0 0

.

0 2 0

0 2 0 1

0 2 0 1

0 0 0

0 0 0

0 0 2 0 0 0

0 2 0

0 0 2 0 0 0

0 2 0

(0 + 0 + 0 + 0) (0 + 0 + 0 + 0) (0 + 4 + 0 + 0) (0 + 0 + 0 + 0)

(0 + 0 + 0 + 0) (0 + 0 + 0 + 0) (0 + 0 + 0 + 0) (0 + 0 + 4 + 0)

= (0 + 0 + 0 + 0) (0 + 0 + 0 + 0) (0 + 0 + 0 + 0) (0 + 0 + 0 + 0) = (0 + 0 + 0 + 0) (0 + 0 + 0 + 0) (0 + 0 + 0 + 0) (0 + 0 + 0 + 0)

(0 + 0 + 0 + 0) (0 + 0 + 0 + 0) (0 + 0 + 0 + 0) (0 + 0 + 0 + 0)

3 = 2 =

0 0 0 0

0 0 0 0

Continuing to multiply by continues to yield the zero matrix, so 100 =

30. =

0 0 0 1

0 2 0 1

0 0 4 0

0 0 0

0 0 0 0

0 0 2 0 0 0

0 2 0

0 0 4 0 0 0 0 4 0 0 0 0 0 0 0 0

0 0 0 4 0 0 0 0

(0 + 0 + 0 + 0) (0 + 0 + 0 + 0) (0 + 0 + 0 + 0) (0 + 8 + 0 + 0)

(0 + 0 + 0 + 0) (0 + 0 + 0 + 0) (0 + 0 + 0 + 0) (0 + 0 + 0 + 0)

= (0 + 0 + 0 + 0) (0 + 0 + 0 + 0) (0 + 0 + 0 + 0) (0 + 0 + 0 + 0) = (0 + 0 + 0 + 0) (0 + 0 + 0 + 0) (0 + 0 + 0 + 0) (0 + 0 + 0 + 0)

(0 + 0 + 0 + 0) (0 + 0 + 0 + 0) (0 + 0 + 0 + 0) (0 + 0 + 0 + 0)

0 0 0 8 0 0 0 0 0 0 0 0 0 0 0 0


4 = 3 =

0 2 0 1 0 0 2

0 0 0 8

0

0 0 0

0 0 0 0

2

0 0 0

0 0 0 0

Solutions Section 5.2

0 0 0 0

0

0 0 0 0

=

0 0 0 0 0 0 0 0 0 0 0 0

Continuing to multiply by continues to yield the zero matrix, so 100 =

31. =

0

10

1 0 1 0

1 0

0 1

1

1 1

3

0

10

1 0 1 0

1 0

1

1

1

=

1 1 TI-83/84 Plus format: [A]*[C]

0 0 0 0 0 0 0 0

(0 1 + 0 + 0)

=

.

4

1

0

0

1 7

Online Matrix Algebra Tool format: A*B

1 1 1

0 0 0 0

(0 + 0 1 + 0) ( 10 + 0 + 3 + 0)

=

5 0 TI-83/84 Plus format: [A]*[B]

32. =

(0 1 + 0 + 5)

0 0 0 0

(0 1 + 0 + 1)

(0 1 + 0 + 1)

(10 + 0 + 1 + 0) ( 10 + 0 + 1 + 0)

=

11 9

Online Matrix Algebra Tool format: A*C

% 0 1 1 1 ( 1 1 * 0 1 0 1 ' 1 1 33. ( ) = * 10 0 1 0 '' 1 3 1 1 * ' 5 0 * & 1 1 ) 1 0 0 1 0 1 0 0 4 1 = = 10 0 1 0 2 2 12 2 4 1 TI-83/84 Plus format: [A]*([B]-[C]) Online Matrix Algebra Tool format: A*(B-C) % 0 1 1 1 ( ' 1 1 1 1 * 0 1 0 1 34. ( ) = ' * 3 1 ' 1 1 * 10 0 1 0 ' 5 0 * & 1 1 ) 1 0 1 1 0 0 0 0 0 0 1 0 1 0 0 0 = = 2 2 10 0 1 0 20 2 2 2 4 1 10 4 1 4 TI-83/84 Plus format: ([B]-[C])*[A] Online Matrix Algebra Tool format: (B-C)*A 35. =

1 1 0

2

3

0

5 1

1 1

2 10

1

2

2 2

0 2 10 2 2 TI-83/84 Plus format: [A]*[B] Online Matrix Algebra Tool format: A*B =


36. =

1 1 0

2

1

4

1 4

Solutions Section 5.2

2

2

8

0 2 2 2 Online Matrix Algebra Tool format: A*C =

8

1 1 3 0 1 1 37. ( + ) = 0 2 + + 5 1 1 4 , 0 2 1 1 3 + 2 + 2 6 + 1 1 + = 0 2 = 10 + 2 2 + 2 10 5 + 1 + 5 0 2 2 2 2 10 10 Online Matrix Algebra Tool format: A*(B+C) 1 1 3 0 1 1 0 2 + + 5 1 1 4 , 0 2 1 1 3+ 1 3 + 1 2 1 + = 0 2 = 5 + 1 + 5 + 17 + 2 5 0 2 Online Matrix Algebra Tool format: (B+C)*A

38. ( + ) =

39. a. - 2 = - - = b. - 4 = - 2 - 2 = c. - 8 = - 4 - 4 =

0.2 0.8 0.2 0.8

0.2 0.8 0.2 0.8

0.2 0.8 0.2 0.8

0.2 0.8 0.2 0.8

0.2 0.8 0.2 0.8

0.2 0.8 0.2 0.8

(0.04 + 0.16) (0.16 + 0.64) (0.04 + 0.16) (0.16 + 0.64)

=

0.2 0.8 0.2 0.8

=

0.2 0.8 0.2 0.8

=

d. - 1,000 = - 8 - 8 - 8 ... - 8 (125 times) = 40. a. - 2 = - - = b. - 4 = - 2 - 2 = c. - 8 = - 4 - 4 =

0.1 0.1 0.9 0.9

0.1 0.1 0.9 0.9

0.1 0.1 0.9 0.9

0.1 0.1 0.9 0.9

0.1 0.1 0.9 0.9

0.1 0.1 0.9 0.9

0.1 0.1 0.9 0.9

=

b. - 4 = - 2 - 2 =

0.1 0.9 0

1

0.01 0.99 0

1

0.1 0.9 0

1

=

0.01 0.99 0

1

again. again.

0.1 0.1 0.9 0.9

.

0.01 0.99 0

=

1

0.0001 0.9999 0

0.1 0.1

.

(0.09 + 0.81) (0.09 + 0.81)

d. - 1,000 = - 8 - 8 - 8 ... - 8 (125 times) = 41. a. - 2 = - - =

0.2 0.8

0.1 0.1

=

0.2 0.8

again.

0.2 0.8

0.9 0.9

=

0.2 0.8

again.

(0.01 + 0.09) (0.01 + 0.09)

=

=

1

0.9 0.9


c. - 8 = - 4 - 4 =

0.0001 0.9999 0

0.0001 0.9999

1

0

1

b. - 4 = - 2 - 2 = c. - 8 = - 4 - 4 =

0

1

0.8 0.2 1

0

0.96 0.04 1

0

0.8 0.2 0

1

0

=

0.96 0.04

0.9984 0.0016 rounded to 4 decimal places.

1

0 1 0 1 1

0.3 0.3 0.4 0.3 0.3 0.4 0.3 0.3 0.4

0 1

0

0.96 0.04 =

1

0

rounded to 4 decimal places.

0.9984 0.0016

0

0.9984 0.0016

d. - 1,000 = - 8 - 8 - 8 ... - 8 (125 times) . 43. a. - 2 = - - =

.

1

d. - 1,000 = - 8 - 8 - 8 ... - 8 (125 times) . 42. a. - 2 = - - =

0 1

Solutions Section 5.2

1 0

=

1

0

0.999997 0.00000256

.

1 0 1 0

1 0

0.3 0.3 0.4

0.3 0.3 0.4 = 0.3 0.3 0.4 0.3 0.3 0.4 = 0.3 0.3 0.4

0.3 0.3 0.4

b. - = - - = - - = - again c. - 8 = - 4 - 4 = - - = - again d. - 1,000 = - 8 - 8 - 8 ... - 8 (125 times) = - again The rows of - are the same, and the entries in each row add up to 1. These calculations suggest that, if is any square matrix with identical rows such that the entries in each row add up to 1, then - - = - . 4

2

2

0.3 0.3 0.3

44. a. - = - - =

0.9

2

0.9

0.4

0.4

0.9 0.4

0.3 0.3 0.3 0.9 0.4

0.9 0.4

0.9 = 0.4

0.3 0.3 0.3 0.9 0.4

0.9 0.4

0.9 = - again 0.4

b. - = - - = - - = - again c. - 8 = - 4 - 4 = - - = - again d. - 1,000 = - 8 - 8 - 8 ... - 8 (125 times) = - again The columns of - are the same, and the entries in each row add up to 1. These calculations suggest that, if - is any square matrix with identical columns such that the entries in each column add up to 1, then - - = -. 4

2

2

45. 4

2

1 4 3 4

1 3

3

= 1

2 + 4

4 + 34 + 13

3

= 1

0 0 3 0 0 3 Equating entries gives the following system of linear equations: 2 + 4 = 3 4 + 34 + 13 = 1 3 = 0


1

13

1 4 3

1 3

+ 4

Solutions Section 5.2

3

= 1

13 3 + 13

3

= 1

2 2 3 0 1 3 + Equating entries gives the following system of linear equations: + 4 = 3 13 3 + 13 = 1 46.

3 + = 2

47.

1 1 0 1 1

1

2 4

=

1

=

2

2

+

+ + 2 + 4

=

1 2

Equating entries gives the following system of linear equations: + = 1 + + 2 + 4 = 2

48.

0

1

6 1

1 5 0 0

9

+ 6 + 5

=

2 9

Equating entries gives the following system of linear equations: + 6 + = 2 5 = 9

49. = 4; //2 = 0 The matrix form is 0 = , where is the matrix of coefficients, 0 is the column matrix of unknowns, and is the column matrix of right-hand sides. 1 1 4 = ,0= , = 2 1 0 Thus, the matrix system is 1 1 4 = . 2 1 0

50. 2 + = 7; // = 9 The matrix form is 0 = , where is the matrix of coefficients, 0 is the column matrix of unknowns, and is the column matrix of right-hand sides. 2 1 7 = ,0= , = 9 1 0 Thus, the matrix system is 2 1 7 = . 9 1 0 51. + = 8; //2 + + = 4; // 3 +2= 1 4

The matrix form is 0 = , where is the matrix of coefficients, 0 is the column matrix of unknowns, and is the column matrix of right-hand sides.


=

1 1 1 2 1

1

0

3 4

0=

1 2

Thus, the matrix system is 1 1 1 2 1 3 4

0

1

8

, =

Solutions Section 5.2

4 1

8

= 4 .

1 2

1

52. + + 2 = 2; //4 + 2 = 8; // 2 3 = 4 =

1 4

1 2

1 2

13

1 2

13

2

1 0 = 0

, =

Thus, the matrix system is 1 1 2 2 4 2 1 = 8 . 0

2 8 4

4

53. We have three prices and three quantities. Arrange the prices as a row matrix and quantities as a column matrix (so that we can multiply them): 50 Price = 15 10 12 Quantity = 40 30

Revenue= Price × Quantity = 15 10 12

50

40 = 750 + 400 + 360 = 1,510 30

54. We have three prices and three quantities. Arrange the prices as a row matrix and quantities as a column matrix (so that we can multiply them): 100 Price = 10 5 8 Quantity = 50 70

Revenue = Price × Quantity = 10 5 8

100 50 70

= 1000 + 250 + 560 = 1,810

55. We have three prices (stated as "costs") and three quantities. Arrange the prices as a row matrix and quantities as a column matrix (so that we can multiply them): 10 Total Cost = Price × Quantity = 1.90 2.50 1.15 20 = 80.5 Therefore, the total cost is $80.5 million.

10

56. We have three prices (stated as "costs") and three quantities. Arrange the prices as a row matrix and


Solutions Section 5.2 quantities as a column matrix (so that we can multiply them): 10 Total Cost = Price × Quantity = 4.50 1.75 1.95 30 = 107.25 5

Therefore, the total cost is $107.25 million. 57. We are asked to write the prices as a column matrix: Price:

Hard 30 10

Plastic 15 We are asked to find the revenue. Since the prices are given as a column matrix, we expect it to go on the right when we multiply rows by columns. So we write the formula for revenue with prices on the right: Revenue = Quantity × Price (Note the reversal of the usual order.) This means that the quantities (hard, soft, plastic) should appear as rows in the quantity matrix: 30 700 1,300 2,000 Quantity: , Price = 10 400 300 500 15 Soft

Revenue = Quantity × Price =

700 1,300 2,000 400

300

500

58. From Exercise 57: Quantity: 1 =

700 1,300 2,000 400

300

500

Price = - =

We are also given costs: 10 Cost = = 5 per book

30

10 = 15

$64,000 $22,500

30

10 per book 15

10 Profit = Revenue − Cost = 1- 1 = 1(- ) 30 10 700 1,300 2,000 %' (* = 10 5 400 300 500 ' * & 15 10 ) =

20 700 1,300 2,000 $30,500 5 = 400 300 $12,000 500 5

59. We are given the mean income per person for each age group as well as the number of people. Total income = Income per person × Number of women in 2030 Thus, we could write the income per person as a row matrix and the number of women in 2030 as a column matrix: Income per person = 20 59 68 45 thousand dollars


21 Number of women in 2030 =

46 41 35

Solutions Section 5.2

million 21 46

Total income = Income per person × Number of women in 2030 = 20 59 68 45

41

= $7,497 thousand million, or $7,497 billion The total income, rounded to two significant digits, is $7,500 billion (or $7.5 trillion).

35

60. We are given the mean income per person for each age group as well as the number of people. 23 Total income = Income per person × Number of men in 2040 = 20 59 68 45

48 44 31

= $7,679 thousand million or $7,679 billion The total income, rounded to two significant digits, is $7,700 billion (or $7.7 trillion). 61. Take 2 to be the income per person; 2 =

20 59 68 45 , take 3 and 4 to be, respectively, the

62. Take 2 to be the income per person; 2 =

20 59 68 45 , take 3 and 4 to be, respectively, the

female and male populations in 2030: 21 22 46 48 3= 4= 41 40 35 29 Then the female income is 23 and the male income is 24. The difference is therefore = 23 24 = 2(3 4), which is the single formula required. Computing, % 21 22 ( 1 ' 46 48 * 2 2(3 4) = 520 59 68 456' * = 520 59 68 456 ' 41 40 * 1 ' 35 29 * 6 & ) = $200 thousand million = $200 billion

female and male populations in 2040: 22 23 47 48 3= 4= 45 44 36 31 Then the female income is 23 and the male income is 24. The total income is therefore = 23 + 24 = 2(3 + 4), which is the single formula required. Computing, % 22 23 ( 45 ' 47 48 * 95 2(3 + 4) = 520 59 68 456' + * = 520 59 68 456 ' 45 44 * 89 ' 36 31 * 67 & ) = $15,572 thousand million (or billion) . $16,000 billion (or $16 trillion)


48

61

1 1 - =

1 1

63. - =

3,802 4,009

Solutions Section 5.2 48

61

3,802 4,009

= ( 48 + 3,802) ( 61 + 4,009) = 3,754 3,948

To interpret this, look at what we are doing when we multiply the matrices: Multiplying 1 1 by the first column of - computes the amount of domestic electric energy consumed in 2000 − the amount of imported electric energy consumed in 2000, giving the amount, in billions of kWh, by which domestic energy consumption exceeded imported energy consumption in 2000, and similarly for the product with the second column of - for the 2020 data. Thus, 1 1 - represents the amounts, in billions of kWh, domestic energy consumption exceeded imported energy consumption in 2000 and 2020. 63 1,245

64. - = -

1

57 3,005

63 1,245

1

63 + 1,245

1,182

1 57 3,005 1 57 + 3,005 2,948 The product represents the increases, in trillions of Btu, from 2000 to 2020 in consumption of solar and wind energy. =

=

=

65. Take to be the matrix given by the table: 54,100 56,200 59,400 = Total number of filings = 23,800 22,400 23,600

9,300 10,600 10,100 The percentages for the three states handled by the firm can be represented by a row matrix: Percentage handled by firm = 0.10 0.05 0.20 (We used a row so that the three percentages match the three states down each column of .) Number of foreclosures filings handled by firm = Percentage handled by firm × Total number 54,100 56,200 59,400 = 0.10 0.05 0.20 23,800 22,400 23,600 = 8,460 8,860 9,140 9,300

10,600 10,100

66. Take to be the matrix given by the table: 54,100 56,200 59,400 = Total number of filings = 23,800 22,400 23,600

9,300 10,600 10,100 The percentages of each month's filings can be represented by a column matrix: 0.1 Percentage handled by firm = 0.3

0.2 (We used a column so that the three percentages match the three months across each row of A.) Number of bankruptcy filings handled by firm = Total number × Percentage handled by firm 54,100 56,200 59,400 0.1 34,150 = 23,800 22,400 23,600 0.3 = 13,820 9,300

10,600 10,100

0.2

6,130


54,100 56,200 59,400

Solutions Section 5.2

67. =

1 1 0 , =

23,800 22,400 23,600

9,300 10,600 10,100 When we multiply by a column of , we are adding the foreclosures in California and Florida for the corresponding month. Therefore, gives number of foreclosures in California and Florida combined in each of the months shown. 68. =

54,100 56,200 59,400

23,800 22,400 23,600 , =

1 1

9,300 10,600 10,100 0 When we multiply a row of by , we are adding the foreclosures in June and July of 2011 for the corresponding state. Therefore, gives the foreclosures in June and July of 2011 combined for each of the states shown. 69. To compute the amount by which the combined foreclosures in California and Texas exceeded the foreclosures in Florida each month, we add the California and Texas entries and subtract the Florida entry in each column. This may be accomplished by multiplying each column by the row matrix 1 1 1 : 1 1 1

54,100 56,200 59,400

23,800 22,400 23,600 = 39,600 44,400 45,900

9,300 10,600 10,100 This product gives the result for each of the three months. To add them up, we can multiply on the right by a column matrix whose entries are all 1: 1 39,600 44,400 45,900 1 = 129,900 1 Therefore, the matrix product we used was 54,100 56,200 59,400 1 1 1

23,800 22,400 23,600 9,300

10,600 10,100

1

1 = 129,900 . 1

70. To compute the amount by which the combined foreclosures in August exceeded foreclosures in June, for each state we subtract the June entry in each row from the corresponding August entry. This may be 1 accomplished by multiplying each row by the column matrix 0 : 54,100 56,200 59,400 23,800 22,400 23,600

1 0

5,300

= 200

1

9,300 10,600 10,100 1 800 This product gives the result for each of the three regions. To add them up, we can multiply on the left by a row matrix whose entries are all 1: 5,300 1 1 1 200 = 5,900

800 Therefore, the matrix product we used was


54,100 56,200 59,400

1

Solutions Section 5.2

1 1 1

23,800 22,400 23,600 9,300

10,600 10,100

0 = 5,900 . 1

71. We are asked to organize the parts required data in a matrix, and the prices per part from each supplier in another. Proc Mem Tubes Quantity of parts: 1 =

Pom II Pom Classic

2

16

1

20

4

40

We multiply this by the prices per part matrix to obtain the total costs. Thus, we should write the price per parts matrix with the prices for each component as columns (to match the rows above): Prices: - =

Motorel Intola 100

Proc

150

50

Mem

40

10

Tubes

15

Total cost = Quantity × Price = 1- =

2 16 20

$1,200

4

40

50

40

1,200 1,240

700 910 10 15 How are these costs organized? The clue is that the rows of the product 1- correspond to the rows of the left-hand matrix, 1. Here 1 has rows corresponding to Pom II and Classic. On the other hand, the columns of the product 1- correspond to the columns of - , which are Motorel and Intola. Thus, the prices are organized as follows: Motorel Intola Pom II

1

100 150

$1,240

=

Pom Classic $700 $910 Solution: Motorel parts on the Pom II: $1,200; Intola parts on the Pom II: $1,240; Motorel parts on the Pom Classic: $700; Intola parts on the Pom Classic: $910 72. From Exercise 71, we have: Quantity of parts: 1 =

Prices: - =

Proc Mem Tubes Pom II

Pom Classic

2

1

16 4

20

40

Motorel Intola Proc Mem Tubes

100 50

10

150 40

15

We are also given the costs to the manufactures: =

25 50

10 10 .

5 7 Thus, Profit = Revenue − Cost = 1- 1 = 1(- ) 100 150 25 50 ( 2 16 20 % 890 840 ' 50 40 10 10 * = = . 1 4 40 ' 435 540 5 7 * 10 15 & )


Solutions Section 5.2 How are these profits organized? The rows of the product 1(- ) correspond to the rows of the lefthand matrix, Q, which has rows corresponding to Pom II and Classic. The columns of the product 1(- ) correspond to the columns of (- ), which are Motorel and Intola. Thus, the profits are organized as follows: Motorel Intola $890

Pom II

$840

Pom Classic $435 $540 Solution: Motorel parts on the Pom II: $890; Intola parts on the Pom II: $840; Motorel parts on the Pom Classic: $435; Intola parts on the Pom Classic: $540 73. =

1,020

440

1,020

440

150

=

120

1,020 150

440 120

0.05 0.04

68.6 12.3

1,650 1,560 1,650 1,560 144.9 In computing this product, we are adding 5% of the number of people visiting Australia and 4% of the number visiting South Africa for each of the three regions. These are exactly the percentages that decide to settle where they visit. Thus, the entries of give the number of people (in thousands) from each of the three regions who settle in the country they visited. 1,020 440 51 17.6 0.05 0 = 150 = 7.5 4.8 120 0 0.04 1,650 1,560 82.5 62.4 Each row of the product is computed by taking 5% of all the tourists to Australia and 4% of those to South Africa separately. Thus, the entries of give the number of people (in thousands) from each of the three regions who settle in Australia, and the number that settle in South Africa. 74. =

150

120

1,650 1,560

=

,

3,234 2,400 ,

=

1.2 1.3 1.1 ,

=

1,224 195

528 156

=

=

1.2 0 0

0

1.3 0

0 0

1.1

1,815 1,716 is obtained by multiplying the number of North American tourists by 1.2, the number of Central and South American tourists by 1.3, and the number of European tourists by 1.1, and then adding them for each of Australia and South Africa. Thus, the entries of give the projected total number of tourists (in thousands) to each of Australia and South Africa in 2040. is obtained by doing the same multiplication of entries as above, except that the results are not totaled for each of the two destinations. Thus, the entries of break those figures down into visitors from each of the three regions. 75. Take - to be the matrix represented by the table: 0.9920 0.0007 0.0058 0.0015 -=

0.0005 0.9935 0.0043 0.0017 0.0013 0.0021 0.9950 0.0016 0.0011 0.0011 0.0048 0.9930

2019 Distribution:

=

56.0 68.3 125.6 78.3

To compute the population distribution in 2020, compute - . 55.8 68.2 126.0 78.2 . Thus, the 2020 distribution was 55.8 million in the Northeast, 68.2 million in the Midwest, 126.0 million in the South, and 78.2 million in the West.


Solutions Section 5.2 76. Take - to be the matrix represented by the table: 0.9920 0.0007 0.0058 0.0015 -=

0.0005 0.9935 0.0043 0.0017 0.0013 0.0021 0.9950 0.0016 0.0011 0.0011 0.0048 0.9930 =

56.0 68.3 125.6 78.3 As in Exercise 75, the population distribution in 2020 is given by - . The distribution in 2021 is therefore given by ( - )- = - 2 . 55.7 68.2 126.3 78.0 . Thus, the 2021 distribution was 55.7 million in the Northeast, 68.2 million in the Midwest, 126.3 million in the South, and 78.0 million in the West. 2019 Distribution:

77. Answers will vary. One example: =

1 2 , =

78. Answers will vary. One example: =

1 2 , =

1 2 3 4 5 6 2 3

Another example: =

Another example: =

1

1 , =

2 , = 3

1 2

2 3 4

79. We find that the addition and multiplication of 1 × 1 matrices is identical to the addition and multiplication of numbers. 80. At this point, the algebra of 1 × 1 matrices lacks division.

81. The claim is correct. Every matrix equation represents the equality of two matrices. When two matrices are equal, each of their corresponding entries must be equal. Equating the corresponding entries gives a system of equations. 82. If is $ × and is × $ ( 7 $)

83. Here is a possible scenario: costs of items , , and in 2013 = 10 20 30 , Percentage increases in these costs in 2014 = 0.5 0.1 0.20 , actual increases in costs = 10 × 0.5 20 × 0.1 30 × 0.20 .

84. Answers will vary.

85. It produces a matrix whose !" entry is the product of the !" entries of the two matrices. 86. - ᄆ (1 ) =

140 30 96 . It gives the individual revenues from the sales of wiper blades, cleaning fluid, and floor mats.


Solutions Section 5.3 Section 5.3 0 1

1. =

, =

0 1

0 1

0 1

1 0

= 8 1 0 1 0 1 0 1 0 0 1 Because and are square matrices with = 8, and are inverses. (We do not have to check that = 8 as well—see the note in the text after the definition of "inverse.") 2 0

2. =

, =

19

=

0

2

2 0

=

19

0 3 0 0 3 2 Because 7 8, and are not inverses. 3. =

2 1 1

19

1 1 1

1 1

0 0

1

0 0

1

0 1 1

1 2 92 0 0 1 1 =

=

19

2

0

0 19

2

1

0 78 3 0 92

=

1 0 0

0 1 0 = 8

0 0 1 0 0 1 0 0 1 Because and are square matrices with = 8, and are inverses. (We do not have to check that = 8 as well—see the note in the text after the definition of "inverse.") 4. =

0 1 1

0

0

1 0 0

0 1 0 = 8

1 =

1

0 0 1 0 0 1 0 0 1 Because and are square matrices with = 8, and are inverses. (We do not have to check that = 8 as well—see the note in the text after the definition of inverse.) 5. =

0 0

0

1

0 =

0

0

1 0 0

0 1 0 78

0 0 0 0 0 0 Because 7 8, and are not inverses. 6. =

0 0 0

0

0

0 0 0 0 #

0

=

0

1

1 0 0

0 1 0 = 8

0 0 1 # 1 Because and are square matrices with = 8, and are inverses. (We do not have to check that = 8 as well—see the note in the text after the definition of inverse. 0

7. To find the inverse of 1 2

1 1 0

1 0 1 :2 2:1

1 0 1 0 1

2

1

1

0

1 1 2 1 →

, we augment with the 2 × 2 identity matrix and row-reduce:

1 0

1

1

1

0 :1 + :2

1 1

=

2 1

.

The right-hand 2 × 2 block is the desired inverse:

2 1

−1

1

0

1 1

0 1 2 1 :2 1 2

1

1

.


8. To find the inverse of 0 1

1 1

1 0

0 1 :2 :1

0 1 1 1 →

Solutions Section 5.3

, we augment with the 2 × 2 identity matrix and row-reduce:

0 1

1

0

:1 ;:2 <

1 0 1 1

0 1

The right-hand 2 × 2 block is the desired inverse:

9. To find the inverse of 0 1 1 0 1 0 0 1

:1 ;:2 <

0 1 1 0

1 1

1 0 0 1 0 1 1 0

4 0 1 0 (1 4):1 0 2 0 1 (1 2):2

.

0 1

2 1

1 0

1 1 0

1 0 1 2:2 :1

1

0 1 1

1 2

3

0

1 0 1 4 0 1

0 1 2 0 1 2:2

2

14.

1

0

1 4

0 :1 :2

2 0

2 1

=

1

0

0

1 2

2 0

1 1

1

−1

4 0 0 2

−1

1 2

1 1

3 0 1 0 (1 3):1 0 1 0 2

3

.

4

2 0 1 :2 2:1

2 1

1

0

.

1

1 1 0

1 1

1

0

.

0

.

0

1 2

2 (1 2):1

0 1 1

2

.

1 0 1 3 0 2

.

1

0 1 −1

.

2

1

−1

0 0 192

1 1 0

0 0 2 1

1 0

.

The right-hand 2 × 2 block is the desired inverse:

13.

.

1 1

=

=

1 0

.

1 0

0

0 1

The right-hand 2 × 2 block is the desired inverse:

12.

−1

1

=

The right-hand 2 × 2 block is the desired inverse:

11.

0 1

, we augment with the 2 × 2 identity matrix and row-reduce:

The right-hand 2 × 2 block is the desired inverse:

10.

1 0 1 1

0

=

1 3 0 0

2

2

.

Since the left-hand 2 × 2 block has a row of zeros, we cannot reduce the matrix to obtain the 2 × 2 identity on the left. Therefore, the matrix is singular. 6

6 0 1 :2 6:1

0 0 6 1

Since the left-hand 2 × 2 block has a row of zeros, we cannot reduce the matrix to obtain the 2 × 2 identity on the left. Therefore, the matrix is singular. 15. To find the inverse of a 3 × 3 matrix, we augment it with the 3 × 3 identity matrix and row-reduce:


1 0 0

1 1 0 0 :1 :2

1 1

1 0 1 0

0

1 0 0 1

1 0

0

1 1 0

Solutions Section 5.3

→ 0 1

1

0 0

0

1

0 :2 :3 → 0 1 0 0

1

0

1 0 0 1 1

0

1

1 1 1

The right-hand 3 × 3 block is the desired inverse: 0 1 1

1 1

= 0

−1

0 0 1

1

0

0 0 1 0 0

1 .

0

1 0

0

1 . 1

1

16. To find the inverse of a 3 × 3 matrix, we augment it with the 3 × 3 identity matrix and row-reduce: 1 0 1 1 2 0 :1 + :3 1 2 3 1 0 0 :1 2:2 1 0 0 1 2 1 0 0

1

2 0 1 0

0

→ 0 1

1 0 0 1

2

0 0

0

1

0 :2 2:3 → 0 1 0 0

1

0

0

1

1 2 3

The right-hand 3 × 3 block is the desired inverse: 0 1 2

0 0 1 0

1 2

= 0

−1

0 0 1

1

1

0

2 .

0

1 0

2 . 1

1

17. To find the inverse of a 3 × 3 matrix, we augment it with the 3 × 3 identity matrix and row-reduce: 1 1 1 1 0 0 1 1 1 1 0 0 :1 + :2 1 1

1

2 0 1 0 :2 :1 → 0

0

1 1 0 0 1 :3 :1

0

2

0 1 0

0

1 0 0

0 1 0

1

2

0

1

1

1 2

1

0 0 1 1 2

0 :1 + :3

1

0

1

1

1 2 .

1

0

0

2 1

0

0 1 0 1 :3 2:2

2

0

0 2:2 + :3 → 0 2

1

1

1 1 1 0

1

0

1 2

0

1

1

1 1

1

1

−1

2

1

The right-hand 3 × 3 block is the desired inverse: 1

1

0

2

1 (1 2):2 →

0

2 1 (1 2):3

1 1 1

=

1

1 2

1 2

1 0

1

1

1 2 . 1 2

18. To find the inverse of a 3 × 3 matrix, we augment it with the 3 × 3 identity matrix and row-reduce: 1 2 3 1 0 0 1 2 3 1 0 0 :1 :2 0 1

2 3 0 1 0

0 1 0 0 1 :3 :1

1 0

0 2

0 0

0

3

1

1 0

1

1

0

1

1

0 0 1

1

1 0 0

1

1

2

3

1

0

1

0

1 0

3

2 3 (1 2):2 →

0 2 2 1 0 1 :3 + :2 1 0 0

0 :2 3:3 → 0 2 0

0 1 0 3 2 1 3 2 . 1

→ 0

1

0 0 1 1

1 1

0

1


1 2 3

Solutions Section 5.3 The right-hand 3 × 3 block is the desired inverse: 0 2 3

19.

1

1

1

0

3

0

1

1 0 0 1 0 :2 :1 → 0

1

2

1 1 0 0

2

3 0 0 1 :3 :1

3

3 1 2

1

0

1

2 1

1 0 3:1 :3

6

1 1 (1 3):1

1 0 0

1

0

1 0 1 1

1

0

0

3

0 3 0 3 2 0

0

1

3 3 1

0

(1 3):2 → 0 1 0

2

20.

1 3 1 0 0

0 1

1 0 0 1 0 0

1

3 0 1 0

6

1

1

1 4

1 2

1

3

8

1 0 0

1 0 0 1 :3 :1 0

0 0 1

1 8

→ 0

0 4:1 + 3:3

0

1 2

4 0

0 8:2 + 3:3 → 0 8

1

1 2 1

0 1 0 3 8

1 1

1 4

1 4

0

1 1

1 1

3

1 3

21. 1

1 1

0

1

0

3 4

3 8 .

1 3 1

1

0

0 1 0 1

1

1

0

0 .

1

3

0

1 3

−1

2 3

=

1 1

1

1 3

1 3

0 0 :1 + :2

2 1 0 1 :3 2:2 0

3

2

3 (1 8):2

−1

1 4

1 0

1

1 2

1

1

1

3

1

0 0 :1 + :2

1

1

0 1 1 0

= 3 8 1 8

0 1 0 1 :3 2:2

2 3

2 3 (1 4):1

1

1 3

2 3 1 3 .

1 0

1 1 3

1 0 1 0 :2 :1 → 0

1 1 0 0 1 :3 :1

2

(1 2):2 →

0 0 8 1 2 1 (1 8):3

1 8

1 1 0 0

2

2

2 3 1 3 .

1

3

0

2 2 (1 2):1

6

3 3 1

The right-hand 3 × 3 block is the desired inverse: 0 1

1

1 0 1 2:3 + :2

The right-hand 3 × 3 block is the desired inverse: 1 1 0 1

1

1 1 1 0

0 0 1 1

(1 3):3

2

0

0

1

0 0 2:1 + :2

1

0 2 1 1 1 0 3:2 + :3 → 0 6 0 6 4 0

1

= 3 2 1 3 2 .

−1

1 2 1 4

1 4

3 4

3 8 .

1 8

0 0 0 1 2 1 Since the left-hand 3 × 3 block has a row of zeros, we cannot reduce the matrix to obtain the 3 × 3 identity on the left. Therefore, the matrix is singular.


22.

1

1 1 1 0 0

0 1

1 0 0 0 1 1

1 1 0 1 0

0 0 0 0 1 :3 :1 1

1 0

0

1

1

0 0 :1 :2

Solutions Section 5.3 1

→ 0

1

1

1

1

0

1 0

0 1 1 1 0 1 :3 + :2

0 .

0 0 0 1 1 1 Since the left-hand 3 × 3 block has a row of zeros, we cannot reduce the matrix to obtain the 3 × 3 identity on the left. Therefore, the matrix is singular. 23. To find the inverse of a 4 × 4 matrix, we augment it with the 4 × 4 identity matrix and row-reduce: 1 0 1 0 1 0 0 0 1 0 1 0 1 0 0 0 1

1 0

1 0 0 1

0 1 0 1 0 0 :2 + :1

1

0

0 1 0 0 1 0 :3 + :1

1 0 1 0 0 0 1 1

0 1 0 0 0 :1 :3 1 1 1 0 0 :2 :3

1

0 0

1

0 0

1 1 0 1 0

2 1 1 0 1 :4 :3

1

1 0 0 0 0 0 1 0 0 0

1 1

0 0 1 0 1 1

2 2

1

.

0

1

1

1

0

1 1 1 1 0 0

0

1 1 1 0 1 0

0 1 0 1 0 0 0 1 :4 + :2

1 0 0 1 0 0 1 0 :1 + :4 0 1 0

0

0 0 1

0 1 1 0

1

0 0 0

1 0

1

0

1 0

1

0 :3 :4

1

0 1 1 1

0 0 0 1 0 1 1 1 The right-hand 4 × 4 block is the desired inverse: 1 0 1 0 −1 0 1 2 1 1 1

1 0

0 1 0 1

0

1 0 1

1

1

=

1 1 0

1

2

1

0

1

.

1

24. To find the inverse of a 4 × 4 matrix, we augment it with the 4 × 4 identity matrix and row-reduce: 0 1 1 0 1 0 0 0 0 1 1 0 1 0 0 0 1

1

1 1 0 1 0 0 :2 :1

0 1 0 0 1 0 :3 :1

0

1 0 1 0 0 0 1 :4 + :1

0

0

1

1 0

0

0

1

1 0 0

0 0 0

0

1

1

0

1

0

0

1 1

0

1

0

0

0

1

1

0 2

0 1 0 0

1

1

0

1

0

1

0 0 :1 + :3 0 0

1 1 0 0

1 2

1 1

1

0 1 :4 + :3

1

0

1 1 :2 1 1

0 1

:3

1 0

0

0

1 1 1 0 0

0 1 1 1 0 1 0 :3 :2 0

0

1

1

1 0 0 0

1

0

0

0 1 0

0

1

0 0 1

0

1

1

1

0 1 0 0 1 1 1 0 0 0 2 2 0 0 1 0 0

1 1 0 1

1

0

1

0 0 0 1 1 1

0

1

1 1

1 1 0 1 1 1 0 1

1 0 1

0 0 :2 :4

1

:1 ;:2 < < >


1 0 0 0 2 2

1

0 1 0 0 1 1 0 0 1 0 0

1

1

1

Solutions Section 5.3

0

1 0

.

0 0 0 1 1 1 1 1 The right-hand 4 × 4 block is the desired inverse: 0 1 1 0 −1 2 2 1 1 1 1

1

1 1

1

0 1

0

1 0 1

0

1

=

1 1 0

1

1

0

1 0

1 1

1

1

.

25. To find the inverse of a 4 × 4 matrix, we augment it with the 4 × 4 identity matrix and row-reduce: 1 0 1 2 1 2 0 0 :1 + :3 1 2 3 4 1 0 0 0 :1 2:2 0 0

0 0

2 3 0 1 0 0

1 2 0 0 1 0

0 1 0 0 0 1

1 0 0

0

1 2

1

0

0 1 0 1 0

1

2 0 :2 + :4

0 1 2 3

0

1

0 0 1

2

0

0

0 :3 2:4

1

0 1 0 0 0 0

2 1 0

3

2

1

0

0

0

0 0 1 2

0

0 0 0 1

0

0 0

1

0 0 0 1 1 0 0 0

26.

1

2

2

0

0 0 1 0 0 1 0 0 0 1 0 0 0 0 1 0

0

0

1 0 0 0 1 2

0 1 0 0 0 0 0 1 0 0

0 0 0 1 0 0 0 0 1 0 0 0 1 The right-hand 4 × 4 block is the desired inverse: 1 2 3 4 −1 1 2 1 0 =

1

1 0 0

0 0 :2 2:3 1 0

0 1 1

2 1 0

0

1

2 1

.

1

1 0 0 0 0 0 0 1

Rearrange rows:

0 1 0 0 0 0 1 0 0 0 1 0 0 1 0 0

1 0 0 0 0 0 0 1 0 0 0 1 1 0 0 0 The right-hand 4 × 4 block is the desired inverse: 0 0 0 1 −1 0 0 0 1 0 0 1 0 0 1 0 0 1 0 0 0 27. 1

# 1

1 1 28.

−1

#

= =

=

0 0 1 0 0 1 0 0

=

1

1

1 0 0 0

1 1

det

.

1

1

1 1

= # = (1)( 1) (1)(1) = 2

1 2 1 2 1 1 1 1 = = # # 2 1 1 1 2 1 2 4 1 0 2

det

1 4 1 8 0

1 2

= # = (4)(2) (0)(1) = 8

.


4 1 0 2 29.

#

3 4

= =

#

1 0 0 1

31.

−1

−1

1 4 1 8 1 1 2 1 = = # # 8 0 4 0 1 2

Solutions Section 5.3

=

1 2

30.

−1

= =

#

3 4

1 6

=

=

1 0

1 0

det

1 6 1 6

0 1

34.

=

35.

1.1 1.2

2

1

3 2 1 2

= # = (1)(1) (0)(0) = 1

0 1

1 0 1 1 1 0 = = # # 1 0 1 0 1 0 1 =

2 1 1

det 0

# 1 1 Singular matrix 1.3 1

1 6

2 1 4 2

1

= # = (1 6)(1 6) ( 1 6)(0) = 1 36

det 0.38

1 1 1 1

6 6 0 6

= # = (2)(2) (1)(4) = 0 0

3 4 0

det

1 1

−1

0

1 6 1 6 1 = 36 = # # 0 1 6

# 3 4 0 Singular matrix 33.

= # = (1)(4) (2)(3) = 2

3 4

det

# 4 2 Singular matrix 32.

1 2

1 0

−1

det

1 1 4 2 = = # # 2 3 1

=

1 6 1 6 0

1 2

= # = (1)(0) (0)(3 4) = 0

= # = (1)(1) (1)(1) = 0

0.45

0.49 0.41

=

TI-83/84 Plus format: [A]-1 Spreadsheet: =MINVERSE(A1:B2) (Assuming the matrix is in cells A1– B2) Online Matrix Algebra Tool format: A^(-1) Student Website → Online Utilities → Matrix Algebra Tool 36.

0.1 3.2 0.1 1.5

−1

=

8.82 18.82 0.59

TI-83/84 Plus format: [A]-1

0.59

Spreadsheet: =MINVERSE(A1:B2) (Assuming the matrix is in cells A1– B2) Online Matrix Algebra Tool format: A^(-1) Student Website → Online Utilities → Matrix Algebra Tool 37.

3.56

1.01

1.23 0

−1

=

0.00 0.99 0.81

2.87


Solutions Section 5.3 TI-83/84 Plus format: [A]-1 Spreadsheet: =MINVERSE(A1:B2) (Assuming the matrix is in cells A1– B2) Online Matrix Algebra Tool format: A^(-1) Student Website → Online Utilities → Matrix Algebra Tool 38.

9.09 5.01 1.01

2.20

−1

1.1

3.1

2.4

0.09

0.20

0.04 0.36

=

TI-83/84 Plus format: [A]-1 Spreadsheet: =MINVERSE(A1:B2) (Assuming the matrix is in cells A1– B2) Online Matrix Algebra Tool format: A^(-1) Student Website → Online Utilities → Matrix Algebra Tool 39. 1.7

2.4

2.3

0.6 0.7 0.1

is singular.

TI-83/84 Plus format: [A]-1 Spreadsheet: =MINVERSE(A1:C3) (Assuming the matrix is in cells A1– C3) Online Matrix Algebra Tool format: A^(-1) Student Website → Online Utilities → Matrix Algebra Tool

40.

2.1 6.1

2.4

3.5

0.1 2.3

−1

0.3 1.2 0.1

=

TI-83/84 Plus format: [A]-1

0.19

0.12 0.06

0.25

0.05 0.72

0.32

0.08

0.64

Spreadsheet: =MINVERSE(A1:C3) (Assuming the matrix is in cells A1– C3) Online Matrix Algebra Tool format: A^(-1) Student Website → Online Utilities → Matrix Algebra Tool 0.01

41.

0.01 0 0

0.32 0

0

0

0.04

0.34

0.32 0.23 0.23 0.41

0

0.01

91.35 8.65

−1

=

0.07 0.07 2.60 2.69

2.60 2.69

0

71.30

0

2.10

0

4.35

2.49

1.37

TI-83/84 Plus format: [A]-1 Spreadsheet: =MINVERSE(A1:D4) (Assuming the matrix is in cells A1– D4) Online Matrix Algebra Tool format: A^(-1) Student Website → Online Utilities → Matrix Algebra Tool 0.01

42.

0.01 0

0.01

0.32 0

0

0

0.04

0.34

0.32 0.23 0.23 0.96 0.23 0.65

is singular.

TI-83/84 Plus format: [A]-1 Spreadsheet: =MINVERSE(A1:D4) (Assuming the matrix is in cells A1– D4) Online Matrix Algebra Tool format: A^(-1) Student Website → Online Utilities → Matrix Algebra Tool 43. + = 4;

= 1 1 1 4 Matrix form: = 1 1 1

0 = 0 = 1 1 1 −1 1 2 1 2 1 = = 1 1 1 2 1 2

See Exercise 27


Thus, 0 = 1 =

1 2

1 2

4

1 2 1 2 So, ( , ) = (5 2, 3 2).

=

5 2

=

1

.

6

.

Solutions Section 5.3

1

3 2

.

44. 2 + = 2; 2 3 = 2 2 1 2 Matrix form: = 2 3 2

0 = 0 = 1 2 1 −1 3 8 1 8 1 = = 2 3 1 4 1 4

Thus, 0 = 1 =

So, ( , ) = (1, 0).

3 8

1 8

2

1 4 1 4

2

0

+ = 0; 3 2

+ = 1 2 1 3 1 2 0 Matrix form: = 1 2 1 1

45.

0 = 0 = 1 1 3 1 2 −1 12 6 1 = = 1 2 1 6 4

Thus, 0 = 1 =

So, ( , ) = (6, 4).

12 6 4

6

0

1

4

=

2 1 = ; = 1 3 2 6 2 2 2 3 1 2 1 6 Matrix form: = 1 2 1 2 1 46.

0 = 0 = 1 2 3 1 2 −1 6 6 1 = = 1 2 1 2 6 8

Thus, 0 = 1 =

So, ( , ) = (7, 9).

6 6 6 8

1 6 1

=

7 9

.

47. + 2 = 0; + 2 = 0; 0 1 2 1 Matrix form: 1 1 2 = 0 2

0

1

6

0 = 0 = 1 3 2 3 1 1 2 1 −1 1 = 1 1 2 = 1 1 1 2

0

1

1

2 3 4 3 1

2 = 6


Thus, 0 = = 1

1 3 2 3 1 1

1

0

1

6

Solutions Section 5.3

0 = 6 .

2 3 4 3 1 So, ( , , ) = (6, 6, 6).

6

6

48. + 2 = 4; = 0; + 3 2 = 5 1 2 0 4 Matrix form: 0 1 1 = 0 1 3 2

5

0 = 0 = 1 2 0 −1 1 4 2 1 = 0 1 1 = 1 2 1 1

1 3 2

Thus, 0 = = 1

1 4 1

2

1 1 So, ( , , ) = (6, 1, 1).

1

1

1

5

2

1

4

6

0 = 1 .

1

1

49. The three systems of equations have the matrix form 0 = , where 1 2 0 1 4 2 1 = 1 ; = 2 1 0 1 1 .

a. =

4 3

1

1

1

0= = 1

8 ( , , ) = (10, 5, 3)

b. =

0

0= =

3

2 ( , , ) = (6, 1, 5)

c. =

0

1

0 = 1 =

0

0 ( , , ) = (0, 0, 0)

1 3 2

1 2

0

4 8

3

1 2

0

0

6

1 2

0

0

0 1 1 1 3 2

0 1 1 1 3 2 0 1 1 1 3 2

10

3 = 5

3 = 1 2

5

0 = 0 0 0

0

50. The three systems of equations have the matrix form 0 = , where 1 2 4 1 4 2 1 = 1 ; = 0 1 0 1 1 . a. =

8 3 8

1

1

1

0= = 1

1 3 4

1 2 4 0 1 1 1 3 4

8

3 = 8

34 5

31


( , , ) = (34, 5, 31) b. =

8 3

0= = 1

2 ( , , ) = (10, 1, 7)

c. =

0 0

0= =

0 ( , , ) = (0, 0, 0)

1

Solutions Section 5.3 1 2 4

8

10

1 2 4

0

0

0 1 1 1 3 4 0 1 1 1 3 4

3 = 1 2

7

0 = 0 0

0

51. Unknowns: = the number of servings of Pork & Beans; = the number of slices of bread a. Arrange the given information in a table with unknowns across the top: Pork & Beans ( ) Bread ( ) Desired

Protein

5

4

20

Carbs.

21

12

80

We can now set up an equation for each of the items listed on the left: Protein: 5 + 4 = 20 Carbs: 21 + 12 = 80. We put this system in matrix form: 5 4 20 = 21 12 80

0 = 0 = 1 5 4 1 2 1 6 1 = = 21 12 7 8 5 24 Thus, 20 10 3 1 2 1 6 0 = 1 = = 7 8 5 24 80 5 6 So, ( , ) = (10 3, 5 6) Solution: Prepare 10/3 servings of beans, and 5/6 slices of bread. b. We must solve the following system: 5 + 4 = ; 21 + 12 = We put this system in matrix form: 5 4 = 21 12 Solving gives 5 4 1 2 1 6 2 + 6 = = = . 21 12 7 8 5 24 7 8 5 24 Solution: Prepare 2 + 6 servings of beans and 7 8 5 24 slices of bread.

52. Unknowns: = the number of servings of Cheerios; = the number of slices of milk a. Arrange the given information in a table with unknowns across the top:


Solutions Section 5.3

Cheerios ( ) Milk ( ) Desired Protein

3

4

26

Carbs.

24

6

78

We can now set up an equation for each of the items listed on the left: Protein: 3 + 4 = 26 Carbs: 24 + 6 = 78. We put this system in matrix form: 3 4 26 = 24 6 78

0 = 0 = 1 3 4 −1 1 13 2 39 1 = = . 24 6 4 13 1 26 Thus, 26 2 1 13 2 39 0 = 1 = = . 4 13 1 26 78 5 So, ( , ) = (2, 5). Solution: Use 2 servings of Cheerios and 5 servings of milk. b. We must solve the following system: 3 + 4 = ; 24 + 6 = . We put this system in matrix form: 3 4 = . 24 6 Solving gives 3 4 1 13 2 39 13 + 2 39 = = = . 24 6 4 13 1 26 4 13 26 Solution: Prepare 13 + 2 39 servings of Cheerios and 4 13 26 servings of milk.

53. Unknowns: = the number of batches of vanilla; = the number of batches of mocha; = the number of batches of strawberry Arrange the given information in a table with unknowns across the top: Vanilla ( ) Mocha ( ) Strawberry ( )

Eggs

2

1

1

Milk

1

1

2

Cream

2

2

1

a. Eggs: Milk: Cream:

2 + + = 350 + + 2 = 350 2 + 2 + = 400

Matrix form: 2 1 1 1 1 2 2 2 1

350

= 350

400

0 = 0 = 1


1 3 1 3

350

100

0 = 0 = 2 1 1 −1 400 1 1 3 1 3 0= 1 1 2 500 = 1 0 1

400

100

0=

2 1 1 1 1 2

−1

350

1

Solutions Section 5.3

350 = 1

0

1

350 =

50

2 2 1 400 0 2 3 1 3 400 100 Solution: Use 100 batches of vanilla, 50 batches of mocha, and 100 batches of strawberry. b. The requirements lead to the following matrix equation: 2 1 1 400 = 1 1 2 500 2 2 1

400

1

500 =

0

2 2 1 400 0 2 3 1 3 400 200 Solution: Use 100 batches of vanilla, no mocha, and 200 batches of strawberry. c. The requirements lead to the following matrix equation: 2 1 1 400 1 1 2 = 500 2 2 1

400

0 = 0 = 2 1 1 −1 1 1 3 1 3 0= 1 1 2 = 0 1 1 1

=

.

3 3 +

2 2 1 0 2 3 1 3 2 3 3 Solution: Use 3 3 batches of Vanilla, + batches of mocha, and 2 3 3 batches of strawberry. .

54. Unknowns: = the number of gallons of PineOrange; = the number of gallons of PineKiwi; = the number of gallons of OrangeKiwi Arrange the given information in a table with unknowns across the top: PineOrange ( ) PineKiwi ( ) OrangeKiwi ( )

Pineapple

2

3

0

Orange

2

0

3

Kiwi

0

1

1

a. Pineapple: Orange: Kiwi:

2 + 3 = 800 2 + 3 = 650 + = 350

Matrix form: 2 3 0

2 0 3 0 1 1

800

= 600

350

0 = 0 = 2 3 0 −1 800 0= 2 0 3 600 = 0 1 1

1

350

1 4 1 6

1 6

1 4

1 6 1 6

3 4 1 2

1 2

800

100

600 = 200 350

150


Solutions Section 5.3 Solution: Make 100 gallons of PineOrange, 200 gallons of PineKiwi, and 150 gallons of OrangeKiwi. b. The requirements lead to the following matrix equation: 2 3 0 650 2 0 3 = 800 0 1 1

350

0 = 0 = 2 3 0 −1 650 0= 2 0 3 800 = 1

1 4 1 6

1 4

1 6

650

3 4 1 2

100

800 = 150 .

0 1 1 350 1 2 350 200 1 6 1 6 Solution: Make 100 gallons of PineOrange, 150 gallons of PineKiwi, and 200 gallons of OrangeKiwi. c. The requirements lead to the following matrix equation: 2 3 0 −1 2 0 3 = 0 1 1

0=

2 3 0 2 0 3

−1

0 1 1

=

1 4 1 6

1 6

1 4

1 6 1 6

3 4 1 2

1 2

=

( + 3 ) 4

( + 3 ) 6 .

( + + 3 ) 6

Make ( + 3 ) 4 gallons of PineOrange, ( + 3 ) 6 gallons of PineKiwi, and ( + + 3 ) 6 gallons of OrangeKiwi. 55. Unknowns: = amount invested in SHPIX, = amount invested in RYURX, = amount invested in RYCWX The total investment was $9,000: You invested an equal amount in RYURX and RYCWX: YTD loss from the first two funds was $1,170:

+ + = 9,000. = , or = 0. 0.17 + 0.16 = 1,170.

Matrix form of the system of equations: 1 1 1 9,000 = 0 1 1 0 0.17 0.16

0

1,170

0 = 0 = 1 1 1 −1 9,000 8 9 8 9 100 9 0= = 0 1 1 0 17 18 17 18 50 9 1

9,000 0

5,000 = 2,000

0.17 0.16 0 1,170 17 18 1 18 50 9 1,170 2,000 Solution: You invested $5,000 in SHPIX, $2,000 in RYURX, $2,000 in RYCWX.

56. Unknowns: = amount invested in SHPIX, = amount invested in RYURX, = amount invested in RYCWX The total investment was $7,000: You invested an equal amount in SHPIX and RYURX: Total loss for the year was $1,410: Matrix form of the system of equations:

+ + = 7,000. = , or = 0. 0.17 + 0.16 + 0.25 = 1,410.


1 1

1

1

1

0

0.17 0.16 0.25

=

0 = 0 = 1 1 1 0= 1 0 1 1

−1

7,000

Solutions Section 5.3

0

1,140

7,000 0

25 17 25 17

9 17

100 17

8 17 100 17

7,000 0

= 2,000

0.17 0.16 0.25 1,410 33 17 1 17 200 17 1,410 Solution: You invested $2,000 in each of SHPIX and RYURX and $3,000 in RYCWX. =

2,000 3,000

57. Unknowns: = the number of shares of WSR, = the number of shares of TKOMY, = the number of shares of STX The total investment was $7,850. Investment in WSR = shares @ $10 = 10 Investment in TKOMY = shares @ $53 = 53 Investment in STX = shares @ $84 = 84 Thus, 10 + 53 + 84 = 7,850. You expected to earn $272 in dividends: WSR dividend = 4% of 10 invested = 0.04(10 ) = 0.4 TKOMY dividend = 4% of 53 invested = 0.04(53 ) = 2.12 STX dividend = 3% of 84 invested = 0.03(84 ) = 2.52 Thus, 0.4 + 2.12 + 2.52 = 272. You purchased a total of 200 shares: + + = 200. We therefore have the following system: + + = 200 10 + 53 + 84 = 7,850 0.4 + 2.12 + 2.52 = 272. 1 1 1 200 Matrix form: 10 53 84 = 7,850 0.4 2.12 2.52

272

0 = 0 = 1 1 1 −1 200 53 43 10 903 775 903 0 = 10 53 84 7,850 = 10 43 53 903 1850 903 1

200

7,850 =

100 50

0.4 2.12 2.52 272 0 1 21 25 21 272 50 Solution: You purchased 100 shares of WSR, 50 shares of TKOMY, and 50 shares of STX.

58. Unknowns: = the number of shares of WSR, = the number of shares of TKOMY, = the number of shares of STX The total investment was $13,150. Investment in WSR = shares @ $10 = 10 Investment in TKOMY = shares @ $53 = 53 Investment in STX = shares @ $84 = 84 Thus, 10 + 53 + 84 = 13,150. You expected to earn $484 in dividends: WSR dividend = 4% of 10 invested = 0.04(10 ) = 0.4 TKOMY dividend = 4% of 53 invested = 0.04(53 ) = 2.12 STX dividend = 3% of 84 invested = 0.03(84 ) = 2.52 Thus, 0.4 + 2.12 + 2.52 = 484. You purchased a total of 300 shares: + + = 300. We therefore have the following system:


Solutions Section 5.3 + + = 300 10 + 53 + 84 = 13,150 0.4 + 2.12 + 2.52 = 484. 1 1 1 300 Matrix form: 10 53 84 = 13,150 0.4 2.12 2.52

484

0 = 0 = 1 1 1 −1 300 53 43 10 903 775 903 0 = 10 53 84 13,150 = 10 43 53 903 1850 903 1

300

100

13,150 = 150

0.4 2.12 2.52 484 0 1 21 25 21 484 Solution: You purchased 100 shares of WSR, 150 shares of TKOMY, 50 shares of STX.

50

59. Take - to be the matrix represented by the table: 0.9920 0.0007 0.0058 0.0015 -=

0.0005 0.9935 0.0043 0.0017 0.0013 0.0021 0.9950 0.0016 0.0011 0.0011 0.0048 0.9930

2020 Distribution:

=

57.6 69.0 126.3 78.6

To compute the population distribution in 2019, compute - 1 . 57.8 69.1 125.9 78.7 (See the solution to Exercise 75 in the preceding section.) TI-83/84 Plus format: [A]*[B]-1 Spreadsheet: =MMULT(A1:D1,MINVERSE(E1:H4)) (Assuming that is in cells A1–D1 and - is in cells E1–H4) Online Matrix Algebra Tool format: A*P^(-1) Student Website → Online Utilities → Matrix Algebra Tool 60. Take - to be the matrix represented by the table: 0.9920 0.0007 0.0058 0.0015 -=

0.0005 0.9935 0.0043 0.0017 0.0013 0.0021 0.9950 0.0016 0.0011 0.0011 0.0048 0.9930 =

57.6 69.0 126.3 78.6

To compute the population distribution in 2018, compute ( - 1) - 1 = - 2 . 2020 Distribution:

58.0 69.1 125.5 78.9 (See the solution to Exercise 76 in the preceding section.) TI-83/84 Plus format: [A]*[B]-1*[B]-1 Spreadsheet: =MMULT(MMULT(A1:D1,MINVERSE(E1:H4)),MINVERSE(E1:H4)) (Assuming that is in cells A1–D1 and - is in cells E1–H4) Online Matrix Algebra Tool format: A*P^(-2) Student Website → Online Utilities → Matrix Algebra Tool 61. : =

=1 2 =1 2 =1 2

=1 2

.

0.7071 0.7071

0.7071 0.7071 a. The coordinates of a rotated point are given by 0.7071 0.7071 2 0.7071 . = . = : 0.7071 0.7071 3 3.5355

Thus, ( , ) . ( 0.7071, 3.5355). b. Multiplication by : rotates points through 45°. To rotate through 90°, multiply by : again, obtaining


: :

= :2

Solutions Section 5.3 .

In other words, multiplying by : 2 will result in a counterclockwise rotation of 90°. To rotate by 135°, multiply yet again by :. This amounts to multiplying the original column vector by : 3. c. Let > be the matrix representing a clockwise rotation through 45°. Since a rotation of 45° clockwise followed by a rotation of 45° counterclockwise results in no change: : > = . = 8 In other words, multiplication by :> should have the same effect as multiplication by the 2 × 2 identity matrix. This will happen if > = : 1. 0.5 0.8660 =3 4 . 1 2 =3 4 0.8660 0.5 a. The coordinates of a rotated point are given by 0.5 0.8660 2 1.598 . = . = > 0.8660 0.5 3 3.232 62. > =

1 2

Thus, ( , ) . ( 1.598, 3.232).

=1 2 =1 2

b. From Exercise 61, multiplication by : =

=1 2

=1 2

.

0.7071 0.7071

rotates points through 0.7071 0.7071 45°. To rotate through 105° = 60° + 45°, rotate first by 60° and then follow by a rotation through 45°. That is, multiply by > and then by :, obtaining : > . = :> In other words, multiplying by :> will result in a counterclockwise rotation of 105°. The same result is obtained if we multiply by >:. To rotate by 135°, multiply yet again by :. This amounts to multiplying the original column vector by : 3. c. Let be the matrix representing a clockwise rotation through 60°. Since a rotation of 60° clockwise followed by a rotation of 60° counterclockwise results in no change: > = . = 8 In other words, multiplication by > should have the same effect as multiplication by the 2 × 2 identity matrix. This will happen if = > 1. 63. First, write the uncoded message as a string of numbers using A = 1, B = 2, C = 3, and so on: "GO TO PLAN B" = 7 15 0 20 15 0 16 12 1 14 0 2 . First, arrange the numbers in a matrix with 2 rows: 7 0 15 16 1 0 Uncoded message = . 15 20 0 12 14 2 Then encode by multiplying by = Coded message =

1 2 3 4

7

1 2 3 4 0

15 20

:

15 16 0

1

0

12 14 2

=

37 40 15 40 29 4 81 80 45 96 59 8

Arrange as a single row: 37 81 40 80 15 45 40 96 29 59 4 8 .

.

64. First, write the uncoded message as a string of numbers using A = 1, B = 2, C = 3, and so on:


"ABANDON SHIP" = 1 2 1 14 4 15 14 0 19 8 9 16 . First, arrange the numbers in a matrix with 2 rows: 1 1 4 14 19 9 Uncoded message = . 2 14 15 0 8 16 Solutions Section 5.3

Then encode by multiplying by = Coded message = Arrange as a single row:

1 2 3 4

1

1 2 3 4

1

4

:

2 14 15

14 19 0

8

9

16

=

5

29 34 14 35 41

11 59 72 42 89 91

5 11 29 59 34 72 14 42 35 89 41 91 .

.

65. Coded message = 33 69 54 126 11 27 20 60 29 59 65 149 41 87 . First, arrange the numbers in a matrix with two rows: 33 54 11 20 29 65 41 Coded message = 69 126 27 60 59 149 87 To decode the message, multiply by 1 : 1 2 −1 1 2 1 = = . 3 4 1.5 0.5 Decoded message: 1 33 54 11 20 29 2 1.5 0.5

65

41

69 126 27 60 59 149 87

=

3

18 5 20

15 18 3

0

1

19

5

14 23 18

Arrange as a single row: 3 15 18 18 5 3 20 0 1 14 19 23 5 18 . Translate to letters using A = 1, B = 2, C = 3, and so on: Decoded message = "CORRECT ANSWER". 66. Coded message = 59 141 43 101 7 21 29 59 65 149 41 87 . First, arrange the numbers in a matrix with two rows: 59 43 7 29 65 41 Coded message = 141 101 21 59 149 87 To decode the message, multiply by 1 : 1 2 −1 1 2 1 = = . 3 4 1.5 0.5 Decoded message 1 59 43 7 29 65 2 1.5 0.5

41

141 101 21 59 149 87

=

23 15 7

1

19

5

18 14 0 14 23 18

Arrange as a single row: 23 18 15 14 7 0 1 14 19 23 5 18 Translate to letters using A = 1, B = 2, C = 3, and so on: Decoded message = "WRONG ANSWER"

67. If = 8 and = 8, then and are inverse matrices. (See the definition of the inverse matrix in the text.) Thus, choice (A) is the correct choice.

68. If is a square matrix with 3 = 8, then ( 2) = 8, so that is invertible (with inverse 2). Thus, choice (B) is the correct choice. 69. The given matrix has two identical rows. If two rows of a matrix are the same, then row reducing it will


Solutions Section 5.3 lead to a row of zeros, and so it cannot be reduced to the identity. Thus, the inverse of the given matrix does not exist; the matrix is singular.

70. Since the only 1 × 1 singular matrix is [0] (all others row-reduce to the 1 × 1 identity [1]), it follows that the invertible matrices are the nonzero numbers.

#

71. We check that the given matrix is the inverse of 1 # #

#

by multiplying the two matrices:

# # 0 1 1 = = # # + # # + # 0 #

=

1 0 0 1

Since the product of the two matrices is the identity, they must be inverse matrices as claimed. In other words, −1 1 (provided # 7 0). = # # # 72.

1 0

# 0 1 :2 #:1

Thus,

( #) 0

#

−1

=

0

:2 ( #)

1 . # #

0 ( #):1 :2

0 # #

:1 ( ( #))

# #

1

1 0

( #)

0 1 # ( #)

( #) ( #)

73. When one or more of the are zero; if that is the case, then the matrix | 8 easily reduces to a matrix that has a row of zeros on the left-hand portion, so is singular. Conversely, if none of the are zero, then | 8 easily reduces to a matrix of the form 8 |

, showing that is invertible.

74. If a square matrix reduces to the identity, then it must be invertible since the process for finding the inverse will work.

75. To check that 1 1 is the inverse of , we multiply these two matrices and check that we obtain the identity matrix: ( )( 1 1) = ( 1) 1 Associative law = 8 1

= 1 = 8.

Since 1 =

Since the product of the given matrices is the identity, they must be inverses as claimed. 76. ( + 0) = Multiply both sides on the left by 1 : 8( + 0) = 1 + 0 = 1 0 = 1 . Multiply both sides on the left by 1 : 80 = 1( 1 ) 0 = 1( 1 ).

.


Solutions Section 5.3 77. If a square matrix reduces to one with a row of zeros, then it cannot have an inverse. The reason is that, if has an inverse, then every system of equations 0 = has a unique solution, namely 0 = 1 . But if reduces to a matrix with a row of zeros, then such a system has either infinitely many solutions or no solution at all.

78. If was invertible, then multiplying both sides of the equation = ? (? is the zero matrix) on the left by 1 would yield = ?, contradicting the fact that is not the zero matrix. Thus cannot possibly be invertible. A similar argument shows that cannot be invertible.


Solutions Section 5.4 Section 5.4 1. = :- =

2. = :- =

3. = :- =

4. = :- =

0 1 0 0

0 0 0 1

2

0 1

3

1 1

1 0

2

0

0 1

1

1

2

0 1

3

1 1

1 0

0 2

0

= 1

= 1 0

0

2 1

0.5

0 1

2

0.5

2 0

0

1 0

0

2 0 3

0

2

0

2

0

1

1 1

0 0.5 0 0.5

0

0 1

0

2 0

0.5 0.5 0 0

1

2

1 0 3

1

0

2 0 2

2

0

1 1

1

2 1

1

0

0.5 0.5 0 0

= 0.25

= 0.75

5. Since we are given a column matrix, we take our row strategy to be := . = :- =

0 1 2 2 0

3

5 4

0

0.25

0.75 = 0

0.75 1

2.25

1 0 5 0.25 = 0.74 + 2.25 + 0.25 . The greatest coefficient is the coefficient of , so we take = 1 and = = = 0. This gives the strategy := 0 0 1 0 and resulting payoff = 0.74(0) 0 + 2.25(1) + 0.25(0) = 2.25. 6. Since we are given a column matrix, we take our row strategy to be := . = :- =

0 1 2 2 0

3

1 0 = (4 3) + (4 3) + (4 3) .

5 4

0

5

1 3

1 3 = 1 3

4 3 4 3 1

4 3

The greatest coefficients are the coefficients of and , so we can either take = 1 and = = = 0, giving the strategy : = 1 0 0 0 , or we could take = 1 and = = = 0, giving the strategy : =


0 1 0 0 and resulting expected payoff = (4 3)(1) + (4 3)(0) + 0 (4 3)(0) = 4 3.

Solutions Section 5.4

7. Since we are given a row matrix, we take our column strategy to be = T = :- =

1 2 0 1 4 1 4

0 1 2 2 0 1

3 0

5

4

0

5

= 1 4 1 4 5 4

= (1 4) + (1 4) + (5 4) The lowest coefficients are the coefficients of and , so we can either take = 1 and = = 0, giving the strategy = 1 0 0 T or we could take = 1 and = = 0, giving the strategy = 0 1 0 T and resulting expected payoff = (1 4)(1) + (1 4)(0) + (5 4)(0) = 1 4. 8. Since we are given a row matrix, we take our column strategy to be = T. = :- =

0.8 0.2 0 0

0 1 2 2 0 1

3 0

5 4

0

5

= 0.4 1.2 4.8

= 0.4 1.2 + 4.8 The lowest coefficient is the coefficient of , so we take = 1 and = = 0, giving the strategy = 0 1 0 T , and resulting expected payoff = 0.4(0) 1.2(1) + 4.8(0) = 1.2.

9. Checking the rows: Neither of rows and dominates the other: 1 is worse than 2, but 10 is better than 4. Checking the columns: Column dominates column because each of the payoffs in column is lower than or equal to the corresponding payoff in column . We therefore eliminate column to obtain

1

2

10

4

This matrix cannot be reduced further.

10. Checking the rows: Neither of rows and dominates the other: 1 is worse than 2, but 10 is better than 4. Checking the columns: Column dominates column because each of the payoffs in column is lower than or equal to the corresponding payoff in column . We therefore eliminate column to obtain

0

4

10

5

Checking the rows: Row now dominates row because each of its payoffs is greater than or equal to the corresponding payoff in row , so we eliminate row to obtain


0

Solutions Section 5.4

10

Checking the columns: Column dominates column , so we reduce further to

0

11. Checking the rows: Row 3 dominates both rows 1 and 2 because each of its payoffs is greater than or equal to the corresponding payoff in row 1 and 2, so we eliminate rows 1 and 2 to obtain 3

5

#

0

1

Checking the columns: Column # dominates columns and because 1 is less than 0 and 5. We therefore eliminate columns and to obtain 3

#

1

12. Checking the rows: Row 3 dominates row 1 because each of its payoffs is greater than or equal to the corresponding payoff in row 1, so we eliminate row 1 to obtain 2 3

3 2

#

10

10 3

4

Neither of the remaining rows dominates the other ( 3 is worse than 2 but 10 is better than 4). Checking the columns: None of the three columns dominates any other, so the matrix cannot be reduced further. 13. Checking the rows: Row dominates rows and @ because each of its payoffs is greater than or equal to the corresponding payoff in row and @, so we eliminate rows and @ to obtain

4

3

0

3

#

2

10

Checking the columns: Column dominates the other two columns (each of its payoffs is less than or equal to the corresponding payoff in the other two columns). We therefore eliminate columns and # to obtain

0

14. Checking the rows: Row dominates rows and @ because each of its payoffs is greater than or equal to the corresponding payoff in row and @, so we eliminate rows and @ to obtain

2

1

4 1

#

9

0

Checking the columns: Column dominates column (each of its payoffs is less than or equal to the corresponding payoff in column ). We therefore eliminate column to obtain


4 1

Solutions Section 5.4

#

9

0

The matrix cannot be reduced further. 15. Circle the row minima and box the row maxima:

1

2

1

1

4

1

2

4

Since the , -entry 1 was both circled and boxed, it is a saddle point, and the game is strictly determined with the row player's optimal strategy being , the column player's optimal strategy being , and the value = saddle point = 1. 16. Circle the row minima and box the row maxima:

2

1 10

1

2

1 10

1

Since no entry is both circled and boxed, there are no saddle points, and so the game is not strictly determined. 17. Circle the row minima and box the row maxima:

2

1

0

3

2 0

2

0

2

3

1

0

Since no entry is both circled and boxed, there are no saddle points, and so the game is not strictly determined. 18. Circle the row minima and box the row maxima:

2 2

1

3

3 2

1

2

3

3

2

2

Since no entry is both circled and boxed, there are no saddle points, and so the game is not strictly determined. 19. Circle the row minima and box the row maxima: -

#

1

1

5

3

3

10

1

4

>

5

:

4 5

-

#

1

1

5

3

3

10

2

1

4

4

>

5

:

4 5

2

4

No entry is both circled and boxed, so there are no saddle points and the game is not strictly determined.


Solutions Section 5.4 20. Circle the row minima and box the row maxima:

-

2

4

:

1

2

1 >

1 1

#

9

1 1

#

-

2

4

9

:

1

2

3

0

1

1

>

3

1 1

1 1

0

1

Since no entry is both circled and boxed, there are no saddle points, and so the game is not strictly determined. 21. a. Row strategy: Take : = 1 , =

1 0

Take : =

1

T

0 1

1

T

= :- =

1

1 , =

A = :- =

1

Graphs of and A :

0

0

2

1

2

0

0

1

1

1

= . = 2 (1 ) = 3 1.

Lower (heavy) portion has its highest point at the intersection of the two lines. The -coordinate of the intersection is given when = A : = 3 1 4 = 1 = 1 4. So the optimal row strategy is : = 1 = 1 4 3 4 . b. Column strategy: Take : = 1 0 , = 1 T = :- =

Take : =

1 0

0 1 , =

= :- =

0 1

1

2

0

1

0

1

1 1

2

T

1

1

= + 2(1 ) = 3 + 2 = 1.


Solutions Section 5.4

Graphs of and A :

The upper (heavy) portion has its lowest point at the intersection of the two lines. The -coordinate of the intersection is given when = A : 3 + 2 = 1 4 = 3 = 3 4. So the optimal column strategy is = 1 T = 3 4 1 4 T .

c. To compute the expected value of the game, compute the -coordinate of the point on either graph with the given -coordinate. For instance, using the second graph (part (b)), = 1 = 3 4 1 = 1 4. Solution: : = 1 4 3 4 , = 3 4 1 4 T , = 1 4 22. a. Row strategy: Take : = 1 , =

1 0

Take : =

1

T

0 1

1

T

= :- =

1

1 , =

A = :- =

1

Graphs of and A :

1

1

0

1

0

0

0

1

1

1

= + (1 ) = 2 + 1. = (1 ) = 1.

Lower (heavy) portion has its highest point at the intersection of the two lines. The -coordinate of the intersection is given when = A : 2 + 1 = 1 3 = 2 = 2 3. So the optimal row strategy is : = 1 = 2 3 1 3 . b. Column strategy: Take : = 1 0 , = = :- =

1 0

1 1 1

0

1

T

1

=


Take : =

0 1 , =

= :- =

0 1

1 1 1

Graphs of and A :

0

Solutions Section 5.4 T

1

1

= (1 ) = 2 1.

The upper (heavy) portion has its lowest point at the intersection of the two lines. The -coordinate of the intersection is given when = A : = 2 1 3 = 1 = 1 3 So the optimal column strategy is = 1 T = 1 3 2 3 T .

c. To compute the expected value of the game, compute the -coordinate of the point on either graph with the given -coordinate. For instance, using the second graph (part (b)), = = 1 3. Solution: : = 2 3 1 3 , = 1 3 2 3 T , = 1 3 23. a. Row strategy: Take : = 1 , =

1 0

T

Take : =

1 2

0 1

1

T

0

= :- =

1

1 , =

A = :- =

1

Graphs of and A :

2

1

1 2 1

2

0

1

= 2(1 ) = 2. = 2 + (1 ) = 3 + 1.

Lower (heavy) portion has its highest point at the intersection of the two lines. The -coordinate of the intersection is given when = A : 2 = 3 + 1 4 = 3 = 3 4. So the optimal row strategy is : = 1 = 3 4 1 4 . b. Column strategy: Take : = 1 0 , =

1

T


= :- =

Take : =

1 0

0 1 , =

= :- =

1 2 2

1

1

1 2

Solutions Section 5.4

T

1

= 2(1 ) = 2

= 2 + (1 ) = 3 + 1. 1 2 1 Note that and A are the same as for part (a). Therefore, their graphs are the same, except that this time we are interested in the lowest point of the upper region: 0 1

This is the same intersection point as in part (a), so = 3 4 and the optimal column strategy is = 1 T = 3 4 1 4 T .

c. To compute the expected value of the game, compute the -coordinate of the point on either graph with the given -coordinate. For instance, using the second graph (part (b)), = 2 = 3 4 2 = 5 4. Solution: : = 3 4 1 4 , = 3 4 1 4 T , = 5 4 24. a. Row strategy: Take : = 1 , =

1 0

T

Take : =

2 1

0 1

1

T

0

= :- =

1

1 , =

A = :- =

1

Graphs of and A :

1 3

2 1 1 3

0

1

= 2 (1 ) = 1. = 3(1 ) = 2 3.

Lower (heavy) portion has its highest point at the intersection of the two lines. The -coordinate of the intersection is given when = A : 1 = 2 3 3 = 2 = 2 3 So the optimal row strategy is : = 1 = 2 3 1 3 . b. Column strategy:


Take : =

1 0 , =

1

Take : =

0 1 , =

2 1

T

1

2 1

T

= :- = = :- =

1 0

1 3

Solutions Section 5.4

1

= 2 (1 ) = 1

= 3(1 ) = 2 3. 1 3 1 Note that and A are the same as for part (a). Therefore, their graphs are the same, except that this time we are interested in the lowest point of the upper region: 0 1

This is the same intersection point as in part (a), so = 2 3 and the optimal column strategy is = 1 T = 2 3 1 3 T .

c. To compute the expected value of the game, compute the -coordinate of the point on either graph with the given -coordinate. For instance, using the second graph (part (b)), = 1 = 2 3 1 = 5 3. Solution: : = 2 3 1 3 , = 2 3 1 3 T , = 5 3 25. The possible outcomes are:

HH: You lose 1 point. Payoff: 1 TH: You win 1 point. Payoff: 1

HT: You win 1 point. Payoff: 1 TT: You lose 1 point. Payoff: 1

Payoff matrix with you as the row player and your friend as the column player: B

B

1

1

1

1

26. The possible outcomes are:

HH: You lose 2 points. Payoff: 2 TH: You win 1 point. Payoff: 1

HT: You win 2 points. Payoff: 2 TT: You lose 1 point. Payoff: 1

Payoff matrix with you as the row player and your friend as the column player: B

B

1

1

2

2

27. Take 3 = France; > = Sweden; 2 = Norway. Your strategies are to invade one of 3 , >, 2, and your opponent's strategies are to defend one of 3 , >, or 2. The possible outcomes in which you invade the same country as your opponent defends are 3 3 , >>, 22, and all of these have a payoff of 1 (you are


Solutions Section 5.4 defeated). Any other combination (you invade a country other than the one your opponent is defending) gives a payoff of 1 point (you are successful). Payoff matrix: 3

>

2

1

1

1

Your Opponent Defends

You Invade

3

1

2

1

>

1

1

1

1

28. Your strategies are to attack by sea or air. Your opponent's strategies are to defend by sea, air, or both. The possible outcomes are: (Sea, Sea) or (Air, Air): Your attack is met by an all-out defense: you lose 200 points. Payoff: -200 (Sea, Air) or (Air, Sea): Your attack is met by no defense: you win 100 points. Payoff: 100 (Sea, Both) or (Air, Both): Your attack is met by a shared defense: you win 50 points. Payoff: 50 Payoff matrix: Your Opponent Defends

Sea

200

You Attack Sea

100

Air

Air

Both

200

50

100

50

29. Take B = Brakpan; N = Nigel; S = Springs. Your strategies are to locate in one of B, N, S, and your opponent's strategies are the same. The possible outcomes are: BB, SS, or SS: You share the total business. No net gain or loss of sales. Payoff: 0 BN or NB: One of you locates at B, the other at N. These two cities provide the same potential market, so again there is no net gain or loss. Payoff: 0 SB or SN: You locate at S (total market: 1,000 burgers/day), your opponent locates at N or B (total market: 2,000 burgers/day). Your net loss is then 1,000 burgers/day. Payoff: 1, 000 BS or NS: You locate at N or B (total market: 2,000 burgers/day), your opponent locates at S (total market: 1,000 bugers/day) Your net gain is then 1,000 burgers/day. Payoff: 1,000 Payoff matrix:

You

2 >

Your Opponent

0

0

0

1,000

2

>

0

1,000

1,000

0

1,000

30. Take B = Brakpan; N = Nigel; S = Springs. Your strategies are to locate in one of B, N, S, and your opponent's strategies are the same. The possible outcomes are: BB, SS, or SS: You share the total business. No net gain or loss of sales. Payoff: 0 BN: You locate at B (total market: 2,500 burgers/day) and your opponent at N (total market: 1,500 burgers/day). Your net gain is then 1,000 burgers/day. Payoff: 1,000 BS: You locate at B (total market: 2,500 burgers/day) and your opponent at S (total market: 1,200 burgers/day). Your net gain is then 1300 burgers/day. Payoff: 1,300 NB: You locate at N (total market: 1,500 burgers/day) and your opponent at B (total market: 2,500 burgers/day). Your net loss is then 1000 burgers/day. Payoff: 1, 000 NS: You locate at N (total market: 1,500 burgers/day) and your opponent at S (total market: 1,200


Solutions Section 5.4 burgers/day). Your net gain is then 300 burgers/day. Payoff: 300 SB: You locate at S (total market: 1,200 burgers/day), and your opponent at B (total market: 2,500 burgers/day). Your net loss is then 1300 burgers/day. Payoff: 1, 300 SN: You locate at S (total market: 1,200 burgers/day), and your opponent at N (total market: 1,500 burgers/day). Your net loss is then 300 burgers/day. Payoff: 300 Payoff matrix:

You

2

2

Your Opponent

0

1,000

1,300

1,300

300

0

1,000

>

>

0

300

31. Let P = Pleasant Tap; T = Thunder Rumble; S = Strike the Gold, N = None. Your strategies: P, T, S. Your "opponent" is Nature, which decides the winner. Opponent's strategies: P, T, S, N. The possible outcomes are: PP: You bet on P and P wins. Odds: 5–2. 2 ! @ CD5. Therefore $10 wins you 5 × $5 = $25. Payoff: 25 TT: You bet on T and T wins. Odds: 7–2. $2 wins you $7. Therefore $10 wins you 5 × $7 = $35. Payoff: 35 SS: You bet on S and S wins. Odds: 4–1. $1 wins you $4. Therefore $10 wins you 10 × $4 = $40. Payoff: 40 All other outcomes: You lose your $10 because your horse fails to win. Payoff: 10 Payoff matrix: -

You Bet

>

-

Winner

>

35

10

2

25

10

10

10

10

10

40

10

10

10

32. Let P = Pleasant Tap; T = Thunder Rumble; S = Strike the Gold, N = None. Your strategies: P, T, S. Your "opponent" is Nature, which decides the winner. Opponent's strategies: P, T, S, N. The possible outcomes are: PP: You bet on P and P wins. Odds: 5-2. $2 wins you $5. Therefore $10 wins you 5 × $5 = $25. Payoff: 25 TT: You bet on T and T wins. Odds: 2-5. $5 wins you $2. Therefore $10 wins you 2 × $2 = $4. Payoff: 4 SS: You bet on S and S wins. Odds: 4-1. $1 wins you $4. Therefore $10 wins you 10 × $4 = $40. Payoff: 40 All other outcomes: You lose your $10 because your horse fails to win. Payoff: -10 Payoff matrix: -

You Bet

>

-

Winner

>

4

10

2

25

10

10

10

10

10

40

10

10

10


60 40

33. - =

30

50

10

40

= :- =

,:=

.50 .50

.50 .50 , =

60 40

30 50 You can expect to lose 39 customers. 34. - =

30

= :- =

50

,:=

.30 .70

10

200 150 140

130 220 130 , : = 110 110 220

= :- =

0.80

0.20

40

= 15 45

0.80

0.80

.30 .70 , =

30 50 You can expect to lose 9.8 customers. 35. - =

0.20

Solutions Section 5.4

0.20

0.80 0.20

0.5 0.3

0.5 =

110 110 220

0.3

The largest coefficient is the coefficient of , so we take : = the suburbs (corresponding to ) is the best option. 20 20 10

36. - =

10 15 20 , : = 10 20 20

= :- =

, 10 15 20 10 20 20

0.6 0.3

0.1

37. a. - = = :- =

90 70 70

0.6 = 0.3

40 90 40 , : =

0.25 0.50 0.25 , =

0.25 0.50 0.25

40 90 40

60 40 90

90 70 70

0.25

175 = 157 + 175 + 143 143

0 1 0 meaning that Option (II): Move to

16 = 17 + 16 + 19 19

0 0 1 , meaning that Rice

0.25 0.50 0.25

0.50 = 0.25 0.50 0.25

60 40 90 0.25 You can expect to get about 66% on the test. 90 70 70 0.25 b. - = 40 90 40 , : = , = 0.50 60 40 90

157

17

The largest coefficient is the coefficient of , so we take : = (corresponding to ) is the best option.

= 9.8

0.1

, =

20 20 10

0.20

= 39

0.2

0.2

130 220 130

0.80

0.80

= 18 23

, =

200 150 140

0.20

0.25

75 65

57.5

= 65.625


= :- =

90 70 70 40 90 40 60 40 90

0.25

0.50 = 0.25

The largest coefficient is the coefficient of , so we take : = (corresponding to ) is the best option. Your expected score is then 75(1) + 65(0) + 57.5(0) = 75

c. - =

90 70 70

40 90 40 , : = 60 40 90

= :- =

0.25 0.50 0.25 , =

0.25 0.50 0.25

75

Solutions Section 5.4

90 70 70 40 90 40 60 40 90

65

57.5

= 75 + 65 + 57.5

1 0 0 , meaning that game theory

= 57.5 72.5 60

= 57.5 + 72.5 + 60

The lowest coefficient is the coefficient of , so we take =

1 0 0 T , meaning that game theory would be worst for you, and you could expect to earn 57.5(1) + 72.5(0) + 60(0) = 57.5 for the test.

38. a. - =

30 0 0

= :- =

0

70 0

20

0 ,:=

70

0.25 0.50 0.25 , =

0.25 0.50 0.25

30 0

0

70

20 0

0.25

0.25 0.50 0.25

0.50 = 0.25 0.50 0.25

0 0 70 0.25 Joe can expect to get 25% on the test. 30 0 20 0.25 b. - = 0 70 0 , : = , = 0.50 0

= :- =

0

70

30 0 0

0

70 0

20 0

70

0.25

0.25

0.50 = 0.25

The largest coefficient is the coefficient of , so we take : = (corresponding to ) is the best option. Joe's expected score is then 12.5(0) + 35(1) + 17.5(0) = 35

c. - =

30 0 0

= :- =

0

70 0

20

0 ,:=

70

0.25 0.50 0.25 , =

0.25 0.50 0.25

30 0 0

0

70 0

20 0

70

12.5 35

17.5

35

17.5

= 25

= 12.5 + 35 + 17.5

0 1 0 , meaning that linear programming

= 7.5 35 22.5

12.5

The lowest coefficient is the coefficient of , so we take =

= 7.5 + 35 + 22.5

1 0 0 T , meaning that game theory would be worst for you, and Joe could expect to earn 7.5(1) + 35(0) + 22.5(0) = 7.5 for the test.

39.


a. - =

500,000 200,000 0

200,000

10,000

100,000

Solutions Section 5.4

10,000 0

200,000 0

200,000 300,000

,:=

, =

10,000

200,000

(based on the past 10 years) = :- = =

500,000 200,000 200,000

0

0

0.3 0.3

,

0.2

0

0.3

10,000

40,000

= 77, 000 40, 000 168, 000

168,000

0.2

0.2

100,000 77,000

0.2

200,000 300,000

0.3

The largest coefficient is the coefficient of , so we take : =

0 1 0 . meaning that laying off 10 workers (corresponding to ) is the best option. The expected payoff is 77, 000(0) 40, 000(1) 168, 000(0) = 40, 000, corresponding to a cost of $40,000. 200,000 500,000 200,000 10,000 b. - = 200,000 , : = 0.50 0 0.50 , = , 0 0 0 100,000 10,000 200,000 300,000 = :- =

=

0.50 0 0.50

500,000 200,000 200,000 100,000

0

10,000

10,000 0

200,000 0

200,000 300,000

300,000 95,000 95,000 50,000

= 300,000 95,000 95,000 50,000

The lowest coefficient is the coefficient of , so we take =

1 0 0 0

T , meaning that 0 inches of

snow would be worst for him, costing him $300,000. c. The strategy from part (a) is : = 0 1 0 ; laying off 10 workers. If he does so, the worst that could happen is 0 inches of snow (which would cost him $20,000). Since he feels that the Gods of Chaos are planning on 0 inches of snow, his best option would be to lay off 15 workers (according to the payoff matrix) and cut his losses to $100,000. 40. a. - =

5 10 5

5

3

5

10 0

3

5 3 10 (based on the past 10 years)

,:=

1 3 1 3 1 3 0 , =

0.25

0.50 , 0.25


Solutions Section 5.4 = :- =

1 3 1 3 1 3 0

Thus, the expected score is 1 3. 5 10 10 b. - =

5

5 5

3

5

3

= :- =

0

3

10

,:=

10

5

10

5

3

5

5

3

0

3

5 10

10

5

10

5

5

3

5

3

3.75

0.25

1 8 1 4 3 8 1 4 , =

1 8 1 4 3 8 1 4

= 0 + 2.125 0.125

5 10

0

3

The lowest coefficient is the coefficient of , so we take =

2.75

0.50 = 1 3 1 3 1 3 0

2

0.25

0.25

= 1 3

= 0 2.125 0.125

0 0 1

T , corresponding to a "laid back"

reviewer. c. From part (b), = 0 + 2.125 0.125 . You would obtain the highest score from the most impressed reviewer, so choose the largest coefficient: the coefficient of , corresponding to a "dead serious" reviewer. 41. a. The payoff matrix is - =

15 60 80

15 60 60 .

10 20 40 Checking the rows: Row 1 dominates each of the other rows, so we eliminate rows 2 and 3, leaving 15 60 80 .

Checking the columns: Column 1 dominates each of the other columns, so we eliminate columns 2 and 3, leaving the 1 × 1 game 15 , which corresponds to the first CE strategy (charge $1,000) and the first GCS strategy (charge $900). So, CE should charge $1,000 and GCS should charge $900. Since the payoff is 15, a 15% gain in market share for CE results. b. Look at the original (unreduced) payoff matrix. CE is aware that GCS is planning to charge $900. For the best market share, CE should charge either $1,000 or $1,200 because either will result in the best market share (15% gain) under the circumstances. Thus, in terms of market share, the added information has no effect. However, in terms of revenue, the better of the two options would be to charge the larger price: $1,200 (the more CE can charge for the same market, the better!). 42. a. The payoff matrix is - =

20 60 60

15 60 60 .

10 20 40 Checking the rows: Row 1 dominates each of the other rows, so we eliminate rows 2 and 3, leaving 20 60 60 .

Checking the columns: Column 1 dominates each of the other columns, so we eliminate columns 2 and 3, leaving the 1 × 1 game 20 ,


Solutions Section 5.4 which corresponds to the first CE strategy (charge $1,000) and the first GCS strategy (charge $900). So, CE should charge $1,000 and GCS should charge $900. Since the payoff is 20, a 20% gain in market share for CE results. b. Because, if the payoff matrix is arranged so that prices decrease from left to right, and top to bottom, the payoffs tend to decrease down the columns and increase along the rows, with the effect that dominance reduces the matrix to the top left corner. More simply, lower prices always dominate higher prices. 43. Take CCC as the row player and MMA as the column player. Thus, CCC's strategies are Pablo, Sal and Edison, while MMA's strategies are Carlos, Marcus, and Noto. To set up the payoff matrix, enter 1 for every combination in which the CCC wrestler beats the MMA wrestler, 1 for every combination in which the MMA wrestler beats the CCC wrestler, and 0 in all the evenly matched combinations. The resulting payoff matrix is MMA

Carlos 1

Marcus

0

1

Noto

0

1 1

0

1

Pablo CCC

Sal Edison

0

Comparing rows, we see that Row 1 dominates the other two rows, so we eliminate Rows 2 and 3, leaving Carlos

Marcus

1

Pablo

1

Noto 0

Comparing columns, we see that Column 3 dominates the other two columns, so we eliminate Columns 1 and 2, leaving us with Noto Pablo

0

.

Thus the game is reduced to Pablo vs. Noto. Since the payoff is 0, the game is evenly matched. 44. Take CCC as the row player and MMA as the column player. Thus, CCC's strategies are Hans, Sal, and Edison, while MMA's strategies are Carlos, Marcus, and Noto. To set up the payoff matrix, enter 1 for every combination in which the CCC wrestler beats the MMA wrestler, 1 for every combination in which the MMA wrestler beats the CCC wrestler, and 0 in all the evenly matched combinations. The resulting payoff matrix is MMA

CCC

Hans Sal Edison

Carlos 1

Marcus

0

1

Noto

0

1 1

0

1

0

Comparing rows, we see that Row 2 dominates the other two rows, so we eliminate Rows 1 and 3 leaving


Solutions Section 5.4 Carlos

Marcus

0

Sal

Noto 0

1

Comparing columns, we see that Column 2 dominates the other two columns, so we eliminate Columns 1 and 3, leaving us with Marcus 1

Sal

Thus the game is reduced to Sal vs. Marcus. Since the payoff is 1, MMA has the advantage. 45. We first reduce by dominance (following the "FAQ" in the textbook): Comparing the rows, we find that neither row dominates the other. Comparing the columns, we see than Column 1 dominates Column 2. So we eliminate Column 2, leaving Northern Route 2

Northern Route

1

Southern Route

Comparing the rows, we now see that Row 1 dominates Row 2, so we eliminate Row 2, leaving us with Northern Route 2

Northern Route

Thus, both commanders should use the northern route, resulting in an estimate of two days' bombing time. 46. We first reduce by dominance (following the "FAQ" in the textbook): Comparing the rows, we find that neither row dominates the other. Comparing the columns, we see than Column 1 dominates Column 2. So we eliminate Column 2, leaving Northern Route 2

Northern Route

1.5

Split Recon

1

Southern Route

Comparing the rows, we now see that Row 1 dominates the other rows, so we eliminate Row 2, leaving us with Northern Route Northern Route

2

Thus, both commanders should use the northern route, resulting in an estimate of two days' bombing time. 47. Take C = Confess and N = Do not confess. If we take the (negative) payoffs as the amount of time Slim faces behind bars, we get: Joe

C Slim

N

C

N

10

5

2

0


Solutions Section 5.4 As Row 1 dominates Row 2, Slim's optimal strategy is to confess. 48. Take C = Confess and N = Do not confess. If we take the payoffs as the amount of time Jane faces behind bars, we get: Prudence

N

10

0

2

C Jane

C

N

0

As Row 1 dominates Row 2, Jane's optimal strategy is to confess. 49. a. Let F represent a visit to Florida and O a visit to Ohio. The outcomes are: FF and OO: Both candidates visit the same state, so Goode still has a 24% chance of winning, and so the payoff is 24 in both cases. FO: Goode visits Florida, giving him a 60 + 10 = 70 chance in that state, and Slick visits Ohio, reducing Goode's chances there to 40 10 = 30 Thus, the probability of Goode winning both states is 0.70 × 0.30 = 0.21 OF: Goode visits Ohio, giving him a 40 + 10 = 50 chance in that state, and Slick visits Florida, reducing Goode's chances there to 60 10 = 50 Thus, the probability of Goode winning both states is 0.50 × 0.50 = 0.25 This gives the following payoff matrix: Slick

F

F Goode

O

24

25

O

21

24

b. In the payoff matrix in part (a), Row 2 dominates Row 1 (meaning that Goode should visit Ohio). In that case): F

O

25

O

24

Column 2 now dominates Column 1, leaving us with the following solution O

O

24

meaning that both candidates should visit Ohio, leaving Goode with a 24% chance of winning the election. 50. a. Let F represent a visit to Florida and O a visit to Ohio. The outcomes are: FF and OO: Both candidates visit the same state, so Goode has a 0.90 × 0.80 = 0.72, or 72% chance of winning, so the payoff is 72 in both cases. FO: Goode visits Florida, giving him a 90 + 10 = 100 chance in that state, and Slick visits Ohio, reducing Goode's chances there to 80 10 = 70 Thus, the probability of Goode winning both states is 1.00 × 0.70 = 0.70.


Solutions Section 5.4 OF: Goode visits Ohio, giving him a 80 + 10 = 90 chance in that state, and Slick visits Florida, reducing Goode's chances there to 90 10 = 80 Thus, the probability of Goode winning both states is 0.80 × 0.90 = 0.72. This gives the following payoff matrix: Slick

F

F Goode

O

72

72

O

70

72

b. In the payoff matrix in part (a), Row 2 dominates Row 1 (meaning that Goode should visit Ohio. in that case): F

O

O

72

72

Each column now dominates the other, so whether Slick visits Florida or Ohio will make no difference. Note, however, that if we first eliminate dominated columns and then eliminate dominated rows, we are reduced to the single solution O

O

72

meaning that both candidates should visit Ohio, leaving Goode with a 72% chance of winning the election. 51. The payoff matrix (payoffs in thousands of dollars) is Splish

WISH Softex

WISH

WASH

0

20

100

20

WASH

We are asked to provide the optimal row strategy. Take : = 1 , = 1 0 T = :- =

Take : =

1

1 , =

A = :- =

1

20 100

1

T

0

0

0 1

20

20 100 0

20

0

1

= 20 + 0(1 ) = 20 = 100 20(1 ) = 120 20


Solutions Section 5.4

Graphs of and A :

Lower (heavy) portion has its highest point at the intersection of the two lines. The -coordinate of the intersection is given when = A : 20 = 120 20 140 = 20 = 1 7. So the optimal row strategy is : = 1 = 1 7 6 7 .

This solution corresponds to allocating 1/7 of its advertising budget to WISH and the rest (6/7) to WASH. The value of the game is then = 20 = 20 7 . 2.86. So Softex can expect to lose approximately $2,860. 52. The payoff matrix (payoffs in millions of dollars) is Labor

5%

15%

10

12

20

Accept Company Offer

Reject

10

We are asked to provide the optimal row strategy. Take : = 1 , = 1 0 T = :- =

Take : =

1

1 , =

A = :- =

1

20 10 10 12

0 1

T

20 10

1 0

= 20 + 10(1 ) = 10 + 10

0

10 12 1 = 10 + 12(1 ) = 2 + 12 Graphs of and A :

Lower (heavy) portion has its highest point at the intersection of the two lines. The -coordinate of the intersection is given when = A :


Solutions Section 5.4 10 + 10 = 2 + 12 12 = 2 = 1 6 So the optimal row strategy is : = 1 = 1 6 5 6 . The company should offer 1 6 × 5 The value of the game is then = 10 + 10 = 10(1 6) + 10 = 70 6 . 11.7. The company can expect to ear around $11.7 million in profits.

53. Like a saddle point in a payoff matrix, the center of a saddle is a low point (minimum height) in one direction and a high point (maximum) in a perpendicular direction.

54. In one trivial case: a 1 × 1 game. Otherwise, it would violate the requirement that a saddle point be a row minimum. 55. Although there is a saddle point in the (2,4) position, you would be wrong to use saddle points (based on the minimax criterion) to reach the conclusion that row strategy 2 is best. The most important reason is that the given table does not represent the payoff matrix of a two-person zero-sum game: Having a job in a category with large numbers of employees does not necessarily represent a benefit to the row player, and there is no opponent deciding what your job will be in such a way as to force you into the least populated job category. 56. What a mixed strategy in this context really means is to choose, at random, only one topic to review with probability as given by the mixed row strategy. The expected test score is then the average you would obtain if you took many tests under these circumstances, each time selecting, at random, only one topic to review. This calculation of expected test score does not truly reflect what you would achieve by studying several topics before the test. 57. If you strictly alternate the two strategies, the column player will know which pure strategy you will play on each move and can choose a pure strategy accordingly. For example, consider the game

1

0

0

1

.

By the analysis of Example 3 (or the symmetry of the game), the best strategy for the row player is 0.5 0.5 , and the best strategy for the column player is 0.5 0.5 T . This gives an expected value of 0.5 for the game. However, suppose that the row player alternates and strictly and that the column player catches on to this. Then, whenever the row player plays the column player will play , and whenever the row player plays the column player will play . This gives a payoff of 0 each time, worse for the row player than the expected value of 0.5. 58. Given the payoff matrix

1 2 1 3

, the row player could use either 1 0 or 0.5 0.5 as a strategy

(or any mixed strategy, for that matter). The column player will always use 1 0 game is 1.

T , and the value of the


Solutions Section 5.5 Section 5.5 1. =

0.2 0.05

; Sector 1 = Paper; Sector 2 = Wood 0.8 0.01 a. Wood → Paper = Sector 2 → Sector 1 ; The corresponding entry of the technology matrix is 21 = 0.8. b. Paper → Paper = Sector 1 → Sector 1 ; The corresponding entry of the technology matrix is 11 = 0.2. c. Paper → Wood = Sector 1 → Sector 2 The corresponding entry of the technology matrix is 12 = 0.05. 2. =

0.01 0.001

; Sector 1 = Processor chips; Sector 2 = Silicon 0.2 0.004 a. Silicon → Silicon = Sector 2 → Sector 2; The corresponding entry of the technology matrix is 22 = 0.004. b. Processor chips → Silicon = Sector 1 → Sector 2; The corresponding entry of the technology matrix is 12 = 0.001. c. Silicon → Processor chips= Sector 2_Sector 1 The corresponding entry of the technology matrix is 21 = 0.2. 3. Sector 1 = Television news, Sector 2 = Radio news 11 = units of Television news needed to produce one unit of Television news = 0.2 12 = units of Television news needed to produce one unit of Radio news = 0.1 21 = units of Radio news needed to produce one unit of Television news = 0.5 22 = units of Radio news needed to produce one unit of Radio news = 0 Thus, =

0.2 0.1 0.5

0

.

4. Sector 1 = Cologne, Sector 2 = Perfume 11 = units of Cologne needed to produce one unit of Cologne = 0 12 = units of Cologne needed to produce one unit of Perfume = 0.1 21 = units of Perfume needed to produce one unit of Cologne = 0.5 22 = units of Perfume needed to produce one unit of Perfume = 0.3 Thus, =

5.

0

0.1

0.5 0.3

.

0 = (8 ) 1 = =

1 0 0 1

2 1.6 0

2

0.5 0.4 0

10,000 20,000

0.5 =

−1

10,000 20,000

52,000 40,000

=

0.5 0.4 0

0.5

−1

10,000 20,000


Solutions Section 5.5 6.

1 0

0 = (8 ) = 1

0 1

2 1.6

=

7.

0

0 = (8 ) =

0 1

1 0

0 = (8 ) = 1

0 1

0 = (8 ) =

25,000 15,000

0.1 0.2 0.4 0.5

24,000 14,000

0.5 0.1

0.5 0.1

0

0

0.5

0

0 = (8 ) =

−1

1 0 0 0 0 1

1

−1

0.2 0.5

0 1 0

1

0

0 0

0

−1

0.5

0

0

14,000

0.1 0.5

0

0.5

10,000

0.9

0.4

−1

25,000

0.9

0.2

−1

24,000

−1

0.2

0.5

15,000

0.4

0.5

14,000

60,000

0

0.5 0.1 0

=

40,000

=

−1

0.5

3,000

1,000

2 0.4 0.08 0

2

0

−1

3,800 = 2,000

1,000 2,000

1,000 =

0

0.5

24,000

1,000

0.5 0.1

0

20,000

−1

50,000

2,000

0

=

50,000

=

0.5

0.5 0.1

0

15,000

0

1 0 0 0 0 1

25,000

0.5 0.1

0.1

0 1 0

20,000

0.5 0.4

=

20,000

0.1 0.4

40 37 90 37

= 11.

10,000

10,000

56,000

=

50 37 20 37

=

10.

20,000

−1

0.5

20 37 90 37

=

9.

0

50 37 40 37

=

8.

2

1 0

1

0.5 0.4

0

0.4 2

1,000

1,000 = 2,000

2,560 2,800 4,000

3,000 3,800 2,000

2 0.4 0.08 0 0

2 0

0.4 2

3,000

3,800 = 2,000

8,000

10,000 4,000


Solutions Section 5.5

0 = (8 ) = 1

=

1 0 0

0.8

0 1 0

0.2 0

27,000 28,000 17,000

0 0 1

0.2 0.6

0.2

0.2

0.2

0

−1

0.2 0

0.2 0.8

0.4 0.2

0

0.2

−1

0.2

16,000 8,000 8,000

16,000 8,000 8,000

=

11 8 1 2 1 8

1 2 2

1 2

1 8

1 2

11 8

16,000 8,000 8,000

=


Solutions Section 5.5

12.

1

0 = (8 ) =

0

1

= =

0 1

0

0.8

0

0

0 1

0.2

11 7 5 7 4 7

0.2

0.2

−1

7,000

0.2

0.4

0.2

0.2

0.2

5 7

4 7

0.6

0.2

0.2

0.2

0.2

15 7

0.2

7,000 7,000

13. Change in Production = (8 ) 1× Change in Demand = =

1 0 0 1

0.1 0.4 0.2 0.5

50 37 40 37 20 37 90 37

50 30

50

−1

=

=

30

100 100

0.9

0.2

0.4 0.5

−1

14. Change in Production = (8 ) 1× Change in Demand =

1 0 0 1

0.5 0.4 0

0.5

1.5 0.1

0

−1

20 10

=

0.5 0.4 0

0.5

−1

20 10

15. Change in Production = (8 ) 1× Change in Demand =

0.2 1.2 0.1 0.1 0.7 1.6

1

40,000 25,000 50 30

=

2 1.6 0

2

20 10

=

56 20

1.5

0 = 0.2 0

25,000

=

14,000

11 7

14,000 7,000

7,000

5 7

5 7

0.2

7,000

14,000

0.8

0.2

−1

0.1

Note that this is the first column of (8 ) 1. In general, the !th column of (8 ) 1 gives the change in production necessary to meet an increase in external demand of one unit for the product of Sector !. 16. Change in Production = (8 ) 1× Change in Demand =

1.5 0.1

0

0.1 1.1 0.1 0

0

1.3

1

1 = 1

1.6 1.3 1.3

Note that this obtained by adding the columns of (8 ) 1.


Solutions Section 5.5 17. Given table:

To A

B

C

A 1,000 2,000 3,000 From

B

0

4,000

0

C

0

1,000 3,000

Total Output 5,000 5,000 6,000 We obtain the technology matrix from the input-output table by dividing each column by its total: 1000 5000 2000 5000 3000 6000 0.2 0.4 0.5 = = 0 4000 5000 0 0 0.8 0 . 0

1000 5000 3000 6000

18. Given table:

0

0.2 0.5

To

From

A

B

C

A

0

100

300

B

500

400

300

C

0

0

600

Total Output 1,000 2,000 3,000 We obtain the technology matrix from the input-output table by dividing each column by its total: 0 100 2000 300 3000 0 0.05 0.1 = 500 1000 400 2000 300 3000 = 0.5 0.2 0.1 . 0

0

600 3000

0

0

0.2

19. First obtain the technology matrix from the input-output table by dividing each column by its total: 10,000 50,000 20,000 40,000 0.2 0.5 = = 5000 50,000 0 0.1 0 Production = (8 ) 1× Demand =

=

1 0 0 1

4 3

0.2 0.5 0.1

2 3

0

−1

45,000

45,000

=

30,000

=

80,000

0.8 .5

.1

1

−1

45,000 30,000

. 2 15 16 15 30,000 30,000 Solution: The Main DR had to produce $80,000 worth of food, while Bits & Bytes had to produce $38,000 worth of food. 20. First obtain the technology matrix from the input-output table by dividing each column by its total: 20 100 10 200 0.2 0.05 = . = 10 100 30 200 0.1 0.15 Production = (8 ) 1× Demand =

1 0 0 1

0.2 0.05 0.1 0.15

−1

270 810

=

0.8 0.05

.1

0.85

−1

270 810


=

34 27

2 27

270

400

Solutions Section 5.5

=

4 27 32 27 810 1,000 Solution: The clubs must write 400 Choral Society papers and 1,000 Football club papers. 21. First obtain the technology matrix from the input-output table by dividing each column by its total: 8,000 50,000 2,000 20,000 0.16 0.1 = = 700 50,000 2,000 20,000 0.014 0.1 Production = (8 ) 1 × Demand =

.

1 0 0 1

0.16

0.1

0.014 0.1

1.19268 0.13252

−1

60,000

60,000 30,000 .

=

75,537

0.84

0.014

0.1 0.9

−1

60,000 30,000

. 0.01855 1.11317 30,000 34,508 Solution: Textiles sector production approximately $76,000 million, Apparel sector production approximately $35,000 million. 22. First obtain the technology matrix from the input-output table by dividing each column by its total: 25,000 120,000 7,000 190,000 0.20833 0.03684 = = 800 120,000 44,000 190,000 0.00667 0.23158 (Results shown to 5 decimal places, but calculations carried out with more accuracy.) Production = (8 ) 1× Demand =

.

1 0 0 1

0.20833 0.03684 0.00667 0.23158

1.26367 0.06059

10,000

−1

.

10,000 20,000

13,848

=

0.79167

0.00667

0.03684 0.76842

−1

10,000 20,000

. 0.01096 1.30190 20,000 26,148 Solution: Wood products need to increase by approximately $14,000 million; paper products need to increase by approximately $26,000 million. 23. (8 ) 1 =

1.019 0.022

0.270 1.614 Sector 1 = Electricity generation, Sector 2 = Electricity distribution a. The missing term is the number of units of Sector 2 needed to produce one additional unit of Sector 1. This quantity is given by the 2,1-entry of (8 ) 1, or 0.270. b. The missing terms refer to the 1,2-entry of (8 ) 1, which gives the number of units of Sector 1 needed to produce one additional unit of Sector 2—in other words, the additional dollars worth of electricity generation that must be produced to meet a one-dollar increase in the demand for electricity distribution. 24. (8 ) 1 =

1.198 0.127

0.019 1.085 Sector 1 = Professional services, Sector 2 = Computer services a. 0.019 is the 2,1-entry of (8 ) 1, and gives the number of units of Sector 2 needed to produce one additional unit of Sector 1. Thus, 0.019 additional dollars worth of computer services must be produced to meet a $1 increase in the demand for professional services. b. The missing quantity is the number of units of Sector 1 needed to produce one additional unit of Sector 1. This quantity is given by the 1,1-entry of (8 ) 1, or 1.198.


Solutions Section 5.5 25. As the technology matrix is obtained from the input-output table by dividing each column by the production total for that column, we perform this process in reverse to obtain the input-output table; that is, multiply each entry in the technology matrix by the production total for that column. Thus, the inputoutput matrix is To Primary Secondary Tertiary

From

Primary

62.4

420

0

Secondary

117

3,080

780

Tertiary

54.6

1,960

1,820

Total output

780

14,000

13,000

(Entries are in billions of pesos.) 26. As the technology matrix is obtained from the input-output table by dividing each column by the production total for that column, we perform this process in reverse to obtain the input-output table; that is, multiply each entry in the technology matrix by the production total for that column. Thus, the inputoutput matrix is To Primary Secondary Tertiary

From

Primary

47.2

330

0

Secondary

88.5

2,420

576

Tertiary

41.3

1,540

1,344

Total output

590

11,000

9,600

(Entries are in billions of pesos.) 27. =

0.08 0.03

0

0.15 0.22 0.06 , 0 = 0.07 0.14 0.14

780

14,000 13,000

Amount available for external use = = 0 0 = =

780

482.4

0.08 0.03

0

14,000

3,977

=

297.6

10,023

780

0.08 0.03

0

14,000 0.15 0.22 0.06 13,000

300

. 10,000

0.07 0.14 0.14

780

14,000 13,000

13,000 3,834.6 9,165.4 9,200 300 billion pesos of raw materials, 10,000 billion pesos of manufactured goods, and 9,200 billion pesos of services. 28. =

0.15 0.22 0.06 , 0 = 0.07 0.14 0.14

590

11,000 9,600


590

Solutions Section 5.5 Amount available for external use = = 0 0 = =

590

377.2

11,000 3,084.5 =

212.8

0.08 0.03

0

11,000 0.15 0.22 0.06

210

9,600

0.07 0.14 0.14

7,915.5 . 7,900

590

11,000 9,600

9,600 2,925.3 6,674.7 6,700 210 billion pesos of raw materials, 7,900 billion pesos of manufactured goods, and 6,700 billion pesos of services. 29. To determine how each sector would need to react to an increase (or decrease) in demand, we compute −1 0.92 0.03 0 1.094 0.043 0.003 1 (8 ) = 0.15 0.78 0.06 . 0.220 1.307 0.091 . 0.07 0.14

0.86

0.125 0.216 1.178

TI-83/84 Plus Format: (Identity(3)-[A])-1 Matrix Algebra Tool: (I-A)^-1 We are given external demand increases of 2,000, 0, and −1,000 billion pesos in the sectors, so we multiply (8 ) 1 by the column matrix =

2,000 0

and obtain //(8 ) . 1

2,185 349

1,000 928 Thus, production in the primary sector would rise by around 2,185 billion pesos, production in the secondary sector would rise by around 349 billion pesos, and production in the tertiary sector would drop by 928 billion pesos. .

30. To determine how each sector would need to react to an increase (or decrease) in demand, we compute −1 0.92 0.03 0 1.094 0.043 0.003 1 (8 ) = 0.15 0.78 0.06 . 0.220 1.307 0.091 . 0.07 0.14

0.86

0.125 0.216 1.178

TI-83/84 Plus Format: (Identity(3)-[A])-1 Matrix Algebra Tool: (I-A)^-1 We are given external demand increases of −1,000, −1,000, and 1,000 billion pesos in the sectors, so we multiply (8 ) 1 by the column matrix =

1,000

1,134

1,000 and obtain //(8 ) . 1,436 . 1

1,000 837 Thus, production in the primary sector would decrease by around 1,134 billion pesos, production in the secondary sector would decrease by around 1,436 billion pesos, and production in the tertiary sector would increase by 837 billion pesos.

31. To determine how each sector would need to react to an increase in demand, we compute (8 ) 1 , where is the increase in demand. First obtain the technology matrix from the input-output table by dividing each column by its total output: .

0.03582 0.00314 0.00359 0.00900 0.00016 0.14063 0.05350 0.00018 0.00889 0.01000 0.13339 0.02362 0.00991 0.00722 0.01081 0.04024

Then compute the matrix (8 ) 1.

.


Solutions Section 5.5 TI-83/84 Plus Format: (Identity(4)-[A])-1 Matrix Algebra Tool: (I-A)^-1 (8 ) 1 .

1.03730 0.00392 0.00466 0.00984 0.00088 1.16451 0.07193 0.00200 0.01094 0.01372 1.15517 0.02854 0.01084 0.00896 0.01360 1.04236

We are given external demand increases of $1,000 million in each sector, so we multiply (8 ) 1 by the column matrix 1,000 1,056 =

1,000 1,000

1,239

and obtain //(8 ) 1 .

1,208

.

1,000 1,076 The entries of this matrix show the amounts, in millions of dollars, by which each sector needs to increase production to meet the additional demand.

32. To determine how each sector would need to react to an increase in demand, we compute (8 ) 1 , where is the increase in demand. First obtain the technology matrix from the input-output table by dividing each column by its total output: .

0.00138 0.00043 0.00572 0.00454 0.00003 0.00096 0.00000 0.01038 0.00085 0.00021 0.00077 0.00110 0.00541 0.00201 0.00323 0.01578

Then compute the matrix (8 ) 1. TI-83/84 Plus Format: (Identity(4)-[A])-1 (8 ) 1 .

.

Matrix Algebra Tool: (I-A)^-1

1.00141 0.00045 0.00574 0.00463 0.00008 1.00098 0.00003 0.01055 0.00086 0.00021 1.00078 0.00112 0.00550 0.00204 0.00332 1.01609

We are given external demand increases of $1,000 million in each sector, so we multiply (8 ) 1 by the column matrix 1,000 1,012 =

1,000 1,000

and obtain //(8 ) 1 .

1,012 1,003

.

1,000 1,027 The columns of this matrix show the amounts, in millions of dollars, by which each sector needs to increase production to meet the additional demand. 33. a. To determine how each sector would need to react to an increase in demand, we compute (8 ) 1 , where is the increase in demand. First obtain the technology matrix from the input-output table by dividing each column by its total output:


0.06245 0.03834 0.01290 0.00048

Solutions Section 5.5

.

0.00128 0.01597 0.00522 0.00000 0.06196 0.06401 0.02278 0.00048 0.00000 0.00000 0.00000 0.02391

.

Then compute the matrix (8 ) 1. TI-83/84 Plus Format: (Identity(4)-[A])-1

Matrix Algebra Tool: (I-A)^-1

1.06762 0.04253 0.01432 0.00053 0.00175 1.01666 0.00545 0.00000

(8 ) 1 .

0.06780 0.06929 1.02458 0.00054

0.00000 0.00000 0.00000 1.02450 We are given external demand increases of $100 in sheep, etc. and 0 in all others, so we multiply (8 ) 1 by the column matrix =

100 0 0

and obtain //0 = (8 ) 1 .

106.76 0.18 6.78

0 0.00 The additional production required from the other agriculture sector (Sector 3) is the (1,3)-entry: $6.78 (rounded to the nearest 1¢). b. The diagonal entries in (8 ) 1 show the additional production required from that sector to meet a $1 increase for the product of that sector. Since the largest diagonal entry is the 1,1-entry: 1.06762, we conclude that Sector 1 (sheep, grains, beef, and dairy cattle) requires the most of its own product in order to meet a $1 increase in external demand for that product. 34. a. To determine how each sector would need to react to an increase in demand, we compute (8 ) 1 , where is the increase in demand. First obtain the technology matrix from the input-output table by dividing each column by its total: .

0.00968 0.00115 0.00084 0.00105 0.00011 0.00398 0.00042 0.00133 0.00019 0.00039 0.17861 0.00078 0.00027 0.00038 0.00017 0.00603

Then compute the matrix (8 ) 1. TI-83/84 Plus Format: (Identity(4)-[A])-1 (8 ) 1 .

.

Matrix Algebra Tool: (I-A)^-1

1.00977 0.00116 0.00103 0.00107 0.00011 1.00400 0.00051 0.00134 0.00023 0.00048 1.21745 0.00096

0.00027 0.00039 0.00021 1.00606 We are given external demand increases of $1,000 in the aircraft manufacturing sector and 0 in all the others, so we multiply (8 ) 1 by the column matrix


=

0 0 0

1.07

Solutions Section 5.5 and obtain //0 = (8 ) 1 .

1.34 0.96

1,000 1006.06 The additional production required from the motor vehicles and parts sector (Sector 1) is the (1,1)-entry: $1.07 (rounded to the nearest 1¢). b. The off-diagonal entries in (8 ) 1 show the additional production required by each sector to meet a $1 increase for the product of some other sector. Since the largest off-diagonal entry is the 2,4-entry: 0.00134, we conclude that Sector 4 (aircraft manufacturing) requires the most of Sector 2 products (ship and boat manufacturing) to meet a $1 increase in external demand. 35. It would mean that all of the sectors require neither their own product nor the product of any other sector. 36. No, since each sector would require all of its own products in its production, and so there could be no net output. 37. The sum of the entries in a row of an input-output table gives the total internal demand for that sector's products. If that total was equal to the total output for that sector, it would mean that all of the output of that sector was used internally in the economy. Thus, none of the output was available for export and no importing was necessary. 38. It would mean that the economy required more of that product than the sector could produce, and so some must have been imported from outside the economy. 39. If an entry in the matrix (8 ) 1 is zero, then an increase in demand for one sector (the column sector) has no effect on the production of another sector (the row sector). 40. An increase in demand for the sector should cause an increase in production to match the new demand. Production will probably have to rise more than the increase in external demand to meet increased internal demand from an expanded economy.

41. The off-diagonal entries in (8 ) 1 show the additional production required by each sector to meet a one-unit increase for the product of some other sector. Usually, to produce one unit of one sector requires less than one unit of input from another. We would expect then that an increase in demand of one unit for one sector would require a smaller increase in production in another sector. 42. An increase in demand for one sector should cause the economy generally to grow. We do not expect an increase in demand for one sector to cause a decline in production in another.


Solutions Chapter 5 Review Chapter 5 Review

1. The sum of two matrices is defined only when they have the same dimensions. Because is 2 × 3 and B is 2 × 2, their dimensions differ, so the sum + is undefined. 1 2 3

2. = 3. =

4 5 6

1 4

3 2 1 1

2

3

1 4

2 5 , so 2 + = 2 2 5 +

3 6

1+3 2+2 3+1

=

3 6

4 1 5 2 6 3

1 0

1

1 =

1 0

4 4 4

=

3 3 3

8

5 11

1

6 13

4. Computing would require taking the product (2 × 3)(2 × 2). Because the number of columns of does not match the number of rows of , the product is undefined. 1 4

5. =

1 1

2 5

0

3 6

1

=

(1 + 0) ( 1 + 4)

(2 + 0) ( 2 + 5) = (3 + 0) ( 3 + 6)

1 3 2 3 3 3

6. Computing 2 would require taking the product (2 × 3)(2 × 3). Because the number of columns does not match the number of rows, the product is undefined. 7. 2 =

1 1 0

1

8. 3 = 2 = 9. + =

1 1 0

1

1 2 0

1

1 2 3 4 5 6

=

(1 + 0) ( 1 1) (0 + 0)

1 1

=

0

1

1

1 +

1 0 0

1

(0 + 1)

=

1 2 0

(1 + 0) ( 1 2) (0 + 0) 1 1 0

1

(0 + 1) 1

=

5

1 11

1

=

+

1 3 0

1

1 1 0

1

2

=

4

1 12

10. is 3 × 3, and is 2 × 2, so the sum + is undefined. 11. To find the inverse of

1 1 0

1 1 1 0 :1 + :2 0

1

0 1

1

, we augment with the 2 × 2 identity matrix and row-reduce: 1 0 1 1 0 1 0 1

.

The right-hand 2 × 2 block is the desired inverse:

12.

1 2 0 0

1 1 0

1

−1

=

1 1 0 1

.

is singular, as it has a row of zeros and therefore cannot be row-reduced to the identity.


Solutions Chapter 5 Review 13. To find the inverse of a 3 × 3 matrix, we augment it with the 3 × 3 identity matrix and row-reduce: 2 0 5 2 1 0 :1 5:3 1 2 3 1 0 0 2:1 :2 0 0

4 0

1 0 1 0

1 0 0 1

2 0 0 2 1 5 (1 2):1 0 4 0 0

→ 0 4

1

0 0

0

1

0

0

1 4 0

0 0 1 0 0 The right-hand 3 × 3 block is the desired inverse: 1 2 3 −1 1 1 2 5 2 = 0 1 4 1 4 . 0 4 1 0 0 1

0

1

1 0 0 1 1 2 5 2

1 (1 4):2 → 0 1 0 0 0 0 1 0 1

1

0 :2 :3

1

0

1 4 . 1

1

14. To find the inverse of a 4 × 4 matrix, we augment it with the 4 × 4 identity matrix and row-reduce: 1 2 3 4 1 0 0 0 1 2 3 4 1 0 0 0 :1 2:2 1 0 0

3 4 2 0 1 0 0 :2 :1 1 2 3 0 0 1 0 0 1 2 0 0 0 1

1 0

1

0 1

8

1

0 0

1

0 0

1

1 0 0

0 3 0

0 0 3

3

2 1 5

2

0

0

0

1

0

1

1

2

0 0 0 3 1

0

2 0 0 :1 :3 1

0 0 :2 :3

1 1 0 0

0

1 2 1

0 1 :4 :3

2 4

2 1

1

1

0

1

0

3

1

2

1 0 0 0 1 0 0 0 1

0 0 0

0

0 1 0 :3 :2

7

2

0

3 5

3

1 0 0 0

(1 3):3

0 1 0 0

0 0 1 1

1

0 1 2 3 0 0 1 2

=

1 3

2 3 1 3

1 3 2 3

1 3

4 3

2 3 1 3

1

1 1

1 3

The right-hand 4 × 4 block is the desired inverse: 1 2 3 4 −1 1 0 1 2 1 3 4 2

2

1 3

0 0 0 1

7 3 5 3

1 3

1 1 0 :1 + :4

2

0 0 1 0 2 3

(1 3):4

1

2

0

7 (1 3):2 5

1 2 1 1 0 0

1 0 3:2 7:4 1

0 3:3 + 5:4

1 1 0

1 3 2 3

1 3

2

4 3

2 3 1 3

1

7 3 5 3

1 3

.

.

15. To find the inverse of a 4 × 4 matrix, we augment it with the 4 × 4 identity matrix and row-reduce: 1 2 3 4 1 0 0 0 1 2 3 4 1 0 0 0 :1 + 2:2 2

3 3 3 0 1 0 0 :2 2:1

1

0

0 0

0

1

6 3 2 0 0 :1 3:3

1

1 2 3 0 0 1 0 0 1 2 0 0 0 1

0 1 0

0

0

0

3 3

1 1

0 0

5 2 1 0 0 :2 3:3 2 2 1 1 0 2

0

0 0 1 :4 + :3

1 0

3 5 2 1 0 0 2

3

0

0 1 0 :3 + :2

0 1

0

1

4

0 0

1

0 0 0

2

0

0

0

0 0 1 3

1 2 2 0

0

2

1 3 0 2 3 0 1 1

1 1

0 1

.

Since the left-hand 4 × 4 block has a row of zeros, we cannot reduce the matrix to obtain the 4 × 4 identity on the left. Therefore, the matrix is singular.


0 16.

Thus,

1 0 0

1 0 0 0

0 0 0 1

0 0 1 0

0 1 0 0 1 0 0 0 0 0 0 1

1

0

0

1

0

0

0

0

0 0 1 0

0

Solutions Chapter 5 Review 1

0 0

Rearrange rows →

1

0

0

1

0

0

0

0

is its own inverse.

0 0 1 0

17. + 2 = 0 3 + 4 = 2 Matrix form: 1 2

0

3 4 2 Solving gives 1 2 −1 0 1 2 = = 3 4 2 3 2 1 2 Solution: ( , ) = (2, 1) =

0 2

=

2

1

.

18. + + = 3 + 2 = 4 = 1 Matrix form: 1 1 1 0 1

2

3

= 4

0 1 1 Solving gives 1 1 1 = 0 1 2

1

−1

3

1 2 3 1 3 = 4 0 1 3 2 3

0 1 1 1 Solution: ( , , ) = (0, 2, 1)

19. + + = 2 + 2 + = 3 + + 2 = 1 Matrix form: 1 1 1 1 2 1

1 1 2 Solving gives

2 = 3

1

0

1 3

1 3

3

0 = 4 2 . 1

1

0 0 1 0

0 0 0 1

0 1 0 0

1 0 0 0

0 0 0 1

0 0 1 0

.


1 1 1

= 1 2 1

2

3

1 1

0

1 2

2

2

Solutions Chapter 5 Review

3 = 1

−1

1 1 2 1 Solution: ( , , ) = (2, 1, 1)

1

1 0

0 1

3 = 1

1 .

1

20. + //////////// = 0 /////// + ///// = 1 /////// + = 0 ////////// = 3 Matrix form: 1 1 0 0 0 1

1 0 0

1 1 0

Solving gives 1

=

0 0 1

0

0

1

1 1 1 0 0

0

0 1

=

0 0

1

1

0

1

1 0 3 −1

1 0 3

=

Solution: ( , , , ) = (1, 1, 2, 2)

1 2

1 2 1 2

1 2 1 2

1 2

1 2

1 2

1 2

0 1

1 2 1 2 1 2

=

1

2 1 3 2

.

1 2

1 2

0 3

1 2

21. Reduce by dominance: Row 1 dominates row 2, so eliminate row 2:

1

2 0 1 3 1

2

2

.

. 0 Row 1 dominates row 2, so we eliminate row 2, leaving just the single entry 1. This means that the optimal strategies are pure strategies, corresponding to the first row and second column: : = 1 0 0 , Now column 2 dominates all the other columns, so eliminate all but column 2:

=

0 1 0 0

T The corresponding expected value is the entry in the first row, second column,

= 1.

22. We begin by trying to reduce by dominance. However, no row dominates any other, and no column dominates any other, so we cannot reduce this game. Since it is larger than 2 × 2, we look to see if it is strictly determined. We circle the row minima and box the column maxima: 3 3 2 1 2

3

2

0 1

.

We do have a saddle point: the 1 in the lower right. So, the game is strictly determined and the optimal strategies are : = 0 0 1 , = 0 0 1 T , with the expected value equal to the saddle point: = 1. 23. We begin by reducing by dominance. The second row dominates the first, so eliminate the first:

1 3 3

Now, the first column dominates the second, so eliminate the second:

0

3 1

.

1 3

0

1

.


Solutions Chapter 5 Review We can't reduce any further, so now we look for the players' optimal mixed strategies. We begin with the row player. Take : = 1 and = 1 0 T . = :- = 4 + 3. If we take : = 1 and =

0 1

T , we get A =

:- = 1. Here are the graphs of and A :

The lower (heavy) portion has its highest point at the intersection of the two lines: 4 + 3 = 1 5 = 4 = 4 5 = 0.8 So the optimal row strategy is : = 0.8 0.2 for the 2 × 2 game, which corresponds to 0 0.8 0.2 for the original game. For the column player we take : = 1 0 and = 1 T , getting = :- = . Taking

:=

0 1 and =

1

T we get A =

:- = 4 1. Here are the graphs of and A :

The upper (heavy) portion has its lowest point at the intersection of the two lines: = 4 1 5 = 1 = 1 5 = 0.2. So, the optimal column strategy is = 0.2 0.8 T for the 2 × 2 game, which corresponds to

0.2 0 0.8 T for the original game. The expected value is the second coordinate of either intersection point above, or the product :- with the optimal strategies. In any case, it is = 0.2. 24. We begin by reducing by dominance. The first row dominates the second, so eliminate the second:

1 4

3

3

2 0 1 2

.

1

3

. 2 1 We can't reduce any further, so now we look for the players' optimal mixed strategies. We begin with the row player. Take : = 1 and = 1 0 T . = :- = + 2. If we take : = 1 and Now, the third column dominates the second and the fourth, so we eliminate both:

=

0 1

T , we get A =

:- = 4 1. Here are the graphs of and A :


Solutions Chapter 5 Review The lower (heavy) portion has its highest point at the intersection of the two lines: + 2 = 4 1 5 = 3 = 3 5 = 0.6 So the optimal row strategy is : = 0.6 0.4 for the 2 × 2 game, which corresponds to 0.6 0 0.4 for the original game. For the column player we take : = 1 0 and = 1 T , getting = :- = 2 + 3.

Taking : =

0 1 and =

T we get A =

1

:- = 3 1. Here are the graphs of and A :

The upper (heavy) portion has its lowest point at the intersection of the two lines: 2 + 3 = 3 1 5 = 4 = 4 5 = 0.8. So, the optimal column strategy is = 0.8 0.2 T for the 2 × 2 game, which corresponds to

0.8 0 0.2 0 T for the original game. The expected value is the second coordinate of either intersection point above, or the product :- with the optimal strategies. In any case, it is = 1.4. 25. (8 ) 1 = 26. (8 ) 1 = 27. (8 )

28. (8 )

1

=

0.7 0.1 0

0.7

0.1

0.3

0.3

0.8

0.5

0.1

0

29. Inventory =

2,500

120,000 70,000

0

0

1,500

0

0.2 0.8

0

5 4

−1

=

−1

0.5

0.1

4,000

3,000

0.1

3,000

0.5

1,000

, and so 0 = (8 ) 1 =

10 7

15 4

=

−1

0

0.1

70,000

0.1

10 7 10 49

=

0.8 0.2 0.2

=

1

−1

5 4

15 4

, and so 0 = (8 ) 1 =

5 4 5 16 25 64 0 0

=

48 23 10 23 2 23

, Sales =

700

6,250 8,750

5 16 , and so 0 = (8 ) =

5 4 0

1,100

5 4

10 23

1

2 23

48,125 22,500 10,000

50 23 10 23 , and so 0 = (8 ) 1 =

10 23 48 23

300 100

500 600

100 200

, Purchases =

400 200

400 400

300 300

To obtain the inventory in the warehouses at the end of June, we subtract the June sales from the June 1 stock (inventory) and add the new books purchased: Inventory Sales + Purchases


=

2,500

4,000

3,000

1,600

2,800

1,100

.

2,600

1,500

3,900

3,000

1,000

3,200

Solutions Chapter 5 Review 300

300 300

=

Purchases each month =

400

400

280

550

100

300

150

100

180

100

500 600

100 200

400 200

+

400 400

30. Change in Inventory = Purchases Sales 400 400 300 300 500 100 100 100 200 = = 200 400 300 100 600 200 100 200 100 31. Inventory on July 1 =

2,600

50

500

Sales each month =

280

1,600 550

3,900 2,800

100 120

3,200 1,100

from Exercise 29

200

400

300 300

To obtain the inventory in the warehouses months after July 1, we subtract times the monthly sales from the July 1 stock and add times the new books purchased: Inventory Sales + Purchases = =

2,600 1,600 2,600 1,600

3,900 2,800 3,900 2,800

3,200 1,100 3,200 1,100

+

50

120

500

150

120

200

+

400 200

400 400

300

;

Nevada Sci Fi inventory is given by the (2,2)-entry: 2 800 + ( 500 + 400) = 2 950 100 , which is zero when = 28 months from July 1, or December 1 of next year. 32. Change in inventory = Purchases Sales=

400 400 300

280 550 100

200 400 300 50 500 120 Texas horror change inventory is given by the (1,3)-entry: (300 100) = 200 , which will reach 1,000 when = 5 months from July 1, or December 1 of this year. 33. Projected July sales figures are Revenue = Quantity × Price =

34.

400 400 300 200 400 300

2

3.5 = 1.5

280 550 100 50

500 120

280 550 100 50

500 120

5 6

.

5.5

=

5,250 Texas

3,910 Nevada

2,650 Texas

2,250 Nevada

35. Unknowns: = Number of shares purchased on July 1, = Number of shares purchased on August 1, = Number of shares purchased on September 1 Given information: Total of 5,000 shares purchased: + + = 5 000 Total of $50,000 invested: 20 + 10 + 5 = 50 000 Total dividends of $300 on shares held as of August 15:


Solutions Chapter 5 Review 0.10( + ) = 300 0.10 + 0.10 = 300 We have three equations in three unknowns: + + = 5 000; 20 + 10 + 5 = 50 000; 0.10 + 0.10 = 300. 1 1 1 5,000 Matrix form: 20 10 5 = 50,000 Solving gives 1 = 20

0.1 0.1 0

1

1

10 5

−1

5,000

300

.5 0.1

5,000

5

50,000 = 0.5 .1

15

1,000

50,000 = 2,000 .

0.1 0.1 0 300 1 0 10 300 2,000 Solution: The company made the following investments: July 1: 1,000 shares, August 1: 2,000 shares, September 1: 2,000 shares

36. Unknowns: = Number of shares purchased on July 1, = Number of shares purchased on August 1, = Number of shares purchased on September 1 Given information: Total of 7,000 shares purchased; + + = 7 000 Total of $150,000 invested: 10 + 20 + 40 = 150 000 Total dividends of $600 on shares held as of July 15: 0.20 = 600 We have three equations in three unknowns: + + = 7 000; 10 + 20 + 40 = 150 000; 0.20 = 600. 1 1 1 7,000 Matrix form: 10 20 40 = 150,000 0.2

0

Solving gives 1 1 1 = 10 20 40

0

−1

600

150,000 =

2

7,000

0

0

5

0.05 7.5

7,000

3,000

150,000 = 2,000 .

0.2 0 0 600 2.5 600 2,000 1 0.05 Solution: The company made the following investments: July 1: 3,000 shares, August 1: 2,000 shares, September 1: 2,000 shares 37. We first need to solve #35: Unknowns: = Number of shares purchased on July 1, = Number of shares purchased on August 1. = Number of shares purchased on September 1 Given information: Total of 5,000 shares purchased: + + = 5 000 Total of $50,000 invested: 20 + 10 + 5 = 50 000 Total dividends of $300 on shares held as of August 15: 0.10( + ) = 300 0.10 + 0.10 = 300 We have three equations in three unknowns: + + = 5 000; 20 + 10 + 5 = 50 000; 0.10 + 0.10 = 300. 1 1 1 5,000 Matrix form: 20 10 5 = 50,000 Solving gives 1 = 20

0.1 0.1 0

1

1

10 5

0.1 0.1 0

−1

5,000

300

.5 0.1

50,000 = 0.5 .1 300

1

0

5 15

10

5,000

1,000

50,000 = 2,000 . 300

2,000


Solutions Chapter 5 Review To answer the current question, let us write the number of shares purchased (calculated in Exercise 35) as a row matrix: Shares Purchased = 1,000 2,000 2,000 .

Then we can calculate the loss by computing the total purchase cost and subtracting the total proceeds (dividends plus selling price): Loss = Number of shares × (Purchase price Dividends Selling price) 20

0.10

3

10 0.10 1

= 1,000 2,000 2,000

5

0

1

= 42,700 .

38. By Exercise 36, the number of stocks purchased (written as a row matrix) is 3,000 2,000 2,000 . Replace this matrix by 6,000 4,000 4,000 to account for the 2-1 split. Then we can calculate the loss by computing the total purchase cost (taking the split into account) and subtracting the total proceeds (dividends plus selling price): Loss = Number of shares × (Purchase price Dividends) Selling price =

5

10

6,000 4,000 4,000

20

0.10 0 0

/(3 000)(20) = $89 400.

39. July 1 customers = 2,000 4,000 4,000

Customers at the end of July = 2,000 4,000 4,000

0.8 0.1 0.1 0.4 0.6 0.2

0

0.8

40. From Exercise 39, August 1 customers = 2,000 4,000 4,000 4,000 2,600 3,400

Customers at the end of August = 4,000 2,600 3,400

=

0

0.8 0.1 0.1 0.4 0.6 0.2

0.8 0.1 0.1 0.4 0.6 0.2

0

4,000 2,600 3,400

0

0.8

0

=

0

0.8

=

4,920 1,960 3,120

41. The matrix shows that no JungleBooks.com customers switched directly to FarmerBooks.com, so the only way to get to FarmerBooks.com is via OHaganBooks.com. 42. Here are three. (1) It is possible for someone to be a customer at two different enterprises. (2) Some customers may stop using all three of the companies. (3) New customers can enter the field. 43. - =

0

30 20

60 40 20 0

= :- =

10 , : = 15

0

30 20

, = 60 40 20 0

10 15

0.4

0.2 , 0.4

0.4

0.2 = 0.4

28 20 14

= 28 + 20 + 14


The largest coefficient is the coefficient of , so we take : =

0 1 0 , meaning that you should go with the "3 for 1" promotion. The resulting effect on your customer base is then = 28(0) + 20(1) + 14(0) = 20, so you will gain 20,000 customers from JungleBooks.com. Solutions Chapter 5 Review

44. - =

0

60 40 20

10 , : =

= :- =

0 0.8 0.2

30 20

0

15

0 0.8 0.2 , = 0

30 20

60 40 20 0

10 15

= 28 + 16 + 11

= 28 16 11

The lowest coefficient is the coefficient of , so we take =

0

0 ,

1 meaning that JungleBooks.com should go with the "3 for 2" promotion. The resulting effect on its customer base is then = 28(0) + 16(0) + 11(1) = 11, so JungleBooks.com will lose 11,000 customers to OHaganBooks.com. 45. Since JungleBooks.com now knows that OHaganBooks.com is assuming it will use the mixed column strategy of Exercise 43, it also knows that OHaganBooks must logically respond by going with the "3 for 1" promotion to counter this. Therefore, its best response will be to use its third strategy "3 for 2" and thus cut its losses to 10,000 customers. But, having seen the e-mail, O'Hagan knows this as well, and so its logical move will be to go with the Finite Math promo and thereby gain 15,000 customers! 46. Since OHaganBooks now knows that JungleBooks.comis assuming it will use the mixed row strategy in Exercise 44, it also knows that JungleBooks.comwill make the logical response of going with "3 for 2." The response of OHaganBooks should then be to go with Finite Math to maximize its gain. Since JungleBooks.comknows this (having seen the correspondence) its logical choice is to go with "2 for 1", resulting in a gain of 0 customers. 47. As neither company is now certain about the strategy of the other, the fundamental principle of game theory comes into effect and so we solve the game to find OHaganBooks.com's optimal minimax strategy. You start by reducing by dominance, which leads to throwing out the "no promotion" options for both players, leaving the following 2 × 2 game: 20 10 -= . 0 15 We need only find your optimal strategy for the row player. Take : = = :- = 20 . If we take : =

the graphs of and A :

1 and =

0 1

1 and =

T T, we get A =

1 0

T.

:- = 5 + 15. Here are


Solutions Chapter 5 Review The lower (heavy) portion has its highest point at the intersection of the two lines: 20 = 5 + 15 25 = 15 = 3 5 = 0.6. So the optimal row strategy is : = 0.6 0.4 for the 2 × 2 game, which corresponds to 0 0.6 0.4 for the original game. The expected value is the height of the point of intersection, = 20(0.6) = 12, corresponding to a gain of 12,000 customers. 48. As neither company is now certain about the strategy of the other, the fundamental principle of game theory comes into effect and so we solve the game using the new payoff matrix to find JungleBooks.com's optimal minimax strategy. We solve the game to find the minimax optimal strategy. Reducing by dominance leads to throwing out the "no promotion" options for both players, leaving the following 2 × 2 game: 20 10 -= . 0 20 We need only find the optimal strategy for the column player. Take = = :- = 20 + 10(1 ) = 10 + 10. With : =

graphs of and A :

1

T and : =

1 0 .

0 1 , we get A = :- = 20 + 20. Here are the

The upper (heavy) portion has its lowest point at the intersection of the two lines: 10 + 10 = 20 + 20 30 = 10 = 1 3. So the optimal column strategy is = 1 3 2 3 for the 2 × 2 game, which corresponds to

0 1 3 2 3 for the original game. The expected value is the height of the point of intersection, = 10(1 3) + 10 . 13.333, corresponding to a gain for O'Hagan books of about 13,333 customers, so a loss for JungleBooks.com of that amount.

49. We obtain the technology matrix by dividing each entry in the input-output table by the total in the last row: 0.1 0.5 = 0.01 0.05 1 0

50. (8 ) 1 =

0 1

0.1

0.5

−1

0.1

0.5

−1

0.01 0.05

=

0.9

.5

−1

0.9

.5

−1

0.01 0.95

19 17 10 17

=

1 85

18 17

The (1, 2)-entry is 10 17 . 0.588. This gives the number of units of Sector 1 that must be produced to meet a one-unit demand for Sector 2 products. In other words, $0.588 worth of paper must be produced in order to meet a $1 increase in the demand for books. 51. First, compute (8 )

1

=

1 0 0 1

0.01 0.05

=

0.01 0.95

=

19 17 10 17 1 85

18 17

We are told that the total (external) demand for Bruno Mills' products is Demand =

.

170

1,700

.


19 17 10 17

170

Solutions Chapter 5 Review

Production = (8 ) 1×Demand =

=

1,190

1 85 18 17 1,700 1,802 Thus, $1,190 worth of paper and $1,802 worth of books must be produced.

52. We are given the Production vector (capacity) and want to compute the associated demand : 0.9 350,000 .5 500,000 = 0 0 = (8 )0 = . = 185,000 0.01 0.95 200,000 Solution: The external demand is $350,000 of paper, $185,000 of books.


Solutions Chapter 5 Case Study Chapter 5 Case Study 1. Employee D has the second-highest rating. While part of the influential group around A, D also serves as the connection to the group G–J. 2. Employee C interacts directly with D as well as A, so is closer to the two highest influencers on the site. 3. The adjacency matrix for this network (with the rows and columns in the order A, B, C, D) is 0 1 1 0 1 0 1 1 = 1 1 0 1 0 1 1 0 Using the power iteration method from the text, we find the following ratings, rounded to 4 decimal places: 0 = [0.2192 0.2808 0.2808 0.2192] Users B and C are tied for being the most influential while users A and D are tied for being the least. 4. The adjacency matrix for this network (with the rows and columns in the order A, B, C, D) is 0 1 1 1 1 0 1 1 = 1 1 0 1 1 1 1 0 Using the power iteration method from the text, we find the following ratings: 0 = [0.25 0.25 0.25 0.25] In fact, this is what we get at each stage if we start with = [0.25/0.25/0.25/0.25]. All users are equally influential. 5. Use the network with the following graph. A B

E

D

C

6. One possibility is the following network. (There are smaller networks possible, with only 6 users.) A

B

E

F

G

C

D

The corresponding ratings are, to four decimal places: 0 = [0.1133 0.2660 0.1527 0.0924 0.1736 0.1416

0.0603]


Solutions Section 6.1 Section 6.1 1. 2𝑥 + 𝑦 ≤ 10 First sketch the graph of 2𝑥 + 𝑦 = 10 (graph on the left).

Choose (0, 0) as a test point: 2(0) + (0) ≤ 10. ✓ Since (0, 0) is in the solution set, we block out the region on the other side of the line as shown above on the right. Since the solution set is not completely enclosed, it is unbounded. 2. 4𝑥 − 𝑦 ≤ 12 First sketch the graph of 4𝑥 − 𝑦 = 12 (graph on the left).

Choose (0, 0) as a test point: 4(0) − (0) ≤ 12. ✓ Since (0, 0) is in the solution set, we block out the region on the other side of the line as shown above on the right. Since the solution set is not completely enclosed, it is unbounded. 3. −𝑥 − 2𝑦 ≤ 8

Choose (0, 0) as a test point: −0 − 2(0) ≤ 8. ✓ Since (0, 0) is in the solution set, we block out the region on the other side of the line as shown above on the right. Since the solution set is not completely enclosed, it is unbounded.


Solutions Section 6.1 4. −𝑥 + 2𝑦 ≥ 4

Choose (0, 0) as a test point: −0 + 2(0) ≥ 4. ✗ Since (0, 0) is not in the solution set, we block out the region on the same side of the line as shown above on the right. Since the solution set is not completely enclosed, it is unbounded. 5. 3𝑥 + 2𝑦 ≥ 5

Choose (0, 0) as a test point: 3(0) + 2(0) ≥ 5. ✗ Since (0, 0) is not in the solution set, we block out the region on the same side of the line as shown above on the right. Since the solution set is not completely enclosed, it is unbounded. 6. 2𝑥 − 3𝑦 ≤ 7

Choose (0, 0) as a test point: 2(0) − 3(0) ≤ 7. ✓ Since (0, 0) is in the solution set, we block out the region on the other side of the line as shown above on the right. Since the solution set is not completely enclosed, it is unbounded. 7. 𝑥 ≤ 3𝑦 To sketch 𝑥 = 3𝑦, solve for 𝑦 to obtain 𝑦 = 13 𝑥. This is a line of slope 13 passing through the origin.


Solutions Section 6.1

As (0, 0) is on the line, we choose another test point—say, (1, 0) : 1 ≤ 3(0). ✗ Since (1, 0) is not in the solution set, we block out the region on the same side of the line as shown above on the right. Since the solution set is not completely enclosed, it is unbounded. 8. 𝑦 ≥ 3𝑥

As (0, 0) is on the line, we choose another test point—say, (1, 0) : 0 ≥ 3(1). ✗ Since (1, 0) is not in the solution set, we block out the region on the same side of the line as shown above on the right. Since the solution set is not completely enclosed, it is unbounded. 3𝑥 𝑦 − ≤1 4 4 To sketch the associated line, replace the inequality by equality and solve for 𝑦 : 𝑦 3𝑥 3𝑥 𝑦 − =1⇒ = − 1 ⇒ 𝑦 = 3𝑥 − 4. 4 4 4 4 9.

3(0) 0 − ≤ 1. ✓ 4 4 Since (0, 0) is in the solution set, we block out the region on the other side of the line as shown above on the right. Since the solution set is not completely enclosed, it is unbounded. Choose (0, 0) as a test point:

𝑥 2𝑦 + ≥2 3 3 To sketch the associated line, replace the inequality by equality and solve for 𝑦 : 10.


Solutions Section 6.1 2𝑦 𝑥 2𝑦 𝑥 1 + =2⇒ = − + 2 ⇒ 𝑦 = − 𝑥 + 3. 3 3 3 3 2

0 2(0) + ≥ 2. ✗ 3 3 Since (0, 0) is not in the solution set, we block out the region on the same side of the line as shown above on the right. Since the solution set is not completely enclosed, it is unbounded. Choose (0, 0) as a test point:

11. 𝑥 ≥ −5 The line 𝑥 = −5 is a vertical line as shown on the left:

Choose (0, 0) as a test point: 0 ≥ −5. ✓ Since (0, 0) is in the solution set, we block out the region on the other side of the line as shown above on the right. Since the solution set is not completely enclosed, it is unbounded. 12. 𝑦 ≤ −4 The line 𝑦 = −4 is the horizontal line shown on the left below:

Choose (0, 0) as a test point: 0 ≤ −4. ✗ Since (0, 0) is not in the solution set, we block out the region on the same side of the line as shown above on the right. Since the solution set is not completely enclosed, it is unbounded. 13. 4𝑥 − 𝑦 ≤ 8 𝑥 + 2𝑦 ≤ 2 The two associated lines are shown below on the left:


Solutions Section 6.1

Choose (0, 0) as a test point for the region 4𝑥 − 𝑦 ≤ 8 : 4(0) − 0 ≤ 8. ✓ Therefore, we shade to the right of the line 4𝑥 − 𝑦 = 8 as shown above center. Choose (0, 0) as a test point for the region 𝑥 + 2𝑦 ≤ 2 : 0 + 2(0) ≤ 2. ✓ Therefore, we shade above the line 𝑥 + 2𝑦 = 2 as shown above on the right. The white region shown is the solution set, which, being not entirely enclosed, is unbounded. For the corner point, we solve the system 4𝑥 − 𝑦 = 8 𝑥 + 2𝑦 = 2. Multiplying the first equation by 2 and adding gives 9𝑥 = 18, so 𝑥 = 2. Substituting 𝑥 = 2 in the first equation gives 8 − 𝑦 = 8, so 𝑦 = 0. Therefore, the corner point is (2, 0). 14. 2𝑥 + 𝑦 ≤ 4 𝑥 − 2𝑦 ≥ 2 The two associated lines are shown below on the left:

Choose (0, 0) as a test point for the region 2𝑥 + 𝑦 ≤ 4 : 2(0) + 0 ≤ 4. ✓ Therefore, we shade to the right of the line 2𝑥 + 𝑦 = 4 as shown above center. Choose (0, 0) as a test point for the region 𝑥 − 2𝑦 ≥ 2 : 0 − 2(0) ≥ 2. ✗ Therefore, we shade above the line 𝑥 − 2𝑦 = 2 as shown above on the right. The white region shown is the solution set, which, being not entirely enclosed, is unbounded. For the corner point, we solve the system 2𝑥 + 𝑦 = 4 𝑥 − 2𝑦 = 2. Multiplying the first equation by 2 and adding gives 5𝑥 = 10, so 𝑥 = 2. Substituting 𝑥 = 2 in the first equation gives 4 + 𝑦 = 4 so 𝑦 = 0. Therefore, the corner point is (2, 0). 15. 3𝑥 + 2𝑦 ≥ 6 3𝑥 − 2𝑦 ≤ 6 𝑥≥0 The three associated lines are shown below on the left. Choose (0, 0) as a test point for the region 3𝑥 + 2𝑦 ≥ 6 : 3(0) + 2(0) ≥ 6. ✗ Therefore, we shade to the left of the line 3𝑥 + 2𝑦 = 6 as shown in the second figure below.


Solutions Section 6.1

Now choose (0, 0) as a test point for the region 3𝑥 − 2𝑦 ≤ 6 : 3(0) − 2(0) ≤ 6. ✓ Therefore, we shade below the line 3𝑥 − 2𝑦 = 6 as shown in the third figure above. Now choose (1, 0) as a test point for the region 𝑥 ≥ 0 : 1 ≥ 0. ✓ Therefore, we shade to the left of 𝑥 = 0 as shown above on the right, leaving the solution set as the unshaded area. Again, the solution set is unbounded, since it is no entirely enclosed. The corner points are shown in the following table: Corner Point Lines through Point Coordinates 𝐴

3𝑥 + 2𝑦 = 6 3𝑥 − 2𝑦 = 6

(2, 0)

𝐵

𝑥=0 3𝑥 + 2𝑦 = 6

(0, 3)

16. 3𝑥 + 2𝑦 ≤ 6 3𝑥 − 2𝑦 ≥ 6 −𝑦 ≥ 2 The three associated lines are shown below on the left. Choose (0, 0) as a test point for the region 3𝑥 + 2𝑦 ≤ 6 : 3(0) + 2(0) ≤ 6. ✓ Therefore, we shade to the right of the line 3𝑥 + 2𝑦 = 6 as shown in the second figure below.


Solutions Section 6.1

Now choose (0, 0) as a test point for the region 3𝑥 − 2𝑦 ≥ 6 : 3(0) − 2(0) ≥ 6. ✗ Therefore, we shade above the line 3𝑥 − 2𝑦 = 6 as shown in the third figure above. Now choose (0, 0) as a test point for the region −𝑦 ≥ 2 : 0 ≥ 2. ✗ Therefore, we shade above 𝑦 = −2 as shown above on the right, leaving the solution set as shown. Again, the solution set is unbounded, since it is not entirely enclosed. The corner points are shown in the following table: Corner Point Lines through Point Coordinates 𝐴

3𝑥 − 2𝑦 = 6 −𝑦 = 2

! 3 , −2"

𝐵

3𝑥 + 2𝑦 = 6 −𝑦 = 2

! 3 , −2"

2

10

17. 𝑥 + 𝑦 ≥ 5 𝑥 ≤ 10 𝑦≤8 𝑥 ≥ 0, 𝑦 ≥ 0 The last two inequalities 𝑥 ≥ 0, 𝑦 ≥ 0 tell us that the solution set is in the first quadrant, so we block out everything else, as shown on the left below:

We then add the lines 𝑥 + 𝑦 = 5, 𝑥 = 10, and 𝑦 = 8 and then shade the appropriate regions, as shown on the right above. The solution set is bounded, since it is entirely enclosed. Corner points: We can easily read these off the graph: 𝐴 : (5, 0), 𝐵 : (10, 0), 𝐶 : (10, 8), 𝐷 : (0, 8), 𝐸 : (0, 5). 18. 2𝑥 + 4𝑦 ≥ 12 𝑥≤5 𝑦≤3 𝑥 ≥ 0, 𝑦 ≥ 0 The last two inequalities 𝑥 ≥ 0, 𝑦 ≥ 0 tell us that the solution set is in the first quadrant, so we block out everything else, as shown on the left below:


Solutions Section 6.1

We then add the lines 2𝑥 + 4𝑦 = 12, 𝑥 = 5, and 𝑦 = 3 and then shade the appropriate regions, as shown on the right above. The solution set is bounded, since it is entirely enclosed. Corner points: We can easily read 𝐴 and 𝐶 off the graph: 𝐴 : (0, 3), 𝐶 : (5, 3). 𝐵 is the intersection of 2𝑥 + 4𝑦 = 12 and 𝑥 = 5, giving 𝐵 : (5, 12 ). 19. 20𝑥 + 10𝑦 ≤ 100 10𝑥 + 20𝑦 ≤ 100 10𝑥 + 10𝑦 ≤ 60 𝑥 ≥ 0, 𝑦 ≥ 0 The last two inequalities 𝑥 ≥ 0, 𝑦 ≥ 0 tell us that the solution set is in the first quadrant, so we block out everything else. We then add the lines 20𝑥 + 10𝑦 = 100, 10𝑥 + 20𝑦 = 100, and 10𝑥 + 10𝑦 = 60 and then shade the appropriate regions, as shown below.

The solution set is the bounded white region. The corner points are shown in the following table: Corner Point Lines through Point Coordinates 𝐴

𝑥 = 0, 𝑦 = 0

(0, 0)

𝐵

𝑦=0 20𝑥 + 10𝑦 = 100

(5, 0)

𝐶

20𝑥 + 10𝑦 = 100 10𝑥 + 10𝑦 = 60

(4, 2)

𝐷

10𝑥 + 10𝑦 = 60 10𝑥 + 20𝑦 = 100

(2, 4)

𝐸

𝑥=0 10𝑥 + 20𝑦 = 100

(0, 5)

20. 30𝑥 + 20𝑦 ≤ 600 10𝑥 + 40𝑦 ≤ 400 20𝑥 + 30𝑦 ≤ 450 𝑥 ≥ 0, 𝑦 ≥ 0 The last two inequalities 𝑥 ≥ 0, 𝑦 ≥ 0 tell us that the solution set is in the first quadrant, so we block out everything else. We then add the lines 30𝑥 + 20𝑦 = 600, 10𝑥 + 40𝑦 = 400, and 120𝑥 + 30𝑦 = 450 and then shade the appropriate regions, as shown below.


Solutions Section 6.1

The solution set is the bounded white region. The corner points are shown in the following table: Corner Point Lines through Point Coordinates 𝐴

𝑥 = 0, 𝑦 = 0

(0, 0)

𝐵

𝑦=0 30𝑥 + 20𝑦 = 600

(20, 0)

𝐶

30𝑥 + 20𝑦 = 600 20𝑥 + 30𝑦 = 450

(18, 3)

𝐷

20𝑥 + 30𝑦 = 450 10𝑥 + 40𝑦 = 400

(12, 7)

𝐸

𝑥=0 10𝑥 + 40𝑦 = 400

(0, 10)

21. 20𝑥 + 10𝑦 ≥ 100 10𝑥 + 20𝑦 ≥ 100 10𝑥 + 10𝑦 ≥ 80 𝑥 ≥ 0, 𝑦 ≥ 0 Proceeding as before, we obtain the unbounded solution set shown below:

The corner points are shown in the following table: Corner Point Lines through Point Coordinates 𝐴

𝑥=0 20𝑥 + 10𝑦 = 100

(0, 10)

𝐵

20𝑥 + 10𝑦 = 100 10𝑥 + 10𝑦 = 80

(2, 6)

𝐶

10𝑥 + 10𝑦 = 80 10𝑥 + 20𝑦 = 100

(6, 2)

𝐷

10𝑥 + 20𝑦 = 100 𝑦=0

(10, 0)

22. 30𝑥 + 20𝑦 ≥ 600 10𝑥 + 40𝑦 ≥ 400 20𝑥 + 30𝑦 ≥ 600 𝑥 ≥ 0, 𝑦 ≥ 0 Proceeding as before, we obtain the unbounded solution set shown below:


Solutions Section 6.1

The corner points are shown in the following table: Corner Point Lines through Point Coordinates 𝐴

𝑥=0 30𝑥 + 20𝑦 = 600

(0, 30)

𝐵

30𝑥 + 20𝑦 = 600 20𝑥 + 30𝑦 = 600

(12, 12)

𝐶

20𝑥 + 30𝑦 = 600 10𝑥 + 40𝑦 = 400

(24, 4)

𝐷

10𝑥 + 40𝑦 = 400 𝑦=0

(40, 0)

23. −3𝑥 + 2𝑦 ≤ 5 3𝑥 − 2𝑦 ≤ 6 Solution set (unbounded):

𝑥 ≤ 2𝑦

𝑥 ≥ 0, 𝑦 ≥ 0

Corner points: We can read two of the corner points, 𝐵 : (0, 0) and 𝐶 : (0, 52 ), directly from the graph. The remaining corner point is 𝐴, at the intersection of the lines 𝑥 = 2𝑦 and 3𝑥 − 2𝑦 = 6. Solving this system of two equations gives the point 𝐴 as 𝐴 : (3, 32 ). 24. −3𝑥 + 2𝑦 ≤ 5 3𝑥 − 2𝑦 ≥ 6 Solution set (unbounded):

𝑦 ≤ 𝑥∕2

𝑥 ≥ 0, 𝑦 ≥ 0

Corner points: We can read off the coordinates of 𝐴 : (2, 0), while 𝐵 is the intersection of 3𝑥 − 2𝑦 = 6 and 𝑦 = 𝑥∕2, giving 𝐵 : (3, 32 ).


Solutions Section 6.1 25. 2𝑥 − 𝑦 ≥ 0 𝑥 − 3𝑦 ≤ 0 𝑥 ≥ 0, 𝑦 ≥ 0 To sketch the lines, solve for 𝑦 in each case: 1 2𝑥 − 𝑦 = 0 ⇒ 𝑦 = 2𝑥 𝑥 − 3𝑦 = 0 ⇒ 𝑦 = 𝑥. 3 Solution set (unbounded):

The only corner point is the origin: (0, 0). 26. −𝑥 + 𝑦 ≥ 0 4𝑥 − 3𝑦 ≥ 0 Solution set (unbounded):

𝑥 ≥ 0, 𝑦 ≥ 0

The only corner point is the origin: (0, 0). 27. To draw the region 2.1𝑥 − 4.3𝑦 ≥ 9.7 using technology, solve the associated equation for 𝑦 : 2.1 8.7 4.3𝑦 = 2.1𝑥 − 9.7 ⇒ 𝑦 = 𝑥− . 4.3 4.3 Solution set (unbounded):

28. To draw the region −4.3𝑥 + 4.6𝑦 ≥ 7.1 using technology, solve the associated equation for 𝑦 : 4.3 7.1 4.6𝑦 = 4.3𝑥 + 7.1 ⇒ 𝑦 = 𝑥+ . 4.6 4.6 Solution set (unbounded):


Solutions Section 6.1

29. To draw the region −0.2𝑥 + 0.7𝑦 ≥ 3.3; 1.1𝑥 + 3.4𝑦 ≥ 0 using technology, solve the associated equations for 𝑦 : 0.2 3.3 2 33 1.1 11 𝑦= 𝑥+ 𝑦=− 𝑥 = − 𝑥. = 𝑥+ 0.7 0.7 7 7 3.4 34

To obtain the coordinates of (the only) corner point, zoom in to it until you can read off the coordinates to two decimal places: (−7.74, 2.50). 30. To draw the region 0.2𝑥 + 0.3𝑦 ≥ 7.2; 2.5𝑥 − 6.7𝑦 ≤ 0 using technology, solve the associated equations for 𝑦 : 2 25 𝑦 = − 𝑥 + 24 𝑦= 𝑥. 3 67

To obtain the coordinates of (the only) corner point, zoom in to it until you can read off the coordinates to two decimal places: (23.08, 8.61). 31. To draw the region 4.1𝑥 − 4.3𝑦 ≤ 4.4; 7.5𝑥 − 4.4𝑦 ≤ 5.7; 4.3𝑥 + 8.5𝑦 ≤ 10 using technology, solve the associated equations for 𝑦 : 4.1 4.4 7.5 5.7 4.3 10 𝑦= 𝑥− 𝑦= 𝑥− 𝑦=− 𝑥+ . 4.3 4.3 4.4 4.4 8.5 4.3


Solutions Section 6.1

To obtain the coordinates of the corner points, zoom in to it until you can read off the coordinates to two decimal places:𝐴 : (0.36, −0.68), 𝐵 : (1.12, 0.61). 32. 2.3𝑥 − 2.4𝑦 ≤ 2.5; 4.0𝑥 − 5.1𝑦 ≤ 4.4; 6.1𝑥 + 6.7𝑦 ≤ 9.6

To obtain the coordinates of the corner points, zoom in to it until you can read off the coordinates to two decimal places:𝐴 : (1.03, −0.06), 𝐵 : (1.32, .23). 33. Unknowns: 𝑥 = Number of quarts of Creamy Vanilla, 𝑦 = Number of quarts of Continental Mocha Arrange the given information in a table with unknowns across the top: Vanilla (𝑥) Mocha (𝑦) Available Eggs

2

1

500

Cream

3

3

900

We can now set up an inequality for each of the items listed on the left: Eggs: Cream:

2𝑥 + 𝑦 ≤ 500 3𝑥 + 3𝑦 ≤ 900 or 𝑥 + 𝑦 ≤ 300

Since the factory cannot manufacture negative amounts, we also have 𝑥 ≥ 0, 𝑦 ≥ 0. The solution and corner points are shown below: Corner Point Lines through Point Coordinates 𝐴

𝑥 = 0, 𝑦 = 0

(0, 0)

𝐵

𝑦=0 2𝑥 + 𝑦 = 500

(250, 0)

𝐶

2𝑥 + 𝑦 = 500 𝑥 + 𝑦 = 300

(200, 100)

𝐷

𝑥 + 𝑦 = 300 𝑥=0

(0, 300)

34. Unknowns: 𝑥 = Number of sections of Finite Math, 𝑦 = Number of sections of Applied Calculus.


Solutions Section 6.1 There are a total of up to 110 sections: 𝑥 + 𝑦 ≤ 110. Information about enrollment: 60𝑥 + 50𝑦 ≤ 6, 000 The numbers of sections cannot be negative: 𝑥 ≥ 0, 𝑦 ≥ 0. The solution and corner points are shown below: Corner Point Lines through Point Coordinates 𝐴

𝑥 = 0, 𝑦 = 0

(0, 0)

𝐵

𝑦=0 60𝑥 + 50𝑦 = 6, 000

(100, 0)

𝐶

60𝑥 + 50𝑦 = 6, 000 𝑥 + 𝑦 = 110

(50, 60)

𝐷

𝑥 + 𝑦 = 110 𝑥=0

(0, 110)

35. Unknowns: 𝑥 = Number of ounces of chicken, 𝑦 = Number of ounces of grain Arrange the given information in a table with unknowns across the top: Chicken (𝑥) Grain (𝑦) Required Protein

10

2

200

Fat

5

2

150

We can now set up an inequality for each of the items listed on the left (note that "at least" is represented by "≥"): Protein: Fat:

10𝑥 + 2𝑦 ≥ 200, or 5𝑥 + 𝑦 ≥ 100 5𝑥 + 2𝑦 ≥ 150

The amounts of ingredients cannot be negative: 𝑥 ≥ 0, 𝑦 ≥ 0. The solution set is the unshaded region shown below: Corner Point Lines through Point Coordinates 𝐴

𝑦=0 5𝑥 + 2𝑦 = 150

(30, 0)

𝐵

5𝑥 + 2𝑦 = 150 5𝑥 + 𝑦 = 100

(10, 50)

𝐶

5𝑥 + 𝑦 = 100 𝑥=0

(0, 100)

36. Unknowns: 𝑥 = Number of Pomegranates, 𝑦 = Number of iZacs Arrange the given information in a table with unknowns across the top: Pomegranates (𝑥) iZacs (𝑦) Required Memory

400

300

48,000

Disc Space

80

100

12,800

We can now set up an inequality for each of the items listed on the left (note that "at least" is represented


Solutions Section 6.1 by "≥"): Memory: Disc Space:

400𝑥 + 300𝑦 ≥ 48, 000 or 4𝑥 + 3𝑦 ≥ 480 80𝑥 + 100𝑦 ≥ 12, 800 or 4𝑥 + 5𝑦 ≥ 640

The values of the unknowns cannot be negative: 𝑥 ≥ 0, 𝑦 ≥ 0. The solution set is the unshaded region shown below: Corner Point Lines through Point Coordinates 𝐴

𝑦=0 4𝑥 + 5𝑦 = 640

(160, 0)

𝐵

4𝑥 + 5𝑦 = 640 4𝑥 + 3𝑦 = 480

(60, 80)

𝐶

4𝑥 + 3𝑦 = 480 𝑥=0

(0, 160)

37. Unknowns: 𝑥 = Number of servings of Multigrain Cereal, 𝑦 = Number of servings of 2nd Foods Mango Arrange the given information in a table with unknowns across the top: Cereal (𝑥) Mango (𝑦) Required Calories

60

80

140

Carbs.

11

19

30

We can now set up an inequality for each of the items listed on the left (note that "at least" is represented by "≥"): Calories: Carbs:

60𝑥 + 80𝑦 ≥ 140 or 3𝑥 + 4𝑦 ≥ 7 11𝑥 + 19𝑦 ≥ 30

The values of the unknowns cannot be negative: 𝑥 ≥ 0, 𝑦 ≥ 0. The solution set is the unshaded region shown below: Corner Point Lines through Point Coordinates 𝐴

𝑦=0 11𝑥 + 19𝑦 = 30

(30∕11, 0)

𝐵

11𝑥 + 19𝑦 = 30 3𝑥 + 4𝑦 = 7

(1, 1)

𝐶

3𝑥 + 4𝑦 = 7 𝑥=0

(0, 7∕4)

38. Unknowns: 𝑥 = Number of servings of Multigrain Cereal, 𝑦 = Number of servings of Custard Pudding Arrange the given information in a table with unknowns across the top:


Solutions Section 6.1 Cereal (𝑥) Pudding (𝑦) Required Calories

60

120

140

Vitamin C

15

45

45

We can now set up an inequality for each of the items listed on the left (note that "at least" is represented by "≥"): Calories: 60𝑥 + 120𝑦 ≥ 140 or 3𝑥 + 6𝑦 ≥ 7 Vit C: 15𝑥 + 45𝑦 ≥ 45 or 𝑥 + 3𝑦 ≥ 3 The values of the unknowns cannot be negative: 𝑥 ≥ 0, 𝑦 ≥ 0. The solution set is the unshaded region shown below: Corner Point Lines through Point Coordinates 𝐴

3𝑥 + 6𝑦 = 7 𝑥=0

(0, 7∕6)

𝐵

3𝑥 + 6𝑦 = 7 𝑥 + 3𝑦 = 3

(1, 2∕3)

𝐶

𝑥 + 3𝑦 = 3 𝑦=0

(3, 0)

39. Unknowns: 𝑥 = Number of dollars in MHI, 𝑦 = Number of dollars in GNMVX Total invested is up to $110,000: 𝑥 + 𝑦 ≤ 110, 000. Interest earned is at least $2,400: 0.04𝑥 + 0.015𝑦 ≥ 2,400 or 8𝑥 + 3𝑦 ≥ 480,000. The values of the unknowns cannot be negative: 𝑥 ≥ 0, 𝑦 ≥ 0. The solution set is the unshaded region shown below: Corner Point Lines through Point

Coordinates

𝐴

𝑦=0 8𝑥 + 3𝑦 = 480,000

(60,000, 0)

𝐵

𝑦=0 𝑥 + 𝑦 = 110,000

(110,000, 0)

𝐶

𝑥 + 𝑦 = 110,000 8𝑥 + 3𝑦 = 480,000

(30,000, 80,000)

40. Unknowns: 𝑥 = Number of dollars in FNORX, 𝑦 = Number of dollars in PRGTX Total invested is up to $60,000: 𝑥 + 𝑦 ≤ 60, 000. Investment grows by at least $6,400: 0.08𝑥 + 0.16𝑦 ≥ 6,400 or 𝑥 + 2𝑦 ≥ 80,000. The values of the unknowns cannot be negative: 𝑥 ≥ 0, 𝑦 ≥ 0. The solution set is the unshaded region shown below:


Solutions Section 6.1 Corner Point Lines through Point

Coordinates

𝐴

𝑥=0 𝑥 + 𝑦 = 60,000

(0, 60,000)

𝐵

𝑥=0 𝑥 + 2𝑦 = 80,000

(0, 40,000)

𝐶

𝑥 + 𝑦 = 60, 000 𝑥 + 2𝑦 = 80,000

(40,000, 20,000)

41. Unknowns: 𝑥 = Number of full-page ads in Sports Illustrated, 𝑦 = Number of full-page ads in GQ Readership: 0.9𝑥 + 0.5𝑦 ≥ 5 At least 3 full-page ads in each magazine: 𝑥 ≥ 3, 𝑦 ≥ 3 The values of the unknowns cannot be negative: 𝑥 ≥ 0, 𝑦 ≥ 0. The solution set is the unshaded region shown below: Corner Point Lines through Point

Coordinates (rounded)

𝐴

𝑦=3 0.9𝑥 + 0.5𝑦 = 5

(4, 3)

𝐵

𝑥=3 0.9𝑥 + 0.5𝑦 = 5

(3, 5)

42. Unknowns: 𝑥 = Number of full-page ads in Sports Illustrated, 𝑦 = Number of full-page ads in Motor Trend Readership: 0.9𝑥 + 0.3𝑦 ≥ 6 At least 4 full-page ads in each magazine: 𝑥 ≥ 4, 𝑦 ≥ 4 The values of the unknowns cannot be negative: 𝑥 ≥ 0, 𝑦 ≥ 0. The solution set is the unshaded region shown below: Corner Point Lines through Point Coordinates (rounded) 𝐴

𝑦=4 0.9𝑥 + 0.3𝑦 = 6

(5, 4)

𝐵

𝑥=4 0.9𝑥 + 0.3𝑦 = 6

(4, 8)

43. Unknowns: 𝑥 = Number of shares of TD; 𝑦 = Number of shares of CNA You have up to $25,800 to invest: 66𝑥 + 42𝑦 ≤ 25,800 or 11𝑥 + 7𝑦 ≤ 4,300. You wish to earn at least $969 in dividends: TD dividend = 4% of 66𝑥 invested = 0.04(66𝑥) = 2.64𝑥 CNA dividend = 3.5% of 42𝑦 invested = 0.035(42𝑦) = 1.47𝑦. Thus, 2.64𝑥 + 1.47𝑦 ≥ 969 or 88𝑥 + 49𝑦 ≥ 32,300. The values of the unknowns cannot be negative: 𝑥 ≥ 0, 𝑦 ≥ 0. The solution set is the unshaded region shown below:


Solutions Section 6.1 Corner Point Lines through Point

Coordinates (rounded)

𝐴

11𝑥 + 7𝑦 = 4,300 88𝑥 + 49𝑦 = 32,300

(200, 300)

𝐵

11𝑥 + 7𝑦 = 4,300 𝑦=0

(391, 0)

𝐶

88𝑥 + 49𝑦 = 32,300 𝑦=0

(367, 0)

44. Unknowns: 𝑥 = Number of shares of PAA; 𝑦 = Number of shares of CVX You have up to $30,000 to invest: 10𝑥 + 100𝑦 ≤ 30,000 or 𝑥 + 10𝑦 ≤ 3,000. You wish to earn at least $1,700 in dividends: PAA dividend = 7% of 10𝑥 invested = 0.07(10𝑥) = 0.7𝑥 CVX dividend = 5% of 100𝑦 invested = 0.05(100𝑦) = 5𝑦. Thus, 0.7𝑥 + 5𝑦 ≥ 1,700 or 7𝑥 + 50𝑦 ≥ 17,000. The values of the unknowns cannot be negative: 𝑥 ≥ 0, 𝑦 ≥ 0. The solution set is the unshaded region shown below: Corner Point Lines through Point

Coordinates (rounded)

𝐴

𝑥 + 10𝑦 = 3,000 7𝑥 + 50𝑦 = 17,000

(1,000, 200)

𝐵

𝑥 + 10𝑦 = 3,000 𝑦=0

(3,000, 0)

𝐶

7𝑥 + 50𝑦 = 17,000 𝑦=0

(2,429, 0)

45. Many of the systems of inequalities in the earlier exercises have unbounded solution sets. Another example is: 𝑥 ≥ 0, 𝑦 ≥ 0, 𝑥 + 𝑦 ≥ 1. 46. An example is 𝑥 + 𝑦 ≥ 5, 𝑥 + 𝑦 ≤ 3. 47. The given triangle is the region enclosed by the lines 𝑥 = 0, 𝑦 = 0, and 𝑥 + 2𝑦 = 2 (see figure).

Thus, the region can be described as the solution set of the system 𝑥 ≥ 0, 𝑦 ≥ 0, 𝑥 + 2𝑦 ≤ 2. 48. If one shades the regions of points that do satisfy the inequalities, then the feasible region for more than one inequality will correspond to the overlap of the shaded regions. This becomes difficult when there are several inequalities, as in Exercises 23 and 24.


Solutions Section 6.1 49. (Answers may vary.) One limitation is that the method is suitable only for situations with two unknown quantities. Another limitation is accuracy, which is never perfect when graphing. 50. The greatest value of 𝑥 + 𝑦 always occurs at a corner point of the region (or everywhere on a line segment joining two corner points). 51. There should be at least 3 more grams of ingredient A than ingredient B. Rephrasing this statement gives: The number of grams of ingredient A exceeds the grams of ingredient B by 3: 𝑥 − 𝑦 ≥ 3 — Choice (C). 52. 𝑥 ≥ 0.25(𝑥 + 𝑦) ⇒ 4𝑥 ≥ 𝑥 + 𝑦 ⇒ 3𝑥 − 𝑦 ≥ 0 — Choice (D) 53. There should be at least 3 parts (by weight) of ingredient A to 2 parts of ingredient B. That is, 3∕2 = 1.5 parts of ingredient A to 1 part of ingredient B. Rephrasing this statement gives: The number of grams of ingredient A is 1.5 times the number of grams of ingredient B: 𝑥 ≥ 1.5𝑦 ⇒ 2𝑥 ≥ 3𝑦 ⇒ 2𝑥 − 3𝑦 ≥ 0 — Choice (B). 54. 𝑥 ≤ 𝑦 ⇒ 𝑥 − 𝑦 ≤ 0 — Choice (B) 55. There are no feasible solutions; that is, it is impossible to satisfy all the constraints. 56. No. the fact that the total number of tickets cannot exceed the capacity of the flight leads to inequalities of the form 𝑥 + 𝑦 ≤ capacity, 𝑥 ≥ 0, 𝑦 ≥ 0. This is always bounded. 57. Answers will vary. 58. Answers will vary.


Solutions Section 6.2 Section 6.2 1. Maximize 𝑝 = 𝑥 + 𝑦 subject to 𝑥 + 2𝑦 ≤ 9, 2𝑥 + 𝑦 ≤ 9, 𝑥 ≥ 0, 𝑦 ≥ 0. Corner Point Lines through Point Coordinates 𝑝 = 𝑥 + 𝑦 𝐴

𝑥 = 0, 𝑦 = 0

(0, 0)

0

𝐵

𝑦=0 2𝑥 + 𝑦 = 9

(4.5, 0)

4.5

𝐶

2𝑥 + 𝑦 = 9 𝑥 + 2𝑦 = 9

(3, 3)

6

𝐷

𝑥 + 2𝑦 = 9 𝑥=0

(0, 4.5)

4.5

Maximum value occurs at point 𝐶 : 𝑝 = 6 for 𝑥 = 3, 𝑦 = 3. 2. Maximize 𝑝 = 𝑥 + 2𝑦 subject to 𝑥 + 3𝑦 ≤ 24, 2𝑥 + 𝑦 ≤ 18, 𝑥 ≥ 0, 𝑦 ≥ 0. Corner Point Lines through Point Coordinates 𝑝 = 𝑥 + 2𝑦 𝐴

𝑥 = 0, 𝑦 = 0

(0, 0)

0

𝐵

𝑦=0 2𝑥 + 𝑦 = 18

(9, 0)

9

𝐶

2𝑥 + 𝑦 = 18 𝑥 + 3𝑦 = 24

(6, 6)

18

𝐷

𝑥 + 3𝑦 = 24 𝑥=0

(0, 8)

16

Maximum value occurs at point 𝐶 : 𝑝 = 18, 𝑥 = 6, 𝑦 = 6. 3. Minimize 𝑐 = 𝑥 + 𝑦 subject to 𝑥 + 2𝑦 ≥ 6, 2𝑥 + 𝑦 ≥ 6, 𝑥 ≥ 0, 𝑦 ≥ 0. Corner Point Lines through Point Coordinates 𝑐 = 𝑥 + 𝑦 𝐴

𝑦=0 𝑥 + 2𝑦 = 6

(6, 0)

6

𝐵

𝑥 + 2𝑦 = 6 2𝑥 + 𝑦 = 6

(2, 2)

4

𝐶

2𝑥 + 𝑦 = 6 𝑥=0

(0, 6)

6

Although the feasible region is unbounded, there is no need to add a bounding rectangle since, by the FAQ at the end of the section in the text: If you are minimizing 𝑐 = 𝑎𝑥 + 𝑏𝑦 with 𝑎 and 𝑏 nonnegative, 𝑥 ≥ 0, and 𝑦 ≥ 0, then optimal solutions always exist. Minimum value occurs at point 𝐵 : 𝑐 = 4, 𝑥 = 2, 𝑦 = 2.


Solutions Section 6.2 4. Minimize 𝑐 = 𝑥 + 2𝑦 subject to 𝑥 + 3𝑦 ≥ 30, 2𝑥 + 𝑦 ≥ 30, 𝑥 ≥ 0, 𝑦 ≥ 0. Corner Point Lines through Point Coordinates 𝑐 = 𝑥 + 2𝑦 𝐴

𝑦=0 𝑥 + 3𝑦 = 30

(30, 0)

30

𝐵

𝑥 + 3𝑦 = 30 2𝑥 + 𝑦 = 30

(12, 6)

24

𝐶

2𝑥 + 𝑦 = 30 𝑥=0

(0, 30)

60

Although the feasible region is unbounded, there is no need to add a bounding rectangle since, by the FAQ at the end of the section in the text: If you are minimizing 𝑐 = 𝑎𝑥 + 𝑏𝑦 with 𝑎 and 𝑏 nonnegative, 𝑥 ≥ 0, and 𝑦 ≥ 0, then optimal solutions always exist. Minimum value occurs at point 𝐵 : 𝑐 = 24, 𝑥 = 12, 𝑦 = 6. 5. Maximize 𝑝 = 3𝑥 + 𝑦 subject to 3𝑥 − 7𝑦 ≤ 0, 7𝑥 − 3𝑦 ≥ 0, 𝑥 + 𝑦 ≤ 10, 𝑥 ≥ 0, 𝑦 ≥ 0. Corner Point Lines through Point Coordinates 𝑝 = 3𝑥 + 𝑦 𝐴

𝑥 = 0, 𝑦 = 0

(0, 0)

0

𝐵

3𝑥 − 7𝑦 = 0 𝑥 + 𝑦 = 10

(7, 3)

24

𝐶

7𝑥 − 3𝑦 = 0 𝑥 + 𝑦 = 10

(3, 7)

16

Maximum value occurs at point 𝐵 : 𝑝 = 24, 𝑥 = 7, 𝑦 = 3. 6. Maximize 𝑝 = 𝑥 − 2𝑦 subject to 𝑥 + 2𝑦 ≤ 8, 𝑥 − 6𝑦 ≤ 0, 3𝑥 − 2𝑦 ≥ 0, 𝑥 ≥ 0, 𝑦 ≥ 0. Corner Point Lines through Point Coordinates 𝑝 = 𝑥 − 2𝑦 𝐴

𝑥 = 0, 𝑦 = 0

(0, 0)

0

𝐵

𝑥 − 6𝑦 = 0 𝑥 + 2𝑦 = 8

(6, 1)

4

𝐶

3𝑥 − 2𝑦 = 0 𝑥 + 2𝑦 = 8

(2, 3)

−4

Maximum value occurs at point 𝐵 : 𝑝 = 4, 𝑥 = 6, 𝑦 = 1.


Solutions Section 6.2 7. Maximize 𝑝 = 3𝑥 + 2𝑦 subject to 0.2𝑥 + 0.1𝑦 ≤ 1, 0.15𝑥 + 0.3𝑦 ≤ 1.5, 10𝑥 + 10𝑦 ≤ 60, 𝑥 ≥ 0, 𝑦 ≥ 0. Corner Point Lines through Point Coordinates 𝑝 = 3𝑥 + 2𝑦 𝐴

𝑥 = 0, 𝑦 = 0

(0, 0)

0

𝐵

𝑦=0 0.2𝑥 + 0.1𝑦 = 1

(5, 0)

15

𝐶

0.2𝑥 + 0.1𝑦 = 1 10𝑥 + 10𝑦 = 60

(4, 2)

16

𝐷

10𝑥 + 10𝑦 = 60 0.15𝑥 + 0.3𝑦 = 1.5

(2, 4)

14

𝐸

0.15𝑥 + 0.3𝑦 = 1.5 𝑥=0

(0, 5)

10

Maximum value occurs at point 𝐶 : 𝑝 = 16, 𝑥 = 4, 𝑦 = 2. 8. Maximize 𝑝 = 𝑥 + 2𝑦 subject to 30𝑥 + 20𝑦 ≤ 600, 0.1𝑥 + 0.4𝑦 ≤ 4, 0.2𝑥 + 0.3𝑦 ≤ 4.5, 𝑥 ≥ 0, 𝑦 ≥ 0. Corner Point Lines through Point Coordinates 𝑝 = 𝑥 + 2𝑦 𝐴

𝑥 = 0, 𝑦 = 0

(0, 0)

0

𝐵

𝑦=0 30𝑥 + 20𝑦 = 600

(20, 0)

20

𝐶

30𝑥 + 20𝑦 = 600 0.2𝑥 + 0.3𝑦 = 4.5

(18, 3)

24

𝐷

0.2𝑥 + 0.3𝑦 = 4.5 0.1𝑥 + 0.4𝑦 = 4

(12, 7)

26

𝐸

0.1𝑥 + 0.4𝑦 = 4 𝑥=0

(0, 10)

20

Maximum value occurs at point 𝐷 : 𝑝 = 26, 𝑥 = 12, 𝑦 = 7. 9. Minimize 𝑐 = 0.2𝑥 + 0.3𝑦 subject to 0.2𝑥 + 0.1𝑦 ≥ 1, 0.15𝑥 + 0.3𝑦 ≥ 1.5, 10𝑥 + 10𝑦 ≥ 80, 𝑥 ≥ 0, 𝑦 ≥ 0. Corner Point Lines through Point Coordinates 𝑐 = 0.2𝑥 + 0.3𝑦 𝐴

𝑦=0 0.15𝑥 + 0.3𝑦 = 1.5

(10, 0)

2

𝐵

0.15𝑥 + 0.3𝑦 = 1.5 10𝑥 + 10𝑦 = 80

(6, 2)

1.8

𝐶

10𝑥 + 10𝑦 = 80 0.2𝑥 + 0.1𝑦 = 1

(2, 6)

2.2

𝐷

0.2𝑥 + 0.1𝑦 = 1 𝑥=0

(0, 10)

3

Although the feasible region is unbounded, there is no need to add a bounding rectangle as the coefficients of 𝑐 are nonnegative. Maximum value occurs at point 𝐵 : 𝑐 = 1.8, 𝑥 = 6, 𝑦 = 2.


Solutions Section 6.2 10. Minimize 𝑐 = 0.4𝑥 + 0.1𝑦 subject to 30𝑥 + 20𝑦 ≥ 600, 0.1𝑥 + 0.4𝑦 ≥ 4, 0.2𝑥 + 0.3𝑦 ≥ 4.5, 𝑥 ≥ 0, 𝑦 ≥ 0. Corner Point Lines through Point Coordinates 𝑐 = 0.4𝑥 + 0.1𝑦 𝐴

𝑦=0 0.1𝑥 + 0.4𝑦 = 4

(40, 0)

16

𝐵

0.1𝑥 + 0.4𝑦 = 4 30𝑥 + 20𝑦 = 600

(16, 6)

7

𝐶

30𝑥 + 20𝑦 = 600 𝑥=0

(0, 30)

3

Although the feasible region is unbounded, there is no need to add a bounding rectangle as the coefficients of 𝑐 are nonnegative. Maximum value occurs at point 𝐶 : 𝑐 = 3, 𝑥 = 0, 𝑦 = 30. 11. Maximize and minimize 𝑝 = 𝑥 + 2𝑦 subject to 𝑥 + 𝑦 ≥ 2, 𝑥 + 𝑦 ≤ 10, 𝑥 − 𝑦 ≤ 2, 𝑥 − 𝑦 ≥ −2. Corner Point Lines through Point Coordinates 𝑝 = 𝑥 + 2𝑦 𝐴

𝑥+𝑦=2 𝑥−𝑦=2

(2, 0)

2 (Min)

𝐵

𝑥−𝑦=2 𝑥 + 𝑦 = 10

(6, 4)

14

𝐶

𝑥 + 𝑦 = 10 𝑥 − 𝑦 = −2

(4, 6)

16 (Max)

𝐷

𝑥 − 𝑦 = −2 𝑥+𝑦=2

(0, 2)

4

Maximum value occurs at point 𝐶 : 𝑝 = 16, 𝑥 = 4, 𝑦 = 6; minimum value occurs at point 𝐴 : 𝑝 = 2, 𝑥 = 2, 𝑦 = 0. 12. Maximize and minimize 𝑝 = 2𝑥 − 𝑦 subject to 𝑥 + 𝑦 ≥ 2, 𝑥 − 𝑦 ≤ 2, 𝑥 − 𝑦 ≥ −2, 𝑥 ≤ 10, 𝑦 ≤ 10. Corner Point Lines through Point Coordinates 𝑝 = 2𝑥 − 𝑦 𝐴

𝑥+𝑦=2 𝑥−𝑦=2

(2, 0)

4

𝐵

𝑥−𝑦=2 𝑥 = 10

(10, 8)

12 (Max)

𝐶

𝑥 = 10, 𝑦 = 10

(10, 10)

10

𝐷

𝑦 = 10 𝑥 − 𝑦 = −2

(8, 10)

6

𝐸

𝑥=0 𝑥+𝑦=2

(0, 2)

−2 (Min)

Maximum value occurs at point 𝐵 : 𝑝 = 12, 𝑥 = 10, 𝑦 = 8; minimum value occurs at point 𝐸 : 𝑝 = −2, 𝑥 = 0, 𝑦 = 2.


Solutions Section 6.2 13. Maximize 𝑝 = 2𝑥 − 𝑦 subject to 𝑥 + 2𝑦 ≥ 6, 𝑥 ≤ 8, 𝑥 ≥ 0, 𝑦 ≥ 0. The feasible region is unbounded, and the problem does not satisfy any of the conditions in the FAQ at the end of the text for this section, so we need to add a bounding rectangle, adding two new corner points to the feasible region as shown: y

y 4 E

4 3

A

3

𝑥 + 2𝑦 = 6

𝑥=8

𝑥 + 2𝑦 = 6

B C 6

D

A

8

𝑥=8

B C

x

6

x

8

Corner Point Lines through Point Coordinates 𝑝 = 2𝑥 − 𝑦 𝐴

𝑥 + 2𝑦 = 6 𝑥=0

(0, 3)

−3

𝐵

𝑥 + 2𝑦 = 6 𝑦=0

(6, 0)

12

𝐶

𝑥 = 8, 𝑦 = 0

(8, 0)

16 (Max)

𝐷

𝑥 = 8, 𝑦 = 4

(8, 4)

12

𝐸

𝑥 = 0, 𝑦 = 4

(0, 4)

−4

The maximum occurs at the point 𝐶, which is a corner point of the original feasible region, so the LP problem has an optimal solution: 𝑝 = 16, 𝑥 = 8, 𝑦 = 0. 14. Maximize 𝑝 = 𝑥 − 3𝑦 subject to 2𝑥 + 𝑦 ≥ 4, 𝑦 ≤ 5, 𝑥 ≥ 0, 𝑦 ≥ 0. The feasible region is unbounded, and the problem does not satisfy any of the conditions in the FAQ at the end of the text for this section, so we need to add a bounding rectangle, adding two new corner points to the feasible region as shown: y

5

A 4B

y 𝑦=5

5

A 4B

2𝑥 + 𝑦 = 4

𝑦=5

E

2𝑥 + 𝑦 = 4

C 2

x

C 2

D 3

x


Solutions Section 6.2 Corner Point Lines through Point Coordinates 𝑝 = 𝑥 − 3𝑦 𝐴

𝑥 = 0, 𝑦 = 5

(0, 5)

−15

𝐵

2𝑥 + 𝑦 = 4 𝑥=0

(0, 4)

−12

𝐶

2𝑥 + 𝑦 = 4 𝑦=0

(2, 0)

2

𝐷

𝑥 = 3, 𝑦 = 0

(3, 0)

3 (Max)

𝐸

𝑥 = 3, 𝑦 = 5

(3, 5)

−12

The maximum occurs only at the point 𝐷, which is not a corner point of the original feasible region, so the LP problem has no optimal solution. (The value of the objective function is unbounded.) 15. Maximize 𝑝 = 2𝑥 + 3𝑦 subject to 0.1𝑥 + 0.2𝑦 ≥ 1, 2𝑥 + 𝑦 ≥ 10, 𝑥 ≥ 0, 𝑦 ≥ 0. This is an unbounded region, and we wish to maximize 𝑝 = 2𝑥 + 3𝑦. According to the FAQ at the end of the text for this section, if the feasible region is unbounded: If you are maximizing 𝑝 = 𝑎𝑥 + 𝑏𝑦 with a and both positive, then there is no optimal solution. Thus, there is no optimal solution.

16. Maximize 𝑝 = 3𝑥 + 2𝑦 subject to 0.1𝑥 + 0.1𝑦 ≥ 0.2, 𝑦 ≤ 10, 𝑥 ≥ 0, 𝑦 ≥ 0. This is an unbounded region, and we wish to maximize 𝑝 = 3𝑥 + 2𝑦. According to the FAQ at the end of the text for this section if the feasible region is unbounded: If you are maximizing 𝑝 = 𝑎𝑥 + 𝑏𝑦 with a and both positive, then there is no optimal solution. Thus, there is no optimal solution.

17. Minimize 𝑐 = 𝑥 − 3𝑦 subject to 3𝑥 + 𝑦 ≥ 5, 2𝑥 − 𝑦 ≥ 0, 𝑥 − 3𝑦 ≤ 0, 𝑥 ≥ 0, 𝑦 ≥ 0. The feasible region is unbounded, and the problem does not satisfy any of the conditions in the FAQ at the end of the text for this section, so we need to add a bounding rectangle, adding two new corner points to the feasible region as shown: y

y 2𝑥 − 𝑦 = 0

2𝑥 − 𝑦 = 0 4

C

𝑦 + 3𝑥 = 5

𝑦 + 3𝑥 = 5

B

B A

𝑥 − 3𝑦 = 0

A

x

D

𝑥 − 3𝑦 = 0

2

x


Solutions Section 6.2 Corner Point Lines through Point Coordinates 𝑐 = 𝑥 − 3𝑦 𝐴

3𝑥 + 𝑦 = 5 𝑥 − 3𝑦 = 0

(3∕2, 1∕2)

0

𝐵

3𝑥 + 𝑦 = 5 2𝑥 − 𝑦 = 0

(1, 2)

−5

𝐶

𝑥=2 2𝑥 − 𝑦 = 0

(2, 4)

−10 (Min)

𝐷

𝑥=2 𝑥 − 3𝑦 = 0

(2, 2∕3)

0

The minimum occurs only at the point 𝐶, which is not a corner point of the original feasible region, so the LP problem has no optimal solution. (The value of the objective function is unbounded.) 18. Minimize 𝑐 = 3𝑥 − 𝑦 subject to 2𝑥 − 𝑦 ≥ 3, 𝑥 − 𝑦 ≥ 0, 𝑥 − 2𝑦 ≤ 0, 𝑥 ≥ 0, 𝑦 ≥ 0. The feasible region is unbounded, and the problem does not satisfy any of the conditions in the FAQ at the end of the text for this section, so we need to add a bounding rectangle, adding two new corner points to the feasible region as shown: y

y 4

2𝑥 − 𝑦 = 3

2𝑥 − 𝑦 = 3

B 𝑥−𝑦=0

D

𝑥−𝑦=0

A

C

B

A

𝑥 − 2𝑦 = 0

𝑥 − 2𝑦 = 0

x

4

x

Corner Point Lines through Point Coordinates 𝑐 = 3𝑥 − 𝑦 𝐴

2𝑥 − 𝑦 = 3 𝑥 − 2𝑦 = 0

(2, 1)

5 (Min)

𝐵

2𝑥 − 𝑦 = 3 𝑥−𝑦=0

(3, 3)

6

𝐶

𝑥=4 𝑥−𝑦=0

(4, 4)

8

𝐷

𝑥=4 𝑥 − 2𝑦 = 0

(4, 2)

10

The minimum occurs at the point 𝐴, which is a corner point of the original feasible region, so the LP problem has an optimal solution: 𝑐 = 5, 𝑥 = 2, 𝑦 = 1.


Solutions Section 6.2 19. Minimize 𝑐 = 2𝑥 + 4𝑦 subject to 0.1𝑥 + 0.1𝑦 ≥ 1, 𝑥 + 2𝑦 ≥ 14, 𝑥 ≥ 0, 𝑦 ≥ 0. Corner Point Lines through Point Coordinates 𝑐 = 2𝑥 + 4𝑦 𝐴

𝑦=0 𝑥 + 2𝑦 = 14

(14, 0)

28

𝐵

𝑥 + 2𝑦 = 14 0.1𝑥 + 0.1𝑦 = 1

(6, 4)

28

𝐶

0.1𝑥 + 0.1𝑦 = 1 𝑥=0

(0, 10)

40

Although the feasible region is unbounded, there is no need to add a bounding rectangle since the coefficients of 𝑐 are nonnegative. Minimum value occurs at points 𝐴 and 𝐵 : 𝑐 = 28; (𝑥, 𝑦) = (14, 0) and (6, 4), and the line connecting them. 20. Maximize 𝑝 = 2𝑥 + 3𝑦 subject to −𝑥 + 𝑦 ≥ 10, 𝑥 + 2𝑦 ≤ 12, 𝑥 ≥ 0, 𝑦 ≥ 0.

Feasible region is empty—no solutions.

21. Minimize 𝑐 = 3𝑥 − 3𝑦 subject to 𝑥4 ≤ 𝑦, 𝑦 ≤ 2𝑥 , 𝑥 + 𝑦 ≥ 5, 𝑥 + 2𝑦 ≤ 10, 𝑥 ≥ 0, 𝑦 ≥ 0. 3 Corner Point Lines through Point Coordinates 𝑐 = 3𝑥 − 3𝑦 𝐴

𝑦 = 𝑥∕4 𝑥+𝑦=5

(4, 1)

9

𝐵

𝑦 = 𝑥∕4 𝑥 + 2𝑦 = 10

(20∕3, 5∕3)

15

𝐶

𝑥 + 2𝑦 = 10 𝑦 = 2𝑥∕3

(30∕7, 20∕7)

30∕7

𝐷

𝑦 = 2𝑥∕3 𝑥+𝑦=5

(3, 2)

3

Minimum value occurs at point 𝐷 : 𝑐 = 3, 𝑥 = 3, 𝑦 = 2.


Solutions Section 6.2 22. Minimize 𝑐 = −𝑥 + 2𝑦 subject to 𝑦 ≤ 2𝑥 , 𝑥 ≤ 3𝑦, 𝑦 ≥ 4.𝑥 ≥ 6, 𝑥 + 𝑦 ≤ 16. 3 Corner Point Lines through Point Coordinates 𝑐 = −𝑥 + 2𝑦 𝐴

𝑦 = 𝑥∕3 𝑥 + 𝑦 = 16

(12, 4)

−4

𝐵

𝑦 = 2𝑥∕3 𝑥 + 𝑦 = 16

(48∕5, 32∕5)

16∕5

𝐶

𝑥=6 𝑦=4

(6, 4)

2

Minimum value occurs at point 𝐴 : 𝑐 = −4, 𝑥 = 12, 𝑦 = 4. 23. Maximize 𝑝 = 𝑥 + 𝑦 subject to 𝑥 + 2𝑦 ≥ 10, 2𝑥 + 2𝑦 ≤ 10, 2𝑥 + 𝑦 ≥ 10, 𝑥 ≥ 0, 𝑦 ≥ 0.

Feasible region is empty—no solutions.

24. Minimize 𝑐 = 3𝑥 + 𝑦 subject to 10𝑥 + 20𝑦 ≥ 100, 0.3𝑥 + 0.1𝑦 ≥ 1, 𝑥 ≥ 0, 𝑦 ≥ 0. Corner Point Lines through Point Coordinates 𝑐 = 3𝑥 + 𝑦 𝐴

𝑦=0 10𝑥 + 20𝑦 = 100

(10, 0)

30

𝐵

10𝑥 + 20𝑦 = 100 0.3𝑥 + 0.1𝑦 = 1

(2, 4)

10

𝐶

0.3𝑥 + 0.1𝑦 = 1 𝑥=0

(0, 10)

10

Minimum values occur at points 𝐵 and 𝐶 : 𝑐 = 10; (𝑥, 𝑦) = (2, 4), (0, 10), and the line connecting them. 25. (To set up the inequalities, see the solution to #33 in the preceding section.) Unknowns: 𝑥 = # quarts of Creamy Vanilla, 𝑦 = # quarts of Continental Mocha Maximize 𝑝 = 3𝑥 + 2𝑦 subject to 2𝑥 + 𝑦 ≤ 500, 3𝑥 + 3𝑦 ≤ 900 (or 𝑥 + 𝑦 ≤ 300), 𝑥 ≥ 0, 𝑦 ≥ 0.


Solutions Section 6.2 Corner Point Lines through Point Coordinates 𝑝 = 3𝑥 + 2𝑦 𝐴

𝑥 = 0, 𝑦 = 0

(0, 0)

0

𝐵

𝑦=0 2𝑥 + 𝑦 = 500

(250, 0)

750

𝐶

2𝑥 + 𝑦 = 500 𝑥 + 𝑦 = 300

(200, 100)

800

𝐷

𝑥 + 𝑦 = 300 𝑥=0

(0, 300)

600

Maximum value occurs at the point 𝐶 : 𝑝 = 800; 𝑥 = 200, 𝑦 = 100. Solution: You should make 200 quarts of Creamy Vanilla and 100 quarts of Continental Mocha. 26. (To set up the inequalities, see the solution to #34 in the preceding section.) Unknowns: 𝑥 = # sections of Finite Math, 𝑦 = # sections of Applied Calculus Maximize 𝑝 = 100𝑥 + 50𝑦 (in $1,000) subject to 𝑥 + 𝑦 ≤ 110, 60𝑥 + 50𝑦 ≤ 6,000, 𝑥 ≥ 0, 𝑦 ≥ 0. Corner Point Lines through Point Coordinates 𝑝 = 100𝑥 + 50𝑦 𝐴

𝑥 = 0, 𝑦 = 0

(0, 0)

0

𝐵

𝑦=0 60𝑥 + 50𝑦 = 6,000

(100, 0)

10,000

𝐶

60𝑥 + 50𝑦 = 6,000 𝑥 + 𝑦 = 110

(50, 60)

8,000

𝐷

𝑥 + 𝑦 = 110 𝑥=0

(0, 110)

5,500

Maximum value occurs at the point 𝐵 : 𝑝 = 10,000; 𝑥 = 100, 𝑦 = 0. Solution: The department should offer 100 sections of Finite Math and no sections of Applied Calculus. 27. (To set up the inequalities, see the solution to #35 in the preceding section.) Unknowns: 𝑥 = # ounces of chicken, 𝑦 = # ounces of grain Minimize 𝑐 = 10𝑥 + 𝑦 subject to 10𝑥 + 2𝑦 ≥ 200, 5𝑥 + 2𝑦 ≥ 150, 𝑥 ≥ 0, 𝑦 ≥ 0. Corner Point Lines through Point Coordinates 𝑐 = 10𝑥 + 𝑦 𝐴

𝑦=0 5𝑥 + 2𝑦 = 150

(30, 0)

300

𝐵

5𝑥 + 2𝑦 = 150 5𝑥 + 𝑦 = 100

(10, 50)

150

𝐶

5𝑥 + 𝑦 = 100 𝑥=0

(0, 100)

100

Although the feasible region is unbounded, there is no need to add a bounding rectangle as the coefficients of 𝑐 are nonnegative. Minimum value occurs at the point 𝐶 : 𝑐 = 800; 𝑥 = 0, 𝑦 = 100. Solution: Ruff, Inc., should use 100 oz of grain and no chicken.


Solutions Section 6.2 28. (To set up the inequalities, see the solution to #36 in the preceding section.) Unknowns: 𝑥 = # Pomegranates, 𝑦 = # iZacs Minimize 𝑐 = 2,000𝑥 + 2,000𝑦 subject to 400𝑥 + 300𝑦 ≥ 48,000 (or 4𝑥 + 3𝑦 ≥ 480), 80x + 100y \geq 12{,}800 (or 4𝑥 + 5𝑦 ≥ 640), 𝑥 ≥ 0, 𝑦 ≥ 0. Corner Point Lines through Point Coordinates 𝑐 = 2,000𝑥 + 2,000𝑦 𝐴

𝑦=0 4𝑥 + 5𝑦 = 640

(160, 0)

320,000

𝐵

4𝑥 + 5𝑦 = 640 4𝑥 + 3𝑦 = 480

(60, 80)

280,000

𝐶

4𝑥 + 3𝑦 = 480 𝑥=0

(0, 160)

320,000

Although the feasible region is unbounded, there is no need to add a bounding rectangle as the coefficients of 𝑐 are nonnegative. Minimum value occurs at the point 𝐵 : 𝑐 = 280,000; 𝑥 = 60, 𝑦 = 80. Solution: The school should buy 60 Pomegranates and 80 iZacs. 29. (To set up the inequalities, see the solution to #37 in the preceding section.) Unknowns: 𝑥 = Number of servings of Multigrain Cereal, 𝑦 = Number of servings of 2nd Foods Mango Minimize 𝑐 = 30𝑥 + 50𝑦 subject to 3𝑥 + 4𝑦 ≥ 7, 11𝑥 + 19𝑦 ≥ 30, 𝑥 ≥ 0, 𝑦 ≥ 0. Corner Point Lines through Point Coordinates 𝑐 = 30𝑥 + 50𝑦 𝐴

𝑦=0 11𝑥 + 19𝑦 = 30

(30∕11, 0)

81.8

𝐵

11𝑥 + 19𝑦 = 30 3𝑥 + 4𝑦 = 7

(1, 1)

80

𝐶

3𝑥 + 4𝑦 = 7 𝑥=0

(0, 7∕4)

87.5

Although the feasible region is unbounded, there is no need to add a bounding rectangle as the coefficients of 𝑐 are nonnegative. Minimum value occurs at the point 𝐵 : 𝑐 = 80; 𝑥 = 1, 𝑦 = 1. Solution: Feed your child 1 serving of cereal and 1 serving of dessert. 30. (To set up the inequalities, see the solution to #38 in the preceding section.) Unknowns: 𝑥 = Number of servings of Multigrain Cereal, 𝑦 = Number of servings of Vanilla Custard Pudding Minimize 𝑐 = 10𝑥 + 24𝑦 subject to 3𝑥 + 6𝑦 ≥ 7, 𝑥 + 3𝑦 ≥ 3, 𝑥 ≥ 0, 𝑦 ≥ 0. Corner Point Lines through Point Coordinates 𝑐 = 10𝑥 + 24𝑦 𝐴

3𝑥 + 6𝑦 = 7 𝑥=0

(0, 7∕6)

28

𝐵

3𝑥 + 6𝑦 = 7 𝑥 + 3𝑦 = 3

(1, 2∕3)

26

𝐶

𝑥 + 3𝑦 = 3 𝑦=0

(3, 0)

30


Solutions Section 6.2 Although the feasible region is unbounded, there is no need to add a bounding rectangle as the coefficients of 𝑐 are nonnegative. Minimum value occurs at the point 𝐵 : 𝑐 = 26; 𝑥 = 1, 𝑦 = 2∕3 Solution: Feed your child 1 serving of cereal and 2∕3 serving of custard pudding. 31. Unknowns: 𝑥 = # LED light bulbs, 𝑦 = # square ft of insulation Maximize 𝑝 = 2𝑥 + 0.2𝑦 subject to 4𝑥 + 𝑦 ≤ 1,200, 𝑥 ≤ 60, 𝑦 ≤ 1,100, 𝑥 ≥ 0, 𝑦 ≥ 0. Corner Point Lines through Point Coordinates 𝑝 = 2𝑥 + 0.2𝑦 𝐴

4𝑥 + 𝑦 = 1,200 𝑥 = 60

(60, 960)

312

𝐵

𝑦 = 1,100 4𝑥 + 𝑦 = 1,200

(25, 1,100)

270

𝐶

𝑦 = 1,100, 𝑥 = 0

(0, 1,100)

220

𝐷

𝑦 = 0𝑥 = 0

(0, 0)

0

𝐸

𝑥 = 60, 𝑦 = 0

(60, 0)

120

Maximum value occurs at the point 𝐴 : 𝑝 = 312; 𝑥 = 60, 𝑦 = 960. Solution: Purchase 60 LED light bulbs and 960 square feet of insulation for a total saving of $312 per year in energy costs. 32. Unknowns: 𝑥 = # LED light bulbs, 𝑦 = # sq. ft. of insulation Maximize 𝑝 = 4𝑥 + 𝑦 subject to 2𝑥 + 0.2𝑦 ≤ 800, 𝑥 ≤ 200, 𝑦 ≤ 3,000, 𝑥 ≥ 0, 𝑦 ≥ 0. Corner Point Lines through Point Coordinates 𝑝 = 4𝑥 + 𝑦 𝐴

2𝑥 + 0.2𝑦 = 800 𝑥 = 200

(200, 2,000)

2,800

𝐵

𝑦 = 3,000 2𝑥 + 0.2𝑦 = 800

(100, 3,000)

3,400

𝐶

𝑦 = 3,000, 𝑥 = 0

(0, 3,000)

3,000

𝐷

𝑦 = 0, 𝑥 = 0

(0, 0)

0

𝐸

𝑥 = 200, 𝑦 = 0

(200, 0)

800

Maximum value occurs at the point 𝐵 : 𝑐 = 3400; 𝑥 = 200, 𝑦 = 2,000. The total savings in annual energy costs is 2(200) + 0.2(2,000) = $80.


Solutions Section 6.2 33. Unknowns: 𝑥 = # servings of Xtend, 𝑦 = # servings of RecoverMode Minimize 𝑐 = 0.8𝑥 + 1.2𝑦 subject to 3𝑦 ≥ 6, 2.5𝑥 + 3𝑦 ≥ 46, 7𝑥 + 5𝑦 ≥ 105, 𝑥 ≥ 0, 𝑦 ≥ 0. Corner Point Lines through Point Coordinates 𝑐 = 0.8𝑥 + 1.2𝑦 𝐴

7𝑥 + 5𝑦 = 105 𝑥=0

(0, 21)

25.2

𝐵

2.5𝑥 + 3𝑦 = 46 7𝑥 + 5𝑦 = 105

(10, 7)

16.4

𝐶

2.5𝑥 + 3𝑦 = 46 𝑦=2

(16, 2)

15.2

Although the feasible region is unbounded, there is no need to add a bounding rectangle as the coefficients of 𝑐 are nonnegative. Minimum value occurs at the point 𝐶 : 𝑐 = 15.2; 𝑥 = 16, 𝑦 = 2. Solution: Mix 16 servings of Xtend and 2 servings of RecoverMode for a cost of $15.20. 34. Unknowns: 𝑥 = # servings of RecoverMode, 𝑦 = # servings of Strongevity Minimize 𝑐 = 1.2𝑥 + 𝑦 subject to 3𝑥 + 3𝑦 ≥ 54, 3𝑥 + 𝑦 ≥ 42, 5𝑥 ≥ 30, 𝑥 ≥ 0, 𝑦 ≥ 0. y

42

Corner Point Lines through Point Coordinates 𝑐 = 1.2𝑥 + 𝑦

x=6

𝐴

3𝑥 + 𝑦 = 42 5𝑥 = 30

(6, 24)

31.2

𝐵

3𝑥 + 3𝑦 = 54 3𝑥 + 𝑦 = 42

(12, 6)

20.4

𝐶

3𝑥 + 3𝑦 = 54 𝑦=0

(18, 0)

21.6

A 3x + y = 42

6

B 3x + 3y = 54 C x 18

Although the feasible region is unbounded, there is no need to add a bounding rectangle as the coefficients of 𝑐 are nonnegative. Minimum value occurs at the point 𝐵 : 𝑐 = 20.4; 𝑥 = 12, 𝑦 = 6. Solution: Combine 12 servings of RecoverMode and 6 servings of Strongevity for a cost of $20.40. 35. Unknowns: 𝑥 = # servings of RecoverMode, 𝑦 = # servings of Strongevity. Constraints: 3𝑥 + 3𝑦 ≥ 72, 3𝑥 + 𝑦 ≥ 42, 5𝑥 ≤ 100, 𝑥 ≥ 0, 𝑦 ≥ 0. In part (a) you are maximizing 𝑝 = 𝑥 − 𝑦 and in part (b) you are maximizing 𝑝 = 𝑦 − 𝑥. The feasible region is unbounded, and the LP problems in both parts (a) and (b) do not satisfy any of the conditions in the FAQ at the end of the text for this section, so we need to add a bounding rectangle, adding two new corner points to the feasible region as shown:


Solutions Section 6.2 Corner Point Lines through Point Coordinates a. 𝑝 = 𝑥 − 𝑦 b. 𝑝 = 𝑦 − 𝑥 𝐴

3𝑥 + 𝑦 = 42 𝑥=0

(0, 42)

−42

42

𝐵

3𝑥 + 3𝑦 = 72 3𝑥 + 𝑦 = 42

(9, 15)

−6

6

𝐶

3𝑥 + 3𝑦 = 72 𝑥 = 20

(20, 4)

16

−16

𝐷

𝑥 = 20, 𝑦 = 50

(20, 50)

−30

30

𝐸

𝑥 = 0, 𝑦 = 50

(0, 50)

−50

50

a. The maximum occurs at the point 𝐶, which is a corner point of the original feasible region, so the LP problem has an optimal solution: 𝑝 = 16, 𝑥 = 20, 𝑦 = 4. Use 20 servings of RecoverMode and 4 servings of Strongevity. b. The maximum occurs only at the point 𝐸, which is not a corner point of the original feasible region, so the LP problem has no optimal solution. (The value of the objective function is unbounded.) How do we understand this result in terms of the supplements? Note that we can make 𝑝 = 𝑦 − 𝑥 as large as we like by increasing 𝑦, which means going vertically upwards in the graph on the left, without leaving the feasible region. Increasing 𝑦 corresponds to increasing the amount of Strongevity. So, you could use as much Strongevity as you like without violating your trainer's specifications (Strongevity contains no BCAAs). 36. Unknowns: 𝑥 = # servings of Xtend, 𝑦 = # servings of Muscle Physique. Constraints: 2𝑦 ≤ 16, 2.5𝑥 + 2𝑦 ≥ 25, 7𝑥 ≥ 42, 𝑥 ≥ 0, 𝑦 ≥ 0. In part (a) you are maximizing 𝑝 = 𝑥 − 𝑦 and in part (b) you are maximizing 𝑝 = 𝑦 − 𝑥. The feasible region is unbounded, and the LP problems in both parts (a) and (b) do not satisfy any of the conditions in the FAQ at the end of the text for this section, so we need to add a bounding rectangle, adding two new corner points to the feasible region as shown: y

y

A

y=8

A

B x=6

y=8

E

B 2.5x + 2y = 25 C 10

x=6

2.5x + 2y = 25 C 10

x

D

15

x

Corner Point Lines through Point Coordinates a. 𝑝 = 𝑥 − 𝑦 b. 𝑝 = 𝑦 − 𝑥 𝐴

𝑦 = 8, 𝑥 = 6

(6, 8)

−2

2

𝐵

𝑥=6 2.5𝑥 + 2𝑦 = 25

(6, 5)

1

−1

𝐶

2.5𝑥 + 2𝑦 = 25 𝑦=0

(10, 0)

10

−10

𝐷

𝑥 = 15, 𝑦 = 0

(15, 0)

15

−15

𝐸

𝑥 = 15, 𝑦 = 8

(15, 8)

7

−7

a. The maximum occurs only at the point 𝐷, which is not a corner point of the original feasible region, so the


Solutions Section 6.2 LP problem has no optimal solution. (The value of the objective function is unbounded.) How do we understand this result in terms of the supplements? Note that we can make 𝑝 = 𝑥 − 𝑦 as large as we like by increasing 𝑥, which means moving to the right in the graph on the left, without leaving the feasible region. Increasing 𝑥 corresponds to increasing the amount of Xtend. So, they could use as much Xtend as they like without violating the specifications (Xtend contains no creatine). b. The maximum occurs at the point 𝐴, which is a corner point of the original feasible region, so the LP problem has an optimal solution: 𝑝 = 2, 𝑥 = 6, 𝑦 = 8. Use 6 servings of Xtend and 8 servings of Muscle Physique. 37. Unknowns: 𝑥 = # Dracula salamis, 𝑦 = # Frankenstein sausages Maximize 𝑝 = 𝑥 + 3𝑦 subject to 𝑥 + 2𝑦 ≤ 1,000, 3𝑥 + 2𝑦 ≤ 2,400, 𝑦 ≤ 2𝑥, 𝑥 ≥ 0, 𝑦 ≥ 0. Corner Point Lines through Point Coordinates 𝑝 = 𝑥 + 3𝑦 𝐴

𝑥 = 0, 𝑦 = 0

(0, 0)

0

𝐵

3𝑥 + 2𝑦 = 2,400 𝑦=0

(800, 0)

800

𝐶

𝑥 + 2𝑦 = 1,000 3𝑥 + 2𝑦 = 2,400

(700, 150)

1,150

𝐷

𝑥 + 2𝑦 = 1,000 𝑦 − 2𝑥 = 0

(200, 400)

1,400

Maximum value occurs at the point 𝐷 : 𝑝 = 1,400; 𝑥 = 200, 𝑦 = 400. Solution: You should make 200 Dracula Salamis and 400 Frankenstein Sausages, for a profit of $1,400. 38. Unknowns: 𝑥 = # high income patients, 𝑦 = # homeless patients Maximize 𝑝 = 8,000𝑥 + 10,000𝑦 subject to 𝑥 ≤ 750, 𝑦 ≥ 1,000, 𝑥 + 𝑦 ≤ 2,100, 𝑦 ≤ 2𝑥, 𝑥 ≥ 0, 𝑦 ≥ 0. Corner Point Lines through Point Coordinates 𝑝 = 8,000𝑥 + 10,000𝑦 𝐴

𝑦 = 1,000 𝑦 − 2𝑥 = 0

(500, 1,000)

14 mill.

𝐵

𝑥 = 750 𝑦 = 1,000

(750, 1,000)

16 mill.

𝐶

𝑥 = 750 𝑥 + 𝑦 = 2,100

(750, 1,350)

19.5 mill.

𝐷

𝑥 + 𝑦 = 2,100 𝑦 − 2𝑥 = 0

(700, 1,400)

19.6 mill.

Maximum value occurs at the point 𝐷 : 𝑝 = 19.6million; 𝑥 = 700, 𝑦 = 1,400. Solution: The company should house 700 high-income patients and 1,400 homeless patients for a profit of $19,600,000. 39. Unknowns: 𝑥 = # spots on Chicago Fire, 𝑦 = # spots on The Conners Maximize 𝐸 = 7.2𝑥 + 3.6𝑦, subject to 𝑥 + 𝑦 ≥ 30, 3𝑥 + 𝑦 ≤ 120, 𝑥 − 𝑦 ≥ 0, 𝑥 ≥ 0, 𝑦 ≥ 0.


Solutions Section 6.2 Corner Point Lines through Point Coordinates 𝐸 = 7.2𝑥 + 3.6𝑦 𝐴

𝑥 + 𝑦 = 30 𝑦=0

(30, 0)

216

𝐵

3𝑥 + 𝑦 = 120 𝑦=0

(40, 0)

288

𝐶

3𝑥 + 𝑦 = 120 𝑥−𝑦=0

(30, 30)

324

𝐷

𝑥 + 𝑦 = 30 𝑥−𝑦=0

(15, 15)

162

Maximum value occurs at the point 𝐶 : 𝐸 = 324; 𝑥 = 30, 𝑦 = 30. Solution: Purchase 30 spots on Chicago Fire and 30 spots on The Conners. 40. Unknowns: 𝑥 = # spots on The Masked Singer, 𝑦 = # spots on The Conners Maximize 𝐸 = 4.6𝑥 + 3.6𝑦 subject to 𝑥 + 𝑦 ≥ 40, 4𝑥 + 𝑦 ≤ 260, 𝑥 − 3𝑦 ≥ 0, 𝑥 ≥ 0, 𝑦 ≥ 0. Corner Point Lines through Point Coordinates 𝐸 = 4.6𝑥 + 3.6𝑦 𝐴

𝑥 + 𝑦 = 40 𝑦=0

(40, 0)

184

𝐵

4𝑥 + 𝑦 = 260 𝑦=0

(65, 0)

299

𝐶

4𝑥 + 𝑦 = 260 𝑥 − 3𝑦 = 0

(60, 20)

348

𝐷

𝑥 + 𝑦 = 40 𝑥 − 3𝑦 = 0

(30, 10)

174

Maximum value occurs at the point 𝐶 : 𝐸 = 348; 𝑥 = 60, 𝑦 = 20. Solution: Purchase 60 spots on The Masked Singer and 20 spots on The Conners. 41. Unknowns: 𝑥 = # CVS shares, 𝑦 = # RCKY shares Minimize 𝑐 = 30𝑥 + 25𝑦 subject to 90𝑥 + 50𝑦 ≤ 90,000 or 9𝑥 + 5𝑦 ≤ 9,000, (0.02)(90𝑥) + (0.01)(50𝑦) ≥ 1,080, or 1.8𝑥 + 0.5𝑦 ≥ 1,080 or 18𝑥 + 5𝑦 ≥ 10,800, 𝑥 ≥ 0, 𝑦 ≥ 0. Corner Point Lines through Point Coordinates 𝑐 = 30𝑥 + 25𝑦 𝐴

9𝑥 + 5𝑦 = 9,000 18𝑥 + 5𝑦 = 10,800

(200, 1,440)

42,000

𝐵

18𝑥 + 5𝑦 = 10,800 𝑦=0

(600, 0)

18,000

𝐶

9𝑥 + 5𝑦 = 9,000 𝑦=0

(1,000, 0)

30,000

Minimum value occurs at the point 𝐵 : 𝑐 = 18,000; 𝑥 = 600, 𝑦 = 0 Solution: Purchase 600 shares of CVS and no shares of RCKY.


Solutions Section 6.2 42. Unknowns: 𝑥 = # GCO shares, 𝑦 = # ATVI shares Minimize 𝑐 = 40𝑥 − 5𝑦 subject to 60𝑥 + 75𝑦 ≤ 42,000 or 4𝑥 + 5𝑦 ≤ 2,800, 0𝑥 + (0.01)(75𝑦) ≤ 150 or 0.75𝑦 ≤ 150, or 𝑦 ≤ 200, 𝑥 ≥ 0, 𝑦 ≥ 0. Corner Point Lines through Point Coordinates 𝑐 = 40𝑥 − 5𝑦 𝐴

𝑥 = 0, 𝑦 = 200

(0, 200)

−1,000

𝐵

4𝑥 + 5𝑦 ≤ 2,800 𝑦 = 200

(450, 200)

17,000

𝐶

4𝑥 + 5𝑦 ≤ 2,800 𝑦=0

(700, 0)

28,000

𝐷

𝑥 = 0, 𝑦 = 0

(0, 0)

0

Minimum value occurs at the point 𝐴 : 𝑐 = −1,000; 𝑥 = 0, 𝑦 = 200. Solution: Purchase no shares of GCO and 200 shares of ATVI. 43. Unknowns: 𝑥 = # TD shares; 𝑦 = # CNA shares Minimize 𝑐 = 3𝑥 + 2𝑦 subject to 66𝑥 + 42𝑦 ≤ 25,800 or 11𝑥 + 7𝑦 ≤ 4,300, 0.04(66𝑥) + 0.035(42𝑦) ≥ 969 or 88𝑥 + 49𝑦 ≥ 32,300, 𝑥 ≥ 0, 𝑦 ≥ 0. Corner Point Lines through Point

Coordinates 𝑐 = 3𝑥 + 2𝑦 (rounded)

𝐴

11𝑥 + 7𝑦 = 4,300 88𝑥 + 49𝑦 = 32,300

(200, 300)

1,200

𝐵

11𝑥 + 7𝑦 = 4,300 𝑦=0

(391, 0)

1,173

𝐶

88𝑥 + 49𝑦 = 32,300 𝑦=0

(367, 0)

1,101

Minimum value occurs at the point 𝐶 : 𝑐 = 1,101; 𝑥 ≈ 367, 𝑦 = 0. Solution: Purchase 367 shares of TD and no shares of CNA. 44. Unknowns: 𝑥 = # PAA shares; 𝑦 = # CVX shares Minimize 𝑐 = 2𝑥 + 3𝑦 subject to 10𝑥 + 100𝑦 ≤ 30,000 or 𝑥 + 10𝑦 ≤ 3,000, 0.07(10𝑥) + 0.05(100𝑦) ≥ 1,700 or 7𝑥 + 50𝑦 ≥ 17,000, 𝑥 ≥ 0, 𝑦 ≥ 0. Corner Point Lines through Point

Coordinates 𝑐 = 2𝑥 + 3𝑦 (rounded)

𝐴

𝑥 + 10𝑦 = 3,000 7𝑥 + 50𝑦 = 17,000

(1,000, 200)

2,600

𝐵

𝑥 + 10𝑦 = 3,000 𝑦=0

(3,000, 0)

6,000

𝐶

7𝑥 + 50𝑦 = 17,000 𝑦=0

(2,429, 0)

4,858

Minimum value occurs at the point 𝐴 : 𝑐 = 2,600; 𝑥 = 1,000, 𝑦 = 200. Solution: Purchase 1,000 shares of PAA and 200 shares of CVX.


Solutions Section 6.2 45. Unknowns: 𝑥 = # hours spent in battle instruction per week, 𝑦 = # hours spent per week in diplomacy instruction Maximize 𝑝 = 50𝑥 + 40𝑦 subject to 𝑥 + 𝑦 ≤ 50, 𝑥 ≥ 2𝑦, 𝑦 ≥ 10, 10𝑥 + 5𝑦 ≥ 400, 𝑥 ≥ 0, 𝑦 ≥ 0. Corner Point Lines through Point Coordinates 𝑝 = 50𝑥 + 40𝑦 𝐴

𝑦 = 10 10𝑥 + 5𝑦 = 400

(35, 10)

2,150

𝐵

𝑥 + 𝑦 = 50 𝑦 = 10

(40, 10)

2,400

𝐶

𝑥 + 𝑦 = 50 𝑥 − 2𝑦 = 0

(100∕3, 50∕3)

7,000∕3

𝐷

𝑥 − 2𝑦 = 0 10𝑥 + 5𝑦 = 400

(32, 16)

2,240

Maximum value occurs at the point 𝐵 : 𝑝 = 2,400; 𝑥 = 40, 𝑦 = 10. Solution: He should instruct in diplomacy for 10 hours per week and in battle for 40 hours per week, giving a weekly profit of 2,400 sestertii. 46. Unknowns: 𝑥 = # hours spent in battle instruction per week, 𝑦 = # hours spent per week in diplomacy instruction Maximize 𝑝 = 50𝑥 + 40𝑦 subject to 𝑥 + 𝑦 ≤ 50, 𝑥 ≥ 𝑦, 𝑦 ≥ 10, 10𝑥 + 5𝑦 ≥ 600, 𝑥 ≥ 0, 𝑦 ≥ 0.

There is no solution: The feasible region is empty.

47. Unknowns: 𝑥 = # sleep spells, 𝑦 = # shock spells a. Minimize 𝑐 = 50𝑥 + 20𝑦 subject to 3𝑥 + 𝑦 ≥ 24, 2𝑥 + 4𝑦 ≥ 26, 𝑥 − 𝑦 ≥ 0, 𝑥 − 3𝑦 ≤ 3, 𝑥 ≥ 0, 𝑦 ≥ 0. y

3𝑥 + 𝑦 = 24

6.5

Corner Point Lines through Point Coordinates 𝑐 = 50𝑥 + 20𝑦

𝑥−𝑦=0

A 𝑥 − 3𝑦 = 3

B

C 2𝑥 + 4𝑦 = 26

8

13

x

𝐴

3𝑥 + 𝑦 = 24 𝑥−𝑦=0

(6, 6)

420

𝐵

3𝑥 + 𝑦 = 24 2𝑥 + 4𝑦 = 26

(7, 3)

410

𝐶

2𝑥 + 4𝑦 = 26 𝑥 − 3𝑦 = 3

(9, 2)

490

Although the feasible region is unbounded, there is no need to add a bounding rectangle since, by the FAQ at the end of the section in the text: If you are minimizing 𝑐 = 𝑎𝑥 + 𝑏𝑦 with 𝑎 and 𝑏 nonnegative, 𝑥 ≥ 0,and 𝑦 ≥ 0, then optimal solutions always exist. Minimum value occurs at point 𝐵 : 𝑐 = 410, 𝑥 = 7, 𝑦 = 3. Use 7 sleep spells and 3 shock spells. b. and c. Minimize (b) 𝑐 = 40𝑥 − 10𝑦 or (c) 𝑐 = 10𝑥 − 40𝑦 subject to 3𝑥 + 𝑦 ≥ 24, 2𝑥 + 4𝑦 ≥ 26, 𝑥 − 𝑦 ≥ 0,


Solutions Section 6.2 𝑥 − 3𝑦 ≤ 3, 𝑥 ≥ 0, 𝑦 ≥ 0. The feasible region is unbounded, and the problem does not satisfy any of the conditions in the FAQ at the end of the text for this section, so we need to add a bounding rectangle, adding three new corner points to the feasible region as shown: y

F

𝑦=8

E

A D B

C

𝑥 = 15

x

Corner Point Lines through Point Coordinates 𝑐 = 40𝑥 − 10𝑦 𝑐 = 10𝑥 − 40𝑦 𝐴

3𝑥 + 𝑦 = 24 𝑥−𝑦=0

(6, 6)

180

−180

𝐵

3𝑥 + 𝑦 = 24 2𝑥 + 4𝑦 = 26

(7, 3)

250

−50

𝐶

2𝑥 + 4𝑦 = 26 𝑥 − 3𝑦 = 3

(9, 2)

340

10

𝐷

𝑥 = 15 𝑥 − 3𝑦 = 3

(15, 4)

560

−10

𝐸

𝑥 = 15, 𝑦 = 8

(15, 8)

520

−170

𝐹

𝑥−𝑦=0 𝑦=8

(8, 8)

240

−240

(b) The minimum occurs at the point 𝐵, which is a corner point of the original feasible region, so the LP problem has an optimal solution: 𝑐 = 180, 𝑥 = 6, 𝑦 = 6 : Use 6 sleep spells and 6 shock spells. (c) The minimum occurs at the point 𝐹 , which is not a corner point of the original feasible region, so the LP problem has no optimal solution. Net expenditure of aural energy can be an arbitrarily large negative number. 48. Unknowns: 𝑥 = # gold payoffs, 𝑦 = # political favors a. Minimize 𝑐 = 2𝑥 + 4𝑦 subject to 𝑥 + 𝑦 ≥ 9, 2𝑥 + 3𝑦 ≥ 24, −5𝑥 + 𝑦 ≤ 3, 2𝑥 − 3𝑦 ≤ 0, 𝑥 ≥ 0, 𝑦 ≥ 0. y

Corner Point Lines through Point Coordinates 𝑐 = 2𝑥 + 4𝑦

−5𝑥 + 𝑦 = 3

9 8

A B C

3

2𝑥−3𝑦 = 0 2𝑥 + 3𝑦 = 24

𝑥+𝑦=9

9

x

𝐴

𝑥+𝑦=9 −5𝑥 + 𝑦 = 3

(1, 8)

34

𝐵

2𝑥 + 3𝑦 = 24 𝑥+𝑦=9

(3, 6)

30

𝐶

2𝑥 + 3𝑦 = 24 2𝑦 − 3𝑥 = 0

(6, 4)

28

Although the feasible region is unbounded, there is no need to add a bounding rectangle since, by the FAQ at the end of the section in the text: If you are minimizing 𝑐 = 𝑎𝑥 + 𝑏𝑦 with 𝑎 and 𝑏 nonnegative, 𝑥 ≥ 0, and 𝑦 ≥ 0, then optimal solutions always exist. Minimum value occurs at point 𝐶 : 𝑐 = 28; 𝑥 = 6, 𝑦 = 4.


Solutions Section 6.2 Solution: He should make 6 gold payoffs and 4 political favors with a net loss to the travel budget of 28 Orbs. b. and c. Minimize (b) 𝑐 = −2𝑥 + 𝑦 or (c) 𝑐 = 6𝑥 − 𝑦 subject to 𝑥 + 𝑦 ≥ 9, 2𝑥 + 3𝑦 ≥ 24, −5𝑥 + 𝑦 ≤ 3, 2𝑥 − 3𝑦 ≤ 0, 𝑥 ≥ 0, 𝑦 ≥ 0. The feasible region is unbounded, and the problem does not satisfy any of the conditions in the FAQ at the end of the test for this section, so we need to add a bounding rectangle, adding three new corner points to the feasible region as shown: y

F A

𝑦 = 10

E D

B C

𝑥=9

x

Corner Point Lines through Point Coordinates 𝑐 = −2𝑥 + 𝑦 𝑐 = 6𝑥 − 𝑦 𝐴

𝑥+𝑦=9 −5𝑥 + 𝑦 = 3

(1, 8)

6

−2

𝐵

2𝑥 + 3𝑦 = 24 𝑥+𝑦=9

(3, 6)

0

12

𝐶

2𝑥 + 3𝑦 = 24 2𝑦 − 3𝑥 = 0

(6, 4)

−8

32

𝐷

𝑥=9 2𝑥 − 3𝑦 = 0

(9, 6)

−12

48

𝐸

𝑥 = 9, 𝑦 = 10

(9, 10)

−8

44

𝐹

−5𝑥 + 𝑦 = 3 𝑦 = 10

(1.4, 10)

7.2

−1.6

(b) The minimum occurs only at the point 𝐷, which is not a corner point of the original feasible region, so the LP problem has no optimal solution. Net loss to the travel budget can be an arbitrarily large negative number. (c) The minimum occurs at the point 𝐴, which is a corner point of the original feasible region, so the LP problem has an optimal solution: 𝑐 = −2, 𝑥 = 1, 𝑦 = 8 : 1 gold payoff and 8 political favors, with a net gain to the travel budget of 2 Orbs.


Solutions Section 6.2 49. Unknowns: 𝑥 = # hours for new customers, 𝑦 = # hours for old customers Maximize 𝑝 = 10𝑥 + 30𝑦 subject to 10𝑥 + 30𝑦 ≥ 1,200, 𝑥 + 𝑦 ≤ 160, 𝑥 ≥ 100, 𝑥 ≥ 0, 𝑦 ≥ 0. Corner Point Lines through Point Coordinates 𝑝 = 10𝑥 + 30𝑦 𝐴

10𝑥 + 30𝑦 = 1,200 𝑥 = 100

(100, 20∕3)

1,200

𝐵

10𝑥 + 30𝑦 = 1,200 𝑦=0

(120, 0)

1,200

𝐶

𝑥 + 𝑦 = 160 𝑦=0

(160, 0)

1,600

𝐷

𝑥 + 𝑦 = 160 𝑥 = 100

(100, 60)

2,800

Maximum value occurs at the point 𝐷 : 𝑝 = 2,800; 𝑥 = 100, 𝑦 = 60. Solution: Allocate 100 hours per week for new customers and 60 hours per week for old customers. 50. a. The mill can produce 8×24×30×4.63 = 26,668.8 yards of material. This is not enough to meet the demand of 16,000 + 12,000 = 28,000 yards. b. Unknowns: 𝑥 = # yards of Fabric 1, 𝑦 = # yards of Fabric 2 Maximize 𝑝 = 0.13𝑥 + 0.17𝑦 + 5,120 subject to 𝑥 + 𝑦 ≤ 26,668.8, 𝑥 ≤ 16,000, 𝑦 ≤ 12,000, 𝑥 ≥ 0, 𝑦 ≥ 0. Corner Point Lines through Point Coordinates

𝑝 = 0.13𝑥 + 0.17𝑦 +5,120

𝐴

𝑥 = 0, 𝑦 = 0

(0, 0)

5,120

𝐵

𝑥 = 16,000 𝑦=0

(16,000, 0)

7200

𝐶

𝑥 = 16,000 𝑥 + 𝑦 = 6,668.8

(16,000, 10,668.8)

9,013.70

𝐷

𝑥 + 𝑦 = 26,668.8 𝑦 = 12,000

(14,668.8, 12,000)

9,066.94

𝐸

𝑥=0 𝑦 = 12,000

(0, 12,000)

7,160

Maximum value occurs at the point 𝐷 : 𝑝 = 9,066.94; 𝑥 = 14,668.8, 𝑦 = 12,000. Solution: The mill should produce 14,668.8 yards of Fabric 1 and 12,000 yards of Fabric 2. 51. By the Fundamental Theorem of Linear Programming, linear programming problems with bounded, nonempty feasible regions always have optimal solutions (Choice (A)). 52. If a linear programming problem has an unbounded, nonempty feasible region, then optimal solutions may or may not exist (Choice (B)). 53. Every point along the line connecting them is also an optimal solution. 54. Answers may vary. One limitation is that the method is only suitable for situations with two unknown quantities. Another is that the graphical representation may need to be very accurate to show the feasible region correctly.


Solutions Section 6.2 55. a. Minimizing 𝑐 = 4𝑥 + 𝑦 satisfies the first condition in the FAQ at the end of the text for this section, so optimal solutions exist, and it is not necessary to add a bounding rectangle. b. Maximizing 𝑝 = 2𝑥 does not satisfy any of the conditions in the FAQ, so we need to add a bounding rectangle to check if optimal solutions exist. c. Maximizing 𝑝 = 4𝑥 − 𝑦 does not satisfy any of the conditions in the FAQ, so we need to add a bounding rectangle to check if optimal solutions exist. d. Maximizing 𝑝 = 2𝑥 + 𝑦 satisfies the second condition in the FAQ, so no optimal solution exists, and it is not necessary to add a bounding rectangle. 56. a. Minimizing 𝑐 = 4𝑥 − 𝑦 does not satisfy any of the conditions in the FAQ at the end of the section, so we need to add a bounding rectangle to check if optimal solutions exist. b. Minimizing 𝑐 = −𝑥 − 𝑦 satisfies the condition (d) in the FAQ, so no optimal solution exists, and it is not necessary to add a bounding rectangle. c. Minimizing 𝑐 = −5𝑦 does not satisfy any of the conditions in the FAQ, so we need to add a bounding rectangle to check if optimal solutions exist. d. Maximizing 𝑝 = −2𝑥 − 𝑦 satisfies condition (c) in the FAQ at the end of the text for this section, so optimal solutions exist, and it is not necessary to add a bounding rectangle. 57. Here are two simple examples: (1) (Empty feasible region) Maximize 𝑝 = 𝑥 + 𝑦 subject to 𝑥 + 𝑦 ≤ 10; 𝑥 + 𝑦 ≥ 11, 𝑥 ≥ 0, 𝑦 ≥ 0. (2) (Unbounded feasible region and no optimal solutions) Minimize 𝑐 = 𝑥 − 𝑦 subject to 𝑥 + 𝑦 ≥ 10, 𝑥 ≥ 0, 𝑦 ≥ 0. 58. Answers may vary. Maximize 𝑝 = 𝑥 + 𝑦 subject to 𝑥 + 𝑦 ≤ 10, 𝑥 ≥ 0, 𝑦 ≥ 0. 59. Answers may vary. 60. Answers may vary. 61. A simple example is the following: Maximize profit 𝑝 = 2𝑥 + 𝑦 subject to 𝑥 ≥ 0, 𝑦 ≥ 0. Then 𝑝 can be made as large as we like by choosing large values of 𝑥 and/or 𝑦. Thus, there is no optimal solution to the problem. 62. A simple example is the following: Minimize cost = 𝑐 = 3𝑥 + 5𝑦 subject to 𝑥 ≥ 2, 𝑦 ≥ 1. Then the only corner point of the feasible region is (2, 1) and the cost cannot be made smaller than 3(2) + 5(1) = 11, which occurs at the corner point. 63. Mathematically, this means that there are infinitely many possible solutions: one for each point along the line joining the two corner points in question. In practice, select those points with integer solutions (since 𝑥 and 𝑦 must be whole numbers in this problem) that are in the feasible region and close to this line, and choose the one that gives the largest profit. 64. Something must be wrong with the way the LP problem was set up, since in a real situation, there must be a value for 𝑥 and for 𝑦 resulting in the largest profit. In other words, there has to be an optimal solution.


Solutions Section 6.2 65. The corner points are shown in the following figure:

The corner points have coordinates 𝐴 : (0, 0), 𝐵 : (2, 0), 𝐶 : (0, 2), and so 𝑝 = 𝑥𝑦 is zero at each of these points. However, at infinitely many points along the diagonal such as 𝐷 : (1, 1) and 𝐸 : (1.5, 0.5), the value of 𝑝 is positive. (For example, at 𝐷, 𝑝 = (1)(1) = 1, and at 𝐸, 𝑝 = (1.5)(0.5) = 0.75.) 66. The figure shows the feasible region together with some curves of the form 𝑥𝑦 = 𝑘 (𝑘 ≥ 0).

Among these curves, the one with the largest value of 𝑘 that touches the feasible region is 𝑥𝑦 = 1, which intersects the region at 𝐷 : (1, 1). Thus, the optimal solution is 𝑥 = 𝑦 = 1, 𝑝 = 1.


Solutions Section 6.3 Section 6.3 1. Introduce slack variables and rewrite the constraints and objective function in standard form, and set up the first tableau: 𝑥 + 2𝑦 + 𝑠 = 6 −𝑥 + 𝑦 + 𝑡 = 4 𝑥+𝑦+𝑢=4 −2𝑥 − 𝑦 + 𝑝 = 0 𝑥

𝑦

𝑠

𝑡

𝑢

𝑝

𝑠 𝑡 𝑢

1

−1 1

2 1 1

1 0 0

0 1 0

0 0 1

0 0 0

6 4 4

𝑝

−2

−1

0

0

0

1

0

The most negative entry in the bottom row is the −2 in the 𝑥-column, so we use this column as the pivot column. The test ratios are: s: 6/1, u: 4/1. The smallest test ratio is u: 4/1. Thus we pivot on the 1 in the u-row. 𝑥

𝑦

𝑠

𝑡

𝑢

𝑝

𝑠

1

2

1

0

0

0

6

𝑅1 − 𝑅3

𝑡

−1

1

0

1

0

0

4

𝑅2 + 𝑅3

𝑢

1

1

0

0

1

0

4

𝑝

−2

−1

0

0

0

1

0

𝑥

𝑦

𝑠

𝑡

𝑢

𝑝

𝑠 𝑡 𝑥

0 0 1

1 2 1

1 0 0

0 1 0

−1 1 1

0 0 0

2 8 4

𝑝

0

1

0

0

2

1

8

𝑅4 + 2𝑅3

As there are no more negative numbers in the bottom row, we are done, and read off the solution: Optimal solution: 𝑝 = 8∕1 = 8; 𝑥 = 4∕1 = 4, 𝑦 = 0. 2. Introduce slack variables and rewrite the constraints and objective function in standard form, and set up the first tableau: 𝑥−𝑦+𝑠=4 −𝑥 + 3𝑦 + 𝑡 = 4 −𝑥 + 𝑝 = 0 𝑥

𝑦

𝑠

𝑡

𝑝

𝑠 𝑡

1 −1

−1 3

1 0

0 1

0 0

4 4

𝑝

−1

0

0

0

1

0


Solutions Section 6.3 The most negative entry in the bottom row is the −1 in the 𝑥-column, so we use this column as the pivot column. The only positive entry in this column is the 1 in the s-row, so we pivot on that entry. 𝑥

𝑦

𝑠

𝑡

𝑝

𝑠

1

−1

1

0

0

4

𝑡

−1

3

0

1

0

4

𝑅2 + 𝑅1

𝑝

−1

0

0

0

1

0

𝑅3 + 𝑅1

𝑥

𝑦

𝑠

𝑡

𝑝

𝑥 𝑡

1 0

−1 2

1 1

0 1

0 0

4 8

𝑝

0

−1

1

0

1

4

The most negative entry in the bottom row is the −1 in the 𝑦-column, so we use this column as the pivot column. The only positive entry in this column is the 2 in the t-row, so we pivot on that entry. 𝑥

𝑦

𝑠

𝑡

𝑝

𝑥

1

−1

1

0

0

4

𝑡

0

2

1

1

0

8

𝑝

0

−1

1

0

1

4

𝑥

𝑦

𝑠

𝑡

𝑝

𝑥 𝑦

2 0

0 2

3 1

1 1

0 0

16 8

𝑝

0

0

3

1

2

16

2𝑅1 + 𝑅2 2𝑅3 + 𝑅2

As there are no more negative numbers in the bottom row, we are done, and read off the solution: Optimal solution: 𝑝 = 16∕2 = 8; 𝑥 = 16∕2 = 8, 𝑦 = 8∕2 = 4. 3. Introduce slack variables and rewrite the constraints and objective function in standard form, and set up the first tableau: 5𝑥 − 5𝑦 + 𝑠 = 20 2𝑥 − 10𝑦 + 𝑡 = 40 −𝑥 + 𝑦 + 𝑝 = 0 𝑥

𝑦

𝑠

𝑡

𝑝

𝑠 𝑡

5 2

−5 −10

1 0

0 1

0 0

20 40

𝑝

−1

1

0

0

1

0

The most negative entry in the bottom row is the −1 in the 𝑥-column, so we use this column as the pivot column. The test ratios are: s: 20/5, t: 40/2. The smallest test ratio is s: 20/5. Thus we pivot on the 5 in the


Solutions Section 6.3 s-row. 𝑥

𝑦

𝑠

𝑡

𝑝

𝑠

5

−5

1

0

0

20

𝑡

2

−10

0

1

0

40

5𝑅2 − 2𝑅1

𝑝

−1

1

0

0

1

0

5𝑅3 + 𝑅1

𝑥

𝑦

𝑠

𝑡

𝑝

𝑥 𝑡

5 0

−5 −40

1 −2

0 5

0 0

20 160

𝑝

0

0

1

0

5

20

As there are no more negative numbers in the bottom row, we are done, and read off the solution: Optimal solution: 𝑝 = 20∕5 = 4; 𝑥 = 20∕5 = 4, 𝑦 = 0. 4. Introduce slack variables and rewrite the constraints and objective function in standard form, and set up the first tableau: 3𝑥 + 8𝑦 + 𝑠 = 24 6𝑥 + 4𝑦 + 𝑡 = 30 −2𝑥 − 3𝑦 + 𝑝 = 0 𝑥

𝑦

𝑠

𝑡

𝑝

𝑠 𝑡

3 6

8 4

1 0

0 1

0 0

24 30

𝑝

−2

−3

0

0

1

0

The most negative entry in the bottom row is the −3 in the 𝑦-column, so we use this column as the pivot column. The test ratios are: s: 24/8, t: 30/4. The smallest test ratio is s: 24/8. Thus we pivot on the 8 in the s-row. 𝑥

𝑦

𝑠

𝑡

𝑝

𝑠

3

8

1

0

0

24

𝑡

6

4

0

1

0

30

2𝑅2 − 𝑅1

𝑝

−2

−3

0

0

1

0

8𝑅3 + 3𝑅1

𝑥

𝑦

𝑠

𝑡

𝑝

𝑦 𝑡

3 9

8 0

1 −1

0 2

0 0

24 36

𝑝

−7

0

3

0

8

72

The most negative entry in the bottom row is the −7 in the 𝑥-column, so we use this column as the pivot column. The test ratios are: y: 24/3, t: 36/9. The smallest test ratio is t: 36/9. Thus we pivot on the 9 in the t-row.


Solutions Section 6.3 𝑥

𝑦

𝑠

𝑡

𝑝

𝑦

3

8

1

0

0

24

𝑡

9

0

−1

2

0

36

𝑝

−7

0

3

0

8

72

𝑥

𝑦

𝑠

𝑡

𝑝

𝑦 𝑥

0 9

24 0

4 −1

−2 2

0 0

36 36

𝑝

0

0

20

14

72

900

3𝑅1 − 𝑅2 9𝑅3 + 7𝑅2

As there are no more negative numbers in the bottom row, we are done, and read off the solution: Optimal solution: 𝑝 = 900∕72 = 25∕2; 𝑥 = 36∕9 = 4, 𝑦 = 36∕24 = 3∕2. 5. Introduce slack variables and rewrite the constraints and objective function in standard form, and set up the first tableau: 5𝑥 + 5𝑧 + 𝑠 = 100 5𝑦 − 5𝑧 + 𝑡 = 50 5𝑥 − 5𝑦 + 𝑢 = 50 −5𝑥 + 4𝑦 − 3𝑧 + 𝑝 = 0 𝑥

𝑦

𝑧

𝑠

𝑡

𝑢

𝑝

𝑠 𝑡 𝑢

5 0 5

0 5

−5

5 −5 0

1 0 0

0 1 0

0 0 1

0 0 0

100 50 50

𝑝

−5

4

−3

0

0

0

1

0

The most negative entry in the bottom row is the −5 in the 𝑥-column, so we use this column as the pivot column. The test ratios are: s: 100/5, u: 50/5. The smallest test ratio is u: 50/5. Thus we pivot on the 5 in the u-row. 𝑥

𝑦

𝑧

𝑠

𝑡

𝑢

𝑝

𝑠

5

0

5

1

0

0

0

100

𝑡

0

5

−5

0

1

0

0

50

𝑢

5

−5

0

0

0

1

0

50

𝑝

−5

4

−3

0

0

0

1

0

𝑥

𝑦

𝑧

𝑠

𝑡

𝑢

𝑝

𝑠 𝑡 𝑥

0 0 5

5 5 −5

5 −5 0

1 0 0

0 1 0

−1 0 1

0 0 0

50 50 50

𝑝

0

−1

−3

0

0

1

1

50

𝑅1 − 𝑅3

𝑅4 + 𝑅3


Solutions Section 6.3 The most negative entry in the bottom row is the −3 in the 𝑧-column, so we use this column as the pivot column. The only positive entry in this column is the 5 in the s-row, so we pivot on that entry. 𝑥

𝑦

𝑧

𝑠

𝑡

𝑢

𝑝

𝑠

0

5

5

1

0

−1

0

50

𝑡

0

5

−5

0

1

0

0

50

𝑥

5

−5

0

0

0

1

0

50

𝑝

0

−1

−3

0

0

1

1

50

𝑥

𝑦

𝑧

𝑠

𝑡

𝑢

𝑝

𝑧 𝑡 𝑥

0 0 5

5 10 −5

5 0 0

1 1 0

0 1 0

−1 −1 1

0 0 0

50 100 50

𝑝

0

10

0

3

0

2

5

400

𝑅2 + 𝑅1 5𝑅4 + 3𝑅1

As there are no more negative numbers in the bottom row, we are done, and read off the solution: Optimal solution: 𝑝 = 400∕5 = 80; 𝑥 = 50∕5 = 10, 𝑦 = 0, 𝑧 = 50∕5 = 10. 6. Introduce slack variables and rewrite the constraints and objective function in standard form, and set up the first tableau: 3𝑥 + 𝑦 + 𝑠 = 15 2𝑥 + 2𝑦 + 2𝑧 + 𝑡 = 20 −6𝑥 − 𝑦 − 3𝑧 + 𝑝 = 0 𝑥

𝑦

𝑧

𝑠

𝑡

𝑝

𝑠 𝑡

3 2

1 2

0 2

1 0

0 1

0 0

15 20

𝑝

−6

−1

−3

0

0

1

0

The most negative entry in the bottom row is the −6 in the 𝑥-column, so we use this column as the pivot column. The test ratios are: s: 15/3, t: 20/2. The smallest test ratio is s: 15/3. Thus we pivot on the 3 in the s-row. 𝑥

𝑦

𝑧

𝑠

𝑡

𝑝

𝑠

3

1

0

1

0

0

15

𝑡

2

2

2

0

1

0

20

3𝑅2 − 2𝑅1

𝑝

−6

−1

−3

0

0

1

0

𝑅3 + 2𝑅1


Solutions Section 6.3 𝑥

𝑦

𝑧

𝑠

𝑡

𝑝

𝑥 𝑡

3 0

1 4

0 6

1

−2

0 3

0 0

15 30

𝑝

0

1

−3

2

0

1

30

The most negative entry in the bottom row is the −3 in the 𝑧-column, so we use this column as the pivot column. The only positive entry in this column is the 6 in the t-row, so we pivot on that entry. 𝑥

𝑦

𝑧

𝑠

𝑡

𝑝

𝑥

3

1

0

1

0

0

15

𝑡

0

4

6

−2

3

0

30

𝑝

0

1

−3

2

0

1

30

𝑥

𝑦

𝑧

𝑠

𝑡

𝑝

𝑥 𝑧

3 0

1 4

0 6

1 −2

0 3

0 0

15 30

𝑝

0

6

0

2

3

2

90

2𝑅3 + 𝑅2

As there are no more negative numbers in the bottom row, we are done, and read off the solution: Optimal solution: 𝑝 = 90∕2 = 45; 𝑥 = 15∕3 = 5, 𝑦 = 0, 𝑧 = 30∕6 = 5. 7. Introduce slack variables and rewrite the constraints and objective function in standard form, and set up the first tableau: 𝑥+𝑦−𝑧+𝑠=3 𝑥 + 2𝑦 + 𝑧 + 𝑡 = 8 𝑥+𝑦+𝑢=5 −7𝑥 − 5𝑦 − 6𝑧 + 𝑝 = 0 𝑥

𝑦

𝑧

𝑠

𝑡

𝑢

𝑝

𝑠 𝑡 𝑢

1 1 1

1 2 1

−1 1 0

1 0 0

0 1 0

0 0 1

0 0 0

3 8 5

𝑝

−7

−5

−6

0

0

0

1

0

The most negative entry in the bottom row is the −7 in the 𝑥-column, so we use this column as the pivot column. The test ratios are: s: 3/1, t: 8/1, u: 5/1. The smallest test ratio is s: 3/1. Thus we pivot on the 1 in the s-row.


Solutions Section 6.3 𝑥

𝑦

𝑧

𝑠

𝑡

𝑢

𝑝

𝑠

1

1

−1

1

0

0

0

3

𝑡

1

2

1

0

1

0

0

8

𝑅2 − 𝑅1

𝑢

1

1

0

0

0

1

0

5

𝑅3 − 𝑅1

𝑝

−7

−5

−6

0

0

0

1

0

𝑅4 + 7𝑅1

𝑥

𝑦

𝑧

𝑠

𝑡

𝑢

𝑝

𝑥 𝑡 𝑢

1 0 0

1 1 0

−1 2 1

1 −1 −1

0 1 0

0 0 1

0 0 0

3 5 2

𝑝

0

2

−13

7

0

0

1

21

The most negative entry in the bottom row is the −13 in the 𝑧-column, so we use this column as the pivot column. The test ratios are: t: 5/2, u: 2/1. The smallest test ratio is u: 2/1. Thus we pivot on the 1 in the u-row. 𝑥

𝑦

𝑧

𝑠

𝑡

𝑢

𝑝

𝑥

1

1

−1

1

0

0

0

3

𝑅1 + 𝑅3

𝑡

0

1

2

−1

1

0

0

5

𝑅2 − 2𝑅3

𝑢

0

0

1

−1

0

1

0

2

𝑝

0

2

−13

7

0

0

1

21

𝑥

𝑦

𝑧

𝑠

𝑡

𝑢

𝑝

𝑥 𝑡 𝑧

1 0 0

1 1 0

0 0 1

0 1 −1

0 1 0

1 −2 1

0 0 0

5 1 2

𝑝

0

2

0

−6

0

13

1

47

𝑅4 + 13𝑅3

The most negative entry in the bottom row is the −6 in the 𝑠-column, so we use this column as the pivot column. The only positive entry in this column is the 1 in the t-row, so we pivot on that entry. 𝑥

𝑦

𝑧

𝑠

𝑡

𝑢

𝑝

𝑥

1

1

0

0

0

1

0

5

𝑡

0

1

0

1

1

−2

0

1

𝑧

0

0

1

−1

0

1

0

2

𝑅3 + 𝑅2

𝑝

0

2

0

−6

0

13

1

47

𝑅4 + 6𝑅2


Solutions Section 6.3 𝑥

𝑦

𝑧

𝑠

𝑡

𝑢

𝑝

𝑥 𝑠 𝑧

1 0 0

1 1 1

0 0 1

0 1 0

0 1 1

1

−2 −1

0 0 0

5 1 3

𝑝

0

8

0

0

6

1

1

53

As there are no more negative numbers in the bottom row, we are done, and read off the solution: Optimal solution: 𝑝 = 53∕1 = 53; 𝑥 = 5∕1 = 5, 𝑦 = 0, 𝑧 = 3∕1 = 3. 8. Introduce slack variables and rewrite the constraints and objective function in standard form, and set up the first tableau: 3𝑥 + 𝑦 + 𝑧 + 𝑠 = 5 𝑥 + 2𝑦 + 𝑧 + 𝑡 = 5 𝑥+𝑦+𝑧+𝑢=4 −3𝑥 − 4𝑦 − 2𝑧 + 𝑝 = 0 𝑥

𝑦

𝑧

𝑠

𝑡

𝑢

𝑝

𝑠 𝑡 𝑢

3 1 1

1 2 1

1 1 1

1 0 0

0 1 0

0 0 1

0 0 0

5 5 4

𝑝

−3

−4

−2

0

0

0

1

0

The most negative entry in the bottom row is the −4 in the 𝑦-column, so we use this column as the pivot column. The test ratios are: s: 5/1, t: 5/2, u: 4/1. The smallest test ratio is t: 5/2. Thus we pivot on the 2 in the t-row. 𝑥

𝑦

𝑧

𝑠

𝑡

𝑢

𝑝

𝑠

3

1

1

1

0

0

0

5

𝑡

1

2

1

0

1

0

0

5

𝑢

1

1

1

0

0

1

0

4

2𝑅3 − 𝑅2

𝑝

−3

−4

−2

0

0

0

1

0

𝑅4 + 2𝑅2

𝑥

𝑦

𝑧

𝑠

𝑡

𝑢

𝑝

𝑠 𝑦 𝑢

5 1 1

0 2 0

1 1 1

2 0 0

−1 1 −1

0 0 2

0 0 0

5 5 3

𝑝

−1

0

0

0

2

0

1

10

2𝑅1 − 𝑅2

The most negative entry in the bottom row is the −1 in the 𝑥-column, so we use this column as the pivot column. The test ratios are: s: 5/5, y: 5/1, u: 3/1. The smallest test ratio is s: 5/5. Thus we pivot on the 5 in the s-row.


Solutions Section 6.3 𝑥

𝑦

𝑧

𝑠

𝑡

𝑢

𝑝

𝑠

5

0

1

2

−1

0

0

5

𝑦

1

2

1

0

1

0

0

5

5𝑅2 − 𝑅1

𝑢

1

0

1

0

−1

2

0

3

5𝑅3 − 𝑅1

𝑝

−1

0

0

0

2

0

1

10

5𝑅4 + 𝑅1

𝑥

𝑦

𝑧

𝑠

𝑡

𝑢

𝑝

𝑥 𝑦 𝑢

5 0 0

0 10 0

1 4 4

2 −2 −2

−1 6 −4

0 0 10

0 0 0

5 20 10

𝑝

0

0

1

2

9

0

5

55

As there are no more negative numbers in the bottom row, we are done, and read off the solution: Optimal solution: 𝑝 = 55∕5 = 11; 𝑥 = 5∕5 = 1, 𝑦 = 20∕10 = 2, 𝑧 = 0. 9. Introduce slack variables and rewrite the constraints and objective function in standard form, and set up the first tableau: 5𝑥1 − 𝑥2 + 𝑥3 + 𝑠 = 1,500 2𝑥1 + 2𝑥2 + 𝑥3 + 𝑡 = 2,500 4𝑥1 + 2𝑥2 + 𝑥3 + 𝑢 = 2,000 −3𝑥1 − 7𝑥2 − 8𝑥3 + 𝑧 = 0 𝑥1

𝑥2

𝑥3

𝑠

𝑡

𝑢

𝑧

𝑠 𝑡 𝑢

5 2 4

−1 2 2

1 1 1

1 0 0

0 1 0

0 0 1

0 0 0

1500 2500 2000

𝑧

−3

−7

−8

0

0

0

1

0

The most negative entry in the bottom row is the −8 in the 𝑥3 -column, so we use this column as the pivot column. The test ratios are: s: 1500/1, t: 2500/1, u: 2000/1. The smallest test ratio is s: 1500/1. Thus we pivot on the 1 in the s-row. 𝑥1

𝑥2

𝑥3

𝑠

𝑡

𝑢

𝑧

𝑠

5

−1

1

1

0

0

0

1500

𝑡

2

2

1

0

1

0

0

2500

𝑅2 − 𝑅1

𝑢

4

2

1

0

0

1

0

2000

𝑅3 − 𝑅1

𝑧

−3

−7

−8

0

0

0

1

0

𝑅4 + 8𝑅1


Solutions Section 6.3 𝑥1

𝑥2

𝑥3

𝑠

𝑡

𝑢

𝑧

𝑥3

5

−1

1

1

0

0

0

1500

𝑡 𝑢

−3 −1

3 3

0 0

−1 −1

1 0

0 1

0 0

1000 500

𝑧

37

−15

0

8

0

0

1

12000

The most negative entry in the bottom row is the −15 in the 𝑥2 -column, so we use this column as the pivot column. The test ratios are: t: 1000/3, u: 500/3. The smallest test ratio is u: 500/3. Thus we pivot on the 3 in the u-row. 𝑥1

𝑥2

𝑥3

𝑠

𝑡

𝑢

𝑧

𝑥3

5

−1

1

1

0

0

0

1500

3𝑅1 + 𝑅3

𝑡

−3

3

0

−1

1

0

0

1000

𝑅2 − 𝑅3

𝑢

−1

3

0

−1

0

1

0

500

𝑧

37

−15

0

8

0

0

1

12000

𝑥1

𝑥2

𝑥3

𝑠

𝑡

𝑢

𝑧

𝑥3

14

0

3

2

0

1

0

5000

𝑡

−2

0

0

0

1

−1

0

500

−1

3

0

−1

0

1

0

500

32

0

0

3

0

5

1

14500

𝑥2

𝑧

𝑅4 + 5𝑅3

As there are no more negative numbers in the bottom row, we are done, and read off the solution: Optimal solution: 𝑧 = 14,500∕1 = 14,500; 𝑥1 = 0, 𝑥2 = 500∕3, 𝑥3 = 5,000∕3. 10. Introduce slack variables and rewrite the constraints and objective function in standard form, and set up the first tableau: 5𝑥1 − 𝑥2 + 𝑥3 + 𝑠 = 1,500 2𝑥1 + 2𝑥2 + 𝑥3 + 𝑡 = 2,500 4𝑥1 + 2𝑥2 + 𝑥3 + 𝑢 = 2,000 −3𝑥1 − 4𝑥2 − 6𝑥3 + 𝑧 = 0 𝑥1

𝑥2

𝑥3

𝑠

𝑡

𝑢

𝑧

𝑠 𝑡 𝑢

5 2 4

−1 2 2

1 1 1

1 0 0

0 1 0

0 0 1

0 0 0

1500 2500 2000

𝑧

−3

−4

−6

0

0

0

1

0

The most negative entry in the bottom row is the −6 in the 𝑥3 -column, so we use this column as the pivot column. The test ratios are: s: 1500/1, t: 2500/1, u: 2000/1. The smallest test ratio is s: 1500/1. Thus we pivot on the 1 in the s-row.


Solutions Section 6.3 𝑥1

𝑥2

𝑥3

𝑠

𝑡

𝑢

𝑧

𝑠

5

−1

1

1

0

0

0

1500

𝑡

2

2

1

0

1

0

0

2500

𝑅2 − 𝑅1

𝑢

4

2

1

0

0

1

0

2000

𝑅3 − 𝑅1

𝑧

−3

−4

−6

0

0

0

1

0

𝑅4 + 6𝑅1

𝑥1

𝑥2

𝑥3

𝑠

𝑡

𝑢

𝑧

𝑥3

5

−1

1

1

0

0

0

1500

𝑡 𝑢

−3 −1

3 3

0 0

−1 −1

1 0

0 1

0 0

1000 500

𝑧

27

−10

0

6

0

0

1

9000

The most negative entry in the bottom row is the −10 in the 𝑥2 -column, so we use this column as the pivot column. The test ratios are: t: 1000/3, u: 500/3. The smallest test ratio is u: 500/3. Thus we pivot on the 3 in the u-row. 𝑥1

𝑥2

𝑥3

𝑠

𝑡

𝑢

𝑧

𝑥3

5

−1

1

1

0

0

0

1500

3𝑅1 + 𝑅3

𝑡

−3

3

0

−1

1

0

0

1000

𝑅2 − 𝑅3

𝑢

−1

3

0

−1

0

1

0

500

𝑧

27

−10

0

6

0

0

1

9000

𝑥1

𝑥2

𝑥3

𝑠

𝑡

𝑢

𝑧

𝑥3

14

0

3

2

0

1

0

5000

𝑡

−2

0

0

0

1

−1

0

500

−1

3

0

−1

0

1

0

500

71

0

0

8

0

10

3

32000

𝑥2

𝑧

3𝑅4 + 10𝑅3

As there are no more negative numbers in the bottom row, we are done, and read off the solution: Optimal solution: 𝑧 = 32,000∕3; 𝑥1 = 0, 𝑥2 = 500∕3, 𝑥3 = 5,000∕3. 11. Introduce slack variables and rewrite the constraints and objective function in standard form, and set up the first tableau: 𝑥+𝑦+𝑧+𝑠=3 𝑦+𝑧+𝑤+𝑡=4 𝑥+𝑧+𝑤+𝑢=5 𝑥+𝑦+𝑤+𝑣=6 −𝑥 − 𝑦 − 𝑧 − 𝑤 + 𝑝 = 0


Solutions Section 6.3 𝑥

𝑦

𝑧

𝑤

𝑠

𝑡

𝑢

𝑣

𝑝

𝑠 𝑡 𝑢 𝑣

1 0 1 1

1 1 0 1

1 1 1 0

0 1 1 1

1 0 0 0

0 1 0 0

0 0 1 0

0 0 0 1

0 0 0 0

3 4 5 6

𝑝

−1

−1

−1

−1

0

0

0

0

1

0

The most negative entry in the bottom row is the −1 in the 𝑥-column, so we use this column as the pivot column. The test ratios are: s: 3/1, u: 5/1, v: 6/1. The smallest test ratio is s: 3/1. Thus we pivot on the 1 in the s-row. 𝑥

𝑦

𝑧

𝑤

𝑠

𝑡

𝑢

𝑣

𝑝

𝑠

1

1

1

0

1

0

0

0

0

3

𝑡

0

1

1

1

0

1

0

0

0

4

𝑢

1

0

1

1

0

0

1

0

0

5

𝑅3 − 𝑅1

𝑣

1

1

0

1

0

0

0

1

0

6

𝑅4 − 𝑅1

𝑝

−1

−1

−1

−1

0

0

0

0

1

0

𝑅5 + 𝑅1

𝑥

𝑦

𝑧

𝑤

𝑠

𝑡

𝑢

𝑣

𝑝

𝑥 𝑡 𝑢 𝑣

1 0 0 0

1 1 −1 0

1 1 0

−1

0 1 1 1

1 0 −1 −1

0 1 0 0

0 0 1 0

0 0 0 1

0 0 0 0

3 4 2 3

𝑝

0

0

0

−1

1

0

0

0

1

3

The most negative entry in the bottom row is the −1 in the 𝑤-column, so we use this column as the pivot column. The test ratios are: t: 4/1, u: 2/1, v: 3/1. The smallest test ratio is u: 2/1. Thus we pivot on the 1 in the u-row. 𝑥

𝑦

𝑧

𝑤

𝑠

𝑡

𝑢

𝑣

𝑝

𝑥

1

1

1

0

1

0

0

0

0

3

𝑡

0

1

1

1

0

1

0

0

0

4

𝑢

0

−1

0

1

−1

0

1

0

0

2

𝑣

0

0

−1

1

−1

0

0

1

0

3

𝑅4 − 𝑅3

𝑝

0

0

0

−1

1

0

0

0

1

3

𝑅5 + 𝑅3

𝑅2 − 𝑅3


Solutions Section 6.3 𝑥

𝑦

𝑧

𝑤

𝑠

𝑡

𝑢

𝑣

𝑝

𝑥 𝑡 𝑤 𝑣

1 0 0 0

1 2 −1 1

1 1 0 −1

0 0 1 0

1 1 −1 0

0 1 0 0

0

−1 1 −1

0 0 0 1

0 0 0 0

3 2 2 1

𝑝

0

−1

0

0

0

0

1

0

1

5

The most negative entry in the bottom row is the −1 in the 𝑦-column, so we use this column as the pivot column. The test ratios are: x: 3/1, t: 2/2, v: 1/1. The smallest test ratio is t: 2/2. Thus we pivot on the 2 in the t-row. 𝑥

𝑦

𝑧

𝑤

𝑠

𝑡

𝑢

𝑣

𝑝

𝑥

1

1

1

0

1

0

0

0

0

3

𝑡

0

2

1

0

1

1

−1

0

0

2

𝑤

0

−1

0

1

−1

0

1

0

0

2

2𝑅3 + 𝑅2

𝑣

0

1

−1

0

0

0

−1

1

0

1

2𝑅4 − 𝑅2

𝑝

0

−1

0

0

0

0

1

0

1

5

2𝑅5 + 𝑅2

𝑥

𝑦

𝑧

𝑤

𝑠

𝑡

𝑢

𝑣

𝑝

𝑥 𝑦 𝑤 𝑣

2 0 0 0

0 2 0 0

1 1 1 −3

0 0 2 0

1 1 −1 −1

−1 1 1 −1

1 −1 1 −1

0 0 0 2

0 0 0 0

4 2 6 0

𝑝

0

0

1

0

1

1

1

0

2

12

2𝑅1 − 𝑅2

As there are no more negative numbers in the bottom row, we are done, and read off the solution: Optimal solution: 𝑝 = 12∕2 = 6; 𝑥 = 4∕2 = 2, 𝑦 = 2∕2 = 1, 𝑧 = 0, 𝑤 = 6∕2 = 3. 12. Introduce slack variables and rewrite the constraints and objective function in standard form, and set up the first tableau: 𝑥+𝑦+𝑧+𝑠=3 𝑦+𝑧+𝑤+𝑡=3 𝑥+𝑧+𝑤+𝑢=4 𝑥+𝑦+𝑤+𝑣=4 −𝑥 + 𝑦 − 𝑧 − 𝑤 + 𝑝 = 0 𝑥

𝑦

𝑧

𝑤

𝑠

𝑡

𝑢

𝑣

𝑝

𝑠 𝑡 𝑢 𝑣

1 0 1 1

1 1 0 1

1 1 1 0

0 1 1 1

1 0 0 0

0 1 0 0

0 0 1 0

0 0 0 1

0 0 0 0

3 3 4 4

𝑝

−1

1

−1

−1

0

0

0

0

1

0


Solutions Section 6.3 The most negative entry in the bottom row is the −1 in the 𝑥-column, so we use this column as the pivot column. The test ratios are: s: 3/1, u: 4/1, v: 4/1. The smallest test ratio is s: 3/1. Thus we pivot on the 1 in the s-row. 𝑥

𝑦

𝑧

𝑤

𝑠

𝑡

𝑢

𝑣

𝑝

𝑠

1

1

1

0

1

0

0

0

0

3

𝑡

0

1

1

1

0

1

0

0

0

3

𝑢

1

0

1

1

0

0

1

0

0

4

𝑅3 − 𝑅1

𝑣

1

1

0

1

0

0

0

1

0

4

𝑅4 − 𝑅1

𝑝

−1

1

−1

−1

0

0

0

0

1

0

𝑅5 + 𝑅1

𝑥

𝑦

𝑧

𝑤

𝑠

𝑡

𝑢

𝑣

𝑝

𝑥 𝑡 𝑢 𝑣

1 0 0 0

1 1 −1 0

1 1 0 −1

0 1 1 1

1 0 −1 −1

0 1 0 0

0 0 1 0

0 0 0 1

0 0 0 0

3 3 1 1

𝑝

0

2

0

−1

1

0

0

0

1

3

The most negative entry in the bottom row is the −1 in the 𝑤-column, so we use this column as the pivot column. The test ratios are: t: 3/1, u: 1/1, v: 1/1. The smallest test ratio is u: 1/1. Thus we pivot on the 1 in the u-row. 𝑥

𝑦

𝑧

𝑤

𝑠

𝑡

𝑢

𝑣

𝑝

𝑥

1

1

1

0

1

0

0

0

0

3

𝑡

0

1

1

1

0

1

0

0

0

3

𝑢

0

−1

0

1

−1

0

1

0

0

1

𝑣

0

0

−1

1

−1

0

0

1

0

1

𝑅4 − 𝑅3

𝑝

0

2

0

−1

1

0

0

0

1

3

𝑅5 + 𝑅3

𝑥

𝑦

𝑧

𝑤

𝑠

𝑡

𝑢

𝑣

𝑝

𝑥 𝑡 𝑤 𝑣

1 0 0 0

1 2 −1 1

1 1 0 −1

0 0 1 0

1 1 −1 0

0 1 0 0

0 −1 1 −1

0 0 0 1

0 0 0 0

3 2 1 0

𝑝

0

1

0

0

0

0

1

0

1

4

𝑅2 − 𝑅3

As there are no more negative numbers in the bottom row, we are done, and read off the solution: Optimal solution: 𝑝 = 4∕1 = 4; 𝑥 = 3∕1 = 3, 𝑦 = 0, 𝑧 = 0, 𝑤 = 1∕1 = 1. Other solutions include 𝑥 = 1, 𝑦 = 0, 𝑧 = 0, 𝑤 = 3 and 𝑥 = 1, 𝑦 = 0, 𝑧 = 2, 𝑤 = 1 13. Introduce slack variables and rewrite the constraints and objective function in standard form, and set up the


Solutions Section 6.3 first tableau: 𝑥+𝑦+𝑠=1 𝑦+𝑧+𝑡=2 𝑧+𝑤+𝑟=3 𝑤+𝑣+𝑞=4 −𝑥 − 𝑦 − 𝑧 − 𝑤 − 𝑣 + 𝑝 = 0 𝑥

𝑦

𝑧

𝑤

𝑣

𝑠

𝑡

𝑟

𝑞

𝑝

𝑠 𝑡 𝑟 𝑞

1 0 0 0

1 1 0 0

0 1 1 0

0 0 1 1

0 0 0 1

1 0 0 0

0 1 0 0

0 0 1 0

0 0 0 1

0 0 0 0

1 2 3 4

𝑝

−1

−1

−1

−1

−1

0

0

0

0

1

0

The most negative entry in the bottom row is the −1 in the 𝑥-column, so we use this column as the pivot column. The only positive entry in this column is the 1 in the s-row, so we pivot on that entry. 𝑥

𝑦

𝑧

𝑤

𝑣

𝑠

𝑡

𝑟

𝑞

𝑝

𝑠

1

1

0

0

0

1

0

0

0

0

1

𝑡 𝑟 𝑞

0 0 0

1 0 0

1 1 0

0 1 1

0 0 1

0 0 0

1 0 0

0 1 0

0 0 1

0 0 0

2 3 4

𝑝

−1

−1

−1

−1

−1

0

0

0

0

1

0

𝑥

𝑦

𝑧

𝑤

𝑣

𝑠

𝑡

𝑟

𝑞

𝑝

𝑥 𝑡 𝑟 𝑞

1 0 0 0

1 1 0 0

0 1 1 0

0 0 1 1

0 0 0 1

1 0 0 0

0 1 0 0

0 0 1 0

0 0 0 1

0 0 0 0

1 2 3 4

𝑝

0

0

−1

−1

−1

1

0

0

0

1

1

𝑅5 + 𝑅1

The most negative entry in the bottom row is the −1 in the 𝑧-column, so we use this column as the pivot column. The test ratios are: t: 2/1, r: 3/1. The smallest test ratio is t: 2/1. Thus we pivot on the 1 in the t-row. 𝑥

𝑦

𝑧

𝑤

𝑣

𝑠

𝑡

𝑟

𝑞

𝑝

𝑥

1

1

0

0

0

1

0

0

0

0

1

𝑡

0

1

1

0

0

0

1

0

0

0

2

𝑟

0

0

1

1

0

0

0

1

0

0

3

𝑞

0

0

0

1

1

0

0

0

1

0

4

𝑝

0

0

−1

−1

−1

1

0

0

0

1

1

𝑅3 − 𝑅2 𝑅5 + 𝑅2


Solutions Section 6.3 𝑥

𝑦

𝑧

𝑤

𝑣

𝑠

𝑡

𝑟

𝑞

𝑝

𝑥 𝑧 𝑟 𝑞

1 0 0 0

1 1 −1 0

0 1 0 0

0 0 1 1

0 0 0 1

1 0 0 0

0 1 −1 0

0 0 1 0

0 0 0 1

0 0 0 0

1 2 1 4

𝑝

0

1

0

−1

−1

1

1

0

0

1

3

The most negative entry in the bottom row is the −1 in the 𝑤-column, so we use this column as the pivot column. The test ratios are: r: 1/1, q: 4/1. The smallest test ratio is r: 1/1. Thus we pivot on the 1 in the r-row. 𝑥

𝑦

𝑧

𝑤

𝑣

𝑠

𝑡

𝑟

𝑞

𝑝

𝑥 𝑧

1 0

1 1

0 1

0 0

0 0

1 0

0 1

0 0

0 0

0 0

1 2

𝑟

0

−1

0

1

0

0

−1

1

0

0

1

𝑞

0

0

0

1

1

0

0

0

1

0

4

𝑅4 − 𝑅3

𝑝

0

1

0

−1

−1

1

1

0

0

1

3

𝑅5 + 𝑅3

𝑥

𝑦

𝑧

𝑤

𝑣

𝑠

𝑡

𝑟

𝑞

𝑝

𝑥 𝑧 𝑤 𝑞

1 0 0 0

1 1 −1 1

0 1 0 0

0 0 1 0

0 0 0 1

1 0 0 0

0 1 −1 1

0 0 1 −1

0 0 0 1

0 0 0 0

1 2 1 3

𝑝

0

0

0

0

−1

1

0

1

0

1

4

The most negative entry in the bottom row is the −1 in the 𝑣-column, so we use this column as the pivot column. The only positive entry in this column is the 1 in the q-row, so we pivot on that entry. 𝑥

𝑦

𝑧

𝑤

𝑣

𝑠

𝑡

𝑟

𝑞

𝑝

𝑥 𝑧 𝑤

1 0 0

1 1 −1

0 1 0

0 0 1

0 0 0

1 0 0

0 1 −1

0 0 1

0 0 0

0 0 0

1 2 1

𝑞

0

1

0

0

1

0

1

−1

1

0

3

𝑝

0

0

0

0

−1

1

0

1

0

1

4

𝑥

𝑦

𝑧

𝑤

𝑣

𝑠

𝑡

𝑟

𝑞

𝑝

𝑥 𝑧 𝑤 𝑣

1 0 0 0

1 1 −1 1

0 1 0 0

0 0 1 0

0 0 0 1

1 0 0 0

0 1 −1 1

0 0 1 −1

0 0 0 1

0 0 0 0

1 2 1 3

𝑝

0

1

0

0

0

1

1

0

1

1

7

𝑅5 + 𝑅4


Solutions Section 6.3 As there are no more negative numbers in the bottom row, we are done, and read off the solution: Optimal solution: 𝑝 = 7∕1 = 7; 𝑥 = 1∕1 = 1, 𝑦 = 0, 𝑧 = 2∕1 = 2, 𝑤 = 1∕1 = 1, 𝑣 = 3∕1 = 3. Another solution is 𝑥 = 1, 𝑦 = 0, 𝑧 = 2, 𝑤 = 0, 𝑣 = 4. 14. Introduce slack variables and rewrite the constraints and objective function in standard form, and set up the first tableau: 𝑥+𝑦+𝑠=1 𝑦+𝑧+𝑡=2 𝑧+𝑤+𝑟=3 𝑤+𝑣+𝑞=4 −𝑥 − 2𝑦 − 𝑧 − 2𝑤 − 𝑣 + 𝑝 = 0 𝑥

𝑦

𝑧

𝑤

𝑣

𝑠

𝑡

𝑟

𝑞

𝑝

𝑠 𝑡 𝑟 𝑞

1 0 0 0

1 1 0 0

0 1 1 0

0 0 1 1

0 0 0 1

1 0 0 0

0 1 0 0

0 0 1 0

0 0 0 1

0 0 0 0

1 2 3 4

𝑝

−1

−2

−1

−2

−1

0

0

0

0

1

0

The most negative entry in the bottom row is the −2 in the 𝑦-column, so we use this column as the pivot column. The test ratios are: s: 1/1, t: 2/1. The smallest test ratio is s: 1/1. Thus we pivot on the 1 in the s-row. 𝑥

𝑦

𝑧

𝑤

𝑣

𝑠

𝑡

𝑟

𝑞

𝑝

𝑠

1

1

0

0

0

1

0

0

0

0

1

𝑡

0

1

1

0

0

0

1

0

0

0

2

𝑟 𝑞

0 0

0 0

1 0

1 1

0 1

0 0

0 0

1 0

0 1

0 0

3 4

𝑝

−1

−2

−1

−2

−1

0

0

0

0

1

0

𝑥

𝑦

𝑧

𝑤

𝑣

𝑠

𝑡

𝑟

𝑞

𝑝

𝑦 𝑡 𝑟 𝑞

1 −1 0 0

1 0 0 0

0 1 1 0

0 0 1 1

0 0 0 1

1 −1 0 0

0 1 0 0

0 0 1 0

0 0 0 1

0 0 0 0

1 1 3 4

𝑝

1

0

−1

−2

−1

2

0

0

0

1

2

𝑅2 − 𝑅1

𝑅5 + 2𝑅1

The most negative entry in the bottom row is the −2 in the 𝑤-column, so we use this column as the pivot column. The test ratios are: r: 3/1, q: 4/1. The smallest test ratio is r: 3/1. Thus we pivot on the 1 in the r-row.


Solutions Section 6.3 𝑥

𝑦

𝑧

𝑤

𝑣

𝑠

𝑡

𝑟

𝑞

𝑝

𝑦 𝑡

1

−1

1 0

0 1

0 0

0 0

1

−1

0 1

0 0

0 0

0 0

1 1

𝑟

0

0

1

1

0

0

0

1

0

0

3

𝑞

0

0

0

1

1

0

0

0

1

0

4

𝑅4 − 𝑅3

𝑝

1

0

−1

−2

−1

2

0

0

0

1

2

𝑅5 + 2𝑅3

𝑥

𝑦

𝑧

𝑤

𝑣

𝑠

𝑡

𝑟

𝑞

𝑝

𝑦 𝑡 𝑤 𝑞

1 −1 0 0

1 0 0 0

0 1 1 −1

0 0 1 0

0 0 0 1

1 −1 0 0

0 1 0 0

0 0 1 −1

0 0 0 1

0 0 0 0

1 1 3 1

𝑝

1

0

1

0

−1

2

0

2

0

1

8

The most negative entry in the bottom row is the −1 in the 𝑣-column, so we use this column as the pivot column. The only positive entry in this column is the 1 in the q-row, so we pivot on that entry. 𝑥

𝑦

𝑧

𝑤

𝑣

𝑠

𝑡

𝑟

𝑞

𝑝

𝑦 𝑡 𝑤

1

−1 0

1 0 0

0 1 1

0 0 1

0 0 0

1

−1 0

0 1 0

0 0 1

0 0 0

0 0 0

1 1 3

𝑞

0

0

−1

0

1

0

0

−1

1

0

1

𝑝

1

0

1

0

−1

2

0

2

0

1

8

𝑥

𝑦

𝑧

𝑤

𝑣

𝑠

𝑡

𝑟

𝑞

𝑝

𝑦 𝑡 𝑤 𝑣

1 −1 0 0

1 0 0 0

0 1 1 −1

0 0 1 0

0 0 0 1

1 −1 0 0

0 1 0 0

0 0 1 −1

0 0 0 1

0 0 0 0

1 1 3 1

𝑝

1

0

0

0

0

2

0

1

1

1

9

𝑅5 + 𝑅4

As there are no more negative numbers in the bottom row, we are done, and read off the solution: Optimal solution: 𝑝 = 9∕1 = 9; 𝑥 = 0, 𝑦 = 1∕1 = 1, 𝑧 = 0, 𝑤 = 3∕1 = 3, 𝑣 = 1∕1 = 1. Another solution is 𝑥 = 0, 𝑦 = 1, 𝑧 = 1, 𝑤 = 2, 𝑣 = 2. 15. You can use the online Pivot and Gauss-Jordan Tool (or the Excel version) in decimal mode to do the pivoting. Introduce slack variables and rewrite the constraints and objective function in standard form, and set up the first tableau: 0.1𝑥 + 𝑦 − 2.2𝑧 + 𝑠 = 4.5 2.1𝑥 + 𝑦 + 𝑧 + 𝑡 = 8 𝑥 + 2.2𝑦 + 𝑢 = 5 −2.5𝑥 − 4.2𝑦 − 2𝑧 + 𝑝 = 0


Solutions Section 6.3 𝑥

𝑦

𝑧

𝑠

𝑡

𝑢

𝑝

𝑠 𝑡 𝑢

0.1 2.1 1

1 1 2.2

−2.2 1 0

1 0 0

0 1 0

0 0 1

0 0 0

4.5 8 5

𝑝

−2.5

−4.2

−2

0

0

0

1

0

The most negative entry in the bottom row is the −4.2 in the 𝑦-column, so we use this column as the pivot column. The test ratios are: s: 4.5/1, t: 8/1, u: 5/2.2. The smallest test ratio is u: 5/2.2. Thus we pivot on the 2.2 in the u-row. 𝑥

𝑦

𝑧

𝑠

𝑡

𝑢

𝑝

𝑠 𝑡

0.1 2.1

1 1

−2.2 1

1 0

0 1

0 0

0 0

4.5 8

𝑢

1

2.2

0

0

0

1

0

5

𝑝

−2.5

−4.2

−2

0

0

0

1

0

𝑥

𝑦

𝑧

𝑠

𝑡

𝑢

𝑝

𝑠 −0.35 𝑡 1.65 𝑦 0.45

0 0 1

−2.2 1 0

1 0 0

0 1 0

−0.45 −0.45 0.45

0 0 0

2.23 5.73 2.27

𝑝 −0.59

0

−2

0

0

1.91

1

9.55

The most negative entry in the bottom row is the −2 in the 𝑧-column, so we use this column as the pivot column. The only positive entry in this column is the 1 in the t-row, so we pivot on that entry. 𝑥

𝑦

𝑧

𝑠

𝑡

𝑢

𝑝

𝑠 −0.35

0

−2.2

1

0

−0.45

0

2.23

𝑡

1.65

0

1

0

1

−0.45

0

5.73

𝑦

0.45

1

0

0

0

0.45

0

2.27

𝑝 −0.59

0

−2

0

0

1.91

1

9.55

𝑥

𝑦

𝑧

𝑠

𝑡

𝑢

𝑝

𝑠 𝑧 𝑦

3.27 1.65 0.45

0 0 1

0 1 0

1 0 0

2.2 1 0

−1.45 −0.45 0.45

0 0 0

14.83 5.73 2.27

𝑝

2.7

0

0

0

2

1

1

21

As there are no more negative numbers in the bottom row, we are done, and read off the solution: Optimal solution: 𝑝 = 21; 𝑥 = 0, 𝑦 = 2.27, 𝑧 = 5.73.


Solutions Section 6.3 16. You can use the online Pivot and Gauss-Jordan Tool (or the Excel version) in decimal mode to do the pivoting. Introduce slack variables and rewrite the constraints and objective function in standard form, and set up the first tableau: 3.1𝑥 + 1.2𝑦 + 𝑧 + 𝑠 = 5.5 𝑥 + 2.3𝑦 + 𝑧 + 𝑡 = 5.5 2.1𝑥 + 𝑦 + 2.3𝑧 + 𝑢 = 5.2 −2.1𝑥 − 4.1𝑦 − 2𝑧 + 𝑝 = 0 𝑥

𝑦

𝑧

𝑠

𝑡

𝑢

𝑝

𝑠 𝑡 𝑢

3.1 1 2.1

1.2 2.3 1

1 1 2.3

1 0 0

0 1 0

0 0 1

0 0 0

5.5 5.5 5.2

𝑝

−2.1

−4.1

−2

0

0

0

1

0

The most negative entry in the bottom row is the −4.1 in the 𝑦-column, so we use this column as the pivot column. The test ratios are: s: 5.5/1.2, t: 5.5/2.3, u: 5.2/1. The smallest test ratio is t: 5.5/2.3. Thus we pivot on the 2.3 in the t-row. 𝑥

𝑦

𝑧

𝑠

𝑡

𝑢

𝑝

𝑠

3.1

1.2

1

1

0

0

0

5.5

𝑡

1

2.3

1

0

1

0

0

5.5

𝑢

2.1

1

2.3

0

0

1

0

5.2

𝑝

−2.1

−4.1

−2

0

0

0

1

0

𝑥

𝑦

𝑧

𝑠

𝑡

𝑢

𝑝

2.58 0.43 1.67

0 1 0

0.48 0.43 1.87

1 0 0

−0.52 0.43 −0.43

0 0 1

0 0 0

2.63 2.39 2.81

𝑝 −0.32

0

−0.22

0

1.78

0

1

9.8

𝑠 𝑦 𝑢

The most negative entry in the bottom row is the −0.32 in the 𝑥-column, so we use this column as the pivot column. The test ratios are: s: 2.63/2.58, y: 2.39/0.43, u: 2.81/1.67. The smallest test ratio is s: 2.63/2.58. Thus we pivot on the 2.58 in the s-row. 𝑥

𝑦

𝑧

𝑠

𝑡

𝑢

𝑝

𝑠

2.58

0

0.48

1

−0.52

0

0

2.63

𝑦 𝑢

0.43 1.67

1 0

0.43 1.87

0 0

0.43 −0.43

0 1

0 0

2.39 2.81

𝑝 −0.32

0

−0.22

0

1.78

0

1

9.8


Solutions Section 6.3 𝑥

𝑦

𝑧

𝑠

𝑥 𝑦 𝑢

1 0 0

0 1 0

0.19 0.35 1.56

0.39

𝑝

0

0

−0.16

𝑡

𝑢

𝑝

−0.2 −0.17 0.52 −0.65 −0.1

0 0 1

0 0 0

1.02 1.95 1.11

0.12

0

1

10.13

1.72

The most negative entry in the bottom row is the −0.16 in the 𝑧-column, so we use this column as the pivot column. The test ratios are: x: 1.02/0.19, y: 1.95/0.35, u: 1.11/1.56. The smallest test ratio is u: 1.11/1.56. Thus we pivot on the 1.56 in the u-row. 𝑥

𝑦

𝑧

𝑥 𝑦

1 0

0 1

0.19 0.35

𝑢

0

0

𝑝

0

𝑠

𝑡

𝑢

𝑝

0.39 −0.2 −0.17 0.52

0 0

0 0

1.02 1.95

1.56

−0.65 −0.1

1

0

1.11

0

−0.16

0.12

1.72

0

1

10.13

𝑥

𝑦

𝑧

𝑠

𝑡

𝑢

𝑝

𝑥 𝑦 𝑧

1 0 0

0 1 0

0 0 1

0.46 −0.19 −0.12 −0.02 0.55 −0.23 −0.41 −0.06 0.64

0 0 0

0.89 1.7 0.71

𝑝

0

0

0

0.06

1

10.24

1.71

0.1

As there are no more negative numbers in the bottom row, we are done, and read off the solution: Optimal solution: 𝑝 = 10.24; 𝑥 = 0.89, 𝑦 = 1.7, 𝑧 = 0.71. 17. You can use the online Pivot and Gauss-Jordan Tool (or the Excel version) in decimal mode to do the pivoting. Introduce slack variables and rewrite the constraints and objective function in standard form, and set up the first tableau: 𝑥 + 2𝑦 + 3𝑧 + 𝑠 = 3 𝑦 + 𝑧 + 2.2𝑤 + 𝑡 = 4 𝑥 + 𝑧 + 2.2𝑤 + 𝑢 = 5 𝑥 + 𝑦 + 2.2𝑤 + 𝑣 = 6 −𝑥 − 2𝑦 − 3𝑧 − 𝑤 + 𝑝 = 0 𝑥

𝑦

𝑧

𝑤

𝑠

𝑡

𝑢

𝑣

𝑝

𝑠 𝑡 𝑢 𝑣

1 0 1 1

2 1 0 1

3 1 1 0

0 2.2 2.2 2.2

1 0 0 0

0 1 0 0

0 0 1 0

0 0 0 1

0 0 0 0

3 4 5 6

𝑝

−1

−2

−3

−1

0

0

0

0

1

0

The most negative entry in the bottom row is the −3 in the 𝑧-column, so we use this column as the pivot column. The test ratios are: s: 3/3, t: 4/1, u: 5/1. The smallest test ratio is s: 3/3. Thus we pivot on the 3 in the s-row.


Solutions Section 6.3 𝑥

𝑦

𝑧

𝑤

𝑠

𝑡

𝑢

𝑣

𝑝

𝑠

1

2

3

0

1

0

0

0

0

3

𝑡 𝑢 𝑣

0 1 1

1 0 1

1 1 0

2.2 2.2 2.2

0 0 0

1 0 0

0 1 0

0 0 1

0 0 0

4 5 6

𝑝

−1

−2

−3

−1

0

0

0

0

1

0

𝑥

𝑦

𝑧

𝑤

𝑠

𝑡

𝑢

𝑣

𝑝

𝑧 0.33 0.67 𝑡 −0.33 0.33 𝑢 0.67 −0.67 𝑣 1 1

1 0 0 0

0 2.2 2.2 2.2

0.33 −0.33 −0.33 0

0 1 0 0

0 0 1 0

0 0 0 1

0 0 0 0

1 3 4 6

𝑝

0

−1

1

0

0

0

1

3

0

0

The most negative entry in the bottom row is the −1 in the 𝑤-column, so we use this column as the pivot column. The test ratios are: t: 3/2.2, u: 4/2.2, v: 6/2.2. The smallest test ratio is t: 3/2.2. Thus we pivot on the 2.2 in the t-row. 𝑥

𝑦

𝑧

𝑤

𝑠

𝑡

𝑢

𝑣

𝑝

0.33

0.67

1

0

0.33

0

0

0

0

1

𝑡 −0.33

0.33

0

2.2

−0.33

1

0

0

0

3

𝑢 𝑣

0.67 1

−0.67 1

0 0

2.2 2.2

−0.33 0

0 0

1 0

0 1

0 0

4 6

𝑝

0

0

0

−1

1

0

0

0

1

3

𝑥

𝑦

𝑧

𝑤

𝑠

𝑡

𝑢

𝑣

𝑝

𝑧 0.33 𝑤 −0.15 𝑢 1 𝑣 1.33

0.67 0.15 −1 0.67

1 0 0 0

0 1 0 0

0.33 −0.15 0 0.33

0 0.45 −1 −1

0 0 1 0

0 0 0 1

0 0 0 0

1 1.36 1 3

𝑝 −0.15

0.15

0

0

0.85

0.45

0

0

1

4.36

𝑧

The most negative entry in the bottom row is the −0.15 in the 𝑥-column, so we use this column as the pivot column. The test ratios are: z: 1/0.33, u: 1/1, v: 3/1.33. The smallest test ratio is u: 1/1. Thus we pivot on the 1 in the u-row.


Solutions Section 6.3 𝑥

𝑦

𝑧

𝑤

𝑠

𝑡

𝑢

𝑣

𝑝

𝑧 0.33 𝑤 −0.15

0.67 0.15

1 0

0 1

0.33

−0.15

0 0.45

0 0

0 0

0 0

1 1.36

𝑢

1

−1

0

0

0

−1

1

0

0

1

𝑣

1.33

0.67

0

0

0.33

−1

0

1

0

3

𝑝 −0.15

0.15

0

0

0.85

0.45

0

0

1

4.36

𝑥

𝑦

𝑧

𝑤

𝑠

𝑡

𝑢

𝑣

𝑝

𝑧 𝑤 𝑥 𝑣

0 0 1 0

1 0 −1 2

1 0 0 0

0 1 0 0

0.33

−0.15 0 0.33

0.33 0.3 −1 0.33

−0.33 0.15 1 −1.33

0 0 0 1

0 0 0 0

0.67 1.52 1 1.67

𝑝

0

0

0

0

0.85

0.3

0.15

0

1

4.52

As there are no more negative numbers in the bottom row, we are done, and read off the solution: Optimal solution: 𝑝 = 4.52; 𝑥 = 1, 𝑦 = 0, 𝑧 = 0.67, 𝑤 = 1.52. Another solution is 𝑥 = 1.67, 𝑦 = 0.67, 𝑧 = 0, 𝑤 = 1.52. 18. You can use the online Pivot and Gauss-Jordan Tool (or the Excel version) in decimal mode to do the pivoting. Introduce slack variables and rewrite the constraints and objective function in standard form, and set up the first tableau: 𝑥 + 1.3𝑦 + 𝑧 + 𝑠 = 3 1.3𝑦 + 𝑧 + 𝑤 + 𝑡 = 3 𝑥 + 𝑧 + 𝑤 + 𝑢 = 4.1 𝑥 + 1.3𝑦 + 𝑤 + 𝑣 = 4.1 −1.1𝑥 + 2.1𝑦 − 𝑧 − 𝑤 + 𝑝 = 0 𝑥

𝑦

𝑧

𝑤

𝑠

𝑡

𝑢

𝑣

𝑝

𝑠 𝑡 𝑢 𝑣

1 0 1 1

1.3 1.3 0 1.3

1 1 1 0

0 1 1 1

1 0 0 0

0 1 0 0

0 0 1 0

0 0 0 1

0 0 0 0

3 3 4.1 4.1

𝑝

−1.1

2.1

−1

−1

0

0

0

0

1

0

The most negative entry in the bottom row is the −1.1 in the 𝑥-column, so we use this column as the pivot column. The test ratios are: s: 3/1, u: 4.1/1, v: 4.1/1. The smallest test ratio is s: 3/1. Thus we pivot on the 1 in the s-row.


Solutions Section 6.3 𝑥

𝑦

𝑧

𝑤

𝑠

𝑡

𝑢

𝑣

𝑝

𝑠

1

1.3

1

0

1

0

0

0

0

3

𝑡 𝑢 𝑣

0 1 1

1.3 0 1.3

1 1 0

1 1 1

0 0 0

1 0 0

0 1 0

0 0 1

0 0 0

3 4.1 4.1

𝑝

−1.1

2.1

−1

−1

0

0

0

0

1

0

𝑥

𝑦

𝑧

𝑤

𝑠

𝑡

𝑢

𝑣

𝑝

𝑥 𝑡 𝑢 𝑣

1 0 0 0

1.3 1.3 −1.3 0

1 1 0 −1

0 1 1 1

1 0 −1 −1

0 1 0 0

0 0 1 0

0 0 0 1

0 0 0 0

3 3 1.1 1.1

𝑝

0

3.53

0.1

−1

1.1

0

0

0

1

3.3

The most negative entry in the bottom row is the −1 in the 𝑤-column, so we use this column as the pivot column. The test ratios are: t: 3/1, u: 1.1/1, v: 1.1/1. The smallest test ratio is u: 1.1/1. Thus we pivot on the 1 in the u-row. 𝑥

𝑦

𝑧

𝑤

𝑠

𝑡

𝑢

𝑣

𝑝

𝑥 𝑡

1 0

1.3 1.3

1 1

0 1

1 0

0 1

0 0

0 0

0 0

3 3

𝑢

0

−1.3

0

1

−1

0

1

0

0

1.1

𝑣

0

0

−1

1

−1

0

0

1

0

1.1

𝑝

0

3.53

0.1

−1

1.1

0

0

0

1

3.3

𝑥

𝑦

𝑧

𝑤

𝑠

𝑡

𝑢

𝑣

𝑝

𝑥 𝑡 𝑤 𝑣

1 0 0 0

1.3 2.6 −1.3 1.3

1 1 0 −1

0 0 1 0

1 1 −1 0

0 1 0 0

0 −1 1 −1

0 0 0 1

0 0 0 0

3 1.9 1.1 0

𝑝

0

2.23

0.1

0

0.1

0

1

0

1

4.4

As there are no more negative numbers in the bottom row, we are done, and read off the solution: Optimal solution: 𝑝 = 4.4; 𝑥 = 3, 𝑦 = 0, 𝑧 = 0, 𝑤 = 1.1. 19. You can use the online Pivot and Gauss-Jordan Tool (or the Excel version) in decimal mode to do the pivoting. Introduce slack variables and rewrite the constraints and objective function in standard form, and set up the first tableau: 𝑥 + 𝑦 + 𝑠 = 1.1 𝑦 + 𝑧 + 𝑡 = 2.2 𝑧 + 𝑤 + 𝑟 = 3.3 𝑤 + 𝑣 + 𝑞 = 4.4 −𝑥 + 𝑦 − 𝑧 + 𝑤 − 𝑣 + 𝑝 = 0


Solutions Section 6.3 𝑥

𝑦

𝑧

𝑤

𝑣

𝑠

𝑡

𝑟

𝑞

𝑝

𝑠 𝑡 𝑟 𝑞

1 0 0 0

1 1 0 0

0 1 1 0

0 0 1 1

0 0 0 1

1 0 0 0

0 1 0 0

0 0 1 0

0 0 0 1

0 0 0 0

1.1 2.2 3.3 4.4

𝑝

−1

1

−1

1

−1

0

0

0

0

1

0

The most negative entry in the bottom row is the −1 in the 𝑥-column, so we use this column as the pivot column. The only positive entry in this column is the 1 in the s-row, so we pivot on that entry. 𝑥

𝑦

𝑧

𝑤

𝑣

𝑠

𝑡

𝑟

𝑞

𝑝

𝑠

1

1

0

0

0

1

0

0

0

0

1.1

𝑡 𝑟 𝑞

0 0 0

1 0 0

1 1 0

0 1 1

0 0 1

0 0 0

1 0 0

0 1 0

0 0 1

0 0 0

2.2 3.3 4.4

𝑝

−1

1

−1

1

−1

0

0

0

0

1

0

𝑥

𝑦

𝑧

𝑤

𝑣

𝑠

𝑡

𝑟

𝑞

𝑝

𝑥 𝑡 𝑟 𝑞

1 0 0 0

1 1 0 0

0 1 1 0

0 0 1 1

0 0 0 1

1 0 0 0

0 1 0 0

0 0 1 0

0 0 0 1

0 0 0 0

1.1 2.2 3.3 4.4

𝑝

0

2

−1

1

−1

1

0

0

0

1

1.1

The most negative entry in the bottom row is the −1 in the 𝑧-column, so we use this column as the pivot column. The test ratios are: t: 2.2/1, r: 3.3/1. The smallest test ratio is t: 2.2/1. Thus we pivot on the 1 in the t-row. 𝑥

𝑦

𝑧

𝑤

𝑣

𝑠

𝑡

𝑟

𝑞

𝑝

𝑥

1

1

0

0

0

1

0

0

0

0

1.1

𝑡

0

1

1

0

0

0

1

0

0

0

2.2

𝑟 𝑞

0 0

0 0

1 0

1 1

0 1

0 0

0 0

1 0

0 1

0 0

3.3 4.4

𝑝

0

2

−1

1

−1

1

0

0

0

1

1.1

𝑥

𝑦

𝑧

𝑤

𝑣

𝑠

𝑡

𝑟

𝑞

𝑝

𝑥 𝑧 𝑟 𝑞

1 0 0 0

1 1 −1 0

0 1 0 0

0 0 1 1

0 0 0 1

1 0 0 0

0 1 −1 0

0 0 1 0

0 0 0 1

0 0 0 0

1.1 2.2 1.1 4.4

𝑝

0

3

0

1

−1

1

1

0

0

1

3.3


Solutions Section 6.3 The most negative entry in the bottom row is the −1 in the 𝑣-column, so we use this column as the pivot column. The only positive entry in this column is the 1 in the q-row, so we pivot on that entry. 𝑥

𝑦

𝑧

𝑤

𝑣

𝑠

𝑡

𝑟

𝑞

𝑝

𝑥 𝑧 𝑟

1 0 0

1 1 −1

0 1 0

0 0 1

0 0 0

1 0 0

0 1 −1

0 0 1

0 0 0

0 0 0

1.1 2.2 1.1

𝑞

0

0

0

1

1

0

0

0

1

0

4.4

𝑝

0

3

0

1

−1

1

1

0

0

1

3.3

𝑥

𝑦

𝑧

𝑤

𝑣

𝑠

𝑡

𝑟

𝑞

𝑝

𝑥 𝑧 𝑟 𝑣

1 0 0 0

1 1 −1 0

0 1 0 0

0 0 1 1

0 0 0 1

1 0 0 0

0 1 −1 0

0 0 1 0

0 0 0 1

0 0 0 0

1.1 2.2 1.1 4.4

𝑝

0

3

0

2

0

1

1

0

1

1

7.7

As there are no more negative numbers in the bottom row, we are done, and read off the solution: Optimal solution: 𝑝 = 7.7; 𝑥 = 1.1, 𝑦 = 0, 𝑧 = 2.2, 𝑤 = 0, 𝑣 = 4.4. 20. You can use the online Pivot and Gauss-Jordan Tool (or the Excel version) in decimal mode to do the pivoting. Introduce slack variables and rewrite the constraints and objective function in standard form, and set up the first tableau: 𝑥 + 𝑦 + 𝑠 = 1.1 𝑦 + 𝑧 + 𝑡 = 2.2 𝑧 + 𝑤 + 𝑟 = 3.3 𝑤 + 𝑣 + 𝑞 = 4.4 −𝑥 + 2𝑦 − 𝑧 + 2𝑤 − 𝑣 + 𝑝 = 0 𝑥

𝑦

𝑧

𝑤

𝑣

𝑠

𝑡

𝑟

𝑞

𝑝

𝑠 𝑡 𝑟 𝑞

1 0 0 0

1 1 0 0

0 1 1 0

0 0 1 1

0 0 0 1

1 0 0 0

0 1 0 0

0 0 1 0

0 0 0 1

0 0 0 0

1.1 2.2 3.3 4.4

𝑝

−1

2

−1

2

−1

0

0

0

0

1

0

The most negative entry in the bottom row is the −1 in the 𝑥-column, so we use this column as the pivot column. The only positive entry in this column is the 1 in the s-row, so we pivot on that entry.


Solutions Section 6.3 𝑥

𝑦

𝑧

𝑤

𝑣

𝑠

𝑡

𝑟

𝑞

𝑝

𝑠

1

1

0

0

0

1

0

0

0

0

1.1

𝑡 𝑟 𝑞

0 0 0

1 0 0

1 1 0

0 1 1

0 0 1

0 0 0

1 0 0

0 1 0

0 0 1

0 0 0

2.2 3.3 4.4

𝑝

−1

2

−1

2

−1

0

0

0

0

1

0

𝑥

𝑦

𝑧

𝑤

𝑣

𝑠

𝑡

𝑟

𝑞

𝑝

𝑥 𝑡 𝑟 𝑞

1 0 0 0

1 1 0 0

0 1 1 0

0 0 1 1

0 0 0 1

1 0 0 0

0 1 0 0

0 0 1 0

0 0 0 1

0 0 0 0

1.1 2.2 3.3 4.4

𝑝

0

3

−1

2

−1

1

0

0

0

1

1.1

The most negative entry in the bottom row is the −1 in the 𝑧-column, so we use this column as the pivot column. The test ratios are: t: 2.2/1, r: 3.3/1. The smallest test ratio is t: 2.2/1. Thus we pivot on the 1 in the t-row. 𝑥

𝑦

𝑧

𝑤

𝑣

𝑠

𝑡

𝑟

𝑞

𝑝

𝑥

1

1

0

0

0

1

0

0

0

0

1.1

𝑡

0

1

1

0

0

0

1

0

0

0

2.2

𝑟 𝑞

0 0

0 0

1 0

1 1

0 1

0 0

0 0

1 0

0 1

0 0

3.3 4.4

𝑝

0

3

−1

2

−1

1

0

0

0

1

1.1

𝑥

𝑦

𝑧

𝑤

𝑣

𝑠

𝑡

𝑟

𝑞

𝑝

𝑥 𝑧 𝑟 𝑞

1 0 0 0

1 1 −1 0

0 1 0 0

0 0 1 1

0 0 0 1

1 0 0 0

0 1 −1 0

0 0 1 0

0 0 0 1

0 0 0 0

1.1 2.2 1.1 4.4

𝑝

0

4

0

2

−1

1

1

0

0

1

3.3

The most negative entry in the bottom row is the −1 in the 𝑣-column, so we use this column as the pivot column. The only positive entry in this column is the 1 in the q-row, so we pivot on that entry. 𝑥

𝑦

𝑧

𝑤

𝑣

𝑠

𝑡

𝑟

𝑞

𝑝

𝑥 𝑧 𝑟

1 0 0

1 1 −1

0 1 0

0 0 1

0 0 0

1 0 0

0 1 −1

0 0 1

0 0 0

0 0 0

1.1 2.2 1.1

𝑞

0

0

0

1

1

0

0

0

1

0

4.4

𝑝

0

4

0

2

−1

1

1

0

0

1

3.3


Solutions Section 6.3 𝑥

𝑦

𝑧

𝑤

𝑣

𝑠

𝑡

𝑟

𝑞

𝑝

𝑥 𝑧 𝑟 𝑣

1 0 0 0

1 1 −1 0

0 1 0 0

0 0 1 1

0 0 0 1

1 0 0 0

0 1 −1 0

0 0 1 0

0 0 0 1

0 0 0 0

1.1 2.2 1.1 4.4

𝑝

0

4

0

3

0

1

1

0

1

1

7.7

As there are no more negative numbers in the bottom row, we are done, and read off the solution: Optimal solution: 𝑝 = 7.7; 𝑥 = 1.1, 𝑦 = 0, 𝑧 = 2.2, 𝑤 = 0, 𝑣 = 4.4. 21. Unknowns: 𝑥 = # calculus texts, 𝑦 = # history texts, 𝑧 = # marketing texts Maximize 𝑝 = 10𝑥 + 4𝑦 + 8𝑧 subject to 𝑥 + 𝑦 + 𝑧 ≤ 650, 2𝑥 + 𝑦 + 3𝑧 ≤ 1,000, 𝑥 ≥ 0, 𝑦 ≥ 0, 𝑧 ≥ 0. 𝑥

𝑦

𝑧

𝑠

𝑡

𝑝

𝑠

1

1

1

1

0

0

650

𝑡

2

1

3

0

1

0

1000

𝑝

−10

−4

−8

0

0

1

0

𝑥

𝑦

𝑧

𝑠

𝑡

𝑝

𝑠 𝑥

0 2

1 1

−1 3

2 0

−1 1

0 0

300 1000

𝑝

0

1

7

0

5

1

5000

2𝑅1 − 𝑅2 𝑅3 + 5𝑅2

Optimal solution: 𝑝 = 5,000∕1 = 5,000; 𝑥 = 1,000∕2 = 500, 𝑦 = 0, 𝑧 = 0. You should purchase 500 calculus texts, no history texts, and no marketing texts. The maximum profit is $5,000 per semester. 22. Unknowns: 𝑥 = # ordinary T-shirts, 𝑦 = # fancy T-shirts, 𝑧 = # very fancy T-shirts Maximize 𝑝 = 4𝑥 + 5𝑦 + 4𝑧 subject to 𝑥 + 𝑦 + 𝑧 ≤ 300, 6𝑥 + 8𝑦 + 10𝑧 ≤ 3,000, 𝑥 ≥ 0, 𝑦 ≥ 0, 𝑧 ≥ 0. 𝑥

𝑦

𝑧

𝑠

𝑡

𝑝

𝑠

1

1

1

1

0

0

300

𝑡

6

8

10

0

1

0

3000

𝑅2 − 8𝑅1

𝑝

−4

−5

−4

0

0

1

0

𝑅3 + 5𝑅1

𝑥

𝑦

𝑧

𝑠

𝑡

𝑝

𝑦 𝑡

1 −2

1 0

1 2

1 −8

0 1

0 0

300 600

𝑝

1

0

1

5

0

1

1500


Solutions Section 6.3 Optimal solution: 𝑝 = 1,500∕1 = 1,500; 𝑥 = 0, 𝑦 = 300∕1 = 300, 𝑧 = 0. The club should order no "ordinary" T-shirts, 300 "fancy" T-shirts, and no "very fancy" T-shirts. The maximum profit is $1,500. 23. Unknowns: 𝑥 = # gallons of PineOrange, 𝑦 = # gallons of PineKiwi, 𝑧 = # gallons of OrangeKiwi Maximize 𝑝 = 𝑥 + 2𝑦 + 𝑧 subject to 2𝑥 + 3𝑦 ≤ 800, 2𝑥 + 3𝑧 ≤ 650, 𝑦 + 𝑧 ≤ 350,𝑥 ≥ 0, 𝑦 ≥ 0, 𝑧 ≥ 0. 𝑥

𝑦

𝑧

𝑠

𝑡

𝑢

𝑝

𝑠

2

3

0

1

0

0

0

800

𝑡

2

0

3

0

1

0

0

650

𝑢

0

1

1

0

0

1

0

350

3𝑅3 − 𝑅1

𝑝

−1

−2

−1

0

0

0

1

0

3𝑅4 + 2𝑅1

𝑥

𝑦

𝑧

𝑠

𝑡

𝑢

𝑝

𝑦

2

3

0

1

0

0

0

800

𝑡

2

0

3

0

1

0

0

650

𝑢

−2

0

3

−1

0

3

0

250

𝑝

1

0

−3

2

0

0

3

1600

𝑅4 + 𝑅3

𝑥

𝑦

𝑧

𝑠

𝑡

𝑢

𝑝

𝑦

2

3

0

1

0

0

0

800

2𝑅1 − 𝑅2

𝑡

4

0

0

1

1

−3

0

400

𝑧

−2

0

3

−1

0

3

0

250

2𝑅3 + 𝑅2

𝑝

−1

0

0

1

0

3

3

1850

4𝑅4 + 𝑅2

𝑥

𝑦

𝑧

𝑠

𝑡

𝑢

𝑝

𝑦 𝑥 𝑧

0 4 0

6 0 0

0 0 6

1 1 −1

−1 1 1

3 −3 3

0 0 0

1200 400 900

𝑝

0

0

0

5

1

9

12

7800

𝑅2 − 𝑅3

Optimal solution: 𝑝 = 7,800∕12 = 650; 𝑥 = 400∕4 = 100, 𝑦 = 1,200∕6 = 200, 𝑧 = 900∕6 = 150. The company makes a maximum profit of $650 by making 100 gallons of PineOrange, 200 gallons of PineKiwi, and 150 gallons of OrangeKiwi. 24. Unknowns: 𝑥 = # Gigahaul trucks, 𝑦 = # Megahaul trucks, 𝑧 = # Picohaul trucks Maximize 𝑝 = 5𝑥 + 5𝑦 + 5𝑧 subject to 𝑥 + 𝑦 + 𝑧 ≤ 30, 6𝑥 + 5𝑦 + 2𝑧 ≤ 130, 6𝑥 + 5𝑦 + 4𝑧 ≤ 150,𝑥 ≥ 0, 𝑦 ≥ 0, 𝑧 ≥ 0.


Solutions Section 6.3 𝑥

𝑦

𝑧

𝑠

𝑡

𝑢

𝑝

𝑠

1

1

1

1

0

0

0

30

𝑡

6

5

2

0

1

0

0

130

𝑢

6

5

4

0

0

1

0

150

𝑅3 − 𝑅2

𝑝

−5

−5

−5

0

0

0

1

0

6𝑅4 + 5𝑅2

𝑥

𝑦

𝑧

𝑠

𝑡

𝑢

𝑝

𝑠

0

1

4

6

−1

0

0

50

𝑅1 − 2𝑅3

𝑥

6

5

2

0

1

0

0

130

𝑅2 − 𝑅3

𝑢

0

0

2

0

−1

1

0

20

𝑝

0

−5

−20

0

5

0

6

650

𝑥

𝑦

𝑧

𝑠

𝑡

𝑢

𝑝

𝑠

0

1

0

6

1

−2

0

10

𝑥

6

5

0

0

2

−1

0

110

𝑧

0

0

2

0

−1

1

0

20

𝑝

0

−5

0

0

−5

10

6

850

𝑥

𝑦

𝑧

𝑠

𝑡

𝑢

𝑝

𝑦 𝑥 𝑧

0 6 0

1 0 0

0 0 2

6 −30 0

1 −3 −1

−2 9 1

0 0 0

10 60 20

𝑝

0

0

0

30

0

0

6

900

6𝑅1 − 𝑅2

𝑅4 + 10𝑅3

𝑅2 − 5𝑅1 𝑅4 + 5𝑅1

Optimal solution: 𝑝 = 900∕6 = 150; 𝑥 = 60∕6 = 10, 𝑦 = 10∕1 = 10, 𝑧 = 20∕2 = 10. The company should purchase 10 of each kind of truck, giving an annual revenue of $15,000,000. Another solution is to buy 15 Gigahaul and 15 Picohaul trucks, for the same maximum revenue. 25. Unknowns: 𝑥 = # sections of Ancient History, 𝑦 = # sections of Medieval History, 𝑧 = # sections of Modern History Maximize 𝑝 = 𝑥 + 2𝑦 + 3𝑧 (in tens of thousands of dollars) subject to 𝑥 + 𝑦 + 𝑧 ≤ 45, 10𝑥 + 5𝑦 + 20𝑧 ≤ 500, 𝑥 + 𝑦 + 2𝑧 ≤ 60, 𝑥 ≥ 0, 𝑦 ≥ 0, 𝑧 ≥ 0. 𝑥

𝑦

𝑧

𝑠

𝑡

𝑢

𝑝

𝑠

1

1

1

1

0

0

0

45

𝑡

10

5

20

0

1

0

0

500

𝑢

1

1

2

0

0

1

0

60

10𝑅3 − 𝑅2

𝑝

−1

−2

−3

0

0

0

1

0

20𝑅4 + 3𝑅2

20𝑅1 − 𝑅2


Solutions Section 6.3 𝑥

𝑦

𝑧

𝑠

𝑡

𝑢

𝑝

𝑠

10

15

0

20

−1

0

0

400

𝑅1 − 3𝑅3

𝑧

10

5

20

0

1

0

0

500

𝑅2 − 𝑅3

𝑢

0

5

0

0

−1

10

0

100

𝑝

10

−25

0

0

3

0

20

1500

𝑥

𝑦

𝑧

𝑠

𝑡

𝑢

𝑝

𝑠

10

0

0

20

2

−30

0

100

𝑧

10

0

20

0

2

−10

0

400

𝑅2 − 𝑅1

𝑦

0

5

0

0

−1

10

0

100

2𝑅3 + 𝑅1

𝑝

10

0

0

0

−2

50

20

2000

𝑅4 + 𝑅1

𝑥

𝑦

𝑧

𝑠

𝑡

𝑢

𝑝

𝑡 𝑧 𝑦

10 0 10

0 0 10

0 20 0

20 −20 20

2 0 0

−30 20 −10

0 0 0

100 300 300

𝑝

20

0

0

20

0

20

20

2100

𝑅4 + 5𝑅3

Optimal solution: 𝑝 = 2,100∕20 = 105; 𝑥 = 0, 𝑦 = 300∕10 = 30, 𝑧 = 300∕20 = 15. The department should offer no Ancient History, 30 sections of Medieval History, and 15 sections of Modern History, for a profit of $105 × 10,000 = $1,050,000. Answers to additional question: The values of the slack variables are: 𝑡 = 1,000∕2 = 500, meaning that there will be 500 students without classes. 𝑠 = 𝑢 = 0, meaning that all time slots and professors are used. 26. Unknowns: 𝑥 = # batches of Creamy Vanilla, 𝑦 = # batches of Continental Mocha, 𝑧 = # batches of Succulent Strawberry a. Maximize 𝑝 = 3𝑥 + 2𝑦 + 4𝑧 subject to 2𝑥 + 𝑦 + 𝑧 ≤ 200, 𝑥 + 𝑦 + 2𝑧 ≤ 120, 2𝑥 + 2𝑦 + 2𝑧 ≤ 200,𝑥 ≥ 0, 𝑦 ≥ 0, 𝑧 ≥ 0. 𝑥 𝑦 𝑧 𝑠 𝑡 𝑢 𝑝 2𝑅1 − 𝑅2

𝑠

2

1

1

1

0

0

0

200

𝑡

1

1

2

0

1

0

0

120

𝑢

2

2

2

0

0

1

0

200

𝑅3 − 𝑅2

𝑝

−3

−2

−4

0

0

0

1

0

𝑅4 + 2𝑅2

𝑥

𝑦

𝑧

𝑠

𝑡

𝑢

𝑝

𝑠

3

1

0

2

−1

0

0

280

𝑅1 − 3𝑅3

𝑧

1

1

2

0

1

0

0

120

𝑅2 − 𝑅3

𝑢

1

1

0

0

−1

1

0

80

𝑝

−1

0

0

0

2

0

1

240

𝑅4 + 𝑅3


Solutions Section 6.3 𝑥

𝑦

𝑧

𝑠

𝑡

𝑢

𝑝

𝑠 𝑧 𝑥

0 0 1

−2 0 1

0 2 0

2 0 0

2 2 −1

−3 −1 1

0 0 0

40 40 80

𝑝

0

1

0

0

1

1

1

320

Optimal solution: 𝑝 = 320∕1 = 320; 𝑥 = 80∕1 = 80, 𝑦 = 0, 𝑧 = 40∕2 = 20. Make 80 batches of vanilla, no mocha, and 20 batches of strawberry, for a profit of $320. b. The values of the slack variables are: 𝑠 = 40∕2 = 20, meaning that you will have 20 eggs left over. 𝑡 = 𝑢 = 0, meaning that you will use all the milk and cream. c. We add the additional constraint 𝑧 ≤ 10 to the linear programming problem in part (a): 𝑥

𝑦

𝑧

𝑠

𝑡

𝑢

𝑣

𝑝

𝑠

2

1

1

1

0

0

0

0

200

𝑅1 − 𝑅4

𝑡

1

1

2

0

1

0

0

0

120

𝑅2 − 2𝑅4

𝑢

2

2

2

0

0

1

0

0

200

𝑅3 − 2𝑅4

𝑣

0

0

1

0

0

0

1

0

10

𝑝

−3

−2

−4

0

0

0

0

1

0

𝑅5 + 4𝑅4

𝑥

𝑦

𝑧

𝑠

𝑡

𝑢

𝑣

𝑝

𝑠

2

1

0

1

0

0

−1

0

190

𝑅1 − 𝑅3

𝑡

1

1

0

0

1

0

−2

0

100

2𝑅2 − 𝑅3

𝑢

2

2

0

0

0

1

−2

0

180

𝑧

0

0

1

0

0

0

1

0

10

𝑝

−3

−2

0

0

0

0

4

1

40

𝑥

𝑦

𝑧

𝑠

𝑡

𝑢

𝑣

𝑝

𝑠 𝑡 𝑥 𝑧

0 0 2 0

−1 0 2 0

0 0 0 1

1 0 0 0

0 2 0 0

−1 −1 1 0

1 −2 −2 1

0 0 0 0

10 20 180 10

𝑝

0

2

0

0

0

3

2

2

620

Optimal solution: 𝑝 = 620∕2 = 310; 𝑥 = 180∕2 = 90, 𝑦 = 0, 𝑧 = 10∕1 = 10. The maximum profit is $10 less than the profit in part (a). 27. Unknowns: 𝑥 = # acres of tomatoes, 𝑦 = # acres of lettuce, 𝑧 = # acres of carrots

2𝑅5 + 3𝑅3


Solutions Section 6.3 Maximize 𝑝 = 20𝑥 + 15𝑦 + 5𝑧 subject to 𝑥 + 𝑦 + 𝑧 ≤ 100, 5𝑥 + 4𝑦 + 2𝑧 ≤ 400, 4𝑥 + 2𝑦 + 2𝑧 ≤ 500,𝑥 ≥ 0, 𝑦 ≥ 0, 𝑧 ≥ 0. (𝑝 is measured in hundreds of dollars.) 𝑥

𝑦

𝑧

𝑠

𝑡

𝑢

𝑝

𝑠

1

1

1

1

0

0

0

100

𝑡

5

4

2

0

1

0

0

400

𝑢

4

2

2

0

0

1

0

500

5𝑅3 − 4𝑅2

𝑝

−20

−15

−5

0

0

0

1

0

𝑅4 + 4𝑅2

𝑥

𝑦

𝑧

𝑠

𝑡

𝑢

𝑝

𝑠 𝑥 𝑢

0 5 0

1 4 −6

3 2 2

5 0 0

−1 1 −4

0 0 5

0 0 0

100 400 900

𝑝

0

1

3

0

4

0

1

1600

5𝑅1 − 𝑅2

Optimal solution: 𝑝 = 1,600∕1 = 1,600; 𝑥 = 400∕5 = 80, 𝑦 = 0, 𝑧 = 0. Plant 80 acres of tomatoes, and no lettuce or carrots, for a maximum profit of $160,000. The slack variable corresponding to the number of acres available is 𝑠. The value of 𝑠 is 𝑠 = 100∕5 = 20, meaning that you will leave 20 acres unplanted. 28. Unknowns: 𝑥 = # acres of soy beans, 𝑦 = # acres of corn, 𝑧 = # acres of wheat Maximize 𝑝 = 3𝑥 + 2𝑦 + 𝑧 subject to 𝑥 + 𝑦 + 𝑧 ≤ 500, 5𝑥 + 2𝑦 + 𝑧 ≤ 3,000, 5𝑥 + 2𝑦 + 2𝑧 ≤ 3,000,𝑥 ≥ 0, 𝑦 ≥ 0, 𝑧 ≥ 0. (𝑝 is measured in hundreds of dollars.) 𝑥

𝑦

𝑧

𝑠

𝑡

𝑢

𝑝

𝑠

1

1

1

1

0

0

0

500

𝑡

5

2

1

0

1

0

0

3000

𝑅2 − 5𝑅1

𝑢

5

2

2

0

0

1

0

3000

𝑅3 − 5𝑅1

𝑝

−3

−2

−1

0

0

0

1

0

𝑅4 + 3𝑅1

𝑥

𝑦

𝑧

𝑠

𝑡

𝑢

𝑝

𝑥 𝑡 𝑢

1 0 0

1 −3 −3

1 −4 −3

1 −5 −5

0 1 0

0 0 1

0 0 0

500 500 500

𝑝

0

1

2

3

0

0

1

1500

Optimal solution: 𝑝 = 1,500∕1 = 1,500; 𝑥 = 500∕1 = 500, 𝑦 = 0, 𝑧 = 0. Plant 500 acres of soy beans, and nothing else, for a profit of $1,500,000. The slack variable corresponding to the available labor is 𝑢. The value of 𝑢 is 𝑢 = 500∕1 = 500, meaning that 500 hours of labor will not be used. In other words, you require only 2,500 of the available 3,000.


Solutions Section 6.3 29. Unknowns: 𝑥 = # servings of granola, 𝑦 = # servings of nutty granola, 𝑧 = # servings of nuttiest granola Maximize 𝑝 = 6𝑥 + 8𝑦 + 3𝑧 subject to 𝑥 + 𝑦 + 5𝑧 ≤ 1,500, 4𝑥 + 8𝑦 + 8𝑧 ≤ 10,000, 2𝑥 + 4𝑦 + 8𝑧 ≤ 4,000,𝑥 ≥ 0, 𝑦 ≥ 0, 𝑧 ≥ 0. 𝑥

𝑦

𝑧

𝑠

𝑡

𝑢

𝑝

𝑠

1

1

5

1

0

0

0

1500

4𝑅1 − 𝑅3

𝑡

4

8

8

0

1

0

0

10000

𝑅2 − 2𝑅3

𝑢

2

4

8

0

0

1

0

4000

𝑝

−6

−8

−3

0

0

0

1

0

𝑥

𝑦

𝑧

𝑠

𝑡

𝑢

𝑝

𝑠

2

0

12

4

0

−1

0

2000

𝑡

0

0

−8

0

1

−2

0

2000

𝑦

2

4

8

0

0

1

0

4000

𝑅3 − 𝑅1

𝑝

−2

0

13

0

0

2

1

8000

𝑅4 + 𝑅1

𝑥

𝑦

𝑧

𝑠

𝑡

𝑢

𝑝

𝑥 𝑡 𝑦

2 0 0

0 0 4

12 −8 −4

4 0 −4

0 1 0

−1 −2 2

0 0 0

2000 2000 2000

𝑝

0

0

25

4

0

1

1

10000

𝑅4 + 2𝑅3

Optimal solution: 𝑝 = 10,000∕1 = 10,000; 𝑥 = 2,000∕2 = 1,000, 𝑦 = 2,000∕4 = 500, 𝑧 = 0. The Choral Society can make a profit of $10,000 by selling 1,000 servings of granola, 500 servings of nutty granola, and no nuttiest granola. To obtain the ingredients left over, look at the values of the slack variables in the final tableau: 𝑠 = 0, so there are no toasted oats left over. 𝑡 = 2,000∕1 = 2,000, so there are 2,000 oz of almonds left over. 𝑢 = 0, so there are no raisins left over. 30. Unknowns: 𝑥 = # servings of granola, 𝑦 = # servings of nutty granola, 𝑧 = # servings of nuttiest granola Maximize 𝑝 = 3𝑥 + 3𝑦 + 3𝑧 subject to 𝑥 + 𝑦 + 5𝑧 ≤ 1,500, 4𝑥 + 8𝑦 + 8𝑧 ≤ 10,000, 2𝑥 + 4𝑦 + 8𝑧 ≤ 4,000,𝑥 ≥ 0, 𝑦 ≥ 0, 𝑧 ≥ 0. 𝑥

𝑦

𝑧

𝑠

𝑡

𝑢

𝑝

𝑠

1

1

5

1

0

0

0

1500

𝑡

4

8

8

0

1

0

0

10000

𝑅2 − 4𝑅1

𝑢

2

4

8

0

0

1

0

4000

𝑅3 − 2𝑅1

𝑝

−3

−3

−3

0

0

0

1

0

𝑅4 + 3𝑅1


Solutions Section 6.3 𝑥

𝑦

𝑧

𝑠

𝑡

𝑢

𝑝

𝑥 𝑡 𝑢

1 0 0

1 4 2

5

−12 −2

1

−4 −2

0 1 0

0 0 1

0 0 0

1500 4000 1000

𝑝

0

0

12

3

0

0

1

4500

Optimal solution: 𝑝 = 4,500∕1 = 4,500; 𝑥 = 1,500∕1 = 1,500, 𝑦 = 0, 𝑧 = 0. The Choral Society can make a profit of $4,500 by selling 1,500 servings of granola and nothing else. To obtain the ingredients left over, look at the values of the slack variables in the final tableau: 𝑠 = 0, so there are no toasted oats left over. 𝑡 = 4,000∕1 = 4,000, so there are 4,000 oz of almonds left over. 𝑢 = 1,000∕1 = 1,000, so there are 1,000 oz of raisins left over. Another solution is to sell 1,000 servings of granola and 500 servings of nutty granola as before, leaving only 2,000 oz of almonds left over. 31. Unknowns: 𝑥 = # axes, 𝑦 = # maces, 𝑧 = # spears. Maximize 𝑝 = 6𝑥 + 6𝑦 + 8𝑧 subject to 8𝑥 + 5𝑦 + 2𝑧 ≤ 50,000, 2𝑥 + 4𝑦 + 6𝑧 ≤ 40,000,𝑧 − 𝑥 ≤ 0, 𝑥 ≥ 0, 𝑦 ≥ 0, 𝑧 ≥ 0. 𝑥

𝑦

𝑧

𝑠

𝑡

𝑢

𝑝

𝑠

8

5

2

1

0

0

0

50000

𝑅1 − 2𝑅3

𝑡

2

4

6

0

1

0

0

40000

𝑅2 − 6𝑅3

𝑢

−1

0

1

0

0

1

0

0

𝑝

−6

−6

−8

0

0

0

1

0

𝑥

𝑦

𝑧

𝑠

𝑡

𝑢

𝑝

𝑠

10

5

0

1

0

−2

0

50000

𝑡

8

4

0

0

1

−6

0

40000

5𝑅2 − 4𝑅1

𝑧

−1

0

1

0

0

1

0

0

10𝑅3 + 𝑅1

𝑝

−14

−6

0

0

0

8

1

0

5𝑅4 + 7𝑅1

𝑥

𝑦

𝑧

𝑠

𝑡

𝑢

𝑝

𝑥 𝑡 𝑧

10 0 0

5 0 5

0 0 10

1 −4 1

0 5 0

−2 −22 8

0 0 0

50000 0 50000

𝑝

0

5

0

7

0

26

5

350000

𝑅4 + 8𝑅3

Optimal solution: 𝑝 = 350,000∕5 = 70,000; 𝑥 = 50,000∕10 = 5,000, 𝑦 = 0, 𝑧 = 50,000∕10 = 5,000. Achlúk can inflict a maximum of 70,000 units of damage using an arsenal of 5,000 axes, no maces, and 5,000 spears.


Solutions Section 6.3 32. Unknowns: 𝑥 = # javelins, 𝑦 = # longswords, 𝑧 = # spears. Maximize 𝑝 = 12𝑥 + 32𝑦 + 24𝑧 subject to 𝑥 + 15𝑦 + 2𝑧 ≤ 30,000, 2𝑥 + 4𝑦 + 6𝑧 ≤ 3,000,𝑦 − 2𝑥 ≤ 0, 𝑥 ≥ 0, 𝑦 ≥ 0, 𝑧 ≥ 0. 𝑥

𝑦

𝑧

𝑠

𝑡

𝑢

𝑝

𝑠

1

15

2

1

0

0

0

30000

𝑅1 − 15𝑅3

𝑡

2

4

6

0

1

0

0

3000

𝑅2 − 4𝑅3

𝑢

−2

1

0

0

0

1

0

0

𝑝

−12

−32

−24

0

0

0

1

0

𝑅4 + 32𝑅3

𝑥

𝑦

𝑧

𝑠

𝑡

𝑢

𝑝

𝑠

31

0

2

1

0

−15

0

30000

10𝑅1 − 31𝑅2

𝑡

10

0

6

0

1

−4

0

3000

𝑦

−2

1

0

0

0

1

0

0

5𝑅3 + 𝑅2

𝑝

−76

0

−24

0

0

32

1

0

5𝑅4 + 38𝑅2

𝑥

𝑦

𝑧

𝑠

𝑡

𝑢

𝑝

𝑠 𝑥 𝑦

0 10 0

0 0 5

−166 6 6

10 0 0

−31 1 1

−26 −4 1

0 0 0

207000 3000 3000

𝑝

0

0

108

0

38

8

5

114000

Optimal solution: 𝑝 = 114,000∕5 = 22,800; 𝑥 = 3,000∕10 = 300, 𝑦 = 3,000∕5 = 600, 𝑧 = 0. Galandir can inflict a maximum of 22,800 units of critical damage using an arsenal of 300 javelins, 600 longswords, and no spears. 33. Unknowns: 𝑥 = millions of gallons of oil allocated to process A, 𝑦 = millions of gallons of oil allocated to process B, 𝑧 = millions of gallons of oil allocated to process C Maximize 𝑝 = 4(0.60)𝑥 + 4(0.55)𝑦 + 4(0.50)𝑧 = 2.4𝑥 + 2.2𝑦 + 2.0𝑧, which we change to 𝑝 = 24𝑥 + 22𝑦 + 20𝑧 to work with integers, subject to 𝑥 + 𝑦 + 𝑧 ≤ 50, 150𝑥 + 100𝑦 + 50𝑧 ≤ 3,000 or 3𝑥 + 2𝑦 + 𝑧 ≤ 60, 𝑥 ≥ 0, 𝑦 ≥ 0, 𝑧 ≥ 0. 𝑥

𝑦

𝑧

𝑠

𝑡

𝑝

𝑠

1

1

1

1

0

0

50

𝑡

3

2

1

0

1

0

60

𝑝

−24

−22

−20

0

0

1

0

3𝑅1 − 𝑅2 𝑅3 + 8𝑅2


Solutions Section 6.3 𝑥

𝑦

𝑧

𝑠

𝑡

𝑝

𝑠

0

1

2

3

−1

0

90

𝑥

3

2

1

0

1

0

60

2𝑅2 − 𝑅1

𝑝

0

−6

−12

0

8

1

480

𝑅3 + 6𝑅1

𝑥

𝑦

𝑧

𝑠

𝑡

𝑝

𝑧 𝑥

0 6

1 3

2 0

3

−3

−1 3

0 0

90 30

𝑝

0

0

0

18

2

1

1020

Optimal solution: 𝑝 = 1,020∕1 = 1,020; 𝑥 = 30∕6 = 5, 𝑦 = 0, 𝑧 = 90∕2 = 45. Allocate 5 million gallons to process A and 45 million gallons to process C. Another solution: Allocate 10 million gallons to process B and 40 million gallons to process C. 34. Add the constraint 𝑧 ≤ 20 to the LP problem in the preceding exercise: Maximize 𝑝 = 24𝑥 + 22𝑦 + 20𝑧 subject to 𝑥 + 𝑦 + 𝑧 ≤ 50, 3𝑥 + 2𝑦 + 𝑧 ≤ 60, 𝑧 ≤ 20, 𝑥 ≥ 0, 𝑦 ≥ 0, 𝑧 ≥ 0. 𝑥

𝑦

𝑧

𝑠

𝑡

𝑢

𝑝

𝑠

1

1

1

1

0

0

0

50

𝑡

3

2

1

0

1

0

0

60

𝑢

0

0

1

0

0

1

0

20

𝑝

−24

−22

−20

0

0

0

1

0

𝑅4 + 8𝑅2

𝑥

𝑦

𝑧

𝑠

𝑡

𝑢

𝑝

𝑠

0

1

2

3

−1

0

0

90

𝑅1 − 2𝑅3

𝑥

3

2

1

0

1

0

0

60

𝑅2 − 𝑅3

𝑢

0

0

1

0

0

1

0

20

𝑝

0

−6

−12

0

8

0

1

480

𝑅4 + 12𝑅3

𝑥

𝑦

𝑧

𝑠

𝑡

𝑢

𝑝

𝑠

0

1

0

3

−1

−2

0

50

2𝑅1 − 𝑅2

𝑥

3

2

0

0

1

−1

0

40

𝑧

0

0

1

0

0

1

0

20

𝑝

0

−6

0

0

8

12

1

720

3𝑅1 − 𝑅2

𝑅4 + 3𝑅2


Solutions Section 6.3 𝑥

𝑦

𝑧

𝑠

𝑡

𝑢

𝑝

𝑠 𝑦 𝑧

−3 3 0

0 2 0

0 0 1

6 0 0

−3 1 0

−3 −1 1

0 0 0

60 40 20

𝑝

9

0

0

0

11

9

1

840

Optimal solution: 𝑝 = 840∕1 = 840; 𝑥 = 0, 𝑦 = 40∕2 = 20, 𝑧 = 20∕1 = 20. Allocate 20 million gallons to process B and 20 million gallons to process C. 35. Unknowns: 𝑥 = # servings of Xtend, 𝑦 = # servings of RecoverMode, 𝑧 = # servings of Strongevity Maximize 𝑝 = 7𝑥 + 5𝑦 subject to 3𝑦 + 3𝑧 ≤ 60, 2.5𝑥 + 3𝑦 + 𝑧 ≤ 70, 𝑥 − 𝑧 ≤ 0, 𝑥 ≥ 0, 𝑦 ≥ 0, 𝑧 ≥ 0. 𝑥

𝑦

𝑧

𝑠

𝑡

𝑢

𝑝

𝑠

0

3

3

1

0

0

0

60

𝑡

2.5

3

1

0

1

0

0

70

𝑢

1

0

−1

0

0

1

0

0

𝑝

−7

−5

0

0

0

0

1

0

𝑥

𝑦

𝑧

𝑠

𝑡

𝑢

𝑝

𝑠

0

3

3

1

0

0

0

60

𝑡

5

6

2

0

2

0

0

140

𝑢

1

0

−1

0

0

1

0

0

𝑝

−7

−5

0

0

0

0

1

0

𝑥

𝑦

𝑧

𝑠

𝑡

𝑢

𝑝

𝑠

0

3

3

1

0

0

0

60

𝑡

0

6

7

0

2

−5

0

140

3𝑅2 − 7𝑅1

𝑥

1

0

−1

0

0

1

0

0

3𝑅3 + 𝑅1

𝑝

0

−5

−7

0

0

7

1

0

3𝑅4 + 7𝑅1

𝑥

𝑦

𝑧

𝑠

𝑡

𝑢

𝑝

𝑧 𝑡 𝑥

0 0 3

3 −3 3

3 0 0

1 −7 1

0 6 0

0 −15 3

0 0 0

60 0 60

𝑝

0

6

0

7

0

21

3

420

2𝑅2

𝑅2 − 5𝑅3 𝑅4 + 7𝑅3

Optimal solution: 𝑝 = 420∕3 = 140; 𝑥 = 60∕3 = 20, 𝑦 = 0, 𝑧 = 60∕3 = 20. Use 20 servings each of Xtend and Strongevity and no servings of RecoverMode for 140 g of BCAAs.


Solutions Section 6.3 36. Unknowns: 𝑥 = # servings of RecoverMode, 𝑦 = # servings of Strongevity, 𝑧 = # servings of Muscle Physique Maximize 𝑝 = 3𝑥 + 𝑦 + 2𝑧 subject to 3𝑥 + 3𝑦 + 2𝑧 ≤ 60, 5𝑥 ≤ 50, 𝑥 − 𝑧 ≤ 0, 𝑥 ≥ 0, 𝑦 ≥ 0, 𝑧 ≥ 0. 𝑥

𝑦

𝑧

𝑠

𝑡

𝑢

𝑝

𝑠

3

3

2

1

0

0

0

60

𝑅1 − 3𝑅3

𝑡

5

0

0

0

1

0

0

50

𝑅2 − 5𝑅3

𝑢

1

0

−1

0

0

1

0

0

𝑝

−3

−1

−2

0

0

0

1

0

𝑅4 + 3𝑅3

𝑥

𝑦

𝑧

𝑠

𝑡

𝑢

𝑝

𝑠

0

3

5

1

0

−3

0

60

𝑅1 − 𝑅2

𝑡

0

0

5

0

1

−5

0

50

𝑥

1

0

−1

0

0

1

0

0

5𝑅3 + 𝑅2

𝑝

0

−1

−5

0

0

3

1

0

𝑅4 + 𝑅2

𝑥

𝑦

𝑧

𝑠

𝑡

𝑢

𝑝

𝑠

0

3

0

1

−1

2

0

10

𝑧

0

0

5

0

1

−5

0

50

𝑥

5

0

0

0

1

0

0

50

𝑝

0

−1

0

0

1

−2

1

50

𝑥

𝑦

𝑧

𝑠

𝑡

𝑢

𝑝

𝑢 𝑧 𝑥

0 0 5

3 15 0

0 10 0

1 5 0

−1 −3 1

2 0 0

0 0 0

10 150 50

𝑝

0

2

0

1

0

0

1

60

2𝑅2 + 5𝑅1 𝑅4 + 𝑅1

Optimal solution: 𝑝 = 60∕1 = 60; 𝑥 = 50∕5 = 10, 𝑦 = 0, 𝑧 = 150∕10 = 15. Combine 10 servings of RecoverMode, no Strongevity, and 15 servings of Muscle Physique for 60 g of L-glutamine. 37. Unknowns: 𝑥 = # DUK shares, 𝑦 = # CMCSA shares, 𝑧 = # CVX shares Maximize 𝑝 = 8𝑥 + 10𝑦 + 40𝑧 subject to 100𝑥 + 55𝑦 + 110𝑧 ≤ 99,000, (0.04)(100𝑥) + (0.02)55𝑦 + (0.05)(110𝑧) ≤ 5,500 or 40𝑥 + 11𝑦 + 55𝑧 ≤ 55,000, 𝑥 ≥ 0, 𝑦 ≥ 0, 𝑧 ≥ 0. 𝑥

𝑦

𝑧

𝑠

𝑡

𝑝

𝑠

100

55

110

1

0

0

99000

𝑡

40

11

55

0

1

0

55000

2𝑅2 − 𝑅1

𝑝

−8

−10

−40

0

0

1

0

11𝑅3 + 4𝑅1


Solutions Section 6.3 𝑥

𝑦

𝑧

𝑠

𝑡

𝑝

𝑧 𝑡

100 −20

55 −33

110 0

1 −1

0 2

0 0

99000 11000

𝑝

312

110

0

4

0

11

396000

Optimal solution: 𝑝 = 396,000∕11 = 36,000; 𝑥 = 0, 𝑦 = 0, 𝑧 = 99,000∕110 = 900. Buy 900 shares of CVX and no others. The broker is wrong. 38. Unknowns: 𝑥 = # DUK shares, 𝑦 = # CMCSA shares, 𝑧 = # CVX shares Maximize 𝑝 = 8𝑥 + 30𝑦 + 40𝑧 subject to 100𝑥 + 55𝑦 + 110𝑧 ≤ 99,000, (0.04)(100𝑥) + (0.02)55𝑦 + (0.05)(110𝑧) ≤ 5,500 or 40𝑥 + 11𝑦 + 55𝑧 ≤ 55,000, 𝑥 ≥ 0, 𝑦 ≥ 0, 𝑧 ≥ 0. 𝑥

𝑦

𝑧

𝑠

𝑡

𝑝

𝑠

100

55

110

1

0

0

99000

𝑡

40

11

55

0

1

0

55000

2𝑅2 − 𝑅1

𝑝

−8

−30

−40

0

0

1

0

11𝑅3 + 4𝑅1

𝑥

𝑦

𝑧

𝑠

𝑡

𝑝

𝑧

100

55

110

1

0

0

99000

𝑡

−20

−33

0

−1

2

0

11000

5𝑅2 + 3𝑅1

𝑝

312

−110

0

4

0

11

396000

𝑅3 + 2𝑅1

𝑥

𝑦

𝑧

𝑠

𝑡

𝑝

𝑦 𝑡

100 200

55 0

110 330

1 −2

0 10

0 0

99000 352000

𝑝

512

0

220

6

0

11

594000

Optimal solution: 𝑝 = 594,000∕11 = 54,000; 𝑥 = 0, 𝑦 = 99,000∕55 = 1,800, 𝑧 = 0. Buy 1,800 shares of CMCSA and no others. The broker is right. 39. Unknowns: 𝑥 = amount allocated to automobile loans (in $ millions), 𝑦 = amount allocated to furniture loans (in $ millions), 𝑧 = amount allocated to signature loans (in $ millions), 𝑤 = amount allocated to other secured loans Maximize 𝑝 = 8𝑥 + 10𝑦 + 12𝑧 + 10𝑤 (100 times the return) subject to 𝑥 + 𝑦 + 𝑧 + 𝑤 ≤ 5, −𝑥 − 𝑦 + 9𝑧 − 𝑤 ≤ 0, −𝑥 + 𝑦 + 𝑤 ≤ 0, −2𝑥 + 𝑤 ≤ 0, 𝑥 ≥ 0, 𝑦 ≥ 0, 𝑧 ≥ 0, 𝑤 ≥ 0.


Solutions Section 6.3 𝑥

𝑦

𝑧

𝑤

𝑠

𝑡

𝑢

𝑣

𝑝

𝑠

1

1

1

1

1

0

0

0

0

5

𝑡

−1

−1

9

−1

0

1

0

0

0

0

𝑢 𝑣

−1 −2

1 0

0 0

1 1

0 0

0 0

1 0

0 1

0 0

0 0

𝑝

−8

−10

−12

−10

0

0

0

0

1

0

3𝑅5 + 4𝑅2

𝑥

𝑦

𝑧

𝑤

𝑠

𝑡

𝑢

𝑣

𝑝

𝑠

10

10

0

10

9

−1

0

0

0

45

𝑅1 − 10𝑅3

𝑧

−1

−1

9

−1

0

1

0

0

0

0

𝑅2 + 𝑅3

𝑢

−1

1

0

1

0

0

1

0

0

0

𝑣

−2

0

0

1

0

0

0

1

0

0

𝑝

−28

−34

0

−34

0

4

0

0

3

0

𝑥

𝑦

𝑧

𝑤

𝑠

𝑡

𝑢

𝑣

𝑝

𝑠

20

0

0

0

9

−1

−10

0

0

45

𝑧

−2

0

9

0

0

1

1

0

0

0

10𝑅2 + 𝑅1

𝑦

−1

1

0

1

0

0

1

0

0

0

20𝑅3 + 𝑅1

𝑣

−2

0

0

1

0

0

0

1

0

0

10𝑅4 + 𝑅1

𝑝

−62

0

0

0

0

4

34

0

3

0

10𝑅5 + 31𝑅1

𝑥

𝑦

𝑧

𝑤

𝑠

𝑡

𝑢

𝑣

𝑝

𝑥 𝑧 𝑦 𝑣

20 0 0 0

0 0 20 0

0 90 0 0

0 0 20 10

9 9 9 9

−1 9 −1 −1

−10 0 10 −10

0 0 0 10

0 0 0 0

45 45 45 45

𝑝

0

0

0

0

279

9

30

0

30

1395

9𝑅1 − 𝑅2

𝑅5 + 34𝑅3

Optimal solution: 𝑝 = 1,395∕30 = 93∕2; 𝑥 = 45∕20 = 9∕4, 𝑦 = 45∕20 = 9∕4, 𝑧 = 45∕90 = 1∕2, 𝑤 = 0. Allocate $2,250,000 to automobile loans, $2,250,000 to furniture loans, and $500,000 to signature loans. Another optimal solution (pivot in the 𝑤-column): 𝑝 = 8,370∕180 = 46.5; 𝑥 = 45∕20 = 2.25, 𝑦 = 0, 𝑧 = 45∕90 = 0.5, 𝑤 = 45∕20 = 2.25. Allocate $2,250,000 to automobile loans, $500,000 to signature loans, and $2,250,000 to other secured loans. In general, allocate $2,250,000 to automobile loans, $500,000 to signature loans, and $2,250,000 to any combination of furniture loans and other secured loans. 40. Unknowns: 𝑥 = amount invested in BSBR, 𝑦 = amount invested in BOH, 𝑧 = amount invested in SAN Maximize 𝑝 = 6𝑥 + 3𝑦 + 2𝑧 (100 times the return) subject to 𝑥 + 𝑦 + 𝑧 ≤ 100,000, 𝑥 + 𝑦 − 𝑧 ≤ 0, 9𝑥 − 𝑦 − 𝑧 ≤ 0, 𝑥 ≥ 0, 𝑦 ≥ 0, 𝑧 ≥ 0.


Solutions Section 6.3 𝑥

𝑦

𝑧

𝑠

𝑡

𝑢

𝑝

𝑠

1

1

1

1

0

0

0

100000

𝑡

1

1

−1

0

1

0

0

0

𝑢

9

−1

−1

0

0

1

0

0

𝑅3 − 9𝑅2

𝑝

−6

−3

−2

0

0

0

1

0

𝑅4 + 6𝑅2

𝑥

𝑦

𝑧

𝑠

𝑡

𝑢

𝑝

𝑠

0

0

2

1

−1

0

0

100000

4𝑅1 − 𝑅3

𝑥

1

1

−1

0

1

0

0

0

8𝑅2 + 𝑅3

𝑢

0

−10

8

0

−9

1

0

0

𝑝

0

3

−8

0

6

0

1

0

𝑥

𝑦

𝑧

𝑠

𝑡

𝑢

𝑝

𝑠

0

10

0

4

5

−1

0

400000

𝑥

8

−2

0

0

−1

1

0

0

5𝑅2 + 𝑅1

𝑧

0

−10

8

0

−9

1

0

0

𝑅3 + 𝑅1

𝑝

0

−7

0

0

−3

1

1

0

10𝑅4 + 7𝑅1

𝑥

𝑦

𝑧

𝑠

𝑡

𝑢

𝑝

𝑦 𝑥 𝑧

0 40 0

10 0 0

0 0 8

4 4 4

5 0 −4

−1 4 0

0 0 0

400000 400000 400000

𝑝

0

0

0

28

5

3

10

2800000

𝑅1 − 𝑅2

𝑅4 + 𝑅3

Optimal solution: 𝑝 = 2,800,000∕10 = 280,000; 𝑥 = 400,000∕40 = 10,000, 𝑦 = 400,000∕10 = 40,000, 𝑧 = 400,000∕8 = 50,000. Invest $10,000 in BSBR, $40,000 in BOH, and $50,000 in SAN. 41. Unknowns: 𝑥 = amount invested in Universal, 𝑦 = amount invested in Sony, 𝑧 = amount invested in Warner, 𝑤 = amount invested in independents Maximize 𝑝 = 0.35𝑥 + 0.20𝑦 + 0.15𝑧 + 0.30𝑤 subject to 0.25𝑥 + 0.20𝑦 + 0.10𝑧 + 0.45𝑤 ≤ 18,000, −4𝑥 + 𝑦 + 𝑧 + 𝑤 ≤ 0, 𝑥 + 𝑦 + 𝑧 + 𝑤 ≤ 90,000, 𝑥 ≥ 0, 𝑦 ≥ 0, 𝑧 ≥ 0, 𝑤 ≥ 0. 𝑥

𝑦

𝑧

𝑤

𝑠

𝑡

𝑟

𝑝

𝑠

0.25

0.20

0.10

0.45

1

0

0

0

18000

𝑡 𝑟

−4 1

1 1

1 1

1 1

0 0

1 0

0 1

0 0

0 90000

0

0

0

1

0

𝑝 −0.35 −0.2 −0.15 −0.3

20𝑅1

20𝑅4


Solutions Section 6.3 𝑥

𝑦

𝑧

𝑤

𝑠

𝑡

𝑟

𝑝

𝑠

5

4

2

9

20

0

0

0

360000

𝑡

−4

1

1

1

0

1

0

0

0

5𝑅2 + 4𝑅1

𝑟

1

1

1

1

0

0

1

0

90000

5𝑅3 − 𝑅1

𝑝

−7

−4

−3

−6

0

0

0

20

0

5𝑅4 + 7𝑅1

𝑥

𝑦

𝑧

𝑤

𝑠

𝑡

𝑟

𝑝

𝑥

5

4

2

9

20

0

0

0

360000

3𝑅1 − 2𝑅3

𝑡

0

21

13

41

80

5

0

0

1440000

3𝑅2 − 13𝑅3

𝑟

0

1

3

−4

−20

0

5

0

90000

𝑝

0

8

−1

33

140

0

0

100

2520000

𝑥

𝑦

𝑧

𝑤

𝑠

𝑡

𝑟

𝑝

𝑥 𝑡 𝑧

15 0 0

10 50 1

0 0 3

35 175 −4

100 500 −20

0 15 0

−10 −65 5

0 0 0

900000 3150000 90000

𝑝

0

25

0

95

400

0

5

300

7650000

3𝑅4 + 𝑅3

Optimal solution: 𝑝 = 7,650,000∕300 = 25,500; 𝑥 = 900,000∕15 = 60,000, 𝑦 = 0, 𝑧 = 90,000∕3 = 30,000, 𝑤 = 0. Invest $60,000 in Universal, $30,000 in Warner, and none in the rest. 42. Unknowns: 𝑥 = amount invested in Universal, 𝑦 = amount invested in Sony, 𝑧 = amount invested in Warner, 𝑤 = amount invested in independents Maximize 𝑝 = 0.35𝑥 + 0.20𝑦 + 0.15𝑧 + 0.30𝑤 subject to 0.25𝑥 + 0.20𝑦 + 0.10𝑧 + 0.45𝑤 ≤ 9,000, −4𝑥 + 𝑦 + 𝑧 + 𝑤 ≤ 0, 𝑥 + 𝑦 + 𝑧 + 𝑤 ≤ 12,000, 𝑥 ≥ 0, 𝑦 ≥ 0, 𝑧 ≥ 0, 𝑤 ≥ 0. 𝑥

𝑦

𝑧

𝑤

𝑠

𝑡

𝑟

𝑝

𝑠

0.25

0.20

0.10

0.45

1

0

0

0

12000

𝑡 𝑟

−4 1

1 1

1 1

1 1

0 0

1 0

0 1

0 0

0 90000

0

0

0

1

0

𝑝 −0.35 −0.2 −0.15 −0.3

20𝑅1

20𝑅4

𝑥

𝑦

𝑧

𝑤

𝑠

𝑡

𝑟

𝑝

𝑠

5

4

2

9

20

0

0

0

240000

𝑡

−4

1

1

1

0

1

0

0

0

5𝑅2 + 4𝑅1

𝑟

1

1

1

1

0

0

1

0

90000

5𝑅3 − 𝑅1

𝑝

−7

−4

−3

−6

0

0

0

20

0

5𝑅4 + 7𝑅1


Solutions Section 6.3 𝑥

𝑦

𝑧

𝑤

𝑠

𝑡

𝑟

𝑝

𝑥

5

4

2

9

20

0

0

0

240000

3𝑅1 − 2𝑅3

𝑡

0

21

13

41

80

5

0

0

960000

3𝑅2 − 13𝑅3

𝑟

0

1

3

−4

−20

0

5

0

210000

𝑝

0

8

−1

33

140

0

0

100

1680000

𝑥

𝑦

𝑧

𝑤

𝑠

𝑡

𝑟

𝑝

𝑥 𝑡 𝑧

15 0 0

10 50 1

0 0 3

35 175 −4

100 500

−20

0 15 0

−10 −65 5

0 0 0

300000 150000 210000

𝑝

0

25

0

95

400

0

5

300

5250000

3𝑅4 + 𝑅3

Optimal solution: 𝑝 = 5,250,000∕300 = 17,500; 𝑥 = 300,000∕15 = 20,000, 𝑦 = 0, 𝑧 = 210,000∕3 = 70,000, 𝑤 = 0. Invest $20,000 in Universal, $70,000 in Warner, and none in the rest. 43. Unknowns: 𝑥 = # boards sent from Tucson to Honolulu 𝑦 = # boards sent from Tucson to Venice Beach 𝑧 = # boards sent from Toronto to Honolulu 𝑤 = # boards sent from Toronto to Venice Beach

Maximize 𝑝 = 𝑥 + 𝑦 + 𝑧 + 𝑤 subject to 𝑥 + 𝑦 ≤ 620, 𝑧 + 𝑤 ≤ 410, 𝑥 + 𝑧 ≤ 500, 𝑦 + 𝑤 ≤ 530, 10𝑥 + 5𝑦 + 20𝑧 + 10𝑤 ≤ 6,550, 𝑥 ≥ 0, 𝑦 ≥ 0, 𝑧 ≥ 0, 𝑤 ≥ 0. 𝑥

𝑦

𝑧

𝑤

𝑠

𝑡

𝑢

𝑣

𝑟

𝑝

𝑠

1

1

0

0

1

0

0

0

0

0

620

𝑡

0

0

1

1

0

1

0

0

0

0

410

𝑢

1

0

1

0

0

0

1

0

0

0

500

𝑣

0

1

0

1

0

0

0

1

0

0

530

𝑟

10

5

20

10

0

0

0

0

1

0

6550

𝑅5 − 10𝑅3

𝑝

−1

−1

−1

−1

0

0

0

0

0

1

0

𝑅6 + 𝑅3

𝑥

𝑦

𝑧

𝑤

𝑠

𝑡

𝑢

𝑣

𝑟

𝑝

𝑠

0

1

−1

0

1

0

−1

0

0

0

120

𝑡 𝑥

0 1

0 0

1 1

1 0

0 0

1 0

0 1

0 0

0 0

0 0

410 500

𝑣

0

1

0

1

0

0

0

1

0

0

530

𝑅4 − 𝑅1

𝑟

0

5

10

10

0

0

−10

0

1

0

1550

𝑅5 − 5𝑅1

𝑝

0

−1

0

−1

0

0

1

0

0

1

500

𝑅6 + 𝑅1

𝑅1 − 𝑅3


Solutions Section 6.3 𝑥

𝑦

𝑧

𝑤

𝑠

𝑡

𝑢

𝑣

𝑟

𝑝

𝑦

0

1

−1

0

1

0

−1

0

0

0

120

15𝑅1 + 𝑅5

𝑡

0

0

1

1

0

1

0

0

0

0

410

15𝑅2 − 𝑅5

𝑥

1

0

1

0

0

0

1

0

0

0

500

15𝑅3 − 𝑅5

𝑣

0

0

1

1

−1

0

1

1

0

0

410

15𝑅4 − 𝑅5

𝑟

0

0

15

10

−5

0

−5

0

1

0

950

𝑝

0

0

−1

−1

1

0

0

0

0

1

620

15𝑅6 + 𝑅5

𝑥

𝑦

𝑧

𝑤

𝑠

𝑡

𝑢

𝑣

𝑟

𝑝

𝑦

0

15

0

10

10

0

−20

0

1

0

2750

𝑅1 − 𝑅5

𝑡

0

0

0

5

5

15

5

0

−1

0

5200

2𝑅2 − 𝑅5

𝑥

15

0

0

−10

5

0

20

0

−1

0

6550

𝑅3 + 𝑅5

𝑣

0

0

0

5

−10

0

20

15

−1

0

5200

2𝑅4 − 𝑅5

𝑧

0

0

15

10

−5

0

−5

0

1

0

950

𝑝

0

0

0

−5

10

0

−5

0

1

15

10250

2𝑅6 + 𝑅5

𝑥

𝑦

𝑧

𝑤

𝑠

𝑡

𝑢

𝑣

𝑟

𝑝

𝑦

0

15

−15

0

15

0

−15

0

0

0

1800

3𝑅1 + 𝑅4

𝑡

0

0

−15

0

15

30

15

0

−3

0

9450

3𝑅2 − 𝑅4

𝑥

15

0

15

0

0

0

15

0

0

0

7500

3𝑅3 − 𝑅4

𝑣

0

0

−15

0

−15

0

45

30

−3

0

9450

𝑤

0

0

15

10

−5

0

−5

0

1

0

950

9𝑅5 + 𝑅4

𝑝

0

0

15

0

15

0

−15

0

3

30

21450

3𝑅6 + 𝑅4

𝑥

𝑦

𝑧

𝑤

𝑠

𝑡

𝑢

𝑣

𝑟

𝑝

𝑦 𝑡 𝑥 𝑢 𝑤

0 0 45 0 0

45 0 0 0 0

−60 −30 60 −15 120

0 0 0 0 90

30 60 15 −15 −60

0 90 0 0 0

0 0 0 45 0

30 −30 −30 30 30

−3 −6 3 −3 6

0 0 0 0 0

14850 18900 13050 9450 18000

𝑝

0

0

30

0

30

0

0

30

6

90

73800

Optimal solution: 𝑝 = 73,800∕90 = 820; 𝑥 = 13,050∕45 = 290, 𝑦 = 14,850∕45 = 330, 𝑧 = 0, 𝑤 = 18,000∕90 = 200. Make the following shipments: Tucson to Honolulu: 290 boards; Tucson to Venice Beach: 330 boards; Toronto to Honolulu: 0 boards; Toronto to Venice Beach: 200 boards, giving 820 boards shipped.


Solutions Section 6.3 44. Unknowns: 𝑥 = # boards sent from Tucson to Honolulu 𝑦 = # boards sent from Tucson to Venice Beach 𝑧 = # boards sent from Toronto to Honolulu 𝑤 = # boards sent from Toronto to Venice Beach

Maximize 𝑝 = 𝑥 + 𝑦 + 𝑧 + 𝑤 subject to 𝑥 + 𝑦 ≤ 620, 𝑧 + 𝑤 ≤ 410, 𝑥 + 𝑧 ≤ 500, 𝑦 + 𝑤 ≤ 530, 10𝑥 + 5𝑦 + 20𝑧 + 10𝑤 ≤ 5,050, 𝑥 ≥ 0, 𝑦 ≥ 0, 𝑧 ≥ 0, 𝑤 ≥ 0. 𝑥

𝑦

𝑧

𝑤

𝑠

𝑡

𝑢

𝑣

𝑟

𝑝

𝑠

1

1

0

0

1

0

0

0

0

0

620

𝑡

0

0

1

1

0

1

0

0

0

0

410

𝑢

1

0

1

0

0

0

1

0

0

0

500

𝑣

0

1

0

1

0

0

0

1

0

0

530

𝑟

10

5

20

10

0

0

0

0

1

0

5050

𝑅5 − 10𝑅3

𝑝

−1

−1

−1

−1

0

0

0

0

0

1

0

𝑅6 + 𝑅3

𝑥

𝑦

𝑧

𝑤

𝑠

𝑡

𝑢

𝑣

𝑟

𝑝

𝑠

0

1

−1

0

1

0

−1

0

0

0

120

5𝑅1 − 𝑅5

𝑡 𝑥

0 1

0 0

1 1

1 0

0 0

1 0

0 1

0 0

0 0

0 0

410 500

𝑣

0

1

0

1

0

0

0

1

0

0

530

𝑟

0

5

10

10

0

0

−10

0

1

0

50

𝑝

0

−1

0

−1

0

0

1

0

0

1

500

𝑥

𝑦

𝑧

𝑤

𝑠

𝑡

𝑢

𝑣

𝑟

𝑝

𝑠

0

0

−15

−10

5

0

5

0

−1

0

550

𝑡

0

0

1

1

0

1

0

0

0

0

410

𝑥

1

0

1

0

0

0

1

0

0

0

500

5𝑅3 − 𝑅1

𝑣

0

0

−10

−5

0

0

10

5

−1

0

2600

𝑅4 − 2𝑅1

𝑦

0

5

10

10

0

0

−10

0

1

0

50

𝑅5 + 2𝑅1

𝑝

0

0

10

5

0

0

−5

0

1

5

2550

𝑅6 + 𝑅1

𝑅1 − 𝑅3

5𝑅4 − 𝑅5 5𝑅6 + 𝑅5


Solutions Section 6.3 𝑥

𝑦

𝑧

𝑤

𝑠

𝑡

𝑢

𝑣

𝑟

𝑝

𝑢

0

0

−15

−10

5

0

5

0

−1

0

550

4𝑅1 + 3𝑅4

𝑡

0

0

1

1

0

1

0

0

0

0

410

20𝑅2 − 𝑅4

𝑥

5

0

20

10

−5

0

0

0

1

0

1950

𝑅3 − 𝑅4

𝑣

0

0

20

15

−10

0

0

5

1

0

1500

𝑦

0

5

−20

−10

10

0

0

0

−1

0

1150

𝑅5 + 𝑅4

𝑝

0

0

−5

−5

5

0

0

0

0

5

3100

4𝑅6 + 𝑅4

𝑥

𝑦

𝑧

𝑤

𝑠

𝑡

𝑢

𝑣

𝑟

𝑝

𝑢

0

0

0

5

−10

0

20

15

−1

0

6700

3𝑅1 − 𝑅4

𝑡

0

0

0

5

10

20

0

−5

−1

0

6700

3𝑅2 − 𝑅4

𝑥

5

0

0

−5

5

0

0

−5

0

0

450

3𝑅3 + 𝑅4

𝑧

0

0

20

15

−10

0

0

5

1

0

1500

𝑦

0

5

0

5

0

0

0

5

0

0

2650

3𝑅5 − 𝑅4

𝑝

0

0

0

−5

10

0

0

5

1

20

13900

3𝑅6 + 𝑅4

𝑥

𝑦

𝑧

𝑤

𝑠

𝑡

𝑢

𝑣

𝑟

𝑝

𝑢 𝑡 𝑥 𝑤 𝑦

0 0 15 0 0

0 0 0 0 15

−20 −20 20 20 −20

0 0 0 15 0

−20 40 5 −10 10

0 60 0 0 0

60 0 0 0 0

40 −20 −10 5 10

−4 −4 1 1 −1

0 0 0 0 0

18600 18600 2850 1500 6450

𝑝

0

0

20

0

20

0

0

20

4

60

43200

Optimal solution: 𝑝 = 43,200∕60 = 720; 𝑥 = 2,850∕15 = 190, 𝑦 = 6,450∕15 = 430, 𝑧 = 0, 𝑤 = 1,500∕15 = 100. Make the following shipments: Tucson to Honolulu: 190 boards; Tucson to Venice Beach: 430 boards; Toronto to Honolulu: 0 boards; Toronto to Venice Beach: 100 boards, giving 720 boards shipped. 45. Unknowns: 𝑥 = # people you fly from Chicago to Los Angeles, 𝑦 = # people you fly from Chicago to New York, 𝑧 = # people you fly from Denver to Los Angeles, 𝑤 = # people you fly from Denver to New York Maximize 𝑝 = 𝑥 + 𝑦 + 𝑧 + 𝑤 subject to 200𝑥 + 150𝑦 + 400𝑧 + 200𝑤 ≤ 2,100, 𝑥 + 𝑦 ≤ 20, 𝑧 + 𝑤 ≤ 10, 𝑥 + 𝑧 ≤ 15, 𝑦 + 𝑤 ≤ 15, 𝑥 ≥ 0, 𝑦 ≥ 0, 𝑧 ≥ 0, 𝑤 ≥ 0.


Solutions Section 6.3 𝑥

𝑦

𝑧

𝑤

𝑠

𝑡

𝑢

𝑣

𝑟

𝑝

𝑠

200

150

300

200

1

0

0

0

0

0

2100

𝑡

1

1

0

0

0

1

0

0

0

0

20

𝑢

0

0

1

1

0

0

1

0

0

0

10

𝑣

1

0

1

0

0

0

0

1

0

0

15

𝑟

0

1

0

1

0

0

0

0

1

0

15

𝑝

−1

−1

−1

−1

0

0

0

0

0

1

0

𝑥

𝑦

𝑧

𝑤

𝑠

𝑡

𝑢

𝑣

𝑟

𝑝

𝑥

200

150

300

200

1

0

0

0

0

0

2100

𝑡

0

50

−300 −200

−1

200

0

0

0

0

1900

𝑢

0

0

0

0

1

0

0

0

10

𝑣

0

−1

0

0

200

0

0

900

𝑅4 + 𝑅1

𝑟

0

1

0

1

0

0

0

0

1

0

15

150𝑅5 − 𝑅1

𝑝

0

−50

100

0

1

0

0

0

0

200

2100

3𝑅6 + 𝑅1

𝑥

𝑦

𝑧

𝑤

𝑠

𝑡

𝑢

𝑣

𝑟

𝑝

𝑦 𝑡 𝑢 𝑣 𝑟

200 −200 0 200 −200

150 0 0 0 0

300 200 −1200 −800 1 1 200 0 −300 −50

1 −4 0 0 −1

0 600 0 0 0

0 0 1 0 0

0 0 0 200 0

0 0 0 0 150

0 0 0 0 0

2100 3600 10 3000 150

𝑝

200

0

600

4

0

0

0

0

600

8400

1

1

−150 −100 −200

200

200𝑅2 − 𝑅1 200𝑅4 − 𝑅1 200𝑅6 + 𝑅1

3𝑅2 − 𝑅1

Optimal solution: 𝑝 = 8,400∕600 = 14; 𝑥 = 0, 𝑦 = 2,100∕150 = 14, 𝑧 = 0, 𝑤 = 0. Fly 14 people from Chicago to New York. 46. Unknowns: 𝑥 = # people you fly from Chicago to Los Angeles, 𝑦 = # people you fly from Chicago to New York, 𝑧 = # people you fly from Denver to Los Angeles, 𝑤 = # people you fly from Denver to New York Maximize 𝑝 = 𝑥 + 𝑦 + 𝑧 + 𝑤 subject to 200𝑥 + 150𝑦 + 200𝑧 + 140𝑤 ≤ 2,000, 𝑥 + 𝑦 ≤ 20, 𝑧 + 𝑤 ≤ 10, 𝑥 + 𝑧 ≤ 15, 𝑦 + 𝑤 ≤ 15, 𝑥 ≥ 0, 𝑦 ≥ 0, 𝑧 ≥ 0, 𝑤 ≥ 0. 𝑥

𝑦

𝑧

𝑤

𝑠

𝑡

𝑢

𝑣

𝑟

𝑝

𝑠

200

150

200

140

1

0

0

0

0

0

2000

𝑡

1

1

0

0

0

1

0

0

0

0

20

𝑢

0

0

1

1

0

0

1

0

0

0

10

𝑣

1

0

1

0

0

0

0

1

0

0

15

𝑟

0

1

0

1

0

0

0

0

1

0

15

𝑝

−1

−1

−1

−1

0

0

0

0

0

1

0

200𝑅2 − 𝑅1 200𝑅4 − 𝑅1 200𝑅6 + 𝑅1


Solutions Section 6.3 𝑥

𝑦

𝑧

𝑤

𝑠

𝑡

𝑢

𝑣

𝑟

𝑝

𝑥

200

150

200

140

1

0

0

0

0

0

2000

𝑅1 − 140𝑅3

𝑡

0

50

−200 −140

−1

200

0

0

0

0

2000

𝑅2 + 140𝑅3

𝑢

0

0

1

1

0

0

1

0

0

0

10

𝑣

0

−150

0

−140

−1

0

0

200

0

0

1000

𝑅4 + 140𝑅3

𝑟

0

1

0

1

0

0

0

0

1

0

15

𝑅5 − 𝑅3

𝑝

0

−50

0

−60

1

0

0

0

0

200

2000

𝑅6 + 60𝑅3

𝑥

𝑦

𝑧

𝑤

𝑠

𝑡

𝑢

𝑣

𝑟

𝑝

𝑥

200

150

60

0

1

0

−140

0

0

0

600

𝑡

0

50

−60

0

−1

200

140

0

0

0

3400

𝑤

0

0

1

1

0

0

1

0

0

0

10

𝑣

0

−150

140

0

−1

0

140

200

0

0

2400

𝑟

0

1

−1

0

0

0

−1

0

1

0

5

150𝑅5 − 𝑅1

𝑝

0

−50

60

0

1

0

60

0

0

200

2600

3𝑅6 + 𝑅1

𝑥

𝑦

𝑧

𝑤

𝑠

𝑡

𝑢

𝑣

𝑟

𝑝

𝑦 𝑡 𝑤 𝑣 𝑟

200 −200 0 200 −200

150 0 0 0 0

60 −240 1 200 −210

0 0 1 0 0

1 −4 0 0 −1

0 600 0 0 0

−140 560 1 0 −10

0 0 0 200 0

0 0 0 0 150

0 0 0 0 0

600 9600 10 3000 150

𝑝

200

0

240

0

4

0

40

0

0

600

8400

3𝑅2 − 𝑅1 𝑅4 + 𝑅1

Optimal solution: 𝑝 = 8,400∕600 = 14; 𝑥 = 0, 𝑦 = 600∕150 = 4, 𝑧 = 0, 𝑤 = 10∕1 = 10. Fly 4 people from Chicago to New York and 10 people from Denver to New York. 47. Yes; the given problem can be stated as: Maximize 𝑝 = 3𝑥 − 2𝑦 subject to −𝑥 + 𝑦 − 𝑧 ≤ 0, 𝑥 − 𝑦 − 𝑧 ≤ 6, 𝑥 ≥ 0, 𝑦 ≥ 0, 𝑧 ≥ 0. 48. Maximize 𝑝 = −3𝑥 − 2𝑦 subject to −𝑥 + 𝑦 − 𝑧 ≤ 0, −𝑥 + 𝑦 + 𝑧 ≤ 6, 𝑥 ≥ 0, 𝑦 ≥ 0, 𝑧 ≥ 0. 49. The graphical method applies only to LP problems in two unknowns, whereas the simplex method can be used to solve LP problems with any number of unknowns. 50. Any nonstandard LP problem with two unknowns. That is, any LP problem with two unknowns in which either the constraints are "≥," or in which the objective function must be minimized rather than maximized. 51. She is correct. Since there are only two constraints, there can only be two active variables, giving two or fewer nonzero values for the unknowns at each stage. 52. He is wrong. Here is a linear programming problem with one constraint that requires two pivoting steps (try


Solutions Section 6.3 it!). Maximize 𝑝 = 3𝑥 + 2𝑦 subject to 2𝑥 + 𝑦 ≤ 10, 𝑥 ≥ 0, 𝑦 ≥ 0. 53. A basic solution to a system of linear equations is a solution in which all the nonpivotal variables are taken to be zero; that is, all variables whose values are arbitrary are assigned the value zero. To obtain a basic solution for a given system of linear equations, one can row-reduce the associated augmented matrix, write down the general solution, and then set all the parameters (variables with "arbitrary" values) equal to zero. 54. a. Solutions in which all the variables are positive are the feasible solutions to the problem. b. Solutions in which some variables are negative are points outside of the feasible region. c. Solutions in which the inactive variables are 0 are the basic solutions. 55. No. Let us assume for the sake of simplicity that all the pivots are 1s. (They may certainly be changed to 1s without affecting the value of any of the variables.) Since the entry at the bottom of the pivot column is negative, the bottom row gets replaced by itself plus a positive multiple of the pivot row. The value of the objective function (bottom right entry) is thus replaced by itself plus a positive multiple of the nonnegative rightmost entry of the pivot row. Therefore, it cannot decrease. 56. Yes; if the rightmost entry of the pivot row is zero, the pivoting operation will not affect the value of the objective function.


Solutions Section 6.4 Section 6.4 1. Introduce slack variables and rewrite the constraints and objective function in standard form, and set up the first tableau: 𝑥 + 2𝑦 − 𝑠 = 6 −𝑥 + 𝑦 + 𝑡 = 4 2𝑥 + 𝑦 + 𝑢 = 8 −𝑥 − 𝑦 + 𝑝 = 0 𝑥

𝑦

𝑠

𝑡

𝑢

𝑝

*𝑠 𝑡 𝑢

1

−1 2

2 1 1

−1 0 0

0 1 0

0 0 1

0 0 0

6 4 8

𝑝

−1

−1

0

0

0

1

0

The first starred row is the s-row, and its largest positive entry is the 2 in the 𝑦-column. Thus, we use this column as the pivot column. The test ratios are: s: 6/2, t: 4/1, u: 8/1. The smallest test ratio is s: 6/2. Thus we pivot on the 2 in the s-row. 𝑥

𝑦

𝑠

𝑡

𝑢

𝑝

*𝑠 𝑡

1

2

−1

0

0

0

6

−1

1

0

1

0

0

4

2𝑅2 − 𝑅1

𝑢

2

1

0

0

1

0

8

2𝑅3 − 𝑅1

𝑝

−1

−1

0

0

0

1

0

2𝑅4 + 𝑅1

𝑥

𝑦

𝑠

𝑡

𝑢

𝑝

𝑦 𝑡 𝑢

1 −3 3

2 0 0

−1 1 1

0 2 0

0 0 2

0 0 0

6 2 10

𝑝

−1

0

−1

0

0

2

6

As there are no more starred rows, we go to Phase 2, and do the standard simplex method. 𝑥

𝑦

𝑠

𝑡

𝑢

𝑝

𝑦

1

2

−1

0

0

0

6

3𝑅1 − 𝑅3

𝑡

−3

0

1

2

0

0

2

𝑅2 + 𝑅3

𝑢

3

0

1

0

2

0

10

𝑝

−1

0

−1

0

0

2

6

3𝑅4 + 𝑅3


Solutions Section 6.4 𝑥

𝑦

𝑠

𝑡

𝑢

𝑝

𝑦

0

6

−4

0

−2

0

8

𝑡

0

0

2

2

2

0

12

𝑥

3

0

1

0

2

0

10

2𝑅3 − 𝑅2

𝑝

0

0

−2

0

2

6

28

𝑅4 + 𝑅2

𝑥

𝑦

𝑠

𝑡

𝑢

𝑝

𝑦 𝑠 𝑥

0 0 6

6 0 0

0 2 0

4 2

−2

2 2 2

0 0 0

32 12 8

𝑝

0

0

0

2

4

6

40

𝑅1 + 2𝑅2

Optimal solution: 𝑝 = 40∕6 = 20∕3; 𝑥 = 8∕6 = 4∕3, 𝑦 = 32∕6 = 16∕3. 2. Introduce slack variables and rewrite the constraints and objective function in standard form, and set up the first tableau: 𝑥 + 3𝑦 − 𝑠 = 6 −𝑥 + 𝑦 + 𝑡 = 4 2𝑥 + 𝑦 + 𝑢 = 8 −3𝑥 − 2𝑦 + 𝑝 = 0 𝑥

𝑦

𝑠

𝑡

𝑢

𝑝

*𝑠 𝑡 𝑢

1 −1 2

3 1 1

−1 0 0

0 1 0

0 0 1

0 0 0

6 4 8

𝑝

−3

−2

0

0

0

1

0

The first starred row is the s-row, and its largest positive entry is the 3 in the 𝑦-column. Thus, we use this column as the pivot column. The test ratios are: s: 6/3, t: 4/1, u: 8/1. The smallest test ratio is s: 6/3. Thus we pivot on the 3 in the s-row. 𝑥

𝑦

𝑠

𝑡

𝑢

𝑝

*𝑠 𝑡

1

3

−1

0

0

0

6

−1

1

0

1

0

0

4

3𝑅2 − 𝑅1

𝑢

2

1

0

0

1

0

8

3𝑅3 − 𝑅1

𝑝

−3

−2

0

0

0

1

0

3𝑅4 + 2𝑅1


Solutions Section 6.4 𝑥

𝑦

𝑠

𝑡

𝑢

𝑝

𝑦 𝑡 𝑢

1

−4 5

3 0 0

−1 1 1

0 3 0

0 0 3

0 0 0

6 6 18

𝑝

−7

0

−2

0

0

3

12

As there are no more starred rows, we go to Phase 2, and do the standard simplex method. 𝑥

𝑦

𝑠

𝑡

𝑢

𝑝

𝑦

1

3

−1

0

0

0

6

5𝑅1 − 𝑅3

𝑡

−4

0

1

3

0

0

6

5𝑅2 + 4𝑅3

𝑢

5

0

1

0

3

0

18

𝑝

−7

0

−2

0

0

3

12

5𝑅4 + 7𝑅3

𝑥

𝑦

𝑠

𝑡

𝑢

𝑝

𝑦

0

15

−6

0

−3

0

12

3𝑅1 + 2𝑅2

𝑡

0

0

9

15

12

0

102

𝑥

5

0

1

0

3

0

18

9𝑅3 − 𝑅2

𝑝

0

0

−3

0

21

15

186

3𝑅4 + 𝑅2

𝑥

𝑦

𝑠

𝑡

𝑢

𝑝

𝑦 𝑠 𝑥

0 0 45

45 0 0

0 9 0

30 15 −15

15 12 15

0 0 0

240 102 60

𝑝

0

0

0

15

75

45

660

Optimal solution: 𝑝 = 660∕45 = 44∕3; 𝑥 = 60∕45 = 4∕3, 𝑦 = 240∕45 = 16∕3. 3. Introduce slack variables and rewrite the constraints and objective function in standard form, and set up the first tableau: 𝑥 + 𝑦 + 𝑠 = 25 𝑥 − 𝑡 = 10 −𝑥 + 2𝑦 − 𝑢 = 0 −12𝑥 − 10𝑦 + 𝑝 = 0 𝑥

𝑦

𝑠

𝑡

𝑢

𝑝

𝑠 *𝑡 *𝑢

1 1

−1

1 0 2

1 0 0

0 −1 0

0 0

−1

0 0 0

25 10 0

𝑝

−12

−10

0

0

0

1

0


Solutions Section 6.4 The first starred row is the t-row, and its largest positive entry is the 1 in the 𝑥-column. Thus, we use this column as the pivot column. The test ratios are: s: 25/1, t: 10/1. The smallest test ratio is t: 10/1. Thus we pivot on the 1 in the t-row. 𝑥

𝑦

𝑠

𝑡

𝑢

𝑝

𝑠

1

1

1

0

0

0

25

*𝑡 *𝑢

1

0

0

−1

0

0

10

−1

2

0

0

−1

0

0

𝑅3 + 𝑅2

𝑝

−12

−10

0

0

0

1

0

𝑅4 + 12𝑅2

𝑥

𝑦

𝑠

𝑡

𝑢

𝑝

𝑠 𝑥 *𝑢

0 1 0

1 0 2

1 0 0

1 −1 −1

0 0

−1

0 0 0

15 10 10

𝑝

0

−10

0

−12

0

1

120

𝑅1 − 𝑅2

The first starred row is the u-row, and its largest positive entry is the 2 in the 𝑦-column. Thus, we use this column as the pivot column. The test ratios are: s: 15/1, u: 10/2. The smallest test ratio is u: 10/2. Thus we pivot on the 2 in the u-row. 𝑥

𝑦

𝑠

𝑡

𝑢

𝑝

𝑠

0

1

1

1

0

0

15

𝑥

1

0

0

−1

0

0

10

*𝑢

0

2

0

−1

−1

0

10

𝑝

0

−10

0

−12

0

1

120

𝑥

𝑦

𝑠

𝑡

𝑢

𝑝

𝑠 𝑥 𝑦

0 1 0

0 0 2

2 0 0

3 −1 −1

1 0 −1

0 0 0

20 10 10

𝑝

0

0

0

−17

−5

1

170

2𝑅1 − 𝑅3

𝑅4 + 5𝑅3

As there are no more starred rows, we go to Phase 2, and do the standard simplex method. 𝑥

𝑦

𝑠

𝑡

𝑢

𝑝

𝑠

0

0

2

3

1

0

20

𝑥

1

0

0

−1

0

0

10

3𝑅2 + 𝑅1

𝑦

0

2

0

−1

−1

0

10

3𝑅3 + 𝑅1

𝑝

0

0

0

−17

−5

1

170

3𝑅4 + 17𝑅1


Solutions Section 6.4 𝑥

𝑦

𝑠

𝑡

𝑢

𝑝

𝑡 𝑥 𝑦

0 3 0

0 0 6

2 2 2

3 0 0

1 1 −2

0 0 0

20 50 50

𝑝

0

0

34

0

2

3

850

Optimal solution: 𝑝 = 850∕3; 𝑥 = 50∕3, 𝑦 = 50∕6 = 25∕3. 4. Introduce slack variables and rewrite the constraints and objective function in standard form, and set up the first tableau: 𝑥 + 𝑦 + 𝑠 = 25 𝑦 − 𝑡 = 10 2𝑥 − 𝑦 − 𝑢 = 0 −𝑥 − 2𝑦 + 𝑝 = 0 𝑥

𝑦

𝑠

𝑡

𝑢

𝑝

𝑠 *𝑡 *𝑢

1 0 2

1 1

−1

1 0 0

0 −1 0

0 0

−1

0 0 0

25 10 0

𝑝

−1

−2

0

0

0

1

0

The first starred row is the t-row, and its largest positive entry is the 1 in the 𝑦-column. Thus, we use this column as the pivot column. The test ratios are: s: 25/1, t: 10/1. The smallest test ratio is t: 10/1. Thus we pivot on the 1 in the t-row. 𝑥

𝑦

𝑠

𝑡

𝑢

𝑝

𝑠

1

1

1

0

0

0

25

*𝑡 *𝑢

0

1

0

−1

0

0

10

2

−1

0

0

−1

0

0

𝑅3 + 𝑅2

𝑝

−1

−2

0

0

0

1

0

𝑅4 + 2𝑅2

𝑥

𝑦

𝑠

𝑡

𝑢

𝑝

𝑠 𝑦 *𝑢

1 0 2

0 1 0

1 0 0

1 −1 −1

0 0 −1

0 0 0

15 10 10

𝑝

−1

0

0

−2

0

1

20

𝑅1 − 𝑅2

The first starred row is the u-row, and its largest positive entry is the 2 in the 𝑥-column. Thus, we use this column as the pivot column. The test ratios are: s: 15/1, u: 10/2. The smallest test ratio is u: 10/2. Thus we pivot on the 2 in the u-row.


Solutions Section 6.4 𝑥

𝑦

𝑠

𝑡

𝑢

𝑝

𝑠

1

0

1

1

0

0

15

𝑦

0

1

0

−1

0

0

10

*𝑢

2

0

0

−1

−1

0

10

𝑝

−1

0

0

−2

0

1

20

𝑥

𝑦

𝑠

𝑡

𝑢

𝑝

𝑠 𝑦 𝑥

0 0 2

0 1 0

2 0 0

3 −1 −1

1 0 −1

0 0 0

20 10 10

𝑝

0

0

0

−5

−1

2

50

2𝑅1 − 𝑅3

2𝑅4 + 𝑅3

As there are no more starred rows, we go to Phase 2, and do the standard simplex method. 𝑥

𝑦

𝑠

𝑡

𝑢

𝑝

𝑠

0

0

2

3

1

0

20

𝑦

0

1

0

−1

0

0

10

3𝑅2 + 𝑅1

𝑥

2

0

0

−1

−1

0

10

3𝑅3 + 𝑅1

𝑝

0

0

0

−5

−1

2

50

3𝑅4 + 5𝑅1

𝑥

𝑦

𝑠

𝑡

𝑢

𝑝

𝑡 𝑦 𝑥

0 0 6

0 3 0

2 2 2

3 0 0

1 1 −2

0 0 0

20 50 50

𝑝

0

0

10

0

2

6

250

Optimal solution: 𝑝 = 250∕6 = 125∕3; 𝑥 = 50∕6 = 25∕3, 𝑦 = 50∕3. 5. Introduce slack variables and rewrite the constraints and objective function in standard form, and set up the first tableau: 𝑥 + 𝑦 + 𝑧 + 𝑠 = 150 𝑥 + 𝑦 + 𝑧 − 𝑡 = 100 −2𝑥 − 5𝑦 − 3𝑧 + 𝑝 = 0 𝑥

𝑦

𝑧

𝑠

𝑡

𝑝

𝑠 *𝑡

1 1

1 1

1 1

1 0

0 −1

0 0

150 100

𝑝

−2

−5

−3

0

0

1

0

The first starred row is the t-row, and its largest positive entry is the 1 in the 𝑥-column. Thus, we use this column as the pivot column. The test ratios are: s: 150/1, t: 100/1. The smallest test ratio is t: 100/1. Thus


Solutions Section 6.4 we pivot on the 1 in the t-row. 𝑥

𝑦

𝑧

𝑠

𝑡

𝑝

𝑠

1

1

1

1

0

0

150

*𝑡

1

1

1

0

−1

0

100

𝑝

−2

−5

−3

0

0

1

0

𝑥

𝑦

𝑧

𝑠

𝑡

𝑝

𝑠 𝑥

0 1

0 1

0 1

1 0

1 −1

0 0

50 100

𝑝

0

−3

−1

0

−2

1

200

𝑅1 − 𝑅2 𝑅3 + 2𝑅2

As there are no more starred rows, we go to Phase 2, and do the standard simplex method. 𝑥

𝑦

𝑧

𝑠

𝑡

𝑝

𝑠

0

0

0

1

1

0

50

𝑥

1

1

1

0

−1

0

100

𝑝

0

−3

−1

0

−2

1

200

𝑥

𝑦

𝑧

𝑠

𝑡

𝑝

𝑠

0

0

0

1

1

0

50

𝑦

1

1

1

0

−1

0

100

𝑅2 + 𝑅1

𝑝

3

0

2

0

−5

1

500

𝑅3 + 5𝑅1

𝑥

𝑦

𝑧

𝑠

𝑡

𝑝

𝑡 𝑦

0 1

0 1

0 1

1 1

1 0

0 0

50 150

𝑝

3

0

2

5

0

1

750

𝑅3 + 3𝑅2

Optimal solution: 𝑝 = 750∕1 = 750; 𝑥 = 0, 𝑦 = 150∕1 = 150, 𝑧 = 0. 6. Introduce slack variables and rewrite the constraints and objective function in standard form, and set up the first tableau: 𝑥 + 𝑦 + 2𝑧 + 𝑠 = 38 2𝑥 + 𝑦 + 𝑧 − 𝑡 = 24 −3𝑥 − 2𝑦 − 2𝑧 + 𝑝 = 0


Solutions Section 6.4 𝑥

𝑦

𝑧

𝑠

𝑡

𝑝

𝑠 *𝑡

1 2

1 1

2 1

1 0

0

−1

0 0

38 24

𝑝

−3

−2

−2

0

0

1

0

The first starred row is the t-row, and its largest positive entry is the 2 in the 𝑥-column. Thus, we use this column as the pivot column. The test ratios are: s: 38/1, t: 24/2. The smallest test ratio is t: 24/2. Thus we pivot on the 2 in the t-row. 𝑥

𝑦

𝑧

𝑠

𝑡

𝑝

𝑠

1

1

2

1

0

0

38

*𝑡

2

1

1

0

−1

0

24

𝑝

−3

−2

−2

0

0

1

0

𝑥

𝑦

𝑧

𝑠

𝑡

𝑝

𝑠 𝑥

0 2

1 1

3 1

2 0

1 −1

0 0

52 24

𝑝

0

−1

−1

0

−3

2

72

2𝑅1 − 𝑅2 2𝑅3 + 3𝑅2

As there are no more starred rows, we go to Phase 2, and do the standard simplex method. 𝑥

𝑦

𝑧

𝑠

𝑡

𝑝

𝑠

0

1

3

2

1

0

52

𝑥

2

1

1

0

−1

0

24

𝑅2 + 𝑅1

𝑝

0

−1

−1

0

−3

2

72

𝑅3 + 3𝑅1

𝑥

𝑦

𝑧

𝑠

𝑡

𝑝

𝑡 𝑥

0 2

1 2

3 4

2 2

1 0

0 0

52 76

𝑝

0

2

8

6

0

2

228

Optimal solution: 𝑝 = 228∕2 = 114; 𝑥 = 76∕2 = 38, 𝑦 = 0, 𝑧 = 0. 7. Introduce slack variables and rewrite the constraints and objective function in standard form, and set up the first tableau: 𝑥 + 2𝑦 + 𝑧 + 𝑠 = 40 2𝑦 − 𝑧 − 𝑡 = 10 2𝑥 − 𝑦 + 𝑧 − 𝑢 = 20 −10𝑥 − 20𝑦 + 15𝑧 + 𝑝 = 0


Solutions Section 6.4 𝑥

𝑦

𝑧

𝑠

𝑡

𝑢

𝑝

𝑠 *𝑡 *𝑢

1 0 2

2 2 −1

1

−1 1

1 0 0

0

−1 0

0 0 −1

0 0 0

40 10 20

𝑝

−10

−20

15

0

0

0

1

0

The first starred row is the t-row, and its largest positive entry is the 2 in the 𝑦-column. Thus, we use this column as the pivot column. The test ratios are: s: 40/2, t: 10/2. The smallest test ratio is t: 10/2. Thus we pivot on the 2 in the t-row. 𝑥

𝑦

𝑧

𝑠

𝑡

𝑢

𝑝

𝑠

1

2

1

1

0

0

0

40

*𝑡 *𝑢

0

2

−1

0

−1

0

0

10

2

−1

1

0

0

−1

0

20

2𝑅3 + 𝑅2

𝑝

−10

−20

15

0

0

0

1

0

𝑅4 + 10𝑅2

𝑥

𝑦

𝑧

𝑠

𝑡

𝑢

𝑝

𝑠 𝑦 *𝑢

1 0 4

0 2 0

2 −1 1

1 0 0

1 −1 −1

0 0 −2

0 0 0

30 10 50

𝑝

−10

0

5

0

−10

0

1

100

𝑅1 − 𝑅2

The first starred row is the u-row, and its largest positive entry is the 4 in the 𝑥-column. Thus, we use this column as the pivot column. The test ratios are: s: 30/1, u: 50/4. The smallest test ratio is u: 50/4. Thus we pivot on the 4 in the u-row. 𝑥

𝑦

𝑧

𝑠

𝑡

𝑢

𝑝

𝑠

1

0

2

1

1

0

0

30

𝑦

0

2

−1

0

−1

0

0

10

*𝑢

4

0

1

0

−1

−2

0

50

𝑝

−10

0

5

0

−10

0

1

100

𝑥

𝑦

𝑧

𝑠

𝑡

𝑢

𝑝

𝑠 𝑦 𝑥

0 0 4

0 2 0

7 −1 1

4 0 0

5 −1 −1

2 0 −2

0 0 0

70 10 50

𝑝

0

0

15

0

−25

−10

2

450

4𝑅1 − 𝑅3

2𝑅4 + 5𝑅3

As there are no more starred rows, we go to Phase 2, and do the standard simplex method.


Solutions Section 6.4 𝑥

𝑦

𝑧

𝑠

𝑡

𝑢

𝑝

𝑠

0

0

7

4

5

2

0

70

𝑦

0

2

−1

0

−1

0

0

10

5𝑅2 + 𝑅1

𝑥

4

0

1

0

−1

−2

0

50

5𝑅3 + 𝑅1

𝑝

0

0

15

0

−25

−10

2

450

𝑅4 + 5𝑅1

𝑥

𝑦

𝑧

𝑠

𝑡

𝑢

𝑝

𝑡 𝑦 𝑥

0 0 20

0 10 0

7 2 12

4 4 4

5 0 0

2 2

−8

0 0 0

70 120 320

𝑝

0

0

50

20

0

0

2

800

Optimal solution: 𝑝 = 800∕2 = 400; 𝑥 = 320∕20 = 16, 𝑦 = 120∕10 = 12, 𝑧 = 0. 8. Introduce slack variables and rewrite the constraints and objective function in standard form, and set up the first tableau: 𝑥 − 𝑦 + 𝑧 + 𝑠 = 12 2𝑥 − 2𝑦 + 𝑧 − 𝑡 = 15 −𝑦 + 𝑧 − 𝑢 = 3 10𝑥 − 10𝑦 − 15𝑧 + 𝑝 = 0 𝑥

𝑦

𝑧

𝑠

𝑡

𝑢

𝑝

𝑠 *𝑡 *𝑢

1 2 0

−1 −2 −1

1 1 1

1 0 0

0 −1 0

0 0 −1

0 0 0

12 15 3

𝑝

10

−10

−15

0

0

0

1

0

The first starred row is the t-row, and its largest positive entry is the 2 in the 𝑥-column. Thus, we use this column as the pivot column. The test ratios are: s: 12/1, t: 15/2. The smallest test ratio is t: 15/2. Thus we pivot on the 2 in the t-row. 𝑥

𝑦

𝑧

𝑠

𝑡

𝑢

𝑝

𝑠

1

−1

1

1

0

0

0

12

*𝑡 *𝑢

2

−2

1

0

−1

0

0

15

0

−1

1

0

0

−1

0

3

𝑝

10

−10

−15

0

0

0

1

0

2𝑅1 − 𝑅2

𝑅4 − 5𝑅2


Solutions Section 6.4 𝑥

𝑦

𝑧

𝑠

𝑡

𝑢

𝑝

𝑠 𝑥 *𝑢

0 2 0

0

−2 −1

1 1 1

2 0 0

1

−1 0

0 0 −1

0 0 0

9 15 3

𝑝

0

0

−20

0

5

0

1

−75

The first starred row is the u-row, and its largest positive entry is the 1 in the 𝑧-column. Thus, we use this column as the pivot column. The test ratios are: s: 9/1, x: 15/1, u: 3/1. The smallest test ratio is u: 3/1. Thus we pivot on the 1 in the u-row. 𝑥

𝑦

𝑧

𝑠

𝑡

𝑢

𝑝

𝑠

0

0

1

2

1

0

0

9

𝑅1 − 𝑅3

𝑥

2

−2

1

0

−1

0

0

15

𝑅2 − 𝑅3

*𝑢

0

−1

1

0

0

−1

0

3

𝑝

0

0

−20

0

5

0

1

−75

𝑥

𝑦

𝑧

𝑠

𝑡

𝑢

𝑝

𝑠 𝑥 𝑧

0 2 0

1 −1 −1

0 0 1

2 0 0

1 −1 0

1 1 −1

0 0 0

6 12 3

𝑝

0

−20

0

0

5

−20

1

−15

𝑅4 + 20𝑅3

As there are no more starred rows, we go to Phase 2, and do the standard simplex method. 𝑥

𝑦

𝑧

𝑠

𝑡

𝑢

𝑝

𝑠

0

1

0

2

1

1

0

6

𝑥

2

−1

0

0

−1

1

0

12

𝑅2 + 𝑅1

𝑧

0

−1

1

0

0

−1

0

3

𝑅3 + 𝑅1

𝑝

0

−20

0

0

5

−20

1

−15

𝑅4 + 20𝑅1

𝑥

𝑦

𝑧

𝑠

𝑡

𝑢

𝑝

𝑦 𝑥 𝑧

0 2 0

1 0 0

0 0 1

2 2 2

1 0 1

1 2 0

0 0 0

6 18 9

𝑝

0

0

0

40

25

0

1

105

Optimal solution: 𝑝 = 105∕1 = 105; 𝑥 = 18∕2 = 9, 𝑦 = 6∕1 = 6, 𝑧 = 9∕1 = 9.


Solutions Section 6.4 9. Introduce slack variables and rewrite the constraints and objective function in standard form, and set up the first tableau: 𝑥 + 𝑦 + 𝑧 + 𝑤 + 𝑠 = 40 2𝑥 + 𝑦 − 𝑧 − 𝑤 − 𝑡 = 10 𝑥 + 𝑦 + 𝑧 + 𝑤 − 𝑟 = 10 −𝑥 + 𝑦 − 3𝑧 − 𝑤 + 𝑝 = 0 𝑥

𝑦

𝑧

𝑤

𝑠

𝑡

𝑟

𝑝

𝑠 *𝑡 *𝑟

1 2 1

1 1 1

1 −1 1

1 −1 1

1 0 0

0 −1 0

0 0 −1

0 0 0

40 10 10

𝑝

−1

1

−3

−1

0

0

0

1

0

The first starred row is the t-row, and its largest positive entry is the 2 in the 𝑥-column. Thus, we use this column as the pivot column. The test ratios are: s: 40/1, t: 10/2, r: 10/1. The smallest test ratio is t: 10/2. Thus we pivot on the 2 in the t-row. 𝑥

𝑦

𝑧

𝑤

𝑠

𝑡

𝑟

𝑝

𝑠

1

1

1

1

1

0

0

0

40

*𝑡 *𝑟

2

1

−1

−1

0

−1

0

0

10

1

1

1

1

0

0

−1

0

10

2𝑅3 − 𝑅2

𝑝

−1

1

−3

−1

0

0

0

1

0

2𝑅4 + 𝑅2

𝑥

𝑦

𝑧

𝑤

𝑠

𝑡

𝑟

𝑝

𝑠 𝑥 *𝑟

0 2 0

1 1 1

3 −1 3

3 −1 3

2 0 0

1 −1 1

0 0 −2

0 0 0

70 10 10

𝑝

0

3

−7

−3

0

−1

0

2

10

2𝑅1 − 𝑅2

The first starred row is the r-row, and its largest positive entry is the 3 in the 𝑧-column. Thus, we use this column as the pivot column. The test ratios are: s: 70/3, r: 10/3. The smallest test ratio is r: 10/3. Thus we pivot on the 3 in the r-row. 𝑥

𝑦

𝑧

𝑤

𝑠

𝑡

𝑟

𝑝

𝑠

0

1

3

3

2

1

0

0

70

𝑅1 − 𝑅3

𝑥

2

1

−1

−1

0

−1

0

0

10

3𝑅2 + 𝑅3

*𝑟

0

1

3

3

0

1

−2

0

10

𝑝

0

3

−7

−3

0

−1

0

2

10

3𝑅4 + 7𝑅3


Solutions Section 6.4 𝑥

𝑦

𝑧

𝑤

𝑠

𝑡

𝑟

𝑝

𝑠 𝑥 𝑧

0 6 0

0 4 1

0 0 3

0 0 3

2 0 0

0

−2 1

2

−2 −2

0 0 0

60 40 10

𝑝

0

16

0

12

0

4

−14

6

100

As there are no more starred rows, we go to Phase 2, and do the standard simplex method. 𝑥

𝑦

𝑧

𝑤

𝑠

𝑡

𝑟

𝑝

𝑠

0

0

0

0

2

0

2

0

60

𝑥

6

4

0

0

0

−2

−2

0

40

𝑅2 + 𝑅1

𝑧

0

1

3

3

0

1

−2

0

10

𝑅3 + 𝑅1

𝑝

0

16

0

12

0

4

−14

6

100

𝑅4 + 7𝑅1

𝑥

𝑦

𝑧

𝑤

𝑠

𝑡

𝑟

𝑝

𝑟 𝑥 𝑧

0 6 0

0 4 1

0 0 3

0 0 3

2 2 2

0 −2 1

2 0 0

0 0 0

60 100 70

𝑝

0

16

0

12

14

4

0

6

520

Optimal solution: 𝑝 = 520∕6 = 260∕3; 𝑥 = 100∕6 = 50∕3, 𝑦 = 0, 𝑧 = 70∕3, 𝑤 = 0. 10. Introduce slack variables and rewrite the constraints and objective function in standard form, and set up the first tableau: 𝑥 + 𝑦 + 𝑧 + 𝑤 + 𝑠 = 50 2𝑥 + 𝑦 − 𝑧 − 𝑤 − 𝑡 = 10 𝑥 + 𝑦 + 𝑧 + 𝑤 − 𝑟 = 20 −𝑥 − 𝑦 − 4𝑧 + 2𝑤 + 𝑝 = 0 𝑥

𝑦

𝑧

𝑤

𝑠

𝑡

𝑟

𝑝

𝑠 *𝑡 *𝑟

1 2 1

1 1 1

1 −1 1

1 −1 1

1 0 0

0 −1 0

0 0 −1

0 0 0

50 10 20

𝑝

−1

−1

−4

2

0

0

0

1

0

The first starred row is the t-row, and its largest positive entry is the 2 in the 𝑥-column. Thus, we use this column as the pivot column. The test ratios are: s: 50/1, t: 10/2, r: 20/1. The smallest test ratio is t: 10/2. Thus we pivot on the 2 in the t-row.


Solutions Section 6.4 𝑥

𝑦

𝑧

𝑤

𝑠

𝑡

𝑟

𝑝

𝑠

1

1

1

1

1

0

0

0

50

*𝑡 *𝑟

2

1

−1

−1

0

−1

0

0

10

1

1

1

1

0

0

−1

0

20

2𝑅3 − 𝑅2

𝑝

−1

−1

−4

2

0

0

0

1

0

2𝑅4 + 𝑅2

𝑥

𝑦

𝑧

𝑤

𝑠

𝑡

𝑟

𝑝

𝑠 𝑥 *𝑟

0 2 0

1 1 1

3 −1 3

3 −1 3

2 0 0

1 −1 1

0 0

−2

0 0 0

90 10 30

𝑝

0

−1

−9

3

0

−1

0

2

10

2𝑅1 − 𝑅2

The first starred row is the r-row, and its largest positive entry is the 3 in the 𝑧-column. Thus, we use this column as the pivot column. The test ratios are: s: 90/3, r: 30/3. The smallest test ratio is r: 30/3. Thus we pivot on the 3 in the r-row. 𝑥

𝑦

𝑧

𝑤

𝑠

𝑡

𝑟

𝑝

𝑠

0

1

3

3

2

1

0

0

90

𝑅1 − 𝑅3

𝑥

2

1

−1

−1

0

−1

0

0

10

3𝑅2 + 𝑅3

*𝑟

0

1

3

3

0

1

−2

0

30

𝑝

0

−1

−9

3

0

−1

0

2

10

𝑥

𝑦

𝑧

𝑤

𝑠

𝑡

𝑟

𝑝

𝑠 𝑥 𝑧

0 6 0

0 4 1

0 0 3

0 0 3

2 0 0

0 −2 1

2 −2 −2

0 0 0

60 60 30

𝑝

0

2

0

12

0

2

−6

2

100

𝑅4 + 3𝑅3

As there are no more starred rows, we go to Phase 2, and do the standard simplex method. 𝑥

𝑦

𝑧

𝑤

𝑠

𝑡

𝑟

𝑝

𝑠

0

0

0

0

2

0

2

0

60

𝑥

6

4

0

0

0

−2

−2

0

60

𝑅2 + 𝑅1

𝑧

0

1

3

3

0

1

−2

0

30

𝑅3 + 𝑅1

𝑝

0

2

0

12

0

2

−6

2

100

𝑅4 + 3𝑅1


Solutions Section 6.4 𝑥

𝑦

𝑧

𝑤

𝑠

𝑡

𝑟

𝑝

𝑟 𝑥 𝑧

0 6 0

0 4 1

0 0 3

0 0 3

2 2 2

0

−2 1

2 0 0

0 0 0

60 120 90

𝑝

0

2

0

12

6

2

0

2

280

Optimal solution: 𝑝 = 280∕2 = 140; 𝑥 = 120∕6 = 20, 𝑦 = 0, 𝑧 = 90∕3 = 30, 𝑤 = 0. 11. Convert the given problem into a maximization problem: Maximize 𝑝 = −6𝑥 − 6𝑦 subject to 𝑥 + 2𝑦 ≥ 20, 2𝑥 + 𝑦 ≥ 20, 𝑥 ≥ 0, 𝑦 ≥ 0. Introduce slack variables and rewrite the constraints and objective function in standard form, and set up the first tableau: 𝑥 + 2𝑦 − 𝑠 = 20 2𝑥 + 𝑦 − 𝑡 = 20 6𝑥 + 6𝑦 + 𝑝 = 0 𝑥

𝑦

𝑠

𝑡

𝑝

*𝑠 *𝑡

1 2

2 1

−1 0

0 −1

0 0

20 20

𝑝

6

6

0

0

1

0

The first starred row is the s-row, and its largest positive entry is the 2 in the 𝑦-column. Thus, we use this column as the pivot column. The test ratios are: s: 20/2, t: 20/1. The smallest test ratio is s: 20/2. Thus we pivot on the 2 in the s-row. 𝑥

𝑦

𝑠

𝑡

𝑝

*𝑠 *𝑡

1

2

−1

0

0

20

2

1

0

−1

0

20

2𝑅2 − 𝑅1

𝑝

6

6

0

0

1

0

𝑅3 − 3𝑅1

𝑥

𝑦

𝑠

𝑡

𝑝

𝑦 *𝑡

1 3

2 0

−1 1

0 −2

0 0

20 20

𝑝

3

0

3

0

1

−60

The first starred row is the t-row, and its largest positive entry is the 3 in the 𝑥-column. Thus, we use this column as the pivot column. The test ratios are: y: 20/1, t: 20/3. The smallest test ratio is t: 20/3. Thus we pivot on the 3 in the t-row.


Solutions Section 6.4 𝑥

𝑦

𝑠

𝑡

𝑝

𝑦

1

2

−1

0

0

20

*𝑡

3

0

1

−2

0

20

𝑝

3

0

3

0

1

−60

𝑥

𝑦

𝑠

𝑡

𝑝

𝑦 𝑥

0 3

6 0

−4 1

2

−2

0 0

40 20

𝑝

0

0

2

2

1

−80

3𝑅1 − 𝑅2 𝑅3 − 𝑅2

As there are no more starred rows, we go to Phase 2, and do the standard simplex method. However, there are no negative numbers in the bottom row, so there are no pivot steps to do in Phase 2. Therefore, we are done. Optimal solution: 𝑝 = −80∕1 = −80; 𝑥 = 20∕3, 𝑦 = 40∕6 = 20∕3. Since 𝑐 = −𝑝, the minimum value of c is 80. 12. Convert the given problem into a maximization problem: Maximize 𝑝 = −3𝑥 − 2𝑦 subject to 𝑥 + 2𝑦 ≥ 20, 2𝑥 + 𝑦 ≥ 10, 𝑥 ≥ 0, 𝑦 ≥ 0. Introduce slack variables and rewrite the constraints and objective function in standard form, and set up the first tableau: 𝑥 + 2𝑦 − 𝑠 = 20 2𝑥 + 𝑦 − 𝑡 = 10 3𝑥 + 2𝑦 + 𝑝 = 0 𝑥

𝑦

𝑠

𝑡

𝑝

*𝑠 *𝑡

1 2

2 1

−1 0

0 −1

0 0

20 10

𝑝

3

2

0

0

1

0

The first starred row is the s-row, and its largest positive entry is the 2 in the 𝑦-column. Thus, we use this column as the pivot column. The test ratios are: s: 20/2, t: 10/1. The smallest test ratio is s: 20/2. Thus we pivot on the 2 in the s-row. 𝑥

𝑦

𝑠

𝑡

𝑝

*𝑠 *𝑡

1

2

−1

0

0

20

2

1

0

−1

0

10

2𝑅2 − 𝑅1

𝑝

3

2

0

0

1

0

𝑅3 − 𝑅1

𝑥

𝑦

𝑠

𝑡

𝑝

𝑦 *𝑡

1 3

2 0

−1 1

0 −2

0 0

20 0

𝑝

2

0

1

0

1

−20

The first starred row is the t-row, and its largest positive entry is the 3 in the 𝑥-column. Thus, we use this


Solutions Section 6.4 column as the pivot column. The test ratios are: y: 20/1, t: 0/3. The smallest test ratio is t: 0/3. Thus we pivot on the 3 in the t-row. 𝑥

𝑦

𝑠

𝑡

𝑝

𝑦

1

2

−1

0

0

20

*𝑡

3

0

1

−2

0

0

𝑝

2

0

1

0

1

−20

𝑥

𝑦

𝑠

𝑡

𝑝

𝑦 𝑥

0 3

6 0

−4 1

2 −2

0 0

60 0

𝑝

0

0

1

4

3

−60

3𝑅1 − 𝑅2 3𝑅3 − 2𝑅2

As there are no more starred rows, we go to Phase 2, and do the standard simplex method. However, there are no negative numbers in the bottom row, so there are no pivot steps to do in Phase 2. Therefore, we are done. Optimal solution: 𝑝 = −60∕3 = −20; 𝑥 = 0∕3 = 0, 𝑦 = 60∕6 = 10. Since 𝑐 = −𝑝, the minimum value of c is 20. 13. Convert the given problem into a maximization problem: Maximize 𝑝 = −2𝑥 − 𝑦 − 3𝑧 subject to 𝑥 + 𝑦 + 𝑧 ≥ 100, 2𝑥 + 𝑦 ≥ 50, 𝑦 + 𝑧 ≥ 50, 𝑥 ≥ 0, 𝑦 ≥ 0, 𝑧 ≥ 0. Introduce slack variables and rewrite the constraints and objective function in standard form, and set up the first tableau: 𝑥 + 𝑦 + 𝑧 − 𝑠 = 100 2𝑥 + 𝑦 − 𝑡 = 50 𝑦 + 𝑧 − 𝑢 = 50 2𝑥 + 𝑦 + 3𝑧 + 𝑝 = 0 𝑥

𝑦

𝑧

𝑠

𝑡

𝑢

𝑝

*𝑠 *𝑡 *𝑢

1 2 0

1 1 1

1 0 1

−1 0 0

0 −1 0

0 0 −1

0 0 0

100 50 50

𝑝

2

1

3

0

0

0

1

0

The first starred row is the s-row, and its largest positive entry is the 1 in the 𝑥-column. Thus, we use this column as the pivot column. The test ratios are: s: 100/1, t: 50/2. The smallest test ratio is t: 50/2. Thus we pivot on the 2 in the t-row. 𝑥

𝑦

𝑧

𝑠

𝑡

𝑢

𝑝

*𝑠 *𝑡

1

1

1

−1

0

0

0

100

2

1

0

0

−1

0

0

50

*𝑢

0

1

1

0

0

−1

0

50

𝑝

2

1

3

0

0

0

1

0

2𝑅1 − 𝑅2

𝑅4 − 𝑅2


Solutions Section 6.4 𝑥

𝑦

𝑧

𝑠

𝑡

𝑢

𝑝

*𝑠 𝑥 *𝑢

0 2 0

1 1 1

2 0 1

−2 0 0

1

−1 0

0 0 −1

0 0 0

150 50 50

𝑝

0

0

3

0

1

0

1

−50

The first starred row is the s-row, and its largest positive entry is the 2 in the 𝑧-column. Thus, we use this column as the pivot column. The test ratios are: s: 150/2, u: 50/1. The smallest test ratio is u: 50/1. Thus we pivot on the 1 in the u-row. 𝑥

𝑦

𝑧

𝑠

𝑡

𝑢

𝑝

*𝑠 𝑥

0

1

2

−2

1

0

0

150

2

1

0

0

−1

0

0

50

*𝑢

0

1

1

0

0

−1

0

50

𝑝

0

0

3

0

1

0

1

−50

𝑥

𝑦

𝑧

𝑠

𝑡

𝑢

𝑝

*𝑠 𝑥 𝑧

0 2 0

−1 1 1

0 0 1

−2 0 0

1

−1 0

2 0 −1

0 0 0

50 50 50

𝑝

0

−3

0

0

1

3

1

−200

𝑅1 − 2𝑅3

𝑅4 − 3𝑅3

The first starred row is the s-row, and its largest positive entry is the 2 in the 𝑢-column. Thus, we use this column as the pivot column. The only positive entry in this column is the 2 in the s-row, so we pivot on that entry. 𝑥

𝑦

𝑧

𝑠

𝑡

𝑢

𝑝

*𝑠 𝑥

0

−1

0

−2

1

2

0

50

2

1

0

0

−1

0

0

50

𝑧

0

1

1

0

0

−1

0

50

2𝑅3 + 𝑅1

𝑝

0

−3

0

0

1

3

1

−200

2𝑅4 − 3𝑅1

𝑥

𝑦

𝑧

𝑠

𝑡

𝑢

𝑝

𝑢 𝑥 𝑧

0 2 0

−1 1 1

0 0 2

−2 0 −2

1 −1 1

2 0 0

0 0 0

50 50 150

𝑝

0

−3

0

6

−1

0

2

−550

As there are no more starred rows, we go to Phase 2, and do the standard simplex method.


Solutions Section 6.4 𝑥

𝑦

𝑧

𝑠

𝑡

𝑢

𝑝

𝑢

0

−1

0

−2

1

2

0

50

𝑥

2

1

0

0

−1

0

0

50

𝑧

0

1

2

−2

1

0

0

150

𝑅3 − 𝑅2

𝑝

0

−3

0

6

−1

0

2

−550

𝑅4 + 3𝑅2

𝑥

𝑦

𝑧

𝑠

𝑡

𝑢

𝑝

𝑢

2

0

0

−2

0

2

0

100

𝑦

2

1

0

0

−1

0

0

50

𝑧

−2

0

2

−2

2

0

0

100

𝑝

6

0

0

6

−4

0

2

−400

𝑥

𝑦

𝑧

𝑠

𝑡

𝑢

𝑝

𝑢 𝑦 𝑡

2 2

−2

0 2 0

0 2 2

−2 −2 −2

0 0 2

2 0 0

0 0 0

100 200 100

𝑝

2

0

4

2

0

0

2

−200

𝑅1 + 𝑅2

2𝑅2 + 𝑅3 𝑅4 + 2𝑅3

Optimal solution: 𝑝 = −200∕2 = −100; 𝑥 = 0, 𝑦 = 200∕2 = 100, 𝑧 = 0. Since 𝑐 = −𝑝, the minimum value of c is 100. 14. Convert the given problem into a maximization problem: Maximize 𝑝 = −2𝑥 − 2𝑦 − 3𝑧 subject to 𝑥 + 𝑧 ≥ 100, 2𝑥 + 𝑦 ≥ 50, 𝑦 + 𝑧 ≥ 50, 𝑥 ≥ 0, 𝑦 ≥ 0, 𝑧 ≥ 0. Introduce slack variables and rewrite the constraints and objective function in standard form, and set up the first tableau: 𝑥 + 𝑧 − 𝑠 = 100 2𝑥 + 𝑦 − 𝑡 = 50 𝑦 + 𝑧 − 𝑢 = 50 2𝑥 + 2𝑦 + 3𝑧 + 𝑝 = 0 𝑥

𝑦

𝑧

𝑠

𝑡

𝑢

𝑝

*𝑠 *𝑡 *𝑢

1 2 0

0 1 1

1 0 1

−1 0 0

0 −1 0

0 0 −1

0 0 0

100 50 50

𝑝

2

2

3

0

0

0

1

0

The first starred row is the s-row, and its largest positive entry is the 1 in the 𝑥-column. Thus, we use this column as the pivot column. The test ratios are: s: 100/1, t: 50/2. The smallest test ratio is t: 50/2. Thus we pivot on the 2 in the t-row.


Solutions Section 6.4 𝑥

𝑦

𝑧

𝑠

𝑡

𝑢

𝑝

*𝑠 *𝑡

1

0

1

−1

0

0

0

100

2

1

0

0

−1

0

0

50

*𝑢

0

1

1

0

0

−1

0

50

𝑝

2

2

3

0

0

0

1

0

𝑥

𝑦

𝑧

𝑠

𝑡

𝑢

𝑝

*𝑠 𝑥 *𝑢

0 2 0

−1 1 1

2 0 1

−2 0 0

1 −1 0

0 0 −1

0 0 0

150 50 50

𝑝

0

1

3

0

1

0

1

−50

2𝑅1 − 𝑅2

𝑅4 − 𝑅2

The first starred row is the s-row, and its largest positive entry is the 2 in the 𝑧-column. Thus, we use this column as the pivot column. The test ratios are: s: 150/2, u: 50/1. The smallest test ratio is u: 50/1. Thus we pivot on the 1 in the u-row. 𝑥

𝑦

𝑧

𝑠

𝑡

𝑢

𝑝

*𝑠 𝑥

0

−1

2

−2

1

0

0

150

2

1

0

0

−1

0

0

50

*𝑢

0

1

1

0

0

−1

0

50

𝑝

0

1

3

0

1

0

1

−50

𝑥

𝑦

𝑧

𝑠

𝑡

𝑢

𝑝

*𝑠 𝑥 𝑧

0 2 0

−3 1 1

0 0 1

−2 0 0

1 −1 0

2 0 −1

0 0 0

50 50 50

𝑝

0

−2

0

0

1

3

1

−200

𝑅1 − 2𝑅3

𝑅4 − 3𝑅3

The first starred row is the s-row, and its largest positive entry is the 2 in the 𝑢-column. Thus, we use this column as the pivot column. The only positive entry in this column is the 2 in the s-row, so we pivot on that entry. 𝑥

𝑦

𝑧

𝑠

𝑡

𝑢

𝑝

*𝑠 𝑥

0

−3

0

−2

1

2

0

50

2

1

0

0

−1

0

0

50

𝑧

0

1

1

0

0

−1

0

50

2𝑅3 + 𝑅1

𝑝

0

−2

0

0

1

3

1

−200

2𝑅4 − 3𝑅1


Solutions Section 6.4 𝑥

𝑦

𝑧

𝑠

𝑡

𝑢

𝑝

𝑢 𝑥 𝑧

0 2 0

−3 1 −1

0 0 2

−2 0 −2

1

−1 1

2 0 0

0 0 0

50 50 150

𝑝

0

5

0

6

−1

0

2

−550

As there are no more starred rows, we go to Phase 2, and do the standard simplex method. 𝑥

𝑦

𝑧

𝑠

𝑡

𝑢

𝑝

𝑢

0

−3

0

−2

1

2

0

50

𝑥

2

1

0

0

−1

0

0

50

𝑅2 + 𝑅1

𝑧

0

−1

2

−2

1

0

0

150

𝑅3 − 𝑅1

𝑝

0

5

0

6

−1

0

2

−550

𝑅4 + 𝑅1

𝑥

𝑦

𝑧

𝑠

𝑡

𝑢

𝑝

𝑡 𝑥 𝑧

0 2 0

−3 −2 2

0 0 2

−2 −2 0

1 0 0

2 2 −2

0 0 0

50 100 100

𝑝

0

2

0

4

0

2

2

−500

Optimal solution: 𝑝 = −500∕2 = −250; 𝑥 = 100∕2 = 50, 𝑦 = 0, 𝑧 = 100∕2 = 50. Since 𝑐 = −𝑝, the minimum value of c is 250. 15. Convert the given problem into a maximization problem: Maximize 𝑝 = −50𝑥 − 50𝑦 + 11𝑧 subject to 2𝑥 + 𝑧 ≥ 3, 2𝑥 + 𝑦 − 𝑧 ≥ 2, 3𝑥 + 𝑦 − 𝑧 ≤ 3, 𝑥 ≥ 0, 𝑦 ≥ 0, 𝑧 ≥ 0. Introduce slack variables and rewrite the constraints and objective function in standard form, and set up the first tableau: 2𝑥 + 𝑧 − 𝑠 = 3 2𝑥 + 𝑦 − 𝑧 − 𝑡 = 2 3𝑥 + 𝑦 − 𝑧 + 𝑢 = 3 50𝑥 + 50𝑦 − 11𝑧 + 𝑝 = 0 𝑥

𝑦

𝑧

𝑠

𝑡

𝑢

𝑝

*𝑠 *𝑡 𝑢

2 2 3

0 1 1

1 −1 −1

−1 0 0

0 −1 0

0 0 1

0 0 0

3 2 3

𝑝

50

50

−11

0

0

0

1

0

The first starred row is the s-row, and its largest positive entry is the 2 in the 𝑥-column. Thus, we use this column as the pivot column. The test ratios are: s: 3/2, t: 2/2, u: 3/3. The smallest test ratio is t: 2/2. Thus we pivot on the 2 in the t-row.


Solutions Section 6.4 𝑥

𝑦

𝑧

𝑠

𝑡

𝑢

𝑝

*𝑠 *𝑡

2

0

1

−1

0

0

0

3

2

1

−1

0

−1

0

0

2

𝑢

3

1

−1

0

0

1

0

3

2𝑅3 − 3𝑅2

𝑝

50

50

−11

0

0

0

1

0

𝑅4 − 25𝑅2

𝑥

𝑦

𝑧

𝑠

𝑡

𝑢

𝑝

*𝑠 𝑥 𝑢

0 2 0

−1 1 −1

2 −1 1

−1 0 0

1 −1 3

0 0 2

0 0 0

1 2 0

𝑝

0

25

14

0

25

0

1

−50

𝑅1 − 𝑅2

The first starred row is the s-row, and its largest positive entry is the 2 in the 𝑧-column. Thus, we use this column as the pivot column. The test ratios are: s: 1/2, u: 0/1. The smallest test ratio is u: 0/1. Thus we pivot on the 1 in the u-row. 𝑥

𝑦

𝑧

𝑠

𝑡

𝑢

𝑝

*𝑠 𝑥

0

−1

2

−1

1

0

0

1

𝑅1 − 2𝑅3

2

1

−1

0

−1

0

0

2

𝑅2 + 𝑅3

𝑢

0

−1

1

0

3

2

0

0

𝑝

0

25

14

0

25

0

1

−50

𝑥

𝑦

𝑧

𝑠

𝑡

𝑢

𝑝

*𝑠 𝑥 𝑧

0 2 0

1 0

−1

0 0 1

−1 0 0

−5 2 3

−4 2 2

0 0 0

1 2 0

𝑝

0

39

0

0

−17

−28

1

−50

𝑅4 − 14𝑅3

The first starred row is the s-row, and its largest positive entry is the 1 in the 𝑦-column. Thus, we use this column as the pivot column. The only positive entry in this column is the 1 in the s-row, so we pivot on that entry. 𝑥

𝑦

𝑧

𝑠

𝑡

𝑢

𝑝

*𝑠 𝑥

0

1

0

−1

−5

−4

0

1

2

0

0

0

2

2

0

2

𝑧

0

−1

1

0

3

2

0

0

𝑅3 + 𝑅1

𝑝

0

39

0

0

−17

−28

1

−50

𝑅4 − 39𝑅1


Solutions Section 6.4 𝑥

𝑦

𝑧

𝑠

𝑡

𝑢

𝑝

𝑦 𝑥 𝑧

0 2 0

1 0 0

0 0 1

−1 0 −1

−5 2 −2

−4 2 −2

0 0 0

1 2 1

𝑝

0

0

0

39

178

128

1

−89

As there are no more starred rows, we go to Phase 2, and do the standard simplex method. However, there are no negative numbers in the bottom row, so there are no pivot steps to do in Phase 2. Therefore, we are done. Optimal solution: 𝑝 = −89∕1 = −89; 𝑥 = 2∕2 = 1, 𝑦 = 1∕1 = 1, 𝑧 = 1∕1 = 1. Since 𝑐 = −𝑝, the minimum value of c is 89. 16. Convert the given problem into a maximization problem: Maximize 𝑝 = 50𝑥 − 11𝑦 − 50𝑧 subject to 3𝑥 + 𝑧 ≥ 8, 3𝑥 + 𝑦 − 𝑧 ≥ 6, 4𝑥 + 𝑦 − 𝑧 ≤ 8, 𝑥 ≥ 0, 𝑦 ≥ 0, 𝑧 ≥ 0. Introduce slack variables and rewrite the constraints and objective function in standard form, and set up the first tableau: 3𝑥 + 𝑧 − 𝑠 = 8 3𝑥 + 𝑦 − 𝑧 − 𝑡 = 6 4𝑥 + 𝑦 − 𝑧 + 𝑢 = 8 −50𝑥 + 11𝑦 + 50𝑧 + 𝑝 = 0 𝑥

𝑦

𝑧

𝑠

𝑡

𝑢

𝑝

*𝑠 *𝑡 𝑢

3 3 4

0 1 1

1 −1 −1

−1 0 0

0 −1 0

0 0 1

0 0 0

8 6 8

𝑝

−50

11

50

0

0

0

1

0

The first starred row is the s-row, and its largest positive entry is the 3 in the 𝑥-column. Thus, we use this column as the pivot column. The test ratios are: s: 8/3, t: 6/3, u: 8/4. The smallest test ratio is t: 6/3. Thus we pivot on the 3 in the t-row. 𝑥

𝑦

𝑧

𝑠

𝑡

𝑢

𝑝

*𝑠 *𝑡

3

0

1

−1

0

0

0

8

3

1

−1

0

−1

0

0

6

𝑢

4

1

−1

0

0

1

0

8

3𝑅3 − 4𝑅2

𝑝

−50

11

50

0

0

0

1

0

3𝑅4 + 50𝑅2

𝑥

𝑦

𝑧

𝑠

𝑡

𝑢

𝑝

*𝑠 𝑥 𝑢

0 3 0

−1 1 −1

2 −1 1

−1 0 0

1 −1 4

0 0 3

0 0 0

2 6 0

𝑝

0

83

100

0

−50

0

3

300

𝑅1 − 𝑅2

The first starred row is the s-row, and its largest positive entry is the 2 in the 𝑧-column. Thus, we use this column as the pivot column. The test ratios are: s: 2/2, u: 0/1. The smallest test ratio is u: 0/1. Thus we


Solutions Section 6.4 pivot on the 1 in the u-row. 𝑥

𝑦

𝑧

𝑠

𝑡

𝑢

𝑝

*𝑠 𝑥

0

−1

2

−1

1

0

0

2

𝑅1 − 2𝑅3

3

1

−1

0

−1

0

0

6

𝑅2 + 𝑅3

𝑢

0

−1

1

0

4

3

0

0

𝑝

0

83

100

0

−50

0

3

300

𝑥

𝑦

𝑧

𝑠

𝑡

𝑢

𝑝

*𝑠 𝑥 𝑧

0 3 0

1 0 −1

0 0 1

−1 0 0

−7 3 4

−6 3 3

0 0 0

2 6 0

𝑝

0

183

0

0

−450 −300

3

300

𝑅4 − 100𝑅3

The first starred row is the s-row, and its largest positive entry is the 1 in the 𝑦-column. Thus, we use this column as the pivot column. The only positive entry in this column is the 1 in the s-row, so we pivot on that entry. 𝑥

𝑦

𝑧

𝑠

𝑡

𝑢

𝑝

*𝑠 𝑥

0

1

0

−1

−7

−6

0

2

3

0

0

0

3

3

0

6

𝑧

0

−1

1

0

4

3

0

0

𝑅3 + 𝑅1

𝑝

0

183

0

0

−450 −300

3

300

𝑅4 − 183𝑅1

𝑥

𝑦

𝑧

𝑠

𝑡

𝑢

𝑝

𝑦 𝑥 𝑧

0 3 0

1 0 0

0 0 1

−1 0 −1

−7 3 −3

−6 3 −3

0 0 0

2 6 2

𝑝

0

0

0

183

831

798

3

−66

As there are no more starred rows, we go to Phase 2, and do the standard simplex method. However, there are no negative numbers in the bottom row, so there are no pivot steps to do in Phase 2. Therefore, we are done. Optimal solution: 𝑝 = −66∕3 = −22; 𝑥 = 6∕3 = 2, 𝑦 = 2∕1 = 2, 𝑧 = 2∕1 = 2. Since 𝑐 = −𝑝, the minimum value of c is 22. 17. Convert the given problem into a maximization problem: Maximize 𝑝 = −𝑥 − 𝑦 − 𝑧 − 𝑤 subject to 5𝑥 − 𝑦 + 𝑤 ≥ 1,000, 𝑧 + 𝑤 ≤ 2,000, 𝑥 + 𝑦 ≤ 500, 𝑥 ≥ 0, 𝑦 ≥ 0, 𝑧 ≥ 0, 𝑤 ≥ 0. Introduce slack variables and rewrite the constraints and objective function in standard form, and set up the first tableau: 5𝑥 − 𝑦 + 𝑤 − 𝑠 = 1,000 𝑧 + 𝑤 + 𝑡 = 2,000 𝑥 + 𝑦 + 𝑢 = 500 𝑥+𝑦+𝑧+𝑤+𝑝=0


Solutions Section 6.4 𝑥

𝑦

𝑧

𝑤

𝑠

𝑡

𝑢

𝑝

*𝑠 𝑡 𝑢

5 0 1

−1 0 1

0 1 0

1 1 0

−1 0 0

0 1 0

0 0 1

0 0 0

1000 2000 500

𝑝

1

1

1

1

0

0

0

1

0

The first starred row is the s-row, and its largest positive entry is the 5 in the 𝑥-column. Thus, we use this column as the pivot column. The test ratios are: s: 1000/5, u: 500/1. The smallest test ratio is s: 1000/5. Thus we pivot on the 5 in the s-row. 𝑥

𝑦

𝑧

𝑤

𝑠

𝑡

𝑢

𝑝

*𝑠 𝑡

5

−1

0

1

−1

0

0

0

1000

0

0

1

1

0

1

0

0

2000

𝑢

1

1

0

0

0

0

1

0

500

5𝑅3 − 𝑅1

𝑝

1

1

1

1

0

0

0

1

0

5𝑅4 − 𝑅1

𝑥

𝑦

𝑧

𝑤

𝑠

𝑡

𝑢

𝑝

𝑥 𝑡 𝑢

5 0 0

−1 0 6

0 1 0

1 1 −1

−1 0 1

0 1 0

0 0 5

0 0 0

1000 2000 1500

𝑝

0

6

5

4

1

0

0

5

−1000

As there are no more starred rows, we go to Phase 2, and do the standard simplex method. However, there are no negative numbers in the bottom row, so there are no pivot steps to do in Phase 2. Therefore, we are done. Optimal solution: 𝑝 = −1,000∕5 = −200; 𝑥 = 1,000∕5 = 200, 𝑦 = 0, 𝑧 = 0, 𝑤 = 0. Since 𝑐 = −𝑝, the minimum value of c is 200. 18. Convert the given problem into a maximization problem: Maximize 𝑝 = −5𝑥 − 𝑦 − 𝑧 − 𝑤 subject to 5𝑥 − 𝑦 + 𝑤 ≥ 1,000, 𝑧 + 𝑤 ≤ 2,000, 𝑥 + 𝑦 ≤ 500, 𝑥 ≥ 0, 𝑦 ≥ 0, 𝑧 ≥ 0, 𝑤 ≥ 0. Introduce slack variables and rewrite the constraints and objective function in standard form, and set up the first tableau: 5𝑥 − 𝑦 + 𝑤 − 𝑠 = 1,000 𝑧 + 𝑤 + 𝑡 = 2,000 𝑥 + 𝑦 + 𝑢 = 500 5𝑥 + 𝑦 + 𝑧 + 𝑤 + 𝑝 = 0 𝑥

𝑦

𝑧

𝑤

𝑠

𝑡

𝑢

𝑝

*𝑠 𝑡 𝑢

5 0 1

−1 0 1

0 1 0

1 1 0

−1 0 0

0 1 0

0 0 1

0 0 0

1000 2000 500

𝑝

5

1

1

1

0

0

0

1

0

The first starred row is the s-row, and its largest positive entry is the 5 in the 𝑥-column. Thus, we use this column as the pivot column. The test ratios are: s: 1000/5, u: 500/1. The smallest test ratio is s: 1000/5.


Solutions Section 6.4 Thus we pivot on the 5 in the s-row. 𝑥

𝑦

𝑧

𝑤

𝑠

𝑡

𝑢

𝑝

*𝑠 𝑡

5

−1

0

1

−1

0

0

0

1000

0

0

1

1

0

1

0

0

2000

𝑢

1

1

0

0

0

0

1

0

500

5𝑅3 − 𝑅1

𝑝

5

1

1

1

0

0

0

1

0

𝑅4 − 𝑅1

𝑥

𝑦

𝑧

𝑤

𝑠

𝑡

𝑢

𝑝

𝑥 𝑡 𝑢

5 0 0

−1 0 6

0 1 0

1 1 −1

−1 0 1

0 1 0

0 0 5

0 0 0

1000 2000 1500

𝑝

0

2

1

0

1

0

0

1

−1000

As there are no more starred rows, we go to Phase 2, and do the standard simplex method. However, there are no negative numbers in the bottom row, so there are no pivot steps to do in Phase 2. Therefore, we are done. Optimal solution: 𝑝 = −1,000∕1 = −1,000; 𝑥 = 1,000∕5 = 200, 𝑦 = 0, 𝑧 = 0, 𝑤 = 0. Since 𝑐 = −𝑝, the minimum value of c is 1,000.If we pivot on the 𝑤 column, we get another optimal solution: 𝑐 = 1,000; 𝑥 = 𝑦 = 0, 𝑧 = 0, 𝑤 = 1,000. 19. You can use the online Pivot and Gauss-Jordan Tool in decimal mode to do the pivoting, or use the online Simplex Method Tool. When entering problems in the Simplex Method Tool there is no need to enter the inequalities 𝑥 ≥ 0, 𝑦 ≥ 0, etc. 𝑥 𝑦 𝑧 𝑤 𝑠 𝑡 𝑢 𝑝 𝑠

1.2

1

1

1

1

0

0

0

40.5

*𝑡 *𝑢

2.2

1

−1

−1

0

−1

0

0

10

1.2

1

1

1.2

0

0

−1

0

10.5

𝑝

−2

−3

−1.1

−4

0

0

0

1

0

𝑥

𝑦

𝑧

𝑤

𝑠

𝑡

𝑢

𝑝

𝑠 𝑥

0 1

0.45 0.45

1.55 1.55 −0.45 −0.45

1 0

0.55 −0.45

0 0

0 0

35.05 4.55

*𝑢

0

0.45

1.55

1.75

0

0.55

−1

0

5.05

𝑝

0

−2.09 −2.01 −4.91

0

−0.91

0

1

9.09

𝑥

𝑦

𝑧

𝑤

𝑠

𝑡

𝑢

𝑝

𝑠

0

0.05

0.18

0

1

0.06

0.89

0

30.58

𝑥 𝑤

1 0

0.57 0.26

−0.05 0.89

0 1

0 0

−0.31 −0.26 0.31 −0.57

0 0

5.86 2.89

𝑝

0

−0.81

2.34

0

0

0.63

1

23.28

−2.81


Solutions Section 6.4 𝑥

𝑦

𝑧

𝑤

𝑠

𝑡

𝑢

𝑝

𝑢

0

0.06

0.2

0

1.13

0.07

1

0

34.54

𝑥

1

0.59

0

0

0.29

−0.29

0

0

14.85

𝑤

0

0.29

1

1

0.65

0.35

0

0

22.68

𝑝

0

−0.65

2.9

0

3.18

0.82

0

1

120.41

𝑥

𝑦

𝑧

𝑤

𝑠

𝑡

𝑢

𝑝

𝑢 𝑦 𝑤

−0.1 1.7 −0.5

0 1 0

0.2 0 1

0 0 1

1.1 0.5 0.5

0.1 −0.5 0.5

1 0 0

0 0 0

33.05 25.25 15.25

𝑝

1.1

0

2.9

0

3.5

0.5

0

1

136.75

Optimal solution: 𝑝 = 136.75; 𝑥 = 0, 𝑦 = 25.25, 𝑧 = 0, 𝑤 = 15.25. 20. You can use the online Pivot and Gauss-Jordan Tool in decimal mode to do the pivoting, or use the online Simplex Method Tool. When entering problems in the Simplex Method Tool there is no need to enter the inequalities 𝑥 ≥ 0, 𝑦 ≥ 0, etc. 𝑥 𝑦 𝑧 𝑤 𝑠 𝑡 𝑢 𝑝 𝑠

1

1.5

1.5

1

1

0

0

0

50.5

*𝑡 *𝑢

2

1.5

−1

−1

0

−1

0

0

10

1

1.5

1

1.5

0

0

−1

0

21

𝑝

−2.2

−2

−1.1

−2

0

0

0

1

0

𝑥

𝑦

𝑧

𝑤

𝑠

𝑡

𝑢

𝑝

𝑠 𝑥

0 1

0.75 0.75

2

−0.5

1.5

−0.5

1 0

0.5

−0.5

0 0

0 0

45.5 5

*𝑢

0

0.75

1.5

2

0

0.5

−1

0

16

𝑝

0

−0.35 −2.2

−3.1

0

−1.1

0

1

11

𝑥

𝑦

𝑧

𝑤

𝑠

𝑡

𝑢

𝑝

𝑠

0

0.19

0.88

0

1

0.13

0.75

0

33.5

𝑥 𝑤

1 0

0.94 0.38

−0.13 0.75

0 1

0 0

−0.38 −0.25 0.25 −0.5

0 0

9 8

𝑝

0

0.81

0.13

0

0

−0.33 −1.55

1

35.8


Solutions Section 6.4 𝑥

𝑦

𝑧

𝑤

𝑠

𝑡

𝑢

𝑝

𝑢 𝑥

0 1

0.25 1

1.17 0.17

0 0

1.33 0.33

0.17

−0.33

1 0

0 0

44.67 20.17

𝑤

0

0.5

1.33

1

0.67

0.33

0

0

30.33

𝑝

0

1.2

1.93

0

2.07

−0.07

0

1

105.03

𝑥

𝑦

𝑧

𝑤

𝑠

𝑡

𝑢

𝑝

𝑢 𝑥 𝑡

0 1 0

0 1.5 1.5

0.5 1.5 4

−0.5 1 3

1 1 2

0 0 1

1 0 0

0 0 0

29.5 50.5 91

𝑝

0

1.3

2.2

0.2

2.2

0

0

1

111.1

Optimal solution: 𝑝 = 111.1; 𝑥 = 50.5, 𝑦 = 0, 𝑧 = 0, 𝑤 = 0. 21. First convert the problem to a maximization problem as in Example 3 by taking 𝑝 to be the negative of 𝑐. Then use the online Pivot and Gauss-Jordan Tool in decimal mode to do the pivoting, or use the online Simplex Method Tool. When entering problems in the Simplex Method Tool there is no need to enter the inequalities 𝑥 ≥ 0, 𝑦 ≥ 0, etc. 𝑥 𝑦 𝑧 𝑠 𝑡 𝑢 𝑝 *𝑠 *𝑡

1

1.5

1.2

−1

0

0

0

100

2

1.5

0

0

−1

0

0

50

*𝑢

0

1.5

1.1

0

0

−1

0

50

𝑝

2.2

1

3.3

0

0

0

1

0

𝑥

𝑦

𝑧

𝑠

𝑡

𝑢

𝑝

*𝑠 𝑦

−1 1.33

0 1

1.2 0

−1 0

1 −0.67

0 0

0 0

50 33.33

*𝑢

−2

0

1.1

0

1

−1

0

0

𝑝

0.87

0

3.3

0

0.67

0

1

−33.33

𝑥

𝑦

𝑧

𝑠

𝑡

𝑢

𝑝

1.18

0

0

−1

−0.09

1.09

0

50

1.33

1

0

0

−0.67

0

0

33.33

𝑧 −1.82

0

1

0

0.91

−0.91

0

0

𝑝

0

0

0

−2.33

3

1

−33.33

*𝑠 𝑦

6.87


Solutions Section 6.4 𝑥

𝑦

𝑧

𝑠

𝑡

𝑢

𝑝

*𝑠 𝑥 𝑧

0

−0.89

0

−1

0.5

1.09

0

20.45

1 0

0.75 1.36

0 1

0 0

0 −0.5 0 −0.91

0 0

25 45.45

𝑝

0

−5.15

0

0

1.1

3

1

−205

𝑥

𝑦

𝑧

𝑠

𝑡

𝑢

𝑝

𝑢

0

−0.81

0

−0.92

0.46

1

0

18.75

𝑥

1

0.75

0

0

−0.5

0

0

25

𝑧

0

0.62

1

−0.83

0.42

0

0

62.5

𝑝

0

−2.71

0

2.75

−0.27

0

1

−261.25

𝑥

𝑦

𝑧

𝑠

𝑡

𝑢

𝑝

1.08 1.33

0 1

0 0

−0.92 −0.08 0 −0.67

1 0

0 0

45.83 33.33

𝑧 −0.83

0

1

−0.83

0.83

0

0

41.67

𝑝

3.62

0

0

2.75

−2.08

0

1

−170.83

𝑥

𝑦

𝑧

𝑠

𝑡

𝑢

𝑝

𝑢 𝑦 𝑡

1 0.67 −1

0 1 0

0.1 0.8 1.2

−1 −0.67 −1

0 0 1

1 0 0

0 0 0

50 66.67 50

𝑝

1.53

0

2.5

0.67

0

0

1

−66.67

𝑢 𝑦

Optimal solution: 𝑝 = −66.67; 𝑥 = 0, 𝑦 = 66.67, 𝑧 = 0. Since 𝑐 = −𝑝, the minimum value of c is 66.67. 22. First convert the problem to a maximization problem as in Example 3 by taking 𝑝 to be the negative of 𝑐. Then use the online Pivot and Gauss-Jordan Tool in decimal mode to do the pivoting, or use the online Simplex Method Tool. When entering problems in the Simplex Method Tool there is no need to enter the inequalities 𝑥 ≥ 0, 𝑦 ≥ 0, etc. 𝑥 𝑦 𝑧 𝑠 𝑡 𝑢 𝑝 *𝑠 *𝑡 *𝑢

3.1 3.1

0 1

1.1 −1.1

−1 0

0 −1

0 0

0 0

28 23

4.2

1

−1.1

0

0

−1

0

28

𝑝

50.3

10.5

50.3

0

0

0

1

0


Solutions Section 6.4 𝑥

𝑦

𝑧

𝑠

𝑡

𝑢

𝑝

*𝑠 *𝑡 𝑥

0

−0.74

1.91

−1

0

0.74

0

7.33

0 1

0.26 0.24

−0.29 −0.26

0 0

−1 0

0.74 −0.24

0 0

2.33 6.67

𝑝

0

−1.48 63.47

0

0

11.98

1

−335.33

𝑥

𝑦

𝑧

𝑠

𝑡

𝑢

𝑝

𝑧

0

−0.39

1

−0.52

0

0.39

0

3.84

*𝑡 𝑥

0

0.15

0

−0.15

−1

0.85

0

3.44

1

0.14

0

−0.14

0

−0.14

0

7.67

𝑝

0

23.03

0

33.2

0

−12.53

1

−578.79

𝑥

𝑦

𝑧

𝑠

𝑡

𝑢

𝑝

𝑧

0

−0.45

1

−0.45

0.45

0

0

2.27

𝑢 𝑥

0 1

0.18 0.16

0 0

−0.18 −1.18 −0.16 −0.16

1 0

0 0

4.05 8.23

𝑝

0

25.25

0

30.98 −14.75

0

1

−528.08

𝑥

𝑦

𝑧

𝑠

𝑡

𝑢

𝑝

𝑡 𝑢 𝑥

0 0 1

−1 −1 0

2.2 2.59 0.35

−1 −1.35 −0.32

1 0 0

0 1 0

0 0 0

5 9.94 9.03

𝑝

0

10.5

32.45 16.23

0

0

1

−454.32

Optimal solution: 𝑝 = −454.32; 𝑥 = 9.03, 𝑦 = 0, 𝑧 = 0. Since 𝑐 = −𝑝, the minimum value of c is 454.32. 23. First convert the problem to a maximization problem as in Example 3 by taking 𝑝 to be the negative of 𝑐. Then use the online Pivot and Gauss-Jordan Tool in decimal mode to do the pivoting, or use the online Simplex Method Tool. When entering problems in the Simplex Method Tool there is no need to enter the inequalities 𝑥 ≥ 0, 𝑦 ≥ 0, etc. 𝑥 𝑦 𝑧 𝑤 𝑠 𝑡 𝑢 𝑝 𝑠

5.12

−1

0

1

1

0

0

0

1000

*𝑡 𝑢

0

0

1

1

0

−1

0

0

2000

1.22

1

0

0

0

0

1

0

500

𝑝

1.1

1

1.5

−1

0

0

0

1

0


Solutions Section 6.4 𝑥

𝑦

𝑧

𝑤

𝑠

𝑡

𝑢

𝑝

𝑠

5.12

−1

0

1

1

0

0

0

1000

𝑧 𝑢

0 1.22

0 1

1 0

1 0

0 0

−1 0

0 1

0 0

2000 500

𝑝

1.1

1

0

−2.5

0

1.5

0

1

−3000

𝑥

𝑦

𝑧

𝑤

𝑠

𝑡

𝑢

𝑝

𝑤 5.12 𝑧 −5.12

−1 1

0 1

1 0

1 −1

0 −1

0 0

0 0

1000 1000

𝑢

1.22

1

0

0

0

0

1

0

500

𝑝

13.9

−1.5

0

0

2.5

1.5

0

1

−500

𝑥

𝑦

𝑧

𝑤

𝑠

𝑡

𝑢

𝑝

𝑤 6.34 𝑧 −6.34 𝑦 1.22

0 0 1

0 1 0

1 0 0

1 −1 0

0 −1 0

1 −1 1

0 0 0

1500 500 500

𝑝 15.73

0

0

0

2.5

1.5

1.5

1

250

Optimal solution: 𝑝 = 250; 𝑥 = 0, 𝑦 = 500, 𝑧 = 500, 𝑤 = 1,500. Since 𝑐 = −𝑝, the minimum value of c is -250. 24. First convert the problem to a maximization problem as in Example 3 by taking 𝑝 to be the negative of 𝑐. Then use the online Pivot and Gauss-Jordan Tool in decimal mode to do the pivoting, or use the online Simplex Method Tool. When entering problems in the Simplex Method Tool there is no need to enter the inequalities 𝑥 ≥ 0, 𝑦 ≥ 0, etc. 𝑥 𝑦 𝑧 𝑤 𝑠 𝑡 𝑢 𝑝 *𝑠 *𝑡 𝑢

5.12

−1

0

1

−1

0

0

0

1000

0 1.12

0 1

1 0

1 0

0 0

−1 0

0 1

0 0

2000 500

𝑝

5.45

1

1.5

1

0

0

0

1

0

𝑥

𝑦

𝑧

𝑤

𝑠

𝑡

𝑢

𝑝

𝑥

1

−0.2

0

0.2

−0.2

0

0

0

195.31

*𝑡 𝑢

0

0

1

1

0

−1

0

0

2000

0

1.22

0

−0.22

0.22

0

1

0

281.25

𝑝

0

2.06

1.5

−0.06

1.06

0

0

1

−1064.45


Solutions Section 6.4 𝑥

𝑦

𝑧

𝑤

𝑠

𝑡

𝑢

𝑝

𝑥

1

−0.2

0

0.2

−0.2

0

0

0

195.31

𝑧 𝑢

0 0

0 1.22

1 0

1 −0.22

0 0.22

−1 0

0 1

0 0

2000 281.25

𝑝

0

2.06

0

−1.56

1.06

1.5

0

1

−4064.45

𝑥

𝑦

𝑧

𝑤

𝑠

𝑡

𝑢

𝑝

5.12

−1

0

1

−1

0

0

0

1000

𝑧 −5.12

1

1

0

1

−1

0

0

1000

𝑢

1.12

1

0

0

0

0

1

0

500

𝑝

8.01

0.5

0

0

−0.5

1.5

0

1

−2500

𝑥

𝑦

𝑧

𝑤

𝑠

𝑡

𝑢

𝑝

𝑤 0 𝑠 −5.12 𝑢 1.12

0 1 1

1 1 0

1 0 0

0 1 0

−1 −1 0

0 0 1

0 0 0

2000 1000 500

𝑝

1

0.5

0

0

1

0

1

−2000

𝑤

5.45

Optimal solution: 𝑝 = −2,000; 𝑥 = 0, 𝑦 = 0, 𝑧 = 0, 𝑤 = 2,000. Since 𝑐 = −𝑝, the minimum value of c is 2,000. 25. Unknowns: 𝑥 = # acres of tomatoes, 𝑦 = # acres of lettuce, 𝑧 = # acres of carrots Maximize 𝑝 = 20𝑥 + 15𝑦 + 5𝑧 (in hundreds of dollars) subject to 𝑥 + 𝑦 + 𝑧 ≤ 100, 5𝑥 + 4𝑦 + 2𝑧 ≥ 400, 4𝑥 + 2𝑦 + 2𝑧 ≤ 500, 𝑥 ≥ 0, 𝑦 ≥ 0, 𝑧 ≥ 0. 𝑥 𝑦 𝑧 𝑠 𝑡 𝑢 𝑝 5𝑅1 − 𝑅2

𝑠

1

1

1

1

0

0

0

100

*𝑡 𝑢

5

4

2

0

−1

0

0

400

4

2

2

0

0

1

0

500

5𝑅3 − 4𝑅2

𝑝

−20

−15

−5

0

0

0

1

0

𝑅4 + 4𝑅2

𝑥

𝑦

𝑧

𝑠

𝑡

𝑢

𝑝

𝑠

0

1

3

5

1

0

0

100

𝑥

5

4

2

0

−1

0

0

400

𝑅2 + 𝑅1

𝑢

0

−6

2

0

4

5

0

900

𝑅3 − 4𝑅1

𝑝

0

1

3

0

−4

0

1

1600

𝑅4 + 4𝑅1


Solutions Section 6.4 𝑥

𝑦

𝑧

𝑠

𝑡

𝑢

𝑝

𝑡 𝑥 𝑢

0 5 0

1 5 −10

3 5 −10

5 5 −20

1 0 0

0 0 5

0 0 0

100 500 500

𝑝

0

5

15

20

0

0

1

2000

Optimal solution: 𝑝 = 2,000∕1 = 2,000; 𝑥 = 500∕5 = 100, 𝑦 = 0, 𝑧 = 0. Plant 100 acres of tomatoes and no other crops. This will give you a profit of $200,000. Since the slack variable corresponding to farm area is 𝑠 = 0, you will be using all 100 acres of farm. 26. Unknowns: 𝑥 = # soy beans, 𝑦 = # acres of corn, 𝑧 = # acres of wheat Maximize 𝑝 = 3𝑥 + 2𝑦+z (in thousands of dollars) subject to 𝑥 + 𝑦 + 𝑧 ≤ 900, 5𝑥 + 2𝑦 + 𝑧 ≤ 3,000, 5𝑥 + 2𝑦 + 2𝑧 ≥ 2,000, 𝑥 ≥ 0, 𝑦 ≥ 0, 𝑧 ≥ 0. 𝑥 𝑦 𝑧 𝑠 𝑡 𝑢 𝑝 𝑠

1

1

1

1

0

0

0

900

5𝑅1 − 𝑅3

𝑡

5

2

1

0

1

0

0

3000

𝑅2 − 𝑅3

*𝑢

5

2

2

0

0

−1

0

2000

𝑝

−3

−2

−1

0

0

0

1

0

𝑥

𝑦

𝑧

𝑠

𝑡

𝑢

𝑝

𝑠

0

3

3

5

0

1

0

2500

𝑡

0

0

−1

0

1

1

0

1000

𝑥

5

2

2

0

0

−1

0

2000

3𝑅3 − 2𝑅1

𝑝

0

−4

1

0

0

−3

5

6000

3𝑅4 + 4𝑅1

𝑥

𝑦

𝑧

𝑠

𝑡

𝑢

𝑝

𝑦

0

3

3

5

0

1

0

2500

𝑅1 − 𝑅2

𝑡

0

0

−1

0

1

1

0

1000

𝑥

15

0

0

−10

0

−5

0

1000

𝑅3 + 5𝑅2

𝑝

0

0

15

20

0

−5

15

28000

𝑅4 + 5𝑅2

𝑥

𝑦

𝑧

𝑠

𝑡

𝑢

𝑝

𝑦 𝑢 𝑥

0 0 15

3 0 0

4 −1 −5

5 0 −10

−1 1 5

0 1 0

0 0 0

1500 1000 6000

𝑝

0

0

10

20

5

0

15

33000

5𝑅4 + 3𝑅3

Optimal solution: 𝑝 = 33,000∕15 = 2,200; 𝑥 = 6,000∕15 = 400, 𝑦 = 1,500∕3 = 500, 𝑧 = 0. Plant 400 acres of soy beans and 500 acres of corn. This will give you a profit of $2,200,000. Since the


Solutions Section 6.4 surplus variable corresponding to labor is 𝑢 = 1,000, there is a surplus of 1,000 hours in labor, and so you use 3,000 hours of labor. 27. Unknowns: 𝑥 = # mailings to the East Coast, 𝑦 = # mailings to the Midwest, 𝑧 = # mailings to the West Coast Minimize 𝑐 = 40𝑥 + 60𝑦 + 50𝑧 subject to100𝑥 + 100𝑦 + 50𝑧 ≥ 1,500, 50𝑥 + 100𝑦 + 100𝑧 ≥ 1,500, 𝑥 ≥ 0, 𝑦 ≥ 0, 𝑧 ≥ 0. 𝑥 𝑦 𝑧 𝑠 𝑡 𝑝 *𝑠 *𝑡

100

100

50

−1

0

0

1500

50

100

100

0

−1

0

1500

2𝑅2 − 𝑅1

𝑝

40

60

50

0

0

1

0

5𝑅3 − 2𝑅1

𝑥

𝑦

𝑧

𝑠

𝑡

𝑝

𝑥

100

100

50

−1

0

0

1500

3𝑅1 − 𝑅2

*𝑡

0

100

150

1

−2

0

1500

𝑝

0

100

150

2

0

5

−3000

𝑥

𝑦

𝑧

𝑠

𝑡

𝑝

𝑥 𝑧

300 0

200 100

0 150

−4 1

2 −2

0 0

3000 1500

𝑝

0

0

0

1

2

5

−4500

𝑅3 − 𝑅2

Optimal solution: 𝑝 = −4,500∕5 = −900; 𝑥 = 3,000∕300 = 10, 𝑦 = 0, 𝑧 = 1,500∕150 = 10. Since 𝑐 = −𝑝, the minimum value of c is 900. Send 10 mailings to the East Coast, none to the Midwest, and 10 to the West Coast. Cost: $900. Another solution resulting in the same cost (pivot on the 𝑦-column in the last tableau above) is no mailings to the East Coast, 15 to the Midwest, None to the West Coast. 28. Unknowns: 𝑥 = # packages from Harvard Paper, 𝑦 = # packages from Yale Paper, 𝑧 = # packages from Dartmouth Paper Minimize 𝑐 = 60𝑥 + 40𝑦 + 50𝑧 subject to 20𝑥 + 10𝑦 + 10𝑧 ≥ 350, 10𝑥 + 10𝑦 + 20𝑧 ≥ 400, 𝑥 ≥ 0, 𝑦 ≥ 0, 𝑧 ≥ 0. 𝑥 𝑦 𝑧 𝑠 𝑡 𝑝 *𝑠 *𝑡

20

10

10

−1

0

0

350

10

10

20

0

−1

0

400

2𝑅2 − 𝑅1

𝑝

60

40

50

0

0

1

0

𝑅3 − 3𝑅1

𝑥

𝑦

𝑧

𝑠

𝑡

𝑝

𝑥

20

10

10

−1

0

0

350

3𝑅1 − 𝑅2

*𝑡

0

10

30

1

−2

0

450

𝑝

0

10

20

3

0

1

−1050

3𝑅3 − 2𝑅2


Solutions Section 6.4 𝑥

𝑦

𝑧

𝑠

𝑡

𝑝

𝑥 𝑧

60 0

20 10

0 30

−4 1

2

−2

0 0

600 450

𝑝

0

10

0

7

4

3

−4050

Optimal solution: 𝑝 = −4,050∕3 = −1,350; 𝑥 = 600∕60 = 10, 𝑦 = 0, 𝑧 = 450∕30 = 15. Since 𝑐 = −𝑝, the minimum value of c is 1,350. Buy 10 packages from Harvard, 15 from Dartmouth, and none from Yale. Cost: $1,350. 29. Unknowns: 𝑥 = # quarts orange juice, 𝑦 = # quarts orange concentrate Minimize 𝑐 = 0.5𝑥 + 2𝑦 subject to 𝑥 ≥ 10,000, 𝑦 ≥ 1,000, 10𝑥 + 50𝑦 ≥ 200,000, 𝑥 ≥ 0, 𝑦 ≥ 0. 𝑥 𝑦 𝑠 𝑡 𝑢 𝑝 *𝑠 *𝑡 *𝑢

1 0 10

0 1 50

−1 0 0

0 −1 0

0 0

−1

0 0 0

10000 1000 200000

𝑝

0.5

2

0

0

0

1

0

𝑥

𝑦

𝑠

𝑡

𝑢

𝑝

1

0

−1

0

0

0

10000

0

1

0

−1

0

0

1000

*𝑢

10

50

0

0

−1

0

200000

𝑅3 − 10𝑅1

𝑝

1

4

0

0

0

2

0

𝑅4 − 𝑅1

𝑥

𝑦

𝑠

𝑡

𝑢

𝑝

𝑥

1

0

−1

0

0

0

10000

*𝑡 *𝑢

0

1

0

−1

0

0

1000

0

50

10

0

−1

0

100000

𝑅3 − 50𝑅2

𝑝

0

4

1

0

0

2

−10000

𝑅4 − 4𝑅2

𝑥

𝑦

𝑠

𝑡

𝑢

𝑝

𝑥

1

0

−1

0

0

0

10000

𝑦

0

1

0

−1

0

0

1000

*𝑢

0

0

10

50

−1

0

50000

𝑝

0

0

1

4

0

2

−14000

𝑥

𝑦

𝑠

𝑡

𝑢

𝑝

𝑥 𝑦 𝑡

1 0 0

0 50 0

−1 10 10

0 0 50

0

−1 −1

0 0 0

10000 100000 50000

𝑝

0

0

5

0

2

50

−450000

*𝑠 *𝑡

2𝑅4

50𝑅2 + 𝑅3 25𝑅4 − 2𝑅3


Solutions Section 6.4 Optimal solution: 𝑝 = −450,000∕50 = −9,000; 𝑥 = 10,000∕1 = 10,000, 𝑦 = 100,000∕50 = 2,000. Since 𝑐 = −𝑝, the minimum value of c is 9,000. Succulent Citrus should produce 10,000 quarts of orange juice and 2,000 quarts of orange concentrate. 30. Unknowns: 𝑥 = # pints of pineapple juice, 𝑦 = # cans of pineapple rings Minimize 𝑐 = 0.2𝑥 + 0.5𝑦 subject to 2𝑥 + 𝑦 ≥ 20,000, 𝑥 ≥ 10,000, 𝑦 ≥ 1,000, 𝑥 ≥ 0, 𝑦 ≥ 0. 𝑥 𝑦 𝑠 𝑡 𝑢 𝑝 *𝑠 *𝑡 *𝑢

2 1 0

1 0 1

−1 0 0

0 −1 0

0 0 −1

0 0 0

20000 10000 1000

𝑝

0.2

0.5

0

0

0

1

0

𝑥

𝑦

𝑠

𝑡

𝑢

𝑝

*𝑠 *𝑡

2

1

−1

0

0

0

20000

1

0

0

−1

0

0

10000

*𝑢

0

1

0

0

−1

0

1000

𝑝

2

5

0

0

0

10

0

𝑅4 − 𝑅1

𝑥

𝑦

𝑠

𝑡

𝑢

𝑝

𝑥

2

1

−1

0

0

0

20000

𝑅1 + 𝑅2

*𝑡 *𝑢

0

−1

1

−2

0

0

0

0

1

0

0

−1

0

1000

𝑝

0

4

1

0

0

10

−20000

𝑥

𝑦

𝑠

𝑡

𝑢

𝑝

𝑥

2

0

0

−2

0

0

20000

𝑠

0

−1

1

−2

0

0

0

*𝑢

0

1

0

0

−1

0

1000

𝑝

0

5

0

2

0

10

−20000

𝑥

𝑦

𝑠

𝑡

𝑢

𝑝

𝑥 𝑠 𝑦

2 0 0

0 0 1

0 1 0

−2 −2 0

0 −1 −1

0 0 0

20000 1000 1000

𝑝

0

0

0

2

5

10

−25000

10𝑅4

2𝑅2 − 𝑅1

𝑅4 − 𝑅2

𝑅2 + 𝑅3 𝑅4 − 5𝑅3

Optimal solution: 𝑝 = −25,000∕10 = −2,500; 𝑥 = 20,000∕2 = 10,000, 𝑦 = 1,000∕1 = 1,000. Since 𝑐 = −𝑝, the minimum value of c is 2,500. Fancy Pineapple should produce 10,000 pints of pineapple juice and 1,000 cans of pineapple rings.


Solutions Section 6.4 31. Unknowns: 𝑥 = # regional music albums, 𝑦 = # pop/rock music albums, 𝑧 = # tropical music albums Maximize 𝑝 = 5𝑥 + 4𝑦 + 6𝑧 subject to 𝑥 − 4𝑧 ≥ 0, 𝑥 + 𝑦 + 𝑧 ≤ 40,000, 𝑦 ≥ 10,000, 𝑥 ≥ 0, 𝑦 ≥ 0, 𝑧 ≥ 0. 𝑥 𝑦 𝑧 𝑠 𝑡 𝑢 𝑝 *𝑠 𝑡

1

0

−4

−1

0

0

0

0

1

1

1

0

1

0

0

40000

*𝑢

0

1

0

0

0

−1

0

10000

𝑝

−5

−4

−6

0

0

0

1

0

𝑥

𝑦

𝑧

𝑠

𝑡

𝑢

𝑝

𝑥

1

0

−4

−1

0

0

0

0

𝑡

0

1

5

1

1

0

0

40000

*𝑢

0

1

0

0

0

−1

0

10000

𝑝

0

−4

−26

−5

0

0

1

0

𝑅4 + 4𝑅3

𝑥

𝑦

𝑧

𝑠

𝑡

𝑢

𝑝

𝑥

1

0

−4

−1

0

0

0

0

5𝑅1 + 4𝑅2

𝑡

0

0

5

1

1

1

0

30000

𝑦

0

1

0

0

0

−1

0

10000

𝑝

0

0

−26

−5

0

−4

1

40000

𝑥

𝑦

𝑧

𝑠

𝑡

𝑢

𝑝

𝑥 𝑧 𝑦

5 0 0

0 0 1

0 5 0

−1 1 0

4 1 0

4 1 −1

0 0 0

120000 30000 10000

𝑝

0

0

0

1

26

6

5

980000

𝑅2 − 𝑅1 𝑅4 + 5𝑅1

𝑅2 − 𝑅3

5𝑅4 + 26𝑅2

Optimal solution: 𝑝 = 980,000∕5 = 196,000; 𝑥 = 120,000∕5 = 24,000, 𝑦 = 10,000∕1 = 10,000, 𝑧 = 30,000∕5 = 6,000. You should sell 24,000 regional music albums, 10,000 pop/rock music albums, and 6,000 tropical music albums per day for a maximum revenue of $196,000. 32. Unknowns: 𝑥 = # regional music albums, 𝑦 = # pop/rock music albums, 𝑧 = # reggaeton albums Maximize 𝑝 = 5𝑥 + 4𝑦 + 3𝑧 subject to 𝑥 − 2𝑧 ≤ 0, 𝑥 + 𝑦 + 𝑧 ≤ 60,000, 𝑥 + 𝑦 ≥ 50,000, 𝑥 ≥ 0, 𝑦 ≥ 0, 𝑧 ≥ 0. 𝑥 𝑦 𝑧 𝑠 𝑡 𝑢 𝑝 𝑠

1

0

−2

1

0

0

0

0

𝑡

1

1

1

0

1

0

0

60000

𝑅2 − 𝑅1

*𝑢

1

1

0

0

0

−1

0

50000

𝑅3 − 𝑅1

𝑝

−5

−4

−3

0

0

0

1

0

𝑅4 + 5𝑅1


Solutions Section 6.4 𝑥

𝑦

𝑧

𝑠

𝑡

𝑢

𝑝

𝑥

1

0

−2

1

0

0

0

0

𝑡

0

1

3

−1

1

0

0

60000

*𝑢

0

1

2

−1

0

−1

0

50000

3𝑅3 − 2𝑅2

𝑝

0

−4

−13

5

0

0

1

0

3𝑅4 + 13𝑅2

𝑥

𝑦

𝑧

𝑠

𝑡

𝑢

𝑝

𝑥

3

2

0

1

2

0

0

120000

𝑅1 − 2𝑅3

𝑧

0

1

3

−1

1

0

0

60000

𝑅2 − 𝑅3

*𝑢

0

1

0

−1

−2

−3

0

30000

𝑝

0

1

0

2

13

0

3

780000

𝑥

𝑦

𝑧

𝑠

𝑡

𝑢

𝑝

𝑥 𝑧 𝑦

3 0 0

0 0 1

0 3 0

3 0 −1

6 3 −2

6 3 −3

0 0 0

60000 30000 30000

𝑝

0

0

0

3

15

3

3

750000

3𝑅1 + 2𝑅2

𝑅4 − 𝑅3

Optimal solution: 𝑝 = 750,000∕3 = 250,000; 𝑥 = 60,000∕3 = 20,000, 𝑦 = 30,000∕1 = 30,000, 𝑧 = 30,000∕3 = 10,000. Sell 20,000 regional music albums, 30,000 pop/rock music albums, and 10,000 reggaeton music albums per day for a maximum revenue of $250,000. 33. Unknowns: 𝑥 = # axes, 𝑦 = # maces, 𝑧 = # spears. Maximize 𝑝 = 6𝑥 + 6𝑦 + 8𝑧 subject to 8𝑥 + 5𝑦 + 2𝑧 ≤ 600, 𝑥 + 𝑦 + 2𝑧 ≥ 200, −𝑦 + 2𝑧 ≤ 0, 𝑥 ≥ 0, 𝑦 ≥ 0, 𝑧 ≥ 0. 𝑥

𝑦

𝑧

𝑠

𝑡

𝑢

𝑝

𝑠

8

5

2

1

0

0

0

600

𝑅1 − 𝑅3

*𝑡 𝑢

1

1

2

0

−1

0

0

200

𝑅2 − 𝑅3

0

−1

2

0

0

1

0

0

𝑝

−6

−6

−8

0

0

0

1

0

𝑅4 + 4𝑅3

𝑥

𝑦

𝑧

𝑠

𝑡

𝑢

𝑝

𝑠

8

6

0

1

0

−1

0

600

𝑅1 − 3𝑅2

*𝑡 𝑧

1

2

0

0

−1

−1

0

200

0

−1

2

0

0

1

0

0

2𝑅3 + 𝑅2

𝑝

−6

−10

0

0

0

4

1

0

𝑅4 + 5𝑅2


Solutions Section 6.4 𝑥

𝑦

𝑧

𝑠

𝑡

𝑢

𝑝

𝑠

5

0

0

1

3

2

0

0

𝑦

1

2

0

0

−1

−1

0

200

3𝑅2 + 𝑅1

𝑧

1

0

4

0

−1

1

0

200

3𝑅3 + 𝑅1

𝑝

−1

0

0

0

−5

−1

1

1000

3𝑅4 + 5𝑅1

𝑥

𝑦

𝑧

𝑠

𝑡

𝑢

𝑝

𝑡 𝑦 𝑧

5 8 8

0 6 0

0 0 12

1 1 1

3 0 0

2 −1 5

0 0 0

0 600 600

𝑝

22

0

0

5

0

7

3

3000

Optimal solution: 𝑝 = 3,000∕3 = 1,000; 𝑥 = 0, 𝑦 = 600∕6 = 100, 𝑧 = 600∕12 = 50. Achlúk can inflict a maximum of 1,000 units of damage using an arsenal of no axes, 100 maces, and 50 spears. 34. Unknowns: 𝑥 = # javelins, 𝑦 = # longswords, 𝑧 = # spears. Maximize 𝑝 = 12𝑥 + 32𝑦 + 24𝑧 subject to 𝑥 + 2𝑦 + 3𝑧 ≤ 3,000, 3𝑥 + 4𝑦 + 4𝑧 ≥ 1,000, 𝑦 − 𝑧 ≤ 0, 𝑥 ≥ 0, 𝑦 ≥ 0, 𝑧 ≥ 0. 𝑥

𝑦

𝑧

𝑠

𝑡

𝑢

𝑝

𝑠

1

2

3

1

0

0

0

3000

𝑅1 − 2𝑅3

*𝑡 𝑢

3

4

4

0

−1

0

0

1000

𝑅2 − 4𝑅3

0

1

−1

0

0

1

0

0

𝑝

−12

−32

−24

0

0

0

1

0

𝑅4 + 32𝑅3

𝑥

𝑦

𝑧

𝑠

𝑡

𝑢

𝑝

𝑠

1

0

5

1

0

−2

0

3000

8𝑅1 − 5𝑅2

*𝑡 𝑦

3

0

8

0

−1

−4

0

1000

0

1

−1

0

0

1

0

0

8𝑅3 + 𝑅2

𝑝

−12

0

−56

0

0

32

1

0

𝑅4 + 7𝑅2

𝑥

𝑦

𝑧

𝑠

𝑡

𝑢

𝑝

𝑠

−7

0

0

8

5

4

0

19000

𝑧

3

0

8

0

−1

−4

0

1000

5𝑅2 + 𝑅1

𝑦

3

8

0

0

−1

4

0

1000

5𝑅3 + 𝑅1

𝑝

9

0

0

0

−7

4

1

7000

5𝑅4 + 7𝑅1


Solutions Section 6.4 𝑥

𝑦

𝑧

𝑠

𝑡

𝑢

𝑝

𝑡

−7

0

0

8

5

4

0

19000

𝑧

8

0

40

8

0

−16

0

24000

𝑦

8

40

0

8

0

24

0

24000

𝑅3 − 𝑅2

𝑝

−4

0

0

56

0

48

5

168000

2𝑅4 + 𝑅2

𝑥

𝑦

𝑧

𝑠

𝑡

𝑢

𝑝

𝑡 𝑥 𝑦

0 8 0

0 0 40

280 40

−40

120 8 0

40 0 0

−80 −16 40

0 0 0

320000 24000 0

𝑝

0

0

40

120

0

80

10

360000

8𝑅1 + 7𝑅2

Optimal solution: 𝑝 = 360,000∕10 = 36,000; 𝑥 = 24,000∕8 = 3,000, 𝑦 = 0∕40 = 0, 𝑧 = 0. Galandir can inflict a maximum of 36,000 units of critical damage using an arsenal of 3,000 javelins, and no longswords or spears. 35. Unknowns: 𝑥 = # axes, 𝑦 = # maces, 𝑧 = # spears. Minimize 𝑐 = 8𝑥 + 5𝑦 + 2𝑧 subject to 3𝑥 + 3𝑦 + 4𝑧 ≥ 1,000, 𝑥 + 𝑦 + 2𝑧 ≥ 200, 𝑥 + 2𝑦 + 3𝑧 ≤ 500, 𝑥 ≥ 0, 𝑦 ≥ 0, 𝑧 ≥ 0. 𝑥

𝑦

𝑧

𝑠

𝑡

𝑢

𝑝

*𝑠 *𝑡

3

3

4

−1

0

0

0

1000

1

1

2

0

−1

0

0

200

𝑢

1

2

3

0

0

1

0

500

2𝑅3 − 3𝑅2

𝑝

8

5

2

0

0

0

1

0

𝑅4 − 𝑅2

𝑥

𝑦

𝑧

𝑠

𝑡

𝑢

𝑝

*𝑠 𝑧

1

1

0

−1

2

0

0

600

3𝑅1 − 2𝑅3

1

1

2

0

−1

0

0

200

3𝑅2 + 𝑅3

𝑢

−1

1

0

0

3

2

0

400

𝑝

7

4

0

0

1

0

1

−200

𝑥

𝑦

𝑧

𝑠

𝑡

𝑢

𝑝

*𝑠 𝑧

5

1

0

−3

0

−4

0

1000

2

4

6

0

0

2

0

1000

5𝑅2 − 2𝑅1

𝑡

−1

1

0

0

3

2

0

400

5𝑅3 + 𝑅1

𝑝

22

11

0

0

0

−2

3

−1000

5𝑅4 − 22𝑅1

𝑅1 − 2𝑅2

3𝑅4 − 𝑅3


Solutions Section 6.4 𝑥

𝑦

𝑧

𝑠

𝑡

𝑢

𝑝

𝑥 𝑧 𝑡

5 0 0

1 18 6

0 30 0

−3 6 −3

0 0 15

−4 18 6

0 0 0

1000 3000 3000

𝑝

0

33

0

66

0

78

15

−27000

Optimal solution: 𝑝 = −27,000∕15 = −1,800; 𝑥 = 1,000∕5 = 200, 𝑦 = 0, 𝑧 = 3,000∕30 = 100. Since 𝑐 = −𝑝, the minimum value of c is 1,800. Use 200 axes, no maces, and 100 spears for a minimum cost of 1,800 gold pieces. 36. Unknowns: 𝑥 = # javelins, 𝑦 = # longswords, 𝑧 = # spears. Minimize 𝑐 = 2𝑥 + 4𝑦 + 6𝑧 subject to 3𝑥 + 8𝑦 + 6𝑧 ≥ 6,000, 3𝑥 + 4𝑦 + 4𝑧 ≥ 1,000, 𝑥 + 15𝑦 + 2𝑧 ≤ 15,000, 𝑥 ≥ 0, 𝑦 ≥ 0, 𝑧 ≥ 0. 𝑥

𝑦

𝑧

𝑠

𝑡

𝑢

𝑝

*𝑠 *𝑡

3

8

6

−1

0

0

0

6000

3

4

4

0

−1

0

0

1000

𝑢

1

15

2

0

0

1

0

15000

𝑝

2

4

6

0

0

0

1

0

𝑥

𝑦

𝑧

𝑠

𝑡

𝑢

𝑝

*𝑠 𝑦

−3

0

−2

−1

2

0

0

4000

3

4

4

0

−1

0

0

1000

2𝑅2 + 𝑅1

𝑢

−41

0

−52

0

15

4

0

45000

2𝑅3 − 15𝑅1

𝑝

−1

0

2

0

1

0

1

−1000

2𝑅4 − 𝑅1

𝑥

𝑦

𝑧

𝑠

𝑡

𝑢

𝑝

𝑡 𝑦 𝑢

−3 3 −37

0 8 0

−2 6 −74

−1 −1 15

2 0 0

0 0 8

0 0 0

4000 6000 30000

𝑝

1

0

6

1

0

0

2

−6000

𝑅1 − 2𝑅2 4𝑅3 − 15𝑅2 𝑅4 − 𝑅2

Optimal solution: 𝑝 = −6,000∕2 = −3,000; 𝑥 = 0, 𝑦 = 6,000∕8 = 750, 𝑧 = 0. Since 𝑐 = −𝑝, the minimum value of c is 3,000. Use 750 longswords and nothing else for a total weight of 3,000 lbs. 37. Unknowns: 𝑥 = # bundles from Nadir, 𝑦 = # bundles from Blunt, 𝑧 = # bundles from Sonny Minimize 𝑐 = 3𝑥 + 4𝑦 + 5𝑧 (in thousands of dollars) subject to 5𝑥 + 10𝑦 + 15𝑧 ≥ 150, 10𝑥 + 10𝑦 + 10𝑧 ≥ 200, 15𝑥 + 10𝑦 + 10𝑧 ≥ 150, 𝑥 ≥ 0, 𝑦 ≥ 0, 𝑧 ≥ 0.


Solutions Section 6.4 𝑥

𝑦

𝑧

𝑠

𝑡

𝑢

𝑝

*𝑠 *𝑡

5

10

15

−1

0

0

0

150

10

10

10

0

−1

0

0

200

3𝑅2 − 2𝑅1

*𝑢

15

10

10

0

0

−1

0

150

3𝑅3 − 2𝑅1

𝑝

3

4

5

0

0

0

1

0

3𝑅4 − 𝑅1

𝑥

𝑦

𝑧

𝑠

𝑡

𝑢

𝑝

𝑧

5

10

15

−1

0

0

0

150

7𝑅1 − 𝑅3

*𝑡 *𝑢

20

10

0

2

−3

0

0

300

7𝑅2 − 4𝑅3

35

10

0

2

0

−3

0

150

𝑝

4

2

0

1

0

0

3

−150

𝑥

𝑦

𝑧

𝑠

𝑡

𝑢

𝑝

𝑧

0

60

105

−9

0

3

0

900

*𝑡 𝑥

0

30

0

6

−21

12

0

1500

2𝑅2 − 𝑅1

35

10

0

2

0

−3

0

150

6𝑅3 − 𝑅1

𝑝

0

30

0

27

0

12

105

−5850

2𝑅4 − 𝑅1

𝑥

𝑦

𝑧

𝑠

𝑡

𝑢

𝑝

𝑦

0

60

105

−9

0

3

0

900

7𝑅1 + 3𝑅3

*𝑡 𝑥

0

0

−105

21

−42

21

0

2100

𝑅2 − 𝑅3

210

0

−105

21

0

−21

0

0

𝑝

0

0

−105

63

0

21

210

−12600

𝑅4 − 3𝑅3

𝑥

𝑦

𝑧

𝑠

𝑡

𝑢

𝑝

𝑦

630

420

420

0

0

−42

0

6300

𝑅1 + 𝑅2

*𝑡 𝑠

−210

0

0

0

−42

42

0

2100

210

0

−105

21

0

−21

0

0

2𝑅3 + 𝑅2

𝑝

−630

0

210

0

0

84

210

−12600

𝑅4 − 2𝑅2

𝑥

𝑦

𝑧

𝑠

𝑡

𝑢

𝑝

𝑦

420

420

420

0

−42

0

0

8400

𝑅1 − 2𝑅3

𝑢

−210

0

0

0

−42

42

0

2100

𝑅2 + 𝑅3

𝑠

210

0

−210

42

−42

0

0

2100

𝑝

−210

0

210

0

84

0

210

−16800

35𝑅4 − 4𝑅3

𝑅4 + 𝑅3


Solutions Section 6.4 𝑥

𝑦

𝑧

𝑠

𝑡

𝑢

𝑝

𝑦 𝑢 𝑥

0 0 210

420 0 0

840

−210 −210

−84 42 42

42

−84 −42

0 42 0

0 0 0

4200 4200 2100

𝑝

0

0

0

42

42

0

210

−14700

Optimal solution: 𝑝 = −14,700∕210 = −70; 𝑥 = 2,100∕210 = 10, 𝑦 = 4,200∕420 = 10, 𝑧 = 0. Since 𝑐 = −𝑝, the minimum value of c is 70. Buy 10 bundles from Nadir, 10 from Blunt, and none from Sonny. Cost: $70,000. Another solution resulting in the same cost (pivot on the 𝑧-column in the last tableau) is 15 bundles from Nadir, none from Blunt, and 5 from Sonny. 38. Unknowns: 𝑥 = # packages from Fred Motor Co., 𝑦 = # packages from Admiral Motors, 𝑧 = # packages from Chrysalis Minimize 𝑐 = 5𝑥 + 4𝑦 + 3𝑧 (in $100,000) subject to 𝑥 + 𝑦 + 2𝑧 ≥ 110, 𝑥 + 2𝑦 + 𝑧 ≥ 100, 2𝑥 + 𝑦 + 𝑧 ≥ 110, 𝑥 ≥ 0, 𝑦 ≥ 0, 𝑧 ≥ 0. 𝑥

𝑦

𝑧

𝑠

𝑡

𝑢

𝑝

*𝑠 *𝑡

1

1

2

−1

0

0

0

110

1

2

1

0

−1

0

0

100

2𝑅2 − 𝑅1

*𝑢

2

1

1

0

0

−1

0

110

2𝑅3 − 𝑅1

𝑝

5

4

3

0

0

0

1

0

2𝑅4 − 3𝑅1

𝑥

𝑦

𝑧

𝑠

𝑡

𝑢

𝑝

𝑧

1

1

2

−1

0

0

0

110

3𝑅1 − 𝑅2

*𝑡 *𝑢

1

3

0

1

−2

0

0

90

3

1

0

1

0

−2

0

110

3𝑅3 − 𝑅2

𝑝

7

5

0

3

0

0

2

−330

3𝑅4 − 5𝑅2

𝑥

𝑦

𝑧

𝑠

𝑡

𝑢

𝑝

𝑧

2

0

6

−4

2

0

0

240

4𝑅1 − 𝑅3

𝑦

1

3

0

1

−2

0

0

90

8𝑅2 − 𝑅3

*𝑢

8

0

0

2

2

−6

0

240

𝑝

16

0

0

4

10

0

6

−1440

𝑥

𝑦

𝑧

𝑠

𝑡

𝑢

𝑝

𝑧 𝑦 𝑥

0 0 8

0 24 0

24 0 0

−18 6 2

6 −18 2

6 6 −6

0 0 0

720 480 240

𝑝

0

0

0

0

6

12

6

−1920

𝑅4 − 2𝑅3

Optimal solution: 𝑝 = −1,920∕6 = −320; 𝑥 = 240∕8 = 30, 𝑦 = 480∕24 = 20, 𝑧 = 720∕24 = 30.


Solutions Section 6.4 Since 𝑐 = −𝑝, the minimum value of c is 320. Buy 30 packages from Fred, 20 from Admiral, and 30 from Chrysalis. Total cost: $32,000,000 39. Unknowns: 𝑥 = # servings of Xtend, 𝑦 = # servings of Gainz, 𝑧 = # servings of Strongevity Minimize 𝑐 = 0.8𝑥 + 1.2𝑦 + 𝑧 subject to 3𝑦 + 3𝑧 ≥ 60, 2.5𝑥 + 3𝑦 + 𝑧 ≥ 45, 7𝑥 + 5𝑦 ≥ 170, 𝑥 ≥ 0, 𝑦 ≥ 0, 𝑧 ≥ 0. You can use the online Pivot and Gauss-Jordan Tool in decimal mode do the pivoting. 𝑥

𝑦

𝑧

𝑠

𝑡

𝑢

𝑝

*𝑠 *𝑡

0

3

3

−1

0

0

0

60

2.5

3

1

0

−1

0

0

45

*𝑢

7

5

0

0

0

−1

0

170

𝑝

0.8

1.2

1

0

0

0

1

0

𝑥

𝑦

𝑧

𝑠

𝑡

𝑢

𝑝

*𝑠 −2.5 𝑦 0.8333 *𝑢 2.8333

0

2

−1

1

0

0

15

1 0

0.3333 −1.6667

0 0

−0.3333 1.6667

0 −1

0 0

15 95

𝑝

−0.2

0

0.6

0

0.4

0

1

−18

𝑥

𝑦

𝑧

𝑠

𝑡

𝑢

𝑝

𝑧 −1.25

0

1

−0.5

0.5

0

0

7.5

𝑦 *𝑢

1.25 0.75

1 0

0 0

0.1667 −0.5 −0.8333 2.5

0 −1

0 0

12.5 107.5

𝑝

0.55

0

0

0.3

0.1

0

1

−22.5

𝑥

𝑦

𝑧

𝑠

𝑡

𝑢

𝑝

𝑡 𝑦

−2.5 0

0 1

2 1

−1 −0.3333

1 0

0 0

0 0

15 20

*𝑢

7

0

−5

1.6667

0

−1

0

70

𝑝

0.8

0

−0.2

0.4

0

0

1

−24

𝑥

𝑦

𝑧

𝑠

𝑡

𝑢

𝑝

𝑡 𝑦 𝑥

0 0 1

0 1 0

0.2143 −0.4048 1 −0.3333 −0.7143 0.2381

1 0 0

−0.3571 0 −0.1429

0 0 0

40 20 10

𝑝

0

0

0.3714 0.2095

0

0.1143

1

−32

Optimal solution: 𝑝 = −32; 𝑥 = 10, 𝑦 = 20, 𝑧 = 0. Since 𝑐 = −𝑝, the minimum value of c is 32. Use 10 servings of Xtend, 20 servings of RecoverMode, and no Strongevity for a total cost of $32. 40. Unknowns: 𝑥 = # servings of RecoverMode, 𝑦 = # servings of Strongevity, 𝑧 = # servings of Muscle


Solutions Section 6.4 Physique Minimize 𝑐 = 1.2𝑥 + 𝑦 + .5𝑧 subject to 3𝑥 + 3𝑦 + 2𝑧 ≥ 70, 3𝑥 + 𝑦 + 2𝑧 ≥ 70, 5𝑥 ≥ 50, 𝑥 ≥ 0, 𝑦 ≥ 0, 𝑧 ≥ 0. You can use the online Pivot and Gauss-Jordan Tool in decimal mode do the pivoting. 𝑥

𝑦

𝑧

𝑠

𝑡

𝑢

𝑝

*𝑠 *𝑡 *𝑢

3 3

3 1

2 2

−1 0

0

−1

0 0

0 0

70 70

5

0

0

0

0

−1

0

50

𝑝

1.2

1

0.5

0

0

0

1

0

𝑥

𝑦

𝑧

𝑠

𝑡

𝑢

𝑝

*𝑠 *𝑡 𝑥

0

3

2

−1

0

0.6

0

40

0 1

1 0

2 0

0 0

−1 0

0.6 −0.2

0 0

40 10

𝑝

0

1

0.5

0

0

0.24

1

−12

𝑥

𝑦

𝑧

𝑦

0

1

*𝑡 𝑥

0

0

1

0

𝑝

0

0

𝑥

𝑦

𝑧

𝑠

𝑡

𝑢

𝑝

𝑦

0

1

0

−0.5

0.5

0

0

0

𝑧 𝑥

0 1

0 0

1 0

0.25 0

−0.75 0.3 0 −0.2

0 0

20 10

𝑝

0

0

0

0.375 −0.125 0.09

1

−22

𝑥

𝑦

𝑧

𝑠

𝑡

𝑢

𝑝

𝑡 𝑧 𝑥

0 0 1

2 1.5 0

0 1 0

−1 −0.5 0

1 0 0

0 0.3 −0.2

0 0 0

0 20 10

𝑝

0

0.25

0

0.25

0

0.09

1

−22

𝑠

𝑡

𝑢

𝑝

0.6667 −0.3333

0

0.2

0

13.3333

1.3333 0.3333

−1

0.4

0

26.6667

0

−0.2

0

10

0

0.04

1

−25.3333

0

0

−0.1667 0.3333

Optimal solution: 𝑝 = −22; 𝑥 = 10, 𝑦 = 0, 𝑧 = 20. Since 𝑐 = −𝑝, the minimum value of c is 22. Use 10 servings of RecoverMode, no Strongevity, and 20 servings of Muscle Physique for a total cost of $22. 41. Unknowns: 𝑥 = # convention-style hotels, 𝑦 = # vacation-style hotels, and 𝑧 = # small motels a. Minimize 𝑐 = 100𝑥 + 20𝑦 + 4𝑧 subject to 500𝑥 + 200𝑦 + 50𝑧 ≥ 1,400, 𝑥 ≥ 1, 𝑧 ≤ 2, 𝑥 ≥ 0, 𝑦 ≥ 0,


Solutions Section 6.4 𝑧 ≥ 0.

𝑥

𝑦

𝑧

𝑠

𝑡

𝑢

𝑝

*𝑠 *𝑡

500

200

50

−1

0

0

0

1400

1

0

0

0

−1

0

0

1

𝑢

0

0

1

0

0

1

0

2

𝑝

100

20

4

0

0

0

1

0

𝑥

𝑦

𝑧

𝑠

𝑡

𝑢

𝑝

*𝑠 𝑥

0

200

50

−1

500

0

0

900

1

0

0

0

−1

0

0

1

𝑢

0

0

1

0

0

1

0

2

𝑝

0

20

4

0

100

0

1

−100

𝑥

𝑦

𝑧

𝑠

𝑡

𝑢

𝑝

𝑡

0

200

50

−1

500

0

0

900

𝑥

500

200

50

−1

0

0

0

1400

𝑢

0

0

1

0

0

1

0

2

𝑝

0

−100

−30

1

0

0

5

−1400

2𝑅4 + 𝑅1

𝑥

𝑦

𝑧

𝑠

𝑡

𝑢

𝑝

𝑦

0

200

50

−1

500

0

0

900

𝑅1 − 50𝑅3

𝑥

500

0

0

0

−500

0

0

500

𝑢

0

0

1

0

0

1

0

2

𝑝

0

0

−10

1

500

0

10

−1900

𝑥

𝑦

𝑧

𝑠

𝑡

𝑢

𝑝

𝑦 𝑥 𝑧

0 500 0

200 0 0

0 0 1

−1 0 0

500 −500 0

−50 0 1

0 0 0

800 500 2

𝑝

0

0

0

1

500

10

10

−1880

𝑅1 − 500𝑅2

𝑅4 − 100𝑅2

500𝑅2 + 𝑅1 5𝑅4 − 𝑅1

𝑅2 − 𝑅1

𝑅4 + 10𝑅3

Optimal solution: 𝑝 = −1,880∕10 = −188; 𝑥 = 500∕500 = 1, 𝑦 = 800∕200 = 4, 𝑧 = 2∕1 = 2. Since 𝑐 = −𝑝, the minimum value of c is 188. Build 1 convention-style hotel, 4 vacation-style hotels, and 2 small motels. The total cost will amount to $188 million. b. Since 20% of the $188 million cost is $37.6 million, you will still be covered by the $50 million subsidy. 42. Unknowns: 𝑥 = # convention-style hotels, 𝑦 = # vacation-style hotels, and 𝑧 = # small motels a. Minimize 𝑐 = 100𝑥 + 20𝑦 + 4𝑧 subject to 500𝑥 + 200𝑦 + 50𝑧 ≥ 1,400, 𝑦 ≥ 2, 𝑧 ≤ 4, 𝑥 ≥ 0, 𝑦 ≥ 0, 𝑧 ≥ 0.


Solutions Section 6.4 𝑥

𝑦

𝑧

𝑠

𝑡

𝑢

𝑝

*𝑠 *𝑡 𝑢

500

200

50

−1

0

0

0

1400

0 0

1 0

0 1

0 0

−1 0

0 1

0 0

2 4

𝑝

100

20

4

0

0

0

1

0

5𝑅4 − 𝑅1

𝑥

𝑦

𝑧

𝑠

𝑡

𝑢

𝑝

𝑥

500

200

50

−1

0

0

0

1400

𝑅1 − 200𝑅2

*𝑡 𝑢

0

1

0

0

−1

0

0

2

0

0

1

0

0

1

0

4

𝑝

0

−100

−30

1

0

0

5

−1400

𝑥

𝑦

𝑧

𝑠

𝑡

𝑢

𝑝

𝑥

500

0

50

−1

200

0

0

1000

𝑦

0

1

0

0

−1

0

0

2

𝑢

0

0

1

0

0

1

0

4

𝑝

0

0

−30

1

−100

0

5

−1200

2𝑅4 + 𝑅1

𝑥

𝑦

𝑧

𝑠

𝑡

𝑢

𝑝

𝑡

500

0

50

−1

200

0

0

1000

𝑅1 − 50𝑅3

𝑦

500

200

50

−1

0

0

0

1400

𝑅2 − 50𝑅3

𝑢

0

0

1

0

0

1

0

4

𝑝

500

0

−10

1

0

0

10

−1400

𝑥

𝑦

𝑧

𝑠

𝑡

𝑢

𝑝

𝑡 𝑦 𝑧

500 500 0

0 200 0

0 0 1

−1 −1 0

200 0 0

−50 −50 1

0 0 0

800 1200 4

𝑝

500

0

0

1

0

10

10

−1360

𝑅4 + 100𝑅2

200𝑅2 + 𝑅1

𝑅4 + 10𝑅3

Optimal solution: 𝑝 = −1,360∕10 = −136; 𝑥 = 0, 𝑦 = 1,200∕200 = 6, 𝑧 = 4∕1 = 4. Since 𝑐 = −𝑝, the minimum value of c is 136. Build no convention-style hotels, 6 vacation-style hotels, and 4 motels. b. The total cost will amount to $136 million and will be covered by the subsidy, since 20% is $28.2 million.


Solutions Section 6.4 43. Unknowns: 𝑥 = # boards sent from Tucson to Honolulu 𝑦 = # boards sent from Tucson to Venice Beach 𝑧 = # boards sent from Toronto to Honolulu 𝑤 = # boards sent from Toronto to Venice Beach

Minimize 𝑐 = 10𝑥 + 5𝑦 + 20𝑧 + 10𝑤 subject to 𝑥 + 𝑦 ≤ 620, 𝑧 + 𝑤 ≤ 410, 𝑥 + 𝑧 ≥ 500, 𝑦 + 𝑤 ≥ 530, 𝑥 ≥ 0, 𝑦 ≥ 0, 𝑧 ≥ 0, 𝑤 ≥ 0. 𝑥 𝑦 𝑧 𝑤 𝑠 𝑡 𝑢 𝑣 𝑝 𝑅1 − 𝑅3

𝑠

1

1

0

0

1

0

0

0

0

620

𝑡

0

0

1

1

0

1

0

0

0

410

*𝑢 *𝑣

1

0

1

0

0

0

−1

0

0

500

0

1

0

1

0

0

0

−1

0

530

𝑝

10

5

20

10

0

0

0

0

1

0

𝑥

𝑦

𝑧

𝑤

𝑠

𝑡

𝑢

𝑣

𝑝

𝑠

0

1

−1

0

1

0

1

0

0

120

𝑡 𝑥

0 1

0 0

1 1

1 0

0 0

1 0

0 −1

0 0

0 0

410 500

*𝑣

0

1

0

1

0

0

0

−1

0

530

𝑅4 − 𝑅1

𝑝

0

5

10

10

0

0

10

0

1

−5000

𝑅5 − 5𝑅1

𝑥

𝑦

𝑧

𝑤

𝑠

𝑡

𝑢

𝑣

𝑝

𝑦

0

1

−1

0

1

0

1

0

0

120

𝑅1 + 𝑅4

𝑡

0

0

1

1

0

1

0

0

0

410

𝑅2 − 𝑅4

𝑥

1

0

1

0

0

0

−1

0

0

500

𝑅3 − 𝑅4

*𝑣

0

0

1

1

−1

0

−1

−1

0

410

𝑝

0

0

15

10

−5

0

5

0

1

−5600

𝑅5 − 15𝑅4

𝑥

𝑦

𝑧

𝑤

𝑠

𝑡

𝑢

𝑣

𝑝

𝑦

0

1

0

1

0

0

0

−1

0

530

𝑅1 − 𝑅4

𝑡

0

0

0

0

1

1

1

1

0

0

𝑥

1

0

0

−1

1

0

0

1

0

90

𝑧

0

0

1

1

−1

0

−1

−1

0

410

𝑝

0

0

0

−5

10

0

20

15

1

−11750

𝑅5 − 10𝑅3

𝑅3 + 𝑅4 𝑅5 + 5𝑅4


Solutions Section 6.4 𝑥

𝑦

𝑧

𝑤

𝑠

𝑡

𝑢

𝑣

𝑝

𝑦 𝑡 𝑥 𝑤

0 0 1 0

1 0 0 0

−1 0 1 1

0 0 0 1

1 1 0 −1

0 1 0 0

1 1 −1 −1

0 1 0 −1

0 0 0 0

120 0 500 410

𝑝

0

0

5

0

5

0

15

10

1

−9700

Optimal solution: 𝑝 = −9,700∕1 = −9,700; 𝑥 = 500∕1 = 500, 𝑦 = 120∕1 = 120, 𝑧 = 0, 𝑤 = 410∕1 = 410. Since 𝑐 = −𝑝, the minimum value of c is 9,700. Make the following shipments: Tucson to Honolulu: 500 boards per week; Tucson to Venice Beach: 120 boards per week; Toronto to Honolulu: 0 boards per week; Toronto to Venice Beach: 410 boards per week. Minimum weekly cost is $9,700. 44. Unknowns: 𝑥 = # boards sent from Tucson to Honolulu 𝑦 = # boards sent from Tucson to Venice Beach 𝑧 = # boards sent from Toronto to Honolulu 𝑤 = # boards sent from Toronto to Venice Beach

Minimize 𝑐 = 10𝑥 + 5𝑦 + 20𝑧 + 10𝑤 subject to 𝑥 + 𝑦 ≤ 1,000, 𝑧 + 𝑤 ≤ 0, 𝑥 + 𝑧 ≥ 500, 𝑦 + 𝑤 ≥ 530, 𝑥 ≥ 0, 𝑦 ≥ 0, 𝑧 ≥ 0, 𝑤 ≥ 0. 𝑥 𝑦 𝑧 𝑤 𝑠 𝑡 𝑢 𝑣 𝑝 𝑅1 − 𝑅3

𝑠

1

1

0

0

1

0

0

0

0

1000

𝑡

0

0

1

1

0

1

0

0

0

0

*𝑢 *𝑣

1

0

1

0

0

0

−1

0

0

500

0

1

0

1

0

0

0

−1

0

530

𝑝

10

5

20

10

0

0

0

0

1

0

𝑥

𝑦

𝑧

𝑤

𝑠

𝑡

𝑢

𝑣

𝑝

𝑠

0

1

−1

0

1

0

1

0

0

500

𝑡 𝑥

0 1

0 0

1 1

1 0

0 0

1 0

0 −1

0 0

0 0

0 500

*𝑣

0

1

0

1

0

0

0

−1

0

530

𝑅4 − 𝑅1

𝑝

0

5

10

10

0

0

10

0

1

−5000

𝑅5 − 5𝑅1

𝑅5 − 10𝑅3


Solutions Section 6.4 𝑥

𝑦

𝑧

𝑤

𝑠

𝑡

𝑢

𝑣

𝑝

𝑦

0

1

−1

0

1

0

1

0

0

500

𝑡

0

0

1

1

0

1

0

0

0

0

𝑥

1

0

1

0

0

0

−1

0

0

500

𝑅3 − 𝑅2

*𝑣

0

0

1

1

−1

0

−1

−1

0

30

𝑅4 − 𝑅2

𝑝

0

0

15

10

−5

0

5

0

1

−7500

𝑅5 − 15𝑅2

𝑥

𝑦

𝑧

𝑤

𝑠

𝑡

𝑢

𝑣

𝑝

𝑦 𝑧 𝑥 *𝑣

0 0 1 0

1 0 0 0

0 1 0 0

1 1 −1 0

1 0 0 −1

1 1 −1 −1

1 0 −1 −1

0 0 0 −1

0 0 0 0

500 0 500 30

𝑝

0

0

0

−5

−5

−15

5

0

1

−7500

𝑅1 + 𝑅2

There are no positive numbers in the first starred row, so there is no solution; there are not enough boogie boards being manufactured to fill the orders. 45. Unknowns: 𝑥 = amount overdrawn from Congressional Integrity Bank, 𝑦 = amount from Citizens' Trust, 𝑧 = amount from Checks R Us Maximize 𝑝 = 𝑥 subject to 𝑥 + 𝑦 + 𝑧 ≤ 10,000, 𝑥 + 𝑦 ≤ 1 ⁄ 4 (𝑥 + 𝑦 + 𝑧) or 3𝑥 + 3𝑦 − 𝑧 ≤ 0, 𝑥 ≥ 2,500, 𝑥 ≥ 0, 𝑦 ≥ 0, 𝑧 ≥ 0. 𝑥 𝑦 𝑧 𝑠 𝑡 𝑢 𝑝 3𝑅1 − 𝑅2

𝑠

1

1

1

1

0

0

0

10000

𝑡

3

3

−1

0

1

0

0

0

*𝑢

1

0

0

0

0

−1

0

2500

3𝑅3 − 𝑅2

𝑝

−1

0

0

0

0

0

1

0

3𝑅4 + 𝑅2

𝑥

𝑦

𝑧

𝑠

𝑡

𝑢

𝑝

𝑠

0

0

4

3

−1

0

0

30000

𝑅1 − 4𝑅3

𝑥

3

3

−1

0

1

0

0

0

𝑅2 + 𝑅3

*𝑢

0

−3

1

0

−1

−3

0

7500

𝑝

0

3

−1

0

1

0

3

0

𝑥

𝑦

𝑧

𝑠

𝑡

𝑢

𝑝

𝑠

0

12

0

3

3

12

0

0

𝑥

3

0

0

0

0

−3

0

7500

4𝑅2 + 𝑅1

𝑧

0

−3

1

0

−1

−3

0

7500

4𝑅3 + 𝑅1

𝑝

0

0

0

0

0

−3

3

7500

4𝑅4 + 𝑅1

𝑅4 + 𝑅3


Solutions Section 6.4 𝑥

𝑦

𝑧

𝑠

𝑡

𝑢

𝑝

𝑢 𝑥 𝑧

0 12 0

12 12 0

0 0 4

3 3 3

3 3 −1

12 0 0

0 0 0

0 30000 30000

𝑝

0

12

0

3

3

0

12

30000

Optimal solution: 𝑝 = 30,000∕12 = 2,500; 𝑥 = 30,000∕12 = 2,500, 𝑦 = 0, 𝑧 = 30,000∕4 = 7,500. Withdraw $2,500 from Congressional Integrity Bank, $0 from Citizens' Trust, $7,500 from Checks R Us. 46. Unknowns: 𝑥 = # hours spent with private citizens, y = # hours spent with corporate executives, z = # hours spent with university professors Maximize 𝑝 = 5𝑥 + 50𝑦 + 10𝑧 subject to 𝑥 − 𝑦 − 𝑧 ≥ 2, −𝑥 − 𝑦 + 𝑧 ≤ 0, 𝑧 ≥ 10, 𝑥 + 𝑦 + 𝑧 ≤ 40, 𝑥 ≥ 0, 𝑦 ≥ 0, 𝑧 ≥ 0. 𝑥 𝑦 𝑧 𝑠 𝑡 𝑢 𝑣 𝑝 *𝑠 𝑡

1

−1

−1

−1

0

0

0

0

2

−1

−1

1

0

1

0

0

0

0

*𝑢 𝑣

0

0

1

0

0

−1

0

0

10

1

1

1

0

0

0

1

0

40

𝑅4 − 𝑅1

𝑝

−5

−50

−10

0

0

0

0

1

0

𝑅5 + 5𝑅1

𝑥

𝑦

𝑧

𝑠

𝑡

𝑢

𝑣

𝑝

𝑥

1

−1

−1

−1

0

0

0

0

2

𝑅1 + 𝑅3

𝑡

0

−2

0

−1

1

0

0

0

2

*𝑢 𝑣

0

0

1

0

0

−1

0

0

10

0

2

2

1

0

0

1

0

38

𝑅4 − 2𝑅3

𝑝

0

−55

−15

−5

0

0

0

1

10

𝑅5 + 15𝑅3

𝑥

𝑦

𝑧

𝑠

𝑡

𝑢

𝑣

𝑝

𝑥

1

−1

0

−1

0

−1

0

0

12

2𝑅1 + 𝑅4

𝑡

0

−2

0

−1

1

0

0

0

2

𝑅2 + 𝑅4

𝑧

0

0

1

0

0

−1

0

0

10

𝑣

0

2

0

1

0

2

1

0

18

𝑝

0

−55

0

−5

0

−15

0

1

160

𝑥

𝑦

𝑧

𝑠

𝑡

𝑢

𝑣

𝑝

𝑥 𝑡 𝑧 𝑦

2 0 0 0

0 0 0 2

0 0 1 0

−1 0 0 1

0 1 0 0

0 2 −1 2

1 1 0 1

0 0 0 0

42 20 10 18

𝑝

0

0

0

45

0

80

55

2

1310

𝑅2 + 𝑅1

2𝑅5 + 55𝑅4


Solutions Section 6.4 Optimal solution: 𝑝 = 1,310∕2 = 655; 𝑥 = 42∕2 = 21, 𝑦 = 18∕2 = 9, 𝑧 = 10∕1 = 10. Spend 21 hours per week with private citizens, 9 hours with corporate executives, and 10 hours per week with university professors, giving a weekly income of $655,000. 47. Unknowns: 𝑥 = # people you fly from Chicago to Los Angeles, 𝑦 = # people you fly from Chicago to New York, 𝑧 = # people you fly from Denver to Los Angeles, 𝑤 = # people you fly from Denver to New York Minimize 𝑐 = 200𝑥 + 150𝑦 + 400𝑧 + 200𝑤 subject to 𝑥 + 𝑦 ≤ 20, 𝑧 + 𝑤 ≤ 10, 𝑥 + 𝑧 ≥ 10, 𝑦 + 𝑤 ≥ 15, 𝑥 ≥ 0, 𝑦 ≥ 0, 𝑧 ≥ 0, 𝑤 ≥ 0. 𝑥 𝑦 𝑧 𝑤 𝑠 𝑡 𝑢 𝑣 𝑝 𝑅1 − 𝑅3

𝑠

1

1

0

0

1

0

0

0

0

20

𝑡

0

0

1

1

0

1

0

0

0

10

*𝑢 *𝑣

1

0

1

0

0

0

−1

0

0

10

0

1

0

1

0

0

0

−1

0

15

𝑝

200

150

400

200

0

0

0

0

1

0

𝑥

𝑦

𝑧

𝑤

𝑠

𝑡

𝑢

𝑣

𝑝

𝑠

0

1

−1

0

1

0

1

0

0

10

𝑡 𝑥

0 1

0 0

1 1

1 0

0 0

1 0

0 −1

0 0

0 0

10 10

*𝑣

0

1

0

1

0

0

0

−1

0

15

𝑝

0

150

200

200

0

0

200

0

1

−2000

𝑥

𝑦

𝑧

𝑤

𝑠

𝑡

𝑢

𝑣

𝑝

𝑦

0

1

−1

0

1

0

1

0

0

10

𝑅1 + 𝑅4

𝑡

0

0

1

1

0

1

0

0

0

10

𝑅2 − 𝑅4

𝑥

1

0

1

0

0

0

−1

0

0

10

𝑅3 − 𝑅4

*𝑣

0

0

1

1

−1

0

−1

−1

0

5

𝑝

0

0

350

200

−150

0

50

0

1

−3500

𝑅5 − 350𝑅4

𝑥

𝑦

𝑧

𝑤

𝑠

𝑡

𝑢

𝑣

𝑝

𝑦

0

1

0

1

0

0

0

−1

0

15

𝑅1 − 𝑅4

𝑡

0

0

0

0

1

1

1

1

0

5

𝑥

1

0

0

−1

1

0

0

1

0

5

𝑧

0

0

1

1

−1

0

−1

−1

0

5

𝑝

0

0

0

−150

200

0

400

350

1

−5250

𝑅5 − 200𝑅3

𝑅4 − 𝑅1 𝑅5 − 150𝑅1

𝑅3 + 𝑅4 𝑅5 + 150𝑅4


Solutions Section 6.4 𝑥

𝑦

𝑧

𝑤

𝑠

𝑡

𝑢

𝑣

𝑝

𝑦 𝑡 𝑥 𝑤

0 0 1 0

1 0 0 0

−1 0 1 1

0 0 0 1

1 1 0 −1

0 1 0 0

1 1 −1 −1

0 1 0 −1

0 0 0 0

10 5 10 5

𝑝

0

0

150

0

50

0

250

200

1

−4500

Optimal solution: 𝑝 = −4,500∕1 = −4,500; 𝑥 = 10∕1 = 10, 𝑦 = 10∕1 = 10, 𝑧 = 0, 𝑤 = 5∕1 = 5. Since 𝑐 = −𝑝, the minimum value of c is 4,500. Fly 10 people from Chicago to each of Chicago and New York, and 5 from Denver to New York for a total cost of $4,500. 48. Unknowns: 𝑥 = # people you fly from Chicago to Los Angeles, 𝑦 = # people you fly from Chicago to New York, 𝑧 = # people you fly from Denver to Los Angeles, 𝑤 = # people you fly from Denver to New York Minimize 𝑐 = 200𝑥 + 150𝑦 + 400𝑧 + 200𝑤 subject to 𝑥 + 𝑦 ≤ 20, 𝑧 + 𝑤 ≤ 10, 𝑥 + 𝑧 ≥ 15, 𝑦 + 𝑤 ≥ 15, 𝑥 ≥ 0, 𝑦 ≥ 0, 𝑧 ≥ 0, 𝑤 ≥ 0. 𝑥 𝑦 𝑧 𝑤 𝑠 𝑡 𝑢 𝑣 𝑝 𝑅1 − 𝑅3

𝑠

1

1

0

0

1

0

0

0

0

20

𝑡

0

0

1

1

0

1

0

0

0

10

*𝑢 *𝑣

1

0

1

0

0

0

−1

0

0

15

0

1

0

1

0

0

0

−1

0

15

𝑝

200

150

400

200

0

0

0

0

1

0

𝑥

𝑦

𝑧

𝑤

𝑠

𝑡

𝑢

𝑣

𝑝

𝑠

0

1

−1

0

1

0

1

0

0

5

𝑡 𝑥

0 1

0 0

1 1

1 0

0 0

1 0

0

−1

0 0

0 0

10 15

*𝑣

0

1

0

1

0

0

0

−1

0

15

𝑝

0

150

200

200

0

0

200

0

1

−3000

𝑥

𝑦

𝑧

𝑤

𝑠

𝑡

𝑢

𝑣

𝑝

𝑦

0

1

−1

0

1

0

1

0

0

5

𝑅1 + 𝑅4

𝑡

0

0

1

1

0

1

0

0

0

10

𝑅2 − 𝑅4

𝑥

1

0

1

0

0

0

−1

0

0

15

𝑅3 − 𝑅4

*𝑣

0

0

1

1

−1

0

−1

−1

0

10

𝑝

0

0

350

200

−150

0

50

0

1

−3750

𝑅5 − 200𝑅3

𝑅4 − 𝑅1 𝑅5 − 150𝑅1

𝑅5 − 350𝑅4


Solutions Section 6.4 𝑥

𝑦

𝑧

𝑤

𝑠

𝑡

𝑢

𝑣

𝑝

𝑦

0

1

0

1

0

0

0

−1

0

15

𝑡

0

0

0

0

1

1

1

1

0

0

𝑥

1

0

0

−1

1

0

0

1

0

5

𝑧

0

0

1

1

−1

0

−1

−1

0

10

𝑝

0

0

0

−150

200

0

400

350

1

−7250

𝑥

𝑦

𝑧

𝑤

𝑠

𝑡

𝑢

𝑣

𝑝

𝑦 𝑡 𝑥 𝑤

0 0 1 0

1 0 0 0

−1 0 1 1

0 0 0 1

1 1 0 −1

0 1 0 0

1 1 −1 −1

0 1 0 −1

0 0 0 0

5 0 15 10

𝑝

0

0

150

0

50

0

250

200

1

−5750

𝑅1 − 𝑅4 𝑅3 + 𝑅4 𝑅5 + 150𝑅4

Optimal solution: 𝑝 = −5,750∕1 = −5,750; 𝑥 = 15∕1 = 15, 𝑦 = 5∕1 = 5, 𝑧 = 0, 𝑤 = 10∕1 = 10. Since 𝑐 = −𝑝, the minimum value of c is 5,750. Fly 15 people from Chicago to LA, 5 from Chicago to New York, 10 from Denver to New York at a total cost of $5,750. 49. Take the unknowns as shown in the figure:

For all parts of the problem, set up the following as constraints: Intersection A: Traffic in = Traffic out: 150 + 𝑥 = 50 + 𝑧, or 𝑧 − 𝑥 = 100. Intersection B: Traffic in = Traffic out: 100 + 𝑤 = 200 + 𝑥, or 𝑤 − 𝑥 = 100 Intersection C: Traffic in = Traffic out: 100 + 𝑧 = 100 + 𝑦, or 𝑦 − 𝑧 = 0 Intersection D: Traffic in = Traffic out: 50 + 𝑦 = 50 + 𝑤, or 𝑦 − 𝑤 = 0 a. Solve four LP problems with the same constraints given above, but with the following four objectives: : Minimize x, Minimize y, Minimize z, and Minimize w. The resulting minima are: Minimum 𝑥: 0 Minimum 𝑦: 100 Minimum 𝑧: 100 Minimum 𝑤: 100 Thus, the minimum traffic on all middle sections is 100 cars per minute except for Bree which has a minimum traffic of zero cars/min. b. Add the constraint 𝑥 = 400 and maximize and minimize 𝑧. The maximum and minimum will be the same, giving the unique traffic flow along the middle section of Simmons Street: 500 cas/min. c. Use the original constraint to maximize z (for Simmons). The resulting LP problem has no optimal solution and the tool indicates that 𝑧 can be made as large as we like. We can visualize this by imagining thousands of cars going around and around the center block without affecting the recorded numbers in the


Solutions Section 6.4 diagram. 50. Take the unknowns as shown in the figure:

For all parts, use the following as constraints: Intersection A: Traffic in = Traffic out: 𝑥 = 50 + 𝑢 ⇒ 𝑥 − 𝑢 = 50 Intersection B: Traffic in = Traffic out: 𝑧 + 𝑦 = 50 + 𝑥 ⇒ −𝑥 + 𝑦 + 𝑧 = 50 Intersection C: Traffic in = Traffic out: 100 + 𝑢 = 𝑦 + 𝑧 ⇒ 𝑦 + 𝑧 − 𝑢 = 100 a. Solve four LP problems with the same constraints given above, but with the following four objectives: : Minimize x, Minimize y, Minimize z, and Minimize u. The resulting minima are: Minimum 𝑥: 50 Minimum 𝑦: 0 Minimum 𝑧: 0 Minimum 𝑢: 0 Thus, the minimum traffic on all stretches is 0 cars/min except for Oak St., which has a minimum traffic of 50 cars/min. b. Add the constraint 𝑢 = 20 and maximize and minimize 𝑥. The maximum and minimum will be the same, giving the unique traffic flow along the middle section of Simmons Street: 70 cars/min. c. Use the original constraint plus the additional constraint 𝑦 = 0 and maximize and minimze 𝑧 (Einstein Ave). The resulting minimum is 100 cars/min, but there is no maximum (the resulting LP problem has an unbounded objective function). We can visualize this by imagining thousands of cars going around and around the outer block without affecting the recorded numbers in the diagram. 51. Unknowns: 𝑥 = # cardiologists hired, 𝑦 = # rehabilitation specialists hired, 𝑧 = # infectious disease specialists hired Maximize 𝑝 = 12𝑥 + 19𝑦 + 14𝑧 (thousands of dollars) subject to 𝑥 + 𝑦 + 3𝑧 ≥ 27, 10𝑥 + 10𝑦 + 10𝑧 ≤ 200 − 30 = 170, 𝑥 ≥ 0, 𝑦 ≥ 0, 𝑧 ≥ 0. 𝑥 𝑦 𝑧 𝑠 𝑡 𝑝 *𝑠 𝑡

1

1

3

−1

0

0

27

1

1

1

0

1

0

17

𝑝

−12

−19

−14

0

0

1

0

𝑥

𝑦

𝑧

𝑠

𝑡

𝑝

𝑧

0.3333

0.3333

1

−0.3333

0

0

9

𝑡

0.6667

0.6667

0

0.3333

1

0

8

𝑝 −7.3333 −14.3333

0

−4.6667

0

1

126


Solutions Section 6.4 𝑥

𝑦

𝑧

𝑠

𝑡

𝑝

𝑧 𝑦

0 1

0 1

1 0

−0.5 0.5

−0.5 1.5

0 0

5 12

𝑝

7

0

0

2.5

21.5

1

298

Optimal solution: 𝑝 = 298; 𝑥 = 0, 𝑦 = 12, 𝑧 = 5. Hire no more cardiologists, 12 rehabilitation specialists, and 5 infectious disease specialists. 52. Unknowns: 𝑥 = # cardiologists hired, 𝑦 = # rehabilitation specialists hired, 𝑧 = # infectious disease specialists hired Maximize 𝑝 = 120 × 12𝑥 + 90 × 19𝑦 + 70 × 14𝑧 (thousand dollars per year) subject to 𝑥 + 𝑦 + 3𝑧 ≥ 27, 120𝑥 + 90𝑦 + 50𝑧 ≤ 1,960 − 360 = 1,600, 𝑥 ≥ 0, 𝑦 ≥ 0, 𝑧 ≥ 0. 𝑥 𝑦 𝑧 𝑠 𝑡 𝑝 *𝑠 𝑡

1

1

3

−1

0

0

27

120

90

50

0

1

0

1600

𝑝 −1440 −1710 −980

0

0

1

0

𝑥

𝑦

𝑧

𝑠

𝑡

𝑝

𝑧

0.3333

0.3333

1

−0.3333

0

0

9

𝑡

103.3333

73.3333

0

16.6667

1

0

1150

0

−326.6667

0

1

8820

𝑠

𝑡

𝑝

𝑝 −1113.3333 −1383.3333 𝑥

𝑦

𝑧

𝑧 −0.1364

0

1

−0.4091 −0.0045

0

3.7727

𝑦

1.4091

1

0

0.2273

0.0136

0

15.6818

𝑝 835.9091

0

0

−12.2727 18.8636

1

30513.1818

𝑥

𝑦

𝑧

𝑠

𝑡

𝑝

𝑧 𝑠

2.4 6.2

1.8 4.4

1 0

0 1

0.02 0.06

0 0

32 69

𝑝

912

54

0

0

19.6

1

31360

Optimal solution: 𝑝 = 31,360; 𝑥 = 0, 𝑦 = 0, 𝑧 = 32. Hire 32 infectious disease specialists and no one else. 53. In a general linear programming problem, the solution 𝑥 = 0, 𝑦 = 0, ... represented by the initial tableau may not be feasible (feasible solutions must satisfy all the constraints), and this shows up as a basic solution with some negative values for surplus variables. In order for a basic solution to be feasible, none of the basic variables (including surplus and slack variables) can be negative. In phase I we use pivoting to arrive at a basic solution where no basic variables are negative.


Solutions Section 6.4 54. The purpose of Phase I is to arrive at a tableau representing a basic solution that is feasible (satisfies all the constraints). However, at the conclusion of Phase I, the basic solution need not be optimal. The purpose of Phase II is therefore to arrive at an optimal solution by pivoting. 55. The basic solution corresponding to the initial tableau has all the unknowns equal to zero, and this is not a feasible solution because it does not satisfy the given inequality. 56. We star all rows whose associated active variables have negative values. In the simplex method, a negative variable indicates a basic solution that is not feasible, so we star those rows to remind ourselves that we have not yet obtained a feasible solution. 57. We can rewrite the given problem as a standard LP problem by multiplying both sides of the first constraint by −1 : Maximize 𝑝 = 𝑥 + 𝑦 subject to −𝑥 + 2𝑦 ≤ 0, 2𝑥 + 𝑦 ≤ 10, 𝑥 ≥ 0, 𝑦 ≥ 0. Therefore, this problem can be solved using the techniques of either section (choice (C)). 58. The constraint 𝑥 − 2𝑦 ≥ 1 cannot be written as a "\leq" constraint without making the right-hand side negative. Therefore, the given problem cannot be written as a standard LP problem. Therefore, the problem must be solved using the techniques of this section (Choice (C)). 59. Answers may vary. Examples are Exercises 1 and 2. 60. Answers may vary. Examples are Exercises 3 and 4. 61. A simple example is: Maximize 𝑝 = 𝑥 + 𝑦 subject to 𝑥 + 𝑦 ≤ 10, 𝑥 + 𝑦 ≥ 20, 𝑥 ≥ 0, 𝑦 ≥ 0. 𝑥 𝑦 𝑠 𝑡 𝑝 𝑠

1

1

1

0

0

10

*𝑡

1

1

0

−1

0

20

𝑅2 − 𝑅1

𝑝

−1

−1

0

0

1

0

𝑅3 + 𝑅1

𝑥

𝑦

𝑠

𝑡

𝑝

𝑥 *𝑡

1 0

1 0

1

−1

0

−1

0 0

10 10

𝑝

0

0

1

0

1

10

We now find it impossible to find a feasible solution, since there are no positive entries in the starred row. This problem has an empty feasible region, and hence no optimal solution. 62. A simple example is: Maximize 𝑝 = 2𝑥 + 𝑦 subject to 2𝑥 + 𝑦 ≤ 10, 𝑥 + 𝑦 ≥ 4, 𝑥 ≥ 0, 𝑦 ≥ 0. The usual simplex method yields the following tableau, giving one solution: 𝑝 = 10; 𝑥 = 5, 𝑦 = 0 : 𝑥 𝑦 𝑠 𝑡 𝑝 𝑡 𝑥

0 2

−1 1

1 1

2 0

0 0

2 10

𝑝 0 0 1 0 1 10 Another solution, 𝑝 = 10; 𝑥 = 0, 𝑦 = 10, can now be obtained by choosing the 𝑦-column as a pivot column. In general, if we choose any column corresponding to an inactive variable with a zero in the bottom row of the final tableau, we do not change the value of the objective function by pivoting in that column, and so we can find other solutions.


Solutions Section 6.4


Solutions Section 6.5 Section 6.5 1. Maximize 𝑝 = 2𝑥 + 𝑦 subject to 𝑥 + 2𝑦 ≤ 6 −𝑥 + 𝑦 ≤ 2 𝑥 ≥ 0, 𝑦 ≥ 0.

1 2 6 1 −1 2 → 2 1 1 −1 1 2 2 1 0 6 2 0

Minimize 𝑐 = 6𝑠 + 2𝑡 subject to 𝑠−𝑡≥2 2𝑠 + 𝑡 ≥ 1 𝑠 ≥ 0, 𝑡 ≥ 0.

2. Maximize 𝑝 = 𝑥 + 5𝑦 subject to 𝑥+𝑦≤6 −𝑥 + 3𝑦 ≤ 4 𝑥 ≥ 0, 𝑦 ≥ 0.

1 1 6 1 −1 1 −1 3 4 → 1 3 5 1 5 0 6 4 0

Minimize 𝑐 = 6𝑠 + 4𝑡 subject to 𝑠−𝑡≥1 𝑠 + 3𝑡 ≥ 5 𝑠 ≥ 0, 𝑡 ≥ 0.

3. Minimize 𝑐 = 2𝑠 + 𝑡 + 3𝑢 subject to 𝑠 + 𝑡 + 𝑢 ≥ 100 2𝑠 + 𝑡 ≥ 50 𝑠 ≥ 0, 𝑡 ≥ 0, 𝑢 ≥ 0. 4. Minimize 𝑐 = 2𝑠 + 2𝑡 + 3𝑢 subject to 𝑠 + 𝑢 ≥ 100 2𝑠 + 𝑡 ≥ 50 𝑠 ≥ 0, 𝑡 ≥ 0, 𝑢 ≥ 0.

5. Maximize 𝑝=𝑥+𝑦+𝑧+𝑤 subject to 𝑥+𝑦+𝑧≤3 𝑦+𝑧+𝑤≤4 𝑥+𝑧+𝑤≤5 𝑥+𝑦+𝑤≤6 𝑥 ≥ 0, 𝑦 ≥ 0 𝑧 ≥ 0, 𝑤 ≥ 0. 6. Maximize 𝑝=𝑥+𝑦+𝑧+𝑤 subject to 𝑥+𝑦+𝑧≤3 𝑦+𝑧+𝑤≤3 𝑥+𝑧+𝑤≤4 𝑥+𝑦+𝑤≤4 𝑥 ≥ 0, 𝑦 ≥ 0 𝑧 ≥ 0, 𝑤 ≥ 0.

1 0 1 1 1

1 0 1 1 1

1 1 1 100

2 1 0 50

2 1 3 0

Maximize 𝑝 = 100𝑥 + 50𝑦 subject to 𝑥 + 2𝑦 ≤ 2 𝑥+𝑦≤1 𝑥 ≤ 3𝑥 ≥ 0, 𝑦 ≥ 0.

1 1 0 1 100 0 2 1 0 50 → 1 2 2 3 0 100

2 1 0 50

2 2 3 0

Maximize 𝑝 = 100𝑥 + 50𝑦 subject to 𝑥 + 2𝑦 ≤ 2 𝑦≤2 𝑥≤3 𝑥 ≥ 0, 𝑦 ≥ 0.

1 1 1 100 2 1 0 50 → 2 1 3 0

1 1 0 1 1

1 1 0 1 1

1 1 1 0 1

1 1 1 0 1

0 1 1 1 1

0 1 1 1 1

3 1 4 1 → 5 1 6 0 0 3

3 3 4 → 4 0

1 1 1 0 3

0 1 1 1 4

0 1 1 1 3

1 0 1 1 5

1 0 1 1 4

1 1 0 1 6

1 1 0 1 4

1 1 1 1 0

Minimize 𝑐 = 3𝑠 + 4𝑡 + 5𝑢 + 6𝑣 subject to 𝑠+𝑢+𝑣≥1 𝑠+𝑡+𝑣≥1 𝑠+𝑡+𝑢≥1 𝑡+𝑢+𝑣≥1 𝑠 ≥ 0, 𝑡 ≥ 0 𝑢 ≥ 0, 𝑣 ≥ 0.

1 1 1 1 0

Minimize 𝑐 = 3𝑠 + 3𝑡 + 4𝑢 + 4𝑣 subject to 𝑠+𝑢+𝑣≥1 𝑠+𝑡+𝑣≥1 𝑠+𝑡+𝑢≥1 𝑡+𝑢+𝑣≥1 𝑠 ≥ 0, 𝑡 ≥ 0 𝑢 ≥ 0, 𝑣 ≥ 0.


Solutions Section 6.5 5 −1 0 1 1,000 0 0 1 −1 2,000 → 1 1 0 0 500 1 3 1 0 0 5 0 1 1 0 1 3 −1 0 1 0 1 1 0 0 −1 1,000 2,000 500 0

7. Minimize 𝑐 = 𝑠 + 3𝑡 + 𝑢 subject to 5𝑠 − 𝑡 + 𝑣 ≥ 1,000 𝑢 − 𝑣 ≥ 2,000 𝑠 + 𝑡 ≥ 500 𝑠 ≥ 0, 𝑡 ≥ 0 𝑢 ≥ 0, 𝑣 ≥ 0.

Maximize 𝑝 = 1,000𝑥 + 2,000𝑦 + 500𝑧 subject to 5𝑥 + 𝑧 ≤ 1 −𝑥 + 𝑧 ≤ 3 𝑦≤1 𝑥−𝑦≤0 𝑥 ≥ 0, 𝑦 ≥ 0, 𝑧 ≥ 0.

8. Minimize 𝑐 = 5𝑠 + 2𝑢 + 𝑣 subject to 𝑠 − 𝑡 + 2𝑣 ≥ 2,000 𝑢 + 𝑣 ≥ 3,000 𝑠 + 𝑡 ≥ 500 𝑠 ≥ 0, 𝑡 ≥ 0 𝑢 ≥ 0, 𝑣 ≥ 0.

1 −1 0 2 2,000 0 0 1 1 3,000 → 1 1 0 0 500 5 0 2 1 0

1 0 1 0 1 −1 0 1 0 2 1 0 2,000 3,000 500

9. Minimize 𝑐 = 𝑠 + 𝑡 subject to 𝑠 + 2𝑡 ≥ 6 2𝑠 + 𝑡 ≥ 6 𝑠 ≥ 0, 𝑡 ≥ 0.

1 2 6 1 2 1 2 1 6 → 2 1 1 1 1 0 6 6 0

Maximize 𝑝 = 6𝑥 + 6𝑦 subject to 𝑥 + 2𝑦 ≤ 1 2𝑥 + 𝑦 ≤ 1 𝑥 ≥ 0, 𝑦 ≥ 0.

5 0 2 1 0

Maximize 𝑝 = 2,000𝑥 + 3,000𝑦 + 500𝑧 subject to 𝑥+𝑧≤5 −𝑥 + 𝑧 ≤ 0 𝑦≤2 2𝑥 + 𝑦 ≤ 1 𝑥 ≥ 0, 𝑦 ≥ 0, 𝑧 ≥ 0.

Solve the dual problem using the standard simplex method: 𝑥 𝑦 𝑠 𝑡 𝑝 2𝑅1 − 𝑅2

𝑠

1

2

1

0

0

1

𝑡

2

1

0

1

0

1

𝑝

−6

−6

0

0

1

0

𝑥

𝑦

𝑠

𝑡

𝑝

𝑠

0

3

2

−1

0

1

𝑥

2

1

0

1

0

1

3𝑅2 − 𝑅1

𝑝

0

−3

0

3

1

3

𝑅3 + 𝑅1

𝑥

𝑦

𝑠

𝑡

𝑝

𝑦 𝑥

0 6

3 0

2 −2

−1 4

0 0

1 2

𝑝

0

0

2

2

1

4

𝑅3 + 3𝑅2

The solution to the primal problem is 𝑐 = 4∕1 = 4; 𝑠 = 2∕1 = 2, 𝑡 = 2∕1 = 2.


10. Minimize 𝑐 = 𝑠 + 2𝑡 subject to 𝑠 + 3𝑡 ≥ 30 2𝑠 + 𝑡 ≥ 30 𝑠 ≥ 0, 𝑡 ≥ 0.

Solutions Section 6.5 Maximize 𝑝 = 30𝑥 + 30𝑦 1 3 30 1 2 1 subject to 𝑥 + 2𝑦 ≤ 1 2 1 30 → 3 1 2 3𝑥 + 𝑦 ≤ 2 1 2 0 30 30 0 𝑥 ≥ 0, 𝑦 ≥ 0.

Solve the dual problem using the standard simplex method: 𝑥 𝑦 𝑠 𝑡 𝑝 3𝑅1 − 𝑅2

𝑠

1

2

1

0

0

1

𝑡

3

1

0

1

0

2

𝑝

−30

−30

0

0

1

0

𝑥

𝑦

𝑠

𝑡

𝑝

𝑠

0

5

3

−1

0

1

𝑥

3

1

0

1

0

2

5𝑅2 − 𝑅1

𝑝

0

−20

0

10

1

20

𝑅3 + 4𝑅1

𝑥

𝑦

𝑠

𝑡

𝑝

𝑦 𝑥

0 15

5 0

3 −3

−1 6

0 0

1 9

𝑝

0

0

12

6

1

24

𝑅3 + 10𝑅2

The solution to the primal problem is 𝑐 = 24∕1 = 24; 𝑠 = 12∕1 = 12, 𝑡 = 6∕1 = 6. 11. Minimize 𝑐 = 6𝑠 + 6𝑡 subject to 𝑠 + 2𝑡 ≥ 20 2𝑠 + 𝑡 ≥ 20 𝑠 ≥ 0, 𝑡 ≥ 0.

1 2 20 1 2 6 2 1 20 → 2 1 6 6 6 0 20 20 0

Maximize 𝑝 = 20𝑥 + 20𝑦 subject to 𝑥 + 2𝑦 ≤ 6 2𝑥 + 𝑦 ≤ 6 𝑥 ≥ 0, 𝑦 ≥ 0.

Solve the dual problem using the standard simplex method: 𝑥 𝑦 𝑠 𝑡 𝑝 2𝑅1 − 𝑅2

𝑠

1

2

1

0

0

6

𝑡

2

1

0

1

0

6

𝑝

−20

−20

0

0

1

0

𝑥

𝑦

𝑠

𝑡

𝑝

𝑠

0

3

2

−1

0

6

𝑥

2

1

0

1

0

6

3𝑅2 − 𝑅1

𝑝

0

−10

0

10

1

60

3𝑅3 + 10𝑅1

𝑅3 + 10𝑅2


Solutions Section 6.5 𝑥

𝑦

𝑠

𝑡

𝑝

𝑦 𝑥

0 6

3 0

2

−2

−1 4

0 0

6 12

𝑝

0

0

20

20

3

240

The solution to the primal problem is 𝑐 = 240∕3 = 80; 𝑠 = 20∕3, 𝑡 = 20∕3. 12. Minimize 𝑐 = 3𝑠 + 2𝑡 subject to 𝑠 + 2𝑡 ≥ 20 2𝑠 + 𝑡 ≥ 10 𝑠 ≥ 0, 𝑡 ≥ 0.

Maximize 𝑝 = 20𝑥 + 10𝑦 subject to 𝑥 + 2𝑦 ≤ 3 2𝑥 + 𝑦 ≤ 2 𝑥 ≥ 0, 𝑦 ≥ 0.

1 2 20 1 2 3 2 1 10 → 2 1 2 3 2 0 20 10 0

Solve the dual problem using the standard simplex method: 𝑥 𝑦 𝑠 𝑡 𝑝 𝑠

1

2

1

0

0

3

𝑡

2

1

0

1

0

2

𝑝

−20

−10

0

0

1

0

𝑥

𝑦

𝑠

𝑡

𝑝

𝑠 𝑥

0 2

3 1

2 0

−1 1

0 0

4 2

𝑝

0

0

0

10

1

20

2𝑅1 − 𝑅2 𝑅3 + 10𝑅2

The solution to the primal problem is 𝑐 = 20∕1 = 20; 𝑠 = 0, 𝑡 = 10∕1 = 10. 13. Minimize 𝑐 = 0.2𝑠 + 0.3𝑡 subject to 2𝑠 + 𝑡 ≥ 10 𝑠 + 2𝑡 ≥ 10 𝑠+𝑡≥8 𝑠 ≥ 0, 𝑡 ≥ 0.

2 1 10 2 1 1 0.2 1 2 10 → 1 2 1 0.3 1 1 8 10 10 8 0 0.2 0.3 0

Maximize 𝑝 = 10𝑥 + 10𝑦 + 8𝑧 subject to 2𝑥 + 𝑦 + 𝑧 ≤ 0.2 𝑥 + 2𝑦 + 𝑧 ≤ 0.3 𝑥 ≥ 0, 𝑦 ≥ 0, 𝑧 ≥ 0.

Solve the dual problem using the standard simplex method. (Note: Do not rewrite the constraints without decimals at this stage—clear them in the first step of the simplex method.) 𝑥 𝑦 𝑧 𝑠 𝑡 𝑝 𝑠

2

1

1

1

0

0

0.2

5𝑅1

𝑡

1

2

1

0

1

0

0.3

10𝑅2

𝑝

−10

−10

−8

0

0

1

0


Solutions Section 6.5 𝑥

𝑦

𝑧

𝑠

𝑡

𝑝

𝑠

10

5

5

5

0

0

1

𝑡

10

20

10

0

10

0

3

𝑅2 − 𝑅1

𝑝

−10

−10

−8

0

0

1

0

𝑅3 + 𝑅1

𝑥

𝑦

𝑧

𝑠

𝑡

𝑝

𝑥

10

5

5

5

0

0

1

3𝑅1 − 𝑅2

𝑡

0

15

5

−5

10

0

2

𝑝

0

−5

−3

5

0

1

1

𝑥

𝑦

𝑧

𝑠

𝑡

𝑝

𝑥

30

0

10

20

−10

0

1

𝑦

0

15

5

−5

10

0

2

2𝑅2 − 𝑅1

𝑝

0

0

−4

10

10

3

5

5𝑅3 + 2𝑅1

𝑥

𝑦

𝑧

𝑠

𝑡

𝑝

𝑧 𝑦

30 −30

0 30

10 0

20 −30

−10 30

0 0

1 3

𝑝

60

0

0

90

30

15

27

3𝑅3 + 𝑅2

The solution to the primal problem is 𝑐 = 27∕15 = 1.8; 𝑠 = 90∕15 = 6, 𝑡 = 30∕15 = 2. 14. Minimize 𝑐 = 0.4𝑠 + 0.1𝑡 subject to 3𝑠 + 2𝑡 ≥ 60 𝑠 + 2𝑡 ≥ 40 2𝑠 + 3𝑡 ≥ 45 𝑠 ≥ 0, 𝑡 ≥ 0.

3 2 60 3 1 2 0.4 1 2 40 → 2 2 3 0.1 2 3 45 60 40 45 0 0.4 0.1 0

Maximize 𝑝 = 60𝑥 + 40𝑦 + 45𝑧 subject to 3𝑥 + 𝑦 + 2𝑧 ≤ 0.4 2𝑥 + 2𝑦 + 3𝑧 ≤ 0.1 𝑥 ≥ 0, 𝑦 ≥ 0, 𝑧 ≥ 0.

Solve the dual problem using the standard simplex method. (Note: Do not rewrite the constraints without decimals at this stage—clear them in the first step of the simplex method.) 𝑥 𝑦 𝑧 𝑠 𝑡 𝑝 𝑠

3

1

2

1

0

0

0.4

5𝑅1

𝑡

2

2

3

0

1

0

0.1

10𝑅2

𝑝

−60

−40

−45

0

0

1

0

𝑥

𝑦

𝑧

𝑠

𝑡

𝑝

𝑠

15

5

10

5

0

0

2

𝑡

20

20

30

0

10

0

1

𝑝

−60

−40

−45

0

0

1

0

4𝑅1 − 3𝑅2 𝑅3 + 3𝑅2


Solutions Section 6.5 𝑥

𝑦

𝑧

𝑠

𝑡

𝑝

𝑠 𝑥

0 20

−40 20

−50 30

20 0

−30 10

0 0

5 1

𝑝

0

20

45

0

30

1

3

The solution to the primal problem is 𝑐 = 3∕1 = 3; 𝑠 = 0, 𝑡 = 30∕1 = 30. 15. Minimize 𝑐 = 2𝑠 + 𝑡 subject to 3𝑠 + 𝑡 ≥ 30 𝑠 + 𝑡 ≥ 20 𝑠 + 3𝑡 ≥ 30 𝑠 ≥ 0, 𝑡 ≥ 0.

3 1 1 2

1 30 3 1 1 2 1 20 → 1 1 3 1 3 30 30 20 30 0 1 0

Maximize 𝑝 = 30𝑥 + 20𝑦 + 30𝑧 subject to 3𝑥 + 𝑦 + 𝑧 ≤ 2 𝑥 + 𝑦 + 3𝑧 ≤ 1 𝑥 ≥ 0, 𝑦 ≥ 0, 𝑧 ≥ 0.

Solve the dual problem using the standard simplex method: 𝑥 𝑦 𝑧 𝑠 𝑡 𝑝 𝑠

3

1

1

1

0

0

2

𝑡

1

1

3

0

1

0

1

3𝑅2 − 𝑅1

𝑝

−30

−20

−30

0

0

1

0

𝑅3 + 10𝑅1

𝑥

𝑦

𝑧

𝑠

𝑡

𝑝

𝑥

3

1

1

1

0

0

2

8𝑅1 − 𝑅2

𝑡

0

2

8

−1

3

0

1

𝑝

0

−10

−20

10

0

1

20

2𝑅3 + 5𝑅2

𝑥

𝑦

𝑧

𝑠

𝑡

𝑝

𝑥

24

6

0

9

−3

0

15

𝑅1 − 3𝑅2

𝑧

0

2

8

−1

3

0

1

𝑝

0

−10

0

15

15

2

45

𝑥

𝑦

𝑧

𝑠

𝑡

𝑝

𝑥 𝑦

24 0

0 2

−24 8

12 −1

−12 3

0 0

12 1

𝑝

0

0

40

10

30

2

50

𝑅3 + 5𝑅2

The solution to the primal problem is 𝑐 = 50∕2 = 25; 𝑠 = 10∕2 = 5, 𝑡 = 30∕2 = 15.


16. Minimize 𝑐 = 𝑠 + 2𝑡 subject to 4𝑠 + 𝑡 ≥ 100 2𝑠 + 𝑡 ≥ 80 𝑠 + 3𝑡 ≥ 150 𝑠 ≥ 0, 𝑡 ≥ 0.

Solutions Section 6.5 4 1 100 4 2 1 1 2 1 80 → 1 1 3 2 1 3 150 100 80 150 0 1 2 0

Maximize p = 100𝑥 + 80𝑦 + 150𝑧 subject to 4𝑥 + 2𝑦 + 𝑧 ≤ 1 𝑥 + 𝑦 + 3𝑧 ≤ 2 𝑥 ≥ 0, 𝑦 ≥ 0, 𝑧 ≥ 0.

Solve the dual problem using the standard simplex method: 𝑥 𝑦 𝑧 𝑠 𝑡 𝑝 3𝑅1 − 𝑅2

𝑠

4

2

1

1

0

0

1

𝑡

1

1

3

0

1

0

2

𝑝

−100

−80

−150

0

0

1

0

𝑥

𝑦

𝑧

𝑠

𝑡

𝑝

𝑠

11

5

0

3

−1

0

1

𝑧

1

1

3

0

1

0

2

11𝑅2 − 𝑅1

𝑝

−50

−30

0

0

50

1

100

11𝑅3 + 50𝑅1

𝑥

𝑦

𝑧

𝑠

𝑡

𝑝

𝑥

11

5

0

3

−1

0

1

𝑧

0

6

33

−3

12

0

21

5𝑅2 − 6𝑅1

𝑝

0

−80

0

150

500

11

1150

𝑅3 + 16𝑅1

𝑥

𝑦

𝑧

𝑠

𝑡

𝑝

𝑦 𝑧

11 −66

5 0

0 165

3 −33

−1 66

0 0

1 99

𝑝

176

0

0

198

484

11

1166

𝑅3 + 50𝑅2

The solution to the primal problem is 𝑐 = 1,166∕11 = 106; 𝑠 = 198∕11 = 18, 𝑡 = 484∕11 = 44. 17. Minimize 𝑐 = 𝑠 + 2𝑡 + 3𝑢 subject to 3𝑠 + 2𝑡 + 𝑢 ≥ 60 2𝑠 + 𝑡 + 3𝑢 ≥ 60 𝑠 ≥ 0, 𝑡 ≥ 0, 𝑢 ≥ 0.

3 2 3 2 1 60 2 1 2 1 3 60 → 1 3 1 2 3 0 60 60

1 2 3 0

Maximize 𝑝 = 60𝑥 + 60𝑦 subject to 3𝑥 + 2𝑦 ≤ 1 2𝑥 + 𝑦 ≤ 2 𝑥 + 3𝑦 ≤ 3 𝑥 ≥ 0, 𝑦 ≥ 0.

Solve the dual problem using the standard simplex method: 𝑥 𝑦 𝑠 𝑡 𝑢 𝑝 𝑠

3

2

1

0

0

0

1

𝑡

2

1

0

1

0

0

2

3𝑅2 − 2𝑅1

𝑢

1

3

0

0

1

0

3

3𝑅3 − 𝑅1

𝑝

−60

−60

0

0

0

1

0

𝑅4 + 20𝑅1


Solutions Section 6.5 𝑥

𝑦

𝑠

𝑡

𝑢

𝑝

𝑥

3

2

1

0

0

0

1

𝑡

0

−1

−2

3

0

0

4

2𝑅2 + 𝑅1

𝑢

0

7

−1

0

3

0

8

2𝑅3 − 7𝑅1

𝑝

0

−20

20

0

0

1

20

𝑅4 + 10𝑅1

𝑥

𝑦

𝑠

𝑡

𝑢

𝑝

𝑦 𝑡 𝑢

3 3 −21

2 0 0

1 −3 −9

0 6 0

0 0 6

0 0 0

1 9 9

𝑝

30

0

30

0

0

1

30

The solution to the primal problem is 𝑐 = 30∕1 = 30; 𝑠 = 30∕1 = 30, 𝑡 = 0, 𝑢 = 0. 18. Minimize 𝑐 = 𝑠 + 𝑡 + 2𝑢 subject to 𝑠 + 2𝑡 + 2𝑢 ≥ 60 2𝑠 + 𝑡 + 3𝑢 ≥ 60 𝑠 ≥ 0, 𝑡 ≥ 0, 𝑢 ≥ 0.

1 2 1 2 2 60 2 1 2 1 3 60 → 2 3 1 1 2 0 60 60

1 1 2 0

Maximize 𝑝 = 60𝑥 + 60𝑦 subject to 𝑥 + 2𝑦 ≤ 1 2𝑥 + 𝑦 ≤ 1 2𝑥 + 3𝑦 ≤ 2 𝑥 ≥ 0, 𝑦 ≥ 0.

Solve the dual problem using the standard simplex method: 𝑥 𝑦 𝑠 𝑡 𝑢 𝑝 2𝑅1 − 𝑅2

𝑠

1

2

1

0

0

0

1

𝑡

2

1

0

1

0

0

1

𝑢

2

3

0

0

1

0

2

𝑅3 − 𝑅2

𝑝

−60

−60

0

0

0

1

0

𝑅4 + 30𝑅2

𝑥

𝑦

𝑠

𝑡

𝑢

𝑝

𝑠

0

3

2

−1

0

0

1

𝑥

2

1

0

1

0

0

1

3𝑅2 − 𝑅1

𝑢

0

2

0

−1

1

0

1

3𝑅3 − 2𝑅1

𝑝

0

−30

0

30

0

1

30

𝑅4 + 10𝑅1

𝑥

𝑦

𝑠

𝑡

𝑢

𝑝

𝑦 𝑥 𝑢

0 6 0

3 0 0

2 −2 −4

−1 4 −1

0 0 3

0 0 0

1 2 1

𝑝

0

0

20

20

0

1

40

The solution to the primal problem is 𝑐 = 40∕1 = 40; 𝑠 = 20∕1 = 20, 𝑡 = 20∕1 = 20, 𝑢 = 0.


Solutions Section 6.5

19. Minimize 𝑐 = 2𝑠 + 𝑡 + 3𝑢 subject to 𝑠 + 𝑡 + 𝑢 ≥ 100 2𝑠 + 𝑡 ≥ 50 𝑡 + 𝑢 ≥ 50 𝑠 ≥ 0, 𝑡 ≥ 0, 𝑢 ≥ 0.

1 2 0 2

1 1 1 1

1 100 1 0 50 1 → 1 50 1 3 0 100

2 1 0 50

0 1 1 50

Maximize 𝑝 = 100𝑥 + 50𝑦 + 50𝑧 subject to 𝑥 + 2𝑦 ≤ 2 𝑥+𝑦+𝑧≤1 𝑥+𝑧≤3 𝑥 ≥ 0, 𝑦 ≥ 0, 𝑧 ≥ 0.

2 1 3 0

Solve the dual problem using the standard simplex method: 𝑥 𝑦 𝑧 𝑠 𝑡 𝑢 𝑝 𝑅1 − 𝑅2

𝑠

1

2

0

1

0

0

0

2

𝑡

1

1

1

0

1

0

0

1

𝑢

1

0

1

0

0

1

0

3

𝑅3 − 𝑅2

𝑝

−100

−50

−50

0

0

0

1

0

𝑅4 + 100𝑅2

𝑥

𝑦

𝑧

𝑠

𝑡

𝑢

𝑝

𝑠 𝑥 𝑢

0 1 0

1 1 −1

−1 1 0

1 0 0

−1 1 −1

0 0 1

0 0 0

1 1 2

𝑝

0

50

50

0

100

0

1

100

The solution to the primal problem is 𝑐 = 100∕1 = 100; 𝑠 = 0, 𝑡 = 100∕1 = 100, 𝑢 = 0. 20. Minimize 𝑐 = 2𝑠 + 2𝑡 + 3𝑢 subject to 𝑠 + 𝑢 ≥ 100 2𝑠 + 𝑡 ≥ 50 𝑡 + 𝑢 ≥ 50 𝑠 ≥ 0, 𝑡 ≥ 0, 𝑢 ≥ 0.

1 2 0 2

0 1 1 2

1 100 1 0 50 0 → 1 50 1 3 0 100

2 1 0 50

0 1 1 50

2 2 3 0

Maximize 𝑝 = 100𝑥 + 50𝑦 + 50𝑧 subject to 𝑥 + 2𝑦 ≤ 2 𝑦+𝑧≤2 𝑥+𝑧≤3 𝑥 ≥ 0, 𝑦 ≥ 0, 𝑧 ≥ 0.

Solve the dual problem using the standard simplex method: 𝑥 𝑦 𝑧 𝑠 𝑡 𝑢 𝑝 𝑠

1

2

0

1

0

0

0

2

𝑡

0

1

1

0

1

0

0

2

𝑢

1

0

1

0

0

1

0

3

𝑅3 − 𝑅1

𝑝

−100

−50

−50

0

0

0

1

0

𝑅4 + 100𝑅1

𝑥

𝑦

𝑧

𝑠

𝑡

𝑢

𝑝

𝑥

1

2

0

1

0

0

0

2

𝑡

0

1

1

0

1

0

0

2

𝑢

0

−2

1

−1

0

1

0

1

𝑝

0

150

−50

100

0

0

1

200

𝑅2 − 𝑅3 𝑅4 + 50𝑅3


Solutions Section 6.5 𝑥

𝑦

𝑧

𝑠

𝑡

𝑢

𝑝

𝑥 𝑡 𝑧

1 0 0

2 3 −2

0 0 1

1 1 −1

0 1 0

0

−1 1

0 0 0

2 1 1

𝑝

0

50

0

50

0

50

1

250

The solution to the primal problem is 𝑐 = 250∕1 = 250; 𝑠 = 50∕1 = 50, 𝑡 = 0, 𝑢 = 50∕1 = 50. 21. Minimize 𝑐 = 𝑠 + 𝑡 + 𝑢 subject to 3𝑠 + 2𝑡 + 𝑢 ≥ 60 2𝑠 + 𝑡 + 3𝑢 ≥ 60 𝑠 + 3𝑡 + 2𝑢 ≥ 60 𝑠 ≥ 0, 𝑡 ≥ 0, 𝑢 ≥ 0.

3 2 1 1

2 1 3 1

1 3 2 1

60 3 2 60 2 1 → 60 1 3 0 60 60

1 3 2 60

1 1 1 0

Maximize 𝑝 = 60𝑥 + 60𝑦 + 60𝑧 subject to 3𝑥 + 2𝑦 + 𝑧 ≤ 1 2𝑥 + 𝑦 + 3𝑧 ≤ 1 𝑥 + 3𝑦 + 2𝑧 ≤ 1 𝑥 ≥ 0, 𝑦 ≥ 0, 𝑧 ≥ 0.

Solve the dual problem using the standard simplex method: 𝑥 𝑦 𝑧 𝑠 𝑡 𝑢 𝑝 𝑠

3

2

1

1

0

0

0

1

𝑡

2

1

3

0

1

0

0

1

3𝑅2 − 2𝑅1

𝑢

1

3

2

0

0

1

0

1

3𝑅3 − 𝑅1

𝑝

−60

−60

−60

0

0

0

1

0

𝑅4 + 20𝑅1

𝑥

𝑦

𝑧

𝑠

𝑡

𝑢

𝑝

𝑥

3

2

1

1

0

0

0

1

7𝑅1 − 𝑅2

𝑡

0

−1

7

−2

3

0

0

1

𝑢

0

7

5

−1

0

3

0

2

7𝑅3 − 5𝑅2

𝑝

0

−20

−40

20

0

0

1

20

7𝑅4 + 40𝑅2

𝑥

𝑦

𝑧

𝑠

𝑡

𝑢

𝑝

𝑥

21

15

0

9

−3

0

0

6

18𝑅1 − 5𝑅3

𝑧

0

−1

7

−2

3

0

0

1

54𝑅2 + 𝑅3

𝑢

0

54

0

3

−15

21

0

9

𝑝

0

−180

0

60

120

0

7

180

𝑥

𝑦

𝑧

𝑠

𝑡

𝑢

𝑝

𝑥 𝑧 𝑦

378 0 0

0 0 54

0 378 0

147 −105 3

21 147

−15

−105 21 21

0 0 0

63 63 9

𝑝

0

0

0

210

210

210

21

630

3𝑅4 + 10𝑅3

The solution to the primal problem is 𝑐 = 630∕21 = 30; 𝑠 = 210∕21 = 10, 𝑡 = 210∕21 = 10,


Solutions Section 6.5 𝑢 = 210∕21 = 10. 22. Minimize 𝑐 = 𝑠 + 𝑡 + 2𝑢 subject to 𝑠 + 2𝑡 + 2𝑢 ≥ 60 2𝑠 + 𝑡 + 3𝑢 ≥ 60 𝑠 + 3𝑡 + 6𝑢 ≥ 60 𝑠 ≥ 0, 𝑡 ≥ 0, 𝑢 ≥ 0.

1 2 1 1

2 1 3 1

2 3 6 2

60 1 2 60 2 1 → 60 2 3 0 60 60

1 3 6 60

1 1 2 0

Maximize 𝑝 = 60𝑥 + 60𝑦 + 60𝑧 subject to 𝑥 + 2𝑦 + 𝑧 ≤ 1 2𝑥 + 𝑦 + 3𝑧 ≤ 1 2𝑥 + 3𝑦 + 6𝑧 ≤ 2 𝑥 ≥ 0, 𝑦 ≥ 0, 𝑧 ≥ 0.

Solve the dual problem using the standard simplex method: 𝑥 𝑦 𝑧 𝑠 𝑡 𝑢 𝑝 2𝑅1 − 𝑅2

𝑠

1

2

1

1

0

0

0

1

𝑡

2

1

3

0

1

0

0

1

𝑢

2

3

6

0

0

1

0

2

𝑅3 − 𝑅2

𝑝

−60

−60

−60

0

0

0

1

0

𝑅4 + 30𝑅2

𝑥

𝑦

𝑧

𝑠

𝑡

𝑢

𝑝

𝑠

0

3

−1

2

−1

0

0

1

𝑥

2

1

3

0

1

0

0

1

3𝑅2 − 𝑅1

𝑢

0

2

3

0

−1

1

0

1

3𝑅3 − 2𝑅1

𝑝

0

−30

30

0

30

0

1

30

𝑅4 + 10𝑅1

𝑥

𝑦

𝑧

𝑠

𝑡

𝑢

𝑝

𝑦 𝑥 𝑢

0 6 0

3 0 0

−1 10 11

2 −2 −4

−1 4 −1

0 0 3

0 0 0

1 2 1

𝑝

0

0

20

20

20

0

1

40

The solution to the primal problem is 𝑐 = 40∕1 = 40; 𝑠 = 20∕1 = 20, 𝑡 = 20∕1 = 20, 𝑢 = 0. 23. Add 𝑘 = 2 to each entry and put 1s to the right and below: 1 3 4 1 4 1 0 1 1 1 1 0 This gives the following LP problem: Maximize 𝑝 = 𝑥 + 𝑦 + 𝑧 subject to 𝑥 + 3𝑦 + 4𝑧 ≤ 1 4𝑥 + 𝑦 ≤ 1 𝑥 ≥ 0, 𝑦 ≥ 0, 𝑧 ≥ 0. Here are the tableaux:


Solutions Section 6.5 𝑥

𝑦

𝑧

𝑠

𝑡

𝑝

𝑠

1

3

4

1

0

0

1

𝑡

4

1

0

0

1

0

1

𝑝

−1

−1

−1

0

0

1

0

𝑥

𝑦

𝑧

𝑠

𝑡

𝑝

𝑠

0

11

16

4

−1

0

3

𝑥

4

1

0

0

1

0

1

𝑝

0

−3

−4

0

1

4

1

𝑥

𝑦

𝑧

𝑠

𝑡

𝑝

𝑧

0

11

16

4

−1

0

3

𝑥

4

1

0

0

1

0

1

11𝑅2 − 𝑅1

𝑝

0

−1

0

4

3

16

7

11𝑅3 + 𝑅1

𝑥

𝑦

𝑧

𝑠

𝑡

𝑝

𝑦 𝑥

0 44

11 0

16 −16

4 −4

−1 12

0 0

3 8

𝑝

0

0

16

48

32

176

80

4𝑅1 − 𝑅2 4𝑅3 + 𝑅2

4𝑅3 + 𝑅1

The solution to the primal problem is 𝑝 = 80∕176 = 5∕11; 𝑥 = 8∕44 = 2∕11, 𝑦 = 3∕11, 𝑧 = 0. So, the column player's optimal strategy is 𝐶 = [2∕5 3∕5 0] 𝑇 and the value of the game is 𝑒 = 11∕5 − 2 = 1∕5. The solution to the dual problem is 𝑠 = 48∕176 = 3∕11, 𝑡 = 32∕176 = 2∕11, so the row player's optimal strategy is 𝑅 = [3∕5 2∕5]. 24. Add 𝑘 = 1 to each entry and put 1s to the right and below: 2 0 3 1 2 3 1 1 1 1 1 0 This gives the following LP problem: Maximize 𝑝 = 𝑥 + 𝑦 + 𝑧 subject to 2𝑥 + 3𝑧 ≤ 1 2𝑥 + 3𝑦 + 𝑧 ≤ 1 𝑥 ≥ 0, 𝑦 ≥ 0, 𝑧 ≥ 0. Here are the tableaux: 𝑥 𝑦 𝑧

𝑠

𝑡

𝑝

𝑠

2

0

3

1

0

0

1

𝑡

2

3

1

0

1

0

1

𝑅2 − 𝑅1

𝑝

−1

−1

−1

0

0

1

0

2𝑅3 + 𝑅1


Solutions Section 6.5 𝑥

𝑦

𝑧

𝑠

𝑡

𝑝

𝑥

2

0

3

1

0

0

1

𝑡

0

3

−2

−1

1

0

0

𝑝

0

−2

1

1

0

2

1

𝑥

𝑦

𝑧

𝑠

𝑡

𝑝

𝑥

2

0

3

1

0

0

1

𝑦

0

3

−2

−1

1

0

0

3𝑅2 + 2𝑅1

𝑝

0

0

−1

1

2

6

3

3𝑅3 + 𝑅1

𝑥

𝑦

𝑧

𝑠

𝑡

𝑝

𝑧 𝑦

2 4

0 9

3 0

1 −1

0 3

0 0

1 2

𝑝

2

0

0

4

6

18

10

3𝑅3 + 2𝑅2

The solution to the primal problem is 𝑝 = 10∕18 = 5∕9; 𝑥 = 0, 𝑦 = 2∕9, 𝑧 = 1∕3. So, the column player's optimal strategy is 𝐶 = [0 2∕5 3∕5] 𝑇 and the value of the game is 𝑒 = 9∕5 − 1 = 4∕5. The solution to the dual problem is 𝑠 = 4∕18 = 2∕9, 𝑡 = 6∕18 = 1∕3, so the row player's optimal strategy is 𝑅 = [2∕5 3∕5]. 25. Add 𝑘 = 2 to each entry and put 1s to the right and below: 1 3 4 1 4 1 0 1 3 4 2 1 1 1 1 0 This gives the following LP problem: Maximize 𝑝 = 𝑥 + 𝑦 + 𝑧 subject to 𝑥 + 3𝑦 + 4𝑧 ≤ 1 4𝑥 + 𝑦 ≤ 1 3𝑥 + 4𝑦 + 2𝑧 ≤ 1 𝑥 ≥ 0, 𝑦 ≥ 0, 𝑧 ≥ 0. Here are the tableaux: 𝑥 𝑦 𝑧

𝑠

𝑡

𝑢

𝑝

𝑠

1

3

4

1

0

0

0

1

𝑡

4

1

0

0

1

0

0

1

𝑢

3

4

2

0

0

1

0

1

4𝑅3 − 3𝑅2

𝑝

−1

−1

−1

0

0

0

1

0

4𝑅4 + 𝑅2

4𝑅1 − 𝑅2


Solutions Section 6.5 𝑥

𝑦

𝑧

𝑠

𝑡

𝑢

𝑝

𝑠

0

11

16

4

−1

0

0

3

𝑥

4

1

0

0

1

0

0

1

𝑢

0

13

8

0

−3

4

0

1

𝑝

0

−3

−4

0

1

0

4

1

𝑥

𝑦

𝑧

𝑠

𝑡

𝑢

𝑝

𝑠

0

−15

0

4

5

−8

0

1

𝑥

4

1

0

0

1

0

0

1

5𝑅2 − 𝑅1

𝑧

0

13

8

0

−3

4

0

1

5𝑅3 + 3𝑅1

𝑝

0

7

0

0

−1

4

8

3

5𝑅4 + 𝑅1

𝑥

𝑦

𝑧

𝑠

𝑡

𝑢

𝑝

𝑡 𝑥 𝑧

0 20 0

−15 20 20

0 0 40

4 −4 12

5 0 0

−8 8 −4

0 0 0

1 4 8

𝑝

0

20

0

4

0

12

40

16

𝑅1 − 2𝑅3

2𝑅4 + 𝑅3

The solution to the primal problem is 𝑝 = 16∕40 = 2∕5; 𝑥 = 4∕20 = 1∕5, 𝑦 = 0, 𝑧 = 8∕40 = 1∕5. So, the column player's optimal strategy is 𝐶 = [1∕2 0 1∕2] 𝑇 and the value of the game is 𝑒 = 5∕2 − 2 = 1∕2. The solution to the dual problem is 𝑠 = 4∕40 = 1∕10, 𝑡 = 0, 𝑢 = 12∕40 = 3∕10, so the row player's optimal strategy is 𝑅 = [1∕4 0 3∕4]. 26. Add 𝑘 = 1 to each entry and put 1s to the right and below: 2 0 3 1 2 3 1 1 . 1 2 2 1 1 1 1 0 This gives the following LP problem: Maximize 𝑝 = 𝑥 + 𝑦 + 𝑧 subject to 2𝑥 + 3𝑧 ≤ 1 2𝑥 + 3𝑦 + 𝑧 ≤ 1 𝑥 + 2𝑦 + 2𝑧 ≤ 1 𝑥 ≥ 0, 𝑦 ≥ 0, 𝑧 ≥ 0. Here are the tableaux: 𝑥 𝑦 𝑧

𝑠

𝑡

𝑢

𝑝

𝑠

2

0

3

1

0

0

0

1

𝑡

2

3

1

0

1

0

0

1

𝑅2 − 𝑅1

𝑢

1

2

2

0

0

1

0

1

2𝑅3 − 𝑅1

𝑝

−1

−1

−1

0

0

0

1

0

2𝑅4 + 𝑅1


Solutions Section 6.5 𝑥

𝑦

𝑧

𝑠

𝑡

𝑢

𝑝

𝑥

2

0

3

1

0

0

0

1

𝑡

0

3

−2

−1

1

0

0

0

𝑢

0

4

1

−1

0

2

0

1

3𝑅3 − 4𝑅2

𝑝

0

−2

1

1

0

0

2

1

3𝑅4 + 2𝑅2

𝑥

𝑦

𝑧

𝑠

𝑡

𝑢

𝑝

𝑥

2

0

3

1

0

0

0

1

11𝑅1 − 3𝑅3

𝑦

0

3

−2

−1

1

0

0

0

11𝑅2 + 2𝑅3

𝑢

0

0

11

1

−4

6

0

3

𝑝

0

0

−1

1

2

0

6

3

𝑥

𝑦

𝑧

𝑠

𝑡

𝑢

𝑝

𝑥 𝑦 𝑧

22 0 0

0 33 0

0 0 11

8 −9 1

12 3 −4

−18 12 6

0 0 0

2 6 3

𝑝

0

0

0

12

18

6

66

36

11𝑅4 + 𝑅3

The solution to the primal problem is 𝑝 = 36∕66 = 6∕11; 𝑥 = 2∕22 = 1∕11, 𝑦 = 6∕33 = 2∕11, 𝑧 = 3∕11. So, the column player's optimal strategy is 𝐶 = [1∕6 1∕3 1∕2] 𝑇 and the value of the game is 𝑒 = 11∕6 − 1 = 5∕6. The solution to the dual problem is 𝑠 = 12∕66 = 2∕11, 𝑡 = 18∕66 = 3∕11, 𝑢 = 6∕66 = 1∕11, so the row player's optimal strategy is 𝑅 = [1∕3 1∕2 1∕6]. 27. The first row is dominated by the last, so we can remove it. That is as far as we can reduce by dominance. Add 𝑘 = 3 to each entry and put 1s to the right and below: 5 2 1 0 1 4 5 3 4 1 . 3 5 6 6 1 1 1 1 1 0 This gives the following LP problem: Maximize 𝑝 = 𝑥 + 𝑦 + 𝑧 + 𝑤 subject to 5𝑥 + 2𝑦 + 𝑧 ≤ 1 4𝑥 + 5𝑦 + 3𝑧 + 4𝑤 ≤ 1 3𝑥 + 5𝑦 + 6𝑧 + 6𝑤 ≤ 1 𝑥 ≥ 0, 𝑦 ≥ 0, 𝑧 ≥ 0, 𝑤 ≥ 0. Here are the tableaux:


Solutions Section 6.5 𝑥

𝑦

𝑧

𝑤

𝑠

𝑡

𝑢

𝑝

𝑠

5

2

1

0

1

0

0

0

1

𝑡

4

5

3

4

0

1

0

0

1

5𝑅2 − 4𝑅1

𝑢

3

5

6

6

0

0

1

0

1

5𝑅3 − 3𝑅1

𝑝

−1

−1

−1

−1

0

0

0

1

0

5𝑅4 + 𝑅1

𝑥

𝑦

𝑧

𝑤

𝑠

𝑡

𝑢

𝑝

𝑥

5

2

1

0

1

0

0

0

1

𝑡

0

17

11

20

−4

5

0

0

1

𝑢

0

19

27

30

−3

0

5

0

2

2𝑅3 − 3𝑅2

𝑝

0

−3

−4

−5

1

0

0

5

1

4𝑅4 + 𝑅2

𝑥

𝑦

𝑧

𝑤

𝑠

𝑡

𝑢

𝑝

𝑥

5

2

1

0

1

0

0

0

1

21𝑅1 − 𝑅3

𝑤

0

17

11

20

−4

5

0

0

1

21𝑅2 − 11𝑅3

𝑢

0

−13

21

0

6

−15

10

0

1

𝑝

0

5

−5

0

0

5

0

20

5

𝑥

𝑦

𝑧

𝑤

𝑠

𝑡

𝑢

𝑝

𝑥 𝑤 𝑧

105 0 0

55 500 −13

0 0 21

0 420 0

15 −150 6

15 270 −15

−10 −110 10

0 0 0

20 10 1

𝑝

0

40

0

0

30

30

50

420

110

21𝑅4 + 5𝑅3

The solution to the primal problem is 𝑝 = 110∕420 = 11∕42; 𝑥 = 20∕105 = 4∕21, 𝑦 = 0, 𝑧 = 1∕21, 𝑤 = 10∕420 = 1∕42. So, the column player's optimal strategy is 𝐶 = [8∕11 0 2∕11 1∕11] 𝑇 and the value of the game is 𝑒 = 42∕11 − 3 = 9∕11. The solution to the dual problem is 𝑠 = 30∕420 = 1∕14, 𝑡 = 30∕420 = 1∕14, 𝑢 = 50∕420 = 5∕42, so the row player's optimal strategy is 𝑅 = [0 3∕11 3∕11 5∕11]. (Note that we put in 0 for the first row that we removed at the beginning.) 28. The first column is dominated by the last, so we can remove it. That is as far as we can reduce by dominance. Add 𝑘 = 2 to each entry and put 1s to the right and below: 1 4 2 1 4 2 3 1 3 3 2 1 . 2 0 4 1 1 1 1 0 This gives the following LP problem: Maximize 𝑝 = 𝑥 + 𝑦 + 𝑧 subject to 𝑥 + 4𝑦 + 2𝑧 ≤ 1


Solutions Section 6.5 4𝑥 + 2𝑦 + 3𝑧 ≤ 1 3𝑥 + 3𝑦 + 2𝑧 ≤ 1 2𝑥 + 4𝑧 ≤ 1 𝑥 ≥ 0, 𝑦 ≥ 0, 𝑧 ≥ 0. Here are the tableaux: 𝑥 𝑦 𝑧

𝑠

𝑡

𝑢

𝑣

𝑝

𝑠

1

4

2

1

0

0

0

0

1

𝑡

4

2

3

0

1

0

0

0

1

𝑢

3

3

2

0

0

1

0

0

1

4𝑅3 − 3𝑅2

𝑣

2

0

4

0

0

0

1

0

1

2𝑅4 − 𝑅2

𝑝

−1

−1

−1

0

0

0

0

1

0

4𝑅5 + 𝑅2

𝑥

𝑦

𝑧

𝑠

𝑡

𝑢

𝑣

𝑝

𝑠

0

14

5

4

−1

0

0

0

3

3𝑅1 − 7𝑅3

𝑥

4

2

3

0

1

0

0

0

1

3𝑅2 − 𝑅3

𝑢

0

6

−1

0

−3

4

0

0

1

𝑣

0

−2

5

0

−1

0

2

0

1

3𝑅4 + 𝑅3

𝑝

0

−2

−1

0

1

0

0

4

1

3𝑅5 + 𝑅3

𝑥

𝑦

𝑧

𝑠

𝑡

𝑢

𝑣

𝑝

𝑠

0

0

22

12

18

−28

0

0

2

𝑥

12

0

10

0

6

−4

0

0

2

11𝑅2 − 5𝑅1

𝑦

0

6

−1

0

−3

4

0

0

1

22𝑅3 + 𝑅1

𝑣

0

0

14

0

−6

4

6

0

4

11𝑅4 − 7𝑅1

𝑝

0

0

−4

0

0

4

0

12

4

11𝑅5 + 2𝑅1

𝑥

𝑦

𝑧

𝑠

𝑡

𝑢

𝑣

𝑝

𝑧

0

0

22

12

18

−28

0

0

2

24𝑅1 + 7𝑅2

𝑥

132

0

0

−60

−24

96

0

0

12

𝑦

0

132

0

12

−48

60

0

0

24

8𝑅3 − 5𝑅2

𝑣

0

0

0

−84

−192

240

66

0

30

2𝑅4 − 5𝑅2

𝑝

0

0

0

24

36

−12

0

132

48

8𝑅5 + 𝑅2

4𝑅1 − 𝑅2


Solutions Section 6.5 𝑥

𝑦

𝑧

𝑧 𝑢 𝑦 𝑣

924 132 −660 −660

0 0 1056 0

528 0 0 0

𝑝

132

0

0

𝑠

𝑡

𝑢

𝑣

𝑝

−132 264 −60 −24 396 −264 132 −264

0 96 0 0

0 0 0 132

0 0 0 0

132 12 132 0

132

0

0

1056

396

264

The solution to the primal problem is 𝑝 = 396∕1,056 = 3∕8; 𝑥 = 0, 𝑦 = 132∕1,056 = 1∕8, 𝑧 = 132∕528 = 1∕4. So, the column player's optimal strategy is 𝐶 = [0 0 1∕3 2∕3] 𝑇 and the value of the game is 𝑒 = 8∕3 − 2 = 2∕3. (Note that we put in 0 for the first column that we removed at the beginning.) A solution to the dual problem is 𝑠 = 132∕1,056 = 1∕8, 𝑡 = 264∕1,056 = 1∕4, 𝑢 = 0, 𝑣 = 0, so the row player's optimal strategy is 𝑅 = [1∕3 2∕3 0 0]. Note We obtain a second solution to the dual problem by choosing the pivot from the fourth row in Tableau 4 (instead of from the second row): 𝑠 = 3∕20, 𝑡 = 1∕5, 𝑢 = 0, 𝑣 = 1∕40, giving an alternative optimal row strategy of [2∕5 8∕15 0 1∕15]. (There are actually infinitely many possible row strategies, obtained by taking combinations of these two.) 29. Unknowns: 𝑠 = # ounces of fish, 𝑡 = # ounces of cornmeal Minimize 𝑐 = 5𝑠 + 5𝑡 subject to 8𝑡 + 4𝑡 ≥ 48, 4𝑠 + 8𝑡 ≥ 48, 𝑠 ≥ 0, 𝑡 ≥ 0. 8 4 48 8 4 5 Dualize: 4 8 48 → 4 8 5 . 5 5 0 48 48 0 Dual problem: Maximize 𝑝 = 48𝑥 + 48𝑦 subject to 8𝑥 + 4𝑦 ≤ 5, 4𝑥 + 8𝑦 ≤ 5, 𝑥 ≥ 0, 𝑦 ≥ 0. Solve the dual problem: 𝑥 𝑦 𝑠 𝑡 𝑝 𝑠

8

4

1

0

0

5

𝑡

4

8

0

1

0

5

2𝑅2 − 𝑅1

𝑝

−48

−48

0

0

1

0

𝑅3 + 6𝑅1

𝑥

𝑦

𝑠

𝑡

𝑝

𝑥

8

4

1

0

0

5

3𝑅1 − 𝑅2

𝑡

0

12

−1

2

0

5

𝑝

0

−24

6

0

1

30

𝑥

𝑦

𝑠

𝑡

𝑝

𝑥 𝑦

24 0

0 12

4 −1

−2 2

0 0

10 5

𝑝

0

0

4

4

1

40

𝑅3 + 2𝑅2

The solution to the primal problem is 𝑐 = 40∕1 = 40, 𝑠 = 4∕1 = 4, 𝑡 = 4∕1 = 4. Meow should use 4 ounces each of fish and cornmeal, for a total cost of 40¢ per can. The shadow costs are the values of 𝑥 and 𝑦 in the final tableau: 𝑥 = 10∕24 = 5∕12¢ per gram of protein, 𝑦 = 5∕12¢ per gram of fat.


Solutions Section 6.5 30. Unknowns: 𝑠 = # pounds of giraffe meat, 𝑡 = # pounds gazelle meat Minimize 𝑐 = 2𝑠 + 4𝑡 subject to 18𝑠 + 36𝑡 ≥ 36,000, 36𝑠 + 18𝑡 ≥ 54,000, 𝑠 ≥ 0, 𝑡 ≥ 0. 18 36 36,000 18 36 2 Dualize: 36 18 54,000 → 36 18 4 .

2 4 0 36,000 54,000 0 Dual problem: Maximize 𝑝 = 36,000𝑥 + 54,000𝑦 subject to 18𝑥 + 36𝑦 ≤ 2, 36𝑥 + 18𝑦 ≤ 4, 𝑥 ≥ 0, 𝑦 ≥ 0. Solve the dual problem: 𝑥 𝑦 𝑠 𝑡 𝑝 𝑠

18

36

1

0

0

2

𝑡

36

18

0

1

0

4

2𝑅2 − 𝑅1

0

0

1

0

𝑅3 + 1500𝑅1

𝑝 −36000 −54000 𝑥

𝑦

𝑠

𝑡

𝑝

𝑦

18

36

1

0

0

2

𝑡

54

0

−1

2

0

6

𝑅2 − 3𝑅1

𝑝 −9000

0

1500

0

1

3000

𝑅3 + 500𝑅1

𝑥

𝑦

𝑠

𝑡

𝑝

𝑥 𝑡

18 0

36 −108

1 −4

0 2

0 0

2 0

𝑝

0

18000 2000

0

1

4000

The solution to the primal problem is 𝑐 = 4,000∕1 = 4,000, 𝑠 = 2,000∕1 = 2,000, 𝑡 = 0. 2,000 pounds of giraffe meat and no gazelle meat for a total cost of $4,000; The shadow costs are the values of 𝑥 and 𝑦 in the final tableau: 𝑥 = 2∕18 = 11 1 ⁄ 9 ¢ per gram of protein, 𝑦 = 0¢ per gram of fat. Note. Another solution, obtained by pivoting on the 54 in the second tableau rather than on the 18, leads to another solution to the primal problem: 𝑐 = 24,000∕6 = 4,000, 𝑠 = 8,000∕6 = 4,000∕3, 𝑡 = 2,000∕6 = 1,000∕3. Use 1,333 1 ⁄ 3 pounds of giraffe meat and 333 1 ⁄ 3 pounds of gazelle meat for a total cost of $4,000; 11 1 ⁄ 9 ¢ per gram of protein, 0¢ per gram of fat. 31. Unknowns: 𝑠 = # ounces of chicken, 𝑡 = # ounces of grain Minimize 𝑐 = 10𝑠 + 𝑡 subject to 10𝑠 + 2𝑡 ≥ 200, 5𝑠 + 2𝑡 ≥ 150, 𝑠 ≥ 0, 𝑡 ≥ 0. 10 2 200 10 5 10 Dualize: 5 2 150 → 2 2 1 10 1 0 200 150 0 Dual problem: Maximize 𝑝 = 200𝑥 + 150𝑦 subject to 10𝑥 + 5𝑦 ≤ 10, 2𝑥 + 2𝑦 ≤ 1, 𝑥 ≥ 0, 𝑦 ≥ 0. Solve the dual problem: 𝑥 𝑦 𝑠 𝑡 𝑝 𝑠

10

5

1

0

0

10

𝑡

2

2

0

1

0

1

0

0

1

0

𝑝

−200 −150

𝑅1 − 5𝑅2 𝑅3 + 100𝑅2


Solutions Section 6.5 𝑥

𝑦

𝑠

𝑡

𝑝

𝑠 𝑥

0 2

−5 2

1 0

−5 1

0 0

5 1

𝑝

0

50

0

100

1

100

The solution to the primal problem is 𝑐 = 100∕1 = 100; 𝑠 = 0, 𝑡 = 100∕1 = 100. Use no chicken and 100 oz of grain for a total cost of $1. The shadow costs are the values of 𝑥 and 𝑦 in the final tableau: 𝑥 = 1∕2¢ per gram of protein, 𝑦 = 0¢ per gram of fat. 32. Unknowns: 𝑠 = # Pomegranates, 𝑡 = # iZacs Minimize 𝑐 = 2𝑠 + 2𝑡 (in thousands of dollars) subject to 4𝑠 + 3𝑡 ≥ 480 (hundreds of GB), 8𝑠 + 10𝑡 ≥ 1,280 (tens of TB), 𝑠 ≥ 0, 𝑡 ≥ 0. 4 3 480 4 8 2 Dualize: 8 10 1,280 → 3 10 2 . 2 2 0 480 1,280 0 Dual problem: Maximize 𝑝 = 480𝑥 + 1 280𝑦 subject to 4𝑥 + 8𝑦 ≤ 2, 3𝑥 + 10𝑦 ≤ 2, 𝑥 ≥ 0, 𝑦 ≥ 0. Solve the dual problem: 𝑥 𝑦 𝑠 𝑡 𝑝 𝑠

4

8

1

0

0

2

𝑡

3

10

0

1

0

2

0

0

1

0

𝑝

−480 −1280

5𝑅1 − 4𝑅2 𝑅3 + 128𝑅2

𝑥

𝑦

𝑠

𝑡

𝑝

𝑠

8

0

5

−4

0

2

𝑦

3

10

0

1

0

2

8𝑅2 − 3𝑅1

𝑝

−96

0

0

128

1

256

𝑅3 + 12𝑅1

𝑥

𝑦

𝑠

𝑡

𝑝

𝑥 𝑦

8 0

0 80

5 −15

−4 20

0 0

2 10

𝑝

0

0

60

80

1

280

The solution to the primal problem is 𝑐 = 280∕1 = 280; 𝑠 = 60∕1 = 60, 𝑡 = 80∕1 = 80. The school should buy 60 Pomegranates and 80 iZacs, for a total cost of $280,000. The shadow costs are the values of 𝑥 and 𝑦 in the final tableau: 𝑥 = 1∕4 thousand dollars per 100 GB of memory, or $2.50/GB of memory and 𝑦 = 5∕40 = 0.125 thousand dollars per 10 TB of disk space, or $12.50/TB of disk space. 33. Unknowns: 𝑠 = # mailings to the East Coast, 𝑡 = # mailings to the Midwest, 𝑢 = # mailings to the West Coast Minimize 𝑐 = 40𝑠 + 60𝑡 + 50𝑢 subject to 100𝑠 + 100𝑡 + 50𝑢 ≥ 1,500, 50𝑠 + 100𝑡 + 100𝑢 ≥ 1,500, 𝑠 ≥ 0, 𝑡 ≥ 0, 𝑢 ≥ 0.


Solutions Section 6.5 100 50 40 100 100 60 . 50 100 50 1,500 1,500 0 Dual problem: Maximize 𝑝 = 1,500𝑥 + 1,500𝑦 subject to 100𝑥 + 50𝑦 ≤ 40; 100𝑥 + 100𝑦 ≤ 60, 50𝑥 + 100𝑦 ≤ 50, 𝑥 ≥ 0, 𝑦 ≥ 0. Solve the dual problem: 𝑥 𝑦 𝑠 𝑡 𝑢 𝑝 100 100 50 1,500 Dualize: 50 100 100 1,500 → 40 60 50 0

𝑠

100

50

1

0

0

0

40

𝑡

100

100

0

1

0

0

60

𝑅2 − 𝑅1

𝑢

50

100

0

0

1

0

50

2𝑅3 − 𝑅1

𝑝 −1500 −1500

0

0

0

1

0

𝑅4 + 15𝑅1

𝑅1 − 𝑅2

𝑥

𝑦

𝑠

𝑡

𝑢

𝑝

𝑥

100

50

1

0

0

0

40

𝑡

0

50

−1

1

0

0

20

𝑢

0

150

−1

0

2

0

60

𝑅3 − 3𝑅2

𝑝

0

−750

15

0

0

1

600

𝑅4 + 15𝑅2

𝑥

𝑦

𝑠

𝑡

𝑢

𝑝

𝑥 𝑦 𝑢

100 0 0

0 50 0

2 −1 2

−1 1 −3

0 0 2

0 0 0

20 20 0

𝑝

0

0

0

15

0

1

900

The solution to the primal problem is 𝑐 = 900∕1 = 900; 𝑠 = 0, 𝑡 = 15∕1 = 15, 𝑢 = 0. Send 15 mailings to the Midwest and none to the East or West Coasts. Cost: $900. The shadow costs are the values of 𝑥 and 𝑦 in the final tableau: 𝑥 = 20∕100 = 1∕5 = 20¢ per Democrat and 𝑦 = 20∕50 = 2∕5 = 40¢ per Republican. Another solution comes from pivoting on the 150 in the 𝑦-column in the second tableau instead of the 50: 𝑠 = 10, 𝑡 = 0, 𝑢 = 10. Send 10 mailings to the East Coast, none to the Midwest, 10 to the West Coast. 34. Unknowns: 𝑠 = # packages from Harvard Paper, 𝑡 = # packages from Yale Paper, 𝑢 = # packages from Dartmouth Paper Minimize 𝑐 = 60𝑠 + 40𝑡 + 50𝑢 subject to 20𝑠 + 10𝑡 + 10𝑢 ≥ 350, 10𝑠 + 10𝑡 + 20𝑢 ≥ 400, 𝑠 ≥ 0, 𝑡 ≥ 0, 𝑢 ≥ 0. 20 10 60 20 10 10 350 10 10 40 Dualize: 10 10 20 400 → . 10 20 50 60 40 50 0 350 400 0 Dual problem: Maximize 𝑝 = 350𝑥 + 400𝑦 subject to 20𝑥 + 10𝑦 ≤ 60, 10𝑥 + 10𝑦 ≤ 40, 10𝑥 + 20𝑦 ≤ 50, 𝑥 ≥ 0, 𝑦 ≥ 0, 𝑧 ≥ 0. Solve the dual problem:


Solutions Section 6.5 𝑥

𝑦

𝑠

𝑡

𝑢

𝑝

𝑠

20

10

1

0

0

0

60

2𝑅1 − 𝑅3

𝑡

10

10

0

1

0

0

40

2𝑅2 − 𝑅3

𝑢

10

20

0

0

1

0

50

𝑝

−350 −400

0

0

0

1

0

𝑅4 + 20𝑅3

𝑥

𝑦

𝑠

𝑡

𝑢

𝑝

𝑠

30

0

2

0

−1

0

70

𝑡

10

0

0

2

−1

0

30

3𝑅2 − 𝑅1

𝑦

10

20

0

0

1

0

50

3𝑅3 − 𝑅1

𝑝

−150

0

0

0

20

1

1000

𝑅4 + 5𝑅1

𝑥

𝑦

𝑠

𝑡

𝑢

𝑝

𝑥 𝑡 𝑦

30 0 0

0 0 60

2 −2 −2

0 6 0

−1 −2 4

0 0 0

70 20 80

𝑝

0

0

10

0

15

1

1350

The solution to the primal problem is 𝑐 = 1,350∕1 = 1,350; 𝑠 = 10∕1 = 10, 𝑡 = 0, 𝑢 = 15∕1 = 15. Buy 10 packages from Harvard, 15 from Dartmouth, none from Yale. Cost: $1,350. The shadow costs are the values of 𝑥 and 𝑦 in the final tableau: 𝑥 = 70∕30 = 7∕3 = $2.33 per ream of white paper, 𝑦 = 80∕60 = 4∕3 = $1.33 per ream of yellow paper. 35. Unknowns: 𝑠 = # full-page ads in Sports Illustrated, 𝑡 = # full-page ads in GQ. Expressing costs and readers in thousands, the primal problem is: Minimize 𝑐 = 400𝑠 + 300𝑡 subject to 900𝑠 + 500𝑡 ≥ 11,000, 𝑠 ≥ 4, 𝑡 ≥ 4, 𝑠 ≥ 0, 𝑡 ≥ 0. 900 500 11,000 900 1 0 400 1 0 4 Dualize: → 500 0 1 300 . 0 1 4 11,000 4 4 0 400 300 0 Dual problem: Maximize 𝑝 = 11,000𝑥 + 4𝑦 + 4𝑧 subject to 900𝑥 + 𝑦 ≤ 400; 500𝑥 + 𝑧 ≤ 300, 𝑥 ≥ 0, 𝑦 ≥ 0, 𝑧 ≥ 0. Solve the dual problem: 𝑥 𝑦 𝑧 𝑠 𝑡 𝑝 𝑠

900

1

0

1

0

0

400

𝑡

500

0

1

0

1

0

300

9𝑅2 − 5𝑅1

𝑝 −11000

−4

−4

0

0

1

0

9𝑅3 + 110𝑅1


Solutions Section 6.5 𝑥

𝑦

𝑧

𝑠

𝑡

𝑝

𝑥

900

1

0

1

0

0

400

𝑡

0

−5

9

−5

9

0

700

𝑝

0

74

−36

110

0

9

44000

𝑥

𝑦

𝑧

𝑠

𝑡

𝑝

𝑥 𝑧

900 0

1 −5

0 9

1 −5

0 9

0 0

400 700

𝑝

0

54

0

90

36

9

46800

𝑅3 + 4𝑅2

The solution to the primal problem is 𝑐 = 46,800∕9 = 5,200; 𝑠 = 90∕9 = 10, 𝑡 = 36∕9 = 4. Place 10 ads in Sports Illustrated and 4 in GQ. Cost: $5,200,000. The shadow cost per reader is the value of 𝑥 in the final tableau: 𝑥 = 400∕900 ≈ $0.44 per reader. 36. Unknowns: 𝑠 = # full-page ads in Sports Illustrated, 𝑡 = # full-page ads in Motor Trend. Expressing costs and readers in thousands, the primal problem is: Minimize 𝑐 = 400𝑠 + 100𝑡 subject to 900𝑠 + 300𝑡 ≥ 7,500, 𝑠 ≥ 5, 𝑡 ≥ 5, 𝑠 ≥ 0, 𝑡 ≥ 0. 900 300 7,500 900 1 0 400 1 0 5 Dualize: → 300 0 1 100 . 0 1 5 7,500 5 5 0 400 100 0 Dual problem: Maximize 𝑝 = 7,500𝑥 + 5𝑦 + 5𝑧 subject to 900𝑥 + 𝑦 ≤ 400; 300𝑥 + 𝑧 ≤ 100, 𝑥 ≥ 0, 𝑦 ≥ 0, 𝑧 ≥ 0. Solve the dual problem: 𝑥 𝑦 𝑧 𝑠 𝑡 𝑝 𝑠

900

1

0

1

0

0

400

𝑡

300

0

1

0

1

0

100

𝑝 −7500

−5

−5

0

0

1

0

𝑥

𝑦

𝑧

𝑠

𝑡

𝑝

𝑠

0

1

−3

1

−3

0

100

𝑥

300

0

1

0

1

0

100

𝑝

0

−5

20

0

25

1

2500

𝑥

𝑦

𝑧

𝑠

𝑡

𝑝

𝑦 𝑥

0 300

1 0

−3 1

1 0

−3 1

0 0

100 100

𝑝

0

0

5

5

10

1

3000

𝑅1 − 3𝑅2 𝑅3 + 25𝑅2

𝑅3 + 5𝑅1

The solution to the primal problem is 𝑐 = 3,000∕1 = 3,000; 𝑠 = 5∕1 = 5, 𝑡 = 10∕1 = 10. Place 5 ads in Sports Illustrated and 10 in Motor Trend. Cost: $3,000,000. The shadow cost per reader is the value of 𝑥 in


Solutions Section 6.5 the final tableau: 𝑥 = 100∕300 ≈ $0.33 per reader. 37. Unknowns: 𝑠 = # sleep spells, 𝑡 = # shock spells Minimize 𝑐 = 50𝑠 + 20𝑡 subject to 3𝑠 + 𝑡 ≥ 48, 3𝑠 + 2𝑡 ≥ 52, −𝑠 + 𝑡 ≥ 0, 𝑠 ≥ 0, 𝑡 ≥ 0. 3 1 48 3 3 −1 50 3 2 52 Dualize: → 1 2 1 20 . −1 1 0 48 52 0 0 50 20 0 Dual problem: Maximize 𝑝 = 48𝑥 + 52𝑦 subject to 3𝑥 + 3𝑦 − 𝑧 ≤ 50, 𝑥 + 2𝑦 + 𝑧 ≤ 20, 𝑥 ≥ 0, 𝑦 ≥ 0, 𝑧 ≥ 0. Solve the dual problem: 𝑥 𝑦 𝑧 𝑠 𝑡 𝑝 2𝑅1 − 3𝑅2

𝑠

3

3

−1

1

0

0

50

𝑡

1

2

1

0

1

0

20

𝑝

−48

−52

0

0

0

1

0

𝑥

𝑦

𝑧

𝑠

𝑡

𝑝

𝑠

3

0

−5

2

−3

0

40

𝑦

1

2

1

0

1

0

20

3𝑅2 − 𝑅1

𝑝

−22

0

26

0

26

1

520

3𝑅3 + 22𝑅1

𝑥

𝑦

𝑧

𝑠

𝑡

𝑝

𝑥

3

0

−5

2

−3

0

40

8𝑅1 + 5𝑅2

𝑦

0

6

8

−2

6

0

20

𝑝

0

0

−32

44

12

3

2440

𝑥

𝑦

𝑧

𝑠

𝑡

𝑝

𝑥 𝑧

24 0

30 6

0 8

6 −2

6 6

0 0

420 20

𝑝

0

24

0

36

36

3

2520

𝑅3 + 26𝑅2

𝑅3 + 4𝑅2

The solution to the primal problem is 𝑐 = 2,520∕3 = 840; 𝑠 = 36∕3 = 12, 𝑡 = 36∕3 = 12. Gillian should use 12 sleep spells and 12 shock spells, costing 840 therms of energy. 38. Unknowns: 𝑠 = # gold payoffs, 𝑡 = # political favors per year Minimize 𝑐 = 4𝑠 + 2𝑡 subject to 2𝑠 + 3𝑡 ≥ 24, −3𝑠 + 𝑡 ≥ 0, 𝑠 ≥ 0, 𝑡 ≥ 0. 2 3 24 2 −3 4 Dualize: −3 1 0 → 3 1 2 . 4 2 0 24 0 0 Dual problem: Maximize 𝑝 = 24𝑥 subject to 2𝑥 − 3𝑦 ≤ 4, 3𝑥 + 𝑦 ≤ 2, 𝑥 ≥ 0, 𝑦 ≥ 0. Solve the dual problem:


Solutions Section 6.5 𝑥

𝑦

𝑠

𝑡

𝑝

𝑠

2

−3

1

0

0

4

𝑡

3

1

0

1

0

2

𝑝

−24

0

0

0

1

0

𝑥

𝑦

𝑠

𝑡

𝑝

𝑠 𝑥

0 3

−11 1

3 0

−2 1

0 0

8 2

𝑝

0

8

0

8

1

16

3𝑅1 − 2𝑅2 𝑅3 + 8𝑅2

The solution to the primal problem is 𝑐 = 16∕1 = 16; 𝑠 = 0, 𝑡 = 8. He should make no gold payoffs and 8 political favors per year, costing him a total of 16 trips. 39. We observe that the first column dominates the last, so we can eliminate the last. In the matrix that remains, the second row dominates the third, so we eliminate the third. We cannot reduce any further. Add 𝑘 = 500 to each entry and put 1s to the right and below: 300 200 1 0 1,000 1 1 1 0 This gives the following LP problem: Maximize 𝑝 = 𝑥 + 𝑦 subject to 300𝑥 + 200𝑦 ≤ 1, 1,000𝑦 ≤ 1, 𝑥 ≥ 0, 𝑦 ≥ 0. Here are the tableaux: 𝑥 𝑦 𝑠 𝑡 𝑝 𝑠

300

200

1

0

0

1

𝑡

0

1000

0

1

0

1

𝑝

−1

−1

0

0

1

0

300𝑅3 + 𝑅1

𝑥

𝑦

𝑠

𝑡

𝑝

𝑥

300

200

1

0

0

1

5𝑅1 − 𝑅2

𝑡

0

1000

0

1

0

1

𝑝

0

−100

1

0

300

1

𝑥

𝑦

𝑠

𝑡

𝑝

𝑥 𝑦

1500 0

0 1000

5 0

−1 1

0 0

4 1

𝑝

0

0

10

1

3000

11

10𝑅3 + 𝑅2

The solution to the primal problem is 𝑝 = 11∕3,000; 𝑥 = 4∕1,500 = 1∕375, 𝑦 = 1∕1,000. So, T. N. Spend's optimal strategy is 𝐶 = [8∕11 3∕11 0] 𝑇 and the value of the game is 𝑒 = 3,000∕11 − 500 = −2,500∕11 ≈ −227.


Solutions Section 6.5 The solution to the dual problem is 𝑠 = 10∕3,000, 𝑡 = 1∕3,000, so T. L. Down's optimal strategy is 𝑅 = [10∕11 1∕11 0]. T. N. Spend should spend about 73% of the days in Littleville, 27% in Metropolis, and skip Urbantown. T. L. Down should spend about 91% of the days in Littleville, 9% in Metropolis, and skip Urbantown. The expected outcome is that T. L. Down will lose about 227 votes per day of campaigning. 40. The payoff matrix is −1,000 1,500 𝑃= 500 −750 with the first row or column representing New York and the second Boston. This game does not reduce. Add 𝑘 = 1,000 to each entry and put 1s to the right and below: 0 2,500 1 1,500 250 1 . 1 1 0 This gives the following LP problem: Maximize 𝑝 = 𝑥 + 𝑦 subject to 2,500𝑦 ≤ 1, 1,500𝑥 + 250𝑦 ≤ 1, 𝑥 ≥ 0, 𝑦 ≥ 0. Here are the tableaux: 𝑥 𝑦 𝑠 𝑡 𝑝 𝑠

0

2500

1

0

0

1

𝑡

1500

250

0

1

0

1

𝑝

−1

−1

0

0

1

0

𝑥

𝑦

𝑠

𝑡

𝑝

𝑠

0

2500

1

0

0

1

𝑥

1500

250

0

1

0

1

10𝑅2 − 𝑅1

𝑝

0

−1250

0

1

1500

1

2𝑅3 + 𝑅1

𝑥

𝑦

𝑠

𝑡

𝑝

𝑦 0 2500 𝑥 15000 0

1 −1

0 10

0 0

1 9

𝑝

1

2

3000

3

0

0

1500𝑅3 + 𝑅2

The solution to the primal problem is 𝑝 = 3∕3,000 = 1∕1,000; 𝑥 = 9∕15,000 = 3∕5,000, 𝑦 = 1∕2,500. So, the iNod's maker's optimal strategy is 𝐶 = [3∕5 2∕5] 𝑇 and the value of the game is 𝑒 = 1,000 − 1,000 = 0. The solution to the dual problem is 𝑠 = 1∕3,000, 𝑡 = 2∕3,000 = 1∕1,500, so your optimal strategy is 𝑅 = [1∕3 2∕3]. You should spend 1/3 of your time in New York and 2/3 in Boston. The iNod's maker should spend 3/5 of its time in New York and 2/5 in Boston. The expected outcome is no net gain or loss of sales. 41. With A the row player and B the column player, the payoff matrix is 2 −1 0 𝑃 = −1 4 −1 .

0 −1 6 This game does not reduce. Add 𝑘 = 1 to each entry and put 1s to the right and below:


Solutions Section 6.5 3 0 1 1 0 5 0 1 . 1 0 7 1 1 1 1 0 This gives the following LP problem: Maximize 𝑝 = 𝑥 + 𝑦 + 𝑧 subject to 3𝑥 + 𝑧 ≤ 1, 5𝑦 ≤ 1, 𝑥 + 7𝑧 ≤ 1, 𝑥 ≥ 0, 𝑦 ≥ 0, 𝑧 ≥ 0. Here are the tableaus: 𝑥 𝑦 𝑧 𝑠 𝑡 𝑢 𝑝 𝑠

3

0

1

1

0

0

0

1

𝑡

0

5

0

0

1

0

0

1

𝑢

1

0

7

0

0

1

0

1

3𝑅3 − 𝑅1

𝑝

−1

−1

−1

0

0

0

1

0

3𝑅4 + 𝑅1

𝑥

𝑦

𝑧

𝑠

𝑡

𝑢

𝑝

𝑥

3

0

1

1

0

0

0

1

𝑡

0

5

0

0

1

0

0

1

𝑢

0

0

20

−1

0

3

0

2

𝑝

0

−3

−2

1

0

0

3

1

5𝑅4 + 3𝑅2

𝑥

𝑦

𝑧

𝑠

𝑡

𝑢

𝑝

𝑥

3

0

1

1

0

0

0

1

20𝑅1 − 𝑅3

𝑦

0

5

0

0

1

0

0

1

𝑢

0

0

20

−1

0

3

0

2

𝑝

0

0

−10

5

3

0

15

8

𝑥

𝑦

𝑧

𝑠

𝑡

𝑢

𝑝

𝑥 𝑦 𝑧

60 0 0

0 5 0

0 0 20

21 0 −1

0 1 0

−3 0 3

0 0 0

18 1 2

𝑝

0

0

0

9

6

3

30

18

2𝑅4 + 𝑅3

The solution to the primal problem is 𝑝 = 18∕30 = 3∕5; 𝑥 = 18∕60 = 3∕10, 𝑦 = 1∕5, 𝑧 = 2∕20 = 1∕10. So, player B's optimal strategy is 𝐶 = [1∕2 1∕3 1∕6] 𝑇 and the value of the game is 𝑒 = 5∕3 − 1 = 2∕3. The solution to the dual problem is 𝑠 = 9∕30 = 3∕10, 𝑡 = 6∕30 = 1∕5, 𝑢 = 3∕30 = 1∕10, so player A's optimal strategy is 𝑅 = [1∕2 1∕3 1∕6]. Each player should show one finger with probability 1/2, two fingers with probability 1/3, and three fingers with probability 1/6. The expected outcome is that player A will win 2/3 point per round, on average.


Solutions Section 6.5 42. With A the row player and B the column player, the payoff matrix is −2 3 −4 𝑃 = 3 −4 5 .

−4 5 −6 This game does not reduce. Add 𝑘 = 6 to each entry and put 1s to the right and below: 4 9 2 1 9 2 11 1 . 2 11 0 1 1 1 1 0 This gives the following LP problem: Maximize 𝑝 = 𝑥 + 𝑦 + 𝑧 subject to 4𝑥 + 9𝑦 + 2𝑧 ≤ 1, 9𝑥 + 2𝑦 + 11𝑧 ≤ 1, 2𝑥 + 11𝑦 ≤ 1, 𝑥 ≥ 0, 𝑦 ≥ 0, 𝑧 ≥ 0. Here are the tableaus: 𝑥 𝑦 𝑧 𝑠 𝑡 𝑢 𝑝 9𝑅1 − 4𝑅2

𝑠

4

9

2

1

0

0

0

1

𝑡

9

2

11

0

1

0

0

1

𝑢

2

11

0

0

0

1

0

1

9𝑅3 − 2𝑅2

𝑝

−1

−1

−1

0

0

0

1

0

9𝑅4 + 𝑅2

𝑥

𝑦

𝑧

𝑠

𝑡

𝑢

𝑝

𝑠

0

73

−26

9

−4

0

0

5

𝑥

9

2

11

0

1

0

0

1

73𝑅2 − 2𝑅1

𝑢

0

95

−22

0

−2

9

0

7

73𝑅3 − 95𝑅1

𝑝

0

−7

2

0

1

0

9

1

73𝑅4 + 7𝑅1

𝑥

𝑦

𝑧

𝑠

𝑡

𝑢

𝑝

𝑦

0

73

−26

9

−4

0

0

5

432𝑅1 + 13𝑅3

𝑥

657

0

855

−18

81

0

0

63

96𝑅2 − 95𝑅3

𝑢

0

0

864

−855

234

657

0

36

𝑝

0

0

−36

63

45

0

657

108

𝑥

𝑦

𝑧

𝑠

𝑡

𝑢

𝑦 0 31536 𝑥 63072 0 𝑧 0 0

0 0 864

−7227 1314 8541 79497 −14454 −62415 657 −855 234

𝑝

0

657

0

0

1314

657

24𝑅4 + 𝑅3

𝑝 0 0 0

2628 2628 36

15768

2628

The solution to the primal problem is 𝑝 = 2,628∕15,768 = 1∕6; 𝑥 = 2,628∕63,072 = 1∕24, 𝑦 = 2,628∕31,536 = 1∕12, 𝑧 = 36∕864 = 1∕24. So, player B's optimal strategy is 𝐶 = [1∕4 1∕2 1∕4] 𝑇 and the value of the game is 𝑒 = 6 − 6 = 0. The solution to the dual problem is 𝑠 = 657∕15,768 = 1∕24, 𝑡 = 1,314∕15,768 = 1∕12,


Solutions Section 6.5 𝑢 = 657∕15,768 = 1∕24, so player A's optimal strategy is 𝑅 = [1∕4 1∕2 1∕4]. Each player should show one finger with probability 1/4, two fingers with probability 1/2, and three fingers with probability 1/4. The expected outcome is that the players will be even, in the long run. 43. Let Colonel Blotto be the row player and Captain Kije the column player. Label possible moves by the number of regiments sent to the first of the two locations; the remaining regiments are sent to the other location. The payoff matrix is then the following: Captain Kije

0 1 2 3 0 4 2 1 0 1 1 3 0 −1 Colonel Blotto 2 −2 2 2 −2 3 −1 0 3 1 0 1 2 4 4 We cannot reduce this game. Add 𝑘 = 2 to each entry and put 1s to the right and below: 6 4 3 2 1 3 5 2 1 1 0 4 4 0 1 . 1 2 5 3 1 2 3 4 6 1 1 1 1 1 0 This gives the following LP problem: Maximize 𝑝 = 𝑥 + 𝑦 + 𝑧 + 𝑤 subject to 6𝑥 + 4𝑦 + 3𝑧 + 2𝑤 ≤ 1 3𝑥 + 5𝑦 + 2𝑧 + 𝑤 ≤ 1 4𝑦 + 4𝑧 ≤ 1 𝑥 + 2𝑦 + 5𝑧 + 3𝑤 ≤ 1 2𝑥 + 3𝑦 + 4𝑧 + 6𝑤 ≤ 1 𝑥 ≥ 0, 𝑦 ≥ 0, 𝑧 ≥ 0, 𝑤 ≥ 0. Here are the tableaux: 𝑥

𝑦

𝑧

𝑤

𝑠

𝑡

𝑢

𝑣

𝑟

𝑝

𝑠

6

4

3

2

1

0

0

0

0

0

1

𝑡

3

5

2

1

0

1

0

0

0

0

1

𝑢

0

4

4

0

0

0

1

0

0

0

1

𝑣

1

2

5

3

0

0

0

1

0

0

1

6𝑅4 − 𝑅1

𝑟

2

3

4

6

0

0

0

0

1

0

1

3𝑅5 − 𝑅1

𝑝

−1

−1

−1

−1

0

0

0

0

0

1

0

6𝑅6 + 𝑅1

𝑥

𝑦

𝑧

𝑤

𝑠

𝑡

𝑢

𝑣

𝑟

𝑝

𝑥

6

4

3

2

1

0

0

0

0

0

1

8𝑅1 − 𝑅5

𝑡 𝑢

0 0

6 4

1 4

0 0

−1 0

2 0

0 1

0 0

0 0

0 0

1 1

𝑣

0

8

27

16

−1

0

0

6

0

0

5

𝑟

0

5

9

16

−1

0

0

0

3

0

2

𝑝

0

−2

−3

−4

1

0

0

0

0

6

1

2𝑅2 − 𝑅1

𝑅4 − 𝑅5 4𝑅6 + 𝑅5


Solutions Section 6.5 𝑥

𝑦

𝑧

𝑤

𝑠

𝑡

𝑢

𝑣

𝑟

𝑝

𝑥

48

27

15

0

9

0

0

0

−3

0

6

𝑡

0

6

1

0

−1

2

0

0

0

0

1

𝑢

0

4

4

0

0

0

1

0

0

0

1

3𝑅3 − 2𝑅2

𝑣

0

3

18

0

0

0

0

6

−3

0

3

2𝑅4 − 𝑅2

𝑤

0

5

9

16

−1

0

0

0

3

0

2

6𝑅5 − 5𝑅2

𝑝

0

−3

−3

0

3

0

0

0

3

24

6

2𝑅6 + 𝑅2

𝑥

𝑦

𝑧

𝑤

𝑠

𝑡

𝑢

𝑣

𝑟

𝑝

𝑥

96

0

21

0

27

−18

0

0

−6

0

3

10𝑅1 − 21𝑅3

𝑦

0

6

1

0

−1

2

0

0

0

0

1

10𝑅2 − 𝑅3

𝑢

0

0

10

0

2

−4

3

0

0

0

1

𝑣

0

0

35

0

1

−2

0

12

−6

0

5

2𝑅4 − 7𝑅3

𝑤

0

0

49

96

−1

−10

0

0

18

0

7

10𝑅5 − 49𝑅3

𝑝

0

0

−5

0

5

2

0

0

6

48

13

2𝑅6 + 𝑅3

𝑥

𝑦

𝑧

𝑤

𝑠

𝑡

𝑢

𝑣

𝑟

𝑝

𝑥 𝑦 𝑧 𝑣 𝑤

960 0 0 0 0

0 60 0 0 0

0 0 10 0 0

0 0 0 0 960

228 −12 2 −12 −108

−96 24 −4 24 96

−63 −3 3 −21 −147

0 0 0 24 0

−60 0 0 −12 180

0 0 0 0 0

9 9 1 3 21

𝑝

0

0

0

0

12

0

3

0

12

96

27

2𝑅1 − 9𝑅2

The solution to the primal problem is 𝑝 = 27∕96 = 9∕32; 𝑥 = 9∕960 = 3∕320, 𝑦 = 9∕60 = 3∕20, 𝑧 = 1∕10, 𝑤 = 21∕960 = 7∕320. So, Captain Kije's optimal strategy is 𝐶 = [1∕30 8∕15 16∕45 7∕90] 𝑇 and the value of the game is 𝑒 = 32∕9 − 2 = 14∕9. In fact, this is only one of several optimal strategies for Captain Kije; which you find depends on the choices of pivots you make in the simplex method. Other optimal strategies are the one given above, in reverse order, and [1∕18 4∕9 4∕9 1∕18] 𝑇 . The solution to the dual problem (which is unique) is 𝑠 = 12∕96 = 1∕8, 𝑡 = 0, 𝑢 = 3∕96 = 1∕32, 𝑣 = 0, 𝑟 = 12∕96 = 1∕8, so Colonel Blotto's optimal strategy is 𝑅 = [4∕9 0 1∕9 0 4∕9]. Write moves as (𝑥, 𝑦), where 𝑥 represents the number of regiments sent to the first location and 𝑦 represents the number sent to the second location. Colonel Blotto should play (0, 4) with probability 4∕9, (2, 2) with probability 1∕9, and (4, 0) with probability 4∕9. Captain Kije has several optimal strategies, one of which is to play (0, 3) with probability 1∕30, (1, 2) with probability 8∕15, (2, 1) with probability 16∕45, and (3, 0) with probability 7∕90. The expected outcome is that Colonel Blotto will win 14∕9 points on average.


Solutions Section 6.5 44. Let Colonel Blotto be the row player and Captain Kije the column player. Label possible moves by the number of regiments sent to the first of the two locations; the remaining regiments are sent to the other location. The payoff matrix is then the following: Captain Kije

0 1 2 3 0 4 1 0 −1 1 1 3 −1 −2 Colonel Blotto 2 −3 2 2 −3 3 −2 −1 3 1 −1 0 1 4 4 We cannot reduce this game. Add 𝑘 = 3 to each entry and put 1s to the right and below: 7 4 3 2 1 4 6 2 1 1 0 5 5 0 1 . 1 2 6 4 1 2 3 4 7 1 1 1 1 1 0 This gives the following LP problem: Maximize 𝑝 = 𝑥 + 𝑦 + 𝑧 + 𝑤 subject to 7𝑥 + 4𝑦 + 3𝑧 + 2𝑤 ≤ 1 4𝑥 + 6𝑦 + 2𝑧 + 𝑤 ≤ 1 5𝑦 + 5𝑧 ≤ 1 𝑥 + 2𝑦 + 6𝑧 + 4𝑤 ≤ 1 2𝑥 + 3𝑦 + 4𝑧 + 7𝑤 ≤ 1 𝑥 ≥ 0, 𝑦 ≥ 0, 𝑧 ≥ 0, 𝑤 ≥ 0. Here are the tableaus: 𝑥 𝑦 𝑧 𝑤 𝑠 𝑡 𝑢 𝑣 𝑟 𝑝 𝑠

7

4

3

2

1

0

0

0

0

0

1

𝑡 𝑢 𝑣 𝑟

4 0 1 2

6 5 2 3

2 5 6 4

1 0 4 7

0 0 0 0

1 0 0 0

0 1 0 0

0 0 1 0

0 0 0 1

0 0 0 0

1 1 1 1

𝑝

−1

−1

−1

−1

0

0

0

0

0

1

0

𝑥

𝑦

𝑧

𝑤

𝑠

𝑡

𝑢

𝑣

𝑟

𝑝

𝑥 𝑡 𝑢 𝑣

1 0 0 0

0.57 3.71 5 1.43

0.43 0.29 5 5.57

0.29 0.14 −0.14 −0.57 0 0 3.71 −0.14

0 1 0 0

0 0 1 0

0 0 0 1

0 0 0 0

0 0 0 0

0.14 0.43 1 0.86

𝑟

0

1.86

3.14

6.43

−0.29

0

0

0

1

0

0.71

𝑝

0

−0.43 −0.57 −0.71

0.14

0

0

0

0

1

0.14


Solutions Section 6.5 𝑥

𝑦

𝑧

𝑤

𝑠

𝑡

𝑢

𝑣

𝑟

𝑝

𝑥

1

0.49

0.29

0

0.16

0

0

0

−0.04

0

0.11

𝑡

0

3.76

0.36

0

−0.58

1

0

0

0.02

0

0.44

𝑢 𝑣 𝑤

0 0 0

5 0.36 0.29

5 3.76 0.49

0 0 1

0 0.02 −0.04

0 0 0

1 0 0

0 1 0

0 −0.58 0.16

0 0 0

1 0.44 0.11

𝑝

0

−0.22 −0.22

0

0.11

0

0

0

0.11

1

0.22

𝑠

𝑡

𝑢

𝑣

𝑟

𝑝

𝑥

𝑦

𝑧

𝑤

𝑥 𝑦

1 0

0 1

0.24 0.09

0 0

0.23 −0.13 −0.15 0.27

0 0

0 0

−0.05 0.01

0 0

0.05 0.12

𝑢

0

0

4.53

0

0.77

−1.33

1

0

−0.03

0

0.41

𝑣 𝑤

0 0

0 0

3.72 0.46

0 1

0.08 0

−0.09 −0.08

0 0

1 0

−0.58 0.15

0 0

0.4 0.08

𝑝

0

0

−0.2

0

0.08

0.06

0

0

0.11

1

0.25

𝑥

𝑦

𝑧

𝑤

𝑠

𝑡

𝑢

𝑣

𝑟

𝑝

𝑥 𝑦 𝑧 𝑣 𝑤

1 0 0 0 0

0 1 0 0 0

0 0 1 0 0

0 0 0 0 1

0.19 −0.06 −0.05 −0.17 0.29 −0.02 0.17 −0.29 0.22 1 −0.56 −0.82 −0.08 0.06 −0.1

0 0 0 1 0

−0.05 0.01 −0.01 −0.56 0.16

0 0 0 0 0

0.03 0.11 0.09 0.07 0.04

𝑝

0

0

0

0

0.11

0

0.11

1

0.27

0

0.04

The solution to the primal problem is 𝑝 ≈ 0.2667; 𝑥 ≈ 0.0314, 𝑦 ≈ 0.1098, 𝑧 ≈ 0.0902, 𝑤 ≈ 0.0353. So, Captain Kije's optimal strategy is 𝐶 = [0.12 0.41 0.34 0.13] 𝑇 and the value of the game is 𝑒 ≈ 1∕0.2667 − 3 ≈ 0.75. In fact, this is only one of several optimal strategies for Captain Kije; which you find depends on the choices of pivots you make in the simplex method. Other optimal strategies are the one given above, in reverse order, and [0.12 0.37 0.37 0.12] 𝑇 . The solution to the dual problem (which is unique) is 𝑠 ≈ 0.1111, 𝑡 = 0, 𝑢 = 0.0444, 𝑣 = 0, 𝑟 = 0.1111, so Colonel Blotto's optimal strategy is 𝑅 = [0.42 0 0.17 0 0.42]. Write moves as (𝑥, 𝑦), where 𝑥 represents the number of regiments sent to the first location and 𝑦 represents the number sent to the second location. Colonel Blotto should play (0, 4) with probability 0.42, (2, 2) with probability 0.17, and (4, 0) with probability 0.42. Captain Kije has several optimal strategies, one of which is to play (0, 3) with probability 0.12, (1, 2) with probability 0.41, (2, 1) with probability 0.34, and (3, 0) with probability 0.13. The expected outcome is that Colonel Blotto will win 0.75 points on average. 45. Two variables and three constraints: In the matrix formulation of an LP problem, the number of rows is one more than the number of constraints, and the number of columns is one more than the number of variables. Thus, the matrix form of the primal problem has three rows and four columns, and so its transpose has 4 rows and 3 columns, translating to three constraints and two variables. 46. The dual problem has more variables than constraints. In the matrix formulation of an LP problem, the number of rows is one more than the number of constraints, and the number of columns is one more than


Solutions Section 6.5 the number of variables. Thus, the matrix form of the primal problem has more rows than columns, and so its transpose has more columns than rows, translating to more variables than constraints. 47. The dual of a standard minimization problem satisfying the nonnegative objective condition is a standard maximization problem, which can be solved by using the standard simplex algorithm, thus avoiding the need to do Phase I. 48. To ensure that the dual of a minimization problem will be a standard maximization problem, the coefficients of the objective function should all be nonnegative, since they are what appear on the righthand sides of the inequalities in the dual problem. Thus, the primal problem should satisfy the nonnegative objective condition. (Choice A) 49. Answers will vary. We use a minimization problem that does not satisfy the nonnegative objective condition. An example is: Minimize 𝑐 = 𝑥 − 𝑦 subject to 𝑥 − 𝑦 ≥ 100, 𝑥 + 𝑦 ≥ 200, 𝑥 ≥ 0, 𝑦 ≥ 0. This problem can be solved using the techniques in the preceding section. 50. Answers will vary. An example is: Minimize 𝑐 = 3𝑥 + 2𝑦 subject to 𝑥 + 𝑦 ≤ 100, 𝑥 + 𝑦 ≥ 50, 𝑥 ≥ 0, 𝑦 ≥ 0. 51. The dual problem is a nonstandard maximization problem, because the right-hand sides of its constraints are the entries in the bottom row of the matrix representation of the primal problem, and at least one of those entries is negative. 52. The dual problem does not satisfy the nonnegative objective condition, because its coefficients are the entries in the rightmost column of the matrix representation of the primal problem, and that column contains negative entries (each ≥ inequality needs to be converted to a ≤ inequality, resulting in a negative sign on the right-hand side). 53. If the given problem is a standard minimization problem satisfying the nonnegative objective condition, its dual is a standard maximization problem and so can be solved using a single-phase simplex method. Otherwise, dualizing may not save any labor, since the dual will not be a standard maximization problem. 54. Answers will vary.


Solutions Chapter 6 Review Chapter 6 Review 1. 2𝑥 − 3𝑦 ≤ 12

2. 𝑥 ≤ 2𝑦

Unbounded Unbounded 3. 𝑥 + 2𝑦 ≤ 20 3𝑥 + 2𝑦 ≤ 30 𝑥 ≥ 0, 𝑦 ≥ 0

4. 3𝑥 + 2𝑦 ≥ 6 2𝑥 − 3𝑦 ≤ 6 3𝑥 − 2𝑦 ≥ 0 𝑥 ≥ 0, 𝑦 ≥ 0

Bounded; Corner points: (0, 0), (0, 10), (5, 15∕2), (10, 0)

Unbounded; Corner points: (2, 0), (3, 0), (1, 3∕2)

5. Maximize 𝑝 = 2𝑥 + 𝑦 subject to 3𝑥 + 𝑦 ≤ 30, 𝑥 + 𝑦 ≤ 12, 𝑥 + 3𝑦 ≤ 30, 𝑥 ≥ 0, 𝑦 ≥ 0. Corner Point Lines through Point Coordinates 𝑝 = 2𝑥 + 𝑦 𝐴

𝑥 = 0, 𝑦 = 0

(0, 0)

0

𝐵

3𝑥 + 𝑦 = 30 𝑦=0

(10, 0)

20

𝐶

3𝑥 + 𝑦 = 30 𝑥 + 𝑦 = 12

(9, 3)

21

𝐷

𝑥 + 𝑦 = 12> 𝑥 + 3𝑦 = 30

(3, 9)

15

𝐸

𝑥 + 3𝑦 = 30 𝑥=0

(0, 10)

10

Maximum value occurs at 𝐶 : 𝑝 = 21; 𝑥 = 9, 𝑦 = 3.


Solutions Chapter 6 Review 6. Maximize 𝑝 = 2𝑥 + 3𝑦 subject to 𝑥 + 𝑦 ≥ 10, 2𝑥 + 𝑦 ≥ 12, 𝑥 + 𝑦 ≤ 20, 𝑥 ≥ 0, 𝑦 ≥ 0. Corner Point Lines through Point Coordinates 𝑝 = 2𝑥 + 3𝑦 𝐴

𝑦=0 𝑥 + 𝑦 = 10

(10, 0)

20

𝐵

𝑦=0 𝑥 + 𝑦 = 20

(20, 0)

40

𝐶

𝑥=0 𝑥 + 𝑦 = 20

(0, 20)

60

𝐷

2𝑥 + 𝑦 = 12 𝑥=0

(0, 12)

36

𝐸

𝑥 + 𝑦 = 10 2𝑥 + 𝑦 = 12

(2, 8)

28

Maximum value occurs at 𝐶 : 𝑝 = 60; 𝑥 = 0, 𝑦 = 20. 7. Minimize 𝑐 = 2𝑥 + 𝑦 subject to 3𝑥 + 𝑦 ≥ 30, 𝑥 + 2𝑦 ≥ 20, 2𝑥 − 𝑦 ≥ 0, 𝑥 ≥ 0, 𝑦 ≥ 0. Corner Point Lines through Point Coordinates 𝑐 = 2𝑥 + 𝑦 𝐴

𝑥 + 2𝑦 = 20 𝑦=0

(20, 0)

40

𝐵

3𝑥 + 𝑦 = 30 𝑥 + 2𝑦 = 20

(8, 6)

22

𝐶

3𝑥 + 𝑦 = 30 2𝑥 − 𝑦 = 0

(6, 12)

24

Minimum value occurs at 𝐵 : 𝑐 = 22; 𝑥 = 8, 𝑦 = 6. 8. Minimize 𝑐 = 3𝑥 + 𝑦 subject to 3𝑥 + 2𝑦 ≥ 6, 2𝑥 − 3𝑦 ≤ 0, 3𝑥 − 2𝑦 ≥ 0, 𝑥 ≥ 0, 𝑦 ≥ 0. Corner Point Lines through Point

Coordinates

𝑐 = 3𝑥 + 𝑦

𝐴

3𝑥 + 2𝑦 = 6 3𝑥 − 2𝑦 = 0

(1, 3∕2)

9∕2

𝐵

3𝑥 + 2𝑦 = 6 2𝑥 − 3𝑦 = 0

(18∕13, 12∕13)

66∕13

Minimum value occurs at 𝐴 : 𝑐 = 9∕2; 𝑥 = 1, 𝑦 = 3∕2 9. Introduce slack variables and rewrite the constraints and objective function in standard form, and set up the first tableau: 𝑥 + 2𝑦 + 2𝑧 + 𝑠 = 60 2𝑥 + 𝑦 + 3𝑧 + 𝑡 = 60 −𝑥 − 𝑦 − 2𝑧 + 𝑝 = 0


Solutions Chapter 6 Review 𝑥

𝑦

𝑧

𝑠

𝑡

𝑝

𝑠 𝑡

1 2

2 1

2 3

1 0

0 1

0 0

60 60

𝑝

−1

−1

−2

0

0

1

0

The most negative entry in the bottom row is the −2 in the 𝑧-column, so we use this column as the pivot column. The test ratios are: s: 60/2, t: 60/3. The smallest test ratio is t: 60/3. Thus we pivot on the 3 in the t-row. 𝑥

𝑦

𝑧

𝑠

𝑡

𝑝

𝑠

1

2

2

1

0

0

60

𝑡

2

1

3

0

1

0

60

𝑝

−1

−1

−2

0

0

1

0

𝑥

𝑦

𝑧

𝑠

𝑡

𝑝

𝑠 𝑧

−1 2

4 1

0 3

3 0

−2 1

0 0

60 60

𝑝

1

−1

0

0

2

3

120

3𝑅1 − 2𝑅2 3𝑅3 + 2𝑅2

The most negative entry in the bottom row is the −1 in the 𝑦-column, so we use this column as the pivot column. The test ratios are: s: 60/4, z: 60/1. The smallest test ratio is s: 60/4. Thus we pivot on the 4 in the s-row. 𝑥

𝑦

𝑧

𝑠

𝑡

𝑝

𝑠

−1

4

0

3

−2

0

60

𝑧

2

1

3

0

1

0

60

4𝑅2 − 𝑅1

𝑝

1

−1

0

0

2

3

120

4𝑅3 + 𝑅1

𝑥

𝑦

𝑧

𝑠

𝑡

𝑝

𝑦 𝑧

−1 9

4 0

0 12

3 −3

−2 6

0 0

60 180

𝑝

3

0

0

3

6

12

540

As there are no more negative numbers in the bottom row, we are done, and read off the solution: Optimal solution: 𝑝 = 540∕12 = 45; 𝑥 = 0, 𝑦 = 60∕4 = 15, 𝑧 = 180∕12 = 15. 10. Introduce slack variables and rewrite the constraints and objective function in standard form, and set up the first tableau: 𝑥 + 2𝑦 + 2𝑧 + 𝑠 = 60 2𝑥 + 𝑦 + 3𝑧 + 𝑡 = 60 𝑥 + 3𝑦 + 6𝑧 + 𝑢 = 60 −𝑥 − 𝑦 − 2𝑧 + 𝑝 = 0


Solutions Chapter 6 Review 𝑥

𝑦

𝑧

𝑠

𝑡

𝑢

𝑝

𝑠 𝑡 𝑢

1 2 1

2 1 3

2 3 6

1 0 0

0 1 0

0 0 1

0 0 0

60 60 60

𝑝

−1

−1

−2

0

0

0

1

0

The most negative entry in the bottom row is the −2 in the 𝑧-column, so we use this column as the pivot column. The test ratios are: s: 60/2, t: 60/3, u: 60/6. The smallest test ratio is u: 60/6. Thus we pivot on the 6 in the u-row. 𝑥

𝑦

𝑧

𝑠

𝑡

𝑢

𝑝

𝑠

1

2

2

1

0

0

0

60

3𝑅1 − 𝑅3

𝑡

2

1

3

0

1

0

0

60

2𝑅2 − 𝑅3

𝑢

1

3

6

0

0

1

0

60

𝑝

−1

−1

−2

0

0

0

1

0

𝑥

𝑦

𝑧

𝑠

𝑡

𝑢

𝑝

𝑠 𝑡 𝑧

2 3 1

3 −1 3

0 0 6

3 0 0

0 2 0

−1 −1 1

0 0 0

120 60 60

𝑝

−2

0

0

0

0

1

3

60

3𝑅4 + 𝑅3

The most negative entry in the bottom row is the −2 in the 𝑥-column, so we use this column as the pivot column. The test ratios are: s: 120/2, t: 60/3, z: 60/1. The smallest test ratio is t: 60/3. Thus we pivot on the 3 in the t-row. 𝑥

𝑦

𝑧

𝑠

𝑡

𝑢

𝑝

𝑠

2

3

0

3

0

−1

0

120

𝑡

3

−1

0

0

2

−1

0

60

𝑧

1

3

6

0

0

1

0

60

3𝑅3 − 𝑅2

𝑝

−2

0

0

0

0

1

3

60

3𝑅4 + 2𝑅2

𝑥

𝑦

𝑧

𝑠

𝑡

𝑢

𝑝

𝑠 𝑥 𝑧

0 3 0

11 −1 10

0 0 18

9 0 0

−4 2 −2

−1 −1 4

0 0 0

240 60 120

𝑝

0

−2

0

0

4

1

9

300

3𝑅1 − 2𝑅2

The most negative entry in the bottom row is the −2 in the 𝑦-column, so we use this column as the pivot column. The test ratios are: s: 240/11, z: 120/10. The smallest test ratio is z: 120/10. Thus we pivot on the 10 in the z-row.


Solutions Chapter 6 Review 𝑥

𝑦

𝑧

𝑠

𝑡

𝑢

𝑝

𝑠

0

11

0

9

−4

−1

0

240

10𝑅1 − 11𝑅3

𝑥

3

−1

0

0

2

−1

0

60

10𝑅2 + 𝑅3

𝑧

0

10

18

0

−2

4

0

120

𝑝

0

−2

0

0

4

1

9

300

𝑥

𝑦

𝑧

𝑠

𝑡

𝑢

𝑝

𝑠 𝑥 𝑦

0 30 0

0 0 10

−198 18 18

90 0 0

−18 18 −2

−54 −6 4

0 0 0

1080 720 120

𝑝

0

0

18

0

18

9

45

1620

5𝑅4 + 𝑅3

As there are no more negative numbers in the bottom row, we are done, and read off the solution: Optimal solution: 𝑝 = 1,620∕45 = 36; 𝑥 = 720∕30 = 24, 𝑦 = 120∕10 = 12, 𝑧 = 0. 11. Introduce slack variables and rewrite the constraints and objective function in standard form, and set up the first tableau: 𝑥 + 𝑦 + 𝑧 − 𝑠 = 100 𝑦 + 𝑧 + 𝑡 = 80 𝑥 + 𝑧 + 𝑢 = 80 −𝑥 − 𝑦 − 3𝑧 + 𝑝 = 0 𝑥

𝑦

𝑧

𝑠

𝑡

𝑢

𝑝

*𝑠 𝑡 𝑢

1 0 1

1 1 0

1 1 1

−1 0 0

0 1 0

0 0 1

0 0 0

100 80 80

𝑝

−1

−1

−3

0

0

0

1

0

The first starred row is the s-row, and its largest positive entry is the 1 in the 𝑥-column. Thus, we use this column as the pivot column. The test ratios are: s: 100/1, u: 80/1. The smallest test ratio is u: 80/1. Thus we pivot on the 1 in the u-row. 𝑥

𝑦

𝑧

𝑠

𝑡

𝑢

𝑝

*𝑠 𝑡

1

1

1

−1

0

0

0

100

0

1

1

0

1

0

0

80

𝑢

1

0

1

0

0

1

0

80

𝑝

−1

−1

−3

0

0

0

1

0

𝑅1 − 𝑅3

𝑅4 + 𝑅3


Solutions Chapter 6 Review 𝑥

𝑦

𝑧

𝑠

𝑡

𝑢

𝑝

*𝑠 𝑡 𝑥

0 0 1

1 1 0

0 1 1

−1 0 0

0 1 0

−1 0 1

0 0 0

20 80 80

𝑝

0

−1

−2

0

0

1

1

80

The first starred row is the s-row, and its largest positive entry is the 1 in the 𝑦-column. Thus, we use this column as the pivot column. The test ratios are: s: 20/1, t: 80/1. The smallest test ratio is s: 20/1. Thus we pivot on the 1 in the s-row. 𝑥

𝑦

𝑧

𝑠

𝑡

𝑢

𝑝

*𝑠 𝑡

0

1

0

−1

0

−1

0

20

0

1

1

0

1

0

0

80

𝑥

1

0

1

0

0

1

0

80

𝑝

0

−1

−2

0

0

1

1

80

𝑥

𝑦

𝑧

𝑠

𝑡

𝑢

𝑝

𝑦 𝑡 𝑥

0 0 1

1 0 0

0 1 1

−1 1 0

0 1 0

−1 1 1

0 0 0

20 60 80

𝑝

0

0

−2

−1

0

0

1

100

𝑅2 − 𝑅1 𝑅4 + 𝑅1

As there are no more starred rows, we go to Phase 2, and do the standard simplex method. 𝑥

𝑦

𝑧

𝑠

𝑡

𝑢

𝑝

𝑦

0

1

0

−1

0

−1

0

20

𝑡

0

0

1

1

1

1

0

60

𝑥

1

0

1

0

0

1

0

80

𝑅3 − 𝑅2

𝑝

0

0

−2

−1

0

0

1

100

𝑅4 + 2𝑅2

𝑥

𝑦

𝑧

𝑠

𝑡

𝑢

𝑝

𝑦 𝑧 𝑥

0 0 1

1 0 0

0 1 0

−1 1 −1

0 1 −1

−1 1 0

0 0 0

20 60 20

𝑝

0

0

0

1

2

2

1

220

Optimal solution: 𝑝 = 220∕1 = 220; 𝑥 = 20∕1 = 20, 𝑦 = 20∕1 = 20, 𝑧 = 60∕1 = 60.


Solutions Chapter 6 Review 12. Introduce slack variables and rewrite the constraints and objective function in standard form, and set up the first tableau: 𝑥 + 2𝑦 − 𝑠 = 12 2𝑥 + 𝑦 + 𝑡 = 12 𝑥+𝑦+𝑢=5 −2𝑥 − 𝑦 + 𝑝 = 0 𝑥

𝑦

𝑠

𝑡

𝑢

𝑝

*𝑠 𝑡 𝑢

1 2 1

2 1 1

−1 0 0

0 1 0

0 0 1

0 0 0

12 12 5

𝑝

−2

−1

0

0

0

1

0

The first starred row is the s-row, and its largest positive entry is the 2 in the 𝑦-column. Thus, we use this column as the pivot column. The test ratios are: s: 12/2, t: 12/1, u: 5/1. The smallest test ratio is u: 5/1. Thus we pivot on the 1 in the u-row. 𝑥

𝑦

𝑠

𝑡

𝑢

𝑝

*𝑠 𝑡

1

2

−1

0

0

0

12

𝑅1 − 2𝑅3

2

1

0

1

0

0

12

𝑅2 − 𝑅3

𝑢

1

1

0

0

1

0

5

𝑝

−2

−1

0

0

0

1

0

𝑥

𝑦

𝑠

𝑡

𝑢

𝑝

*𝑠 𝑡 𝑦

−1 1 1

0 0 1

−1 0 0

0 1 0

−2 −1 1

0 0 0

2 7 5

𝑝

−1

0

0

0

1

1

5

𝑅4 + 𝑅3

There is no positive entry in the starred row, meaning that the feasible region is empty, so there is no solution. 13. Introduce slack variables and rewrite the constraints and objective function in standard form, and set up the first tableau: 3𝑥 + 2𝑦 + 𝑧 − 𝑠 = 60 2𝑥 + 𝑦 + 3𝑧 − 𝑡 = 60 𝑥 + 2𝑦 + 3𝑧 + 𝑝 = 0 𝑥

𝑦

𝑧

𝑠

𝑡

𝑝

*𝑠 *𝑡

3 2

2 1

1 3

−1 0

0 −1

0 0

60 60

𝑝

1

2

3

0

0

1

0

The first starred row is the s-row, and its largest positive entry is the 3 in the 𝑥-column. Thus, we use this column as the pivot column. The test ratios are: s: 60/3, t: 60/2. The smallest test ratio is s: 60/3. Thus we pivot on the 3 in the s-row.


Solutions Chapter 6 Review 𝑥

𝑦

𝑧

𝑠

𝑡

𝑝

*𝑠 *𝑡

3

2

1

−1

0

0

60

2

1

3

0

−1

0

60

3𝑅2 − 2𝑅1

𝑝

1

2

3

0

0

1

0

3𝑅3 − 𝑅1

𝑥

𝑦

𝑧

𝑠

𝑡

𝑝

𝑥 *𝑡

3 0

2

−1

1 7

−1 2

0

−3

0 0

60 60

𝑝

0

4

8

1

0

3

−60

The first starred row is the t-row, and its largest positive entry is the 7 in the 𝑧-column. Thus, we use this column as the pivot column. The test ratios are: x: 60/1, t: 60/7. The smallest test ratio is t: 60/7. Thus we pivot on the 7 in the t-row. 𝑥

𝑦

𝑧

𝑠

𝑡

𝑝

𝑥

3

2

1

−1

0

0

60

*𝑡

0

−1

7

2

−3

0

60

𝑝

0

4

8

1

0

3

−60

𝑥

𝑦

𝑧

𝑠

𝑡

𝑝

𝑥 𝑧

21 0

15 −1

0 7

−9 2

3 −3

0 0

360 60

𝑝

0

36

0

−9

24

21

−900

7𝑅1 − 𝑅2 7𝑅3 − 8𝑅2

As there are no more starred rows, we go to Phase 2, and do the standard simplex method. 𝑥

𝑦

𝑧

𝑠

𝑡

𝑝

𝑥

21

15

0

−9

3

0

360

𝑧

0

−1

7

2

−3

0

60

𝑝

0

36

0

−9

24

21

−900

𝑥

𝑦

𝑧

𝑠

𝑡

𝑝

𝑥 𝑠

42 0

21 −1

63 7

0 2

−21 −3

0 0

1260 60

𝑝

0

63

63

0

21

42

−1260

2𝑅1 + 9𝑅2 2𝑅3 + 9𝑅2

Optimal solution: 𝑝 = −1,260∕42 = −30; 𝑥 = 1,260∕42 = 30, 𝑦 = 0, 𝑧 = 0. Since 𝑐 = −𝑝, the minimum value of c is 30. 14. Introduce slack variables and rewrite the constraints and objective function in standard form, and set up the first


Solutions Chapter 6 Review tableau: 𝑥 + 𝑦 + 4𝑧 − 𝑠 = 30 2𝑥 + 𝑦 + 3𝑧 − 𝑡 = 60 5𝑥 + 4𝑦 + 3𝑧 + 𝑝 = 0 𝑥

𝑦

𝑧

𝑠

𝑡

𝑝

*𝑠 *𝑡

1 2

1 1

4 3

−1 0

0 −1

0 0

30 60

𝑝

5

4

3

0

0

1

0

The first starred row is the s-row, and its largest positive entry is the 4 in the 𝑧-column. Thus, we use this column as the pivot column. The test ratios are: s: 30/4, t: 60/3. The smallest test ratio is s: 30/4. Thus we pivot on the 4 in the s-row. 𝑥

𝑦

𝑧

𝑠

𝑡

𝑝

*𝑠 *𝑡

1

1

4

−1

0

0

30

2

1

3

0

−1

0

60

4𝑅2 − 3𝑅1

𝑝

5

4

3

0

0

1

0

4𝑅3 − 3𝑅1

𝑥

𝑦

𝑧

𝑠

𝑡

𝑝

𝑧 *𝑡

1 5

1 1

4 0

−1 3

0 −4

0 0

30 150

𝑝

17

13

0

3

0

4

−90

The first starred row is the t-row, and its largest positive entry is the 5 in the 𝑥-column. Thus, we use this column as the pivot column. The test ratios are: z: 30/1, t: 150/5. The smallest test ratio is t: 150/5. Thus we pivot on the 5 in the t-row. 𝑥

𝑦

𝑧

𝑠

𝑡

𝑝

𝑧

1

1

4

−1

0

0

30

*𝑡

5

1

0

3

−4

0

150

𝑝

17

13

0

3

0

4

−90

𝑥

𝑦

𝑧

𝑠

𝑡

𝑝

𝑧 𝑥

0 5

4 1

20 0

−8 3

4 −4

0 0

0 150

𝑝

0

48

0

−36

68

20

−3000

5𝑅1 − 𝑅2 5𝑅3 − 17𝑅2

As there are no more starred rows, we go to Phase 2, and do the standard simplex method.


Solutions Chapter 6 Review 𝑥

𝑦

𝑧

𝑠

𝑡

𝑝

𝑧

0

4

20

−8

4

0

0

𝑥

5

1

0

3

−4

0

150

𝑝

0

48

0

−36

68

20

−3000

𝑥

𝑦

𝑧

𝑠

𝑡

𝑝

𝑧 𝑠

40 5

20 1

60 0

0 3

−20 −4

0 0

1200 150

𝑝

60

60

0

0

20

20

−1200

3𝑅1 + 8𝑅2 𝑅3 + 12𝑅2

Optimal solution: 𝑝 = −1,200∕20 = −60; 𝑥 = 0, 𝑦 = 0, 𝑧 = 1,200∕60 = 20. Since 𝑐 = −𝑝, the minimum value of c is 60. 15. Since technology is indicated, we use the online Simplex Method Tool. When entering the problems in this tool there is no need to enter the inequalities 𝑥 ≥ 0, 𝑦 ≥ 0, etc. Tableau #1 x y 3 2 2 1 1 3 1 -2

z -1 3 -2 4

s1 -1 0 0 0

s2 0 -1 0 0

s3 0 0 -1 0

-c 0 0 0 1

10 20 30 0

z 3 11 7 10

s1 0 0 1 0

s2 -1 -3 -2 -2

s3 0 1 0 0

-c 0 0 0 1

20 30 30 40

... Tableau #6 x y 2 1 5 0 1 0 5 0

Looking at the sixth tableau, we find that there is no possible pivot above the most negative entry in the bottom row, indicating that the feasible region is unbounded (and no optimal solution exists). 16. Since technology is indicated, we use the online Simplex Method Tool. When entering the problems in this tool there is no need to enter the inequalities 𝑥 ≥ 0, 𝑦 ≥ 0, etc. Tableau #1 x y 3 2 2 1 1 3 1 1 ...

z 1 3 2 -1

s1 -1 0 0 0

s2 0 -1 0 0

s3 0 0 -1 0

-c 0 0 0 1

60 60 60 0


Solutions Chapter 6 Review Tableau #6 x y 5 1 3 2 7 5 4 3

z 0 1 0 0

s1 -2 -1 -3 -1

s2 0 0 1 0

s3 1 0 0 0

-c 0 0 0 1

60 60 120 60

Looking at the sixth tableau, we find that there is no possible pivot above the most negative entry in the bottom row, indicating that the feasible region is unbounded (and no optimal solution exists). 17. Introduce slack variables and rewrite the constraints and objective function in standard form, and set up the first tableau: 𝑥 + 𝑦 − 𝑠 = 30 𝑥 + 𝑧 − 𝑡 = 20 𝑥 + 𝑦 − 𝑤 + 𝑢 = 10 𝑦 + 𝑧 − 𝑤 + 𝑣 = 10 𝑥+𝑦+𝑧+𝑤+𝑝=0 𝑥

𝑦

𝑧

𝑤

𝑠

𝑡

𝑢

𝑣

𝑝

*𝑠 *𝑡 𝑢 𝑣

1 1 1 0

1 0 1 1

0 1 0 1

0 0 −1 −1

−1 0 0 0

0 −1 0 0

0 0 1 0

0 0 0 1

0 0 0 0

30 20 10 10

𝑝

1

1

1

1

0

0

0

0

1

0

The first starred row is the s-row, and its largest positive entry is the 1 in the 𝑥-column. Thus, we use this column as the pivot column. The test ratios are: s: 30/1, t: 20/1, u: 10/1. The smallest test ratio is u: 10/1. Thus we pivot on the 1 in the u-row. 𝑥

𝑦

𝑧

𝑤

𝑠

𝑡

𝑢

𝑣

𝑝

*𝑠 *𝑡

1

1

0

0

−1

0

0

0

0

30

𝑅1 − 𝑅3

1

0

1

0

0

−1

0

0

0

20

𝑅2 − 𝑅3

𝑢

1

1

0

−1

0

0

1

0

0

10

𝑣

0

1

1

−1

0

0

0

1

0

10

𝑝

1

1

1

1

0

0

0

0

1

0

𝑥

𝑦

𝑧

𝑤

𝑠

𝑡

𝑢

𝑣

𝑝

*𝑠 *𝑡 𝑥 𝑣

0 0 1 0

0 −1 1 1

0 1 0 1

1 1 −1 −1

−1 0 0 0

0 −1 0 0

−1 −1 1 0

0 0 0 1

0 0 0 0

20 10 10 10

𝑝

0

0

1

2

0

0

−1

0

1

−10

𝑅5 − 𝑅3

The first starred row is the s-row, and its largest positive entry is the 1 in the 𝑤-column. Thus, we use this column as the pivot column. The test ratios are: s: 20/1, t: 10/1. The smallest test ratio is t: 10/1. Thus we pivot on the 1 in the t-row.


Solutions Chapter 6 Review 𝑥

𝑦

𝑧

𝑤

𝑠

𝑡

𝑢

𝑣

𝑝

*𝑠 *𝑡

0

0

0

1

−1

0

−1

0

0

20

0

−1

1

1

0

−1

−1

0

0

10

𝑥

1

1

0

−1

0

0

1

0

0

10

𝑅3 + 𝑅2

𝑣

0

1

1

−1

0

0

0

1

0

10

𝑅4 + 𝑅2

𝑝

0

0

1

2

0

0

−1

0

1

−10

𝑅5 − 2𝑅2

𝑥

𝑦

𝑧

𝑤

𝑠

𝑡

𝑢

𝑣

𝑝

*𝑠 𝑤 𝑥 𝑣

0 0 1 0

1 −1 0 0

−1 1 1 2

0 1 0 0

−1 0 0 0

1 −1 −1 −1

0 −1 0 −1

0 0 0 1

0 0 0 0

10 10 20 20

𝑝

0

2

−1

0

0

2

1

0

1

−30

𝑅1 − 𝑅2

The first starred row is the s-row, and its largest positive entry is the 1 in the 𝑦-column. Thus, we use this column as the pivot column. The only positive entry in this column is the 1 in the s-row, so we pivot on that entry. 𝑥

𝑦

𝑧

𝑤

𝑠

𝑡

𝑢

𝑣

𝑝

*𝑠 𝑤

0

1

−1

0

−1

1

0

0

0

10

0

−1

1

1

0

−1

−1

0

0

10

𝑥 𝑣

1 0

0 0

1 2

0 0

0 0

−1 −1

0 −1

0 1

0 0

20 20

𝑝

0

2

−1

0

0

2

1

0

1

−30

𝑥

𝑦

𝑧

𝑤

𝑠

𝑡

𝑢

𝑣

𝑝

𝑦 𝑤 𝑥 𝑣

0 0 1 0

1 0 0 0

−1 0 1 2

0 1 0 0

−1 −1 0 0

1 0 −1 −1

0 −1 0 −1

0 0 0 1

0 0 0 0

10 20 20 20

𝑝

0

0

1

0

2

0

1

0

1

−50

𝑅2 + 𝑅1

𝑅5 − 2𝑅1

As there are no more starred rows, we go to Phase 2, and do the standard simplex method. However, there are no negative numbers in the bottom row, so there are no pivot steps to do in Phase 2. Therefore, we are done. Optimal solution: 𝑝 = −50∕1 = −50; 𝑥 = 20∕1 = 20, 𝑦 = 10∕1 = 10, 𝑧 = 0, 𝑤 = 20∕1 = 20. Since 𝑐 = −𝑝, the minimum value of c is 50.Another solution is: 𝑥 = 30, 𝑦 = 0, 𝑧 = 0, 𝑤 = 20. 18. Introduce slack variables and rewrite the constraints and objective function in standard form, and set up the first tableau: 𝑥 + 𝑦 − 𝑠 = 30 𝑦 − 𝑧 + 𝑡 = 20 𝑧 − 𝑤 + 𝑢 = 10 4𝑥 + 𝑦 + 𝑧 + 𝑤 + 𝑝 = 0


Solutions Chapter 6 Review 𝑥

𝑦

𝑧

𝑤

𝑠

𝑡

𝑢

𝑝

*𝑠 𝑡 𝑢

1 0 0

1 1 0

0 −1 1

0 0 −1

−1 0 0

0 1 0

0 0 1

0 0 0

30 20 10

𝑝

4

1

1

1

0

0

0

1

0

The first starred row is the s-row, and its largest positive entry is the 1 in the 𝑥-column. Thus, we use this column as the pivot column. The only positive entry in this column is the 1 in the s-row, so we pivot on that entry. 𝑥

𝑦

𝑧

𝑤

𝑠

𝑡

𝑢

𝑝

*𝑠 𝑡 𝑢

1

1

0

0

−1

0

0

0

30

0 0

1 0

−1 1

0 −1

0 0

1 0

0 1

0 0

20 10

𝑝

4

1

1

1

0

0

0

1

0

𝑥

𝑦

𝑧

𝑤

𝑠

𝑡

𝑢

𝑝

𝑥 𝑡 𝑢

1 0 0

1 1 0

0 −1 1

0 0 −1

−1 0 0

0 1 0

0 0 1

0 0 0

30 20 10

𝑝

0

−3

1

1

4

0

0

1

−120

𝑅4 − 4𝑅1

As there are no more starred rows, we go to Phase 2, and do the standard simplex method. 𝑥

𝑦

𝑧

𝑤

𝑠

𝑡

𝑢

𝑝

𝑥

1

1

0

0

−1

0

0

0

30

𝑡

0

1

−1

0

0

1

0

0

20

𝑢

0

0

1

−1

0

0

1

0

10

𝑝

0

−3

1

1

4

0

0

1

−120

𝑥

𝑦

𝑧

𝑤

𝑠

𝑡

𝑢

𝑝

𝑥

1

0

1

0

−1

−1

0

0

10

𝑦

0

1

−1

0

0

1

0

0

20

𝑅2 + 𝑅1

𝑢

0

0

1

−1

0

0

1

0

10

𝑅3 − 𝑅1

𝑝

0

0

−2

1

4

3

0

1

−60

𝑅4 + 2𝑅1

𝑅1 − 𝑅2

𝑅4 + 3𝑅2


Solutions Chapter 6 Review 𝑥

𝑦

𝑧

𝑤

𝑠

𝑡

𝑢

𝑝

𝑧 𝑦 𝑢

1 1 −1

0 1 0

1 0 0

0 0 −1

−1 −1 1

−1 0 1

0 0 1

0 0 0

10 30 0

𝑝

2

0

0

1

2

1

0

1

−40

Optimal solution: 𝑝 = −40∕1 = −40; 𝑥 = 0, 𝑦 = 30∕1 = 30, 𝑧 = 10∕1 = 10, 𝑤 = 0. Since 𝑐 = −𝑝, the minimum value of c is 40. 19. Minimize 𝑐 = 2𝑠 + 𝑡 subject to 3𝑠 + 2𝑡 ≥ 60, 2𝑠 + 𝑡 ≥ 60, 𝑠 + 3𝑡 ≥ 60, 𝑠 ≥ 0, 𝑡 ≥ 0. 3 2 60 3 2 1 2 2 1 60 Dualize: → 2 1 3 1 . 1 3 60 60 60 60 0 2 1 0 Dual problem: Maximize 𝑝 = 60𝑥 + 60𝑦 + 60𝑧 subject to 3𝑥 + 2𝑦 + 𝑧 ≤ 2, 2𝑥 + 𝑦 + 3𝑧 ≤ 1, 𝑥 ≥ 0, 𝑦 ≥ 0, 𝑧 ≥ 0. Solve the dual problem: 𝑥 𝑦 𝑧 𝑠 𝑡 𝑝 2𝑅1 − 3𝑅2

𝑠

3

2

1

1

0

0

2

𝑡

2

1

3

0

1

0

1

𝑝

−60

−60

−60

0

0

1

0

𝑥

𝑦

𝑧

𝑠

𝑡

𝑝

𝑠

0

1

−7

2

−3

0

1

𝑥

2

1

3

0

1

0

1

𝑅2 − 𝑅1

𝑝

0

−30

30

0

30

1

30

𝑅3 + 30𝑅1

𝑥

𝑦

𝑧

𝑠

𝑡

𝑝

𝑦

0

1

−7

2

−3

0

1

10𝑅1 + 7𝑅2

𝑥

2

0

10

−2

4

0

0

𝑝

0

0

−180

60

−60

1

60

𝑥

𝑦

𝑧

𝑠

𝑡

𝑝

𝑦 𝑧

14 2

10 0

0 10

6 −2

−2 4

0 0

10 0

𝑝

36

0

0

24

12

1

60

𝑅3 + 30𝑅2

𝑅3 + 18𝑅2

Solution to the primal problem: 𝑐 = 60; 𝑠 = 24, 𝑡 = 12. Another possible solution: 𝑐 = 60; 𝑠 = 0, 𝑡 = 60 (pivot on the 1 in the second row in the second tableau). Since the original unknowns were 𝑥 and 𝑦, we write these optimal solutions as: 𝑐 = 60; 𝑥 = 24, 𝑦 = 12 or 𝑐 = 60; 𝑥 = 0, 𝑦 = 60.


Solutions Chapter 6 Review 20. Minimize 𝑐 = 2𝑠 + 𝑡 + 2𝑢 subject to 3𝑠 + 2𝑡 + 𝑢 ≥ 100, 2𝑠 + 𝑡 + 3𝑢 ≥ 200, 𝑡 ≥ 0, 𝑡 ≥ 0, 𝑢 ≥ 0. 3 2 2 3 2 1 100 2 1 1 Dualize: 2 1 3 200 → . 1 3 2 2 1 2 0 100 200 0 Dual problem: Maximize 𝑝 = 100𝑥 + 200𝑦 subject to 3𝑥 + 2𝑦 ≤ 2, 2𝑥 + 𝑦 ≤ 1, 𝑥 + 3𝑦 ≤ 2, 𝑥 ≥ 0, 𝑦 ≥ 0. Solve the dual problem: 𝑥 𝑦 𝑠 𝑡 𝑢 𝑝 𝑠

3

2

1

0

0

0

2

3𝑅1 − 2𝑅3

𝑡

2

1

0

1

0

0

1

3𝑅2 − 𝑅3

𝑢

1

3

0

0

1

0

2

0

0

0

1

0

3𝑅4 + 200𝑅3

5𝑅1 − 7𝑅2

𝑝

−100 −200 𝑥

𝑦

𝑠

𝑡

𝑢

𝑝

𝑠

7

0

3

0

−2

0

2

𝑡

5

0

0

3

−1

0

1

𝑦

1

3

0

0

1

0

2

5𝑅3 − 𝑅2

𝑝

−100

0

0

0

200

3

400

𝑅4 + 20𝑅2

𝑥

𝑦

𝑠

𝑡

𝑢

𝑝

𝑠 𝑥 𝑦

0 5 0

0 0 15

15 0 0

−21 3 −3

−3 −1 6

0 0 0

3 1 9

𝑝

0

0

0

60

180

3

420

Solution to the primal problem: 𝑐 = 420∕3 = 140; 𝑠 = 0, 𝑡 = 60∕3 = 20, 𝑢 = 180∕3 = 60. Since the original unknowns were 𝑥, 𝑦, and 𝑧, we write the optimal solution as: 𝑐 = 140; 𝑥 = 0, 𝑦 = 20, 𝑧 = 60. 21. First rewrite the given problem as a standard minimization problem with all the constraints using "≥": Minimize 𝑐 = 2𝑠 + 𝑡 subject to 3𝑠 + 2𝑡 ≥ 10, −2𝑠 + 𝑡 ≥ −30, 𝑠 + 3𝑡 ≥ 60, 𝑠 ≥ 0, 𝑡 ≥ 0. 3 2 10 3 −2 1 2 −2 1 −30 Dualize: → 2 1 3 1 . 1 3 60 10 −30 60 0 2 1 0 Dual problem: Maximize 𝑝 = 10𝑥 − 30𝑦 + 60𝑧 subject to 3𝑥 − 2𝑦 + 𝑧 ≤ 2, 2𝑥 + 𝑦 + 3𝑧 ≤ 1, 𝑥 ≥ 0, 𝑦 ≥ 0, 𝑧 ≥ 0. Solve the dual problem: 𝑥 𝑦 𝑧 𝑠 𝑡 𝑝 𝑠

3

−2

1

1

0

0

2

𝑡

2

1

3

0

1

0

1

𝑝

−10

30

−60

0

0

1

0

3𝑅1 − 𝑅2 𝑅3 + 20𝑅2


Solutions Chapter 6 Review 𝑥

𝑦

𝑧

𝑠

𝑡

𝑝

𝑠 𝑧

7 2

−7 1

0 3

3 0

−1 1

0 0

5 1

𝑝

30

50

0

0

20

1

20

Solution to the primal problem: 𝑐 = 20∕1 = 20; 𝑠 = 0, 𝑡 = 20∕1 = 20. Since the original unknowns were 𝑥 and 𝑦, we write the optimal solution as: 𝑐 = 20; 𝑥 = 0, 𝑦 = 20. 22. First rewrite the given problem as a standard minimization problem with all the constraints using "≥": Minimize 𝑐 = 2𝑠 + 𝑡 + 2𝑢 subject to 3𝑠 − 2𝑡 + 𝑢 ≥ 100, −2𝑠 − 𝑡 + 3𝑢 ≥ −200, 𝑠 ≥ 0, 𝑡 ≥ 0, 𝑢 ≥ 0. 3 −2 2 3 −2 1 100 −2 −1 1 Dualize: −2 −1 3 −200 → . 1 3 2 2 1 2 0 100 −200 0 Maximize 𝑝 = 100𝑥 − 200𝑦 subject to 3𝑥 − 2𝑦 ≤ 2, −2𝑥 − 2𝑦 ≤ 1, 𝑥 + 3𝑦 ≤ 2, 𝑥 ≥ 0, 𝑦 ≥ 0. Solve the dual problem: 𝑥 𝑦 𝑠 𝑡 𝑢 𝑝 𝑠

3

−2

1

0

0

0

2

𝑡

−2

−2

0

1

0

0

1

3𝑅2 + 2𝑅1

𝑢

1

3

0

0

1

0

2

3𝑅3 − 𝑅1

𝑝

−100

200

0

0

0

1

0

3𝑅4 + 100𝑅1

𝑥

𝑦

𝑠

𝑡

𝑢

𝑝

𝑥 𝑡 𝑢

3 0 0

−2 −10 11

1 2 −1

0 3 0

0 0 3

0 0 0

2 7 4

𝑝

0

400

100

0

0

3

200

Solution to the primal problem: 𝑐 = 200∕3; 𝑠 = 100∕3, 𝑡 = 0, 𝑢 = 0. Since the original unknowns were 𝑥, 𝑦 and 𝑧, we write the optimal solution as: 𝑐 = 200∕3; 𝑥 = 100∕3, 𝑦 = 0, 𝑧 = 0. 23. The first column is dominated by the last, so we can eliminate it. We cannot reduce any further. Add 𝑘 = 2 to each entry and put 1s to the right and below: 4 1 1 0 3 1 . 1 2 1 1 1 0 This gives the following LP problem: Maximize 𝑝 = 𝑥 + 𝑦 subject to 4𝑥 + 𝑦 ≤ 1, 3𝑦 ≤ 1, 𝑥 + 2𝑦 ≤ 1, 𝑥 ≥ 0, 𝑦 ≥ 0. Here are the tableaux:


Solutions Chapter 6 Review 𝑥

𝑦

𝑠

𝑡

𝑢

𝑝

𝑠

4

1

1

0

0

0

1

𝑡

0

3

0

1

0

0

1

𝑢

1

2

0

0

1

0

1

4𝑅3 − 𝑅1

𝑝

−1

−1

0

0

0

1

0

4𝑅4 + 𝑅1

𝑥

𝑦

𝑠

𝑡

𝑢

𝑝

𝑥

4

1

1

0

0

0

1

3𝑅1 − 𝑅2

𝑡

0

3

0

1

0

0

1

𝑢

0

7

−1

0

4

0

3

3𝑅3 − 7𝑅2

𝑝

0

−3

1

0

0

4

1

𝑅4 + 𝑅2

𝑥

𝑦

𝑠

𝑡

𝑢

𝑝

𝑥 𝑦 𝑢

12 0 0

0 3 0

3 0

−3

−1 1 −7

0 0 12

0 0 0

2 1 2

𝑝

0

0

1

1

0

4

2

The solution to the primal problem is 𝑝 = 2∕4 = 1∕2; 𝑥 = 2∕12 = 1∕6, 𝑦 = 1∕3. So, the column player's optimal strategy is 𝐶 = [0 1∕3 2∕3] 𝑇 and the value of the game is 𝑒 = 2 − 2 = 0. The solution to the dual problem is 𝑠 = 1∕4, 𝑡 = 1∕4, 𝑢 = 0, so the row player's optimal strategy is 𝑅 = [1∕2 1∕2 0]. 24. The first row dominates the second, so remove the second. We cannot reduce any further. Add 𝑘 = 3 to each entry and put 1s to the right and below: 0 3 4 1 3 2 1 1 . 1 1 1 0 This gives the following LP problem: Maximize 𝑝 = 𝑥 + 𝑦 + 𝑧 subject to 3𝑦 + 4𝑧 ≤ 1, 3𝑥 + 2𝑦 + 𝑧 ≤ 1, 𝑥 ≥ 0, 𝑦 ≥ 0, 𝑧 ≥ 0. Here are the tableaux: 𝑥 𝑦 𝑧 𝑠 𝑡 𝑝 𝑠

0

3

4

1

0

0

1

𝑡

3

2

1

0

1

0

1

𝑝

−1

−1

−1

0

0

1

0

𝑥

𝑦

𝑧

𝑠

𝑡

𝑝

𝑠

0

3

4

1

0

0

1

𝑥

3

2

1

0

1

0

1

4𝑅2 − 𝑅1

𝑝

0

−1

−2

0

1

3

1

2𝑅3 + 𝑅1

3𝑅3 + 𝑅2


Solutions Chapter 6 Review 𝑥

𝑦

𝑧

𝑠

𝑡

𝑝

𝑧 𝑥

0 12

3 5

4 0

1 −1

0 4

0 0

1 3

𝑝

0

1

0

1

2

6

3

The solution to the primal problem is 𝑝 = 3∕6 = 1∕2; 𝑥 = 3∕12 = 1∕4, 𝑦 = 0, 𝑧 = 1∕4. So, the column player's optimal strategy is 𝐶 = [1∕2 0 1∕2] 𝑇 and the value of the game is 𝑒 = 2 − 3 = −1. The solution to the dual problem is 𝑠 = 1∕6, 𝑡 = 2∕6 = 1∕3, so the row player's optimal strategy is 𝑅 = [1∕3 0 2∕3]. 25. This game cannot be reduced. Add 𝑘 = 3 to each entry and put 1s to the right and below: 0 1 6 1 4 3 3 1 . 1 5 4 1 1 1 1 0 This gives the following LP problem: Maximize 𝑝 = 𝑥 + 𝑦 + 𝑧 subject to 𝑦 + 6𝑧 ≤ 1, 4𝑥 + 3𝑦 + 3𝑧 ≤ 1, 𝑥 + 5𝑦 + 4𝑧 ≤ 1, 𝑥 ≥ 0, 𝑦 ≥ 0, 𝑧 ≥ 0. Here are the tableaux: 𝑥 𝑦 𝑧 𝑠 𝑡 𝑢 𝑝 𝑠

0

1

6

1

0

0

0

1

𝑡

4

3

3

0

1

0

0

1

𝑢

1

5

4

0

0

1

0

1

4𝑅3 − 𝑅2

𝑝

−1

−1

−1

0

0

0

1

0

4𝑅4 + 𝑅2

𝑥

𝑦

𝑧

𝑠

𝑡

𝑢

𝑝

𝑠

0

1

6

1

0

0

0

1

17𝑅1 − 𝑅3

𝑥

4

3

3

0

1

0

0

1

17𝑅2 − 3𝑅3

𝑢

0

17

13

0

−1

4

0

3

𝑝

0

−1

−1

0

1

0

4

1

𝑥

𝑦

𝑧

𝑠

𝑡

𝑢

𝑝

𝑠

0

0

89

17

1

−4

0

14

𝑥

68

0

12

0

20

−12

0

8

89𝑅2 − 12𝑅1

𝑦

0

17

13

0

−1

4

0

3

89𝑅3 − 13𝑅1

𝑝

0

0

−4

0

16

4

68

20

89𝑅4 + 4𝑅1

17𝑅4 + 𝑅3


Solutions Chapter 6 Review 𝑥

𝑦

𝑧

𝑠

𝑡

𝑧 𝑥 𝑦

0 6052 0

0 0 1513

89 0 0

17

1 −4 −204 1768 −1020 −221 −102 408

𝑝

0

0

0

68

1428

𝑢

340

𝑝 0 0 0

14 544 85

6052

1836

The solution to the primal problem is 𝑝 = 1 836∕6 052 = 27∕89; 𝑥 = 544∕6 052 = 8∕89, 𝑦 = 85∕1 513 = 5∕89, 𝑧 = 14∕89. So, the column player's optimal strategy is 𝐶 = [8∕27 5∕27 14∕27] 𝑇 and the value of the game is 𝑒 = 89∕27 − 3 = 8∕27. The solution to the dual problem is 𝑠 = 68∕6 052 = 1∕89, 𝑡 = 1 428∕6 052 = 21∕89, 𝑢 = 340∕6 052 = 5∕89, so the row player's optimal strategy is 𝑅 = [1∕27 21∕27 5∕27] = [1∕27 7∕9 5∕27]. 26. This game cannot be reduced. Add 𝑘 = 4 to each entry and put 1s to the right and below: 0 2 1 1 5 1 2 1 . 1 5 0 1 1 1 1 0 This gives the following LP problem: Maximize 𝑝 = 𝑥 + 𝑦 + 𝑧 subject to 2𝑦 + 𝑧 ≤ 1, 5𝑥 + 𝑦 + 2𝑧 ≤ 1, 𝑥 + 5𝑦 ≤ 1, 𝑥 ≥ 0, 𝑦 ≥ 0, 𝑧 ≥ 0. Here are the tableaux: 𝑥 𝑦 𝑧 𝑠 𝑡 𝑢 𝑝 𝑠

0

2

1

1

0

0

0

1

𝑡

5

1

2

0

1

0

0

1

𝑢

1

5

0

0

0

1

0

1

5𝑅3 − 𝑅2

𝑝

−1

−1

−1

0

0

0

1

0

5𝑅4 + 𝑅2

𝑥

𝑦

𝑧

𝑠

𝑡

𝑢

𝑝

𝑠

0

2

1

1

0

0

0

1

12𝑅1 − 𝑅3

𝑥

5

1

2

0

1

0

0

1

24𝑅2 − 𝑅3

𝑢

0

24

−2

0

−1

5

0

4

𝑝

0

−4

−3

0

1

0

5

1

6𝑅4 + 𝑅3

𝑥

𝑦

𝑧

𝑠

𝑡

𝑢

𝑝

𝑠

0

0

14

12

1

−5

0

8

25𝑅1 − 7𝑅2

𝑥

120

0

50

0

25

−5

0

20

𝑦

0

24

−2

0

−1

5

0

4

25𝑅3 + 𝑅2

𝑝

0

0

−20

0

5

5

30

10

5𝑅4 + 2𝑅2


Solutions Chapter 6 Review 𝑥

𝑦

𝑧

𝑠

𝑡

𝑢

𝑝

𝑠 𝑧 𝑦

−840 120 120

0 0 600

0 50 0

300 0 0

−150 25 0

−90 −5 120

0 0 0

60 20 120

𝑝

240

0

0

0

75

15

150

90

The solution to the primal problem is 𝑝 = 90∕150 = 3∕5; 𝑥 = 0, 𝑦 = 120∕600 = 1∕5, 𝑧 = 20∕50 = 2∕5. So, the column player's optimal strategy is 𝐶 = [0 1∕3 2∕3] 𝑇 and the value of the game is 𝑒 = 5∕3 − 4 = −7∕3. The solution to the dual problem is 𝑠 = 0, 𝑡 = 75∕150 = 1∕2, 𝑢 = 15∕150 = 1∕10, so the row player's optimal strategy is 𝑅 = [0 5∕6 1∕6]. 27. Apply the simplex method to the given problem: 𝑥 𝑦 𝑠 𝑡 𝑝 *𝑠 *𝑡

−2

1

−1

0

0

1

1

−2

0

−1

0

1

𝑅2 + 2𝑅1

𝑝

1

2

0

0

1

0

𝑅3 − 2𝑅1

𝑥

𝑦

𝑠

𝑡

𝑝

𝑦 *𝑡

−2 −3

1 0

−1 −2

0 −1

0 0

1 3

𝑝

5

0

2

0

1

−2

As there are no positive entries in the 𝑡-row (other than the right-hand side), we cannot go to the next step. Thus, it is impossible to get into the feasible region by completing Phase 1. In other words, there are no feasible solutions (choice A). Another way of seeing this would be to graph the constraints. This would show an empty feasible region. 28. Apply the simplex method to the given problem: 𝑥 𝑦 𝑠 𝑡 𝑝 𝑠

−2

1

1

0

0

1

𝑡

1

−2

0

1

0

2

𝑝

−1

−1

0

0

1

0

𝑥

𝑦

𝑠

𝑡

𝑝

𝑠 𝑥

0 1

−3 −2

1 0

2 1

0 0

5 2

𝑝

0

−3

0

1

1

2

𝑅1 + 2𝑅2 𝑅3 + 𝑅2

There are now no positive entries to use as pivot in the 𝑦-column. This indicates that the objective function is


Solutions Chapter 6 Review unbounded (choice B). Another way of seeing this would be to set up the problem graphically. This would show an unbounded feasible region with nonnegative coefficients of 𝑝. Thus, 𝑝 can be made as large as we like. 29. First note that the objective function, 𝑍 = 𝑥1 + 4𝑥2 + 2𝑥3 − 10, is not linear because of the "−10". However, maximizing 𝑝 = 𝑥1 + 4𝑥2 + 2𝑥3 will also maximize 𝑍, provided we remember to subtract 10 from the optimal value of the objective function when we are done. Thus, we solve the problem using 𝑝 = 𝑥1 + 4𝑥2 + 2𝑥3 as our objective function: 𝑥1 𝑥2 𝑥3 𝑠1 𝑠2 𝑧 𝑠1

4

1

1

1

0

0

45

𝑅1 − 𝑅2

𝑠2

−1

1

2

0

1

0

0

𝑧

−1

−4

−2

0

0

1

0

𝑥1

𝑥2

𝑥3

𝑠1

𝑠2

𝑧

𝑠1

5

0

−1

1

−1

0

45

𝑥2

−1

1

2

0

1

0

0

5𝑅2 + 𝑅1

𝑧

−5

0

6

0

4

1

0

𝑅3 + 𝑅1

𝑥1

𝑥2

𝑥3

𝑠1

𝑠2

𝑧

𝑥1

5

0

−1

1

−1

0

45

𝑥2

0

5

9

1

4

0

45

𝑧

0

0

5

1

3

1

45

𝑅3 + 4𝑅2

Optimal Solution: 𝑝 = 45; 𝑥1 = 45∕5 = 9, 𝑥2 = 45∕5 = 9, 𝑥3 = 0. Since the maximum value of 𝑝 is 45, the maximum value of 𝑍 is 45 − 10 = 35. 30. First note that the objective function, 𝑍 = 𝑥1 + 4𝑥2 + 2𝑥3 + 𝑥4 + 40, is not linear because of the "+ 40". However, minimizing 𝑐 = 𝑥1 + 4𝑥2 + 2𝑥3 + 𝑥4 will also minimize 𝑍, provided we remember to add 40 to the optimal value of the objective function when we are done. Thus, we solve the problem using 𝑐 = 𝑥1 + 4𝑥2 + 2𝑥3 + 𝑥4 as our objective function: 𝑥1

𝑥2

𝑥3

𝑥4

𝑠1

𝑠2

𝑝

𝑠1

4

1

1

0

1

0

0

45

*𝑠2

−1

2

0

1

0

−1

0

40

𝑝

1

4

2

1

0

0

1

0

𝑅3 − 2𝑅2

𝑥1

𝑥2

𝑥3

𝑥4

𝑠1

𝑠2

𝑝

𝑠1

9

0

2

−1

2

1

0

50

𝑅1 + 𝑅2

𝑥2

−1

2

0

1

0

−1

0

40

𝑝

3

0

2

−1

0

2

1

−80

2𝑅1 − 𝑅2

𝑅3 + 𝑅2


Solutions Chapter 6 Review 𝑥1

𝑥2

𝑥3

𝑥4

𝑠1

𝑠2

𝑝

𝑠1

8

2

2

0

2

0

0

90

𝑥4

−1

2

0

1

0

−1

0

40

𝑝

2

2

2

0

0

1

1

−40

Optimal Solution: 𝑐 = 40; 𝑥1 = 0, 𝑥2 = 0, 𝑥3 = 0, 𝑥4 = 40. Thus, 𝑍 = 𝑐 + 40 = 80. 31. Unknowns: 𝑥 = # packages from Duffin House, 𝑦 = # packages from Higgins Press Minimize 𝑐 = 50𝑥 + 80𝑦 subject to 5𝑥 + 5𝑦 ≥ 4 000, 5𝑥 + 10𝑦 ≥ 6 000, 𝑥 ≥ 0, 𝑦 ≥ 0. Corner Point Lines through Point Coordinates 𝑐 = 50𝑥 + 80𝑦 𝐴

5𝑥 + 5𝑦 = 4 000 𝑥=0

(0, 800)

64 000

𝐵

5𝑥 + 5𝑦 = 4 000 5𝑥 + 10𝑦 = 6 000

(400, 400)

52 000

𝐶

5𝑥 + 10𝑦 = 6 000 𝑦=0

(1 200, 0)

60 000

Purchase 400 packages from each for a minimum cost of $52,000. 32. Unknowns: 𝑥 = # packages from McPhearson, 𝑦 = # packages from O'Conell Minimize 𝑐 = 80𝑥 + 90𝑦 subject to 10𝑥 + 10𝑦 ≥ 5 000, 5𝑥 + 10𝑦 ≥ 4 000, 𝑥 ≥ 0, 𝑦 ≥ 0. Corner Point Lines through Point Coordinates 𝑐 = 80𝑥 + 90𝑦 𝐴

10𝑥 + 10𝑦 = 5 000 𝑥=0

(0, 500)

45 000

𝐵

10𝑥 + 10𝑦 = 5 000 5𝑥 + 10𝑦 = 4 000

(200, 300)

43 000

𝐶

5𝑥 + 10𝑦 = 4 000 𝑦=0

(800, 0)

64 000

Solution: Purchase 200 packages from McPhearson and 300 from O'Conell for a minimum cost of $43,000. 33. a. Since the lowest cost is given by ordering the same number of packages from each supplier (point 𝐵 in the feasible region in Exercise 31), changing that by ordering at least 20% more packages from Duffin as from Higgins will result in a higher cost, since the solution will no longer be point 𝐵 but another point in the feasible region (choice B), assuming it is possible to meet all the conditions. On the other hand, it is conceivable that the extra constraint will result in an empty feasible region (choice D). Thus, the correct answers are (B) and (D). b. Unknowns: as in Exercise 31. Minimize 𝑐 = 50𝑥 + 80𝑦 subject to 5𝑥 + 5𝑦 ≥ 4 000, 5𝑥 + 10𝑦 ≥ 6 000, 𝑥 ≥ 1.2𝑦 or 𝑥 − 1.2𝑦 ≥ 0, 𝑥 ≥ 0, 𝑦 ≥ 0.


Solutions Chapter 6 Review Corner Point Lines through Point Coordinates 𝑐 = 50𝑥 + 80𝑦 𝐴

5𝑥 + 10𝑦 = 6 000 𝑥 − 1.2𝑦 = 0

(450, 375)

52 500

𝐵

5𝑥 + 10𝑦 = 6 000 𝑦=0

(1 200, 0)

60 000

Solution: Purchase 450 packages from Duffin House, 375 from Higgins Press for a minimum cost of $52,500. 34. Here is the feasible region for Exercise 32:

The new constraints are 𝑥 + 𝑦 ≤ 500 and 𝑥 ≥ 𝑦. Adding the first removes everything above the line passing through 𝐴 and 𝐵, leaving only the segment 𝐴𝐵 as feasible. However, every point on that segment has 𝑦 > 𝑥, violating the second new constraint. 35. Unknowns: 𝑥 = # shares in EEE, 𝑦 = # shares in RRR Minimize 𝑐 = 2.0𝑥 + 3.0𝑦 subject to 50𝑥 + 55𝑦 ≤ 12 100, 2.25𝑥 + 2.75𝑦 ≥ 550, 𝑥 ≥ 0, 𝑦 ≥ 0. Corner Point Lines through Point Coordinates 𝑐 = 2.0𝑥 + 3.0𝑦 𝐴

𝑥=0 50𝑥 + 55𝑦 = 12 100

(0, 220)

660

𝐵

𝑥=0 2.25𝑥 + 2.75𝑦 = 550

(0, 200)

600

𝐶

50𝑥 + 55𝑦 = 12 100 2.25𝑥 + 2.75𝑦 = 550

(220, 20)

500

Solution: Buy 220 shares of EEE and 20 shares of RRR. The minimum total risk index is 𝑐 = 500. 36. Unknowns: 𝑥 = # shares in CNM, 𝑦 = # shares in SS Minimize 𝑐 = 1.0𝑥 + 1.5𝑦 subject to 40𝑥 + 25𝑦 ≤ 30 000, 2.2𝑥 + 1.875𝑦 ≥ 1 650, 𝑥 ≥ 0, 𝑦 ≥ 0. Corner Point Lines through Point Coordinates 𝑐 = 1.0𝑥 + 1.5𝑦 𝐴

𝑥=0 40𝑥 + 25𝑦 = 30 000

(0, 1 200)

1 800

𝐵

𝑥=0 2.2𝑥 + 1.875𝑦 = 1 650

(0, 880)

1 320

𝐶

40𝑥 + 25𝑦 = 30 000 2.2𝑥 + 1.875𝑦 = 1 650

(750, 0)

750


Solutions Chapter 6 Review Solution: Buy 750 shares of CNM and no shares of SS. The minimum total risk index is 𝑐 = 750. 37. Take 𝑥 = # Sprinkles, 𝑦 = # Storms, 𝑧 = # Hurricanes. Maximize 𝑝 = 𝑥 + 𝑦 + 2𝑧 subject to 𝑥 + 2𝑦 + 2𝑧 ≤ 600, 2𝑥 + 𝑦 + 3𝑧 ≤ 600, 𝑥 + 3𝑦 + 6𝑧 ≤ 600, 𝑥 ≥ 0, 𝑦 ≥ 0, 𝑧 ≥ 0. 𝑥 𝑦 𝑧 𝑠 𝑡 𝑢 𝑝 𝑠

1

2

2

1

0

0

0

600

3𝑅1 − 𝑅3

𝑡

2

1

3

0

1

0

0

600

2𝑅2 − 𝑅3

𝑢

1

3

6

0

0

1

0

600

𝑝

−1

−1

−2

0

0

0

1

0

3𝑅4 + 𝑅3

𝑥

𝑦

𝑧

𝑠

𝑡

𝑢

𝑝

𝑠

2

3

0

3

0

−1

0

1200

3𝑅1 − 2𝑅2

𝑡

3

−1

0

0

2

−1

0

600

𝑧

1

3

6

0

0

1

0

600

3𝑅3 − 𝑅2

𝑝

−2

0

0

0

0

1

3

600

3𝑅4 + 2𝑅2

𝑥

𝑦

𝑧

𝑠

𝑡

𝑢

𝑝

𝑠

0

11

0

9

−4

−1

0

2400

10𝑅1 − 11𝑅3

𝑥

3

−1

0

0

2

−1

0

600

10𝑅2 + 𝑅3

𝑧

0

10

18

0

−2

4

0

1200

𝑝

0

−2

0

0

4

1

9

3000

𝑥

𝑦

𝑧

𝑠

𝑡

𝑢

𝑝

𝑠 𝑥 𝑦

0 30 0

0 0 10

−198 18 18

90 0 0

−18 18 −2

−54 −6 4

0 0 0

10800 7200 1200

𝑝

0

0

18

0

18

9

45

16200

5𝑅4 + 𝑅3

Optimal solution: 𝑝 = 16,200∕45 = 360; 𝑥 = 7,200∕30 = 240, 𝑦 = 1,200∕10 = 120, 𝑧 = 0. Make 240 Sprinkles, 120 Storms, and no Hurricanes. 38. Unknowns: 𝑥 = # paperbacks, 𝑦 = # quality paperbacks, 𝑧 = # hardcovers Maximize 𝑝 = 𝑥 + 2𝑦 + 3𝑧 subject to 3𝑥 + 2𝑦 + 𝑧 ≤ 6 000, 2𝑥 + 𝑦 + 3𝑧 ≤ 6 000, 𝑥 + 𝑦 + 𝑧 ≤ 2 200, 𝑥 ≥ 0, 𝑦 ≥ 0, 𝑧 ≥ 0.


Solutions Chapter 6 Review 𝑥

𝑦

𝑧

𝑠

𝑡

𝑢

𝑝

𝑠

3

2

1

1

0

0

0

6000

𝑡

2

1

3

0

1

0

0

6000

𝑢

1

1

1

0

0

1

0

2200

3𝑅3 − 𝑅2

𝑝

−1

−2

−3

0

0

0

1

0

𝑅4 + 𝑅2

𝑥

𝑦

𝑧

𝑠

𝑡

𝑢

𝑝

𝑠

7

5

0

3

−1

0

0

12000

2𝑅1 − 5𝑅3

𝑧

2

1

3

0

1

0

0

6000

2𝑅2 − 𝑅3

𝑢

1

2

0

0

−1

3

0

600

𝑝

1

−1

0

0

1

0

1

6000

𝑥

𝑦

𝑧

𝑠

𝑡

𝑢

𝑝

𝑠 𝑧 𝑦

9 3 1

0 0 2

0 6 0

6 0 0

3 3 −1

−15 −3 3

0 0 0

21000 11400 600

𝑝

3

0

0

0

1

3

2

12600

3𝑅1 − 𝑅2

2𝑅4 + 𝑅3

Optimal solution: 𝑝 = 12,600∕2 = 6,300; 𝑥 = 0, 𝑦 = 600∕2 = 300, 𝑧 = 11,400∕6 = 1,900. Duffin should print no paperbacks, 300 quality paperbacks, and 1,900 hardcovers for a total daily profit of $6,300. 39. Unknowns: 𝑥 = # packages from Duffin House, 𝑦 = # packages from Higgins Press, 𝑧 = # packages from Ewing Books. Minimize 𝑐 = 50𝑥 + 150𝑦 + 100𝑧 subject to 5𝑥 + 10𝑦 + 5𝑧 ≥ 4 000, 2𝑥 + 10𝑦 + 5𝑧 ≥ 6 000, 𝑦 ≥ 0.5(𝑥 + 𝑦 + 𝑧) or 𝑥 − 𝑦 + 𝑧 ≤ 0, 𝑥 ≥ 0, 𝑦 ≥ 0, 𝑧 ≥ 0𝑧 ≥ 0. Convert the given problem to a maximization problem: Maximize 𝑝 = −50𝑥 − 150𝑦 − 100𝑧 subject to 5𝑥 + 10𝑦 + 5𝑧 ≥ 4 000, 2𝑥 + 10𝑦 + 5𝑧 ≥ 6 000, 𝑥 − 𝑦 + 𝑧 ≤ 0, 𝑥 ≥ 0, 𝑦 ≥ 0, 𝑧 ≥ 0. 𝑥 𝑦 𝑧 𝑠 𝑡 𝑢 𝑝 *𝑠 *𝑡

5

10

5

−1

0

0

0

4000

2

10

5

0

−1

0

0

6000

𝑅2 − 𝑅1

𝑢

1

−1

1

0

0

1

0

0

10𝑅3 + 𝑅1

𝑝

50

150

100

0

0

0

1

0

𝑅4 − 15𝑅1

𝑥

𝑦

𝑧

𝑠

𝑡

𝑢

𝑝

𝑦

5

10

5

−1

0

0

0

4000

𝑅1 + 𝑅2

*𝑡 𝑢

−3

0

0

1

−1

0

0

2000

15

0

15

−1

0

10

0

4000

𝑅3 + 𝑅2

𝑝

−25

0

25

15

0

0

1

−60000

𝑅4 − 15𝑅2


Solutions Chapter 6 Review 𝑥

𝑦

𝑧

𝑠

𝑡

𝑢

𝑝

𝑦 𝑠 𝑢

2

−3 12

10 0 0

5 0 15

0 1 0

−1 −1 −1

0 0 10

0 0 0

6000 2000 6000

𝑝

20

0

25

0

15

0

1

−90000

Optimal solution: 𝑝 = −90,000∕1 = −90,000; 𝑥 = 0, 𝑦 = 6,000∕10 = 600, 𝑧 = 0. Since 𝑐 = −𝑝, the minimum value of c is 90,000. Order 600 packages from Higgins and none from the others, for a total cost of $90,000. 40. Unknowns: 𝑥 = # packages from McPhearson 𝑦 = # packages from O'Conell, 𝑧 = # packages from the U.S. Treasury. Minimize 𝑐 = 50𝑥 + 90𝑦 + 120𝑧 subject to 10𝑥 + 10𝑦 ≥ 5 000, 10𝑦 + 20𝑧 ≥ 4 000, 2𝑥 + 2𝑦 − 𝑧 ≤ 0, 𝑥 ≥ 0, 𝑦 ≥ 0, 𝑧 ≥ 0. 𝑥 𝑦 𝑧 𝑠 𝑡 𝑢 𝑝 𝑅1 − 5𝑅3

*𝑠 *𝑡

10

10

0

−1

0

0

0

5000

0

10

20

0

−1

0

0

4000

𝑢

2

2

−1

0

0

1

0

0

𝑝

50

90

120

0

0

0

1

0

𝑅4 − 25𝑅3

𝑥

𝑦

𝑧

𝑠

𝑡

𝑢

𝑝

*𝑠 *𝑡

0

0

5

−1

0

−5

0

5000

4𝑅1 − 𝑅2

0

10

20

0

−1

0

0

4000

𝑥

2

2

−1

0

0

1

0

0

20𝑅3 + 𝑅2

𝑝

0

40

145

0

0

−25

1

0

4𝑅4 − 29𝑅2

𝑥

𝑦

𝑧

𝑠

𝑡

𝑢

𝑝

*𝑠 𝑧

0

−10

0

−4

1

−20

0

16000

0

10

20

0

−1

0

0

4000

𝑅2 + 𝑅1

𝑥

40

50

0

0

−1

20

0

4000

𝑅3 + 𝑅1

𝑝

0

−130

0

0

29

−100

4

−116000

𝑅4 − 29𝑅1

𝑥

𝑦

𝑧

𝑠

𝑡

𝑢

𝑝

𝑡 𝑧 𝑥

0 0 40

−10 0 40

0 20 0

−4 −4 −4

1 0 0

−20 −20 0

0 0 0

16000 20000 20000

𝑝

0

160

0

116

0

480

4

−580000

Optimal solution: 𝑝 = −580,000∕4 = −145,000; 𝑥 = 20,000∕40 = 500, 𝑦 = 0, 𝑧 = 20,000∕20 = 1,000. Since 𝑐 = −𝑝, the minimum value of c is 145,000. Order 500 packages from McPhearson, none from O'Conell, and 1,000 from the U.S. Treasury for a total cost of $145,000.


Solutions Chapter 6 Review 41. Take 𝑥 = the number of credits of Sciences, 𝑦 = the number of credits of Fine Arts, 𝑧 = the number of credits of Liberal Art, 𝑤 = the number of credits of Mathematics. Given information: (1) The total number of credits is at least 120: 𝑥 + 𝑦 + 𝑧 + 𝑤 ≥ 120. (2) At least as many Science credits as Fine Arts credits: 𝑥 ≥ 𝑦, or 𝑥 − 𝑦 ≥ 0. (3) At most twice as many Mathematics credits as Science credits. Rephrasing: The number of Mathematics credits is at most twice the number of Science credits: 𝑤 ≤ 2𝑥 ⇒ −2𝑥 + 𝑤 ≤ 0. (4) Liberal Arts credits exceed Mathematics credits by no more than one third of the number of Fine Arts credits. Rephrasing: The number of Liberal Arts credits minus the number of Mathematics credits is at most one third of the number of Fine Arts credits: 𝑧 − 𝑤 ≤ 1 ⁄ 3 𝑦 ⇒ 3𝑧 − 3𝑤 ≤ 𝑦 ⇒ −𝑦 + 3𝑧 − 3𝑤 ≤ 0 Thus, the linear programming problem is: Minimize 𝑐 = 300𝑥 + 300𝑦 + 200𝑧 + 200𝑤 subject to 𝑥 + 𝑦 + 𝑧 + 𝑤 ≥ 120, 𝑥 − 𝑦 ≥ 0, −2𝑥 + 𝑤 ≤ 0, −𝑦 + 3𝑧 − 3𝑤 ≤ 0, 𝑥 ≥ 0, 𝑦 ≥ 0, 𝑧 ≥ 0, 𝑤 ≥ 0. b. Using technology (Website → On Line Utilities → Simplex Method Tool) We obtain the following solution: 𝑐 = 26 400; 𝑥 = 24, 𝑦 = 0, 𝑧 = 48, 𝑤 = 48. Billy-Sean should take the following combination: Sciences: 24 credits, Fine Arts: no credits, Liberal Arts: 48 credits, Mathematics: 48 credits, for a total cost of $26,400. 42. a. Take 𝑥 = the number of credits of Liberal Arts, 𝑦 = the number of credits of Sciences, 𝑧 = the number of credits of Verbal Expression, 𝑤 = the number of credits of Mathematics. Given information: (1) The total number of credits is at least 120: 𝑥 + 𝑦 + 𝑧 + 𝑤 ≥ 120. (2) At most as many Science as Liberal Arts credits: 𝑦 ≤ 𝑥, or −𝑥 + 𝑦 ≤ 0. (3) At least twice as many Verbal Expression credits as Science credits and Fine Arts credits combined. Rephrasing: The number of Verbal Expression credits is at least twice the sum of the numbers of Science and Liberal Arts credits: 𝑧 ≥ 2(𝑥 + 𝑦) ⇒ 2𝑥 + 2𝑦 − 𝑧 ≤ 0. (4) Liberal Arts credits exceed Mathematics credits by at least one quarter of the number of Verbal Expression Credits. Rephrasing: The number of Liberal Arts credits minus the number of Mathematics credits is at least one quarter of the number of Verbal Expression credits: 𝑥 − 𝑤 ≥ 1 ⁄ 4 𝑧 ⇒ 𝑥 − 1 ⁄ 4 𝑧 − 𝑤 ≥ 0. Thus, the linear programming problem is: Minimize 𝑐 = 400𝑥 + 300𝑦 + 400𝑧 + 400𝑤 subject to 𝑥 + 𝑦 + 𝑧 + 𝑤 ≥ 120, −𝑥 + 𝑦 ≤ 0, 2𝑥 + 2𝑦 − 𝑧 ≤ 0, 𝑥 − (1∕4)𝑧 − 𝑤 ≥ 0, 𝑥 ≥ 0, 𝑦 ≥ 0, 𝑧 ≥ 0, 𝑤 ≥ 0. b. Using technology (Website → On Line Utilities → Simplex Method Tool) We obtain the following solution: 𝑐 = 46 000; 𝑥 = 20, 𝑦 = 20, 𝑧 = 80, 𝑤 = 0. Solution: Billy-Sean is now forced to take exactly the following combination: Liberal Arts: 20 credits, Sciences: 20 credits, Verbal Expression: 80 credits, Mathematics: 0 credits for a total cost of $46,000! 43. Unknowns: 𝑥 = # packages from New York to O'Hagan.com, 𝑦 = # packages form New York to FantasyBooks.com, 𝑧 = # packages from Illinois to O'Hagan.com, 𝑤 = # packages form Illinois to FantasyBooks.com Minimize 𝑐 = 20𝑥 + 50𝑦 + 30𝑧 + 40𝑤 subject to 𝑥 + 𝑦 ≤ 600, 𝑧 + 𝑤 ≤ 300, 𝑥 + 𝑧 ≥ 600, 𝑦 + 𝑤 ≥ 200, 𝑥 ≥ 0, 𝑦 ≥ 0, 𝑧 ≥ 0, 𝑤 ≥ 0. 𝑥 𝑦 𝑧 𝑤 𝑠 𝑡 𝑢 𝑣 𝑝 𝑠

1

1

0

0

1

0

0

0

0

600

𝑡

0

0

1

1

0

1

0

0

0

300

*𝑢 *𝑣

1

0

1

0

0

0

−1

0

0

600

0

1

0

1

0

0

0

−1

0

200

𝑝

20

50

30

40

0

0

0

0

1

0

𝑅1 − 𝑅3

𝑅5 − 20𝑅3


Solutions Chapter 6 Review 𝑥

𝑦

𝑧

𝑤

𝑠

𝑡

𝑢

𝑣

𝑝

𝑠

0

1

−1

0

1

0

1

0

0

0

𝑡 𝑥

0 1

0 0

1 1

1 0

0 0

1 0

0 −1

0 0

0 0

300 600

*𝑣

0

1

0

1

0

0

0

−1

0

200

𝑅4 − 𝑅1

𝑝

0

50

10

40

0

0

20

0

1

−12000

𝑅5 − 50𝑅1

𝑥

𝑦

𝑧

𝑤

𝑠

𝑡

𝑢

𝑣

𝑝

𝑦

0

1

−1

0

1

0

1

0

0

0

𝑅1 + 𝑅4

𝑡

0

0

1

1

0

1

0

0

0

300

𝑅2 − 𝑅4

𝑥

1

0

1

0

0

0

−1

0

0

600

𝑅3 − 𝑅4

*𝑣

0

0

1

1

−1

0

−1

−1

0

200

𝑝

0

0

60

40

−50

0

−30

0

1

−12000

𝑥

𝑦

𝑧

𝑤

𝑠

𝑡

𝑢

𝑣

𝑝

𝑦

0

1

0

1

0

0

0

−1

0

200

𝑡

0

0

0

0

1

1

1

1

0

100

𝑥

1

0

0

−1

1

0

0

1

0

400

𝑅3 + 𝑅1

𝑧

0

0

1

1

−1

0

−1

−1

0

200

𝑅4 − 𝑅1

𝑝

0

0

0

−20

10

0

30

60

1

−24000

𝑅5 + 20𝑅1

𝑥

𝑦

𝑧

𝑤

𝑠

𝑡

𝑢

𝑣

𝑝

𝑤 𝑡 𝑥 𝑧

0 0 1 0

1 0 1 −1

0 0 0 1

1 0 0 0

0 1 1 −1

0 1 0 0

0 1 0 −1

−1 1 0 0

0 0 0 0

200 100 600 0

𝑝

0

20

0

0

10

0

30

40

1

−20000

𝑅5 − 60𝑅4

Optimal solution: 𝑝 = −20,000∕1 = −20,000; 𝑥 = 600∕1 = 600, 𝑦 = 0, 𝑧 = 0∕1 = 0, 𝑤 = 200∕1 = 200. Since 𝑐 = −𝑝, the minimum value of c is 20,000. 44. Unknowns: 𝑥 = # people you fly from Austin to Houston, 𝑦 = # people you fly from Austin to Cleveland, 𝑧 = # people you fly from San Diego to Houston, 𝑤 = # people you fly from San Diego to Cleveland Minimize 𝑐 = 200𝑥 + 150𝑦 + 400𝑧 + 200𝑤 subject to 𝑥 + 𝑦 ≤ 25, 𝑧 + 𝑤 ≤ 10, 𝑥 + 𝑧 ≥ 15, 𝑦 + 𝑤 ≥ 15, 𝑥 ≥ 0, 𝑦 ≥ 0, 𝑧 ≥ 0, 𝑤 ≥ 0.


Solutions Chapter 6 Review 𝑥

𝑦

𝑧

𝑤

𝑠

𝑡

𝑢

𝑣

𝑝

𝑠

1

1

0

0

1

0

0

0

0

25

𝑡

0

0

1

1

0

1

0

0

0

10

*𝑢 *𝑣

1

0

1

0

0

0

−1

0

0

15

0

1

0

1

0

0

0

−1

0

15

𝑝

200

150

400

200

0

0

0

0

1

0

𝑥

𝑦

𝑧

𝑤

𝑠

𝑡

𝑢

𝑣

𝑝

𝑠

0

1

−1

0

1

0

1

0

0

10

𝑡 𝑥

0 1

0 0

1 1

1 0

0 0

1 0

0

−1

0 0

0 0

10 15

*𝑣

0

1

0

1

0

0

0

−1

0

15

𝑝

0

150

200

200

0

0

200

0

1

−3000

𝑥

𝑦

𝑧

𝑤

𝑠

𝑡

𝑢

𝑣

𝑝

𝑦

0

1

−1

0

1

0

1

0

0

10

𝑅1 + 𝑅4

𝑡

0

0

1

1

0

1

0

0

0

10

𝑅2 − 𝑅4

𝑥

1

0

1

0

0

0

−1

0

0

15

𝑅3 − 𝑅4

*𝑣

0

0

1

1

−1

0

−1

−1

0

5

𝑝

0

0

350

200

−150

0

50

0

1

−4500

𝑅5 − 350𝑅4

𝑥

𝑦

𝑧

𝑤

𝑠

𝑡

𝑢

𝑣

𝑝

𝑦

0

1

0

1

0

0

0

−1

0

15

𝑅1 − 𝑅4

𝑡

0

0

0

0

1

1

1

1

0

5

𝑥

1

0

0

−1

1

0

0

1

0

10

𝑧

0

0

1

1

−1

0

−1

−1

0

5

𝑝

0

0

0

−150

200

0

400

350

1

−6250

𝑥

𝑦

𝑧

𝑤

𝑠

𝑡

𝑢

𝑣

𝑝

𝑦 𝑡 𝑥 𝑤

0 0 1 0

1 0 0 0

−1 0 1 1

0 0 0 1

1 1 0 −1

0 1 0 0

1 1 −1 −1

0 1 0 −1

0 0 0 0

10 5 15 5

𝑝

0

0

150

0

50

0

250

200

1

−5500

𝑅1 − 𝑅3

𝑅5 − 200𝑅3

𝑅4 − 𝑅1 𝑅5 − 150𝑅1

𝑅3 + 𝑅4 𝑅5 + 150𝑅4

Optimal solution: 𝑝 = −5,500∕1 = −5,500; 𝑥 = 15∕1 = 15, 𝑦 = 10∕1 = 10, 𝑧 = 0, 𝑤 = 5∕1 = 5. Since 𝑐 = −𝑝, the minimum value of c is 5,500. Fly 15 people from Austin to Houston, 10 from Austin to Cleveland, none from San Diego to Houston, 5 from San Diego to Cleveland at a total cost of $5,500.


Solutions Chapter 6 Review 45. First reduce the game: We can eliminate O'Hagan's option of offering no promotion, and then we can eliminate FantasyBook's no promotion option. The entries in the remaining payoff matrix are nonnegative, so we put 1s to the right and below: 20 10 15 1 0 15 10 1 . 1 1 1 0 This gives the following LP problem: Maximize 𝑝 = 𝑥 + 𝑦 + 𝑧 subject to 20𝑥 + 10𝑦 + 15𝑧 ≤ 1, 15𝑦 + 10𝑧 ≤ 1, 𝑥 ≥ 0, 𝑦 ≥ 0, 𝑧 ≥ 0. Here are the tableaus: 𝑥 𝑦 𝑧 𝑠 𝑡 𝑝 𝑠

20

10

15

1

0

0

1

𝑡

0

15

10

0

1

0

1

𝑝

−1

−1

−1

0

0

1

0

20𝑅3 + 𝑅1

𝑥

𝑦

𝑧

𝑠

𝑡

𝑝

𝑥

20

10

15

1

0

0

1

3𝑅1 − 2𝑅2

𝑡

0

15

10

0

1

0

1

𝑝

0

−10

−5

1

0

20

1

𝑥

𝑦

𝑧

𝑠

𝑡

𝑝

𝑥 𝑦

60 0

0 15

25 10

3 0

−2 1

0 0

1 1

𝑝

0

0

5

3

2

60

5

3𝑅3 + 2𝑅2

The solution to the primal problem is 𝑝 = 5∕60 = 1∕12; 𝑥 = 1∕60, 𝑦 = 1∕15, 𝑧 = 0. So, FantasyBook's optimal strategy is 𝐶 = [0 1∕5 4∕5 0] 𝑇 and the value of the game is 𝑒 = 12. The solution to the dual problem is 𝑠 = 3∕60 = 1∕20, 𝑡 = 2∕60 = 1∕30, so O'HaganBook's optimal strategy is 𝑅 = [0 3∕5 2∕5]. 46. This game does not reduce by dominance, so we solve it via the following associated LP problem: Maximize 𝑝 = 𝑥 + 𝑦 + 𝑧 subject to 30𝑥 + 20𝑧 ≤ 1, 70𝑦 ≤ 1, 70𝑧 ≤ 1, 𝑥 ≥ 0, 𝑦 ≥ 0, 𝑧 ≥ 0. Here are the tableaus: 𝑥 𝑦 𝑧 𝑠 𝑡 𝑢 𝑝 𝑠

30

0

20

1

0

0

0

1

𝑡 𝑢

0 0

70 0

0 70

0 0

1 0

0 1

0 0

1 1

𝑝

−1

−1

−1

0

0

0

1

0

30𝑅4 + 𝑅1


Solutions Chapter 6 Review 𝑥

𝑦

𝑧

𝑠

𝑡

𝑢

𝑝

𝑥

30

0

20

1

0

0

0

1

𝑡

0

70

0

0

1

0

0

1

𝑢

0

0

70

0

0

1

0

1

𝑝

0

−30

−10

1

0

0

30

1

7𝑅4 + 3𝑅2

𝑥

𝑦

𝑧

𝑠

𝑡

𝑢

𝑝

𝑥

30

0

20

1

0

0

0

1

7𝑅1 − 2𝑅3

𝑦

0

70

0

0

1

0

0

1

𝑢

0

0

70

0

0

1

0

1

𝑝

0

0

−70

7

3

0

210

10

𝑥

𝑦

𝑧

𝑠

𝑡

𝑢

𝑝

𝑥 𝑦 𝑧

210 0 0

0 70 0

0 0 70

7 0 0

0 1 0

−2 0 1

0 0 0

5 1 1

𝑝

0

0

0

7

3

1

210

11

𝑅4 + 𝑅3

The solution to the primal problem is: 𝑝 = 11∕210; 𝑥 = 5∕210, 𝑦 = 1∕70, 𝑧 = 1∕70. All that is required is Pat's strategy (the row strategy). The solution to the dual problem is 𝑠 = 7∕210, 𝑡 = 3∕210, 𝑢 = 1∕210, so his strategy is 𝑅 = [7∕11 3∕11 1∕11] ≈ [0.636 0.273 0.091] and 𝑒 = 210∕11 ≈ 19.1. Pat should study game theory 63.6% of the night, linear programming 27.3% of the night, and matrix algebra the rest of the night for an expected score of 19.1%.


Solutions Chapter 6 Case Study Chapter 6 Case Study 1. Note: To solve the LP problems in this section, enter them in the online Simplex Method Tool. When entering the problems, there is no need to enter the inequalities 𝑥1 ≥ 0, 𝑥2 ≥ 0, 𝑒𝑡𝑐. When the objective is to minimize cost (Scenario 1), note that tuna does not appear either. The protein sources that do appear in the optimal solutions supply nutrients not supplied by chicken, such as carbs, fiber, and vitamin C. Including enough of those other foods to supply these missing nutrients ends up supplying sufficient protein as well. When the objective is to minimize caloric intake (Scenario 2) and cost is not a factor, tuna is selected over chicken due to the fact that each serving of chicken supplies more than twice the calories of a serving of tuna, but less than twice the protein, so tuna can supply the necessary protein with fewer calories. 2. The constraint corresponding to the protein requirement is the next-to-last one: 9.4𝑥1 + 42.2𝑥2 + 8.2𝑥3 + 1𝑥4 + 1.2𝑥5 + 2.2𝑥6 + 7𝑥7 + 6.1𝑥8 + 7.7𝑥9 + 22.7𝑥10 ≥ 60. Increasing the protein requirement 10 grams at a time corresponds to changing the right-hand-side to 70, 80, 90, ... Chicken first becomes necessary when the protein requirement reaches 110 g. 3. 𝑥2 ≤ 2, 𝑥4 ≤ 0, 𝑥9 ≤ 0, resulting in a 799-calorie diet of tofu, wheat bread, and tuna: 𝑥1 = 2.99152, 𝑥2 = 0, 𝑥3 = 0, 𝑥4 = 0, 𝑥5 = 6.71351, 𝑥6 = 0, 𝑥7 = 0, 𝑥8 = 0, 𝑥9 = 0, 𝑥10 = 1.04949. 4. Maximize 𝑃 = 9.4𝑥1 + 42.2𝑥2 + 8.2𝑥3 + 1𝑥4 + 1.2𝑥5 + 2.2𝑥6 + 7𝑥7 + 6.1𝑥8 + 7.7𝑥9 + 22.7𝑥10 Subject to 0.31𝑥1 + 0.84𝑥2 + 0.78𝑥3 + 0.27𝑥4 + 0.15𝑥5 + 0.05𝑥6 + 0.25𝑥7 + 0.82𝑥8 + 0.07𝑥9 + 0.69𝑥10 ≤ 6 88.2𝑥1 + 277.4𝑥2 + 358.2𝑥3 + 25.8𝑥4 + 61.6𝑥5 + 65𝑥6 + 112.7𝑥7 + 145.1𝑥8 + 188.5𝑥9 + 115.6𝑥10 ≥ 2, 200 5.5𝑥1 + 10.8𝑥2 + 12.3𝑥3 + 0.4𝑥4 + 0.2𝑥5 + 1𝑥6 + 9.3𝑥7 + 2.3𝑥8 + 16𝑥9 + 2.1𝑥10 ≥ 20 2.2𝑥1 + 58.3𝑥3 + 5.7𝑥4 + 15.4𝑥5 + 12.4𝑥6 + 0.4𝑥7 + 25.3𝑥8 + 6.9𝑥9 ≤ 60 1.4𝑥1 + 11.6𝑥3 + 1.4𝑥4 + 3.1𝑥5 + 1.3𝑥6 + 4𝑥8 + 2.1𝑥9 ≥ 25 0.1𝑥1 + 27.9𝑥3 + 23.5𝑥4 + 69.7𝑥5 ≥ 90. Optimal Solution: 𝑃 = 199.804; 𝑥1 = 12.6542, 𝑥2 = 0.930915, 𝑥3 = 0, 𝑥4 = 0, 𝑥5 = 1.27309, 𝑥6 = 0, 𝑥7 = 3.9721, 𝑥8 = 0, 𝑥9 = 1.58933, 𝑥10 = 0 That is, the diet consists of 12.7 servings of tofu, 0.93 servings of chicken, 1.27 servings of oranges, 3.97 servings of cheddar cheese, and 1.59 servings of peanut butter, and provides 200 g of protein. 5. No; including the constraints 𝑥6 ≤ 0 and 𝑥9 ≤ 0 in Scenario 1 results in a diet costing at least $4.80 per day.


Solutions Section 7.1 Section 7.1

1. The elements of are the four seasons: spring, summer, fall, winter. Thus, = {spring, summer, fall, winter}. 2. The elements of are the authors of this book: Waner, Costenoble. Thus, = {Waner, Costenoble}.

3. The elements of are all the positive integers no greater than 6, namely 1, 2, 3, 4, 5, 6. Thus, = {1, 2, 3, 4, 5, 6}.

4. The elements of are all the negative integers greater than 3, namely 2, 1. Thus, = { 2, 1} 5. = { | is a positive integer and 0 < < 3} = {1, 2, 3} (Note that 0 is not positive, so we exclude it.) 6. = { | is a positive integer and 0 < < 8} = {1, 2, 3, 4, 5, 6, 7} 7. = { | is an even positive integer and 0 < < 8} = {2, 4, 6, 8} 8. = { | is an odd positive integer and 0 < < 8} = {1, 3, 5, 7}

9. a. If the coins are distinguishable, = {(H, H), (H, T), (T, H), (T, T)}. (Note that (H, T) and (T, H) are different outcomes, since the first coin is distinguished from the second.) b. If the coins are indistinguishable, then (H, T) and (T, H) are the same outcome, and so = {(H, H), (H, T), (T, T)}. 10. a. If the coins are distinguishable, = {(H, H, H), (H, H, T), (H, T, H), (H, T, T), (T, H, H), (T, H, T), (T, T, H), (T, T, T)}. (Note that, for example, (H, H, T) and (T, H, T) are different outcomes when the coins are distinguishable.) b. If the coins are indistinguishable, then the outcomes are characterized only by the number of heads and tails, so = {(H, H, H), (H, H, T), (H, T, T), (T, T, T)}. 11. If the dice are distinguishable, then the outcomes can be thought of as ordered pairs. Thus, since the numbers add to 6, = {(1, 5), (2, 4), (3, 3), (4, 2), (5, 1)}. 12. If the dice are distinguishable, then the outcomes can be thought of as ordered pairs. Thus, since the numbers add to 8, = {(2, 6), (3, 5), (4, 4), (5, 3), (6, 2)}. 13. If the dice are indistinguishable, then the outcomes are characterized only by the numbers that come up, and not by the order. Thus, since the numbers add to 6, = {(1, 5), (2, 4), (3, 3)}. 14. If the dice are indistinguishable, then the outcomes are characterized only by the numbers that come up, and not by the order. Thus, since the numbers add to 8, = {(2, 6), (3, 5), (4, 4)}. 15. As the numbers facing up can never add to 13 (the largest sum is 12), there are no such outcomes. In other words, = . 16. As the numbers facing up can never add to 1 (the smallest sum is 2), there are no such outcomes. In


other words, =

.

Solutions Section 7.1

17.

18.

19.

20.

21. = {Romeo, Juliet, Anthony, Cleopatra, Gandhi, Mandela}, = {Juliet, Mandela, Cleopatra} is the set of all elements that are in or (or both): Romeo, Juliet, Anthony, Cleopatra, Gandhi, Mandela. Thus, = {Romeo, Juliet, Anthony, Cleopatra, Gandhi, Mandela } = .

22. = {Romeo, Juliet, Anthony, Cleopatra, Gandhi, Mandela}, = {Isabella, Nureyev, Fonteyn, Elizabeth, Mandela} = {Romeo, Juliet, Anthony, Cleopatra, Gandhi, Mandela}, = {Juliet, Mandela, Cleopatra}, = {Isabella, Nureyev, Fonteyn, Elizabeth, Mandela} is the set of all elements that are in or (or both): Romeo, Juliet, Anthony, Cleopatra, Gandhi, Mandela, Isabella, Nureyev, Fonteyn, Elizabeth. Thus, = {Romeo, Juliet, Anthony, Cleopatra, Gandhi, Mandela, Isabella, Nureyev, Fonteyn, Elizabeth}. 23. = {Romeo, Juliet, Anthony, Cleopatra, Gandhi, Mandela} is the set of all elements that are in or (or both). Since has no elements, every set . 24. = {Juliet, Mandela, Cleopatra} is the set of all elements that are in or (or both). Since has no elements, every set .

= for = for


Solutions Section 7.1 25. = {Romeo, Juliet, Anthony, Cleopatra, Gandhi, Mandela}, = {Juliet, Mandela, Cleopatra}, = {Isabella, Nureyev, Fonteyn, Elizabeth, Mandela} = {Juliet, Mandela, Cleopatra} {Isabella, Nureyev, Fonteyn, Elizabeth, Mandela} = {Juliet, Cleopatra, Isabella, Nureyev, Fonteyn, Elizabeth, Mandela} Therefore, ( ) = {Romeo, Juliet, Anthony, Cleopatra, Gandhi, Mandela } {Juliet, Cleopatra, Isabella, Nureyev, Fonteyn, Elizabeth, Mandela}= {Romeo, Juliet, Anthony, Cleopatra, Gandhi, Mandela, Isabella, Nureyev, Fonteyn, Elizabeth}. 26. = {Romeo, Juliet, Anthony, Cleopatra, Gandhi, Mandela}, = {Juliet, Mandela, Cleopatra}, = {Isabella, Nureyev, Fonteyn, Elizabeth, Mandela} = {Romeo, Juliet, Anthony, Cleopatra, Gandhi, Mandela} Therefore, ( ) = {Romeo, Juliet, Anthony, Cleopatra, Gandhi, Mandela } {Isabella, Nureyev, Fonteyn, Elizabeth, Mandela} = {Romeo, Juliet, Anthony, Cleopatra, Gandhi, Mandela, Isabella, Nureyev, Fonteyn, Elizabeth}. 27. = {Juliet, Mandela, Cleopatra}, = {Isabella, Nureyev, Fonteyn, Elizabeth, Mandela} is the set of all elements that are common to both and . Thus, = {Mandela}

28. = {Romeo, Juliet, Anthony, Cleopatra, Gandhi, Mandela}, = {Isabella, Nureyev, Fonteyn, Elizabeth, Mandela} is the set of all elements that are common to both and . Thus, = {Mandela}. 29. = {Romeo, Juliet, Anthony, Cleopatra, Gandhi, Mandela} is the set of all elements that are common to both and . Since has no elements, for every set . 30. = {Juliet, Mandela, Cleopatra} is the set of all elements that are common to both and . Since has no elements, for every set .

= =

31. = {Romeo, Juliet, Anthony, Cleopatra, Gandhi, Mandela}, = {Juliet, Mandela, Cleopatra}, = {Isabella, Nureyev, Fonteyn, Elizabeth, Mandela} = {Juliet, Mandela, Cleopatra} Therefore, ( ) = {Juliet, Mandela, Cleopatra } {Isabella, Nureyev, Fonteyn, Elizabeth, Mandela } = {Mandela}.

32. = {Juliet, Mandela, Cleopatra}, = {Isabella, Nureyev, Fonteyn, Elizabeth, Mandela} = {Mandela} Therefore, ( ) = {Romeo, Juliet, Anthony, Cleopatra, Gandhi, Mandela } {Mandela } = {Mandela}.

33. = {Romeo, Juliet, Anthony, Cleopatra, Gandhi, Mandela}, = {Juliet, Mandela, Cleopatra}, = {Isabella, Nureyev, Fonteyn, Elizabeth, Mandela} = {Juliet, Mandela, Cleopatra} Therefore, ( ) = {Juliet, Mandela, Cleopatra } {Isabella, Nureyev, Fonteyn, Elizabeth, Mandela} = {Juliet, Cleopatra, Mandela, Isabella, Nureyev, Fonteyn, Elizabeth}.


Solutions Section 7.1 34. = {Romeo, Juliet, Anthony, Cleopatra, Gandhi, Mandela}, = {Juliet, Mandela, Cleopatra}, = {Isabella, Nureyev, Fonteyn, Elizabeth, Mandela} = {Mandela} Therefore, ( ) = {Romeo, Juliet, Anthony, Cleopatra, Gandhi, Mandela } {Mandela } = .

35. = {Juliet, Mandela, Cleopatra} and = {Isabella, Nureyev, Fonteyn, Elizabeth, Mandela}, = {Mandela} Therefore, ( ) = {Romeo, Juliet, Anthony, Cleopatra, Gandhi, Elizabeth, Isabella, Nureyev, Fonteyn, Zelda}.

36. = {Juliet, Mandela, Cleopatra} and = {Isabella, Nureyev, Fonteyn, Elizabeth, Mandela}, = {Juliet, Mandela, Cleopatra, Elizabeth, Isabella, Nureyev, Fonteyn} Therefore, ( ) = {Romeo, Anthony, Gandhi, Zelda}.

37. = {Juliet, Mandela, Cleopatra} and = {Isabella, Nureyev, Fonteyn, Elizabeth, Mandela}, = {Romeo, Anthony, Gandhi, Elizabeth, Isabella, Nureyev, Fonteyn, Zelda}, = {Romeo, Juliet, Anthony, Cleopatra, Gandhi, Zelda} Therefore, = {Romeo, Juliet, Anthony, Cleopatra, Gandhi, Elizabeth, Isabella, Nureyev, Fonteyn, Zelda}. 38. = {Juliet, Mandela, Cleopatra} and = {Isabella, Nureyev, Fonteyn, Elizabeth, Mandela}, = {Romeo, Anthony, Gandhi, Elizabeth, Isabella, Nureyev, Fonteyn, Zelda}, = {Romeo, Juliet, Anthony, Cleopatra, Gandhi, Zelda} Therefore, = {Romeo, Anthony, Gandhi, Zelda} 39.

40.

= { | is an integer with 0 5 or is an even integer with 0 10} = {0, 1, 2, 3, 4, 5, 6, 8, 10}. = { | is an integer with 0 5 or is an odd integer with 0 10} = {0, 1, 2, 3, 4, 5, 7, 9}. = { | is an even integer and 0 5} = {0, 2, 4}.

41. 42. 43. 44.

= { | is an odd integer and 0 5} = {1, 3, 5}.

= { | is an even integer and 5 < 10} = {6, 8, 10}.

= { | is an odd integer and 0 5} = {1, 3, 5}.

45. = {small, medium, large }, = {triangle, square} × is the set of all ordered pairs ( , ) with and : Therefore, × = {(small, triangle), (small, square), (medium, triangle), (medium, square), (large, triangle), (large, square)}. 46. = {blue, green }, = {triangle, square} × is the set of all ordered pairs ( , ) with and : Therefore, × = {(blue, triangle), (blue, square), (green, triangle), (green, square)}. 47. = {small, medium, large}, = {blue, green}


Solutions Section 7.1 × is the set of all ordered pairs ( , ) with and : Therefore, × = {(small, blue), (small, green), (medium, blue), (medium, green), (large, blue), (large, green)}. 48. = {small, medium, large}, = {blue, green}, = {triangle, square} The elements of × × are the ordered triples ( , , ) with , , and .. Therefore, × = {(small, blue, triangle), (small, blue, square), (small, green, triangle), (small, green, square), (medium, blue, triangle), (medium, blue, square), (medium, green, triangle), (medium, green, square), (large, blue, triangle), (large, blue, square), (large, green, triangle), (large, green, square)}.

49. = {blue, green}, = {triangle, square} To represent × we use the elements of = {blue, green} for the row headings, and the elements of = {triangle, square} for the column headings:

50. = {small, medium, large}, = {triangle, square} To represent × we use the elements of = {small, medium, large} for the row headings, and the elements of = {triangle, square} for the column headings:

51. = {small, medium, large}, = {blue, green} To represent × we use the elements of = {small, medium, large} for the row headings, and the elements of = {blue, green} for the column headings:

52. = {small, medium, large} To represent × we use the elements of = {small, medium, large} for both the row and column headings:

53. If a die is rolled and a coin is tossed, each outcome is a pair ( , ), where is an outcome when a die is rolled and is an outcome when a coin is tossed. Thus, the set of outcomes is × = {1H, 1T, 2H, 2T, 3H, 3T, 4H, 4T, 5H, 5T, 6H, 6T}.


Solutions Section 7.1 54. If a coin is tossed twice, each outcome is a pair ( 1 , 2 ), where 1 and 2 are outcomes when a coin is tossed. Thus, the set of outcomes is × = {HH, HT, TH, TT}. 55. If a coin is tossed 3 times, each outcome is a triple ( 1 , 2 , 3 ), where 1 , 2 , and 3 are outcomes when a coin is tossed. Thus, the set of outcomes is × × = {HHH, HHT, HTH, HTT, THH, THT, TTH, TTT}.

56. If a coin is tossed twice and then a die is rolled, each outcome is a triple ( 1 , 2 , ), where 1 and 2 are outcomes when a coin is tossed and is an outcome when a die is rolled. Thus, the set of outcomes is × × = {HH1, HH2, HH3, HH4, HH5, HH6, HT1, HT2, HT3, HT4, HT5, HT6, TH1, TH2, TH3, TH4, TH5, TH6, TT1, TT2, TT3, TT4, TT5, TT6}. 57. is the set of outcomes in which at least one die shows an even number. Thus, outcomes in which both dice show an odd number: = {(1, 1), (1, 3), (1, 5), (3, 1), (3, 3), (3, 5), (5, 1), (5, 3), (5, 5)}.

is the set of

58. is the subset of outcomes in which at least one die shows an odd number. Thus, is the set of outcomes in which both dice show an even number: = {(2, 2), (2, 4), (2, 6), (4, 2), (4, 4), (4, 6), (6, 2), (6, 4), (6, 6)}.

59. is the set of outcomes in which either at least one die is even, or at least one is odd. Since this includes all the outcomes (in every outcome there is either an odd or even outcome), its complement is empty: ( ) = .

60. is the set of outcomes in which at least one die is even and at least one is odd. In other words, one is even and one is odd. Thus, its complement is the set of outcomes for which this is not true—that is, either both are even, or both are odd: ( ) = {(1, 1), (1, 3), (1, 5), (3, 1), (3, 3), (3, 5), (5, 1), (5, 3), (5, 5), (2, 2), (2, 4), (2, 6), (4, 2), (4, 4), (4, 6), (6, 2), (6, 4), (6, 6)}.

61. By Exercise 47, is the set of outcomes in which both dice show an odd number: By Exercise 48, is the set of outcomes in which both dice show an even number. Thus, is the set of outcomes in which either both are even, or both are odd: = {(1, 1), (1, 3), (1, 5), (3, 1), (3, 3), (3, 5), (5, 1), (5, 3), (5, 5), (2, 2), (2, 4), (2, 6), (4, 2), (4, 4), (4, 6), (6, 2), (6, 4), (6, 6)}. 62. By Exercise 47, is the set of outcomes in which both dice show an odd number: By Exercise 48, is the set of outcomes in which both dice show an even number. Thus, is the set of outcomes in which both are even and both are odd. Since this is impossible, there are no such outcomes: = .


Solutions Section 7.1 63. ( ) is the region outside , while is the overlap of the region outside with that outside (the gray region in the diagram below): ( ) =

64. ( ) is the region outside , while outside (the gray region in the diagram below): ( ) =

is the union of the region outside and that

65. ( ) is the overlap of and , which is the same as the overlap of all three sets: , , . Similarly for ( ) (the gray region in the diagram below): ( ) ) = (


Solutions Section 7.1 66. ( ) is the union of and , which is the same as the union of all three sets: , , . Similarly for ( ) (the gray region in the diagram below): ( ) ) = (

67. ( ) is the union of with both and . This is the same as( ) ( ) = ( ) ( )

, and so consists of all elements in together with those in ( ) (the gray region in the diagram below):

68. ( ) is the intersection of with , and so consists of all elements in that are also in or in . This is the same as( ) ( ) (the gray region in the diagram below): ( ) = ( ) ( )


69. =

70.

Solutions Section 7.1

=

71. The set of clients who owe her money is = {Acme, Crafts, Effigy, Global}. The set of clients who have done at least $10,000 worth of business with her is = {Acme, Brothers, Crafts, Dion}. Therefore, the set of clients who owe her money and have done at least $10,000 worth of business with her is = {Acme, Crafts}. 72. The set of clients who owe her money is = {Acme, Crafts, Effigy, Global}. The set of clients who have done at least $10,000 worth of business with her is = {Acme, Brothers, Crafts, Dion}. Therefore, the set of clients who owe her money or have done at least $10,000 worth of business with her is = {Acme, Brothers, Crafts, Dion, Efigy, Global}. 73. The set of clients who have done at least $10,000 worth of business with her is = {Acme, Brothers, Crafts, Dion}. The set of clients who have employed her in the last year is = {Acme, Crafts, Dion, Effigy, Global, Hilbert}. Therefore, the set of clients who have done at least $10,000 worth of business with her or have employed her in the last year is = {Acme, Brothers, Crafts, Dion, Effigy, Global, Hilbert}. 74. The set of clients who have done at least $10,000 worth of business with her is = {Acme, Brothers, Crafts, Dion}. The set of clients who have employed her in the last year is = {Acme, Crafts, Dion, Effigy, Global, Hilbert}.


Solutions Section 7.1 Therefore, the set of clients who have done at least $10,000 worth of business with her and have employed her in the last year is = {Acme, Crafts, Dion}. 75. The set of clients who do not owe her money is = {Brothers, Dion, Floyd, Hilbert}. The set of clients who have employed her in the last year is = {Acme, Crafts, Dion, Effigy, Global, Hilbert}. Therefore, the set of clients who do not owe her money and have employed her in the last year is = {Dion, Hilbert}. 76. The set of clients who do not owe her money is = {Brothers, Dion, Floyd, Hilbert}. The set of clients who have employed her in the last year is = {Acme, Crafts, Dion, Effigy, Global, Hilbert}. Therefore, the set of clients who do not owe her money or have employed her in the last year is = . 77. The clients who owe her money is = {Acme, Crafts, Effigy, Global}. The set of clients who have not done at least $10,000 worth of business with her is = {Effigy, Floyd, Global, Hilbert}. The set of clients who have not employed her in the last year is = {Brothers, Floyd}. Therefore, the set of clients who owe her money, have not done at least $10,000 worth of business with her, and have not employed her in the last year is = . 78. The set of clients who do not owe her money is = {Brothers, Dion, Floyd, Hilbert}. The set of clients who have done at least $10,000 worth of business with her is = {Acme, Brothers, Crafts, Dion}. The set of clients who have employed her in the last year is = {Acme, Crafts, Dion, Effigy, Global, Hilbert}. Therefore, the set of clients who either do not owe her money, have done at least $10,000 worth of business with her, or have employed her in the last year is = . 79. You can organize the spreadsheet as follows: For the row headings, use the years 2018, 2019, 2020, and 2021. For the column headings, use Sailboats, Motor Boats, and Yachts:

This setup gives a tabular representation of the Cartesian product {2018, 2019, 2020, 2021} × {Sailboats, Motor Boats, Yachts}. Alternatively, you could use the years for the column headings and {Sailboats, Motor Boats, Yachts} for the row headings and obtain a representation of {Sailboats, Motor Boats, Yachts} × {2018, 2019, 2020, 2021}.


Solutions Section 7.1 80. You can organize the spreadsheet as follows: For the row headings, use the decades 70s, 80s, 90s, and for the column headings, use Prescription Drugs, Nursing Homes, Hospital care, and Professional Services:

This setup gives a representation of the Cartesian product {70s, 80s, 90s} × {Prescription Drugs, Nursing Homes, Hospital Care, Professional Services}. Alternatively, you could use the decades for the column headings and the categories of health care for the row headings, and obtain a representation of {Prescription Drugs, Nursing Homes, Hospital Care, Professional Services} × {70s, 80s, 90s}. 81. The collection of all iPads and jPads combined is the set of all items that are either iPads (that is, in the set ) or jPads (that is, in the set ), and is therefore the union of the two sets: . 82. Each choice consists of a pair: (model, color), and so the set of all choices is the Cartesian product, × . 83. Techno music that is neither European nor Dutch: Techno AND NOT (European OR Dutch) Techno ∩ (European Dutch) (Choice B).

84. WWII OR (Comix AND NOT Aliens)WWII (Comix ∩ Aliens )`(Choice C)

85. Answers will vary. Let = {1, 2, 3}, = {1} and = {2}. Then ( ) = {1, 2} = {3}, but = {2, 3} {1, 3} = {1, 2, 3}. In the figure are shown the (shaded) regions corresponding to ( ) and .

By DeMorgan's law, (

) =

.

By DeMorgan's law, (

) =

.

86. Answers will vary. Let = {1, 2, 3}, = {1} and = {2}. Then ( = {2, 3} {1, 3} = {3}.

87. Let = {1}, = {2}, and

= {1, 2}.Then (

)

) =

= {1, 2} but

(

= {1, 2, 3}, but

) = {1}. In general,


Solutions Section 7.1 ( ) must be a subset of , but ( ) need not be; also, ( subset, but ( ) need not.

)

must contain

as a

88. Let = people who play soccer, = people who play rugby, and = people who play cricket. Then the given English sentence could be translated as either "He is in ( ) " or "He is in ( )." Just as in Exercise 75, these are two different sets. 89. A universal set is a set containing all "things" currently under consideration. When discussing sets of positive integers, the universe might be the set of all positive integers, or the set of all integers (positive, negative, and 0), or any other set containing the set of all positive integers. 90. No; the set is missing some ordered pairs. For instance, (1, 2) but (2, 1) .

91. ( ) means , and either or not . Thus, for instance, take as the set of suppliers who deliver components on time, as the set of suppliers whose components are known to be of high quality, and as the set of suppliers who do not promptly replace defective components. Then selecting suppliers in ( ) means selecting those who deliver components on time and are either companies whose components are of high quality or are suppliers who promptly replace defective components.

92. is the set of suppliers who deliver pre-assembled wrist watch mechanisms on time, is the set of suppliers whose wrist watch components are unassembled but known to be of high quality, and is the set of suppliers who do not deliver their watch components on time.

93. Let = {movies that are violent}, = {movies that are shorter than 2 hours}, = {movies that have a tragic ending}, and = {movies that have an unexpected ending}. The given sentence can be rewritten as "She prefers movies in ( ) ." It can also be rewritten as "She prefers movies in ." 94. Let = events that have more than 1,000 people, = events lasting at least 3 hours, and = events within a 50-mile radius of Toronto. The given sentence is "He will cater for any event in ." 95. Removing the comma would cause the statement to be ambiguous, as it could then correspond to either WWII (Comix Aliens ) or to (WWII Comix) Aliens . (See Exercise 76.)

96. With the comma included, the sentence could correspond to either WWII (Comix Aliens ) or to (WWII Comix) Aliens . However, these have the same meaning by the associative law for intersections of sets (see Exercise 55).


Solutions Section 7.2 Section 7.2

1. ( ) = Number of elements in = 4, ( ) = Number of elements in = 5. Therefore, ( ) + ( ) = 4 + 5 = 9. 2. ( ) = Number of elements in = 4, ( ) = Number of elements in ( ) + ( ) = 4 + 2 = 6. 3. 4.

= {Dirk, Johan, Frans, Sarie, Tina, Klaas, Henrika}, and so (

5.

6.

= 2. Therefore,

= { Dirk, Johan, Frans, Sarie, Hans }, and so (

) = 5.

= { Frans, Sarie, Tina, Klaas, Henrika, Frans}}, ( )) = 2.

(

= {Frans},

(

(

) = 7.

) = {Dirk, Johan, Frans, Sarie}, and so (

(

)) = 4.

) = { Frans, Sarie }, and so

7. From Exercise 3, we know that ( ) = 7. On the other hand, ( ) + ( ) ( ) = 4 + 5 2 = 7. Therefore, ( ) = ( ) + ( ) ( ).

8. From Exercise 4, we know that ( ) = 5. On the other hand, ( ) + ( ) ( ) = 4 + 2 1 = 5. Therefore, ( ) = ( ) + ( ) ( ).

9. ( × ) = ( ) ( ) = 2 × 2 = 4

10. ( × ) = ( ) ( ) = 6 × 6 = 36 11. ( × ) = ( ) ( ) = 6 × 3 = 18

12. ( × ) = ( ) ( ) = 2 × 3 = 6

13. ( × × ) = ( ) ( × ) = ( ) ( ) ( ) = 2 × 6 × 6 = 72

14. ( × × ) = ( ) ( × ) = ( ) ( ) ( ) = 2 × 6 × 3 = 36

15. (

16. (

) = ( ) + ( ) ( ) = ( ) + ( ) (

17. ( ) = ( ) + ( ) ( ( ) = 60 + 60 100 = 20. 18. (

) = ( ) + ( ) (

) = 43 + 20 3 = 60 ) = 60 + 20 1 = 79

) 100 = 60 + 60 (

), and so

) 150 = 100 + ( ) 40, and so ( ) = 150 100 + 40 = 90.

19. ( ) = ( ) ( ) = 10 4 = 6 20. ( ) = ( ) ( ) = 10 3 = 7


21. ((

22. ((

) ) = ( ) (

) ) = ( ) (

Solutions Section 7.2 ) = 10 1 = 9 ) = 10 6 = 4

23. Use = {BA, MU, SO, SU, LI, MS, WA, RT, DU, LY}, = {SO, LI, MS, RT}, = {BA, MU, SO}. = {BA, MU, SU, WA, DU, LY}, = {SU, LI, MS, WA, RT, DU, LYSU, WA, DU, LY} and so ( ) = 4.

24. Use = {BA, MU, SO, SU, LI, MS, WA, RT, DU, LY}, = {SO, LI, MS, RT}, = {BA, MU, SO}. = {BA, MU, SU, WA, DU, LY}, = {SU, LI, MS, WA, RT, DU, LY BA, MU, SU, WA, DU, LY, LI, MS, RT } and so ( ) = 9.

25. (( ) ) = 9 from Exercise 21, and so ( ) + ( ) (( Therefore, (( ) ) = ( ) + ( ) (( ) ). 26. (

) = 9 from Exercise 24, and so (

) + (

) ) = 6 + 7 4 = 9.

) = 4 + 6 = 10 = ( ).

27. Assign labels to the regions of the diagram where the quantities are unknown:

( ) = 20 + 16 = 20 = 4 ( )= 8 2+ = 8 = 6 ( ) = 20 6 + + = 20 6 + + 6 = 20 = 8 ( ) = 28 12 + + = 28 12 + + 6 = 28 = 10 ( ) = 50 + + + + + 16 = 50 4 + 8 + 10 + 6 + + 16 = 50 = 6 Thus, the completed diagram is


Solutions Section 7.2 28. Assign labels to the regions of the diagram where the quantities are unknown:

( ) = 11 + 6 = 11 = 5 ( ) = 16 + + 11 = 16 + 5 + 11 = 16 = 0 ( ) = 30 + + 15 = 3 5 + + 15 = 30 = 10 ( ) = 40 + + + + 16 = 40 0 + 5 + 10 + + 16 = 40 = 9 Thus, the completed diagram is

29. Assign labels to the regions of the diagram where the quantities are unknown:

( ) = 10 + + 7 = 10 + = 3 ( ) = 19 + + 14 = 19 + = 5 ( ) = 140 + + + 132 = 140 + + = 8 This is a system of 3 linear equations in 3 unknowns: + = 3

(1) + = 5(2) + + = 8.(3)

To solve, we can substitute (1) into (3), giving 3+ = 8 = 5 (2) now gives + 5 = 5 = 0. (1) now gives + 0 = 3 = 3. Thus, the solution is ( , , ) = (3, 0, 5). The completed diagram is:


Solutions Section 7.2

30. Assign labels to the regions of the diagram where the quantities are unknown:

( ) = 30 + + 5 = 30 + = 25 ( ) = 30 + + 25 = 30 + = 5 ( ) = 35 + + 25 = 35 + = 10 This is a system of 3 linear equations in 3 unknowns: + = 5

(1) + = 5(2) + = 10.(3)

To solve, add (1) and (2) to obtain + 2 + = 10 Using (3), this becomes 10 + 2 = 10 = 0. (1) now gives + 0 = 5 = 5. (2) now gives 0 + = 5 = 5. Thus, the solution is ( , , ) = (5, 0, 5). The completed diagram is:

31. Let be the set of websites containing "asteroid" and let be the set of websites containing the word "comet." We are told that ( ) = 10.4, ( ) = 26.6, ( ) = 30.6. The formula ( ) = ( ) + ( ) ( ) gives 30.6 = 10.4 + 26.6 ( ) = 37.0 ( ) so ( ) = 37.0 30.6 = 6.4 million sites


Solutions Section 7.2 32. Let be the set of websites containing "Mars mission" and let be the set of websites containing the phrase "Moon mission." We are told that ( ) = 1.7, ( ) = 0.8, ( ) = 0.2. Therefore, ( ) = ( ) + ( ) ( ) = 1.7 + 0.8 0.2 = 2.3 million sites.

33. Let be the set of people who had black hair, and let be the set of people who had a whole row to themselves. We are told that ( ) = 37, ( ) = 33, and ( ) = 6.We are asked to find ( ). ( ) = ( ) + ( ) ( ) 37 = 33 + 6 ( ) So, ( ) = 33 + 6 37 = 2.

34. Let be the set of desserts that include yogurt, and let be the set of desserts that include fruit. We are told that ( ) = 14, ( ) = 8, and ( ) = 9. We are asked to find ( ). ( ) = ( ) + ( ) ( ) 14 = 8 + 9 ( ) So, ( ) = 8 + 9 14 = 3.

35. Let be the set of gamers who used smartphones, and let be the set of gamers who used tablets. We are told that ( ) = 132, ( ) = 123, and ( ) = 44 (working in millions of gamers). We are asked to find the number who used tablets excluding those who used both: ( ) ( ), so we first calculate ( ) : ( ) = ( ) + ( ) ( ) 132 = 123 + 44 ( ) So, ( ) = 123 + 44 132 = 35, so 35 million gamers used both. Therefore, ( ) ( ) = 44 35 = 9 million gamers. 36. Let be the set of gamers who used smartphones, and let be the set of gamers who used tablets. We are told that ( ) = 208, ( ) = 194, and ( ) = 73 (working in millions of gamers). We are asked to find the number who did not use tablets; that is, the number who used smartphones excluding those who used both: ( ) ( ), so we first calculate ( ) : ( ) = ( ) + ( ) ( ) 208 = 194 + 73 ( ) So, ( ) = 194 + 73 208 = 59, so 59 million gamers used both. Therefore, ( ) ( ) = 194 59 = 135 million users. 37.

is the set of authors who are both successful and new. is the set of authors who are either successful or new (or both). ( ) = 30, ( ) = 20, ( ) = 5, ( ) = 45 ( ) = ( ) + ( ) ( ) 45 = 30 + 20 5 ✓

38. is the set of authors who are both new and unsuccessful. is the set of authors who are either new or unsuccessful (or both). ( ) = 20, ( ) = 70, ( ) = 15, ( ) = 75 ( ) = ( ) + ( ) ( ) 75 = 20 + 70 15 ✓

39. is the set of authors who are successful but not new—in other words, the set of authors who are successful and established. ( ) = 25

40. is the set of authors who are either unsuccessful or not established (or both)—in other words, the set of authors who are unsuccessful or new. ( ) = 5 + 15 + 55 = 75

41. Of the 80 established authors, 25 are successful. Thus, the percentage of established authors who are successful is 25 ! 0.3125, or 31.25%. 80


Solutions Section 7.2 Of the 30 successful authors, 25 are established. Thus, the percentage of successful authors who are established is 25 ! 0.8333, or 83.33%. 30 42. Of the 20 new authors, 15 are unsuccessful. Thus, the percentage of new authors who are unsuccessful is 15 = 0.75, or 75%. 20 Of the 70 unsuccessful authors, 15 are new. Thus, the percentage of unsuccessful authors who are new is 15 ! 0.2143, or 21.43%. 70 43. The set of housing starts in the Midwest in 2020 is ( ) = 140 thousand units

.

44. The set of housing starts either in the West or in 2018 is " (" ) = 670 + 870 220 = 1,320 thousand units

.

46. The set of housing starts in the Northeast after 2018 is ( ) = 180 60 = 120 thousand units

.

45. The set of housing starts in 2019 excluding housing starts in the South is ( # ) = 890 500 = 390 thousand units

# .

47. The set of housing starts in 2019 in the West and Midwest is (" ). ( (" )) = (( " ) ( )) = 210 + 120 0 = 330 thousand units

48. The set of housing starts in the South and West in years other than 2018 is ( (# " )) = ((# " )) ( (# " )) = 1,520 + 670 (470 + 220) = 1,500 thousand units 49. The set of non-financial industries that increased is $ $

%

6

5

0

11

&

4

0

3

7

2

3

1

6

#

6

5

3

14

0

5

1

6

Totals

18

18

8

44

($

) = 18 4 = 14

Totals

.

(#

" ).


Solutions Section 7.2 50. The set of industries in the financial sector that did not increase is $

%

6

5

0

11

&

4

0

3

7

2

3

1

6

#

6

5

3

14

0

5

1

6

Totals

18

18

8

44

(

$ .

Totals

$ ) = 0 + 3 = 3 or (

$ ) = 7 4 = 3

51. ( %) is the number of industries that either were not in the industrial sector or were unchanged in value (or both). $

%

6

5

0

11

&

4

0

3

7

2

3

1

6

#

6

5

3

14

0

5

1

6

Totals

18

18

8

44

(

Totals

%) = 44 (6 + 5) = 33

52. ( % ) is the number of industries that were either in the industrial sector or were not unchanged (or both). $

%

6

5

0

11

&

4

0

3

7

2

3

1

6

#

6

5

3

14

0

5

1

6

Totals

18

18

8

44

(

Totals

% ) = 44 (0 + 3 + 1 + 1) = 39


53. (&

) = 3; ( ) = 18

Solutions Section 7.2

$

%

6

5

0

11

&

4

0

3

7

2

3

1

6

#

6

5

3

14

0

5

1

6

Totals

18

18

8

44

Totals

(& ) 3 1 = = . ( ) 18 6 This is the fraction of industries that decreased that were from the health care sector. 54. (#

) = 5; (# ) = 6 $

%

6

5

0

11

&

4

0

3

7

2

3

1

6

#

6

5

3

14

0

5

1

6

Totals

18

18

8

44

Totals

(# ) 5 = . (# ) 6 This is the fraction of information technology industries that decreased.

55. Let be the set of all adults in the study, let be the set of those who had high exposure to dust and let be the set of those living in rural areas. We are told that ( ) = 5,366, ( ) = 463, ( ) = 1,699, and ( ) = 360. Represent the given information in a Venn diagram: S

D 463 – 360 = 103

R 360

1,699 – 360 = 1,339

5,336 – (103 + 360 + 1,339) = 3,564

a. From the diagram, ( ) = 103. b. From the diagram, ( ) = 3,564.

56. Let be the set of all adults in the study, let be the set of those who had high exposure to fumes,


Solutions Section 7.2 and let be the set of those living in urban areas. We are told that ( ) = 5,366, ( ) = 96, ( ) = 3,667, and ( ) = 55. Represent the given information in a Venn diagram: S

F 96 – 55 = 41

U 55

3,667 – 55 = 3,612

5,336 – (41 + 55 + 3,612) = 1,628

a. From the diagram, ( b. From the diagram, (

) = 41. ) = 1,628.

57. a. Following is a Venn diagram with most of the unknowns marked with labels:

15 had seen a science fiction movie and a horror movie 5 + ' = 15 ' = 10 5 had seen an adventure movie and a horror movie 5 + ( = 5 ( = 0 25 had seen a science fiction movie and an adventure movie 5 + = 25 = 20 35 had seen a horror movie 35 had seen a horror movie 5 + ' + ( + = 35 5 + 10 + 0 + = 35 = 20 55 had seen an adventure movie 5 + + + ( = 55 5 + 20 + + 0 = 55 = 30 40 had seen a science fiction movie 5 + + + ' = 40 5 + + 20 + 10 = 40 = 5 Finally, the total number of people in the survey was 100: 5 + + + + ' + ( + + = 100 5 + 5 + 20 + 30 + 10 + 0 + 20 + = 100 = 10 Here is the completed diagram:

b. Of the 40 people who had seen science fiction, 15 saw a horror movie. Therefore, the percentage of science fiction movie fans who are also horror movie fans can be estimated as 15 = 0.375, or 37.5%. 40


Solutions Section 7.2 58. a. Following is a Venn diagram with most of the unknowns marked with labels:

16 play both lacrosse and football ( + 0 = 16 ( = 166 play both soccer and football '+ 0= 6 '= 6 4 play both soccer and lacrosse + 0 = 4 = 4 96 play football ' + ( + + 0 = 96 6 + 16 + = 96 = 74 60 play lacrosse + + ( + 0 = 60 4 + + 16 = 60 = 40 50 play soccer + + ' + 0 = 50 + 4 + 6 = 50 = 40 Finally, the total number of students is 4,700: + + + ' + ( + + 0 + = 4700 40 + 4 + 40 + 6 + 16 + 74 + = 4700 = 4520 Here is the completed diagram:

b. Of the 50 soccer players, 6 + 4 = 10 play another sport as well. Therefore, the percentage of soccer players who also play one other sport is 10 = 0.20, or 20%. 50

59. Let be the set of students who liked rock music, and let be the set of those who liked classical music. ( ) = 22, ( )= 5 We are also given other information that we do not need. Since 5 of the 22 people who liked rock also liked classical, the other 17 did not like classical. Therefore, 17 of those that enjoyed rock did not enjoy classical music.


Solutions Section 7.2 60. Here is a Venn diagram with some pf the unknown regions marked with letters.

We are asked only for the quantity + . We are told: + + + ' + ( + + + ) = 100 + + ' + ( = 21 + + + = 22 ' + ( + + = 27 + (= 5 ( = 5, ) = 53. Substituting ( = 5 in the equation immediately above it, we get = 0. From the Venn diagram, is the number of students who enjoyed both classical and rock but not house, and so we have answered the question: No students enjoyed both classical and rock but not house. 61. If # = {1, 2, 3, 4, 5, 6} is the set of outcomes for one throw of the die, then (# ) = 6. The set of outcomes for three throws of the die is the Cartesian product # × # × # , and (# × # × # ) = (# ) × (# ) × (# ) = (# ) 3 = 6 3 = 216. For 10 throws, the number of outcomes is (# ) 10 = 6 10 = 60,466,176.

62. If # = {1, 2, . . . , 12} is the set of outcomes for one throw of the die, then (# ) = 12. The set of outcomes for three throws of the die is the Cartesian product # × # × # , and (# × # × # ) = (# ) × (# ) × (# ) = (# ) 3 = 12 3 = 1,728. For 10 throws, the number of outcomes is (# ) 10 = 12 10 = 61,917,364,224.

63. Since every element of is in , and since contains at least one more element than (otherwise they would be the same set), ( ) < ( ). 64. Since every element of is in , and since contains at least one more element than (otherwise and would be equal) ( ) > ( ).

65. The number of elements in the Cartesian product of two finite sets is the product of the number of elements in the two sets. 66. The union; the number of elements in the union of two disjoint sets is the sum of the number of elements in the two sets. 67. Answers will vary. 68. Answers will vary. 69. Since (

) = ( ) + ( ) (

), we get (

) * ( ) + ( ) when (

) * 0; that


is, when

*

.

Solutions Section 7.2

70. Since ( × ) = ( ) ( ), it follows that ( × ) = ( ) whenever ( ) = 1 or = both ( × ) and ( ) zero).

71. Since + when + .

, the only way they can have the same number of elements is if =

72. Since + , the only way they can have the same number of elements is if when + . 73. (

74. ( ) = (

(making

) = ( ) + ( ) + ( ) ( ) < (

) (

) (

) + (

) = ( ) < ( × ) = ( × ) < ( × )

; that is,

= ; that is, )


Solutions Section 7.3 Section 7.3 1. The setups are alternatives, and the different ways a setup can be done are the choices for that alternative. Thus, Alternative 1: 2 choices Alternative 2: 3 choices Alternative 3: 5 choices Total number of resulting options: 2 + 3 + 5 = 10

2. The methodologies are alternatives, and the different ways a methodology can be done are the choices for that alternative. Thus, Alternative 1: 4 choices Alternative 2: 3 choices Alternative 3: 2 choices Alternative 4: 2 choices Total number of resulting options: 4 + 3 + 2 + 2 = 11

3. Step 1: 2 choices Step 2: 3 choices Step 3: 5 choices Total number of resulting options: 2 × 3 × 5 = 30

4. Step 1: 4 outcomes Step 2: 3 choices Step 3: 2 choices Step 4: 2 choices Total number of resulting options: 4 × 3 × 2 × 2 = 48

5. Alternative 1: 1 × 2 = 2 outcomes Alternative 2: 2 × 2 = 4 outcomes Total number of resulting options: 2 + 4 = 6

6. Alternative 1: 1 × 2 × 2 = 4 choices Alternative 2: 2 × 2 = 4 choices Total number of resulting options: 4 + 4 = 8

7. Step 1: 1 + 2 = 3 resulting options Step 2: 2 + 2 + 1 = 5 resulting options Total number of resulting options: 3 × 5 = 15

8. Step 1: 1 + 2 + 2 = 5 outcomes Step 2: 2 + 2 = 4 outcomes Total number of outcomes: 5 × 4 = 20

9. Alternative 1: (3 + 1) × 2 = 8 options Alternative 2: 5 options Total number of resulting options: 8 + 5 = 13

10. Alternative 1: 2 outcomes Alternative 2: (4 + 1) × 2 = 10 options Total number of options: 2 + 10 = 12

11. Step 1: (3 × 1) + 2 = 6 choices Step 2: 5 choices Total number of options: 6 × 5 = 30

12. Step 1: 2 choices Step 2: (4 × 1) + 2 = 6 choices Total number of options: 2 × 6 = 12

13. Decision algorithm: Start with 4 empty slots, and select slots in which to place the letters. Step 1: Select a slot to place the b: 4 choices. Step 2: Place the "a"s in the remaining slots: 1 choice. Total number of outcomes: 4 × 1 = 4 14. Decision algorithm: Start with 5 empty slots, and select slots in which to place the letters. Step 1: Select a slot to place the b: 5 choices. Step 2: Select a slot to place the c: 4 choices. Step 3: Place the "a"s in the remaining slots: 1 choice. Total number of outcomes: 5 × 4 × 1 = 20 15. Decision algorithm:


Solutions Section 7.3 Step 1: Select a flavor: 31 choices. Step 2: Select cone, cup, or sundae: 3 choices. Total number of desserts: 31 × 3 = 93 16. Decision algorithm: Step 1: Select a flavor: 1,300 choices. Step 2: Select cone, cup, or sundae: 3 choices. Step 3: Select a topping: 12 choices. Total number of desserts: 1,300 × 3 × 12 = 46,800 17. Decision algorithm: Step 1: Select the first bit: 2 choices. Step 2: Select the second bit: 2 choices. Step 3: Select the third bit: 2 choices. Step 4: Select the fourth bit: 2 choices. Total number of nybbles: 2 × 2 × 2 × 2 = 16 18. Decision algorithm: Step 1: Select the first bit: 3 choices. Step 2: Select the second bit: 3 choices. Step 3: Select the third bit: 3 choices. Step 4: Select the fourth bit: 2 choices. Step 5: Select the fifth bit: 3 choices. Step 6: Select the sixth bit: 3 choices Total number of sequences: 3 × 3 × 3 × 3 × 3 × 3 = 729 19. Decision algorithm: Start with 6 empty slots, and select slots in which to place the ternary digits. Step 1: Select a slot to place the 1: 6 choices. Step 2: Select a slot to place the 2: 5 choices. Step 3: Place 0s in the remaining slots: 1 choice. Total number of sequences: 6 × 5 × 1 = 30 20. Decision algorithm: Start with 4 empty slots, and select slots in which to place the binary digits. Step 1: Select a slot to place the 1: 4 choices. Step 2: Place 0s in the remaining slots: 1 choice. Total number of nybbles: 4 × 1 = 4 21. Alternative 1 (Gummy candy): Step 1: Select a size: 3 choices. Step 2: Select a shape: 3 choices. Alternative 2 (Licorice nibs): Step 1: Select a size: 2 choices. Step 2: Select a color: 2 choices. Total number of options: 3 × 3 + 2 × 2 = 13

22. Alternative 1 (Writing assignment): Step 1: Select a topic: 2 choices. Step 2: Select a length: 3 choices. Alternative 2 (Reading assignment): Step 1: Select a biography: 5 choices. Step 2: Select an essay volume: 2 choices. Total number of options: 2 × 3 + 5 × 2 = 16

23. Step 1: Answer 1st t/f question: 2 choices. Step 2: Answer 2nd t/f question: 2 choices. ... Step 10: Answer 10th t/f question: 2 choices. Step 11: Answer 1st multiple choice question: 5 choices. Step 12: Answer 2nd multiple choice question: 5 choices. Total number of answer sheets: (2 × 2 × ... × 2) × (5 × 5) = 2 10 × 5 2 = 25,600


Solutions Section 7.3 24. Step 1: Answer 1st t/f question: 2 choices. Step 2: Answer 2nd t/f question: 2 choices. ... Step 20: Answer 20th t/f question: 2 choices. Step 21: Answer 1st multiple choice question: 6 choices. Step 22: Answer 2nd multiple choice question: 6 choices. Step 23: Answer 3rd multiple choice question: 6 choices. Total number of answer sheets:(2 × 2 × ... × 2) × (6 × 6 × 6) = 2 20 × 6 3 = 25,600 = 1,048,576 × 216 = 226,492,416 25. Alternative 1: Do Part A: Steps 1–8: Answer the 8 t/f questions; 2 choices each, giving 2 8 possible choices. Alternative 2: Answer Part B: Steps 1-5: Answer 5 multiple choice questions with 5 choices each, giving 5 5 possible choices. Total number of answer sheets: 2 8 + 5 5 = 3,381 26. Step 1: Do Part A: Steps 1–4: Answer the 4 t/f questions; 2 choices each, giving 2 × 2 × 2 × 2 = 2 4possible choices Step 2: Do the rest of the test: Alternative 1: Choose Part B: Steps 1–4: Choose 1 answer out of 5 each time: 5 × 5 × 5 × 5 = 5 4 choices. Alternative 2: Choose Part C: Steps 1–3: Choose 1 answer out of 6 each time: 6 × 6 × 6 = 6 3choices. Total number of answer sheets: 2 4 × (5 4 + 6 3) = 16(625 + 216) = 13,456 27. a. Step 1: Select a mutual fund: 4 choices. Step 2: Select a muni. bond fund: 3 choices. Step 3: Select a stock: 8 choices. Step 4: Select a precious metal: 3 choices. Total number of portfolios: 4 × 3 × 8 × 3 = 288 b. ,, Step 1: Select 3 mutual funds: 4 choices (select which one to leave out). Step 2: Select 2 municipal bond funds: 3 choices (select which one to leave out). Step 3: Select one stock: 8 choices. Step 4: Select 2 precious metals 3 choices (select which one to leave out). Total number of portfolios: 4 × 3 × 8 × 3 = 288 28. Alternative 1: Dessert & no appetizer: Step 1: Select a soup: 2 choices. Step 2: Select a main course: 4 choices. Step 3: Select a dessert: 5 choices. Alternative 2: dessert & no appetizer: Step 1: Choose an appetizer: 5 choices. Step 2: Select a soup: 2 choices. Step 3: Select a main course: 4 choices. Total number of meals: (2 × 4 × 5) + (5 × 2 × 4) = 80 29. The number of possible characters that can be represented equals the number of possible bytes: Steps 1–8: Select 0 or 1 for each bit: 2 × 2 × ... × 2 = 28 choices. Thus, the number of possible characters is 2 8 = 256. 30. A single byte can represent 2 8 = 256 different characters. Two bytes can therefore represent 256 × 256 = 65,536 different characters.


Solutions Section 7.3 Thus, two bytes is more than sufficient to encode all 50,000 characters. 31. Decision algorithm to obtain a symmetry of the five-pointed star: Alternative 1: Pure rotation: 5 choices Alternative 2: Rotation followed by a flip: 5 choices Total number of symmetries: 5 + 5 = 10 32. Decision algorithm to obtain a symmetry of the six-pointed star: Alternative 1: Pure rotation: 6 choices Alternative 2: Rotation followed by a flip: 6 choices Total number of symmetries: 6 + 6 = 12 33. As the variable name must end in a digit, it cannot have length 1 (as that would be a single letter only). Alternative 1: Length two: Letter, digit: Step 1: Choose a letter (uppercase or lowercase): 52 choices. Step 2: Choose a digit: 10 choices. Alternative 2: Length three: Letter, (letter or digit), digit Step 1: Choose a letter: 52 choices. Step 3: Choose a letter or digit: 62 choices. Step 2: Choose a digit: 10 choices. Total number of variables = 52 × 10 + 52 × 62 × 10 = 32,760 34. Alternative 1: One letter plus digit: Step 1: Choose a letter: 26 choices. Step 2: Choose a digit: 10 choices. Alternative 2: Two letters plus digit: Step 1: Choose first letter: 26 choices. Step 2: Choose 2nd letter: 26 choices. Step 3: Choose a digit: 10 choices Alternative 3: Three letters plus digit: Step 1: Choose first letter: 26 choices. Step 2: Choose 2nd letter: 26 choices. Step 3: Choose 3rd letter: 26 choices. Step 4: Choose a digit: 10 choices. Total number of employee codes = 26 × 10 + 26 2 × 10 + 26 3 × 10 = 182,780 35. Step 1: Select a winner of the North Carolina-Central Connecticut game: 2 choices\gap[40] Step 2: Select a winner of the Virginia-Syracuse game: 1 choice (we know that Syracuse is the winner).\gap[40] Step 3: Select the overall winner: 2 choices.Total number of choices: 2 × 1 × 2 = 4 36. For a decision algorithm, fill in the diagram starting from the right. Step 1: Decide which to place De Paul (top or bottom) in the final: 2 choices. (This results in having to fill in De Paul for the appropriate first round and Hofstra in the other first round.) Step 3: Select a winner of the first round game played by Hofstra: 2 choices. Total number of choices: 2 × 2 = 4 37. a. Step 1: Choose the first digit: 8 choices Steps 2–7: Choose the remaining 6 digits: 10 6 choices Total number: 8 × 10 6 = 8,000,000 b. To count the numbers beginning with 463, 460, or 400, use the following decision algorithm: Alternative 1: Start with 463. Steps 1–4: Choose the remaining 4 digits: 10 4 choices.


Solutions Section 7.3 Alternative 2: Start with 460. Steps 1–4: Choose the remaining 4 digits: 10 4 choices. Alternative 3: Start with 400. Steps 1–4: Choose the remaining 4 digits: 10 4 choices. Total number of choices: 10 4 + 10 4 + 10 4 = 30,000 c. Use the following decision algorithm: Step 1: Select the 1st digit (other than 0 or 1): 8 choices. Step 2: Select a 2nd digit different from the 1st digit: 9 choices. Step 2: Select a 3rd digit different from the 2nd digit: 9 choices. ... Step 7: Select a 7th digit different from the 6th digit: 9 choices. Total number of choices: 8 × 9 6 = 4,251,528

38. a. Steps 1–9 Choose 9 digits: 10 9 choices for Social Security numbers b. Alternative 1: Start with 023. Steps 1–8: Choose the remaining 6 digits: 10 6 choices. Alternative 2: Start with 003. Steps 1–3: Choose the remaining 6 digits: 10 6 choices. Total number of choices: 10 6 + 10 6 = 2,000,000 c. Use the following decision algorithm: Step 1: Select the 1st digit: 10 choices. Step 2: Select a 2nd digit different from the 1st digit: 9 choices. Step 2: Select a 3rd digit different from the 2nd digit: 9 choices. ... Step 9: Select a 9th digit different from the 8th digit: 9 choices. Total number of choices: 10 × 9 8 = 430,467,210 39. a. Use the following decision algorithm: Step 1: Select the first digit: 2 choices. Step 2 Select digits 2 through 6: 1 choice (a single issuer) Step 3: Select digits 7 through 15 (customer id): 10 9 choices. Step 4: Select the check digit: 1 choice (as it is determined by the digits that precede it). Total number of choices: 2 × 10 9 = 2 billion possible card numbers b. Use the following decision algorithm: Step 1: Select the first digit: 1 choice (6 for Discover) Step 2 Select digits 2 through 6: 1 choice (a single issuer) Step 3: Select digits 7 through 15 (customer id): 10 9 choices. Step 4: Select the check digit: 9 choices (exclude the correct check digit). Total number of choices: 10 9 × 9 = 9 billion possible card numbers 40. a. Use the following decision algorithm: Step 1: Select the first two digits: 2 choices. Step 2 Select digits 3 and 4: 2 choices (dollars or pesos) Step 3: Select digits 5 through 11 (account id): 1 choice. Step 4: Select digits 12 through 14 (card id): 10 3 = 1,000 choices. Step 5: Select the check digit: 1 choice (as it is determined by the digits that precede it). Total number of choices: 2 × 2 × 10 3 = 4,000 possible card numbers b. Use the following decision algorithm: Step 1: Select the first two digits: 2 choices. Step 2 Select digits 3 and 4: 1 choice (dollars) Step 3: Select digits 5 through 11: 10 7 choices. Step 4: Select digits 12 through 14 (card id): 10 3 = 1,000 choices. Step 5: Select the check digit: 9 choice (exclude the correct check digit).


Solutions Section 7.3 Total number of choices: 2 × 10 7 × 10 3 × 9 = 18 × 10 10 = 180 billion possible card numbers 41. a. Steps 1–3: Choose three bases (4 choices each): 4 3 = 64 choices b. Steps 1– : Choose bases (4 choices each): 4 choices c. From part (b) with = 2.1 × 10 10, we obtain a total of 4 = 4 2.1×10

10

c. From part (b) with = 2.1 × 10 10, we obtain a total of 6 = 6 2.1×10

10

possible DNA chains.

42. a. Steps 1–4: Choose four bases (6 choices each): 6 4 = 1,296 choices b. Steps 1– : Choose bases (6 choices each): 6 choices

possible expanded DNA chains.

43. a. Steps 1–6: Select 6 hexadecimal digits (16 choices per digit): 16 6 = 16,777,216 possible colors b. Step 1: Choose the 1st repeating pair of digits: 16 choices. Step 2: Choose the 2nd repeating pair of digits: 16 choices. Step 3: Choose the 3rd repeating pair of digits: 16 choices. Total number of colors: 16 3 = 4,096 c. Step 1: Choose the digit : 16 choices. Step 2: Choose the digit : 16 choices. Total number of grayscale shades: 16 2 = 256 d. Step 1: Choose the position for the sequence : 3 choices. Step 2: Choose the values of and : 16^{2} choices. Total number of choices: 3 × 16 2 choices. However, the above decision algorithm leads to the sequence 000000 in three ways. The number of pure colors is therefore 3 × 16 2 2 = 766. 44. Steps 1–2: Select 2 digits in the range 2–9: 8 choices per step: 8 2 choices. Steps 3–7: Select 5 digits in the range 0–9; 10 choices per step: 10 5 choices. Total: 8 2 × 10 5 = 6,400,000 telephone numbers

45. Step 1: Choose a male actor to play Escalus: 10 choices. Step 2: Choose a male actor to play Paris: 9 choices. ... Step 6: Choose a male actor to play Tybalt: 5 choices. Step 7: Choose a male actor to play Friar Lawrence: 4 choices. Step 8: Choose a female actor to play Lady Montague: 8 choices. Step 9: Choose a female actor to play Lady Capulet: 7 choices. Step 10: Choose a female actor to play Juliet: 6 choices. Step 11: Choose a female actor to play Juliet's Nurse: 5 choices. Total number of casts: (10 × 9 × 8 × 7 × 6 × 5 × 4) × (8 × 7 × 6 × 5) = 1,016,064,000 46. Step 1: Choose a female actor to play Prince Siegfried: 4 choices Step 2: Choose a female actor to play Rotbart: 3 choices. Step 3: Choose a male actor to play Friar Siegfried's Mother: 12 choices. Step 4: Choose a male actor to play Princess Odette: 11 choices. Step 5: Choose a male actor to play Odile: 10 choices. Step 6: Choose a female actor to play Cygnet #1: 9 choices. Step 7: Choose a female actor to play Cygnet #2: 8 choices. Step 8: Choose a female actor to play Cygnet #3: 7 choices. Total number of casts: (4 × 3) × (12 × 11 × 10 × 9 × 8 × 7) = 7,983,360

47. a. Steps 1–3: Choose 3 letters: 26 3 choices. Steps 4–6: Choose 3 digits: 10 3 choices. Total number of license plates: 26 3 × 10 3 = 17,576,000


Solutions Section 7.3 b. Steps 1–2 Choose 2 letters: 26 2 choices. Step 3: Choose a letter other than I, O, or Q: 23 choices. Steps 4–6: Choose 3 digits: 10 3 choices. Total number of license plates: 26 2 × 23 × 10 3 = 15,548,000 c. A decision algorithm for the number of reserved plates: Step 1: Choose VET, MDZ, or DPZ: 3 choices. Step 2–4: Choose 3 digits: 10 3 choices. Total number of reserved plates: 3 × 10 3 From part (b), the total number of possible plates is 15,548,000. Therefore, the number of unreserved plates is 15,548,000 3 × 10 3 = 15,545,000.

48. a. Step 1: Choose three digits: 10 3 choices Step 2: Choose to have either one, two or three letters Alternative 1: One letter, 26 choices Alternative 2: Two letters, 26 2 choices Alternative 3: Three letters, 26 3 choices 26 + 26 2 + 26 3 = 18,278 total choices in Step 2 Total number of license plates: 10 3 × 18,278 = 18,278,000. b. Step 1: Choose one nonzero digit: 9 choices Step 2: Choose two digits from 0–9: 10 2 choices Step 3: Same as Step 2 in part (a): 26 + 26 2 + 26 3 = 18,278 choices for the letters. Total number of license plates: 9 × 10 2 × 18,278 = 16,450,200. Another solution would be to determine the number of reserved plates, which is 10 2 × 18,278, and then subtract from the answer from part (a). 49. a. Start with 4 empty slots in which to place the letters R and D. Step 1: Select a slot for the single D: 4 choices. Step 2: Place Rs in the remaining slots: 1 choice. Total number of sequences: 4 × 1 = 4 b. Each route in the maze is a sequence of 4 moves: either right (R) or down (D). Since only one down move is possible, the number of such routes equals the number of four-letter sequences that contain only the letters R and D, with D occurring only once: 4 possibilities, by part (a). c. If we allowed left and/or up moves, we could get an unlimited number of routes, such as RRRD, RLRRRD, RLRLRRRD, ... . 50. a. Start with 6 empty slots in which to place the letters R and D. Step 1: Select a slot for the single D: 6 choices. Step 2: Place Rs in the remaining slots: 1 choice. Total number of sequences: 6 × 1 = 6 b. Each route in the maze is a sequence of 6 moves: either right (R) or down (D). Since only one down move is possible, the number of such routes equals the number of four-letter sequences that contain only the letters R and D, with D occurring only once: 6 possibilities, by part (a). c. If we allowed left and/or up moves, we could get an unlimited number of routes, such as RRRRRD, RLRRRRRD, RLRLRRRRRD, ... . 51. a. Step 1: Choose a 1st cylinder: 6 choices. Step 2: Choose a 2nd cylinder on the opposite side: 3 choices. Step 3: Choose a 3rd cylinder on the same side as the first: 2 choices. Step 4: Chose a 4th cylinder on the same side as the second: 2 choices. Step 5: Chose a 5th cylinder on the same side as the first: 1 choice. Step 6: Chose a 6th cylinder on the same side as the second: 1 choice. Total number of firing sequences: 6 × 3 × 2 × 2 × 1 × 1 = 72


Solutions Section 7.3 b. Use the same decision algorithm as for part (a), except that, in Step 1, we only have 3 choices (the cylinders on the left). Total number of firing sequences: 3 × 3 × 2 × 2 × 1 × 1 = 36 52. a. Step 1: Choose a 1st cylinder: 8 choices. Step 2: Choose a 2nd cylinder on the opposite side: 4 choices. Step 3: Choose a 3rd cylinder on the same side as the first: 3 choices. Step 4: Chose a 4th cylinder on the same side as the second: 3 choices. Step 5: Chose a 5th cylinder on the same side as the first: 2 choices. Step 6: Chose a 6th cylinder on the same side as the second: 2 choices. Step 7: Chose a 7th cylinder on the same side as the first: 1 choice. Step 8: Chose a 8th cylinder on the same side as the second: 1 choice. Total number of firing sequences: 8 × 4 × 3 × 3 × 2 × 2 × 1 × 1 = 1152 b. Use the same decision algorithm as for part (a), except that, in Step 1, we only have 4 choices (the cylinders on the left). Total number of firing sequences: 4 × 4 × 3 × 3 × 2 × 2 × 1 × 1 = 576 53. Decision algorithm for producing a painting: Steps 1–5: Select blue or gray for each of the odd-numbered lines: 2^{5} choices. Step 6: Select a single color for the remaining lines: 3 choices. Total number of paintings: 2 5 × 3 = 96 54. Decision algorithm for the number of possible combinations: Step 1: Choose the 1st number: 40 choices. Step 2: Choose the 2nd number: 2 choices. Step 3: Choose the third number (from 5, 15, 25, 35): 4 choices. Total number of combinations: 40 × 2 × 4 = 320 Since each combination takes 10 seconds to try, the total amount of time is 320 × 10 = 3,200 seconds (or 53 1-3 minutes). 55. a. Decision algorithm for incorrect codes that will open the lock:Step 1: Decide which digit will be the wrong one: 4 choices. Step 2: Choose an incorrect digit in that place: 9 choices (one is correct, leaving 9 incorrect ones). Total number of incorrect codes that will open the lock: 4 × 9 = 36 b. From part (a), there are 36 incorrect codes that will open the lock. Also, there is only 1 correct code. Therefore, the total number of codes that will open the lock is 36 + 1 = 37. 56. Decision algorithm for generating a 4-letter sequence: Steps 1–4: Select a letter that corresponds to the each digit in the sequence; 3 choices per step: 3 4 = 81 possible 4-letter sequences. 57. Decision algorithm for creating a calendar: Step 1: Choose a day of the week for Jan 1: 7 choices. Step 2: Decide whether or not it is a leap year: 2 choices. Total number of possible calendars: 7 × 2 = 14 58. Step 1: Choose a day of the week for Feb 12: 3 choices. Step 2: Decide whether or not it is a leap year: 2 choices. Total number of possible calendars: 3 × 2 = 6 59. Decision algorithm to select a particular iteration: Step 1: Choose a value for . : 10 choices. Step 2: Choose a value for / : 19 choices (19 integers in the range 2–20).


Solutions Section 7.3 Step 3: Choose a value for ) : 10 choices. Total number of iterations: 10 × 19 × 10 = 1,900 60. Decision algorithm to select a particular iteration: Step 1: Choose a value for . : 2 choices. Step 2: Choose a value for / : 2 choices. Step 3: Choose a value for ) : 2 choices. Total number of iterations: 2 3 = 8

61. Decision algorithm to place a single 1 × 1 block: Step 1: Choose a position in the left-right direction: 0 choices. Step 2: Choose a position in the front-back direction: choices. Step 3: Choose a position in the up-down direction: 1 choices. Total number of possible solids: 0 1 62. Decision algorithm to select and entry of an 0 × matrix: Step 1: Choose a row: 0 choices. Step 2: Choose a column: choices. Total number of entries: 0

63. The number of possible sequences of length 1 is 2 (either a dot: 2 or a dash: ) The number of possible sequences of length 2 is 2 × 2 = 4 (2 steps, 2 choices per step): 22, 2 , 2, The number of possible sequences of length 3 is 2 3 = 8 (3 steps, 2 choices per step). The number of possible sequences of length 4 is 2 4 = 16 (3 steps, 2 choices per step). So far, using different lengths from 1 to 4, we can encode 1 + 2 + 4 + 8 + 16 = 31 possible letters, which is enough to include the whole alphabet. (Using lengths 1–3 will not work, since that will give only 1 + 2 + 4 + 8 = 15 possible letters.) Thus, we need to use sequences of up to 4 dots and dashes. 64. Odd numbers are those that end in an odd digit: 1, 3, 5, 7, 9. Step 1: Choose an odd second digit: 5 choices. Step 2: Choose a first digit in the range 1–9 that is different from the first: 8 choices. Total number of choices: 5 × 8 = 40 numbers 65. The multiplication principle is based on the cardinality of the Cartesian product of two sets. 66. The addition principle is based on the cardinality of the union of two disjoint sets. 67. The decision algorithm produces every pair of shirts twice, first in one order and then in the other. Therefore, it is not valid. The actual number of pairs of shirts is half of the number computed by the algorithm: 9032 = 45. 68. The decision algorithm produces each sequence multiple times; for instance, "yyybg" is produced in several ways by choosing the yellow squares in different orders. 69. Think of placing the five squares in a row of five empty slots. Step 1: Choose a slot for the blue square: 5 choices. Step 2: Choose a slot for the green square: 4 choices. Step 3: Choose the remaining 3 slots for the yellow squares: 1 choice. Hence, there are 20 possible five-square sequences. 70. Here is one example: You are buying a home computer and must decide between a Mac and PC, and also between a laptop and a desktop.


Solutions Section 7.4 Section 7.4

1. 6! = 6 × 5 × 4 × 3 × 2 × 1 = 720

3.

2. 7! = 7 × 6 × 5 × 4 × 3 × 2 × 1 = 5,040

8! 8 × 7 × 6 × 5 × 4 × 3 × 2 × 1 = = 56 6! 6×5×4×3×2×1

5. 4 (6, 4) = 6 × 5 × 4 × 3 = 360

7.

3×2 = 3 2×1 Alternatively, (3, 2) =

8.

(3, 3 2) =

(3, 1) = 3

11. (10, 8) = (10, 2) (because 10 8 = 2) 10 × 9 = = 45 2×1 13. (20, 1) =

10! 10 × 9 × 8 × 7 × 6 × 5 × 4 × 3 × 2 × 1 = 8! 8×7×6×5×4×3×2×1 = 90

6. 4 (8, 3) = 8 × 7 × 6 = 336

4 (6, 4) 6 × 5 × 4 × 3 = = 15 4! 4×3×2×1

9. (3, 2) =

4.

4 (8, 3) 8 × 7 × 6 = = 56 3! 3×2×1

4×3×2 = 4 3×2×1 Alternatively, (4, 3) = (4, 4 3) = 10. (4, 3) =

(4, 1) = 4

12. (11, 9) = (11, 2) (because 11 9 = 2) 11 × 10 = = 55 2×1

20 = 20 1

14. (30, 1) =

15. (100, 98) = (100, 2) (because 100 98 = 2) 100 × 99 = = 4,950 2×1

30 = 30 1

16. (100, 97) = (100, 3) (because 100 97 = 3) 100 × 99 × 98 = = 161,700 3×2×1

17. The number of ordered lists of 4 items chosen from 6 is 4 (6, 4) = 6 × 5 × 4 × 3 = 360.

18. The number of ordered lists of 3 items chosen from 7 is 4 (7, 3) = 7 × 6 × 3 = 210.

19. The number of unordered lists of 3 items chosen from 7 is (7, 3) =

20. The number of unordered sets of 4 objects chosen from 6 is (6, 4) =

Alternatively, (6, 4) =

(6, 2) =

6×5 = 15. 2×1

7×6×3 = 35. 3×2×1

6×5×4×3 = 15. 4×3×2×1

21. Each 5-letter sequence containing the letters b, o, g, e, y is a list of 5 letters chosen from the above 5 (or a permutation of the 5 letters) and the number of these is 4 (5, 5) = 5! = 5 × 4 × 3 × 2 × 1 = 120.


Solutions Section 7.4 22. Each 6-letter sequence containing the letters q, u, a, k, e, s is a list of 6 letters chosen from the above 6 (or a permutation of the 6 letters) and the number of these is 4 (67, 6) = 6! = 6 × 5 × 4 × 3 × 2 × 1 = 720. 23. Each 3-letter sequence is a list of 3 letters chosen from the given 6, and so the number of 3-letter sequences is 4 (6, 3) = 6 × 5 × 4 = 120. 24. Each 3-letter sequence is a list of 3 letters chosen from the given 5, and so the number of 3-letter sequences is 4 (5, 3) = 5 × 4 × 3 = 60. 25. Each 3-letter unordered set is a list of 3 letters chosen from the given 6, and so the number of 3-letter sets is 6×5×4 (6, 3) = = 20. 3×2×1 26. Each 3-letter unordered set is a list of 3 letters chosen from the given 5, and so the number of 3-letter sets is 5×4×3 (5, 3) = = 10. 3×2×1 27. Since there are repeated letters, we use a decision algorithm to construct such a sequence. Start with 6 empty slots. Step 1: Choose a slot for the k: 6 choices. Step 2: Choose 3 slots from the remaining 5 for the a's: (5, 3) choices. Step 3: Choose 2 slots from the remaining 2 for the u's: (2, 2) slots. Total number of sequences: 6 × (5, 3) × (2, 2) = 60 28. Since there are repeated letters, we use a decision algorithm to construct such a sequence. Start with 6 empty slots. Step 1: Choose 2 slots for the a's: (6, 2) choices. Step 2: Choose 4 slots from the remaining 4 for the f's: (4, 4) choices. Total number of sequences: (6, 2) × (4, 4) = 15 29. There are a total of 10 marbles in the bag. The number of sets of 4 chosen from 10 marbles is (10, 4) = 210. 30. There are a total of 10 marbles in the bag. The number of sets of 3 chosen from 10 marbles is (10, 3) = 120. 31. Decision algorithm for assembling a collection of 4 marbles that includes all the red ones: Step 1: Choose 3 red marbles: (3, 3) = 1 choice. Step 2: Choose 1 non-red marble: (7, 1) = 7 choices. (There are 7 non-red marbles to choose from.) Total number of sets = (3, 3) (7, 1) = 7 32. Decision algorithm for assembling a collection of 3 marbles that includes all the yellow ones: Step 1: Choose 2 yellow marbles: (2, 2) = 1 choice. Step 2: Choose 1 non-yellow marble: (8, 1) = 8 choices. (There are 8 non-yellow marbles to choose from.) Total number of sets = (2, 2) (8, 1) = 8


Solutions Section 7.4 33. Decision algorithm for assembling a collection of 4 marbles that includes no red ones: Step 1: Choose 4 non-red marbles: (7, 4) = 35 choices. (There are 7 non-red marbles to choose from.) Total number of sets = (7, 4) = 35. 34. Decision algorithm for assembling a collection of 3 marbles that includes no yellow ones: Step 1: Choose 3 non-yellow marble: (8, 3) = 56 choices. (There are 8 non-yellow marbles to choose from.) Total number of sets = (8, 3) = 56. 35. Decision algorithm for assembling a collection as specified: Step 1: Choose 1 red marble: 3 choices. Step 2: Choose 1 green marble: 2 choices. Step 3: Choose 1 yellow marble: 2 choices. Step 4: Choose 1 orange marble: 2 choices. Total number of sets: 3 × 2 × 2 × 2 = 24 36. Decision algorithm for assembling a collection as specified: Step 1: Choose 1 red marble: 3 choices. Step 2: Choose 1 green marble: 2 choices. Step 3: Choose 1 lavender marble: 1 choice. Step 4: Choose 1 yellow marble: 2 choices. Step 5: Choose 1 orange marble: 2 choices. Total number of sets: 3 × 2 × 1 × 2 × 2 = 24 37. A set containing at least 2 red marbles must contain either 2 or 3 red marbles. Alternative 1: Decide on 2 red marbles. Step 1: Choose 2 red marbles: (3, 2) choices. Step 2: Choose 3 non-red marbles: (7, 3) choices. Alternative 3: Decide on 3 red marbles. Step 1: Choose 3 red marbles: (3, 3) choices. Step 2: Choose 2 non-red marbles: (7, 2) choices. Total number of choices: (3, 2) (7, 3) + (3, 3) (7, 2) = 126 38. A set containing at least 1 yellow marbles must contain either 1 or 2 yellow marbles. Alternative 1: Decide on 1 yellow marble. Step 1: Choose 1 yellow marble: (2, 1) choices. Step 2: Choose 4 non-yellow marbles: (8, 4) choices. Alternative 3: Decide on 2 yellow marbles. Step 1: Choose 2 yellow marbles: (2, 2) choices. Step 2: Choose 3 non-yellow marbles: (8, 3) choices. Total number of choices: (2, 1) (8, 4) + (2, 2) (8, 3) = 196 39. A set containing at most 1 yellow marbles must contain either 0 or 1 yellow marbles. Alternative 1: Decide on 0 yellow marbles. Step 1: Choose 5 non-yellow marbles: (8, 5) choices. Alternative 2: Decide on 1 yellow marble. Step 1: Choose 1 yellow marble: (2, 1) choices. Step 2: Choose 4 non-yellow marbles: (8, 4) choices. Total number of choices: (8, 5) + (2, 1) (8, 4) = 196 40. A set containing at most 1 red marbles must contain either 0 or 1 red marbles.


Solutions Section 7.4 Alternative 1: Decide on 0 red marbles. Step 1: Choose 5 non-red marbles: (7, 5) choices. Alternative 2: Decide on 1 red marble. Step 1: Choose 1 red marble: (3, 1) choices. Step 2: Choose 4 non-red marbles: C(7, 4) choices. Total number of choices: (7, 5) + (3, 1) (7, 4) = 126 41. Decision algorithm for assembling a collection as specified: Alternative 1: Use the lavender marble but no yellow ones. Step 1: Select the lavender marble: (1, 1) = 1 choice. Step 2: Select 4 non-lavender non-yellow marbles: (7, 4) choices. Alternative 2: Use a yellow marble but no lavender ones. Step 1: Select 1 yellow marble: (2, 1) choices. Step 2: Select 4 non-lavender non-yellow marbles: (7, 4) choices. Total number of sets: (1, 1) (7, 4) + (2, 1) (7, 4) = 105 42. Decision algorithm for assembling a collection as specified: Alternative 1: Decide on 1 yellow marble. Step 1: Select 1 yellow marble: (2, 1) choices. Step 2: Select 4 non-yellow non-green marbles: (6, 4) choices. Alternative 2: Decide on 2 yellow marbles. Step 1: Select 2 yellow marbles: (2, 2) choices. Step 2: Select 3 non-yellow non-green marbles: (6, 3) choices. Total number of sets: (2, 1) (6, 4) + (2, 2) (6, 3) = 50 43. Think of the outcome of a sequence of 30 dice throws as a sequence of "words" of length 30 using the "letters" 1, 2, 3, ... , 6. Using this interpretation, a sequence with five 1s is a 30-letter word containing five 1s. For the decision algorithm, start with 30 empty slots and choose numbers to fill the slots: Step 1: Choose five slots for the 1s: (30, 5) choices. Steps 2–26: Choose one of 2, 3, 4, 5, 6 to fill each of the remaining 25 slots: 5 × 5 × ... × 5 = 5 25 choices. Total number of sequences with five 1s: (30, 5) × 5 25 Since there are 6 30 different sequences possible, the fraction of sequences with five 1s is (30, 5) × 5 25 ! 0.192. 6 30 44. Think of the outcome of a sequence of 30 dice throws as a sequence of "words" of length 30 using the "letters" 1, 2, 3, ... , 6. Using this interpretation, a sequence with five 1s and five 2s is a 30-letter word containing five 1s and five 2s. For the decision algorithm, start with 30 empty slots and choose numbers to fill the slots: Step 1: Choose five slots for the 1s: (30, 5) choices. Step 2: Choose five slots for the 2s: (25, 5) choices. Steps 3–22: Choose one of 3, 4, 5, 6 to fill each of the remaining 20 slots: 4 × 4 × ... × 4 = 4 20 choices Total number of sequences with five 1s and five 2s: (30, 5) × (25, 5) × 4 20 Since there are 6^{30} different sequences possible, the fraction of sequences with five 1s and five 2s is (30, 5) (25, 5) × 4 20 ! 0.0377. 6 30 45. For the decision algorithm, start with 30 empty slots and choose numbers to fill the slots: Step 1: Choose 15 slots for the even numbers: (30, 15) choices. Next 15 steps: Choose 2, 4, or 6 for each of these 15 slots: 3 15 choices.


Solutions Section 7.4 Next 15 steps: Choose 1, 3, or 5 for each of the remaining 15 slots: 3 15 choices. Since there are 6^{30} different sequences possible, the fraction of sequences with exactly 15 even numbers is (30, 15) × 3 15 × 3 15 ! 0.144. 6 30 46. For the decision algorithm, start with 30 empty slots and choose numbers to fill the slots: Step 1: Choose 10 slots for the numbers less than or equal to 2: (30, 10) choices. Next 10 steps: Choose 1 or 2 for each of these 10 slots: 2 10 choices. Next 20 steps: Choose 3, 4, 5, or 6 for each of the remaining 20 slots: 4 20 choices. Since there are 6 30 different sequences possible, the fraction of sequences with exactly 10 numbers less than or equal to 2 is (30, 10) × 2 10 × 4 20 ! 0.153. 6 30 47. Start with 11 empty slots. Decision algorithm: Step 1: Select a slot for the m: (11, 1) = 11 choices. Step 2: Select 4 slots for the i's: (10, 4) choices. Step 3: Select 6 slots for the s's: (6, 4) choices. Step 4: Select 2 slots for the p's: (2, 2) = 1 choice. Total number of sequences: (11, 1) (10, 4) (6, 4) (2, 2) 48. Start with 11 empty slots. Decision algorithm: Step 1: Select 2 slots for the m's: (11, 2)choices. Step 2: Select 1 slot for the e: (9, 1) = 9 choices. Step 3: Select 1 slot for the s: (8, 1) = 8 choices. Step 4: Select 2 slots for the o's: (7, 2) choices. Step 5: Select 1 slot for the p: (5, 1) = 5 choices. Step 6: Select 1 slot for the t: (4, 1) = 4 choices. Step 7: Select 2 slots for the 1's: (3, 2) choices. Step 8: Select 1 slot for the i: (1, 1) = 1 choice. Total number of sequences: (11, 2) (9, 1) (8, 1) (7, 2) (5, 1) (4, 1) (3, 2) (1, 1) 49. Start with 11 empty slots. Decision algorithm: Step 1: Select 2 slots for the m's: (11, 2)choices. Step 2: Select 1 slot for the e: (9, 1) = 9 choices. Step 3: Select 1 slot for the g: (8, 1) = 8 choices. Step 4: Select 3 slots for the a's: (7, 3) choices. Step 5: Select 1 slot for the l: (4, 1) = 4 choices. Step 6: Select 1 slot for the o: (3, 1) = 3 choices. Step 7: Select 1 slot for the n: (2, 1) = 2 choices. Step 8: Select 1 slot for the i: (1, 1) = 1 choice. Total number of sequences: (11, 2) (9, 1) (8, 1) (7, 3) (4, 1) (3, 1) (2, 1) (1, 1) 50. Start with 13 empty slots. Decision algorithm: Step 1: Select 1 slot for the s: (13, 1) = 13choices. Step 2: Select 1 slot for the c: (12, 1) = 12 choices. Step 3: Select 2 slots for the h's (11, 2) choices. Step 4: Select 2 slots for the i's (9, 2) choices. Step 5: Select 1 slot for the z: (7, 1) = 7 choices. Step 6: Select 1 slot for the o: (6, 1) = 6 choices.


Solutions Section 7.4 Step 7: Select 1 slot for the p: (5, 1) = 5 choices. Step 8: Select 1 slot for the r: (4, 1) = 4 choices. Step 9: Select 1 slot for the e: (3, 1) = 3 choices. Step 10: Select 1 slot for the n: (2, 1) = 2 choices. Step 11: Select 1 slot for the a: (1, 1) = 1 choice. Total number of sequences: (13, 1) (12, 1) (11, 2) (9, 2) (7, 1) (6, 1) (5, 1) (4, 1) (3, 1) (2, 1) (1, 1) 51. Start with 10 empty slots. Decision algorithm: Step 1: Select 2 slots for the c's: (10, 2)choices. Step 2: Select 4 slots for the a's: (8, 4) choices. Step 3: Select 1 slot for the s: (4, 1) = 4 choices. Step 4: Select 1 slot for the b: (3, 1) = 3 choices. Step 5: Select 1 slot for the l: (2, 1) = 2 choices. Step 6: Select 1 slot for the n: (1, 1) = 1 choice. Total number of sequences: (10, 2) (8, 4) (4, 1) (3, 1) (2, 1) (1, 1) 52. Start with 10 empty slots. Decision algorithm: Step 1: Select 2 slots for the d's: (10, 2)choices. Step 2: Select 2 slots for the e's: (8, 2) choices. Step 3: Select 1 slot for the s: (6, 1) = 6 choices. Step 4: Select 1 slot for the m: (5, 1) = 5 choices. Step 5: Select 1 slot for the o: (4, 1) = 4 choices. Step 6: Select 1 slot for the r: (3, 1) = 3 choices. Step 7: Select 1 slot for the l: (2, 1) = 2 choices. Step 8: Select 1 slot for the a: (1, 1) = 1 choice. Total number of sequences: (10, 2) (8, 2) (6, 1) (5, 1) (4, 1) (3, 1) (2, 1) (1, 1) 53. Each itinerary is a list of 4 venues chosen from 4. Thus, the number of itineraries is 4 (4, 4) = 4! = 4 × 3 × 2 × 1 = 24. 54. Each itinerary is a list of 3 venues chosen from 3 (the 4th stop is already decided.) Thus, the number of itineraries is 4 (3, 3) = 3! = 3 × 2 × 1 = 6. 55. Decision algorithm for constructing a two pair hand: Step 1: Select 2 denominations for the pairs: (13, 2) choices. Step 2: Select two cards of the lowest-ranked denomination above: (4, 2) choices. Step 3: Select two cards of the other denomination above: (4, 2) choices. Step 4: Select a single card that belongs to neither of the two denominations: (44, 1) = 44 choices. (52 8 = 44) Total number of two pair hands: (13, 2) (4, 2) (4, 2) (44, 1) = 123,552 56. Decision algorithm for constructing a three of a kind hand: Step 1: Select a denomination for the three cards: (13, 1) = 13 choices. Step 2: Select 3 cards of the above denomination: (4, 3) = 4 choices. Step 3: Select 2 denominations for the singles: (12, 2) choices. Step 4: Select 1 card of the lowest-ranked denomination above: (4, 1) = 4 choices. Step 5: Select 1 card of the other denomination: (4, 1) = 4 choices. Total number of three of a kind hands: 13 × 4 × (12, 2) × 4 × 4 = 54,912 57. Decision algorithm for constructing a two of a kind hand:


Solutions Section 7.4 Step 1: Select a denomination for the two cards: (13, 1) = 13 choices. Step 2: Select 2 cards of the above denomination: (4, 2) choices. Step 3: Select 3 denominations for the singles: (12, 3) choices. Step 4: Select 1 card of the lowest-ranked denomination above: (4, 1) = 4 choices. Step 5: Select 1 card of the next-ranked denomination: (4, 1) = 4 choices. Step 6: Select 1 card of the highest-ranked: (4, 1) = 4 choices. Total number of two of a kind hands: 13 × (4, 2) (12, 3) × 4 × 4 × 4 = 1,098,240 58. Decision algorithm for constructing a two pair hand: Step 1: Select a denomination for the four cards: (13, 1) = 13 choices. Step 2: Select 4 cards of the above denomination: (4, 4) = 1 choice. Step 3: Select a denomination for the single: (12, 1) choices. Step 4: Select 1 card of that denomination: (4, 1) = 4 choices. Total number of two pair hands: 13 × 1 × (12, 1) × 4 = 624 59. A straight is a run of 5 cards of consecutive denominations: A, 2, 3, 4, 5 up through 10, J, Q, K, A, but not all of the same suit. If we ignore the restriction about the suits, we get the following decision algorithm: Step 1: Select a starting denomination for the straight: (A, 2, ..., 10): 10 choices. Steps 2–6: Select a card of each of the above denominations: 4 × 4 × 4 × 4 × 4 = 4 5 choices. Total number of runs of 5 cards: 10(4 5 However, these runs include hands in which all 5 cards are of the same suit (straight flushes). There are 10 × 4 of these (Step 1: Select a starting card; Step 2: Select a single suit). Excluding the straight flushes gives Total number of straights: 10 × 4 5 10 × 4 = 10,200. 60. First compute the number of hands in which all 5 cards are of the same suit: Step 1: Select a suit: 4 choices. Step 2: Select 5 cards of that suit: (13, 5) choices. Total number of choices: 4 × (13, 5) Among these are hands the 5 cards are consecutive: A, 2, 3, 4, 5 up through 10, J, Q, K, A (straight flushes). The number of straight flushes is 4 × 10 (Step 1: Select a suit; Step 2: Select a starting card for the run). Excluding these gives Total number of flushes: 4 × (13, 5) 4 × 10 = 5,108. 61. a. Since a portfolio consists of a set of five stocks chosen from the 10, the number of possible portfolios is (10, 5) = 252 portfolios. b. Decision algorithm for assembling a portfolio as per part (b): Step 1: Choose VZ and CSCO: C(2, 2) = 1 choice Step 2: Choose 3 more stocks from 6 (VZ, CSCO, WBA, MRK excluded): (6, 3) = 20 Total number of choices: 1 × 20 = 20 portfolios c. Five of the listed stocks have yields above 4%. Decision algorithm for assembling a portfolio as per part (c): Alternative 1: Exactly 4 stocks have yields above 4%: Step 1: Choose 4 stocks with yields above 4%: (5, 4) = 5 choices. Step 2: Choose 1 stock with a yield not above 4%: (5, 1) = 5 choices. Total number of choices for Alternative 1: 5 × 5 = 25 choices. Alternative 2: All 5 stocks have yields above 4%: Step 1: Choose 5 stocks with yields above 4%: (5, 5) = 1 choice. Total number of choices for Alternative 2: 1 choice. Total number of portfolios = 25 + 1 = 26


Solutions Section 7.4 62. a. Since a portfolio consists of a set of six stocks chosen from the 10, the number of possible portfolios is (10, 6) = 210 portfolios. b. Decision algorithm for assembling a portfolio as per part (b): Step 1: Choose DOW: (1, 1) = 1 choice. Step 2: Choose 5 more stocks from 8 (DOW, MMM excluded): (8, 5) = 56 choices. Total number of choices: 1 × 56 = 56 portfolios c. At most one stock priced above $100. Decision algorithm for assembling a portfolio as per part (c): Alternative 1: Exactly 1 stock is priced above $100: Step 1: Choose 1 stock priced above $100: (3, 1) = 3 choices. Step 2: Choose 5 stocks not priced above $100: (7, 5) = 21 choices. Total number of choices for Alternative 1: 3 × 21 = 63 choices. Alternative 2: No stocks priced above $100: Step 1: Choose 6 stocks not priced above $100: (7, 6) = 7 choices. Total number of choices for Alternative 2: 7 choices Total number of portfolios = 63 + 7 = 70 63. a. Decision algorithm for assembling a collection of stocks: Step 1: Choose 3 tech stocks: (6, 3) = 20 choices. Step 2: Choose 2 non-tech stocks: (6, 2) = 15 choices. Total number of choices: 20 × 15 = 300 collections b. Decision algorithm for assembling a collection of stocks that declined in value: Step 1: Choose 3 declining tech stocks: (5, 3) = 10 choices. Step 2: Choose 2 declining non-tech stocks: (3, 2) = 3 possible choices. Total number of choices: 10 × 3 = 30 collections c. Since only 30 collections of out a possible 300 consist entirely of stocks that declined in value, the chances of selecting one of those collections is 30 in 300; that is, 1 in 10, or .1.

64. a. The number of collections of 3 chosen from 12 is (12, 3) = 220 b. Decision algorithm for assembling a collection of stocks as per part (b): Step 1: Choose 2 tech stocks that decreased: (5, 2) = 10 choices. Step 2: Choose 1 stock other than a tech stock that decreased: (7, 1) = 7 choices. Total number of choices: 10 × 7 = 70 collections c. Since 70 collections of out a possible 220 included exactly two tech stocks that decreased in value, the chances of selecting one of those collections is 70 in 220, or about 0.318.

65. a. As a seeding consists of a list of 16 teams, the number of seedings is 4 (16, 16) = 16! b. To choose a seeding of the designated type, we have the following decision algorithm: Choose the order in which the games 1 vs 16, 2 vs 15, etc. appear: 4 (8, 8) = 8! In each of these 8 games, choose the order of the teams: 2 8. So, the number is 4 (8, 8) × 2 8 = 8! × 2 8. 66. a. As a seeding consists of a list of 8 teams, the number of seedings is 4 (8, 8) = 8! b. To choose a seeding of the designated type, we have the following decision algorithm: Choose the order in which the games 1 vs 2, 3 vs 4, etc. appear: 4 (4, 4) = 4! In each of these 4 games, choose the order of the teams: 2 4. So, the number is 4 (4, 4) × 2 4 = 4! × 2 4.

67. There are 32 first-round games, 16 second-round games, 8 third round games, 4 fourth-round games, 2 fifth-round games, and one final, giving a total of 63 games. Decision algorithm for picking the winners with 15 upsets in the first four rounds: Step 1: Choose 15 games in which the upsets occur: (60, 15) choices Step 2: Choose the winners for each of the three games in the last two rounds: 2 × 2 × 2 = 8.


Solutions Section 7.4 Total number of possiblties: (60, 15) × 8 = 425,552,713,541,760. 68. There are 32 first-round games, 16 second-round games, 8 third round games, and 4 fourth-round games, giving a total of 60 games in the first four rounds. There are two more rounds to complete the tourmament (called "the final four" and the finals): 2 fifth-round games, and one final. Decision algorithm for picking the winners with 10 upsets in the first round, 4 in the second round, and three in each of the third and fourth rounds: Step 1: Choose 10 fist-round games games in which the upsets occur: (32, 10) choices Step 2: Choose 4 second-round games games in which the upsets occur: (16, 4) choices Step 3: Choose 3 third-round games games in which the upsets occur: (8, 3) choices Step 4: Choose 3 fourth-round games games in which the upsets occur: (4, 3) choices Step 2: Choose the winners for each of the three games in the last two rounds: 2 × 2 × 2 = 8. Total number of possiblties: (32, 10) (16, 4) (8, 3) (4, 3) × 8 = 210,402,800,025,600.

69. a. The number of groups of 4 chosen from 10 is (10, 4) = 210. b. Decision algorithm for assembling a group of 4 movies that satisfies the Lara twins: Alternative 1: Exactly one of "Snake Eyes" or "The Tomorrow War" Step 1: Choose 1 out of "Snake Eyes" or "The Tomorrow War": (2, 1) = 2 choices. Step 2: Choose 3 from the remaining 7 ("A Quiet Place Part II" excluded): (7, 3) = 35 choices. Total number of groups in Alternative 1 is therefore 2 × 35 = 70. Alternative 2: Both "Snake Eyes" and "The Tomorrow War" Step 1: Choose 2 out of "Snake Eyes" and "The Tomorrow War": (2, 2) = 1 choice. Step 2: Choose 2 from the remaining 7 ("Snake Eyes", "The Tomorrow War", and "CODA" excluded): (7, 2) = 21 choices. Total number of groups in Alternative 2 is therefore 1 × 21 = 21. Total number of groups that make the Lara twins happy is therefore 70 + 21 = 91. c. As 91 of the 210 groups (less than half) make the Lara twins happy, they are less likely than not to be satisfied with a random selection.

70. a. The number of groups of 3 chosen from 10 is (10, 3) = 120. b. Decision algorithm for assembling a group of 3 movies that satisfies the Pelogrande twins: Alternative 1: One of the movies ranked 7 or 8 is in the group of 3. Step 1: Choose 1 of the movies ranked 7 or 8 : (2, 1) = 2 choices. Step 2: Choose 2 other movies ranked higher than 7 : (6, 2) = 15 choices. Total number of choices for alternative 1: 2 × 15 = 30 choices. Alternative 2: None of the movies ranked 7 to 10 in the group of 3. Choose 3 movies from the remaining 6 : (6, 3) = 20 choices. Total number of choices = 30 + 20 = 50. c. As 50 of the 120 groups (less than half) make the Pelogrande twins happy, they are less likely than not to be satisfied with a random selection. 71. a. Each itinerary is a permutation of the 23 listed cities: 23! of them altogether. b. Once the first five stops are determined, a decision algorithm for completing the itinerary is: Steps 1–18: Select a different city from the remaining 18 for each remaining stop: 18 × 17 × ... × 2 × 1 choices. Total number of itineraries: 18 × 17 × ... × 2 × 1 = 18! c. Decision algorithm for constructing an itinerary of the required type (think of making an itinerary as filling a sequence of 23 empty slots with different cities): Step 1: Choose a starting point in the itinerary to insert the sequence of 5 named cities: 1 through 19: 19 choices. Next 18 steps: Select a different city from the remaining 18 for each remaining slot: 18 × 17 × ... × 2 × 1 choices.


Total number of itineraries: 19 × 18!

Solutions Section 7.4

72. a. To complete the itinerary: Steps 1–22: Select a different city from the remaining 22 for each remaining stop: 22 × 21 × ... × 2 × 1 choices. Total number of itineraries: 22! b. Think of an itinerary as a sequence of 25 slots in which the first and last are already filled (with Detroit) and the understanding that Chicago will appear twice: Steps 1–21: Choose slots of the unfilled 23 slots for the 21 non-repeating cities: 4 (23, 21) choices. Total number of itineraries: 4 (23, 21) c. Decision algorithm for making an itinerary: Start with 22 empty slots: Step 1: Choose a starting point to insert the sequence of 5 named cities: 1 through 18: 18 choices. Next 17 steps: Select a different city from the remaining 17 for each remaining slot: 17 × ... × 2 × 1 choices. Total number of itineraries: 18 × 17! 73. We wish to compute the number of groups of 4 chosen from 5 contestants (since we are excluding Ben and Ann): (5, 4) = 5 possible groups. (Choice A). 74. Decision algorithm: Step 1: Choose Ben: 1 choice. Step 2: Choose 3 more from the remaining group excluding Ann: (5, 3) = 10 choices. Total number of possible groups: 1 × 10 = 30 (Choice D). 75. Each handshake corresponds to a pair of people, so the question is really asking how many pairs of people there are in a group of 10: (10, 2) = 45 (Choice D). 76. Each exchange of business cards corresponds to a pair of business people, and the number of pairs of business people in a group of 12 is (12, 2) = 66. Since each pair of people results in 2 business cards changing hands, the total number of cards exchanged is twice the number: 2 × 66 = 132 (Choice C) 77. a. Note: In a set of 3 numbers, all 3 are different. The number of combinations = the number of sets of 3 numbers chosen from 40: (40, 3) = 9,880. b. Decision algorithm for constructing a combination in which a number appears twice: Step 1: Select the number that appears twice: 40 choices. Step 3: Select another number: 39 choices. Total number of new combinations: 40 × 39 = 1,560 c. Decision algorithm for constructing a combination in which a number appears 3 times: Step 1: Select the number that appears 3 times: 40 choices. This gives 40 new combinations. Total number of combinations: 9,880 + 1,560 + 40 = 11,480 78. a. Note: In a set of 3 numbers, all 3 are different. The number of combinations = the number of sets of 3 numbers chosen from 20: (20, 3) = 1,140 b. Decision algorithm for constructing a combination in which a number appears twice: Step 1: Select the number that appears twice: 20 choices. Step 2: Select another number: 19 choices. Total number of new combinations: 20 × 19 = 380 c. Decision algorithm for constructing a combination in which a number appears 3 times: Step 1: Select the number that appears 3 times: 20 choices. This gives 20 new combinations. Total number of combinations: 1,140 + 380 + 20 = 1,540


Solutions Section 7.4 79. a. The number of combinations of 2 equations chosen from the 20 constraints is (20, 2) = 190 systems of equations to solve. b. Replace 20 by , getting ( , 2).

80. a. The number of combinations of 3 equations chosen from the 20 constraints is (20, 3) = 1,140 systems of equations to solve. b. Replace 20 by , getting ( , 3). 81. You should choose the multiplication principle because the multiplication principle can be used to solve all problems that call for the formulas for permutations, as well as others. 82. No; ( , 1) is the number of possible sets of 1 objects chosen from , and hence is always a whole number; if ( , 1) is computed as a fraction, the denominator will always cancel. 83. A permutation is an ordered list, or sequence, and only choices (A) and (D) can be represented as ordered lists. (In a presidential cabinet, each portfolio is distinct.) 84. Since only (A) and (B) represent unordered collections, only these represent combinations. 85. A permutation. Changing the order in a list of driving instructions can result in a different outcome; for instance, "1. Turn left. 2. Drive one mile." and "1. Drive one mile. 2. Turn left." will take you to different locations. 86. Wrong; you are really selecting an unordered set, or combination, of 5 students, and then arranging them in alphabetical order. The order in which the students on the list is selected is not relevant; therefore what you have is a combination. 87. Urge your friend not to focus on formulas but instead to learn to formulate decision algorithms and use the principles of counting. 88. If a counting procedure has 5 alternatives, each of which has 4 steps of 2 choices each, then there are 2 4 + 2 4 + 2 4 + 2 4 + 2 4 = < ' > 80 < 3' > outcomes. On the other hand, if there are 5 steps, each of which has 4 alternatives of 2 choices each, then there are 8 × 8 × 8 × 8 × 8 = < ' > 32,768 < 3' > outcomes. 89. It is ambiguous on the following point: Are the three students to play different characters, or are they to play a group of three, such as "three guards"? This should be made clear in the exercise. 90. ( + ) 6 = ( + )( + )( + )( + )( + )( + ) When we use the distributive rule to multiply it out, we get a term 2 4 for each choice of 2 's and 4 's to multiply together. Each of these choices comes from a different factor ( + ). Step 1: Choose 2 factors ( + ) for the : (6, 2) choices Step 2: Choose 4 remaining factors ( + ) for the : (4, 4) = 1 choice Total number of choices: (6, 2) × (1, 1) = (6, 2) 91. Here is a decision algorithm: Alternative 1: include the last element. Step 1: Choose 1 1 of the first 1 elements, ( 1, 1 1) choices. Step 2: Choose the last element, 1 choice Total number of choices possible in Alternative 1: ( 1, 1 1). Alternative 2: do not include the last element.


Solutions Section 7.4 We must choose our 1 elements from the first 1, so there are ( 1, 1) possible choices in Alternative 2. Thus, there are ( 1, 1 1) + ( 1, 1) possible ways to choose 1 items from , but we know that the same number is given by ( , 1), so we must have ( , 1) = ( 1, 1 1) + ( 1, 1).

92. When 1 = 0, the equation says that ( , 0) = ( 1, 1) + ( 1, 0). Because ( , 0) = 1 = ( 1, 0), this means that we should define ( 1, 1) = 0 for the formula to continue to work in this case. Similarly, when 1 = , we see that we should define ( 1, ) = 0. In fact, it is convenient to define ( , 1) = 0 whenever 1 < 0 or 1 > .


Solutions Chapter 7 Review Chapter 7 Review

1. The negative integers greater than or equal to 3 are 3, 2, and 1. Thus, = { 3, 2, 1}

2. There are 2 × 2 × 2 × 2 × 2 = 2 5 = 32 possible outcomes: = {HHHHH, HHHHT, HHHTH, HHHTT, HHTHH, HHTHT, HHTTH, HHTTT, HTHHH, HTHHT, HTHTH, HTHTT, HTTHH, HTTHT, HTTTH, HTTTT, THHHH, THHHT, THHTH, THHTT, THTHH, THTHT, THTTH, THTTT, TTHHH, TTHHT, TTHTH, TTHTT, TTTHH, TTTHT, TTTTH, TTTTT} 3. If the dice are distinguishable, then the outcomes can be thought of as ordered pairs. Thus, since the numbers are different, is the set of all ordered pairs of distinct numbers in the range 1–6: = {(1, 2), (1, 3), (1, 4), (1, 5), (1, 6), (2, 1), (2, 3), (2, 4), (2, 5), (2, 6), (3, 1), (3, 2), (3, 4), (3, 5), (3, 6), (4, 1), (4, 2), (4, 3), (4, 5), (4, 6), (5, 1), (5, 2), (5, 3), (5, 4), (5, 6), (6, 1), (6, 2), (6, 3), (6, 4), (6, 5)} 4. = {1, 2, 3, 4, 5}, = {3, 4, 5}, = {1, 2, 5, 6, 7} = {3, 4, 5} ( ) = {3, 4, 5} = {1, 2, 3, 4, 5, 6, 7} = {1, 2, 3, 4, 5, 6, 7} ( ) = {1, 2, 3, 4, 5, 6, 7} = {1, 2, 3, 4, 5} 5. = {a, b}, = {b, c}, = {a, b, c, d}, = {a, d} = {a, b} {a, d} = {a, b, d} × = {(a, a), (a, d), (b, a), (b, d)}

6. The set of all customers who owe money but owe less than $1,000 is the set of all customers in and not in : ' 7. The set of outcomes when a day in August and a time of that day are selected is the set of pairs (day in August, time). That is, it is the set × . 8. The set of outcomes in which both dice show an even number or sum to 7 is the set of all outcomes not in or in (or both): 9. The set of all integers that are not odd perfect squares is the set of integers other than those in the odd integers) and 5 : ( 5) or 5

(

10. The set of all even integers that are neither negative nor multiples of three is the set of integers in and neither in nor # : ( # ) or #

11. Let be the set of all novels in your home; ( ) = 400. Let be the event that you have read a novel: ( ) = 150. Let be the event that Roslyn has read a novel: ( ) = 200. We are also told that ( ) = 50. To compute ( ) we use ( ) = ( ) + ( ) ( ) = 150 + 200 50 = 300. The event that neither of you has read a novel is ( ) , and its cardinality is given by [( ) ] = ( ) ( ) = 400 300 = 100. Note that the formula used for the last calculation is ( ) = ( ) ( ). 12. (

) = ( ) + ( ) (

) 32 = 24 + 24 (

) (

) = 24 + 24 32 = 16

=


Solutions Chapter 7 Review 13. The number of outcomes from rolling the dice is ( ( ), where is the set of outcomes of the red dice and is the set of outcomes of the green one. ( ( ) = ( ) ( ) = 6 2 6 = 36 The set of losing combinations has the form , where here is the set of doubles and is the set of outcomes in which the green die shows an odd number and the red die shows an even number. To compute its cardinality, we use ( ) = ( ) + ( ) ( ) = 6 + 3 × 3 0 = 15. The set of winning combinations is the complement of the set of losing combinations and is given by ( ) = ( ) ( ) = 36 15 = 21.

14. A choice of a model and color corresponds to a pair (model, color); that is, to an element of × , where is the set of models; ( ) = 3,and is the set of colors; ( ) = 5. ( × ) = ( ) ( ) = 3 2 5 = 15

15. Decision algorithm for constructing a two of a kind hand with no Aces: Step 1: Select a denomination other than Ace for the pair: (12, 1) choices. Step 2: Select 2 cards of that denomination: C(4, 2) choices. Step 3: Select 3 denominations (other than Ace or the one already selected above) for the remaining singles: (11, 3) choices. Step 4: Select a suit for the highest-ranked denomination just chosen: (4, 1) choices. Step 5: Select a suit for the next highest-ranked denomination just chosen: (4, 1) choices. Step 6: Select a suit for the lowest-ranked denomination just chosen: (4, 1) choices. Total number of hands: (12, 1) (4, 2) (11, 3) (4, 1) (4, 1) (4, 1) 16. Decision algorithm for constructing a full house with either two Kings and three Queens or two Queens and three Kings: Alternative 1: Two Kings and three Queens Step 1: Select a pair of suits for the Kings: (4, 2) choices. Step 2: Select three suite for the Queens: (4, 3) choices. Total for Alternative 1: (4, 2) (4, 3) choices. The second alternative (two Queens and three Kings) gives the same number of choices. Therefore, the total number of such hands is 2 (4, 2) (4, 3). 17. Decision algorithm for constructing a straight flush: Step 1: Select a suit for the flush: (4, 1) choices. Step 2: Select a starting denomination for the consecutive run: (10, 1) choices. (A, 2, 3, 4, 5 up through 10, J, Q, K, A) Total number of hands: (4, 1) (10, 1) 18. Decision algorithm for constructing a three of a kind hand with no Aces: Step 1: Select a denomination other than Ace for the triple: (12, 1) choices. Step 2: Select 3 cards of that denomination: C(4, 3) choices. Step 3: Select 2 denominations (other than Ace or the one already selected above) for the remaining singles: (11, 2) choices. Step 4: Select a suit for the highest-ranked denomination just chosen: (4, 1) choices. Step 5: Select a suit for the lowest-ranked denomination just chosen: (4, 1) choices. Total number of hands: (12, 1) (4, 3) (11, 2) (4, 1) (4, 1)

19. There are a total of 4 + 2 = 6 red and green marbles. Thus, there are (6, 5) = 6 possible sets of such marbles.


Solutions Chapter 7 Review 20. There are a total of 1 + 3 + 2 = 6 marbles that are neither red nor green. Thus, there are (6, 5) = 6 possible sets of such marbles. 21. Decision algorithm for constructing a set of five marbles that include all the red ones: Step 1: Select 4 red marbles: (4, 4) choices. Step 2: Select 1 non-red marble: (8, 1) choices. Total number of choices: (4, 4) (8, 1) = 8

22. The number of sets of 5 marbles chosen from 12 is (12, 5) = 792. By the preceding exercise, 8 of these 792 possible sets of 5 marbles include all the red ones. Therefore, 792 8 = 784 do not. 23. Decision algorithm for constructing a set of 5 marbles that include at least 2 yellow ones: Alternative 1: Use 2 yellow ones: Step 1: Select 2 yellow marbles: (3, 2) choices. Step 2: Select 3 non-yellow marbles: (9, 3) choices. Alternative 2: Use 3 yellow ones: Step 1: Select 3 yellow marbles: (3, 3) choices. Step 2: Select 2 non-yellow marbles: (9, 2) choices. Total number of choices: (3, 2) (9, 3) + (3, 3) (9, 2) = 288 24. Decision algorithm for constructing a set of 5 marbles that include at most one of the red ones but no yellow ones: Alternative 1: Use no red ones: Step 1: Select 5 non-red, non-yellow marbles: (5, 5) choices. Alternative 2: Use 1 red one: Step 1: Select 1 red marble: (4, 1) choices. Step 2: Select 4 non-red, non-yellow marbles: (5, 4) choices. Total number of choices: (5, 5) + (4, 1) (5, 4) = 21 25. # is the set of books that are either sci fi or stored in Texas (or both); ( # ) = ( ) + (# ) ( # ) = 33,000 + 94,000 15,000 = 112,000

26. & is the set of horror books in California; (& (Intersection of the Horror column and California row)

) = 12,000

27. is the set of books that are either stored in California or not sci fi. To compute its cardinality, add the corresponding entries (shown below in thousands): S

H

R

O

Total

W

10

12

12

30

64

C

8

12

6

16

42

T

15

15

20

44

94

Total

33

39

38

90

200

(

) = 175,000


28. (

#)

Solutions Chapter 7 Review &is the set of romance books stored in Texas together with all the horror books.

S

H

R

O

Total

W

10

12

12

30

64

C

8

12

6

16

42

T

15

15

20

44

94

Total

33

39

38

90

200

29.

(#

((

#)

&) = 59,000

&) is the set of romance books that are also horror books or stored in Texas.

S

H

R

O

Total

W

10

12

12

30

64

C

8

12

6

16

42

T

15

15

20

44

94

Total

33

39

38

90

200

(

(#

&)) = 20,000

30. ( " ) (& stored in California.

S

H

R

O

Total

W

10

12

12

30

64

C

8

12

6

16

42

T

15

15

20

44

94

Total

33

39

38

90

200

((

")

(&

)is the set of sci fi books stored in Washington together with the horror books not

)) = 37,000

31. Let & denote the set of OHaganBooks.com customers, the set of JungleBooks.com customers, and the set of FarmerBooks.com customers.

We are told that OHaganBooks.com had 3500 customers: + + + 1,000 = 3,500. 2,000 customers were shared with JungleBooks.com: + 1,000 = 2,000, = 1,000. 1,500 customers were shared with FarmerBooks.com: + 1,000 = 1,500 = 500. We can now compute from the first equation: + 1,000 + 500 + 1,000 = 3,500 = 1,000.


Solutions Chapter 7 Review We are asked for the number that are exclusive OHaganBooks.com customers: = 1,000.

32. Let & denote the set of OHaganBooks.com customers, the set of JungleBooks.com customers, and the set of FarmerBooks.com customers. Let us start with the information from Exercise 31 filled in:

JungleBooks.com has a total of 3,600 customers: + + 2,000 = 3,600 + = 1,600 FarmerBooks.com has 3,400 customers: + + 1,500 = 3,400 + = 1,900. JungleBooks.com and FarmerBooks.com share 1,100 customers between them: + 1,000 = 1,100 = 100 giving + 100 = 1,600 = 1,500 100 + = 1900 = 1,800. The customers they share that are not OHaganBooks.com customers constitute + + = 1,500 + 100 + 1,800 = 3,400.

33. Let & denote the set of OHaganBooks.com customers, the set of JungleBooks.com customers, and the set of FarmerBooks.com customers. Let us start with the information from Exercise 31 filled in:

JungleBooks.com has a total of 3,600 customers: + + 2,000 = 3,600 + = 1,600. FarmerBooks.com has 3,400 customers: + + 1,500 = 3,400 + = 1,900. JungleBooks.com and FarmerBooks.com share 1,100 customers between them: + 1,000 = 1,100 = 100 giving + 100 = 1,600 = 1,500 100 + = 1,900 = 1,800. Thus, OHaganBooks has 1,000 exclusive customers, JungleBooks has = 1,500 exclusive customers, and FarmerBooks has = 1,800 exclusive customers—the most.

34. Let & denote the set of OHaganBooks.com customers, the set of JungleBooks.com customers, and the set of FarmerBooks.com customers. Here is the complete data from Exercise 31 above:


Solutions Chapter 7 Review

OHaganBooks has the smallest number of exclusive customers: 1,000.

35. Let & denote the set of OHaganBooks.com customers, the set of JungleBooks.com customers, and the set of FarmerBooks.com customers. Here is the complete data from Exercise 31 above:

(& ) = 3,500 + 1,500 + 100 = 5,100 (& ) = 3,500 + 100 + 1,800 = 5,400 Thus, OHaganBooks should merge with FarmerBooks.com for a combined customer base of 5{,}400.

36. Let & denote the set of OHaganBooks.com customers, the set of JungleBooks.com customers, and the set of FarmerBooks.com customers. Here is the complete data from Exercise 31 above:

If OHaganBooks merged with JungleBooks, their combined exclusive customer base would be [(& ) ] = 5,100 (1,000 + 500 + 100) = 3,500. If OHaganBooks merged with FarmerBooks, their combined exclusive customer base would be [(& ) ] = 5,400 (1,000 + 1,000 + 100) = 3,300. The larger of the two results from a merger with JungleBooks.com: 3{,}500 exclusive customers. 37. Decision algorithm for constructing a 3-letter code: Step 1: Choose the 1st letter: 26 choices. Step 2: Choose the 2nd letter: 26 choices. Step 3: Choose the 3rd letter: 26 choices. Total number of codes: 26 × 26 × 26 = 26 3 = 17,576 38. Decision algorithm for constructing a 3-letter code with 3 different letters: Step 1: Choose the 1st letter: 26 choices.


Solutions Chapter 7 Review Step 2: Choose the 2nd letter: 25 choices. Step 3: Choose the 3rd letter: 26 choices. Total number of codes: 26 × 25 × 24 = 15,600 39. Decision algorithm for constructing a code: Step 1: Choose the 1st letter: 26 choices. Step 2: Choose the 2nd letter: 25 choices. Step 3: Choose the 1st digit: 9 choices. Step 4: Choose the 2nd digit: 10 choices. Total number of codes: 26 × 25 × 9 × 10 = 58,500

40. 2 different letters, 3 different digits not starting with zero: 26 × 25 × 9 × 9 × 8 = 421,200 2 different letters, 4 different digits not starting with zero: 26 × 25 × 9 × 9 × 8 × 7 = 2,948,400 41. Decision algorithm for constructing a course schedule meeting the minimum requirements: Step 1: Select 3 liberal arts courses: (6, 3) choices. Step 2: Select 3 math courses: (6, 3) choices. (Note: At this stage, you have must select a minimum of 4 courses to make up the required 10, and so you must select 2 sciences and 2 fine arts.) Step 3: Select 2 science courses: (5, 2) choices. Step 4: Select 2 fine arts courses: (6, 2) choices. Total number of schedules: (6, 3) (6, 3) (5, 2) (6, 2) = 20 × 20 × 10 × 15 = 60,000 42. Decision algorithm for constructing a course schedule meeting the minimum requirements, including Calc I, and bearing in mind the fine arts complication: Step 1: Select 3 liberal arts courses: (6, 3) choices. Step 2: Select Calc I: 1 choice. Step 3: Select 2 other math courses: (5, 2) choices. Step 4: Select 2 science courses: (5, 2) choices. Step 5: Select 2 fine arts courses other than the single pair that cannot be taken in the first year: (6, 2) 1 choices. Total number of schedules: (6, 3) (5, 2) (5, 2)[ (6, 2) 1] = 20 × 10 × 10 × 15 1 = 28,000 43. Decision algorithm for constructing a course schedule meeting all of the above requirements, and the physics complication: Alternative 1: Take neither Physics I nor Physics II. Step 1: Select 3 liberal arts courses: (6, 3) choices. Step 2: Select Calc I: 1 choice. Step 3: Select 2 other math courses: (5, 2) choices. Step 4: Select 2 science courses other than Physics I and II: (3, 2) choices. Step 5: Select 2 fine arts courses other than the single pair that cannot be taken in the first year: (6, 2) 1 choices Total number of schedules for Alternative 1: (6, 3) (5, 2) (3, 2)[ (6, 2) 1] = 20 × 10 × 3 × (15 1) = 8,400 Alternative 2: Take Physics I but not Physics II. Step 1: Select 3 liberal arts courses: (6, 3) choices. Step 2: Select Calc I: 1 choice. Step 3: Select 2 other math courses: (5, 2) choices. Step 4: Select Physics I:1 choice. Step 5: Select 1 science course other than Physics I and II: (3, 1) choices. Step 6: Select 2 fine arts courses other than the single pair that cannot be taken in the first year:


Solutions Chapter 7 Review (6, 2) 1 choices. Total number of schedules for Alternative 1: (6, 3) (5, 2) (3, 1)[ (6, 2) 1] = 20 × 10 × 3 × (15 1) = 8,400 Alternative 3: Take Physics I and Physics II. Step 1: Select 3 liberal arts courses: (6, 3) choices. Step 2: Select Calc I: 1 choice. Step 3: Select 2 other math courses: (5, 2) choices. Step 4: Select Physics I & II: 1 choice. Step 5: Select 2 fine arts courses other than the single pair that cannot be taken in the first year: (6, 2) 1 choices. Total number of schedules for Alternative 1: (6, 3) (5, 2)[ (6, 2) 1] = 20 × 10 × (15 1) = 2,800 Total number of course schedules overall: 8,400 + 8,400 + 2,800 = 19,600


Solutions Chapter 7 Case Study Chapter 7 Case Study 1. Decision algorithm to manufacture a two-colored cube: Choose two colors to use (3 choices). Now choose three faces to have the same color; there are exactly two possibilities: The three faces meet at a corner, or they consist of two opposite faces and one of the other four faces (the faces receiving the other color will be the same). Thus there are 2(3 × 2) = 12 additional cubes required (remember that each cube appears twice in the kit).

2. First select which color will appear on only one face, which on two, and which on three (3! = 6 choices). Select the three faces of the same color. If they meet at a corner, there is only one way to complete the coloring. If they consist of two opposite faces and one of the faces between, then there are two distinct ways of choosing the two faces of the same color: opposite one another or sharing an edge. In any case the remaining face gets the remaining color. Thus there are 2 × 6(1 + 2) = 36 additional cubes required. 3. First count how many have two faces of one color and two of the other: There is only one way to do this (any pair of faces meets at an edge). Next count how many have three faces of one color and the fourth of the other color: There are two choices for the colors, but only one arrangement with a given choice of colors. Thus, the kit will require 2 × (1 + 2) = 6 tetrahedra. 4. Decision algorithm: Alternative 1: One color: 3 choices. Alternative 2: Two colors: Step 1: Choose 2 colors: (3, 2) = 3 choices of colors. Subalternative a: One face of one color, 5 of the other. 2 choices as to which color is which, one arrangement once that is decided. Total: 2 choices. Subalternative b: Two faces of one color, 4 of the other. 2 choices as to which color is which Given the colors, two faces can be chosen in two distinct ways (sharing an edge or opposite). Total: 2 × 2 = 4 choices Subalternative c: Three faces of each color. 2 choices, as in Exercise 1 Total for Alternative 2: 3 × (2 + 4 + 2) = 24 choices. Alternative 3: Use all three colors Subalternative a: One of one color, one of a second, four of the third. Three ways to choose which color appears on four faces. Given the colors, two distinct ways of choosing the two faces with single colors. Total: 3 × 2 = 6 choices Subalternative b: One of one color, two of a second, three of the third. 18 choices, as in Exercise 2 Subalternative c: Two faces of each color. 6 choices, as in the text Total for Alternative 3 : 6 + 18 + 6 = 30 choices Thus, there are 3 + 24 + 30 = 57 such cubes, so the kit should contain 114 cubes.


Solutions Section 8.1 Section 8.1

1. = {HH, HT, TH, TT}; = {HH, HT, TH} 2. = {HH, HT, TH, TT}; = {HH, HT, TH}

3. = {HHH, HHT, HTH, HTT, THH, THT, TTH, TTT}; = {HTT, THT, TTH, TTT} 4. = {HHH, HHT, HTH, HTT, THH, THT, TTH, TTT}; = {HTT, THT, TTH, TTT} (1, 1), (2, 1), (3, 1), 5. = (4, 1), (5, 1), (6, 1), (1, 1), (2, 1), (3, 1), 6. = (4, 1), (5, 1), (6, 1),

(1, 2), (2, 2), (3, 2), (4, 2), (5, 2), (6, 2), (1, 2), (2, 2), (3, 2), (4, 2), (5, 2), (6, 2),

(1, 3), (2, 3), (3, 3), (4, 3), (5, 3), (6, 3), (1, 3), (2, 3), (3, 3), (4, 3), (5, 3), (6, 3),

(1, 1), (1, 2), (1, 3), (2, 2), (2, 3), (3, 3), 7. =

(1, 4), (2, 4), (3, 4), (4, 4), (5, 4), (6, 4), (1, 4), (2, 4), (3, 4), (4, 4), (5, 4), (6, 4),

(1, 4), (2, 4), (3, 4), (4, 4),

(1, 5), (2, 5), (3, 5), (4, 5), (5, 5), (6, 5), (1, 5), (2, 5), (3, 5), (4, 5), (5, 5), (6, 5),

(1, 5), (2, 5), (3, 5), (4, 5), (5, 5),

(1, 6), (2, 6), (3, 6), (4, 6), (5, 6), (6, 6) (1, 6), (2, 6), (3, 6), (4, 6), (5, 6), (6, 6)

(1, 6), (2, 6), (3, 6), (4, 6), (5, 6), (6, 6)

= {(1, 4), (2, 3), (3, 2), (4, 1)}

= {(3, 6), (4, 5), (5, 4), (6, 3)}

= {(1, 3), (2, 2)}

(1, 1), (1, 2), (1, 3), (1, 4), (1, 5), (1, 6), (2, 2), (2, 3), (2, 4), (2, 5), (2, 6), (3, 3), (3, 4), (3, 5), (3, 6), 8. = (4, 4), (4, 5), (4, 6), (5, 5), (5, 6), (6, 6) = {(1, 2), (1, 4), (1, 6), (2, 3), (2, 5), (3, 4), (3, 6), (4, 5), (5, 6)}

9. same as Exercise 7; = {(2, 2), (2, 3), (2, 5), (3, 3), (3, 5), (5, 5)}

10. same as Exercise 8; = {(1, 1), (1, 4), (1, 6), (4, 1), (4, 4), (4, 6), (6, 1), (6, 4), (6, 6)} 11. = {m, o, z, a, r, t}; = {o, a}

12. = {m, o, z, a, r, t}; = {o, z, r, t}

13. There are 4 × 3 = 12 possible sequences of 2 different letters chosen from sore: = {(s, o), (s, r), (s, e), (o, s), (o, r), (o, e), (r, s), (r, o), (r, e), (e, s), (e, o), (e, r)};


Solutions Section 8.1 = {(o, s), (o, r), (o, e), (e, s), (e, o), (e, r)}.

14. There are 4 × 3 = 12 possible sequences of 2 different letters chosen from hear: = {(h, e), (h, a), (h, r), (e, h), (e, a), (e, r), (a, h), (a, e), (a, r), (r, h), (r, e), (r, a)} = {(h, r), (e, h), (e, r), (a, h), (a, r), (r, h)}. 15. There are 5 × 4 = 20 possible sequences of 2 different digits chosen from 0–4: = {01, 02, 03, 04, 10, 12, 13, 14, 20, 21, 23, 24, 30, 31, 32, 34, 40, 41, 42, 43} = {10, 20, 21, 30, 31, 32, 40, 41, 42, 43}. 16. There are 5 × 4 = 20 possible sequences of 2 different digits chosen from 0–4: = {01, 02, 03, 04, 10, 12, 13, 14, 20, 21, 23, 24, 30, 31, 32, 34, 40, 41, 42, 43} = {00, 21, 42}.

17. = {domestic car, imported car, van, antique car, antique truck}; = {van, antique truck} 18. = {Psychology 1, Psychology 2, Economics 1, General Economics, Math for Poets}; = {Psychology 1, Psychology 2, Math for Poets}

19. a. The sample space is the set of all sets of 4 gummy candies chosen from the packet of 12. b. The event that April will get the combination she desires is the set of all sets of 4 gummy candies in which two are strawberry and two are black currant. 20. a. The sample space is the set of all sets of 3 marbles chosen from the bag of 12. b. The event that Alexandra gets one of each color is the set of all sets of 3 marbles containing one of each color. 21. a. The sample space is the set of all lists of 15 people chosen from 20. b. The event that Dr. Janet Yellen is the Secretary of State is the set of all lists of 15 people chosen from 20, in which Dr. Yellen occupies the twelfth (Secretary of the Treasury) position. 22. a. The sample space is the set of all sets of 5 cards chosen from a standard deck of 52. b. The event "a full house" is the set of all sets of 5 cards, chosen from a standard deck of 52, in which three are of one denomination and two are of another denomination.

23. : The red die shows 1; : The numbers add to 4. "The red die shows 1 and the numbers add to 4" is the event There is only one possible outcome in : = (1, 3). Therefore, ( ) = 1.

.

24. : The red die shows 1; : The numbers do not add to 11. "The red die shows 1 but the numbers do not add to 11" is the event = (1, 1), (1, 2), (1, 3), (1, 4), (1, 5), (1, 6); ( ) = 6 25. : The numbers add to 4. "The numbers do not add to 4" is the event . = (1, 3), (2, 2), (3, 1); ( ) = ( ) ( ) = 36 3 = 33 26. : The numbers do not add to 11. "The numbers add to 11" is the event . = (6, 5), (5, 6); ( ) = 2

And =

.

But =


Solutions Section 8.1 27. "The numbers do not add to 4" is the event . "The numbers add to 11" is the event . Therefore, "The numbers do not add to 4 but they do add to 11" is the event . = (6, 5), (5, 6), and so ( ) = 2 28. "The numbers add to 11" is the event . "The red die shows 1" is the event . Therefore, "Either the numbers add to 11 or the red die shows a 1" is the event . = (6, 5), (5, 6), (1, 1), (1, 2), (1, 3), (1, 4), (1, 5), (1, 6), and so ( ) = 8

29. "At least one of the numbers is 1" is the event . "The numbers add to 4" is the event . Therefore, "At least one of the numbers is 1 or the numbers add to 4" is the event . = (1, 1), (1, 2), (1, 3), (1, 4), (1, 5), (1, 6), (6, 1), (5, 1), (4, 1), (3, 1), (2, 1), (2, 2) ( ) = 12

30. "The numbers add to 4" is the event . "The numbers add to 11" is the event . "At least one of the numbers is 1" is the event . Therefore, "Either the numbers add to 4, or they add to 11, or at least one of them is 1" is the event . = (1, 1), (1, 2), (1, 3), (1, 4), (1, 5), (1, 6), (6, 1), (5, 1), (4, 1), (3, 1), (2, 1), (2, 2), (6, 5), (5, 6) ( ) = 14 31. : You will use the Website tonight. : Your math grade will improve. Therefore, "You will use the Website tonight and your math grade will improve." is the event And =

.

32. : You will use the Website tonight. : Your math grade will improve. Therefore, "You will use theWebsite tonight or your math grade will not improve." is the event . Or = 33. : You will use the Website every night. : Your math grade will improve. Therefore, "Either you will use the Website every night, or your math grade will not improve." is the event . Or =

34. : Your math grade will improve. : You will use the Website every night. Therefore, "Your math grade will not improve even though you use the Website every night." is the event . Even though =

35. : Your math grade will improve. : You will use the Website tonight. : You will use the Website every night. The given statement is: "Either your math grade will improve, or you will use the Website tonight but you will not use it every night." The comma after "improve" breaks the statement into two pieces: Either your math grade will improve: or you will use the Website tonight but you will not use it every night: ( ). Therefore, the given statement is the event ( ). 36. : Your math grade will improve. : You will use the Website tonight. : You will use the Website


Solutions Section 8.1 every night. The given statement is: "You will either use the Website tonight or you will use it very night, and your grade will improve." The comma after "night" breaks the statement into two pieces: You will either use the Website tonight or you will use it very night,: ( ) and your grade will improve: . Therefore, the given statement is the event ( ) .

37. : Your math grade will improve. : You will use the Website tonight. : You will use the Website every night. The given statement is: "Either your math grade will improve or you will use the Website tonight, but you will not use it not every night." The comma after "tonight" breaks the statement into two pieces: Either your math grade will improve or you will use the Website tonight, ( ) but you will not use it every night: . Therefore, the given statement is the event ( ) . 38. : Your math grade will improve. : You will use the Website tonight. : You will use the Website every night. The given statement is: "You will either use the Website tonight, or you will use it every night and your grade will improve." The comma after "tonight" breaks the statement into two pieces: You will either use the Website tonight: or you will use it every night and your grade will improve: ( ) Therefore, the given statement is the event ( ). 39. The sample space consists of all sets of 3 marbles chosen from 8. The event consists of all sets of 3 non-red marbles chosen from 4.

( ) = (8, 3) = 56 ( ) = (4, 3) = 4

40. The sample space consists of all sets of 3 marbles chosen from 8. ( ) = (8, 3) = 56 The event consists of all sets of 3 red marbles chosen from 4. ( ) = (4, 3) = 4 41. Using the multiplication principle: ( ) = (4, 1) × (2, 1) × (2, 1) = 16

42. The number of sets of 3 marbles all of the same color is the number of sets of 3 red marbles (since there are only 2 of each of the other colors): (4, 3) = 4 The total number of collections of 3 marbles is ( ) = (8, 3) = 56. Therefore, the number of sets that are not all of the same color is 56 4 = 52. 43. Looking at the map, we find the following regions that saw an increase in housing prices of 11% or more: Pacific, Mountain, New England, and Middle Atlantic. Therefore, = {Pacific, Mountain, New England, Middle Atlantic}. 44. Looking at the map, we find the following regions that saw an increase in housing prices of less than 10%: West North Central and West South Central. Therefore, = {West North Central, West South Central}. 45. : The region you choose saw an increase in housing prices of 11% or more. : The region you choose is on the east coast.


Solutions Section 8.1 translates to or : You choose a region that saw an increase in housing prices of 11% or more or is on the east coast. = {Pacific, Mountain, New England, Middle Atlantic, South Atlantic} translates to and : You choose a region that saw an increase in housing prices of 11% or more and is on the east coast. = {New England, Middle Atlantic}

46. : The region you choose saw an increase in housing prices of less than 11%. : The region you choose is not on the east coast. translates to or : You choose a region that saw an increase in housing prices of less than 11% or is not on the east coast. = {South Atlantic, East North Central, East South Central, West North Central, West South Central, Mountain, Pacific}, which we can express as a complement: {New England, Middle Atlantic} . translates to and : You choose a region that saw an increase in housing prices of less than 11% and is not on the east coast. = {East North Central, East South Central, West North Central, West South Central} 47. Two events are mutually exclusive if they have no outcomes in common. a. : You choose a region from among the two with the lowest percentage increase in housing prices. = {West North Central, West South Central} : You choose a region that is not on the east or west coast. = {Mountain, West North Central, West South Central, East North Central, East South Central} Since = {West North Central, West South Central}, , so they are not mutually exclusive. b. : You choose a region from among the two with the lowest increase in housing prices. = {West North Central, West South Central} : You choose a region that is on the east coast. = {New England, Middle Atlantic, South Atlantic} Since = , they are mutually exclusive. 48. Two events are mutually exclusive if they have no outcomes in common. a. : You choose a region from among the three with the highest increase in housing prices. = {New England, Middle Atlantic, Mountain} : You choose a region from among the central divisions. = {West North Central, West South Central, East North Central, East South Central} Since = , they are mutually exclusive. b. : You choose a region from among the three with the highest increase in housing prices. = {New England, Middle Atlantic, Mountain} : You choose a region on the east or west coast. = {New England, Middle Atlantic, South Atlantic, Pacific} Since = {New England, Middle Atlantic}, , so they are not mutually exclusive. 49. In the table, consists of the states in rows 1 and 2 and consists of the states in rows 2 and 3. Thus their intersection is the set of states in row 2, with rates between 45% and 49%. As there are 6 of those, is the event that the state had a vaccination rate between 45% and 49%; ( ) = 6.

50. In the table, consists of the states in row 5, and consists of the states in rows 3, 4, 5. Thus their intersection is the set of states in row 5, with rates between 60% and 64%. As there are 10 of those, is the event that the state had a vaccination rate between 60% and 64%; ( ) = 10. 51. By Exercise 49, is the event that the state had a vaccination rate between 45% and 49%, and ( ) = 6. Thus, its complement ( ) is the event that the state did not have a vaccination rate between 45% and 49%; ( ) = 50 6 = 44. It is also the event by DeMorgan's law.


Solutions Section 8.1 52. By Exercise 50, is the event that the state had a vaccination rate between 60% and 64%, and ( ) = 10. Thus, its complement ( ) is the event that the state did not have a vaccination rate between 60% and 64%; ( ) = 50 10 = 40. It is also the event by DeMorgan's law.

53. The only events that are not disjoint from are and . Thus, , , and are all disjoint from . The largest of these is , consisting of the states in rows 3–5. Rather than count all of them, count those in the complement (in rows 1, 2, and 6): there are 18 of them, so ( ) = 50 18 = 32. 54. The two events with the lowest cardinalities are ( ) = 10. 55. One possible answer is ( ) . As elements.

and , and they are also disjoint. ( ) = 7 and

has 21 elements, its complement has 50 21 = 29

56. One possible answer is ( ) . The states in question are those in the first two rows: 7 in all. 57. : an author is successful. : an author is new. is the event that an author is successful and new. is the event that an author is either successful or new ( ) = 5 (see table) ( ) = 45(see table)

Total

5

25

30

15

55

70

20

80

100

Total

Total

5

25

30

15

55

70

20

80

100

Total

58. : an author is new. : an author is unsuccessful. is the event that an author is new and unsuccessful. is the event that an author is either new or unsuccessful (or both). ( ) = 15 (See table) ( ) = 75 (See table)

Total

5

25

30

15

55

70

20

80

100

Total

Total

5

25

30

15

55

70

20

80

100

Total

59. Since and have no outcomes in common (an author cannot be both new and established), they are mutually disjoint. and are not mutually disjoint, since ( ) = 5 (see table). and are not mutually disjoint, since ( ) = 25 (see table).

Total

5

25

30

15

55

70

20

80

100

Total


Solutions Section 8.1 60. and are not mutually disjoint, since ( ) = 55 (see table). and are mutually exclusive, since there are no authors who are both successful and unsuccessful. and are not mutually disjoint, since ( ) = 5 (see table).

Total

5

25

30

15

55

70

20

80

100

Total

61. : an author is successful. : an author is new. is the event that an author is successful but not a new author. ( ) = 25 (see table)

Total

= Total

5

25

30

15

55

70

20

80

100

62. : an author is unsuccessful. : an author is established. is the event that an author is either unsuccessful or not established (or both). ( ) = 75 (see table)

Total

=

5

25

30

15

55

70

20

80

100

Total

63. The total number of established authors is 80. Of these, 25 are successful. Thus, the percentage that are successful is 25 = 31.25 80 The total number of successful authors is 30. Of these, 25 are established. Thus, the percentage that are established is 25 83.33 30 64. The total number of new authors is 20. Of these, 15 are unsuccessful. Thus, the percentage that are unsuccessful is 15 = 75 20 The total number of unsuccessful authors is 70. Of these, 15 are new. Thus, the percentage that are new is 15 21.43 70


Solutions Section 8.1 65. "An industry increased in value but was not in the financial sector" is the event

6

5

0

11

4

0

3

7

2

3

1

6

6

5

3

14

0

5

1

6

Totals

18

18

8

44

(

Totals

) = 18 4 = 14

66. "An industry was in the financial sector and did not increase in value" is the event

6

5

0

11

4

0

3

7

2

3

1

6

6

5

3

14

0

5

1

6

Totals

18

18

8

44

(

.

.

Totals

) = 0 + 3 = 3

67. = {Industrial} {Unchanged in value}; the event that an industry was either not in the industrial sector or was unchanged in value (or both). ( ) = 44 (6 + 5) = 33 (see table)

6

5

0

11

4

0

3

7

2

3

1

6

6

5

3

14

0

5

1

6

Totals

18

18

8

44

Totals


Solutions Section 8.1 68. = {Health care} {Unchanged in value} ; the event that an industry was either in the industrial sector or not unchanged in value (or both). ( ) = 44 (0 + 3 + 1 + 1) = 39 (see table)

6

5

0

11

4

0

3

7

2

3

1

6

6

5

3

14

0

5

1

6

Totals

18

18

8

44

Totals

69. For a pair of events to be mutually exclusive, they must have no outcomes in common. Three kinds can be identified: (1) Pairs of events corresponding to separate rows: and , and , and (2) Pairs of events corresponding to separate columns: and , and , and (3) Pairs corresponding to a row and column whose intersection is empty: and , and 70. For a pair of events to not be mutually exclusive, their intersection must contain at least one outcome. Such pairs of events are pairs corresponding to a row and column with nonempty intersection, and also pairs of the form " and ": and , and , and , and , and , and , and . 71. : The dog's flight drive is strongest. : The dog's fight drive is weakest. : The dog's fight drive is strongest. a. "The dog's flight drive is not strongest and its fight drive is weakest" is the event . b. "The dog's flight drive is strongest or its fight drive is weakest" is the event . c. "Neither the dog's flight drive nor fight drive are strongest" is the event = ( ) . 72. : The dog's flight drive is strongest. : The dog's flight drive is weakest. : The dog's fight drive is strongest. : The dog's fight drive is weakest. a. "The dog's flight drive is weakest and its fight drive is not weakest" is the event . b. "The dog's flight drive is not strongest or its fight drive is weakest" is the event . c. "Either the dog's flight drive or fight drive fail to be strongest" is the event = (

)'.

73. a. The dog's fight and flight drives are both strongest: Intersection of rightmost column and bottom row: {9} b. The dog's fight drive is strongest, but its flight drive is neither weakest nor strongest: Intersection of rightmost column and middle row: {6} 74. a. Neither the dog's fight drive nor its flight drive is strongest: Intersection of left two columns with top two rows: {1, 2, 4, 5} b. The dog's fight drive is weakest, but its flight drive is neither weakest nor strongest: Intersection of leftmost column and middle row: {4} 75. a. {1, 4, 7} This is the leftmost column: The dog's fight drive is weakest. b. {1, 9}: The dog's fight and flight drives are either both strongest or both weakest. c. {3, 6, 7, 8, 9} This is the union of the rightmost column and the bottom row: Either the dog's fight drive


Solutions Section 8.1 is strongest, or its flight drive is strongest. 76. a. {7, 8, 9} This is the bottom row: The dog's flight drive is strongest. b. {3, 7} Either its flight drive is strongest and its fight drive is weakest, or the other way around. c. {1, 2, 3, 4, 7} This is the union of the leftmost column and the top row: Either the dog's fight drive is weakest, or its flight drive is weakest.

77. is the set of sets of 4 gummy bears chosen from 6; ( ) = (6, 4) = 15. Decision algorithm for constructing a set including the raspberry gummy bear: Step 1: Select the raspberry one: (1, 1) = 1 choice Step 2: Select 3 non-raspberry ones: (5, 3) = 10 choices Total number of choices: ( ) = 1 × 10 = 10

78. is the set of sets of 5 items chosen from 20; ( ) = (20, 5) = 15 504. Decision algorithm for constructing a set including the cashew: Step 1: Select the cashew: (1, 1) = 1 choice Step 2: Select 4 chocolates: (19, 4) = 3 876 choices Total number of choices: ( ) = 1 × 3 876 = 3 876

79. a. Decision algorithm for constructing a finish (winner, second place and third place) for the race: Step 1: Select the winner: 7 choices Step 2: Select second place: 6 choices Step 3: Select third place: 5 choices Total number of finishes: ( ) = 7 × 6 × 5 = 210 b. : Discretionary Marq is in second or third place. : Sanctuary City is the winner. Therefore, is the event that Sanctuary City wins and Discretionary Marq is in second or third place. In other words, it is the set of all lists of three horses in which Sanctuary City is first and Discretionary Marq is second or third. Decision algorithm for constructing a finish in : Alternative 1: Discretionary Marq is second: Step 1: Select Sanctuary City as the winner: 1 choice Step 2: Select Discretionary Marq for second place: 1 choice Step 3: Select third place: 5 choices Total number of finishes for Alternative 1: 1 × 1 × 5 = 5 Alternative 2: Discretionary Marq is third: Step 1: Select Sanctuary City as the winner: 1 choice Step 2: Select Discretionary Marq for third place: 1 choice Step 3: Select second place: 5 choices Total number of finishes for Alternative 2: 1 × 1 × 5 = 5 Therefore, ( ) = 5 + 5 = 10. Shorter (alternative) decision algorithm: Step 1: Select Sanctuary City as the winner: 1 choice Step 2: Select a slot (second or third place) for Discretionary Marq: 2 choices Step 3: Select a horse for the missing slot: 5 choices Total number of finishes: 1 × 2 × 5 = 10 80. a. An outcome is an ordered list of two teams chosen from 5. Therefore, ( ) = (5, 2) = 5 × 4 = 20. b. : The City Slickers are runners-up. : The Independent Wildcats are neither the winners nor runners-up. Therefore, is the event that either the City Slickers are runners-up, or the Independent Wildcats are neither the winners nor runners-up. ( ) = ( ) + ( ) ( )


Solutions Section 8.1 Decision algorithm for constructing a finish in : City Slickers are runners-up: Step 1: Select City Slickers are runners-up: 1 choice Step 2: Select a winner: (4, 1) = 4Total number of finishes: ( ) = 1 × 4 = 4 Decision algorithm for constructing a finish in : The Independent Wildcats are neither the winners nor runners-up: Step 1: Select a winner: (4, 1) = 4 choices Step 2: Select a runner-up: (3, 1) = 3 choices Total number of finishes: ( ) = 4 × 3 = 12 Decision algorithm for constructing a finish in : City Slickers are runners-up and the Independent Wildcats are neither the winners nor runners-up: Step 1: Select City Slickers are runners-up: 1 choice Step 2: Select a winner other than the Independent Wildcats: (3, 1) = 3 Total number of finishes: ( ) = 1 × 3 = 3 ( ) = ( ) + ( ) ( ) = 4 + 12 3 = 13 81. An event is a subset of the sample space.

82. Two events and are mutually exclusive if their intersection is empty. 83. ( ) is the complement of the event . Since ( ) is the event that and do not both occur. 84. (

is the event that both and occur,

) is the event that neither nor occurs.

85. Choice (B): The event should be a subset of the sample space, and only (B) has the property (the collection of tall, dark strangers you meet is a subset of the set of people you meet). 86. Choice (C): The event should be a subset of the sample space, and only (C) has the property (the collection of Porsches is a subset of the set of models available).

87. True; Consider the following experiment: Select an element of the set at random. Then the sample space is the set of elements of . In other words, the sample space is . 88. False; for instance, consider the following experiment: Flip a coin until you get heads, and observe the number of times you flipped the coin. 89. Answers may vary. Cast a die, and record the remainder when the number facing up is divided by 2. 90. Answers may vary. Flip two coins, and count the total number of heads: = {0, 1, 2}. 91. Yes. For instance, = {(2, 5), (5, 1)} and = {(4, 3)} are two such events.

92. Answers may vary: Experiment: Cast two dice, and record the numbers facing up as well as the day of the week. : The dice are cast on a Monday; : The dice are cast on a Tuesday.


Solutions Section 8.2 Section 8.2

"#( ) 40 1. ! ( ) = = = .4 100 "#( ) 300 2. ! ( ) = = = .6 500

3. = 800, "#( ) = 640; ! ( ) =

"#( ) 640 = = .8 800

"#( ) 160 4. = 800, "#( ) = 800 640 = 160; ! ( ) = = = .2 800

"#( ) 8 + 8 + 8 + 12 36 5. = {1, 2, 3, 4}; ! ( ) = = = = .6 60 60 6. = {4, 5, 6}; ! ( ) =

"#( ) 30 + 15 + 15 60 = = = .667 90 90

7. We obtain the relative frequency distribution by dividing the frequencies by = 4,000 : 1,100 = .275 4,000

Outcome Frequency Relative Frequency HH

1,100

HT

950

TH

1,200

TT

750

950 = .2375 4,000 1,200 = .3 4,000

750 = .1875 4,000

8. From Exercise 7, the relative frequency is Outcome HH

HT

TH

TT

Probability .275 .2375

.3

.1875

The event that heads comes up at least once is = { , , } $ ! ( ) = .275 + .2375 + .3 = .8125. 9. Refer to the relative frequency distribution in the solution to Exercise 7. The event the that the second coin lands with heads up is = { , } $ ! ( ) = .275 + .3 = .575. 10. Refer to the relative frequency distribution in the solution to Exercise 8. The event the that the first coin lands with heads up is = { , } $ ! ( ) = .275 + .2375 = .5125. 11. From the solution to Exercise 9, the second coin comes up heads approximately 58% of the time.


Solutions Section 8.2 Therefore, the second coin seems slightly biased in favor of heads, especially in view of the fact that the coin was tossed 4,000 times. On the other hand, it is conceivable that the coin is fair and that heads came up 58% of the time purely by chance. Deciding which conclusion is more reasonable requires some knowledge of inferential statistics. 12. From the solution to Exercise 10, the first coin comes up heads approximately 51% of the time. Therefore, the first coin seems fair, since heads comes up close to 50% of the time. On the other hand, it is conceivable that the coin is biased and that heads came up close to 50% of the time purely by chance. Deciding which conclusion is more reasonable requires some knowledge of inferential statistics. 13. Yes: Since the relative frequencies are between 0 and 1 (inclusive) and add up to 1, the given distribution can be a relative frequency distribution. 14. No: The relative frequencies given add up to .6; they are supposed to add up to 1. 15. No: Relative frequencies cannot be negative. 16. No: Relative frequencies cannot exceed 1. 17. Yes: Since the relative frequencies are between 0 and 1 (inclusive) and add up to 1, the given distribution can be a relative frequency distribution. 18. Yes: Since the relative frequencies are between 0 and 1 (inclusive) and add up to 1, the given distribution can be a relative frequency distribution. 19. Relative frequency distribution: Outcome

1

2

3

4

Rel. Frequency

.2

.3

.1

.1

5

%

Since the probabilities of all the outcomes add to 1, .2 + .3 + .1 + .1 + % = 1 $ .7 + % = 1 $ % = .3. a. From the properties of relative frequency distributions, ! (1, 3, 5) = ! (1) + ! (3) + ! (5) = .2 + .1 + .3 = .6. b. Since = {1, 2, 3}, = {4, 5}, ! ( ) = ! (4, 5) = ! (4) + ! (5) = .1 + .3 = .4 20. Relative frequency distribution: Outcome

1

Rel. Frequency

.4

2

%

3

4

5

.3

.1

.1

Since the probabilities of all the outcomes add to 1, .4 + % + .3 + .1 + .1 = 1 $ .9 + % = 1 $ % = .1. a. From the properties of relative frequency distributions, ! (2, 3, 4) = ! (2) + ! (3) + ! (4) = .1 + .3 + .1 = .5. b. Since = {3, 4}, = {1, 2, 5}, ! ( ) = ! (1, 2, 5) = ! (1) + ! (2) + ! (5) = .4 + .1 + .1 = .6. 21. Use the following formulas to generate binary digits (0 represents heads, say, and 1 represents tails): TI-83/84 Plus: randInt(0,1) To obtain randInt, follow [MATH] → PRB.


Solutions Section 8.2 Spreadsheet: =RANDBETWEEN(0,1) Answers will vary. The estimated probability that heads comes up should be around .5. 22. Use the following formulas to generate digits in the range 1–6: TI-83/84 Plus: randInt(1,6) To obtain randInt, follow [MATH] → PRB. Spreadsheet: =RANDBETWEEN(1,6) Answers will vary. The estimated probability that a six comes up should be around .1667. 23. Simulate tossing two coins by generating two random binary digits (0 represents heads, say, and 1 represents tails). If the outcome is one head and one tail, the digits should add to 1. Otherwise, they will add to 0 or 2. The sum of these digits can be obtained as follows: TI-83/84 Plus: randInt(0,1)+randInt(0,1) To obtain randInt, follow [MATH] → PRB. Spreadsheet: =RANDBETWEEN(0,1)+RANDBETWEEN(0,1) Then count how many 1s you get. Answers will vary. The estimated probability that the outcome is one head and one tail should be around .5. 24. Simulate tossing two fair dice by generating two random integers in the range 1–6. If the outcome is a double six, the digits should add to 12. The sum of these digits can be obtained as follows: TI-83/84 Plus: randInt(1,6)+randInt(1,6) To obtain randInt, follow [MATH] → PRB. Spreadsheet: =RANDBETWEEN(1,6)+RANDBETWEEN(1,6) Then count how many 12s you get. Answers will vary. The estimated probability that the outcome is a double six should be around .028. "#( ) 480 = = .96 500 "#( ) 500 5 b. ! ( ) = = = .99 500 "#( ) 5 + 480 c. ! ( ) = = = .97 500 25. a. ! ( ) =

"#( ) 9 = = .018 500 b. Using the answer from part (a), ! ( ) = 1 0.018 = .982 "#( ) 9 + 485 c. ! ( ) = = = .988 500 26. a. ! ( ) =


Solutions Section 8.2 27. a. We obtain the relative frequency distribution by dividing the frequencies by = 134 + 52 + 9 + 5 = 200 : Outcome

134 = .67 200

Frequency Rel. Frequency

Current

134

Past Due

52

In Foreclosure

9

Repossessed

5

52 = .26 200

9 = .045 200 5 = .025 200

b. Take to be the event that a randomly selected subprime mortgage in Texas was not current. = {Past Due, In Foreclosure, Repossessed}; ! ( ) = .26 + .045 + .025 = .33 28. a. We obtain the relative frequency distribution by dividing the frequencies by = 110 + 65 + 60 + 15 = 250 : Outcome

110 = .44 250

Frequency Rel. Frequency

Current

110

Past Due

65

In Foreclosure

60

Repossessed

15

65 = .26 250 60 = .24 250 15 = .06 250

b. Take to be the event that a randomly selected subprime mortgage in Florida was neither in foreclosure nor repossessed. = {Current, Past Due}; ! ( ) = .44 + .26 = .70 29. a. Using the result of Quick Example #3, the relative frequency distribution is obtained by converting the percentages into decimals: Age 0–14 15–29 30–64 & 65

Rel. Frequency

.25

.25

.42

.08

b. The event that a resident of Mexico is not between 15 and 64 is shown by the shaded parts of the distribution: Age 0–14 15–29 30–64 & 65

Rel. Frequency

.25

.25

.42

.08

Thus the relative frequency is ! ( ) = .25 + .08 = .33. 30. a. Using the result of Quick Example #3, the relative frequency distribution is obtained by converting the percentages into decimals:


Solutions Section 8.2

Age 0–14 15–29 30–64 & 65 Rel. Frequency

.18

.20

.45

.17

b. The event that a resident of the United States is 30 or older is shown by the shaded parts of the distribution: Age 0–14 15–29 30–64 & 65

Rel. Frequency

.18

.20

.45

.17

Thus the relative frequency is ! ( ) = .45 + .17 = .62. 31. a. We obtain the relative frequency distribution by dividing the frequencies by = 20 = 1 + 7 + 9 + 2 + 1: Predicted Reliability Frequency Rel. Frequency

1

2

3

4

5

1 7 9 2 1 = .05 = .35 = .45 = .1 = .05 20 20 20 20 20 1

7

9

2

1

b. Take to be the event that a randomly selected compact SUV will have a predicted reliability of average (3) or better. = {3, 4, 5}; ! ( ) = .45 + .1 + .05 = .6 32. a. We obtain the relative frequency distribution by dividing the frequencies by = 16 = 2 + 0 + 12 + 1 + 1: Predicted Reliability Frequency Rel. Frequency

1

2

3

4

5

2 0 12 1 1 = .125 = 0 = .75 = .0625 = .0625 16 16 16 16 16 2

0

12

1

1

b. Take to be the event that a randomly selected small car will have a predicted reliability worse than average. = {1, 2}; ! ( ) = .125 + 0 = .125 33. The number of households is = 2,000. To calculate "#( ) in each case, we use "#( ) ! ( ) = , giving "#( ) = ! ( ) × . Here is a chart showing the frequencies and their calculation: Outcome

Dial-up

Cable Modem

DSL

Other

Rel. Frequency

.628

.206

.151

.015

Frequency .628 × 2,000 = 1,256 .206 × 2,000 = 412 .151 × 2,000 = 302 .015 × 2,000 = 30

34. The number of households is = 5,000. To calculate "#( ) in each case, we use "#( ) ! ( ) = , giving "#( ) = ! ( ) × . Here is a chart showing the frequencies and their calculation:


Solutions Section 8.2 Outcome

Wired

Satellite

Dialup

Other

Rel. Frequency

.9462

.0454

.0060

.0024

Frequency

.9462 × 5,000 .0454 × 5,000 .0060 × 5,000 .0024 × 5,000 = 4,731 = 227 = 30 = 12

35. Using the given classification of the data, we get the following: Outcome

Surge

Frequency

4 6 10 = .2 = .3 = .5 20 20 20

Rel. Frequency

4

Plunge 6

Steady 10

36. Using the given classification of the data, we get the following: Outcome

Surge

Frequency

4 7 9 = .2 = .35 = .45 20 20 20

Rel. Frequency 37. ! (

Total

) =

Plunge

4

7

"#( ) 25 = = .25 100

5

25

30

15

55

70

20

80

100

Total

"#( ) 20 39. ! ( ) = = = .2 100

Total

Steady

5

25

30

15

55

70

20

80

100

Total

9

38. ! (

Total

) =

"#( ) 15 = = .15 100

5

25

30

15

55

70

20

80

100

Total

"#( ) 30 40. ! ( ) = = = .3 100

Total

5

25

30

15

55

70

20

80

100

Total


"#( ) 70 41. ! ( ) = = = .7 100

Total

5

25

30

15

55

70

20

80

100

"#( ) 80 42. ! ( ) = = = .8 100

Solutions Section 8.2

Total

Total

5

25

30

15

55

70

20

80

100

Total

43. Restrict attention to the successful authors only: 25 5 ! (Successful author is established) = = 30 6

44. Restrict attention to the unsuccessful authors only: 55 11 ! (Unsuccessful author is established) = = 70 14

Total

Total

5

25

30

45. Restrict attention to the established authors only: 25 5 ! (Established author is successful) = = 80 16

25

Total

80

55

15

55

70

46. Restrict attention to the new authors only: 15 ! (New author is successful) = = .75 20

Total

5

15 20

47. There were 70 Phoenix residents in the survey who had not been infected during the year, so: 70 ! ( ) = = .07 1,000 48. There were 220 N.Y. City residents in the survey who had been infected during the year, so: 220 ! ( ) = = .22 1,000 49. There were a total of 450 people who had been infected, so: 450 ! ( ) = = .45 1,000 50. There were a total of 550 people who had not been infected, so: 550 ! ( ) = = .55 1,000 51. Restrict attention to Chicago residents only: 80 ! (Chicago resident was infected) = = .50 160


Solutions Section 8.2 52. Restrict attention to Los Angeles residents only: 120 .52 ! (Los Angeles resident was infected) = 230 53. Restrict attention to residents who had been infected only: 120 ! (Infected person was from Los Angeles) = .27 450 54. Restrict attention to residents who had not been infected only: 290 ! (Non infected person was from N.Y. City) = .53 550

55. ! (Contaminated) = .8 Therefore, ! ( ) = 1 .8 = .2. Of the contaminated chicken, 20% had the strain resistant to antibiotics. Therefore, ! (') = .2 × .8 = .16 (20% of 80%). The other 80% of contaminated chicken is not resistant. Therefore, ! ( ) = .8 × .8 = .64 (80% of 80%). This gives the following relative frequency distribution: Outcome Rel. Freq.

'

.2

.64

.16

56. ! (Contaminated) = .58 Therefore, ! ( ) = 1 .58 = .42. Of the contaminated turkey, 84% had the strain resistant to antibiotics. Therefore, ! (') = .84 × .58 = .4872 (84% of 58%). The other 16% of contaminated turkey is not resistant. Therefore, ! ( ) = .16 × .58 = .0928 (16% of 58%). This gives the following relative frequency distribution: Outcome Rel. Freq.

.42

'

.0928 .4872

57. Conventionally grown produce: The events are: No pesticide ( ), Single pesticide ( ), Multiple pesticides (( ). We are told that 60% of conventionally grown foods had residues from at least one pesticide. Therefore, ! ( ) = 1 .60 = .40. We are also told that conventionally grown foods were 4 times as likely to contain multiple pesticides as organic foods, and that 11% of organic foods had multiple pesticides. Therefore, ! (( ) = 4 × .11 = .44. This leaves ! ( ) = 1 (.40 + .44) = .16. Probability distribution for conventionally grown produce: Outcome

Probability

.40

(

.16

.44

Organic produce: We are told that ! (( ) = .11 and that 26% had either single or multiple pesticide residues. Therefore,


Solutions Section 8.2 ! ( ) = .26 .11 = .15. This leaves ! ( ) = 1 (.11 + .15) = .74. Probability distribution for organic produce: Outcome

Probability

(

.15

.11

.74

58. Conventionally grown produce: The events are: No pesticide ( ), Single pesticide ( ), Multiple pesticides (( ). We are told that conventionally grown foods were 5 times as likely to contain multiple pesticides as organic foods, of which 8% contained multiple pesticides. Therefore, ! (( ) = 5 × .08 = .40. We are also told that 55% had either single or multiple pesticide residues. Therefore, ! ( ) = .55 .40 = .15. This leaves ! ( ) = 1 (.15 + .40) = .45. Probability distribution for conventionally grown produce: Outcome

Probability

(

.15

.40

.45

Organically grown produce: We are told that ! (( ) = .08 and that 23% had either single or multiple pesticide residues. Therefore, ! ( ) = .23 .08 = .15. This leaves ! ( ) = 1 (.15 + .08) = .77. Probability distribution for organic produce: Outcome

Probability

(

.15

.08

.77

59. ! (False negative) =

10 = .025 400

60. ! (False exoneration) =

100 = .3333 300

! (False positive) =

10 = .05 200

! (False accusation) =

200 = .4 500

61. Answers will vary. 62. Answers will vary.

63. The estimated probability of an event is defined to be the fraction of times occurs. 64. Wrong. The relative frequency can be different each time a number of observations is made.

65. 101; "#( ) can be any number beween 0 and 100 inclusive, so the possible answers are 0)100 = 0, 1)100 = .01, 2)100 = .02, ..., 99)100 = .99, 100)100 = 1.


Solutions Section 8.2 66. A possible experiment: Approach a student and ask whether he or she approves of the student council president. The student council president's popularity rating is the relative frequency of a favorable answer. 67. Wrong. For a pair of fair dice, the probability of a pair of matching numbers is 1/6, as Ruth says. However, it is quite possible, although not very likely, that if you cast a pair of fair dice 20 times, you will never obtain a matching pair. (In fact, there is approximately a 2.6% chance that this will happen.) In general, a nontrivial claim about theoretical probability can never be absolutely validated or refuted experimentally. All we can say is that the evidence suggests that the dice are not fair. 68. Wrong. Repeating the experiment could result in any number of such outcomes from 0 to 24. In general, a nontrivial claim about probability can never be absolutely validated or refuted experimentally. All we can say is that the evidence suggests that the Juan is probably right. 69. For a (large) number of days, record the temperature prediction for the next day, and then check the actual high temperature the next day. Record whether the prediction was accurate (within, say, 2°F of the actual temperature). The fraction of times the prediction was accurate is the relative frequency. 70. No. If, for example, you got 500 heads in the next 1000 tosses, you would have an relative frequency of 570)1,100 0.518, which is much closer to 50%.


Solutions Section 8.3 Section 8.3 1. Outcome

a

b

c

d

Probability

.1

.05

.6

.05

e

%

Since the probabilities of all the outcomes add to 1, .1 + .05 + .6 + .05 + % = 1 $ .8 + % = 1 $ % = .2. a. ({a, c, e}) = (*) + (+) + (,) = .1 + .6 + .2 = .9 b. ( ) = ({a, b, c, e}) = (a) + (b) + (c) + (e) = .1 + .05 + .6 + .2 = .95 c. ( ) = (-, .) = (-) + (.) = .05 + .05 = .1 d. ( ) = ({+, ,}) = (+) + (,) = .6 + .2 = .8 2. Outcome

a

Probability

.1

b

%

c

d

e

.65

.1

.05

Since the probabilities of all the outcomes add to 1, .1 + % + .65 + .1 + .05 = 1 $ .9 + % = 1 $ % = .1. a. ({a, c, e}) = (*) + (+) + (,) = .1 + .65 + .05 = .8 b. ( ) = ({a, b, c, e}) = (a) + (b) + (c) + (e) = .1 + .1 + .65 + .05 = .9 c. ( ) = ({-, .}) = (-) + (.) = .1 + .1 = .2 d. ( ) = ({+, ,}) = (+) + (,) = .65 + .5 = .7 3. ( ) =

( ) 5 1 = = ( ) 20 4

4. ( ) =

( ) 4 1 = = ( ) 8 2

5. ( ) =

( ) 10 = = 1 ( ) 10

6. ( ) =

( ) 0 = = 0 ( ) 10

7. ( ) = 4, ( ) = 3 $ ( ) =

( ) 3 = ( ) 4

8. ( ) = 5, ( ) = 2 $ ( ) =

9. = {HH, HT, TH, TT}; ( ) = 4; = {HH, HT, TH}; ( ) = 3 $ ( ) = 10. = {HH, HT, TH, TT}; ( ) = 4; = {HH, HT, TH}; ( ) = 3 $ ( ) =

( ) 2 = ( ) 5

( ) 3 = ( ) 4

( ) 3 = ( ) 4

11. = {HHH, HHT, HTH, HTT, THH, THT, TTH, TTT}; ( ) = 8; = {HTT, THT, TTH, TTT}; ( ) = 4 ( ) 4 1 ( ) = = = ( ) 8 2

12. = {HHH, HHT, HTH, HTT, THH, THT, TTH, TTT}; ( ) = 8; = {HTT, THT, TTH, TTT};


( ) = 4

( ) =

Solutions Section 8.3

( ) 4 1 = = ( ) 8 2

13. ( ) = 36, ( ) = 4 $ ( ) = 14. ( ) = 36, ( ) = 4 $ ( ) = 15. ( ) = 36, ( ) = 0 $ ( ) = 16. ( ) = 36, ( ) = 18 $ ( ) =

( ) 4 1 = = ( ) 36 9 ( ) 4 1 = = ( ) 36 9 ( ) 0 = = 0 ( ) 36

( ) 18 1 = = ( ) 36 2

17. ( ) = 36, = {(2, 2), (2, 3), (2, 5), (3, 2), (3, 3), (3, 5), (5, 2), (5, 3), (5, 5)}; ( ) = 9 ( ) 9 1 ( ) = = = ( ) 36 4 18. ( ) = 36, = {(1, 1), (1, 4), (1, 6), (4, 1), (4, 4), (4, 6), (6, 1), (6, 4), (6, 6)}; ( ) = 9 ( ) 9 1 ( ) = = = ( ) 36 4

19. = {(4, 4), (2, 3)} The outcomes for a pair of indistinguishable dice are not equally likely, so we cannot use ( ) = ( )) ( ). Using the Example in the textbook, 1 1 (4, 4) = , and (2, 3) = . 36 18 1 1 1 Therefore, ( ) = . + = 36 18 12 The corresponding event for distinguishable dice is {(4, 4), (2, 3), (3, 2)}. ( ) 3 1 (We can also use this to compute ( ): ( ) = .) = = ( ) 36 12 20. = {(5, 5), (2, 5), (3, 5)} The outcomes for a pair of indistinguishable dice are not equally likely, so we cannot use ( ) = ( )) ( ). Using the Example in the textbook, 1 1 (5, 5) = and (2, 5) = (3, 5) = . 36 18 1 1 1 5 Therefore, ( ) = . + + = 36 18 18 36 The corresponding event for distinguishable dice is {(5, 5), (2, 5), (5, 2), (3, 5), (5, 3)}. ( ) 5 (We can also use this to compute ( ): ( ) = .) = ( ) 36 21. Start with Outcome Probability

1

%

2

2%

3

%

4

2%

5

%

6

2%


Solutions Section 8.3 Since the sum of the probabilities of the outcomes must be 1, we get 1 9% = 1, and so % = . 9 This gives: Outcome

1

2

3

4

5

6

Probability

1/9

2/9

1/9

2/9

1/9

2/9

1

2

3

4

5

6

({1, 2, 3}) =

1 2 1 4 + + = 9 9 9 9

22. Start with Outcome

3%

3%

%

%

%

%

Outcome

1

2

3

4

5

6

Probability

.3

.3

.1

.1

.1

.1

1

2

3

4

Probability

Since the sum of the probabilities of the outcomes must be 1, we get 1 10% = 1, and so % = . 10 This gives:

(2, 4, 6) = .3 + .1 + .1 = .5

23. Start with Outcome

8%

4%

2%

%

1

2

3

4

Probability 8/15

4/15

2/15

1/15

2

3

4

Probability

Since the sum of the probabilities of the outcomes must be 1, we get 1 15% = 1, and so % = . 15 This gives: Outcome

24. Start with

1

Outcome

%

Probability

2%

3%

5

4%

6

5%

7

6%

8

7%

9

8%

9%

Since the sum of the probabilities of the outcomes must be 1, we get 1 78% = 1, and so % = . 78 This gives: Outcome

10

11

12

10%

11%

12%

10

11

12

1

2

3

4

5

6

7

8

9

Prob 1/78

2/78

3/78

4/78

5/78

6/78

7/78

8/78

9/78 10/78 11/78 12/78


25. ( ) = .1, ( ) = .6, ( ( )= ( )+ ( ) (

26. ( ) = .3, ( ) = .4, ( ( )= ( )+ ( ) ( 27.

28.

29.

30.

Solutions Section 8.3 ) = .05 ) = .1 + .6 .05 = .65 ) = .02 ) = .3 + .4 .02 = .68

= , ( ) = .3, ( ) = .4 ( )= ( )+ ( ) (Mutually exclusive) .4 = .3 + ( ) $ ( ) = .4 .3 = .1

= , ( ) = .8, ( ) = .8 ( )= ( )+ ( ) (Mutually exclusive) 8 = ( ) + .8 $ ( ) = .8 .8 = 0 = , ( ) = .3, ( ) = .4 ( ) = ( ) + ( ) = .3 + .4 = .7 = , ( ) = .2, ( ) = .3 ( ) = ( ) + ( ) = .2 + .3 = .5

(Mutually exclusive)

(Mutually exclusive)

31. ( ) = .9, ( ) = .6, ( ) = .1 ( )= ( )+ ( ) ( ) .9 = ( ) + .6 .1 $ ( ) = .9 .6 + .1 = .4 32. ( ) = 1, ( ) = .6, ( ) = .1 ( )= ( )+ ( ) ( ) 1 = .6 + ( ) .1 $ ( ) = 1 .6 + .1 = .5

33. ( ) = .75 $ (

34. ( ) = .22 $ (

) = 1 ( ) = 1 .75 = .25

) = 1 ( ) = 1 .22 = .78

35. ( ) = .3, ( ) = .4, ( ) = .3 Since , , and are mutually exclusive, (

36. ( ) = .2, ( ) = .6, ( ) = .1 Since , , and are mutually exclusive, (

) =

( ) + ( ) + ( ) = .3 + .4 + .3 = 1.

) =

( ) + ( ) + ( ) = .2 + .6 + .1 = .9.

37. ( ) = .3, ( ) = .4 Since and are mutually exclusive, ( ) = ( ) + ( ) = .3 + .4 = .7 $ (( ) ) = 1 ( ) = 1 .7 = .3. 38. ( ) = .4, ( ) = .4 Since and are mutually exclusive, ( ) = ( ) + ( ) = .4 + .4 = .8 $ (( ) ) = 1 ( ) = 1 .8 = .2.

39. Since = , ( ) = 1. Since = , and are mutually exclusive, so ( ) = Substituting ( ) = 1 gives 1 = ( ) + ( ).

( ) + ( ).


40. ( ) = .3 =

Solutions Section 8.3 ( )+ ( ) ( ) ( ) + ( ) .1 $ ( ) + ( ) = .3 + .1 = .4

41. ( ) = .2, ( ) = .1; ( ) = .4 ( ) is more than ( ) + ( ). But, since ( ) = ( ) + ( ) ( ), ( ) cannot be more than ( ) + ( ). Therefore, the given information does not describe a probability distribution. 42. ( ) = .2, ( ) = .4; ( ) = .2 No; ( ) should be & ( ), since

/ .

44. ( ) = .2, ( ) = .4; ( ) = .3 No; ( ) should be 0 ( ), since

1 .

43. ( ) = .2, ( ) = .4; ( ) = .2 Since ( ) is 0 both ( ) and ( ), the given information is consistent with a probability distribution.

45. ( ) = .1, ( ) = 0; ( ) = 0 No; ( ) should be & ( ), since

/ .

46. ( ) = .1, ( ) = 0; ( )= 0 Yes; for instance, could be and could be empty.

47. a. = {Current, Past Due}; ( ) = 136,330 + 53,310 = 189,640 ( ) 189,640 ( ) = .93 = ( ) 203,480 b. = {Past Due, In Foreclosure, Repossessed}; ( ) = 203,480 136,330 = 67,150 ( ) 67,150 ( ) = .33 = ( ) 203,480 48. a. = {In Foreclosure, Repossessed}; ( ) = 72,380 + 17,000 = 89,380 ( ) 89,380 ( ) = .31 = ( ) 293,040 b. = {Current, Past Due, In Foreclosure}; ( ) = 293,040 17,000 = 276,040 ( ) 276,040 ( ) = .94 = ( ) 293,040

49. The probability distribution is obtained by converting the percentages to decimals: Outcome Hispanic or Latino White (not Hispanic) African American Asian Other Probability

.553

.217

.052

(neither African American nor Asian) = 1 .052 .095 = .853

.095

.083

50. The probability distribution is obtained by converting the percentages to decimals: Outcome Hispanic or Latino White (not Hispanic) African American Asian Other Probability

.26

.56

.10

(neither Hispanic, Latino, nor African American) = 1 .26 .10 = .64

.05

.03


Solutions Section 8.3 51. a. = {Stock market success, Sold to other concern, Fail} 2 3 b. (Stock market success) = (Sold to other concerns) = = .2 = .3 10 10 (Fail) = 1 (.2 + .3) = .5 c. To realize profits for early investors, a start-up venture must be either a stock market success or sold to another concern, so (Profit) = .3 + .2 = .5. 52. = { = trading above first day close, ( = trading below first day close but above initial offering, = trading below initial offering} 73 53 17 17 b. ( ) = .58 (() = .29 ( )= .13 126 126 126 c. (Either below initial offering price or above the price it closed on its first day of trading) = .58 + .13 = .71

53. The outcomes of the experiment are: SUV, pickup, passenger car, and minivan. The given information tells us that (SUV) = .5 and (pickup) = .2. We are also told that (passenger car) is five times (minivan). Take (minivan) = %. Then we have: Outcome SUV Pickup Passenger Car Minivan Probability

.5

5%

.2

%

Since the sum of the probabilities must be 1, we get .5 + .2 + 5% + % = 1 $ .7 + 6% = 1 $ 6% = 1 .7 = .3 $ % = .05. So, (minivan) = .05, and (passenger car) = 5% = .25. This gives the required distribution: Outcome SUV Pickup Passenger Car Minivan Probability

.5

.2

.25

.05

54. The outcomes of the experiment are: pickup, passenger car, SUV, and minivan. The given information tells us that (pickup) = .15 and (passenger car) = .55. We are also told that (SUV) = 2 (minivan). This gives: Outcome Pickup Passenger Car SUV Minivan Probability

.15

2%

.55

%

Since the sum of the probabilities must be 1, we get .70 + 3% = 1 $ % = .10. This gives the required distribution: Outcome Pickup Passenger Car SUV Minivan Probability

.15

.55

.20

.10

55. Start with Outcome Probability

1

0

2

%

3

%

4% = 1, so % = 1)4 = .25, giving

4

%

5

%

6

0


Solutions Section 8.3 Outcome

1

2

3

4

5

6

Probability

0

.25

.25

.25

.25

0

Outcome

1

2

%

5

%

6

.5

%

4

Probability

%

3

Outcome

1

2

3

4

5

6

Probability

.5

.125

.125

.125

.125

0

1

2

3

4

5

6

(odd) = 0 + .25 + .25 = .5

56. Start with

.5 + 4% = 1, so % = .125, giving

(odd) = .5 + .125 + .125 = .75

0

57. Start with Outcome

%

2%

2%

2%

2%

%

Outcome

1

2

3

4

5

6

Probability

.1

.2

.2

.2

.2

.1

1

2

3

4

5

6

Probability

10% = 1, so % = .1, giving

(odd) = .1 + .2 + .2 = .5

58. Start with Outcome

2%

%

%

%

%

2%

Outcome

1

2

3

4

5

6

Probability

.25

.125

.125

.125

125

.25

Probability

8% = 1, so % = 1)8 = .125, giving

(odd) = .25 + .125 + .125 = .5

59. Let % = (matching numbers). Then (Mismatching numbers) = 2%. There are 6 matching pairs and 30 mismatching ones. Since the probabilities add to 1, 1 6% + 30(2%) = 1 $ 66% = 1, so % = . 66 This gives: 1 (1, 1) = (2, 2) = 2 2 2 = (6, 6) = 66 2 1 (1, 2) = 2 2 2 = (6, 5) = . = 66 33 To obtain an odd sum, the pair must consist of an even and odd number, and there are nine such


(odd sum) = 9 ×

(mismatching) pairs:

Solutions Section 8.3

6 1 . = 33 11

60. Since mismatching numbers never come up, (1, 2) = 2 2 2 = (6, 5) = 0. This leaves us with 6 equally likely matching pairs, so that 1 (1, 1) = (2, 2) = 2 2 2 = (6, 6) = . 6 To obtain an odd sum, the pair must consist of an even and odd number, and there are nine such (mismatching) pairs: (odd sum) = 9 × 0 = 0.

61. Take (2) = 15%. This gives Outcome Probability

38% = 1, so % = Outcome Probability (odd) =

1

2

3

4

5

6

5%

15%

5%

3%

5%

5%

5 38

15 38

5 38

4

3 38

5

5 38

5 38

4

5

6

1 , giving 38

1

2

3

5 5 5 15 + + = 38 38 38 38

6

62. Take (2) = 12%. This gives Outcome Probability

1

3%

26% = 1, so % =

Outcome Probability (odd) =

3 26 1

2

3

12%

3%

2%

3%

3%

12 26

3 26

2 26

3 26

3 26

1 , giving 26 2

3 3 3 9 + + = 26 26 26 26

3

4

5

6

63. Take : I will meet a tall dark stranger; ( ) = 1)3. : I will travel; ( ) = 2)3. Also, ( ) = 1)6. 1 2 1 5 ( ) = ( ) + ( ) ( ) = + = 3 3 6 6

64. Take : Amazon.com will go up this afternoon; ( ) = .5. : Alphabet Inc. will go up this afternoon; ( ) = .2. Also, ( ) = .2. ( ) = ( ) + ( ) ( ) = .5 + .2 .2 = .5


Solutions Section 8.3 65. Take : A randomly selected person selected from one of the three cities was a New Yorker and had been infected with COVID-19 at some point during the year. We are told that ( ) = .25, and we are asked to find ( ). ( ) = 1 ( ) = 1 .25 = .75 66. Take : A randomly selected person selected from one of the three cities was either an Angelino or had been infected at some point during the year. We are told that ( ) = .41, and we are asked to find ( ). ( ) = 1 ( ) = 1 .41 = .59

67. Take : A randomly chosen electric car was manufactured by Tesla; ( ) = .30. : It was manufactured by Nissan; ( ) = .35. The events and are mutually exclusive, so ( ) = ( ) + ( ) = .30 + .35 = .65. We are asked to find [( ) ] = 1 ( ) = 1 .65 = .35.

68. Take : A randomly chosen electric vehicle was manufactured by Tesla; ( ) = .66. : It was manufactured by Chevrolet; ( ) = .10. The events and are mutually exclusive, so ( ) = ( ) + ( ) = .66 + .10 = .76. We are asked to find [( ) ] = 1 ( ) = 1 .76 = .24. 69. = set of all applicants; ( ) = 108,877 = set of admitted applicants; ( ) = 15,644 ( ) 15,644 ( ) = .14 = ( ) 108,877

70. = set of all applicants; ( ) = 108,877 = set of applicants who had a GPA below 3.53; ( ) = 26,131 ( ) 26,131 ( ) = .24 = ( ) 108,877

71. = set of all applicants; ( ) = 108,877 = set of admitted applicants who had a GPA below 3.53; ( ) = 313 ( ) 313 ( ) = .0029 .00 = ( ) 108,877

72. = set of all applicants; ( ) = 108,877 = set of admitted applicants who had a GPA of 3.95 or above; ( ) = 10,683 ( ) 10,683 ( ) = .10 = ( ) 108,877 73. = set of all applicants; ( ) = 108,877 = set of applicants not admitted; ( ) = 93,233 ( ) 93,233 ( ) = .86 = ( ) 108,877

74. : An applicant had a GPA below 3.53. ( ) = (

) = 1 ( ) 1 .24 = .76

( ) 26,131 .24 = ( ) 108,877


Solutions Section 8.3 75. ': An applicant had a GPA in the range 3.53–3.94. : An applicant was admitted. 55,527 15,644 4,693 (') = .51 ( )= .14 (' )= .04 108,877 108,877 108,877 (' ) = (') + ( ) (' ) .51 + .14 .04 = .61

76. ': An applicant had a GPA of 3.95 or above. : An applicant was admitted. 27,219 15,644 10,683 (') = .250 ( )= .144 (' )= .098 108,877 108,877 108,877 (' ) = (') + ( ) (' ) .250 + .144 .098 .30 (Note that the answer will come out to 0.29 if intermediate results are rounded to two decimal places.) 77. ': An applicant had a GPA in the range 3.53–3.94. : An applicant was admitted. We are asked to find [(' ) ]. In Exercise 75 we computed (' ) .61. So [(' ) ] = 1 (' ) 1 .61 = .39. 78. ': An applicant had a GPA of 3.95 or above. : An applicant was admitted. We are asked to find [(' ) ]. In Exercise 76 we computed (' ) .30. So [(' ) ] = 1 (' ) 1 .30 = .70.

79. = set of all applicants who had a GPA below 3.53; ( ) = 26,131 = set of admitted applicants who had a GPA below 3.53; ( ) = 313 ( ) 313 ( ) = .01 = ( ) 26,131

80. = set of all applicants who had a GPA of 3.95 or above; ( ) = 27,219 = set of admitted applicants who had a GPA of 3.95 or above; ( ) = 10,683 ( ) 10,683 ( ) = .39 = ( ) 27,219 81. = set of all admitted applicants; ( ) = 15,644 = set of admitted applicants who had a GPA of 3.95 or above; ( ) = 10,683 ( ) 10,683 ( ) = .68 = ( ) 15,644 82. = set of all admitted applicants; ( ) = 15,644 = set of admitted applicants who had a GPA below 3.53; ( ) = 313 ( ) 313 ( ) = .02 = ( ) 15,644

83. = set of all rejected applicants; ( ) = 93,233 = set of rejected applicants who had a GPA below 3.95; ( ) = 50,834 + 25,818 = 76,652 ( ) 76,652 ( ) = .82 = ( ) 93,233

84. = set of all rejected applicants; ( ) = 93,233 = set of rejected applicants who had a GPA of at least 3.53; ( ) = 16,581 + 50,834 = 67,415 ( ) 67,415 ( ) = .72 = ( ) 93,233 85. Use the following events:


Solutions Section 8.3 : Agree that the federal budget deficit is a moderately to very big problem. : Agree that the condition of infrastructure is a moderately to very big problem. ( ) = ( ) + ( ) ( ) ( ) = .82 + .74 ( ) = 1.56 ( ) . . . (1) Since ( ) must be less than or equal to 1, ( ) must be at least .56, so the smallest percentage of people that could have agreed with both statements is 56%. For the second part of the question, we ask how large ( ) can be for formula (1) to make sense. The largest conceivable value for ( ) is .74 (because is a subset of and and hence its probability cannot exceed either .82 or .74). Thus, the largest percentage of people that could have agreed with both statements is 74%. 86. Use the following events: : Agree that racism is a very big problem. : Agree that sexism is a very big problem. ( ) = ( ) + ( ) ( ) ( ) = .45 + .23 ( ) = .68 ( ) . . . (1) ( ) assumes its largest possible value of .68 when ( ) = 0, and so the largest percentage of people that could have agreed with at least one of these statements is 68%. To make ( ) as small as possible, we make ( ) as large as possible. The largest conceivable value for ( ) is .23 (because is a subset of and and hence its probability cannot exceed either .45 or .23). This gives ( ) = .68 .23 = .45. Thus, the smallest percentage of people that could have agreed with at least one of these statements is 45%. 87. Use the following events: : Failed it backward; ( ) = We are also told that (

) =

5 . 12

2 3 ; : Failed it sideways; ( ) = . 3 4

2 3 5 + = 1 3 4 12 Therefore, all of them failed it either backwards or sideways, so all were disqualified. ( ) =

( ) + ( ) (

) =

88. Use the following events: (: Encounter the Myrmidons; (() = .50; : Encounter the Balrog; ( ) = .20. We are also told that (( ) = .10. (( ) = (() + ( ) (( ) = .50 + .20 .10 = .60 $ [(( ) ] = 1 .60 = .40

89. Use the following events: 3 : Vaccinated; (3 ) = .80; : Infected; ( ) = .10. We are also told that 2% of the vaccinated population gets infected. Therefore, the percentage that are vaccinated and also get infected is (3 ) = .02 (3 ) = .02 × .80 = .016. Now (3 ) = (3 ) + ( ) (3 ) = .80 + .10 .016 = .884. Thus, 88.4% of the population either gets vaccinated or get infected.

90. Use the following events: 3 : Vaccinated; (3 ) = .75; : Become infected; ( ) = .10. We are also told that 4% of the vaccinated population becomes infected. Therefore, the percentage that are vaccinated and also become infected is (3 ) = .04 (3 ) = .04 × .75 = .03. Now (3 ) = (3 ) + ( ) (3 ) = .75 + .10 .03 = .82.


Solutions Section 8.3 Therefore, [(3 ) ] = 1 (3 ) = 1 .82 = .18. 91. Here is one possible experiment: Roll a die, and observe which of the following outcomes occurs: Outcome : 1 or 2 facing up; ( ) = 1)3, Outcome : 3 or 4 facing up; ( ) = 1)3, Outcome : 5 or 6 facing up; ( ) = 1)3. 92. Here is one possible experiment: Roll a die, and observe which of the following outcomes occurs: Outcome A: 1 facing up; ( ) = 1)6, Outcome : 2 or 3 facing up; ( ) = 1)3, Outcome : 4, 5, or 6 facing up; ( ) = 1)2. 93. He is wrong. It is possible to have a run of losses of any length. Each time he plays the game, the chances of losing are the same, regardless of his history of wins or losses. Tony may have grounds to suspect that the game is rigged, but he has no proof. 94. Her reasoning is suspect. Based on the four games to date, she is correct that the relative frequency of winning is 100%. However, four samples are too few to trust that the relative frequency closely approximates the winning (modeled) probability. In fact, most casino games have winning probabilities slightly less than .5. 95. The probability of the union of two events is the sum of the probabilities of the two events if they are mutually exclusive. 96. It may rain and snow the same day. If is the event that it rains and is the event that it snows, and may not be disjoint, so ( ) + ( ) may be larger than ( ); in particular, ( ) + ( ) can be larger than 1. 97. Wrong. For example, the modeled probability of winning a state lottery is small but nonzero. However, the vast majority of people who play the lottery every day of their lives never win, no matter how frequently they play, so the relative frequency is zero for these people. 98. When the modeled probability of an event is extremely close to zero (like the lottery), it may require a very large number of repetitions (a lot more than 600) to see the limit. For instance, if the probability of winning the lottery is 1 in a million, then, after 600 plays, one would expect to win about 600/1,000,000 times, or only 0.0006 times on average. 99. When = we have ( ( )= ( )+ ( ) (

)=

( ) = 0, so )= ( )+ ( ) 0=

( ) + ( ).

100. Let = {41 , 42 , . . . , 4 } with = {41 , 42 , . . . , 4 }, so = {4 +1 , 4 +2 , . . . , 4 }. Then ( ) = (4 +1 ) + 2 2 2 + (4 ) = (41 ) + 2 2 2 + (4 ) + (4 +1 ) + 2 2 2 + (4 ) [ (41 ) + 2 2 2 + (4 )] = 1 ( ). 101. Zero. According to the assumption, no matter how many thunderstorms occur, lightning cannot strike a given spot more than once, so, after trials the relative frequency will never exceed 1) and so will approach zero as the number of trials gets large. Since the modeled probability models the limit of relative frequencies as the number of trials gets large, it must therefore be zero. 102. Wrong. The assertion amounts to the claim that the relative frequency of such an event must always be zero. What must be true is that, as the number of trials gets larger, the relative frequency of the event must approach zero. That does not mean that the relative frequency is always zero.


103. (

104. (

) =

Solutions Section 8.3 ( ) + ( ) + ( ) ( ) ( ) (

) =

( ) + ( ) + ( ) + ( ) 3 (

) + (

)

)


Solutions Section 8.4 Section 8.4

1. ( ) = (10, 5) = 252 ( ) = (4, 4) (6, 1) = 6 ( ) 6 1 ( ) = = = ( ) 252 42 2. ( ) = (10, 5) = 252 ( ) = (6, 5) = 6 ( ) 6 1 ( ) = = = ( ) 252 42

(4 red, 1 non-red)

(5 non-red)

3. ( ) = (10, 5) = 252 : At least 1 white marble The complementary event is : No white ones. ( ) = (8, 5) = 56 (5 non-white) ( ) 56 2 ( ) = = = ( ) 252 9 Therefore, 2 7 ( ) = 1 ( ) = 1 = . 9 9 4. ( ) = (10, 5) = 252 : At least 1 green marble The complementary event is : No green ones. ( ) = (7, 5) = 21 (5 non-green) ( ) 21 1 ( ) = = = ( ) 252 12 Therefore, 1 11 ( ) = 1 ( ) = 1 . = 12 12 5. ( ) = (10, 5) = 252 ( ) = (4, 2) (3, 1) (2, 1) (1, 1) = 36 ( ) 36 1 ( ) = = = ( ) 252 7 6. ( ) = (10, 5) = 252 ( ) = (3, 2) (4, 1) (2, 1) (1, 1) = 24 ( ) 24 2 ( ) = = = ( ) 252 21

(2 red, 1 green, 1 white, 1 purple)

(2 green, 1 red, 1 white, 1 purple)

7. ( ) = (10, 5) = 252 ( ) = (7, 5) + (3, 1) (7, 4) = 21 + 105 = 126 ( ) 126 1 ( ) = = = ( ) 252 2 8. ( ) = (10, 5) = 252 ( ) = (8, 5) + (2, 1) (8, 4) = 56 + 140 = 196 ( ) 196 7 ( ) = = = ( ) 252 9

(5 non-green or 1 green, 4 non-green)

(5 non-white or 1 white, 4 non-white)


Solutions Section 8.4 9. : She does not have all the red ones. The complementary event is : She has all the red ones. In Exercise 1, we computed this probability: 1 ( ) = . 42 Therefore, 1 41 ( ) = 1 . = 42 42 10. ( ) = (10, 5) = 252 : She does not have all the green ones The complementary event is : She has all the green ones. ( ) = (3, 3) (7, 2) = 21 (3 green, 2 non-green) ( ) 21 1 ( ) = = = ( ) 252 12 Therefore, 1 11 ( ) = 1 . = 12 12

11. ( ) = (10, 2) = 45 There are only 3 stocks listed with yields of 5.00% or more. Therefore, ( ) 3 1 ( ) = (3, 2) = 3 $ ( ) = . = = ( ) 45 15

12. ( ) = (10, 3) = 120 There are only 3 stocks listed with yields of 5.00% or more. Therefore, ( ) 1 ( ) = (3, 3) = 1 $ ( ) = . = ( ) 120

13. ( ) = (10, 4) = 210; ( ) = (1, 1) (8, 3) = 56 (Choose CVX and then 3 out of the 8 that remain when you exclude CVX and KO.) ( ) 56 4 ( ) = = = ( ) 210 15

14. ( ) = (10, 4) = 210; ( ) = (2, 2) (6, 2) = 15 (Choose VZ and CSCO and then 2 out of the 6 that remain when you exclude VZ, CSCO, WBA, and MRK.) ( ) 15 1 ( ) = = = ( ) 210 14 15. ( ) = (10, 2) = 45 Ending up with 200 shares of IBM means that IBM was one of the stocks you chose: ( ) = (1, 1) (9, 1) = 9 (Choose IBM and then 1 out of the 9 that remain.) ( ) 9 1 ( ) = = = . ( ) 45 5

16. ( ) = (10, 3) = 120 Ending up with 200 shares of IBM means that IBM was one of the stocks you chose: ( ) = (1, 1) (9, 2) = 36 (Choose IBM and then 2 out of the 9 that remain.) ( ) 36 3 ( ) = . = = ( ) 120 10

17. Number of possible completed tests: ( ) = 2 8 × 5 5 × 5! (8 true/false, 5 multiple choice, 5 matching)


Number of correct answers: ( ) = 1 ( ) 1 ( ) = = ( ) 2 8 × 5 5 × 5!

Solutions Section 8.4

18. Number of possible completed tests: ( ) = 2 4 × 5 4 × 6! (4 true/false, 4 multiple choice, 6 matching) Number of correct answers: ( ) = 1 ( ) 1 ( ) = = 4 ( ) 2 × 5 4 × 6! 19. ( ) = (52, 5) = 2,598,960; ( ) = 1,098,240 (6.4 Exercise 57) ( ) 1,098,240 ( ) = .4226 = ( ) 2,598,960 20. ( ) = (52, 5) = 2,598,960; ( ) = 54,912 (6.4 Exercise 56) ( ) 54,912 ( ) = .0211 = ( ) 2,598,960

21. ( ) = (52, 5) = 2,598,960; ( ) = 123,552 (6.4 Exercise 55) 123,552 ( ) = .0475 2,598,960 22. ( ) = (52, 5) = 2,598,960 Decision algorithm for constructing a straight flush: Step 1: Select a starting card (A–9): 9 choices. Step 2: Select a suit: 4 choices. This gives ( ) = 36. ( ) 36 ( ) = .00001 = ( ) 2,598,960

23. ( ) = (52, 5) = 2,598,960 Decision algorithm for computing the number of flushes: First compute the number of hands in which all 5 cards are of the same suit: Step 1: Select a suit: 4 choices. Step 2: Select 5 cards of that suit: (13, 5) choices. Total number of choices: 4 × (13, 5) Among these are hands in which the 5 cards are consecutive: A, 2, 3, 4, 5 up through 10, J, Q, K, A (straight or royal flushes). The number of straight or royal flushes is 4 × 10 (Step 1: Select a suit; Step 2: Select a starting card for the run). Excluding these gives Total number of flushes: ( ) = 4 × (13, 5) 4 × 10 = 5,108 ( ) 5,108 ( ) = .0020. = ( ) 2,598,960 24. ( ) = (52, 5) = 2,598,960 Decision algorithm for computing the number of straights: If we ignore the restriction about the suits, we get the following decision algorithm: Step 1: Select a starting denomination for the straight: (A, 2, ..., 10): 10 choices. Steps 2–6: Select a card of each of the above denominations: 4 × 4 × 4 × 4 × 4 = 4 5 choices. Total number of runs of 5 cards: 10 × 4 5 However, these runs include hands in which all 5 cards are of the same suit (straight or royal flushes). There are 10 × 4 of these (Step 1: Select a starting card; Step 2: Select a single suit). Excluding the


Solutions Section 8.4 straight and royal flushes gives Total number of straights: ( ) = 10 × 4 5 10 × 4 = 10,200 ( ) 10,200 ( ) = .0039. = ( ) 2,598,960 25. ( ) = 27 39; ( ) = 1 (The correct sequence) ( ) 1 ( ) = = ( ) 27 39

26. ( ) = 88 4; ( ) = 1 (The Beethoven sequence) ( ) 1 ( ) = = ( ) 88 4

27. ( ) = (7, 4) = 35; ( ) = (5, 4) = 5 $ ( ) =

( ) 5 1 = = ( ) 35 7

28. ( ) = (7, 4) = 35; ( ) = (1, 1) (5, 3) = 10 $ ( ) = 29. ( ) = (50, 5) = 2,118,760

( ) 10 2 = = ( ) 35 7

( ) 1 .000000472 = ( ) 2,118,760 Small-Fry Winner: ( ) = (5, 4) (45, 1) = 225 (4 winning numbers & 1 losing number) ( ) 225 ( ) = .000106194 = ( ) 2,118,760 226 ( ) = ( ) + ( ) = .000106666 (Mutually exclusive) 2,118,760 Big Winner: ( ) = 1 $ ( ) =

30. ( ) =

(50, 3) = 117,600

( ) 1 .000008503 = ( ) 117,600 ( ) 5 Booby Prize Winner: ( ) = 3! 1 = 5 $ ( ) = .000042517 = ( ) 117,600 6 ( ) = ( ) + ( ) = .00005102 (Mutually exclusive) 117,600 Winner: ( ) = 1 $ ( ) =

31. ( ) = (700, 400) a. ( ) = (100, 100) (600, 300) = (600, 300) 100 managers, 300 non-managers ( ) (600, 300) ( ) = = ( ) (700, 400) b. ( ) = (1, 1) (699, 399) = (699, 399) You, 399 others ( ) (699, 399) ( ) = = ( ) (700, 400) Note that (699, 399) 699!)(399! × 300!) 699! × 400! 400 . = = = (700, 400) 700!)(400! × 300!) 700! × 399! 700

32. ( ) = (100, 50) (100, 50) (500, 300) (50 managers, 50 workers, 300 misc. staff) ( ) = (1, 1) (99, 49) (100, 50) (500, 300) (You, 49 other managers, 50 workers, 300 misc.


Solutions Section 8.4

staff)

( ) =

( ) (1, 1) (99, 49) (100, 50) (500, 300) (99, 49) = = ( ) (100, 50) (100, 50) (500, 300) (100, 50)

33. ( ) = 10 × 10 × 10 = 10 3; ( ) = 10 × 9 × 8 = ( ) 720 ( ) = = = .72 ( ) 1,000

(10, 3) = 720

34. ( ) = 10 × 10 × 10 = 10 3 Decision algorithm for selecting a sequence of 3 digits in which 2 are the same: Step 1: Select a position for the non-repeating digit: 3 choices. Step 2: Select the nonrepeating digit: 10 choices. Step 3: Select the repeating digit: 9 choices. ( ) 270 ( ) = 3 × 10 × 9 = 270 $ ( ) = = = .27 ( ) 1,000

35. The number of possible outcomes is ( ) = 2 × 2 × 2 = 8. (Select a winner 3 times) The only way North Carolina (NC) will beat Central Connecticut (CC) but lose to Virginia is if NC beats CC, Virginia beats Syracuse, and CC loses to Virginia. Therefore, ( ) 1 ( ) = 1 $ ( ) = = . ( ) 8

36. The number of possible outcomes is ( ) = 2 × 1 × 2 × 2 × 2 = 16. (Decision algorithm: Select a team to oppose Colgate, select the remaining team to oppose Hofstra, select the winner 3 times.) The number of ways Hofstra can play Colgate in the finals and win is ( ) = 2 × 1 = 2 since the outcomes of all the playoffs are determined. ( ) 2 1 ( ) = = = ( ) 16 8 37. As a seeding consists of a list of 16 teams, ( ) = the number of possible seedings = To choose a seeding of the designated type, we have the following decision algorithm: Choose the order in which the games 1 vs 16, 2 vs 15, etc. appear: (8, 8) = 8! In each of these 8 games, choose the order of the teams: 2 8. So, ( ) = (8, 8) × 2 8 = 8! × 2 8 ( ) 8! × 2 8 ( ) = = ( ) 16!

(16, 16) = 16!

38. As a seeding consists of a list of 8 teams, ( ) = the number of teams = (8, 8) = 8! To choose a seeding of the designated type, we have the following decision algorithm: Choose the order in which the games 1 vs 2, 3 vs 4, etc. appear: (4, 4) = 4! In each of these 4 games, choose the order of the teams: 2 4. So, ( ) = (4, 4) × 2 4 = 4! × 2 4 ( ) 4! × 2 4 ( ) = = ( ) 8! 39. There are 32 first-round games, 16 second-round games, 8 third round games, and 4 fourth-round games, giving a total of 60 games in the first four rounds. There are two more rounds to complete the tourmament (called "the final four" and the finals): 2 fifth-round games, and one final. Thus, there are ( ) = 2 × 2 × 2 2 2 × 2 = 2 63 possible brackets. a. Because only one of these brackets is the correct one, ( ) = 1, so ( ) = 1)2 63.


Solutions Section 8.4 b. Decision algorithm for picking the winners with 15 upsets in the first four rounds: Step 1: Choose 15 games in which the upsets occur: (60, 15) choices Step 2: Choose the winners for each of the three games in the last two rounds: 2 × 2 × 2 = 8. Total number of possiblties: ( ) = (60, 15) × 8 = 425,552,713,541,760 possibilities ( ) (60, 15) × 8 425,552,713,541,760 ( ) = = = 63 ( ) 9,223,372,036,854,775,808 2 40. There are 32 first-round games, 16 second-round games, 8 third round games, 4 fourth-round games, 2 fifth-round games, and one final, giving a total of 63 games. Thus, ( ) = 2 × 2 × 2 2 2 × 2 = 2 63 Decision algorithm for picking the winners with 10 upsets in the first round, 4 in the second round, and three in each of the third and fourth rounds: Step 1: Choose 10 fist-round games games in which the upsets occur: (32, 10) choices Step 2: Choose 4 second-round games games in which the upsets occur: (16, 4) choices Step 3: Choose 3 third-round games games in which the upsets occur: (8, 3) choices Step 4: Choose 3 fourth-round games games in which the upsets occur: (4, 3) choices Step 2: Choose the winners for each of the three games in the last two rounds: 2 × 2 × 2 = 8. Total number of possiblties: ( ) = (32, 10) (16, 4) (8, 3) (4, 3) × 8 = 210,402,800,025,600 possibilities ( ) (32, 10) (16, 4) (8, 3) (4, 3) × 8 210,402,800,025,600 ( ) = = = ( ) 9,223,372,036,854,775,808 2 63 41. A perfect progression is a sequence of 8 "Won" scores in the range 0–7 that are all different. is the set of all sequences of 8 digits in the range 0–7. ( ) 8! ( ) = 8 8; ( ) = 8! $ ( ) = = ( ) 8 8 42. is the set of all sequences of 8 digits in the range 0–7; ( ) = 8 8. is the set of all sequences of 8 digits in which the first digit is a 7; ( ) = 7! ( ) 7! ( ) = . = ( ) 8 8

43. Each of the two random moves from the starting position can be up, down, left, or right. Since this gives 4 choices per move, there are ( ) = 4 × 4 = 16 possible sequences of two moves. Only two of these sequences will get you to the Finish node: right + down and down + right. Therefore, ( ) 2 1 ( ) = 2 $ ( ) = = = . ( ) 16 8 44. Each of the two random moves from the starting position can be up, down, left, or right. Since this gives 4 choices per move, there are ( ) = 4 × 4 = 16 possible sequences of two moves. Of these sequences, exactly 3 will get you to a Finish node: up + up, right + down and down + right. Therefore, ( ) 3 ( ) = 3 $ ( ) = . = ( ) 16

45. The number of possible sequences of 4 digits in the range 0–9 is ( ) = 10 × 10 × 10 × 10 = 10,000. Number of correct codes: 1 Number of codes that are correct except for a single digit: Select a slot for the incorrect digit: 4 choices.


Solutions Section 8.4 Select the incorrect digit: 9 choices. Fill in the remaining slots with the correct digits: 1 choice. Total number of incorrect codes: 4 × 9 × 1 = 36 ( ) 37 Therefore, ( ) = 1 + 36 = 37 $ ( ) = = = .0037. ( ) 10,000 46. Since you already know the first digit, you are left with 3, of which one is allowed to be incorrect. ( ) = 10 × 10 × 10 = 1,000 Number of correct codes: 1 Number of codes that are correct except for a single digit: Select a slot for the incorrect digit: 3 choices. Select the incorrect digit: 9 choices. Fill in the remaining slots with the correct digits: 1 choice. Total number of incorrect codes: 3 × 9 × 1 = 27 ( ) 28 Therefore, ( ) = 1 + 27 = 28 $ ( ) = = = .028. ( ) 1,000 47. a. Decision algorithm for forming a committee: Select a chief investigator: (6, 1) choices. Select an assistant investigator: (6, 1) choices. Select 2 at-large investigators: (10, 2) choices. Select 5 ordinary members: (8, 5) choices. This gives ( ) = (6, 1) (6, 1) (10, 2) (8, 5) = 90,720. b. Decision algorithm to make Larry happy: Alternative 1: Larry is chief and Otis is assistant: Select Larry for chief and Otis for assistant: 1 choice. Select 2 at-large investigators: (10, 2) choices. Select 5 ordinary members: (8, 5) choices. This gives (10, 2) (8, 5) choices for this alternative. Alternative 2: Larry is not on the committee: Select a chief investigator: (5, 1) choices. Select an assistant investigator: (6, 1) choices. Select 2 at-large investigators: (9, 2) choices. Select 5 ordinary members: (7, 5) choices. This gives (5, 1) (6, 1) (9, 2) (7, 5) choices for this alternative. Adding gives a total of ( ) = (10, 2) (8, 5) + (5, 1) (6, 1) (9, 2) (7, 5) = 25,200. ( ) 25,200 c. ( ) = .28 = ( ) 90,720 48. a. Decision algorithm for forming a committee: Select a chair: (10, 1) choices. Select 3 hagglers: (9, 3) choices. Select 4 do-nothings: (6, 4) choices. Total number of committees: ( ) = (10, 1) (9, 3) (6, 4) = 12,600 b. Decision algorithm for forming a committee with Norman as chair: Select Norman as chair: (1, 1) choice. Select 3 hagglers: (9, 3) choices. Select 4 do-nothings: (6, 4) choices. Total number of committees:


Solutions Section 8.4 ( ) 1,260 ( ) = (1, 1) (9, 3) (6, 4) = 1,260 $ ( ) = = = .1 ( ) 12,600 c. To make Norma happy, both she and Norman should serve. Since there are a total of 8 people out of the pool of 10 who will serve, making Norma happy means excluding exactly two other members: Step 1: Select 2 members to exclude (other than Norma and Norman): (8, 2) choices. There are now 8 people left and so all (including Norma and Norman) will serve: Steps 2–4: Form the committee from the remaining 8: (8, 1) (7, 3) (4, 4) choices. ( ) 7,840 28 ( ) = (8, 2) (8, 1) (7, 3) (4, 4) = 7,840 $ ( ) = .6222 = = ( ) 12,600 45 d. Decision algorithm to make Norma happy: Alternative 1: Exclude Oona from the committee. Step 1: Exclude Oona: 1 choice. Step 2: Exclude another member (so that she and Norman will get selected for something): (7, 1)choices. Steps 3–5 Form the committee from the remaining 8: (8, 1) (7, 3) (4, 4) choices. (7, 1) (8, 1) (7, 3) (4, 4) choices for Alternative 1 Alternative 2: Oona on the committee, but not as chair Step 1: Exclude two members other than Oona, Norma, and Norman: (7, 2) choices. Step 2: Select a chair (excluding Oona): (7, 1) choices. Steps 3–4: form the rest of the committee from the remaining 7: (7, 3) (4, 4) choices. (7, 2) (7, 1) (7, 3) (4, 4) choices Adding gives ( ) = (7, 1) (8, 1) (7, 3) (4, 4) + (7, 2) (7, 1) (7, 3) (4, 4) = 7,105 ( ) 7,105 ( ) = .5639. = ( ) 12,600 49. The four outcomes listed are not equally likely; for example, (red, blue) can occur in four ways. The methods of this section yield a probability for (red, red) of (2, 2)) (4, 2) = 1)6. 50. The methods of this section assume that the outcomes are equally likely. They are not in this experiment (see Example 2 in Section 7.3).

51. No. If we do not pay attention to order, the probability is (5, 2)) (9, 2) = 10)36 = 5)18. If we do pay attention to order, the probability is (5, 2)) (9, 2) = 20)72 = 5)18 again. The difference between permutations and combinations cancels when we compute the probability. 52. Most events that refer to the first and second marble will do. For example, the event that the first marble is red and the second green. The probability is 5 × 4) (9, 2) = 5)18. Compare this to the probability that she gets one red and one green marble, which is 5 × 4) (9, 2) = 5)9. 53. Answers will vary. 54. Answers will vary.


Solutions Section 8.5 Section 8.5

(

1. ( | ) = 3. (

)=

5. (

)=

( )

)

=

.2 = .4 .5

( | ) ( ) = (.2)(.4) = .08

( | ) ( ) .3 .3 = .4 ( ) $ ( ) = = .75 .4 7. If

and

8. If

and

9. If

and

10. If

and

11. As

12. As

2. ( | ) =

are independent, then ( are independent, then (

)= )=

are independent, then ( | ) =

are independent, then ( | ) =

and are independent, ( ( )= ( )+ ( ) (

and are independent, ( ( )= ( )+ ( ) (

4. (

)=

6. (

)=

(

( )

)

=

.3 = .5 .6

( | ) ( ) = (.1)(.5) = .05

( | ) ( ) .1 .1 = .4 ( ) $ ( ) = = .25 .4

( ) ( ) = (.5)(.4) = .2.

( ) ( ) = (.2)(.2) = .04.

( ) = .5.

( ) = .6.

) = ( ) ( ) = (.3)(.2) = .06. Thus, ) = .3 + .2 .06 = .44.

) = ( ) ( ) = (.8)(.1) = .08. Thus, ) = .8 + .1 .08 = .82.

13. 10% of all Anchovians detest anchovies ( ). Rewording: The probability that an Anchovian detests anchovies is .10 : ( ) = .10. 30% of all married Anchovians (() detest anchovies. Rewording this as in Example 2, we get: The probability that an Anchovian detests anchovies, given that he or she is married, is .30; ( |() = .30. 14. 95% of all music composers can read music ((). Rewording: The probability that a music composer can read music is .95.; (() = .95. 99% of all classical music composers ( ) can read music. Rewording this as in Example 2, we get: The probability that a music composer can read music, given that he or she is a classical music composer, is .99; ((| ) = .99. 15. 30% of all lawyers who lost clients (5) were antitrust lawyers ( ). Rewording this as in Example 2, we get: The probability that a lawyer was an antitrust lawyer, given that she lost clients, is .30; ( |5) = .30. 10% of all antitrust lawyers lost clients. Rewording this as in Example 2, we get: The probability that a lawyer lost clients, given that she was an antitrust lawyer, was .10; (5| ) = .10. 16. 2% of all items bought on my auction site ( ) were works of art ( ). Rewording this as in Example 2, we get:


Solutions Section 8.5 The probability that an item was a work of art, given that it was bought, was .02; ( | ) = .02. 1% of all works of art on the site were bought. Rewording this as in Example 2, we get: The probability that an item was bought, given that it was a work of art, was .01; ( | ) = .01. 17. 55% of those who go out in the midday sun (M) are Englishmen (E). Rewording this as in Example 2, we get: The probability that someone is an Englishman, given that he goes out in the midday sun, is .55; ( |() = .55. 5% of those who do not go out in the midday sun are Englishmen. Rewording this as in Example 2, we get: The probability that someone is an Englishman, given that he does not go out in the midday sun, is .05; ( |( ) = .05. 18. 80% of those who have a Mac now (() will purchase a Mac next time ( ). Rewording this as in Example 2, we get: The probability that someone will purchase a Mac next time, given that she has a Mac now, is .80; ( |() = .80. 20% of those who do not have a Mac now will purchase a Mac next time. Rewording this as in Example 2, we get: The probability that someone will purchase a Mac next time, given that she does not have a Mac now, is .80; ( |( ) = .20. : The green one is not a 1. 3 1 30 5 (3, 2), (2, 3), (1, 4) = ( )= = = 36 12 36 6

19. : The sum is 5. (

)=

: The green one is either 4 or 3. 2 1 12 1 (2, 4), (3, 3) = ( )= = = 36 18 36 3

20. : The sum is 6. (

)=

1 (5, 1) = 36

21. : The red one is 5. (

( )=

)=

1 (4, 4) = 36

(

)=

( | )=

5 36

( | )=

: The green one is a 4. 1 ( )= ( | )= 6

(

(

( )

( ) )

=

)

)=

( )

( )

)

: The dice have opposite parity. 4 1 18 1 (4, 1), (3, 2), (2, 3), (1, 4) = ( )= = = 36 9 36 2

=

=

1)12 1 = 5)6 10

1)18 1 = 1)3 6

1)36 1 = 5)36 5

( | )=

24. : The sum is 6. : The dice have opposite parity. ( ) = ( ) = 0 (If the sum is 6, the dice must be either both odd or both even.) ( ) 18 1 0 ( )= ( | )= = = = 0 36 2 ( ) 1)2 25. : She gets all 3 red ones.

=

)

1)36 1 = 1)6 6

23. : The sum is 5. (

(

(

: The sum is 6.

(1, 5), (2, 4), (3, 3), (4, 2), (5, 1) =

22. : The red one is 4.

( | )=

: She gets the fluorescent pink one.

(

( )

)

=

1)9 2 = 1)2 9


Solutions Section 8.5 ( ) = (10, 4) = 210 (1, 1) (9, 3) (1, 1) (3, 3) 1 84 ( )= ( )= = = 210 210 210 210 1)210 ( ) 1 ( | )= = = ( ) 84)210 84 26. : She gets all 3 red ones. : She does not get the fluorescent pink one. ( ) = (10, 4) = 210 (3, 3) (6, 1) (9, 4) 126 6 ( )= ( )= = = 210 210 210 210 6)210 ( ) 6 1 ( | )= = = = ( ) 126)210 126 21 27. : She gets no red ones. : She gets the fluorescent pink one. ( ) = (10, 4) = 210 (1, 1) (6, 3) (1, 1) (9, 3) 20 84 ( )= ( )= = = 210 210 210 210 ( ) 20)210 5 ( | )= = = ( ) 84)210 21 28. : She gets one of each color other than fluorescent pink. ( ) = (10, 4) = 210 (

)=

( | )=

(

( ) = 0 ( )

)

=

( contradicts .)

0 = 0 84)210

( )=

29. : She gets one of each color other than fluorescent pink. Notice that ( ) = (10, 4) = 210 = . (3, 1) (2, 1) (2, 1) (2, 1) 24 ( )= ( )= = 210 210 24)210 ( ) 24 ( | )= = = ( ) 175)210 175

: She gets the fluorescent pink one.

(1, 1) (9, 3) 84 = 210 210

: She gets at least one red one. ( )= 1

30. : She gets at least two red ones. : She gets at least one green one. ( ) = (10, 4) = 210 (3, 2) (2, 2) + (3, 2) (2, 1) (5, 1) + (3, 3) (2, 1) 35 ( )= = 210 210 (8, 4) 70 140 ( )= 1 = 1 = 210 210 210 35)210 ( ) 1 ( | )= = = ( ) 140)210 4

(7, 4) 35 175 = 1 = 210 210 210


Solutions Section 8.5 31.

Since the probabilities leaving each branching point must add to 1, we can fill in the missing probabilities:

The probability of each outcome is obtained by multiplying the probabilities along the branches leading to the corresponding node: ( ) = .3 × .5 = .15 ( ) = .3 × .5 = .15 ( ) = .7 × .2 = .14 ( ) = .7 × .8 = .56. 32.

Since the probabilities leaving each branching point must add to 1, we can fill in the missing probabilities:

The probability of each outcome is obtained by multiplying the probabilities along the branches leading to the corresponding node: ( ) = .6 × .2 = .12 ( ) = .6 × .8 = .48 ( ) = .4 × .1 = .04 ( ) = .4 × .9 = .36.


Solutions Section 8.5 33.

Since the probabilities leaving each branching point must add to 1, we can fill in some of the missing probabilities (below left):

To get the remaining two (above right), use the given information about the probabilities of the outcomes: .14 ( ) = .7 × % = .14 $ % = = .2 $ 6 = 1 .2 = .8. .7 Finally, the probability of each outcome is obtained by multiplying the probabilities along the branches leading to the corresponding node: ( ) = .2 × .5 = .10 ( ) = .2 × .5 = .10 ( ) = .1 × .1 = .01 ( ) = .1 × .9 = .09 ( ) = .7 × .2 = .14 ( 7) = .7 × .8 = .56. 34.

Since the probabilities leaving each branching point must add to 1, we can fill in some of the missing probabilities (below left):


Solutions Section 8.5

To get the remaining ones (above right), use the given information about the probabilities of the outcomes: ( ) = .1 × 6 = 0 $ 6 = 0 $ % = .5 (since .5 + % + 0 = 1). .09 Also, ( ) = .9 × 8 = .09 $ 8 = = .1 $ 9 = .2. .9 Finally, the probability of each outcome is obtained by multiplying the probabilities along the branches leading to the corresponding node: ( ) = .1 × .5 = .05 ( ) = .1 × .5 = .05 ( ) = .1 × 0 = 0 ( ) = .9 × .1 = .09 ( ) = .9 × .2 = .18 ( 7) = .9 × .7 = .63. 35. : Your new skateboard design is a success. : Your new skateboard design is a failure. If your new skateboard design is a success, it cannot be a failure, and vice versa. Therefore, and mutually exclusive.

are

36. : Your new skateboard design is a success. : There is life in the Andromeda galaxy. The likelihood of has no effect on the likelihood of , Therefore, and are independent. 37. : Your new skateboard design is a success. : Your competitor's new skateboard design is a failure. Since the likelihood of does affect the likelihood of , the events are not independent. Since it is possible for and to occur together, they are not mutually exclusive. Therefore, the correct choice is "neither." 38. : Your first coin flip results in heads. : Your second coin flip results in heads. The likelihood of no effect on the likelihood of . Therefore, and are independent. 39. : The red die is 1, 2, or 3; ( ) =

1 2

: The red die is 1, 2, or 3 and the green one is even; ( 1 1 1 × = 2 2 4 ) = ( ) ( ),

( ) ( )=

Since (

40. : The red die is 1; ( ) =

1 4 are independent.

(

)=

and

1 6

1 1 1 × = 6 2 12 ) = ( ) ( ),

Since (

(

and

1 2 3 1 )= = 36 12

: The sum is even; ( ) =

: The red die is 1 and the sum is even; (

( ) ( )=

1 2 9 1 )= = 36 4

: The green die is even; ( ) =

1 12 are independent. )=

has


Solutions Section 8.5 10 5 1 41. : Exactly one die is 1; ( ) = : The sum is even; ( ) = = 36 18 2 4 1 : Exactly one die is 1 and the sum is even; ( )= = 36 9 5 1 5 1 ( ) ( )= ( )= × = 18 2 36 9 Since ( ) ( ) ( ), and are dependent. 42. : Neither die is 1 or 6; ( ) =

16 36

: The sum is even; ( ) =

: Neither die is 1 or 6 and the sum is even; ( 16 1 8 8 ( )= × = 36 2 36 36 ) = ( ) ( ), and are independent.

( ) ( )=

Since (

43. : Neither die is 1; ( ) =

25 36

: Exactly one die is 2; ( ) =

: Neither die is 1 and exactly one is 2; ( 25 5 .1929 × 36 18 ) ( ) ( ), and

( ) ( )=

Since (

44. : Both dice are 1; ( ) =

1 36

(

1 25 × 36 36 ) ( ) ( ),

Since (

(

and

)=

)=

are dependent.

8 2 = 36 9

2 .2222 9

: Neither die is 2; ( ) =

: Both dice are 1 and neither is 2; (

( ) ( )=

8 36

)=

)=

1 36 are dependent. )=

( )=

1 2

10 5 = 36 18

25 36

1 36

45. The outcome of each coin toss is independent of the others. Therefore, 1 1 1 1 1 ( ) = ( ) ( ) ( ) dots ( ) = 2 2 dots 2 = 11 = . 2 2 2 2 2,048 46. The outcome of each roll of the die is independent of the others. Therefore, 1 1 1 1 1 1 (4, 3, 2, 1) = 2 2 2 = . = 6 6 6 6 6 4 1,296 47. The two events we are interested in are: : A person in the U.S. declared personal bankruptcy. (: A person in the U.S. recently experienced medical-related issues. We are told that: The probability that a person in the U.S. would declare personal bankruptcy was .0016. That is, ( ) = .0016. The probability that a person in the U.S. would declare personal bankruptcy and had recently experienced medical-related issues was .0011. That is, (( ) = .0011. We are asked to find the probability that a person had recently experienced medical-related issues given that they had declared personal bankruptcy. That is, we are asked to find ((| ). (( ) .0011 ((| ) = .69 = ( ) .0016 48. The two events we are interested in are:


Solutions Section 8.5 : A person in the U.S. declared personal bankruptcy. 5: A person in the U.S. had student loans they could not repay. We are told that: The probability that a person in the U.S. would declare personal bankruptcy was .0016. That is, ( ) = .0016. The probability that a person in the U.S. would declare personal bankruptcy and had student loans they could not repay was .0004. That is, (5 ) = .0004. We are asked to find the probability that a person had student loans they could not repay, given that they had declared personal bankruptcy. That is, we are asked to find (5| ). (5 ) .0004 (5| ) = = = .25 ( ) .0016 49. : A home sale took place in the West. 7 : A home sale took place in July 2021. a. We are asked for ( |7). 7 is the event that a home sale took place in the West and in July 2021. 120)6,200 120 120 580 ( 7) = ; (7) = $ ( |7) = .21 = 6,200 6,200 580)6,200 580 b. We are asked for (7| ). 120)6,200 1,300 120 120 ( 7) = ; ( ) = $ (7| ) = .092 = 6,200 6,200 1,300)6,200 1,300

50. : A home sale took place outside the West. 7 : A home sale took place in July 2021. a. We are asked for ( |7). 7 is the event that home sale took place outside the West and in July 2021. (580 120))6,200 580 120 580 120 580 ( 7) = ; (7) = $ ( |7) = .79 = 6,200 6,200 580)6,200 580 b. We are asked for (7| ). 580 120 ( 7) = ; 6,200 (580 120))6,200 6,200 1,300 580 120 ( ) = $ (7| ) = .094 = 6,200 (6,200 1,300))6,200 6,200 1,300

51. Use the following events: : a crash involved speeding. : a crash involved a young driver. We are told that ( ) = .086 We are also told that, of the PDO crashes involving speeding, 23% involved a young driver. Rephrasing this, the probability that the driver was young, given that they were speeding, was 23%. Hence, ( | ) = .23. We are asked for ( ), but we know that ( ) ( | ) = , ( ) so ( ) = ( | ) ( ) = (.23)(.086) .020. Note that the above step is just the multiplication principle.

52. Use the following events: : a crash involved speeding. : a crash involved a young driver. We are told that ( ) = .12 We are also told that, of the crashes involving speeding, 21% involved a young driver. Rephrasing this, the probability that the driver was young, given that they were speeding, was 21%. Hence, ( | ) = .21. We are asked for ( ), but we know that ( ) ( | ) = , ( ) so ( ) = ( | ) ( ) = (.21)(.12) .025. Note that the above step is just the multiplication principle.


Solutions Section 8.5 53. Use the following events: : Involved in daytime crash. :: Involved in crash involving property damage only. We are told that ( :) = .012. We are also told that the probability that a daytime crash would involve only property damage was .72. Rephrasing this, The probability that a crash involves only property damage, given that it occurs in the daytime, is .72: (:| ) = .72. We are asked to find ( ). Start with the formula: ( :) .012 .012 (:| ) = $ .72 = $ .72 ( ) = .012 $ ( ) = .017. ( ) ( ) .72

54. Use the following events: : Involved in nighttime crash. :: Involved in crash involving property damage only. We are told that ( :) = .0050. We are also told that the probability that a nighttime crash would involve only property damage was .69. Rephrasing this, The probability that a crash involves only property damage, given that it occurs in the nighttime, is .69: (:| ) = .69. We are asked to find ( ). Start with the formula: ( :) .0050 .0050 (:| ) = $ .69 = $ .69 ( ) = .0050 $ ( ) = .0072. ( ) ( ) .69 55. The two events are : Agreed that the affordability of health care was a very big problem . ( ) = .56 : Agreed that climate change was a very big problem. ( ) = .40 a. By independence, ( ) = ( ) ( ) = (.56)(.40) .22. b. We want ( ), which we can calculate by ( )= ( )+ ( ) ( ) = ( ) + ( ) ( ) ( ), by independence of and . Thus, ( ) = .56 + .40 (.56)(.40) .74. c. We want ( ). As complements of independent events are also independent, ( ) = ( ) ( ) = (1 .56)(1 .40) .26. Alternatively, notice that the event in part (c) is the complement of the event in part (b): = ( ) , so ( ) = (( ) ) 1 .74 = .26. 56. The two events are : Disagreed that violent crime was a very big problem. ( ) = .52 : Disagreed that domestic terrorism was a very big problem. ( ) = .65 a. By independence, ( ) = ( ) ( ) = (.52)(.65) .34. b. We want ( ), which we can calculate by ( )= ( )+ ( ) ( ) = ( ) + ( ) ( ) ( ), by independence of and . Thus, ( ) = .52 + .65 (.52)(.65) .83. c. We want ( ). As complements of independent events are also independent, ( ) = ( ) ( ) = (1 .52)(1 .65) .17. Alternatively, notice that the event in part (c) is the complement of the event in part (b): = ( ) , so ( ) = (( ) ) 1 .83 = .17. 57. Take : Used Brand X; ( ) = .40; : Gave up doing laundry; ( ) = .05


Solutions Section 8.5 ( ) = .04, ( ) ( ) = .40 × .05 = .02 ( ). Therefore, and are not independent. ( ) .04 ( | ) = = = .1 ( ) .40 which is larger than ( ). Therefore, a user of Brand X is more likely to give up doing laundry than a randomly chosen person.

58. Use the following events: : Used Brand Z computers; ( ) = .60; ;: Quit their job; (;) = .05 ( ;) = .03; ( ) (;) = .60 × .05 = .03 = ( ;). Therefore, and ; are independent. (;| ) = (;) = .05 A user of Brand Z computers is as likely to quit a job as a randomly chosen person. 59. ( | ) =

Total

5

25

30

15

55

70

20

80

100

60. ( | ) =

Total

Total

Total

( ) .25 5 or ( | ) = = = ( ) .80 16

5

25

30

15

55

70

20

80

100

( ) 25 5 = = ( ) 80 16

Total

( ) .15 = = .75 or ( | ) = ( ) .20

5

25

30

15

55

70

20

80

100

62. ( | ) =

( ) 25 5 = = ( ) 30 6

Total

61. ( | ) =

( ) .25 5 or ( | ) = = = ( ) .30 6

( ) 15 = = .75 ( ) 20

Total

( ) .15 3 or ( | ) = = = ( ) .70 14

5

25

30

15

55

70

20

80

100

Total

( ) 15 3 = = ( ) 70 14


Solutions Section 8.5 (% ) .55 11 (% ) 55 11 63. (%| ) = or (%| ) = = = = = .80 16 80 16 ( ) ( )

%

Total

" 5

25

30

15

55

70

20

80

100

64. ( |%) =

%

Total

Total

"

( %) .55 11 ( %) 55 11 or ( |%) = = = = = (%) .70 14 (%) 70 14 Total

5

25

30

15

55

70

20

80

100

65. (An unsuccessful author is established) = ( |%) ( %) .55 11 ( %) 55 11 ( |%) = or ( |%) = = = = = (%) .70 14 (%) 70 14

%

Total

"

Total

5

25

30

15

55

70

20

80

100

66. (An established author is successful) = ( | ) ( ) .25 ( ) .25 5 5 ( | ) = or ( | ) = = = = = ( ) .80 16 ( ) .80 16

%

Total

"

Total

5

25

30

15

55

70

20

80

100

67.

Note that the probability of each outcome was obtained by multiplying the probabilities of the corresponding edges.


Solutions Section 8.5 68.

Note that the probability of each outcome was obtained by multiplying the probabilities of the corresponding edges. 69. Notice that the total number of vehicles adds up to 100, so that the numbers give the probabilities for the first branches of the tree:

Note that the probability of each outcome was obtained by multiplying the probabilities of the corresponding edges. 70.

Note that the probability of each outcome was obtained by multiplying the probabilities of the corresponding edges. 71.

Note that the probability of each outcome was obtained by multiplying the probabilities of the corresponding edges. From the tree, (No rain today or tomorrow) = .25.


Solutions Section 8.5 72.

Note that the probability of each outcome was obtained by multiplying the probabilities of the corresponding edges. From the tree, (Snow by the end of tomorrow) = .04 + .16 + .16 = .36.

73. Use the following events: : Employed; : Bachelor's degree or higher ( ) 57.5 ( | ) = .69. = ( ) 83.5

74. Use the following events: : Employed; : Less than a high school diploma ( ) 7.9 ( | ) = .40. = ( ) 19.9 75. Use the following events: : Bachelor's degree or higher; : Employed ( ) 57.5 ( | ) = .44. = ( ) 130.6

76. Use the following events: : Less than a high school diploma; : Employed ( ) 7.9 ( | ) = .06. = ( ) 130.6 77. Rephrasing the question, we want to find the probability that someone was not in the labor force, given that he or she had not completed a Bachelor's degree or higher. : Not in labor force; : Not completed a Bachelor's degree or higher ( ) 82.3 23.0 ( | ) = .43 = ( ) 222.8 83.5 78. Rephrasing the question, we want to find the probability that someone was not in the labor force, given that he or she had completed at least a high school diploma. : Not in labor force; : Completed at least a high school diploma ( ) 82.3 11.0 ( | ) = .35 = ( ) 222.8 19.9 79. Rephrasing the question, we want to find the probability that someone was employed, given that he or she had completed a Bachelor's degree or higher and was in the labor force. : Employed; : Bachelor's degree or higher and in the labor force ( ) 57.5 ( | ) = .95 = ( ) 83.5 23.0 80. Rephrasing the question, we want to find the probability that someone was employed, given that he or she had completed less than a high school diploma and was in the labor force. : Employed; : Less than a high school diploma and in the labor force ( ) 7.9 ( | ) = .89 = ( ) 19.9 11.0


Solutions Section 8.5 81. We compare (High school diploma only|Unemployed) with (High school diploma only|Employed). 3.1 (High school diploma only|Unemployed) = .32 9.8 31.6 (High school diploma only|Employed) = .24 130.6 Your friend is right: The probability that an unemployed person has a high school diploma only is .32, while the corresponding figure for an employed person is .24. 82. We compare (Less than high school diploma|Not in Labor Force) with (Less than high school diploma|Employed). 11.0 (Less than high school diploma|Not in Labor Force) = .13 82.3 7.9 (Less than high school diploma|Employed) = .06 130.6 Your friend is right: The probability that a person not in the labor force has less than a high school diploma is .13, while the corresponding figure for an employed person is .06. 83. a. We are given ( ||%) = .5 and the efficacy =

.90 =

( ||%) ( ||& ) ( ||%) .5 ( ||& )

= .90. Use the given formula

.5 ' .5 ( |& ) = (.90)(.5) = .45 ' ( |& ) = .5 .45 = .05 b. As .5 is 10 times .05, being unvaccinated makes one ten times as likely to get severe COVID disease as being vaccinated with the Janssen vaccine. 84. a. We are given ( |%) = .8 and the efficacy =

.85 =

( |%) ( |& ) ( ||%) .8 ( |& )

= .85. Use the given formula

.8 ' .8 ( ||& ) = (.85)(.8) = .68 ' ( ||& ) = .8 .68 = .12

.8 20 6.7, being unvaccinated makes one about 6.7 times as likely to get symptomatic COVID = .12 3 disease as being vaccinated with the Pfizer vaccine. b. As

( ||%) ( ||& ) ( ||%) ' ( ||%) = ( ||%) ( ||& ) (Multiply both sides by ( ||%).) ' ( ||& ) = ( ||%) ( ||%) = (1 ) ( ||%). b. For the given situation the efficacy is = .96 and we are told that ( ||%) = 1. Thus, by part (a),

85. a.

=

( ||& ) = (1 ) ( ||%) = (1 .96)(1) = .04,

so the vaccinated person would have only a 4% chance of contracting symptomatic COVID. 86. a. By part (a) of the preceding exercise, ( ||& ) = (1 ) ( ||%). Multiplying both sides by

1 1


1 ( ||& ). 1 b. For the given situation the efficacy is (a),

Solutions Section 8.5

( ||%) =

gives the result:

( ||%) =

1 1

( ||& ) =

= .90 and we are told that ( ||& ) = 1 50 = .02. Thus, by part

1 (.02) = .2, .1

a 1 in 5 chance.

87. If and & are independent, then by the hint, so are and %, so ( |%) = ( |& ) = ( ), and =

( |%) ( |& ) ( ) ( ) = = 0. ( ) ( ||%)

88. Given that ( ||%) = =

4 ( ||& ), we get 3

4 4 ( ||%) ( ||& ) 3 ( |& ) ( |& ) 3 1 1 = = = . 4 4 4 ( |%) ( ||& ) 3

3

An easier calculation can be obained by using the result of Exercise 86(a). 89. From the table, the probability that an Infiniti Q50 was reported stolen was .00139. In other words, The probability that a vehicle was reported stolen given that it was an Infiniti Q50 was .00139; ((|$) = .00139. 90. From the table, the probability that a Dodge Charger was reported stolen was .000986. In other words, The probability that a vehicle was reported stolen given that it was a Dodge Charger was .000986. ((| 2) = .000986 91. From the table, the probability that a Chrysler 300 was reported stolen was .000781. In other words, The probability that a vehicle was reported stolen given that it was a Chrysler 300 was .000781. This is exactly what Choice (D) says. 92. From the table, ((| 1) = .00101. Therefore, (( | 1) = 1 .00101 = .99899 (Choice B).

93. Consider the following events: ": My Nissan Maxima will be stolen; (") = .000938. : My Dodge Durango will be stolen; ( ) = .000728. a. As the events are independent, (" ) = (") ( ) = .000938 × .000728 .000000683. b. (" ) = (") + ( ) (" ) = .000938 + .000728 .000000683 .00167 94. Consider the following events: 1: My Dodge Challenger was stolen; ( 1) = .00101. : My Chevrolet Silverado 1500 was stolen; ( ) = .000757. a. Since the events are independent, ( 1 ) = ( 1 ) ( ) = (1 .00101) × .000757 .000756. b. From part (a), ( 1 ) = .000756, whereas ( ) = .000757. Therefore, the probability that my Silverado was stolen is greater than the probability that only my Silverado was stolen.


Solutions Section 8.5 95. Use the following events: & : Tests positive; %: Use steroids. (% & ) (%|& ) = .90, (% & ) = .10; (%|& ) = (& ) .10 .10 .90 = .90 (& ) = .10 (& ) = .11 or 11% (& ) .90

96. Use the following events: : Pass fitness test; : candidate for the soccer team. ( ) ( | ) = .80, ( ) = .20; ( | ) = ( ) .20 .20 .80 = .80 ( ) = .20. (& ) = .25 or 25% ( ) .80

97. Use the following events: : Contaminated by Salmonella; : Contaminated by a strain of Salmonella resistant to at least three antibiotics. We are told that ( ) = .038, and also that the probability that a Salmonella-contaminated sample was contaminated by a strain resistant to at least three antibiotics was .419. Rewriting this gives: The probability that a sample was contaminated by a strain resistant to at least three antibiotics given that it was contaminated by Salmonella is .419: ( | ) = .419. We are asked to find ( ) (which is actually the same as ( )), so we use the formula: ( ) ( ) ( | ) = .419 = ( ) .038 and obtain ( ) = .419 × .038 .016.

98. Use the following events: : Contaminated by Salmonella; : Contaminated by a strain of Salmonella resistant to at least one antibiotic. We are told that ( ) = .038, and also that a Salmonella-contaminated sample was contaminated by a strain resistant to at least one antibiotic was .819. Rewriting this gives: The probability that a sample was contaminated by a strain resistant to at least one antibiotic given that it was contaminated by Salmonella is .819: ( | ) = .819. We are asked to find ( ) (which is actually the same as ( )), so we use the formula: ( ) ( ) ( | ) = .819 = ( ) .038 and obtain ( ) = .819 × .038 .031.

99. Take : Contaminated by Salmonella 1: Contaminated by a strain of Salmonella resistant to at least one antibiotic 3: Contaminated by a strain of Salmonella resistant to at least three antibiotics We are told: ( ) = .038, ( 1| ) = .819, ( 3| ) = .419. We are asked to find ( 3| 1). ( 3 1) ( 3) ( 3 ) ( 3| 1) = = = ( 1) ( 1) ( 1 ) because 3 1 = 3 = 3 , and 1 = 1 . But we have ( 1 ) ( 1| ) = = .819 ( ) ( 3 ) ( 3| ) = = .419. ( ) Taking their ratio gives ( 3 ) .419 .512. = ( 1 ) .819 Therefore,


( 3 ) ( 3| 1) = .512. ( 1 )

Solutions Section 8.5

100. We obtain the following tree:

From the tree, the probability that someone eating a randomly chosen chicken sample will not become seriously ill with a strain of Salmonella resistant to at least two antibiotics is .0225 + .975 .998.

101. Here is the given information with the event names as in the text, and with the unknown used for the event as shown: Purchased Game ( )

Saw Ad ( ) Did Not See Ad ( )

Did Not Purchase Game ( ) Total

20

40

180

40 +

200

Total 60

180 +

240 +

As the survey showed the ad had no effect on on sales, the estimated probabilities ( ) and ( | ) are equal. So, 60 20 1 = = 240 + 200 10 Cross-multiplying gives 600 = 240 + )) )) = 360. Thus, the entire table is Saw Ad Did Not See Ad Total Purchased Game

20

40

60

Did Not Purchase Game

180

360

540

Total

200

400

600

102. Here is the given information with the event names as in the text, and with the unknown used for the event as shown: Purchased Game ( )

Did Not Purchase Game ( ) Total

Saw Ad ( ) Did Not See Ad ( ) 20

60

20 +

180

240

Total 80

180 + 260 +

As the survey showed the ad had no effect on on sales, the estimated probabilities ( ) and ( | ) are equal. So,


80 20 = 260 + 20 + Cross-multiplying gives 1,600 + 80 = 5,200 + 20 )) Thus, the entire table is

Solutions Section 8.5 ))60 = 3,600))

)) = 60.

Saw Ad Did Not See Ad Total Purchased Game

20

60

80

Did Not Purchase Game

60

180

240

Total

80

240

320

103. Let be the event that someone misspells "Waner", and let be the event that someone misspells "Costenoble." As these events are independent, the probability of misspelling both names is ( ) = ( ) ( ) = * × * = * 2 Thus the percentage of people who misspell both names is expected to be 100* 2 percent. 104. Let be the event that misspells "Waner", and let be the event that someone misspells "Costenoble." Then ( ) = 1 * and ( ) = 1 +. As these events are independent, the probability of misspelling both names is ( ) = ( ) ( ) = (1 *)(1 +).

105. Answers will vary. Here is a simple one: : The first toss is a head, : The second toss is a head, : The third toss is a head. 106. Answers will vary. Here are some examples: (a) The first die shows a 1 and the second die shows a 2. (b) The first die is even and the second die is odd. (c) The dice add up to 7 and the first die shows a 1.

107. The probability you seek is ( | ), or should be. If, for example, you were going to place a wager on whether occurs or not, it is crucial to know that the sample space has been reduced to . (You know that did occur.) If you base your wager on ( ) rather than ( | ), you will misjudge your likelihood of winning. 108. Suppose that, in fact, 100,000 people saw the ad and only 3,000 did not. Then the probability that someone bought your product, given that he or she saw your ad, is only 1/10, while the probability that someone bought your product, given that he or she did not see your ad, is 2/3. You have a better probability of selling your product to someone who did not see your ad than to someone who did!

109. You might explain that the conditional probability of is not the a priori probability of , but it is the probability of in a hypothetical world in which the outcomes are restricted to be what is given. In the example she is citing, yes, the probability of throwing a double-six is 1/36 in the absence of any other knowledge. However, by the "conditional probability" of throwing a double-six given that the sum is larger than 7, we might mean the probability of a double-six in a situation in which the dice have already been thrown, but all we know is that the sum is greater than 7. Since there are only 15 ways in which that can happen, the conditional probability is 1/15. For a more extreme case, consider the conditional probability of throwing a double-six given that the sum is 12. 110. He is wrong. Here is an example which proves him wrong: In an experiment in which a coin is tossed, take to be the event that you get heads, and the event that you get tails. Then ( ) = .5, whereas ( | ) = 0.


Solutions Section 8.5

111. If , , then = , so ( ) = ( ) and ( | ) = 112. If , , then = , so ( ) = ( ) and ( | ) =

( ) ( ) . = ( ) ( )

( ) ( ) = = 1. ( ) ( )

113. Your friend is correct. If and are mutually exclusive, then ( ) = 0. On the other hand, if and are independent, then ( ) = ( ) ( ). Thus, ( ) ( ) = 0. If a product is 0, then one of the factors must be 0, so either ( ) = 0 or ( ) = 0. Thus, it cannot be true that and are mutually exclusive, have nonzero probabilities, and are independent all at the same time.

114. Your friend is wrong. If and are mutually exclusive, then ( ) = 0. On the other hand, if and are not dependent, then ( ) = ( ) ( ). Thus, ( ) ( ) = 0, meaning that either or has zero probability, contrary to the assumption. 115. Suppose that and are independent, so that ( ) = ( ) ( ). Then ( ) = 1 ( ) = 1 [ ( ) + ( ) ( )] = 1 [ ( ) + ( ) ( ) ( )] [since ( ) = ( ) ( )] = (1 ( ))(1 ( )) = ( ) ( ). Therefore, ( ) = ( ) ( ), and so and are independent. 116. If and are independent, then ( | ) = ( ) and also ( | ) + ( | ) = 1. Therefore, ( | ) = 1 ( | ) = 1 ( ) = ( ), and so and are independent.


Solutions Section 8.6 Section 8.6

1. ( | ) = .8, ( ) = .2, ( | ) = .3, ( ) = 1 ( ) = 1 .2 = .8 ( | ) ( ) (.8)(.2) .16 ( | )= = = = .4 ( | ) ( ) + ( | ) ( ) (.8)(.2) + (.3)(.8) .16 + .24

2. ( | ) = .6, ( ) = .3, ( | ) = .5, ( ) = 1 ( ) = 1 .3 = .7 ( | ) ( ) (.6)(.3) .18 ( | )= .3396 = = ( | ) ( ) + ( | ) ( ) (.6)(.3) + (.5)(.7) .18 + .35

3. ( | ) = .8, ( ) = .3, ( | ) = .5, ( ) = 1 ( ) = 1 .3 = .7, ( | ) ( ) (.8)(.7) .56 ( | ) = .7887 = = ( | ) ( ) + ( | ) ( ) (.8)(.7) + (.5)(.3) .56 + .15 4. ( | ) = .6, ( ) = .4, ( | ) = .3, ( ) = 1 ( ) = 1 .4 = .6 ( | ) ( ) (.6)(.6) .36 ( | ) = = = = .75 ( | ) ( ) + ( | ) ( ) (.6)(.6) + (.3)(.4) .36 + .12

5. ( | ) = .6, ( ) = .3, ( | ) = .5, ( ) = 1 ( ) = 1 .3 = .7 ( | ) ( ) ( | ) = ( | ) ( ) + ( | ) ( ) We need ( | ) and ( | ). By the hint, they are ( | ) = 1 ( | ) = 1 .6 = .4 and ( | ) = 1 ( | ) = 1 .5 = .5. so, ( | ) ( ) (.4)(.3) .12 ( | ) = .2553 = = ( | ) ( ) + ( | ) ( ) (.4)(.3) + (.5)(.7) .12 + .35

6. ( | ) = .8, ( ) = .3, ( | ) = .4, ( ) = 1 ( ) = 1 .3 = .7, ( | ) ( ) ( | ) = ( | ) ( ) + ( | ) ( ) We need ( | ) and ( | ). By the hint, they are ( | ) = 1 ( | ) = 1 .8 = .2 and ( | ) = 1 ( | ) = 1 .4 = .6. so, ( | ) ( ) (.2)(.7) .14 ( | ) = = = = .4375 ( | ) ( ) + ( | ) ( ) (.2)(.7) + (.6)(.3) .14 + .18

7. ( | 1 ) = .4, ( | 2 ) = .5, ( | 3 ) = .6, ( 1 ) = .8, ( 2 ) = .1, ( 3 ) = 1 (.8 + .1) = .1 ( | 1 ) ( 1 ) (.4)(.8) ( 1 | ) = = ( | 1 ) ( 1 ) + ( | 2 ) ( 2 ) + ( | 3 ) ( 3 ) (.4)(.8) + (.5)(.1) + (.6)(.1) .32 .7442 = .32 + .05 + .06 8. ( | 1 ) = .2, ( | 2 ) = .3, ( | 3 ) = .6, ( 1 ) = .3, ( 2 ) = .4, ( 3 ) = 1 (.3 + .4) = .3 ( | 1 ) ( 1 ) (.2)(.3) ( 1 | ) = = ( | 1 ) ( 1 ) + ( | 2 ) ( 2 ) + ( | 3 ) ( 3 ) (.2)(.3) + (.3)(.4) + (.6)(.3) .06 .1667 = .06 + .12 + .18 9. ( | 1 ) = .4, ( | 2 ) = .5, ( | 3 ) = .6, ( 1 ) = .8, ( 2 ) = .1, ( 3 ) = 1 (.8 + .1) = .1


=

( 2 | ) =

Solutions Section 8.6 ( | 2 ) ( 2 ) (.5)(.1) = ( | 1 ) ( 1 ) + ( | 2 ) ( 2 ) + ( | 3 ) ( 3 ) (.4)(.8) + (.5)(.1) + (.6)(.1)

.05 .1163 .32 + .05 + .06

10. ( | 1 ) = .2, ( | 2 ) = .3, ( | 3 ) = .6, ( 1 ) = .3, ( 2 ) = .4 ( 3 ) = 1 (.3 + .4) = .3 ( | 2 ) ( 2 ) (.3)(.4) ( 2 | ) = = ( | 1 ) ( 1 ) + ( | 2 ) ( 2 ) + ( | 3 ) ( 3 ) (.2)(.3) + (.3)(.4) + (.6)(.3) .12 .3333 = .06 + .12 + .18 11. Use the following events: : Decreased spending on music; : Internet user ( | ) = .11, ( ) = .40, ( | ) = .2. We are asked to compute ( | ). ( | ) ( ) (.11)(.4) .044 ( | ) = .27, or 27% = = ( | ) ( ) + ( | ) ( ) (.11)(.4) + (.2)(.6) .044 + .12

12. Use the following events: : Decreased spending on music : Experienced user ( | ) = .36, ( ) = .03, ( | ) = .2. We are asked to compute ( | ). ( | ) ( ) (.36)(.03) .0108 ( | ) = .05, or 5% = = ( | ) ( ) + ( | ) ( ) (.36)(.03) + (.2)(.97) .0108 + .194 13. Use the following events: : Snows in Greenland; : Glaciers grow; 1 ( )= = .04, ( | ) = .20, ( | ) = .04. We are asked to compute ( | ). 25 ( | ) ( ) (.2)(.04) .008 ( | )= .17 = = ( | ) ( ) + ( | ) ( ) (.2)(.04) + (.04)(.96) .008 + .0384 14. Use the following events: ': Rains in Spain; : Hurricanes happen in Hartford. 1 (') = = .1, ( |') = .02, ( |' ) = .01. We are asked to compute ('| ). 10 ( |') (') (.02)(.1) .002 ('| ) = .18 = = ( |') (') + ( |' ) (' ) (.02)(.1) + (.01)(.9) .002 + .009

15. We need to calculate ( | ), the probability that an asymptomatic person is infected given that they test negative. Bayes' theorem states that ( | ) =

( | ) ( ) ( | ) ( ) + ( | ) ( )

From the table and the given information,

( ) = 1)100 = .01, so ( ) = 1 ( ) = .99 ( | ) = .58, == ( | ) = .011, so ( | ) = 1 .58 = .42, == ( | ) = 1 .011 = .989 ( | ) ( ) (.42)(.01) ( | ) = .0043 = ( | ) ( ) + ( | ) ( ) (.42)(.01) + (.989)(.99)

16. We need to calculate ( | ), the probability that an asymptomatic person is not infected given that they test positive. Bayes' theorem states that ( | ) =

( | ) ( ) ( | ) ( ) + ( | ) ( )


Solutions Section 8.6 From the table and the given information,

( ) = 1)500 = .002, so ( ) = 1 ( ) = .998 ( | ) = .58, == ( | ) = .011 ( | ) ( ) (.011)(.998) ( | ) = .90 = ( | ) ( ) + ( | ) ( ) (.011)(.998) + (.58)(.002)

17. We need to calculate ( | ), the probability that a symptomatic person is infected given that they test positive. Bayes' theorem states that ( | ) =

( | ) ( ) ( | ) ( ) + ( | ) ( )

From the table and the given information,

( ) = 1)100 = .01, so ( ) = 1 ( ) = .99 ( | ) = .72, == ( | ) = .005 ( | ) ( ) (.72)(.01) ( | ) = .59 = ( | ) ( ) + ( | ) ( ) (.72)(.01) + (.005)(.99)

18. We need to calculate ( | ), the probability that a symptomatic person is infected given that they test negative. Bayes' theorem states that ( | ) =

( | ) ( ) ( | ) ( ) + ( | ) ( )

From the table and the given information,

( ) = 1)40 = .025, so ( ) = 1 ( ) = .975 ( | ) = .72, == ( | ) = .005, so ( | ) = 1 .72 = .28, == ( | ) = 1 .005 = .995 ( | ) ( ) (.28)(.025) ( | ) = .0072 = ( | ) ( ) + ( | ) ( ) (.28)(.025) + (.995)(.975)

19. We need to calculate ( ) given that ( | ) is .5. Thus, we take ( ) = % and use Bayes' theorem with the asymptomatic figures: ( | ) ( ) ( | ) ( ) + ( | ) ( ) .42% .5 = .42% + .989(1 %) .5(.42% + .989(1 %)) = .42% .21% + .4945 .4945% = .42% .7045% = .4945 .4945 %= 0.7019 .7045 ( | ) =

Rounding down to .70 would result in less than .5 for the probability of being infected, so we round up and conclude that ( ) would need to be at least .71, meaning that at least 71% of the population would need to be infected.

20. We need to calculate ( ) given that ( | ) is .5. Thus, we take ( ) = % and use Bayes' theorem with the symptomatic figures:


( | ) ( ) ( | ) ( ) + ( | ) ( ) .28% .5 = .28% + .995(1 %) .5(.28% + .995(1 %)) = .28% .14% + .4975 .4975% = .28% .6375% = .4975 .4975 %= .780392 .6375 ( | ) =

Solutions Section 8.6

Rounding down to .78 would result in less than .5 for the probability of being infected, so we round up and conclude that ( ) would need to be at least .79, meaning that at least 79% of the population would need to be infected. 21. Use the following events: : a crash involved speeding. : a crash involved a young driver. ( ) = .086, ( ) = 1 ( ) = .914, ( | ) = .23, ( | ) = .19. We are asked to compute ( | ). ( | ) ( ) (.23)(.086) ( | ) = .10 = ( | ) ( ) + ( | ) ( ) (.23)(.086) + (.19)(.914) 22. Use the following events: : a crash involved speeding. : a crash involved a young driver. ( ) = .12, ( ) = 1 ( ) = .88, ( | ) = .21, ( | ) = .19. We are asked to compute ( | ). ( | ) ( ) (.21)(.12) ( | ) = .13 = ( | ) ( ) + ( | ) ( ) (.21)(.12) + (.19)(.88) 23. Use the following events: : Collision with another vehicle; : Crash is fatal. We are given ( ) = .718. Also, the information in the table tells us that ( | ) = .0027, ( | ) = .0105. We are asked to find ( | ). Bayes' theorem states ( | ) ( ) (.0027)(.718) ( | ) = .40. = ( | ) ( ) + ( | ) ( ) (.0027)(.718) + (.0105)(1 .718)

24. Use the following events: : Collision with another vehicle; : High-speed crash is fatal. We are given ( ) = .576. Also, the information in the table tells us that ( | ) = .0086, ( | ) = .0195. We are asked to find ( | ). Bayes' theorem states ( | ) ( ) (.0086)(.576) ( | ) = .37. = ( | ) ( ) + ( | ) ( ) (.0086)(.576) + (.0195)(1 .576) 25. Use the following events: : Fit enough to play; : Failed the fitness test and therefore dropped from the team. ( | ) = .5, ( | ) = 1, ( ) = .45, so ( ) = .55 We are asked to compute the probability that Mona was justifiably dropped, which is ( | ). ( | ) ( ) (1)(.55) .55 ( | ) = .71 = = ( | ) ( ) + ( | ) ( ) (.5)(.45) + (1)(.55) .225 + .55

26. Use the following events: : Fail the test; : Do not go on to a career in physics. ( | ) = 1, ( | ) = .6, ( ) = .75, so ( ) = .25 We are asked to compute ( | ). ( | ) ( ) (1)(.25) .25 ( | ) = .36 = = ( | ) ( ) + ( | ) ( ) (1)(.25) + (.6)(.75) .25 + .45


Solutions Section 8.6 27. Use the following events: : Head-on collision; : Not involving another vehicle; :: Other type of collision; : Crash is fatal. We are given ( ) = .0267, ( ) = .282, so (:) = 1 .0267 .282 = .6913. Also, the information in the table tells us that ( | ) = .0200, ( | ) = .0105, ( |:) = .0020. We are asked to find the probability that a crash was a head-on collision, given that it was fatal, that is, ( | ). Bayes' theorem tells us ( | ) ( ) ( | ) = ( | ) ( ) + ( | ) ( ) + ( |:) (:) (.0200)(.0267) .11. = (.0200)(.0267) + (.0105)(.282) + (.0020)(.6913) 28. Use the following events: : Head-on collision; : Not involving another vehicle; :: Other type of collision; : Crash is fatal. We are given ( ) = .0156, ( ) = .424, so (:) = 1 .0156 .424 = .5604. Also, the information in the table tells us that ( | ) = .0574, ( | ) = .0195, ( |:) = .0073. We are asked to find the probability that a crash was a head-on collision, given that it was fatal, that is, ( | ). Bayes' theorem tells us ( | ) ( ) ( | ) = ( | ) ( ) + ( | ) ( ) + ( |:) (:) (.0574)(.0156) .07. = (.0574)(.0156) + (.0195)(.424) + (.0073)(.5604) 29. Use the following events: : Admitted into UCLA; : California applicant; : Applicant from another U.S. state; : International applicant. ( | ) = .12, ( | ) = .16, ( | ) = .08, ( ) = .62, ( ) = .21, ( ) = .17 We are asked to compute ( | ). ( | ) ( ) ( | ) = ( | ) ( ) + ( | ) ( ) + ( | ) ( ) (.12)(.62) .61, = (.12)(.62) + (.16)(.21) + (.08)(.17) or approximately 61% 30. Use the following events: : Admitted into UCLA; : California applicant; : Applicant from another U.S. state; : International applicant. ( | ) = .26, ( | ) = .18, ( | ) = .13, ( ) = .86, ( ) = .11, ( ) = .03 We are asked to compute ( | ). ( | ) ( ) ( | ) = ( | ) ( ) + ( | ) ( ) + ( | ) ( ) (.26)(.86) .9042, = (.26)(.86) + (.18)(.11) + (.13)(.03) or approximately 90%

31. Use the following events: : Used e-mail; : Age 15–24; : Age 25–44; : Age 45–64; : 65 or older. ( | ) = .684, ( | ) = .717, ( | ) = .608, ( | ) = .403, ( ) = .172, ( ) = .327, ( ) = .328, ( ) = 1 (.172 + .327 + .328) = .173 We are asked to compute ( | ). ( | ) ( ) ( | ) = ( | ) ( ) + ( | ) ( ) + ( | ) ( ) + ( | ) ( ) (.684)(.172) = (.684)(.172) + (.717)(.327) + (.608)(.328) + (.403)(.173)


.19, or approximately 19%

Solutions Section 8.6

32. Use the following events: : Used e-mail; : Age 15–24; : Age 25–44; : Age 45–64; : 65 or older. ( | ) = .761, ( | ) = .805, ( | ) = .743, ( | ) = .581, ( ) = .160, ( ) = .327, ( ) = .313, ( ) = 1 (.160 + .327 + .313) = .200 We are asked to compute ( | ). ( | ) ( ) ( | ) = ( | ) ( ) + ( | ) ( ) + ( | ) ( ) + ( | ) ( ) (.581)(.200) = (.761)(.160) + (.805)(.327) + (.743)(.313) + (.581)(.200) .16, or approximately 16%

33. Use the following events: : Afoul of the law; : No preschool education. ( | ) = .55, ( | ) = .36. a. ( ) = .20, ( ) = 1 .20 = .80 ( | ) ( ) (.55)(.8) ( | ) = .86 or 86% = ( | ) ( ) + ( | ) ( ) (.55)(.8) + (.36)(.2) b. Changing ( ) to .80 gives ( | ) ( ) (.55)(.2) ( | ) = .28, or 28%. = ( | ) ( ) + ( | ) ( ) (.55)(.2) + (.36)(.8) c. No; part (b) applies to a community where only 20% had not had preschool, but a larger percentage, 28%, of those who ran afoul of the law had not had preschool. In other words, the set of people who had no preschool is over-represented among those who ran afoul of the law.

34. Use the following events: : Earning at least $20,000 by age 40; : Attended preschool. ( | ) = .60, ( | ) = .40 a. ( ) = .90, ( ) = 1 .90 = .10 ( | ) ( ) (.60)(.90) ( | ) = .93, or 93% = ( | ) ( ) + ( | ) ( ) (.60)(.90) + (.40)(.10) b. Changing ( ) to .10 gives ( | ) ( ) (.60)(.10) ( | ) = .14, or 14% = ( | ) ( ) + ( | ) ( ) (.60)(.10) + (.40)(.90) c. Only 14% of those earning at least $20,000 had gone to preschool, but then again only 10% went to preschool. So those who did go to preschool are over-represented in the group earning at least $20,000, thus it was beneficial to them to have done so. So no, children were not better off not attending preschool. 35. Use the following events: : Former student of Prof. A.; : Earned C− or lower. All of Professor A's former students wound up with a C− or lower. In other words, ( | ) = 1.

1 . 3 Three quarters of Professor F's class consisted of former students of Professor A, so ( ) = .75. We are asked to find what percentage of the students who got C− or worse were former students of Prof. A: ( | ). ( | ) ( ) (1)(.75) ( | ) = = = .9 ( | ) ( ) + ( | ) ( ) (1)(.75) + (1)3)(.25) Therefore, we estimate that 9 of the 10 students in the delegation were former students of Prof. A. Two thirds of students not from Prof. A's class got better than a C−. In other words, ( |

36. Use the following events: ': It rains; : Sagittarius is in the shadow of Jupiter. 1 4 1 ( |') = , (') = = .08, ( |' ) = = .2 3 50 5

)=


Solutions Section 8.6 (1)3)(.08) .08)3 ( |') (') ('| ) = .1266 = = ( |') (') + ( |' ) (' ) (1)3)(.08) + (.2)(.92) .08)3 + .184 Therefore, she is right approximately 13% of the time. Replace her.

37. Use the following events: : Husband employed; : wife employed. ( ) = .95, ( | ) = .71 Since either the husband or wife in a couple with earnings had to be employed, ( | ) = 1. We are asked to find ( | ). ( | ) ( ) (.71)(.95) .6745 ( | ) = .93 = = ( | ) ( ) + ( | ) ( ) (.71)(.95) + (1)(.05) .6745 + .05 38. Use the following events: : Husband employed; : wife employed. ( ) = .90, ( | ) = .70 Since either the husband or wife in a couple with earnings had to be employed, ( | ) = 1. We are asked to find ( | ). ( | ) ( ) (.70)(.90) ( | ) = .86 = ( | ) ( ) + ( | ) ( ) (.70)(.90) + (1)(1 .90) 39. Use the following events: : Arrested by age 14; : Become a chronic offender. ( | ) = 17.9 ( | ), ( ) = .001 ( | ) ( ) 17.9 ( | )(.001) ( | ) = = ( | ) ( ) + ( | ) ( ) 17.9 ( | )(.001) + ( | )(.999) Now cancel the ( | ) to get (17.9)(.001) .0179 .0176, or 1.76%. = (17.9)(.001) + .999 .0179 + .999

40. Use the following events: : Become a chronic offender; 3 : Commit violent crime. (3 | ) = 14.3 (3 | ), ( ) = .002 (3 | ) ( ) 14.3 (3 | )(.002) ( |3 ) = = (3 | ) ( ) + (3 | ) ( ) 14.3 (3 | )(.002) + (3 | )(.998) Now cancel the (3 | ) to get (14.3)(.002) .0286 .03. = (14.3)(.002) + .998 .0286 + .998 41. Use the following events: : has diabetes; : very active. 1 ( | ) = .5 ( | ), ( ) = 3 ( | ) ( ) .5 ( | ) ( ) ( | ) = = ( | ) ( ) + ( | ) ( ) .5 ( | ) ( ) + ( | ) ( Cancel the ( | ) to obtain (.5)(1)3) 1)6 .5 ( ) = = = .2. .5 ( ) + ( ) (.5)(1)3) + 2)3 1)6 + 2)3 42. Use the following events: : has diabetes; : very active. ( | ) = .5 ( | ), ( ) = .1 ( | ) ( ) .5 ( | ) ( ) ( | ) = = ( | ) ( ) + ( | ) ( ) .5 ( | ) ( ) + ( | ) ( Cancel the ( | ) to obtain .5 ( ) (.5)(.1) .05 .0526. = = .5 ( ) + ( ) (.5)(.1) + .9 .05 + .9

)

)


Solutions Section 8.6 43. Use the following events: @: The child was killed; : The air bag deployed. (@| ) = 1.31 (@| ), ( ) = .25 We are asked to compute ( |@). (@| ) ( ) 1.31 (@| ) ( ) ( |@) = = (@| ) ( ) + (@| ) ( ) 1.31 (@| ) ( ) + (@| ) ( ) Cancel the terms (@| ) to obtain 1.31 ( ) (1.31)(.25) .3275 .30. = = 1.31 ( ) + ( ) (1.31)(.25) + .75 .3275 + .75 44. Use the following events: @: The child was killed; : The air bag deployed. (@| ) = 1.84 (@| ), ( ) = .25 We are asked to compute ( |@). (@| ) ( ) 1.84 (@| ) ( ) ( |@) = = (@| ) ( ) + (@| ) ( ) 1.84 (@| ) ( ) + (@| ) ( ) Cancel the terms (@| ) to obtain 1.84 ( ) (1.84)(.25) .46 .38. = = 1.84 ( ) + ( ) (1.84)(.25) + .75 .46 + .75

45. Show him an example such as Example 1 of this section, where ( | ) = .95 but ( | ) .64. 46. To use Bayes' formula to compute ( | ), you need to be given P(F|E), P(E), and P(F|E ).

47. Suppose that the steroid test gives 10% false negatives and that only 0.1% of the tested population uses steroids. Then the probability that an athlete uses steroids, given that he or she has tested positive, is (0.9)(0.001) .083. (0.9)(0.001) + (0.01)(0.999) 48. Suppose that the steroid test gives 10% false negatives and that 80% of the tested population uses steroids. Then the probability that an athlete uses steroids, given that he or she has tested positive, is (.9)(.8) .92. (.9)(.8) + (.3)(.2) 49. Draw a tree in which the first branching shows which of '1 , '2 , or '3 occurred and the second branching shows which of or then occurred. There are three final outcomes in which occurs: ('1 ) = ( |'1 ) ('1 ) ('2 ) = ( |'2 ) ('2 ) ('3 ) = ( |'3 ) ('3 ). In only one of these, the first, does '1 occur. Thus, ('1 ) ( |'1 ) ('1 ) ('1 | ) = . = ( ) ( |'1 ) ('1 ) + ( |'2 ) ('2 ) + ( |'3 ) ('3 ) 50. ('1 | ) =

( |'1 ) ('1 ) ( |'1 ) ('1 ) + ( |'2 ) ('2 ) + ( |'3 ) ('3 ) + ( |'4 ) ('4 )

51. The reasoning is flawed. Let be the event that a Democrat agrees with Safire's column, and let and ( be the events that a Democrat reader is female and male, respectively. Then A. D. makes the following argument: ((| ) = .9, ( | ) = .9. Therefore,


Solutions Section 8.6 ( |() = .9. According to Bayes' theorem, we cannot conclude anything about ( |() unless we know ( ), the percentage of all Democrats who agreed with Safire's column. This was not given.

52. Take: : Agreed with Safire's column; : The Democrat reader is female; (: The Democrat reader is male. ((| ) = .9, ( | ) = .1, so ((| ) = 1 .1 = .9. Also, ( |() = .9. We are asked to find ( ). Using Bayes' theorem, ((| ) ( ) ( |() = ((| ) ( ) + ((| ) ( ) .9 ( ) .9 = . .9 ( ) + .9(1 ( )) Canceling the .9s on the right gives ( ) .9 = = ( ). So, ( ) = .9, or 90%. ( ) +1 ( )


Solutions Section 8.7 Section 8.7 1. The transition matrix is organized as follows: 1A1 1A2 . = 2A1 2A2 Therefore, the diagram

has

=

1)4 3)4 1)2 1)2

has .

3. The transition matrix is organized as follows: 1A1 1A2 . = 2A1 2A2 Therefore, the diagram

has

5.

=

0

1

1)6 5)6

2. The transition matrix is organized as follows: 1A1 1A2 . = 2A1 2A2 Therefore, the diagram

1)2 1)2 1)6 5)6

.

=

0 1 1 0

Organization

=

.

4. The transition matrix is organized as follows: 1A1 1A2 . = 2A1 2A2 Therefore, the diagram

has

Diagram

=

1A1 1A2 1A3 2A1 2A2 2A3 3A1 3A2 3A3

.

Transition Matrix

=

0 .8 .2

.9 0 .1 0

0

1


Solutions Section 8.7 6.

Diagram

Organization 1A1 1A2 1A3

=

7.

2A1 2A2 2A3 3A1 3A2 3A3

Diagram

Organization

1A1 1A2 1A3

=

2A1 2A2 2A3 3A1 3A2 3A3

9. The diagram

1

has

=

2)3

0

0

0

2)3

0

0

0 0

0 0

0

1)3

0

0

0

1)3

0

0

2)3 0

0

2)3

0

0

0

1)3 0

1

0

0

0

0

1)3 0

.

.5 0 .5

=

3A1 3A2 3A3

Organization

.9 0 .1

Transition Matrix

2A1 2A2 2A3

Diagram

.1 .8 .1

=

1A1 1A2 1A3

=

8.

Transition Matrix

1 0 0 0 1 0 0 0 1

Transition Matrix

=

0 1 0 0 0 1 1 0 0


Solutions Section 8.7 10. The diagram

has

11. a.

1

2)3

=

0 0 0

2

=

0

0

0

0

1)3

0

0

1)2 1)2 0

1

0

2)3 1)3 0

0

0

0

.5 .5 0

0

0 .

1

.5 .5 1

0

=

.25 .75

b. Distribution after one step: B = After two steps: B

2

After three steps: B

12. a.

2

=

1

=

3

0

= 1

.5 .5

.5 .5

0

.25 .75 0

.5 .5

=

2

After three steps: B

13. a.

2

=

=

3

.2 .8

=

.5 .5

1

.4 .6

1

.4 .6

=

After three steps: B

2

=

3

=

.3 .7

0

1

0 1

.5 .5 1

0

.5 .5

.32 .68

.5 .5

.2 .8 .4 .6

.34 .66

1

= .5 .5

= .125 .875

1

0

.5 .5

= .5 .5

= .75 .25

.36 .64

b. Distribution after one step: B = After two steps: B

.5 .5

0

0

= .25 .75

1

0

.5 .5

0

.75 .25

.75 .25

.2 .8

1

1

.5 .5

b. Distribution after one step: B = After two steps: B

0

= .875 .125

.2 .8 .4 .6

= .34 .66

.2 .8 .4 .6

= .3 .7

= .332 .668


14. a.

2

=

1)3 2)3 1)2 1)2

1)3 2)3 1)2 1)2

After two steps: B

2

After three steps: B

15. a.

2

=

=

3

=

1)2 1)2 1

0

=

B =

b. Distribution after one step:

4)9

11)24 13)24

5)12 7)12 1)4 3)4

1)3 2)3

1

0

b. Distribution after one step: B =

=

1)3 2)3 1)2 1)2

= 11)24 13)24

= 61)144 83)144

1)2 1)2

61)144 83)144 1)2 1)2

5)9

Solutions Section 8.7

1)3 2)3 1)2 1)2

= 371)864 493)864

3)4 1)4 1)2 1)2

1)2 1)2

2)3 1)3

1

0

= 2)3 1)3

After two and three steps, the result will remain the same: 2)3 1)3 . 16. a.

2

=

0

1

1)4 3)4

0

1

1)4 3)4

b. Distribution after one step: B =

=

1)4

3)4

3)16 13)16 0

1)5 4)5

1

1)4 3)4

= 1)5 4)5

After two and three steps, the result will remain the same: 1)5 4)5 . 17. a.

2

=

3)4 1)4 3)4 1)4

3)4 1)4 3)4 1)4

b. Distribution after one step: B = After two steps: B

=

3)4 1)4

2)3 1)3

2)3 1)3

2

=

3)4 1)4 3)4 1)4

3)4 1)4

1)2 1)2

3)4 1)4

3)4 1)4

= 3)4 1)4

= 3)4 1)4

3)4 1)4

After three steps, the result will remain the same: 3)4 1)4 . 18. a.

2

=

2)3 1)3

2)3 1)3

b. Distribution after one step: B = After two steps: B

=

2)3 1)3

.5 .5 0

.5 .5 0

2

=

2)3 1)3 2)3 1)3

2)3 1)3

1)7 6)7

2)3 1)3

2)3 1)3

= 2)3 1)3

= 2)3 1)3

2)3 1)3

After three steps, the result will remain the same: 2)3 1)3 .

19. a.

2

=

0

1

0

0 .5 .5

0

1

0

0 .5 .5

=

.25 .75 0 0

1

0 0

.75 .25


.5 .5 0

Solutions Section 8.7 b. Distribution after one step: B =

1 0 0

After two steps: B

0

2

After three steps: B

20. a.

2

=

=

.5 .5 0 .25 .75 0

.5 0 .5

.5 0 .5

1

0

0

0 .5 .5

1

0

0

0 .5 .5

b. Distribution after one step: B = After two steps: B

2

After three steps: B

21. a.

2

=

0

= 3

1 0 0 =

1

0

1)3 1)3 1)3 1

0

0

After two steps: B After three steps: B

22. a.

2

=

= 3

1)2 1)2 1)2 1)2 1)2

0

1

.5 .5 0

.25 .25 .5

0

1)2

0

.5

0

1

0

0

= 1 0 0

= .5 0 .5

.5 0 .5 0

0

= .25 .25 .5

0 .5 .5 1

.5

0 .5 .5

0 .5 .5 1

.5

.5 0 .5

0

= .5 .5 0

= .125 .875 0

.25 .25

.5 0 .5

0

1)3 1)3 1)3

1)3 1)3 1)3 = 4)9 4)9 1)9 1

0

0

0

1

0

1

0

0

1)3 1)3 1)3 = 1)2 1)2 0 1

1

0

0

0

1)3 1)3 1)3 = 1)6 2)3 1)6

1)2 1)2 1)2 1)2 1)2

0

1)2 0 1)2

1)6 2)3 1)6 0

0

0 .5 .5

1)2 1)2 0 =

0

0

= .25 .75 0

0 .5 .5

0 1 0

0

b. Distribution after one step: B = 2

0

=

1

.5 0 .5

1

1

0 .5 .5

.5 .5 0

=

3

0

0

1

0

0

1

0

0

1)3 1)3 1)3 = 7)18 7)18 2)9 1

0 0

1)2

0

0

1)2 1)2 = 1)2 1)2

0 0

1)2 1)4 1)4


1)2 1)2

0

Solutions Section 8.7 b. Distribution after one step: B = After two steps: B

2

After three steps: B

23. a.

2

=

= 3

=

1

0

0 .2 .8

0

1

0

0 .2 .8

B. Distribution after one step: B = After two steps: B

2

After three steps: B

24. a.

2

=

= 3

.1 .1 .8 .5 0 .5 .5 0 .5

After two steps: B After three steps: B

= 3

1)2

.01 .99 0

1

0

.5 0 .5

0

= 1)2 1)4 1)4

0 0

1)2

= 1)2 3)8 1)8

0

1

0 = .05 .55 .4

0 .2 .8

.005 .675 .32

0

1

0 .2 .8

= .005 .675 .32

.1 .9 0 0

0 .2 .8

= .0005 .7435 .256

.46 .01 .53 .3 .3

0 1 0

.1 .1 .8

.05 .65 .05 .65

.1 .1 .8

.5 0 .5 = .5 0 .5 .5 0 .5

.5 0 .5 = .3 .05 .65 .5 0 .5

.1 .1 .8

.3 .05 .65

.5 0 .5 = .38 .03 .59

25. To find the steady-state vector B = % +6 = 1 1)2 1)2 = % 6 % 6 1 0 The above equations give:. % +6 = 1 %)2 + 6 = %

0

1)2

= 1)2 0 1)2

.1 .9 0

0

.5 0 .5

1)2

0

0

0

.36 .64

.1 .9 0

.5 0 .5 =

0

1)2 1)2

=

.1 .1 .8

0

1)2 1)2

1

.5 0 .5 =

0

0

b. Distribution after one step: B = 2

1)2

.05 .55 .4 =

1)2

1)2 1)2

1)2 1)4 1)4 .1 .9 0

1)2 1)2

1)2 1)2

1)2 0 1)2

.1 .9 0 0

0 0 1

.5 0 .5

% 6 , we solve

%)2 = 6


Solutions Section 8.7 Rewriting in standard form and dropping the last equation (which is the same as the next-to-last), we get % +6 = 1 %)2 + 6 = 0. Solving this system gives % = 2)3, 6 = 1)3. So, B = 2)3 1)3 26. To find the steady-state vector B = % +6 = 1 % 6

0

1

% 6 , we solve

= % 6 1)4 3)4 The above equations give: % +6 = 1 6)4 = % % + 36)4 = 6. Rewriting in standard form and dropping the last equation (which is the same as the next-to-last), we get % +6 = 1 % + 6)4 = 0. Solving this system gives % = 1)5, 6 = 4)5. So, B = 1)5 4)5 27. To find the steady-state vector B =

% 6 , we solve

28. To find the steady-state vector B =

% 6 , we solve

29. To find the steady-state vector B =

% 6 , we solve

% +6 = 1 1)3 2)3 = % 6 % 6 1)2 1)2 The above equations give: % +6 = 1 %)3 + 6)2 = % 2%)3 + 6)2 = 6. Rewriting in standard form and dropping the last equation (which is the same as the next-to-last), we get % +6 = 1 2%)3 + 6)2 = 0. Solving this system gives % = 3)7, 6 = 4)7. So, B = 3)7 4)7 % +6 = 1 .2 .8 = % 6 % 6 .4 .6 The above equations give: % +6 = 1 .2% + .46 = % .8% + .66 = 6. Rewriting in standard form and dropping the last equation (which is the same as the next-to-last), we get % +6 = 1 .8% + .46 = 0. Solving this system gives % = 1)3, 6 = 2)3. So, B = 1)3 2)3 % +6 = 1 .1 .9 = % 6 % 6 .6 .4 The above equations give: % +6 = 1 .1% + .66 = % .9% + .46 = 6. Rewriting in standard form and dropping the last equation (which is the same as the next-to-last), we get % +6 = 1 .9% + .66 = 0. Solving this system gives % = 2)5, 6 = 3)5.


So, B =

2)5 3)5

Solutions Section 8.7

30. To find the steady-state vector B =

% 6 , we solve

31. To find the steady-state vector B =

% 6 8 , we solve

% +6 = 1 .2 .8 = % 6 % 6 .7 .3 The above equations give: % +6 = 1 .2% + .76 = % .8% + .36 = 6. Rewriting in standard form and dropping the last equation (which is the same as the next-to-last), we get % +6 = 1 .8% + .76 = 0. Solving this system gives % = 7)15, 6 = 8)15. So, B = 7)15 8)15 % +6 +8 = 1 .5 0 .5 % 6 8

1

0

0

= % 6 8

0 .5 .5 The above equations give: % +6 = 1 .5% + 6 = % .58 = 6 .5% + .58 = 8. Rewriting in standard form, we get % +6 +8 = 1 .5% .58 = 0. .5% + 6 = 0 6 + .58 = 0 Solving this system gives % = 2)5, 6 = 1)5, 8 = 2)5. So, B = 2)5 1)5 2)5 32. To find the steady-state vector B = % +6 +8 = 1 0 .5 .5 % 6 8

.5 .5 0

% 6 8 , we solve

= % 6 8

1 0 0 The above equations give: % +6 = 1 .56 + 8 = % .5% + .56 = 6 .5% = 8. Rewriting in standard form, we get % +6 +8 = 1 .5% .56 = 0 .5% 8 = 0. % + .56 + 8 = 0 Solving this system gives % = 2)5, 6 = 2)5, 8 = 1)5. So, B = 2)5 2)5 1)5 33. To find the steady-state vector B = % +6 +8 = 1 % 6 8

0

1

0

% 6 8 , we solve

1)3 1)3 1)3 = % 6 8

1 0 0 The above equations give: % +6 = 1 6)3 + 8 = % % + 6)3 = 6 6)3 = 8. Rewriting in standard form, we get % +6 +8 = 1 % 26)3 = 0 6)3 8 = 0. % + 6)3 + 8 = 0


Solutions Section 8.7 Solving this system gives % = 1)3, 6 = 1)2, 8 = 1)6. So, B = 1)3 1)2 1)6 34. To find the steady-state vector B = % +6 +8 = 1 1)2 1)2 % 6 8

1)2 1)2

0 0

% 6 8 , we solve

= % 6 8

1)2 0 1)2 The above equations give: % +6 = 1 %)2 + 6)2 + 8)2 = % %)2 + 6)2 = 6 8)2 = 8. Rewriting in standard form, we get % +6 +8 = 1 %)2 6)2 = 0 %)2 + 6)2 + 8)2 = 0 8)2 = 0. Solving this system gives % = 1)2, 6 = 1)2, 8 = 0. So, B = 1)2 1)2 0 35. To find the steady-state vector B = % +6 +8 = 1 .1 .9 0 % 6 8

0

1

0

% 6 8 , we solve

= % 6 8

0 .2 .8 The above equations give: % +6 = 1 .1% = % .9% + 6 + .28 = 6 Rewriting in standard form, we get % +6 +8 = 1 .9% + .28 = 0 .9% = 0 Solving this system gives % = 0, 6 = 1, 8 = 0. So, B = 0 1 0 36. To find the steady-state vector B = % +6 +8 = 1 .1 .1 .8 % 6 8

.88 = 8.

.28 = 0.

% 6 8 , we solve

.5 0 .5 = % 6 8

.5 0 .5 The above equations give: % +6 = 1 .1% + .56 + .58 = % .1% = 6 .8% + .56 + .58 = 8. Rewriting in standard form, we get % +6 +8 = 1 .1% 6 = 0 .8% + .56 .58 = 0. .9% + .56 + .58 = 0 Solving this system gives % = 5)14, 6 = 1)28, 8 = 17)28. So, B = 5)14 1)28 17)28 37. Take 1 = Sorey State, 2 = C&T. 1)2 1)2 = 1)4 3)4 We are asked to find the (1, 1) entry of the 2-step transition probability matrix. 2

=

1)2 1)2

1)2 1)2

1)4 3)4 1)4 3)4 The (1, 1) entry is 3)8 = .375.

=

3)8

5)8

5)16 11)16


Solutions Section 8.7 38. Take 1 = Business major, 2 = Non–business major. .9 .1 .9 .1 .9 .1 .83 .17 ; 2= = = .2 .8 .2 .8 .2 .8 .34 .66 We are asked for the (1, 2) entry of

2

:(

)12 = .17.

2

39. a. Take 1 = Not checked in, 2 = Checked in. .4 .6 .4 .6 .4 .6 .16 .84 ; 2= = = 0 1 0 1 0 1 0 1 3

=

.4 .6 0

1

.16 .84 0

1

=

.064 .936 0

1

b. 1 hour: 12 = .6 2 hours: ( )12 = .84 3 hours: ( c. Eventually, all the roaches will have checked in. 2

)12 = .936

3

40. a. Take 1 = Employed by the DAA, 2 = Not employed by the DAA. .9 .1 .9 .1 .9 .1 .81 .19 ; 2= = = 0 1 0 1 0 1 0 1 3

=

.9 .1 0

1

.81 .19 0

1

=

.729 .271 0

1

b. 1 week: 11 = .9 2 weeks ( )11 = .81 3 weeks: ( 3)11 = .729 c. Not very good: the probability of being employed after week is .9 . 2

41. Take 1 = High risk, 2 = Low risk. .50 .50 = .10 .90

To find the steady-state vector % 6 , we solve % +6 = 1 .50 .50 = % 6 . % 6 .10 .90 The above equations give: % +6 = 1 .5% + .16 = % .5% + .96 = 6. Rewriting in standard form and dropping the last equation, we get % +6 = 1 .5% + .16 = 0. Solving this system gives % = 1)6, 6 = 5)6. So, % 6 = 1)6 5)6 .

In the long term, 1)6 16.67 fall into the high-risk category, and 5)6 83.33 into the low-risk category. 42. Take 1 = good credit rating, 2 = poor credit rating. .80 .20 = .40 .60

To find the steady-state vector % 6 , we solve % +6 = 1 .80 .20 = % 6 . % 6 .40 .60 The above equations give: % +6 = 1 .8% + .46 = % .2% + .66 = 6. Rewriting in standard form and dropping the last equation, we get


Solutions Section 8.7 % +6 = 1 .2% + .46 = 0. Solving this system gives % = 2)3, 6 = 1)3. So, % 6 = 2)3 1)3 .

In the long term, 2/3 or approximately 66.67% have good ratings and 1/3 or approximately 33.33% have bad ratings. 43. a. Take 1 = User, 2 = Non-User. To set up the transition matrix, note that the entries in each row have to add up to 1: 2)3 1)3 . = 1)10 9)10 The 2-year transition matrix is 2)3 1)3 2)3 1)3 43)90 47)90 2 = . = 1)10 9)10 1)10 9)10 47)300 253)300 Non-user A User in 2 steps: ( 2)21 = 47)300 .156667 b. To find the steady-state vector % 6 , we solve % +6 = 1 2)3 1)3 = % 6 . % 6 1)10 9)10 The above equations give: % +6 = 1 (2)3)% + (1)10)6 = % (1)3)% + (9)10)6 = 6. Rewriting in standard form and dropping the last equation, we get % +6 = 1 (1)3)% + (1)10)6 = 0. Solving this system gives % = 3)13, 6 = 10)13. So, % 6 = 3)13 10)13 .

In the long term, 3/13 of the college instructors will be users of this book. 44. a. Take 1 = Scores, 2 = Fails to score. To set up the transition matrix, note that the entries in each row have to add up to 1: 2)3 1)3 . = 1)4 3)4 The 2-year transition matrix is 2)3 1)3 2)3 1)3 19)36 17)36 2 = . = 1)4 3)4 1)4 3)4 17)48 31)48 Failed to score $ Scored in 2 steps: ( 2)21 = 17)48 .35 b. To find the steady-state vector % 6 , we solve % +6 = 1 2)3 1)3 = % 6 . % 6 1)4 3)4 The above equations give: % +6 = 1 (2)3)% + (1)4)6 = % (1)3)% + (3)4)6 = 6. Rewriting in standard form and dropping the last equation, we get % +6 = 1 (1)3)% + (1)4)6 = 0. Solving this system gives % = 3)7, 6 = 4)7. So, % 6 = 3)7 4)7 . In the long term, 3/7 of the shots are successful.


Solutions Section 8.7 45. a. Take 1 = Paid Up, 2 = 0–90 Days, 3 = Bad Debt. .5 .5 0 = .5 .3 .2 0 .5 .5

To find the steady-state vector % 6 8 , we solve % +6 +8 = 1 .5 .5 0 % 6 8 .5 .3 .2 = % 6 8 .

0 .5 .5 The above equations give: % +6 +8 = 1 .5% + .56 = % .5% + .36 + .58 = 6 Rewriting in standard form and dropping the last equation, we get % +6 +8 = 1 .5% .76 + .58 = 0. .5% + .56 = 0 Solving this system gives % = 5)12, 6 = 5)12, 8 = 1)6. So, % 6 = 5)12 5)12 1)6 .

.26 + .58 = 8.

5/12 or approximately 41.67% of the customers will be in the Paid Up category, 5/12 or approximately 41.67% in the 0–90 Days category, and 1/6 or approximately 16.67% in the Bad Debt category. 46. a. Take 1 = Paid Up, 2 = 0–90 Days, 3 = Bad Debt. .8 .2 0 = .5 .3 .2 0 .5 .5

To find the steady-state vector % 6 8 , we solve % +6 +8 = 1 .8 .2 0 % 6 8 .5 .3 .2 = % 6 8 .

0 .5 .5 The above equations give: % +6 +8 = 1 .8% + .56 = % .2% + .36 + .58 = 6 Rewriting in standard form and dropping the last equation, we get % +6 +8 = 1 .2% .76 + .58 = 0. .2% + .56 = 0 Solving this system gives % = 25)39, 6 = 10)39, 8 = 4)39. So, % 6 = 25)39 10)39 4)39 .

.26 + .58 = 8.

25/39 or approximately 64.10% of the customers will be in the Paid Up category, 10/39 or approximately 25.64% in the 0–90 Days category, and 4/39 or approximately 10.26% in the Bad Debt category.

47. a. The transition matrix is C.43 .27 .17 .09 .04F E.25 .24 .18 .19 .14H = EE.14 .2 .23 .24 .19HH E H E.09 .2 .23 .24 .24H ED.08 .1 .19 .23 .4 HG We want the three-step transition matrix, which is C.233860 .217111 .195909 .183781 E.207312 .206054 .198909 .194401 3 EE.190827 .199406 .200872 .201071 E.183387 .196041 .201627 .203941 E ED.171819 .189643 .202458 .208030

.169339F .193324H .207824HH .215004HH .228050HG


Solutions Section 8.7 The probability that, of families in the first quintile, the family income 3 generations later is in the top 3 quintile, is 15 .17.

b. We use technology to compute a high enough power of so that the rows are approximately the same: C.19731 .201592 .199968 .198296 .202834F E.19731 .201592 .199968 .198296 .202834H 100 E.19731 .201592 .199968 .198296 .202834H EE HH E.19731 .201592 .199968 .198296 .202834H ED.19731 .201592 .199968 .198296 .202834HG To the nearest percent, the long-term distribution of family incomes is [.20=.20=.20=.20=.20]. This makes sense, as one fifth of families should be in each quintile, by definition. 48. a. The transition matrix is C .1 .37 .26 .17 .1 F E.12 .14 .1 .27 .37H = E.07 .15 .21 .26 .31H EE HH E.05 .1 .23 .27 .35H ED.04 .06 .16 .23 .51HG We want the three-step transition matrix, which is C.064885 .124124 .185835 .246243 .378913F E.062149 .116690 .185047 .246595 .389519H 3 EE.061832 .116764 .185860 .246555 .388989HH E H E.061005 .114980 .186039 .246594 .391382H ED.059297 .111262 .185687 .246069 .397685HG The probability that, of families in the middle quintile, the family income 3 generations later is in the top 3 quintile, is 35 .39.

b. We use technology to compute a high enough power of so that the rows are approximately the same: C.0608556 .114605 .185742 .246359 .392439F E.0608556 .114605 .185742 .246359 .392439H 100 E.0608556 .114605 .185742 .246359 .392439H EE HH E.0608556 .114605 .185742 .246359 .392439H DE.0608556 .114605 .185742 .246359 .392439GH To the nearest percent, the long-term distribution of family incomes is [.06=.11=.19=.25=.39]. We should not expect the uniform long-term distribution of the preceding exercise, because the quintiles are based on all families but we are not looking at all families, only "collegiate" families. 49. a. Number the states as follows: 1: Verizon, 2: T-Mobile, 3: AT&T, 4: Other. The figures next to the arrows show all the transition probabilities (obtained by dividing by 100) except the ones from each state to itself. To compute the missing probabilities, use the fact that the entries in each row add to 1. This leads to: C.9688 .0174 .0111 .0027F E.0166 .9634 .0136 .0064H = E H. E.0111 .0134 .9739 .0016H E.0151 .0176 .0123 .9550H D G b. Each time-step is one quarter. At the end of the second quarter, the distribution is given as I = J0.286 0.247 0.231 0.236K. We are asked to find the distribution B at the beginning of that quarter: one time step earlier. We know that Distribution after one quarter = B = J0.286 0.247 0.231 0.236K

To obtain B, multiply both sides on the right by 1 : B = J0.286 0.247 0.231 0.236K 1. On the Matrix Algebra Tool, use the format w*P^-1: We obtain (rounding to 3 decimal places): B = J0.285 0.244 0.227 0.244K. Thus, the market shares at the beginning of the quarter were:


Solutions Section 8.7 Verizon: 28.5%, T-Mobile: 24.4%, AT&T: 22.7%, Other: 24.4%. c. The end of 2023 is 10 quarters from the end of the second quarter of 2021. Therefore, the distribution is predicted as I 10 = J0.296 0.272 0.260 0.172K Verizon: 29.6%, T-Mobile: 27.2%, AT&T: 26.0%, Other: 17.2%. (Technology format: w*P^10) The biggest gainer is AT&T, gaining 2.9%. 50. a. Number the states as follows: 1: Verizon, 2: T-Mobile, 3: AT&T, 4: Other. The figures next to the arrows show all the transition probabilities (obtained by dividing by 100) except the ones from each state to itself. To compute the missing probabilities, use the fact that the entries in each row add to 1. This leads to: C.9688 .0174 .0111 .0027F E.0166 .9634 .0136 .0064H . = E E.0111 .0134 .9739 .0016HH E.0151 .0176 .0123 .9550H D G b. Each time step is one quarter. At the end of the second quarter, the distribution is given as I = J0.286 0.247 0.231 0.236K. We are asked to find the distribution B four time steps earlier. We know that Distribution after four quarters = B 4 = J0.286 0.247 0.231 0.236K. To obtain B, multiply both sides on the right by ( 1) 4 : B = J0.286 0.247 0.231 0.236K( 4). On the Matrix Algebra Tool, use the format w*P^-4: We obtain (rounding to 3 decimal places): B = J0.280 0.232 0.216 0.272K. Thus, the market shares one year earlier were: Verizon: 28.0%, T-Mobile: 23.2%, AT&T: 21.6%, Other: 27.2%. c. The end of 2029 is 34 quarters from the end of the second quarter of 2021. Therefore, the distribution is predicted as I 34 = J.306 .294 .297 .103K Verizon: 30.6%, T-Mobile: 29.4%, AT&T: 29.7%, Other: 10.3%. (Technology format: w*P^34) The biggest loser is Other, losing 13.3%. 51. From the diagram, the transition matrix is 1)2 1)2 0 0 0 =

1)2 0 0

0

1)2 0

1)2 0

1)2

0

1)2 0

0

0

1)2

.

0 0 0 1)2 1)2 For the steady-state vector, we solve: % +6 +8 +9 +B = 1 (1)2)% + (1)2)6 = % (1)2)% + (1)2)8 = 6 (1)2)6 + (1)2)9 = 8 (1)2)8 + (1)2)B = 9 (1)2)9 + (1)2)B = B. Rewriting in standard form and dropping the last equation, we get the system % +6 +8 +9 +B = 1 (1)2)% + (1)2)6 = 0 (1)2)% 6 + (1)2)8 = 0 (1)2)6 8 + (1)2)9 = 0 (1)2)8 9 + (1)2)B = 0. Solving gives % = 1)5, 6 = 1)5, 8 = 1)5, 9 = 1)5, B = 1)5.


Hence the steady-state vector is 1)5 1)5 1)5 1)5 1)5 . The system spends an average of 1/5 of the time in each state. Solutions Section 8.7

52. From the diagram, the transition matrix is 2)3 1)3 0 0 0 =

2)3 0 0

0

2)3 0

1)3 0

2)3

0

1)3 0

0

0

1)3

.

0 0 0 2)3 1)3 For the steady-state vector, we solve: % +6 +8 +9 +B = 1 (2)3)% + (2)3)6 = % (1)3)% + (2)3)8 = 6 (1)3)6 + (2)3)9 = 8 (1)3)8 + (2)3)B = 9 (1)3)9 + (2)3)B = B. Rewriting in standard form and dropping the last equation, we get the system % +6 +8 +9 +B = 1 (1)3)% + (2)3)6 = 0 (1)3)% 6 + (2)3)8 = 0 (1)3)6 8 + (2)3)9 = 0 (1)3)8 9 + (2)3)B = 0. Solving gives % = 16)31, 6 = 8)31, 8 = 4)31, 9 = 2)31, B = 1)31. Hence the steady-state vector is 16)31 8)31 4)31 2)31 1)31 . 53. Answers will vary. 54. Answers will vary. 55. There are two assumptions made by Markov systems that may not be true about the stock market: the assumption that the transition probabilities do not change over time and the assumption that the transition probability depends only on the current state. 56. Yes, it can. The two states are (1) heads and (2) tails, and all the transition probabilities equal 1/2.

57. If ? is a row of ;, then by assumption, ? = ?. Thus, when we multiply the rows of ; by , nothing changes, and ; = ;.

58. An example is the system with the transition matrix

=

0.5 0.5 0.5 0.5 0 0

0 0

0 0

0 0

0.5 0.5 0.5 0.5

.

59. At each step only 0.4 of the population in state 1 remains there, and nothing enters from any other state. Thus, when the first entry in the steady-state distribution vector is multiplied by 0.4, it must remain unchanged. The only number for which this is true is 0. 60. The steady-state distribution is 0 0.5 0.5 . The first entry is 0 for the same reason as in Exercise


Solutions Section 8.7 61. The other two entries are both 0.5 because of the symmetry of these two states. 61. An example is

Its transition matrix is

=

.3 .3 .4

.3 .3 .4 and it is easy to see then that B = B for B = .3 .3 .4

.3 .3 .4 .

62. An example is

Its transition matrix is

.

=

.6 .3 0 .1 .6 .3 0 .1 .6 .3 0 .1 .6 .3 0 .1

63. If B = B and I = I, then 1 (B + I) = 12 B + 12 I = 2

and it is easy to see then that B = B for B =

.6 .3 0 .1

1 B + 12 I = 12 (B + I). 2

Further, if the entries of B and I add up to 1, then so do the entries of (B + I))2.

64. Assume that we can make be as close to ; as we like by making sufficiently large. Then, for sufficiently large , ; = +1 ;, with the approximations as close as we like. Thus, ; ; to any desired degree of accuracy, which can only be true if in fact ; = ;. From this it follows that each row of ; must be a steady-state vector.


Solutions Chapter 8 Review Chapter 8 Review

1. ( ) = 2 × 2 × 2 = 8;

= {HHT, HTH, HTT, THH, THT, TTH, TTT}; ( ) = 2. ( ) = 2 × 2 × 2 × 2 = 16;

( ) 7 = ( ) 8

= {HTTT, THTT, TTHT, TTTH, TTTT}; ( ) =

( ) 5 = ( ) 16

= {(1, 6), (2, 5), (3, 4), (4, 3), (5, 2), (6, 1)}; ( ) =

( ) 6 1 = = ( ) 36 6

3. ( ) = 6 × 6 = 36; 4. ( ) = 6 3 = 216

= {(1, 1, 3), (1, 3, 1), (3, 1, 1), (1, 2, 2), (2, 1, 2), (2, 2, 1)}; ( ) =

( ) 6 1 = = ( ) 216 36

5. ( ) = 6 and = {2}. However, the outcomes are not equally likely, so we need the probability distribution. Start with Outcome

1

2

3

4

5

6

2%

%

%

%

%

2%

Outcome

1

2

3

4

5

6

Probability

.25

.125

.125

.125

.125

.25

Probability

Since the sum of the probabilities of the outcomes must be 1, we get 1 8% = 1, so % = = .125. 8 This gives:

( ) =

(2) = .125 (or 1)8)

6. ( ) = 21 and = {(1, 6), (2, 5), (3, 4)}. However, the outcomes are not equally likely. To compute ( ), we pretend that the dice were distinguishable (the dice don't behave any differently if they are different colors). For distinguishable dice, ( ) 6 1 ( ) = 36, = {(1, 6), (2, 5), (3, 4), (4, 3), (5, 2), (6, 1)} $ ( ) = = = . ( ) 36 6 7. (2 heads) =

"#(2 heads) 12 = 50

(At least 1 tail) = 1 (2 heads) = 1

12 38 = = .76 50 50


Solutions Chapter 8 Review 8. The given information can be placed in a table: Internet Other Total Increased

7

Decreased

2

Total

8

10

We can now fill in the missing quantities (starting with the totals): Internet Other Total Increased

6

1

7

Decreased

2

1

3

Total

8

2

10

: Increased Internet; ( ) =

"#( ) 9 = = .9 10

Internet Other Total Increased

6

1

7

Decreased

2

1

3

Total

8

2

10

9. We use the following events for a randomly selected novel: : You have read the novel. ' : Roslyn has read the novel. 150 200 50 300 ( ') = ( ) + (') ( ') = + = = .75 400 400 400 400 Therefore, the probability that a novel has been read by neither you nor your sister is [( ') ] = 1 ( ') = 1 .75 = .25. 10. For the sum of the numbers to be even, either both numbers must be odd or both numbers must be even. This occurs 8 times out of the 10 (since only on 2 rolls is one number odd and the other even). Therefore, "#( ) 8 ( ) = = = .8. 10 : A student is in category A; 24 ( ) = 1, ( ) = ( ) = = .75 32 We are asked to find ( ). ( )= ( )+ ( ) ( ) 1 = .75 + .75 ( )$ ( ) = .75 + .75 1 = .5 11. Let us use the following events:

: A student is in category B.

12. ( ) = 36, and there are 15 losing combinations, shown boxed (the first number represents the green die):


(1, 1), (2, 1), (3, 1), (4, 1), (5, 1), (6, 1),

(1, 2), (1, 3), (2, 2), (2, 3), (3, 2), (3, 3), (4, 2), (4, 3), (5, 2), (5, 3), (6, 2), (6, 3), (Lose) 15 (Lose) = = ( ) 36

(1, 4), (2, 4), (3, 4), (4, 4), (5, 4), (6, 4),

(1, 5), (2, 5), (3, 5), (4, 5), (5, 5), (6, 5),

(1, 6), (2, 6), (3, 6), (4, 6), (5, 6), (6, 6)

Solutions Chapter 8 Review

Therefore, (Win) = 1 (Lose) = 1

15 21 7 . = = 36 36 12

: Is a Model A; ' : Is orange. 1 1 1 7 ( ) + (') ( ') = + = 3 5 15 15

13. Use the following events: ( ') =

14. Use the following events: ; tornadoes; ( : monsoon. ( () = ( ) + (() ( () = .5 + .2 .1 = .6 Therefore, (Neither) = [( () ] = 1 ( () = 1 .6 = .4. 15. ( ) = (12, 5) = 792; ( ) = 16. ( ) = (12, 5) = 792; ( ) = 17. ( ) = (12, 5) = 792; ( ) =

( ) (4, 4) (8, 1) 8 = = ( ) 792 792

( ) (2, 2) (10, 3) 120 5 = = = ( ) 792 792 33

( ) (4, 1) (2, 1) (1, 1) (3, 1) (2, 1) 48 = = ( ) 792 792

18. ( ) = (All are red) = 0 (There are only 4 red ones.) Therefore, ( ) = 1 ( ) = 1 0 = 1.

19. ( ) = (12, 5) = 792 Decision algorithm for constructing a set with at least 2 yellow: Alternative 1: 2 yellow: (3, 2) (9, 3) = 252 choices Alternative 2: 3 yellow: (3, 3) (9, 2) = 36 choices ( ) 288 This gives a total of 252 + 36 = 288 choices, so ( ) = . = ( ) 792

20. ( ) = (12, 5) = 792 Decision algorithm for constructing a set with none yellow and at most one red: Alternative 1: No yellow, no red: (5, 5) = 1 choice Alternative 2: No yellow, 1 red: (4, 1) (5, 4) = 20 choices ( ) 21 This gives a total of 1 + 20 = 21 choices, so ( ) = . = ( ) 792 21. ( ) = (52, 5); ( ) =

( ) (8, 5) = ( ) (52, 5)


22. ( ) = (52, 5); ( ) = 23. ( ) = (52, 5); ( ) =

Solutions Chapter 8 Review ( ) (12, 5) = ( ) (52, 5) ( ) (4, 3) (1, 1) (3, 1) = ( ) (52, 5)

24. ( ) = (52, 5) Decision algorithm for constructing a prime full house: Step 1: Select the denomination for the triple (2, 3, 5, 7, 11, 13): (6, 1) choices. Step 2: Select one of the remaining 5 denominations for the double: (5, 1) choices. Step 3: Select 3 cards of the chosen denomination for the triple: (4, 3) choices. Step 4: Select 2 cards of the chosen denomination for the double: (4, 2) choices. ( ) (6, 1) (5, 1) (4, 3) (4, 2) ( ) = = ( ) (52, 5)

25. ( ) = (52, 5) Decision algorithm for constructing a full house of commons: Step 1: Select the denomination for the triple (2, 3, 4, 5, 6, 7, 8, 9, 10): (9, 1) choices. Step 2: Select one of the remaining 8 denominations for the double: (8, 1) choices. Step 3: Select 3 cards of the chosen denomination for the triple: (4, 3) choices. Step 4: Select 2 cards of the chosen denomination for the double: (4, 2) choices. ( ) (9, 1) (8, 1) (4, 3) (4, 2) ( ) = = ( ) (52, 5)

26. ( ) = (52, 5) Decision algorithm for constructing a black two pair: Step 1: Select the two denominations for the pairs: (13, 2) choices. Step 2: Choose 2 black cards from the highest-ranked denomination chosen in Step 1: (2, 2) = 1 choice. Step 3: Choose 2 black cards from the other denomination chosen in Step 1: (2, 2) = 1 choice. Step 4: Choose a denomination for the single: (11, 1) choices. Step 5: Choose a single black card of that denomination: (2, 1) choices. ( ) (13, 2) (11, 1) (2, 1) ( ) = = ( ) (52, 5) : The sum is 5. : The green one is not a 1 and the yellow one is 1. : The sum is 5, the green one is not 1, and the yellow one is 1. = {(2, 1), (3, 1), (4, 1), (5, 1), (6, 1)}; = {(4, 1)} 1 5 ( )= , ( )= 36 36 ( ) 1)36 1 ( | )= = = ( ) 5)36 5 4 1 ( )= = 36 9 Since ( | ) ( ), the events and are dependent. 27.

: The sum is 6. : The green one is either 1 or 3 and the yellow one is 1. : The sum is 6, the green one is either 1 or 3, and the yellow one is 1. = {(1, 1), (3, 1)}; = 2 ( ) = 0, ( ) = 36

28.


( | )=

(

)

Solutions Chapter 8 Review 0 = = 0 2)36

( ) 4 1 ( )= = 36 9 Since ( | ) ( ), the events

and

are dependent.

: The yellow one is 4. : The green one is 4. : Both the yellow and green dice are 4. 1 1 ( )= , ( )= 36 6 ( ) 1)36 1 ( | )= = = ( ) 1)6 6 1 ( )= = ( | ). 6 Therefore, the events and are independent. 29.

: The yellow one is 5. : The sum is 6. : The yellow one is 5 and the sum is 6. 1 5 ( )= , ( )= 36 36 ( ) 1)36 1 ( | )= = = ( ) 5)36 5 1 ( )= ( | ). 6 Therefore, the events and are dependent. 30.

: The dice have the same parity. : Both dice are odd. : The dice have the same parity and are both odd. Note that = . 9 9 ( )= , ( )= 36 36 ( ) 9)36 ( | )= = = 1 ( ) 9)36 18 1 ( )= = 36 2 Since ( | ) ( ), the events and are dependent. 31.

: The sum is 7. : The dice do not have the same parity. : The sum is 7 and the dice do not have the same parity. Note that = . 6 18 L ( )= , ( )= 36 36 6)36 ( ) 6 1 ( | )= = = = ( ) 18)36 18 3 6 1 ( )= = 36 6 Since ( | ) ( ), the events and are dependent. 32.

33. Take 1 = Brand A, 2 = Brand B.

=

1)2 1)2 1)4 3)4


1)2 1)2

Solutions Chapter 8 Review =

34. Take 1 = Brand A, 2 = Brand B. =

2

=

3

1)2 1)2

1)2 1)2

1)4 3)4

5)16 11)16

1)4 3)4

1)2 1)2

1)4 3)4 3)8

=

5)8

1)4 3)4

3)8

5)8

5)16 11)16

11)32 21)32

21)64 43)64 21 Prob. of Brand → Brand B in 3 years = ( 3)12 = 32 35. From Exercise 33, =

2 3

Take B =

=

=

1)2 1)2 1)4 3)4

1)2 1)2

1)2 1)2

1)4 3)4

5)16 11)16

1)4 3)4

1)2 1)2

2)3 1)3

1)4 3)4 3)8

Distribution after 3 years is B

=

5)8

3

=

=

.

3)8

5)8

5)16 11)16 =

11)32 21)32 21)64 43)64

2)3 1)3

11)32 21)32

21)64 43)64 Brand A: 65)192 .339, Brand B: 127)192 .661

= 65)192 127)192 .

36. To find the steady-state vector % 6 , we solve % +6 = 1 1)2 1)2 = % 6 . % 6 1)4 3)4 The above equations give: % +6 = 1 %)2 + 6)4 = % %)2 + 36)4 = 6. Rewriting in standard form and dropping the last equation (see the note in the textbook before Example 4), we get % +6 = 1 %)2 + 6)4 = 0. Solving this system gives % = 1)3, 6 = 2)3. So, % 6 = 1)3 2)3 ; Brand A: 1/3, Brand B: 2/3.

37. Take the first letter of each category shown in the table to stand for the corresponding event: : The book is Sci Fi; : The book is stored in Washington, and so on. 94 + 33 15 112 14 ( ) = = = 200 200 25 S

H

R

O

Total

W

10

12

12

30

64

C

8

12

6

16

42

T

15

15

20

44

94

Total

33

39

38

90

200

38. Take the first letter of each category shown in the table to stand for the corresponding event: : The


Solutions Chapter 8 Review book is Sci Fi; : The book is stored in Washington, and so on. 15 3 ( ) = = 200 40 S

H

R

O

Total

W

10

12

12

30

64

C

8

12

6

16

42

T

15

15

20

44

94

Total

33

39

38

90

200

39. Take the first letter of each category shown in the table to stand for the corresponding event: : The book is Sci Fi; : The book is stored in Washington, and so on. ( ) 15 ( | ) = = ( ) 94 S

H

R

O

Total

W

10

12

12

30

64

C

8

12

6

16

42

T

15

15

20

44

94

Total

33

39

38

90

200

40. Take the first letter of each category shown in the table to stand for the corresponding event: : The book is Sci Fi; : The book is stored in Washington, and so on. ( ) 15 5 ( | ) = = = ( ) 33 11 S

H

R

O

Total

W

10

12

12

30

64

C

8

12

6

16

42

T

15

15

20

44

94

Total

33

39

38

90

200

41. Take the first letter of each category shown in the table to stand for the corresponding event: : The book is Sci Fi; : The book is stored in Washington, and so on. ( ) 79 ( | ) = = ( ) 167 S

H

R

O

Total

W

10

12

12

30

64

C

8

12

6

16

42

T

15

15

20

44

94

Total

33

39

38

90

200


Solutions Chapter 8 Review 42. Take the first letter of each category shown in the table to stand for the corresponding event: : The book is Sci Fi; : The book is stored in Washington, and so on. ( ) 18 6 ( | ) = = = ( ) 33 11 S

H

R

O

Total

W

10

12

12

30

64

C

8

12

6

16

42

T

15

15

20

44

94

Total

33

39

38

90

200

43. Take : Visited OHaganBooks.com; ( ) = .02. ( ) = 1 ( ) = 1 .02 = .98, or 98%

44. Take : Visited a competitor; ( ) = .05. ( ) = 1 ( ) = 1 .05 = .95, or 95%

45. Take : Visited OHaganBooks.com; ( ) = .02; : Visited a competitor; ( ) = .05. Since and are independent, ( ) = ( ) ( ) = .02 × .05 = .001. Therefore, ( ) = ( ) + ( ) ( ) = .02 + .05 .001 = .069, or 6.9%.

46. Take : Visited OHaganBooks.com; ( ) = .02; : Visited a competitor; ( ) = .05. Since and are independent, ( ) = ( ) ( ) = .02 × .05 = .001. Therefore, ( ) = ( ) ( ) = .02 .001 = .019, or 1.9%. 47. Take : Visited OHaganBooks.com; ( ) = .02; : Visited a competitor; ( ) = .05. [( ) ] = 1 ( ) = 1 .069 = .931

48. Take : Visited OHaganBooks.com; ( ) = .02; : Visited a competitor; ( ) = .05. Since and are mutually exclusive, ( ) = 0 Therefore, ( ) = ( ) + ( ) = .02 + .05 = .07. So [( ) ] = 1 ( ) = 1 .07 = .93. 49. We are told that an online shopper visiting a competitor was more likely to visit OHaganBooks.com that a randomly selected online shopper. That is, ( | ) > ( ). Multiplying both sides by ( ) gives ( | ) ( ) > ( ) ( ). That is, ( ) > ( ) ( ). Therefore, ( ) is greater.

50. Take : Visited OHaganBooks.com; ( ) = .02; : Visited a competitor; ( ) = .05. We are told that ( | ) = .25 and are asked to compute ( ).Since we are given ( | ), we can also compute ( ) using the multiplication principle: ( ) = ( | ) ( ) = .25 × .05 = .0125. Now we can find ( ) : ( ) = ( ) ( ) = .02 .0125 = .0075, or 0.75%.


Solutions Chapter 8 Review 51. Take : Visited OHaganBooks.com; : Purchased books. ( | ) = .08, ( ) = .02, ( | ) = .005 We are asked to compute ( | ). Using Bayes' theorem, ( | ) ( ) (.08)(.02) .0016 ( | ) = .246. = = ( | ) ( ) + ( | ) ( ) (.08)(.02) + (.005)(.98) .0016 + .0049

52. Take : Visited OHaganBooks.com; : Purchased books. ( | ) = .08, ( ) = .01, ( | ) = .005 We are asked to compute ( | ). Using Bayes' theorem, ( | ) ( ) (.08)(.01) .0008 ( | ) = .139. = = ( | ) ( ) + ( | ) ( ) (.08)(.01) + (.005)(.99) .0008 + .00495 53. Use the following events: : Admitted; : In-state. ( ) = .72, ( | ) = .56, ( | ) = .15 We are asked to compute ( | ). ( | ) ( ) (.56)(.72) .4032 ( | ) = .91 = = ( | ) ( ) + ( | ) ( ) (.56)(.72) + (.15)(.28) .4452

54. Use the following events: : Admitted; : from the U.S. ( ) = .75, ( | ) = .22, ( | ) = .14 We are asked to compute ( | ). ( | ) ( ) (.22)(.75) .165 ( | ) = .825 = = ( | ) ( ) + ( | ) ( ) (.22)(.75) + (.14)(.25) .2

55. Number the states in the order shown in the table, so Starting distribution (July 1): B =

.2 .4 .4

=

.8 .1 .1

.4 .6 0 . .2 0 .8

Distribution after 1 month: .8 .1 .1 B = .2 .4 .4 .4 .6 0 = .40 .26 .34

.2 0 .8 40% for OHaganBooks.com, 26% for JungleBooks.com, and 34% for FarmerBooks.com

56. Number the states in the order shown in the table, so Starting distribution (July 1): B =

.2 .4 .4

Distribution after 1 month: .8 .1 .1 B = .2 .4 .4 .4 .6 0 = .40 .26 .34

=

.8 .1 .1

.4 .6 0 . .2 0 .8

.2 0 .8 Distribution after 2 months (End of August): .8 .1 .1 (B ) = .40 .26 .34 .4 .6 0 = .492 .196 .312 .2 0 .8


Solutions Chapter 8 Review 49.2% for OHaganBooks.com, 19.6% for JungleBooks.com, and 31.2% for FarmerBooks.com 57. Here are three factors that the Markov model does not take into account: (1) It is possible for someone to be a customer at two different enterprises. (2) Some customers may stop using all three of the companies. (3) New customers can enter the field. 58. Steady-state vector: % +6 +8 = 1 .8 .1 .1 % 6 8

.4 .6 0

= % 6 8

.2 0 .8 The above equations give: % +6 +8 = 1 .8% + .46 + .28 = % .1% + .66 = 6 Rewriting in standard form and dropping the last equation, we get % +6 +8 = 1 .1% .46 = 0. .2% + .46 + .28 = 0 Solving gives % = 4)7, 6 = 2)7, 8 = 1)7. So, % 6 8 = 4)7 1)7 2)7 = B .

.1% + .88 = 8.

Answer: OHaganBooks.com: 4/7, JungleBooks.com: 1/7, FarmerBooks.com: 2/7


Solutions Chapter 8 Case Study Chapter 8 Case Study 1. With this strategy there is a 50-50 chance that the prize is behind door A, and so it does not matter whether you switch or not. Let , , , and be the events given in the text. As before, ( ) = ( ) = ( ) = 1)3. ( | ) = 1)2 as before also. Now, ( | ) = 0 still since is the event that Monty opens door B and the Prize is not behind it. On the other hand, ( | ) = 1)2 since he chooses either door B or door C at random (and if he does open door B, the prize will not be behind it). Thus, ( | ) =

( | ) ( ) = ( | ) ( ) + ( | ) ( ) + ( | ) ( )

1 1 2 2 3 = 1 1 1 1 1 2 0 2 2 + + 2 3 3 2 3

1 . 2

You can also do this calculation using a tree, as in the text. 2. a. Let us repeat the original calculation, this time using the known strategy of opening door B if possible. As before, let be the event that the Big Prize is behind door A, the event that it is behind door B, and the event that it is behind door C. Let be the event that Monty has opened door B and revealed that the Prize is not there. We compute ( | ) using Bayes' theorem. As before, 1 ( ) = ( ) = ( ) = . 3 Also, ( | ) = 0, ( | ) = 1, but this time, ( | ) = 1 (not 1/2 as before) since Monty no longer makes that decision at random. The probability of winning upon switching to C is ( | ) =

1 2 13 ( | ) ( ) 1 = = . ( | ) ( ) + ( | ) ( ) + ( | ) ( ) 1 2 1 + 0 2 1 + 1 2 1 2 3 3 3

Therefore, it makes no difference whether or not you switch!. b. If Monty opens door C, that means that he could not open door B, so the prize must be there. So, you are guaranteed to win if you switch.

3. Let , , , and be the events that the Prize is behind doors A, B, C, and D, respectively. Let be the event that Monty opens door B and reveals that the Prize is not there. Let us calculate ( | ). First, ( ) = ( ) = ( ) = ( ) = 1)4. Next, ( | ) = 1)3 and ( | ) = 0 by arguments similar to those in the text. ( | ) = 1)2 since, if the Prize is behind door C, Monty will pick at random either door B or door D to open. ( | ) = 1)2 similarly. Thus, ( | ) = =

( | ) ( ) ( | ) ( ) + ( | ) ( ) + ( | ) ( ) + ( | ) ( )

1 1 2 3 4 = 1 1 1 1 1 1 1 2 0 2 2 2 + + + 3 4 4 2 4 2 4

1 , 4

Just as before, Monty has not changed the probability that the Prize is behind door A. Clearly the probabilities ( | ) and ( | ) are equal, and ( | ) + ( | ) + ( | ) = 1. It follows that ( | ) = ( | ) = 3)8. Thus, switching from door A to either door C or door D (it does not matter which) increases your probability of finding the Prize from 1)4 to 3)8. 1 4. If there are 1,000 doors, not switching results in a probability of 1 chance in 1,000, or 1,000 of winning.

Switching doors increases the probability slightly, to


1 1 1,000

998

=

999 . 998,000

Solutions Chapter 8 Case Study


Solutions Section 9.1 Section 9.1

1. = the sum of the numbers facing up when you roll two dice. The possible sums are 2, 3, . . ., 12. Thus, is finite and has the set {2, 3, . . . , 12} of possible values.

2. = the page number when you open a 500-page book. The possible page numbers are 1, 2, . . . , 500. Thus, is finite and has the set {1, 2, . . . , 500} of possible values.

3. is the profit, to the nearest dollar, earned in a year if you purchase one share of a stock selected at random. The possible values of are 0, ±1, ±2, . . . (negative numbers indicate a loss). This gives an infinite number of discrete possible values for . Therefore, is a discrete infinite random variable with the set of possible values {0, 1, 1, 2, 2, . . .}. 4. is the exact amount of electricity supplied in a year by a randomly selected utility company, in gigawatt hours. This is a continuous random variable, since can assume any nonnegative real value.

5. is the time the second hand of your watch reads in seconds. This can be any real number between 0 and 60. Therefore, is a continuous random variable that can assume any value between 0 and 60 (including 0).

6. The number of goals scored in a soccer game can be 0, 1, 2, 3, . . . so we can think of as a discrete infinite random variable with values {0, 1, 2, . . .}. Alternatively, we can think of as finite, since there is an upper limit to the number of goals that can be scored in the time it takes to complete a soccer game. 7. The total number of goals that can be scored, up to a maximum of 10, is 1, 2, 3, . . ., 9, or 10. Therefore, is a finite random variable with values in the set {0, 1, 2, . . . , 10}. 8. The (whole) number of points that can be scored, up to a maximum of 100, is 1, 2, 3, . . ., 99, or 100. Therefore, is a finite random variable with values in the set {0, 1, 2, . . . , 100}. 9. The possible energies of an electron in a hydrogen atom are 1, 4, 9, 16, . . . . Thus, is a discrete infinite random variable with set of possible values { 1, 4, 9, 16, . . .}. 10. The energy of a hydrogen atom can be any positive value. Therefore, is a continuous random variable that can assume any positive value. 11. a. = {HH, HT, TH, TT} b. is the rule that assigns to each outcome the number of tails. c. Counting the number of tails in each outcome gives us the following values of : Outcome HH

HT

TH

TT

Value of X

1

1

2

0

12. a. = {HHH, HHT, HTH, HTT, THH, THT, TTH, TTT} b. is the rule that assigns to each outcome the largest number of consecutive times heads comes up in a row. c. Counting the number of consecutive heads gives us the following values of :


Solutions Section 9.1 Outcome HHH HHT HTH HTT THH THT TTH TTT

Value of

3

2

1

1

2

1

1

0

13. a. = {(1, 1), (1, 2), . . . , (1, 6), (2, 1), (2, 2), . . . , (6, 6)} b. is the rule that assigns to each outcome the sum of the two numbers. c. Computing the sum of the numbers in each outcome gives us the following values of : Outcome (1, 1) (1, 2) (1, 3)

Value of

2

3

4

... ...

(6, 6) 12

14. a. = {(1, 1), (1, 2), . . . , (1, 6), (2, 1), (2, 2), . . . , (6, 6)} b. is the rule that assigns to each outcome the largest number. c. Computing the largest of the two numbers in each outcome gives us the following values of : Outcome (1, 1) (1, 2) (1, 3)

Value of

1

2

3

... ...

(2,1) 2

... ...

(6, 6) 6

15. a. Take each outcome to be a pair of numbers (# of red marbles, # of green ones). Thus, = {(4, 0), (3, 1), (2, 2)}. (The pairs (1, 3) and (0, 4) are impossible since there are only 2 green marbles.) b. is the rule that assigns to each outcome the number of red marbles. c. Writing down the number of red marbles (the first coordinate) in each outcome gives the following values of . Outcome (4, 0) (3, 1) (2, 2)

Value of

4

3

2

16. a. Take each outcome to be a pair of numbers (# of red marbles, # of green ones). Thus, = {(3, 1), (2, 2)}. (The pairs (4, 1) (1, 3) and (0, 4) are impossible since there are only 3 red marbles and 2 green ones.) b. is the rule that assigns to each outcome the number of green marbles. c. Writing down the number of green marbles (the second coordinate) in each outcome gives the following values of . Outcome (3, 1) (2, 2)

Value of

1

2

17. a. = the set of students in the study group. b. is the rule that assigns to each student his or her final exam score. c. The values of , in the order given, are 89%, 85%, 95%, 63%, 92%, 80%.

18. a. = the set of dormitory suite mates b. is the rule that assigns to each student the capacity of his or her hard drive. c. The values of , in the order given, are 1,000 GB, 1,500 GB, 2,000 GB, 2,500 GB, 3,000 GB, and 3,500 GB.


Solutions Section 9.1 19. a. Assign letters to the missing values (we use the same letter for both since they are given to be equal):

( = )

2

4

.1

.2

6

8

10 .1

Since the probabilities add to 1, we get .1 + .2 + 2 + .1 = 1 \quad ⇒ \quad .4 + 2 = 1 \quad ⇒ \quad 2 = .6 \quad ⇒ \quad = .3. The completed table is:

( = )

2

4

6

8

10

.1

.2

.3

.3

.1

b. ( 6) = ( = 6) + ( = 8) + ( = 10) = .3 + .3 + .1 = .7 (2 < < 8) = ( = 4) + ( = 6) = .2 + .3 = .5

20. a. ( 0) = ( = 0) + ( = 1) + ( = 2) = .4 + .1 + .1 = .6 Since the probabilities in a probability distribution add to 1, ( < 0) = ( = 1) + ( = 2) = 1 ( 0) = 1 .6 = .4. b. Assign letters to the missing values (we use the same letter for both since they are given to be equal):

( = )

2

1

0

1

2

.4

.1

.1

2 + .4 + .1 + .1 = 1 \quad ⇒ \quad 2 = .4 \quad ⇒ \quad = .2

21. Since the probability that any specific number faces up is 1/6, the probability distribution for is given by the following table and histogram:

( = )

1 1 6

2 1 6

3 1 6

4 1 6

5 1 6

6 1 6

From the distribution, ( < 5) = ( = 1) + ( = 2) + ( = 3) + ( = 4) =

4 2 = . 6 3

22. The values of are the squares of the 1, 2, . . ., 6; that is, 1, 4, 9, . . ., 36. The probability of any one of these outcomes is the same as the probability that any specific number faces up: 1/6.

( = )

1 1 6

4 1 6

9 1 6

16 1 6

25 1 6

36 1 6


Solutions Section 9.1

From the distribution, ( > 9) = ( = 16) + ( = 25) + ( = 36) =

3 1 = . 6 2

23. The number of heads showing when you toss 3 fair coins is 0, 1, 2, or 3. The corresponding probabilities are 1/8, 3/8, 3/8, and 1/8 (see Example 3). The corresponding values of are the squares of 0, 1, 2, and 3; that is, 0, 1, 4, and 9. This gives us the following distribution and histogram:

( = )

0 1 8

1 3 8

From the distribution, (1

4

9

9) = ( = 1) + ( = 4) + ( = 9) =

3 8

1 8

7 . 8

24. The number of heads showing when you toss 3 fair coins is 0, 1, 2, or 3. The corresponding probabilities are 1/8, 3/8, 3/8, and 1/8 (see Example 3). The corresponding numbers of tails are 3, 2, 1, and 0. Therefore, the corresponding values of are 3, 1, 1, and 3. This gives us the following distribution and histogram:

( = )

3 1 8

1 3 8

From the distribution, (1

1

3

9) = ( = 3) + ( = 1) =

4 1 = . 8 2

4

5

10

3 8

1 8

25. When two distinguishable dice are thrown, there are 36 possible outcomes. The values of are the possible sums of the numbers facing up: 2, 3, ... , 11, 12. To calculate their probabilities, we can use 1 ( = 2) = (1, 1) = 36 2 ( = 3) = (1, 2), (2, 1) = 36 3 ( = 4) = (1, 3), (2, 2), (3, 1) = 36 and so on. The completed probability distribution is as follows:

( = )

2

1 36

3

2 36

3 36

4 36

6

5 36

7

6 36

8

5 36

9

4 36

3 36

11 2 36

12 1 36


Solutions Section 9.1

From the distribution, (

7) = 1 ( = 7) = 1

6 30 5 = = . 36 36 6

26. The event that = 0 is (1, 1), (2, 2), . . . , (6, 6) 6 1 ( = 0) = = . 36 6 The event that = 1 is the complement of the above event; (1, 2), (2, 1), . . . , (5, 6). 30 5 ( = 1) = = 36 6

( = )

0 1 6

1 5 6

As can never be > 1, ( > 1) = 0.

27. The possible values of are 1, 2, ... , 6 (the largest of the two numbers facing up). We calculate their probabilities as follows: 1 ( = 1) = (1, 1) = 36 3 ( = 2) = (1, 2), (2, 2), (2, 1) = 36 5 ( = 3) = (1, 3), (2, 3), (3, 3), (3, 2), (3, 1) = 36 and so on. The completed probability distribution is as follows:

( = )

1

1 36

2

3 36

From the distribution, (

3

5 36

4

7 36

5

9 36

6

11 36

3) = ( = 1) + ( = 2) + ( = 3) =

9 1 = . 36 4

28. The possible values of are 1, 2, ... , 6 (the smallest of the two numbers facing up). We calculate their probabilities as follows:


1 ( = 6) = (6, 6) = 36

Solutions Section 9.1

( = 5) = (6, 5), (5, 5), (5, 6) =

3 36

5 36 and so on. The completed probability distribution is as follows: ( = 4) = (6, 4), (5, 4), (4, 4), (4, 5), (4, 6) =

( = )

1

11 36

2

9 36

3

7 36

4

5 36

5

3 36

6

1 36

From the distribution, ( 4) = ( = 4) + ( = 5) + ( = 6) =

9 1 = . 36 4

29. a. For the values of we use the rounded midpoints of the measurement classes given: (35 + 44.9) 2 (65 + 74.9) 2

40 70

(45 + 54.9) 2 (75 + 84.9) 2

50 80

(55 + 64.9) 2 (85 + 94.9) 2

60 90

To obtain the probabilities, divide each frequency by the sum of all the frequencies, 50:

( = )

40

50

60

70

80

90

.02

.48

.34

.16

0

0

b. The histogram above has the area corresponding to < 60 shaded. ( < 60) = .02 + .48 = .50

30. a. For the values of we use the rounded midpoints of the measurement classes given: (35 + 44.9) 2 (65 + 74.9) 2

40 70

(45 + 54.9) 2 (75 + 84.9) 2

50 80

(55 + 64.9) 2 (85 + 94.9) 2

60 90

To obtain the probabilities, divide each frequency by the sum of all the frequencies, 50:

( = )

40

50

60

70

80

90

0

.06

.42

.24

.22

.06


Solutions Section 9.1

b. The histogram above has the area corresponding to > 60 shaded. ( < 60) = .24 + .22 + .06 = .52

31. a. For the values of we use the rounded midpoints of the measurement classes given: (24.9 + 15) 2 (54.9 + 45) 2 (84.9 + 75) 2

20 50 80

(34.9 + 25) 2 (64.9 + 55) 2

30 60

(44.9 + 35) 2 (74.9 + 65) 2

40 70

To obtain the probabilities, divide each frequency by the sum of all the frequencies, 1,000:

20

( = ) .222

30

40

50

60

70

80

.229

.173

.149

.125

.073

.029

b. The histogram above has the area corresponding to < 50 shaded. ( < 50) = .222 + .229 + .173 = .624

32. a. For the values of we use the rounded midpoints of the measurement classes given: (24.9 + 15) 2 (54.9 + 45) 2 (84.9 + 75) 2

20 50 80

(34.9 + 25) 2 (64.9 + 55) 2

30 60

(44.9 + 35) 2 (74.9 + 65) 2

40 70

To obtain the probabilities, divide each frequency by the sum of all the frequencies, 1,000:

20

( = ) .175

30

40

50

60

70

80

.215

.170

.154

.146

.090

.050


Solutions Section 9.1

b. The histogram above has the area corresponding to < 50 shaded. ( < 50) = .175 + .215 + .170 = .560

33. The associated random variable is shown on the axis of the histogram: = age of a (randomly selected) resident in Mexico. To determine the probability distribution of , use a frequency table. For the values of , use the rounded midpoints of the measurement classes given: (0 + 14) 2 = 7 (15 + 29) 2 = 22 (30 + 64) 2 = 47 (65 + 74) 2 70. To obtain the relative frequencies, divide each frequency by the sum of all the frequencies, 250:

Freq

7

22

47

70

65

64

108

13

.256

.432

.052

( = ) .260

34. The associated random variable is shown on the axis of the histogram: = age of a (randomly selected) resident in the U.S. To determine the probability distribution of , use a frequency table. For the values of , use the rounded midpoints of the measurement classes given: (0 + 14) 2 = 7 (15 + 29) 2 = 22 (30 + 64) 2 = 47 (65 + 74) 2 70. To obtain the relative frequencies, divide each frequency by the sum of all the frequencies, 250:

Freq

7

22

47

70

49

54

120

27

.216

.480

.108

( = ) .196

35. To obtain the frequency table, count how many of the scores fall in each measurement class: Class 1.1–2.0 2.1–3.0 3.1–4.0 Freq.

4

7

9

To obtain the probability distribution, divide each frequency by the sum of all the frequencies, 20:

( = )

1.5

2.5

3.5

.20

.35

.45


Solutions Section 9.1 36. To obtain the frequency table, count how many of the scores fall in each measurement class: Class 2.1–4.0 4.1–6.0 6.1–8.0 8.1–10.0 Freq.

1

1

11

7

To obtain the probability distribution, divide each frequency by the sum of all the frequencies, 20:

( = )

3.0

5.0

7.0

9.0

.5

.5

.55

.35

37. a. The values of are the possible tow ratings: 6,000, 6,500, 7,000, 7,500, 8,000. b. The following table shows the frequency (number of models with each tow rating) and the resulting probabilities (divide each frequency by the sum of all the frequencies): 6,000 6,500 7,000 7,500 8,000

Freq.

c. (

( = )

3

1

3

2

1

.3

.1

.3

.2

.1

6,500) = .3 + .1 = .4

38. a. The values of are the possible percent increases: 75, 100, 125, 150, 175, 200, 225, 250, 275, 300 (100, 175, 200, 250, and 275 are optional) b. The following table shows the frequency (number of regions with each percent increase) and the resulting probabilities (divide each frequency by the sum of all the frequencies):

Freq.

( = )

c. ( > 200) =

75

100

125

150

175

200

225

250

275

300

1 9

0

2 9

1 3

0

0

2

0

1

0

0

2 9

0 0

0

1

2 1 1 + = 9 9 3

0

2

3

1 9

39. a. The values of are the (rounded) changes of the Dow: 700, 600, 500, 400, 300, 200, 100, 0, 100, 200, 300, 400, 500, 600, 700, 800, 900 ( 600, 0, 100, 300, 500, 600, 700, and 800 are optional). b. The following table shows the frequency (number of days with each specified change in the Dow) and the resulting probabilities (divide each frequency by the sum of all the frequencies, 20): 700 600 500 400 300 200 100

Freq.

( = )

2

0

2

1

1

4

4

0

.1

0

.1

.05

.05

.2

.2

0

200

300

400

500

600

700

800

900

0

2

0

2

0

0

0

0

2

0

.1

0

.1

0

0

0

0

.1

100

Freq.

( = )

0

c. ( < 200) = .05 + .05 + .1 + .1 = .3


Solutions Section 9.1 40. a. The values of are the (rounded) changes of the Dow: 700, 600, 500, 400, 300, 200, 100, 0, 100, 200, 300, 400, 500, 600 ( 600 is optional). b. The following table shows the frequency (number of days with each specified change in the Dow) and the resulting probabilities (divide each frequency by the sum of all the frequencies, 20): 700 600 500 400 300 200 100

Freq

1

0

1

4

1

2

1

.05

0

.05

.2

.05

.1

.05

0

100

200

300

400

500

600

Freq

1

1

3

2

1

1

1

.05

.05

.15

.1

.05

.05

.05

( = ) ( = )

c. ( < 200) = .05 + .2 + .05 + .05 = .35 41. First, construct the probability distribution:

0

1

2

3

4

5

6

7

8

9

10

Freq 140

350

450

650

200

140

50

10

5

15

10

.07

.175

.225

.325

.1

.06

.025

.005 .0025 .0075 .005

( = )

( < 6) = .07 + .175 + .225 + .325 + .1 + .06 = .955. Therefore, 95.5% of cars are newer than 6 years old. 42. First, construct the probability distribution (probabilities are rounded to 4 decimal places and are obtained by dividing the frequencies by the total number of cars, 147):

Freq

0

1

2

3

4

5

6

7

8

9

10

0

2

5

5

10

10

15

20

20

20

40

( = ) .0000 .0136 .0340 .0340 .0680 .0680 .1020 .1361 .1361 .1361 .2721

( = 5) = .0680, ( = 6) = .1020. Therefore, the relative frequency of getting on the Dean's List goes up by .1020 .0680 = .034, or 3.4%.

43. The frequency distribution for is the first row (small cars) of the given table, and we use that to compute the relative frequency distribution (divide each frequency by the sum of the frequencies, 16):

1

( = ) .125

2

3

0

.75

4

5

.0625 .0625

44. The frequency distribution for is the second row (compact SUVs) of the given table, and we use that to compute the relative frequency distribution (divide each frequency by the sum of the frequencies, 20):

( = )

1

2

3

4

5

.05

.35

.45

.1

.05


45. ( 3) = ( = 3) + ( = 4) + ( = 5) = .75 + .0625 + .0625 = .875 The relative frequency that a randomly selected small car is rated Average or better is .875. Solutions Section 9.1

46. ( 2) = ( = 1) + ( = 2) = .05 + .35 = .4 The relative frequency that a randomly selected compact SUV is rated worse than Average is .4. and :

47. The following tables give the relative frequency distributions for

( = )

1

2

3

4

5

.05

.35

.45

.1

.05

( = )

1

2

3

4

5

0

.417

.25

.333

0

( 3) = ( = 3) + ( = 4) + ( = 5) = .45 + .1 + .05 = .6 ( 3) = ( = 3) + ( = 4) + ( = 5) .25 + .333 + 0 = .583 Since a reliability rating of at least 3 indicates Average or better, the data suggest that compact SUVs are a little more reliable than midsize SUVs. 48. The following tables give the relative frequency distributions for and :

1

( = ) .125

2

3

0

.75

4

5

.0625 .0625

( = )

1

2

3

4

0

0.75 0.125 0.125

5 0

( 3) = ( = 1) + ( = 2) + ( = 3) = .125 + 0 + .75 = .875 ( 3) = ( = 1) + ( = 2) + ( = 3) = 0 + .75 + .125 = .875 Since a reliability rating of 3 or less indicates Average or worse, the data suggest that small cars and large SUVs have a similar chance of having Average or worse reliability. 1 1 2 ( = 5) = ( = 5) = .167 = .0625 = .05 16 20 12 0 0 1 ( = 5) = ( = 5) = ( = 5) = = 0 = 0 = .05 12 8 20 The highest is ( = 5), for midsize cars.

49. ( = 5) =

12 9 ( = 3) = = .75 = .45 16 20 3 1 ( = 3) = ( = 3) = = .25 = .125 12 8 The lowest is ( = 3), for large SUVs. 50. ( = 3) =

( = 3) =

7 12

( = 3) =

.583

5 = .25 20

51. The relative frequency that a randomly selected small car will be rated at least 3 is 14 ( 3) = = .875. 16 The relative frequency that a randomly selected compact SUV will be rated at least 3 is 12 ( 3) = = .6. 20 Since these two events are independent (the vehicles are selected at random from their respective populations), the relative frequency that both will be rated at least 3 is ( 3) × ( 3) = .875 × .6 = .525.


Solutions Section 9.1 52. The relative frequency that a randomly selected small car will be rated at most 2 is 2 ( 2) = = .125. 16 The relative frequency that a randomly selected full-sized pickup will be rated at most 2 is 11 ( 2) = = .55. 20 Since these two events are independent (the vehicles are selected at random from their respective populations), the relative frequency that both will be rated at most 2 is ( 2) × ( 2) = .125 × .55 = .06875.

53. The sample space is the set of all sets of 4 tents selected from 7; ( ) = (7, 4) = 35. The possible values of are 1, 2, 3, and 4. ( cannot equal 0, since that would require 4 green tents, and there are only 3.) (4, 1) (3, 3) 4 = 35 35 (4, 3) (3, 1) 12 ( = 3) = = 35 35

(4, 2) (3, 2) 18 = 35 35 (4, 4) (3, 0) 1 ( = 4) = = 35 35

( = 1) =

( = 2) =

Probability distribution:

( = )

1

4 35

2

18 35

3

12 35

( 2) = 1 ( = 1) = 1

4

1 35

4 31 = 35 35

.886

54. The sample space is the set of all sets of 4 knapsacks selected from 9; ( ) = (9, 4) = 126. The possible values of are 0, 1, 2, 3, and 4. (5, 0) (4, 4) 1 = 126 126 (5, 2) (4, 2) 60 ( = 2) = = 126 126 (5, 4) (4, 0) 5 ( = 4) = = 126 126

(5, 1) (4, 3) 20 = 126 126 (5, 3) (4, 1) 40 ( = 3) = = 126 126

( = 0) =

( = 1) =

Probability distribution:

( = )

(

0

1 126

1

20 126

2

60 126

3

40 126

4

5 126

2) = ( = 0) + ( = 1) + ( = 2) =

1 20 60 81 + + = 126 126 126 126

.643

55. Answers will vary. 56. Answers will vary.

57. No; for instance, if is the number of times you must toss a coin until heads comes up, then is infinite but not continuous.


Solutions Section 9.1 58. Yes; if is continuous, then it must have infinitely many possible values, since it takes on whole ranges of all values in some interval; for instance, 1.1, 1.01, 1.001, . . . .

59. By measuring the values of for a large number of outcomes and then using the estimated probability (relative frequency). 60. As the measurement classes shrink, there are likely to be fewer outcomes in each class, so you expect the probability of each measurement class to decrease. 61. Here are two examples: (1) Let be the number of times you have read a randomly selected book. (2) Let be the number of days a diligent student waits before beginning to study for an exam scheduled in 10 days' time. 62. Here are two examples: (1) Let be the number of books you have never read from a random collection of 10. (2) Let be the number of days a procrastinating student waits before beginning to study for an exam scheduled in 10 days' time. 63. The bars should be 1 unit wide so that their height is numerically equal to their area. 64. The bars should be 1 unit wide, so that their height is numerically equal to their area. Then the area of any portion of the histogram is the sum of the areas of the individual bars, and therefore equals the corresponding probability. 65. Answers may vary. If we are interested in exact page counts, then the number of possible values is very large, and the values are (relatively speaking) close together, so using a continuous random variable might be advantageous. In general, the finer and more numerous the measurement classes, the more likely it becomes that a continuous random variable could be advantageous. 66. Answers may vary. If we are interested in measurement classes that are fairly wide, then the number of possible classes is small, so using a discrete random variable might be advantageous. In general, the bigger the measurement classes, the fewer possible values we need to use.


Solutions Section 9.2 Section 9.2 1. Throwing a double has the same probability of 1/6 each time you throw the dice, so the experiment is a Bernoulli trial with = 1 6, = 5 6. 2. The probability of a change is always 1/2, so the experiment is a Bernoulli trial with = = 1 2.

3. Not a Bernoulli trial; the probability of success increases every time the experiment results in failure until the first success, after which it changes to 0. 4. Not a Bernoulli trial, as the probability of success increases every time the experiment results in failure. 5. Not a Bernoulli trial, the probability of success decreases every time the experiment results in success and increases every time the experiment results in failure. 6. Not a Bernoulli trial; the experiment will result in a repeating sequence of the same two outcomes.

7. A Bernoulli trial as there is always a 3 1,000 = .003 chance of selecting someone who is infected; = .003, = .997. 8. Not a Bernoulli trial; a positve result would affect the probabilily of subsequent positive results depending on when the experiment is next done. 9. = 5, = .1, = .9; ( = 2) = (5, 2)(.1) 2(.9) 3 = .0729

10. = 5, = .1, = .9; ( = 3) = (5, 3)(.1) 3(.9) 2 = .0081

11. = 5, = .1, = .9; ( = 0) = (5, 0)(.1) 0(.9) 5 = .59049 12. No failures means 5 successes. = 5, = .1, = .9; ( = 5) = (5, 5)(.1) 5(.9) 0 = .00001

13. = 5, = .1, = .9; ( = 5) = (5, 5)(.1) 5(.9) 0 = .00001 14. All failures means 0 successes. = 5, = .1, = .9; ( = 0) = (5, 0)(.1) 0(.9) 5 = .59049

15. = 5, = .1, = .9; ( 2) = ( = 0) + ( = 1) + ( = 2) ( = 0) = (5, 0)(.1) 0(.9) 5 = .59049 ( = 1) = (5, 1)(.1) 1(.9) 4 = .32805 ( = 2) = (5, 2)(.1) 2(.9) 3 = .0729 Therefore, ( 2) = .59049 + .32805 + .0729 = .99144. 16. = 5, = .1, = .9; ( 4) = ( = 4) + ( = 5) ( = 4) = (5, 4)(.1) 4(.9) 1 = .00045 ( = 5) = (5, 5)(.1) 5(.9) 0 = .00001 Therefore, ( 4) = .00045 + .00001 = .00046.

17. = 5, = .1, = .9; ( 3) = ( = 3) + ( = 4) + ( = 5) ( = 3) = (5, 3)(.1) 3(.9) 2 = .0081


Solutions Section 9.2 ( = 4) = (5, 4)(.1) 4(.9) 1 = .00045 ( = 5) = (5, 5)(.1) 5(.9) 0 = .00001 Therefore, ( 3) = .0081 + .00045 + .00001 = .00856.

18. = 5, = .1, = .9; ( 3) = ( = 0) + ( = 1) + ( = 2) + ( = 3) ( = 0) = (5, 0)(.1) 0(.9) 5 = .59049 ( = 1) = (5, 1)(.1) 1(.9) 4 = .32805 2 3 ( = 2) = (5, 2)(.1) (.9) = .0729 ( = 3) = (5, 3)(.1) 3(.9) 2 = .0081 Therefore, ( 3) = .59049 + .32805 + .0729 + .0081 = .99954. 19. = 6, = .4, = 1 = .6; ( = 3) = (6, 3)(.4) 3(.6) 3 = .27648 20. = 6, = .4, = 1 = .6; ( = 4) = (6, 4)(.4) 4(.6) 2 = .13824

21. = 6, = .4, = 1 = .6; ( 2) = ( = 0) + ( = 1) + ( = 2) ( = 0) = (6, 0)(.4) 0(.6) 6 = .046656 ( = 1) = (6, 1)(.4) 1(.6) 5 = .18662 ( = 2) = (6, 2)(.4) 2(.6) 4 = .31104 Therefore, ( 2) = .046656 + .18662 + .31104 = .54432.

22. = 6, = .4, = 1 = .6; ( 1) = ( = 0) + ( = 1) ( = 0) = (6, 0)(.4) 0(.6) 6 = .046656 ( = 1) = (6, 1)(.4) 1(.6) 5 = .18662 Therefore, ( 1) = .046656 + .18662 = .23328.

23. = 6, = .4, = 1 = .6; ( 5) = ( = 5) + ( = 6) ( = 5) = (6, 5)(.4) 5(.6) 1 = .036864 ( = 6) = (6, 6)(.4) 6(.6) 0 = .004096 Therefore, ( 5) = .036864 + .004096 = .04096.

24. = 6, = .4, = 1 = .6; ( 4) = ( = 4) + ( = 5) + ( = 6) ( = 4) = (6, 4)(.4) 4(.6) 2 = .13824 ( = 5) = (6, 5)(.4) 5(.6) 1 = .036864 ( = 6) = (6, 6)(.4) 6(.6) 0 = .004096 Therefore, ( 4) = .13824 + .036864 + .004096 = .1792.

25. = 6, = .4, = 1 = .6; (1 3) = ( = 1) + ( = 2) + ( = 3) ( = 1) = (6, 1)(.4) 1(.6) 5 = .18662 ( = 2) = (6, 2)(.4) 2(.6) 4 = .31104 ( = 3) = (6, 3)(.4) 3(.6) 3 = .27648 Therefore, (1 3) = .18662 + .31104 + .27648 = .77414.

26. = 6, = .4, = 1 = .6; (3 5) = ( = 3) + ( = 4) + ( = 5) ( = 3) = (6, 3)(.4) 3(.6) 3 = .27648 ( = 4) = (6, 4)(.4) 4(.6) 2 = .13824 ( = 5) = (6, 5)(.4) 5(.6) 1 = .036864 Therefore, (3 5) = .27648 + .13824 + .036864 = .45158. 1 3 , = 4 4 We used Excel to generate the distribution. Format: BINOMDIST(x,n,p,0)

27. = 5, =


Solutions Section 9.2

TI-83/84 Plus: binompdf(5,0.25,x)

1 2 , = 3 3 We used Excel to generate the distribution. Format: BINOMDIST(x,n,p,0) 28. = 5, =

TI-83/84 Plus: binompdf(5,1/3,x)

1 2 , = ; ( 2) 3 3 We used Excel to generate the distribution. Format: BINOMDIST(x,n,p,0). (TI-83/84 Plus: binompdf(4,1/3,x)) The resulting values are shown on the histogram, with the portion corresponding to ( 29. = 4, =

(

2)

2) shaded.

.1975 + .3951 + .2963 = .8889

1 3 , = ; ( 1) 4 4 We used Excel to generate the distribution. Format: BINOMDIST(x,n,p,0). (TI-83/84 Plus: binompdf(4,0.25,x)) The resulting values are shown on the histogram, with the portion corresponding to (

30. = 4, =

1) shaded.


Solutions Section 9.2

(

1)

.3164 + .4219 = .7383

31. Take "success" = check their phone within ten minutes of waking. = 5, = .58, = 1 .58 = .42; ( = 2) = (5, 2)(.58) 2(.42) 3 .25 (to two decimal places) 32. Take "success" = of pension age. = 4, = .8, = 1 .8 = .2; ( = 4) = (4, 4)(.8) 4(.2) 0

.41 (to two decimal places).

33. Take "success" = test positive. = 5, = .011, = 1 .011 = .989;. Rather than computing ( 1) directly, use the complementary event = 0 (no successes): ( 1) = 1 ( = 0) = 1 (5, 0) 0 5 = 1 (.011) 0(.989) 5 = 1 (.989) 5 .054. 34. Take "success" = test negative. = 5, = .28, = 1 .28 = .72;. Rather than computing ( 2) directly, use the complementary event ( 2) = 1 ( 1) = 1 (5, 0) 0 5 (5, 1) 1 4 = 1 (.28) 0(.72) 5 5(.28) 1(.72) 4 .430

1 (0 or 1 success):

35. Take "success" = a stock market success. = 10, = .2, = 1 .2 = .8; ( 1) = 1 ( = 0) ( = 0) = (10, 0)(.2) 0(.8) 10 .10737 Therefore, ( 1) 1 10737 = .8926. 36. Take "success" = trading at or above their initial offering price. = 5, = 1 .135 = .865, = .135; ( 4) = ( = 4) + ( = 5) ( = 4) = (5, 4)(.865) 4(.135) 1 .37789 ( = 5) = (5, 5)(.865) 5(.135) 0 .48426 Therefore, ( 4) .37789 + .48426 .8622.

37. Take "success" = selecting a male. = 3, = .5, = 1 .5 = .5 ( 1) = 1 ( = 0) ( = 0) = (3, 0)(.5) 0(.5) 3 = .125 Therefore, ( 1) = 1 125 = .875. 38. Take "success" = drawing a male. = 5, = .5, = 1 .5 = .5 ( 2) = 1 ( 1) ( = 0) = (5, 0)(.5) 0(.5) 5 = .03125 ( = 1) = (5, 1)(.5) 1(.5) 4 = .15625


Solutions Section 9.2 Therefore, ( 2) = 1 (.03125 + .15625) .8125.

39. Take "success" = selecting a defective bag. = 5, = .1, = 1 .1 = .9 a. ( = 3) = (5, 3)(.1) 3(.9) 2 = .0081 b. ( 2) = 1 ( 1) ( = 0) = (5, 0)(.1) 0(.9) 5 = .59049 ( = 1) = (5, 1)(.1) 1(.9) 4 = .32805 Therefore, ( 2) = 1 (.59049 + .32805) = .08146. 40. Take "success" = selecting a defective bag. = 5, = .1, = 1 .1 = .9 ( = 0) = (5, 0)(.1) 0(.9) 5 = .59049 ( = 2) = (5, 2)(.1) 2(.9) 3 = .0729 ( = 4) = (5, 4)(.1) 4(.9) 1 = .00045

( = 1) = (5, 1)(.1) 1(.9) 4 = .32805 ( = 3) = (5, 3)(.1) 3(.9) 2 = .0081 ( = 5) = (5, 5)(.1) 5(.9) 0 = .00001

We get the following probability distribution:

0

1

2

3

4

5

( = ) .59049 .32805 .07290 .0081 .00045 .00001 The event that at least one bag will be defective and at least one will not is (1 4) = .32805 + .0729 + .0081 + .00045 = .4095.

41. Take "success" = spends more than hour a day online. = 10, = .69, = 1 .69 = .31 ( 8) = ( = 8) + ( = 9) + ( = 10) ( = 9) = (10, 9)(.69) 9(.31) 1 .110

Therefore, ( 8)

.222 + .110 + .024

.36.

( = 8) = (10, 8)(.69) 8(.31) 2 .222 ( = 10) = (10, 10)(.69) 10(.31) 0 .024

42. Take "success" = spends at least two hours a day online. = 10, = .46, = 1 .46 = .54 ( 4) = ( = 0) + ( = 1) + ( = 2) + ( = 3) + ( = 4) ( = 0) = (10, 0)(.46) 0(.54) 10 .0021 ( = 2) = (10, 2)(.46) 2(.54) 8 .0688 ( = 4) = (10, 4)(.46) 4(.54) 6 .2331

Therefore, (

4)

( = 1) = (10, 1)(.46) 1(.54) 9 ( = 3) = (10, 3)(.46) 3(.54) 7

.0021 + .0180 + .0688 + .1564 + .2331

.0180 .1564

.48.

43. a. The probability of one chosen man having an accident is Number of accidents 6.90 = .0611 = Number of drivers 113 b. The probability that at least 2 out of 10 Bernoulli trials is a "success" (where success means having an accident) is ( 2) = 1 ( = 0) ( = 1) = 1 (10, 0)(.0611) 0(.9389) 10 (10, 1)(.0611) 1(.9389) 9 .121 44. a. The probability of one chosen woman having an accident is Number of accidents 5.22 = = = .045 Number of drivers 116 b. The probability that at least 2 out of 10 Bernoulli trials is a "success" (where success means having an accident) is


Solutions Section 9.2 ( 2) = 1 ( = 0) ( = 1) = 1 (10, 0)(.045) 0(.955) 10 (10, 1)(.045) 1(.955) 9

.072

45. a. Take "success" = in foreclosure. = 10, = .24, = 1 .24 = .76 ( = 5) = (10, 5)(.24) 5(.76) 5 .0509 b. Following is the distribution generated by the utility at Website → On-line Utilities → Binomial Distribution Utility:

c. The value of with the largest probability is = 2. So, the most likely number of homes to have been in foreclosure was 2 . 46. a. Take "success" = in foreclosure. = 10, = .67, = 1 .67 = .33 ( = 4) = (10, 4)(.67) 4(.33) 6 .0547 b. Following is the distribution generated by the utility at Website → On-line Utilities → Binomial Distribution Utility:

c. The value of with the largest probability is = 7. So, the most likely number of homes to have been current was 7 . 47. Take "success" = computer malfunction. = 3, = .01, = 1 .01 = .99 ( 2) = ( = 2) + ( = 3) ( = 2) = (3, 2)(.01) 2(.99) 1 = .000297 ( = 3) = (3, 3)(.01) 3(.99) 0 = .000001


Solutions Section 9.2 Therefore, ( 2) = .000297 + .000001 = .000298.

48. Take "success" = qualifying for Mensa. = 10, = .02, = 1 .02 = .98 ( 2) = 1 ( 1) ( = 0) = (10, 0)(.02) 0(.98) 10 .81707 ( = 1) = (10, 1)(.02) 1(.98) 9 .16675 Therefore, ( 1) 1 (.81707 + .16675) .0162.

49. Take "success" = answering a question correctly. = 100, = .80 We use technology to compute (75 85) = ( 85) ( 74) TI-83/84 Plus: binomcdf(100,0.8,85)-binomcdf(100,0.8,74) Spreadsheet: BINOMDIST(85,100,0.8,1)-BINOMDIST(74,100,0.8,1) Answer: (75 85) .8321

50. Take "success" = answering a question correctly. = 100, = .80 We use technology to compute ( 90) = ( 100) ( 89) TI-83/84 Plus: binomcdf(100,0.8,100)-binomcdf(100,0.8,89 Spreadsheet: BINOMDIST(100,100,0.8,1)-BINOMDIST(89,100,0.8,1) Answer: ( 90) .0057

51. Take "success" = containing more than 10 grams of fat. = 50, = .43 We use technology to generate the probability distribution: Spreadsheet: BINOMDIST(x,50,0.43,0) TI-83/84 Plus: binompdf(50,0.43,x)

a. Since 43% of all the burgers contain more than 10 grams of fat, we can expect about 0.43 × 50 = 21 of them to contain more than 10 grams of fat. b. The probability that or more patties contain more than 10 grams of fat is ( ) = 1 ( 1) We want this to equal approximately .71: 1 ( 1) .71 ( 1) .29. To answer this question, use the cumulative probability distribution: Spreadsheet: BINOMDIST(x,50,0.43,1) TI-83/84 Plus: binomcdf(50,0.43,x)


Solutions Section 9.2

From the table, 1 = 19, so = 20. There is approximately a 71% chance that a batch of 50 ZeroFat patties contains 20 or more patties with at least 10 grams of fat. c. Graphs

= 50

= 20

The graph for = 50 trials is more widely distributed than the graph for = 20.

52. Take "success" = containing more than 1,000 calories. = 50, = .65 We use technology to generate the probability distribution: Spreadsheet: BINOMDIST(x,50,0.65,0) TI-83/84 Plus: binompdf(50,0.65,x)

a. Since 65% of all the burgers contain more than 1,000 calories, we can expect about 0.65 × 50 them to contain more than 1,000 calories. b. The probability that or more patties contain more than 10 grams of fat is ( ) = 1 ( 1). We want this to equal approximately .73:

33 of


Solutions Section 9.2 1 ( 1) .73 ( 1) .27 To answer this question, use the cumulative probability distribution: Spreadsheet: BINOMDIST(x,50,0.65,1) TI-83/84 Plus: binomcdf(50,0.65,x)

From the table, 1 = 30, so = 31. There is approximately a 73% chance that a batch of 50 ZeroCal patties contains 31 or more patties with more than 1,000 calories. c. Graphs

= 50

= 20

The graph for = 50 trials is more widely distributed than the graph for = 20.

53. Take "success" = bad bulb. = .01, = ? We want ( 1) .5 1 ( = 0) .5 ( = 0) ( , 0)(.01) 0(.99) .5 1 × 1 × .99 .5 .99 .5 Using technology, we compute the values of .99 for = 0, 1, 2, ... :

.5


Solutions Section 9.2

.99

0

1

1

0.99

68

0.50488589

69

0.49983703

The probability first dips below .5 when = 69 trials.

54. Take "success" = bad bulb. = .01, = ? We want ( 2) .5. That is, 1 ( 1) .5 1 [ ( = 0) + ( = 1)] .5 ( = 0) + ( = 1) ( , 0)(.01) 0(.99) + ( , 1)(.01) 1(.99) 1 .5 .99 + (.01)(.99) 1 .5. Using technology, we compute the values of .99 + (.01)(.99) 1 for = 1, 2, ... :

.99 + (.01)(.99) 1

1

1

2

0.9999

167

0.50156121

168

0.49841231

.5

The probability first dips below .5 when = 168 trials.

55. Let be the number of cases of mad cow disease found among the 243,000 tested. Then is a 5 binomial random variable with = 1.1111 × 10 7 and = 243,000. 45,000,000 We are asked to compute the probability of at least one "success": ( 1). ( 1) = 1 ( = 0) = 1 ( , 0) 0(1 ) 0 = 1 (1)(1)(1 1.1111 × 10 7) 243,000 = 1 (.999999889) 243,000 1 .9734 = .0266 Since there is only a 2.66% chance of detecting the disease in a given year, the government's claim seems dubious. 56. Let be the number of cases of mad cow disease found among the 223,000 tested. Then is a 5 binomial random variable with = .000002, = 223,000. 500,000 We are asked to compute the probability of at least one "success": ( 2). ( 2) = 1 ( 1) = 1 [ ( , 0) 0(1 ) 0 + ( , 1) 1(1 ) 1] = 1 [(1)(1)(.999998) 223,000 + (223,000)(.000002)(.999998) 222,999] = 1 .9257 = .0743 Since there is only a 7.42% chance of detecting the disease in a given year, your associate's claim seems


Solutions Section 9.2 dubious. 57. No; in the given scenario, the probability of success depends on the outcome of the previous shot. However, in a sequence of Bernoulli trials, the occurrence of one success does not affect the probability of success on the next attempt. 58. We must assume that the event of scoring on any one attempt is a fixed number; not dependent on whether or not the player has scored earlier in the sequence of shots. 59. No; if life is a sequence of Bernoulli trials, then the occurrence of one misfortune ("success") does not affect the probability of a misfortune on the next trial. Hence, misfortunes may very well not "occur in threes." 60. No; the more times you have checked your battery, the more likely it is to be dead on the next check, since the lifespan of a battery is finite. In a sequence of Bernoulli trials, the probability of "success" does not change for the duration of the experiment. 61. Think of performing the experiment as a Bernoulli trial with "success" being the occurrence of . Performing the experiment times independently in succession would then be a sequence of Bernoulli trials.

62. Wrong; each time you perform the experiment you alter the number of gummy bears and so the probability of choosing a lime one can change. In a sequence of Bernoulli trials, the probability of success must be identical in each trial. 63. The probability of selecting a red marble changes after each selection, as the number of marbles left in the bag decreases. This violates the requirement that, in a sequence of Bernoulli trials, the probability of "success" is contant.

64. Strictly speaking, is not a binomial random variable, since the probability of selecting a defective one does change after each selection. However, since there are so many in the batch, the probability changes by a (very) small amount. Thus, we can approximate the random variable by a binomial one. (For instance, removing the first item changes the probability of "success," .1, by less than .0001.)


Solutions Section 9.3 Section 9.3

1 + 5 + 5 + 7 + 14 = 6 5 To compute the median, arrange the scores in order and take the (average of the) middle score(s). 1, 5, 5, 7, 14 Median = middle score = 5; Mode = most frequent score(s) = 5 1. =

2+ 6+ 6+ 7 1 = 4 5 To compute the median, arrange the scores in order and take the (average of the) middle score(s). 1, 2, 6, 6, 7 Median = middle score = 6; Mode = most frequent score(s) = 6 2. =

2+ 5+ 6+ 7 1 1 = 3 6 To compute the median, arrange the scores in order and take the (average of the) middle score(s). 1, 1, 2, 5, 6, 7 2+ 5 Median = average of middle scores = = 3.5; Mode = most frequent score(s) = 1 2 3. =

3+ 1+ 6 3+ 0+ 5 = 2 6 To compute the median, arrange the scores in order and take the (average of the) middle score(s). 3, 0, 1, 3, 5, 6 1+ 3 Median = average of middle scores = = 2; Mode = most frequent score(s) = Every value 2

4. =

5. In decimal notation, the given scores are: 0.5, 1.5, 4, 1.25 0.5 + 1.5 4 + 1.25 = 0.1875 = 4 To compute the median, arrange the scores in order and take the (average of the) middle score(s). 4, 0.5, 1.25, 1.5 0.5 + 1.25 Median = average of middle scores = = 0.875; Mode = most frequent score(s) = Every value 2 6. In decimal notation, the given scores are: 1.5, 0.375, 1, 2.5 1.5 + 0.375 1 + 2.5 = 0.09375 = 4 To compute the median, arrange the scores in order and take the (average of the) middle score(s). 1.5, 1, 0.375, 2.5 1 + 0.375 Median = average of middle scores = = 0.3125; Mode = most frequent score(s) = Every 2 value 2.5 5.4 + 4.1 0.1 0.1 = 0.2 5 To compute the median, arrange the scores in order and take the (average of the) middle score(s). 5.4, 0.1, 0.1, 2.5, 4.1 Median = middle score = 0.1; Mode = most frequent score(s) = 0.1 7. =

4.2 3.2 + 0 + 1.7 + 0 = 0.54 5 To compute the median, arrange the scores in order and take the (average of the) middle score(s). 8. =


Solutions Section 9.3 3.2, 0, 0, 1.7, 4.2 Median = middle score = 0; Mode = most frequent score(s) = 0 9. Answers may vary. Two examples are: 0, 0, 0, 0, 0, 6 and 0, 0, 0, 1, 2, 3. = 1, Median = 0 for each sample. 10. Answers may vary. Two examples are: 0, 0, 1, 1, 498 and 1, 1, 1, 1, 496. 11. We use the tabular method described in Example 3 in the textbook:

( = )

( = )

0

1

2

3

.5

.2

.2

.1

0

0.2

0.4

0.3

( ) = Sum of entries in the bottom row = 0.9 12. We use the tabular method described in Example 3 in the textbook:

( = )

( = )

1

2

3

4

.1

.2

.5

.2

0.1

0.4

1.5

0.8

( ) = Sum of entries in the bottom row = 2.8 13. We first convert the fractions into decimals, and then use the tabular method described in Example 3 in the textbook:

( = )

( = )

10

20

30

40

.3

.4

.2

.1

3

8

6

4

( ) = Sum of entries in the bottom row = 21 14. We first convert the fractions into decimals, and then use the tabular method described in Example 3 in the textbook:

( = )

( = )

2

4

6

8

.05

.75

.1

.1

0.1

3

0.6

0.8

( ) = Sum of entries in the bottom row = 4.5 15. We use the tabular method described in Example 3 in the textbook:

5

1

( = )

.2 -1

( = )

0

2

5

10

.3

.2

.1

.2

0

-0.3

0

0.2

1

0


Solutions Section 9.3 ( ) = Sum of entries in the bottom row = 0.1 16. We use the tabular method described in Example 3 in the textbook: 20

10

.2 -4

( = )

( = )

0

10

20

30

.4

.2

.1

0

.1

-4

0

1

0

3

( ) = Sum of entries in the bottom row = 4 17. The probability distribution of is

( = )

1

2

3

4

5

6

1/6

1/6

1/6

1/6

1/6

1/6

Using the tabular method described in Example 3, we get

( = )

( = )

1

2

3

4

5

6

1/6

1/6

1/6

1/6

1/6

1/6

1/6

2/6

3/6

4/6

5/6

6/6

( ) = sum of numbers in bottom row = 3.5

18. The probability distribution of is

( = )

1

2

3

4

1/4

1/4

1/4

1/4

Using the tabular method described in Example 3, we get

( = )

( = )

1

2

3

4

1/4

1/4

1/4

1/4

1/4

2/4

3/4

4/4

( ) = sum of numbers in bottom row = 2.5

19. is a binomial random variable with = 2 and = .5. Therefore, ( ) = = 2(.5) = 1.

20. is a binomial random variable with = 3 and = .5. Therefore, ( ) = = 3(.5) = 1.5.

21. We compute the probability distribution of (see the solution to Exercise 27 in Section 8.1) and then use the tabular method of Example 3 to compute ( ):


Freq

( = )

( = )

Solutions Section 9.3 1

2

3

4

5

6

1 36

3 36

5 36

7 36

9 36

11 36

161 36

4.4722

1

1 36

3

6 36

5

15 36

7

28 36

( ) = Sum of entries in the bottom row =

9

45 36

11

66 36

22. We compute the probability distribution of (see the solution to Exercise 28 in Section 8.1) and then use the tabular method of Example 3 to compute ( ):

Freq

( = )

( = )

1

2

3

4

5

6

11 36

9 36

7 36

5 36

3 36

1 36

91 36

2.5278

40 15

2.6667

1

11 36

3

18 36

5

21 36

7

20 36

( ) = Sum of entries in the bottom row =

9

15 36

11

6 36

23. Number of sets of 4 marbles = (6, 4) = 15. If is the number of red marbles, then the possible values of are 2, 3, 4, and (4, ) (2, 4 ) ( = ) = . 15

( = )

( = )

2

6 15 12 15

3

8 15 24 15

4

1 15 4 15

( ) = Sum of entries in the bottom row =

24. Number of sets of 4 marbles = (5, 4) = 5. If is the number of red marbles, then the possible values of are 1, 2, and (2, ) (3, 4 ) ( = ) = . 5

( = )

( = )

1

2

.4

.6

.4

1.2

( ) = Sum of entries in the bottom row = 1.6

25. is a binomial random variable with = 20 and = .1. Therefore,


( ) = = 20(.1) = 2.

Solutions Section 9.3

26. is a binomial random variable with = 30 and = .2. Therefore, ( ) = = 30(.2) = 6.

27. The number of possible hands of 5 cards is (52, 5) = 2,598,960. The possible values of are 0, 1, 2, 3, 4, and (4, ) (48, 5 ) ( = ) = . 2,598,960

0

( = ) .6588

1

2

3

4

.2995

.0399

.0017

.0000

( = ) 0.0000 0.2995 0.0799 0.0052 0.0001

( ) = Sum of entries ( = )

0.3846

28. The number of possible hands of 5 cards is (52, 5) = 2,598,960. The possible values of are 0, 1, 2, 3, 4, 5, and (26, ) (26, 5 ) ( = ) = . 2,598,960

0

( = ) .6588

1

2

3

4

5

.1496

.3251

.3251

.1496

.0253

( = ) 0.0000 0.1496 0.6503 0.9754 0.5982 0.1266

( ) = Sum of entries ( = ) = 2.5

29. The sum of the given Dow changes is 1,500. Therefore, =

1,500 = 150. 10

If we arrange the Dow changes in order, we get 700, 700, 500, 400, 200, 100, 100, 100, 400, 900. 200 + ( 100) The two middle scores are 200 and 100. Therefore, Median = = 150. 2 There were as many days with a change in the Dow above −150 points as there were with changes below that. (See the definition of the median.) 30. The sum of the given Dow changes is 300. Therefore, =

300 = 30. 10

If we arrange the Dow changes in order, we get 500, 300, 200, 200, 100, 100, 200, 200, 400, 900.

100 + ( 100) = 100. 2 There were as many days with a change in the Dow above −100 points as there were with changes below that. (See the definition of the median.) The two middle scores are 100 and 100. Therefore, Median =

31. The sum of the prices is 17,923. Therefore, =

17,923 = 1,792.3. 10

Arranging the scores in order gives: 1,751, 1,757, 1,792, 1,794, 1,794, 1,795, 1,799, 1,800, 1,807, 1,834. 1,794 + 1,795 The two middle scores are 1,794 and 1,795. Therefore, Median = = 1,794.5. 2


Solutions Section 9.3 The mode is the most frequently occurring score: 1,794, which appears twice. Over the 10-business-day period sampled, the price of gold averaged $1,792.30 per ounce. It was above $1,794.50 as many times as it was below that price, and stood at $1,794 per ounce more often than at any other price. 238.0 = 23.80. 10

32. The sum of the prices is 238.0. Therefore, =

Arranging the scores in order gives: 22.3, 22.8, 23.8, 23.8, 23.9, 23.9, 24.1, 24.2, 24.4, 24.8. 23.9 + 23.9 The two middle scores are both 23.9. Therefore, Median = = 23.90. 2 The modes are the most frequently occurring scores: 23.8 and 23.9, each of which appears twice. Over the 10-business day period sampled, the price of silver averaged $23.80 per ounce. It was above $23.9 as many times as it was below that price, and stood at $23.8 and $23.9 per ounce more often than at any other price. 33. a. We use the tabular method described in Example 3 in the textbook:

( = )

( = )

0

1

2

3

4

5

6

7

8

9

.01

.05

.14

.24

.25

.18

.09

.02

.01

.01

0

.05

.28

.72

1.00

.90

.54

.14

.08

.09

= ( ) = sum of values ( = ) = 3.8. The average driver will have 3.8 accidents over the course of their lifetime. b. ( < ) = ( < 3.8) = .01 + .05 + .14 + .24 = .44 ( > ) = ( > 3.8) = 1 .44 = .56, and is thus larger. Most drivers will have more than the average number of crashes. 34. a. We use the tabular method described in Example 3 in the textbook:

( = )

( = )

1

2

3

4

5

6

7

8

9

10

.01

.01

.01

.02

.07

.17

.26

.26

.15

.04

.01

.02

.03

.08

.35

1.02

1.82

2.08

1.35

.40

= ( ) = sum of values ( = ) 7.2 There are, on average, 7.2 record stores in a city with more than 100,000 inhabitants. b. ( < ) = ( < 7.2) = .01 + .01 + .01 + .02 + .07 + .17 + .26 = .55 ( > ) = ( > 7.2) = 1 .55 = .45, and is thus smaller. Most cities have fewer than the average number of record stores.

35. a. For the values of we use the rounded midpoints of the measurement classes given: (35 + 44.9) 2 (65 + 74.9) 2

40 70

(45 + 54.9) 2 (75 + 84.9) 2

50 80

(55 + 64.9) 2 (85 + 94.9) 2

60 90

To obtain the probabilities, divide each frequency by the sum of all the frequencies, 50:

( = )

40

50

60

70

80

90

.02

.48

.34

.16

0

0

To calculate ( ), we use the tabular method described in Example 3 in the textbook:


( = )

( = )

Solutions Section 9.3 40

50

60

70

80

90

.02

.48

.34

.16

0

0

0.8

24

20.4

11.2

0

0

= ( ) = sum of values ( = ) = 56.4 On average, a randomly selected U.S. state had 56.4% of its population fully vaccinated. b. The fraction of states with vaccination rates below the mean is ( < 56.4) = .02 + .48 = .5, so 50 of states had a vaccination rate below the mean. 36. a. For the values of we use the rounded midpoints of the measurement classes given: (35 + 44.9) 2 (65 + 74.9) 2

40 70

(45 + 54.9) 2 (75 + 84.9) 2

50 80

(55 + 64.9) 2 (85 + 94.9) 2

60 90

To obtain the probabilities, divide each frequency by the sum of all the frequencies, 50:

( = )

40

50

60

70

80

90

0

.06

.42

.24

.22

.06

40

50

60

70

80

90

0

.06

.42

.24

.22

.06

0

3

25.2

16.8

17.6

5.4

To find ( ), we use the tabular method described in Example 3 in the textbook:

( = )

( = )

= ( ) = sum of values ( = ) = 68.0 On average, a randomly selected U.S. state had 68% of its population vaccinated with at least one dose. b. The fraction of states with vaccination rates above the mean is ( > 68.0) = .24 + .22 + .06 = .52, so 52 of states had a vaccination rate above the mean. 37. Using the rounded midpoints of the given measurement classes, we get the following table. (The probabilities are obtained by dividing the given frequencies by their sum, 75 and then rounding to 2 decimal places.)

( = )

( = )

5

10.5

16

20

26

32.5

45

.16

.37

.21

.13

.08

.01

.03

0.8

3.885 3.36

2.6

2.08 0.325 1.35

( ) = sum of values ( = ) = 14.4 Interpretation: The average age of a a U.S. resident who attended an educational institution in 2019 was 14.4.


Solutions Section 9.3 38. Using the rounded midpoints of the given measurement classes, we get the following table. (The probabilities are obtained by dividing the given frequencies by their sum, 79, and then rounding to 2 decimal places.)

( = )

( = )

5

10.5

16

20

26

32.5

45

.16

.35

.20

.13

.09

.03

.04

0.8

3.675

3.2

2.6

2.34 0.975

1.8

( ) = sum of values ( = ) = 15.39 Interpretation: The average age of a U.S. resident who attended an educational institution in 2010 was approximately 15.4. 39. For the values of , use the rounded midpoints of the measurement classes given: (0 + 14) 2 = 7 (15 + 29) 2 = 22 (30 + 64) 2 = 47 (65 + 74) 2 70 To estimate the average age, we use the tabular method described in Example 3 in the textbook:

Freq

7

22

47

70

65

64

108

13

.256

.432

.052

( = ) .260

( = ) 1.82 5.632 20.304 3.64

Average age = expected value of = sum of values ( = )

31.4.

40. For the values of , use the rounded midpoints of the measurement classes given: (0 + 14) 2 = 7 (15 + 29) 2 = 22 (30 + 64) 2 = 47 (65 + 74) 2 70 To estimate the average age, we use the tabular method described in Example 3 in the textbook:

Freq

7

22

47

70

49

54

120

27

( = ) 0.196 0.216 0.480 0.108

( = ) 1.372 4.752 22.56 7.56

Average age = expected value of = sum of values ( = )

36.2.

41. Using the rounded midpoints of the age brackets, we get the following table:

20

30

40

50

60

70

80

( = ) 4.44

.229

.173

.149

.125

.073

.029

6.87

6.92

7.45

7.5

5.11

2.32

( = ) .222

The sum of the entries in the bottom row is 40.61, so, rounding to the nearest year, the average age is 41. 42. Using the rounded midpoints of the age brackets, we get the following table:


Solutions Section 9.3

( = ) .175

30

40

50

60

70

80

.215

.170

.154

.146

.090

.050

3.5

6.45

6.8

7.7

8.76

6.3

4

( = )

20

The sum of the entries in the bottom row is 43.51, so, rounding to the nearest year, the average age is 44. 43. The midpoints of the ranges are (3.95 + 4.00) 2 4.0, (3.53 + 3.94) 2 3.75, and (3.00 + 3.52) 2 = 3.25. Using the rounded mid-points of the given ranges and the technique in the textbook, we obtain the following:

4.0

3.75

Freq 10,638 4,693

313

.68

.02

( = )

( = )

7

2.72

.3

3.25

1.125 0.065

Average GPA = expected value of = sum of values ( = ) accepted student was about 3.9.

3.9. The average GPA of an

44. The midpoints of the ranges are (3.95 + 4.00) 2 4.0, (3.53 + 3.94) 2 3.75, and (3.00 + 3.52) 2 = 3.25. Using the rounded mid-points of the given ranges and the technique in the textbook, we obtain the following:

7

4.0

3.75

3.25

Freq 16,581 50,834 25,818

( = )

( = )

.18

0.55

0.28

0.72

2.0625

0.91

Average GPA = expected value of admitted applicant was about 3.7.

= sum of values ( = )

3.7. The average GPA of a non-

45. The probabilities in the tables below are obtained by dividing the given frequencies by the sum, 16 for and 20 for :

1

( = ) .125

( = ) 0.125

2

3

4

5

0

.75

.0625 .0625

0

2.25

0.25 0.3125

( ) = sum of values in the bottom row = 2.9375

( = )

1

2

3

4

5

.05

.35

.45

.1

.05

0.7

1.35

0.4

0.25

( = ) 0.05

( ) = sum of values in the bottom row = 2.75

Because small cars ( ) have a higher average rating, small cars had a better predicted reliability.


Solutions Section 9.3 46. The probabilities in the tables below are obtained by dividing the given frequencies by the sum, 12 for and 12 for :

1

( = ) .0833

( = ) 0.0833

2 0 0

3

4

.5833 .1667

5 .1667

( = )

1.75 0.6667 0.8333 ( = )

( ) = sum of values in the bottom row = 3.3333

1

2

3

4

5

0

.4167

.25

.3333

0

0

0.8333 0.75 1.3333

0

( ) = sum of values in the bottom row = 2.9167

Because midsize cars ( ) have a higher average rating, midsize cars had a better predicted reliability. 47. The probabilities in the tables below are obtained by dividing the given frequencies by their sums: Small Cars:

Midsize Cars:

1

( = ) .125

( = ) 0.125

( ) = 2.9375

2

3

4

0

.75

0

2.25

5

.0625 .0625

( =

( ) = 2.6

) )

1

( = ) .0833

0.25 0.3125 ( = ) 0.0833 ( ) = 3.3333

Full-sized Pickup Trucks: ( =

1

2

3

4

5

.1

.45

.25

.15

.05

0.1

0.9

0.75

0.6

0.25

Of the three, midsize cars ( ) were most reliable.

2

3

4

5

0

.5833 .1667

.1667

0

1.75 0.6667 0.8333


Solutions Section 9.3 48. The probabilities in the tables below are obtained by dividing the given frequencies by their sums: Compact SUVs:

( = )

Midsize SUVs:

1

2

3

4

5

.05

.35

.45

.1

.05

0.7

1.35

0.4

0.25

1

2

3

4

5

0

.75

.125

.125

0

0

1.5

0.375

0.5

0

( = ) 0.05

( ) = 2.75

( = )

( = )

( ) = 2.9167

1

2

3

4

5

0

.4167

.25

.3333

0

0

0.8333 0.75 1.3333

0

Large SUVs:

( = )

( = )

( ) = 2.375

Of the three, midsize SUVs ( ) were most reliable.

49. Either you lose $1 ( = 1) or win $1 ( = 1).

20 . 38 18 There are 18 winning numbers out of 38, so ( = 1) = . 38 There are 20 losing numbers out of 38, so ( = 1) =

( = )

1 20 38

( = )

( ) =

20 38

20 18 2 + = 38 38 38

1

18 38 18 38

0.53

Expect to lose 53¢.

50. Either you lose $1 ( = 1) or win $17 ( = 17). There are a total of (38, 2) = 703 pairs of numbers to select. Of these, 37 include the winning number. So, 37 1 ( = 1) = . = 703 19 1 18 If you don't win, you lose, so ( = 1) = 1 = 19 19

( = )

1 18 19

( = )

( ) =

18 19

18 17 1 + = 19 19 19

17 1 19 17 19

0.53

Expect to lose 53¢.


Solutions Section 9.3 51. The given experiment consists of = 40 Bernoulli trials with = .69. Thus, the expected number of children spending more than an hour a day online is = = 40 × .69 = 27.6.

52. The given experiment consists of = 30 Bernoulli trials with = .46. Thus, the expected number of teenagers spending at least two hours a day online is = = 30 × .46 = 13.8.

53. a. The given experiment consists of = 20 Bernoulli trials with = .10. Therefore, the expected number of defective air bags is = = 20(.10) = 2. b. = .10, = 12 and is unknown. 12 = 12 = .10 = = 120 air bags .10

54. a. The given experiment consists of = 20 Bernoulli trials with = .12. Therefore, the expected number of guests Spider will bite is = = 20 × .12 = 2.4. b. = .12, = 6 and is unknown 6 = 6 = .12 = = 50 guests .12

55. The probability distribution for = number of red tents is derived in the solution to Exercise 51 in Section 1 of this chapter. Here, we add a new row to compute the expected value:

( = )

( = )

1

4 35 4 35

2

18 35 36 35

3

4

12 35

1 35

80 35

2.2857 tents

2

3

36 35

( ) = Sum of bottom row entries =

4 35

56. The probability distribution for = number of green knapsacks is derived in the solution to Exercise 52 in Section 1 of this chapter. Here, we add a new row to compute the expected value:

( = )

( = )

0

1 126 0

1

20 126 20 126

60 126 120 126

( ) = Sum of bottom row entries =

280 126

40 126 120 126

4

5 126 20 126

2.2222 tents

57. According to Exercise 37 in the section on Probability and counting techniques, the probability of winning this bet is (8! × 2 8) 16!. Your expected winnings are therefore 8! × 2 8 8! × 2 8 1,000,000 × 1 × �1 � 0.51. 16! 16! So, you expect to lose about 51¢ on this bet, on average. 58. According to Exercise 38 in the section on Probability and counting techniques, the probability of winning this bet is (4! × 2 4) 8!. Your expected winnings are therefore 4! × 2 4 4! × 2 4 100 × 1 × �1 � 0.04. 8! 8!


Solutions Section 9.3 So, you expect to lose about 4¢ on this bet, on average. 59. According to Exercise 39 in the section on Probability and counting techniques, the probability of someone picking all the correct winners at random was 1 2 63. If was the expected number of winners, then was a binomial variable with = 1 2 63 and = 50,000,000, hence the expected value of was 50,000,000 = . 2 63 The expected payout was therefore 50,000,000 1,000,000,000 × 0.005. 2 63 In other words, Quicken Loans expected to pay out about half a cent. Put another way, they really didn't expect to have to pay out anything.

60. Let be the number of upsets. There are 60 games played in the first four rounds and we are assuming that the probability of an upset is a fixed number in each, and that the results of the games are independent. Therefore, is a binomial random variable with = 60 and unknown. We do know that 20 = ( ) = = 60 , so 20 1 = = . 60 3 This is the probability of the higher-ranked team losing, so the probability of it winning is 2/3. 61. Let = rate of return of Fastforward Funds, and let = rate of return of SolidState Securities. The following worksheets show the computation of the expected values of and :

From the worksheets, we read off the following expected rates of return: FastForward: 3.97%; SolidState: 5.51%. SolidState gives the higher expected return.

62. Let = rate of return of Fastforward Funds, and let = rate of return of SolidState Securities. The following worksheets show the computation of the expected values of and :

From the worksheets, we read off the following expected rates of return:


Solutions Section 9.3 FastForward: 30.51; SolidState: 29.19. SolidState gives the lower expected loss.

63. If a driver wrecks a car, the net cost to the insurance company is $100,000 $5,000 = $95,000, so = 95,000. If a driver does not wreck a car, the net profit for the insurance company is the premium: $5,000, so = 5,000. To compute the probability distribution, use the following tree:

Probability of no wreck is .9 4 = .6561, Probability of a wreck is 1 .9 4 = .3439, 95, 000

( = )

.3439

5000 .6561

( = ) 32,670.5 3280.5

( ) = sum of entries in bottom row = 29,390. The company can expect to lose $29,390 per driver. 64. In the event of a fatal plane crash, the insurance company loses $100,000 $20 = $99,980, so = 99,980. In the event of a nonfatal plane crash, the insurance company loses $25,000 $20 = $24,980, so = 24,980. In the event of no crash, the insurance company earns a profit of $20, so = 20 The probability of a fatal air crash is .000 004 1 × .1 = .000 000 41. The probability of a nonfatal air crash is .000 004 1 × .9 = .000 003 69. The probability of no crash is 1 .000 004 1 = .999 995 9

99,980

24,980

-0.04099

-0.09218

20

( = ) .000 000 41 .000 003 69 .999 995 9

( = )

( ) = sum of entries in bottom row

19.99992

19.87. The company can expect a profit of $19.87 per passenger.

65. Since there are as many scores above the median as below the median, the correct choice is (A): The median and mean are equal. 66. Since the most frequently occurring score is the mode, the correct choice is (C): The mode and median are equal. 67. He is wrong; for example, the collection 0, 0, 300 has mean 100 and median 0.


Solutions Section 9.3 68. She is wrong; for example, the collection 1, 1, 100, 101, 102 has mode 1 and median 100. 69. No; the expected number is the average number of times you will hit the bull's-eye per 50 shots; the average of a set of whole numbers need not be a whole number. 70. Wrong; the most likely score to occur is the mode. The expected value gives the average score expected. 71. Not necessarily; it might be the case that only a small fraction of people in the class scored better than you but received exceptionally high scores that raised the class average. Suppose, for instance, that there are 10 people in the class. Four received 100%, you received 80%, and the rest received 70%. Then the class average is 83%, 5 people have lower scores that you, but only 4 have higher scores. 72. Wrong; it might be the case that there were some very low scores that brought the average down without affecting the median. Suppose, for instance, that there are 10 people in the class. Six get 100%, you got 80%, and the rest got 0%. Then the class average is 68%, considerably lower than your score. 73. No; the mean of a very large sample is only an estimate of the population mean. The means of larger and larger samples approach the population mean as the sample size increases.

74. If is a random variable, you can estimate ( ) by randomly sampling a large number of values of , and then taking their mean. 75. Wrong; the statement attributed to President Bush asserts that the mean tax saving would be $1,000, whereas the statements referred to as "The Truth" suggest that the median tax saving would be close to $100 (and that the 31st percentile would be zero). 76. Wrong; if the median size of a habitat is at least 400 square feet, then at least half of them are 400 square feet or more. It follows that the mean cannot be less than 200 square feet; even if the rest of them had zero square feet, the mean would be at least 200 square feet.

77. Select a U.S. household at random, and let be the income of that household. The expected value of is then the population mean of all U.S. household incomes.

78. A sample mean of measurements of is the average value of a sample of measurements of . The expected value is the limiting value of the sample mean for larger and larger samples. For example, consider the experiment of selecting a resident of Johannesburg at random, and let be that person's age. The average age of a sample of 10 residents of Johannesburg may be 38 (a sample mean) whereas the average age of all Johannesburg citizens may be 42.3 (the expected value of ).


Solutions Section 9.4 Section 9.4

1. For ease of computation, we arrange the data in a table. The first row lists the values of , the second row lists the numbers ( ), and the third row lists their squares. The totals are shown in the last column.

( )

( ) 2

30 = 6 = 5

1

5

5

7

1

1

7 1

8

0

49

1

1

1

64

116

! ( ) = 116

14

30

=

2

2

! ( ) 2 1

=

116 = 29 5 1

= " 2

5.39

46 = 11.5 5 1

= " 2

3.39

= " 2

3.52

= " 2

3.35

2. For ease of computation, we arrange the data in a table. The first row lists the values of , the second row lists the numbers ( ), and the third row lists their squares. The totals are shown in the last column.

( )

=

( ) 2 20 = 4 5

2

2 4

6

6

7

2

2

3

4

4

9

! ( ) 2 = 46

1

2 =

5

20

25

46

0

! ( ) 2 1

=

3. Below are the data arranged in a table, with the totals in the last column.

( )

=

( ) 2 18 = 3 6

2

1 1

5

6

7

2

3

4

4

9

16

! ( ) 2 = 62

1

1

4 16 2 =

4

18

16

62

0

! ( ) 2 1

=

62 = 12.4 6 1

4. Below are the data arranged in a table, with the totals in the last column.

( )

=

( ) 2 12 = 2 6

3

1

1 1

1

6

1

16

3

4

0

5

2

25

4

! ( ) 2 = 56

2 =

5

12

3

0

9

56

! ( ) 2 1

=

56 = 11.2 6 1

5. In the following table, we first converted all the fractions to decimals.

0.5

1.5

4

1.25

( ) 0.6875 1.6875 3.8125 1.4375

0.75 0

( ) 2 0.4727 2.8477 14.5352 2.0664 19.9219


=

0.75 = 0.1875 4

Solutions Section 9.4 ! ( ) 2 19.9219 2 = 1

! ( ) 2

19.9219 4 1

6.64

= " 2

2.58

6. In the following table, we first converted all the fractions to decimals.

1.5

1

0.375

2.5

0.375

( ) 1.59375 0.28125 1.09375 2.40625

=

( ) 2

2.54

0.375 = 0.09375 4

0.0791

1.1963

5.79

! ( ) 2

9.6055

2 =

0 9.6055

! ( ) 2 1

9.6055 4 1

3.20

= " 2

1.79

7. Below are the data arranged in a table, with the totals in the last column.

( )

=

5.4

2.5

5.6

2.3

0.1 0.1

4.1

0.3 0.3

3.9

( ) 2 5.29 31.36 15.21 0.09 1 = 0.2 5

! ( ) 2

1 0

0.09 52.04

52.04

! ( ) 2

2 =

1

52.04 = 13.01 5 1

=

= " 2

3.61

8. Below are the data arranged in a table, with the totals in the last column.

4.2

( )

3.66

3.2

0

3.74

1.7

0.54

0

0.54

1.16

2.7 0

( ) 2 13.3956 13.9876 0.2916 1.3456 0.2916 29.312

2.7 = 0.54 = 5

! ( )

29.312

2

= 2

! ( ) 2 1

9. We use the tabular method described in Example 3 in the textbook:

( = )

( = )

0

1

2

3

.5

.2

.2

.1

0

0.2

0.4

0.3

0.1

1.1

2.1

0.01

1.21

4.41

= Sum of entries in the bottom row = 0.9 0.9

( ) 2 0.81

( ) 2 ( = ) 0.405 0.002 0.242 0.441

# 2 = Sum of entries in bottom row = 1.09

# = "# 2

1.04

=

29.312 5 1

7.3

= " 2

2.71


Solutions Section 9.4 10. We use the tabular method described in Example 3 in the textbook:

( = )

( = )

1

2

3

4

.1

.2

.5

.2

0.1

0.4

1.5

0.8

= Sum of entries in the bottom row = 2.8 1.8 0.8

( ) 2 3.24

0.64

0.2

1.2

0.04

1.44

( ) 2 ( = ) 0.324 0.128 0.02 0.288

# 2 = Sum of entries in bottom row = 0.76

# = "# 2

0.87

11. We first convert the fractions into decimals, and then use the tabular method described in Example 3 in the textbook:

10

20

30

40

.3

.4

.2

.1

3

8

6

4

11

1

( ) 2 ( = ) 36.3

( = )

( = )

= Sum of entries in the bottom row = 21 ( ) 2 121

9

19

1

81

361

0.4

16.2

36.1

# 2 = Sum of entries in bottom row = 89

# = "# 2

9.43

12. We first convert the fractions into decimals, and then use the tabular method described in Example 3 in the textbook:

( = )

( = )

2

4

6

8

.05

.75

.1

.1

0.1

3

0.6

0.8

2.5

0.5

6.25

0.25

= Sum of entries in the bottom row = 4.5 ( ) 2

1.5

3.5

2.25 12.25

( ) 2 ( = ) 0.3125 0.1875 0.225 1.225

# 2 = Sum of entries in bottom row = 1.95

# = "# 2

1.40


Solutions Section 9.4 13. We use the tabular method described in Example 3 in the textbook:

5

1

( = )

.2

.3

( = )

1

0.3

0

2

5

10

.2

.1

.2

0

0

0.2

1

0

0.1

2.1

5.1

0.01

4.41 26.01 102.01

= Sum of entries in the bottom row = 0.1 4.9 0.9

( ) 2 24.01 0.81

10.1

( ) 2 ( = ) 4.802 0.243 0.002 0.441 5.202

# 2 = Sum of entries in bottom row = 10.69

# = "# 2

0

3.27

14. We use the tabular method described in Example 3 in the textbook: 20

10

.2

.4

0

10

20

30

4

.2

.1

0

.1

0

1

0

3

16

6

4

14

24

( ) 2 ( = ) 51.2

36

16

196

576 1,156

14.4

3.2

19.6

0

( = )

( = )

4

= Sum of entries in the bottom row = 4 ( ) 2 256

# 2 = Sum of entries in bottom row = 200.4

# = "# 2

34

115.6

14.28

15. The probability distribution and expected value calculated in Exercise 17 of Section 8.3:

( = )

( = )

1

2

3

4

5

6

1/6

1/6

1/6

1/6

1/6

1/6

1/6

2/6

3/6

4/6

5/6

6/6

0.5

1.5

2.5

= sum of numbers in bottom row = 3.5

2.5 1.5 0.5

( ) 2 6.25

2.25

0.25

0.25

2.25

6.25

# 2 = Sum of entries in bottom row

2.92

# = "# 2

1.71

( ) 2 ( = ) 1.042 0.375 0.042 0.042 0.375 1.042

16. The probability distribution and expected value calculated in Exercise 18 of Section 8.3:


( = )

( = )

Solutions Section 9.4 1

2

3

4

1/4

1/4

1/4

1/4

1/4

2/4

3/4

4/4

= sum of numbers in bottom row = 2.5 1.5

( ) 2 2.25

0.5 0.25

0.5

1.5

0.25

2.25

( ) 2 ( = ) 0.562 0.0625 0.0625 0.5625

# 2 = Sum of entries in bottom row = 1.25

# = "# 2

1.12

17. is a binomial random variable with = 2 and = .5. Therefore, = ( ) = = 2(.5) = 1 # 2 = = 2(.5)(.5) = 0.5 # = "# 2 18. is a binomial random variable with = 3 and = .5. Therefore, = ( ) = = 3(.5) = 1.5 # 2 = = 3(.5)(.5) = 0.75

0.71.

# = "# 2

0.87.

19. The probability distribution was calculated in Exercise 27 of Section 8.1. To continue the calculation, we use decimal approximations of the fractions:

1

2

3

4

5

6

.0833

.1389

.1944

.25

.3056

= Sum of entries in the bottom row

4.47

( = ) .0278

( = ) 0.0278 0.1667 0.4167 0.7778 1.25 1.8333 3.4722 2.4722 1.4722 0.4722 0.5278 1.5278

( ) 2 12.0563

( ) 2 ( = ) 0.3349

6.1119

2.1674

0.223

0.5093

0.301

0.0434 0.0696 0.7132

# 2 = Sum of entries in bottom row

1.9715

# = "# 2

0.2785 2.3341 1.40

20. The probability distribution was calculated in Exercise 28 of Section 8.1. To continue the calculation, we use decimal approximations of the fractions:

1

( = ) .3056

( = ) 0.3056

2

3

4

5

6

.25

.1944

.1389

.0833

.0278

0.5

0.5833 0.5556 0.4167 0.1667

= Sum of entries in the bottom row

2.53

1.5278 0.5278 0.4722 1.4722 2.4722 3.4722

( ) 2 2.3341

( ) 2 ( = ) 0.7132

0.2785

0.223 2.1674 6.1119 12.0563

0.0696 0.0434 0.301 0.5093 0.3349


# = Sum of entries in bottom row

1.9715

# = "# 2

1.40

Solutions Section 9.4

2

21. The probability distribution was calculated in Exercise 23 of Section 8.3. To continue the calculation, we use decimal approximations of the fractions:

2

( = )

( = )

3

4

.4

.5333 .0667

0.8

1.6

0.2667

= Sum of entries in the bottom row =

40 15

2.67

0.6667 0.3333 1.3333

( ) 2 0.4444

0.1111 1.7778

( ) 2 ( = ) 0.1778 0.0593 0.1185

# 2 = Sum of entries in bottom row

0.36

# = "# 2

0.60

# = "# 2

0.49

22. Number of sets of 4 marbles = (5, 4) = 5. If is the number of red marbles, then the possible values of are 1, 2, and (2, ) (3, 4 ) ( = ) = . 5

( = )

( = )

1

2

.4

.6

.4

1.2

= Sum of entries in the bottom row = 1.6 0.6

0.4

( ) 2 0.36

0.16

( ) 2 ( = ) 0.144 0.096

# 2 = Sum of entries in bottom row = 0.36

23. is a binomial random variable with = 20 and = .1. Therefore, = ( ) = = 20(.1) = 2 # 2 = = 20(.1)(.9) = 1.8 # = "# 2 24. is a binomial random variable with = 30 and = .2. Therefore, = ( ) = = 30(.2) = 6 # 2 = = 30(.2)(.8) = 4.8 # = "# 2 25. a. Below are the data arranged in a table, with the totals in the last column.

( )

( ) 2

3 0 0

2

0

1

3

9

1

9

36

6

1

2

15

4

50

0

1.34 2.19.


Solutions Section 9.4 15 50 2 ! ( 2 = 12.5 = " 2 3.54 = 3 = ) = 50 5 5 1 b. The empirical rule states that approximately 68% of the class will rank you between and = 3 3.54 = 0.54 + = 3 + 3.54 = 6.54. That is, in the interval [0, 6.54] (We replaced the negative score by 0, since no rankings can be negative.) We must assume that the population distribution is bell shaped and symmetric. 26. a. Below are the data arranged in a table, with the totals in the last column.

2

8

7

10

5

8

( ) 4.6667 1.3333 0.3333 3.3333 1.6667 1.3333

40 0

( ) 2 21.7778 1.7778 0.1111 11.1111 2.7778 1.7778 39.3333

40 39.3333 6.67 ! ( 2 = 7.8667 = " 2 2.80 ) 2 39.3333 6 6 1 b. The empirical rule states that approximately 95% of the class will rank Sally between and 2 6.67 2(2.80) = 1.07 + 2 6.67 + 2(2.80) = 12.27. That is, in the interval [1.07, 10] (We replaced the 12.27 by 10, since no rankings can be higher than 10.) We must assume that the population distribution is bell shaped and symmetric. =

27. a. By Chebyshev's rule, at least 3/4 of the scores must fall within two standard deviations of the mean: 2 = 1,792 47.2 = 1,744.8 and + 2 = 1,792 + 47.2 1,839.2. b. As $1,840 is outside the range specified by part (a), the price could not have been higher more than twice; otherwise 30% or more of the prices would be outside the range, contradicting Chebyshev's rule. 28. a. The empirical rule states that approximately 68% of the data will fall within one standard deviation of the mean: = 23.80 0.74 = 23.06 and + = 23.80 + 0.74 = 24.54.b. By part (a), one would have expected about 32% of the prices to be outside the range given in part (a): less than $ 23.06 or greater than $ 24.54. But at most one of the prices fall in those ranges, so we conclude that the distribution cannot be bellshaped and symmetric. 29. a. Below are the data arranged in a table, with the totals in the last column. 400

( ) 250

500 350

200 50

700 550

100 50

900 1050

100 50

700 550

400 550

100

1, 500

50

0

( ) 2 62,500 12,2500 2,500 302,500 2,500 1,102,500 2,500 302,500 302,500 2,500 2,205,000

2,205,000 1,500 ! ( 2 = = " 2 495 = 150 = 245,000 ) 2 = 2,205,000 10 10 1 b. The empirical rule states that approximately 68% of the data will fall between 150 495 and + 150 + 495 = 345. Thus, 100 68 = 32 of the time the data fall outside that range, and so, by symmetry, 16% of the data will be below 645. That is, 16% of the time, the Dow will drop by more than 645 points. To obtain the actual percentage of times the Dow fell by more than 645 points, count how many times this actually happened: twice. Since 2 scores is 20% of the original 10, we conclude that the Dow actually fell by more than 645 points 20% of the time. =

30. a. Below are the data arranged in a table, with the totals in the bottom row.


100

( ) 130

400 370

200 230

Solutions Section 9.4

500 530

200 170

300 330

200 230

900 870

100 130

200

300

170

0

( ) 2 16,900 136,900 52,900 280,900 28,900 108,900 52,900 756,900 16,900 28,900 1,481,000

1,481,000 300 ( 2 = 164,555.556 = 2 406 = 30 ) 2 = 1,481,000 10 10 1 b. The empirical rule states that approximately 95% of the data will fall between 2 30 2(406) = 782 and + 2 30 + 2(406) = 842. Thus, 100 95 = 5 of the time the data fall outside that range, and so, by symmetry, 2.5% of the data will be above 842. That is, 2.5% of the time, the Dow will rise by more than 842 points. To obtain the actual percentage of times the Dow rose by more than 842 points, count how many times this actually happened: once. Since only one of the 10 scores (that is, 10% of the scores) is above 842, we conclude that the Dow actually rose by more than 842 points 10% of the time. =

31. a. Below is a worksheet computation of the variance: From the worksheet, 2 26.8407 = 2 5.18. b. Chebyshev's inequality predicts that at least 8/9 of the scores will fall between 3 29.07 3(5.18) 13.5 and + 3 29.07 + 3(5.18) 44.6. That is, they will fall in the interval [13.5, 44.6]. c. Every single score, or 100% of the scores fall in the range [13.5, 44.61]. Since the empirical rule predicts that 99.7% of the scores will fall in the given range, the empirical rule is a more accurate predictor than Chebyshev's inequality in this case.


Solutions Section 9.4 32. a. Below is a worksheet computation of the variance: From the worksheet, 2 = 29.6 = 2 5.44 b. Chebyshev's inequality predicts that at least 75% (3/4) of the scores will fall between 2 20.5 2(5.44) 9.6 and + 2 20.5 + 2(5.44) 31.4. That is, they will fall in the interval [9.6, 31.4]. c. A total of 15 of the 16 scores fall the range [9.6, 31.4] (only the single score of 37 falls outside it). Thus, the fraction of scores falling in the range is 15/16 or 93.75%. Since the empirical rule predicts that 95% of the scores will fall in the given range, the empirical rule is a more accurate predictor than Chebyshev's inequality in this case.

33. We use the tabular method of arranging the data described in Example 3.

( = )

( = )

0

1

2

3

4

.4

.1

.2

.2

.1

0

0.1

0.4

0.6

0.4

0.5

1.5

2.5

0.25

2.25

6.25

= Sum of entries in the bottom row = 1.5

1.5 0.5

( ) 2 2.25

( ) 2 ( = )

0.9

0.25

0.025 0.05

0.45 0.625

= Sum of entries in bottom row = 2.05 = 2 1.43 The range of within two standard deviations of the mean is given by Lower limit = 2 2.05 2(1.43) = 0.81 Upper limit = + 2 2.05 + 2(1.43) = 4.91 All (100%) of the values of are within the interval [ 0.81, 4.91]. Therefore, 100% of malls have a number of movie theater screens within two standard deviations of . 2

34. We use the tabular method of arranging the data described in Example 3.

( = )

( = )

0

1

2

3

4

.5

.1

.2

.1

.1

0

0.1

0.4

0.3

0.4

= Sum of entries in the bottom row = 1.2


1.2 0.2

( ) 2 1.44

0.04

Solutions Section 9.4 0.8

1.8

2.8

0.64

3.24

7.84

( ) 2 ( = ) 0.72 0.004 0.128 0.324 0.784

# 2 = Sum of entries in bottom row = 1.96 # = "# 2 1.4 The range of within two standard deviations of the mean is given by Lower limit = 2# 1.2 2(1.4) = 1.6 Upper limit = + 2# 1.2 + 2(1.4) = 4 Eleven (100%) of the values of are within the interval [ 0.81, 4.91]. Therefore, is within two standard deviations of for 100% of students. 35. a. The rounded midpoints of the measurement classes (ages) are: (15 + 24.9) 2 20 (25 + 54.9) 2 40 (55 + 64.9) 2 60 In the following table, the probabilities have been rounded to two decimal places.

$

20

40

60

Total

10

54

14

78

0.69

0.18

1

2.6

27.6

10.8

41

-21

-1

19

441

1

361

( = ) 0.13

( = )

( ) 2

( ) 2 ( = ) 57.33 0.69 64.98 123 Expected value = = Sum of entries in 4th row = 41 yrs old Variance = # 2 = Sum of entries in bottom row = 123 St. deviation = # = "# 2 11.09 years b. According to the empirical rule approximately 68% of all employed men fall in the interval [ #, + #] = [41 11.09, 41 + 11.09] = [29.91, 52.09] [30, 52]. So, approximately 68% of all employed men are 30–52 years old. 36. a. The rounded midpoints of the measurement classes (ages) are: (15 + 24.9) 2 20 (25 + 54.9) 2 40 (55 + 64.9) 2 60 In the following table, the probabilities have been rounded to two decimal places.

$

20

40

60

Total

17

47

13

77

0.61

0.17

1

4.4

24.4

10.2

39

-19

1

21

361

1

441

( = ) 0.22

( = )

( ) 2

( ) 2 ( = ) 79.42 0.61 74.97 155 Expected value = = Sum of entries in 4th row = 39 yrs old Variance = # 2 = Sum of entries in bottom row = 155


"# 2

St. deviation = # = 12.45 years b. According to the empirical rule approximately 68% of all employed men fall in the interval [ #, + #] = [39 12.45, 39 + 12.45] = [26.55, 51.45] [27, 51]. So, approximately 68% of all employed women are 27–51 years old. Solutions Section 9.4

37. a. For the values of we use the rounded midpoints of the measurement classes given: (35 + 44.9) 2 (65 + 74.9) 2

40 70

(45 + 54.9) 2 (75 + 84.9) 2

50 80

(55 + 64.9) 2 (85 + 94.9) 2

60 90

To obtain the probabilities, divide each frequency by the sum of all the frequencies, 50:

( = )

40

50

60

70

80

90

.02

.48

.34

.16

0

0

40

50

60

70

80

90

.02

.48

.34

.16

0

0

0.8

24

20.4

11.2

0

0

For and # we use the tabular method described in Example 3 in the textbook:

( = )

( = )

= ( ) = sum of values ( = ) = 56.4 16.4 6.4

3.6

13.6

23.6

33.6

19.66 4.41

29.59

0

0

( ) 2 268.96 40.96 12.96 184.96 556.96 1128.96

( ) 2 ( = )

5.38

# 2 = Sum of entries in bottom row = 59.04 # = "# 2 7.68 b. The distribution appears to bell-shaped but not symmetric. The empirical rule states that approximately 68% of the data will fall between # 56.4 7.68 = 48.72 and + # 56.4 + 7.68 = 64.08. 68% of the states would correspond to 34 states. Referring to the original table of percentages (with percentage brackets), we can estimate that the actual number is somewhat less than 24 + 17 = 41 (although we cannot say by exactly how much), so the empirical rule prediction is consistent with the data given. 38. a. For the values of we use the rounded midpoints of the measurement classes given: (35 + 44.9) 2 (65 + 74.9) 2

40 70

(45 + 54.9) 2 (75 + 84.9) 2

50 80

(55 + 64.9) 2 (85 + 94.9) 2

60 90

To obtain the probabilities, divide each frequency by the sum of all the frequencies, 50:

( = )

40

50

60

70

80

90

0

.06

.42

.24

.22

.06

For and # we use the tabular method described in Example 3 in the textbook:


( = )

( = )

Solutions Section 9.4 40

50

60

70

80

90

0

.06

.42

.24

.22

.06

0

3

25.2

16.8

17.6

5.4

28

18

-8

2

12

22

64

4

144

484

= ( ) = sum of values ( = ) = 68.0 ( ) 2 784

( ) 2 ( = )

0

324

19.44 26.88 0.96 31.68 29.04

# 2 = Sum of entries in bottom row = 108 # = "# 2 10.39 b. The distribution appears to bell-shaped but not symmetric. The empirical rule states that approximately 68% of the data will fall between # 68 10.39 = 57.61 and + # 68 + 10.39 = 78.39. 68% of the states would correspond to 34 states. Referring to the original table of percentages (with percentage brackets), we can estimate that the actual number is somewhat less than 21 + 12 + 11 = 44 (although we cannot say by exactly how much), so the empirical rule prediction is consistent with the data given. 39. Since the probability distribution is highly skewed, we need to use Chebyshev's rule. = 2, # = 0.15 Following is a representation of the mean ± several standard deviations (each division is one standard deviation in width):

The range [1.4, 2.6] represents the interval ± 4#, so by Chebyshev's rule, at least 15/16 of companies have a lifespan in this range. Therefore, at most 1/16 have a lifespan outside this range (the gray regions above). Since the distribution is not symmetric, we cannot conclude that half of the 1/16 is in the range on the right (companies at least as old as yours). Therefore, all we can say is that at most 1/16, or 6.25%, of all companies are in the range on the right (at least as old as yours). 40. Since the probability distribution is highly skewed, we need to use Chebyshev's rule. = 3, # = 0.2 Following is a representation of the mean ± several standard deviations (each division is one standard deviation in width):

The range [2.6, 3.4] represents the interval ± 2#, so by Chebyshev's rule, at least 3/4 of companies have a lifespan in this range. Therefore, at most 1/4 have a lifespan outside this range (the gray regions above). Since the distribution is symmetric, at most half of these: 1/8 of the car-compounding services, have life spans less than 2.6 years (the gray region on the left). Therefore, at most 1/8 = 12.5% of car-compounding services last at most as long as yours. 41. Since the probability distribution is not known to be bell shaped, we need to use Chebyshev's rule. = 9, # = 2 Following is a representation of the mean ± several standard deviations (each division is one standard deviation in width):


Solutions Section 9.4

The range [5, 13] represents the interval ± 2#, so by Chebyshev's rule, at least 3/4 of all Batmobiles have a lifespan in this range. Therefore, at most 1/4 have a lifespan outside this range (the gray regions above). Since the distribution is symmetric, at most half of these, 1/8 of all Batmobiles, have life spans more than 13 years (the gray region on the right). Therefore, there is at most (Choice B) a 12.5% chance that your new Batmobile will last 13 years or more. 42. Since the probability distribution is not known to be bell shaped or symmetric, we need to use Chebyshev's rule. = 8, # = 2 Following is a representation of the mean ± several standard deviations (each division is one standard deviation in width):

The range [4, 12] represents the interval ± 2#, so by Chebyshev's rule, at least 3/4 of all Batmobiles have a life span in this range. Therefore, at most 1/4, or 25%, have a life span outside this range (the gray regions above). Since the distribution is not known to be symmetric, we cannot divide the 1/4 between the two gray regions on the ends. So all we can say is the following: Therefore, there is at most (Choice B) a 25% chance that your new Spiderman Coupé will last less than 4 years. 43. a. Take "success" to mean spending more than an hour a day online. The given distribution is a binomial distribution with = 40, = .69, = 1 .69 = .31 = = 40 × .69 = 27.6 children # = " = "40 × .69 × .31 2.93 b. Since the binomial distribution is symmetric and bell shaped, we can use the empirical rule, which says there is a 95% chance that between 2# = 27.6 2(2.93) 21.7 and + 2# = 27.6 + 2(2.93) 33.5 children in the sample spend more than an hour a day online. Therefore, there is a 5% chance of this not happening—that is, a 5% chance that either 21 or fewer children spend more than an hour a day online, or that 34 or more do. Since the distribution is symmetric, there is a 2.5% chance that 34 or more children in the group spend more than an hour a day online. 44. a. Take "success" to mean spending at least two hours a day online. The given distribution is a binomial distribution with = 30, = .46, = 1 .46 = .54 = = 30 × .46 = 13.8 teenagers # = " = "30 × .46 × .54 2.73 b. Since the binomial distribution is symmetric and bell shaped, we can use the empirical rule, which says there is a 68% chance that that between # = 13.8 2.73 11.1 and + # = 13.8 + 2.73 16.5 teenagers in the sample spend at least two hours a day online. Therefore, there is a 32% chance of this not happening—that is, a 32% chance that either 11 or fewer teenagers spend at least two hours a day online, or that 17 or more do. Since the distribution is symmetric, there is a 16% chance that 17 or more teenagers in the group spend at least two hours a day online. 45. a. Take "success" to mean playing video games. The given distribution is a binomial distribution with = 1,000, = 1 .27 = .73, = .27 = = 1,000 × .73 = 730 teenagers # = " = "1,000 × .73 × .27 14.0 b. Since the binomial distribution is symmetric and bell shaped, we can use the empirical rule, which says there is a 95% chance that that between 2# 730 2(14.0) = 702 and + 2# 730 + 2(14.0) = 758 teenagers in


Solutions Section 9.4 the sample will play video games. 46. a. Take "success" to mean not playing Super Smash Bros. online. The given distribution is a binomial distribution with = 2,000, = 1 .12 = .88, = .12 = = 2, 000 × .88 = 1,760 teenagers # = " = "2,000 × .88 × .12 14.53 14.5 b. Since the binomial distribution is symmetric and bell shaped, we can use the empirical rule, which says there is a 99.7% chance that that between 3# = 1,760 3(14.53) 1,716 and + 3# = 1,760 + 3(14.53) 1,804 teenagers in the sample will not play Super Smash Bros. online. Note: If the rounded value of # is used, then 3# rounds to 1,717. 47. a. Below is a table giving the computations (best done using technology, such as a spreadsheet).

( = )

( = )

0

1

2

3

4

5

6

7

8

9

.01

.05

.14

.24

.25

.18

.09

.02

.01

.01

0

.05

.28

.72

1.00

.90

.54

.14

.08

.09

Total

3.80

( ) 2 ( = ) 0.144 0.392 0.454 0.154 0.010 0.259 0.436 0.205 0.176 0.270 2.50 From the table, = 3.8 # 2 2.5 # = "# 2 1.6. b. According to Chebyshev's inequality, at least 3/4 or 75% of all drivers will have between 2# 3.8 2(1.6) = 0.6 and + 2# 3.8 + 2(1.6) = 7.0 crashes. 48. a. Below is a table giving the computations (best done using technology, such as a spreadsheet).

( = )

( = )

1

2

3

4

5

6

7

8

9

10

.01

.01

.01

.02

.07

.17

.26

.26

.15

.04

.01

.02

.03

.08

.35

1.02

1.82

2.08

1.35

.40

Total

7.16

( ) 2 ( = ) 0.379 0.266 0.173 0.200 0.327 0.229 0.007 0.183 0.508 0.323 2.59 From the worksheet, = 7.16 7.2 # 2 = 2.59 2.6 # = "# 2 1.6. b. According to Chebyshev's inequality, at least 3/4 or 75% of all cities will have between 2# 7.2 2(1.6) = 4.0 and + 2# 7.2 + 2(1.6) = 10.4 (really, 10, the top of the possible values of ) record stores. The largest (whole) number of record stores below this range is 3.

49. The household income of a low-income family in the U.S. is 49,000 1.3(34,000) = $4,800 or less.

50. The household income of a low-income family in Switzerland is 43,000 1.3(22,000) = $14,400 or less.

51. The household income of a high-income family in the U.S. is 49,000 + 1.3(34,000) = $93,200 or more.

52. The household income of a high-income family in Belgium is 32,000 + 1.3(14,000) = $50,200 or more.


Solutions Section 9.4 53. Cutoffs for low-income families (1.3 standard deviations): U.S.: 49,000 1.3(34,000) = $4,800 Switzerland: 43,000 1.3(22,000) = $14,400 Belgium: 32,000 1.3(14,000) = $13,800

Canada: 38,000 1.3(20,000) = $12,000 Germany: 33,000 1.3(16,000) = $12,200

The U.S. has the lowest cutoff. 54. Cutoffs for high-income families (1.3 standard deviations): U.S.: 49,000 + 1.3(34,000) = $93,200 Switzerland: 43,000 + 1.3(22,000) = $71,600 Belgium: 32,000 + 1.3(14,000) = $50,200

Canada: 38,000 + 1.3(20,000) = $64,000 Germany: 33,000 + 1.3(16,000) = $53,800

The U.S. has the highest cutoff.

55. The gap between high and low incomes is measured by 2 × 1.3 = 2.6 standard deviations. Since the U.S. has the largest standard deviation listed, it has the largest gap between high and low incomes.

56. The gap between high and low incomes is measured by 2 × 1.3 = 2.6 standard deviations. Since Belgium has the smallest standard deviation listed, it has the smallest gap between high and low incomes. 57. An income of $15,000 is 1 standard deviation below the U.S. mean income. By the empirical rule, approximately 68% earned within 1 standard deviation, so that approximately 32% earn outside the onestandard-deviation interval. Half of that, approximately 16%, earn less. 58. An income of $117,000 is two standard deviations above the U.S. mean income. By the empirical rule, approximately 95% earned within two standard deviations, so that approximately 5% earn outside the twostandard-deviation interval. Half of that, 2.5%, earn more. 59. By the empirical rule, approximately 99.7% earned within three standard deviations of the mean. This is the range 33,000 3(16,000) = 15,000 to 33,000 + 3(16,000) = 81,000 Since income can't be negative, the answer is $0–$81,000. 60. By the empirical rule, approximately 99.7% earned within three standard deviations of the mean. This is the range 38,000 3(20,000) = 22,000 to 38,000 + 3(20,000) = 98,000 Since income can't be negative, the answer is $0–$98,000. 61. Using technology: TI: Enter the data in L1 and then 1-Var Stats 2000 data: = 12.56%, # 1.8885% 2010 data: = 13.30%, #

Spreadsheet: =STDEVP() 1.6643%

62. Using technology: TI: Enter the data in L1 and then 1-Var Stats 2010 data: = 13.30%, # 1.6643% 2020 data: = 17.02%, #

Spreadsheet: =STDEVP() 1.9848%

63. The mean for of the aging populations in 2010 is larger than that for 2000, suggesting that the population was older in 2010. The standard deviation for 2010 is less than that for 2000, so that the variation across states of percentage of the aging population is lower, suggesting that the population was less diverse with respect to age in 2010. (Choice (B)) 64. The mean of the aging populations in 2010 is smaller than that for 2020, suggesting that the population was


Solutions Section 9.4 younger in 2010. The standard deviation for 2010 is smaller than that for 2020, so that the variation across states of percentage of the aging population is smaller, suggesting that the population was less diverse with respect to age in 2010. (Choice (D)) 65. The empirical rule predicts approximately 68%. The one-standard-deviation interval based on the calculations in Exercise 61 is # = 13.30 1.6643 11.64 + # = 13.30 + 1.6643 14.96 The scores in that range are shown in bold: 8, 9, 10, 11, 11, 11, 12, 12, 12, 12, 12, 12, 12, 13, 13, 13, 13, 13, 13, 13, 13, 13, 14, 14, 14, 14, 14, 14, 14, 14, 14, 14, 14, 14, 14, 14, 14, 14, 14, 14, 14, 14, 15, 15, 15, 15, 15, 16, 16, 17 These are 36 states, representing 36/50 = 72% of all states. This differs substantially from the empirical rule prediction. One reason for the discrepancy is that the associated probability distribution is roughly bell shaped but not symmetric, as shown in the following figure:

66. The empirical rule predicts approximately 68%. The one-standard-deviation interval based on the calculations in Exercise 62 is # = 17.02 1.9848 15.04 + # = 17.02 + 1.9848 19.00. The scores in that range are shown in bold: 11, 12, 13, 14, 15, 15, 16, 16, 16, 16, 16, 16, 16, 16, 16, 16, 16, 16, 16, 17, 17, 17, 17, 17, 17, 17, 17, 17, 17, 17, 18, 18, 18, 18, 18, 18, 18, 18, 18, 18, 18, 18, 19, 19, 19, 20, 20, 21, 21, 21 These are 39 states, representing 39/50 = 78% of all states. This differs substantially from the empirical rule prediction. One reason for the discrepancy is that the associated probability distribution is not particularly bell shaped nor symmetric, as shown in the following figure:

67. The two-standard-deviation interval based on the calculations in Exercise 61 is 2# = 13.30 2(1.6643) 9.971 + 2# = 13.30 + 2(1.6643) 16.63. The scores in that range are shown in bold: 8, 9, 10, 11, 11, 11, 12, 12, 12, 12, 12, 12, 12, 13, 13, 13, 13, 13, 13, 13, 13, 13, 14, 14, 14, 14, 14, 14, 14, 14, 14, 14, 14, 14, 14, 14, 14, 14, 14, 14, 14, 14, 15, 15, 15, 15, 15, 16, 16, 17


Solutions Section 9.4 These are 47 states, representing 47/50 = 94% of all states. Chebyshev's rule is valid, since it predicts that at least 75% of the scores are in this range. This is not surprising, as Chebyshev's rule is always valid. 68. The two-standard-deviation interval based on the calculations in Exercise 62 is 2# = 17.02 2(1.9848) 13.05 + 2# = 17.02 + 2(1.9848) 20.99. The scores in that range are shown in bold: 11, 12, 13, 14, 15, 15, 16, 16, 16, 16, 16, 16, 16, 16, 16, 16, 16, 16, 16, 17, 17, 17, 17, 17, 17, 17, 17, 17, 17, 17, 18, 18, 18, 18, 18, 18, 18, 18, 18, 18, 18, 18, 19, 19, 19, 20, 20, 21, 21, 21 These are 44 states, representing 44/50 = 88% of all states. Chebyshev's rule is valid, since it predicts that at least 75% of the scores are in this range. This is not surprising, as Chebyshev's rule is always valid. 69. (A) The graph shows standard deviations, not the actual power grid frequency, so we cannot conclude (A). (B) The standard deviation indicates the variability in the power supply frequency. Since it was lower in mid-1999 than in 1995, this indicates greater stability in mid-1999 than in 1995, so the assertion is true. (C) The standard deviation indicates the variability in the power supply frequency. Since it was higher in mid-2002 than in mid-1995, this indicates less stability in mid-2002, so we cannot conclude (C). (D) The standard deviation was greatest in 2001–2002, indicating that the greatest fluctuations in the power grid frequency occurred during that period, so the assertion is true. (E) Around January 1995, the standard deviation was closest to its average of 0.9 but was lower around January 1999, so the power grid was more stable around January 1999 than around January 1995. Thus, (E) is false. 70. (A) and (B): From 2000 on, the plotted means tend to get larger, and the fluctuations also tend to get larger. Therefore, both the mean and standard deviation show an upward trend, indicating that (A) is true and (B) is false. (C) What peaked in the second half of 2001 was the frequency. The graph does not show the demand for electric power, so we cannot conclude (C). (D) Although the frequency was higher in the second half of 2002 than in the corresponding period for 1999, it fluctuated less in the second half of 2002, indicating that the standard deviation then was smaller, not larger. (E) The mean of the monthly means in 2000 was no more than 4.0, whereas the mean of the 2002 monthly means was considerably higher. On the other hand, those means fluctuated less in 2002 than in 2000, so the statement is true. 71. (A), (B), (C) From 2002 on, the mean is definitely trending upward, but the fluctuations appear to be trending downward, so the standard deviation is not increasing (choice (B)). (D) False; what was greater in 2003 was the mean. The fluctuations in 2001 were significantly greater (E) The mean of the monthly means in 2002 was greater than 4.0, whereas the mean of the 2000 monthly means was considerably lower. On the other hand, those means fluctuated more in 2000 than in 2002, so the statement is true. 72. (A), (B) and (C): From 2000 on, the plotted values, and hence the annual means, tend to get larger, although the fluctuations remain roughly the same. Therefore, the annual mean, but not the standard deviation, shows a significant upward trend, indicating that (B) is true and (A) and (C) are false. (D) The data in the second half of 2000 vary in a range of around 100, whereas the data in the first half vary by around 180, suggesting a significantly larger standard deviation in the first half, and not the second half. Thus, (D) is false. (E) The data in 2000 fluctuated in a significantly larger range (around 180) in 2000 than that in 2001 (around120) so the statement is true. 73. The sample standard deviation is bigger; the formula for sample standard deviation involves division by the smaller term 1 instead of , which makes the resulting number larger. 74. Closer. For large values of , division of a fixed number by and by 1 yields approximately the same result.


Solutions Section 9.4 75. The grades in the first class were clustered fairly close to 75. By Chebyshev's inequality, at least 88% of the class had grades in the range 60–90. On the other hand, the grades in the second class were widely dispersed. The second class had a much wider spread of ability than did the first class. 76. You would rate the first employee higher, because he or she is much more consistent. If we use the empirical rule as a guide, approximately 68% of the parts produced by the first employee are likely to be in the range 50.1 ± 0.15 mm, and all parts in that range are usable. For the second employee, the corresponding range is 50.0 ± 0.4 mm, suggesting that less than 68% will be usable. Moreover, it is likely that a minor modification in technique would bring the first employee's average closer to 50.0, but it is usually more difficult to make someone more consistent. 77. Since the standard deviation is 0, there is no variability at all; that is, the variable must be constant. Therefore, the variable must take on only the value 10, with probability 1. 78. The mean of such a population is zero, and the variance is the sum of the squares of differences from the mean—that is, the sum of copies of 1. Dividing the total by yields a variance of 1.

79. Since there are 2 data points, the mean is midway between them, at a distance of ( ) 2 from each. Summing the square of this distance twice gives ( ) 2 ( ) 2 ( ) 2 ! ( ) 2 = . + = 4 4 2 Dividing by 2 gives the variance: ( ) 2 #2 = . 4 Therefore, # = . 2 80. Since there are 2 data points, the mean is midway between them, at a distance of ( ) 2 from each. Summing the square of this distance twice gives ( ) 2 ( ) 2 ( ) 2 ! ( ) 2 = . + = 4 4 2 Dividing by 2 1 = 1 gives the variance: ( ) 2 #2 = . 2 Therefore, # = . "2


Solutions Section 9.5 Section 9.5 1. Note: Answers for this section were computed by using the 4-digit table in the Appendix, and may differ slightly from the more accurate answers generated by using technology. We use the table to find (0

0.00

0.4

0.01

0.5) = .1915.

0.02

.1554 .1591 .1628

0.5

.1915 .1950 .1985

0.6

.2257 .2291 .2324

3. The table gives us (0

0.6

0.00

0.01

0.71) = .2611.

0.02

0.00

1.4

0.01

0.02

.4192 .4207 .4222

1.5

.4332 .4345 .4357

1.6

.4452 .4463 .4474

4. The table gives us (0

0.00

0.01

1.71) = .4564.

0.02

0.7

.2580 .2611 .2642

1.6 1.7

.4554 .4564 .4573

0.8

.2881 .2910 .2939

1.8

.4641 .4649 .4656

( 0.71 shown:

.2257 .2291 .2324

2. We use the table to find (0 1.5) = .4332.

( 0.71

0.71) is twice this area, as

0.71) = 2(.2611) = .5222

( 1.71 shown:

.4452 .4463 .4474

( 1.71

1.71) is twice this area, as

1.71) = 2(.4564) = .9128


5. The table gives us (0

1.2

0.03

0.04

Solutions Section 9.5

1.34) = .4099.

6. The table gives us (0

0.05

0.03

0.23) = .0910.

0.04

1.3

.4082 .4099 .4115

0.1 0.2

.0871 .0910 .0948

1.4

.4236 .4251 .4265

0.3

.1255 .1293 .1331

(0

0.6

.3907 .3925 .3944

0.02

0.71) = .2611 0.00

0.01

(0

0.02

.2257 .2291 .2324

.0478 .0517 .0557

1.71) = .4564 0.00

0.01

0.02

0.7

.2580 .2611 .2642

1.6

.4452 .4463 .4474

1.7

.4554 .4564 .4573

0.8

.2881 .2910 .2939

1.8

.4641 .4649 .4656

( 0.71 1.34) is obtained by adding these areas (see figure).

( 1.71 0.23) is obtained by adding these areas (see figure).

( 0.71

( 1.71

1.34) = .4099 + .2611 = .6710

0.23) = .0910 + .4564 = .5474

7. From the table, (0 1.5) = .4332 (0 0.5) = .1915. To obtain (0.5 1.5), subtract the smaller from the larger (see figure).

8. From the table, (0 1.82) = .4656 (0 0.71) = .2611. To obtain (0.71 1.82), subtract the smaller from the larger (see figure).

(0.5

(0.71

1.5) = .4332 .1915 = .2417

1.82) = .4656 .2611 = .2045


Solutions Section 9.5 9. From the table, (0 0.71) = .2611. To obtain ( 0.71), subtract (0 0.71) from .5 (see Example 1(e)). ( 0.71) = .5 .2611 = .2389

10. From the table, (0 1.82) = .4656 To obtain ( 1.82), add (0 1.82) to .5 (see Example 1(f)). ( 1.82) = .5 + .4656 = .9656

11. From the table, (0 1.34) = .4099. To obtain ( 1.34), subtract (0 1.34) from .5 (see Example 1(e)). ( 1.34) = .5 .4099 = .0901

12. From the table, (0 0.23) = .0910. To obtain ( 0.23), add (0 0.23) to .5 (see Example 1(f)). ( 0.23) = .5 + .0910 = .5910

13. = 50, # = 10. Standardize the given problem: % & (% &) = � � # # 35 50 65 50 (35 65) = � � = ( 1.5 10 10 = 2(.4332) = .8664

1.5)

0.25)

14. = 40, # = 20. Standardize the given problem: % & (% &) = � � # # 35 40 45 40 (35 45) = � � = ( 0.25 20 20 = 2(.0987) = .1974 15. = 50, # = 10. Standardize the given problem: % & (% &) = � � # # 30 50 62 50 (30 62) = � � = ( 2 10 10 = .3849 + .4772 = .8621

16. = 40, # = 20. Standardize the given problem: % & (% &) = � � # # 30 40 53 40 (30 53) = � � = ( 0.5 20 20 = .1915 + .2422 = .4337 17. = 100, # = 15. Standardize the given problem: % & (% &) = � � # # 110 100 130 100 (110 130) = � � 15 15 = .4772 .2486 = .2286 18. = 100, # = 15. Standardize the given problem: % & (% &) = � � # #

1.2)

0.65)

(0.67

2)


70 100 (70 80) = � 15 = .4772 .4082 = .0690.

Solutions Section 9.5 80 100 � ( 2 15

1.33)

19. The -value measures the number of standard deviations form the mean. Therefore, the given problem translates to ( 0.5 0.5) = 2(.1915) = .3830.

20. The -value measures the number of standard deviations form the mean. Therefore, the given problem translates to ( 1.5 1.5) = 2(.4332) = .8664.

21. This is the probability that is either > 23 or < 23 . The complement of this event is the event that 23

2 . 3

( 0.67 0.67) = 2(.2486) = .4972 Therefore, the desired probability is 1 .4972 = .5028.

22. This is the probability that is either > 53 or < 53 . The complement of this event is the event that 53

5 . 3

( 1.67 1.67) = 2(.4525) = .9050 Therefore, the desired probability is 1 .9050 = .0950.

& 100 � = .3. 10 Looking inside the table, we see that (0 0.84) & 100 = 0.84 10 & = 10 × 0.84 + 100 = 108.4. (100

&) = �0

23. Normalizing, we have

& 10 � = .4. 5 Looking inside the table, we see that (0 1.28) & 10 = 1.28 5 & = 5 × 1.28 + 10 = 16.4. (10

&) = �0

24. Normalizing, we have

.3, so

.4, so

% 100 � = .04. 10 Because ( &) = .5 (0 &), we look inside the table to find (0 ( 1.75) .04. So, % 100 = 1.75 10 % = 10 × 1.75 + 100 = 117.5. ( %) = �

25. Normalizing, we have

26. Normalizing, we have

1.75)

.46, hence


Solutions Section 9.5 % 10 ( %) = � � = .03. 5 Because ( &) = .5 (0 &), we look inside the table to find (0 ( 1.88) .03. So, % 10 = 1.88 5 % = 5 × 1.88 + 10 = 19.4. 27. (10

15) = (9.5

15.5), where

standard deviation of # = " = '100 × 16 × 56

28. (15

15.5) = �

20) = (14.5

20.5), where

(14.5

20.5) = �

standard deviation of # = " = '100 × 16 × 56

29. ( < 25) = (0

= =

200 × 16

( 0.5

has a mean of = = 100 × 16

3.7268. We now standardize :

24.5), where

24.5 33.3333 0.5 33.3333 � 5.2705 5.2705 ( 6.42 1.68) = .5000 .4535 = .0465

200 × 16

200) = (40.5

200.5), where

(40.5

has a mean of

40.5 33.3333 200.5 33.3333 � 5.2705 5.2705 (1.36 31.72) = .5000 .4131 = .0869

200.5) = �

31. = 500, # = 100

450 500 100 = .1915 + .1915 = .3830

550) = �

32. = 151, # = 7

(

144) = �

144 151 � = ( 7

(110

110 100 16

33. = 100, # = 16

140) = �

550 500 � = ( 0.5 100

.09. 0.5)

1) = .5 .3413 = .1587 140 100 � 16

(0.63

16.6667 and a

16.6667 and a

5.2705. We now

.05.

33.3333 and a standard deviation of # = " = '200 × 16 × 56

standardize :

.47, hence

.57.

has a mean of

24.5) = �

30. ( > 40) = (41

(450

.35.

33.3333 and a standard deviation of # = " = '200 × 16 × 56

standardize :

= =

3.7268. We now standardize :

14.5 16.6667 20.5 16.6667 � 3.7268 3.7268 ( 0.58 1.03) = .3485 + .2190 = .5675 24) = ( 0.5

1.88)

has a mean of = = 100 × 16

9.5 16.6667 15.5 16.6667 � 3.7268 3.7268 ( 1.92 0.31) = .4726 .1217 = .3509

(9.5

2.5)

5.2705. We now


= .4938 .2357 = .2581, approximately 26% 34. = 100, # = 16

80 100 16 = .3944 .2357 = .1587, approximately 16% (80

90) = �

Solutions Section 9.5

90 100 � 16

( 1.25

0.63)

35. = 100, # = 16

120 100 � = ( 1.25) = .5 .3944 = .1056 16 The total number of such people in the United States is .1056 × 333,000,000 35,200,000, to three significant digits. ( 120) = �

36. = 100, # = 16

140 100 � = ( 2.5) = .5 .4938 = .0062 16 The total number of such people in the United States is .0062 × 333,000,000 2,100,000, to two significant digits. ( 140) = �

37. = 500, # = 100. We seek % such that ( %) = .05. Normalizing, we have % 500 ( %) = � � = .05. 100 Because ( &) = .5 (0 &), we look inside the table to find (0 appears to be halfway between 1.64 and 1.65), hence ( 1.645) .05. So, % 500 = 1.645 100 % = 100 × 1.645 + 500 665. 38. = 151, # = 7. We seek % such that ( %) = .02. Normalizing, we have % 151 ( %) = � � = .02. 7 Because ( &) = .5 (0 &), we look inside the table to find (0 ( 2.05) .02. So, % 151 = 2.05 7 % = 7 × 2.05 + 151 165.

1.645)

.45 (it

2.05)

.48, hence

39. = 0.250, # = 0.03

0.400 0.250 � = ( 5) = .5 .5000 = .0000 0.03 to 4 decimal places. The total number of such batters expected is therefore 0. ( 0.400) = �

40. = 0.250, # = 0.05.

0.400 0.250 � = ( 3) = .5 .4987 = .0013 0.05 The total number of such batters expected is .0013 × 250 0.33; that is, about 1 batter every 3 seasons. ( 0.400) = �

41. = 7.5, # = 1

9 7.5 � = ( 1.5) = .5 .4332 = .0668 1 The total number of jars is therefore .0668 × 100,000 6,680. ( 9) = �


42. = 12, # = 2.5

Solutions Section 9.5

9 12 � = ( 1.2) = .5 .3849 = .1151 2.5 The total number of bottles is therefore .1151 × 300,000 34,530. (

9) = �

43. = 49, # = 34 (in thousands of dollars) 60 49 ( 60) = � � ( 0.32) = .5 .1255 = .3745; 34 approximately 37% 44. = 33, # = 16 (in thousands of dollars) 60 33 ( 60) = � � ( 1.69) = .5 .4545 = .0455; 16 approximately 5%

45. = 43, # = 22 (in thousands of dollars) 100 43 ( 100) = � � ( 2.59) = .5 .4952 = .0048 22 12 43 ( 12) = � � ( 1.41) = .5 .4207 = .0792; 22 .0048 + .0792 8% 46. = 32, # = 14 (in thousands of dollars) 100 32 ( 100) = � � ( 4.86) = .5 .5000 = 0 14 12 32 ( 12) = � � ( 1.43) = .5 .4236 = .0764; 11 approximately 8%

47. United States: = 49, # = 34 (in thousands of dollars) 12 49 ( 12) = � � ( 1.09) = .5 .3621 = .1379 34 Canada: = 38, # = 20 (in thousands of dollars) 12 38 ( 12) = � � = ( 1.30) = .5 .4032 = .0968 20 The United States had a higher proportion of families with incomes of $12,000 or less. 48. Canada: = 38, # = 20 (in thousands of dollars) 12 38 ( 12) = � � = ( 1.30) = .5 .4032 = .0968 20 Switzerland: = 43, # = 22 (in thousands of dollars) 12 43 ( 12) = � � ( 1.41) = .5 .4207 = .0792 22 Canada had a higher proportion of families with incomes of $12,000 or less.

49. Wechsler; as this test has a smaller standard deviation, a greater percentage of scores fall within 20 points of the mean. 50. Stanford-Binet. Because this test has a larger standard deviation, a smaller percentage of scores fall within 20 points of the mean. 51. = 6, # = 1. The -value corresponding to = 1 month is 1 6 = = 5. 1


Solutions Section 9.5 This is surprising, because the time between failures was more than 5 standard deviations away from the mean, which happens with an extremely small probability. 52. Because the time between failures was only one standard deviation away from the mean, this is not unusual. 53. Task 1: = 11.4, # = 5.0 10 11.4 ( 10) = � � 5

54. Task 3: = 7.3, # = 3.9 10 7.3 ( 10) = � � 3.9

( 0.28) = .5 + .1103 = .6103 ( .69) = .5 .2549 = .2451

55. Task 1: By Exercise 51, ( 1 10) = .6103. Task 2: = 11.9, # = 9 10 11.9 ( 2 10) = � � ( 0.21) = .5 + .0832 = .5832 9 Because the times taken to complete the tasks are independent, ( 1 10 and 2 10) = ( 1 10) × ( 2 10) = .6103 × .5832 .3559

56. Task 3: By Exercise 52, ( 3 10) = .2451. Task 4: = 9.1, # = 5.5 10 9.1 ( 4 10) = � � ( 0.16) = .5 .0636 = .4364 5.5 Because the times taken to complete the tasks are independent, ( 3 10 and 4 10) = ( 3 10) × ( 4 10) = .2451 × .4364

.1070

57. The total time it takes to complete tasks 1 and 2 is = 1 + 2 , which is normal with = 11.4 + 11.9 = 23.3 and # = "5 2 + 9 2 10.2956 20 23.3 ( 20) = � � ( 0.32) = .5 + .1255 = .6255. 10.2956

58. The total time it takes to complete tasks 3 and 4 is = 3 + 4 , which is normal with = 7.3 + 9.1 = 16.4 and # = "3.9 2 + 5.5 2 6.7424 20 16.4 ( 20) = � � ( 0.53) = .5 .2019 = .2981. 6.7424

59. = = 1,000 × .89 = 890 # = " = "1,000 × 0.89 × 0.11 9.894 We compute 3# 860 and + 3# 920. Because these are between 0 and = 1,000, the normal approximation of the binomial distribution is valid. 879.5 890 (880 ) (879.5 ) = � � ( 1.06 ) = .5 + .3554 = .8554 9.894

60. = = 5,000 × .25 = 1,250 # = " = "5,000 × 0.25 × 0.75 30.6 We compute 3# 1,158 and + 3# 1,342. Because these are between 0 and = 5,000, the normal approximation of the binomial distribution is valid. 1,299.5 1,250 (1,300 ) (1,299.5 ) = � � (1.62 ) = .5 .4474 = .0536 30.6


Solutions Section 9.5 61. = 10,000,000, = .00000311 = = 31.1; # = " = "31.1 × .99999689 5.58 We compute 3# 14 and + 3# 48. Because these are between 0 and = 10,000,000, the normal approximation of the binomial distribution is valid. 39.5 31.1 ( < 40) = ( 39) = ( 39.5) = � � ( 1.51) 5.58 = .4348 + .5 = .9348 62. = 5,000,000, = .00000410 = = 20.5; # = " = "20.5 × .99999590 4.53 We compute 3# 7 and + 3# 34. Because these are between 0 and = 5,000,000, the normal approximation of the binomial distribution is valid. 31.5 20.5 ( > 30) = ( 31) = ( 30.5) = � � ( 2.21) 4.53 = .5 .4864 = .0136

63. = 10,000,000, = .00000311 = = 31.1; # = " = "31.1 × .99999689 5.58 We compute 3# 14 and + 3# 48. Because these are between 0 and = 10,000,000, the normal approximation of the binomial distribution is valid. Suppose there are crashes. Since 10 people buy insurance, the payout is 10 × 1,000,000 = $10,000,000 per crash; that is, 10,000,000 . On the other hand, the premium you receive is 10 × 3.50 × 10,000,000 = $350,000,000. For breakeven you must have 10,000,000 = 350,000,000 so = 35 flights. To lose money, > 35: 35.5 31.1 ( > 35) = ( 36) = ( 35.5) = � � ( 0.79) 5.58 = .5 .2852 = .2148. 64. = 10,000,000, = .00000410 = = 41.0; # = " = "41.0 × .99999590 6.40 We compute 3# 22 and + 3# 60. Because these are between 0 and = 10,000,000, the normal approximation of the binomial distribution is valid. Suppose there are crashes. Since 10 people buy insurance, the payout is 10 × 1,000,000 = $10,000,000 per crash; that is, 10,000,000 . On the other hand, the premium you receive is 10 × 5 × 10,000,000 = $500,000,000. For breakeven you must have 10,000,000 = 500,000,000 so = 50 flights. To lose money, > 50: 50.5 41.0 ( > 50) = ( 51) = ( 50.5) = � � ( 1.48) 6.40 = .5 .4306 = .0694. 65. Let "success" = a person polled says that he or she prefers Goode. = .9 × .55 + .1 × .45 = .54 = 1,000 = = 540 # = " = "1,000 × .54 × .46 15.761 We compute 3# 493 and + 3# 587. Because these are between 0 and = 1,000, the normal approximation of the binomial distribution is valid.


Solutions Section 9.5 520.5 540 ( > 520) = ( 521) = ( 520.5) = � � 15.761 = .5 + .3925 = .8925

( 1.25)

66. Let "success" = a person polled says that he or she prefers Goode. Since, in fact, only 49% prefer Goode, = .9 × .49 + .1 × .51 = .492 = 1,000 = = 492 # = " = "1,000 × .492 × .508 15.809. We compute 3# 445 and + 3# 539. Because these are between 0 and = 1, 000, the normal approximation of the binomial distribution is valid. For Goode to poll at least 51% means that 510: 509.5 492 ( 510) = ( 509.5) = � � ( 1.11) 15.809 = .5 .3665 = .1335.

67. = 100, # unknown. Let the -score of someone who just barely qualifies be . To be in the top 2%, ( ) = .02. Therefore, (0 ) = .5 .02 = .48. 148 100 Looking in the table for the closest score to .48 (which is .4798) gives 2.05. Since = , # we have 148 100 48 = 2.05 = = # # 2.05# = 48, so 48 #= 23.4. 2.05 Note: A more accurate guess at would be midway between 2.05 and 2.06. Using = 2.055 gives the same answer to one decimal place. 68. = 1,000, # unknown. Let the -score of someone who just barely qualifies be . To be in the top 2%, ( ) = .02. Therefore, (0 ) = .5 .02 = .48. Looking in the table for the closest score to .48 (which is .4798) gives 2.05. Since 1,250 1,000 = , we have # 1,250 1,000 250 = 2.05 = = # # 2.05# = 250, so 250 #= 122. 2.05 Note: A more accurate guess at would be midway between 2.05 and 2.06. Using = 2.055 gives the same answer to three significant digits.

69. Since the empirical rule is based on the normal distribution (see the remarks before Example 3 in the textbook), the empirical rule gives the exact results when the distribution is exactly normal. 70. The only possible value is 0, since this probability corresponds to zero area under the density curve.

71. Neither. They are equal, because they differ by ( = %), which is zero for a continuous random variable.


72. ( curve.

&) (%

Solutions Section 9.5 &), because the left-hand side, in general, corresponds to more area under the

73. The total area under the curve must be equal to 1, and the area is a rectangle with width (& %). 1 Therefore, its height must be . & % 74. The normal distribution, which has a variance of 1. Since the variance is the expected value of the square of the distance from the mean, and since the distances to the mean are all 1 or less for the uniform distribution, one would expect its variance to be (significantly) smaller than 1. 75. A normal distribution with standard deviation 0.5, because it is narrower near the mean but must enclose the same amount of area as the standard curve, so it must be higher. 76. (A). Under the assumption in (A), long times for Task 1 will be more likely to be balanced by shorter times for Task 2 than under the assumptions in (B) and vice versa, so that the combined times will be more likely to be near the mean, leading to a smaller standard deviation.


Solutions Chapter 9 Review Chapter 9 Review

1. = the number of boys. is a binomial random variable with = .5, = 2. ( = 0) = (2, 0)(.5) 0(.5) 2 = 1 4 ( = 1) = (2, 1)(.5) 1(.5) 1 = 1 2 ( = 2) = (2, 2)(.5) 2(.5) 0 = 1 4 Probability distribution:

0

( = ) 1 4

1

1 2

2

1 4

Histogram:

2. = the number of girls. is a binomial random variable with = .5, = 3. ( = 0) = (3, 0)(.5) 0(.5) 3 = .125 ( = 1) = (3, 1)(.5) 1(.5) 2 = .375 ( = 2) = (3, 2)(.5) 2(.5) 1 = .375 ( = 3) = (3, 3)(.5) 3(.5) 0 = .125 Probability distribution:

0

( = ) .125

1

2

3

.375

.375

.125

Histogram:

3. = the sum of the two numbers when a four-sided dice is rolled twice. There are 4 × 4 = 16 possible outcomes: = 2: Outcomes {(1, 1)} = 3: Outcomes {(1, 2), (2, 1)} = 4: Outcomes {(1, 3), (2, 2), (3, 1)} = 5: Outcomes {(1, 4), (2, 3), (3, 2), (4, 1)} = 6: Outcomes {(2, 4), (3, 3), (4, 2)}

( = 2) = 1 16 ( = 3) = 2 16 ( = 4) = 3 16 ( = 5) = 4 16 ( = 6) = 3 16


= 7: Outcomes {(3, 4), (4, 3)} = 8: Outcomes {(4, 4)}

( = 7) = 2 16 ( = 2) = 1 16

Solutions Chapter 9 Review

Probability distribution:

2

( = ) 1/16

3

4

5

6

7

8

2/16

3/16

4/16

3/16

2/16

1/16

25

35

45

.386

.116

.016

Histogram:

4. = age of an Xbox player. Probability distribution:

15

( = ) .482 Histogram:

5. = number of defective joysticks chosen when 3 are selected from a bin that contains 20 defective joysticks and 30 good ones. (20, 0) (30, 3) ( = 0) = .2071 (50, 3) (20, 1) (30, 2) ( = 1) = .4439 (50, 3) (20, 2) (30, 1) ( = 2) = .2908 (50, 3) (20, 3) (30, 1) ( = 3) = .0582 (50, 3)


Solutions Chapter 9 Review Probability distribution:

0

1

2

3

( = ) .2071 .4439 .2908 .0582 Histogram:

6. = the number of ones that face up when 2 weighted dice are thrown. Probability distribution for each weighted die: Take to be the probability of a 2. The given information implies that the probability distribution for the die is

1

( = )

2

2

3

4

5

6

2

( = )

1

2

3

4

5

6

1/4

1/8

1/8

1/8

1/8

1/4

0

Since 8 = 1, we have = 1 8, so the probability distribution for a single die is

It follows that the probability of rolling a 1 is 1 4. Take this as "success." 1 0 3 2 9 ( = 0) = (2, 0)( ) ( ) = 4 4 16 1 1 3 1 3 ( = 1) = (2, 1)( ) ( ) = 4 4 4 1 2 3 0 1 ( = 2) = (2, 2)( ) ( ) = 4 4 16 Probability distribution: ( = ) 9/16

1

2

3/4

1/16


Solutions Chapter 9 Review Histogram:

7. The first row lists the values of , giving us the mean . The second row lists the numbers ( ), and the third row lists their squares.

( )

( ) 2

1 3

2

9

0

0

0

2

3

6

10

1

4

0

4

1

16

30

The right column shows the sums. ! ( ) 2 10 30 2 2 2 ! ( ) 30 = = = = = = 7.5 5 1 5 1 For the median *, arrange the scores in order and select the middle score: * = 2. 1, 0, 2, 3, 6

= " 2

2.7386

8. The first row lists the values of , giving us the mean . The second row lists the numbers ( ), and the third row lists their squares.

( )

( ) 2

4

4

1

1

5

6

6

25

0

1

1

0

1

1

0

1

1

4

The right column shows the sums. ! ( ) 2 4 25 ! ( 2 = = " 2 = 1 = 5 = = 1 = ) 2 = 4 5 1 4 For the median *, arrange the scores in order and select the middle score: 4, 4, 5, 6, 6 * = 5. 9. Two examples are: 0, 0, 0, 4 and 1, 1, 1, 5.

10. Since the standard deviation is 0, the scores must all be equal. Therefore, the only possibility is 2, 2, 2, 2, 2, 2.

11. An example is 1, 1, 1, 1, 1, 1. (Also see Exercise 78 in Section 9.4.) Here is the calculation of the population standard deviation (the right column shows the sums):

( )

( ) 2

1 1 1

1 1 1

1 1

1

1

1

0

1

1

1

0

1

1

1

1

6


=

Solutions Chapter 9 Review ! ( ) 2 6 ! ( #2 = = = 1 ) 2 = 6 6

0 = 0 6

# = "# 2 = 1

12. An example is 1, 1, 0, 1, 1. Here is the calculation of the sample standard deviation (the right column shows the sums):

1

( )

=

1

( ) 2

1

0 = 0 5

1 1

0

1

1

0

0

1

1

0

1

0

1

1

4

! ( ) 2 = 4

2 =

! ( ) 2 1

=

4 = 1 4

= " 2 = 1

13. For each of the following 8 solutions: To construct the probability distribution for the weighted die, take to be the probability of a 1. The given information implies that the probability distribution for the die is

( = )

1

2

3

4

5

6

2

1

2

3

4

5

6

Since 7 = 1, we have = 1 7, so the probability distribution for a single die is

( = )

1 7

1 7

1 7

1 7

1 7

2 7

Throwing the die 4 times is a sequence of 4 Bernoulli trials with = 2 7 and = 1 2 7 = 5 7. 2 1 5 3 ( = 1) = (4, 1)( ) ( ) .4165 7 7 2 3 5 1 14. ( = 3) = (4, 3)( ) ( ) 7 7

.0666

15. The probability that 6 comes up at most twice is ( 2 0 5 4 ( = 0) = (4, 0)( ) ( ) .2603 7 7 2 1 5 3 ( = 1) = (4, 1)( ) ( ) .4165 7 7 2 2 2 5 ( = 2) = (4, 2)( ) ( ) .2499 7 7 Therefore, ( 2) .2603 + .4165 + .2499 = .9267.

16. The probability that 6 comes up at most once is ( 2 0 5 4 ( = 0) = (4, 0)( ) ( ) .2603 7 7 2 1 5 3 ( = 1) = (4, 1)( ) ( ) .4165 7 7 Therefore, ( 2) .2603 + .4165 = .6768.

2) = ( = 0) + ( = 1) + ( = 2).

1) = ( = 0) + ( = 1).


2 4 5 0 17. The probability that is more than 3 is ( > 3) = ( = 4) = (4, 4)( ) ( ) 7 7 Solutions Chapter 9 Review

18. The probability that is at least 2 is ( 2) = 1 ( 1) 1 (.6768) (from Exercise 16) = .3232.

19. (1 Exercise 15.

3) = ( = 1) + ( = 2) + ( = 3). We computed ( = 1) and ( = 2) in

2 3 5 1 ( = 3) = (4, 3)( ) ( ) .0666 7 7 Therefore, (1 3) = ( = 1) + ( = 2) + ( = 3) 20. (

.0067.

2 4 5 0 3) = 1 ( = 4) = 1 (4, 4)( ) ( ) 7 7

.4165 + .2499 + .0666 = .7330.

1 .0067 = .9933

21. Think of the experiment as a sequence of 3 Bernoulli trials with "success" = girl, = 3 and = .5. = = 3(.5) = 1.5 # = " = "3 × .5 × .5 0.8660 To answer the last part, [ #, + #] = [0.634, 2.366] This interval does not include all 3 values. [ 2#, + 2#] = [ 0.232, 3.232] This interval includes all the scores, so all values of lie within 2 standard deviations of the expected value.

22. Think of the experiment as a sequence of 4 Bernoulli trials with "success" = boy, = 4 and = .25. = = 4(.25) = 1 # = " = "4 × .25 × .75 0.8660 To answer the last part, compute [ #, + #] = [0.134, 1.866] [ 2#, + 2#] = [ 0.732, 2.732] [ 3#, + 3#] = [ 1.598, 3.598] [ 4#, + 4#] = [ 2.464, 4.464]. Only the last interval includes all the scores, so all values of lie within 4 standard deviations of the expected value. 23. The frequencies add up to 16, so we obtain the probabilities by dividing the frequencies by 16:

3

( = ) 0.0625

2

1

0.125

0.1875

0

( = ) 0.1875 0.25 0.1875

( ) 2

0

3

2

1

9

4 0.5

( = ) × ( ) 2 0.5625

2

3

0.25 0.1875 0.125 0.0625

= sum of entries in right-hand column = 0

1

0.1875 0.25 0.1875

0

1

2

3

1

0

1

4

9

0.1875

0

0.1875

0.5

0.5625

# 2 = sum of entries in right-hand column = 2.5


"# 2

#= 1.5811 For the last part, note that 14 of the 16 scores in the frequency table (exclude the first and last value) are in the interval [ 2, 2]; that is, in the interval [ 2, + 2] because = 0. The number of standard deviations that 2 represents is approximately 2 1.3 standard deviations. 1.5811 Therefore, 87.5% (or 14/16) of the time, is within 1.3 standard deviations of the expected value. Solutions Chapter 9 Review

24. The frequencies add up to 20, so we obtain the probabilities by dividing the frequencies by 20:

4

( = ) 0.15

2

0.15

( = ) 0.6 0.3

0

2

4

6

0.2

0.25

0.15

0.1

0

0.5

0.6

0.6

= sum of entries in right-hand column = 0.8 4.8 2.8 0.8

( ) 2 23.04 7.84

0.64

1.2

3.2

5.2

1.44 10.24 27.04

( = ) × ( ) 2 3.456 1.176 0.128 0.36 1.536 2.704

# 2 = sum of entries in right-hand column = 9.36 # = "# 2 3.059 For the last part, we compute [ #, + #] = [ 2.259, 3.859] This interval includes 3 + 4 + 5 = 12 of the 20 scores, or 60%. Therefore, 60 percent of the values of lie within one standard deviation of the expected value. 25. By Chebyshev's rule, is guaranteed to lie within standard deviations with of with a probability 1 of at least 1 2 . Thus, 1 1 2 = .90 1 = 1 .90 = .10 2 1 2 = = 10 .10 = "10 3.162. The associated interval is therefore [ #, + #] [100 3.162(16), 100 + 3.162(16)] [49.4, 150.6].

26. By Chebyshev's rule, is guaranteed to lie within standard deviations of with a probability of at 1 least 1 2 . Since the distribution is symmetric, this means that it exceeds + # at most half of 1 2, or 1 (2 2) of the time. 1 = .10 2 2 1 2 = = 5 .20


Solutions Chapter 9 Review = "5 2.236 The associated value of is therefore + # 200 + (2.236)(5) 211.2.

27. Since is bell-shaped and symmetric, the empirical rule applies so approximately 99.7% of samples of are within the interval [ 3#, + 3#]. This means that approximately 0.3 2 = 0.15 of the samples of are greater than + 3# = 200 + 3(20) = 260.

28. Since is bell-shaped and symmetric, the empirical rule applies so approximately 95% of samples of are within the interval [ 2#, + 2#] = [100 2(30), 100 + 2(30] = [40, 160]. 29. = ; From the table, (0

30. = ; (

1.5) = .4332.

1.5) = .5 .4322 = .0668

31. = ; (| | 2.1) = 2(.5 .4821) = .0358

32. Standardize the given problem: % & (% &) = ( # # ) 80 100 120 100 (80 120) = ( ) = ( 1.25 16 16 = 2(.3944) = .7888.

1.25)


Solutions Chapter 9 Review 33. Standardize the given problem: & ( &) = ( # ) 1 0 ( 1) = ( = ( 0.5) = .5 .1915 = .3085. 2 ) 34. Standardize the given problem: % ( %) = ( # ) 1 ( 1) . ( 1) = + = ( 4) = .5 .5000 = .0000. 0.5 /0 ,35. The frequency distribution for the price is 5.50

12( = )

1

10

12

15

2

3

4

Dividing the frequencies by the sum, 10, gives the probability distribution. We also add a row for the computation of ( ): 5.50

( = )

( = )

10

12

15

.1

.2

.3

.4

0

0.2

0.4

0.3

( ) = sum of entries in bottom row = $12.15 36. The frequency distribution for the weekly sales is 6,200 3,500 3,000 1,000

12( = )

1

2

3

4

Dividing the frequencies by the sum, 10, gives the probability distribution. We also add a row for the computation of ( ): 6,200 3,500 3,000 1,000

( = )

( = )

.1

.2

.3

.4

620

700

900

400

( ) = sum of entries in bottom row = 2,620 copies

37. Revenue = Price per copy sold × Number of copies sold. The values of the weekly revenue are obtained by multiplying the prices by the weekly sales: (Revenue) 34,100 35,000 36,000 15,000 ( = )

.1

( = ) 3,410

.2

.3

.4

7,000 10,800 6,000

( ) = sum of entries in bottom row = $27,210


Solutions Chapter 9 Review 38. Loss = Loss per copy sold × Number of copies sold. The values of the weekly losses are obtained by multiplying the losses per book ($20 selling price) by the weekly sales: (Loss) 89,900 35,000 24,000 5000

( = )

.1

.2

( = ) 8,990

.3

7,000

.4

7,200 2,000

( ) = sum of entries in bottom row = $25,190

39. False; let = price and = weekly sales. Then weekly revenue = . However, 27,210 12.15 × 2,620. In other words, ( ) ( ) ( ).

40. False; let = weekly loss and = weekly sales. Then weekly loss per book = and ( ) = $7.85. However, 25,190 2,620 $9.61. In other words, ( ) ( ) ( ).

41. a. Take to be the number of orders per million residents. For the values of , use the rounded midpoints of the measurement classes given. To obtain the probabilities, divide each frequency by the sum of all the frequencies, 100:

Freq

( = )

2

4

6

8

10

25

35

15

15

10

.25

.35

.15

.15

.10

2

4

6

8

10

.25

.35

.15

.15

.10

0.5

1.4

0.9

1.2

1

To compute the expected value, add the ( = ) row as usual:

( = )

( = )

= sum of entries in bottom row = 5

( )

2

3

1

9

( = ) × ( ) 2 2.25

1

3

5

1

1

9

25

0.35

0.15

1.35

2.5

# 2 = sum of entries in bottom row = 6.6 # = "# 2 2.5690 b. The empirical rule predicts that 68% of all orders will lie in the interval [ #, + #] [5 2.569, 5 + 2.569] = [2.431, 7.569]. The empirical rule does not apply because the distribution is not symmetric. (However, it does give a fairly accurate prediction in this case.) c. The original frequency distribution is 1–2.9 3–4.9 5–6.9 7–8.9 9–10.9

Freq

25

35

15

15

10

The shaded cells correspond to the cities from which you obtain between 3 and 8 orders per million residents. These cells account for a total of 35 of the cities. Therefore, at least 50% of the cities gave orders between 3 and 8 per million residents.


Solutions Chapter 9 Review However, we know more: The additional 15 cities in the 7–8.9 category could conceivably all be 8, so we can conclude that up to 50 of the cities gave orders between 3 and 8 per million residents. Conclusion: Between 50% and 65% of the cities gave orders between 3 and 8 per million residents [choice (A)]. 42. a. Take to be the pollen count on a given day. For the values of , use the rounded midpoints of the measurement classes given. To obtain the probabilities, divide each frequency by the sum of all the frequencies, 20:

Freq

( = )

1

3

5

7

9

11

3

5

7

2

1

2

.15

.25

.35

.10

.05

.10

1

3

5

7

9

11

.15

.25

.35

.1

.05

.1

0.75

1.75

0.7

0.45

1.1

0.1

2.1

4.1

0.01

4.41

16.81 37.21

To compute the expected value, add the ( = ) row as usual:

( = )

( = ) 0.15

= Expected pollen count = sum of entries in bottom row = 4.9

3.9

1.9

( ) 2 15.21

3.61

6.1

( = ) × ( ) 2 2.2815 0.9025 0.0035 0.441 0.8405 3.721

# 2 = sum of entries in bottom row = 8.19 # = "# 2 2.8618 b. The empirical rule predicts that 95% of all orders will lie in the interval [ 2#, + 2#] [4.9 2(2.8618), 4.9 + 2(2.8618)] = [ 0.8236, 10.6236]. The empirical rule does not apply because the distribution is not symmetric. (However, it does give a fairly accurate prediction in this case.) c. The original frequency distribution is 0–1.9 2–3.9 4–5.9 6–7.9 8–9.9 10–11.9

Freq

3

5

7

2

1

2

The shaded cells correspond to the days on which the pollen count is between 2 and 7. These cells account for a total of 14 20 = 70 of the days. Therefore, at most 70% of the days had pollen counts between 2 and 7. However, the 2 days in the 6–7.9 range might all have had pollen counts greater than 7, so we can only be sure that at least 12 20 = 60 had pollen counts between 2 and 7. Conclusion: Between 60% and 70% of the days had pollen counts between 2 and 7 [choice (D)]. 43. For each of the following 6 solutions: Let be the number of Mac OS users who will order books in the next hour, and let be the number of Windows users who will order books in the next hour. and are both binomial random variables with the following properties: : = 10, = .05 : = 20, = .10 ( = 3) = (20, 3)(.10) 3(.90) 17 .190.

44. (

3) = ( = 0) + ( = 1) + ( = 2) + ( = 3)


Solutions Chapter 9 Review ( = 0) = (20, 0)(.10) 0(.90) 20 .1216 ( = 1) = (20, 1)(.10) 1(.90) 19 .2702 ( = 2) = (20, 2)(.10) 2(.90) 18 .2852 ( = 3) = (20, 3)(.10) 3(.90) 17 .1901 Therefore, ( 3) .1216 + .2702 + .2852 + .1901 .867.

45. ( = 1) = (10, 1)(.05) 1(.95) 9 .3151 ( = 3) = (20, 3)(.10) 3(.90) 17 .1901 By independence, ( = 1 and = 3) = ( = 1) ( = 3) = .3151 × .1901

.060.

46. The event that a Mac OS visitor to the site orders books is independent of the event that a Windows visitor to the site orders books. 47. ( ) = = (10)(.05) = 0.5

48. ( ) = = (20)(.10) = 2

49. Skin cream: = 38, # = 21 (in thousands of dollars) 50 38 ( 50) = ( ( 0.571) = .5 .2157 21 ) 50. Hair products: = 34, # = 14 (in thousands of dollars) 50 34 ( 50) = ( ( 1.14) = .5 + .3729 14 )

51. Skin cream: = 38, # = 21 (in thousands of dollars) 12 38 ( 12) = ( ( 1.238) = .5 .3925 21 ) 52. Hair products: = 34, # = 14 (in thousands of dollars) 12 34 ( 12) = ( ( 1.57) = .5 .4418 14 )

.284 .873 .108 .058

53. People in the Three Sigma Club have IQs with -values of at least 3. ( 3) .5 .4987 = .0013 Given a population of 333 million, this gives .0013 × 333,000,000 433,000 people. (A more accurate answer using technology is 450,000 people.) 54. To qualify for Mensa, your IQ must be at least 132. = 100, # is unknown, = 132 132 100 32 = . = # # 32 ( ) = .02 # (Your IQ must be in the top 2%.) Therefore, 32 (0 = .5 .02 = .48 #) Using the table backward, we get 32 2.06 #


so

#

32 2.06

Solutions Chapter 9 Review 16.

55. Given = 100, # = 16, to get into the Three Sigma club, your IQ must be at least 3 standard deviations above the mean; that is + 3# = 100 + 3(16) = 148.

56. We first need to determine the standard deviation #: 600 500 100 ( ) = ( # ) = .02 # 100 (0 = .5 .02 = .48 # ) Using the table backward, we get 100 2.06 # so 100 # 48.5. 2.06 To get into the Three Sigma Club, Billy-Sean's score must be at least 3 standard deviations above the mean; that is, at least + 3# = 500 + 3(48.5) 646.


Solutions Chapter 9 Case Study Chapter 9 Case Study 1. Distances between cities in France, measured in kilometers should follow Benfords law. 2. Distances between cities in France, measured in miles should follow Benfords law. 3. The grades (0–100) in your math instructor's grade book should not be expected to follow Benfords law; they tend to be normally distributed, so that the starting digits should be concentrated near that of the mean. For example, if the mean is 72%, then the starting digits 6, 7, and 8 will occur most frequently. 4. The Dow Jones averages for the past 100 years should follow Benford's law. 5. Verbal SAT scores of college-bound high school seniors should not be expected to follow Benfords law; they tend to be normally distributed, so that the starting digits should be concentrated near that of the mean. For example, if the mean SAT is 500, then the starting digits 4, 5, and 6 will occur most frequently. 6. Life spans of companies should follow Benfords law. 7. Following is the spreadsheet computation of SSE for Good Neighbor Inc.'s tax return:

The entries in column B are obtained by entering the formula =LOG10(1+1/A2) in cell B2 and then copying down the column. The entries in column D are obtained by entering the formula =(C2-B2)^2/B2 in cell D2 and then copying down the column. SSE is the sum of the entries above it multiplied by = 1,000: =1000*SUM(D2:D10) Since SSE < chi-squared, we cannot be 95% certain that Good Neighbor Inc.'s tax return violates Benford's law. In other words, the tax return is not suspect on the basis of Benfords law. 8. Following is the spreadsheet computation of SSE for the Honest Growth Funds Stockholder Report:


Solutions Chapter 9 Case Study

The entries in column B are obtained by entering the formula =LOG10(1+1/A2) in cell B2 and then copying down the column. The entries in column D are obtained by entering the formula =(C2-B2)^2/B2 in cell D2 and then copying down the column. SSE is the sum of the entries above it multiplied by = 400: =400*SUM(D2:D10) Since SSE > chi-squared, we can be 95% certain that the Honest Growth Funds Stockholder Report violates Benford's law. In other words, the report is suspect on the basis of Benfords law.


Solutions Section 10.1 Section 10.1 1. Table:

1.9

1.99

1.999

1.9999

( ) 5.975 5.9975 5.99975 5.999975

2

4

2.0001

2.001

2.01

2.1

440,000 44,000 4,400 440

As approaches 2 from the left (left-most four values), the corresponding values of ( ) are 5.975, 5.9975, 5.99975, and 5.999975, which appear to be approaching 6. Thus, we estimate lim ( ) = 6. 2

As approaches 2 from the right (right-most four values), the corresponding values of ( )(right to left) are 440, 4,400, 44,000, and 440,000, which appear to be increasing without bound. Thus, we estimate lim ( ) = . 2+

Because the left and right limits do not agree, lim ( ) does not exist. 2

The value of at 2 is specified in the table: (2) = 4. 2. Table:

2.1

2.01

2.001

2.0001

( ) 1.12 11.12 111.12 1,111.12

2

1.9999

1.999

1.99

1.9

0.00003 0.00031 0.00311 0.03111

As approaches 2 from the left (left-most four values), the corresponding values of ( ) are 1.12, 11.12, 111.12, and 1,111.12, which appear to be decreasing without bound. Thus, we estimate lim ( ) = . 2

As approaches 2 from the right (right-most four values), the corresponding values of ( )(right to left) are 0.03111, 0.00311, 0.00031, and 0.00003, which appear to be approaching 0. Thus, we estimate lim ( ) = 0. 2 +

Because the left and right limits do not agree, lim ( ) does not exist. 2

No value for ( 2) is specified, so ( 2) is undefined. 3. Table:

5.1

5.01

5.001

5.0001

( ) 3.12 31.12 311.12 3,111.12

5

4.9999

4.999

4.99

4.9

4,111.12 411.12 41.12 4.12

As approaches 5 from the left (left-most four values), the corresponding values of ( ) are 3.12, 31.12, 311.12, and 3,111.12, which appear to be decreasing without bound. Thus, we estimate lim ( ) = . 5

As approaches 5 from the right (right-most four values), the corresponding values of ( )(right to left) are 4.12, 41.12, 411.12, and 4,111.12, which appear to be decreasing without bound. Thus, we estimate lim ( ) = . 5

Because the left and right limits agree, lim ( ) = as well. 5

No value for ( 5) is specified, so ( 5) is undefined.


Solutions Section 10.1 4. Table:

0.1

0.01

0.001

0

0.0001

( ) 1.0303 1.00303 1.0003 1.00003

0.0001

0

0.001

0.01

0.1

1.00003 1.0003 1.00303 1.0303

As approaches 0 from the left (left-most four values), the corresponding values of ( ) are 1.0303, 1.00303, 1.0003, and 1.00003, which appear to be approaching 1. Thus, we estimate lim ( ) = 1. 0

As approaches 0 from the right (right-most four values), the corresponding values of ( )(right to left) are 1.0303, 1.00303, 1.0003, and 1.00003, which appear to be approaching 1. Thus, we estimate lim ( ) = 1. 0+

Because the left and right limits do not agree, lim ( ) does not exist. 0

The value of at 0 is specified in the table: (0) = 0. 2 Technology formula: x^2/(x+1) + 1 Limit is 0. Table (broken to fit on page): 5. ( ) =

0.1

0.01

0.001

0.0001

0

0

0.0001

0.001

0.01

0.1

( ) 0.01111 0.000101 1.001 × 10 6 1.0001 × 10 6 ( )

9.999 × 10 9 9.99 × 10 7 9.901 × 10 5 0.009091

3 Technology formula: (x-3)/(x-1) 1 Limit is 3. Table: 6. ( ) =

0.1

0.01

0.001

0.0001

( ) 2.8181818 2.9801980 2.998002 2.9998000

0

2 4 Technology formula: (x^2-4)/(x-2) 2 Limit is 4. Table:

0.0001

0.001

0.01

0.1

3.0002000 3.002002 3.0202020 3.2222222

7. ( ) =

( )

1.9

3.9

1.99 1.999 1.9999

3.99 3.999 3.9999

2

2.0001 2.001 2.01

4.0001 4.001 4.01

2 1 Technology formula: (x^2-1)/(x-2) 2 Limit does not exist. Table:

2.1

4.1

8. ( ) =

1.9

1.99

1.999

1.9999

( ) 26.1 296.01 2,996.001 29,996 9. ( ) =

2

2.0001

2.001

2.01

2.1

30,004.0001 3,004.001 304.01 34.1

2 + 1 Technology formula: (x^2+1)/(x+1) + 1


Solutions Section 10.1 Limit does not exist. Table: 1.1

1.01

1.001

1.0001

1

( ) 22.1 202.01 2,002.001 20,002

0.9999

0.999

19,998.0001 1,998.001 198.01 18.1

2 + 2 + 1 Technology formula: (x^2+2*x+1)/(x+1) + 1 Limit is 0. Table: 10. ( ) =

1.1 1.01 1.001 1.0001

1

( ) 0.1 0.01 0.001 0.0001

0.9999 0.999 0.99 0.9 0.0001

0.001

1 Technology formula: (x-1)/sqrt(x-1) 1 Limit is 0. Table:

0.01

11. ( ) =

1

( )

1.0001 1.001 0.01

3

12. ( ) =

3 Limit is . Table:

2.9

( ) 3.1623

0.0316

1.01

1.1

0.1

0.3162

Technology formula: sqrt(3-x)/(3-x)

2.99 10

2.999

31.6228

2.9999 100

3

2 + 4 + 3 Technology formula: (x^2+4*x+3)/(x+3) + 3 Limit is 3. Table: 13. ( ) =

2

( )

2.0001 2.001 2.01 3.0001 3.001 3.01

2.1

3.1

2 4 + 4 Technology formula: (x^2-4*x+4)/(x-2) 2 Limit is 0. Table: 14. ( ) =

1.9

1.99

1.999

1.9999

( ) 0.1 0.01 0.001 0.0001

2

9 Technology formula: (x-9)/(sqrt(x)-3) 3 Limit is 6. Table: 15. ( ) =

8.9

8.99

8.999

8.9999

( ) 5.98329 5.99833 5.99983 5.99998

9

0.99 0.9

0.1


16. ( ) =

5

25 Limit is 0.1. Table:

( )

25

Solutions Section 10.1 Technology formula: (sqrt(x)-5)/(x-25)

25.0001

25.001

25.01

25.1

0.0999999 0.099999 0.09999 0.0999

4 Technology formula: 4/(x-3)^2 ( 3) 2 Diverges to . Table: 17. ( ) =

2.9

2.99

2.999

2.9999

3

( ) 400 40,000 4,000,000 400,000,000

3 Technology formula: 3/(x-4)^(1/3) ( 4) 1 3 However, spreadsheets and some calculators will not recognize this as a cube root and give errors for < 4. As the cube root of a negative number is just the negative of the cube root of its absolute value, you can instead use -3/(4-x)^(1/3) when < 4. Diverges to . Table: 18. ( ) =

3.9

3.99

3.999 3.9999

4

( ) 6.463 13.925 30 64.633

| + 2| Technology formula: abs(x+2)/(x+2)^(1/6) ( + 2) 1 6 Limit is 0. Table: 19. ( ) =

( )

2

1.9999 1.999

1.99

1.9

0.00046 0.00316 0.02154 0.14678

( + 3) 2 3 Technology formula: (x+3)^(2/3)/abs(x+3) | + 3| However, spreadsheets and some calculators will not recognize this as a cube root and give errors for < 3. To avoid errors, write the technology formula as ((x+3)^2)^(1/3)/abs(x+3) so that the 1 3 power is being taken of a nonnegative number. Diverges to . Table: 20. ( ) =

3.1 3.01 3.001 3.0001

( ) 2.15

4.64

10

100

1, 000

21.54

3

2.9999 2.999 2.99 2.9 21.54

10

4.64

2.15

3 2 + 10 1 Technology formula: (3*x^2+10*x-1)/(2*x^2-5*x) 2 2 5 Limit is 1.5. Table: 21. ( ) =

10

10, 000 100, 000

( ) 2.66 1.58969 1.50877 1.50088 1.50009


Solutions Section 10.1 6 2 + 5 + 100 22. ( ) = Technology formula: (6*x^2+5*x+100)/(3*x^2-9) 3 2 9 Limit is 2. Table:

10

100

1, 000

10, 000 100, 000

( ) 2.57732 2.02061 2.00171 2.00017 2.00002

5 1, 000 4 Technology formula: (x^5-1000*x^4)/(2*x^5+10000) 2 5 + 10, 000 Limit is 0.5. Table: 23. ( ) =

10

( ) 53.15789

1, 000 100, 000 10, 000, 000 1, 000, 000, 000 1

0.50500

0.50005

0.50000

6 + 3, 000 3 + 1, 000, 000 2 6 + 1, 000 3 Technology formula: (x^6+3000*x^3+1000000)/(2*x^6+1000*x^3) Limit is 0.5. Table: 24. ( ) =

10

( )

1

1, 000 10, 000 100, 000

100

0.49876 0.50000 0.50000

0.50000

10 2 + 300 + 1 Technology formula: (10*x^2+300*x+1)/(5*x+2) 5 + 2 Diverges to . Table: 25. ( ) =

10

100

1, 000

10, 000

100, 000

( ) 76.9 259.0 2,059.2 20,059.2 200,059.2

2 4 + 20 3 Technology formula: (2*x^4+20*x^3)/(1000*x^6+6) 1,000 6 + 6 Limit is 0. Table: 26. ( ) =

10

100

1,000

10,000

100,000

( ) 4 × 10 5 2.2 × 10 7 2.02 × 10 9 2.002 × 10 11 2.0002 × 10 13

10 2 + 300 + 1 Technology formula: (10*x^2+300*x+1)/(5*x^3+2) 5 3 + 2 Limit is 0. Table: 27. ( ) =

10

100

1,000

10,000

100,000

( ) 0.79988 0.02600 0.00206 0.00020 2.0006 × 10 5


Solutions Section 10.1 2 4 + 20 3 28. ( ) = Technology formula: (2*x^4+20x^3)/(1000*x^3+6) 1,000 3 + 6 Diverges to . Table:

10

100 1, 000 10,000 100,000

( ) 0.04

0.22

2.02

20.02

200.02

1.999

1.9999

29. ( ) = 2 Technology formula: e^(x-2) or exp(x-2) Limit is 1. Table:

1.9

1.99

2

( ) 0.90484 0.99005 0.99900 0.99990

2.0001

2.001

2.01

2.1

1.00010 1.00100 1.01005 1.10517

30. ( ) = Technology formula: e^(-x) or exp(-x) Limit is 0. Table:

10

100

( ) 4.53999 × 10 5 3.72008 × 10 44

1,000 10,000 100,000 0

0

0

(The last three values of ( ), while not mathematically 0, are too small to be represented in various technologies, which just give the values as 0.) 31. ( ) = Technology formula: x*e^(-x) or x*exp(-x) Limit is 0. Table:

10

100

( ) 0.000454 3.7201 × 10 42

1,000 10,000 100,000 0

0

0

(The last three values of ( ), while not mathematically 0, are too small to be represented in various technologies, which just give the values as 0.) 32. ( ) = Technology formula: x*e^x or x*exp(x) Limit is 0. Table:

10

100

( ) 0.00045 3.7201 × 10 42

1, 000 10, 000 100, 000 0

0

0

(The last three values of ( ), while not mathematically 0, are too small to be represented in various technologies, which just give the values as 0.) 33. ( ) = ( 10 + 2 5 + 1) Technology formula: (x^10+2*x^5+1)*e^x or (x^10+2*x^5+1)*exp(x) Limit is 0. Table:

10

100

( ) 453, 990 3.72008 × 10 24

1, 000 10, 000 100, 000 0

0

0

(The last three values of ( ), while not mathematically 0, are too small to be represented in various technologies, which just give the values as 0.)


Solutions Section 10.1 34. ( ) = ( 50 + 30 + 1) Technology formula: (x^50+x^30+1)*e^(-x) or (x^50+x^30+1)*exp(-x) Limit is 0. Table:

10

100

( ) 4.53999 × 10 45 3.72008 × 10 56

1, 000 10, 000 100, 000 0

0

0

(The last three values of ( ), while not mathematically 0, are too small to be represented in various technologies, which just give the values as 0.) 35. a. 2

b. 1

36. a. 2

b. 1

38. a. 1

b. 1

37. a. 2

b. 1

c. 0

c. 1

d.

d.

39. a. 0 b. 2: As approaches 0 from the right, ( ) approaches the solid dot at height 2. c. 1: As approaches 0 from the left, ( ) approaches the open dot at height 1. The fact that (0) = 2 is irrelevant. d. Does not exist: Parts (b) and (c) show that the one-sided limits, though they both exist, do not agree. e. 2: The solid dot indicates the actual value of (0). f. 40. a. 1 b. 1: As approaches 1 from the right, ( ) approaches the open dot at height 1. The closed dot at height 1 is irrelevant. Similarly: c. 1 d. 1: The two one-sided limits exist and have the same value, 1. e. 1: The value of the function itself is given by the closed dot on the graph. f. 41. a. 1

42. a. 1

b. 1: Similar to Exercise 23.

b. 3: Similar to Exercise 24

c. 2

c. 2

d. Does not exist d. Does not exist

e. 1

e. 2

f. 2

f. 0

43. a. 1 b. = 1 is a left endpoint of the domain of . Thus, the limit from the left cannot exist, as there are no points in the domain of to the left of = 1. c. Does exist and equals the right limit 1, as we care only about points near = 1 and in the domain; i.e., to the right of 1. d. 1

44. a. Does not exist ( = 0 is a right endpoint of the domain) b. 2 c. Does exist and equals the left limit 2, as we care only about points near = 0 and in the domain; i.e., to the left of 0. d. 2 45. a. 1 46. a. 0

b. b.

c.

c.

d. e. Undefined

f. 1

d. Does not exist

e. Undefined

f. 2

47. a. 1: Approaching from the left or the right, the value of ( ) approaches the height of the open dot, 1. b. c. d. Does not exist e. 2 f. 1 : The value of the function is given by the closed dot on the graph. 48. a.

b. 0

c.

d. 0 e. 0

f. 1


Solutions Section 10.1 49. Here is a table of values for ( ): Technology formulas: TI-83/84 Plus: (2.5x^2-2x+12)e^(-0.05x^2)+6 Spreadsheet: (2.5*x^2-2*x+12)*EXP(-0.05*x^2)+6 ( )

0

10

50

100

18

7.631 6.000 6.000

At gets larger and larger, the values of ( ) are getting closer and closer to 6. Thus, we estimate lim ( ) = 6 million housing starts per year

In the long term, the model predicts that there will be 6 million housing starts per year in the U.S.

50. Here is a table of values for ( ): Technology formulas: TI-83/84 Plus: (10x^2-x+6)e^(-0.04x^2)+3 Spreadsheet: (10*x^2-x+6)*EXP(-0.04*x^2)+3 ( )

0

10

9

50

100

21.242 3.000 3.000

At gets larger and larger, the values of ( ) are getting closer and closer to 3. Thus, we estimate lim ( ) = 3 million housing starts per year

In the long term, the model predicts that there will be 3 million housing starts per year in the South.

51. a. Here is a table of values for ( ): Technology formulas: TI-83/84 Plus: 4.7/(1+139*e^(-X)) Spreadsheet: 4.7/(1+139*EXP(-X)) 0

10

20

30

40

( ) 0.03357 4.67053 4.70000 4.70000 4.70000

As gets larger and larger, the values of ( ) are getting closer and closer to 4.7. Thus, we estimate lim ( ) = 4.7.

The model predicts that, had spending on NASA continued to follow the pattern leading up to 1966, annual spending on NASA in the long term would have amounted to 4.7\% of the U.S. federal budget. b. Recent spending (as of 2020) is less than 0.5\% of the U.S. federal budget, so the prediction of 4.7\% is not even close. 52. a. Here is a table of values for ( ): Technology formula: 4.5/(1.07^((X-8)^2)) 0

10

20

30

40

( ) 0.05924 3.43303 0.00026 2.7 × 10 14 3.7 × 10 30

As gets larger and larger, the values of ( ) are decreasing toward 0. Thus, we estimate lim ( ) = 0.

The model predicts that, had spending on NASA continued to follow the pattern leading up to 1966, annual spending on NASA in the long term would have decreased toward zero. b. Current spending (as of 2014) is less than 0.5\% of the U.S. federal budget, so the prediction of 0\% is, unfortunately, quite close. 53. Technology formula: 90.6-63.7/(1+(1.78)*1.37^(-t))


Solutions Section 10.1 To find the long-term projection of ( ), take the limit as (we can estimate this limit numerically or graphically): 63.7 = 26.9% lim ( ) = lim 90.6 1 1.78(1.37) +

54. Technology formula: 28.6/(1+10.4(1.30)^(-t)) To find the long-term projection of ( ), take the limit as (we can estimate this limit numerically or graphically): 28.6

lim ( ) = lim

1 + 10.4(1.30)

= 28.6%

55. Here is a table of values for ( ): Technology formula: 573-133*(0.987)^x

0

10

50

100

200

1000

10,000

( ) 440 456.3128 503.8632 537.0609 563.2886 572.9997 573.0000

At gets larger and larger, the values of ( ) are getting closer and closer to 573. Thus, we estimate lim ( ) = 573.

Since ( ) represents the average SAT math score of students whose household income is thousand dollars per year, we interpret the result as follows: Students whose parents earn an exceptionally large income would score an average of 573 on the SAT math test. 56. Here is a table of values for ( ): Technology formula: 550-136*(0.985)^x

0

10

50

100

200

1000

10,000

( ) 414 433.076660 486.12213 519.99719 543.38111 549.99996 550.00000

At gets larger and larger, the values of ( ) are getting closer and closer to 550. Thus, we estimate lim ( ) = 550.

Since ( ) represents the average SAT math score of students whose household income is thousand dollars per year, we interpret the result as follows: Students whose parents earn an exceptionally large income would score an average of 550 on the SAT critical reading test.

57. a. As approaches 14.75 (representing 2:45 pm) from the left, ( ) is approaching 21. Similarly, as approaches 14.75 from the right, ( ) is approaching 21. Thus, the left and right limits agree, and the limit exists as well: lim ( ) = lim ( ) = lim ( ) = 21.On the other hand, the solid dot at (14.75, 0.01) tells 14.75

14.75 +

14.75

you that (14.75) = 0.01 (so that the stock was briefly worth only a penny). b. Just before 2:45 pm, the stock was approaching $21, but it then fell suddenly to a penny ($0.01) at 2:45 exactly, after which time it jumped back to values close to $21. (Note that the $50 value of the stock that prevailed for most of the day has nothing to do with the limit at 14.75.)

58. a. As approaches 14.75 (representing 2:45 pm) from the left, ( ) is approaching 66. As approaches 14.75 from the right, ( ) is approaching 10. Thus, lim

14.75

( ) = 66,

lim

14.75 +

( ) = 10, lim ( ) does not exist. 14.75


Solutions Section 10.1 The solid dot at (14.75, 10) tells you that (14.75) = 10. b. Just prior to 2:45 pm, the S\&P market depth was approaching 66 million shares, but then fell suddenly to 10 million shares at 2:45 exactly, after which time it began to recover at values close to 10 million shares. 59. As gets larger and larger, ( ) oscillates about, and approaches 175, so we estimate lim ( ) = 175.

In the long term, the home price index will level off at 175 points.

60. As gets larger and larger, ( ) appears to increase without bound, so we estimate lim ( ) = .

In the long term, the home price index will increase without bound.

61. lim ( ) = 0.06, lim ( ) = 0.08, so lim ( ) does not exist. 1

1+

1

11

11 +

11

62. lim ( ) = 35, lim ( ) = 20, so lim ( ) does not exist.

63. lim ( ) = , lim ( ( ) ( )) 2.5. In the long term, U.S. imports from China will rise without bound and be 2.5 times U.S. exports to China. In the real world, imports and exports cannot rise without bound. Thus, the given models should not be extrapolated far into the future.

64. lim ( ) = , lim ( ( ) ( )) 0.4.In the long term, U.S. exports to China will rise without bound and be 0.4 times U.S. imports from China. In the real world, imports and exports cannot rise without bound. Thus, the given models should not be extrapolated far into the future.

65. To approximate lim ( ) numerically, choose values of closer and closer to and on either side of = , and evaluate ( ) for each of them. The limit (if it exists) is then the number that these values of ( ) approach. A disadvantage of this method is that it may never give the exact value of the limit, but only an approximation. (However, we can make the approximation as accurate as we like.)

66. To approximate lim ( ) graphically, draw the graph of either by hand or with technology. Then place a pencil point (or the trace cursor) on the graph and to the left of the point where = , and move it along the curve toward the point where = , reading off the !-coordinates as you go. The left limit (if it exists) is the value that the !-coordinates are approaching. Similarly, starting to the right of the point where = and moving left along the graph gives the right limit. If both limits exist and are the same, the common value is the limit. This method has the same disadvantage as the numerical method: We obtain only an approximate value. 67. Any situation in which there is a sudden change can be modeled by a function in which lim + ( ) is not the same as lim ( ). One example is the value of a stock market index before and after a crash: lim ( ) is the value immediately before the crash at time = , while lim + ( ) is the value immediately after the crash. Another example might be the price of a commodity that is suddenly increased from one level to another. 68. Answers may vary. In Examples 1(a) and (b), putting = though both limits exist.

leads to an undefined expression, even

69. It is possible for lim ( ) to exist even though ( ) is not defined. An example is


2 3 + 2 lim . 1 1

Solutions Section 10.1

70. In Example 4(b) (0) is defined, but lim ( ) does not exist. 71. An example is ( ) = ( 1)( 2).

72. It could, by expanding more and more slowly as it approached the limit of 130,000 billion light years. 73. These limits are all 0. 74. These limits are all 0.


Solutions Section 10.2 Section 10.2

1. The natural domain of ( ) = 4 2 2 consists of all real numbers. Therefore, = 2 is in its domain, and so Theorem C applies, and lim ( 4 2 2) = 4(2 2) 2 = 18. 2

2. The natural domain of ( ) =

1 consists of all real numbers. Therefore, = 1 is in its domain, 2 + 1

and so Theorem C applies, and 1 1 1 lim = = . 2 1 2 + 1 ( 1) + 1 2

3. The natural domain of ( ) = + 1 consists of all real numbers " 1. Therefore, = 2 is in its domain and so Theorem C applies, and lim + 1 = 3. 2

4. The natural domain of ( ) = 1 consists of all real numbers " 1. Therefore, = 10 is in its domain and so Theorem C applies, and lim 1 = 9 = 3. 10

3 consists of all real numbers except when the denominator is 2 + 1 zero: 2 + 1 = 0 # = 1 2. If is any other number, then Theorem C applies, and 3 3 lim . = ( ) = 2 + 1 2 + 1 5. The natural domain of ( ) =

3 4 consists of all real numbers except when the denominator is + 1 zero: + 1 = 0 # = 1. If is any other number, then Theorem C applies, and 3 4 3 4 lim . = ( ) = + 1 + 1 6. The natural domain of ( ) =

7. No singular points: The function is defined everywhere, including at = 2.

8. The only point at which the function is not defined is = 2, so it is the only singular point.

9. The only point at which the function is not defined is = 2, so it is the only singular point.

10. The only point at which the function is not defined is = 1, so it is the only singular point.

11. Substituting = 2 leads to the indeterminate form 0 0, so = 2 is not in the domain of the function, and we try factoring: ( 2) + 2 1 1 lim = lim = lim = = 1. 2 2 5 + 6 2 ( 2)( 3) 2 3 2 3 12. Substituting = 1 2 leads to the indeterminate form 0 0, so = 1 2 is not in the domain of the function, and we try factoring: (2 + 1)(3 2) 6 2 2 1 7 lim lim lim (3 2) = 3$ % 2 = . = = 1 2 1 2 1 2 2 + 1 2 + 1 2 2


Solutions Section 10.2 13. Substituting = 1 leads to the indeterminate form 0 0, so = 1 is not in the domain of the function, and we try factoring: (3 + 1)( + 1) 3 2 + 4 + 1 3 + 1 3( 1) + 1 lim = lim = lim = = 1. 1 1 ( 1)( + 1) 1 1 ( 1) 1 2 1 14. Substituting = 1 leads to the indeterminate form 0 0, so = 1 is not in the domain of the function, and we try factoring: ( + 1)( + 2) 2 + 3 + 2 + 2 ( 1) + 2 lim = lim = lim = = 1. 1 1 1 ( + 1) 1 2 + 15. You can check that = 1 5 is not in the domain of the function, so you need to factor the denominator and cancel. 5 + 1 5 + 1 lim lim = 3 1 5 125 + 1 1 5 (5 + 1)(25 2 5 + 1) 1 1 1 lim = = = . 2 1 5 25 2 5 + 1 25( 1 5) 5( 1 5) + 1 3

16. Substituting = 3 leads to the indeterminate form 0 0, so = 3 is not in the domain of the function, and we try factoring: 3+ 3+ 1 1 1 lim . = lim = lim = = 3 27 + 3 3 (3 + )(9 3 + 2) 3 9 3 + 2 9 3( 3) + ( 3) 2 27

17. Substituting = 1 4 leads to the indeterminate form 0 0, so = 1 4 is not in the domain of the function, and we try factoring: (4 1)(16 2 + 4 + 1) 64 3 1 64 3 1 lim lim lim = = 1 4 4 2 23 + 6 1 4 (4 2 + 23 6) 1 4 (4 1)( + 6) 2 2 16(1 4) 4(1 4) 1 + + 16 + 4 + 1 12 = lim = = . 1 4 (1 4) + 6 25 ( + 6) 18. Substituting = 5 2 leads to the indeterminate form 0 0, so = 5 2 is not in the domain of the function, and we try factoring: (8 3 125) (2 5)(4 2 + 10 + 25) 8 3 + 125 lim lim lim = = 5 2 2 2 + 7 30 5 2 2 2 + 7 30 5 2 (2 5)( + 6) 2 2 4(5 2) + 10(5 2) + 25 (4 + 10 + 25) 150 . = lim = = 5 2 + 6 (5 2) + 6 17

19. Substituting = 0 leads to 60 0, which is the determinate form & 0. Since 4 is nonnegative, the quantity 60 4 is nonnegative, so the limit diverges to .

20. Substituting = 0 leads to 0 0, an indeterminate form. Canceling 2 from top and bottom leads to lim 2 , and substituting = 0 leads to the determinate form & 0. Since , and hence the entire 0

expression 2 , could be either positive or negative depending on whether 0 + or 0 , the limit does not exist.

21. Substituting = 0 leads to 1 0, which is the determinate form & 0. Since the denominator 3, and hence the entire expression ( 3 1) 3, could be either positive or negative depending on whether 0 + or 0 , the limit does not exist. 22. Substituting = 0 leads to 2 0, which is the determinate form & 0. Since 2 is nonnegative, the quantity 2 2 is negative, so the limit diverges to .


Solutions Section 10.2 23. ( ) has = 1 as a singular point. Substituting = 1 in the expression gives the indeterminate form 0 0. Factoring the numerator and denominator and simplifying gives ( + 1)( + 2) + 2 ( ) = = + 1 ( + 1) 2 Substituting = 1 leads to the determinate form 1 0. As 1 , the denominator is negative, so the left limit has sign opposite that of the numerator, 1. As 1 + , the denominator is positive, so the right limit has the same sign as the numerator, 1. as the left- and right-limits disagree, the two-sided limit does not exist.

24. Substituting = 2 leads to 13 0, which is the determinate form & 0. Since 2, and hence the entire expression ( 2 + 3 + 3) ( 2), could be either positive or negative depending on whether 2 + or 2 , the limit does not exist. 25. The function has = 2 as a singular point and substituting = 2 gives the indeterminate form 0 0. Factoring the numerator and denominator and simplifying gives 2 2 6 ( 2)(2 + 3) 2 + 3 . = = 2 2 4 + 4 ( 2) 2 Substituting = 2 now leads to 7 0, which is the determinate form & 0. Since 2, and hence the entire expression (2 + 3) ( 2), could be either positive or negative depending on whether 2 + or 2 , the limit does not exist.

26. The function has = 2 as a singular point and substituting = 2 gives the determinate form 1 0. However, it is difficult as written to see what the two one-sided limits are. Factoring the numerator and denominator and simplifying gives 2 2 + 5 + 3 ( + 1)(2 + 3) 2 + 3 . = = ( + 1)( + 2) + 2 2 + 3 + 2 Substituting = 2 leads to the determinate form 1 0. As 2 , the denominator is negative, so the left limit has sign opposite that of the numerator, 2. As 2 + , the denominator is positive, so the right limit has the same sign as the numerator, 2. as the left- and right-limits disagree, the two-sided limit does not exist. 27. The function has = 3 as a singular point and substituting = 3 gives the indeterminate form 0 0. Factoring the numerator and denominator and simplifying gives 2 2 5 3 ( 3)(2 + 1) 2 + 1 . = = 5 5 32( 3) 4 (2 6) 32( 3) Substituting = 3 now leads to 7 0, which is the determinate form & 0. Since ( 3) 4 is positive, the quantity 7 (32( 3) 4) is positive, so the limit diverges to .

28. The function has = 2 as a singular point and substituting = 2 gives the indeterminate form 0 0. Factoring the numerator and denominator and simplifying gives 3 2 + 7 + 2 ( + 2)(3 + 1) 3 + 1 . = = 3 3 (2 + 4) 8( + 2) 8( + 2) 2 Substituting = 2 now leads to 5 0, which is the determinate form & 0. Since ( + 2) 2 is positive, the quantity 5 (8( + 2) 2) is negative, so the limit diverges to . 29. Substituting = leads to + 5, which is the determinate form & = , so the limit diverges to .

30. Substituting = 0 leads to 4 0, which is the determinate form & 0. Since the denominator , and hence the entire expression (2 2 + 4) , could be either positive or negative depending on whether


Solutions Section 10.2 0 + or 0 , the limit does not exist.

31. Substituting = leads to 4 , which is the determinate form & = 0, so the limit is 0.

32. Substituting = leads to (60 + ) (2 Thus, the limit is (60 + 0) (2 0) = 30.

), where

is the determinate form & = 0.

33. Substituting = 0 leads to the indeterminate form 0 0. Canceling the 3 gives us the limit of 1 3. 34. Substituting = leads to the determinate form + & = . Thus, the limit diverges to .

35. Substituting = leads to the indeterminate form . Canceling the 3 gives us 1 3 3, resulting in the determinate form ( 1) ( ) = 0. 36. Substituting = leads to the indeterminate form . Canceling the 3 gives us 3 3, resulting in the determinate form 3 = . Thus, the limit diverges to .

37. Substituting = leads to the determinate form 4 ( + 2) = 4 = 0. Thus, the limit is 0.

38. Substituting = leads to the determinate form

= 0. Thus, the limit is 0.

39. Substituting = leads to the determinate form 60 ( 1) = 60 (0 1) = 60. Thus, the limit is 60. 40. Substituting = leads to the determinate form 2 ( + 3) = 2 = 0. Thus, the limit is 0.

41. You can check that = 8 is in the domain of the function, so Theorem C applies. lim (3 2 + 19 + 6) = 3( 8) 2 + 19( 8) + 6 = 46 8

42. You can check that = 8 is in the domain of the function, so Theorem C applies. lim ( 2 4 + 12) = ( 8) 2 4( 8) + 12 = 20 8

43. You can check that = 1 is in the domain of the function, so Theorem C applies. 1 3 3 lim = = 2 2 1 2 + 15 + 7 2( 1) + 15( 1) + 7 2

44. ( ) has = 3 as a singular point, so factor the numerator and denominator and simplify if possible. 4 ( ) = ( 3) 2 Substituting = 3 leads to the determinate form 4 0. As 3 from either side, the denominator, having an even exponent, is positive, so both the left and right limits are the same, and have the same sign as 4. Thus 4 lim = . 3 2 6 + 9 45. ( ) has = 2 as a singular point, so factor the numerator and denominator and simplify if possible. 5 ( ) = ( 2) 2 Substituting = 2 leads to the determinate form 5 0. As 2 from either side, the denominator, having


Solutions Section 10.2 an even exponent, is positive, so both the left and right limits are the same, and have the same sign as 5. Thus 5 lim = . 2 2 4 + 4 46. You can check that = 1 is in the domain of the function, so Theorem C applies. 1 1 1 lim 2 = = 2 1 + 2 35 36 ( 1) + 2( 1) 35

47. Substituting = 0 leads to 1 0, which is the determinate form & 0. Since the denominator 3, and hence the entire expression ( 1) 3, could be either positive or negative depending on whether 0 + or 0 , the limit does not exist. 48. Substituting = 0 leads to 2 0, which is the determinate form & 0. Since 2 is nonnegative, the quantity 2 2 is negative, so the limit diverges to .

49. Substituting = 1 leads to 2 0, which is the determinate form & 0. Since the denominator ( 1) 3, and hence the entire expression ( + 1) ( 1) 3, could be either positive or negative depending on whether 1 + or 1 , the limit does not exist. 50. Substituting = 2 leads to 2 0, which is the determinate form & 0. Since ( 2) 2 is nonnegative, the quantity 2 ( 2) 2 is negative, so the limit diverges to . 51. The given function has = 1 in its domain, and so Theorem C applies: 4 2 7 2 4( 1) 2 7( 1) 2 9 lim = = . 1 2 + 1 ( 1) + 1 52. The given function has = 3 in its domain, and so Theorem C applies: 3 3 3 0 lim = = = 0. 2 2 3 2 + 5 3 2(3) + 5(3) 3 30

53. Substituting = 2 leads to the indeterminate form 0 0, so = 2 is not in the domain of the function, so we try factoring: (2 + 1)( + 2) 2 2 + 5 + 2 2 + 1 2( 2) + 1 lim = lim = lim = = 3. 2 2 + 3 + 2 2 ( + 1)( + 2) 2 + 1 2 + 1

54. Substituting = 6 leads to the indeterminate form 0 0, so = 6 is not in the domain of the function, so we try factoring: ( 6)( + 1) 2 5 6 + 1 6+ 1 7 lim lim . = = lim = = 6 2 36 6 ( 6)( + 6) 6 + 6 6 + 6 12 55. Substituting = 1 gives the determinate form 12 0. The denominator factors as ( + 1) 2, so is always positive. Therefore the left and right limits are both . Thus, the limit diverges to as well.

56. Substituting = 2 gives the determinate form 6 0. The denominator factors as ( 2) 2, so is always positive. Therefore the left and right limits are both . 57. Substituting = 2 gives the indeterminate form 0 0. The fraction simplifies: ( 2) 2 2 4 + 4 2 2 2 lim 2 = lim = lim = = 0. 2 2 2 ( 2)( + 1) 2 + 1 2+ 1


Solutions Section 10.2 58. Substituting = 1 gives the indeterminate form 0 0. The fraction simplifies: ( + 1) 2 2 + 2 + 1 + 1 1 + 1 lim lim = = lim = = 0. 1 4 2 4 1 4( + 1)( 1) 1 4( 1) 4( 1 1)

59. Substituting = 1 2 leads to the indeterminate form 0 0, so = 1 2 is not in the domain of the function, so we try factoring: (2 1)(4 2 + 2 + 1) 8 3 1 lim = lim 1 2 2 1 1 2 2 1 = lim (4 2 + 2 + 1) = 4(1 2) 2 + 2(1 2) + 1 = 3. 1 2

60. The given function has = 4 in its domain, and so Theorem C applies: 3 4 (4) 3 4 lim = = 30. 4 2 4 2

61. Substituting = 1 3 leads to the indeterminate form 0 0, so = 1 3 is not in the domain of the function, so we try factoring: (3 2 + 11 4) (3 1)( + 4) 3 2 11 + 4 lim lim = = lim 1 3 1 3 1 3 (3 1)(9 2 + 3 + 1) 27 3 1 27 3 1 (1 3) + 4 ( + 4) 13 = lim = = . 2 1 3 9 2 + 3 + 1 9 9(1 3) + 3(1 3) + 1 62. Substituting = 2 3 leads to the indeterminate form 0 0, so = 2 3 is not in the domain of the function, so we try factoring: (3 2)(2 3) 6 2 13 + 6 6 2 13 + 6 lim lim = = lim 3 3 2 3 27 + 8 2 3 (27 8) 2 3 (3 2)(9 2 + 6 + 4) 2(2 3) 3 2 3 5 . = lim = = 2 2 2 3 (9 + 6 + 4) 9(2 3) + 6(2 3) + 4 36 2

63. lim

2 + 2

64. lim 3

65. lim

9

3

3

3

9

= =

lim

2+

lim

2 2 2

3

3 3 3

= lim

9

( 3)( + 3) ( 9)( + 3)

= =

lim 2 = 2 2 = 0

2+

lim

1

3 3

= lim

=

9

9 ( 9)( + 3)

= lim

1

9 + 3

= 1 6

( 4)( + 2) ( 4)( + 2) 4 = lim = lim = lim ( + 2) = 4 4 2 4 ( 2)( + 2) 4 4 4

66. lim

3 2 + 10 1 3 2 3 3 lim = = lim = 2 2 2 2 2 5 2

67. lim

6 2 + 5 + 100 6 2 6 lim = = lim = 2 3 2 3 3 2 9

68. lim

5 1, 000 4 5 1 1 = lim = lim = 5 5 2 + 10, 000 2 2 2

69. lim


Solutions Section 10.2 6 + 3, 000 3 + 1, 000, 000 6 1 1 70. lim = lim = lim = 6 3 6 2 2 2 2 + 1, 000 10 2 + 300 + 1 10 2 = lim = lim 2 = 5 5 + 2

71. lim

2 4 + 20 3 2 4 lim = = lim = 1, 000 3 + 6 1, 000 3 500

72. lim

3 2 + 10 1 3 2 3 3 lim = = lim = 2 2 2 2 2 2 5

73. lim

6 2 + 5 + 100 6 2 lim = = lim 2 = 2 3 2 3 2 9

74. lim

5 1, 000 4 5 1 1 = lim = lim = 2 5 + 10, 000 2 5 2 2

75. lim

6 + 3, 000 3 + 1, 000, 000 6 1 1 = lim = lim = 6 3 6 2 2 2 2 + 1, 000

76. lim

10 2 + 300 + 1 10 2 = lim = lim 2 = 5 5 + 2

77. lim

2 4 + 20 3 2 4 lim = = lim = 1, 000 3 + 6 1, 000 3 500

78. lim

10 2 + 300 + 1 10 2 2 lim = = lim = 0 5 3 5 3 + 2

79. lim

2 4 + 20 3 2 4 1 = lim = lim = 0 1, 000 6 + 6 1, 000 6 500 2

80. lim

81. lim (4 3 + 12) =

lim

4

$ 3

This limit has the determinate form 82. lim

2

5 5.3 3

=

lim

2

5 5.3(3 3 )

4 + 12 = 0 + 12 = 12.

2

5 5.3 3

This limit has the determinate form 83. lim

+ 12%

2 2 2 = = . 5 5.3 5 0 5

has the determinate form

2 2 = = 0. 5 5.3( )

84. lim (4.1 2 3 ) has the determinate form (4.1 2( )) = .

2 3 2 3 lim . = 1 + 5.3 1 + 5.3

85. lim


Solutions Section 10.2 This limit has the determinate form = = . 1 + 5.3 1 + 0 4.2

86. lim

2 3 2

has the determinate form

4.2 4.2 = = 2.1. 2 3 2 0

0 3 2 3 has the determinate form = = 0. 2 + 2 + 2 + 0

87. lim

2 3 2 0 has the determinate form = = 0. 1 + 5.3 1 + 5.3 1+ 0

88. lim

89. lim ' ( ) = lim ( 0.32 2 + 3.6 + 13) = '( ) 0.32 0.32 2 + 3.6 + 13 0.32 2 lim lim 1.78 = lim = = lim 2 2 (( ) 0.18 0.18 + 2.8 + 17 0.18 In the long term, the popularity of Twitter among social media sites will decrease without bound and be about 1.78 times the popularity of LinkedIn. However, a percentage cannot decrease to less than 0, so extrapolating the models to obtain long-term predictions gives meaningless results.

90. lim ) ( ) = lim ( 0.26 2 + 3.9 + 55) = )( ) 0.26 2 + 3.9 + 55 0.26 2 0.26 lim lim 0.72 = lim = = lim 2 2 ( ) 0.36 0.36 + 6.6 + 10 0.36 In the long term, the popularity of Facebook among social media sites will decrease without bound and be about 0.72 times the popularity of Instagram. However, a percentage cannot decrease below 0, so extrapolating the models to obtain long-term predictions gives meaningless results. *( )

1.745 + 29.84 1.745 1.745 1.59 = lim = lim 1.097 + 10.65 1.097 If the trend continued indefinitely, the annual spending on police would be 1.59 times the annual spending on courts in the long run. tage of newspapers' revenue from movie advertising jumped suddenly in 1999. 91. lim

( )

= lim

1.097

*( )

1.745 + 29.84 1.745 1.745 0.37 = lim = lim 4.761 + 52.85 4.761 If the trend continued indefinitely, the annual spending on police would be 37% of the total spending on law enforcement (police, courts, and prisons) in the long run. 92. lim

* ( ) + ( ) + +( )

= lim

4.761

93. As , the term 1.37 =

1 0, so 1.37

63.7 = 90.6 63.7 = 26.9 lim ( ) = lim 90.6 1+ 0 1 1.78(1.37) +

Interpretation: If the model is extrapolated to the indefinite future, it would predict an R&D expenditure attributable to the U.S. approaching 26.9% in the long term. 94. As , the term 1.30 = lim ( ) = lim

1 0, so 1.30

28.6

1 + 10.4(1.30)

=

28.6 = 28.6 1+ 0

Interpretation: If the model is extrapolated to the indefinite future, it would predict an R&D expenditure attributable to China approaching 28.6% in the long term.


Solutions Section 10.2 95. Here is a table of values for ( ): Technology formula: 573-133*(0.987)^x

0

10

50

100

200

1000

10,000

( ) 440 456.3128 503.8632 537.0609 563.2886 572.9997 573.0000

At gets larger and larger, the values of ( ) are getting closer and closer to 573. Thus, we estimate lim ( ) = 573.

Since ( ) represents the average SAT math score of students whose household income is thousand dollars per year, we interpret the result as follows: Students whose parents earn an exceptionally large income would score an average of 573 on the SAT math test. 96. Here is a table of values for ( ): Technology formula: 550-136*(0.985)^x

0

10

50

100

200

1000

10,000

( ) 414 433.076660 486.12213 519.99719 543.38111 549.99996 550.00000

At gets larger and larger, the values of ( ) are getting closer and closer to 550. Thus, we estimate lim ( ) = 550.

Since ( ) represents the average SAT math score of students whose household income is thousand dollars per year, we interpret the result as follows: Students whose parents earn an exceptionally large income would score an average of 550 on the SAT critical reading test.

97. a. Since ,( ) = 400 2,200 when < 25 and is a closed-form function in this range, we compute the limit lim 25 ,( ) by substituting = 25 to get lim 25 ,( ) = lim 25 (400 6,200) = 400(25) 6,200 = 3,800. Thus, shortly before 2005 ( = 25), the speed of Intel processors was approaching 3,800 MHz. Since ,( ) = 0.45( 25) 3 + 3,800 when > 25, lim 25 + ,( ) = lim 25 + [0.45( 25) 3 + 3,800] = 0.45(25 25) 3 + 3,800 = 3,800- as well. Thus, shortly after 2005 ( = 25), the speed of Intel processors was 3,800 MHz. b. Since lim 25 ,( ) = lim 25 + ,( ) = 3,800, the processor speed of Intel processors did not change abruptly at the start of 2005 ( = 25). 98. a. Since ,( ) = 3 when < 20 and is a closed-form function in this range, we compute the limit lim 20 ,( ) by substituting = 20 to get lim 20 ,( ) = lim 20 3 = 60. Thus, shortly before 1990 ( = 20), the speed of Intel processors was approaching 60 MHz. Since ,( ) = 174 3,420 when > 20, lim 20 + ,( ) = lim 20 + (174 3,420) = 174(20) 3,420 = 60- as well. Thus, shortly after 1990 ( = 20), the speed of Intel processors was close to 60 MHz. b. Since lim 20 ,( ) = lim 20 + ,( ) = 60, the processor speed of Intel processors did not change abruptly at the start of 1990 ( = 20).

99. To evaluate lim ( ) algebraically, first check whether ( ) is a closed-form function. Then check whether = is in its domain. If so, the limit is just ( ); that is, it is obtained by substituting = . If not, then try to first simplify ( ) in such a way as to transform it into a new function such that = is in its domain, and then substitute. A disadvantage of this method is that it is sometimes extremely difficult to evaluate limits algebraically, and rather sophisticated methods are often needed.


Solutions Section 10.2 100. A closed-form function is any function that can be obtained by combining constants, powers of , exponential functions, radicals, logarithms, and trigonometric functions (and some other functions we shall not encounter in this text) into a single mathematical formula by means of the usual arithmetic operations and composition of functions. Limits of closed form funtions at points in their domain can be obtained by substitution; that is, if is in the domain of the closed-form function , then lim ( ) exists and equals ( ).

101. = 3 is not in the domain of the given function , so, yes, the function is undefined at = 3. However, the limit may well be defined. In this case, it leads to the indeterminate form 0/0, telling us that we need to try to simplify, and that leads us to the correct limit of 2

3

27 27:lim 3 3 = lim 3 ( 3)( 3+ 3 + 9) = lim 3 ( 2 + 3 + 9) = 3 2 + 3(3) + 9 = 27.

102. 0 0 is neither defined nor equal to zero. = 1 is not in the domain of the function, and the indeterminate form 0/0 tells us that we need to try to simplify, and that leads us to the correct answer: : 1 lim 1 2 1 = lim 1 1 2 = lim 1 1 = . 2 + 1

( 1)

103. She is wrong. The limits of closed form functions are guaranteed to exist only at points in their domains, and = 2 is not in the domain of the closed-form function ( ) = 1 ( 2) 2. (It is a singular point.) 104. An example is ( ) =

.0

if / 2 . is not a closed-form function. if > 0 3

8 105. Answers may vary. (1) See Quick Example 9: lim 2 2 which leads to the indeterminate form

0/0, but the limit is 12. (2) lim 60 which leads to the indeterminate form but where the limit 2 exists and equals 30.

106. Answers may vary. (1) lim 0 1 , which leads to the determinate form & 0 and where the limit does

not exist. (2) lim 1

1 , which leads to the indeterminate form 0 0 and where the limit does not exist. ( 1) 2

107. As the numerator is a polynomial, we know that ( ) ± as , and that as . Thus, the resulting form is ± . The limits are zero, as can be checked graphically. This suggests that limits resulting in ( ) are zero. 108. ± 0 0 The limits are zero. This suggests that limits resulting in ( ) are zero.

109. The statement may not be true if is not a closed-form function such as ( ) = matleft * , if / 2!0, if > 0, where (2) is defined and equals 2, but the limit does not exist as the left and right limits disagree. The statement can be corrected by requiring that be a closed-form function: "If is a closed form function, and ( ) is defined, then lim ( ) exists and equals ( )." 110. The statement may not be true, as in Example 1(b). The limit lim ( ) does not depend on the value at the point . 111. Answers may vary. (1) lim (( + 5) ) = lim (5) = 5


Solutions Section 10.2 (2) lim ( 2 ) = lim ( 1) = (3) lim (( 5) ) = lim ( 5) = 5 112. Answers may vary. (1) lim 1 = lim 1 = 1 (2) lim (1 + 1 ) = 2

(3) lim (1 + 1 ) =


Solutions Section 10.3 Section 10.3 1. Continuous on its domain 2. Continuous on its domain (0 is a singular point, and so not in its domain.) 3. Continuous on its domain (1 is a singular point, and so not in its domain.) 4. Continuous on its domain

5. Discontinuous at 0: lim 0 + ( ) 1 lim 0 ( ), so lim 0 ( ) does not exist. 6. Discontinuous at 1: lim 1 ( ) = 1 1 (1)

7. Discontinuous at 1: lim 1 + ( ) 1 lim 1 ( ), so lim 1 ( ) does not exist. 8. Discontinuous at 0: lim 0 + ( ) 1 lim 0 ( ), so lim 0 ( ) does not exist. 9. Continuous on its domain. In particular, it is continuous at the end-point 1: lim 1 ( ) = ( 1) = 1. 10. Continuous on its domain. In particular, it is continuous at the end-point 0: lim 0 ( ) = (0) = 2.

11. Continuous on its domain: Note that (0) is not defined, so 0 is not in the domain of .

12. Continuous on its domain: Note that (0) is not defined, so 0 is not in the domain of .

13. Discontinuous at 1 and 0: - lim 1 ( ) = 1 1 ( 1) and lim 0 ( ) does not exist. Note that (0) is defined; (0) = 2, so 0 is in the domain of . 14. Discontinuous at 0 and 1:- lim 1 ( ) = 0 1 (1) and lim 0 ( ) does not exist. Note that (0) is defined; (0) = 0, so 0 is in the domain of .

15. (A), (B), (D), (E) are continuous on their domains: Note that 1 is not in the domain in (B) and (D) and that the domain in (E) is ( , 1] (1, ); the "horizontal break" in the graph in (E) does not make the function discontinuous ( 1 is a domain endpoint). (C) is discontinuous at 1 because the limit there does not equal the function's value, and (F) is discontinuous at the endpoint of its domain. 16. (A), (D), (E), (F) are continuous on their domains: In (A), 2 is not in the domain; in (D) the domain is ( , 1] [2, ) and the "horizontal break" in the graph does not make the function discontinuous, because 1 and 2 are now endpoints of the domain. In (B) and (C) the one-sided limits at 2 exist but do not agree; the function is defined at 2 in each case, in contrast to (A). 17. Continuous: See Quick Example 3. 18. Singular:

= 0 is an isolated point at which is not defined.


Solutions Section 10.3 19. Discontinuous: The left and right limits at = 1 do not agree. 20. Continuous: See Quick Example 3. 21. Singular:

= 1 is an isolated point at which is not defined.

22. Discontinuous: The left and right limits at

= 2 do not agree.

23. Continuous: The left and right limits agree and equal the value of function at 24. Singular:

= 2 is an isolated point at which is not defined.

= 1.

25. The given function is a closed-form function, and therefore continuous on its domain. 26. The given function is a closed-form function, and therefore continuous on its domain. 27. The given function is a closed-form function, and therefore continuous on its domain. 28. The given function is a closed-form function, and therefore continuous on its domain.

+ 2 if < 0 .2 1 if " 0 The only possible point of discontinuity is where changes from one closed-form function to another, at 0: 29. ( ) =

lim ( ) =

0

lim ( ) =

0+

lim + 2 = 2.

0

lim 2 1 = 1.

0+

As the left and right limits disagree, lim 0 ( ) does not exist, and so the function is discontinuous at 0.

1 if / 1 . 1 if > 1 The only possible point of discontinuity is where changes from one closed-form function to another, at 1: 30. ( ) =

lim ( ) =

1

lim ( ) =

1+

lim 1 = 0.

1

lim 1 = 0.

1+

Thus, lim ( ) = 0. 1

Also, (1) = 1 1 = 0,

so lim 1 ( ) exists and equals (1), so the function is continuous at 1. As has no discontinuities, it is therefore continuous on its domain. 3 | | if 1 0 31. 2( ) = 6 4 6 if = 0 50 The only possible point of discontinuity is where 2 changes from one closed-form function to another, at = 0:


lim 2( ) =

0

lim 2( ) =

0+

lim | | = 1

0

lim | | = 1

0+

(As | | = for < 0).

Solutions Section 10.3 (As | | = for > 0).

As the left and right limits disagree, lim 0 2( ) does not exist, and so the function is discontinuous at 0. 3 1 if 1 0 2 32. 2( ) = 6 4 62 if = 0 5 The only possible point of discontinuity is where 2 changes from one closed-form function to another, at 0: lim 2( ) =

0

lim |1 2 = .

0

As the left limit is infinite, lim 0 2( ) does not exist in the sense that it is a finite number, and so the function is discontinuous at 0.

+ 2 if < 0 .2 + 2 if " 0 The only possible point of discontinuity is where changes from one closed-form function to another, at 0: 33. 7( ) =

lim 7( ) =

0

lim 7( ) =

0+

lim + 2 = 2.

0

lim 2 + 2 = 2.

0+

Thus, lim 7( ) = 2. 0

Also, 7(0) = 0 + 2 = 2,

So lim 0 7( ) exists and equals 7(1), and, and so the function is continuous at 0. As 7 has no discontinuities, it is therefore continuous on its domain.

1 if / 1 The only possible point of discontinuity is where 2 changes from one . + 1 if > 1 closed-form function to another, at 1: 34. 7( ) =

lim 7( ) =

1

lim 7( ) =

1+

lim 1 = 0

1

lim + 1 = 2

1+

As the left and right limits disagree, lim 1 7( ) does not exist, and so the function is discontinuous at 1.Discontinuity at 1

( 1) 2 2 2 + 1 = lim = lim ( 1) = 0. As the limit exists (and is finite), 1 1 1 1 1

35. lim ( ) = lim 1


Solutions Section 10.3 the singularity is removable, and setting (1) = 0 makes it continuous at 1.

( + 1)( + 2) 2 + 3 + 2 = lim = lim ( + 2) = 1. As the limit exists (and 1 1 1 1 + 1 + 1 is finite), the singularity is removable, and setting ( 1) = 1 makes it continuous at 1. 36. lim ( ) =

lim

37. lim ( ) = lim 0

0 3 2

= lim

0 (3 1)

= lim

1

0 3 1

= 1. As the limit exists (and is finite), the

singularity is removable, and setting (0) = 1 makes it continuous at 0.

2 3 does not exist; the left limit is on the right limit is . As the limit 4 4 + 4 does not exist, the singularity is essential. 38. lim ( ) =

lim

39. lim ( ) = lim 0

3

0 3 2

does not exist; the left limit is on the right limit is . As the limit does

not exist, the singularity is essential.

1 . As the limit exists (and is 3 finite), the singularity is removable, and setting (1) = 1 3 makes it continuous at 1. 40. lim ( ) = lim 1

1

1 3 1

= lim

1

1 ( 1)( 2 + + 1)

= lim

1

1 2 + + 1

=

41. We use a graph. (You could also use a table of values. If you use technology to show the graph, the hollow dot will not appear.)

Technology Formula: TI-83/84 Plus: (1-e^x)/x Excel: (1-exp(x))/x 1 From the graph we see that lim = 1, so the singularity is removable, and setting (0) = 1 will 0 make it continuous at 1. 42. We use a graph. (You could also use a table of values. If you use technology to show the graph you might see a vertical line at the asymptote.)

Technology Formula: TI-83/84 Plus: (1+e^x)/(1-e^x) Excel: (1+exp(x))/(1-exp(x))


Solutions Section 10.3 1 + From the graph we see that lim does not exist, the singularity is essential, and no value of (0) 0 1 will make it continuous at 0. 43. As is a point in the domain of but lim ( ) does not exist, is discontinuous at . (Choice (B)) 44. As is an isolated point not in the domain of , is singular at . (Choice (A))

45. As is defined only at , the limit at is defined to be ( ) (see the sidenote next to the definition of a continuous function), so the limit exists and equals ( ), making continuous at (Choice (C)). 46. As is defined everywhere except at , is an isolated point not in the domain of , so is singular at (Choice (A)).

47. Not unless the domain of the function consists of all real numbers. (It is impossible for a function to be continuous at points not in its domain.) For example, ( ) = 1 is continuous on its domain—the set of nonzero real numbers—but not at 0. 48. False. For example, ( ) = 1 is continuous on its domain, but its graph is not a single continuous curve.

49. True. If the graph of a function has a break in its graph at any point , then it cannot be continuous at the point . 50. True. If the graph is continuous, then the limit exists at every point and equals the value of the function.

51. Answers may vary. ( ) = 1 [( 1)( 2)( 3)] is such a function; it is undefined at = 1, 2, 3, so its graph consists of three distinct curves.

52. Answers may vary. ( ) = is such a function, since 1 is not in the domain of . (For a function to be discontinuous at , that point must be in the domain of .) 53. Answers may vary.


Solutions Section 10.3 54. Answers may vary.

55. Answers may vary. The price of OHaganBooks.com stock suddenly drops by $10 as news spreads of a government investigation. Let ( ) = Price of OHaganBooks.com stock. 56. Answers may vary. You turn on the light as you enter your room. Let ( ) = Light intensity in your room.


Solutions Section 10.4 Section 10.4 1.

2.

3.

4.

5.

6.

7.

8.

9.

(3) (1) 1 5 = = 3 3 1 2

(2) (0) 2 ( 1) = = 1.5 2 0 2

( 1) ( 1) 1.5 ( 2.1) = = 0.3 2 1 ( 3) (1) ( 1) 6.5 ( 0.5) = = 3.5 1 ( 1) 2

8(6) 8(2) 20.1 20.2 = = $25,000 per month 6 2 4 (3) (1) 4.00 2.20 = = £0.90 per kilo 3 1 2

9(5.5) 9(5) 300 400 = = 200 items per dollar 5.5 5 0.5

:(0.2) :(0.1) 6 3 = = 30 miles per hour 0.2 0.1 0.1 (5) (2) 27 23 $1.33 per month = 5 2 3

10.

(5) (1) 21 28 $1.75 per month = 5 1 4

(4) (0) 8 5 = = 0.75 percentage point increase in unemployment per 1 percentage point 4 0 4 increase in the deficit 11.

(4) (0) 2.5 2 = = 0.125 percentage point increase in inflation per 1 percentage point increase 4 0 4 in the deficit 12.

13.

14.

15.

(3) (1) 6 ( 2) = = 4 3 1 2

(2) ( 1) 12 6 = = 2 2 ( 1) 3 (0) ( 2) 4 0 = = 2 0 ( 2) 2


(4) (1) 1 4 1 1 16. = = 4 1 3 4 17.

18.

Solutions Section 10.4

(3) (2) 9 2 + 1 3 (2 + 1 2) 7 = = 3 2 1 3 (4) (3) 46 (27 3 2) 41 = = 4 3 1 2

( + 2) ( ) (2) (0) because = 0. = 2 2 (2) (0) (1) (0) 2 0 2= 1: = = = 2 2 1 1 (2) (0) (0.1) (0) 0.02 0 2 = 0.1 : = = = 0.2 2 0.1 0.1 Technology can be used to compute the remaining cases. All the values are shown in the following table: 19. Average rate of change =

2

Avg. Rate of Change

1

0.1

2

0.01 0.001 0.0001

0.2

0.02 0.002 0.0002

0.1

0.01

( + 2) ( ) (1 + 2) (1) because = 1. = 2 2 (1 + 2) (1) (2) (1) 2 0.5 2= 1: = = = 1.5 2 1 1 (1 + 2) (1) (1.1) (1) 0.605 0.5 2 = 0.1 : = = = 1.05 2 0.1 0.1 Technology can be used to compute the remaining cases. All the values are shown in the following table: 20. Average rate of change =

2

Avg. Rate of Change

1

1.5

0.001

0.0001

1.05 1.005 1.0005 1.00005

( + 2) ( ) (2 + 2) (2) because = 2. = 2 2 (2 + 2) (2) (3) (2) 1 3 1 2 2= 1: = = = 1 6 0.1667 2 1 1 (2 + 2) (2) (2.1) (2) 1 2.1 1 2 2 = 0.1 : 0.2381 = = 2 0.1 0.1 Technology can be used to compute the remaining cases. All the values are shown in the following table: 21. Average rate of change =

2

1

0.1

0.01

0.001

0.0001

Avg. Rate of Change 0.1667 0.2381 0.2488 0.2499 0.24999

( + 2) ( ) (1 + 2) (1) because = 1. = 2 2 (1 + 2) (1) (2) (1) 1 2 2= 1: = = = 1 2 1 1 (1 + 2) (1) (1.1) (1) 2 1.1 2 2 = 0.1 : 1.8182 = = 2 0.1 0.1 Technology can be used to compute the remaining cases. All the values are shown in the following table: 22. Average rate of change =


2 Avg. Rate of Change

1

1

0.1

Solutions Section 10.4 0.01

0.001

0.0001

1.8182 1.9802 1.998 1.9998

( + 2) ( ) (3 + 2) (3) because = 3. = 2 2 (3 + 2) (3) (4) (3) 24 15 2= 1: = = = 9 2 1 1 (3 + 2) (3) (3.1) (3) 15.81 15 2 = 0.1 : = = = 8.1 2 0.1 0.1 Technology can be used to compute the remaining cases. All the values are shown in the following table: 23. Average rate of change =

2

Avg. Rate of Change

1

9

0.1

0.01 0.001 0.0001

8.1

8.01 8.001 8.0001

0.1

0.01

( + 2) ( ) (2) (0) because = 0. = 2 2 (2) (0) (1) (0) 1 0 2= 1: = = = 1 2 1 1 (2) (0) (0.1) (0) 0.17 0 2 = 0.1 : = = = 1.7 2 0.1 0.1 Technology can be used to compute the remaining cases. All the values are shown in the following table:

24. Average rate of change =

2

Avg. Rate of Change

1

1

0.001

0.0001

1.7 1.97 1.997 1.9997

25. a. 2014–2016 corresponds to the interval [4, 6]. 1,760 1,730 30 Average change over [4, 6] = = = $15 billion per year. 6 4 2 World military expenditure increased at an average rate of $15 billion per year during 2014–2016. 2016–2018 corresponds to the interval [6, 8]. 1,900 1,760 140 Average change over [6, 8] = = = $70 billion per year. 8 6 2 World military expenditure increased at an average rate of $70 billion per year during 2016–2018. (8) (4) 1,900 1,730 170 b. Average rate of change over [4, 8] = = = = $42.5 billion per year. 8 4 8 4 4 15 + 70 85 The average of the two rates calculated in part (a) is = = $42.5 billion per year, the same as 2 2 the average rate over [4, 8]. 26. a. 2012–2016 corresponds to the interval [2, 6]. (6) (2) 1,760 1,775 15 Average rate of change over [2, 6] = = = = $3.75 billion per year. 6 2 16 12 4 World military expenditure decreased at an average rate of $3.75 billion per year during 2012–2016. 2016–2020 corresponds to the interval [6, 10]. (10) (6) 1,980 1,760 220 Average rate of change over [6, 10] = = = = $55 billion per year. 10 6 10 6 4 World military expenditure increased at an average rate of $55 billion per year during 2016–2020. (10) (2) 1,980 1,775 205 b. Average rate of change over [2, 10] = = = = $25.625 billion per 10 2 10 2 8 year.


Solutions Section 10.4 3.75 + 55 51.25 The average of the two rates calculated in part (a) is = = $25.625 billion per year, 2 2 equal to the average rate over [2, 10].

8.6 5.3 3.3 = = 1.65; During the two-week period starting 2 0 2 at week 0, the percentage of vaccinated people increased by an average of 1.65 percentage points per week. 10.7 6.7 4 Average rate of change over [1, 3] = = = 2; During the two-week period starting at week 1, 3 1 2 the percentage of vaccinated people increased by an average of 2 percentage points per week. 13.3 8.6 4.7 Average rate of change over [2, 4] = = = 2.35; During the two-week period starting at 4 2 2 week 2, the percentage of vaccinated people increased by an average of 2.35 percentage points per week. 16.2 10.7 5.5 Average rate of change over [3, 5] = = = 2.75; During the two-week period starting at 5 3 2 week 3, the percentage of vaccinated people increased by an average of 2.75 percentage points per week. b. The rates of change in part (a) are increasing in value (choice (A)). 27. a. Average rate of change over [0, 2] =

33.3 26.4 6.9 = = 3.45; During the two-week period 2 0 2 starting at week 0, the percentage of vaccinated people increased by an average of 3.45 percentage points per week. 36.3 29.9 6.4 Average rate of change over [1, 3] = = = 3.2; During the two-week period starting at 3 1 2 week 1, the percentage of vaccinated people increased by an average of 3.2 percentage points per week. 39 33.3 5.7 Average rate of change over [2, 4] = = = 2.85; During the two-week period starting at 4 2 2 week 2, the percentage of vaccinated people increased by an average of 2.85 percentage points per week. 41.3 36.3 5 Average rate of change over [3, 5] = = = 2.5; During the two-week period starting at week 5 3 2 3, the percentage of vaccinated people increased by an average of 2.5 percentage points per week. b. The rates of change in part (a) are decreasing in value (choice (B)). 28. a. Average rate of change over [0, 2] =

29. a. See the following graph:

;* 10 1.7. ; 6 Interpretation: The percentage of mortgages classified as subprime was increasing at an average rate of around 1.7 percentage points per year between 2000 and 2006. b. The rates of change of * ( ) over 2-year periods are given by the slopes of line segments joining points two units apart horizontally. The steepest of these corresponds to the interval [4, 6], corresponding to 2004–2006.

From the graph, rate of change over [0, 6] =


Solutions Section 10.4 30. a. See the following graph:

;< 700 = 175 ; 4 Interpretation: The subprime mortgage debt outstanding was increasing at an average rate of around $175 billion per year between 2002 and 2006. b. The rates of change of < ( ) over 2-year periods are given by the slopes of line segments joining points two units apart horizontally. The segment with the least slope corresponds to the interval [6, 8], corresponding to 2006–2008. From the graph, rate of change over [2, 6] =

31. a. See the following graph:

The 2-year rate of change with the greatest magnitude corresponds to the steepest of the three line segments * 8, = , 8> . The steepest is * 8 with an average rate of change of (53 42) 2 = 5.5. Interpretation: During 2010–2012, immigration to Ireland was increasing at an average rate of 5,500 people per year. b. The 2-year rate of change with the least magnitude corresponds to the least steep of the three line segments shown: = , with an average rate of change of (56 53) 2 = 1.5. Interpretation: During 2011–2013, immigration to Ireland was increasing at an average rate of 1,500 people per year.


Solutions Section 10.4 32. a. See the following graph:

The 2-year rate of change with the greatest magnitude corresponds to the steepest of the three line segments * 8, = , 8> . The steepest is * 8 with an average rate of change of (87 69) 2 = 9. Interpretation: During 2010–2012, emigration from Ireland was increasing at an average rate of 9,000 people per year. b. The 2-year rate of change with the least magnitude corresponds to the least steep of the three line segments shown: 8> , with an average rate of change of (82 87) 2 = 2.5. Interpretation: During 2012–2014, emigration from Ireland was decreasing at an average rate of 2,500 people per year.

33. a. The one-year average rates of change are equal to the successive annual changes ; of : Average rates of change: (1) (0) [0, 1]: Average rate of change = = (1) (0) = ; = 400 320 = 80 1 0 [1, 2]: Average rate of change = ; = 510 400 = 110 [2, 3]: Average rate of change = ; = 630 510 = 120 [3, 4]: Average rate of change = ; = 770 630 = 140 [4, 5]: Average rate of change = ; = 950 770 = 180 These differences are increasing (choice (A)). b. One-year percentage changes of : (1) (0) 400 320 [0, 1]: Percentage change = 0.25. = (0) 320 (2) (1) 510 400 [1, 2]: Percentage change = 0.28. = (1) 400 (3) (2) 630 510 [2, 3]: Percentage change = 0.24. = (2) 510 (4) (3) 770 630 [3, 4]: Percentage change = 0.22. = (3) 630 (5) (4) 950 770 [4, 5]: Percentage change = 0.23. = (4) 770 These values fluctuate between 0.22 and 0.28 (Choice (C)). c. Worldwide solar power capacity was increasing at an increasing rate during 2016–2021, rising between 22 and 28 percent each year. (2) (0) 270 200 = = 35 2 0 2 (4) (2) 370 270 [2, 4]: Average rate of change = = = 50 4 2 2 (6) (4) 478 370 [4, 6]: Average rate of change = = = 54 6 4 2 34. a. Average rates of change:

[0, 2]: Average rate of change =


Solutions Section 10.4 (8) (6) 590 478 [6, 8]: Average rate of change = = = 56 2 8 6 (10) (8) 710 590 [8, 10]: Average rate of change = = = 60 10 8 2 These differences are increasing (choice (A)). b. Two-year percentage changes of : (2) (0) 270 200 [0, 2]: Percentage change = 0.35. = (0) 200 (4) (2) 370 270 [2, 4]: Percentage change = 0.37. = (2) 270 (6) (4) 478 370 [4, 6]: Percentage change = 0.29. = (4) 370 (8) (6) 590 478 [6, 8]: Percentage change = 0.23. = (6) 478 (10) (8) 710 590 [8, 10]: Percentage change = 0.20. = (8) 590 These values fluctuate between 0.20 and 0.37 (Choice (C)). c. Worldwide production of wind power was increasing at an increasing rate during 2010–2020, rising between 20 and 37 percent each year.

35. a. The four-year period beginning in 2013 corresponds to the interval [3, 7] in . 1,090 1,054 = 9 teams per year. 4 b. The least steep slope of the graph over all possible four-year periods is from = 7 to = 11, representing the period 2017–2021 (the only four-year period with a decrease in the number of teams).

36. a. The four-year period beginning in 2017 corresponds to the interval [7, 11] in . 1,097 1,087 = 2.5 teams per year 4 b. The steepest slope of the graph over all possible four-year periods is from = 0 to = 4, representing the period 2010–2014.

37. a. (A): The average rate of change of U.S. funding over [2, 8] is the slope of the line passing joining the corresponding points on the graph. As this line is steeper downward than the regression line, its slope is more negative, and hence less than that of the regression line. b. (C): The line joining the points on the graph corresponding to [6, 10] is approximately parallel to the regression line. c. (B): The line joining the points on the graph corresponding to [4, 16] has a greater slope than the regression line. d. The average rate of change of per capita spending over [4, 10] is approximately 4.00 4.50 $0.083 per year. 10 4 We can estimate the slope of the regression line using the approximate coordinates of two points it passes through: (2, 5.25) and (18, 3.25). 3.25 5.25 Slope of regression line = $0.125 per year. 18 2 Thus the actual rate of change over [4, 10] was greater (less negative) than the slope of the regression line. 38. a. (C): The average rate of change of U.K. funding over [1, 14] is the slope of the line passing joining the corresponding points on the graph. This line is approximately parallel to the regression line. b. (A): The line joining the points on the graph corresponding to [4, 12] has negative slope, and hence less than the slope of the regression line. c. (B): The line joining the points on the graph corresponding to [3, 10] is steeper upward than the


Solutions Section 10.4 regression line. d. The average rate of change of per capita spending over [10, 14] is approximately 18.5 22.0 = $0.875 per year. 14 10 We can estimate the slope of the regression line using the approximate coordinates of two points it passes through: (2, 19.5) and (14, 20). 20 19.5 Slope of regression line $0.042 per year. 14 2 Thus the actual rate of change over [10, 14] was less than the slope of the regression line. 39. From 1991 to 1995 the volatility decreased at an average rate of 0.2 points per year, so decreased a total of 4 × 0.2 = 0.8 points. Since its value in 1995 was 1.1, its value in 1991 must have been 1.1 + 0.8 = 1.9. Similarly, from 1995 to 1999 the volatility increased a total of 4 × 0.3 = 1.2 to end at 1.1 + 1.2 = 2.3. In between these points we've found almost anything could happen, but the graph might look something like the following:

40. From 1992 to 1995 the volatility had an average rate of change of 0, so the value in 1992 must have been 1.1, the same as the value in 1995. From 1995 to 1998 the volatility increased by 3 × 0.2 = 0.6 points, so in 1998 the volatility was 1.1 + 0.6 = 1.7. In between these points we've found almost anything could happen, but the graph might look something like the following:

(2) (0) 1, 600 1, 000 = = 300. 2 0 2 The index was increasing at an average rate of 300 points per day. 41.

(5) (2) 1, 000 1, 600 = = 200. 5 2 3 The index was increasing at an average rate of 300 points per day.

42.

43. a. The 9-year period beginning at the start of 2012 corresponds to the period [0, 9]. Average rate of change of * is * (9) * (0) 58.5 108 = = $5.50 per year. 9 0 9 The price per barrel of crude oil was decreasing at an average rate of about $5.50 per year over the 9-year


Solutions Section 10.4 period beginning at the start of 2012. b. No. The graph of the given quadratic model is a parabola whose lowest point is at =

? 19 6.33 = 2 3

So, according to the model, during the last 2.67 years ( = 6.33 to = 9) of the 9-year period, the price of oil went up from around $48 to $58.50. In general, the average rate of change of a function over [ , ?] is not affected by the values of the function between and ?. c. Calculations of the rates of change over the intervals [5, 5 + 2] for 2 = 1, 0.1, 0.01, 0.001, and 0.0001: * (5 + 2) * (5) * (6) * (5) 48 50.5 2= 1: = = = 2.5 thousand $/year 2 1 1 * (5 + 2) * (5) * (5.1) * (5) 50.115 50.5 2 = 0.1 : = = = 3.85 thousand $/year 2 0.1 0.1 * (5 + 2) * (5) * (5.01) * (5) 50.46015 50.5 2 = 0.01 : = = = 3.985 thousand $/year 2 0.01 0.01 * (5 + 2) * (5) * (5.001) * (5) 50.4960015 50.5 2 = 0.001 : = = = 3.9985 thousand $/year 2 0.001 0.001 * (5 + 2) * (5) * (5.0001) * (5) 50.499600015 50.5 2 = 0.0001 : = = = 3.99985 thousand $/year 2 0.0001 0.0001 At the very beginning of 2015, the price of oil was decreasing at a rate of about $4.00 per year. 44. a. The 5-year period beginning at the start of 2010 corresponds to the interval [0, 5]. Average rate of change of * is * (5) * (0) 227 209 = = $3.6 thousand per year, or $3,600 per year. 5 0 5 The median home price was increasing at an average rate of about $3,600 per year over the 5-year period beginning at the start of 2010. b. Nothing. In general, the average rate of change of a function over [ , ?] is not affected by specific values of the function between and ?. c. Calculations of the rates of change over the intervals [10, 10 + 2] for 2 = 1, 0.1, 0.01, 0.001, and 0.0001: * (10 + 2) * (10) * (11) * (10) 302.06 285.5 2= 1: = = = 16.56 thousand $/year 2 1 1 * (10 + 2) * (10) * (10.1) * (10) 287.0831 285.5 2 = 0.1 : = = = 15.831 thousand $/year 2 0.1 0.1 * (10 + 2) * (10) * (10.01) * (10) 285.657581 285.5 2 = 0.01 : = = = 15.7581 thousand $/year 2 0.01 0.01 * (10 + 2) * (10) * (10.001) * (10) 285.51575081 285.5 2 = 0.001 : = = = 15.75081 thousand 2 0.001 0.001 $/year * (10 + 2) * (10) * (10.0001) * (10) 285.5015750081 285.5 2 = 0.0001 : = = = 15.750081 2 0.0001 0.0001 thousand $/year Home prices were increasing at a rate of about $ 15,750 per year at the very beginning of 2020. 45. a. = 0.037 2 + 0.02 + 0.4 (1) (0) Rate of change over [0, 1] = = 0.457 0.4 = 0.057 1 (2) (1) Rate of change over [1, 2] = = 0.588 0.457 = 0.131 1 (3) (2) Rate of change over [2, 3] = = 0.793 0.588 = 0.205 1


Solutions Section 10.4 (4) (3) Rate of change over [3, 4] = = 1.072 0.793 = 0.279 1 ! !1 0.131 0.057 b. Slope = 2 = = 0.074. The rate of change of the Sun's radius will be increasing by 2 1 1 0.074 AU/million years per million years 46. a. = 0.037 2 + 0.02 + 0.4 (2) (0) 0.588 0.4 Rate of change over [0, 2] = = = 0.094 2 2 (3) (1) 0.793 0.457 Rate of change over [1, 3] = = = 0.168 2 2 (4) (2) 1.072 0.588 Rate of change over [2, 4] = = = 0.242 2 2 ! !1 0.168 0.094 b. Slope = 2 = = 0.074. The rate of change of the Sun's radius will be increasing by 2 1 1 0.074 AU/million years per million years 47. a. The period March 17 to March 23 corresponds to [0, 6]. Average rate of change of @ is @(6) @(0) 450.8 167 = 47.3 new cases per day. 6 0 6 Interpretation: The number of SARS cases was growing at an average rate of 47.3 new cases per day over the period March 17 to March 23. b. The graph of @( ) = 167(1.18) is increasingly steep as increases. Therefore, the average rates of change increase with , so the number of reported cases was increasing at a faster and faster rate (choice (A)). 48. a. The period April 19 to April 29 corresponds to [18, 28]. Average rate of change of @ is @(28) @(18) 5, 409.7 3, 654.6 175.5 new cases per day. 28 18 10 Interpretation: The number of SARS cases was growing at an average rate of 175.5 new cases per day over the period April 19 to April 29. b. The graph of @( ) = 1804(1.04) is increasingly steep as increases. Therefore, the average rates of change increase with t, so the number of reported cases was increasing at a faster and faster rate (choice (A)). 49. a. ( ) = 95.9 0.72 The approximate values of ( ) are given in the following table: 0

1

2

3

4

5

6

( ) 95.900 197.020 404.765 831.562 1,708.389 3,509.771 7,210.590 Average rates of change: (2) (0) 404.765 95.9 [0, 2] : 154.43 cases per month 2 2 (3) (1) 831.562 197.02 [1, 3] : 317.27 cases per month 2 2 (4) (2) 1,708.389 404.765 [2, 4] : 651.81 cases per month 2 2 (5) (3) 3,509.771 831.562 [3, 5] : 1,339.1 cases per month 2 2 (6) (4) 7,210.589 1,708.389 [4, 6] : 2,751.1 cases per month 2 2 b. Each rate of change in part (a) is approximately 2.05 times the preceding rate, so the average rates


Solutions Section 10.4 follow an exponential model. Interpretation: The 2-month average rate of increase in the number of Ebola cases increased by a factor of about 2.05 each month. (This is the same ratio as that of successive values of .) 50. a. :( ) = 90.52 0.60 The approximate values of :( ) are given in the following table: 0

1

2

3

4

5

6

:( ) 90.520 164.938 300.537 547.614 997.818 1,818.143 3,312.872 Average rates of change: :(2) :(0) 300.537 90.52 [0, 2] : 105.01 deaths per month 2 2 :(3) :(1) 547.614 164.938 [1, 3] : 191.34 deaths per month 2 2 :(4) :(2) 997.818 300.537 [2, 4] : 348.64 deaths per month 2 2 :(5) :(3) 1,818.143 547.614 [3, 5] : 635.26 deaths per month 2 2 :(6) :(4) 3,312.872 997.818 [4, 6] : 1,157.53 deaths per month 2 2 b. Each rate of change in part (a) is approximately 1.82 times the preceding rate, so the average rates follow an exponential model. Interpretation: The 2-month average rate of increase in the number of Ebola deaths increased by a factor of about 1.82 each month. (This is the same ratio as that of successive values of :.) 51. a. Average rates of change: (2) (0) 93,780 81,000 [0, 2] : 6,400 cases per day 2 2 (4) (2) 108,576 93,780 [2, 4] : 7,400 cases per day 2 2 (6) (4) 125,707 108,576 [4, 6] : 8,600 cases per day 2 2 b. Percentage changes: (2) (0) 93,780 81,000 [0, 2] : 0.158 (0) 81,000 (4) (2) 108,576 93,780 [2, 4] : 0.158 (2) 93,780 (6) (4) 125,707 108,576 [4, 6] : 0.158 (4) 108,576 The percentage changes are all the same; the number of new cases was growing by around 15.8% every two days. 52. a. Average rates of change: (2) (0) 4,360 4,000 [0, 2] : 180 cases per day 2 2 (4) (2) 4,752 4,360 [2, 4] : 196 cases per day 2 2 (6) (4) 5,179 4,752 [4, 6] : 214 cases per day 2 2 b. Percentage changes: (2) (0) 4,360 4,000 [0, 2] : 0.090 (0) 4,000


Solutions Section 10.4 (4) (2) 4,752 4,360 [2, 4] : 0.090 4,360 (2) (6) (4) 5,179 4,752 [4, 6] : 0.090 (4) 4,752 The percentage changes are all the same; the number of new cases was growing by around 9.0% every two days.

(6) (5) 27.6 18.75 = = 8.85 manatee deaths per 100,000 boats; 6 5 1 (8) (7) 66.6 43.55 = = 23.05 manatee deaths per 100,000 boats 8 7 1 b. More boats result in more manatee deaths per additional boat. 53. a.

(7) (5) 43.55 18.75 = = 12.4 manatee deaths per 100,000 boats; 7 5 2 (8) (6) 66.6 27.6 = = 19.5 manatee deaths per 100,000 boats 8 6 2 b. They would be equal because the average rate of change of a linear function equals the slope no matter what interval we use. 54. a.

55. Here is a worksheet that computes the successive rates of change:

This worksheet leads to the following values, showing the desired rates of change in the rightmost column:

The answers are obtained by rounding those entries to two decimal places. b. From the table, the average rate of change of over [15, 20] is 0.96. The units of measurement are units of per unit of : thousands of dollars per additional year of professional programming experience. Thus, for programmers in the U.S. with between 15 and 20 years of professional programming experience, their median salary grows at an average rate of about $960 per additional year of professional programming experience. c. All the rates of change obtained in part (a) are positive, but with successively smaller values, showing that the salaries increase, but at a slower and slower rate (choice (C)). 56. Here is a worksheet that computes the successive rates of change:


Solutions Section 10.4

This worksheet leads to the following values, showing the desired rates of change in the rightmost column:

The answers are obtained by rounding those entries to two decimal places. b. From the table, the average rate of change of A over [15, 20] is 1.15. The units of measurement are units of A per unit of : thousands of dollars per additional year of professional programming experience. Thus, for programmers in Germany with between 15 and 20 years of professional programming experience, their median salary grows at an average rate of about $1,150 per additional year of professional programming experience. c. All the rates of change obtained in part (a) are positive, but with successively smaller values, showing that the salaries increase, but at a slower and slower rate (choice (C)). 57. The average rate of change of over an interval [ , ?] can be determined numerically, using a table of values; graphically, by measuring the slope of the corresponding line segment through two points on the graph; or algebraically, using an algebraic formula for the function. Of these, the least precise is the graphical method, because it relies on reading coordinates of points on a graph. 58. The average rate of change is given by the slope: units of ! per unit of , regardless of the interval.

59. No, the formula for the average rate of a function over [ , ?] depends only on ( ) and (?), and not on any values of between and ?. 60. No. All it means is that ( ) and (?) are equal, and says nothing about values of between 61. Answers may vary. Here is one possibility:

62. Answers may vary. Here is one possibility:

and ?.


Solutions Section 10.4

63. For every change of 1 in C, B changes by 3, so A changes by 2 × 3 = 6 units of quantity A per unit of quantity C. 64. For every change of 1 in B, A changes by 2. For A to change by 1 B must change by only 1/2, so the rate of change of B with respect to A is 1/2 unit of quantity B per unit of quantity A. 65. (A): The secant line given by = 1 and = 1 + 2 is steeper for smaller values of 2.

66. (B): The secant line given by = 1 and = 1 + 2 has lesser slope (is more steeply falling) for smaller values of 2. 67. Yes. Here is an example, in which the average rate of growth for 2000–2003 is negative, but the average rates of growth for 2000–2001 and 2001–2002 are positive: Year 2000 2001 2002 2003 Revenue ($ billion)

10

20

30

5

68. No. The slope of * = cannot exceed both the slopes * 8 and 8=, as illustrated in the following graphs. In the first, the slope of * = is greater than that of 8= but must be less than that of * 8. The situation is reversed in the second graph.

69. (A): This can be checked by algebra: (2) (1) (3) (2) + � � (3) (1) 2 1 3 2 . = 2 3 1 70. (A): Again, this can be checked by algebra: (2) (1) (3) (2) (4) (2) + + � � (4) (1) 2 1 3 2 4 2 . = 3 4 1 71. In an exponential model the successive ratios of values over periods of the same length are all equal, and the percentage change over the period [ , ?] can be written as


Solutions Section 10.4 (?) ( ) (?) ( ) (?) = = 1 ( ) ( ) ( ) ( )

(?) (?) are constant over all periods [ , ?] of the same length, so are the quantities 1, ( ) ( ) which are equal to the percentage rates of change. As the ratios

72. If has the property that the average rate of change of over all intervals equals the same constant , then, if is a fixed real number and is any other number, then ( ) ( ) = ,

so that

( ) ( ) = ( ) ( ) = ( ) + ( ) = + ( ( ) ).

which has the form of a linear function + ?. Now this works under the assumption that 1 . However, if = , then the last formula above holds as well (it just says that ( ) = ( )), so it holds for every , meaning that the function is linear.)


Solutions Section 10.5 Section 10.5

1. 6: The average rates of change are approaching 6 for both positive and negative values of 2 approaching 0. 2. 5: The average rates of change are approaching 5 for both positive and negative values of 2 approaching 0. 3. 5.5 : The average rates of change are approaching 5.5 for both positive and negative values of 2 approaching 0. 4. 0.6: The average rates of change are approaching 0.6 for both positive and negative values of 2 approaching 0.

8( + 2) 8( ) . Here, = 5. 2 8(5 + 1) 8(5) 8(6) 8(5) 39 2= 1: = = = 39 1 1 1 8(5 + 0.1) 8(5) 8(5.1) 8(5) 3.99 2 = 0.1 : = = = 39.9 0.1 0.1 0.1 8(5 + 0.01) 8(5) 8(5.01) 8(5) 0.3999 2 = 0.01 : = = = 39.99 0.01 0.01 0.01 Table: 5. The average rate of change is

2

Avg. rate

1

0.1

0.01

39

39.9 39.99

The average rates are approaching an instantaneous rate of $40 per day. 8( + 2) 8( ) . Here, = 3. 2 8(3 + 1) 8(3) 8(4) 8(3) 46 2= 1: = = = 46 1 1 1 8(3 + 0.1) 8(3) 8(3.1) 8(3) 4.78 2 = 0.1 : = = = 47.8 0.1 0.1 0.1 8(3 + 0.01) 8(3) 8(3.01) 8(3) 0.4798 2 = 0.01 : = = = 47.98 0.01 0.01 0.01 Table: 6. The average rate of change is

2

Avg. rate

1

0.1

0.01

46

47.8 47.98

The average rates are approaching an instantaneous rate of $48 per day. 8( + 2) 8( ) . Here, = 1. 2 8(1 + 1) 8(1) 8(2) 8(1) 140 2= 1: = = = 140 1 1 1 8(1 + 0.1) 8(1) 8(1.1) 8(1) 6.62 2 = 0.1 : = = = 66.2 0.1 0.1 0.1 8(1 + 0.01) 8(1) 8(1.01) 8(1) 0.60602 2 = 0.01 : = = = 60.602 0.01 0.01 0.01 7. The average rate of change is


Solutions Section 10.5 Table:

2

1

0.1

Avg. rate 140

0.01

66.2 60.602

The average rates are approaching an instantaneous rate of $60 per day.

8( + 2) 8( ) . Here, = 2. 2 8(2 + 1) 8(2) 8(3) 8(2) 31 2= 1: = = = 31 1 1 1 8(2 + 0.1) 8(2) 8(2.1) 8(2) 3.739 2 = 0.1 : = = = 37.39 0.1 0.1 0.1 8(2 + 0.01) 8(2) 8(2.01) 8(2) 0.379399 2 = 0.01 : = = = 37.9399 0.01 0.01 0.01 Table: 8. The average rate of change is

2

Avg. rate

1

0.1

0.01

31

37.39 37.9399

The average rates are approaching an instantaneous rate of $38 per day. 9. The average cost to manufacture 2 more items is the average rate of change:

= 1,000. (1,000 + 10) (1,000) (1,010) (1,000) 47.99 2 = 10 : = = = 4.799 10 10 10 (1,000 + 1) (1,000) (1,001) (1,000) 4.7999 2= 1: = = = 4.7999 1 1 1 (1,000) = $4.80 per item Table: 2

10

( + 2) ( ) . Here, 2

1

avg 4.799 4.7999 10. The average cost to manufacture 2 more items is the average rate of change:

( + 2) ( ) . Here, 2

= 10,000. (10,000 + 10) (10,000) (10,010) (10,000) 59.995 2 = 10 : = = = 5.9995 10 10 10 (10,000 + 1) (10,000) (10,001) (10,000) 5.99995 2= 1: = = = 5.99995 1 1 1 (10,000) = $6 per item Table: 2

10

1

avg 5.9995 5.99995 11. The average cost to manufacture 2 more items is the average rate of change: = 100.

( + 2) ( ) . Here, 2


Solutions Section 10.5 (100 + 10) (100) (110) (100) 999.0909091 2 = 10 : 99.91 = = 10 10 10 (100 + 1) (100) (101) (100) 99.9009901 2= 1: 99.90 = = 1 1 1 (100) = $99.90 per item Table: 2

10

1

avg 99.91 99.90 12. The average cost to manufacture 2 more items is the average rate of change: = 100.

(100 + 10) (100) (110) (100) 490.9090909 49.09 = = 10 10 10 (100 + 1) (100) (101) (100) 49.009901 2= 1: 49.01 = = 1 1 1 (100) = $49 per item Table: 2 = 10 :

2

10

( + 2) ( ) . Here, 2

1

avg 49.09 49.01

13. The derivative is the slope of the tangent line shown. The tangent line passes through (0, 2) and (6, 5). Therefore its slope is (5 2) (6 0) = 1 2. 14. The derivative is the slope of the tangent line shown. The tangent line passes through (0, 0) and (3, 6). Therefore its slope is 2. 15. The derivative is the slope of the tangent line shown. The tangent line is horizontal. Therefore its slope is 0. 16. The derivative is the slope of the tangent line shown. The tangent line passes through (0, 4) and (6, 1). Therefore its slope is 1 2. 17. a. The derivative is greatest at the point at which the graph is rising with the steepest slope: 8. b. The derivative is least at the point at which the graph is rising with the shallowest slope: * .

18. a. The derivative is greatest at the point at which the graph is falling with the shallowest slope: * . b. The derivative is least at the point at which the graph is falling with the steepest slope: 8. 19. a. The derivative is greatest at the point at which the graph is rising with the steepest slope: * . b. The derivative is least at the point at which the graph is falling with the steepest slope: 8.

20. a. The derivative is greatest at the point at which the graph is rising with the steepest slope: =. b. The derivative is least at the point at which the graph is falling with the steepest slope: 8. 21. a. The derivative is greatest at the point at which the graph is rising with the steepest slope: =. b. The derivative is least at the point at which the graph is rising with the shallowest slope: * .


Solutions Section 10.5 22. a. The derivatives (slopes) at = and 8 are negative, so the derivative is greatest at * , where the slope is zero. b. The derivative is least at the point at which the graph is falling with the steepest slope: =. 23. Estimate the slope of the tangent line at each of the three points. a. = b. 8 c. * 24. Estimate the slope of the tangent line at each of the three points. a. 8 b. = c. * 25. Estimate the slope of the tangent line at each of the three points. a. 8 b. = c. * 26. Estimate the slope of the tangent line at each of the three points. a. * b. = c. 8 27. a. The only point where the tangent line has slope 0 is (1, 0). b. None; the graph never rises. c. The only point where the tangent line has slope 1 is ( 2, 1). 28. a. The only point where the tangent line has slope 0 is (1, 1). b. The only point where the tangent line has slope 1 is ( 2, 0). c. None; the graph never falls.

29. a. The points where the tangent line has slope 0 are ( 2, 0.3), (0, 0), and (2, 0.3). b. None; the graph never rises that steeply. c. None; the graph never falls that steeply. 30. a. The points where the tangent line has slope 0 are (0, 1) and (2, 0.5). b. The points where the tangent line has slope 1 are ( 1, 0) and (3, 0). c. The only point where the tangent line has slope 1 is (1, 0). 31. ( , ( )); ( )

32. Secant; the points ( , ( )) and ( + 2, ( + 2)) 33. (B): The derivative is the slope of the tangent line. It is not any particular average rate of change or difference quotient; these only approximate the derivative. 34. (A): Again, the derivative at each point is the slope of the tangent line, not the approximate slope or any average rate of change or difference quotient.

35. a. (A): The graph rises above the tangent line at = 2. b. (C): The secant line is roughly parallel to the tangent line at = 0. c. (B): The slopes of the tangent lines are decreasing. d. (B): The slopes of the tangent lines decrease to 0 then increase again. e. (C): The height of the graph is approximately 0.7 while the slope of the tangent line at = 4 is approximately 1.

36. a. (B): The graph starts above the tangent line at = 100 and ends below it; hence its average rate of change is less than the slope of the tangent line.


Solutions Section 10.5 b. (C): The secant line from the beginning to the endpoint is roughly parallel to the tangent line at = 150. c. (C): The slopes of the tangent lines are increasing toward 0. d. (B): The graph is always falling. e. (A): The graph is falling more steeply at = 25 than at = 100. 37.

(2 + 0.0001) (2 0.0001) = 2 0.0002

38.

( 3 + 0.0001) ( 3 0.0001) = 1 3 0.0002

39.

( 1 + 0.0001) ( 1 0.0001) 1.5 0.0002

40.

(2 + 0.0001) (2 0.0001) = 2.25 0.0002

41. We use the balanced difference quotient: 7(1 + 0.0001) 7(1 0.0001) 5. 0.0002

42. We use the balanced difference quotient: B( 2 + 0.0001) B( 2 0.0001) 0.1875. 0.0002

43. We use the balanced difference quotient: !(2 + 0.0001) !(2 0.0001) = 16. 0.0002

44. We use the balanced difference quotient: !( 1 + 0.0001) !( 1 0.0001) = 2.\qquad 0.0002

45. We use the balanced difference quotient: B( 2 + 0.0001) B( 2 0.0001) = 0. 0.0002

46. We use the balanced difference quotient: B(2 + 0.0001) B(2 0.0001) = 3.\qquad 0.0002

47. We use the balanced difference quotient: 8(20 + 0.0001) 8(20 0.0001) 0.0025. 0.0002

48. We use the balanced difference quotient: 8(400 + 0.0001) 8(400 0.0001) 0.025. 0.0002

( 1 + 0.0001) ( 1 0.0001) 3 0.0002 b. The equation of the line through ( 1, 1) with slope 3 is ! = 3 + 2. 49. a.

(0 + 0.0001) (0 0.0001) = 0 0.0002 b. The equation of the line through (0, 0) with slope 0 is ! = 0. 50. a.


Solutions Section 10.5

51. a.

(2 + 0.0001) (2 0.0001) 3 0.0002 4

b. The equation of the line through (2, 2.5) with slope 3 4 is ! =

3 + 1. 4

(1 + 0.0001) (1 0.0001) 2 0.0002 b. The equation of the line through (1, 1) with slope 2 is ! = 2 + 3. 52. a.

53. a.

(4 + 0.0001) (4 0.0001) 1 0.0002 4

b. The equation of the line through (4, 2) with slope 1/4 is ! =

+ 1. 4

( 1 + 0.0001) ( 1 0.0001) 2 (Note that the slope of the tangent is the slope of , which is 0.0002 a line.) b. The equation of the line through ( 1, 2) with slope 2 is ! = 2 + 4. (Note that the tangent line is the given line itself.) 54. a.


Solutions Section 10.5

55.

56.

57.

58.

0.0001 1.000 0.0002

0.0001

2 1+ 0.0001 2 1 0.0001 5.437 0.0002

ln(1 + 0.0001) ln(1 0.0001) 1.000 0.0002

ln(2 + 0.0001) ln(2 0.0001) 0.500 0.0002

59. (C): The graph of is a falling straight line, so must have the same negative slope (derivative) at every point; must be a negative constant. 60. (E): The graph of is a rising straight line, so must have the same positive slope (derivative) at every point; must be a positive constant.

61. (A): The function decreases until = 0, where it turns around and starts to increase. Its derivative must be negative until = 0, where the derivative is 0; past that the derivative becomes positive. This is exactly what (A) illustrates. 62. (D): The function increases until = 0, where it turns around and starts to decrease. Its derivative must be positive until = 0, where the derivative is 0; past that the derivative becomes negative. This is exactly what (D) illustrates.

63. (F): The function increases slowly at first, it becomes steeper around = 0, then it returns to slowly rising. Its derivative starts as a small positive number, increases to become largest around = 0, then decreases back toward 0. This is the behavior seen in (F).

64. (B). The function decreases slowly at first, it becomes steeper around = 0, then it returns to slowly falling. Its derivative starts as a negative number near 0, becomes more negative until it is most negative around = 0, then returns toward 0. This is the behavior seen in (B). 65. Increasing (sloping up) for < 0; decreasing (sloping down) for > 0 66. Increasing (sloping up) for < 1; decreasing (sloping down) for > 1

67. Increasing (sloping up) for < 1 and > 1; decreasing (sloping down) for 1 < < 1

68. Increasing (sloping up) for 1 < < 0 and 1 < < 2; decreasing (sloping down) for 2 < < 1 and0 < < 1


Solutions Section 10.5 69. Increasing (derivative positive) for > 1; decreasing (derivative negative) for < 1 70. Increasing (derivative positive) for < 1; decreasing (derivative negative) for > 1 71. Increasing (derivative positive) for < 0; decreasing (derivative negative) for > 0

72. Increasing (derivative positive) for 1 < < 0 and > 1; decreasing (derivative negative) for < 1 and 0 < < 1 73. Below is the graph of the derivative. The tangent line is horizontal where the derivative is 0 (crosses the -axis): = 1.5, = 0.

74. Below is the graph of the derivative. The tangent line is horizontal where the derivative is 0 (crosses the -axis): = 2, = 0.


Solutions Section 10.5 75. Setup:

Values:

Graph: The top curve is ! = ( ); the bottom curve is ! = ( ).

76. Setup:

Values:


Solutions Section 10.5

Graph: The top curve is ! = ( ); the bottom curve is ! = ( ).

77. ( ) = 5 2 + 50 80; 7:00 am corresponds to = 1 (one hour after 6:00 am). Temeperature at 7:00 am = (1) = 5 + 50 80 = −35°F. The tables below show the average rates of change of over [1, 1 + 2] for 2 = 1, 0.1, 0.01, 0.001, and 0.0001 and for 2 = 1, 0.1, 0.01, 0.001, and 0.0001. Average rate of change = (1 + 2) (1) 5(1 + 2) 2 + 50(1 + 2) 80 ( 35) 5(1 + 2) 2 + 50(1 + 2) 45 : = = 2 2 2 2

Avg. Rate of change

2

Avg. Rate of change

1

35

1 45

0.1

0.01

0.001

0.0001

39.5 39.95 39.995 39.9995

0.1 0.01 0.001 0.0001 40.5 40.05 40.005 40.0005

The tables suggest that (1) = 40 : The temperature was rising at a rate of 40°F per hour.

78. ( ) = 880 + 2 2 20 ; 8:00 pm corresponds to = 3 (three hours after 5:00 pm). Temeperature at 8:00 pm = (3) = 880 + 2(3) 2 20(3) = 838°F. The tables below show the average rates of change of over [3, 3 + 2] for 2 = 1, 0.1, 0.01, 0.001, and 0.0001 and for 2 = 1, 0.1, 0.01, 0.001, and 0.0001. (3 + 2) (3) 880 + 2(3 + 2) 2 20(3 + 2) 838 Average rate of change = : = 2 2 2

Avg. Rate of change

1

6

0.1

0.01

0.001

0.0001

7.8 7.98 7.998 7.9998


2

1

Solutions Section 10.5

0.1 0.01 0.001 0.0001

Avg. Rate of change 10 8.2 8.02 8.002 8.0002

The tables suggest that (3) = 8 : The temperature was dropping at a rate of 8°F per hour.

79. 9(100) = 50,000, 9 (100) = 500 (use one of the quick approximations). A total of 50,000 pairs of sneakers can be sold at a price of $100, but the demand is decreasing at a rate of 500 pairs per $1 increase in the price. 80. 9(190) = 500, 9 (190) = 2.5 (use one of the quick approximations). A total of 500 TV sets can be sold at a price of $190, but the demand is decreasing at a rate of 2.5 sets per $1 increase in the price.

81. a. The tangent line passes through the two points (0, 1,200) and (7, 500). Therefore its slope is ;! 500 1,200 = = 100. ; 7 0 Let ! = ( ) be the equation of the original curve, so that (3) = 900 (value of ) from the graph, and (3) = 100 (slope of tangent). We conclude that, in 2013, daily oil imports from Mexico were 900,000 barrels and declining at a rate of 100,000 barrels per year. b. Increasing throughout, as the slope is increasing. (Even through the function first decreases then increases, the slope is increasing.)

82. a. The tangent line passes through the two points (2, 2.6) and (10, 1.8). Therefore its slope is ;! 1.8 2.6 = = 0.1. ; 10 2 Let ! = ( ) be the equation of the original curve, so that (5) = 2.3 (value of ) from the graph, and (5) = 0.1 (slope of tangent). We conclude that, in 2015, daily oil production by Pemex was 2.3 million barrels and decreasing at a rate of 0.1 million barrels (or 100,000 barrels) per year. b. Decreasing, as the slope decreases throughout. 83. a. (B): The graph is getting less steep. b. (B): The graph goes from above the tangent line to below it, so the slope of the tangent line is greater than the average rate of change. c. (A): From 0 to about 4 the graph is getting steeper, so the instantaneous rate of change is increasing; from that point on the graph is getting less steep, so the instantaneous rate of change is decreasing. d. 2004: This is the point ( = 4) where the graph is steepest. e. Reading values from the graph, we get the approximation (5.5) (2.5) 1.55 1.45 0.1 Balanced approximation = 0.033. = 3 3 3 In 2004 the U.S. prison population was increasing at a rate of 0.033 million prisoners (33,000 prisoners) per year. 84. a. (A): The rate of change is negative but approaching 0 as the graph gets less steep, so is increasing. b. (A): The graph goes from below the tangent line to above it, hence the slope of the tangent line is less than the average rate of change. c. (B): From 0 to arond 12.5 the graph is getting steeper, so the instantaneous rate of change is decreasing (remember that the rate of change is negative); from that point on the graph is getting less steep, so the instantaneous rate of change is increasing. d. Days 10–15, corresponding to March 11–March 16: This is the 5-day interval during which the graph is steepest. e. Reading values from the graph, we get the approximation (41 42) 10 = 0.1: On day 30 (March 31) the number of county jail prisoners was decreasing at a rate of approximately 100 prisoners per day.


Solutions Section 10.5 B(4) B(2) 85. a. = 96 ft/sec 4 2 B(4 + 0.0001) B(4 0.0001) b. = 128 ft/sec 0.0002 B(3) B(1) = 184 ft/sec 3 1 B(3 + 0.0001) B(3 0.0001) b. = 216 ft/sec 0.0002 86. a.

87. a. Average rate of change of * is * (9) * (5) 58.5 50.5 = = $2 per year. 9 5 4 The price per barrel of crude oil was increasing at an average rate of about $2 per year over the 4-year period beginning at the start of 2015. b. Instantaneous rate of change of * ( ) at = 5 is (using the quick approximation) * (5 + 0.0001) * (5 0.0001) 50.4996000 50.5004000 = 4. 0.0002 0.0002 The price per barrel of crude oil was decreasing at an instantaneous rate of about $4.00 per year at the start of 2015. c. The price of oil was decreasing in January 2015 but then began to increase (making the average rate of change in part (a) positive). 88. a. Average rate of change of * is * (10) * (5) 285.5 227 = = $11.7 thousand per year. 10 5 4 The median home price was increasing at an average rate of about $11,700 per year over the 5-year period beginning at the start of 2015. b. Instantaneous rate of change of * ( ) at = 5 is (using the quick approximation) * (5 + 0.0001) * (5 0.0001) 227.0007650 226.9992350 = 7.65 0.0002 0.0002 The median home price was increasing at an instantaneous rate of about $7,650 per year at the start of 2015. c. The median home price was increasing at a faster and faster rate during the five-year period starting at the start of 2015, making the average rate of change greater than the instantaneous rate at the start of that period.

89. a. Instantaneous rate of change of ( ) at = 3 is (using the quick approximation) (3 + 0.0001) (3 + 0.0001) 100,907.8642 100,906.3859 7,390 new cases/day 0.0002 0.0002 Interpretation: The number of COVID-19 cases was growing at a rate of about 7,390 new cases per day on April 4. b. As we know from the general form of the graph of an exponential function, the slope of the tangent to the graph of ( ) is increasingly steep as increases. Therefore, the instantaneous rates of change increase with (choice (A)). 90. a. Instantaneous rate of change of @( ) at = 10 is (using the quick approximation) (4 + 0.0001) (4 + 0.0001) 4,751.8624 4,751.8215 205 new cases/day 0.0002 0.0002 Interpretation: The number of COVID-19 cases was growing at a rate of about 205 new cases per day on June 24. b. As we know from the general form of the graph of an exponential function, the slope of the tangent to the graph of ( ) is increasingly steep as increases. Therefore, the instantaneous rates of change increase


with (choice (A)).

Solutions Section 10.5

91. (5) = 95.9 0.72(5) (5 + 0.0001) (5 0.0001) 2,527 = 5 0.0002 Five months after the outbreak, the number of cases was around 3,510 and increasing at a rate of about 2,527 per month.

92. (5) = 90.52 0.60(5) 1,818 (5 + 0.0001) (5 0.0001) 1,091 = 5 0.0002 Five months after the outbreak, the number of deaths was around 1,818 and increasing at a rate of about 1,091 per month.

93. (0) = 4.5 million because gives the number of subscribers; (0) = 60, 000 subscribers per week because gives the rate at which the number of subscribers is changing.

94. (0) = 1.6 million because gives the number of subscribers; (0) is negative because gives the rate of change of the number of subscribers and the number is decreasing. 95. a. 60% of children can speak at the age of 10 months. At the age of 10 months this percentage is increasing by 18.2 percentage points per month. b. As increases, approaches 100 percentage points (almost all children eventually learn to speak), and approaches zero because the percentage stops increasing. 96. a. (7) = 100 and

= 0: Once all the children learn to read, the percentage stops changing and so = 7

its rate of change is 0. 25.3 25 b. = 3.6 percentage points per year. = 5 1 12

15 12.0 1 + 8.6(1.8) 6 (6 + 0.0001) (6 0.0001) 11.97284169 11.97255763 (6) 1.4 0.0002 0.0002 Interpretation: At the start of 2006, about 12% of U.S. mortgages were subprime, and this percentage was increasing at a rate of about 1.4 percentage points per year. b. Graphs: Graph of : 97. a. (6) =


Graph of :

Solutions Section 10.5

From the graphs, ( ) approaches 15 as becomes large (in terms of limits, lim + ( ) = 15) and ( ) approaches 0 as t becomes large (in terms of limits, lim + ( ) = 0). Interpretation: If the trend modeled by the function had continued indefinitely, in the long term 15% of U.S. mortgages would have been subprime, and this percentage would not be changing. 1, 350 1, 225 1, 230 1 + 4.2(1.7) 7 (7 + 0.0001) (7 0.0001) 1, 224.657476 1, 224.645409 (7) 60.3 0.0002 0.0002 Interpretation: At the start of 2007, there was about $1,230 billion in subprime mortgage debt outstanding, and this figure was increasing at a rate of about $60.3 billion per year. b. Graphs: Graph of : 98. a. (7) =

Graph of :

From the second graph, is highest around = 3, or 2003. Interpretation: subprime mortgage debt outstanding was increasing fastest around 2003. 99. Graph of :

a. (D): The graph of the derivative is rising.


Solutions Section 10.5 b. 33 days after the egg was laid: That is where the graph of the derivative is highest. c. 50 days after the egg was laid: That is where the graph of the derivative is lowest in the range 20 50. 100. Graph of :

a. (A): The graph of the derivative rises and then falls. b. 17 days after the egg was laid: That is where the graph of the derivative is highest. c. 30 days after the egg was laid: That is where the graph of the derivative is lowest in the range 8 30. (0.95 + 0.0001) (0.95 0.0001) 304.2 meters/warp. 0.0002 Thus, at a speed of warp 0.95, the spaceship has an observed length of 31.2 meters, and its length is decreasing at a rate of 304.2 meters per unit warp, or 3.042 meters per increase in speed of 0.01 warp.

101. (0.95) 31.2 meters and (0.95)

102. (0.95) 3.20; (0.95) 31.20 seconds/warp. Thus, at a speed of warp 0.95, the spaceship's clocks are observed to take 3.20 seconds to register 1 second earth-time, and this amount of time is increasing at a rate of 31.20 seconds per unit warp, or 0.3120 seconds per increase in speed of 0.01 warp. 103. In (A),

is not differentiable because lim 0

( + ) ( ) does not exist (its left and right limits

disagree). In (B), is not differentiable because is not in the domain of . In (C), is not differentiable because is not an interior point of the domain of . ( + ) ( ) In (D), is not differentiable because lim 0 is not finite. 104. In (A), exists. In (B), In (C), In (D),

is differentiable at = 4; is an interior point of the domain and lim 0

( + ) ( )

is not differentiable because is not an interior point of the domain of . is not differentiable because is not in the domain of . ( + ) ( ) is differentiable at : is an interior point of the domain and lim 0 exists,

105. The difference quotient is not defined when = 0 because there is no such number as 0/0. 106. The interval [ 0.0001, + 0.0001]

107. (10) = 50 tells you that the membership in 2030 ( = 10) was 50 million. (10) = 6 tells you that the rate of change of membership in 2030 ( = 10) was 6 million per year; that is, that the membership was decreasing at a rate of 6 million per year. These statements give choice (D) as the correct answer.


Solutions Section 10.5 108. (100) = 10 tells you that net earnings in 3120 ( = 100) was $10 million; that is, the company lost $10 million. (100) = 60 tells you that the rate of change of net earnings in 3120 ( = 100) was $60 million per year; that is, net earnings were increasing at a rate of $60 million per year. These statements give choice (A) as the correct answer. 109. The derivative is positive (sales are still increasing) and decreasing toward zero (sales are leveling off). 110. The derivative is negative (sales are decreasing) and increasing toward zero (sales are bottoming out, so not decreasing as quickly). 111. Company B. Although the company is currently losing money, the derivative is positive, showing that the profit is increasing. Company A, on the other hand, has profits that are declining. 112. While Company C is currently more profitable, Company D's profits are increasing faster, so Company D seems like a better investment. 113. (C) is the only graph in which the instantaneous rate of change on January 1 is greater than the 1-month average rate of change. 114. (C) is the only graph in which the instantaneous rate of change on January 1 is negative while the 1-month average rate of change is positive. 115. The tangent to the graph is horizontal at that point, so the graph is almost horizontal near that point. 116. Various graphs are possible.

117. Various graphs are possible.

118. Various graphs are possible.

119. If ( ) = + , then its average rate of change over any interval [ , + ] is ( + ) + ( + ) = . Because this does not depend on , the instantaneous rate is also equal to . 120. If is a linear function, then its average rate of change over any interval is the number of units it increases per unit of increase in . Because is linear, this rate of increase is constant, and equals the slope , which is the instantaneous rate of change at every point.


Solutions Section 10.5 121. Increasing, because the average rate of change appears to be rising as we get closer to 5 from the left (see the bottom row). 122. Decreasing, because the average rate of change appears to be dropping as we get closer to 7 from the left (see the bottom row). 123. Various graphs are possible.

124. Various graphs are possible.

125.

126.

127. (B): His average speed was 60 miles per hour. If his instantaneous speed was always 55 mph or less, he could not have averaged more than 55 mph. 128. (D): Her average speed was 50 miles per hour. If her instantaneous speed was always 55 mph or more, she could not have averaged less than 55 mph. 129. Answers will vary.

130. Answers will vary.


Solutions Section 10.6 Section 10.6

(2 + 2) (2) (2 + 2) 2 + 1 (2 2 + 1) = lim 0 2 2 4 + 42 + 2 2 + 1 5 42 + 2 2 = lim 0 = lim 0 = lim 0 (4 + 2) = 4 2 2

1. (2) = lim 0

(1 + 2) (1) (1 + 2) 2 3 (1 2 3) = lim 0 2 2 1 + 22 + 2 2 3 + 2 22 + 2 2 = lim 0 = lim 0 = lim 0 (2 + 2) = 2 2 2

2. (1) = lim 0

( 1 + 2) ( 1) 3( 1 + 2) 4 ( 3 4) = lim 0 2 2 32 3 + 32 4 + 7 = lim 0 = lim 0 = lim 0 3 = 3 2 2

3. ( 1) = lim 0

( 1 + 2) ( 1) 2( 1 + 2) + 4 (2 + 4) = lim 0 2 2 2 22 + 4 6 22 = lim 0 = lim 0 = lim 0 ( 2) = 2 2 2

4. ( 1) = lim 0

(1 + 2) (1) 3(1 + 2) 2 + (1 + 2) (3 + 1) = lim 0 2 2 2 3 + 62 + 32 + 1 + 2 4 72 + 32 2 = lim 0 = lim 0 = lim 0 (7 + 32) = 7 2 2

5. (1) = lim 0

( 2 + 2) ( 2) 2( 2 + 2) 2 + ( 2 + 2) (8 2) = lim 0 2 2 2 2 8 82 + 22 2 + 2 6 72 + 32 = lim 0 = lim 0 = lim 0 ( 7 + 32) = 7 2 2

6. ( 2) = lim 0

( 1 + 2) ( 1) 2( 1 + 2) ( 1 + 2) 2 ( 2 1) = lim 0 2 2 2 2 42 2 2 + 22 1 + 22 2 + 3 = lim 0 = lim 0 = lim 0 (4 2) = 4 2 2

7. ( 1) = lim 0

8. (0) = lim 0

(0 + 2) (0) 2 2 2 0 = lim 0 = lim 0 ( 1 2) = 1 2 2

(2 + 2) (2) (2 + 2) 3 + 2(2 + 2) (8 + 4) = lim 0 2 2 8 + 122 + 62 2 + 2 3 + 4 + 22 12 142 + 62 2 + 2 3 = lim 0 = lim 0 2 2 = lim 0 (14 + 62 + 2 2) = 14

9. (2) = lim 0

(1 + 2) (1) (1 + 2) 2(1 + 2) 3 (1 2) = lim 0 2 2 1 + 2 2 62 62 2 22 3 + 1 52 62 2 22 3 = lim 0 = lim 0 2 2 = lim 0 ( 5 62 22 2) = 5

10. (1) = lim 0


Solutions Section 10.6 1 (1 + 2) ( 1) (1 2) (1) + 1 + (1 + 2) 11. (1) = lim 0 = lim 0 = lim 0 2 2 2(1 + 2) 2 1 = lim 0 = lim 0 = 1 2(1 + 2) 1+ 2

2 (5 + 2) (2 5) (5 + 2) (5) 10 2(5 + 2) = lim 0 = lim 0 2 2 52(5 + 2) 2 22 2 = lim 0 = lim 0 = 52(5 + 2) 5(5 + 2) 25

12. (5) = lim 0

(43 + 2) (43) (43 + 2) + ? (43 + ?) = lim 0 2 2 43 + 2 + ? 43 ? 2 = lim 0 = lim 0 = lim 0 = 2 2

13. (43) = lim 0

(12 + 2) & ? (12 & ?) (12 + 2) (12) = lim 0 2 2 12 & + 2 & ? 12 & + ? 2 & = lim 0 = lim 0 = lim 0 1 & = 1 & 2 2

14. (12) = lim 0

( + 2) ( ) ( + 2) 2 + 1 ( 2 + 1) = lim 0 2 2 2 + 2 2 + 2 2 + 1 2 1 2 2 + 2 2 = lim 0 = lim 0 = lim 0 (2 + 2) = 2 2 2

15. ( ) = lim 0

( + 2) ( ) ( + 2) 2 3 ( 2 3) = lim 0 2 2 2 + 2 2 + 2 2 3 2 + 3 2 2 + 2 2 = lim 0 = lim 0 = lim 0 (2 + 2) = 2 2 2

16. ( ) = lim 0

( + 2) ( ) 3( + 2) 4 (3 4) = lim 0 2 2 3 + 32 4 3 + 4 32 = lim 0 = lim 0 = lim 0 3 = 3 2 2

17. ( ) = lim 0

( + 2) ( ) 2( + 2) + 4 ( 2 + 4) = lim 0 2 2 2 22 + 4 + 2 4 22 = lim 0 = lim 0 = lim 0 ( 2) = 2 2 2

18. ( ) = lim 0

( + 2) ( ) 3( + 2) 2 + ( + 2) (3 2 + ) = lim 0 2 2 3 2 + 6 2 + 32 2 + + 2 3 2 6 2 + 32 2 + 2 = lim 0 = lim 0 2 2 = lim 0 (6 + 32 + 1) = 6 + 1

19. ( ) = lim 0

( + 2) ( ) 2( + 2) 2 + ( + 2) (2 2 + ) = lim 0 2 2 2 2 + 4 2 + 22 2 + + 2 2 2 4 2 + 22 2 + 2 = lim 0 = lim 0 2 2 = lim 0 (4 + 22 + 1) = 4 + 1

20. ( ) = lim 0


Solutions Section 10.6 ( 2) ( ) + 2( + 2) ( + 2) 2 (2 2) 21. ( ) = lim 0 = lim 0 2 2 2 + 22 2 2 2 2 2 2 + 2 22 2 2 2 2 = lim 0 = lim 0 2 2 = lim 0 (2 2 2) = 2 2

( + 2) ( ) ( + 2) ( + 2) 2 ( 2) = lim 0 2 2 2 2 2 2 2 2 2 + + 2 2 2 2 2 = lim 0 = lim 0 2 2 = lim 0 ( 1 2 2) = 1 2

22. ( ) = lim 0

( + 2) ( ) ( + 2) 3 + 2( + 2) ( 3 + 2 ) = lim 0 2 2 3 + 3 22 + 3 2 2 + 2 3 + 2 + 22 3 2 3 22 + 3 2 2 + 2 3 + 22 = lim 0 = lim 0 2 2 = lim 0 (3 2 + 3 2 + 2 2 + 2) = 3 2 + 2

23. ( ) = lim 0

( + 2) ( ) ( + 2) 2( + 2) 3 ( 2 3) = lim 0 2 2 + 2 2 3 6 22 6 2 2 22 3 + 2 3 2 6 22 6 2 2 22 3 = lim 0 = lim 0 2 2 2 2 2 = lim 0 (1 6 6 2 22 ) = 1 6

24. ( ) = lim 0

1 ( + 2) ( 1 ) ( + 2) ( ) + ( + 2) = lim 0 = lim 0 2 2 2 ( + 2) 2 1 1 = lim 0 = lim 0 = 2 ( + 2) ( + 2) 2

25. ( ) = lim 0

2 ( + 2) (2 ) ( + 2) ( ) 2 2( + 2) = lim 0 = lim 0 2 2 2 ( + 2) 2 22 2 = lim 0 = lim 0 = 2 2 ( + 2) ( + 2)

26. ( ) = lim 0

( + 2) ( ) ( + 2) + ? ( + ?) = lim 0 2 2 + 2 + ? ? 2 = lim 0 = lim 0 = lim 0 = 2 2

27. ( ) = lim 0

( + 2) & ? ( & ?) ( + 2) ( ) = lim 0 2 2 & + 2 & ? & + ? 2 & = lim 0 = lim 0 = lim 0 1 & = 1 & 2 2

28. ( ) = lim 0

8(2 + 2) 8(2) 0.3(2 + 2) 2 ( 0.3 × 2 2) = lim 0 2 2 1.2 1.22 0.32 2 + 1.2 1.22 0.32 2 = lim 0 = lim 0 = lim 0 ( 1.2 0.32) = 1.2 2 2

29. 8 (2) = lim 0

30. ( 1) = lim 0

( 1 + 2) ( 1) 1.4( 1 + 2) 2 1.4( 1) 2 = lim 0 2 2


Solutions Section 10.6 1.4 2.82 + 1.52 2 1.4 2.82 + 1.52 2 = lim 0 = lim 0 = lim 0 ( 2.8 + 1.52) = 2.8 2 2 (3 + 2) (3) 5.1(3 + 2) 2 + 5.1 (5.1 × 9 + 5.1) = lim 0 2 2 45.9 + 30.62 + 5.12 2 + 5.1 51 30.62 + 5.12 2 = lim 0 = lim 0 2 2 = lim 0 (30.6 + 5.12) = 30.6

31. (3) = lim 0

(4 + 2) (4) 1.3(4 + 2) 2 + 1.1 ( 1.3 × 16 + 1.1) = lim 0 2 2 20.8 10.42 1.32 2 + 1.1 + 20.8 1.1 10.42 1.32 2 = lim 0 = lim 0 2 2 = lim 0 ( 10.4 1.32) = 10.4

32. (4) = lim 0

33.

( + 2) ( ) 1.3( + 2) 2 4.5( + 2) ( 1.3 2 4.5 ) C = lim = lim 0 0 C 2 2 2 2 2 1.3 2.6 2 1.32 4.5 4.52 + 1.3 + 4.5 2.6 2 1.32 2 4.52 = lim = lim 0 0 2 2 = lim ( 2.6 1.32 4.5) = 2.6 4.5 0

Evaluating at = 1 gives 34.

C = 2.6(1) 4.5 = 7.1. C D = 1

( + 2) ( ) 5.1( + 2) 2 1.1( + 2) (5.1 2 1.1 ) C = lim = lim 0 0 C 2 2 5.1 2 + 10.2 2 5.12 2 1.1 1.12 5.1 2 + 1.1 10.2 2 5.12 2 1.12 = lim = lim 0 0 2 2 = lim (10.2 5.12 1.1) = 10.2 1.1 0

Evaluating at = 1 gives

C = 10.2(1) 1.1 = 9.1. C D = 1

(( + 2) (( ) 4.25( + 2) 5.01 (4.25 5.01) C( = lim = lim 0 0 C 2 2 4.25 + 4.252 5.01 4.25 + 5.01 4.252 = lim = lim = lim 4.25 = 4.25 0 0 2 0 2 C( Evaluating at = 1.2 gives = 4.25. C D = 1.2 35.

(( + 2) (( ) 1.02( + 2) + 5.7 ( 1.02 + 5.7) C( = lim = lim 0 0 C 2 2 1.02 1.022 + 5.7 + 1.02 5.7 1.022 = lim = lim = lim 1.02 = 1.02 0 0 0 2 2 C( Evaluating at = 1.2 gives = 1.02. C D = 3.1 36.

2.4 2.4 � + + 3.1� � + 3.1� C9 9( + 2) 9( ) 37. = lim = lim 0 0 C 2 2 2.4 2.4( + ) 2.4 2.4 � + � � ( + ) � 2.4 2.4 2.42 = lim = lim = lim 0 0 0 2 2 2 ( + 2)


Solutions Section 10.6 2.4 2.42 2.4 = lim = lim = 2 0 2 ( + 2) 0 ( + 2) C9 | 2.4 Evaluating at = 2 gives || = = 0.6. C | = 2 4

1 1 � 0.5( + ) 3.1� � 0.5 3.1� C9 9( + 2) 9( ) 38. = lim = lim 0 0 C 2 2 0.5 0.5( + ) 1 1 � 0.5( + ) 0.5 � � 0.25 ( + ) � 0.5 0.5 0.52 = lim = lim = lim 0 0 0 2 2 0.252 ( + 2) 1 0.52 0.5 = lim = lim = 0 0.252 ( + 2) 0 0.25 ( + 2) 0.5 2 C9 | 1 Evaluating at = 2 gives || = = 0.5. C | = 2 0.5(4)

39. Find the slope by finding the derivative: (2 + 2) (2) (2 + 2) 2 3 (2 2 3) = (2) = lim 0 = lim 0 2 2 4 + 42 + 2 2 3 1 42 + 2 2 = lim 0 = lim 0 = lim 0 (4 + 2) = 4. 2 2 The tangent line has slope 4 and goes through (2, (2)) = (2, 1), so has equation ! = 4 7. 40. Find the slope by finding the derivative: (2 + 2) (2) (2 + 2) 2 + 1 (2 2 + 1) = (2) = lim 0 = lim 0 2 2 4 + 42 + 2 2 + 1 5 42 + 2 2 = lim 0 = lim 0 = lim 0 (4 + 2) = 4. 2 2 The tangent line has slope 4 and goes through (2, (2)) = (2, 5), so has equation ! = 4 3. 41. Find the slope by finding the derivative: (3 + 2) (3) 2(3 + 2) 4 ( 2 × 3 4) = (3) = lim 0 = lim 0 2 2 6 22 4 + 10 22 = lim 0 = lim 0 = lim 0 ( 2) = 2. 2 2 The tangent line has slope 2 and goes through (3, (3)) = (3, 10), so has equation ! = 2 4. 42. Find the slope by finding the derivative: (1 + 2) (1) 3(1 + 2) + 1 (3 × 1 + 1) = (1) = lim 0 = lim 0 2 2 3 + 32 + 1 4 32 = lim 0 = lim 0 = lim 0 3 = 3. 2 2 The tangent line has slope 3 and goes through (1, (1)) = (1, 4), so has equation ! = 3 + 1. 43. Find the slope by finding the derivative: ( 1 + 2) ( 1) ( 1 + 2) 2 ( 1 + 2) [( 1) 2 ( 1)] = ( 1) = lim 0 = lim 0 2 2 1 22 + 2 2 + 1 2 2 32 + 2 2 = lim 0 = lim 0 = lim 0 ( 3 + 2) = 3. 2 2 The tangent line has slope 3 and goes through ( 1, ( 1)) = ( 1, 2), so has equation ! = 3 1. 44. Find the slope by finding the derivative:


Solutions Section 10.6 ( 1 2) + ( 1) ( 1 + 2) 2 + ( 1 + 2) [( 1) 2 + ( 1)] = ( 1) = lim 0 = lim 0 2 2 1 22 + 2 2 1 + 2 0 2 + 2 2 = lim 0 = lim 0 = lim 0 ( 1 + 2) = 1. 2 2 The tangent line has slope 1 and goes through ( 1, ( 1)) = ( 1, 0), so has equation ! = 1. B( + 2) B( ) 400 16( + 2) 2 (400 16 2) = lim 0 2 2 2 2 2 400 16 32 2 162 400 + 16 = lim 0 2 32 2 162 2 = lim 0 = lim 0 ( 32 162) = 32 2 B (4) = 32(4) = 128 ft/sec.

45. B ( ) = lim 0

B( + 2) B( ) 1, 000 120( + 2) 16( + 2) 2 (1, 000 120 16 2) = lim 0 2 2 2 2 1, 000 120 1202 16 32 2 162 1, 000 + 120 + 16 2 = lim 0 2 1202 32 2 162 2 = lim 0 = lim 0 ( 120 32 162) = 120 32 2 B (3) = 120 32(3) = 216 ft/sec 46. B ( ) = lim 0

( + 2) ( ) 11( + 2) 2 170( + 2) + 1,300 (11 2 170 + 1,300) C = lim 0 = lim 0 C 2 2 11( 2 + 2 2 + 2 2) 170( + 2) + 1,300 (11 2 170 + 1,300) = lim 0 2 2 22 2 + 112 1702 (The rest of the terms cancel.) = lim 0 2 2(22 + 112 170) (Cancel the 2 and then let 2 0.) = lim 0 = 22 170 2 C At time = 8, this becomes D = 22(8) 170 = 6 thousand barrels per year. C = 8 Daily oil imports were inrceasing at a rate of 6,000 barrels per year.

47.

* ( + 2) * ( ) C* = lim 0 C 2 0.01( + 2) 2 + 0.01( + 2) + 2.5 ( 0.01 2 + 0.01 + 2.5) = lim 0 2 0.01( 2 + 2 2 + 2 2) + 0.01( + 2) + 2.5 ( 0.01 2 + 0.01 + 2.5) = lim 0 2 2 0.02 2 0.012 + 0.012 (The rest of the terms cancel.) = lim 0 2 2( 0.02 0.012 + 0.01) (Cancel the 2 and then let 2 0.) = lim 0 = 0.02 + 0.01 2 C* At time = 6, this becomes = 0.02(6) + 0.01 = 0.11 million barrels per year; daily oil C D = 6 production was decreasing at a rate of 0.11 million barrels per year. 48.

( + 2) ( ) 0.4( + 2) 2 + ( + 2) + 26.5 (0.4 2 + + 26.5) = lim 0 2 2 0.8 2 + 0.42 2 + 2 = lim 0 = 0.8 + 1 2

49. a. ( ) = lim 0


Solutions Section 10.6 b. ( ) = 0.8 + 1 is a linear function of with positive values for 0 / / 5 and also positive slope, so its value is positive and increases with time (choice (A)). c. As ( ) is the rate at which the annual revenue of food apps increases, the answer to part (b) tells us that the annual revenue of food apps was projected to increase at a faster and faster rate, or accelerate, over the given period of time.

( + 2) ( ) 0.2( + 2) 2 + 3( + 2) + 18 ( 0.2 2 + 3 + 18) = lim 0 2 2 0.4 2 0.22 2 + 32 = lim 0 = 0.4 + 3 2 b. ( ) = 0.4 + 3 is a linear function of with positive values for 0 / / 5 and negative slope, so its value is positive but decreases with time (choice (B)). c. As ( ) is the rate at which the annual revenue of food apps increases, the answer to part (b) tells us that the annual revenue of food apps was projected to increase at a decreasing rate over the given period of time. 50. a. ( ) = lim 0

51. a. Revenue 8 = 9( ) = ( 2.5 + 1,850) = 2.5 2 + 1,850 million dollars. 8( + 2) 8( ) 2.5( + 2) 2 + 1,850( + 2) ( 2.5 2 + 1,850 ) 8 ( ) = lim 0 = lim 0 2 2 5 2 2.52 2 + 1,8502 = lim 0 = 5 + 1,850 2 b. 8(380) = 2.5(380) 2 + 1,850(380) = 342,000 million dollars, or $342 billion. 8 (380) = 5(380) + 1,850 = 50 million dollars per $1 increase in the price . Selling 5G phones at an average price of $380 will result in $342 billion in revenue, and the revenue will decrease at a rate of $50 million per $1 increase in the price. c. The derivative in part (b) can also be interpreted as stating that the revenue will increase at a rate of $50 million per $1 decrease in the price, so the selling price should be decreased in order to increase the total revenue. 52. a. Revenue 8 = 9( ) = ( 10 + 4,360) = 10 2 + 4,360 million dollars. 8( + 2) 8( ) 10( + 2) 2 + 4,360( + 2) ( 10 2 + 4,360 ) 8 ( ) = lim 0 = lim 0 2 2 20 2 102 2 + 4,3602 = lim 0 = 20 + 4,360 2 b. 8(210) = 10(210) 2 + 4,360(210) = 474,600 million dollars, or $474.6 billion. 8 (210) = 20(210) + 4,360 = 160 million dollars per $1 increase in the price . Selling 4G phones at an average price of $210 will result in $474.6 billion in revenue, and the revenue will increase at a rate of $160 million per $1 increase in the price. c. As the revenue increases at a rate of $160 million per $1 decrease in the price, the selling price should be increased in order to increase the total revenue. 53. a. The only possible point of discontinuity could be at = 20. However, (20) = 700 and lim 20 ( ) = 31.1(20) + 78 = 700

lim 20 + ( ) = 90(20) 1, 100 = 700

So, lim 20 ( ) = 700 = (20),

showing that the function is continuous at = 20. b. The graph of comes to a point at = 20 (the slope immediately before that point is 31.1 and the slope immediately after is 90). Thus, is not differentiable at = 20. ( ) = 31.1 if < 20 and ( ) = 90 if > 20, so


lim 20 ( ) = 31.1

Solutions Section 10.6

lim 20 + ( ) = 90.

Until 1990 the cost of a Super Bowl ad was increasing at a rate of $31,100 per year. Immediately thereafter, it was increasing at a rate of $90,000 per year. 54. a. The only possible point of discontinuity could be at = 20. However, (20) = 1, 100 and lim 20 ( ) = 43.9(20) + 222 = 1, 100

lim 20 + ( ) = 140(20) 1, 700 = 1, 100

So, lim 20 ( ) = 1, 100 = (20),

showing that the function is continuous at = 20. b. The graph of comes to a point at = 20 (the slope immediately before that point is 43.9 and the slope immediately after is 140). Thus, is not differentiable at = 20. ( ) = 43.9 if < 20 and ( ) = 140 if > 20, so lim 20 ( ) = 43.9 lim 20 + ( ) = 140. Until 2000 the cost of a Super Bowl ad was increasing at a rate of $43,900 per year. Immediately thereafter, it was increasing at a rate of $140,000 per year. 55. The algebraic method, because it gives the exact value of the derivative. The other two approaches give only approximate values (except in some special cases). 56. The difference quotient is not defined when 2 = 0 because there is no such number as 0/0.

57. The error is in the second line: ( + 2) is not equal to ( ) + 2. For instance, if ( ) = 2, then ( + 2) = ( + 2) 2, whereas ( ) + 2 = 2 + 2.

58. There are two serious errors: (1) In the second line: ( + 2) is not equal to ( ) + (2). For instance, if ( ) = 2, then ( + 2) = ( + 2) 2, whereas ( ) + (2) = 2 + 2 2. (2) The last step makes no sense; you cannot cancel 2 from within (2). 59. The error is in the second line: One could cancel the 2 only if it were a factor of both the numerator and denominator; it is not a factor of the numerator.

60. There are two serious errors: (1) In the second line: ( + 2) is not equal to ( ) + 2. (2) The cancellation in the third line is illegal; One could only cancel the 2 if it were a factor of both the numerator and denominator; it is not a factor of the numerator.

61. Because the algebraic computation of ( ) is exact, and not an approximation, it makes no difference whether one uses the balanced difference quotient or the ordinary difference quotient in the algebraic computation. (3 + 2) (3 2) (3 + 2) 2 (3 2) 2 = lim 0 22 22 9 + 62 + 2 2 9 + 62 2 2 122 = lim 0 = lim 0 = 6, 22 22

62. (3) = lim 0


Solutions Section 10.6 the same answer you get using the ordinary difference quotient. 63. The computation results in a limit that cannot be evaluated.

64. By the time we reach the end of the computation, we have canceled the offending 2 in the denominator, so that putting 2 = 0 makes sense. Put another way, the difference quotient is not defined when 2 = 0, but, after canceling the 2 we change it to a closed-form function (at least in the calculations done in the textbook) that is defined when 2 = 0, so that the limit can be obtained by substituting 2 = 0.


Solutions Chapter 10 Review Chapter 10 Review 1. From the following table, we estimate the limit to be 5.

( )

2.9

4.9

2.99 2.999 2.9999

4.99 4.999 4.9999

3

3.0001 3.001 3.01

5.0001 5.001 5.01

3.1

5.1

2. From the following table, we see that the limit does not exist.

2.9

2.99

2.999

2.9999

( ) 33.9 303.99 3,003.999 30,003.9999

3

3.0001

1.1

1.01 1.001 1.0001

( ) 0.3226 0.3322 0.3332

0.3333

1

0.9999 0.999

1.1

1.01

1.001

1.0001

( ) 0.0529 0.0050 0.0005 5 × 10 5

3.1

0.99

0.9

0.3333 0.3334 0.3344 0.3448

4. From the following table, we estimate the limit to be 0.

3.01

29,996 2,995.999 295.99 25.9

3. From the following table, we see that the limit does not exist.

3.001

1

0.9999 0.999

0.99

0.9

5 × 10 5 0.0005 0.0050 0.0478

5. a. 1: As approaches 0 from the left or right, ( ) approaches the open dot at height 1. The fact that (0) = 3 is irrelevant. b. 3: As approaches 1 from the left or right, ( ) approaches the point on the graph corresponding to = 1, whose !-coordinate is 3. c. Does not exist: As approaches 2 from the left, ( ) approaches the open dot at height 2. As approaches 2 from the right, ( ) approaches the solid dot at height 1. Thus, the one-sided limits, though they both exist, do not agree. 6. a. Does not exist: As approaches 0 from the left, ( ) approaches the open dot at height 1. As approaches 0 from the right, ( ) approaches the solid dot at height 3. Thus, the one-sided limits, though they both exist, do not agree. b. 2: As approaches 2 from the left or right, ( ) approaches the point on the graph corresponding to = 2, whose !-coordinate is 2. c. 1: As approaches 2 from the left or right, ( ) approaches the open dot at height 1. The fact that (2) = 3 is irrelevant.

2 is a closed-form function whose domain includes = 2. Therefore we can substitute to 3 obtain the limit: ( 2) 2 2 lim = = 4 5 2 3 2 3 7. ( ) =

2 9 is a closed-form function whose domain does not include = 3, but we can simplify: 2 6 ( 3)( + 3) 2 9 + 3 3+ 3 lim = lim = lim = = 3. 3 2 6 3 3 2 2( 3) 2

8. ( ) =


Solutions Chapter 10 Review 2 4 9. ( ) = 3 is a closed-form function whose domain does not include = 2, but we can + 2 2 simplify: ( 2)( + 2) 2 4 2 2 2 lim 3 = lim = lim = = 1. 2 2 + 2 2 2 2 ( + 2) 2 ( 2) 2

2 9 is a closed-form function whose domain includes = 1. Therefore we can substitute 2 6 to obtain the limit: 2 9 1 9 lim = = 1. 1 2 6 2 6 10. ( ) =

11. ( ) = lim

2 2

0 2 2

is a closed-form function whose domain does not include = 0, but we can simplify:

= lim

0 (2 1)

= lim

1

0 2 1

=

1 = 1. 0 1

2 9 is a closed-form function whose domain does not include = 1, nor can we simplify to 1 cancel the ( 1) term. However, the numerator approaches 8 and the denominator approaches 0, giving us the determinate form 8 0 = ± . Specifically, 2 9 2 9 lim = - and - lim 1 + = . 1 1 1 Since the left and right limits disagree, the overall limit does not exist. 12. ( ) =

( + 3) 2 + 3 = lim 1 2 2 1 ( 2)( + 1)

13. lim

( + 3) 2 , so lim = . 1 ( 2)( + 1) ( 3)(0 ) ( + 3) 2 As 1 + , the ratio has the determinate form , so lim = . ( 3)(0 + ) 1 + ( 2)( + 1) 2 + 3 Since the left and right limits disagree, lim 2 does not exist. 1 2 As 1 , the ratio has the determinate form

2 + 1 2 + 1 lim = 1 + 2 + 3 + 2 1 + ( + 2)( + 1) + As 1 , the numerator approaches 2 and the factor in the denominator ( + 1) is positive and 2 approaches zero. Thus, the limit has the determinate form , which is positive, and so diverges to (1)(0+ ) . 14.

lim

( 8)( + 2) 2 6 16 + 2 10 = lim = lim = 8 2 9 + 8 8 ( 1)( 8) 8 1 7

15. lim

( + 3) 4 ( 4) 2 As 4 from either side, the numerator approaches 28 and the denominator is positive and approaches zero, so the limit has the determinate form 28 0 + , telling us that the limit diverges to . 16. lim

2 + 3

4 2 8 + 16

= lim

2 + 8 2 + 8 lim = 4 2 2 8 4 ( + 2)( 4)

17. lim


Solutions Chapter 10 Review As 4 + , the numerator approaches 24 and the factor in the denominator ( 4) is positive and 24 which is positive, and so the limit approaches zero. Thus, the limit has the determinate form (6)(0 + ) diverges to . As 4 , the factor ( 4) is negative and approaches zero. Thus, the limit has the determinate form 24 which is negative, and so the limit diverges to . (6)(0 ) 2 + 8 Since the left and right limits disagree, lim 2 does not exist. 4 2 8 ( 6)( + 1) 2 5 6 + 1 7 = lim = lim = 6 2 36 6 ( 6)( + 6) 6 + 6 12

18. lim

2 + 8 2 + 8 lim = 1 2 4 2 4 + 1 1 2 (2 1) 2 As 1 2 from either side, the numerator approaches 8.25 and the denominator is positive and approaches zero, so the limit has the determinate form 8.25 0 + , telling us that the limit diverges to . 19. lim

( + 3) 2 + 3 = lim 2 1 2 2 + 3 1 1 2 (2 1)( + 2) 1.75 As 1 2 we get the determinate form which is negative, and so the limit diverges to . (0 )(2.5) 1.75 As 1 2 + we get + , which is positive, and so the limit diverges to . (0 )(2.5) 2 + 3 Since the left and right limits disagree, lim does not exist. 1 2 2 2 + 3 1 20. lim

21. Ignoring all but the highest terms in numerator and denominator, we get: 10 2 + 300 + 1 10 2 2 lim lim = = lim = 0. 3 3 5 5 + 2 22. Ignoring all but the highest terms in numerator and denominator, we get: 2 4 + 20 3 2 4 1 lim = lim = lim = 0. 1,000 6 + 6 1,000 6 500 2 23. Ignoring all but the highest terms in numerator and denominator, we get: 2 6 2 lim = lim = lim = . 3 So the limit diverges to . 24. Ignoring all but the highest terms in numerator and denominator, we get: 2 6 2 1 1 lim lim = = lim = . 2 2 4 3 4 4 4 So the limit diverges to .

25. As , 3 2 as well, so the denominator 5 + 5.3(3 2 ) . Thus 5 5 lim = 0, because = 0. 5 + 5.3(3 2 )

26. As , 4 as well, so 2 4 0, whence 3 + 2 4 3 + 0 = 3.


27. As , 4

3

=

4

Solutions Chapter 10 Review 4 2 2 2 0, becuase = 0. Thus, lim = = . 3 5 + 4 3 5+ 0 5

28. As , 4 3 as well, whence 4 3 + 12 diverges to .

29. As , the term 2 3 is 1 2 3 and so approaches zero. Thus the entire numerator approaches 1 + 0 = 1. The term 5.3 in the denominator is 5.3 and so approaches 0, so the denominator approaches 1 + 0 = 1 as well. Hence, the entire expression converges to 1 1 = 1.

30. As , the numerator is 8 + 0.5 and hence has the form 8 + 0.5 = 8 + (1 0.5) = 8 + 2 , which diverges to . The denominator approaches 2, since 3 = 0. Hence the entire expression has the form 2 = , and hence diverges to . 31. ( ) =

1 ;- = 0 + 1

The average rate of change is

( + 2) ( ) 2

2 = 1: (0 + 1) (0) (1) (0) 0.5 = = = 0.5 1 1 1 2 = 0.01: (0 + 0.01) (0) (0.01) (0) 0.009901 = = = 0.99001 0.01 0.01 0.01 2 = 0.001: (0 + 0.001) (0) (0.001) (0) 0.000999 = = = 0.9990. 0.001 0.001 0.001 Table: 2

1

0.01

0.001

Avg. Rate of Change 0.5 0.9901 0.9990

The slope of the tangent is the limit as 2 0, which appears to be about 1. 32. ( ) = ; - = 2

The average rate of change is

( + 2) ( ) 2

2 = 1: (2 + 1) (2) (3) (2) 23 = = = 23 1 1 1 2 = 0.01: (2 + 0.01) (2) (2.01) (2) 0.068404 = = 6.8404 0.01 0.01 0.01 2 = 0.001: (2 + 0.001) (2) (2.001) (2) 0.0067793 = = 6.7793. 0.001 0.001 0.001 Table: 2

Avg. Rate of Change

1

23

0.01

0.001

6.8404 6.7793


Solutions Chapter 10 Review The slope of the tangent is the limit as 2 0, which appears to be about 6.8.

33. ( ) = 2 ; - = 0; Technology formula for ( ): TI-83/84 Plus Y1= e^(2x) Spreadhseet: EXP(2*x) To compute the average rate of change on the TI-83/84 Plus, use the following formulas: 2 = 1: (Y1(0+1)-Y1(0))/1 2 = 0.01: (Y1(0+.01)-Y1(0))/.01 2 = 0.001: (Y1(0+.001)-Y1(0))/.001Table: 2

1

0.01

0.001

Avg. Rate of Change 6.3891 2.0201 2.0020

The slope of the tangent is the limit as 2 0, which appears to be about 2.

34. ( ) = ln (2 ); - = 1 Technology formula for ( ): ln(2x) To compute the average rate of change on the TI-83/84 Plus, use the following formulas: 2 = 1: (Y1(1+1)-Y1(1))/1 2 = 0.01: (Y1(1+.01)-Y1(1))/.01 2 = 0.001: (Y1(1+.001)-Y1(1))/.001 Table: 2

1

0.01

0.001

Avg. Rate of Change 0.6931 0.9950 0.9995

The slope of the tangent is the limit as 2 0, which appears to be about 1. 35. a. *

36. a. None 37. a. =

38. a. None

b. =

b. 8

b. None b. 8

c. 8

c. =

d.

c. None

d. *

c. * and

d. None d. =

39. a. (B): The graph starts on the tangent line and falls below it. b. (B): The graph starts above the tangent line and ends below it. c. (B): The graph is getting less steep. d. (A): The graph gets steeper until = 0 and then gets less steep. e. (C): The value of (2) is the height of the graph at = 2, which is about 2.5; the rate of change is the slope of the tangent line at that point, which is approximately 0.5. 40. a. (B): The graph starts on the tangent line and falls below it. b. (C): The average rate of change of over [0, 2] is 0, and the tangent is horizontal at = 1. c. (A): The slope increases from a negative value at = 2 to a positive value at = 0. d. (A): The graph gets steeper until = 0 and then gets less steep. e. (A): The value of (0) is the height of the graph at = 0, which is 0; the rate of change is the slope of the tangent line at that point, which is approximately 1.5. 41. ( ) = 2 +


Solutions Chapter 10 Review ( 2) ( ) + ( + 2) 2 + ( + 2) ( 2 + ) ( ) = lim = lim 0 0 2 2 2 + 2 2 + 2 2 + + 2 2 2 2 + 2 2 + 2 = lim = lim = lim (2 + 2 + 1) = 2 + 1 0 0 0 2 2

42. ( ) = 3 2 + 1 ( + 2) ( ) 3( + 2) 2 ( + 2) + 1 (3 2 + 1) ( ) = lim = lim 0 0 2 2 2 2 2 3 + 6 2 + 32 2 + 1 (3 + 1) = lim 0 2 2(6 + 32 1) 6 2 + 32 2 2 = lim = lim = lim (6 + 32 1) = 6 1 0 0 0 2 2

43. ( ) = 1

2

1 2 ( + 2) (1 2 ) 2 ( + 2) + 2 ( + 2) ( ) = lim 0 = lim 0 2) 2 2 2 + 2( + 2) 22 2 2 = lim 0 = lim 0 = lim 0 = 2 ( + 2) 2 ( + 2) ( + 2) 2

( ) = lim 0

1 + 1 1 ( + 2) + 1 (1 + 1) 1 ( + 2) 1 ( + 2) ( ) ( ) = lim 0 = lim 0 = lim 0 2 2 2 ( + 2) 1 2 1 = lim 0 = lim 0 = lim 0 = 2 2 ( + 2) 2 ( + 2) ( + 2) 44. ( ) =

45. y1 = 10x^5+(1/2)x^4-x+2 y2 = nDeriv(Y1,X,X) (TI-83/84 Plus) y2 = deriv(y1) (Website)

46. y1 = 10/x^5+1/(2x^4)-1/x+2 y2 = nDeriv(Y1,X,X) (TI-83/84 Plus) y2 = deriv(y1) (Website)

47. y1 = 3x^3 + 3x^(1/3) y2 = nDeriv(Y1,X,X) (TI-83/84 Plus) y2 = deriv(y1) (Website)


Solutions Chapter 10 Review

48. y1 = 2/x^2.1-x^0.1/2 y2 = nDeriv(Y1,X,X) (TI-83/4 Plus) y2 = deriv(y1) (Website)

49. a. * (3) {}= value of * ( ) at = 3 {}= 25. As approaches 3 from the left. the !-coordinate of the corresponding point on the graph approaches 25. Therefore, lim * ( ) = 25 3

As approaches 3 from the right. the !-coordinate of the corresponding point on the graph approaches 10. Therefore, lim * ( ) = 10 3

Since the left and right limits do not agree, lim * ( ) does not exist. 3

Interpretation: * (3) = 3: O'Hagan purchased the stock at $25. lim 3 * ( ) = 25: The value of the stock had been approaching $25 up to the time he bought it. lim 3 + * ( ) = 10: The value of the stock dropped to $10 immediately after he bought it. b. As approaches 6 from either side, * ( ) approaches 5, which is also the value of * (6). In other words, lim * ( ) = 5 = * (6). 6

showing that * is continuous at = 6. On the other hand, the graph comes to a sharp point at = 6 so * is not differentiable at = 6. Interpretation: the stock changed continuously but suddenly reversed direction (and started to go up) the instant O'Hagan sold it.

50. a. * (6) = 30: Robles sold the stock at $30. lim 6 * ( ) = 30: The value of the stock had been approaching $30 up the time he sold it. lim 6 + * ( ) = 15: The value of the stock dropped to $15 immediately after he sold it. b. Continuous but not differentiable. Interpretation: the stock price changed continuously but suddenly reversed direction (and started to go up) the instant Robles purchased it.

51. a. lim 3 ( ) 40; lim ( ) = . Close to 2007 ( = 3), the home price index was about 40. In the long term, the home price index will rise without bound. b. 10. (The slope of the linear portion of the curve is 10.) In the long term, the home price index will rise about 10 points per year.

52. a. lim 2 ( ) = 8,000; lim ( ) = 10,000. Close to 2 weeks after the start of the marketing campaign, the weekly cost was about $8,000. In the long term, the weekly cost of the marketing campaign


Solutions Chapter 10 Review will be around $10,000. b. 0; In the long term, the weekly cost will remain approximately constant.

9,000 6,500 = 500 books per week 5 b. [3, 4] (600 books per week), [4, 5] (700 books per week) c. [3, 5], when the average rate of increase was 650 books per week 53. a.

54. a. Average rate of change of sea level =

(125) (0) 390 0 = = 3.12 mm/year 125 0 125

b. The individual 25-year rates of change are (60 0) 25, (140 60) 25, (240 140) 25, (310 240) 25, (390 310) 25, which are, respectively, 2.4, 3.2, 4, 2.8, and 3.2. Those that exceeded 3.12 were for the periods [25, 50], [50, 75] and [100, 125]. c. Since 2 meters = 2,000 mm, it would take another 2,000 3.12 641 years before the sea level rises to her condominium.

55. a. The 10-year period beginning 2004 corresponds to the interval [0, 10] for . (10) (0) 40 10 Average rate of change = = 3 percentage points per year 10 0 10 (10) (3) 40 40 b. = 0 percentage points per year 10 3 7 c. Choice (D): The slope of the tangent decreases from 4 to around 5, and then it starts to increase (the graph curves less steeply downward). This means that the rate of change of the index first decreased, and then increased. (6) (0) 10,000 2,000 8,000 $1, 333 per week = 6 0 6 6 (6) (2) 10,000 8,000 2,000 b. = = $500 per week 6 2 4 4 c. Choice (B): The graph becomes less steep as changes from 1 to 6, so the slope is deceasing. This means that the rate of change of cost is decreasing. Also, the cost itself is increasing. 56. a. Average rate of change =

G( + 2) G( ) 2 36( + 2) 2 + 250( + 2) + 6,240 (36 2 + 250 + 6,240) = lim 0 2 72 2 + 362 2 + 2502 = lim 0 = 72 + 250; 2 b. G (1) = 72(1) + 250 = 322 books per week c. G (7) = 72(7) + 250 = 754 books per week 57. a. G ( ) = lim 0

B( + 2) B( ) 2 0.002( + 2) 2 + 3( + 2) 6.4 (0.002 2 + 3 6.4) = lim 0 2 0.004 2 + 0.0022 2 + 32 = lim 0 = 0.004 + 3 2 b. B (100) = 0.004(100) + 3 = 3.4 mm per year c. B (200) = 0.004(200) + 3 = 3.8 mm per year

58. a. B ( ) = lim 0


Solutions Chapter 10 Case Study Chapter 10 Case Study

1. Using the notation of the Case Study, we have : (9) = (9) & and we want : (8,000,000) = 0, so we need (8,000,000) & = 0 or & = (8,000,000). We are given (8,000,000) = $270 per ton, so the annual emission charge should be & = $270 per ton. 2. Changing 8 to 12 in the solution to Exercise 1, we get that the emission charge should be $779 per ton. 3. Since the emissions charge obtained in the Case Study is $4,000,000,000 3609, the marginal emissions charge is the derivative, $360 per ton. This marginal emissions charge is the negative of the marginal cost of reducing sulfur emissions (before the charge is imposed) at the optimal reduction level.

4. For 12 million tons to be the optimal reduction the cost function would have to change in such a way that the marginal cost would satisfy (12,000,000) = $360 per ton. Just as in the Case Study, this would make : (12,000,000) = 0 after applying the emissions charge and hence make 12 million tons the optimal reduction. 5. Following the reasoning in Exercise 4, would have to change so that (8,000,000) = $360 per ton. 6. The EPA would have to change & to equal the (presumably higher) new value of (10,000,000) and recalculate the emissions charge accordingly.

7. As we did in the Case Study, we can change the emissions charge by any constant and still have 10 million tons be the optimal reduction. If we simply charge $360 per ton of sulfur released, the industry would have to pay 25,000,000(360) 3609 = 9,000,000,000 3609. Giving back a subsidy of $H would have the effect of changing the emissions charge to 9,000,000,000 H 3609. If the industry is to pay no charge at a reduction level of 10 million tons, then we should have 9,000,000,000 H 360(10,000,000) = 0, or H = 9,000,000,000 3,600,000,000 = 5,400,000,000. Thus, the plan would be to charge $360 per ton of sulfur released into the environment and give back to the industry a subsidy of $5.4 billion.

8. As in the Case Study, we put everything in terms of 9, the amount of the reduction in sulfur emissions. The annual emissions would once again be 25,000,000 9, so the emissions charge would now be &(25,000,000 9) 2. The total cost to utilities would be :(9) = (9) + &(25,000,000 9) 2 = (9) &(625 × 10 12 50,000,0009 + 9 2) = (9) 625 × 10 12& 50,000,000&9 + &9 2.

Thus, : (9) = (9) 50,000,000& + 2&9. To make : (10,000,000) = 0 once more, we need to have (10,000,000) 50,000,000& + 20,000,000& = 0, or & = (10,000,000) 30,000,000 = 360 30,000,000 = 0.000012.


Solutions Section 11.1 Section 11.1

1. ( ) has the form so its derivative is 1 = 5 4. 2. ( ) has the form so its derivative is 1 = 4 3.

3. ( ) has the form constant × so its derivative is ×constant × 1 = 2( 2 3) = 4 3. 4. ( ) has the form constant × so its derivative is ×constant × 1 = 3( 2) = 3 2. 5. 0.25 0.75 6. 0.5 1.5 7. Write

1 5 as 5, so its derivative is 5 6, which equals . 5 6

8. Write

1 8 as 8, so its derivative is 8 9, which equals 9 . 8

9. Write

1 3

10. Write

11.

12.

13.

14.

1

1

1 1 as 1 3, so its derivative is 4 3, which equals . 3 3 4 3

1 8

1 1 as 1 8, so its derivative is 9 8, which equals . 8 8 9 8

= 10(0) = 0 (constant multiple and power rule) = 3 2 (power rule)

2 ( ) + ( ) (sum rule) = 2 + 1 (power rule) =

( ) (5) (difference rule) = 1 (power and constant multiple) =

(4 3) + (2 ) (1) (sum and difference) = 3 = 4 ( ) + 2 ( ) (1) (constant multiples) = 12 2 + 2 (power rule) 15.

(4 1) (2 ) (10) (differences) = 1 = 4 ( ) 2 ( ) 10 (1) (constant multiples) = 4 2 2 (power rule) 16.

17. ( ) = 2 3

18. ( ) = 9 2 4 + 1


19. ( ) = 1 + 0.5 0.5

Solutions Section 11.1

20. ( ) = 0.5 0.5 1.5 21. ( ) = 2 3 + 3 2 22. ( ) = 2 2 8 3 23. ( ) = 24. ( ) = 25. ( ) = 26. ( ) = 27. ( ) = 28. ( ) = ( ) =

29.

30.

31.

32.

( ) = ( ) = ( ) =

33. ( ) = 34. ( ) = ( ) =

35.

36.

( ) =

1 1 2 ( 2) = 2 + 2 3 = 2 + 3 2 2 3 ( + 3) = 2 3 3 4 = 3 4 0.8 (2 0.4) = 0.8 1.4 = 1.4

0.1 ( 0.5 0.2) = 0.1 1.2 = 1.2

2 2 6 ( + 2 3) = 0.1 1.2 = 2 3 6 4 = 3 4

2 6 4 (2 1 2 3 + 4) = 2 2 + 6 4 4 5 = 2 + 4 5

2 1 1 0.1 2 0.1 1.1 2 0.1 = 2 + 1.1 � � = 2 + 3 2 3 2 3 2 4 2 9 8 3.2 � + 3.2� = 3 3.2 4.2 = 3 4.2 3 3 3

2 1 4 2 0.1 0.9 4.4 2.1 2 0.1 4.4 = 0.9 2.1 � 0.1 + 1.1 2� = 3 2 3 3 2 3 3 2 3 4 2 1 3.2 2 2 8 3.2 2.2 4 3 8 3.2 2.2 4 + = + + 3 � + + 4� = + 3 6 3 3 6 3 3 6 3

(| | + 1) = | | 2 = | | 1 2

| | 1 1 2 3| | 1 (3| | 1 2) = 3 = 2 2

1 2 1 1 1 1 ( + 1 2) = 1 2 3 2 = 2 2 2 2 7 7 ( + 7 1 2) = 1 3 2 = 1 2 2


Solutions Section 11.1 1 37. ( ) = � � (assuming 0) = 3 1, and so ( ) = 3 2. 2

2 38. ( ) = 1� � = 1 2 2, and so 39. ( ) = 40. ( ) =

( ) = 4 3.

2 2 3 2 2 3 = = 2 2, and so

2 + 2 = 2 + , and so

41. 2.6 0.3 + 1.2 2.2

( ) = 1 4 .

( ) = 1.

42. 8.6 3.3 + 0.6 0.4 43. 1.2(1 | | )

44. 4(2 + 3| | )

45. 3 2 4 (Remember to treat as a constant, i.e., a number.)

46. 2 + (Remember to treat , , as constants, i.e., numbers.) 47. 5.15 9.3 99 2

48. 0.4 0.2 0.45 0.1 49.

50.

1 2.31 0.3 = �2.3 + 2.1 1.1 0.6� = 2.31 2.1 + 0.3 0.4 = 2.1 0.4 2 1.1 2.2 1.2 (2 = + 1.2) = 2.2 2.1 1.2 2.2 = 2.1 2.2

51. 4 2 52. 8

53. Slope of tangent = derivative evaluated at the indicated -value: ( ) = 3 2, so ( 1) = 3.

54. Slope of tangent = derivative evaluated at the indicated -value: ( ) = 4 3, so ( 2) = 32. 55. Slope of tangent = derivative evaluated at the indicated -value: ( ) = 2, so (2) = 2.

56. Slope of tangent = derivative evaluated at the indicated -value: ( ) = 1 3, so ( 3) = 1 3. 57. Slope of tangent = derivative evaluated at the indicated -value: 5 5 ( ) = ( ) = 5 6 = , so (1) = 5. 6


Solutions Section 11.1 58. Slope of tangent = derivative evaluated at the indicated -value: 3 3 3 ()= ( ) = 3 4 = 4 , so ( 2) = . 16

59. ( ) = 3 2, so ( 1) = 3. The line with slope 3 passing through ( 1, ( 1)) = ( 1, 1) is = 3 + 2.

60. ( ) = 2 , so (0) = 0. The line with slope 0 passing through (0, (0)) = (0, 0) is = 0.

61. ( ) = + 1, so ( ) = 1 2 = 1

passing through (2, (2)) = (2, 5 2) is =

1 ; (2) = 1 1 4 = 3 4. The line with slope 3 4 2

3 + 1. 4

2 62. ( ) = 2, so ( ) = 2 3 = 3 ; (1) = 2. The line with slope 2 passing through (1, (1)) = (1, 1) is = 2 + 3.


Solutions Section 11.1

63. ( ) = 1 2, so ( ) = (4, (4)) = (4, 2) is =

1 1 2 1 1 ; (4) = . The line with slope 1 4 passing through = 2 4 2

1 + 1. 4

64. ( ) = 2, so ( 1) = 2. The line with slope 2 passing through ( 1, ( 1)) = ( 1, 2) is = 2 + 4. (Note that, since the graph of is a line, the tangent line is the given line again.)

65. The tangent line is horizontal when its slope is zero; that is, the derivative is zero: = 4 + 3 = 0 when = 3 4. 66. The tangent line is horizontal when its slope is zero; that is, the derivative is zero: = 6 1 = 0 when = 1 6. 67. The tangent line is horizontal when its slope is zero; that is, the derivative is zero: However, = 2 and thus cannot be 0, so there are no such values of . 68. The tangent line is horizontal when its slope is zero; that is, the derivative is zero: However, = 1 and thus cannot be 0, so there are no such values of .


Solutions Section 11.1 69. The tangent line is horizontal when its slope is zero; that is, the derivative is zero: = 1 2 = 1 1 2 = 0 when 2 = 1, so = 1 or 1. 70. The tangent line is horizontal when its slope is zero; that is, the derivative is zero: 1 1 = 1 1 2 = 1 = 0 when 2 = 1, so = (1 2) 2 = 1 4. 2 2

( + ) 4 4 4 4 + 4 3 + 6 2 2 + 4 3 + 4 4 ( ) = lim 0 = lim 0 4 3 + 6 2 2 + 4 3 + 4 3 2 = lim 0 = lim 0 (4 + 6 + 4 2 + 3) = 4 3 71.

72. ( + ) 5 5 4 5 + 5 4 + 10 3 2 + 10 2 3 + 5 4 + 5 5 ( ) = lim 0 = lim 0 5 4 + 10 3 2 + 10 2 3 + 5 4 + 5 4 = lim 0 = lim 0 (5 + 10 3 + 10 2 2 + 5 3 + 4) = 5 4 73. ( ) = 1 3; and so ( ) = 1

1 2 3 1 . = 1 3 3 2 3

1 2 = . 3 3 b. = 0 : (0) is not defined (0 in denominator) and so is not differentiable at 0. a. = 1 : (1) is defined, and so is differentiable at 1; (1) = 1 74. ( ) = 2 + 4 3; and so ( ) = 2 +

4 1 3 . 3

4 2 3 22 8 . = 3 3 4 b. = 0 : (0) is defined, and so is differentiable at 0; (0) = 2 + 0 2 3 = 2. 3 a. = 8 : (8) is defined, and so is differentiable at 1; (8) = 2 +

75. ( ) = 5 4 1 and so ( ) =

5 1 4 . 4

76. ( ) = 1 5 + 5; and so ( ) =

1 4 5 1 . = 5 5 4 5

5 1 4 5 16 = . 4 2 b. = 0 : Although the formula for (0) makes sense, 0 is an endpoint of the domain of and so is not differentiable at 0. a. = 16 : (16) is defined, and so is differentiable at 16; (16) =

1 . 5 b. = 0 : (0) is not defined (0 in denominator) and so is not differentiable at 0. a. = 1 : (1) is defined, and so is differentiable at 1; (1) = 77. Since putting = 1 yields 0 0, L'Hospital's rule applies. 2 2 + 1 2 2 0 lim = lim = = 0 1 1 2 1 1 2

78. Since putting = 1 yields 0 0, L'Hospital's rule applies. 2 + 3 + 2 2 + 3 1 lim = lim = = 1 2 1 1 2 + 1 1 +


Solutions Section 11.1 79. Since putting = 2 yields 0 0, L'Hospital's rule applies. 3 8 3 2 12 lim = lim = = 12 2 2 2 1 1

80. Putting = 0 yields 8 2 = 4. Since this is not an indeterminate form, L'Hospital's rule does not apply, and the limit is 4 (closed-form function).

81. Putting = 1 yields 6 2 = 3. Since this is not an indeterminate form, L'Hospital's rule does not apply, and the limit is 3 (closed-form function). 82. Since putting = 2 yields 0 0, L'Hospital's rule applies. 3 + 8 3 2 12 lim 2 = lim = = 12 2 + 3 + 2 2 2 + 3 1

83. Since putting = yields , L'Hospital's rule applies. 3 2 + 10 1 6 + 10 6 3 lim = lim = lim = 2 4 5 4 2 2 5 84. Since putting = yields , L'Hospital's rule applies. 6 2 + 5 + 100 12 + 5 12 lim = lim = lim = 2 2 6 6 3 9 85. Since putting = yields , L'Hospital's rule applies. 10 2 + 300 + 1 20 + 300 lim = lim 5 + 2 5 This limit is in determinate form and diverges to .

86. Since putting = yields , L'Hospital's rule applies. 2 4 + 20 3 8 3 + 60 2 8 + 60 lim lim \quad (Cancel 2.) = = lim 1,000 3 + 6 3,000 2 3,000 This limit is in determinate form and diverges to . 87. Since putting = yields , L'Hospital's rule applies. 3 100 3 2 6 lim lim = = lim 2 2 + 50 4 4 This limit is in determinate form and diverges to .

88. Since putting = yields , L'Hospital's rule applies. 2 + 30 2 + 30 2 2 lim = lim = lim = = 0 2 6 + 10 12 3 + 10 36 2 +

89. ( ) = 3 10 2 + 24 + 39, so ( ) = 3 2 20 + 24 (5) = 3(5) 2 20(5) + 24 = 1 thousand dollars per month. The price of Bitcoin was decreasing at a rate of around $1,000 per month at the start of August 2021 ( = 5).

90. ( ) = 0.05 3 + 0.3 2 0.8 + 19, so ( ) = 0.15 2 + 0.6 0.8 (2) = 0.15(2) 2 + 0.6(2) 0.8 = 0.2 hundred dollars per month The price per ounce of gold was decreasing at a rate of about $20 per month at the start of May 2021 ( = 2).


Solutions Section 11.1 91. To find the rate of change of spending on food ( ) with respect to we take the derivative using the power rule: 18.8 = = 18.8 1.05, so = (18.8)( 1.05) 1.05 1 = 19.74 2.05. 1.05 We evaluate this at = 2%: | = 19.74(2) 2.05 4.8 percentage points per one percentage point increase in spending on | = 2 education. Because the rate of change is negative, the rate of decrease is about 4.8 percentage points per one percentage point increase in spending on education.

92. To find the rate of change of spending on food ( ) with respect to we take the derivative using the power rule: 688 = 1.99 = 688 1.99, so = (688)( 1.99) 1.99 1 = 1369.12 2.99. We evaluate this at = 6%: | = 1369.12(6) 2.99 6.5 percentage points per one percentage point increase in spending on | = 6 education. Because the rate of change is negative, the rate of decrease is about 6.5 percentage points per one percentage point increase in spending on education.

93. a. ( ) = 32 ; (0) = 0, (1) = 32, (2) = 64, (3) = 96, (4) = 128 ft/sec b. ( ) = 0 when 400 16 2 = 0, so 2 = 400 16 = 25, so at = 5 seconds; the stone is traveling at the velocity (5) = 160, so downward at 160 ft/sec.

94. a. ( ) = 120 32 ; (0) = 120, (1) = 152, (2) = 184, (3) = 216, (4) = 248 ft/sec b. ( ) = 0 when 1, 000 120 16 2 = 0. \\From the quadratic formula or from the factorization, 1, 000 120 16 2 = (5 )(200 + 16 ), so ( ) = 0 when = 5 seconds. At that time the stone has velocity (5) = 280, so it is falling downward at 280 ft/sec.

95. a. ( ) = 76 1.9 2 m, so ( ) = 76 3.8 m/sec. At the highest point, stops increasing and begins to decrease, so its rate of change, , changes from positive to negative, meaning it must be zero. 76 ( ) = 0 76 3.8 = 0 = = 20 sec. 3.8 b. The highest point is reached when = 20 sec, so the height is (20) = 76(20) 1.9(20) 2 = 760 m. 96. a. ( ) = 0 5.6 2 m, so ( ) = 0 11.2 m/sec. At the heighest point, 0 ( ) = 0 0 11.2 = 0 = sec. 11.2 0 1 b. The highest point is reached when = sec, so 0 = 5.6 m/sec, and so ( ) = 5.6 5.6 2. = 11.2 2 Height at that instant is ( 12 ) = 5.6( 12 ) 5.6( 12 ) 2 = 1.4 m.

97. a. ( ) = 6 2 44 + 1,840, so ( ) = 12 44 2013 corresponds to = 3, and (3) = 12(3) 44 = 8 billion dollars per year. Interpretation: Worldwide annual military expenditure was decreasing at a rate of $8 billion per year in 2013 ( = 3) b. Geometrically: Notice that the graph is falling over the interval [0, 3] (military expenditure was decreasing) but that the slope of the tangent is getting less steep (the rate of decrease was slowing). Therefore, the value of decreased at a slower and slower rate (choice (D)). Algebraically: The slope of the military expenditure curve is the derivative: ( ) = 12 44. Its value on


Solutions Section 11.1 [0, 3] is negative, so the rate of change of military expenditure is negative (annual military expenditure was decreasing). However, as increases from 0 to 3, ( ) increases in value, so the rate of increase of annual military expenditure is increasing (that is, military expenditure decreased more slowly). Therefore, the value of decreased at a slower and slower rate (choice (D)).

98. a. ( ) = 0.073 2 + 0.18 + 73.25 so ( ) = 0.146 + 0.18 2016 corresponds to = 3, and (3) = 0.146(3) + 0.18 = 0.258 billion dollars per year. Interpretation: Annual spending on education was decreasing at a rate of $0.258 billion per year in 2016 ( = 3). b. Geometrically: Notice that the graph is dropping over the interval [2, 7] (spending on education was decreasing) and that the slope of the tangent is becoming more negative (the rate of decrease was speeding up). Therefore, the value of decreased at a faster and faster rate (choice (C)). Algebraically: The slope of the education spending curve is the derivative: ( ) = 0.146 + 0.18. Its value on [2, 7] is negative, so the rate of change of spending on education is negative (spending was decreasing). Also, as increases from 2 to 7, ( ) decreases in value, so the rate of change of spending on education is decreasing as well (that is, spending on education decreased more quickly). Therefore, the value of decreased at a faster and faster rate (choice (C)). 99. a. ( ) = 3.55 2 30.2 + 81, so ( ) = 7.1 30.2; (8) = 7.1(8) 30.2 = 26.6 manatees per 100,000 boats. At a level of 800,000 boats, manatee deaths are increasing at a rate of 26.6 deaths each year per 100,000 additional boats. b. ( ) = 7.1 30.2 is linear with positive slope, and thus increasing. The number of manatees killed per additional 100,000 boats increases as the number of boats increases.

100. a. ( ) = 0.27 2 32 + 1,200, so ( ) = 0.54 32. (45) = 0.27(45) 2 32(45) + 1,200 = 306.75 accidents/100 million miles (45) = 0.54(45) 32 = 7.7 accidents/100 million miles per additional year of driver's age 45-year old drivers have an average of around 306.75 accidents/100 million miles of driving, and this frequency is decreasing at a rate of around 7.7 accidents/100 million miles per additional year of the driver's age. b. ( ) = 0.54 32 is linear with positive slope, and thus increasing with increasing . It is negative for 59 and positive above that. The frequency of accidents decreases at a slower and slower rate as a driver gets older, and eventually begins to increase.

101. a. The function has values ( ) ( ), which measures the amount by which the Chrome market share exceeds the Firefox market share. Its derivative ( ) measures the rate at which this difference is changing. b. ( ) ( ), whose absolute value is the vertical distance between the two graphs, starts off negative (the Chrome curve is below the Firefox curve) and then gradually increases through zero and becomes positive a little before = 2, and continues to increase through = 4. Thus the difference is increasing (choice (A)). c. ( ) ( ) = ( ) ( ) = ( 1.30 + 13) ( 0.0213 2 + 0.46 4) = 0.0213 2 1.76 + 17 percentage points per year. One could check from its graph that this function remains positive on [1, 4], or one could reason as follows: this quadratic function has a concave up graph whose vertex is at = (2 ) = 1.76 0.0426 41.3. At = 1 and = 4 the derivative is positive and these points are to the left of the vertex, so it decreases in the interval [1, 4] and so remains positive throughout the interval (choice (A)). Because ( ) measures the rate at which Chrome's market share advantage over Firefox is


Solutions Section 11.1 changing, the result tells us that Chrome's advantage over Firefox was increasing during the stated period 2011–2014. d. ( ) ( ) = 0.0213 2 1.76 + 17, so ( ) (3) = 0.0213(3) 2 1.76(3) + 17 11.9 percentage points per year. Interpretation: Chrome's market share advantage over Firefox was increasing at a rate of around 11.9 percentage points per year at the start of 2013 ( = 3).

102. a. The function has values ( ) ( ), which measures the amount by which the Firefox market share exceeds the Safari market share. Its derivative ( ) measures the rate at which this difference is changing. b. ( ) ( ), whose absolute value is the vertical distance between the two graphs, starts off positive (the Firefox curve is above the Safari curve) and then gradually decreases through zero and becomes negative around = 10.5, and continues to decrease through = 11. Thus the difference is decreasing (choice (B)). c. ( ) ( ) = ( ) ( ) = ( 0.0213 2 + 0.46 4) (0.164 0.6) = 0.0213 2 + 0.296 3.4 percentage points per year. One could check from its graph that this function remains negative on [8, 11], or one could reason as follows: this quadratic function has a concave down graph whose vertex is at = (2 ) = 0.296 0.0426 6.9. At = 8 and = 11 the derivative is negative and these points are to the right of the vertex, so it decreases in the interval [8, 11] and so remains negative throughout the interval (choice (B)). Because ( ) measures the rate at which Firefox's market share advantage over Safari is changing, the result tells us that Firefox's advantage over Safari was decreasing during the stated period 2018–2021. d. ( ) ( ) = 0.0213 2 + 0.296 3.4, so ( ) (10) = 0.0213(10) 2 + 0.296(10) 3.4 2.6 percentage points per year. Interpretation: Firefox's market share advantage over Safari was decreasing at a rate of around 2.6 percentage points per year at the start of 2020 ( = 10). | = 6 | = 1 = 6. | = 1 After graphing the curve = 3 2, draw the line passing through ( 1, 3) with slope 6.

103. The slope of the tangent line at ( 1, 3) is the value of the derivative,

104. After graphing the curve = 4 , draw the line through (0.5, 8) with slope 1 16.

105. The slope of the tangent line of is twice the slope of the tangent line of because ( ) = 2 ( ), so ( ) = 2 ( ).

106. The slopes are the same: ( ) = ( ) + 3, so ( ) = ( ). 107. ( ) = ( ) 108. ( ) = ( )

109. The left-hand side is not equal to the right-hand side. The derivative of the left-hand side is equal to the right-hand side, so your friend should have written \\ (3 4 + 11 5) = 12 3 + 55 4. 110. The left-hand side is not equal to the right-hand side. The derivative of the left-hand side is equal to the right-hand side, so you should have written \\ ( ) = 1.


Solutions Section 11.1 1 1 1 1 1 111. is not equal to 2 . Your friend should have written \\ = = 1, so = 2. 2 2 2 2 112. We cannot apply the power rule to terms in the denominator. To take the derivative of it in exponent form as

3 2 3 3 and then use the power rule to get 3 or 3 . 4 2 2

3 , first write 4 2

113. The derivative of a constant times a function is the constant times the derivative of the function, so that ( ) = (2)(2 ) = 4 . Your enemy mistakenly computed the derivative of the constant times the derivative of the function. (The derivative of a product of two functions is not the product of the derivative of the two functions. The rule for taking the derivative of a product is discussed later in the chapter.) 114. We cannot apply the power rule separately to terms in the numerator and denominator. To take the 3 3 3 derivative of , first write it in exponent form as 1 and then use the power rule to get 2 or 4 4 4 3 2. 4 115. For a general function , the derivative of is defined to be\\ ( ) = lim 0

( + ) ( ) .

One then finds by calculation that the derivative of the specific function is 1. \\In short, 1 is the derivative of a specific function: ( ) = ; it is not the definition of the derivative of a general function or even the definition of the derivative of the function ( ) = . 116. For a general function , the derivative of is defined to be\\ ( ) = lim 0

( + ) ( ) .

A possible response is that the power rule applies only to a function that happens to be given by a constant power of . Since there certainly are functions that have nothing to do with constant powers of , the power rule is very limited, and we need the general definition to deal with other kinds of functions. 117. Answers may vary; here is one possibility. At one point its derivative is not defined but it has a tangent line: The tangent line at that point is vertical so has undefined slope.\\


Solutions Section 11.1 118. Answers may vary; here is one possibility. At two points it has a vertical tangent line, which has undefined slope, so its derivative is undefined there.\\


Solutions Section 11.2 Section 11.2

1. ( ) = 5 0.0002 ; (1,000) = $4.80 per item 2. ( ) = 7 0.0001 ; (10,000) = $6 per item 3. ( ) = 100 4. ( ) = 50

1000 ; (100) = $99.90 per item 2

10,000 ; (100) = $49 per item 2

5. ( ) = 4 , so ( ) = 4. ( ) = 8 0.001 2, so ( ) = 8 0.002 . ( ) = ( ) ( ) = ( ) = 8 0.001 2 4 = 4 0.001 2, so ( ) = 4 0.002 . ( ) = 0 when = 2,000. Thus, at a production level of 2,000, the profit is stationary (neither increasing nor decreasing) with respect to the production level. This may indicate a maximum profit at a production level of 2,000. 6. ( ) = 5 2; ( ) = 3 + 7 + 10 ( ) = 10 ; ( ) = 3 2 + 7 ( ) = ( ) ( ) = 3 5 2 + 7 + 10; ( ) = 3 2 10 + 7 ( ) = 0 when = 1 and when = 7 3. Thus, at these production levels, the profit is stationary with respect to the production level. This may indicate a maximum or a minimum profit at each of these production levels.

7. a. (B): The slope of the graph decreases and then increases. b. (C): This is where the slope of the graph is least. c. (C): At = 50, the height of the graph is about 3,000, so the cost is $3,000. The tangent line at that point passes roughly through (100, 4,000), so has a slope of roughly 20; hence the cost is increasing at a rate of about $20 per item. 8. a. (B): (C): The slope of the graph is increasing. b. (C): The tangent line at = 100 passes through or close to the origin. The slope of the line through the origin is the average cost, so the marginal and average costs are approximately equal. c. (A) At = 150, the height of the graph is roughly 4,400, so the cost is approximately $4,400. The tangent line passes through about (175, 5, 400), so has a slope of about 40; hence the cost is increasing at a rate of about $40 per item.

9. a. ( ) = 20 + 5,600 + 0.05 2, so ( ) = 5,600 + 0.1 . (4) = 5,600 + 0.1(4) = 5,600.4 thousand dollars; that is, $5,600,400. The cost is going up at a rate of $5,600,400 per television commercial. The exact cost of airing the fifth television commercial is (5) (4) = 28,021.25 22,420.80 = 5,600.45 thousand dollars, or $5,600,450. 20 b. ( ) = (4) = 5,605.2 thousand dollars, or $5,605,200 per television + 5,600 + +0.05 ; commercial. The average cost of airing the first four television commercials is $5,605,200.

10. a. ( ) = 40 0.002 . The cost is increasing at a rate of (100) = $39.80 per teddy bear. The exact cost of producing the 101st teddy bear is (101) (100) = $39.799. b. ( ) = 100 + 40 0.001 ; (100) = $39.90 per teddy bear. The average cost of producing the first


Solutions Section 11.2 100 teddy bears is $39.90.

11. a. ( ) = 590 + 0.002 , so (10,000) = $610 per iPhone. The cost is increasing at the rate of $520 per iPhone. The exact cost of producing the 10,001st iPhone is (10,001) (10,000) = 6,400,610.001 6,400,000 = $610.001. 400,000 b. ( ) = + 590 + 0.001 . The average cost to produce the first 10,000 iPhones is (10,000) = $640 per iPhone c. The marginal cost from (a) is lower than the average cost from (b). This means that the average cost is falling at a production level of 10,000 iPhones.

12. a. ( ) = 510 + 0.001 , so (60,000) = $570 per console. The cost is increasing at the rate of $570 per console. The exact cost of producing the 60,001st console is (60,001) (60,000) = 33,200,570.0005 33,200,000 = $570.0005. 800,000 b. ( ) = + 510 + 0.0005 . The average cost to produce the first 60,000 consoles is (60,000) $553.33 per console. c. The marginal cost from (a) is higher than the average cost from (b). This means that the average cost is rising at a production level of 60,000 consoles. 13. a. ( ) = 0.90 Marginal revenue = ( ) = 0.90 ( ) = ( ) ( ) = 0.90 (70 + 0.10 + 0.001 2) = 70 + 0.80 0.001 2 Marginal Profit = ( ) = 0.80 0.002 b. ( ) = 0.90 ; (500) = 0.90(500) = $450 The total revenue from the sale of 500 copies is $450. ( ) = 70 + 0.80 0.001 2; (500) = 70 + 0.80(500) 0.001(500) 2 = $80 The profit from the production and sale of 500 copies is $80. ( ) = 0.90, so (500) = 0.90 Approximate revenue from the sale of the 501st copy is 90¢. ( ) = 0.80 0.002 ; (500) = 0.80 0.002(500) = 0.2 Approximate loss from the sale of the 501st copy is 20¢. (Negative marginal profit indicates a loss.) c. The marginal profit ( ) is zero when 0.80 0.002 = 0 = 0.80 0.002 = 400 copies. The graph of the profit function is a parabola with a vertex at = 400, so the profit is a maximum when you produce and sell 400 copies. 14. a. ( ) = 1.1 Marginal revenue = ( ) = 1.1 ( ) = ( ) ( ) = 1.1 (350 + 0.10 + 0.002 2) = 350 + 0.002 2 Marginal Profit = ( ) = 1 0.004 b. ( ) = 1.1 ; (200) = 1.1(200) = $220 The total revenue from the sale of 200 plates is $220. ( ) = 350 + 0.002 2; (200) = 350 + 200 0.002(200) 2 = $230 The society will lose $230 from the sale of 200 plates. ( ) = 1.1, so (200) = 1.1 Approximate revenue from the sale of the 201st plate is $1.10. ( ) = 1 0.004 ; (200) = 1 0.004(200) = 0.2 Approximate profit from the sale of the 201st plate is 20¢. c. The marginal profit ( ) is zero when


Solutions Section 11.2 1 0.004 = 0 = 1 0.004 = 250 plates. The graph of the profit function is a parabola with a vertex at = 250, so the loss is a minimum when you produce and sell 250 plates. (There is still a loss: (250) = $225.) 15. (1,000) represents the profit in dollars on the sale of 1,000 vinyl records. (1,000) = 3,000, so the profit on the sale of 1,000 vinyl records is $3,000. (1,000) represents the rate of increase of the profit as a function of . (1,000) = 3, so the profit is decreasing at a rate of $3 per additional record sold. 16. The loss on the sale of 50 Type M cars is $5,000 and the loss is decreasing at a rate of $200 per additional Type M car sold.

17. = 5 + . Your current profit is (50) = 5(50) + 50 $257.07. The marginal profit is 1 1 ( ) = 5 + , so (50) = 5 + 5.07. 2 2 50 The derivative is measured in dollars per additional magazine sold. Thus, your current profit is $257.07 per month, and this would increase at a rate of $5.07 per additional magazine in sales. 18. = 2

1

. 2 Thus, when = 100, $190, and ( ) 1.95. Your current profit is $190 per month, and this would increase at a rate of $1.95 per new subscriber. 19. a.

=

, so ( ) = 2

20,000 ! 1.5

20,000 $2.50 per pound. 400 1.5 20,000 20,000 b. (!) = ! = != 1.5 ! ! 0.5 c. (400) = $1000. This is the monthly revenue that will result from setting the price at $2.50 per pound. 10,000 10,000 (!) = , so (400) = $1.25 per pound of tuna. 1.5 ! 400 1.5 Thus, at a demand level of 400 pounds per month, the revenue is decreasing at a rate of $1.25 per pound. d. Since the revenue goes down with increasing demand, the fishery should raise the price to reduce the demand and hence increase revenue. When ! = 400, =

20. a. When ! = 400, =

60 $3.00 per pound 400 0.5

b. (!) = ! = 60! 0.5 c. (400) = $1, 200. This is the monthly revenue that will result from setting the price at $3.00 per pound. (!) = 30 ! 0.5, so (400) = $1.50 per pound of tuna. Thus, at a demand level of 400 pounds per month, the revenue is increasing at a rate of $1.50 per pound. d. The fishery should lower the price to increase the demand and hence increase revenue. 21. " = 40 0.05 2, so " ( ) = 40 0.1 , giving " (50) = 35 cars/worker. This means that, at an employment level of 50 workers, the firm's daily production will increase at a rate of 35 cars washed per additional worker it hires. 22. " = 10 + 2.5 2 0.0005 4, so " ( ) = 10 + 5 0.002 3, giving " (50) = 10 + 5(50) 0.002(50) 3 = 10 cars/worker.


Solutions Section 11.2 This means that, at an employment level of 50 workers, the firm's daily production will decrease at a rate of 10 cars washed per additional worker it hires. 23. a. (B): ( ) = 0.002 + 0.3 decreases as increases. 500 b. (B): ( ) = 0.001 + 0.3 + decreases as increases. (100) = 5.2 c. (C): (100) = 0.1,

24. a. (A): ( ) = 0.2 3.5 increases as increases. 500 b. (D): ( ) = 0.1 3.5 + decreases and then increases (seen most clearly by graphing it). (100) = 11.5 c. (A): (100) = 16.5,

25. a. ( ) = 500,000 + 1,900,000 100,000 50,000 ( ) = 1,900,000 ( ) 500,000 100,000 ( ) = = + 1,900,000 b. (3) $1,870,000 per spot; (3) $2,010,000 per spot Since the marginal cost is less than the average cost, the cost of the fourth ad is lower than the average cost of the first three, so the average cost will decrease as increases. 26. a. Since 10 carpenters are currently being used, we set # = 10, and so ( ) = 15,000 + 50(10) 2 + 60 2 = 20,000 + 60 2 ( ) = 120 ( ) 20,000 ( ) = = + 60 . b. (15) = 120(15) = $1, 800 per electrician 20,000 (15) = + 60(15) $2,233.33 per electrician 15 Since the marginal cost is less than the average cost, the cost of the 16th electrician is lower than the average cost of the first 16, so the average cost will decrease as increases.

27. a. (!) = 200!, so (10) = $2,000 per 1-pound reduction in emissions. b. (!) = 500. Thus (!) = (!) when 500 = 200!, or ! = 2.5 pounds per day reduction. c. $(!) = (!) (!) = 100! 2 500! + 4,000. This is a parabola with lowest point (vertex) given by ! = 2.5. The net cost at this production level is $(2.5) = $3, 375 per day. The value of ! is the same as that for part (b). The net cost to the firm is minimized at the reduction level for which the cost of controlling emissions begins to increase faster than the subsidy. This is why we get the answer by setting these two rates of increase equal to each other.

28. a. $(%) = %[1 & (%)] = % 0.001% 1.5 b. $ (%) = 1 0.0015% 0.5. $ (100, 000) $0.5257 per dollar income; $ (500,000) per dollar income c. The marginal after-tax income becomes negative when $ (%) = 0, or 1 0.0015% 0.5 = 0, giving % = $444, 444.44. This gives a net annual income of $(444, 444.44) = $148, 148.15. At any income level above $444,44.44, an individual begins to pay back more than $1 for each additional $1 earned, so his or her net income begins to drop. d. $148,148.15 29. ( ) =

3, 600 2 1 . So, (3600 1 + ) 2


Solutions Section 11.2 2 3, 600(10) 1 (10) = 0.0002557 mpg/mph (3, 600(10) 1 + 10) 2 This means that, at a speed of 10 mph, the fuel economy is increasing at a rate of 0.0002557 miles per gallon per 1-mph increase in speed. 3, 600(60) 2 1 (60) = = 0 mpg/mph (3, 600(60) 1 + 60) 2 This means that, at a speed of 60 mph, the fuel economy is neither increasing nor decreasing with increasing speed. 3, 600(70) 2 1 (70) = 0.00001799. (3, 600(70) 1 + 10) 2 This means that, at 70 mph, the fuel economy is decreasing at a rate of 0.00001799 miles per gallon per 1-mph increase in speed. Thus 60 mph is the most fuel-efficient speed for the car.

30. (3) $0.65, so that when the price is set at $3 per shirt, the revenue is increasing at a rate of 62¢ per additional $1 they charge. (4) 0, so that the revenue is neither increasing nor decreasing with increasing price. (5) $0.65, so that the revenue is decreasing at a rate of 65¢ per additional $1 they charge. Thus they should sell the shirts at $4 each to maximize revenue. 31. (C): If the marginal cost were lower in one plant than another, moving some production from the higher cost plant to the lower would result in a lower cost for the same production level. 32. (C): If the marginal product was higher for one course than another, allocating more hours to the course with the higher marginal product and fewer hours to the other would result in a higher grade with the same number of hours of study.

33. (D): The marginal product per dollar of salary of a senior professor is only 1.5 2 = 0.75 times that of a junior professor. Therefore, discharging senior professors and hiring more junior professors will result in a higher quantity of output for the same amount of money.

34. (C): The marginal product per dollar of salary of a senior professor is 2 (3 2) 1.33 times that of a junior professor. Therefore, hiring more senior professors and discharging junior professors will result in a higher quantity of output for the same amount of money. 35. (B): (In most cases) This is why we use the marginal cost as an estimate of the actual cost of the item. 36. (A): Both are equal to '.

37. Cost is often measured as a function of the number of items . Thus, ( ) is the cost of producing (or purchasing, as the case may be) items. a. The average cost function ( ) is given by ( ) = ( ) . The marginal cost function is the derivative, ( ), of the cost function. b. The average cost ( ) is the slope of the line through the origin and the point on the graph where = . The marginal cost of the th unit is the slope of the tangent to the graph of the cost function at the point where = . c. The average cost function ( ) gives the average cost of producing the first items. The marginal cost function ( ) is the rate at which cost is changing with respect to the number of items , or the incremental cost per item, and approximates the cost of producing the ( + 1)st item. 38. The average cost will decrease because the information on marginal cost tells you that it will cost only (approximately) $2,500 to manufacture the next piano—less than the average cost. Thus, the average cost will decrease if your company manufactures more pianos.


Solutions Section 11.2 39. Answers may vary. An example is ( ) = 300 . 40. Answers may vary. An example is ( ) = 300 + 10.

41. The marginal cost: If the average cost is rising, then the cost of the next piano must be larger than the average cost of the pianos already built. 42. Advise the company to increase production, since profit will rise with increasing production. 43. Not necessarily. For example, it may be the case that the marginal cost of the 101st item is larger than the average cost of the first 100 items (even though the marginal cost is decreasing). Thus, adding this additional item will raise the average cost. 44. Not necessarily. For example, it may be the case that the marginal cost of the 101st item is increasing, but still lower than the average cost of the first 100 items. Then, adding this additional item will lower the average cost. 45. The circumstances described suggest that the average cost function is at a relatively low point at the current production level, and so it would be appropriate to advise the company to maintain current production levels; raising or lowering the production level will result in increasing average costs. 46. Acceleration of cost is measured in dollars per unit, per unit. It is the rate at which the marginal cost is changing. Thus, if cost is accelerating (positive acceleration of cost), then the marginal cost is increasing with increasing production.


Solutions Section 11.3 Section 11.3

1. a. Constant multiple rule: ( ) = 3(1) = 3 b. Product rule: ( ) = (0) + 3(1) = 3

2. a. ( ) = 2(2 ) = 4 b. Product rule: ( ) = (0) 2 + 2(2 ) = 4

3. a. ( ) = ( 2 = 3; ( ) = 3 2 b. Product rule: ( ) = (1) 2 + (2 ) = 3 2

4. a. ( ) = ( = 2; ( ) = 2 b. Product rule: ( ) = (1) + (1) = 2

5. a. ( ) = ( + 3) = 2 + 3 ; ( ) = 2 + 3 b. Product rule: ( ) = (1)( + 3) + (1) = 2 + 3

6. a. ( ) = (1 + 2 ) = + 2 2; ( ) = 1 + 4 b. Product rule: ( ) = (1)(1 + 2 ) + (2) = 1 + 4

7. a. ( ) = 100(2.1 1.1) = 210 1.1 b. Product rule: ( ) = (0) 2.1 + 100(2.1 1.1) = 210 1.1

8. a. ( ) = 0.2( 2) = 0.2 2 b. Product rule: ( ) = (0) 1 + 0.2( 2) = 0.2 2

9. a. ( ) = 2 1;

10. a. ( ) =

( ) = 2 2 = 2 2 (0) 2(1) 2 b. Quotient rule: ( ) = = 2 2

1 2 1 2 ; ) ( ) = (2 ) = 3 3 3 2 (2 )3 (0) 2 b. Quotient rule: ) ( ) = = 2 3 3

1 1 ; ( ) = 3 3 (1)3 (0) 1 b. Quotient rule: ( ) = = 3 32 6 ( ) = 6 3 = 3 (0) 2 3(2 ) 6 b. Quotient rule: ( ) = = 3 ( 2) 2

11. a. )( ) =

12. a. ( ) = 3 2;

13. = 3 (4 2 1) = 3(4 2 1) + 3 (8 ) = 36 2 3

14. = 3 2(2 + 1) = 6 (2 + 1) + 3 2(2) = 18 2 + 6

15. = 3(1 2) = 3 2(1 2) + 3( 2 ) = 3 2 5 4

16. = 5(1 ) = 5 4(1 ) + 5( 1) = 5 4 6 5

17. = (2 + 3) 2 = 2(2 + 3) + (2 + 3)(2) = 8 + 12

18. = (4 1) 2 = 4(4 1) + (4 1)(4) = 32 8

4 5 2 4(5 2) (4 )(5) 8 = = (5 2) 2 (5 2) 2

19. =

3 3 + 2 3( 3 + 2) (3 )( 3) 6 = = ( 3 + 2) 2 ( 3 + 2) 2

20. =


2 + 4 3 1 2(3 1) 3(2 + 4) 14 = = 2 (3 1) (3 1) 2

3 9 2 + 4 3(2 + 4) 2(3 9) 30 = = 2 (2 + 4) (2 + 4) 2

Solutions Section 11.3

21. =

| | | | ( ) | |(1)� | | | | � = = = 0 2 2

23. =

22. =

24. =

| |

| | ( ) � �(1)| | = | | | | = 0 = 2 2

| | 2 | | 2 ( ) | |(2 )� | | 2 | | | | | | � = = = = 3 2 4 4 ( 2)

25. =

26. =

2 | |

| | ( 2) � �(2 )| | = 2 | | | | = | | = | | = 2 2 2 | |

27. = + = = 2

+

3 = 2 2

28. = 2 2 5 = 2 + = 2 + = 2 2 2 29. = ( + 1)( 2 1) 2 5 = 2 + = 2 + = 2 2 2

30. = (4 2 + )( 2) = (8 + 1)( 2) + (4 2 + )(1 2 ) = 16 3 + 9 2 + 2 31. = (2 0.5 + 4 5)( 1) = ( 0.5 + 4)( 1) + (2 0.5 + 4 5)(1 + 2)

32. = ( 0.7 4 5)( 1 + 2) = (0.7 0.3 4)( 1 + 2) + ( 0.7 4 5)( 2 2 3) 33. = (2 2 4 + 1) 2


Solutions Section 11.3 2 2 = (4 4)(2 4 + 1) + (2 4 + 1)(4 4) = 8(2 2 4 + 1)( 1)

34. = (2 0.5 2) 2 = ( 0.5 2 )(2 0.5 2) + (2 0.5 2)( 0.5 2 ) = 2(2 0.5 2)( 0.5 2 ) 3.2 + �( 2 + 1) 3.2 1 3.2 3.2 = � 2 �( 2 + 1) + � + �(2 ) 3.2 3.2

35. = �

2.1 2 + 2.1 �(7 1) 7 4.2 3.2 = �0.3 1.1 3.1 �(7 1) + � + �(7) 3.2

36. = �

37. = 2(2 + 3)(7 + 2) = 2 (2 + 3)(7 + 2) + 2 2(7 + 2) + 7 2(2 + 3)

38. = ( 2 3)(2 2 + 1) = ( 2 3)(2 2 + 1) + 2 2(2 2 + 1) + 4 2( 2 3)

39. = (5.3 1)(1 2.1)( 2.3 3.4) = 5.3(1 2.1)( 2.3 3.4) 2.1 1.1(5.3 1)( 2.3 3.4) 2.3 3.3(5.3 1)(1 2.1)

40. = (1.1 + 4)( 2.1 )(3.4 2.1) = 1.1( 2.1 )(3.4 2.1) + (1.1 + 4)(2.1 1.1 1)(3.4 2.1) + 2.1 3.1(1.1 + 4)( 2.1 )

41. = ( + 1)� +

1 � 2

1 1 1 2 = � � + 2 � + ( + 1)� 2 2 3 2 � 2 1 2 = �8 �� 2 � + (4 2 2

42. = (4 2

)�

)�

1

2

+

4 � 3

2 2 + 4 + 1 3 1 (4 + 4)(3 1) 3(2 2 + 4 + 1) 6 2 4 7 = = (3 1) 2 (3 1) 2

43. =

44. =

3 2 9 + 11 2 + 4


Solutions Section 11.3 (6 9)(2 + 4) 2(3 2 9 + 11) 6 2 + 24 58 = = (2 + 4) 2 (2 + 4) 2

2 4 + 1 2 + + 1 (2 4)( 2 + + 1) ( 2 4 + 1)(2 + 1) 5 2 5 = = ( 2 + + 1) 2 ( 2 + + 1) 2

45. =

2 + 9 1 2 + 2 1 (2 + 9)( 2 + 2 1) ( 2 + 9 1)(2 + 2) 7( 2 + 1) = = ( 2 + 2 1) 2 ( 2 + 2 1) 2

46. =

0.23 5.7 1 2.9 (0.23 0.77 5.7)(1 2.9) 2.9 3.9( 0.23 5.7 ) = (1 2.9) 2

47. =

8.43 0.1 0.5 1 3.2 + 2.9 ( 0.843 1.1 + 0.5 2)(3.2 + 2.9) 2.9 1.9(8.43 0.1 0.5 1) = (3.2 + 2.9) 2

48. =

+1 1 ½ 1 2( 1 2 1) ½ 1 2( 1 2 + 1) = = ( 1 2 1) 2

49. =

1 +1 ½ 1 2( 1 2 + 1) ½ 1 2( 1 2 1) = = ( 1 2 1) 2

50. =

51. =

� + 1

1 � 2 + 2

+ 1 � 2

First simplify the function:

1 . (Cancel the ( + 1).) (1 + ) 3 3 Therefore, = 4. =

52. =

�1

=

1 � 2

2 1 First simplify the function: =

2 1 � 2 = 2 1

1 . (Cancel the (1 2).) 2

1 ( 1) 2 1 ( + 1) 2


2 Therefore, = 3.

Solutions Section 11.3

( + 3)( + 1) 3 1 [( + 1) + ( + 3)](3 1) 3( + 3)( + 1) 3 2 2 13 = = (3 1) 2 (3 1) 2

53. =

( 5)( 4) ( 5)( 4) [( 4) + ( 5)] 2 + 20 = = [( 5)( 4)] 2 [( 5)( 4)] 2

54. =

( + 3)( + 1)( + 2) 3 1 [( + 1)( + 2) + ( + 3)( + 2) + ( + 3)( + 1)](3 1) 3( + 3)( + 1)( + 2) = (3 1) 2 6 3 + 15 2 12 29 = (3 1) 2

55. =

3 1 ( 5)( 4)( 1) 3( 5)( 4)( 1) (3 1)[( 4)( 1) + ( 5)( 1) + ( 5)( 4)] = ( 5)2( 4)2( 1) 2 3 2 6 + 33 20 31 = ( 5) 2( 4) 2( 1) 2

56. =

57.

58.

[( 2 + )( 2 )] = (2 + 1)( 2 ) + ( 2 + )(2 1) = 4 3 2

[( 2 + 3)( + 1)] = (2 + 3 2)( + 1) + ( 2 + 3) = 4 3 + 6 2 + 2

[( 3 + 2 )( 2 )] || = [(3 2 + 2)( 2 ) + ( 3 + 2 )(2 1)] || = 2 = 2 4 3 2 | = (5 4 + 6 4 )| = 64

59.

= 2

60.

61.

[( 2 + )( 2 )] || = [(2 + 1)( 2 ) + ( 2 + )(2 1)] || = (4 3 2 )|| = 2 = 1 = 1 = 1

2 [( 0.5)( 0.5 + 0.5)] || = [(2 0.5 0.5)( 0.5 + 0.5) + ( 2 0.5)(0.5 0.5 0.5 1.5)] || = 1 = 1 1.5 0.5 | = (2.5 + 1.5 1)| = 3 = 1

62.

2 [( + 0.5)( 0.5 0.5)] || = [(2 + 0.5 0.5)( 0.5 0.5) + ( 2 + 0.5)(0.5 0.5 + 0.5 1.5)] || = 1 = 1 1.5 0.5 | = (2.5 1.5 + 1)| = 2 = 1

63. The calculation thought experiment tells us that the expression for is a difference:


Solutions Section 11.3 4 4 2 [ ( + 120)(4 1)] = [ ] [( 2 + 120)(4 1)]. Now use the power rule for the first expression and the product rule for the second: = 4 3 [(2 )(4 1) + ( 2 + 120)(4)] = 4 3 12 2 + 2 480.

64. The calculation thought experiment tells us that the expression for is a difference: 4 2 + 120 4 2 + 120 [ ] � � �= �. 4 1 4 1 Now use the power rule for the first expression and the quotient rule for the second: (2 )(4 1) ( 2 + 120)(4) 4 2 2 480 3 4 . = 4 3 = (4 1) 2 (4 1) 2

65. The calculation thought experiment tells us that the expression for is a sum: [ ] + [1] + 2 � � + 1 + 2� �� = �. +1 + 1 Now use the power rule for the expressions on the left and the quotient rule for the one on the right: (1)( + 1) (1) 2 . = 1+0+2 = 1+ 2 ( + 1) ( + 1) 2 66. The calculation thought experiment tells us that the expression for is a difference: 1 1 [ + 2] 4 �( 2 )� + ��. �( + 2) 4( 2 )� + �� = Now use the power rule for the expression on the left and the product rule for the one on the right: 1 1 = 1 4�(2 1)� + � + ( 2 1)�1 2 ��.

67. Since the last operation we would do is multiply, the calculation thought experiment tells us that the given expression is a product. By the product rule: ( + 2) ( + 2)� . � = (1)� � + ( + 2) � �= � �+ +1 +1 + 1 +1 ( + 1) 2 68. Since the last operation we would do is divide, the calculation thought experiment tells us that the given expression is a quotient. By the quotient rule: [( + 2) ]( + 1) [( + 2) ](1) (2 + 2)( + 1) ( + 2) . = = 2 ( + 1) ( + 1) 2 69. The calculation thought experiment tells us that the expression for is a difference: [( + 1)( 2)] 2 � �( + 1)( 2) 2� �� = �. +1 + 1 Now use the product rule for the expressions on the left and the quotient rule for the one on the right: (1)( + 1) (1) 2 . = (1)( 2) + ( + 1)(1) 2 = 2 1 ( + 1) 2 ( + 1) 2 70. The calculation thought experiment tells us that the expression for is a sum: +2 +2 [( + 1)( 2)]. + ( + 1)( 2)� = � � �= + 1 + 1 Now use the quotient rule for the expressions on the left and the product rule for the one on the right: (1)( + 1) ( + 2)(1) 1 = + (1)( 2) + ( + 1)(1) = + 2 1. 2 ( + 1) ( + 1) 2

71. ( ) = 2 ( 3 + ) + ( 2 + 1)(3 2 + 1) = 5 4 + 6 2 + 1, so (1) = 12 is the slope. The tangent line


Solutions Section 11.3 passes through (1, (1)) = (1, 4), so its equation is = 12 8.

72. ( ) = 0.5 0.5( 2 + ) + ( 0.5 + 1)(2 + 1) = 2.5 1.5 + 2 + 1.5 0.5 + 1, so (1) = 7. The tangent line passes through (1, (1)) = (1, 4), so its equation is = 7 3. ( + 2) ( + 1) 1 , so (0) = 1 4. The tangent line passes through = 2 ( + 2) ( + 2) 2 (0, (0)) = (0, 1 2), so its equation is = 4 + 1 2. 73. ( ) =

1

1

( + 1) 1 2 , so (4) = 1 64. The tangent line passes = ( + 2) 2 2 ( + 2) 2 through (4, (4)) = (4, 3 4), so its equation is = 64 + 11 16. 74. ( ) = 2

( + 2)

2 ( ) ( 2 + 1) 2 1 , so ( 1) = 0. The tangent line passes through = 2 2 ( 1, ( 1)) = ( 1, 2), so its equation is = 2. 75. ( ) =

( 2 + 1) (2 ) 1 2 , so (1) = 0. The tangent line passes through = ( 2 + 1) 2 ( 2 + 1) 2 (1, (1)) = (1, 1 2), so its equation is = 1 2.

76. ( ) =

77. Rate of change of monthly sales = ! ( ) = 2,000 200 . When = 5 : ! (5) = 2,000 200(5) = 1,000 units/month. Therefore, sales are increasing at a rate of 1,000 units per month. Rate of change of price = ( ) = 2 . When = 5 : (5) = 2(5) = $10/month. Therefore, the price of a sound system is dropping at a rate of $10 per month. Revenue: ( ) = ( )!( ) = (1,000 2)(2,000 100 2) Rate of change of revenue: ( ) = ( )!( ) + ( )! ( ) = ( 2 )(2,000 100 2) + (1,000 2)(2,000 200 ) (5) = [ 2(5)][2,000(5) 100(5) 2] + [1,000 (5) 2][2,000 200(5)] = $900,000/month Therefore, revenue is increasing at a rate of $900,000 per month. 78. Rate of change of monthly sales = ! ( ) = 2,000 200 . When = 6 : ! (6) = 2, 000 200(6) = 800 units/month. Therefore, sales are increasing at a rate of 800 units per month. Rate of change of price = ( ) = 2 . When = 6 : (6) = 2(6) = $12/month. Therefore, the price of an iSun is dropping at a rate of $12 per month. Revenue: ( ) = ( )!( ) = (100 2)(2,000 100 2) Rate of change of revenue: ( ) = ( )!( ) + ( )! ( ) = ( 2 )(2,000 100 2) + (100 2)(2,000 200 ) (6) = [ 2(6)][2,000(6) 100(6) 2] + [100 (6) 2][2,000 200(6)] = $49,600/month Therefore, revenue is decreasing at a rate of $49,600 per month.

79. Daily revenue: ( ) = ( )"( ) million dollars In 2018, = 8, so (8) = (8)"(8) = [0.32(8) 2 8.2(8) + 100][0.59(8) + 76] 4,430 million dollars, or $4.43 billion. To obtain the rate of change of daily revenue, we compute ( ).


Solutions Section 11.3 By the product rule, ( ) = ( )"( ) + ( )" ( ) = (0.64 8.2)(0.59 + 76) + (0.32 2 8.2 + 100)(0.59). Therefore, (8) = (0.64(8) 8.2)(0.59(8) + 76) + (0.32(8) 2 8.2(8) + 100)(0.59) $216 million per year. Daily world oil revenue in 2018 was around $4.43 billion and decreasing at a rate of about $216 million per year. (Notice that is in years, so the rate of change of daily revenue is in millions of dollars per year.)

80. Daily revenue: ( ) = ( )"( ) million dollars In 2015, = 5, so (5) = (5)"(5) = [0.32(5) 2 8.2(5) + 100][ 0.074(5) + 33] 2,186 million dollars, or $2.186 billion. To obtain the rate of change of daily revenue, we compute ( ). By the product rule, ( ) = ( )"( ) + ( )" ( ) = (0.64 8.2)( 0.074 + 33) + (0.32 2 8.2 + 100)( 0.074). Therefore, (5) = (0.64(5) 8.2)( 0.074(5) + 33) + (0.32(5) 2 8.2(5) + 100)( 0.074) $168 million per year. OPEC's daily oil revenue in 2015 was around $2.186 billion and decreasing at a rate of about $168 million per year. (Notice that is in years, so the rate of change of daily revenue is in millions of dollars per year.) 81. Let ( ) be the number of T-shirts sold per day. If = 0 is now, we are told that (0) = 20 and (0) = 3. Let ( ) be the price of T-shirts. We are told that (0) = 7 and (0) = 1. The revenue is then ( ) = ( ) ( ), so (0) = (0) (0) + (0) (0) = 3(7) + 20(1) = 1. So, revenue is decreasing at a rate of $1 per day.

82. Let ( ) be the number of T-shirts sold per day. If = 0 is now, we are told that (0) = 20 and (0) = 3. Let ( ) be the price of T-shirts. We are told that (0) = 7. The revenue is ( ) = ( ) ( ) and we would like to have (0) = 10. This gives 10 = (0) = (0) (0) + (0) (0) = 3(7) + 20 (0), and so 20 (0) = 10 + 21 = 31, hence (0) = 31 20 = 1.55. So she should increase the price at a rate of $1.55 per shirt every day. 83. The cost per passenger is "( ) = ( ) ( ) = (10,000 + 2) (1,000 + 2). So, 2 (1,000 + 2) 2 (10,000 + 2) 18,000 " ( ) = , = 2 2 (1,000 + ) (1,000 + 2) 2 and so " (6) 0.10. The cost per passenger is decreasing at a rate of $0.10 per month.

84. The cost per passenger is "( ) = ( ) ( ) = (100 + 2) (1,000 + 2). So, 2 (1,000 + 2) 2 (100 + 2) 800 " ( ) = , = (1,000 + 2) 2 (1,000 + 2) 2 and so " (6) 0.0045. The cost per passenger is increasing at a rate of $0.0045 per month.

3,000(3,600 2 1) ; (10) 0.7670 mpg/mph. This means that, at a speed of 10 mph, ( + 3,600 1) 2 the fuel economy is increasing at a rate of 0.7670 miles per gallon per 1 mph increase in speed. (60) = 0 mpg/mph. This means that, at a speed of 60 mph, the fuel economy is neither increasing nor decreasing with increasing speed. (70) 0.0540. This means that, at 70 mph, the fuel economy is decreasing at a rate of 0.0540 miles per gallon per 1 mph increase in speed. 60 mph is the most fuel85. ( ) =


Solutions Section 11.3 efficient speed for the car (in the next chapter we shall discuss how to locate largest values in general).

4,000(3,025 2 1) ; (40) > 0, (55) = 0, and (60) < 0. This means that, at 40 ( + 3,025 1) 2 mph, fuel economy is increasing with increasing speed, while at 60 mph it is decreasing with increasing speed. The best fuel economy occurs at 55 mph. (In the next chapter we shall discuss how to locate largest values in general.) 86. ( ) =

87. a. (* +)( ) = *( ) +( ) = DoorDash percentage − Uber Eats percentage, and therefore represents the percentage of the (food app) market by which DoorDash exceeds Uber Eats at time . *( ) * , + -( ) = +( ) = (DoorDash market share)/(Uber Eats market share); DoorDash's market share as a multiple of that of Uber Eats. *(1) * 44 b. , -(1) = = = 1.6; + +(1) 27.5 By the quotient rule (or by the formula in Exercise 101) * . *( ) 1 (13)( 2.5 + 30) (13 + 31)( 2.5) , + - ( ) = 0 +( ) 3 = ( 2.5 + 30) 2 / 2 (13)(30) (31)( 2.5) 467.5 . = = 2 ( 2.5 + 30) ( 2.5 + 30) 2 * . *( ) 1 | 467.5 0.62 per year. = , + - (1) = 0 +( ) 3 ||| ( 2.5 + 30) 2 / 2 = 1 At the start of 2020 ( = 1) Doordash's market share was around 1.6 times that of Uber Eats, and that ratio was increasing at a rate of around 0.62 per year. 88. a. (+ 4)( ) = +( ) 4( ) = Uber Eats percentage − Grubhub percentage, and therefore represents the percentage of the (food app) market by which Uber Eats exceeds Grubhub at time . +( ) + , 4 -( ) = 4( ) = (Uber Eats market share)/(Grubhub market share); Uber Eats's market share as a multiple of that of Grubhub. +(1) 27.5 + b. , -(1) = 1.5; = 4 4(1) 18 By the quotient rule (or by the formula in Exercise 101) + . +( ) 1 ( 2.5)( 2 + 20) ( 2.5 + 30)( 2) , 4 - ( ) = 0/ 4( ) 32 = ( 2 + 20) 2) ( 2.5)(20) (30)( 2) 10 . = = ( 2 + 20) 2 ( 2 + 20) 2 + . +( ) 1 | 10 0.031 per year. = , 4 - (1) = 0/ 4( ) 32 ||| ( 2 + 20) 2 = 1 At the start of 2020 ( = 1) Uber Eats' market share was around 1.5 times that of Grubhub, and that ratio was increasing at a rate of around 0.031 per year. 89. Cost: 1,250 + 160,000 ( since 2014) Personnel: 10,000 + 1,360,000 Rate of change at = 2 is (1,250)( 10,000(2) + 1,360,000) + (1,250(2) + 160,000)( 10,000) = 50,000,000. Total military personnel costs were projected to increase at a rate of about $50 million per year in 2016. 90. Cost: 2,000 + 70,000 Personnel: 0.05 + 2 ( since 1990). Rate of change at = 5 is


Solutions Section 11.3 (2,000)( 0.05(5) + 2) + (2,000(5) + 70,000)( 0.05) = 500. Total military personnel costs were decreasing at a rate of about $500 million per year in 1995.

5.625 , so (4) = 2.5 thousand organisms per hour per 1,000 organisms. This (1 + 0.125 ) 2 means that the reproduction rate of organisms in a culture containing 4,000 organisms is declining at a rate of 2,500 organisms per hour per 1,000 additional organisms. 91. ( ) =

0.08 , so (2) = 0.0408 thousand organisms per hour, per 1,000 organisms. This (1 + 0.2 ) 2 means that the reproduction rate of organisms in a culture containing 2,000 organisms is declining at a rate of 40.8 organisms per hour, per 1,000 additional organisms. 92. ( ) =

93. Let ( ) be the number of eggs; then ( ) = 30 . The total oxygen consumption is ( ) ( ) and its rate of change is ( ) ( ) + ( ) ( ) = ( 1)( 0.016 4 + 1.1 3 11 2 + 3.6 ) + (30 )( 0.064 3 + 3.3 2 22 + 3.6). At = 25 this is approximately 1,572 1,600. Thus, oxygen consumption is decreasing at a rate of about 1,600 milliliters per day. This must be because the number of eggs is decreasing, because (25) is positive. 94. Let ( ) be the number of eggs; then ( ) = 100 2 . The total oxygen consumption is ( ) ( ) and its rate of change is ( ) ( ) + ( ) ( ) = 2( 0.0071 4 + 0.95 3 22 2 + 95 ) + (100 2 )( 0.0284 3 + 2.85 2 44 + 95). At = 40 this is approximately 900. Oxygen consumption is decreasing at a rate of 900 milliliters per day. This must be due to the fact that the number of eggs is decreasing, because (40) is positive. 95. We are given that (3) = 5, (3) = 2, (3) = 4, (3) = 5. Thus, has the value (3) (3) = (5)(4) = 20. By the product rule, ( ) (3) = (3) (3) + (3) (3) = (2)(4) + (5)(5) = 33 units per second equals 20 and is rising at a rate of 33 units per second. 96. We are given that (2) = 3, (2) = 4, (2) = 5, (2) = 6. Thus, has the value (2) (2) = (3)(5) = 15. By the product rule, ( ) (2) = (2) (2) + (2) (2) = (4)(5) + (3)(6) = 38 units per second equals 15 and is rising at a rate of 38 units per second.

97. We are given that (3) = 5, (3) = 2, (3) = 4, (3) = 5. Thus, has the value (3) (3) = 5 4. By the quotient rule, (3) (3) (3) (3) (2)(4) (5)(5) 17 ( ) (3) = = = units per second. 16 (3) 2 42 equals 5/4 and is changing at a rate of −17/16 units per second. 98. We are given that (2) = 3, (2) = 4, (2) = 5, (2) = 6. Thus, has the value (2) (2) = 3 5. By the quotient rule,


Solutions Section 11.3 (2) (2) (2) (2) (4)(5) (3)(6) 2 ( ) (2) = units per second = = 2 25 (2) 52 equals 3/5 and is rising at a rate of 2/25 units per second. 99. The analysis is suspect, because it seems to be asserting that the annual increase in revenue, which we can think of as , is the product of the annual increases, in price, and ! in sales. However, because = !, the product rule implies that is not the product of and ! but is instead ! !+ . = 100. The analysis is wrong because it suggests that the derivative of the quotient, ( ) = ( ) , is the quotient, ( ) 1, of the derivatives. However, the quotient rule tells us that ( ) ( ( ) ( 1 ( ) ( ) ( ) ( ) ( ) = . = 2 = 2 101. By the quotient rule,

( + ) ( + )( ) + = , + ( + ) 2 = ( + ) 2

102. By the quotient rule,

2 ( 2 + ) ( 2 + )(2 ) 2 + = 5 2 + 6 ( 2 + ) 2 2 ( ) = ( 2 + ) 2

103. Answers will vary; ! = + 1,000 is one example: ( ) = ( ) = 2 + 1,000 and (100) = 800 > 0.

! = 2 + 1,000 , so

104. ( ) = !( ), so ( ) = !( ) + ! ( ). We want (100) < 0, so we must have !(100) + 100! (100) < 0, or !(100) < 100! (100). 105. Mine; it is increasing twice as fast as yours. The rate of change of revenue is given by ( ) = ( )!( ) because ! ( ) = 0 for both of us. Thus, in this case, ( ) does not depend on the selling price ( ). 106. Mine. The rate of change of revenue is given by ( ) = ( )! ( ) because Thus, in this case, ( ) does not depend on the quantity sold !( ).

( ) = 0 for both of us.

107. (A): If the marginal product was greater than the average product, it would force the average product to increase. (See the formula for the rate of change of the average given in the solution to Exercise 100.) 108. (C): If the marginal cost was less than the average cost, it would force the average cost to decrease. (See the formula for the rate of change of average cost given in the solution to Exercise 100.)


Solutions Section 11.4 Section 11.4

1. ( ) = (2 + 1) 2, so ( ) = 2(2 + 1)(2) = 4(2 + 1) 2. ( ) = (3 1) 2, so ( ) = 2(3 1)(3) = 6(3 1)

3. ( ) = ( 1) 1, so ( ) = ( 1) 2(1) = ( 1) 2

4. ( ) = (2 1) 2, so ( ) = 2(2 1) 3(2) = 4(2 1) 3 5. ( ) = (2 ) 2, so ( ) = 2(2 ) 3( 1) = 2(2 ) 3 6. ( ) = (1 ) 1, so ( ) = (1 ) 2( 1) = (1 ) 2

7. ( ) = (2 + 1) 0.5, so ( ) = 0.5(2 + 1) 0.5(2) = (2 + 1) 0.5

8. ( ) = ( + 2) 1.5, so ( ) = 1, 5( + 2) 0.5( 1) = 1.5( + 2) 0.5 1 , so 3 1 ( ) = (3 1) 2(3) = 3 (3 1) 2 9. ( ) =

1 , so ( + 1) 2 ( ) = 2( + 1) 3(1) = 2 ( + 1) 3 10. ( ) =

11. ( ) = ( 2 + 2 ) 4, so ( ) = 4( 2 + 2 ) 3. ( 2 + 2 ) = 4( 2 + 2 ) 3(2 + 2) 12. ( ) = ( 3 ) 3, so ( ) = 3( 3 ) 2 ( 3 ) = 3( 3 ) 2(3 2 1)

13. ( ) = (2 2 2) 1, so ( ) = (2 2 2) 2 (2 2 2) = 4 (2 2 2) 2

14. ( ) = (2 3 + ) 2, so ( ) = 2(2 3 + ) 3 (2 3 + ) = 2(2 3 + ) 3(6 2 + 1)


Solutions Section 11.4 15. ( ) = ( 2 3 1) 5, so ( ) = 5( 2 3 1) 6 ( 2 3 1) = 5(2 3)( 2 3 1) 6 16. ( ) = (2 2 + + 1) 3, so ( ) = 3(2 2 + + 1) 4 (2 2 + + 1) = 3(4 + 1)(2 2 + + 1) 4 17. ( ) =

1

( 2 + 1) 3

, so

( ) = 3( 2 + 1) 4

2 ( + 1) = 6 ( 2 + 1) 4

1 , so ( 2 + + 1) 2 ( ) = 2( 2 + + 1) 3 ( 2 + + 1) = 2(2 + 1) ( 2 + + 1) 3 18. ( ) =

19. ( ) = (0.1 2 4.2 + 9.5) 1.5, so ( ) = 1.5(0.1 2 4.2 + 9.5) 0.5 (0.1 2 4.2 + 9.5) = 1.5(0.2 4.2)(0.1 2 4.2 + 9.5) 0.5 20. ( ) = (0.1 4.2 1) 0.5, so ( ) = 0.5(0.1 4.2 1) 0.5 (0.1 4.2 1) = 0.5(0.1 + 4.2 2)(0.1 4.2 1) 0.5 21. ( ) = ( 2

0.5 4

) , so ( ) = 4( 2 0.5) 3 ( 2

22. ( ) = (2 +

0.5 1

) , so ( ) = (2 + 0.5) 2 (2 +

23.

24.

1 2 =

+ 2 =

) = 4(2 0.5 0.5)( 2

0.5

) = (2 + 0.5 0.5)(2 +

0.5

)

0.5 2

1 (1 2) 1 2 = (1 2) 1 2( 2 ) = 2 1 2

1 1 + 2 ( + 2) 1 2 = ( + 2) 1 2(1 + 2 ) = 2 2 + 2

|)| ) |)| = , so ) |3 6| 3|3 6| |3 6| = (3 6) = 3 6 3 6

25.

|)| ) |)| = , so ) | 5 + 1| 5| 5 + 1| | 5 + 1| = ( 5 + 1) = 5 + 1 5 + 1

26.

27.

)

0.5 3

|)| ) |)| = , so )


Solutions Section 11.4 3 | 5 | ( 3 2 + 5)| 3 + 5 | + 3 | 3 + 5 | = ( 5 ) + = 3 + 5 3 + 5 |)| ) |)| = , so ) | 4| (1 4 3)| 4| 4 | 4| = ( ) = 4 4

28.

1 2[( + 1)( 2 1)] 1 2 = 2� �[( + 1)( 2 1)] 3 2. [( + 1)( 2 1)] 2 = [( + 1)( 2 1)] 3 2[(1)( 2 1) + ( + 1)(2 )] = [( + 1)( 2 1)] 3 2(3 1)( + 1) 29.

1 3[(2 1)( 1)] 1 3 = 3[(2 1)( 1)] 4 3. [(2 1)( 1)] 3 = [(2 1)( 1)] 4 3[(2)( 1) + (2 1)(1)] = [(2 1)( 1)] 4 3(4 3) 30.

31. ( ) = (3.1 2) 2 (3.1 2) 2, so

( ) = 2(3.1 2) 1(3.1) ( 2)(3.1 2) 3(3.1) = 6.2(3.1 2) + 2 1 � , so 3.1 2 1 1 ( ) = 2�3.1 2 2 �. �3.1 2 2 � 3.1 2 3.1 2 1 3.1 = 2�3.1 2 2 ��6.2 + � 3.1 2 (3.1 2) 2

32. ( ) = �3.1 2 2

6.2 (3.1 2) 3

33. ( ) = [(6.4 1) 2 + (5.4 2) 3] 2, so ( ) = 2[(6.4 1) 2 + (5.4 2) 3]. [(6.4 1) 2 + (5.4 2) 3] = 2[(6.4 1) 2 + (5.4 2) 3][12.8(6.4 1) + 16.2(5.4 2) 2] 34. ( ) = (6.4 3) 2 + (4.3 1) 2, so ( ) = 12.8(6.4 3) 3 8.6(4.3 1) 3

[( 2 3 ) 2(1 2) 0.5] = [( 2 3 ) 2] ( (1 2) 0.5 + ( 2 3 ) 2 [(1 2) 0.5] = 2( 2 3 ) 3(2 3)(1 2) 0.5 ( 2 3 ) 2(1 2) 0.5 35.

[(3 2 + )(1 2) 0.5] = [3 2 + ] ( (1 2) 0.5 + (3 2 + ) [(1 2) 0.5] = (6 + 1)(1 2) 0.5 (3 2 + )(1 2) 0.5 36.

2 + 4 2 2 + 4 2 + 4 � � = 2� � � � 3 1 3 1 3 1 4( + 2) 2(3 1) (2 + 4)(3) 56( + 2) = � = �( 3 1 (3 1) 2 (3 1) 3 37.

38.

3 9 3 3 9 2 3 9 � � = 3� � � � 2 + 4 2 + 4 2 + 4


Solutions Section 11.4 27( 3) 2 3(2 + 4) (3 9)(2) 810( 3) 2 = � = �( (2 + 4) 2 (2 + 4) 2 (2 + 4) 4 3 2 7 7 7 � � = 3� � � � 7 1 + 7 2 1 + 7 2 7 1 + 7 2 (1 + 7 2) 7(27) 37 2(1 7 2) 37 2 ( = � = � (1 + 7 2) 2 (1 + 7 2) 2 (1 + 7 2) 4

39.

72 1 72 72 � � = 2� � � � 7 1 + 7 1 + 7 7 1 + 7 27(1 + 7) 7 2(1) 27 3(7 + 2) 27 2 = � = �( 1+7 (1 + 7) 2 (1 + 7) 3 40.

[(1 + 2 ) 4 (1 ) 2] 3 = 3[(1 + 2 ) 4 (1 ) 2] 2 [(1 + 2 ) 4 (1 ) 2] = 3[(1 + 2 ) 4 (1 ) 2] 2[8(1 + 2 ) 3 + 2(1 )] 41.

[(3 1) 2 + (1 ) 5] 2 = 2[(3 1) 2 + (1 ) 5] [(3 1) 2 + (1 ) 5] = 2[(3 1) 2 + (1 ) 5].[6(3 1) 5(1 ) 4] 42.

43.

44.

3|3 1| [(3 1)|3 1|] = 3|3 1| + (3 1) = 3|3 1| + 3|3 1| = 6|3 1| 3 1

|( 3) 1 3| |( 3) 1 3| |( 3) 1 3| |( 3) 1 3| = [( 3) 1 3] = ( 3) 2 3 = 3( 3) ( 3) 1 3 3( 3) 1 3

| (2 3) 1 2| | (2 3) 1 2| = [ (2 3) 1 2] (2 3) 1 2 | (2 3) 1 2| [1 (2 3) 1 2] = 1 2 (2 3) 45.

46.

6(3 |3 1|) 3|3 1| (3 |3 1|) 2 = 2(3 |3 1|) 3 (3 |3 1|) = 3 1

( 2 + 1 2) 1 = ( 2 + 1 2) 2 [(2 + 1) 1 2 2] 1 �2 � 2 + 1 2 2 1 1 2 (2) 2 � = = ( 2 + 1 ) � (2 + 1) 2 ( 2 + 1 2) 2 47.

48.

( +1+

= 3( + 1 +

[ + 1 + ] 1 1 3 ) 2� ( + 1) 1 2 + 1 2� = ( + 1 + ) 2� 2 2 2 ) 3 = 3( + 1 +

) 2

1 1 + � +1

3 �1 + (1 + (1 + 2 ) 3) 3� = 3{1 + [1 + (1 + 2 ) 3] 3} 2 {1 + [1 + (1 + 2 ) 3] 3} 3 3 2 3 2 [1 + (1 + 2 ) 3] = 9{1 + [1 + (1 + 2 ) ] } [1 + (1 + 2 ) ]

49.


Solutions Section 11.4 = 27{1 + [1 + (1 + 2 ) 3] 3} 2[1 + (1 + 2 ) 3] 2(1 + 2 ) 2(2) = 54(1 + 2 ) 2[1 + (1 + 2 ) 3] 2(1 + [1 + (1 + 2 ) 3] 3) 2

�2 + (2 + (2 + 1) 3) 3� = 2 + 3[2 + (2 + 1) 3] 2 [2 + (2 + 1) 3] = 2 + 3[2 + (2 + 1) 3] 2[2 + 6(2 + 1) 2] 50.

51.

52.

53.

54.

55.

56.

57.

58.

59.

60.

61.

62.

63.

64.

65.

) = = 2) ( 1 = 2( + 2) )

) = = 3) 2 ( 1 = 3( 1) 2 )

= = (2 + 1)(2) = 4(2 1) + 2

= = (3 2 1)( 4) = 12(1 4 ) 2 + 4

1 1 = = 3 1 1 = = 8

1 1 1 = = = 2 2

1 1 3 = = = = 3 2 2 3 3 3 ( ) 2 3 5 = = 3

4 = = = 8 1 2 6 = = = 2

3

2 2 = = = 32 3 2 3

1 1 | 1 1 , and so | = = = = | 6 2 = 1 6(1) 2 4

1 1 | 1 2 , and so || = = = = 3 + 2 2 | = 2 3 + 2 2 2 7 | 3(1) 2 32 3 , and so | = = = = 2 +2 | = 1 2(1) + 2 4


Solutions Section 11.4 | 2(2) 2 4 66. , and so || = = = = 2 2 = 2 6(2) + 1 25 6 +1 67.

68.

69.

70.

71.

72.

73.

74.

= 100 99 99 2 = (100 99 99 2)

= 0.5 0.5 + 1.5 0.5 = (0.5 0.5 + 1.5 0.5) = 3 4 + 0.5 0.5 = ( 3 4 + 0.5 0.5) =

2

8 = 4 2

= (1

2

)

9 = 8

| = 3 2 2 , so || = 3(2) 2( 1) 2 2( 1) = 47 4 | = 1

1 1 2 1 3 2 1 1 2 1 ( 1) 9 3 2( 1) = 4 27 = = 9 2 2 2 2

75. $( ) = 2.7( 2010) 3 + 44( 2010) 2 84( 2010) + 95, so $ ( ) = 8.1( 2010) 2 + 88( 2010) 84, using that ( 2010) = 1.

$ (2015) = 8.1(2015 2010) 2 + 88(2015 2010) 84 = 153.5 games per year. The number of games in the App Store was increasing at a rate of 153.5 games per year in 2015.

76. $( ) = 2.3( 2010) 3 + 26( 2010) 2 + 6( 2010) + 20, so $ ( ) = 6.9( 2010) 2 + 52( 2010) + 6, using that ( 2010) = 1.

$ (2018) = 6.9(2018 2010) 2 + 52(2018 2010) + 6 = 19.6 games per year. The number of games in the Play Store was decreasing at a rate of 19.6 games per year in 2018.

77. = 5 + 2 + 10. Your current profit is (50) = 5(50) + 110 $260.49. The marginal profit is 1 1 1 ( ) = 5 + (2 + 10) 1 2 ( 2 = 5 + , so (50) = 5 + 5.10. 2 2 + 10 110 The derivative is measured in dollars per additional magazine sold. Thus, your current profit is $260.49 per month, and this would increase at a rate of $5.10 per additional magazine in sales. 3 . 2 3 + 100 Thus, when = 100, (100) = $180, and ( ) 1.93. Your current profit is $180 per month, and this would increase at a rate of $1.93 per new subscriber. 78. ( ) = 2

3 + 100, so ( ) = 2


Solutions Section 11.4 79. Write ( ) = 3,000( + 3,600 1) 1, to find 3,000(1 3,600 2) ( ) = 3,000( + 3,600 1) 2(1 3,600 2) = ( + 3,600 1) 2 (10) 0.7670 mpg/mph. This means that, at a speed of 10 mph, the fuel economy is increasing at a rate of 0.7670 miles per gallon per 1 mph increase in speed. (60) = 0 mpg/mph. This means that, at a speed of 60 mph, the fuel economy is neither increasing nor decreasing with increasing speed. (70) 0.0540. This means that, at 70 mph, the fuel economy is decreasing at a rate of 0.0540 miles per gallon per 1 mph increase in speed. 60 mph is the most fuel-efficient speed for the car. (In the next chapter we shall discuss how to locate largest values in general.) 80. Write ( ) = 4,000( + 3,025 1) 1, to find

4,000(1 3,025 2) ( + 3,025 1) 2 (40) > 0, (55) = 0, and (60) < 0. This means that, at 40 mph, fuel economy is increasing with increasing speed, while at 60 mph it is decreasing with increasing speed. The best fuel economy occurs at 55 mph. (In the next chapter we shall discuss how to locate largest values in general.) ( ) = 4,000( + 3,025 1) 2(1 3,025 2) =

" = 5000 0.5" and = 30 + 0.02 " When = 10, " = 30(10) + 0.01(10) 2 = 301. | " Hence, = = (5, 000 0.5(301))(30 + 0.02(10)) = 146, 454.9. | || = 10 " At an employment level of 10 engineers, Paramount will increase its annual profit at a rate of $146,454.90 per additional engineer hired. 81.

82. (") = 100, 000 ! + 5, 000 0.25", so that

= 100, 000 " 2 0.25 "

" = = (100, 000 301 2 0.25)(30 + 0.02(10)) = 25.78 " when = 10 : The marginal average profit at an employee level of 10 engineers is $25.78 per engineer, so that the average profit per computer will increase by $25.78 for each additional engineer hired. 83. To express as a function of , take the given equation for : = 35 0.25 and substitute for x using = 7 + 0.2 : = 35(7 + 0.2 ) 0.25. The rate of change of spending on food is given by . By the chain rule, = 35( 0.25)(7 + 0.2 ) 1.25 (7 + 0.2 ) = 8.75(7 + 0.2 ) 1.25(0.2) = 1.75(7 + 0.2 ) 1.25 percentage points per month. (The units of are units of are unit of ; that is, percentage points per month.) January 1 is represented by = 0, and so November 1 is given by = 10 : | = 1.75(7 + 0.2(10) 1.25 0.11 percentage points per month. | | = 10

84. To express as a function of , take the given equation for : = 33 0.63 and substitute for using = 3.5 + 0.1 : = 33(3.5 + 0.1 ) 0.63. The rate of change of spending on food is given by . By the chain rule, = 20.79(3.5 + 0.1 ) 1.63 (3.5 + 0.1 ) = 2.079(3.5 + 0.1 ) 1.63 percentage points per month.


Solutions Section 11.4 (The units of are units of y are unit of t; that is, percentage points per month.) January 1 is represented by = 0, and so November 1 is given by = 10 : | = 2.079(3.5 + 0.1(10) 1.63 .18 percentage points per month. | | = 10 85. We are told that = $40 per $1 increase in the price = $0.75 per additional ruby sold ! Marginal revenue = = = (40)( 0.75) = $30 per additional ruby sold. ! ! Interpretation: The revenue is decreasing at a rate of $30 per additional ruby sold. 86. We are told that = €500 per €1 increase in the price = €0.45 per additional emerald sold ! Marginal revenue = = = ( 500)( 0.45) = €225 per additional emerald sold. ! ! Interpretation: The revenue is increasing at a rate of €225 per additional emerald sold.

87. We are given = 3 and we need to find . From the linear relation we have = = (1.5)( 2) = 3. Hence, = 3 murders per 100,000 residents per year each year.

88. We are given = 22 and = 2.5 in 1996. From the model we have = = (0.2 3) , so in 1996, = (0.2(22) 3) ( ( 2.5) = 3.5 murders per 100,000 residents per year each year. 89. Using the Quick Example in the text: 3 11 . = = 2 10 Evaluating at = 2 gives 3(2) 11 5 | 0.833. = = | || = 2 2(2) 10 6 Interpretation: Note that units of are percentage points of home sales per percentage points of price, relative to the 2003 levels. So we can say that, relative to the 2003 levels, home sales in 2008 ( = 2) were changing at a rate of about 0.833 percentage points per percentage point change in price. Equivalently, home sales were dropping at a rate of about 0.833 percentage points per percentage point drop in price. 90. Using the Quick Example in the text: 8 + 4 . = = 12 + 27 Evaluating at = 2 gives


Solutions Section 11.4 8(2) + 4 | 12 = = = 4. | | = 2 12(2) + 27 3 Interpretation: Note that units of are percentage points of home sales per percentage points of price, relative to the 2003 levels. So we can say that, relative to the 2003 levels, home sales in 2006 ( = 2) were changing at a rate of 4 percentage points per percentage point change in price. (Equivalently, home sales were dropping at a rate of 4 percentage points per percentage point increase in price.) 91.

92.

9 = 2 = 2 (3)(2) = 12 mi2/hour 9 2 (4)(0.3) = 2.4 cm2/day = 2

93. First calculate how fast the volume is growing: 8 = 4 2 = 4 (10) 2(0.5) = 200 ft 3/week. Now calculate how fast the cost is growing: 8 = 10008 , so = 1000 = $200, 000 /week $628, 000/week. 94.

= 8 = 8 (10)(4) = 320 cm2/sec

95. a. ! (4) (!(4 + 0.0001) !(4 0.0001)) 0.0002 333 units per month b. = 800!, so ! = $800/unit: The marginal revenue is the selling price. ! c. 800(333) $267, 000 per month = !

96. a. ! (2) (!(2 + 0.0001) !(2 0.0001)) 0.0002 1.652 billion packets per year b. (!) = $5 per million packets ! c. 5(1,652) = $8,260 per year. = ! (Note that we write ! as 1,652 million packets per year so that the units agree.) 97. Keeping and fixed, , = 1.2 0.4 0.3 1.2 0.4 0.3 so . = = 0.6 0.6 0.3 2 We are given = 5% per year, so = 0.6(5) = 3% per year.

98. Keeping and fixed, , = 0.6 0.6 1.3 0.6 0.6 1.3 so . = = 0.3 0.6 0.3 2 We are given = 5% per year, so = 0.3(5) = 1.5% per year. 99. Keeping fixed,


Solutions Section 11.4 0.6 0.3 0.4 0.3 , = 1.2 + 2 so = 0.6 + = 0.6(5) + 5 = 8% per year.

= 1.2 0.4 0.3 . 0.6 0.6 1.3 . + 2 0.6 0.3. , so = 0.6 0.3 + = 0.6(5) 0.3(2) + ( 3) = 0.6% per year.

100.

101. The derivative of one over a glob is 1 over the glob squared, times the derivative of the glob. 102. The derivative of the square root of a glob is 1 over twice the square root of the glob, times the derivative of the glob. 103. The derivative of a quantity cubed is three times the original quantity squared times the derivative of the quantity, not three times the derivative of the quantity squared. Thus, the correct answer is 3(3 3 ) 2(9 2 1). 104. The derivative of a quantity cubed is three times the original quantity squared times the derivative of 3 2 1 2 6 (2 2) (3 2 1)(2) the quantity. So, the correct result (before simplifying) is 3� � � �. 2 2 (2 2) 2 105. First, the derivative of a quantity cubed is three times the original quantity squared times the derivative of the quantity; not three times the derivative of the quantity squared. Second, the derivative of a quotient is not the quotient of the derivatives; the quotient rule needs to be used in calculating the 3 2 1 derivative of . Thus, the correct result (before simplifying) is 2 2 3 2 1 2 6 (2 2) (3 2 1)(2) 3� � � �. 2 2 (2 2) 2 106. First, the derivative of a quantity to the fourth power is four times the (original) quantity cubed, times the derivative of the quantity; not four times the derivative of the quantity cubed. Second, the product rule needs to be used in calculating the derivative of (3 3 )(2 + 1). Thus, the correct result (before simplifying) is 4[(3 3 )(2 + 1)] 3[(9 2 1)(2 + 1) + (3 3 )(2)]. 107. Following the calculation thought experiment, pretend that you were evaluating the function at a specific value of . If the last operation you would perform is addition or subtraction, look at each summand separately. If the last operation is multiplication, use the product rule first; if it is division, use the quotient rule first; if it is any other operation (such as raising a quantity to a power or taking a radical of a quantity), then use the chain rule first. 108. An example is ( ) = �1 +

(2 + 1) 2 3 � . 2

109. An example is ( ) =

+ : + ;

<

+

+

+ 1.

110. Composites of linear functions are linear. The slope of the composite is the product of the slopes of the individual lines.


Solutions Section 11.5 Section 11.5 1.

2.

3.

4.

1 1 1 [ln( 1)] = ( 1) = (1= 1 1 1 1 1 1 [ln( + 3)] = ( + 3) = (1= + 3 +3 +3

1 2 1 2 [ln( 2 + 3)] = 2 ( + 3) = 2 ( 2 = 2 + 3 +3 +3 1 1 2 ln |2 4| = (2 4) = (2= 2 4 2 4 2 4

5. 1 ( ln 2) (from the formula in the textbook) 6. 1 ( ln 3) (from the formula in the textbook) 7.

8.

9.

1 1 log2 ( + 1) = ( + 1) = ( + 1) ln 2 ( + 1) ln 2

1 2 2 + 1 log3 ( 2 + ) = 2 ( + ) = 2 + ) ln 3 ( + ) ln 3 log3 ( + 1 ) =

10. log3 ( + 11.

12.

13.

14.

)=

1

( +1 )= + 1 ) ln 3

( +

1

[ + ) ln 3

]=

+ 3 = = = + 3 ( + 3) = = + 3(1) = = + 3

1

+ 1 ) ln 3 1

( +

2

1

(1 1

) ln 3

2 2 2 2 = = = ( 2) = = (2 ) = 2 = 2 2 + 1 = = (2 1)= + 1

2 2 2 + 1 = = (4 1 1 2)= 2 + 1

2 1 2 2 1 (= ) = 2= 2 1 = = 2= 2 1(2= 2 1) = 4(= 2 1) 2 Alternatively, simplify first to get: 2 1 2 4 2 (= ) = = = 4= 4 2 or 4(= 2 1) 2 15.

2 2 2 2 2 3 (= ) = 3(= 2 ) 2(4 = 2 ) = 12 (= 2 ) 3 Alternatively, simplify first to get:

16.

)=

2

1 1 2 ( + 1 ) ln 3


Solutions Section 11.5 2 2 2 3 6 2 6 2 (= ) = = or 12 (= 2 ) 3 = 12 =

17. 4 ln 4 (from the formula in the textbook) 18. 5 ln 5 (from the formula in the textbook) 19.

20.

21.

22.

23.

24.

25.

26.

27.

28.

2 2 2 1 2 = 2 1 ln 2 ( 2 1) = 2 12 ln 2

2 2 2 3 = 3 ln 3 ( 2 ) = 3 (2 1) ln 3

1 2 + 1 [( 2 + 1) ln ] = 2 ln + ( 2 + 1)� � = 2 ln +

1 4 2 [4 2 ) ln ] = (8 1) ln + (4 2 )� � = (8 1) ln +

( 2 + 1) 5 1 [( 2 + 1) 5 ln ] = 5( 2 + 1) 4(2 ) ln + ( 2 + 1) 5� � = 10 ( 2 + 1) 4 ln +

( + 1) 0.5 1 [( + 1) 0.5 ln ] = 0.5( + 1) 0.5 ln + ( + 1) 0.5� � = 0.5( + 1) 0.5 ln + 1 1 4 ln |2 2 + 1| = (2 2 + 1) = ( 4 = 2 2 + 1 2 2 + 1 2 2 + 1 1 2 1 2 1 ln | 2 | = 2 ( ) = 2 (2 1) = 2

1 2 1 ln( 2 2.1 0.3) = 2 ( 2.1 0.3) = 2 (2 0.63 0.7) 2.1 0.3 2.1 0.3 2 0.63 0.7 = 2 2.1 0.3

1 1 1 + 3.1 2 ln( 3.1 1) = ( 3.1 1) = (1 + 3.1 2) = 3.1 1 3.1 1 3.1 1

( ) = ln[( 2 + 1)( + 1)]] = ln( 2 + 1) + ln( + 1), and so ( ) =

1 2 . + 2 + 1 + 1

( ) = ln[(3 + 1)( + 1)] = ln(3 + 1) + ln( + 1), and so ( ) =

3 1 . 3 + 1 + 1

29. First simplify the function as in Example 1(b):

30. First simplify the function as in Example 1(b):

31. First simplify the function as in Example 1(b): 3 + 1 3 4 ( ) = ln� . � = ln(3 + 1) ln(4 2), and so ( ) = 4 2 3 + 1 4 2 32. First simplify the function as in Example 1(b):


Solutions Section 11.5 9 1 4 ( ) = ln� . � = ln(9 ) ln(4 2), and so ( ) = 4 2 4 2 33. First simplify the function as in Example 1(b): ( + 1)( 3) | ( ) = ln|| = ln | + 1| + ln | 3| ln | 2 9|, and so 2 9 | 1 1 1 1 2 2 ( ) = . + = + + 1 3 2 9 + 1 3 2 + 9 34. First simplify the function as in Example 1(b): ( + 1) | ( ) = ln|| = ln |( + 1| ln |3 4| ln | 9|, and so (3 4)( 9) | 3 1 1 ( ) = . + 1 3 4 9 ( ) = ln[(4 2) 1.3] = 1.3 ln(4 2), and so

( ) = 1.3

35. First simplify the function as in Example 1(a):

( ) = ln[( 8) 2] = 2 ln( 8), and so

( ) =

36. First simplify the function as in Example 1(a):

4 5.2 . = 4 2 4 2

2 . 8

37. First simplify the function using the rules for logarithms: ( + 1) 2 | = ln[|( + 1| 2] ln[|3 4| 3] ln | 9| ( ) = ln|| | (3 4) 3( 9) 2 9 1 . = 2 ln |( + 1| 3 ln |3 4| ln | 9|, and so ( ) = + 1 3 4 9 38. First simplify the function using the rules for logarithms: ( + 1) 2( 3) 4 | 2 4 ( ) = ln|| | = ln[|( + 1| ] + ln[| 3| ] ln |2 + 9| 2 + 9 2 4 2 . = 2 ln |( + 1| + 4 ln | 3| ln |2 + 9|, and so ( ) = + + 1 3 2 + 9 39.

40.

41.

42.

43.

44.

2 ln | | (ln | |) 2 = 2 ln | | [ln | |] =

1 1 1 � � = (ln | |) 2� � = ln | | (ln | |) 2

2 1 2 2 ln( 1) [ln( 2) [ln( 1)] 2] = 2[ln( 1)] = 1 1

4 ln( 2) 8 ln 2 (ln( 2)) 2 = 2 ln( 2) ln( 2) = 2 ln( 2) (2 ln ) = 2 ln( 2)� � = = ( = ) = = + = = = (1 + )

(2= 2= ) = 2= (2 = + 2= ) = = (2 2 2)


Solutions Section 11.5 1 1 3 45. [ln( + 1) + 3 = ] = + 9 2= + 3 3= = + 3= ( 3 + 3 2) 1+ 1+ 46.

47.

48.

49.

50.

51.

52.

1 1 + = ln | + = | = ( = ) + = + = + =

1 (= ln | |) = = ln | | + = � � = = (ln | | + 1 )

1 (= log2 | |) = = log2 | | + = � � = = [log2 | | + 1 ( ln 2)] ln 2

2 2 1 ( = ) = 2 = 2 1 + 2(2= 2 1) = 2 = 2 1(1 + )

4= 4 1( 3 1) 3 2= 4 1 (4 3 3 2 4)= 4 1 = 4 1 = � 3 �= 1 ( 3 1) 2 ( 3 1) 2 2 + 1 (3 + = 3 + 1) = 2 ( 3 2 + 1 ln 3 + 3= 3 + 1

2 2 (= 4 ) = (= 2 )(2)(4 2 ) + = 2 (4 2 ) ln 4(2) = 2= 2 4 2 (1 + ln 4)

53. )( ) = ) ( ) =

2

3 2 + 1

2

2

2

3 ln 3(2 )( 2 + 1) 3 (2 ) 2 3 [( 2 + 1) ln 3 1] = ( 2 + 1) 2 ( 2 + 1) 2 2

54. )( ) = ( 2 + 1)4 1 2

2

2

) ( ) = (2 )4 1 + ( 2 + 1)4 1 ln 4(2 ) = (2 )4 1[1 + ( 2 + 1) ln 4]

= + = = = (= = )(= = ) (= + = )(= + = ) = 2 2 + = 2 (= 2 + 2 + = 2 ) ( ) = = (= = ) 2 (= = ) 2 4 = (= = ) 2 55. ( ) =

56. ( ) =

1 = = , so ( ) = = + = = + = ) 2

57. Simplify first:

58. Simplify first:

3 1 2 5 3 (= = = )= = = 5= 5 3. + 3 2 1 + 11 13 (= = = )= = = 0.


Solutions Section 11.5 1 � ( + (1)� 1 ln + 1 59. � = �= 2 ln ( ln ) ( ln ) 2 60. Simplify first: = 2 (1 + 2 ) (= 2 + 2 = 2 ) = 1 1 + 2 = = 2 2 . � � = � 2 � = 2 4 2 4 = = = = =

61. Note that ln(= ) = , so ( ) = 2 2 ln(= ) = 2 2 , so ( ) = 2 2 = 2( 1). 62. Note that = ln = , so ( ) = (= ln ) 4 = 4, so ( ) = 1 4 3. 63.

1 1 ln | ln | = ln = ln ln

1 1 1 ln | ln | ln || = ln | ln | = � � ln ln | ln | ln | ln | ln 1 1 1 1 = � �� � = ln | ln | ln ln ( ln | ln | 64.

1 ln(ln ), and so 2 1 1 1 1 1 1 ( ) = ln = ( = 2 ln 2 ln 2 ln

( ) = ln

ln = ln(ln ) 1 2 =

65. Simplify first:

66. ( ) =

ln(ln ) 1 1 1 1 ( ) = ln(ln ) = ( = 2 ln(ln ) 2 ln(ln ) ln 2 ln ln(ln )

= = when = 1. = = log2 + = ln 2 ln 2 Therefore the slope of the desired line is = ln 2, and a point on the line is (1, 0). So, the equation of the line is = (= ln 2)( 1) 3.92( 1). 67.

68.

= = = = 0 when = 0. So, the equation of the line is = 2.


Solutions Section 11.5 1 1 69. < > = "% ' " % > >= = " )=" > ln(2 + 1)< >= = = 1 when 2 2 + 1 = 0. The tangent line has slope 1 and passes through (0, 0), so its equation is = .

1 2 2 < > = "% ' " % > >= = " )=" > ln(2 2 + 1)< >= = = 2 2 2 + 1 3 when = 1. The tangent line has slope 2 3 and passes through (1, (ln 3) 2), so its equation is = (2 3)( 1) + (ln 3) 2 (2 3) 0.1174.

70.

2 = 2 = = 2= when = 1. The line at right angles to the graph has slope 1 (2=) and passes through (1, =), so its equation is = [1 (2=)]( 1) + = 0.1839 + 2.9022.

71.

3 3 4 ln 2 when = 1. The line at right angles to the graph has slope and = = (3 + 1) ln 2 4 ln 2 3 passes through the point (1, log2 4) = (1, 2), so its equation is 4 ln 2 = ( 1) + 2 0.9242 + 2.9242. 3 72.

73. The limit has the indeterminate form , so we can use l'Hospital's rule: +2 1 0 lim = lim = lim = 0 = = =


Solutions Section 11.5 74. The limit has the indeterminate form , so we can use l'Hospital's rule: 2 + + 1 2 + 1 2 0 lim = lim = lim = lim = 0 = = = = 75. lim

2 + 3 2 0 = lim = lim = 0 2 2 = 4= 2 2=

2 2 + 1 2 2 2 0 = lim = lim = lim = 0 3= 3 9= 3 27= 3 = 3

76. lim

77. The limit has the indeterminate form 0 0, so we can use l'Hospital's rule: = 1 = lim = lim = =0 = 1 0 0 1 78. The limit has the indeterminate form 0 0, so we can use l'Hospital's rule: = 1 = 1 = =0 1 lim lim lim = = = = 0 0 2 0 2 2 2 2 79. Over-65 population in 2040 ( = 40) is given by $(40) = 32 ln 40 36 82 million. The rate of change of the over-65 population is given by the derivative: 32 $ ( ) =

32 = 0.8 million, or 800,000 per year. 40 The number of U.S. residents over 65 in 2040 is projected to be 82 million and increasing at a rate of 0.8 million per year. $ (40) =

80. Over-85 population in 2050 ( = 50) is given by $(50) = 8 ln 50 16 15.3 million. The rate of change of the over-65 population is given by the derivative: 8 $ ( ) =

8 = 0.16 million, or 160,000 per year. 50 The number of U.S. residents over 85 in 2050 is projected to be 15.3 million and increasing at a rate of 0.16 million per year. $ (50) =

81. Over-65 population in 2040 ( = 30) is given by $(30) = 32 ln(0.325(30) + 3.25) 82 million. The rate of change of the over-65 population is given by the derivative: 32 10.4 $ ( ) = × 0.325 = 0.325 + 3.25 0.325 + 3.25 10.4 $ (30) = = 0.8 million, or 800,000 per year. 0.325(30) + 3.25 The number of U.S. residents over 65 in 2040 is projected to be 82 million and increasing at a rate of 0.8 million per year. This answer is the same as that of Exercise 79. 82. Over-85 population in 2050 ( = 40) is given by $(40) = 8 ln(0.138(40) + 1.38) 15.5 million. The rate of change of the over-85 population is given by the derivative:


Solutions Section 11.5 8 1.104 $ ()= × 0.138 = 0.138 + 1.38 0.138 + 1.38 1.104 $ (40) = = 0.16 million, or 160,000 per year. 0.138(40) + 1.38 The number of U.S. residents over 85 in 2050 is projected to be 15.5 million and increasing at a rate of 0.16 million per year. The first answer is slightly higher than that in Exercise 80 although the second answers agree to two decimal places.

1 1 1 log0.999879 (0.1 ) = (0.1 ) = 0.1 = = 0.1 ln 0.999879 0.1 ln 0.999879 ln 0.999879 Thus 1 1, 653 years per gram. = ? = 5 5 ln 0.999879 The age of the specimen is decreasing at a rate of about 1,653 years per additional 1 gram of carbon 14 present in the sample. (Equivalently, the age of the specimen is increasing at a rate of about 1,653 years per additional 1 gram less of carbon 14 in the sample.)

83.

1 1 1 log0.999567 (0.1 ) = (0.1 ) = 0.1 = = 0.1 ln 0.999567 0.1 ln 0.999567 ln].0.999567 Thus 1 289 years per gram. = ? = 8 8 ln 0.999567 The age of the specimen is decreasing at a rate of about 289 years per additional one gram of iodine 131 present in the sample. (Equivalently, the age of the specimen is increasing at a rate of about 289 years per additional one gram less of iodine 131 in the sample.) 84.

85. In 2003, = 9, and so the price of an apartment was (3) = 0.33= 0.16(9) = 0.33= 1.44 $1.4 million dollars. For the rate of change, we compute the derivative: ()= (0.33= 0.16 ) = (0.33)(0.16)= 0.16 = 0.0528= 0.16 . In 2003, (9) = 0.0528= 0.16(9) = 0.0528= 1.44 0.22 million dollars/year, or $220,000 per year. Thus, in 2003 the average price of a two-bedroom apartment in downtown New York City was increasing at a rate of about $220,000 per year.

86. In 2002, = 8, and so the price of an apartment was (3) = 0.14= 0.10(8) = 0.14= 0.80 $0.31 million dollars, or $310,000. For the rate of change, we compute the derivative: ()= (0.14= 0.10 ) = (0.14)(0.10)= 0.10 = 0.014= 0.10 . In 2002, (8) = 0.014= 0.10(8) = 0.014= 0.80 0.031 million dollars/year, or $31,000 per year. Thus, in 2002 the average price of a two-bedroom apartment in uptown New York City was increasing at a rate of about $31,000 per year.

87. a. ( ) = 320= 0.22 The year 2019 corresponds to = 3. (3) = 320= 0.22(3) 619.1 620 GW (rounded to two significant digits) ( ) = 320= 0.22 (0.22) = 70.4= 0.22 GW/year (3) = 70.4= 0.22(3) 136.2 140 GW/year The constants in the model are specified to two significant digits, so we cannot expect the answer to be


Solutions Section 11.5 accurate to more than two digits. In other words, all digits from the third on are probably meaningless. b. Residential capacity = 55% of total capacity = 0.55(320= 0.22 ). Its rate of change = [0.55(320= 0.22 )] = 0.55 [320= 0.22 ], which is 55% of the rate of change of total capacity. c. The rate of change of is the derivative ( ) = 70.4= 0.22 , which is also an exponential function. Hence the rate of change of solar capacity increased exponentially, answer (C).

88. a. @ ( ) = 210= 0.13 The year 2016 corresponds to = 6. @ (6) = 210= 0.13(6) 458.1 460 GW (rounded to two significant digits) @ ( ) = 210= 0.13 (0.13) = 27.3= 0.13 GW/year @ (6) = 27.3= 0.13(6) 59.55 60 GW/year The constants in the model are specified to two significant digits, so we cannot expect the answer to be accurate to more than two digits. In other words, all digits from the third on are probably meaningless. b. U.S. generation = 14% of world generation = 0.14(210= 0.13 ). Its rate of change = [0.14(210= 0.13 )] = 0.14 [210= 0.13 ], which is 14% of the rate of change of total capacity. c. The rate of change of @ is the derivative @ ( ) = 27.3= 0.13 , which is also an exponential function. Hence the rate of change of wind energy production increased exponentially, answer (C). 89. From the continuous compounding formula, the value of the balance at time years is 9( ) = 10,000= 0.04 . Its derivative is 9 ( ) = 400= 0.04 , so, after 3 years, the balance is growing at the rate of 9 (3) = $451.00 per year. 90. From the continuous compounding formula, the value of the balance at time years is 9( ) = 20,000= 0.035 . Its derivative is 9 ( ) = 700= 0.035 , so, after 3 years, the balance is growing at the rate of 9 (3) = $777.50 per year. 91. From the compound interest formula, the value of the balance at time years is 9( ) = 10, 000(1 + 0.04 2) 2 = 10, 000(1.02) 2 . Its derivative is 9 ( ) = [20, 000 ln(1.02)](1.02) 2 , so, after 3 years, the balance is growing at the rate of 9 (3) = $446.02 per year. 92. From the compound interest formula, the value of the balance at time years is 9( ) = 20, 000(1 + 0.035 2) 2 = 20, 000(1.0175) 2 . Its derivative is 9 ( ) = [40, 000 ln(1.0175)](1.0175) 2 , so, after 3 years, the balance is growing at the rate of 9 (3) = $770.07 per year.

93. The desired model is ( ) = 9 . At time = 0 (April 1, 2021) the number of active cases was 615,000, so 9 = 615,000. Since the number was increasing by 6.9% each day, that number is multiplied by 1.069 each day, so = 1.069. Hence, the model is ( ) = 615,000(1.069) . ( ) = 615,000(1.069) ln 1.069 41,035(1.069) cases per day. As April 21, 2021 corresponds to = 20, the number of cases was increasing at a rate of (20) = 41,035(1.069) 20 156,000 new cases per day (to three significant digits).

94. The desired model is ( ) = 9 . At time = 0 (April 1, 2003) the number of cases was 1,804, so 9 = 1804. Since the number was increasing by 4% each day, the number of cases is multiplied by 1.04 each day, so = 1.04. Hence, the model is 9( ) = 1, 804(1.04) .


Solutions Section 11.5 The rate of spread of the epidemic is the rate of change, 9 : = 1, 804(1.04) ln 1.04 cases per day. Since April 30, 2003 corresponds to = 29, the rate of spread of the epidemic on April 30 was | = 1, 804(1.04) 29 ln 1.04 221 new cases per day. | = 29

95. The desired model is ( ) = 9= . At time = 0 (April 1, 2014) the number of cases was 100, so 9 = 100. Since the number was increasing exponentially with a monthly growth constant of 72%, = 0.72. Hence, the model is ( ) = 100= 0.72 . The rate of increase is ( ) = 72= 0.72 . Since August 1, 2014 corresponds to = 4, the number of cases was increasing at a rate of (4) = 72= 0.72×4 1,280 new cases per month.

96. The desired model is *( ) = 9= . At time = 0 (April 1, 2014) the number of deaths was 90, so 9 = 90. Since the number was increasing exponentially with a monthly growth constant of 60%, = 0.60. Hence, the model is *( ) = 90= 0.60 . The rate of increase is * ( ) = 54= 0.60 . Since October 1, 2014 corresponds to = 6, the number of deaths was increasing at a rate of * (6) = 54= 0.60×6 1,980 new deaths per month.

97. a. (A): The data would best be modeled by a function that approaches a limit of around 140 as A . The function in (A) increases to the limit , so would be appropriate for the correct value of . The function in (B) decreases to the limit of , the one in (C) increases without bound, and the one in (D) decreases without bound. b. + ( ) = 5.8= 0.1 , so + (6) 3.18. At an experience level of 6 years, a developer's salary increases by approximately $3,180 per one-year increase in experience. c. + ( ) decreases with increasing , so as a programmer's experience increases, the effect on salary decreases. 98. a. (C): The data would best be modeled by a function that approaches a limit a little over 100 as A . The function in (C) increases to the limit , so would be appropriate for the correct value of . The function in (A) decreases to the limit of , the one in (B) decreases without bound, and the one in (D) increases without bound. b. 4( ) = 55= 0.05 , so 4 ( ) = 2.75= 0.05 , giving 4 (1) 2.62. At an experience level of one year, a developer's salary in Germany increases by approximately $2,620 per one-year increase in experience. c. 4 ( ) decreases with increasing , so as a programmer's experience increases, the effect on salary decreases. 99. a. ( ) = 120(0.172) ln(0.172), so that (2) = 120(0.172) 2 ln(0.172) 6.25 years per child (We rounded to three significant digits because the given coefficients are only given to three digits.) The answer tells us that, when the fertility rate is 2 children per woman, the average age of a population is dropping at a rate of 6.26 years per 1-child increase in the fertility rate. b. From part (a), the average age of a population is dropping at a rate of 6.26 years per 1-child increase in the fertility rate. In other words: Given: 1 child per woman increase → 6.26 year drop in average age Want: child per woman increase → 1 year drop in average age Answer: = 1 6.25 = 0.160 children per woman. 100. ( ) = 128(0.181) ln(0.181), so that (2.5) = 128(0.181) 2.5 ln(0.181) 3.05 years per child (We rounded to three significant digits because the given coefficients are only given to three digits.) The answer tells us that, when the fertility rate is 2.5 children per woman, the average age of a population is


Solutions Section 11.5 dropping at a rate of 2.43 years per 1-child increase in the fertility rate. b. From the preceding exercise, ( ) = 120(0.172) ln(0.172), so that (2.5) = 120(0.172) 2.5 ln(0.172) 2.59 years per child. Since this has a smaller absolute value than (2.5), we conclude that the European population is affected more than the combined population. 101. =

51 = 51(1 + 140= 1.12 ) 1. 1.12 1 + 140=

( ) = 51(1 + 140= 1.12 ) 2[ 1.12(140= 1.12 )] =

7,996.8= 1.12 . (1 + 140= 1.12 ) 2

(5) 13% per month (7) 3% per month (Although, strictly speaking, is not differentiable at the endpoint = 7 of its domain, we can reasonably asume that the formula works for at least a small interval to the right of = 7.)

102. =

72 = 72(1 + 104= 1.06 ) 1. 1 + 104= 1.06

( ) = 72(1 + 104= 1.06 ) 2[ 1.06(104= 1.06 )] = (4) 18% per month (6) 10% per month

7,937.28= 1.06 . (1 + 104= 1.06 ) 2

15.0 = 15(1 + 8.6= 0.59 ) 1 1 + 8.6= 0.59 The rate of change of the percentage is the derivative, 103. 9( ) =

9 ( ) = 15(1 + 8.6= 0.59 ) 2(8.6= 0.59 )( 0.59) =

(15)(8.6)(0.59)= 0.59 76.11= 0.59 . = (1 + 8.6= 0.59 ) 2 (1 + 8.6= 0.59 ) 2

The start of 2003 corresponds to = 3 so 76.11= 0.59(3) 9 (3) = 2.1 percentage points per year. (1 + 8.6= 0.59(3)) 2 Referring to the graph, the rate of change is the slope of the tangent at = 3. This is also approximately the average rate of change over [2, 4], which is about 4 2 = 2, in approximate agreement with the answer.

1,350 = 1,350(1 + 4.2= 0.53 ) 1 1 + 4.2= 0.53 The rate of change of the percentage is the derivative,

104. 9( ) =

9 ( ) = 1,350(1 + 4.2= 0.53 ) 2(4.2= 0.53 )( 0.53) =

3,005.1= 0.53 . (1 + 4.2= 0.53 ) 2 The start of 2005 corresponds to = 5 so =

(1,350)(4.2)(0.53)= 0.53 (1 + 4.2= 0.53 ) 2


3,005.1= $126 billion per year. (1 + 4.2= 0.53(5)) 2 Referring to the graph, the rate of change is the slope of the tangent at = 5. This is also approximately the average rate of change over [4, 6], which is about 250 2 = $125 billion, in approximate agreement with the answer. 9 (5) =

0.53(5)

Solutions Section 11.5

15.0 = 15(1 + 8.6(1.8) ) 1 1 + 8.6(1.8) The rate of change of the percentage is the derivative,

105. a. 9( ) =

9 ( ) = 15(1 + 8.6(1.8) ) 2( 8.6(1.8) ) ln(1.8) =

(15)(8.6) ln(1.8)(1.8) (1 + 8.6(1.8) ) 2

75.82(1.8) . (1 + 8.6(1.8) ) 2 The start of 2003 corresponds to = 3 so 75.82(1.8) 3 9 (3) = 2.1 percentage points per year. (1 + 8.6(1.8) 3) 2 15.0 b. lim 9( ) = lim 1 + 8.6(1.8) 15.0 Since (1.8) A 0 as A , the limit is = 15. 1+0 Interpretation: Had the trend continued indefinitely, the percentage of mortgages that were subprime would have approached 15% in the long term. From part (a), 75.82(1.8) lim 9 ( ) lim . (1 + 8.6(1.8) ) 2 0 Since (1.8) A 0 as A , the limit is = 0. (1 + 0) 2 Interpretation: Had the trend continued indefinitely, the rate of change of the percentage of mortgages that were subprime would have approached 0 percentage points per year in the long term.

1,350 = 1,350(1 + 4.2(1.7) ) 1 1 + 4.2(1.7) The rate of change of the percentage is the derivative,

106. 9( ) =

9 ( ) = 1,350(1 + 4.2(1.7) ) 2( 4.2(1.7) )(ln 1.7) =

(1,350)(4.2)(ln 1.7)(1.7) (1 + 4.2(1.7) ) 2

3,008.7(1.7) . (1 + 4.2(1.7) ) 2 The start of 2005 corresponds to = 5 so 3,008.7(1.7) 5 9 (5) $126 billion per year. (1 + 4.2(1.7) 5) 2 1,350 b. lim 9( ) = lim 1 + 4.2(1.7) 1,350 Since (1.7) A 0 as A , the limit is = 1,350. 1+0 Interpretation: Had the trend continued indefinitely, the value of subprime mortgage debt outstanding would have approached $1,350 billion in the long term. From part (a), 3,008.7(1.7) lim 9 ( ) lim . (1 + 4.2(1.7) ) 2 0 Since (1.7) A 0 as A , the limit is = 0. (1 + 0) 2


Solutions Section 11.5 Interpretation: Had the trend continued indefinitely, the rate of change in the value of subprime mortgage debt outstanding would have approached $0 billion per year in the long term. 107. For an exponential model ( ) = 9 of the population, where is time in years since 2010, use the two points on its graph: (0, 4,000,000) and (10, 8,000,000). Substituting gives 4,000,000 = 9 8,000,000 = 9 10 = 4,000,000 10 so 10 = 2, giving = 2 1 10 and the model is ( ) = 4,000,000(2 1 10) = 4,000,000(2 10). Its derivative is ( ) = [400,000 ln 2]2 10, so, at the start of 2010, the population was growing at the rate of (0) 277,000 people per year. 108. For an exponential model ( ) = 9 of the population, where is time in years since 2011, use the two points on its graph: (0, 3,000,000) and (7, 6,000,000). Substituting gives 3,000,000 = 9 6,000,000 = 9 7 = 3,000,000 7 so 7 = 2, giving = 2 1 7, and the model is ( ) = 3,000,000(2 1 7) = 3,000,000(2 7). Its derivative is ( ) = [(3,000,000 7) ln 2]2 7, so, at the start of 2011, the population was growing at the rate of (0) 297,000 people per year. 109. The amount of plutonium 239 left after years is ( ) = 10(0.5) 24,400. Its derivative is ( ) = [(10 24,400) ln 0.5](0.5) 24,400, so, after 100 years, the rate of change is (100) 0.000283 grams per year. That is, the plutonium is decaying at the rate of 0.000283 grams per year. 110. The amount of carbon 14 left after years is ( ) = 20(0.5) 5,730. Its derivative is ( ) = [(20 5,730) ln 0.5](0.5) 5,730, so, after 100 years, the rate of change is (100) 0.00239 grams per year. That is, the carbon is decaying at the rate of 0.00239 grams per year.

111. The revenue per subscriber at time is ( ) (11 + 88.3)(1 + 50= 1.25 ) billions of dollars per million subscribers, or thousands of dollars per = $( ) 1,380 subscriber (5) 1 + 50= 1.25(5) 0.114 thousand dollars, or $114 per subscriber. = (11(5) + 88.3) $(5) 1,380 1 � ( ) = 11(1 + 50= 1.25 ) + (11 + 88.3)( 62.5= 1.25 )� by the product rule, so 1,380 (5) 0.00379 thousand dollars per subscriber per year, or −$3.79 per subscriber per year. 112. Percentage return on investment is given by ( ) B( ) 100 ( ) 100(11 + 88.3) ( ) = 100 ( = 100 = 100 B( ) B( ) 26.8 + 8.05= 0.335 100(11(4) + 88.3) (4) = 100 358% (not a bad return!) 26.8 + 8.05= 0.335(4)


Solutions Section 11.5 0.335 11(26.8 8.05= ) (11 + + + 88.3)(2.69675= 0.335 ) ( ) = 100 by the quotient rule, so (26.8 + 8.05= 0.335 ) 2 (4) 49.2 percentage points per year. 113. = raised to the glob, times the derivative of the glob. 114. one over the glob, times the derivative of the glob. 115. 2 raised to the glob, times the derivative of the glob, times the natural logarithm of 2. 116. one over ln(2) times the glob, multiplied by the derivative of the glob. 117. The derivative of ln |)| is not

1 ) 1 ) 3 ; it is . Thus, the correct result is . |)| ) 3 + 1

118. The derivative of 2 raised to a quantity is 2 raised to the quantity, times the derivative of the quantity, times ln 2. Thus, the correct answer is 2 2 2 ln 2. 119. The power rule does not apply when the exponent is not constant. The derivative of 3 raised to a quantity is 3 raised to the quantity, times the derivative of the quantity, times ln 3. Thus, the correct answer is 3 2 2 ln 3. 120. The derivative of ln ) is not

1 1 ) 1 6 ; it is . Thus, the correct answer is 2 6 = . ) ) 3 1 3 2 1

121. No. If $( ) is exponential, so is its derivative.

122. Yes. If $( ) is exponential decay, then its derivative increases with time. 123. If ( ) = = , then the fractional rate of change is ( ) #= = = #, ( ) = the growth constant. 124. The rate of change of ln ( ) is given by

( ) ln ( ) = , the fractional rate of change of . ( )

125. If 9( ) is growing exponentially, then 9( ) = 90 = for constants 90 and #. Its percentage rate of 9 ( ) #90 = change is then = = #, a constant. 9( ) 90 = 126. If 9( ) is the amount of money in an account that compounds interest periodically, then 9( ) = �1 +

� = for constants , , and '. ' Thus, its fractional rate of change is 9 ( ) ' ln( ) = = ' ln( ), 9( ) a constant. This constant is the interest rate that, compounded continuously, would give the same interest as the given account: = ln( ) = (= ln( )) = .


Solutions Section 11.6 Section 11.6 1. Implicit differentiation: 2+3 = 0 = 2 3 Solving for first and then taking the derivative: = (7 2 ) 3, so = 2 3 2. Implicit differentiation: 4 5 = 0 = 4 5 Solving for first and then taking the derivative: = (4 9) 5, so = 4 5 3. Implicit differentiation: 2 2 = 0 = Solving for first and then taking the derivative: = ( 2 6) 2, so, = 4. Implicit differentiation: 3 + 2 = 0 = 2 3 Solving for first and then taking the derivative: = (5 2) 3, so, = 2 3 5. Implicit differentiation: 2+3 = + (3 ) = 2 = ( 2) (3 ) Solving for first and then taking the derivative: = 2 (3 ), so 2(3 ) 2 2 2 2 2 = = = + = 2 2 3 3 3 3 (3 (3 ) 6. Implicit differentiation: 1 = +


Solutions Section 11.6 (1 + ) = 1 = (1 ) (1 + ) Solving for first and then taking the derivative: = (1 + ) (1 + ) 1 1 1 = = = = 2 2 1 + (1 + ) 1+ 1+ 1+ (1 + ) 7. Implicit differentiation: = + = = 0 = Solving for first and then taking the derivative: = 1 = = = , so = = = 8. Implicit differentiation: = + = = 0 (1 = ) = = = = 1 = Solving for first and then taking the derivative: = �2 (= 1)� = = 2 (= 1), so, = 2= (= 1) 2 = = 1 = 1 = 9. Implicit differentiation: ln + + = 0 (1 + ln ) = = (1 + ln ) Solving for first and then taking the derivative: 2 2 1 2 1 = , so ( = ( = = 1 + ln 1 + ln (1 + ln ) (1 + ln ) (1 + ln ) 2 10. Implicit differentiation: � ln � = 1 2 ln = 2 ln = (1 + ) (1 + ) = ln Solving for first and then taking the derivative: = (ln ) (2 ), so


Solutions Section 11.6 (2 ) + ln (ln ) ( ) + ln [using 2 = (ln ) ] = = [(ln ) ] 2 (2 ) 2 (ln )(1 + ) (1 + ) = = ln (ln ) 2

11. 2 + 2

12. 4 2

= 0, so = = 0, so = 2

13. 2 + 2 14. 2 + 2 15. 3 + 3 16.

2 = 0

= 1

17. 2

(4 )

= 2

= 1 2

( 3 2)

6 = 0, so = 3

18. 2 2 19. 2

(2 1)

2 = 2

+ 2 = 0 2 2

( 2 2 )

⇒ ⇒

= (6 + 9 2 )

=

= 2 ( 2 2 )

= (1 2) (2 1)

6 + 9 2 = 3 9 2

= (4 )

+ 2 2 + 2 = 0, so = 2( 2 + ) (2 2) = ( 2 + 1) ( )

⇒ (2 ! 10 ! 2) = ! = 10 ! 2 + 10 2! ! ! ! ! ⇒ = ( + 10 2!) (2 ! 10 ! 2) !

+ 10 2!

⇒ (! + 10 ! 2) = 2! 10 2! = 10 ! 2 + 10 2! ! ! ! 2 2 ⇒ = (2! 10 !) (! + 10 ! ) !

20. 2! !

21. = + =

= = = 0

22. 2 = + 2= 23. = � 24. =

2

2 = =

+ �= 2

�2

+

2

( = = )

( 2= 2 )

(2 = )

= 0

=

(2 =

= 2

= = =

= = 2 =

)

⇒ =

= = = = =

= = 2 =

=

2

2

= 2 = = 2= 2

2

2= = 2 = 2


= 2 2 = ( ) 25. = = 4 26.

= = ( ) (= ) 2

1 + = = 9= + =

27.

28.

+ +

2 1 . +1= 2

= 9

+ ( ) ln =

+ 2 � 29. = = 2 + = + 2 = 2= � = 1 + =

= � +

(9= + = )

�(2 1) ( 2 )�

� +

30.

Solutions Section 11.6 ⇒ (2= + 3= ) = =

= = 2= + 3=

= 1 + =

= ( 2 )

( 2 )

= =

33. a. 2 2 2 = 4 4 2 , so = + = + 2 | 4( 1) 2 = = 2. ||| ( 1,2) 1 + 2(2) b. Point: ( 1, 2); Slope: 2; Intercept: = 1 ' 1 = 2 ( 2)( 1) = 0 Equation of tangent line: = 2 4 + , so = 6

ln . = 2

= = 1 + = =

32. a. 3 2 2 = 11 6 2 ⇒ = 0 = 3 , so | = 3( 2) 1 = 6. | ||( 2,1) b. Point: ( 2, 1); Slope: 6; Intercept: = 1 ' 1 = 1 ( 6)( 2) = 11 Equation of tangent line: = 6 11

2 = 1 + 3 2

=

31. a. 4 2 + 2 2 = 12 8 + 4 ⇒ = 0 = 2 , so | = 2(1) ( 2) = 1. | |(1, 2) b. Point: (1, 2); Slope: 1; Intercept: = 1 ' 1 = 2 (1)(1) = 3 Equation of tangent line: = 3

34. a. 2 2 + = 3 2 4 + + = 6

= ln .

� + 2 = ( + 2)�

(1 + = = )


Solutions Section 11.6 | 4( 1) + ( 1) = = 1. | |( 1, 1) 6( 1) ( 1) b. Point: ( 1, 1); Slope: 1; Intercept: = 1 ' 1 = 1 (1)( 1) = 0 Equation of tangent line: =

35. a. 2 2 + = 1 2 2 + 2 ( 2 ) = (1 + 2 ) 2 +1= 0 | 1 + 2(1)(0) = 2 = 1 | |(1,0) 1 2(0) b. Point: (1, 0); Slope: 1; Intercept: = 1 ' 1 = 0 ( 1)(1) = 1 Equation of tangent line: = + 1

1 + 2 , so = 2 2

36. a. ( ) 2 + = 8 2 ( + ) + ( + ) 1 = 0 2 2 + 2 2 + + 1= 0 1 2 2 ⇒ (2 2 + ) = 1 2 2 , and so = 2 2 + | 1 2(8)(0) 2 0 = = 1 8. || ( 8,0) 2(8) 2(0) 8 b. Point: ( 8, 0); Slope: 1 8; Intercept: = 1 ' 1 = 0 ( 1 8)( 8) = 1 Equation of tangent line: = 8 1

37. a. 2,000 = + = (1 ) = = 1 When = 2, the corresponding value of is obtained from the original equation 2000 = : 2000 = 2 2000 = = 2,000. We now evaluate the derivative: 2,000 = = = 2,000. 1 1 2 b. Point: (2, 2,000); Slope: 2,000; Intercept: = 1 ' 1 = 2,000 ( 2,000)(2) = 6,000 Equation of tangent line: = 2,000 + 6,000

38. a. 2 10 = 200 2 10 5 2 10� + � = 0 2 10 10 = 0 = = 10 5 When = 10, the corresponding value of is obtained from the original equation 2 10 = 200 : 2 10 = 200 100 100 = 200 = 1. We now evaluate the derivative: 5 10 + 5 = = = 0.3. 5 50 b. Point: (10, 1); Slope: 0.3; Intercept: = 1 ' 1 = 1 (0.3)(10) = 4 Equation of tangent line: = 0.3 4

39. a. ln( + ) = 3 2 1 + 1 = 6 = (6 + 1)( + ) 1 + When = 0, the corresponding value of is obtained from the original equation ln( + ) = 3 2 by


Solutions Section 11.6 substituting: ln = 0, = = 0 = 1. We can now evaluate the derivative: = (0 + 1)(0 + 1) 1 = 0. b. Point: (0, 1); Slope: 0; Intercept: = 1 ' 1 = 1 Equation of tangent line: = 0 + 1, or = 1

40. a. ln( ) + 1 = 3 2 1 = 6 = 1 6 ( ) When = 0, the corresponding value of is obtained from the original equation ln( ) + 1 = 3 2 by substituting: ln( ) + 1 = 0 ln( ) = 1 = = 1 = = 1 0.3679, = 1 0 = 1 (notice that we didn't actually need to find to find ). b. Point: (0, = 1); Slope: 0; Intercept: = 1 ' 1 = = 1 (1)(0) = = 1 0.36788 Equation of tangent line: = 0 = 1, so = = 1 or = 0.3679 41. a. = = 4 5 = = , + - 1 = 4 = = When = 3, the corresponding value of is obtained from the original equation = = 4 by substituting: = 3 3 = 12 = 3 = 15 3 = ln .15 1 = ln .15 0.902683. 3 5 (0.902683)(15) Thus 0.18978 (using the fact that = 3 = 15). 3(15) b. Point: (3, 0.9027); Slope: 0.1898; Intercept: 0.9027 3( 0.18978) = 1.4720 Equation of tangent line: = 0.1898 + 1.4720

42. a. = + 2 = 1 2 = � + �= + 2 = 0 = = When = 1, = 2 = 1, so = = 3. 2 1.09861(3) Thus, = ln .3 1.09861, and 0.4319 (using the fact that = = 3). ( 1)(3) b. Point: ( 1, 1.0986); Slope: 0.1898; Intercept: 1.0986 0.4319( 0.1898) = 1.5305 Equation of tangent line: = 0.4319 + 1.5305 2 + 1 4 2 2 + 1 ln . = ln� � = ln(2 + 1) ln(4 2) 4 2 Take of both sides: 43. =


Solutions Section 11.6 1 2 4 = 2 + 1 4 2 2 4 2 + 1 2 4 . = C = D C 2 + 1 4 2 4 2 2 + 1 4 2 D

44. = (3 + 2)(8 5) ln . = ln(3 + 2) + ln(8 5) Take of both sides: 1 3 8 = + 3 + 2 8 5 3 8 3 8 . = C + = (3 + 2)(8 5)C + 3 + 2 8 5 D 3 + 2 8 5 D

(3 + 1) 2 4 (2 1) 3 (3 + 1) 2 ln . = ln = ln(3 + 1) 2 ln(4 ) ln(2 1) 3 = 2 ln(3 + 1) ln(4 ) 3 ln(2 1) E 4 (2 1) 3 F Take of both sides: 1 6 1 6 = 3 + 1 2 1 (3 + 1) 2 6 1 6 6 1 6 . = C = 3 + 1 2 1 D 4 (2 1) 3 C 3 + 1 2 1 D 45. =

2(3 + 1) 2 (2 1) 3 2(3 + 1) 2 ln . = ln = ln( 2) + ln(3 + 1) 2 ln(2 1) 3 = 2 ln . + 2 ln(3 + 1) 3 ln(2 1) E (2 1) 3 F Take of both sides: 1 2 6 6 = + 3 + 1 2 1 2(3 + 1) 2 2 2 6 6 6 6 . = C + = + D C 3 3 + 1 2 1 3 + 1 2 1 D (2 1) 46. =

47. = (8 1) 1 3( 1)

ln . = ln [(8 1) 1 3( 1)] = ln(8 1) 1 3 + ln( 1) =

1 ln(8 1) + ln( 1) 3

Take of both sides: 1 8 1 = + 3(8 1) 1 8 1 J 8 1 J . = G + = (8 1) 1 3( 1)G + HI 3(8 1) 1 KL HI 3(8 1) 1 KL

(3 + 2) 2 3 3 1 (3 + 2) 2 3 2 ln . = ln E = ln(3 + 2) 2 3 ln(3 1) = ln(3 + 2) ln(3 1) 3 1 F 3 Take of both sides: 1 2 3 = 3 + 2 3 1 (3 + 2) 2 3 2 3 2 3 . = C = 3 + 2 3 1 D 3 1 C 3 + 2 3 1 D 48. =


49. = ( + ) 3

3 + 2

Solutions Section 11.6

ln . = ln M( 3 + ) 3 + 2N = ln( 3 + ) +

1 ln( 3 + 2) 2

Take of both sides: 1 3 2 + 1 1 3 2 = + 3 + 2 3 + 2 3 2 + 1 1 3 2 3 2 + 1 1 3 2 = � 3 + = + � � �. + 2 3 + 2 3 + 2 3 + 2

1 ; 2 + 2 1 1 ln . = ln = �ln( 1) ln( 2 + 2)� ; 2 + 2 2 Take of both sides: 1 1 1 2 = � 2 � 2 1 + 2 1 1 2 1 1 1 2 ( (� = ( (� �= �. ; 2 + 2 2 1 2 + 2 2 1 2 + 2 50. =

51. ln = ln( ) = ln 1 1 = ln + � � = 1 + ln = (1 + ln )

52. ln = ln( ) = ln 1 1 = ln � � = (1 + ln ) = (1 + ln ) 53. = 0.6 0.4 Taking of both sides gives

0.6 0.6 0.4 0.4 = 0.4 0.6 0.6 0.4 0.4 3 = = 2 0.4 0.6 0.5 | 3(200,000) = = $3,000 per worker. | || = 100, = 200,000 2(100) The monthly budget to maintain production at the fixed level is decreasing by approximately $3,000 per additional worker at an employment level of 100 workers and a monthly operating budget of $200,000. In other words, increasing the workforce by one worker will result in a saving of approximately $3,000 per month. 0 = 0.6 0.4 0.4 + 0.4 0.6 0.6

54. = 0.5 0.5 Taking of both sides gives

0.5 0.5 0.5 = 0.5 0.5 0.5 0 = 0.5 0.5 0.5 + 0.5 0.5 0.5


Solutions Section 11.6 0.5 0.5 0.5 = = 0.5 0.5 0.5 | 100,000 = = $500 per worker. | || = 200, = 100,000 200 The monthly budget to maintain production at the fixed level is decreasing by approximately $500 per additional worker at an employment level of 200 workers and a monthly operating budget of $100,000. 55. 2,000 = Taking of both sides gives + = = (1 ) . = 1 When = 5, the corresponding value of is obtained from the original equation 2,000 = : 2,000 = 5 2,000 = 4 = 2,000 = 500. We now evaluate the derivative: 500 = = = 125 T-shirts per dollar. Thus, when the price is set at $5, the demand is 1 1 5 dropping by 125 T-shirts per $1 increase in price. 56. 2 10 = 200 2 10� + � = 0 2 10 10 = 0 10 = 2 10 When = 1, the corresponding value of is obtained from the original equation 2 10 = 200 : 2 100 = 200 2 100 200 = 0 ( 20)( + 10) = 0 = 20 (reject = 10, which doesn't make sense in the application). We now evaluate the derivative: 10 5(20) = = = 6.67¢ per gallon. 2 10 20 5(1)

57. Set = 200,000 and differentiate the equation with respect to = : # 0 = 100# + 120= = # 120= 6= = = . = 100# 5# When = = 15 200,000 = 15,000 + 50# 2 + 60(15) 2 = 50# 2 + 28,500, so 50# 2 = 171,500, giving # 58.57, so 6(15) # | 0.307 carpenters per electrician. = ||| = = 15 5(58.57) This means that, for a $200,000 house whose construction employs 15 electricians, adding one more electrician would cost as much as approximately 0.307 additional carpenters. In other words, one electrician is worth approximately 0.307 carpenters.


Solutions Section 11.6 58. Set 8 = 200 and differentiate the equation with respect to : = 0 = 6= + 15 2 15 2 5 2 = . = = 6= 2= When = 3.0, 200 = 3= 2 + 5(3.0) 3 = 3= 2 + 135 3= 2 = 65, = 4.655, so 5(3.0) 2 = | 4.83 years of experience per grade point. = ||| = 3.0 2(4.655) This means that, at a value level of 200, an increase by 1.0 for a candidate with a 3.0 grade-point average is worth 4.83 years of experience.

59. a. Set = 3.0 and = 80 and solve for : 80 = 12 0.2 2 90, 0.2 2 12 + 170 = 0, 22.93 hours by the quadratic formula. (The other root is rejected because it is larger than 30.) b. Set = 80 and differentiate the equation with respect to : 0 = 4 + 4 0.4 20 , 4 20 5 , so = = 0.4 4 0.1 22.93 5(3.0) | 11.2 hours per grade point. ||| = 3.0 0.1(22.93) 3.0 This means that, for a 3.0 student who scores 80 on the examination, 1 grade point is worth approximately 11.2 hours. 60. a. 80 = 30 0.2 2 90, 5.90 hours. (The other root is rejected because it is larger than 10.) b. 0 = 10 + 10 0.4 20 , so 10 20 2 , so = = 0.4 10 0.04 5.90 2(3.0) | 0.0362 hours per grade point. ||| = 3.0 0.04(5.90) 3.0 Note that this is positive, implying that the required study hours go up as the grade-point average increases. This means that, for a 3.0 student who scores 80 on the examination, 1 grade point is worth 0.0367 hours of study. In other words, the higher your GPA, the harder you have to study for the test! 61. 0 = 1.2 0.4 0.3 0.6 0.6 1.3

by the chain rule = 2 , so = = 2

62. 0 = 1.2 0.4 0.3 + 2 0.6 0.3

= 0.6 , so = = 0.6

63. , , , 64. , , ,

65. Let = ( ) ( ). Then ln = ln ( ) + ln ( ), and


Solutions Section 11.6 ( ) ( ) 1 , so = + ( ) ( ) ( ) ( ) ( ) ( ) = � + + � = ( ) ( )� � = ( ) ( ) + ( ) ( ). ( ) ( ) ( ) ( )

( ) . Then ln = ln ( ) ln ( ), and ( ) 1 ( ) ( ) , so = ( ) ( ) ( ) ( ) ( ) ( ) ( ) ( ) ( ) ( ) ( ) ( ) = � + �= � �= � � ( ) ( ) ( ) ( ) ( ) ( ) ( ) ( ) ( ) ( ) ( ) ( ) . = ( ) 2

66. Let =

67. Writing = ( ) specifies as an explicit function of . This can be regarded as an equation giving as an implicit function of . The procedure of finding by implicit differentiation is then the same as finding the derivative of as an explicit function of : we take of both sides. 68. If is not a function of x, then its graph will "fail" the vertical line test, so that there is a vertical line, = , say, that passes through more than one point of the graph. If the slopes at two of these points are not the same, the derivative cannot be a function of either because it too has (at least two) different values at = . 69. Differentiate both sides of the equation = ( ) with respect to to get 1 = ( ) ( , giving 1 1 . = = ( )

70. The graph of the equation should be such that, if the vertical line = passes through more than one point of the graph, then the slope at each of those points is the same.


Solutions Chapter 11 Review Chapter 11 Review

1. ( ) = 50 4 + 2 3 1

2. ( ) = 10 5 + 4 2 1 + 2, so ( ) = 50 6 4 5 2 + 2 = 50 6 2 5 + 1 2 3. ( ) = (3 3 + 3 1 3), so ( ) = 9 2 + 2 3

4. ( ) = 2 2.1 0.1 2, so ( ) = 4.2 3.1 0.1 0.9 2 = 4.2 3.1 0.1 (2 0.9) 5. ( ) = + 2, so ( ) = 1 2 3, or 1 2 3 6. ( ) = 2 1 so ( ) = 2 + 2, or 2 + 1 2

4 1 1 1.1 2 0.1 + 4, so 3 3.2 4 1.1 0.1 4 0.2 1.1 0.1 ( ) = 2 + 0.2 1.1 + , or 2 + 1.1 + 3 3.2 3.2 3

7. ( ) =

1 | |, so 4 1 | | 4 1 | | ( ) = 4 2 + , or 2 +

8. ( ) = 4 1 +

9. ( ) = = ( 2 1) is a product so we use the product rule: ( ) = = ( 2 1) + = (2 ) = = ( 2 + 2 1). 2 + 1 is a quotient so we use the quotient rule: 2 1 2 ( 2 1) ( 2 + 1)(2 ) 4 ( ) = . = 2 2 2 ( 1) ( 1) 2

10. ( ) =

| | + 1 is a quotient so we use the quotient rule: 3 2 + 1 (| | )(3 2 + 1) (| | + 1)(6 ) 3 | | + | | 6 ( ) = . = (3 2 + 1) 2 (3 2 + 1) 2

11. ( ) =

12. ( ) = (| | + )(2 3 2) is a product so we use the product rule: ( ) = (| | + 1)(2 3 2) 6 3(| | + ). 13. ( ) = (4 1) 1, so ( ) = (4 1) 2(4) = 4(4 1) 2

14. ( ) = ( + 7) 2, so ( ) = 2( + 7) 3(1) = 2( + 7) 3.

15. ( ) = ( 2 1) 10 is a power, so we use the chain rule: ( ) = 10( 2 1) 9(2 ) = 20 ( 2 1) 9.


Solutions Chapter 11 Review 1 2 10 16. ( ) = is a power, so we use the chain rule: = ( 1) ( 2 1) 10 20 ( ) = 10( 2 1) 11(2 ) = . ( 2 1) 11 17.

18.

19.

20.

21.

22.

[2 + ( + 1) 0.1] 4.3 = 4.3[2 + ( + 1) 0.1] 3.3 [2 + ( + 1) 0.1] = 0.43( + 1) 1.1[2 + ( + 1) 0.1] 3.3

[( + 1) 0.1 4 ] 5.1 = 5.1[( + 1) 0.1 4 ] 6.1 [( + 1) 0.1 4 ] = 5.1[0.1( + 1) 0.9 4].[( + 1) 0.1 4 ] 6.1 2 + 1 = = 2= 2 + 1 4 5 = = 4= 4 5

2 4 3 = 3 2 4 ln .3 (2 4) = 2 ( 3 2 4 ln .3

+ 5 4 = 4 + 5 ln .4 ( + 5) = 4 + 5 ln .4

23. ( ) = = ( 2 + 1) 10 ( ) = = ( 2 + 1) 10 + = ( 10( 2 + 1) 9(2 ) = = ( 2 + 1) 9( 2 + 20 + 1) 1 3 � is a power, so we use the chain rule: 3 + 1 1 2 (3 + 1) 3( 1) 12( 1) 2 ( ) = � . = � 3 + 1 (3 + 1) 4 (3 + 1) 4

24. ( ) = �

25. ( ) =

3 ln 3( 1) 3 (1) 3 [( 1) ln 3 1] 3 Quotient rule: ( ) = = 1 ( 1) 2 ( 1) 2

26. ( ) = 4 ( + 1) Product rule: ( ) = (4 ) ln 4( 1)( + 1) + 4 (1) = 4 [ ( + 1) ln 4 + 1] 2

2

27. ( ) = = 1 Chain rule: ( ) = 2 = 1 2

2

2

2

28. ( ) = ( 2 + 1)= 1 Product rule: ( ) = 2 = 1 + ( 2 + 1)(2 )= 1 = 2 ( 2 + 2)= 1 29.

30.

1 1 3 ln |3 1| = (3 1) = (3= 3 1 3 1 3 1

1 1 9 ln |5 9 | = (5 9 ) = ( 9) = 5 9 5 9 5 9

31. ( ) = ln( 2 1) Chain rule: ( ) =

2

2 1


Solutions Chapter 11 Review ln( 2 1) 32. ( ) = Quotient rule: 2 1 2 ( 2 1) 2 ln( 2 1) 2 1 2 2 ln( 2 1) ( ) = = ( 2 1) 2 ( 2 1) 2 33. Since the slope of the tangent line is the derivative, the tangent line is horizontal when its slope is 0; that is, = 0. = 3 2 + 7 1 = 6 + 7 = 0 when = 7 6 34. Since the slope of the tangent line is the derivative, the tangent line is horizontal when its slope is 0; that is, = 0. = 5 2 2 + 1 = 10 2 = 0 when = 1 5 2 + 2 1 2 1 2 = 2 = 0 when = 2 . Thus, 2 = 4, giving = ±2. 2 2

35. =

2 8 2 2 16 16 = + 3 = 0 when = 3 , or 4 = 16 This equation has no real solutions; so there are no points where the tangent is horizontal.

36. =

37. = = 2 1, so = 1 2= 2 1. Set = 0 : 1 2= 2 1 = 0 2 1 = ln(1 2) = ln 2 2

2

2

= (1 ln 2) 2.

38. = = , so = 2 = . Set = 0 : 2 = = 0 when = 0.

+1 ( + 1) 1 = = 2 ( + 1) ( + 1) 2 This is never 0, so there are no points at which the tangent line is horizontal. 39. =

40. =

( 1) 3 1 = 1 2 1 2 = 0 when 3 1 2 = 1 2, giving 3 = 1, so = 1 3. 2 2

41. 2 2 = Take of both sides: 2 2 = 1. 2 1 Solve for :\quad . = 2


Solutions Chapter 11 Review 42. 2 + 2 = Take of both sides: . 2 + 2 + 2 = 2 Solve for :\quad . = 2( + ) 1 43. = + = 2 Take of both sides: = 0. � + �= + + (= + 1) Solve for : = = . (= + 1) 44. ln� � = Take of both sides: 1 ( ). = � � 2 . = Solve for : . = (1 )

(2 1) 4(3 + 4) ( + 1)(3 1) 3 We use logarithmic differentiation: (2 1) 4(3 + 4) ln = ln� � = 4 ln(2 1) + ln(3 + 4) ln( + 1) 3 ln(3 1) ( + 1)(3 1) 3 1 8 3 1 9 = + 2 1 3 + 4 + 1 3 1 (2 1) 4(3 + 4) 8 3 1 9 8 3 1 9 = � + + �= � �. 2 1 3 + 4 + 1 3 1 ( + 1)(3 1) 3 2 1 3 + 4 + 1 3 1 45. =

46. = 13 We use logarithmic differentiation: ln = ( 1) ln + ln 3 1 = (1) ln + ( 1) + ln 3 = [ln + ( 1) + ln 3] = 13 [ln + ( 1) + ln 3].

47. = ( 2 3 ) 2 When = 1, = (1 3) 2 = 1 4 = 2( 2 3 ) 3(2 3) = 2(1 3) 3(2 3) = 1 4. ? = 1 Tangent line: Slope: ' = 1 4; Intercept: = 1 ' 1 = 1 4 ( 1 4)(1) = 1 2 = 4 + 1 2


Solutions Chapter 11 Review 48. = (2 2 3) 3 When = 1, = (2 3) 3 = 1 = 3(2 2 3) 4(4 ) = 3(2 3) 4( 4) = 12. ? = 1 Tangent line: Slope: ' = 12; Intercept: = 1 ' 1 = 1 (12)( 1) = 11 = 12 + 11 49. = 2= When = 1, = = 1 = = = 2 = + 2= ( 1) = = (2 2) = =( 2 1) = 3=. ? = 1 Tangent line: Slope: ' = 3=; Intercept: = 1 ' 1 = = ( 3=)( 1) = 2= = 3= 2= 50. =

1 + =

0 = 0 1+1 1 + = = = (1 + = ) 2 1+1 0 = = 1 2. ? = 0 (1 + 1) 2 Tangent line: Slope: ' = 1 2; Intercept: = 1 ' 1 = 0 (1 2)(0) = 0 = 2 When = 0, =

51. 2 = 2 3 + 2 = 2 ( 2 ) = 2 2 = 2 2( 1) 1 = = 1 ? ( 1,1) 1 2(1) Tangent line: Point: ( 1, 1); Slope: 1; Intercept: = 1 ' 1 = 1 (1)( 1) = 2 = +2 52. ln( ) + 2 = 1 1 = 0 � + � + 2 (1 + 2 ) = 1 1 = (1 + 2 )


Solutions Chapter 11 Review 1 = = 1 3 ?( 1, 1) ( 1)(1 ( 1) + 2( 1)) Tangent line: Point: ( 1, 1); Slope: 1 3; Intercept: = 1 ' 1 = 1 ( 1 3)( 1) = 4 3 = 3 4 3 53. Use l'Hospital's rule: 2 2 1 lim = lim = lim = 0. 2 2 2 = 2 = = lim = =

54. Rewrite as

lim

=

,

which has the indeterminate form , so we can use l'Hospital's rule: 1 lim = lim = 0. = = 55. Use l'Hospital's rule: 2 2 1 lim 2 = lim = lim 2 = 1. 0 = 1 0 2 = 2 0 = 56. Use l'Hospital's rule several times: 3 3 2 6 6 lim lim = = lim = lim = 6. 2 0 = 1 2 0 = 1 0 = 1 0 =

57. a. O ( ) = 11.1 2 + 149.2 + 135.5, so O (1) 274 books per week b. O (7) = 636 books per week c. It would not be realistic to use the function O through week 20: It begins to decrease after = 14. Graph:

d. Since the data suggest an upward-curving parabola, the long-term prediction of sales for a quadratic model would be that sales will increase without bound, in sharp contrast to (c). 58. a. The rate of change of the lea level is given by the derivative: P ( ) = 0.0003 2 + 0.04 + 2.2 P (100) = 0.0003(100) 2 + 0.04(100) + 2.2 = 3.2 mm per year. b. P (125) = 0.0003(125) 2 + 0.04(125) + 2.2 2.5 mm per year c. Graph:


Solutions Chapter 11 Review The model predicts the sea level dropping after around 2075 (due to the fact that we chose a cubic model) with no basis in reality. d. The long-term prediction of a model depends on the choice of the type of function used in any model. For instance, a quadratic regression model would result in a curve that predicted the sea level rising without bound (because the data curve upward), whereas a logistic model would predict the sea level eventually leveling off; all with exactly the same data.

59. a. ( ) = 0.00004 + 3.2, so (8,000) = $2.88 per book b. ( ) = 0.00002 + 3.2 + 5,400 , (8,000) = $3.715\ \ per book c. ( ) = 0.00002 5,400 2, so (8,000) $0.000104\ \ per book per additional book sold d. At a sales level of 8,000 books per week, the cost is increasing at a rate of $2.88 per book (so that the 8,001st book costs approximately $2.88 to sell), and it costs an average of $3.715 per book to sell the first 8,000 books. Moreover, the average cost is decreasing at a rate of $0.000104 per book per additional book sold. 60. a. ( ) = 50 5.2, so (10) = $494.80 per intern b. ( ) = 25 5.2 + 4,000 , (10) = $644.80\ \ per intern 2 c. ( ) = 25 4,000 , so (10) $15\ \ per intern per additional intern employed d. At an employment level of 10 interns, the cost is increasing at a rate of $494.80 per additional intern, and it costs an average of $644.80 per intern to to employ the first 10 interns. Moreover, the average cost is decreasing at a rate of $15 per intern per additional intern employed.

61. a. Let !( ) be the number of books sold per week and let ( ) be the price per book. We are given that !(0) = 1,000 and ! (0) = 200 (taking = 0 as now), and that (0) = 20 and (0) = 1. Since revenue is ( ) = ( )!( ), we have (0) = (0)!(0) + (0)! (0) = ( 1)(1,000) + 20(200) = $3,000 per week (rising). b. Let !( ) be the number of books sold per week and let ( ) be the price per book. We are given that !(0) = 1,000, (0) = 20 and (0) = 1 (taking = 0 as now). We desire (0) = 5,000 and need to compute ! (0). Since ( ) = ( )!( ), we have ( 1)(1,000) + 20! (0) = 5,000 ! (0) = 300 books per week.

62. a. Let !( ) be the number of interns transferred per week and let ( ) be the charge per intern. We are given that !(0) = 5 and ! (0) = 4 (taking = 0 as now), and that (0) = 400 and (0) = 20. Since revenue is ( ) = ( )!( ), we have (0) = (0)!(0) + (0)! (0) = ( 20)(5) + 400(4) = $1,500 per week (rising). b. Let !( ) be the number of interns transferred per week and let ( ) be the charge per intern. We are given that !(0) = 5, (0) = 400 and (0) = 20 (taking = 0 as now). We desire (0) = 3,900 and need to compute ! (0). Since ( ) = ( )!( ), we have ( 20)(5) + 400! (0) = 3,900 ! (0) = 10 interns per week. 63. = ! gives = ! + ! . Thus, = ( !) = ( ! + ! ) (We can also derive this equation starting from ln = ln + ln !.)

!=

+ ! !.

64. Let be the stock price and the earnings. We are given = 100, = 50, = 1, and = 0.10. 50(1) 100(0.10) = = 40 � �= 2 12 so the P/E ratio was rising at a rate of 40 units per year.


Solutions Chapter 11 Review

65. Let be the stock price and the earnings. We are given = 100, = 1, = 0.10, and � �= 2 100(0.10) 100 = 12 Solve for to get = $110 per year.

� � = 100.

66. " = gives " = ( ) 2. Thus, " " = " ( ) = ( ) = . 67. a. The rate of increase of weekly sales is 4500( 0.55)= 0.55( 4.8) 2,475= 0.55( 4.8) ()= = (1 + = 0.55( 4.8)) 2 (1 + = 0.55( 4.8)) 2 = 6 : (6) 556 books per week. b. As A + , the expression = 0.55( 4.8) A 0, so 2,475= 0.55( 4.8) 0 lim ( ) = lim = = 0. + + (1 + = 0.55( 4.8)) 2 (1 + 0) 2 In the long term, the rate of increase of weekly sales slows to zero. 68. a. The rate at which the sea level is rising is 418(17.2)( 0.041)= 0.041 294.774= 0.041 P ( ) = (1 + 17.2= 0.041 ) 2 (1 + 17.2= 0.041 ) 2 = 100 : P (100) 2.96 mm per year. b. As A + , the expression = 0.041 A 0, so 294.774= 0.041 0 lim ( ) = lim = = 0. + + (1 + 17.2= 0.041 ) 2 (1 + 0) 2 In the long term, the rate of increase of the sea level slows to zero.

69. If Q( ) is the daily number of hits weeks into the year, then Q( ) = 1,000(1.05) . So, Q ( ) = 1,000 ln(1.05)(1.05) , and Q (52) = 1,000 ln(1.05).(1.05) 52 616.8 hits per day per week.

70. If Q( ) is the daily number of hits weeks into the year, then Q( ) = 100(1.15) . So, Q ( ) = 100 ln(1.15)(1.15) and Q (85) = 100 ln(1.15) ( (1.15) 85 2, 016.997 hits per day per day.

! ! + 2! = 0 ! 250! = 250 + 2! When = 50 and ! = 1,000, ! 250(1,000) 17.24 copies per $1. = 250(50) + 2(1,000) The demand for the gift edition of The Complete Larry Potter is dropping at a rate of about 17.24 copies per $1 71. a. 250! + 250


Solutions Chapter 11 Review increase in the price. b. Since = !, ! 1,000 + 50( 17.24) $138 per dollar increase in price. = !+ The derivative is positive, so the price should be raised.

! ! + 2! = 0 100! = ! 100 + 2! When = 40 and ! = 1,000, 100(1,000) 16.67 copies per $1. = ! 100(40) + 2(1,000) The demand for the gift edition of Lord of the Fields is dropping at a rate of about 16.67 copies per $1 increase in the price. b. Since = !, ! 1,000 + 40( 16.67) $333 per dollar increase in price. = !+ The derivative is positive, so the price should be raised. 72. a. 100! + 100


Solutions Chapter 11 Case Study Chapter 11 Case Study

1. In general, the derivative ( ) is highest when the graph of is steepest. A logistic function of the form ( ) =

$ , 1 + ( 0 )

is steepest at its point of inflection at = ( 0 ) =

$ ( 0 0 ) ln $ ln . = ) ( 2 4 0 0 (1 + )

0 , so the height of the derivative at that point would be

Thus, the heights of the individual three waves were

$1 ln 1 9,163,000 ln 1.107 232,900 cases per day, = 4 4 $2 ln 2 14,440,000 ln 1.038 134,600 cases per day, = 4 4 $2 ln 2 45,580,000 ln 1.035 392,000 cases per day. = 4 4

These results give us the peak numbers of cases per day for each of the three waves.

$ , then 1 + ( 0 ) ( ) $ ( 0 ) ln 1 + ( 0 ) $ ( 0 ) ln ln ( = = = ) ) ( ( ( 2 ( ) $ 0 ) 0 ) (1 + $(1 + ( 0 ) + 1) To the left of the point of inflection, < 0 , the exponent in the denominator is negative, meaning that decreasing increases the denominator and decreases the numerator, resulting in a slower percentage rate of change. 2. If ( ) =

$ ( 0 ) ln . (1 + ( 0 )) 2 a. The height of ( ) is the value at the point of inflection, $ ( 0 0 ) ln $ ln ( 0 ) = . = 4 (1 + ( 0 )) 2 Thus, if = 0.9 (a decrease of 10%), then the ratio of heights is $ ln ln ln(0.9 ) ln 0.9 + ln = = = $ ln ln > ln ln 0.9 0.1054 1 , = 1+ ln ln which is smaller than 1, and gives the ratio by which the height decreases. 3. The daily cases curves have the form ( ) =

b. With the change of as in part (a), the ratio of the widths is ln , ln the reciprocal of the ratio in part (a). Hence the width increases by the reciprocal of the factor by which the height decreases. c. If is decreased by percent (expressed as a decimal), then the calculation in part (a) shows that we ln(1 ) + ln 1 would require = , so ln 2


Solutions Chapter 11 Case Study

ln = 2(ln + ln(1 )) R ln = 2 ln(1 ) = ln(1 ) 2 R =

Thus, = 1

1 (1 ) 2

1 1 0.017, a 1.7% decrease in the magnitude of (to around 1.017). = 1 ; ; 1.035

4. Answers will vary.


Solutions Section 12.1 Section 12.1

1. Absolute min: ( 3, 1), relative max: ( 1, 1), relative min: (1, 0), absolute max: (3, 2)

2. Relative min: ( 3, 0), absolute max: ( 1, 2), absolute min: (1, 1), relative max: (3, 1) 3. Absolute min: (3, 1) and ( 3, 1), absolute max: (1, 2)

4. Relative max: ( 3, 0), absolute min: ( 2, 1), absolute max: (1, 1), relative min: (3, 0) 5. Absolute min: ( 3, 0) and (1, 0), absolute max: ( 1, 2) and (3, 2)

6. Absolute max: ( 3, 2), absolute min: ( 1, 1), relative max: (3, 1) 7. Relative min: ( 1, 1)

8. Relative max: ( 1, 0)

9. Absolute min: ( 3, 1), relative max: ( 2, 2), relative min: (1, 0), absolute max: (3, 3)

10. Relative min: ( 3, 1), singular nonextreme point: ( 1, 0), absolute max: (0, 2), absolute min: (2, 2), relative max: (3, 1) 11. Relative max: ( 3, 0), absolute min: ( 2, 1), stationary nonextreme point: (1, 1)

12. Relative min: ( 3, 0), relative max: ( 2, 1), absolute min: ( 1, 1), stationary nonextreme point: (1, 1) 13. Relative min: ( 3, 2), (0, 1), absolute max: ( 2, 3), absolute min: (3, 2)

14. Absolute min: ( 2, 2), (3, 2), relative max: ( 3, 1), non-extreme singular point: (0, 0)

15. Relative max: ( 3, 1), (0, 2), absolute min: ( 2, 2), absolute max: (3, 3), relative min: (1, 1) 16. Absolute max: ( 2, 3), relative max: (0, 0), absolute min: (1, 1)

17. ( ) = 2 4 + 1 with domain [0, 3], so ( ) = 2 4. ( ) = 0 when 2 4 = 0, giving = 2 ( ) is defined for all in the interior of the domain, so there are no singular points. Thus, we have a stationary point at = 2 and the endpoints = 0 and = 3.

( )

0

1

2

3

3

2

Using the Method of plotting points: A plot of these points shows that the graph must decrease from = 0 until = 2, then increase again until = 3 as shown in in the figure.


Solutions Section 12.1

This gives an absolute max at (0, 1), an absolute min at (2, 3), and a relative max at (3, 2). Using the First derivative test: Add test points on either side iof the critical point and calculate the derivative at the test poingts and critical points Type of point Endpoint Test Point Critical Point Test Point Endpoint

0

( ) = 2 4

Direction of Graph

1

2

2

0

2.5 1

3

The table suggests the shape of the curve in the above figure, and we again conclude that there is an absolute max at (0, 1), an absolute min at (2, 3), and a relative max at (3, 2). 18. ( ) = 2 2 2 + 3 with domain [0, 3], so ( ) = 4 2. ( ) = 0 when 4 2 = 0, giving = 1 2 ( ) is defined for all in the interior of the domain, so there are no singular points. Thus, we have a stationary point at = 1 2 and the endpoints = 0 and = 3.

( )

0

1/2

3

3

5/2

15

Using the Method of plotting points: A plot of these points shows that the graph must decrease from = 0 until = 1 2, then increase again until = 3.

This gives a relative max at (0, 3), an absolute min at (1 2, 5 2), and an absolute max at (3, 15). Using the First derivative test: Type of point Endpoint Test Point Critical Point Test Point Endpoint

( ) = 4 2 Direction of Graph

0

1 4

1 2

1

0

2

6

3

The table suggests the shape of the curve in the above figure, and we again conclude that there is a relative max at (0, 3), an absolute min at (1 2, 5 2), and an absolute max at (3, 15). 19. ( ) = 3 12 with domain [ 4, 4], so ( ) = 3 2 12. ( ) = 0 when 3 2 12 = 0, giving = ±2


Solutions Section 12.1 ( ) is defined for all in the interior of the domain, so there are no singular points. Thus, we have stationary points at = ±2 and the endpoints = ±4:

( )

-4

-2

2

4

-16

16

-16

16

Using the Method of plotting points: A plot of these points shows that the graph increases from = 4 until = 2, then decreases until = 2, then increases until = 4.

This gives an absolute min at ( 4, 16), an absolute max at ( 2, 16), an absolute min at (2, 16), and an absolute max at (4, 16). Using the First derivative test: Type of point Endpoint Test Point Critical Point Test Point Critical Point Test Point Endpoint

4

( ) = 3 2 12

3 16

Direction of Graph

2 0

0

2

12

0

3

15

4

The table suggests the shape of the curve in the above figure, and we again conclude that there is an absolute min at ( 4, 16), an absolute max at ( 2, 16), an absolute min at (2, 16), and an absolute max at (4, 16).

20. ( ) = 2 3 6 + 3 with domain [ 2, 2], so ( ) = 6 2 6. ( ) = 0 when 6 2 6 = 0, giving = ±1. ( ) is defined for all in the interior of the domain, so there are no singular points. Thus, we have stationary points at = ±1 and the endpoints = ±2:

( )

2 1

1 7

1

1

2

7

Using the Method of plotting points: A plot of these points shows that the graph increases from = 2 until = 1, then decreases until = 1, then increases until = 2.

This gives an absolute min at ( 2, 1), an absolute max at ( 1, 7), an absolute min at (1, 1), and an absolute max at (2, 7).


Solutions Section 12.1 Using the First derivative test: Type of point Endpoint Test Point Critical Point Test Point Critical Point Test Point Endpoint

( ) = 6 2 6 Direction of Graph

2

1.5 7.5

1 0

0

1

6

0

1.5

7.5

2

The table suggests the shape of the curve in the above figure, and we again conclude that there is an absolute min at ( 2, 1), an absolute max at ( 1, 7), an absolute min at (1, 1), and an absolute max at (2, 7).

21. ( ) = 3 + with domain [ 2, 2], so ( ) = 3 2 + 1. ( ) = 0 when 3 2 + 1 = 0, which has no solution, so there are no stationary points. ( ) is defined for all in the interior of the domain [ 2, 2], so there are no singular points. Thus, there are no critical points in the domain, just the endpoints ±2: 2

( ) 10

2

10

Using the Method of plotting points: A plot of these points shows that the graph increases from = 2 until = 2.

This gives an absolute min at ( 2, 10) and an absolute max at (2, 10). Using the First derivative test: As there are no critical points in the domain, the first derivative test does not apply, so we use the information from the plotted endpoints, with the same conclusion as above. 22. ( ) = 2 3 3 with domain [ 1, 1], so ( ) = 6 2 3. ( ) = 0 when 6 2 3 = 0, which has no solution, so there are no stationary points. ( ) is defined for all in the interior of the domain [ 1, 1], so there are no singular points. Thus, there are no critical points in the domain, just the endpoints ±1: ( )

1 5

1

5

Using the Method of plotting points: A plot of these points shows that the graph decreases from = 1 until = 1.

This gives an absolute max at ( 1, 5) and an absolute min at (1, 5).


Solutions Section 12.1 Using the First derivative test: As there are no critical points in the domain, the first derivative test does not apply, so we use the information from the plotted endpoints, with the same conclusion as above.

23. ( ) = 2 3 + 3 2 with domain [ 2, ), so ( ) = 6 2 + 6 . ( ) = 0 when 6 2 + 6 = 0, giving = 0 or = 1. ( ) is defined for all in the interior of the domain [ 2, ), so there are no singular points. Thus, we have stationary points at = 0 and = 1 and the endpoint at = 2. In addition to these we test one point to the right of = 0: 2

()

4

1 1

0

1

0

5

Using the Method of plotting points: A plot of these points shows that the graph increases from = 2 until = 1, then decreases until = 0, then increases from that point on. (Remember that = 1 is just a test point to see whether the graph is increasing or decreasing to the right of = 0.)

This gives an absolute min at ( 2, 4), a relative max at ( 1, 1), and relative min at (0, 0). Using the First derivative test: Type of point Endpoint Test Point Critical Point Test Point Critical Point Test Point 2

( )= 6 2+ 6

1.5 4.5

Direction of Graph

1

0.5

0

0

1.5

0

1

12

The table suggests the shape of the curve in the above figure, and we again conclude that there is an absolute min at ( 2, 4), a relative max at ( 1, 1), and relative min at (0, 0).

24. ( ) = 3 3 2 with domain [ 1, ), so ( ) = 3 2 6 . ( ) = 0 when 3 2 6 = 0, giving = 0 or = 2. ( ) is defined for all in the interior of the domain [ 1, ), so there are no singular points. Thus, we have stationary points at = 0 and = 2 and the endpoint at = 1. In addition to these we test one point to the right of = 2: ()

1 4

0 0

2

4

3

0

Using the Method of plotting points: A plot of these points shows that the graph increases from = 1 until = 0, then decreases until = 2, then increases from that point on.


Solutions Section 12.1

This gives an absolute min at ( 1, 4), a relative max at (0, 0), and an absolute min at (2, 4). Using the First derivative test: Type of point Endpoint Test Point Critical Point Test Point Critical Point Test Point 1

( )= 3 2 6

0.5 3.75

Direction of Graph

0

0

1

2

3

0

3

9

The table suggests the shape of the curve in the above figure, and we again conclude that there is an absolute min at ( 1, 4), a relative max at (0, 0), and an absolute min at (2, 4).

25. ( ) = 4 4 3 with domain [ 1, ), so ( ) = 4 3 12 2. ( ) = 0 when 4 3 12 2 = 0, giving = 0 or = 3. ( ) is defined for all in the interior of the domain [ 1, ), so there are no singular points. Thus, we have stationary points at = 0 and = 3 and the endpoint at = 1. In addition to these we test one point to the right of = 3:

( )

1 5

0

3

0

4

0

27

Using the Method of plotting points: A plot of these points shows that the graph increases from = 1 until = 0, continues to decrease until = 3, then increases from that point on.

This gives a relative max at ( 1, 5) and an absolute min at (3, 27). Using the First derivative test: Type of point Endpoint Test Point Critical Point Test Point Critical Point Test Point

( ) = 4 3 12 2 Direction of Graph

1

0.5

0

3.5

0

1

3

8

0

4

64

The table suggests the shape of the curve in the above figure, and we again conclude that there is a relative max at ( 1, 5) and an absolute min at (3, 27). 26. ( ) = 3 4 2 3 with domain [ 1, ), so ( ) = 12 3 6 2. ( ) = 0 when 12 3 6 2 = 0, giving = 0 or = 1 2. ( ) is defined for all in the interior of the domain [ 1, ), so there are no singular points.


Solutions Section 12.1 Thus, we have stationary points at = 0 and = 1 2 and the endpoint at = 1. In addition to these we test one point to the right of = 1 2:

1

( )

5

0

1 2

0

1 16

1

1

Using the Method of plotting points: A plot of these points shows that the graph decreases from = 1 until = 0, continues to decrease until = 1 2, then increases from that point on.

This gives a relative max at ( 1, 5) and an absolute min at (1 2, 1 16). Using the First derivative test: Type of point Endpoint Test Point Critical Point Test Point Critical Point Test Point

( ) = 12 3 6 2

1

1 2

0

Direction of Graph

3

0

1 4

1 2

3 16

0

1

1

The table suggests the shape of the curve in the above figure, and we again conclude that there is a relative max at ( 1, 5) and an absolute min at (1 2, 1 16).

1 4 2 3 1 2 with domain ( , ), so ( ) = 3 2 2 + . + 4 3 2 ( ) = 0 when 3 2 2 + = 0, giving ( 1) 2 = 0, so = 0 or = 1. ( ) is defined for all , so there are no singular points. Thus, we have stationary points at = 0 and = 1. Since we have no endpoints, we test a point to the left of = 0 and a point to the right of = 1: 27. ( ) =

1

( ) 17 12

0

0

1

1 12

2

2 3

Using the Method of plotting points: A plot of these points shows that the graph decreases until = 0, increases until = 1, then continues to increase to the right of = 1.

This gives an absolute min at (0, 0) and no other extrema.


Solutions Section 12.1 Using the First derivative test: Type of point Test Point Critical Point Test Point Critical Point Test Point ( ) =

3

2 2+

1

0

0

4

Direction of Graph

0.5

1

1 8

2

0

2

The table suggests the shape of the curve in the above figure, and we again conclude that there is an absolute min at (0, 0) and no other extrema.

28. ( ) = 3 4 16 3 + 24 2 + 1 with domain ( , ), so ( ) = 12 3 48 2 + 48 . ( ) = 0 when 12 3 48 2 + 48 = 0, giving 12 ( 2) 2 = 0, so = 0 or = 2. ( ) is defined for all , so there are no singular points. Thus, we have stationary points at = 0 and = 2. Since we have no endpoints, we test a point to the left of = 0 and a point to the right of = 2: ( )

1 44

0

1

2

17

3

28

Using the Method of plotting points: A plot of these points shows that the graph decreases until = 0, increases until = 2, then continues to increase to the right of = 2.

This gives an absolute min at (0, 1) and no other extrema. Using the First derivative test: Type of point Test Point Critical Point Test Point Critical Point Test Point ( ) = 12 3 48 2 + 48 Direction of Graph

1

0

08

0

1

12

2 0

3

36

The table suggests the shape of the curve in the above figure, and we again conclude that there is an absolute min at (0, 1) and no other extrema.

29. ( ) = ( 1) 2 3 with domain [0, 2]. Endpoints: 0, 2 2 2 ( ) = ( 1) 1 3 = 3 3( 1) 1 3 2 Stationary points: ( ) = 0 when = 0, which is impossible, so there are no stationary points. 3( 1) 1 3 Singular points: ( ) is undefined at the interior point = 1, so we have a singular point at = 1.


Solutions Section 12.1

( )

0

1

2

1

0

1

Using the Method of plotting points:

This gives absolute maxima at (0, 1) and (2, 1) and an absolute min at (1, 0). Using the First derivative test: Type of point Endpoint Test Point ( ) =

0

2 3( 1) 1 3

Direction of Graph

0.5

Critical Point 1

0.41997 \text{Undefined}

Test Point Endpoint 1.5

0.41997

2

The table suggests the shape of the curve in the above figure, and we again conclude that there are absolute maxima at (0, 1) and (2, 1) and an absolute min at (1, 0). 30. ( ) = ( + 1) 2 5 with domain [ 2, 0]. Endpoints: 2, 0 2 2 ( ) = ( + 1) 3 5 = 5 5( + 1) 3 5 2 Stationary points: ( ) = 0 when = 0, which is impossible, so there are no stationary points. 5( + 1) 3 5 Singular points: ( ) is undefined at the interior point = 1, so we have a singular point at = 1.

2

( )

1

0

1 0

1

Using the Method of plotting points:

This gives absolute maxima at ( 2, 1) and (0, 1) and an absolute min at ( 1, 0). Using the First derivative test: Type of point Endpoint Test Point ( ) =

2 5( + 1) 3 5

Direction of Graph

2

1.5

Critical Point 1

0.2639 \text{Undefined}

Test Point Endpoint 0.5

0.2639

0

The table suggests the shape of the curve in the above figure, and we again conclude that there are absolute


Solutions Section 12.1 maxima at ( 2, 1) and (0, 1) and an absolute min at ( 1, 0).

2 + ( + 1) 2 3 with domain ( , 0]. Endpoint: = 0 3 2 2 2 2 ( ) = + ( + 1) 1 3 = + 3 3 3 3( + 1) 1 3 Stationary points: ( ) = 0 when 2 2 + = 0 3 3( + 1) 1 3 2 2 = 3 3( + 1) 1 3 Cross-multiply: 6( + 1) 1 3 = 6 ( + 1) 1 3 = 1 ( + 1) = ( 1) 3 = 1 = 2 So, we have a stationary point at = 2. Singular points: ( ) is undefined when = 1, so we have a singular point at = 1. Thus we have an endpoint at = 0, a stationary point at = 2, and a singular point at = 1. In addition we use a test point to the left of = 2: 31. ( ) =

3

2

1

( ) 0.41 1 3 2 3

0

1

Using the Method of plotting points: Notice that the singular point ( 1, 2 3) is not an absolute minimum because the graph eventually gets lower on the left.

From the graph we see that we have a relative maximum at ( 2, 1 3), a relative minimum at ( 1, 2 3), and an absolute maximum at (0, 1). Using the First derivative test: Type of point Test Point Critical Point Test Point Critical Point Test Point Endpoint ( ) =

2 2 + 3 3( + 1) 1 3

Direction of Graph

3

0.13753

2 0

1.5

0.17328

1

Unde ned

0.5

1.50661

0

The table suggests the shape of the curve in the above figure, and we again conclude that there is a relative maximum at ( 2, 1 3), a relative minimum at ( 1, 2 3), and an absolute maximum at (0, 1).

2 ( 1) 2 5 with domain [0, ). Endpoint: = 0 5 2 2 2 2 ( ) = ( 1) 3 5 = 5 5 5 5( 1) 3 5 Stationary points: ( ) = 0 when 32. ( ) =


Solutions Section 12.1 2 2 = 0 5 5( 1) 3 5 2 2 = 5 5( 1) 3 5 Cross-multiply: 10( 1) 3 5 = 10 ( 1) 3 5 = 1 ( 1) = 1 5 3 = 1 = 2 So we have a stationary point at = 2. Singular points: ( ) is undefined when = 1, so we have a singular point at = 1. Thus we have an endpoint at = 0, a stationary point at = 2, and a singular point at = 1. In addition we use a test point to the right of = 2:

( )

0

1

1

2

3

2 5 1 5 0.12

Using the Method of plotting points: Notice that the singular point (1, 2 5) is not an absolute maximum because the graph eventually gets higher on the right.

From the graph we see that we have an absolute minimum at (0, 1), a relative maximum at (1, 2 5), and a relative minimum at (2, 1 5). Using the First derivative test: Type of point Endpoint Test Point Critical Point Test Point Critical Point Test Point ( ) =

2 2 5 5( 1) 3 5

Direction of Graph

0

0.5

1.00629

1

Unde ned

1.5

2

0.20629

0

3

0.1361

The table suggests the shape of the curve in the above figure, and we again conclude that there is an absolute minimum at (0, 1), a relative maximum at (1, 2 5), and a relative minimum at (2, 1 5). 33. ( ) =

2

+ 1

; 2 2, ±1

2 1 2

2 ( 1) 2 ( 2 + 1) 4 = 2 2 2 ( 1) ( 1) 2 ( ) = 0 when = 0; ( ) is not defined when = ±1, but these points are not in the domain of , so there are no singular points. Thus, we have a stationary point at = 0 and the endpoints at = ±2. We also test points on either side of = ±1 to see how behaves near these points where it goes to ± (that is, it has vertical asymptotes there, because the denominator goes to 0): ( ) =


2

( ) 5 3

3 2 1 2 13 5 5 3

0

1

Solutions Section 12.1

1 2

3 2

5 3 13 5

2

5 3

Using the Method of plotting points: The graph increases from = 2, approaching the vertical asymptote at = 1; on the other side of the asymptote it increases until = 0 and then decreases as it approaches another vertical asymptote at = 1; on the other side of the second asymptote it decreases until = 2.

This gives a relative min at ( 2, 5 3), a relative max at (0, 1), and a relative min at (2, 5 3). Using the First derivative test: As ( ) is undefined at = ±1 we can use test points on either side of these as we did in the method of plotting points to determine the direction of the curve there. Type of point EndPt Test Pt 4

( ) =

( 2 1) 2

2

Direction of Graph

3 2

96 25

1

Test Pt Critical Pt Test Pt 1 2 32 9

0

0

1 2

32 9

1

Test Pt EndPt 3 2

96 25

2

The table suggests the shape of the curve in the above figure, and we again conclude that there is a relative min at ( 2, 5 3), a relative max at (0, 1), and a relative min at (2, 5 3). 34. ( ) =

2

1

, with domain [ 2, 2]

2+ 1 2

2 ( + 1) 2 ( 2 1) 4 = ( 2 + 1) 2 ( 2 + 1) 2 ( ) = 0 when = 0; ( ) is defined for all . Thus, we have a stationary point at = 0 and the endpoints at = ±2: ( ) =

2

( ) 3 5

0

1

2

3 5

Using the Method of plotting points: The graph decreases from = 2 until = 0, then increases until = 2.

This gives an absolute max at ( 2, 3 5), an absolute min at (0, 1), and an absolute max at (2, 3 5).


Solutions Section 12.1 Using the First derivative test: Type of point Endpoint Test Point Critical Point Test Point Endpoint ( ) =

4 ( 2 + 1) 2

2

1

0

Direction of Graph

1

0

1

2

1

The table suggests the shape of the curve in the above figure, and we again conclude that there is an absolute max at ( 2, 3 5), an absolute min at (0, 1), and an absolute max at (2, 3 5).

35. ( ) = ( 1); 0 3 1 ( ) = 1 2 1 2 2 2 3 1 2 1 1 2 ( ) = 0 when = 0, or 3 1 = 0, giving = 1 3. 2 2 ( ) is not defined at = 0. However, = 0 is not an interior point of the domain [0, ) of ; it is an endpoint. Thus, = 0 does not count as a singular point; there are no singular points. So, we have a stationary point at = 1 3 and an endpoint at = 0. In addition, we test a point to the right of = 1 3:

( )

0 0

1/3

1

2 3 9

0

Using the Method of plotting points: The graph decreases from = 0 until = 1 3, then increases from that point on.

This gives a relative max at (0, 0) and an absolute min at (1 3, 2 3 9). Using the First derivative test: Type of point Endpoint Test Point Critical Point Test Point ( ) =

3 1 2 1 1 2 2 2

Direction of Graph

0

1 4

1 3

1 4

0

1

1

The table suggests the shape of the curve in the above figure, and we again conclude that there is a relative max at (0, 0) and an absolute min at (1 3, 2 3 9). 36. ( ) = ( + 1); 0 3 1 ( ) = 1 2 + 1 2 2 2 3 1 2 1 1 2 ( ) = 0 when + = 0, or 3 + 1 = 0, giving = 1 3, but this is not a valid solution 2 2


Solutions Section 12.1 ( ( 1 3) is not defined) and is not in the domain of anyway, so there are no stationary points. ( ) is not defined at = 0. However, = 0 is not an interior point of the domain [0, ) of ; it is an endpoint. Thus, = 0 does not count as a singular point; there are no singular points. So, we have only the endpoint = 0 and no critical points. We test one point to the right of = 0:

( )

0

1

0

2

Using the Method of plotting points: The graph increases from = 0 on.

Thus, there is an absolute min at (0, 0) and no other extrema.Using the Method of plotting points: The graph decreases from = 0 until = 1 3, then increases from that point on.

This gives a relative max at (0, 0) and an absolute min at (1 3, 2 3 9). Using the First derivative test: As there are no critical points in the domain, the first derivative test does not apply, so we use the information from the plotted endpoints, with the same conclusion as above.

37. ( ) = 2 4 Since the domain of is not specified, we take it to be the set of all for which is defined, which is [0, ). 2 ( ) = 2 2 2 ( ) = 0 when 2 , giving = 1, or 3 2 = 1, which implies that = 1. = 0, or 2 = ( ) is not defined at = 0. However, = 0 is not an interior point of the domain [0, ) of ; it is an endpoint. Thus, = 0 does not count as a singular point; there are no singular points. So, we have only the endpoint = 0 and a stationary point at = 1. We test one point to the right of = 1:

( )

0

0

1

3

2

1.7

Using the Method of plotting points: The graph decreases from = 0 to = 1, then increases from that point on.

Thus, there is a relative max at (0, 0) and an absolute min at (1, 3).


Solutions Section 12.1 Using the First derivative test: Type of point Endpoint Test Point Critical Point Test Point ( ) = 2

2

0

1

1.82

Direction of Graph

0.5

2

0

2.59

The table suggests the shape of the curve in the above figure, and we again conclude that there is a relative max at (0, 0) and an absolute min at (1, 3).

1 1 2 Since the domain of is not specified, we take it to be the set of all for which is defined, which is 0. 1 2 ( ) = 2 + 3 1 2 1 2 ( ) = 0 when 2 + 3 = 0, or 2 = 3 , giving 2 2 = 3, which implies that = 2 (the solution = 0 is rejected as 0 is not in the domain). ( ) is not defined at = 0, but = 0 is not in the domain of ; there are no singular points. Further, the domain has no endpoints either. Thus, we have only one stationary point, at = 2. In addition, we test two points to the left of = 0 and one on either side of = 2: 38. ( ) =

2

( ) 3 4

1

1

2

0

2

1 4

3

2 9

Using the Method of plotting points: The graph is decreasing approaching the vertical asymptote at = 0. On the other side of the asymptote it increases until = 2, then decreases from that point on.

Thus, there is an absolute max at (2, 1 4). Using the First derivative test: As ( ) is undefined at = 0 we can use test points on either side of 0 as we did in the method of plotting points to determine the direction of the curve there. Type of point Test Point Undefined Test Point Critical Point Test Point

1 2 ( ) = 2 + 3

Direction of Graph

1 3

0

1 1

2 0

3

1 27

The table suggests the shape of the curve in the above figure, and we again conclude that there is an absolute max at (2, 1 4).


Solutions Section 12.1 39. is given by 3 ( ) = 2 + 3 The natural domain of is ( , ). 3 2( 2 + 3) 3(2 ) 4 + 9 2 ( ) = = ( 2 + 3) 2 ( 2 + 3) 2 4 2 + 9 ( ) = 0 when = 0, or 4 + 9 2 = 0, giving 2( 2 + 9) = 0, which implies that = 0. ( 2 + 3) 2 ( ) is always defined, so there are no singular points. Further, the domain has no endpoints either. Thus, we have only one stationary point, at = 0. In addition, we test a point on either side:

1

( ) 1 4

0

1

0

1 4

Using the Method of plotting points: The graph is always increasing.

Thus, there are no relative extrema. Using the First derivative test: Type of point Test Point Critical Point Test Point ( ) =

4 + 9 2 ( 2 + 3) 2

Direction of Graph

0

1

5 8

1

0

5 8

The table suggests the shape of the curve in the above figure, and we again conclude that there are no relative extrema.

3 2 3 The natural domain of is all ± 3. 3 2( 2 3) 3(2 ) 4 9 2 ( ) = = ( 2 3) 2 ( 2 3) 2 4 2 2 2 ( ) = 0 when 9 = 0, or ( 9) = 0, giving = ±3 or = 0. ( ) is defined for all in the domain of , so there are no singular points. Further, the domain has no endpoints either. Thus, we have three stationary points, at = 3, 0, and 3. In addition, we test points to the left of 3, to the right of 3, and on either side of the asymptotes at = ± 3: 40. ( ) =

4

3

( ) 64 13 9 2

2 8

1

1 2

0

0

1

1 2

2

8

3

4

9 2 64 13

Using the Method of plotting points: The graph rises until = 3, then decreases approaching the asymptote at = 3; on the other side of


Solutions Section 12.1 the asymptote it decreases approaching the second asymptote at = 3; on the other side of the second asymptote it decreases until = 3 and then increases.

Thus, there is a relative max at ( 3, 9 2) and a relative min at (3, 9 2). Using the First derivative test: As ( ) is undefined at = ±1 we can use test points on either side of these as we did in the method of plotting points to determine the direction of the curve there. Type of point Test Point Critical Point Test Point Undefined

( ) =

4 9 2 ( 2 3) 2

Direction of Graph

4

3

0.66

2

3

0

20

Type of point Test Point Critical Point Test Point Undefined ( ) =

4 9 2 ( 2 3) 2

Direction of Graph

1

0

2

1

3

0

2

Type of point Test Point Critical Point Test Point ( ) =

4 9 2 ( 2 3) 2

Direction of Graph

2

3

4

20

0

0.66

The tables suggests the shape of the curve in the above figure, and we again conclude that there is a relative max at ( 3, 9 2) and a relative min at (3, 9 2).


Solutions Section 12.1 41. ( ) = ln with domain (0, ) 1 ( ) = 1 1 ( ) = 0 when 1 = 0, so = 1. ( ) is defined for all in the domain of , so there are no singular points. Further, the domain has no endpoints either. We test the one stationary point at = 1 and a point on either side: 1 2

( ) 1.19

1

2

1

1.31

Using the Method of plotting points: The graph decreases until = 1 and then increases from that point on.

Thus, there is an absolute min at (1, 1). Using the First derivative test: Type of point Test Point Critical Point Test Point ( ) = 1

1

1 2

1

1

Direction of Graph

0

2

1 2

The table suggests the shape of the curve in the above figure, and we again conclude that there is an absolute min at (1, 1).

42. ( ) = ln 2 with domain (0, ) 2 ( ) = 1 2 ( ) = 0 when 1 = 0, so = 2. ( ) is defined for all in the domain of , so there are no singular points. Further, the domain has no endpoints either. We test the one stationary point at = 2 and a point on either side:

( )

1

1

2

0.61

3

0.80

Using the Method of plotting points: The graph decreases until = 2 and then increases from that point on.


Solutions Section 12.1 Thus, there is an absolute min at (2, 2 ln 4). Using the First derivative test: Type of point Test Point Critical Point Test Point ( ) = 1

2

1

2

0

1

Direction of Graph

3

1 3

The table suggests the shape of the curve in the above figure, and we again conclude that there is an absolute min at (2, 2 ln 4).

43. ( ) = with domain [ 1, 1] ( ) = 1 ( ) = 0 when 1 = 0, or = 1, giving = 0. ( ) is defined for all in the domain of , so there are no singular points. Endpoints: 1, 1 We need to test only the stationary point and the endpoints: 0

1

( ) 1.37

1

1

1.72

Using the Method of plotting points: The graph decreases from = 1 to = 0 and then increases to = 1.

Thus, there is a relative max at ( 1, 1 + 1), an absolute min at (0, 1), and an absolute max at (1, 1). Using the First derivative test: Type of point Endpoint Test Point Critical Point Test Point Endpoint 1

( ) = 1

Direction of Graph

0.5

0

0.39

0

0.5

0.65

1

The table suggests the shape of the curve in the above figure, and we again conclude that there is a relative max at ( 1, 1 + 1), an absolute min at (0, 1), and an absolute max at (1, 1). 2

44. ( ) = with domain ( , ( ) = 2

2

2

)

( ) = 0 when 2 = 0, giving = 0. ( ) is defined for all in the domain of , so there are no singular points. No endpoints We test the stationary point at = 0 and a point on either side:


1

( ) 0.37

0

Solutions Section 12.1

1

1

0.37

Using the Method of plotting points: The graph increases until = 0 and then decreases.

Thus, there is an absolute max at (0, 1). Using the First derivative test: Type of point Test Point Critical Point Test Point ( ) = 2

2

0

1

0.32

1

0

0.74

Direction of Graph

The table suggests the shape of the curve in the above figure, and we again conclude that there is an absolute max at (0, 1).

2 2 24 + 4 The natural domain of is all 4. 4 ( + 4) (2 2 24) 2 2 + 16 + 24 ( ) = = ( + 4) 2 ( + 4) 2 ( ) = 0 when 2 2 + 16 + 24 = 0 2( + 2)( + 6) = 0 = 2 or = 6. ( ) is defined for all in the domain of , so there are no singular points. Also, there are no endpoints, so we have only the stationary points at = 2 and = 6. We test points on either side of the stationary points and between them and the asymptote at = 4: 45. ( ) =

7

6

( ) 24.7 24

5

26

3 6

2 8

1

7.3

Using the Method of plotting points: The graph increases to = 6 and then decreases approaching the asymptote at = 4. On the other side of the asymptote the graph decreases to = 2 and then increases again.

Thus, there is a relative max at ( 6, 24) and a relative min at ( 2, 8). Using the First derivative test: As ( ) is undefined at = 4 we can use test points on either side of 4 as we did in the method of


Solutions Section 12.1 plotting points to determine the direction of the curve there. Type of point Test Pt Critical Pt Test Pt ( ) =

2 2 + 16 + 24 ( + 4) 2

7

1.11

Direction of Graph

6

5

0

6

4

Test Pt Critical Pt Test Pt 3

2

0

6

1

1.11

The table suggests the shape of the curve in the above figure, and we again conclude that there is a relative max at ( 6, 24) and a relative min at ( 2, 8).

4 2 + 20 The natural domain of is the set of all real numbers. ( 2 + 20) ( 4)(2 ) 2 + 8 + 20 ( ) = = ( 2 + 20) 2 ( 2 + 20) 2 ( ) = 0 when 2 + 8 + 20 = 0 ( + 2)( 10) = 0 = 2 or = 10. ( ) is defined for all , so there are no singular points. Also, there are no endpoints, so we have only the stationary points at = 2 and = 10. We test the two stationary points and a point on either side: 46. ( ) =

3

10

2

11

( ) 0.24 0.25 0.05 0.0496 Using the Method of plotting points: The graph decreases to = 2, then increases to = 10, then decreases from that point on.

Thus, there is an absolute min at ( 2, 1 4) and an absolute max at (10, 1 20). Using the First derivative test: Type of point Test Point Critical Point Test Point Critical Point Test Point ( ) =

2 + 8 + 20 ( 2 + 20) 2

Direction of Graph

3

2

0.02

0

0

0.05

10 0

11

0.0007

The table suggests the shape of the curve in the above figure, and we again conclude that there is an absolute min at ( 2, 1 4) and an absolute max at (10, 1 20). 2

47. ( ) = 1 The natural domain of is the set of all real numbers.


1 2

1 2

2

Solutions Section 12.1

( ) = + ( 2 ) = (1 2 2) 1 ( ) = 0 when 1 2 2 = 0, giving = ±1 2. ( ) is defined for all , so there are no singular points. Also, there are no endpoints, so we have only the stationary points at = ±1 2. We test the two stationary points and a point on either side:

( )

1

1

1 2 1 2 1.17

1.17

1

1

Using the Method of plotting points: The graph decreases until = 1 2, increases until = 1 2, then decreases again.

Thus, there is an absolute max at (1 2, 2), and an absolute min at ( 1 2, 2). Using the First derivative test: Type of point Test Point Critical Point Test Point Critical Point Test Point

( ) = (1 2 2) 1

1

1 2

1

2

Direction of Graph

0

0

1 2 0

1

1

The table suggests the shape of the curve in the above figure, and we again conclude that there is an absolute max at (1 2, 2), and an absolute min at ( 1 2, 2).

48. ( ) = ln with domain (0, ) ( ) = ln + 1 ( ) = 0 when ln + 1 = 0, giving ln = 1, so that = 1 = 1 . ( ) is defined for all in the domain of , so there are no singular points. Also, there are no endpoints, so we have only the stationary point at = 1 . We test the stationary point and a point on either side:

0.1

1

( ) 0.23 0.37

1

0

Using the Method of plotting points: The graph decreases until = 1 and then increases from that point on. [Note: You can evaluate the limit ln lim 0 + ( ) using l'Hospital's rule by writing ( ) = .] 1

Thus, there is an absolute min at (1 , 1 ). Using the First derivative test:


Solutions Section 12.1 Type of point Test Point Critical Point Test Point

( ) = ln + 1 Direction of Graph

0.1

1

1.3

0

1

1

The table suggests the shape of the curve in the above figure, and we again conclude that there is an absolute min at (1 , 1 ). 1 49. = 2 + 2 , and so = 2

1 . ( 2) 2

The graphs of and of look like this:

Looking closely at the graph of , we can see three places where it crosses the -axis. Zooming in we can locate these points approximately as 0.15, 1.40, and 2.45. Thus, we have relative minima at (0.15, 0.52) and (2.45, 8.22) and a relative maximum at (1.40, 0.29). 50. = 2 10( 1) 2 3, and so ( 1) 1 3. = 2 20 3

The graphs of and of look like this:

The graph of crosses the -axis in two places; we can zoom in and approximate these points as 2.25 and 2.76. This give us two stationary points of . We also have a singular point at = 1. Thus, we have an absolute min at ( 2.25, 16.88), an absolute max at (1, 1), and a relative min at (2.76, 6.96). 51. ( ) = ( 5) 2( + 4)( 2) with domain [ 5, 6], and so ( ) = 2( 5)( + 4)( 2) + ( 5) 2( 2) + ( 5)2( + 4) = ( 5)[2( + 4)( 2) + ( 5)( 2) + ( 5)( + 4)] = ( 5)(4 2 4 26). The graphs of and of look like this:

The graph of crosses the -axis in three places; hence has three stationary points. Zooming in we can approximate these as 2.10, 3.10, and = 5. Substituting these and the endpoints 5 and 6 into , we find that we have an absolute maximum at ( 5, 700), relative maxima at (3.10, 28.19) and (6, 40), an absolute minimum at ( 2.10, 392.69), and a relative minimum at (5, 0).


Solutions Section 12.1 52. ( ) = ( + 3) 2( 2) 2 with domain [ 5, 5], and so ( ) = 2( + 3)( 2) 2 + 2( + 3) 2( 2) = 2( + 3)( 2)( 2 + + 3) = 2( + 3)( 2)(2 + 1) The graphs of and of look like this:

The graph of crosses the -axis in three places; hence has three stationary points. Zooming in (or solving ( ) = 0, which is easy in this case), we see that the three stationary points are = 3, = 0.5, and = 2. Substituting these and the endpoints into we see that we have an absolute maximum at (5, 576), relative maxima at ( 5, 196) and ( 0.50, 39.06), and absolute minima at ( 3, 0) and (2, 0). 53. The derivative is zero at = 1. To the left of = 1 the derivative is negative, indicating that the graph of is decreasing. To the right of = 1 the derivative is positive, indicating that the graph of is increasing. Thus, has a stationary minimum at = 1.

54. The derivative is zero at = 1. To the left of = 1 the derivative is positive, indicating that the graph of is increasing. To the right of = 1 the derivative is negative, indicating that the graph of is decreasing. Thus, has a stationary maximum at = 1. 55. Stationary minima at = 2 and = 2, stationary maximum at = 0 (see the solution to Exercise 49). 56. Stationary nonextremum at = 1 (the graph of is decreasing both to the left and right of = 1) stationary minimum at = 2 (see the solution to Exercise 50).

57. Singular minimum at = 0. At = 1, the derivative of is also zero, but to both the left and right of = 0 the derivative is positive, indicating that the graph of is increasing on both sides of = 0. Thus, has a stationary nonextreme point at = 1. 58. Singular minimum at = 0, stationary maximum at = 1

59. Stationary minimum at = 2, singular nonextreme points at = 1 and = 1, stationary maximum at = 2

60. Singular nonextreme point at = 1, stationary maximum at = 0, singular minimum at = 1

61. a. The EVT applies as is continuous on the closed interval [ 3, 3]. b. Endpoints and critical points: Type Endpoint Critical Point Critical Point Endpoint

( )

3 1

1 1

1

0

3

2

Absolute minimum at ( 3, 1), absolute maximum at (3, 2).

62. a. The EVT applies as is continuous on the closed interval [ 3, 3]. b. Endpoints and critical points:


Solutions Section 12.1 Type Endpoint Critical Point Critical Point Endpoint

( )

3 0

1

1 2

1

3

1

Absolute maximum at ( 1, 2), absolute minimum at (1, 1)

63. a. The EVT applies as is continuous on the closed interval [ 3, 3]. b. Endpoints and critical points: Type Endpoint Critical Point Endpoint

( )

3 1

1 2

3

1

Absolute minima at ( 3, 1) and (3, 1), absolute maximum at (1, 2)

64. a. The EVT applies as is continuous on the closed interval [ 3, 3]. b. Endpoints and critical points: Type Endpoint Critical Point Critical Point Endpoint

( )

3 0

1

2

1

1

3 0

Absolute minimum at ( 2, 1), absolute maximum at (1, 1)

65. a. The EVT applies as is continuous on the closed interval [ 3, 3]. b. Endpoints and critical points: Type Endpoint Critical Point Critical Point Endpoint

( )

3 0

1

1 2

0

3

2

Absolute minima at ( 3, 0) and (1, 0), absolute maxima at ( 1, 2) and (3, 2) 66. a. The EVT applies as is continuous on the closed interval [ 3, 3]. b. Endpoints and critical points: Type Endpoint Critical Point Endpoint

( )

3 2

1 1

3 1

Absolute maximum at ( 3, 2), absolute minimum at ( 1, 1)

67. a. The EVT does not apply as the domain of is not a closed interval. b. No endpoints. The only critical point occurs at ( 1, 1), which is not an absolute extremum (it is a relative minimum). 68. a. The EVT does not apply as the domain of is not a closed interval.


Solutions Section 12.1 b. No endpoints. The only critical point occurs at ( 1, 0), which is not an absolute extremum (it is a relative maximum). 69. a. The EVT applies as is continuous on the closed interval [ 3, 3]. b. Endpoints and critical points: Type Endpoint Critical Point Critical Point Endpoint

( )

3 1

2 2

1

0

3

3

Absolute minimum at ( 3, 1), absolute maximum at (3, 3)

70. a. The EVT applies as is continuous on the closed interval [ 3, 3]. b. Endpoints and critical points: Type Endpoint Critical Point Critical Point Critical Point Endpoint

( )

3 1

1 0

0

2

2

2

Absolute maximum at (0, 2), absolute minimum at (2, 2)

3

1

71. a. The EVT does not apply as the domain of is not a closed interval. b. Endpoints and critical points: Type Endpoint Critical Point Critical Point

( )

3 0

2 1

Absolute minimum at ( 2, 1)

1 1

72. a. The EVT does not apply as the domain of is not a closed interval. b. Endpoints and critical points: Type Endpoint Critical Point Critical Point Critical Point

( )

3 0

2 1

Absolute minimum at ( 1, 1)

1

1

1

1

73. a. The EVT does not apply as is not continuous. b. Endpoints and critical points: Type Endpoint Critical Point Critical Point Endpoint

( )

3 2

2 3

0

1

3

2

Absolute maximum at ( 2, 3), absolute minimum at (3, 2)


Solutions Section 12.1 74. a. The EVT does not apply as is not continuous. b. Endpoints and critical points: Type Endpoint Critical Point Critical Point Endpoint

( )

3

0

2

1

3

0

2

2

Absolute minima at ( 2, 2) and (3, 2), no absolute maximum. 75. a. The EVT does not apply as is not continuous. b. Endpoints and critical points:

Type Endpoint Critical Point Critical Point Critical Point Endpoint

( )

3

0

2

1

1

2

2

1

Absolute minimum at ( 2, 2), absolute maximum at (3, 3).

3

3

76. a. The EVT does not apply as is not continuous. b. Endpoints and critical points: Type Critical Point Critical Point Critical Point

( )

2 3

0

0

1

1

Absolute maximum at ( 2, 3), absolute minimum at (1, 1). 77. Answers will vary.

78. Answers will vary.

79. Answers will vary.

80. Answers will vary.

81. Not necessarily; it could be neither a relative maximum nor a relative minimum, as in the graph of = 3 at the origin.

82. Not necessarily, both could be relative maxima, as in the case of = 2; 1 1.


Solutions Section 12.1 83. Answers will vary.

84. Answers will vary.

85. The graph oscillates faster and faster above and below zero as it approaches the endpoint at 0, so 0 cannot be either a relative minimum or maximum. Here is the graph:

86. a. The graph appears to level off to a horizontal line as 0, and seems to possess a horizontal tangent line there (although it continues to oscillate) suggesting that has a stationary point at = 0. If we use technology to estimate the derivative at = 0, this further confirms our suspicion that (0) = 0. b. Even though the graph appears to level off toward = 0, it oscillates faster and faster above and below zero as it approaches that point, so 0 cannot be either a relative minimum or maximum. See the graph:


Solutions Section 12.2 Section 12.2

1. Solve the constraint equation + = 10 for = 10 and substitute in the objective: = (10 ) = 10 2. Stationary points: Set ( ) = 0 and solve for : ( ) = 10 2 , so ( ) = 0 when = 5. Since the graph of ( ) is a parabola opening downward, = 5 must be its vertex, hence gives the maximum. The corresponding value of is = 10 = 5. Hence, = = 5 and = 25.

2. Solve the constraint equation + 2 = 40 for = 40 2 and substitute in the objective: = (40 2 ) = 40 2 2. Stationary points: Set ( ) = 0 and solve for : ( ) = 40 4 , so ( ) = 0 when = 10. Since the graph of ( ) is a parabola opening downward, = 10 must be its vertex, hence gives the maximum. The corresponding value of is = 40 2 = 20. Hence, = 20, = 10, and = 200.

3. Solve the constraint equation = 9 for = 9 and substitute in the objective: = + 9 with domain > 0. Stationary points: Set ( ) = 0 and solve for : ( ) = 1 9 2, so ( ) = 0 when = 3 ( = 3 is not in the domain). Testing points on either side of = 3, we see that we have the minimum. Hence, = = 3 and = 6.

4. Solve the constraint equation = 2 for = 2 and substitute in the objective: = + 4 with domain > 0. Stationary points: Set ( ) = 0 and solve for : ( ) = 1 4 2, so ( ) = 0 when = 2 ( = 2 is not in the domain). Testing points on either side of = 2, we see that we have the minimum. Hence, = 2, = 1 and = 4.

5. Solve the constraint equation + 2 = 10 for = 10 2 and substitute in the objective: ( ) = (10 2 ) 2 + 2 = 100 40 + 5 2. Stationary points: Set ( ) = 0 and solve for : ( ) = 40 + 10 , so ( ) = 0 when = 4. Since the graph of is a parabola opening upward, = 4 must be its vertex, hence gives the minimum. Hence, = 2, = 4, and = 20. 6. Solve the constraint equation 2 = 16 for 2 = 16 and substitute in the objective: ( ) = 2 + 16 . Notice that, since = 16 2, we must always have > 0, so this is the domain of . Stationary points: Set ( )= 0 and solve for : ( ) = 2 16 2, so ( ) = 0 when 2 16 2 = 0, giving 2 = 16 2 3 = 8, = 2. Substituting points on either side of = 2, we see that we have the minimum. Hence, = 2, = ±2 2, and = 12.

7. Since appears in both constraints, we can solve for the other two variables in terms of : = 30 and = 30 . Substitute: = (30 ) (30 ) = (30 ) 2. Since all the variables must be nonnegative, we have 0 30 as the domain. Stationary points: Set ( )= 0 and solve for : ( ) = (30 ) 2 2 (30 ) = (30 )(30 2 ) = (30 )(30 3 ). ( ) = 0 when = 10 or = 30. Substituting these values and the other endpoint = 0 into , we find that the maximum occurs when = 10. Thus, = 20, = 10, = 20, and = 4,000.


Solutions Section 12.2 8. Since appears in both constraints, we can solve for the other two variables in terms of : = 12 and = 12 . Substitute: = (12 ) 2. Since all the variables must be nonnegative, we have 0 12 as the domain. Stationary points: Set ( ) = 0 and solve for : ( ) = (12 ) 2 2 (12 ) = (12 )(12 2 ) = (12 )(12 3 ). ( ) = 0 when = 4 or = 12. Substituting these values and the other endpoint = 0 into , we find that the maximum occurs when = 4. Hence, = 8, = 8, = 4 and = 256.

9. Let and be the dimensions. Then we want to maximize = subject to 2 + 2 = 20. Solve for = 10 and substitute in the objective: = (10 ) = 10 2. Stationary points: Set ( ) = 0 and solve for : ( ) = 10 2 , so ( ) = 0 when = 5. Since the graph of ( ) is a parabola opening downward, = 5 must be its vertex, hence gives the maximum. The corresponding value of is = 10 = 5. Hence, the rectangle should have dimensions 5 × 5. 10. Let and be the dimensions. Then we want to minimize = 2 + 2 subject to = 100, with > 0 and > 0. Solve for = 100 and substitute in the objective: = 2 + 200 . Stationary points: Set ( ) = 0 and solve for : ( ) = 2 200 2, so ( ) = 0 when = 10 ( = 10 is not in the domain). Substituting points on either side of = 10, we find that the minimum does occur at = 10. The corresponding value of is = 100 = 10. Hence, the rectangle should have dimensions 10 × 10.

20 11. ( ) = 20 + 5,600 + 0.05 2, so the average cost function is ( ) = + 5,600 + 0.05 . 20 ( ) = 2 + 0.05 Stationary points: 20 2 + 0.05 = 0 20 = 0.05 2 20 2 = = 400, 0.05 giving = 400 = 20 ads. (The solution = 400 = 20 is outside the domain.) There are no other critical points. We create a table showing the values of ( ) at the endpoints 10 and 50 and the critical point:

10

20

50

( ) 5,602.5 5,602 5,602.9 By the Extreme Value Theorem, the average cost has an absolute minimum value of $5,602 thousand dollars per ad when = 20 ads.

490 12. ( ) = 490 + 320 + 0.001 2, so the average cost function is ( ) = + 320 + 0.001 . 490 ( ) = 2 + 0.001


Solutions Section 12.2 Stationary points: 490 2 + 0.001 = 0 490 = 0.001 2 490 2 = = 490, 000, 0.001 giving = 490,000 = 700 ads. (The solution = 490,000 = 700 is outside the domain.) There are no other critical points. We create a table showing the values of ( ) at the endpoints 100 and 1,000 and the critical point: 100

700

1,000

( ) 325 321.4 321.49 By the Extreme Value Theorem, the average cost has an absolute minimum value of approximately $321 billion dollars per ad when = 700 ads. 13. ( ) = 400,000 + 590 + 0.001 2, so the average cost function is ( ) =

400,000 + 590 + 0.001 .

14. ( ) = 800,000 + 510 + 0.0005 2, so the average cost function is ( ) =

800,000 + 510 + 0.0005 .

400,000 ( ) = + 0.001 2 Stationary points: 400,000 + 0.001 = 0 2 400,000 = 0.001 2 400,000 2 = = 400,000,000, 0.001 giving = 400,000,000 = 20,000 iPhones per hour. (The solution = 400,000,000 = 20,000 is outside the domain.) Testing points on either side shows that this gives a minimum, and a consideration of the graph shows that this minimum is absolute. The resulting average cost is 400,000 (20,000) = + 590 + 0.001(20,000) = $630. 20,000 For the resulting marginal cost, ( ) = 590 + 0.002 (20,000) = 590 + 0.002(20,000) = 590 + 40 = $630 as well. 800,000 ( ) = + 0.0005 2 Stationary points: 800,000 + 0.0005 = 0 2 800,000 = 0.0005 2 800,000 2 = = 1,600,000,000, 0.0005 giving = 1,600,000,000 = 40,000 consoles per hour. (The solution = 1,600,000,000 = 40,000 is outside the domain.) Testing points on either side shows that this gives a minimum, and a consideration of the graph shows that this minimum is absolute. The resulting average cost is


Solutions Section 12.2 800,000 (40,000) = + 510 + 0.0005(40,000) = $550. 40,000 At less than the optimal production level, the average cost is larger than the minimum and decreases as production steps up, so the cost of each successive unit needs to be lower than the current average in order to drive down the average as more units are made. In other words, the marginal cost is less than the average cost at that production level.

15. Unknown: !, the number of pounds of pollutant removed per day Objective function: average cost (!) 4,000 + 100! 2 4,000 (!) = = = + 100!, with domain ! > 0. ! ! ! Stationary points: Set (!) = 0 and solve for !: 4,000 (!) = 2 + 100 = 0 ! 2 ! = 4,000 100 = 40 ! = 40 6.32 pounds of pollutant per day. Testing points on either side shows that this gives a minimum, and a consideration of the graph shows that this minimum is absolute. Thus, average cost is minimized when we remove about 6.32 pounds per day, giving an average cost of 4,000 + 100(40) ( 40) = $1,265 per pound. 40 16. Unknown: !, the number of pounds of pollutant removed per day Objective function: average cost (!) 2,000 + 200! 2 2,000 (!) = = = + 200!, with domain ! > 0. ! ! ! Stationary points: Set (!) = 0 and solve for !: 2,000 (!) = 2 + 200 = 0 ! 2 ! = 2,000 200 = 10 ! = 10 3.16 pounds of pollutant per day. Testing points on either side shows that this gives a minimum, and a consideration of the graph shows that this minimum is absolute. Thus, average cost is minimized when we remove about 3.16 pounds per day, giving an average cost of 2,000 + 200(10) ( 10) = $1,265 per pound. 10

17. Net cost is " = (!) 500! = 4,000 + 100! 2 500!, with ! 0. Stationary points: Set " (!) = 0 and solve for !: " (!) = 200! 500. " (!) = 0 when ! = 2.5; " (!) is defined for all !. Testing the endpoint ! = 0, the stationary point ! = 2.5, and one more point to the right of 2.5, we see that the net cost is minimized when ! = 2.5 pounds of pollutant per day. 18. Net cost is " = (!) 100! = 2,000 + 200! 2 100!, with ! 0. Stationary points: Set " (!) = 0 and solve for !: " (!) = 400! 100 " (!) = 0 when ! = 0.25; " (!) is defined for all !. Testing the endpoint ! = 0, the stationary point ! = 0.25, and one more point to the right of 0.25, we see that the net cost is minimized when ! = 0.25 pounds of pollutant per day.

19. Let be the length of the east and west sides and let be the length of the north and south sides. The area is =


Solutions Section 12.2 and the cost of the fence is 2 # 4 + 2 # 2 = 8 + 4 . (We multiply by 2 because there are two sides of length and two sides of length .) So, our problem is to maximize = subject to 8 + 4 = 80. Solve the constraint equation for : = 20 2 and substitute into the objective: ( ) = (20 2 ) = 20 2 2. Since and must both be nonnegative, we have 0 10. Stationary points: Set ( ) = 0 and solve for : ( ) = 20 4 . ( ) = 0 when 20 4 = 0, giving = 5; ( ) is always defined. Testing the endpoints 0 and 10 as well as the stationary point 5, we see that the maximum area occurs when = 5. The corresponding value of is = 10, so the largest area possible is 5 × 10 = 50 square feet.

20. Let be the length of the east and west sides and let be the length of the south side. The area is = and the cost of the fence is 4 + 2 # 2 = 4 + 4 . So, our problem is to maximize = subject to 4 + 4 = 80. Solve the constraint equation for = 20 and substitute in the objective: ( ) = (20 ) = 20 2. Since and must both be nonnegative, we have 0 20. Stationary points: Set ( )= 0 and solve for : ( ) = 20 2 . ( ) = 0 when = 10; ( ) is always defined. Testing the endpoints 0 and 20 as well as the stationary point 10, we see that the maximum area occurs when = 10. The corresponding value of is = 10, so the largest area possible is 10 × 10 = 100 square feet. 21. Let be the length of the bottom fence and let be the length of the fence on the left. The area is = 2 and the cost of the fence is + 5 . So our problem is: Maximize = 2 subject to + 5 = 100. Solve the constraint equation for and substitute into the objective: = 100 5 , giving = (100 5 ) 2 = 50 2.5 2. Since and must be nonnegative, we must have 0 20. Stationary points: = 50 5 = 0 when = 10. (10) = 50(10) 2.5(10) 2 = 250 sq. ft. Endpoints: Each of the endpoints = 0 and 20 gives an area of zero, so the stationary point gives the maximum. When = 10, = 100 5(10) = 50. Thus the dimensions (E-W) × (N-S) are 50 ft × 10 ft for an area of 250 sq. ft. 22. Let be the length of the bottom fence and let be the length of the fence on the left. The area is = 2 and the cost of the fence is 2 + 8 . So our problem is: Maximize = 2 subject to 2 + 8 = 400. Solve the constraint equation for and substitute into the objective: = 200 4 , giving = (200 4 ) 2 = 100 2 2. Since and must be nonnegative, we must have 0 50. Stationary points: = 100 4 = 0 when = 25. (25) = 100(25) 2(25) 2 = 1,250 sq. ft. Endpoints: Each of the endpoints = 0 and 50 gives an area of zero, so the stationary point gives the maximum. When = 25, = 200 4(25) = 100. Thus the dimensions (E-W) × (N-S) are 100 ft × 25 ft for an area of 1,250 sq. ft. 23. Let be the length of the east and west sides and let be the length of the north and south sides. The area is


Solutions Section 12.2 = = 242 and the cost of the fence is = 2(4 ) + 2(2 ) = 8 + 4 . (We multiply by 2 because there are two sides of length and two sides of length .) Since we want to minimize cost, this time the cost is the objective, and the constraint is = 242. Solving the constraint equation for gives = 242 . Substituting in the objective: = 8 + 4(242 ) = 8 + 968 . Since cannot be negative nor zero, the domain of is > 0. Stationary points: = 8 968 2. This is zero when 968 8= 8 2 = 968 2 = 121, giving = 11. 2 From the graph of , we see that = 11 is the absolute minimum of :

Thus, for the cheapest fencing, = 11, = 242 11 = 22.

24. Let be the length of the east and west sides and let be the length of the north and south sides. The area is = = 324. and the cost of the fence is = 4 + 2(2 ) = 4 + 4 . Since we want to minimize cost, this time the cost is the objective, and the constraint is = 324. Solving the constraint equation for gives = 324 . Substituting in the objective: = 4 + 4(324 ) = 4 + 1, 296 . Since cannot be negative nor zero, the domain of is > 0. Stationary points: = 4 1,296 2. This is zero when 1, 296 4= 4 2 = 1, 296 2 = 324 = 18 2 From the graph of , we see that = 18 is the absolute minimum of :

Thus, for the cheapest fencing, = 18, = 324 18 = 18.

25. $ = %! = %(200,000 10,000%) = 200,000% 10,000% 2 For % and ! to both be nonnegative, we must have 0 % 20.


Solutions Section 12.2 Stationary points: Set $ (%) = 0 and solve for %: $ (%) = 200,000 20,000%, so $ (%) = 0 when % = 10; $ (%) is always defined, so there are no singular points. Testing the endpoints 0 and 20 as well as the stationary point 10, we see that the maximum revenue occurs when the price is % = $10.

26. $ = %! and = 4!, so the profit is = $ = %! 4! = %(200,000 10,000%) 4(200,000 10,000%) = 10,000% 2 + 240,000% 800,000. For % and ! to both be nonnegative, we must have 0 % 20. Stationary points: Set (%)= 0 and solve for %: (%) = 20,000% + 240,000, so (%) = 0 when % = 12; (%) is always defined, so there are no singular points. Testing the endpoints 0 and 20 as well as the stationary point 12, we see that the maximum profit occurs when the price is % = $12. 27. The optimization problem is: Maximize $ = %! subject to the constraint ! = 2.5% + 1,850. For % and ! to both be nonnegative, we must have 0 % 740. To solve, substitute the constraint equation in the objective function: $(%) = %( 2.5% + 1,850) = 2.5% 2 + 1,850% million dollars. $ (%) = 5% + 1,850, so $ (%) = 0 when % = 1,850 5 = $370. $ (%) is always defined, so there are no singular points. Testing the endpoints 0 and 740 as well as the stationary point 370, we see that the maximum revenue occurs when the price is % = $370. At this price, the annual revenue would be $(370) = 2.5(370) 2 + 1,850(370) = $342,250 million, or $342.25 billion. 28. The optimization problem is: Maximize $ = %! subject to the constraint ! = 10% + 4,360. For % and ! to both be nonnegative, we must have 0 % 436. To solve, substitute the constraint equation in the objective function: $(%) = %( 10% + 4,360) = 10% 2 + 4,360% million dollars. $ (%) = 20% + 4,360, so $ (%) = 0 when % = 4,360 20 = $218.00. $ (%) is always defined, so there are no singular points. Testing the endpoints 0 and 436 as well as the stationary point 218, we see that the maximum revenue occurs when the price is % = $218.00. At this price, the annual revenue would be $(218) = 10(218) 2 + 4,360(218) = $475,240 million, or $475.24 billion. 29. The optimization problem is: Maximize $ = %! subject to the constraint ! = 4,500% + 41,500. For % and ! to both be nonnegative, we must have 0 % 41,500 4,500 = 83 9 9.22.


Solutions Section 12.2 To solve, substitute the constraint equation in the objective function: $(%) = %( 4,500% + 41,500) = 4,500% 2 + 41,500% $ (%) = 9,000% + 41,500. $ (%) = 0 when % = 41,500 9, 000 $4.61. $ (%) is always defined, so there are no singular points. Testing the endpoints 0 and 9.22 as well as the stationary point 4.61, we see that the maximum revenue occurs when the price is % = $4.61. At this price, the daily revenue would be $(80) = 4,500(4.61) 2 + 41,500(4.61) = $95,680.55. 30. The optimization problem is: Maximize $ = %! subject to the constraint ! = 2% + 24. For % and ! to both be nonnegative, we must have 0 % 12. To solve, substitute the constraint equation in the objective function: $(%) = %( 2% + 24) = 2% 2 + 24% million zonars $ (%) = 4% + 24. $ (%) = 0 when % = 24 4 = Ż6. $ (%) is always defined, so there are no singular points. Testing the endpoints 0 and 12 as well as the stationary point 6, we see that the maximum revenue occurs when the price is % = Ż6. At this price, the daily revenue would be $(80) = 2(6) 2 + 24(6) = Ż72 million. 500,000 500,000 #!= = 500,000! 0.5 1.5 ! ! 0.5 We are given that ! 5,000. Stationary points: Set $ (!) = 0 and solve for !: $ (!) = 250,000! 1.5. $ (!) is never 0, so there are no stationary points. $ (!) is defined for all ! > 0, so there are no singular points. We test the single endpoint ! = 5,000 and a point to the right: $(5,000) = 7,071.07 and $(10,000) = 5,000. Thus, the revenue is decreasing and its maximum value occurs at the endpoint ! = 5,000. The corresponding price is % = $1.41 per pound. b. As found in part (a), the maximum occurs at ! = 5,000 pounds. c. The maximum revenue is $(5,000) = $7,071.07 per month. 31. a. $ = %! =

6,570,000 6,570,000 #!= = 6,570,000! 0.3 ! 1.3 ! 0.3 We are given that ! 10,000. Stationary points: Set $ (!) = 0 and solve for !: $ (!) = 1,971,000! 1.3. $ (!) is never 0, so there are no stationary points. $ (!) is defined for all ! > 0, so there are no singular points. We test the single endpoint ! = 10,000 and a point to the right: $(10,000) = 414,538.98 and $(20,000) = 336,710.28. Thus, the revenue is decreasing and its maximum value occurs at the endpoint ! = 10,000. The corresponding price is $41.45 per bushel. b. As found in part (a), the maximum occurs at ! = 10,000 bushels per year. c. The maximum revenue is $(10,000) = $414,538.98 per year. 32. a. $ = %! =

33. We are given two points: (%, !) = (25, 22) and (14, 27.5). The equation of the line through these two points is


Solutions Section 12.2 ! = % 2 + 34.5. The revenue is $ = %! = %( % 2 + 34.5) = % 2 2 + 34.5%. For % and ! to both be nonnegative, we must have 0 % 69. Stationary points: Set $ (%) = 0 and solve for %: $ (%) = % + 34.5. $ (%) = 0 when % = 34.5; $ (%) is defined for all %, so there are no singular points. Testing the endpoints 0 and 69 as well as the stationary point 34.5, we find that the revenue is maximized when the price is % = 34.5¢ per pound, for an annual (per capita) revenue of $5.95.

34. We are given two points: (%, !) = (10, 120) and (30, 0). The equation of the line through these two points is ! = 6% + 180. The revenue is $ = %! = %( 6% + 180) = 6% 2 + 180%. For % and ! to both be nonnegative, we must have 0 % 30. Stationary points: Set $ (%) = 0 and solve for %: $ (%) = 12% + 180. $ (%) = 0 when % = 15; $ (%) is defined for all %, so there are no singular points. Testing the endpoints 0 and 30 as well as the stationary point 15, we find that the revenue is maximized when the price is % = $15 per book, for a revenue of $1,350. 35. The profit function is = Revenue − Cost = %! 100! The optimization problem is: Maximize = %! 100! subject to the constraint ! = 2.5% + 1,850. For % and ! to both be nonnegative, we must have 0 % 740. To solve, substitute the constraint equation in the objective function: (%) = %( 2.5% + 1,850) 100( 2.5% + 1,850) = 2.5% 2 + 2,100% 185,000 million dollars (%) = 5% + 2,100. (%) = 0 when % = 2,100 5 = $420 (%) is always defined, so there are no singular points. Testing the endpoints 0 and 740 as well as the stationary point 420, we see that the maximum revenue occurs when the price is % = $420.00. At this price, the annual profit would be (420) = 2.5(420) 2 + 2,100(420) 185,000 = $256,000 million, or $256 billion. 36. The profit function is = Revenue − Cost = %! 100! The optimization problem is: Maximize = %! 100! subject to the constraint ! = 10% + 4,360. For % and ! to both be nonnegative, we must have 0 % 436. To solve, substitute the constraint equation in the objective function: (%) = %( 10% + 4,360) 100( 10% + 4,360) = 10% 2 + 5,360% 436,000 million dollars (%) = 20% + 5,360. (%) = 0 when % = 5,360 20 = $268. (%) is always defined., so there are no singular points. Testing the endpoints 0 and 436 as well as the stationary point 268, we see that the maximum revenue occurs when the price is % = $268. At this price, the annual profit would be (268) = 10(268) 2 + 5,360(268) 436,000 = $282,240 million, or $282.24 billion. 37. a. $ = %! and = 100!, so the profit is


Solutions Section 12.2 1,000 = $ = %! 100! = (!) 100! = 1,000! 0.7 100!. ! 0.3 For % and ! to be defined and nonnegative, we need ! > 0. Stationary points: Set (!) = 0 and solve for !: (!) = 700! 0.3 100. (!) = 0 when ! = 7 1 0.3 656; (!) is defined for all ! > 0, so there are no singular points. Testing the stationary point at approximately 656 and points on either side, we see that the profit is maximized when you sell 656 headsets, for a profit of (656) $28,120. b. The corresponding price is % $143 per headset.

38. a. $ = %! and = 100!, so the profit is 800 = $ = %! 100! = (!) 100! = 800! 0.65 100!. 0.35 ! For % and ! to be defined and nonnegative we need ! > 0. Stationary points: Set (!) = 0 and solve for !: (!) = 520! 0.35 100. (!) = 0 when ! = 5.2 1 0.35 111; (!) is defined for all ! > 0, so there are no singular points. Testing the stationary point at approximately 111 and points on either side, we see that the profit is maximized when you sell 111 headsets, for a profit of (111) $5,982. b. The corresponding price is % $154 per headset. 39. Take the height of the can as and the radius of the base as &. The objective function is the total surface area of the can: = '& 2 + 2'& . Bottom disc plus side Constraint: ( = '& 2 = 27,000 27,000 Solve the constraint for : = '& 2 Substituting in the objective function, we get 27,000 54,000 = '& 2 + 2'& , where 0 < & < + . = '& 2 + 2 & '& Stationary points: 54,000 (&) = 2'& = 0 when &2 54,000 27,000 2'& = , giving 2'& 3 = 54,000, so & 3 = . 2 ' & 27,000 1 3 &= � 20.4835 20.48 cm � ' 27,000 27,000 20.48 cm = '& 2 '(20.4835) 2 (There are no singular points or endpoints in the domain.) As usual, check that this gives a minimum. 40. Take the height of the drum as and the radius of the base as &. The objective function is the total surface area of the drum: = '& 2 + 2'& . Bottom disc plus side 2 Constraint: ( = '& = 1 1 Solve the constraint for : = '& 2 Substituting in the objective function, we get. 1 2 = '& 2 + 2'& 2 = '& 2 + , where 0 < & < + . & '& Stationary points: 2 (&) = 2'& 2 = 0 when &


Solutions Section 12.2 2 1 3 3 2'& = 2 , giving 2'& = 2, so & = . ' & 1 1 3 0.68278 0.6828 m &= � � ' 1 1 0.6828 m = '& 2 '(0.68278) 2 (There are no singular points or endpoints in the domain.) As usual, check that this gives a minimum.

41. Take the height of the can as and the radius of the base as &. The objective function is the cost of the metal in the can: = 0.02 × 2'& 2 + 0.01 × 2'& = 0.04'& 2 + 0.02'& Top and bottom plus side 2 Constraint: ( = '& = 250 250 Solve the constraint for : = . '& 2 Substituting in the objective function, we get 250 5 = 0.04'& 2 + 0.02'& 2 = 0.04'& 2 + , where 0 < & < + . & '& Stationary points: 5 (&) = 0.08'& 2 = 0 when & 5 5 0.08'& = 2 , giving 0.08'& 3 = 5, so & 3 = . 0.08' & 1 3 5 &= � 2.7096 2.71 cm � 0.08' 250 250 10.84 cm = '& 2 '(2.7096) 2 (There are no singular points or endpoints in the domain.) As usual, check that this gives a minimum. The ratio height/radius is about 10.84 2.71 = 4.

42. Take the height of the drum as and the radius of the base as &. The objective function is the cost of the metal in the drum: = 3 × 2'& 2 + 2 × 2'& = 3'& 2 + 4'& . Top and bottom plus side 2 Constraint: ( = '& = 2 2 Solve the constraint for : = . '& 2 Substituting in the objective function, we get 2 8 = 3'& 2 + 4'& 2 = 3'& 2 + , where 0 < & < + . & '& Stationary points: 8 (&) = 6'& 2 = 0 when & 8 8 6'& = 2 , giving 6'& 3 = 8, so & 3 = . 6' & 8 1 3 &= � � 0.7517 m 6' 2 2 1.127 cm = '& 2 '(0.7517) 2 (There are no singular points or endpoints in the domain.) As usual, check that this gives a minimum. The ratio height/radius is about 1.127 0.7515 = 4. 43. Let be the length of one side of the square cut out of each corner, as in the figure:


Solutions Section 12.2

When the sides are folded up, the resulting box will have volume ( = (16 2 )(6 2 ) = 4 3 44 2 + 96 . For the sides to have nonnegative lengths, we must have 0 3. Stationary points: Set ( ( ) = 0 and solve for : ( ( ) = 12 2 88 + 96 = 4( 6)(3 4). ( ( ) = 0 when = 4 3 or = 6, but = 6 is outside of the domain; there are no singular points. Testing the endpoints 0 and 3 and the stationary point 4/3, we find that the largest volume occurs when = 4 3 in. Thus, the box with the largest volume has dimensions 13⅓ in × 3⅓ in × 1⅓ in, and it has volume ( (4 3) = 1,600 27 59 cubic inches.

44. Let be the length of one side of the square cut out of each corner. When the sides are folded up, the resulting box will have volume ( = (12 2 ) 2 = 4 3 48 2 + 144 . For the sides to have nonnegative lengths, we must have 0 6. Stationary points: Set ( ( ) = 0 and solve for : ( ( ) = 12 2 96 + 144 = 12( 2)( 6). ( ( ) = 0 when = 2 or = 6; note that = 6 is also an endpoint. Testing the endpoints 0 and 6 and the stationary point 2, we find that the largest volume occurs when = 2". The box with the largest volume has dimensions 8" × 8" × 2" and has volume 128 cubic inches. 45. Let be the width and depth of the box and let be the height. The amount of material used will be = 2 2 + 4 , counting the top, bottom (each of which has area 2), and four sides (each of which has area ). We are told that the volume is 125 cm 3, so we must have 2 = 125. So, our problem is to maximize = 2 2 + 4 subject to 2 = 125. Also, > 0 for and to be nonnegative. Solve for = 125 2 and substitute in the objective: = 2 2 + 500 . Stationary points: Set ( ) = 0 and solve for : ( ) = 4 500 2. ( ) = 0 when 3 = 500 4 = 125, so = 5. (The corresponding value of is = 125 2 = 5 also.) Testing this stationary point and a point on either side, we see that the least material is used building a box of dimensions 5 × 5 × 5 cm. 46. Let be the width and the depth of the box and let be the height. The amount of material used will be = 2 + 4 , counting the bottom and the four sides. We are told that the volume is 108 cm 3, so we must have 2 = 108. So, our problem is to maximize = 2 + 4 subject to 2 = 108. Also, > 0 for and to be nonnegative. Solve for = 108 2 and substitute in the objective: = 2 + 4 (108 2) = 2 + 432 . Stationary points: Set ( ) = 0 and solve for :


Solutions Section 12.2 ( ) = 2 432 2. ( ) = 0 when 3 = 216, = 6. (The corresponding value of is = 108 6 2 = 3.) Testing this stationary point and a point on either side, we see that the least material is used building a box of dimensions 6 × 6 × 3 cm.

47. Let ) = length, * = width, and = height. We want to maximize the volume ( = )* , but we are restricted by ) + * + 62. Since we're looking for the largest volume, we can assume that ) + * + = 62. We are also told that = *. Thus, our problem is to maximize ( = )* subject to ) + * + = 62 and = *. Substitute = * in the other constraint to get ) + 2* = 62, then solve for ): ) = 62 2*. Substitute in the objective to get ( = (62 2*)* 2 = 62* 2 2* 3. For all dimensions to be nonnegative we need 0 * 31. Stationary points: ( (*) = 124* 6* 2, and so ( (*) = 0 when 124* 6* 2 = 0 2*(62 3*) = 0, giving * = 0 or * = 62 3. Singular points: None; ( (*) is defined for all *. Testing the endpoints 0 and 31 as well as the (other) stationary point 62/3, we see that the volume is maximized when * = 62 3. The corresponding values of the other dimensions are ) = = 62 3. Thus, the largest volume bag has dimensions ) = * = 20.67in, and volume ( 8,827 in 3 .

48. Let ) = length, * = width, and = height. We want to maximize the volume ( = )* , but we are restricted by ) + * + 45. Since we're looking for the largest volume, we can assume that ) + * + = 45. We are told that ) = 2 . Thus, our problem is to maximize ( = )* subject to ) + * + = 45 and ) = 2 . Substitute ) = 2 into the other constraint to get 3 + * = 45. Solve for * : * = 45 3 and substitute to get ( = 2 2(45 3 ) = 90 2 6 3. For all dimensions to be nonnegative we need 0 15. Stationary points: ( ( ) = 180 18 2 ( ( ) = 0 when = 0 or = 10; ( ( ) is defined for all . Testing the endpoints 0 and 15 as well as the (other) stationary point 10, we see that the volume is maximized when = 10. The corresponding values of the other dimensions are ) = 20 and * = 15. Thus, the largest volume bag has dimensions ) = 20 in, = 10 in, and * = 15 in, and the largest possible volume is ( = 3,000 in 3 . 49. Let ) = length, * = width, and = height. We want to maximize the volume ( = )* , but we are restricted by ) + * = 45 and * + = 45. Solve for ) = 45 * and = 45 *. Substitute to get ( = *(45 *) 2. For all dimensions to be nonnegative we need 0 * 45. Stationary points: ( (*) = (45 *) 2 2*(45 *) = (45 *)(45 * 2*) = (45 *)(45 3*); ( (*) = 0 when * = 15 or * = 45; ( (*) is defined for all *. Testing the endpoints 0 and 45 as well as the stationary point 15, we see that the volume is maximized when * = 15. The corresponding values of the other dimensions are


)=

Solutions Section 12.2 = 30. Thus, the largest volume bag has dimensions ) = 30 in, * = 15 in, and

= 30 in.

50. Let ) = length, * = width, and = height. We want to maximize the volume ( = )* but we are restricted by ) + * = 36 and ) + + 2* = 72. Solve the first constraint for ) = 36 * and substitute in the second to get 36 + + * = 72. Solve for = 36 * and substitute to get ( = *(36 *) 2. For all the dimensions to be nonnegative we need 0 * 36. Stationary points: ( (*) = (36 *) 2 2*(36 *) = (36 *)(36 3*) ( (*) = 0 when * = 12 or * = 36; ( (*) is defined for all *. Testing the endpoints 0 and 36 as well as the stationary point 12, we see that the volume is maximized when * = 12. The corresponding values of the other dimensions are )= = 24. Thus, the largest volume bag has dimensions ) = 24 in, * = 12 in, = 24 in.

51. Let ) = length, * = width, and = height. We want to maximize the volume ( = )* but we are restricted by ) + 2(* + ) 108. Since we're looking for the largest volume, we can assume that ) + 2(* + ) = 108. We are also told that * = . Substitute to get ) + 2(2*) = 108, or ) + 4* = 108. Solve for ) = 108 4* and substitute to get ( = * 2(108 4*) = 108* 2 4* 3. For all the dimensions to be nonnegative we need 0 * 27. Stationary points: ( (*) = 216* 12* 2 = 12*(18 *); ( (*) = 0 when * = 0 or * = 18; ( (*) is defined for all *. Testing the endpoints 0 and 27 as well as the stationary point 18, we see that the volume is maximized when * = 18. The corresponding values of the other dimensions are = 18 and ) = 36. Thus, the largest volume package has dimensions ) = 36 in and * = = 18 in, and has volume ( = 11,664 in 3 .

52. Let ) = length, * = width, and = height. We want to maximize the volume ( = )* but we are restricted by ) 108 and ) + 2(* + ) 165. We may assume that ) + 2(* + ) = 165, but we cannot assume that ) = 108 : If ) + 2(* + ) < 165 we can increase one of the dimensions without decreasing any others, thereby increasing ( . However, if ) < 108 we may not be able to increase ) without decreasing one of the other dimensions. We will use ) 108 to help determine the domain. Including the condition that the front face be square, our problem is to maximize ( = )* subject to ) + 2(* + ) = 165 and * = , with ) 108. Substitute * for to get ) + 4* = 165. Solve for ) : ) = 165 4* and substitute to get ( = * 2(165 4*). For all dimensions to be nonnegative and for ) 108 we must have (165 108) 4 * 165 4, so 14.25 * 41.25. Stationary points: ( (*) = 330* 12* 2 = 2*(165 6*). ( (*) = 0 when * = 0 or * = 165 6 = 27.5 ; ( (*) is defined for all *. Testing the endpoints 14.25 and 41.25 and the stationary point 27.5 (the point 0 is outside the domain), we see that the maximum volume occurs when * = 27.5. The corresponding values of the other dimensions are ) = 55 and = 27.5. Thus, the largest volume package has dimensions ) = 55 in and * = = 27.5 in, and has volume ( = 41,593.75 in 3 . 53. ( ( ) = (11 + 88.3)1.06 : Stationary points: ( ( ) = (11)1.06 (11 + 88.3)(ln 1.06)1.06 = 1.06 [ 11(ln 1.06) + 11 88.3 ln 1.06] ( ( ) = 0 + 11(ln 1.06) + 11 88.3 ln 1.06 = 0 (the term 1.06 can never be 0). Solving for gives 11 88.3 ln 1.06 9.1 years, which is during 2029; Testing points on either side, we see that this is an absolute = 11 ln 1.06 maximum, and ( (9.1) = (11(9.1) + 88.3)1.06 9.1 $111 billion.


Solutions Section 12.2 54. ( ( ) = (11 + 88.3)1.03 : Stationary points: ( ( ) = (11)1.03 (11 + 88.3)(ln 1.03)1.03 = 1.03 [ 11(ln 1.03) + 11 88.3 ln 1.03] ( ( ) = 0 + 11(ln 1.03) + 11 88.3 ln 1.03 = 0 (the term 1.03 can never be 0). Solving for gives 11 88.3 ln 1.03 25.8 years, which is during 2045; Testing points on either side, we see that this is an absolute = 11 ln 1.03 maximum, and ( (25.8) = (11(25.8) + 88.3)1.03 25.8 $174 billion.

$( ) 3( 2 6 + 250) 1 2 6 + 250 # = = ( ) 6( + 20) 2 + 20 Stationary points: 1 2 + 40 370 ( ) = # = 0 2 ( + 20) 2 when 2 + 40 370 = 0. This is a quadratic with a single positive root at around = 7.75, or three quarters of the way through 2017. Testing points on either side of 7.75, we see that this represents the absolute minimum return. The value of at that point is 1 (7.75) 2 6(7.75) + 250 (7.75) = 4.75 2 7.75 + 20 Three quarters of the way through 2017, the return on investment in research and development reached a minimum of approximately $4.75 per $1 invested in research and development. 55. Let ( ) =

$( ) 3( 2 + 6 + 100) 1 2 + 6 + 100 = = ( ) 6( + 20) 2 + 20 Stationary points: 1 2 + 40 + 20 ( ) = = 0 2 ( + 20) 2 when 2 + 40 + 20 = 0. This is a quadratic with no positive roots, so there are no stationary points in the given domain. Checking the endpoints, we see that the minimum return ocurs at = 0, where (0) = 2.5. At the start of 2010, the return on investment in research and development was at a minimum of $2.5 per $1 invested in research and development. 56. Let ( ) =

57. % = , 0.05 = (300,000 + 1,000 2) 0.05 , ( 5) Endpoint: = 5 Stationary points: % = 2,000 0.05 0.05(300,000 + 1,000 2) 0.05 = ( 15,000 + 2,000 50 2) 0.05 % = 0 when 15,000 + 2,000 50 2 = 0 50( 30)( 10) = 0 so = 10 or = 30; % is always defined. Testing = 5, 10, 30, and 40, we see that the maximum discounted value occurs = 30 years from now. 58. % = , 0.05 =

20 0.05 1 + 0.05

Stationary points: (20 0.05 0.05 )(1 + 0.05 ) 0.05 ( 0.05 2 + 20) 0.05 % = = (1 + 0.05 ) 2 (1 + 0.05 ) 2 % = 0 when 0.05 2 + + 20 = 0, so 12 (the other solution is negative); % is always defined. Testing = 0, 12, and 20, we see that the maximum


Solutions Section 12.2 discounted value occurs 12 years from now.

59. $ = (100 + 2 )(400,000 2,500 ) = 40,000,000 + 550,000 5,000 2 Stationary points: $ = 550,000 10,000 $ = 0 when = 55; $ is defined for all . Testing = 55 and points on either side of it (or recognizing that the graph of $ is a parabola), we see that the release should be delayed for 55 days. 60. $ = (200 + 4 )(300,000 1,500 ) = 60,000,000 + 900,000 6,000 2 Stationary points: $ = 900,000 12,000 $ = 0 when = 75; $ is defined for all . Testing = 75 and points on either side of it (or recognizing that the graph of $ is a parabola), we see that the release should be delayed for 75 days.

61. $ = 500 , = 10,000 + 2 , so the total profit is = $ = 500 (10,000 + 2 ) and the average profit per copy is = = 500 1 2 10,000 1 + 2. Stationary points: = 250 3 2 + 10,000 2 = 0 when = (10,000 250) 2 = 1,600 is defined for all > 0. Testing the stationary point 1,600 and a point on either side, we see that the average profit is maximized at = 1,600 copies. For this many copies, the average profit is (1,600) = $8.25/copy. Since = 250 1 2 + 2, the marginal profit is (1,600) = $8.25/copy also. At this value of , average profit equals marginal profit; beyond this the marginal profit is smaller than the average and so the average declines. 62. $ = 600 , = 9,000 + 2 , so the total profit is = $ = 600 (9,000 + 2 ) and the average profit per copy is = 600 1 2 9,000 1 + 2. Stationary points: 300 3 2 + 9,000 2 = 0 when = (9,000 300) 2 = 900 is defined for all > 0. Testing the stationary point 900 and a point on either side, we see that the average profit is maximized at = 900 copies. For this many copies, the average profit is (900) = $12/copy. Since = 300 1 2 + 2, the marginal profit is (900) = $12/copy also. At this value of , average profit equals marginal profit; beyond this the marginal profit is smaller than the average and so the average declines. 63. Minimize cost = 100 + 16 subject to = 10,000. Solve the constraint for : = 10,000 . Substitute in the objective: = 100 + 160,000

( > 0).


Solutions Section 12.2 160,000 160,000 Stationary points: ( ) = 100 . Solving: = 0 when 100 = 2 2 100 2 = 160,000 2 = 1,600 = 40 laborers This represents the absolute minimum because the objective function increases without bound as approaches either 0 or + (graph it to see). The corresponding value of is given by the constraint equation: = 10,000 40 = 250 robots

64. Minimize cost = 200 + 8 subject to = 1,000,000. Solve the constraint for : = 1,000,000 . Substitute in the objective: = 200 + 8,000,000 ( > 0). 8,000,000 8,000,000 Stationary points: ( ) = 200 . Solving: = 0 when 200 = 2 2 200 2 = 8,000,000 2 = 40,000 = 200 androids This represents the absolute minimum because the objective function increases without bound as approaches either 0 or + (graph it to see). The corresponding value of is given by the constraint equation: = 1,000,000 200 = 5,000 robots. 65. We want to minimize = 20,000 + 365 subject to 0.4 0.6 = 1,000. Solve for : = (1,000 0.4) 1 0.6 = 1,000 5 3 2 3 and substitute: = 20,000 + 365(1,000 5 3) 2 3. The domain is > 0. Stationary points: 730 ( ) = 20,000 1,000 5 3 5 3 3 ( ) = 0 when 3 5 60,000 = � 71. � 5 3 730 × 1,000 Testing the stationary point at 71 and a point on either side, we see that has its minimum at 71. So, you should hire 71 employees. 66. We want to minimize = 20,000 + 365 subject to 0.5 0.5 = 1,000. Solve for : = (1,000 0.5) 1 0.5 = 1,000 2 1 = 1,000,000 1 and substitute: = 20,000 + 365,000,000 1. The domain is > 0. Stationary points: ( ) = 20,000 365,000,000 2 ( ) = 0 when 1 2 20,000 = � 135. � 565,000,000 Testing the stationary point at 135 and a point on either side, we see that has its minimum at 135. So, you should hire 135 employees.

67. We are being asked to find the extreme values of the derivative, " ( ). Call this function -( ). -( ) = " ( ) = 0.12 2 4 + 40, so - ( ) = 0.24 4 - ( ) = 0 when


Solutions Section 12.2 0.24 = 4 = 4 0.24 16.667 17 (representing 2007) to the nearest year. Testing the endpoints 0 and 18 as well as the stationary point 16.667, we see that - has an absolute minimum of -(16.667) 6.667 and an absolute maximum of -(0) = 40. Hence, " was increasing most rapidly in 1990 and increasing least rapidly in 2007.

68. We are being asked to find the extreme values of the derivative, " ( ). Call this function -( ). -( ) = " ( ) = 0.06 2 4 + 100 - ( ) = 0.12 4 - ( ) = 0 when 0.12 = 4 = 4 0.12 33.333 33 to the nearest year. This is outside the domain of the function, so we discard it. Testing the endpoints 0 and 18, we see that - has an absolute maximum of -(0) = 100 and an absolute minimum of -(18) = 47.44. Hence, " was increasing most rapidly in 1990 and increasing least rapidly in 2008.

69. Let &( ) = . ( ) = 0.195 3 + 6.8 22. & ( ) = 0.39 + 6.8; & ( ) = 0 when 17. Testing the endpoints 8 and 30 as well as the stationary point 17, we see that . ( ) has its maximum when = 17 days. This means that the embryo's oxygen consumption is increasing most rapidly 17 days after the egg is laid. 70. Let &( ) = . ( ) = 0.084 2 + 5.8 44.& ( ) = 0.168 + 5.8; & ( ) = 0 when 35. Testing the endpoints 20 and 50 as well as the stationary point 35, we see that . ( ) has its maximum when = 35 days. This means that the embryo's oxygen consumption is increasing most rapidly 35 days after the egg is laid. 71. Graph of derivative:

If we zoom in, we can determine that the maximum value of around 2.2 occurs around = 3.7; during the year 2003. Since ( ) represents the rate of change of the percentage of mortgages that were subprime, we can say that the percentage of mortgages that were subprime was increasing most rapidly during 2003, when it increased at a rate of around 2.2 percentage points per year. 72. Graph of derivative:

If we zoom in, we can determine that a maximum value of around 179 occurs around = 2.7; during the year 2002. Since ( ) represents the rate of change of the value of subprime mortgage debt, we can say that the value of subprime mortgage debt was increasing most rapidly during 2002, when it increased at a rate of around $179 billion per year. 10,000(1.05) 1 + 500 0.5 Technology formula: Excel: 10000*(1.05)^(-x)/(1+500*exp(-0.5*x)) 73. % = ,(1.05) =


Solutions Section 12.2 TI-83/84 Plus: 10000*(1.05)^(-x)/(1+500*e^(-0.5*x)) Here are the graphs of %( ) and % ( ): Graph of %( );

Graph of % ( ):

From the graph, we see that the maximum occurs between = 15and = 20. The maximum is more accurately seen in the graph of % ( ) where it crosses the -axis. Zooming in on the graph of % ( ), we see the following:

and so the maximum is very close to = 17 years. To obtain the value, substitute = 17 in the formula for %( ) or else use the trace feature to see the -coordinate of the highest point in the graph of %( ): approximately $3,960. 45 × 22,514(1.03) 1,013,130(1.03) = 1 + 22,514 2.55 1 + 22,514 2.55 Technology formula: 1013130*(1.03)^(-x)/(1+22514*x^(-2.55)) Here are the graphs of %( ) and % ( ): Graph of %( ); 74. % = ,(1.03) =

Graph of % ( ):

From the graph, we see that the maximum occurs between = 40 and = 50. The maximum is more accurately seen in the graph of % ( ) where it crosses the -axis. Zooming in on the graph of % ( ), we see the following:


Solutions Section 12.2

and so the maximum is at approximately = 47 years. To obtain the value, substitute = 47 in the formula for %( ): approximately $113,430

75. Let ( ) be the annual yield per tree when there are trees; ( ) = 100 ( 50) = 150 . If / ( ) is the total annual yield from trees, then / ( ) = # ( ) = (150 ) = 150 2 / ( ) = 150 2 . / ( ) = 0 when = 75. Since the graph of / is a parabola opening downward, we know that this gives the maximum value of / . Hence, the total annual yield is largest when there are 75 trees, or 25 additional trees beyond the 50 we already have.

76. Let ( ) be the annual yield per tree when there are trees. We are given two data points: ( , ) = (50, 75) and (40, 80). The equation of the line through these two points is 1 = + 100. 2 The total yield is 1 / ( ) = = 2 + 100 2 / ( ) = + 100. / ( ) = 0 when = 100. Since the graph of / is a parabola opening downward, we know that this gives the maximum value of / . Hence, the total annual yield is largest when there are 100 trees, so you should plant 60 more. 77.

0(1 $) 0(1 $) 3 , which is (D). = 3 2.2; = 0 when 2 = 02 02 2.

78. Total revenue is 1 $ = 2 = ( 4 + 3) = 4 2 + 3 . (A).

0(1 $) 0(1 $) 3 , which is = 24 + 3; = 0 when = 0 0 24

79. The objective is to maximize the height, so the objective function is 5 (choice (A)). One of the constraints is that the height 5 should not exceed eight times the distance 6 from the road, so for maximum height, we can say 86 = 5 (choice (B)). 80. The objective is to minimize the cost, so the objective function is (choice (C)). One of the constraints is that the height 5of at least 8 times the distance 6 from the road, so for minimum cost, we can say 5 86 = 0 (choice (A)).

81. The problem is uninteresting because the company can accomplish the objective by cutting away the entire sheet of cardboard, resulting in a box with surface area zero. 82. In most cost functions, cost is minimized at a production level of zero. It is more useful to minimize average cost. 83. Not all absolute extrema occur at stationary points; some may occur at an endpoint or singular point of the domain, as in Exercises 31 and 32.


Solutions Section 12.2 84. Increasing revenue beyond a certain value may be offset by even higher costs that decrease the profit, as in Exercises 35 and 36. 85. The minimum of 0! 0% is the fastest that the demand is dropping in response to increasing price.

86. First substitute = ( , ) into the objective , obtaining as a function of just and , and then substitute for as a function of , obtaining as a function of only. Then find the maximum of this function of . 87. Solving the constraint equation for gives = ± 1 2. Substituting in the objective gives 7 = ( 2) 2 + 1 2 = 5 4 .

If we have not followed Step 6, We would find that 7, being a linear function, has no exrreme. However, the expression for implies that the quantity under the radical shuld not be negative, so 1 - x^2 \geq 0 \implies x^2 \leq 1 \implies -1 \leq x \leq 1,

leading to a domain of [ 1, 1] for 7( ). Although 7 has no critical points, its domain does have endpoints, and by the Extreme Value Theorem, 7( ) = 5 4 has an absolute minimum at 1 and an absolute maximum at 1.

88. Solving the constraint equation for gives = 1. Substituting in the objective gives = ( 1)( 1) = ( 1) 2.

If we have not followed Step 6, We would find that A 4849438:); <=9=<;<4 (1, 0).5:* , & =8%:=9 .499: 3 8;38 = ; 0=9 .:98 &4=9 !;4 =:9.> * :)):* %6, * 9 0 :& 8 &=. to 1 as the solution for is valid only when $x \ne 1#.

0:<4=9: A%


Solutions Section 12.3 Section 12.3 1.

0 0 2 = 6 ; 2 = 6 0 0

2.

0 0 2 = 2 + 1; 2 = 2 0 0

3.

0 2 0 2 4 = 2; 2 = 3 0 0

4.

0 4 0 2 12 = 3; 2 = 4 0 0

5.

0 0 2 = 1.6 0.6 1; 2 = 0.96 1.6 0 0

6.

0 0 2 = 0.02 1.1; 2 = 0.022 2.1 0 0

7.

0 0 2 = ( 1) 1; 2 = ( 1) 0 0

8.

0 0 2 = + ; 2 = + 0 0

9.

0 1 1 0 2 2 1 = 2 ; 2 = 3 + 2 0 0

10. 0 0 2 6 1 = 2 3 + 1; 2 = 6 4 2 = 4 2 0 0

11. a. 8 = 12 + 3 16 2 08 ,= = 3 32 0 0, 4= = 32 ft/sec2 0 b. 4(2) = 32 ft/sec2 13. a. 8 =

1

+

1

2

08 1 2 = 2 3 0 0, 2 6 4= = 3 + 4 ft/sec2 0 b. 4(1) = 2 + 6 = 8 ft/sec2 ,=

15. a. 8 = + 2 08 1 1 2 ,= = + 2 0 2 0, 1 4= = 3 2 + 2 ft/sec2 0 4 63 b. 4(4) = ft/sec2 32

17. (1, 0) : changes from concave down to concave up

12. a. 8 = 12 + 16 2 08 ,= = 1 32 0 0, 4= = 32 ft/sec2 0 b. 4(2) = 32 ft/sec2 14. a. 8 =

1

1 2

08 1 2 = 2+ 3 0 0, 2 6 4= = 3 4 ft/sec2 0 1 b. 4(2) = ft/sec2 8 ,=

16. a. 8 = 2 + 3 08 ,= = 1 2 + 3 2 0 0, 1 4= = 3 2 + 6 ft/sec2 0 2 11 b. 4(1) = ft/sec2 2


Solutions Section 12.3 18. ( 1, 1) : changes from concave down to concave up 19. (1, 0) : changes from concave down to concave up 20. (1, 1) : changes from concave up to concave down 21. None: always concave down

22. ( 1, 1) : changes from concave up to concave down; (1, 1) : changes from concave down to concave up

23. ( 1, 0) : changes from concave up to concave down; (1, 1) : changes from concave down to concave up 24. None: always concave down

25. Remember that a point of inflection of corresponds to a relative extreme point of that is internal, not an endpoint. Points of inflection at = 1 (relative max of ) and = 1 (relative min of ) 26. Remember that a point of inflection of corresponds to a relative extreme point of that is internal, not an endpoint. Points of inflection at = 2 (relative min of ) and = 1 (relative max of )

27. Remember that a point of inflection of corresponds to a relative extreme point of that is internal, not an endpoint. One point of inflection, at = 2 (relative min of ). Note that has a stationary point at = 1 but not a relative extreme point there. 28. Remember that a point of inflection of corresponds to a relative extreme point of that is internal, not an endpoint. Points of inflection at = 2 (relative max of ) and = 1 (relative min of ). Note that has a stationary point at = 1 but not a relative extreme point there. 29. Points of inflection where the graph of crosses the -axis: at = 2, = 0, and = 2.

30. One point of inflection where the graph of crosses the -axis: at = 2. Note that has a zero at = 1 but does not change sign there. 31. Points of inflection where the graph of crosses the -axis at = 2 and = 2.

32. Points of inflection at = 0, where the graph of crosses the -axis, and = 1, where is not defined but changes sign. 33. ( ) = 2 4 + 1, ( ) = 2 4, ( ) = 2 Stationary points: 2 4 = 0 = 2 (2) = 2, which is positive, so has a relative minimum at = 2. 34. ( ) = 2 2 2 + 3, ( ) = 4 2, ( ) = 4


Solutions Section 12.3 Stationary points: 4 2 = 0 when = 1 2 (1 2) = 4, which is positive, so has a relative minimum at = 1 2.

35. ( ) = 3 12 , ( ) = 3 2 12, ( ) = 6 Stationary points: 3 2 12 = 0 3( 2)( + 2) = 0 = 2, 2 ( 2) = 6( 2) = 12, which is negative, so has a relative maximum at = 2. (2) = 6(2) = 12, which is positive, so has a relative minimum at = 2.

36. ( ) = 2 3 6 + 3, ( ) = 6 2 6, ( ) = 12 Stationary points: 6 2 6 = 0 6( 1)( + 1) = 0 = 1, 1 ( 1) = 12( 1) = 12, which is negative, so has a relative maximum at = 1. (1) = 12(1) = 12, which is positive, so has a relative minimum at = 1.

37. ( ) = 3 , ( ) = 3 2 1, ( ) = 6 Stationary points: 3 2 1 = 0 3 2 = 1 = 1 3, 1 3 ( 1 3) = 6( 1 3) = 6 3 which is negative, so has a relative maximum at = 1 3. (1 3) = 6(1 3) = 6 3, which is positive, so has a relative minimum at = 1 3.

38. ( ) = 2 3 + 3 , ( ) = 6 2 + 3, ( ) = 12 Stationary points: 6 2 + 3 = 0 6 2 = 3 = 1 2, 1 2 ( 1 2) = 12( 1 2) = 12 2, which is positive, so has a relative minimum at = 1/ 2. (1 2) = 12(1 2) = 12 2, which is negative, so has a relative maximum at = 1/ 2.

39. ( ) = 4 4 3, ( ) = 4 3 12 2, ( ) = 12 2 24 Stationary points: 4 3 12 2 = 0 4 2( 3) = 0 = 0, 3 (0) = 0;the second derivative test is inconclusive. However, the first derivative is 4 2( 3) and is negative on both sides of = 0, so the first derivative test tells us that has neither a relative maximum nor minimum at 0. (3) = 12(3) 2 24(3) = 36, which is positive, so has a relative minimum at = 3. 40. ( ) = 3 4 2 3, ( ) = 12 3 6 2, ( ) = 36 2 12 Stationary points: 12 3 6 2 = 0 6 2(2 1) = 0 = 0, 1 2 (0) = 0;the second derivative test is inconclusive. However the first derivative is 6 2(2 1) and is negative on both sides of = 0, so the first derivative test tells us that has neither a relative maximum nor minimum at 0. (1 2) = 36(1 2) 2 12(1 2) = 3, which is positive, so has a relative minimum at = 1 2. 2

2

2

41. ( ) = , ( ) = 2 , ( ) = [ 2 + 4 2] Stationary points: 2

2 = 0 = 0 since the exponential term is never zero. (0) = 0[ 2 + 4(0)] = 2, which is negative, so has a relative maximum at = 0.


2 2

42. ( ) = , ( ) = 2 Stationary points:

2

2 2

2

Solutions Section 12.3

, ( ) = 2 [ 2 + 4 2]

2 2 = 0 = 0 since the exponential term is never zero. (0) = 2[ 2 + 4(0)] = 2 2, which is negative, so has a relative maximum at = 0. 2

2

2

2

43. ( ) = 1 , ( ) = 1 [1 2 2], ( ) = 1 [ 4 2 (1 2 2)] = 1 [4 3 6 ] 2

= 1 [2 (2 2 3)] Stationary points: 2

1 [1 2 2] = 0 = 1 2, 1 2, since the exponential term is never zero. 4 H 1 B ? , which is positive, so has a relative minimum at = 1 2. = 1 2E @A 2 CD FG 2 IJ 1 B 4 H ? , which is negative, so has a relative maximum at = 1 2. = 1 2E A@ 2 DC GF 2 JI 2

2

44. ( ) = , ( ) = [1 2 2], 2

2

2

( ) = [ 4 2 (1 2 2)] = [4 3 6 ] = [2 (2 2 3)] Stationary points: 2

[1 2 2] = 0 = 1 2, 1 2, since the exponential term is never zero. 4 H 1 B ? , which is positive, so has a relative minimum at = 1 2. = 1 2E A@ 2 DC GF 2 JI 1 B 4 H ? , which is negative, so has a relative maximum at = 1 2. = 1 2E A@ 2 DC GF 2 JI 45. Take the derivative repeatedly: ( ) = 4 2 + 1 ( ) = 8 1 ( ) = 8 ( ) = (4)( ) = # # # = ( )( ) = 0.

47. Take the derivative repeatedly: ( ) = 4 + 3 2 ( ) = 4 3 + 6 ( ) = 12 2 + 6 ( ) = 24 (4)( ) = 24 (5)( ) = (6)( ) = # # # = ( )( ) = 0.

46. Take the derivative repeatedly: ( ) = 3 3 + 4 ( ) = 9 2 + 4 ( ) = 18 ( ) = 18 (4)( ) = (5)( ) = # # # = ( )( ) = 0. 48. Take the derivative repeatedly: ( ) = 4 + 3 ( ) = 4 3 + 3 2 ( ) = 12 2 + 6 ( ) = 24 + 6 (4)( ) = 24 (5)( ) = (6)( ) = # # # = ( )( ) = 0.


Solutions Section 12.3 49. Take the derivative repeatedly: ( ) = (2 + 1) 4 ( ) = 4(2 + 1) 3(2) = 8(2 + 1) 3 ( ) = 24(2 + 1) 2(2) = 48(2 + 1) 2 ( ) = 96(2 + 1)(2) = 192(2 + 1) (4)( ) = 384 (5)( ) = (6)( ) = # # # = ( )( ) = 0.

50. Take the derivative repeatedly: ( ) = ( 2 + 1) 3 ( ) = 3( 2 + 1) 2( 2) = 6( 2 + 1) 2 ( ) = 12( 2 + 1)( 2) = 24( 2 + 1) ( ) = 48 (4)( ) = (5)( ) = # # # = ( )( ) = 0.

51. Take the derivative repeatedly: ( ) = ( ) = ( 1) = ( ) = ( 1) = ( ) = ( 1) = (4)( ) = ( 1) = ### ( )( ) = ( 1) .

52. Take the derivative repeatedly: ( ) = 2 ( ) = 2 (2) = 2 2 ( ) = 2 2 (2) = 4 2 ( ) = 4 2 (2) = 8 2 (4)( ) = 8 2 (2) = 16 2 ### ( )( ) = 2 2 .

53. Take the derivative repeatedly: ( ) = 3 1 ( ) = 3 1(3) = 3 3 1 ( ) = 3 3 1(3) = 9 3 1 ( ) = 9 3 1(3) = 27 3 1 (4)( ) = 27 3 1(3) = 81 3 1 ### ( )( ) = 3 3 1.

54. Take the derivative repeatedly: ( ) = 2 + 3 ( ) = 2 + 3( 1) = 2 + 3 ( ) = 2 + 3( 1) = 2 + 3 ( ) = 2 + 3( 1) = 2 + 3 (4)( ) = 2 + 3( 1) = 2 + 3 ### ( )( ) = ( 1) 2 + 3.

55. 8( ) = 40 1.9 2 ,( ) = 8 ( ) = 3.8 4( ) = 8 ( ) = 3.8 m/s2

56. 8( ) = 100 + 10 0.8 2 ,( ) = 8 ( ) = 10 1.6 4( ) = 8 ( ) = 1.6 m/s2

57. 8( ) = 3 2 ,( ) = 8 ( ) = 3 2 2 4( ) = 8 ( ) = 6 2 ft/s2 4(1) = 6(1) 2 = 4 ft/s2 Since this is positive, the velocity is increasing.

58. 8( ) = 3 8 2 ,( ) = 8 ( ) = 3 16 4( ) = 8 ( ) = 3 16 ft/s2 4(1) = 3 16 7.85 ft/s2 Since this is negative, the velocity is decreasing.


59. &( ) = 0.4 + + 26.5, & ( ) = 0.8 + 1, $ ( ) = 0.8 billion dollars/yr2 As this is a constant, its value is independent of . Accelerating by $0.8 billion/yr2 2

Solutions Section 12.3

60. &( ) = 0.2 2 + 3 + 18, & ( ) = 0.4 + 3, $ ( ) = 0.4 billion dollars/yr2 As this is a constant, its value is independent of . Decelerating by $0.4 billion/yr2

61. a. .( ) = 0.065 3 + 3.4 2 22 + 3.6, so .(20) = 0.065(20) 3 + 3.4(20) 2 22(20) + 3.6 400 ml b. . ( ) = 0.195 2 + 6.8 22, so . (20) = 0.195(20) 2 + 6.8(20) 22 = 36 ml/day c. . ( ) = 0.39 + 6.8, so . (20) = 0.39(20) + 6.8 = 1 ml/day2

62. a. .( ) = 0.028 3 + 2.9 2 44 + 95 so .(40) = 0.028(40) 3 + 2.9(40) 2 44(40) + 95 1, 200 ml b. . ( ) = 0.084 2 + 5.8 44, so . (40) = 0.084(40) 2 + 5.8(40) 44 54 ml/day c. . ( ) = 0.168 + 5.8, so . (40) = 0.168(40) + 5.8 = 0.92 ml/day2

63. a. >( ) = 0.03 3 + 0.5 2 0.2 + 260, so > ( ) = 0.09 2 + 0.2 > ( ) 0.09 2 + 0.2 Inflation Rate: = >( ) 0.03 3 + 0.5 2 0.2 + 260 > (2) 0.09(2) 2 + (2) 0.2 1.44 January 2021 ( = 2): 0.0055, or 0.55% = = 3 2 >(2) 261.36 0.03(2) + 0.5(2) 0.2(2) + 260 b. > ( ) = 0.09 2 + 0.2, so, > ( ) = 0.18 + 1, and > (2) = 0.18(2) + 1 = 0.64 Since > (2) is positive, inflation was speeding up in January 2021. c. Points of inflection occur when > ( ) = 0: 0.18 + 1 = 0 1 5.56. = 0.18 Looking at the graph, we see that it is concave up to the left of = 5.56 (around mid April 2021) and concave down to the right. Therefore, inflation was speeding up for < 5.56 and slowing for > 5.56 (after that time).

64. a. >( ) = 0.025 3 0.38 2 + 2 + 253, so > ( ) = 0.075 2 0.76 + 2 > ( ) 0.075 2 0.76 + 2 Inflation Rate: = >( ) 0.025 3 0.38 2 + 2 + 253 > (1) 0.075(1) 2 0.76(1) + 2 March 2019 ( = 1): 0.00516, or 0.516% = >(1) 0.025(1) 3 0.38(1) 2 + 2(1) + 253 b. > ( ) = 0.075 2 0.76 + 2 so, > ( ) = 0.15 0.76, and > (1) = 0.15 0.76 = 0.61 Since > (1) is negative, inflation was slowing down in March 2019. c. Points of inflection occur when > ( ) = 0: 0.15 0.76 = 0 0.76 = = 5.07. 0.15 Looking at the graph we see that it is concave down to the left of = 5.07 and concave up to the right. Therefore, inflation was slowing down for < 5.07 (around the beginning of July) and slowing for > 5.07 (after that time). 65. a. >( ) = 0.06 3 0.8 2 + 3.1 + 195, so > ( ) = 0.18 2 1.6 + 3.1 > ( ) 0.18 2 1.6 + 3.1 Inflation Rate: = >( ) 0.06 3 0.8 2 + 3.1 + 195


Solutions Section 12.3 > (5) 0.18(5) 2 1.6(5) + 3.1 0.4 December 2005 ( = 5): 0.00202, or 0.202% = = 198 >(5) 0.06(5) 3 0.8(5) 2 + 3.1(5) + 195 (deflation rate of 0.202%) > (7) 0.18(7) 2 1.6(7) + 3.1 0.72 February 2006 ( = 7): 0.00363, or 0.363% = = 3 2 >(7) 198.08 0.06(7) 0.8(7) + 3.1(7) + 195 b. > ( ) = 0.18 2 1.6 + 3.1 so, > ( ) = 0.36 1.6, and > (7) = 0.36(7) 1.6 = 0.92 Since > (7) is positive, inflation was speeding up in February 2006. c. Points of inflection occur when > ( ) = 0: 0.36 1.6 = 0 1.6 4.44. = 0.36 Looking at the graph we see that it is concave up to the right of = 4.44 and concave up to the left. Therefore, inflation was speeding up for > 4.44 (after mid-November) and decreasing for < 4.44 (prior to that time). 66. a. >( ) = 0.005 3 + 0.12 2 0.01 + 190, so > ( ) = 0.015 2 + 0.24 0.01 > ( ) 0.015 2 + 0.24 0.01 Inflation Rate: = >( ) 0.005 3 + 0.12 2 0.01 + 190 > (10) 0.015(10) 2 + 0.24(10) 0.01 0.89 July 2005 ( = 10): 0.00452, or 0.452% = = 3 2 >(10) 0.005(10) + 0.12(10) 0.01(10) + 190 196.9 b. > ( ) = 0.015 2 + 0.24 0.01 so, > ( ) = 0.030 + 0.24, and > (10) = 0.030(10) + 0.24 = 0.06 Since > (10) is negative, inflation was slowing down in July 2005. c. Points of inflection occur when > ( ) = 0: 0.030 + 0.24 = 0 0.24 = = 8. 0.030 Looking at the graph we see that it is concave up to the left of = 8 and concave down to the right. Therefore, inflation was speeding up for < 8 (before May) and slowing for > 8 (after May).

67. The graph of is concave up when is positive, and concave down when is negative. From the graph of , we see that it is negative until about = 2, at which time it turns positive. Therefore: The graph of is concave down for 0 < < 2, concave up for 2 < < 10, and there is a point of inflection around = 2 (when = 0). Interpretation: From the graph of we see that has a minimum at around = 2, meaning that the percentage of world R&D spending attributable to the U.S. was decreasing most rapidly at time = 2 (2012).

68. The graph of is concave up when is positive, and concave down when is negative. From the graph of , we see that it is positive until about = 9, at which time it turns negative. Therefore: The graph of is concave up for 0 < < 9, concave down for 9 < < 10, and there is a point of inflection around = 9 (when = 0). Interpretation: From the graph of we see that has a maximum at around = 9, meaning that the percentage of world R&D spending attributable to China was increasing most rapidly at around = 9 (2019).

69. a. The graph of . is concave down throughout the range [8, 30], and therefore has no points of inflection: choice (B). b. At = 18 (the point of inflection) the graph of . has a maximum. Since . measures the rate of change of daily oxygen consumption, it means that oxygen consumption is increasing at a maximum rate at around = 18: choice (B). c. For > 18, the graph of . is increasing but concave down, so that oxygen consumption is increasing at a decreasing rate: choice (A).


Solutions Section 12.3 70. a. The graph of . has a point of inflection at around = 35, as we see in the graph of . , which is zero at that point: choice (A). b. At 35 (the point of inflection) the graph of c' has a maximum. Since . measures the rate of change of daily oxygen consumption, it means that the rate of change of daily oxygen consumption is a maximum: choice (A). c. For < 35, the graph of . is increasing and concave up, so that oxygen consumption is increasing at an increasing rate: choice (A). 71. Graphs: ( ):

( ):

( ):

The point of inflection occurs when the second derivative is zero: Around 4. It is concave up when < 4 (where ( ) is positive) and concave down when > 4 (where ( ) is negative). The rate of change of ( ) (that is, the derivative ( )) is a maximum at that value of , meaning that the percentage of U.S. mortgages that were subprime was increasing fastest at the beginning of 2004. 72. Graphs: ( ):


( ):

Solutions Section 12.3

( ):

The point of inflection occurs when the second derivative is zero: Around 3 (to the nearest year). It is concave up when < 3 (where ( ) is positive) and concave down when > 3 (where ( ) is negative). The rate of change of ( ) (that is, the derivative ( )) is a maximum at that value of , meaning that the value of subprime mortgage debt was increasing fastest at the beginning of 2003. 73. a. Where the graph is steepest: 2 years into the epidemic b. At the point of inflection 2 years into the epidemic: There the steepness stops increasing and starts to decline, so the rate of new infections starts to drop. 74. a. Where the graph is steepest: three years since their release. b. Sales start off slowly but the rate of sales increases as the product becomes more popular. At a certain point (three years) the rate of sales starts to decline as newer products start to take over or as the market begins to become saturated.

75. a. 2024 ( = 4): The point where the graph is increasing and steepest b. 2026 ( = 6): The point where the graph is decreasing and steepest c. 2022 ( = 2): Since the graph is steepest downward at = 2 compared with nearby points, the rate of change of industrial output reached a minimum in 2022 compared with nearby years (choice (A)).

76. a. Around 2035 ( = 15): The point where the graph is increasing and steepest b. Around 2027 ( = 7); The point where the graph is decreasing and steepest c. Around 2031 ( = 11): When the graph stopped decreasing and started increasing

77. a. (9) 1757(9 180) 2.325; (9) 4085(9 180) 3.325; (9) is never zero and is always defined for 9 > 180. So, there are no points of inflection in the graph of . b. Since the graph is concave up ( (9) > 0 for 9 > 180), the derivative of is increasing, and so the rate of decrease of SAT scores with increasing numbers of prisoners was diminishing. In other words, the apparent effect on SAT scores of increasing numbers of prisoners was diminishing. 78. a. To detect points of inflection on the graph of we examine its second derivative, ( ) 13, 580(9 180) 4.325, which is never zero and always defined for 9 > 180. So, there are no points of inflection in the graph of


Solutions Section 12.3 . b. Since the graph of is concave down ( (9) < 0 for 9 > 180), the derivative of is decreasing, and so the rate of increase of with increasing numbers of prisoners was diminishing. In other words, the apparent effect of increasing numbers of prisoners on the rate of change of SAT scores was diminishing. 0 29 = 21.494. Thus, for a firm with annual sales of $3 million the rate at which new patents 08 2 K = 3 are produced decreases with increasing firm size. This means that the returns (as measured in the number of new patents per increase of $1 million in sales) are diminishing as the firm size increases. 0 29 b. 2 K = 13.474. Thus, for a firm with annual sales of $7 million the rate at which new patents are 08 = 7 produced increases with increasing firm size by 13.474 new patents per $1 million increase in annual sales. c. There is a point of inflection when 8 5.4587, so that in a firm with sales of $5,458,700 per year the number of new patents produced per additional $1 million in sales is a minimum.

79. a.

80. a. 9 ( ) = 8 + 4 1.2 2; 9 ( ) = 4 2.4 ; 9 (1) = 1.6; 9 (3) = 3.2 With $100,000 invested in R&D, the number of new products per $100,000 is increasing at a rate of 1.6 new products per $100,000 per $100,000, and with $300,000 invested in R&D, it is decreasing at a rate of 3.2 new products per $100,000 per $100,000. b. The point of inflection at $166,666.67 81. >( ) ( ) represents the fraction, or percentage, of Mexico-produced oil exported to the United States. Graphs: >( ) ( ):

[>( ) ( )] :

The graph of >( ) ( ) is decreasing, so the percentage of oil produced in Mexico that was exported to the United States was decreasing. Since the derivative of >( ) ( ) is also decreasing, the graph of >( ) ( ) is concave down. The concavity tells us that >( ) ( ) was decreasing at a faster rate. Thus, the percentage of oil produced in Mexico that was exported to the United States was decreasing at a faster rate (choice (D)). 82. >( ) ( ) represents the fraction, or percentage, of Mexico-produced oil exported to the United States.Graphs: >( ) ( ):


Solutions Section 12.3

[>( ) ( )] :

The graph of >( ) ( ) is decreasing, so the percentage of oil produced in Mexico that was exported to the United States was decreasing. Since the derivative of >( ) ( ) is increasing, the graph of >( ) ( ) is concave up. The concavity tells us that >( ) ( ) was decreasing at a slower rate. Thus, the percentage of oil produced in Mexico that was exported to the United States was decreasing at a slower rate (choice (C)). 83. To locate candidates for points of inflection, we set ( ) = 0 and solve for . " ( ) = 1 + 3 " 3 ( ) = ( 3 ln .3)( 1) " ln .3 = (1 + 3 ) 2 (1 + 3 ) 2 3 ln 3(1 + 3 ) 2 3 2(1 + 3 )( 3 ln 3)( 1) ( ) = " ln .3 # (1 + 3 ) 4 (1 + 3 ) + 2 3 2 1 + 3 \qquad = " (ln .3) 2(1 + 3 )3 # " (ln .3) 3 = (1 + 3 ) 4 (1 + 3 ) 3 For this to be zero, the numerator of the fraction on the right must be zero: 1 + 3 = 0, or 3 = 1, giving 3 = 1 Taking natural logarithms and solving for : ln .3 = ln(1 ) = ln . , or = ln . ln .3, which is the only possible candidate for a point of inflection. We already know from the shape of the logistic curve that there must be a point of inflection somewhere, and so it must occur at = ln . /ln 3. (Alternatively, notice that ( ) changes sign when the numerator ( 1 + 3 ) changes sign, meaning that the graph is concave up on one side of = ln . /ln 3 and concave down on the other, so, again, there must be a point of inflection at = ln . /ln 3.) 84. To locate candidates for points of inflection we set ( ) = 0 and solve for . " ( ) = 1 + " ( ) = ( ) " = (1 + ) 2 (1 + ) 2 2 (1 + ) 2(1 + )( ) ( ) = " # (1 + ) 4 (1 + ) + 2 1 + \qquad = " (1 + ) # = " 2 4 (1 + ) (1 + ) 3


Solutions Section 12.3 For this to be zero, the numerator of the fraction on the right must be zero: 1 + = 0, or = 1 . Taking natural logarithms and solving for x: = ln(1 ) = ln . , or = ln . , which is the only possible candidate for a point of inflection. We already know from the shape of the logistic curve that there must be a point of inflection somewhere, and so it must occur at = ln . . (Alternatively, notice that ( ) changes sign when the numerator ( 1 + ) changes sign, meaning that the graph is concave up on one side of = ln . and concave down on the other, so, again, there must be a point of inflection at = ln . .) 85. Exercise 83 tells us that the point of inflection occurs when = ln . ln .3, where here = 1.1466 and 3 = 1.0357. Thus, = ln .1.1466 ln .1.0357 3.900 4. Since the rate of change of reaches a maximum at the point of inflection, we can say that the population of Puerto Rico was increasing fastest in 1954. 86. Exercise 83 tells us that the point of inflection occurs when = ln . ln .3, where here = 2.3596 and 3 = 1.0767. Thus, = ln .2.3596 ln .1.0767 11.6 12. Since the rate of change of reaches a maximum at the point of inflection, we can say that the population of the Virgin Islands was increasing fastest in 1962. 87. % = , # (1.05) =

10, 000(1.05) 1 + 500 0.5

Technology formula: Spreadsheet: 10000*(1.05)^(-x)/(1+500*exp(-0.5*x)) TI-83/84 Plus: 10000*(1.05)^(-x)/(1+500*e^(-0.5*x)) Here are the graphs of %( ), % ( ), and % ( ): Graph of %( );

Graph of % ( ):


Graph of % ( ):

Solutions Section 12.3

The greatest rate of increase of % occurs when the derivative is greatest. This high point on the graph of % ( ) is located accurately by determining where the graph of % ( ) crosses the -axis: at approximately = 12. The value of the greatest rate of increase at this point is the -coordinate of % ( ) at = 12, which we can determine from the graph of % ( ) as approximately $570 per year. 45 × 22,514(1.03) 1,013,130(1.03) = 1 + 22,514 2.55 1 + 22,514 2.55 Technology formula: 1013130*(1.03)^(-x)/(1+22514*x^(-2.55)) Here are the graphs of %( ), % ( ), and % ( ): Graph of %( ); 88. % = ,(1.03) =

Graph of % ( ):

Graph of % ( ):

The greatest rate of increase of % occurs when the derivative is greatest. This high point on the graph of % ( ) is located accurately by determining where the graph of % ( ) crosses the -axis: at approximately = 21. The value of the greatest rate of increase at this point is the -coordinate of % ( ) at = 21, which we can determine from the graph of % ( ) as approximately $4,140 per year (for example, by graphing


Solutions Section 12.3 % ( ) on a graphing calculator and using the trace feature, or using the calculator to calculate a numerical approximation to the derivative at = 21).

89. %( ) = , 0.05 = (300, 000 + 1, 000 2) 0.05 Technology formula: Excel: (300000+1000*x^2)*exp(-0.05*x) TI-83/84 Plus: (300000+1000*x^2)*e^(-0.05*x) Graph of %( );

Graph of % ( ):

Graph of % ( ):

The greatest rate of increase of % occurs when the the derivative is greatest. This high point on the graph of % ( ) is located accurately by determining where the graph of % ( ) crosses the -axis: at approximately = 17.7. %( ) is decreasing most rapidly at the point where the derivative % ( ) is a minimum, which occurs at = 0 (see the graph of % ( )). 90. %( ) = , 0.05 =

20 0.05 1 + 0.05

Technology formula: Excel: 20*x*exp(-0.05*x)/(1+0.05*x) TI-83/84 Plus: 20*x*e^(-0.05*x)/(1+0.05*x)


Graph of %( );

Solutions Section 12.3

Graph of % ( ):

Graph of % ( ):

The greatest rate of increase of % occurs when the the derivative is greatest. From the graph of % ( ) we see that this occurs at = 0. %( ) is decreasing most rapidly at the point where the derivative % ( ) is a minimum. This low point on the graph of % ( ) is located accurately by determining where the graph of % ( ) crosses the -axis: at approximately = 25.4. 91. Nonnegative 92. Increasing 93. Daily sales were decreasing most rapidly in June 2002. 94. a. Daily sales will increase at the slowest rate next year b. Daily sales will level off in the long term.


Solutions Section 12.3 95.

96.

97. At a point of inflection the graph of a function changes either from concave up to concave down or vice versa. If it changes from concave up to concave down, then the derivative changes from increasing to decreasing and hence has a relative maximum. Similarly, if it changes from concave down to concave up, the derivative has a relative minimum. 98. 8 ( ) measures the rate of change of acceleration; it is positive if acceleration is increasing and negative if acceleration is decreasing. One everyday situation in which it arises is in a car. Typically, a car departs from a standing position with small acceleration, then undergoes increasing acceleration (8 ( ) positive) until the desired cruising speed is approached, and then undergoes a decrease of acceleration (8 ( ) negative) as the speed levels off. If you have a heavy foot on the accelerator, you'll feel a sudden push backward when you step on the gas; if you slam on the brakes you'll feel yourself thrown forward. For this reason 8 ( ) is sometimes called the jerk in the context of motion.


Solutions Section 12.4 Section 12.4

1. a. -intercepts: 2 + 2 + 1 = 0 when ( + 1) 2 = 0, so = 1 -intercept: (0) = 1 b. Extrema: The only extrema are stationary points (no endpoints or singular points of ). ( ) = 2 + 2, so ( ) = 0 when = 1 Absolute minimum at (0, 1) c. Points of inflection: ( ) = 2 0, so no points of inflection d. No singular points of e. as ± Graph:

2. a. -intercepts: 2 2 1 = 0 when ( + 1) 2 = 0, so = 1 -intercept: (0) = 1 b. Extrema: The only extrema are stationary points (no endpoints of singular points of ). ( ) = 2 2, so ( ) = 0 when = 1 Absolute maximum at (0, 1) c. Points of inflection: ( ) = 2 0, so no points of inflection d. No singular points of e. as ± Graph:

3. a. -intercepts: 3 12 = 0 when ( 2 12) = 0, so 12, 0, 12 -intercept: (0) = 0 b. Extrema: Stationary points: ( ) = 3 2 12 = 3( 2 4), so ( ) = 0 when = ±2 Absolute max: ( 2, 16), absolute min: (2, 16) Endpoints: = 4, 4 Absolute min: ( 4, 16) and absolute max: (4, 16) No singular points of (the derivative is defined for all in the domain) c. Points of inflection: ( ) = 6 ; ( ) = 0 when = 0; point of inflection at (0, 0) d. None; the domain is [ 4, 4].


Solutions Section 12.4 Graph:

4. a. -intercepts: 2 3 6 = 0 when (2 2 6) = 0, so 3, 0, 3 -intercept: (0) = 0 b. Extrema: Stationary points: ( ) = 6 2 6 = 6( 2 1), so ( ) = 0 when = ±1 Relative max: ( 1, 4), relative min at (1, 4) Endpoints: = 4, 4 Absolute min: ( 4, 104), absolute max: (4, 104) No singular points of (the derivative is defined for all in the domain). c. Points of inflection: ( ) = 12 ; ( ) = 0 when = 0; point of inflection at (0, 0) d. None; the domain is [ 4, 4]. Graph:

5. a. -intercepts (from the graph) 3.6, 0, 5.1 -intercept: (0) = 0 b. Extrema: The only extrema are the stationary points: ( ) = 6 2 6 36 = 6( 3)( + 2); 6( 3)( + 2) = 0 when = 2 and 3. Relative max: ( 2, 44), relative min at (3, 81) c. ( ) = 12 6; 12 6 = 0 when = 0.5 Point of inflection at (0.5, 18.5) d. None e. as ; as Graph:

6. a. -intercepts (from the graph) 5.1, 0, 3.6


Solutions Section 12.4 -intercept: (0) = 0 b. The only extrema are the stationary points: ( ) = 6 2 6 + 36 = 6( + 3)( 2); 6( + 3)( 2) = 0 when = 3 and 2. Relative min at ( 3, 81), relative max: (2, 44) c. ( ) = 12 6; 12 6 = 0 when = 0.5 Point of inflection at ( 0.5, 18.5) d. None e. as ; as Graph:

7. a. -intercepts (from the graph) 3.3, 0.1, 1.8 -intercept: (0) = 1 b. The only extrema are the stationary points: ( ) = 6 2 + 6 12 = 6( + 2)( 1); 6( + 2)( 1) = 0 when = 2 or = 1 Relative max: ( 2, 21), relative min at (1, 6) c. ( ) = 12 + 6; ( ) = 0 when = 1 2 Point of inflection at ( 1/2, 15 2) d. None e. as ; as Graph:

8. a. -intercept (from the graph) 1.1 -intercept: (0) = 2 b. The only extrema are the stationary points: ( ) = 12 2 + 6 = 6 (2 + 1); 6 (2 + 1) = 0 when = 1 2 or = 0 Relative max: ( 1/2,9 4), relative min at (0, 2) c. ( ) = 24 + 6; ( ) = 0 when = 1 4 Point of inflection at ( 1 4, 17 8) d. None e. as ; as


Solutions Section 12.4 Graph:

9. a. -intercepts (from the graph) 2.9, 4.2 -intercept: (0) = 10 b. The only extrema are the stationary points: ( ) = 12 3 + 12 2 + 72 = 12 ( 2 6) = 12 ( 3)( + 2); 12 ( 3)( + 2) = 0 when = 0, 2, or 3. Relative max: ( 2, 74), relative min at(0, 10),absolute max: (3, 199) c. ( ) = 12(3 2 2 6); ( ) = 0 when 3 2 2 6 = 0 The quadratic formula gives the solutions as 1.12, 1.79 Points of inflection at ( 1.12, 44.8), (1.79, 117.1) d. None e. as ; as Graph:

10. a. -intercepts (from the graph) 4.2, 2.9 -intercept: (0) = 10 b. The only extrema are the stationary points: ( ) = 12 3 + 12 2 72 = 12 ( 2 + 6) = 12 ( + 3)( 2); 12 ( + 3)( 2) = 0 when = 0, 3, or 2. Absolute min: ( 3, 199), relative max: (0, 10), relative min at (2, 74) c. ( ) = 12(3 2 + 2 6); ( ) = 0 when 3 2 + 2 6 = 0 The quadratic formula gives the solutions as 1.79, 1.12 Points of inflection at ( 1.79, 117.3), (1.12, 44.8). d. None e. as ; as


Solutions Section 12.4 Graph:

11. a. -intercepts 1 4 2 3 1 2 1 2 1 + = 0 when 2� 2 + � = 0. 4 3 2 4 3 2 The quadratic in parentheses has no solution (its discriminant 3 2 44. is negative). Thus, the only solution, and hence -intercept, is = 0. -intercept: (0) = 0 b. The only extrema are the stationary points: ( ) = 3 2 2 + = ( 2 2 + 1) = ( 1) 2; ( 1) 2 = 0 when = 0, 1 Note that increases from the stationary point at = 0 to the one at = 1, then continues to increase to the right (as is clear from the graph or from a test point to the right of = 1). So, has an absolute min: (0, 0). c. ( ) = 3 2 4 + 1; 3 2 4 + 1 = 0 when (3 1)( 1) = 0, or = 1 3 or = 1 Points of inflection at (1 3, 11 324) and (1, 1 12) d. None e. as ; as Graph:

12. a. -intercepts: From the graph we see that there are no -intercepts. -intercept: (0) = 1 b. The only extrema are the stationary points: ( ) = 12 3 48 2 + 48 = 12 ( 2) 2; 12 ( 2) 2 = 0 when = 0, 2. Note that increases from the stationary point at = 0 to the one at = 2, then continues to increase to the right (as is clear from the graph or from a test point to the right of = 2). So, has an absolute min: (0, 1). c. ( ) = 36 2 96 + 48 = 12(3 2)( 2); 12(3 2)( 2) = 0 when = 2 3 or = 2. Points of inflection at (2 3, 203 27) and (2, 17) d. None e. as ; as


Solutions Section 12.4 Graph:

1 Multiply both sides by to get 2 + 1 = 0. This equation has no real solutions, so there are no -intercepts. -intercept: (0) is not defined (the domain of includes all numbers except 0). Thus, there is no -intercept. b. The only extrema are the stationary points: 1 ( ) = 1 2 ; ( ) = 0 when 2 = 1, = 1, + 1. has a relative min at (1, 2), and a relative max at ( 1, 2). 2 c. ( ) = 3 and thus is never zero. So there are no points of inflection. d. is singular at 0. 1 1 As 0 , = + . As 0 + , = + , so there is a vertical asymptote at = 0. e. as ; as Graph: 13. a. -intercepts: 0 = +

1 2 Multiply both sides by 2 to get 4 + 1 = 0. This equation has no real solutions, so there are no -intercepts. -intercept: (0) is not defined (the domain of includes all numbers except 0). Thus, there is no -intercept. b. The only extrema are the stationary points: 2 ( ) = 2 3 ; ( ) = 0 when 4 = 1, = 1, 1. has an absolute min: ( 1, 2) and (1, 2). 6 c. ( ) = 2 + 4 and thus is never zero. So there are no points of inflection. 1 1 d. is singular at 0. As 0 , = 2 + 2 . As 0 + , = 2 + 2 , so there is a vertical asymptote at = 0. e. as ; as 14. a. -intercepts: 0 = 2 +


Solutions Section 12.4 Graph:

15. a. -intercepts: = 0 when 3 ( 2 + 3) = 0, so the numerator must be zero: = 0 -intercept: (0) = 0 b. The only extrema are the stationary points: 3 2( 2 + 3) 3(2 ) 4 + 9 2 ( ) = . = ( 2 + 3) 2 ( 2 + 3) 2 ( ) = 0 when 4 + 9 2 = 0 ⇒ 2( 2 + 9) = 0 ⇒ = 0 From the graph (or by using test points or the first derivative test), we see that = 0 is a nonextreme stationary point. Thus, there are no local extrema. (4 3 + 18 )( 2 + 3) 2 ( 4 + 9 2)(2)( 2 + 3)(2 ) c. ( ) = ( 2 + 3) 4 2 3 ( + 3)( 6 + 54 ) 6 ( 2 9) = = ( 2 + 3) 4 ( 2 + 3) 3 ( ) = 0 when = 0 or = 3, + 3 It is difficult to tell from the graph, but the points at = ±3 are points of inflection. We can tell by computing, for example, (2) = 60 7 3 > 0 and (4) = 168 19 3 < 0, which shows that the concavity changes at = 3. So, has points of inflection at (0, 0), ( 3, 9 4), and (3, 9 4). d. No singular points of e. as ; as Graph:

16. a. -intercepts: = 0 when 3 ( 2 3) = 0, so the numerator must be zero: = 0 -intercept: (0) = 0 b. The only extrema are the stationary points: 3 2( 2 3) 3(2 ) 4 9 2 ( ) = . = ( 2 3) 2 ( 2 3) 2 ( ) = 0 when 4 9 2 = 0 ⇒ 2( 2 9) = 0 ⇒ = 0 or = ±3 From the graph (or by using test points or the first derivative test) we see that = 0 is a nonextreme stationary point. The other two stationary points are a relative max: ( 3, 9 2), and a relative min at (3, 9 2). (4 3 18)( 2 3) 2 ( 4 9 2)(2)( 2 3)(2 ) c. ( ) = ( 2 3) 4


Solutions Section 12.4 ( 2 3)(6 3 + 54 ) 6 ( 2 + 9) = = ( 2 3) 4 ( 2 3) 3 ( ) = 0 when = 0 Point of inflection at (0, 0) + d. As 3 , . As 3 , , so there is a vertical asymptote at = 3. + As 3 , . As 3 , , so there is a vertical asymptote at = 3. e. as ; as Graph:

17. a. -intercepts: 0 =

2

18. a. -intercepts: 0 =

2

+ 1

2 1

. Thus the numerator must be zero: 2 + 1 = 0. This equation has no real

solutions, so there are no -intercepts. -intercept: (0) = 1 b. The only extrema are the stationary points: 2 ( 2 1) ( 2 + 1)(2 ) 4 ( ) = , = ( 2 1) 2 ( 2 1) 2 and is zero when the numerator, = 0. This gives a relative max: (0, 1). Endpoints: = 2, 2 These give a relative min at ( 2, 5 3) and at (2, 5 3). 4( 2 1) 2 + 4 (2)( 2 1)(2 ) ( 2 1)[ 4( 2 1) + 16 2] 12 2 + 4 c. ( ) = , = = ( 2 1) 4 ( 2 1) 4 ( 2 1) 3 and thus is never zero (because the numerator cannot be zero). So there are no points of inflection. d. is singular at = ±1. As 1 , . As 1 + , , so there is a vertical asymptote at = 1. As 1 , . As 1 + , , so there is a vertical asymptote at = 1. e. The domain of is [ 2, 2] so there is no limiting behavior at infinity. Graph:

1

2+ 1

Thus the numerator must be zero: 2 1 = 0. So the -intercepts are = ±1.

-intercept: (0) = 1 b. The only extrema are the stationary points: 2 ( 2 + 1) ( 2 1)(2 ) 4 ( ) = ; ( ) = 0 when = 0. = ( 2 + 1) 2 ( 2 + 1) 2 This gives an absolute min: (0, 1). Endpoints: = 2, 2. These give absolute maxes at ( 2, 3 5) and (2, 3 5).


Solutions Section 12.4 2 2 2 4( 1) 4 (2)( 1)(2 ) ( 2 + 1)[4( 2 + 1) 16 2] 4 12 2 + + c. ( ) = = = ( 2 + 1) 4 ( 2 + 1) 4 ( 2 + 1) 3 ( ) = 0 when 4 12 2 = 0 = ±1 3 = ± 3 3 This gives points of inflection at ( 3 3, 1 2) and ( 3 3, 1 2). d. None e. The domain of is [ 2, 2] so there is no limiting behavior at infinity. Graph:

2 + ( + 1) 2 3 3 a. -intercepts: From the graph: 0.7; -intercept: (0) = 1 b. Relative extrema: 2 ( ) = + ( + 1) 2 3 3 2 2 2 2 ( ) = + ( + 1) 1 3 = + 3 3 3 3( + 1) 1 3 2 2 2 2 Stationary points: ( ) = 0 when + ⇒ = 0; = 3 3( + 1) 1 3 3 3( + 1) 1 3 Cross-multiply: 6( + 1) 1 3 = 6, giving ( + 1) 1 3 = 1, so that ( + 1) = ( 1) 3 = 1 ⇒ so we have a stationary point at = 2. Singular points of : ( ) is undefined when = 1, so we have a singular point at = 1. In addition, we use a test point to the left of = 2 and to the right of = 1 : 19. ( ) =

3

2

1

( ) 0.41 1 3 2 3

= 2,

0

1

From the graph we see that we have a relative maximum at ( 2, 1 3) and a relative minimum at ( 1, 2 3). c. Points of inflection: 2 2 2 ( ) = , so ( ) = + 3 3( + 1) 1 3 9( + 1) 4 3 ( ) is never zero, but it is not defined when = 1. So, the only candidate for a point of inflection is = 1, which we see from the graph is not one. No points of inflection. d. Behavior near singular points of : The domain of this function consists of all real numbers, so there are no such points. e. Behavior at infinity: lim ( ) = lim �2 3 + ( + 1) 2 3� = (computing ( ) for large negative values of gives large negative numbers) lim ( ) = lim �2 3 + ( + 1) 2 3� = (computing ( ) for large positive values of gives large positive numbers)


Solutions Section 12.4 Graph:

2 ( 1) 2 5 5 a. -intercepts: From the graph: 0.9, 1.1, and 3.7; -intercept: (0) = 1 b. Relative extrema: 2 2 2 2 ( ) = ( 1) 3 5 = 5 5 5 5( 1) 3 5 2 2 2 2 Stationary points: ( ) = 0 when ⇒ = 0; = 5 5( 1) 3 5 5 5( 1) 3 5 Cross-multiply: 10( 1) 3 5 = 10, giving ( 1) 3 5 = 1, so that ( 1) = 1 5 3 = 1 ⇒ = 2, so we have a stationary point at = 2. Singular points of : ( ) is undefined when = 1, so we have a singular point at = 1. Thus a stationary point at = 2 and a singular point at = 1. In addition we use test points to the left of = 1 and to the right of = 2 : 20. ( ) =

( )

0

1

1

2

3

2 5 1 5 0.12

From the graph we see that we have a relative maximum at (1, 2 5) and a relative minimum at (2, 1 5). c. Points of inflection: 2 2 6 ( ) = , so ( ) = . 5 5( 1) 3 5 25( 1) 8 5 ( ) is never zero, but it is not defined when = 1. So, the only candidate for a point of inflection is = 1, which we see from the graph is not one. No points of inflection. d. Behavior near singular points of : The domain of this function consists of all real numbers, so there are no such points. e. Behavior at infinity: lim ( ) = lim �2 5 ( 1) 2 5� = (computing ( ) for large negative values of gives large negative numbers) lim ( ) = lim �2 5 ( 1) 2 5� = (computing ( ) for large positive values of gives large positive numbers)


Solutions Section 12.4 Graph:

21. a. -intercepts: = 0 when ln . = 0, so = ln . . In exponential form this equation is = . However, > for every , and so the equation has no solution. Hence there are no -intercepts. -intercept: (0) is not defined, so there is no -intercept either. b. Extrema: 1 Stationary: ( ) = 1 , so that ( ) = 0 when = 1, so there is a stationary point at = 1 : an absolute minimum at (1, 1). Since ( ) is defined for all in the domain of , there are no singular points of . 1 c. ( ) = 2 , so ( ) is never 0, and hence there are no points of inflection. d. As 0 + , , so there is a vertical asymptote at = 0. e. as Graph:

22. a. -intercepts: = 0 when ln . 2 = 0, so = ln . 2. In exponential form this equation is = 2. However, > 2 for every , and so the equation has no solution. Hence there are no -intercepts. -intercept: (0) is not defined, so there is no -intercept either. b. Extrema: 2 Stationary: ( ) = 1 , so that ( ) = 0 when = 2, so there is a stationary point at = 2 : an absolute minimum at (2, 2 ln .4). Since ( ) is defined for all in the domain of , there are no singular points of . 2 c. ( ) = 2 , so ( ) is never 0, and hence there are no points of inflection. d. As 0 + , , so there is a vertical asymptote at = 0. e. as


Solutions Section 12.4 Graph:

23. a. -intercepts: From the graph, ±0.8. -intercept: (0) is not defined, so there is no -intercept. b. Extrema: 2 2 2 2 Stationary points: ( ) = 2 + 2 = 2 + . This is zero when 2 + = 0 ⇒ 2 = and so 2 2 = 2, which has no real solutions, so ( ) is never 0 and there are no stationary points. Since ( ) is defined for all in the domain of , there are no singular points of . There are no endpoints either, and hence no extrema. 2 2 2 c. ( ) = 2 2 . Thus, ( ) = 0 when 2 2 = 0, giving 2 = 2 ⇒ 2 = 2 2 ⇒ 2 = 1 ⇒ = ±1 Thus, there are points of inflection at (1, 1) and ( 1, 1). d. As 0, , so there is a vertical asymptote at = 0. e. as ±

24. a. -intercepts: From the graph, 0.5. -intercept: (0) is not defined, so there is no -intercept. b. Extrema: 1 1 Stationary points: ( ) = 4 + . This is zero when 4 = ⇒ 4 2 = 1, which has no real solutions, so ( ) is never 0 and there are no stationary points. Since ( ) is defined for all in the domain of , there are no singular points of . There are no endpoints either, and hence no extrema. 1 1 1 1 1 c. ( ) = 4 2 . Thus, ( ) = 0 when 4 2 = 0, giving 4 = 2 ⇒ 2 = ⇒ = 4 2 Thus, there is a point of inflection at (1 2, 1 2 ln .2). d. As 0 + , , so there is a vertical asymptote at = 0. e. as


Solutions Section 12.4 25. a. -intercepts: = 0 when = , which has no real solutions as > for every . -intercept: (0) = 0 = 1 b. Extrema: Stationary points: ( ) = 1. So, ( ) = 0 when = 1 ⇒ = 0. This gives an absolute minimum at (0, 1). Since ( ) is defined for all in the interior of the domain of there are no singular points of . Endpoints: = 1, = 1 Absolute max: (1, 1), and a relative max: ( 1, 1 + 1) c. ( ) = , which can never be zero d. No singular points of e. None (domain is [ 1, 1])

2

2

26. a. -intercepts: = 0 has no real solutions as > 0 for every . -intercept: (0) = 0 = 1 b. Extrema: 2

Stationary points: ( ) = 2 , so ( ) = 0 when = 0, giving an absolute maximum at (0, 1) Since ( ) is defined for all in the domain of , there are no singular points of . Endpoints: None 2

2

2

c. ( ) = 2 2 ( 2 ) = 2(2 2 1) ; ( ) = 0 when 2 2 1 = 0 Points of inflection at ( 1 2, 1 2) and (1 2, 1 2) d. No singular points of e. As ± , 0 (horizontal asymptote: = 0)

= ±1 2.

27. ( ) = 4 2 3 + 2 2 + 1 ( ) = 4 3 6 2 + 2 2; by graphing ( ), we see that ( ) = 0 for 1.40; ( ) is always 1 3 defined. ( ) = 12 2 12 + 2; ( ) = 0 for = ± (by the quadratic formula), 0.21 or 2 6 0.79; ( ) is always defined.

has an absolute min: (1.40, 1.49) and points of inflection at (0.21, 0.61) and (0.79, 0.55).


Solutions Section 12.4 28. ( ) = 4 + 3 + 2 + + 1 ( ) = 4 3 + 3 2 + 2 + 1; by graphing ( ), we see that ( ) = 0 for 0.61; ( ) is always defined. ( ) = 12 2 + 6 + 2; for this quadratic, 3 2 44. = 36 96 < 0, so ( ) is never 0; ( ) is always defined.

has an absolute min: ( 0.61, 0.67) and no points of inflection.

29. ( ) = 3 ( ) = 3 2; by graphing ( ), we see that ( ) = 0 for 0.46, 0.91, and 3.73; ( ) is always defined. ( ) = 6 ; by graphing ( ), we see that ( ) = 0 for 0.20 and 2.83; ( ) is always defined.

has a relative min at ( 0.46, 0.73), a relative max: (0.91, 1.73), an absolute min: (3.73, 10.22), and points of inflection at (0.20, 1.22) and (2.83, 5.74). 4 4 ( ) = 3; by graphing ( ), we see that ( ) = 0 for 1.86 and 4.54; ( ) is always defined. ( ) = 3 2; by graphing ( ), we see that ( ) = 0 for 0.46, 0.91, and 3.73. 30. ( ) =

has a relative max: (1.86, 3.43), a relative min at (4.54, 12.52), and points of inflection at ( 0.46, 0.62), (0.91, 2.31), and (3.73, 6.71).

31. -intercept: 150; -intercepts: none. The home price index was around 150 at the start of 2010 ( = 0). Extrema: Relative maximum: (0, 150); absolute min: (2, 140); no absolute maximum (the curve apporaches a high of around 250 but never reaches it). The home price index was higher at the start of 2010 ( = 0) than soon after and was at its lowest point of around 140 at the start of 2012 ( = 2) . Point of inflection at (7, 190). The home price index was increasing most rapidly at the start of 2017 ( = 7) when it was around 190. No singular points. Behavior at infinity: As , 250. Assuming that the trend shown in the graph continued


Solutions Section 12.4 indefinitely, the home price index would approach a value of 250 in the long term.

32. -intercept: 140; -intercepts: none. The home price index was around 140 at the start of 2010 ( = 0). Extrema: Relative minimum: (0, 140); absolute maximum: (10, 235); no absolute minimum (the curve apporaches a low of around 130 but never reaches it). The home price index was lower at the start of 2010 ( = 0) than soon after and at its highest point of around 235 at the start of 2020 ( = 0). Points of inflection at (5, 190). and (15, 190). The home price index was increasing most rapidly at the start of 2015 ( = 5).when it was around 190, and decreasing most rapidly at the start of 2025 ( = 15) when it was again around 190. No singular points. Behavior at infinity: As , 130. Assuming that the trend shown in the graph continued indefinitely, the home price index would approach a value of 130 in the long term. 33. a. &-intercept: slightly more than 1; -intercept: 5.5. At the start of the period shown, the radius of the Earth's orbit will be slightly more than one AU and, five and a half million years later, it will be zero after it has spiraled into the core of the Sun. Extrema: Absolute minimum at (5.5, 0); absolute maximum at (5, 1.07). The Earth's orbital radius will reach a maximum of 1.07 AU at = 5 after which point it will spiral into the core of the Sun (& = 0 AU) at = 5.5. Point of inflection at (4, 1.05) : The radius will be increasing most rapidly at = 4, when it will be 1.05 AU. Singular points of : None. Behavior at infinity: As , & 1. At times much earlier than the period shown, the radius of the Earth was close to 1 AU. b. During the period 4 < < 5, the graph is rising, so that & will increase. However, the slope of the tangent is decreasing toward the stationary point at 5, so the rate of change will decrease. 34. a. &-intercept: 0.2; -intercept: none. At the start of the period shown, the radius of the Odyssia's orbit was 0.2 LU, and was never 0 LU. Extrema: Absolute minimum at (5, 0.14); relative maximum at (0, 0.2). Odyssia's orbital radius reached a minimum of 0.14 LU at = 5. At time = 0 it was further out than at subsequent nearby times. Point of inflection: (4, 0.16): The radius was decreasing most rapidly at = 4, when it was 0.16 LU. Singular point of at 5.5; As 5.5 , & ; as approached 5.5, the planet was flung out to infinity. Behavior at infinity does not apply as the domain is [0, 5.5). b. During the period 4 < < 5, the graph is dropping, so that & was decreasing. However, the slope of the tangent is increasing toward the stationary point at 5, so the rate of change was increasing.

35. a. Intercepts: No -intercept (seen from the graph); -intercept at >(0) = 260. The CPI was never zero during the period under consideration; in June 2020 ( = 0) the CPI was 260. Extrema: Stationary points: > ( ) = 0.09 2 + 0.2. Using the quadratic formula, we see that > ( ) = 0 when 0.20 and 10.9. Only the first of these is in the domain so we reject the second. The value of >(0.2) is only fractionally below 260, at 259.979764. This is the absolute minimum. Endpoints: 0, 8. The corresponding values of >( ) are 260 and approximately 275. Absolute max: (8, 275), absolute min: (.2, 259.97976), relative max: (0, 260). The CPI dropped to a low fractionally under 260 just after the start of November 2020 and rose to a high of around 275 at the start of July 2021. Points of inflection occur when > ( ) = 0: 0.18 + 1 = 0 1 5.56. = 0.18 There is a point of inflection at around (5.56, 269) The rate of change of the CPI (inflation) reached a


Solutions Section 12.4 maximum around = 5.56 (around mid April 2021) when the CPI was around 269. b. At the stationary point shortly after the start of Novemeber 2020, > ( ) = 0, making the inflation rate, > ( ) >( ) zero at that point as well.

36. a. Intercepts: No -intercept (seen from the graph); -intercept at >(0) = 253 The CPI was never zero during the period under consideration; at the start of February 2019 ( = 0) the CPI was 253. Extrema: Stationary points: > ( ) = 0.075 2 0.76 + 2. The discriminant 3 2 44. of ths quadratic is negative, so there are no real solutions, and hence no stationary points. Endpoints: 0, 8. The corresponding values of >( ) are 253 and 257.5. Absolute min: (0, 253), absolute max: (8, 257.5) The CPI was at a low of 253 at the start of February 2019, and rose to a high of around 258 at the start of October 2019. Points of inflection occur when > ( ) = 0: > ( ) = 0.075 2 0.76 + 2 so, > ( ) = 0.15 0.76 0.76 0.15 0.76 = 0 + = = 5.07. 0.15 There is a point of inflection at around (5.07, 257). The rate of change of the CPI (inflation) reached a minimum around = 5.07 (around the beginning of July) when the CPI was 257. b. As > ( ) is never zero (there are no stationary points), it is positive throught the period shown, so the inflation rate was positive throughout the period as well. 37. Extrema: Endpoint: = 0 Stationary points: 8 ( ) = 6 2 6 = 6 ( 1); 8 ( ) = 0 when = 0 and = 1 (Note that the derivative is not defined at = 0 so it is an endpoint; not a stationary point.) No singular points of The corresponding values of 8 are: 8(0) = 100; 8(1) = 2(1) 3 3(1) 2 + 100 = 99. Relative max: (0, 100), absolute min: (1, 99) Points of inflection: 8 ( ) = 12 6; 8 ( ) = 0 when = 1 2 The corresponding value of 8 is: 8(1 2) = 2(1 2) 3 3(1 2) 2 + 100 = 199 2 = 99.5. Point of inflection at (0.5, 99.5) Behavior at infinity: As , 8( ) = 2(2 3) + 100 as well. At time = 0 seconds, the UFO is 100 ft away from the observer, and begins to move closer. At time = 0.5 seconds, when the UFO is 99.5 feet away, its distance is decreasing most rapidly (it is moving toward the observer most rapidly). It then slows down to a stop at = 1 sec. when it is at its closest point (99 ft away) and then begins to move further and further away. Graph:

38. Extrema:


Solutions Section 12.4 Endpoint: = 0 Stationary points: 8 ( ) = 6( 1) 2 6( 1) = 6( 1)( 2); 8 ( ) = 0 when = 1 and = 2 No singular points of The corresponding values of 8 are 8(1) = 2(1 1) 3 3(1 1) 2 + 100 = 100 8(2) = 2(2 1) 3 3(2 1) 2 + 100 = 99. Endpoint: = 0. The corresponding value of 8 is 8(0) = 2( 1) 3 3( 1) 2 + 100 = 95. Absolute min: (0, 95), relative max: (1, 100), relative min at (2, 99) Points of inflection: 8 ( ) = 12( 1) 6; 8 ( ) = 0 when = 3 2 The corresponding value of 8 is 8(3 2) = 2(3 2 1) 3 3(3 2 1) 2 + 100 = 99.5. Point of inflection at (1.5, 99.5). Behavior at infinity: As , 8( ) = ( 1) 2[2( 1) 3] + 100 as well. At time = 0 seconds the orbiter is closest to you at 95 km and moving away. At = 1 second, the orbiter is 100 km away, has stopped moving away, and begins to move toward you. At time = 1.5 seconds, when the orbiter is 99.5 km away, its distance is decreasing most rapidly (it is moving toward you most rapidly). It then slows down to a stop at = 2 seconds when it is at its closest point (99 km away) and then begins to move further and further away. Graph:

39. ( ) = 400,000 + 590 + 0.001 2, so the average cost function is 400,000 ( ) = + 590 + 0.001 ( > 0). Intercepts: -intercepts occur when ( ) = 0. However, for > 0, ( ) is positive and never zero. So, there are no -intercepts. -intercept: (0) is not defined; no -intercept Extrema: Stationary points: 400,000 400,000 400,000 ( ) = 0 + 0.001 = 0 = 0.001 2 = = 400,000,000, 2 2 0.001 giving = 400,000,000 = 20,000 iPhones per hour 400,000 The resulting average cost is (20,000) = + 590 + 0.001(20,000) = $630. 20,000 Endpoints: None; Singular points of : None in the domain Absolute minimum at (20,000, 630) 800,000 Points of inflection: ( ) = , which can never be zero. No points of inflection. 3 Behavior near singular points of : ( ) is not defined when = 0. As 0 + , ( ) , so there is a vertical asymptote at = 0. Behavior at infinity: As , ( ) Interpretation: The average cost is never zero, nor is it defined for zero iPhone 12 Pros. The average cost is a minimum ($630) when 20,000 iPhone 12 Pros are manufactured per hour. The average cost becomes


Solutions Section 12.4 extremely large for very small or very large numbers of iPhone 12 Pros. Graph:

40. ( ) = 800,000 + 510 + 0.0005 2, so the average cost function is 800,000 ( ) = + 510 + 0.0005 ( > 0). Intercepts: -intercepts occur when ( ) = 0. However, for > 0, ( ) is positive and never zero. So, there are no -intercepts. -intercept: (0) is not defined; no -intercept Extrema: Stationary points: 800,000 800,000 800,000 ( ) = 0 + 0.0005 = 0 = 0.0005 2 = = 1,600,000,000, 0.0005 2 2 giving = 1,600,000,000 = 40,000 consoles per hour 800,000 The resulting average cost is (40,000) = + 510 + 0.0005(40,000) = $550. 40,000 Endpoints: None; Singular points of : None in the domain Absolute minimum at (40,000, 550) 1,600,000 Points of inflection: ( ) = , which can never be zero. No points of inflection. 3 Behavior near singular points of : ( ) is not defined when = 0. As 0 + , ( ) , so there is a vertical asymptote at = 0. Behavior at infinity: As , ( ) Interpretation: The average cost is never zero, nor is it defined for zero consoles. The average cost is a minimum ($550) when 40,000 consoles are manufactured per hour. The average cost becomes extremely large for very small or very large numbers of consoles. Graph:

41.


Solutions Section 12.4 Graph of derivative:

If we zoom in, we can determine that the maximum occurs around = 3.7; during the year 2003. Since ( ) represents the rate of change of the percentage of mortgages that were subprime, we can say that the percentage of mortgages that were subprime was increasing most rapidly during 2003, when it increased at a rate of around 2.2 percentage points per year. As , ( ) 0. In the long term, assuming the trend shown in the model continues, the rate of change of the percentage of mortgages that were subprime approaches zero; that is, the percentage of mortgages that were subprime approaches a constant value. 42. Graph of derivative:

If we zoom in, we can determine that a maximum value of around 179 occurs around = 2.7, during the year 2002. Since ( ) represents the rate of change of the value of subprime mortgages, we can say that the value of subprime mortgages was increasing most rapidly during 2002, when it increased at a rate of around $179 billion per year. As , ( ) 0. In the long term, assuming the trend shown in the model continues, the rate of change of the value of subprime mortgages approaches zero; that is, the value of subprime mortgages approaches a constant value.

43. No; yes. Near a vertical asymptote the value of increases without bound, and so the graph could not be included between two horizontal lines; hence no vertical asymptotes are possible. Horizontal asymptotes are possible, as, for instance, in the graph in Exercise 31. 44. Yes; yes. One can have at a vertical asymptote as, for instance, in = 1 2. Horizontal asymptotes are also possible, as for instance in the graph of = 1 2.

45. It too has a vertical asymptote at = 4; the magnitude of the derivative increases without bound as 4. 46. It too has a horizontal asymptote at = 4; the derivative increases toward zero as

.

47. No. If the leftmost critical point is a relative maximum, the function will decrease from there until it reaches the rightmost critical point, so it can't have a relative maximum there. 48. Yes, if at least one of the critical points is a singular point of . One example is ( ) = (L L 1) 2.

49. Between every pair of zeros of ( ) there must be a local extremum, which must be a stationary point


of ( ), hence a zero of ( ).

Solutions Section 12.4

50. Between every pair of zeros of ( ) there must be a local extremum of ( ), which gives a point of inflection of ( ).


Solutions Section 12.5 Section 12.5

1. ( ) = 5 + 2 a. The exact value of M is M = ( + M ) ( ):

M = ( + M ) ( ) = 5( + M ) + 2 ( 5 + 2) = 5 5M + 2 ( 5 + 2) = 5M

b. M ( )M = 5M The approximate difference is exact.

2. ( ) = 2 3 a. The exact value of M is M = ( + M ) ( ):

M = ( + M ) ( ) = 2( + M ) 3 (2 3) = 2 + 2M 3 (2 3) = 2M

b. M ( )M = 2M The approximate difference is exact.

3. ( ) = 2 2 4 a. The exact value of M is M = ( + M ) ( ):

M = ( + M ) ( ) = 2( + M ) 2 4 (2 2 4) = 2 2 + 4 M + 2(M ) 2 4 (2 2 4) = 4 M + 2(M ) 2

b. M ( )M = 4 M The approximate difference is off by 2(M ) 2.

4. ( ) = 5 2 + 3 a. The exact value of M is M = ( + M ) ( ):

M = ( + M ) ( ) = 5( + M ) 2 + 3 ( 5 2 + 3) = 5 2 10 M 5(M ) 2 + 3 ( 5 2 + 3) = 10 M 5(M ) 2

b. M ( )M = 10 M The approximate difference is off by 5(M ) 2.

5. ( ) = 4 2 + 2 a. The exact value of M is M = ( + M ) ( ):

M = ( + M ) ( ) = 4( + M ) 2 + 2( + M ) ( 4 2 + 2 ) = 4 2 8 M 4(M ) 2 + 2 + 2M ( 4 2 + 2 ) = 8 M + 2M 4(M ) 2 = ( 8 + 2)M 4(M ) 2

b. M ( )M = ( 8 + 2)M The approximate difference is off by 4(M ) 2.


Solutions Section 12.5 6. ( ) = 5 2 3 a. The exact value of M is M = ( + M ) ( ):

M = ( + M ) ( ) = 5( + M ) 2 3( + M ) (5 2 3 ) = 5 2 + 10 M + 5(M ) 2 3 3M (5 2 3 ) = 10 M 3M + 5(M ) 2 = (10 3)M + 5(M ) 2

b. M ( )M = (10 3)M The approximate difference is off by 5(M ) 2.

2 a. The exact value of M is M = ( + M ) ( ): 7. ( ) =

2 2 + M 2 2( + M ) = ( + M ) 2M = ( + M )

M = ( + M ) ( ) =

2 2M b. M ( )M = 2 M = 2 The approximate difference is off by

2M ? 2M B 2 A ( + M ) DC @

2M 1 1 N + M O 2M ? ( + M ) B = A@ ( + M ) DC 2M ? M B = @A ( + M ) CD 2(M ) 2 = 2 ( + M ) =

3 a. The exact value of M is M = ( + M ) ( ): 8. ( ) =

3 3 + M 3 + 3( + M ) = ( + M ) 3M = ( + M )

M = ( + M ) ( ) =

3 3M M = 2 2 The approximate difference is off by b. M ( )M =

3M 3M 3M 1 1 = N 2 ( M ) M O + +


Solutions Section 12.5 ( M ) + B 3M ? = A@ ( + M ) DC M 3M ? B = A@ ( + M ) DC 3(M ) 2 = 2 ( + M )

9. ( ) = 5 + 2; 4 = 3, M = 0.01 a. By the calculation in Exercise 1 part (a), M = 5M = 5(0.01) = 0.05.

b. By the calculation in Exercise 1 part (b), M 5M = 5(0.01) = 0.05.

The approximate difference is exact.

10. ( ) = 2 3; 4 = 2, M = 0.001 a. By the calculation in Exercise 2 part (a), M = 2M = 2( 0.001) = 0.002.

b. By the calculation in Exercise 2 part (b), M 2M = 2( 0.001) = 0.002.

The approximate difference is exact.

11. ( ) = 2 2 4; 4 = 2, M = 0.001 a. By the calculation in Exercise 3 part (a),

M = 4 M + 2(M ) 2 = 4(2)( 0.001) + 2( 0.001) 2 = 0.007998.

b. By the calculation in Exercise 3 part (b),

M 4 M = 4(2)( 0.001) = 0.008.

The approximate difference differs from the actual difference by | 0.008 ( 0.007998)| = 0.000002.

12. ( ) = 5 2 + 3; 4 = 1, M = 0.002 a. By the calculation in Exercise 4 part (a),

M = 10 M 5(M ) 2 = 10( 1)(0.002) 5(0.002) 2 = 0.01998.

b. By the calculation in Exercise 4 part (b),

M 10 M = 10( 1)(0.002) = 0.02.

The approximate difference differs from the actual difference by |0.02 0.01998| = 0.00002.

13. ( ) = 4 2 + 2 ; 4 = 2, M = 0.0002 a. By the calculation in Exercise 5 part (a),

M = ( 8 + 2)M 4(M ) 2 = ( 8( 2) + 2)( 0.0002) 4( 0.0002) 2 = 0.00360016.

b. By the calculation in Exercise 5 part (b),


M ( 8 + 2)M = ( 8( 2) + 2)( 0.0002) = 0.0036.

Solutions Section 12.5

The approximate difference differs from the actual difference by | 0.0036 ( 0.00360016)| = 0.00000016. 14. ( ) = 5 2 3 ; 4 = 3, M = 0.003 a. By the calculation in Exercise 6 part (a),

M = (10 3)M + 5(M ) 2 = (10(3) 3)( 0.003) + 5( 0.003) 2 = 0.080955.

b. By the calculation in Exercise 6 part (b),

M (10 3)M = (10(3) 3)( 0.003) = 0.081.

The approximate difference differs from the actual difference by | 0.081 ( 0.080955)| = 0.000045. 2 ; 4 = 0.5, M = 0.01 a. By the calculation in Exercise 7 part (a),

15. ( ) =

M =

2( 0.01) 2M 0.08163. = ( + M ) (0.5)(0.5 0.01)

b. By the calculation in Exercise 7 part (b), M

2( 0.01) 2M = = 0.08. 2 (0.5) 2

The approximate difference differs from the actual difference by |0.08 0.08163| = 0.00163.

3 16. ( ) = ; 4 = 0.25, M = 0.001 a. By the calculation in Exercise 8 part (a), M =

3(0.001) 3M 0.04781. = ( + M ) 0.25(0.25 + 0.001)

b. By the calculation in Exercise 8 part (b), M

2( 0.01) 3M 3(0.001) = = = 0.048. 2 (0.25) 2 (0.5) 2

The approximate difference differs from the actual difference by |0.048 0.04781| = 0.00019.

17. = 12 5; = 3, M = 0.001 0 0 a. 0 = 120 . = 12, so 0 = 0 0 b. Take 0 = M = 0.001, so the approximate change in is 0 = 120 = 12( 0.001) = 0.012.

18. = 2.5 + 12; = 3, M = 0.0001 0 0 a. 0 = 2.50 . = 2.5, so 0 = 0 0 b. Take 0 = M = 0.0001, so the approximate change in is


0 = 2.50 = 2.5(0.0001) = 0.00025.

Solutions Section 12.5

19. = 3 2 4 + 5; = 1, M = 0.002 0 0 a. 0 = (6 4)0 . = 6 4, so 0 = 0 0 b. Take 0 = M = 0.002, so the approximate change in is 0 = (6 4)0 = (6( 1) 4)(0.002) = 0.02.

2 + 5 5; = 2, M = 0.003 2 0 0 a. 0 = ( + 5)0 . = + 5, so 0 = 0 0 b. Take 0 = M = 0.003, so the approximate change in is 20. =

0 = ( + 5)0 = ( 2 + 5)( 0.003) = 0.009.

21. 8 =

; = 2, M = 0.001 2 + 1 08 1 08 1 0 a. , so 08 = 0 = 0 = . = 0 0 (2 + 1) 2 (2 + 1) 2 (2 + 1) 2 b. Take 0 = M = 0.001, so the approximate change in 8 is 08 =

0 0.001 = = 0.00004. 2 (2 + 1) (2(2) + 1) 2

3 ; = 3, M = 0.001 2 1 08 08 30 3 3 a. , so 08 = 0 = 0 = . = 0 0 (2 1) 2 (2 1) 2 (2 1) 2 b. Take 0 = M = 0.001, so the approximate change in 8 is 22. 8 =

08 =

30 0.003 = = 0.00012. 2 (2 1) (2(3) 1) 2

23. ! = ; % = 1.5, M% = 0.01 0! 0! a. 0% = 0%. = , so 08 = 0% 0% b. Take 0% = M% = 0.01, so the approximate change in ! is 0! = 0% = 1.5( 0.01) 0.00223.

24. ! = 2 ; % = 0.25, M% = 0.003 0! 0! a. 0% = 2 0%. = 2 , so 08 = 0% 0% b. Take 0% = M% = 0.003, so the approximate change in ! is 0! = 2 0% = 2 0.25(0.003) 0.00770.

25. = ln(1 + ); = 1, M = 0.02 0 0 1 1 0 a. , so 0 = 0 = 0 = . = 0 1 + 0 1+ 1+ b. Take 0 = M = 0.02, so the approximate change in is


0 0.02 0 = = = 0.01. 1+ 1+ 1

Solutions Section 12.5

26. = ln( 1); = 3, M = 0.04 0 0 1 1 0 a. , so 0 = 0 = 0 = . = 0 1 0 1 1 b. Take 0 = M = 0.04, so the approximate change in is 0 =

0 0.04 = = 0.02. 1 3 1

27. ( ) = 12 5; 4 = 3 (4) = 12(3) 5 = 31, ( ) = 12, so (4) = 12. 7( ) = (4) + (4)( 4) = 31 + 12( 3) = 12 5 28. ( ) = 8 + 3; 4 = 1 (4) = 8( 1) + 3 = 11, ( ) = 8, so (4) = 8. 7( ) = (4) + (4)( 4) = 11 8( + 1) = 8 + 3

29. ( ) = 2 + 3 6; 4 = 1 (4) = ( 1) 2 + 3( 1) 6 = 10, ( ) = 2 + 3, so (4) = 2( 1) + 3 = 5. 7( ) = (4) + (4)( 4) = 10 + 5( + 1) = 5 5 30. ( ) = 2 2 5 + 6; 4 = 2 (4) = 2(2) 2 5(2) + 6 = 4, ( ) = 4 5, so (4) = 4(2) 5 = 3. 7( ) = (4) + (4)( 4) = 4 + 3( 2) = 3 2

2 ; 4 = 1 3 1 2(1) 2 2 1 (4) = , so (4) = = 1, ( ) = = . 2 2 3(1) 1 2 (3 1) (3(1) 1) 1 1 3 7( ) = (4) + (4)( 4) = 1 ( 1) = + 2 2 2 31. ( ) =

2 ; 4 = 1 2 + 3 2( 1) 6 6 (4) = , so (4) = = 2, ( ) = = 6. 2 2( 1) + 3 (2 + 3) (2( 1) + 3) 2 7( ) = (4) + (4)( 4) = 2 6( + 1) = 6 4 32. ( ) =

33. ( ) = ; 4 = 0 (4) = 0 = 1, ( ) = , so (4) = 0 = 1. 7( ) = (4) + (4)( 4) = 1 + 1( 0) = 1 +

34. ( ) = ; 4 = 0 (4) = 0 = 1, ( ) = , so (4) = 0 = 1. 7( ) = (4) + (4)( 4) = 1 1( 0) = 1 35. ( ) = 2 0.5 ; 4 = 2 (4) = 2 0.5( 2) = 2 1 =

2 1 , ( ) = 0.5 , so (4) = 0.5( 2) = 1 = .


Solutions Section 12.5 2 1 1 4 + 4 7( ) = (4) + (4)( 4) = + ( + 2) = + =

3 ; 4 = 1 2 3 3 3 3 3 (4) = 1 = , ( ) = , so (4) = 1 = . 2 2 2 2 2 3 3 3 3 3 + 6 7( ) = (4) + (4)( 4) = ( 1) = + = 2 2 2 2 36. ( ) =

37. ( ) = ln(1 + ); 4 = 0

1 1 , so (4) = = 1. 1+ 1+ 0 7( ) = (4) + (4)( 4) = 0 + 1( 0) = (4) = ln(1 + 0) = 0, ( ) =

38. ( ) = ln( 1); 4 = 2

1 1 , so (4) = = 1. 1 2 1 7( ) = (4) + (4)( 4) = 0 + 1( 2) = 2 (4) = ln(2 1) = 0, ( ) =

39. ( ) = ; 4 = 1

(4) = 1 = 1, ( ) =

1

2

, so (4) =

7( ) = (4) + (4)( 4) = 1 +

40. ( ) = 1 + ; 4 = 0

1 . 2

1 1 ( 1) = ( + 1) 2 2

1 1 , so (4) = . 2 2 1 + 1 1 7( ) = (4) + (4)( 4) = 1 + ( 0) = + 1 2 2 (4) = 1 + 0 = 1, ( ) =

41. a. By Exercise 33, 1 +

if is close to 0. Thus,

0.01 1 + 0.01 = 1.01.

b. The actual value of 0.01 is around 1.010050, so the answer is accurate to three decimal places, as rounding it to at most three decimal places gives the same result as part (a). 42. a. By Exercise 34, 1

if is close to 0. Thus,

0.001 1 0.001 = 0.999.

b. The actual value of 0.001 is around 0.9990004998, so the answer is accurate to 6 decimal places, as rounding it to at most 6 decimal places gives the same result as part (a). 43. a. By Exercise 37,


ln(1 + )

Solutions Section 12.5

if is close to 0. Thus,

ln(1.003) = ln(1 + 0.003) 0.003.

b. The actual value of ln(1.003) is around 0.00299551, so the answer is accurate to 5 decimal places, as rounding it to at most 5 decimal places gives the same result as part (a). 44. a. By Exercise 38,

ln( 1) 2

if is close to 2. Thus,

ln(1.0023) = ln(2.0023 1) 2.0023 2 = 0.0023.

b. The actual value of ln(1.0023) is around 0.0022974, so the answer is accurate to 5 decimal places, as rounding it to at most 5 decimal places gives the same result as part (a). 45. a. By Exercise 39,

1 ( + 1) 2

if is close to 1. Thus, 0.9987

1 (0.9987 + 1) = 0.99935. 2

b. The actual value of 0.9987 is around 0.9993497886, so the answer is accurate to 6 decimal places, as rounding it to at most 6 decimal places gives the same result as part (a). 46. a. By Exercise 40, 1 +

1 + 1 2

if is close to 0. Thus,

0.9987 = 1 + ( 0.0013)

1 ( 0.0013) + 1 = 0.99935. 2

b. The actual value of 0.9987 is around 0.9993497886, so the answer is accurate to 6 decimal places, as rounding it to at most 6 decimal places gives the same result as part (a). 47. The area of a disc of radius & is = '& 2, so is

M

0 M& = 2'&M& = 2'(6)(0.05) 1.885 cm2 0&

48. The area of a disc of radius & is = '& 2, so is

0 = 2'&. If & changes by M&, the resulting change in 0&

0 = 2'&. If & changes by M&, the resulting change in 0&


Solutions Section 12.5 0 M M& = 2'&M& = 2'(4)(0.02) 0.503 cm2 0&

49. !(%) = 32 +

2,000 2,000 , so ! (%) = 2 . If % changes by M%, the resulting change in !(%) is % %

M! ! (%)M% =

2,000 2,000 M% = (50) = 0.025 thousand students, %2 2,000 2

or 25 students enrolled. Thus, student enrollment would decline by around 25 students. 50. !(%) = 32 +

2,000 2,000 , so ! (%) = 2 . If % changes by M%, the resulting change in !(%) is % %

M! ! (%)M% =

2,000 2,000 M% = M% = 0.00032M% thousand students, 2 % 2,500 2

We want the change in ! to be 100 students, or 0.1 thousand students, thus we want 0.00032M% = 0.1 + M% =

0.1 = 312.5. 0.00032

Thus, tuition would need to decrease by around $312.50.

51. ( ) = 2,000 + 10 + 0.2 2, so ( ) = 10 + 0.4 , and so 0 = ( )0 = (10 + 0.4 )0 . If = 100 and changes by 0 = 105 100 = 5, then 0 = (10 + 0.4 )0 = (10 + 0.4(100))(5) = 250,

so the resulting change in cost is around $250.

52. ( ) = 3,000 + 8 + 0.1 2, so ( ) = 8 + 0.2 , and so 0 = ( )0 = (8 + 0.2 )0 . If = 150 and changes by 0 = 160 150 = 10, then 0 = (8 + 0.2 )0 = (8 + 0.2(150))(10) = 380,

so the resulting change in cost is around $380.

53. The circumference of a disc with radius & is given by = 2'&. If & is changed by M&, then the corresponding change in circumference is given by M

0 M& = 2'M& 0& = 2'(1) 6.28 m.

54. The circumference of a disc with radius & is given by = 2'&. If & is changed by M&, then the corresponding change in circumference is given by M

0 M& = 2'M& 0& = 2'(1) 6.28 m.

Note that the answer is independent of the radius of the planet! Thus, the same result holds for any spherical object, no matter how large.

55. When the function is linear, the exact difference M is equal to the approximate difference (as


Solutions Section 12.5 measured by 0 ), because the linear approximation is the same as the original function. See, for instance, Exercises 1 and 2.

56. When the function is linear, the linear approximation coincides with the original function—the linear approximation is linear, so the original function must also be linear for the two to be the same. See, for instance, Exercises 27 and 28. 57. The differential of associated with a change 0 in at = 4 is the difference in the linear approximation of at 4 _.

58. The vertical distance, near = 4, between a curve = ( ) and its tangent line at 4 measures the difference between M 0 ____ and __ __.

59. If M is a change in and we take 0 = M , then, if is a function of , 0 gives the approximate associated change in . 60. If M is a change in and we take 0 = M , then, if is a function of , M gives the actual associated change in .


Solutions Section 12.6 Section 12.6

1. The population is currently 10,000 and its rate of change is 1,000 per year: = 10,000 and 0 = 1,000. 0 2. Let 9 be the number of cases of Bangkok flu. 9 = 400 and

09 = 30. 0

3. Let $ be the annual revenue of my company and let ! be annual sales. $ is currently $7,000 and its rate of change is $700 each year. Find how fast ! is changing: 0! 0$ $ = 7,000 and . = 700. Find 0 0 4. Let be the distance from the top of the ladder to the floor, and let be the distance from the base of the ladder to the wall. 0 0 . = 3. Find 0 0 5. Let % be the price of a pair of shoes, and let ! be the demand for shoes. 0% 0! . = 5. Find 0 0 6. Let % be the price of stocks and let , be the value of my portfolio. 0% 0, . = 1,000. Find 0 0

7. Let 1 be the average global temperature and let ! be the number of Bermuda shorts sold per year. 0! 01 1 = 60 and . = 0.1. Find 0 0 8. Let be the population of the country and let ! be the number of diapers sold per year. 0! 0 = 260,000,000 and . = 1,000,000. Find 0 0

9. a. Changing quantities: the radius & and the area 0 0& The problem: when & = 10,000. = 1,200. Find 0 0 2 The relationship: = '& 0 0& Derived equation (0 0 of both sides): = 2'& 0 0 0& Substitute: 1,200 = 2'(10,000) 0 0& so = 6 (100') 0.019km/sec. 0 0& b. This time the problem is to find when = 640,000. From part (a) we have the derived equation 0 0 0& = 2'& . 0 0 Since & appears in the derived equation but not , we need to find & from = '& 2: 640,000 = '& 2 & = 640,000 ' = 800 ' Substituting these values in the derived equation:


0 0& = 2'& 0 0

1,200 = 2'(800 ')

Solutions Section 12.6 0& 0

0& = 3 (4 ') 0.4231 km/sec. 0

10. a. Changing quantities: the radius & and the area 0& 0 The problem: when & = 10. = 5. Find 0 0 The relationship: = '& 2 0 0& Derived equation (0 0 of both sides): = 2'& 0 0 0 2 Substitute: = 2'(10)(5) = 100' 314 cm /sec 0 0 b. This time the problem is to find when = 36. From part (a) we have the derived equation 0 0 0& = 2'& . 0 0 Since & appears in the derived equation but not , we need to find & from = '& 2: 36 = '& 2, & = 36 ' = 6 ' Substituting these values in the derived equation: 0 0& = 2'& . 0 0 0 = 2'(6 ')(5) = 60 ' 106 cm2/sec. 0 11. Changing quantities: The volume ( and the radius &. 0( 0& The problem: when & = 1. = 3. Find 0 0 The relationship: ( = 43 '& 3 0( 0& Derived equation (0 0 of both sides): = 4'& 2 0 0 2 0& Substitute: 3 = 4'(1) 0 0& 3 Solve: 0.24 ft/min = 0 4'

12. Changing quantities: The volume ( and the radius & 0( 0& The problem: when & = 10. = 10. Find 0 0 The relationship: ( = 43 '& 3 0( 0& Derived equation (0 0 of both sides): = 4'& 2 0 0 2 0& Substitute: 10 = 4'(10) 0 0& 1 Solve: 0.08 cm/sec = 0 40'

13. Changing quantities: The volume ( and the radius &. 0& 0( The problem: when & = 93 × 10 6 = 0.003. Find 0 0 The relationship: ( = 43 '& 3


Solutions Section 12.6 0( 0& Derived equation (0 0 of both sides): = 4'& 2 0 0 0( 6 2 Substitute: = 4'(93 × 10 ) (0.003) 326 × 10 12 cubic miles per hour 0 14. Changing quantities: The volume ( and the radius &. 0& 0( The problem: when & = 67 × 10 6 = 0.002. Find 0 0 The relationship: ( = 43 '& 3 0( 0& Derived equation (0 0 of both sides): = 4'& 2 0 0 0( 6 2 Substitute: = 4'(67 × 10 ) (0.002) 113 × 10 12 cubic miles per hour 0

15. Changing quantities: 3 = the distance of the base of the ladder from the wall and = the height of the top of the ladder 03 0 The problem: when 3 = 30. = 10. Find 0 0 The relationship: 3 2 + 2 = 50 2 03 0 Derived equation (0 0 of both sides): 23 + 2 = 0 0 0 We need the value of : 30 2 + 2 = 50 2 gives = 2,500 900 = 40 0 Substitute: 2(30)(10) + 2(40) = 0 0 0 Solve: = 600 80 = 7.5 ft/sec, so the top of the ladder is sliding down at 7.5 ft/sec. 0 16. Changing quantities: 3 = the distance of the base of the ladder from the wall and top of the ladder 0 03 The problem: when = 3. = 10. Find 0 0 The relationship: 3 2 + 2 = 5 2 03 0 Derived equation (0 0 of both sides): 23 + 2 = 0 0 0 We need the value of : 3 2 + 2 = 5 2 gives = 25 9 = 4 03 Substitute: 2(4) + 2(3)( 10) = 0 0 03 Solve: = 60 8 = 7.5 ft/sec 0

= the height of the

17. Changing quantities: = the height of the vehicle 8 = the distance, as show in in the figure.

0 08 when 8 = 1,000. = 100. Find 0 0 The relationship: 600 2 + 2 = 8 2

The problem:


Solutions Section 12.6 0 08 Derived equation (0 0 of both sides): 2 = 28 0 0 2 2 2 We need the value of : 600 + = 1,000 gives = 1,000,000 360,000 = 800 08 Substitute: 2(800)(100) = 2(1,000) 0 08 Solve: = 160,000 2,000 = 80 m/sec 0

18. Changing quantities: = the height of the elevator 8 = the distance, as show in in the figure.

0 08 when 8 = 500. = 16. Find 0 0 The relationship: 300 2 + 2 = 8 2 0 08 Derived equation (0 0 of both sides): 2 = 28 0 0 We need the value of : 300 2 + 2 = 500 2 gives = 500 2 300 2 = 400 0 Substitute: 2(400) = 2(500)( 16) 0 0 Solve: = 16,000 800 = 20 ft/sec 0 The elevator is descending at 20 ft/sec. The problem:

19. Changing quantities: The average cost and the number of CD players 0 0 The problem: = 3,000 and . = 100. Find 0 0 The relationship: ( ) = 150,000 1 + 20 + 0.0001 0 0 0 Derived equation (0 0 of both sides): = 150,000 2 + 0.0001 0 0 0 0 Substitute: = 150,000(3,000) 2(100) + 0.0001(100) 1.66 0 The average cost is decreasing at a rate of $1.66 per gramophone per week.

20. Changing quantities: The average cost and the number of gramophones 0 0 The problem: = 3,000 and . = 100. Find 0 0 The relationship: ( ) = 150,000 1 + 20 + 0.01 0 0 0 Derived equation (0 0 of both sides): = 150,000 2 + 0.01 0 0 0 0 Substitute: = 150,000(3,000) 2(100) + 0.01(100) 0.67 0 The average cost is decreasing at a rate of $0.67 per player per week.

21. Changing quantities: The number ! of T-shirts sold per month and the price % per T-shirt


Solutions Section 12.6 0% 0! The problem: % = 15 and . = 2. Find 0 0 The relationship: This is the given demand equation, ! = 500 100% 0.5. 0! 0% Derived equation (0 0 of both sides): = 50% 0.5 0 0 0! 0.5 Substitute: (2) 26 = 50(15) 0 Monthly sales will drop at a rate of 26 T-shirts per month.

22. Changing quantities: The number ! of memory sticks supplied per month and the price % per memory stick 0% 0! The problem: % = 40 and . = 2. Find 0 0 The relationship: This is the given supply equation, ! = 0.1% 2 + 3%. 0! 0% 0% Derived equation (0 0 of both sides): = 0.2% + 3 0 0 0 0! Substitute: = 0.2(40)( 2) + 3( 2) = 22 0 The supply will decrease at a rate of 22 memory sticks per week. 23. Changing quantities: The price %, the weekly demand !, and the weekly revenue $ 0! 0% 0$ The problem: ! = 50, % = 30¢, and if = 5. Find = 0. 0 0 0 The relationship: $ = %! Revenue = price × quantity 0! 0$ 0% Derived equation (0 0 of both sides): !+ % = 0 0 0 0% 0% Substitute and solve: 0 = (50) + (30)( 5) and so = 150 50 = 3. 0 0 You must raise the price by 3¢ per week.

24. Changing quantities: The price %, the monthly demand !, and the monthly revenue $ 0! 0% 0$ The problem: ! = 40,p= 20,000, and for which is nonnegative. = 3. Find the least value of 0 0 0 0% 0$ This is equivalent to finding the value of for which = 0. 0 0 The relationship: $ = %! Revenue = price × quantity 0! 0$ 0% Derived equation (0 0 of both sides): !+ % = 0 0 0 0% 0% Substitute and solve: 0 = (40) + (20,000)(3) and so = 60,000 40 = 1,500. 0 0 So, you could drop your price by $1,500 per month. 25. Changing quantities: The price %, the daily production !, and the daily revenue $ 0% 0$ The problem: % = 45, ! = 0.01 2 + 0.01 + 2.5, and when = 9 (2019). = 16. Find 0 0 The relationship: $ = %! Revenue = price × quantity 0! 0$ 0% Derived equation (0 0 of both sides): !+ % = 0 0 0 0! We need to know ! and at = 9, which we can get from the formula for !: 0 !( ) = 0.01 2 + 0.01 + 2.5 ! ( ) = 0.02 + 0.01 In 2019: !(9) = 0.01(9) 2 + 0.01(9) + 2.5 = 1.78; ! (9) = 0.02(9) + 0.01 = 0.17 0$ Substitute : = (16)(1.78) + (45)( 0.17) = $20.83 million per year 0


Solutions Section 12.6 Thus the revenue was increasing at a rate of $20.83 million per year

26. Changing quantities: The price %, the daily imports !, and the daily expenditure $ 0% 0$ The problem: % = 50, !( ) = 11 2 170 + 1,300, and when = 10 (2020). = 1.50 Find 0 0 The relationship: $ = %! Revenue = price × quantity 0! 0$ 0% Derived equation (0 0 of both sides): !+ % = 0 0 0 0! We need to know ! and at = 10, which we can get from the formula for !: 0 !( ) = 11 2 170 + 1,300 ! ( ) = 22 170. In 2020: !(10) = 11(10) 2 170(10) + 1,300 = 700; ! (10) = 22(10) 170 = 50 0$ Substitute : = ( 1.50)(700) + (50)(50) = $1,450 thousand, or $1.45 million per year 0

27. Changing quantities: The total food apps revenue $, Uber Eats' market share , and Uber Eats' annual revenue P. 0 0P The problem: $( ) = 0.4 2 + + 26.5, = 27 and . = 2.5. Find 0 0 The relationship: P = $ Uber Eats' revenue 100 = Uber Eats' percentage of the U.S. market × Total U.S. revenue

Derived equation (0 0 of both sides):

0P 1 0 0$ $+ = Q 0 100 0 0 R

0$ at = 1, which we can get from the formula for $: 0 $( ) = 0.4 2 + + 26.5 $ ( ) = 0.8 + 1. In 2021: $(1) = 0.4(1) 2 + 1 + 26.5 = 27.9, $ (1) = 0.8(1) + 1 = 1.8 0P 1 0 0$ 1 Substitute : $+ = = [( 2.5)(27.9) + (27)(1.8)] 0.212 billion dollars, or 0 100 Q 0 0 R 100 $212 million dollars per year. Thus Uber Eats' annual U.S. revenue was projected to decrease at a rate of about $212 million dollars per year. We need to know $ and

28. Changing quantities: The total food apps revenue $, Deliveroo's market share , and Deliveroo's annual revenue 6. 0 06 The problem: $( ) = 0.2 2 + 3 + 18, = 23 and . = 2.5. Find 0 0 The relationship: 6 = $ Deliveroo's revenue 100

= Deliveroo's percentage of the European market × Total European revenue

Derived equation (0 0 of both sides):

06 1 0 0$ $+ = Q 0 100 0 0 R

0$ at = 1, which we can get from the formula for $: 0 $( ) = 0.2 2 + 3 + 18 $ ( ) = 0.4 + 3. In 2021: $(1) = 0.2(1) 2 + 3(1) + 18 = 20.8, $ (1) = 0.4(1) + 3 = 2.6 06 1 0 0$ 1 Substitute : $+ = = [(2.5)(20.8) + (23)(2.6)] = 1.118 billion dollars per year. 0 100 Q 0 0 R 100 Thus Deliveroo's annual European revenue was projected to increase at a rate of about $1.118 billion dollars per year. We need to know $ and


Solutions Section 12.6 29. Changing quantities: The number of laborers, and the number of robots 0 0 The problem: = 400 and . = 16. Find 0 0 The relationship: = 10,000 0 0 Derived equation (0 0 of both sides): + = 0 0 0 We need the value of when = 400: (400) = 10,000, and so = 25. 0 0 400 Substitute and solve: (400) + 25(16) = 0, and so = = 1. 0 0 400 You are laying off 1 laborer per month.

30. Changing quantities: The number of androids, and the number of robots 0 0 The problem: = 5,000 and . = 200. Find 0 0 The relationship: = 1,000,000 0 0 Derived equation (0 0 of both sides): + = 0 0 0 We need the value of when = 5,000: 5,000 = 1,000,000, and so = 200. 0 0 40,000 Substitute and solve: 200(200) + 5,000 = 0, and so = = 8. 0 0 5,000 You are scrapping 8 robots per month.

31. Changing quantities: The number of automobiles produced per year, the number of employees, and the daily operating budget 0 0 The problem: is constant at 1,000. = 150 and . = 10. Find 0 0 The relationship: = 10 0.3 0.7 0 0 Derived equation (0 0 of both sides): 0 = 3 0.7 0.7 (We are told that is constant + 7 0.3 0.3 0 0 so its derivative is zero.) 0 The solution to the problem is a bit simpler if we first solve for before substituting values: 0 0 3 0.7 0.7 3 0 . = 0.3 0.3 = 0 7 0 7 We need the value of : 1,000 = 10(150) 0.3 0.7, and so = (100 150 0.3) 1 0.7 = 100 10 7 150 3 7. 3(100 1 0.7) 150 3 7 0 30(100 10 7) Substitute: (10) = 2.40 = 0 7(150) 7(150) 10 7 The daily operating budget is dropping at a rate of $2.40 per year.

32. Changing quantities: The number of automobiles produced per year, the number of employees, and the daily operating budget 0 0 The problem: = 1,000 and . = 100. = 200 is constant. Find 0 0 The relationship: = 10 0.3 0.7 0 0 Derived equation (0 0 of both sides): (We are told that is constant so its derivative = 7 0.3 0.3 0 0 is zero.) 0 The solution to the problem is a bit simpler if we first solve for before substituting values: 0 0 1 0.3 0 . = N O 0 7 0 We need the value of : 1,000 = 10(200) 0.3 0.7, and so = (100 200 0.3) 1 0.7 = 100 10 7 200 3 7.


Solutions Section 12.6 0.3

200 0 1 100 100 100 3 7 Substitute: 10.61 = S = T 0 200 7 7 200 3 7 The daily operating budget should be increasing at a rate of $10.61 per year. 10 7

3 7

33. Changing quantities: The number ! of pounds of tuna that can be sold in one month, the price % in dollars per pound 0! 0% The problem: ! = 900 and . = 100. Find 0 0 The relationship: This is the demand equation, %! 1.5 = 50,000. 0% 1.5 0! Derived equation (0 0 of both sides): ! + 1.5%! 0.5 = 0 0 0 We will also need the value of %: %(900) 1.5 = 50,000, and so % = 50,000 (900) 1.5. 0% Substitute and solve: (900) 1.5 + 1.5[50,000 (900) 1.5](900) 0.5(100) = 0, giving 0 0% = (75,000 9) (900) 1.5 0.31. 0 The price is decreasing at a rate of approximately 31¢ per pound per month. 34. Changing quantities: The number ! of rubies that can be sold per week, the price % per ruby 0! 0% The problem: ! = 20 and . = 1. Find 0 0 4 The relationship: This is the demand equation, ! + % = 80. 3 0! 4 0% Derived equation (0 0 of both sides): + = 0 0 3 0 0% 3 4 0% Substitute and solve: 1 + = 0 = 3 0 0 4 The price is increasing by 75¢ per ruby per week.

35. Changing quantities: Let be the distance of the Mona Lisa from Montauk and let be the distance of the Dreadnaught from Montauk. Let be the distance between the two ships. 0 0 0 The problem: = 50, . = 30, = 40, and = 20. Find 0 0 0 The relationship: 2 = 2 + 2 0 0 0 Derived equation (0 0 of both sides): 2 = 2 + 2 0 0 0 We need the value of : 2 = 50 2 + 40 2 = 4,100, and so = 4,100. 2,300 0 0 Substitute and solve: 2 4,100 36 = 2(50)(30) + 2(40)(20) = 4,600, and so = 0 0 4,100 miles/hour 36. Changing quantities: Let be the distance of my aunt from the intersection, let be the distance of myself from the intersection, and let be the distance between us. 0 0 0 The problem: = 1 20, . = 10, = 1 10, and = 60. Find 0 0 0 The relationship: 2 = 2 + 2 0 0 0 Derived equation (0 0 of both sides): 2 = 2 + 2 0 0 0 5 1 1 5 2 We need the value of : = , and so = . + = 400 100 400 20 5 0 1 1 0 130 Substitute and solve: 2 58 miles/hour = 2 (10) + 2 (60) = 13, and so = 20 0 20 10 0 5


Solutions Section 12.6 37. Changing quantities: Let be the distance of the batter from home base, and let third base as shown here:

0 0 when = 45. = 24, find 0 0 The relationship: 2 + 90 2 = 2, or 2 + 8,100 = 2 0 0 Derived equation (0 0 of both sides): 2 = 2 0 0 We need the value of : 2 = 8,100 + 45 2 = 8,100 + 2,025 = 10,125, so 0 0 45 Substitute and solve: (24) 10.7 ft/sec = = 0 0 10,125

be the distance from

The problem: Given that

= 10,125 100.62.

38. Changing quantities: Let be the distance of the batter from third base, and let second base as shown here:

0 0 when = 60. = 30, find 0 0 The relationship: 2 + 90 2 = 2, or 2 + 8,100 = 2 0 0 Derived equation (0 0 of both sides): 2 = 2 0 0 We need the value of : 2 = 8,100 + 60 2 = 8,100 + 3,600 = 11,700, so 0 0 60 Substitute and solve: (30) 16.6 ft/sec = = 0 0 11,700

be the distance from

The problem: Given that

= 11,700 108.17.

39. Changing quantities: The -coordinate and the -coordinate of a point on the graph 0 0 The problem: . = 4 and = 2. Find 0 0 The relationship: Given by the equation of the curve, = 1. 0 0 Derived equation (0 0 of both sides): = 2 0 0 We need the value of : 2 = 1 = 1 2 0 Substitute: = (1 2) 2(4) = 16 0 The -coordinate is decreasing at a rate of 16 units per second.


Solutions Section 12.6 40. Changing quantities: The -coordinate and the -coordinate of a point on the circle 0 0 The problem: = 2, = 3, and . = 1. Find 0 0 The relationship: Given by the equation of the circle, 2 + ( 1) 2 = 8. 0 0 Derived equation (0 0 of both sides): 2 + 2( 1) = 0 0 0 0 0 Substitute and solve: 2( 2)( 1) + 2(3 1) = 0, and so = 4 4 = 1. 0 0 The -coordinate is decreasing at a rate of 1 unit per second.

41. Changing quantities: > and 9 09 1 0> The problem: 9 = 13 and . = . Find 0 3 09 The relationship: >(9) = 2.9299 3 115.99 2 + 1,5309 6,760 0> 09 09 09 Derived equation (0 0 of both sides): = 8.7879 2 231.89 + 1,530 0 0 0 0 0> 2 Substitute: = 8.787(13) (1 3) 231.8(13)(1 3) + 1,530(1 3) 0.534 thousand dollars/year, or $534 0 per year 42. Changing quantities: > and 9 0> 09 The problem: 9 = 14 and . = 5. Find 0 0 The relationship: >(9) = 2.9299 3 115.99 2 + 1,5309 6,760 0> 09 Derived equation (0 0 of both sides): (8.7879 2 231.89 + 1,530) = 0 0 09 09 09 5 Substitute and solve: 5 = (8.787(14) 2 231.8(14) + 1,530) = 7.052 , and so 0.71 0 0 0 7.052 years of schooling per year 43. Changing quantities: ( , ,

The problem: ( is constant at 200. = 3.0 and The relationship: ( = 3 2 + 5 3

0 0 . = 0.2. Find 0 0

0 0 + 15 2 0 0 2 3 We need the value of : 200 = 3 + 5(3.0) , and so = 65 3 4.655. 0 0 27 2 Substitute and solve: 0 = 6(4.655) + 15(3.0) ( 0.2) 0.97. 0 0 27.93 Their prior experience must increase at a rate of approximately 0.97 years every year. Derived equation: (0 0 of both sides): 0 = 6

44. Changing quantities: ( , ,

The problem: is constant at 3.0.

= 15 and

0 0 . = 10. Find 0 0

The relationship: = 4 0.2 2 10 2 0 0 0 0 Derived equation: (0 0 of both sides): = 4 0.4 = (4 0.4 ) 0 0 0 0 0 0 10 Substitute and solve: 10 = [4(3.0) 0.4(15)] , and so 1.7. = 0 0 6 The average study time is decreasing by about 1.7hours per year. 45. Changing quantities: ( , 0( 0 The problem: . = 100 and ( = 200'. Find 0 0


Solutions Section 12.6 The relationship: The given formula expresses ( in terms of both and &. To get the relationship between ( and , we need to know how & is related to . Looking at the vessel from the side, we can see that, for any given value of , the corresponding radius & must satisfy & = 30 50, the ratio at the brim of the vessel. So, 3 &= . 5 1 Substituting into ( = '& 2 , we get the relationship we want: 3 3 3 ( = ' . 25 0( 9 0 Derived equation (0 0 of both sides): ' 2 = 0 25 0 3 We need the value of : 200' = ' 3, and so = (5,000 3) 1 3. 25 2,500 ? 3 B 2 3 9' 5,000 2 3 0 0 Substitute and solve: 100 = 0.63 m/sec = N O 25 3 0 0 9' @A 5,000 CD 46. Changing quantities: ( , 0( 0 The problem: . = 10 and ( = 20. Find 0 0 The relationship: The given formula expresses ( in terms of both and &. To get the relationship between ( and , we need to know how & is related to . Looking at the vessel from the side, we can see that, for any given value of , the corresponding radius & must satisfy & = 20 50, the ratio at the brim of the vessel. So, 2 &= . 5 1 Substituting into ( = '& 2 , we get the relationship we want: 3 4 3 ( = ' 75 0( 4 0 Derived equation (0 0 of both sides): ' 2 = 0 25 0 4 3 We need the value of : 20 = ' , and so = (375 ') 1 3. 75 4' 375 2 3 0 0 125 ' 2 3 Substitute and solve: 10 = 0.82 cm/sec = N O 25 ' 0 0 2' N 375 O 47. Changing quantities: ( ,

The problem: & is constant at 2. ( = 4 2 and

= 2. Find

0 . 0

( . 4' 0 1 0( 1 Derived equation (0 0 of both sides): (8 1) = = 0 4' 0 4' We need the value of when = 2. We first find ( = 4' = 8' so 8' = 4 2 , or 4 2 8' = 0. 1 ± 1 + 128' This equation has solution = by the quadratic formula. We take the positive solution 8 1 + 1 + 128' , = 8 where the volume is rising. The relationship: From ( = '& 2 = 4' we get

=


0 Substitute: = 0

1 + 128'

4'

48. Changing quantities: ( ,

Solutions Section 12.6

1.6 cm/sec

0( 0 . = 6 and = 60. Find 0 0 The relationship: ( = '& 2 = 900' 0( 0 0 1 0( Derived equation (0 0 of both sides): = 900' = 0 0 0 900' 0 0 6 Substitute: 0.0021 cm/sec = 0 900' The problem: & is constant at 30.

49. Changing quantities: ! and 0! 0 The problem: = 30,000 and . = 2,000. Find ! and 0 0 The relationship: ! = 0.3454 ln . 3.047 0! 0.3454 0 Derived equation (0 0 of both sides): = 0 0 0! 0.3454 Substitute: (2,000) 0.0230 = 0 30,000 We are also asked for the value of ! (the number of computers per household), so we substitute = 30,000 in the original equation: ! = 0.3454 ln(30,000) 3.047 0.5137. So, there are approximately 0.5137 computers per household, increasing at a rate of 0.0230 computers per household per year.

50. Changing quantities: ! and 0! 0 The problem: ! = 0.5 and . = 0.02. Find and 0 0 The relationship: ! = 0.3454 ln . 3.047 0! 0.3454 0 Derived equation (0 0 of both sides): = 0 0 We need the value of : 0.5 = 0.345 ln . 3.047, so ln . = 3.547 0.3454, giving = 3.547 0.3454 28,830. 28,830 0.3454 0 0 Substitute and solve: 0.02 = 1,670 = (0.02) 28,830 0 0 0.3454 So, the average income is approximately $28,830, increasing at a rate of $1,670 per year. 51. Changing quantities: and 9 09 0 The problem: 9 = 475 and . = 35. Find and 0 0 1,326 The relationship: (9) = 904 + (9 180) 1.325 Computation of : (475) 904.71 from the formula 0 09 Derived equation (0 0 of both sides): = 1,326( 1.325)(9 180) 2.325 0 0 0 2.325 Substitute and solve: (35) 0.11 = 1,756.95(475 180) 0 The average SAT score was 904.71, decreasing at a rate of 0.11 per year. 52. Changing quantities: and 9 0 09 The problem: = 940 and . = 10. Find 9 and 0 0 1,326 The relationship: (9) = 904 + (9 180) 1.325


Solve for 9:

(9 180) 1.325 =

Solutions Section 12.6

1,326 1,326 = 904 36

1,326 1 1.325 + 180 195.2. 36 O 0 09 Derived equation (0 0 of both sides): = 1,326( 1.325)(9 180) 2.325 0 0 2.325 10(15.2) 09 09 Substitute and solve: 10 = 1,756.95(195.2 180) 2.325 , so 3.2 = 0 0 1,756.95 The prison population was 195,200, increasing at a rate of 3,200 per year. 9= N

0& 0 = 0.05. Find 0(&). Note that & = 1.1 is in the range where 0(&) = 40& + 74 and that 0 0 we stay in that range because we are interested in the slope at that point. Thus, 0 0& 0(&) = 40 = 40(0.05) = 2. The divorce rate is decreasing by 2 percentage points per year. 0 0 53. & = 1.1 and

0& 0 130& 103 and = 0.03. Find 0(&). Note that & = 1.5 is in the range where 0(&) = 0 0 3 3 that we stay in that range because we are interested in the slope at that point. Thus, 0 130 0& 130 0(&) = ( 0.03) = 1.3. The divorce rate is decreasing by 1.3 percentage points per year. = 0 3 0 3 54. & = 1.5 and

55. The section is called "Related Rates" because the goal is to compute the rate of change of a quantity based on a knowledge of the rate of change of a related quantity. The relationship between the quantities gives a relationship between their rates of change. 56. You need to know an equation that relates the two changing quantities as well as the values of the quantities that appear in the derived equation (obtained by taking the derivative with respect to time ), to enable you to solve the derived equation for the desired rate of change. 57. Answers may vary: A rectangular solid has dimensions 2 cm × 5 cm × 10 cm, and each side is expanding at a rate of 3 cm/second. How fast is the volume increasing? 58. You would take the derivative of both sides with respect to time and then substitute the known rates of change to solve for the unknown rate. 59. $ = %!, so $ = % ! + %! , where the derivatives are with respect to time . Divide by $ = %! to get $ % ! %! % ! as claimed. = + = + $ %! %! % !

60. If $ denotes the floor space per employee, then $ = ". Taking derivatives with respect to time, we " " $ " " " " get $ = . Divide by $ " to get as claimed. = = = $ " "2 "2 61. The derived equation is linear in the unknown rate U. This follows from the chain rule, since if 2 is a 02 0 quantity and (2) is any expression in 2, we have (2) = (2) , which is linear in the derivative 0 0 02 02 . The presence of other variables may add terms not containing , but those maintain the linearity. 0 0 62. No. If you take derivative with respect to time of an algebraic equation relating two varying quantities and , then you obtain an algebraic equation in which 0 0 and 0 0 occur to the power 1.


Solutions Section 12.6 63. Let = my grades and = your grades. If 0 0 = 20 0 , then 0 0 = (1 2)0 0 . 64. If = < + 3, then 0 0 = < # 0 0 .


Solutions Section 12.7 Section 12.7

20 = ( 20) = 1,000 20 1,000 20 When = 30, = 1.5 : The demand is going down 1.5% per 1% increase in price at that price level. = 1 when 20 = 1 20 = 1,000 20 = 25 1,000 20 Revenue is maximized when = $25; weekly revenue at that price is = = 25(1,000 20 25) = $12, 500. 1. =

10 = ( 10) = 1,000 10 1,000 10 When = 30, = 3 7 : The demand is going down 3% per 7% increase in price at that price level. = 1 when 10 = 1 10 = 1,000 10 = 50 1,000 10 Revenue is maximized when = $50; weekly revenue at that price is = = 50(1,000 10 50) = $25,000. 2. =

3. a. =

= ( 1) . = + 57 + 57

b. Note that is is thousands of dollars. When = 39.30, =

39.30 39.30 + 57

2.22 : The demand was

going down 2.22% per 1% increase in price at that price level. c. For maximum revenue, set = 1. = 1 = + 57 2 = 57 = 28.5 (thousand dollars). + 57 Revenue would have been maximized when = $28.5 thousand or $28,500. Total annual U.S. revenue at that price would have been = = 28.5( 28.5 + 57) = 812.25. The units of measuement are Units of × Units of : Thousands of dollars per vehicle × millions of vehicles = Billions of dollars. So, total revenues would have been around $812 billion. 4. a. =

0.15 = ( 0.15) . = 0.15 + 23 0.15 + 23

0.15(38.40) 0.334 : The demand 0.15(38.40) + 23 was going down 0.334% per 1% increase in price at that price level. c. For maximum revenue, set = 1. 0.15 = 1 0.15 = 0.15 + 23 0.3 = 23 76.7 (thousand dollars). 0.15 + 23 Revenue would have been maximized when = $76.7 thousand or $76,700(!). Total annual U.S. revenue at that price would have been = = 76.7( 0.15(76.7) + 23) 882. The units of measuement are Units of × Units of : Thousands of dollars per vehicle × millions of vehicles = Billions of dollars. So, total revenues would have been around $882 billion. b. Note that is is thousands of dollars. When = 38.40, =

2.2 = ( 2.2) = 9,900 2.2 9,900 2.2 2.2(2,900) (2,900) = 1.81 9,900 2.2(2,900) 2.2(2,200) (2,200) = 0.96 9,900 2.2(2,200) Thus, decreasing the price from $2,900 to $2,200 would have caused the elasticity to drop from 1.81 to

5. a. =


Solutions Section 12.7 0.96, suggesting that the tuition for maximum annual revenue, which corresponds to unit elasticity, would have been between these two values. b. Revenue is maximized when = 1 : 2.2 = 1 2.2 = 9,900 2.2 4.4 = 9,900 = 9,900 4.4 = $2,250 per student, 9,900 2.2 and this would have resulted in an enrollment of about = 9,900 2.2(2,250) = 4,950 students, giving an annual revenue of about = 2,250(4,950) = $11,137,500.

4,500 = ( 4,500) = 4,500 + 41, 500 4,500 + 41, 500 4,500(3) (3) = 0.48 4,500(3) + 41, 500 4,500(5) (5) = 1.18 4,500(5) + 41, 500 Thus, increasing the price from $3 per ride to $5 caused the elasticity to increase from 0.48 to 1.18, suggesting that the fare for maximum daily revenue, which corresponds to unit elasticity, would have been between these two values. b. Revenue is maximized when = 1 : 4,500 = 1 4,500 = 4,500 + 41,500 9,000 = 41,500 4,500 + 41, 500 = 41,500 9,000 $4.61, and this would have resulted in an daily ridership of = 4,500(4.61) + 41,500 = 20,755 rides, giving a daily revenue of = (4.61)(20,755) = $95,680.55.

6. a. =

2 = ( 2)(100 ) = 2 100 (100 ) When = 30, = 6 7. The demand is going down 6% per 7% increase in price at that price level. Thus, a price increase is in order. 2 b. = 1 when = 1 2 = 100 = 100 3. 100 Revenue is maximized when = 100 3 $33.33. c. Demand would be (100 100 3) 2 = (200 3) 2 4,444 cases per week.

7. a. =

4 = 2( 2)(100 2 ) = 2 100 2 (100 2 ) 4 = 1 when = 1 4 = 100 2 = 100 6 16.67 100 2 Revenue is maximized when = $16.67 per dumbbell. 8. a. =

4 33 = ( 4 + 33) = 2 2 + 33 2 + 33 4(10) 33 7 b. (10) = 0.54 = 2(10) + 33 13 2 Interpretation: The demand for = T-shirts is going down by about 0.54% per 1% increase in the price. c. Revenue is maximized when = 1 : 4 33 = 1 4 33 = 2 + 33 6 = 66 = $11. 2 + 33 Therefore, the club should charge $11 per T-shirt to maximize revenue. At this price, the total revenue is given by = = ( 2 2 + 33 ) = (11)( 2(11) 2 + 33(11)) = $1,331. 9. a. =


Solutions Section 12.7 100 2 (400 ) 10. a. = = = 2 50 400 (400 ) 2(40) 80 (40) = 0.22 = 400 40 360 Interpretation: The demand for Iguanawoman comics is going down by about 0.22% per 1% increase in the price. b. Revenue is maximized when = 1 : 2 = 1 2 = 400 3 = 400 $133.33. 400 Therefore, the publisher should charge $133.33 per copy to maximize revenue. At this price, the weekly revenue is given by (400 ) 2 400 (400 400 3) 2 = = $94, 815. = 100 3 100

760 2 760 = = 2 760 760 760 760 (400) = 2.11 > 1 and (700) = 12.67 > 1 760 400 760 700 760 As increases with increasing , it cannot possibly attain the value of 1 within the given range, so 760 the price for maximum revenue must occur at an endpoint ($400). 11. =

= 31( 0.7) 0.7 = 0.7 31 0.7 (3) = 0.7(3) = 2.1 > 1 and (5) = 0.7(5) = 3.5 > 1 The price that would maximize daily revenue would result in = 1, and the elasticity is linear in the price in this case, so the price that results in = 1 cannot lie between 3 and 5, but would be less than Ż3. 12. =

6,570,000 1 1.3 175,502 13. Solve for : = 10 13 175,502 10 175,502 23 13 10 = = ( 10 13) (constant) showing = = 23 13 175,502 10 13 13 23 13 175,502 13 an inelastic demand. Thus, increasing the price will result in increasing revenue (regardless of the price!).

40 1 1.5 11.6961 14. Solve for : = 2 3 11.6961 = = ( 2 3) = 2 3 (constant) showing an inelastic demand. Thus they 5 3 11.6961 2 3 should raise the price per serving in order to increase revenue.

2 = ( 6 + 1)100 3 + = (6 1). 100 3 2+ When = 3, = 51 : The demand is going down 51% per 1% increase in price at that price level. Thus, a large price decrease is advised. b. = 1 when (6 1) = 1 6 2 1 = 0 (3 + 1)(2 1) = 0 = 1 2 = 0.50 (We reject the solution = 1 3 because we must have > 0.) Revenue is maximized when the price is ¥50. c. Demand would be = 100 3 4+ 1 2 78 paint-by-number sets per month.

15. a. =

2 = (1 3 )100 3 2 = (3 1) 2 100 3 2 When = 3, = 24 : The demand is going down 24% per 1% increase in price at that price level. Thus, a

16. a. =


Solutions Section 12.7 large price decrease is advised.

b. = 1 when (3 1) = 1 3 2 1 = 0 = it. Using the positive solution gives =

1 + 13 6 2

c. Demand would be = 100 (0.77) 3(0.77) 2 ¥77.

1 ± 13 . One solution is negative, so we reject 6

0.77. So, revenue is maximized when the price is

89 paint-by-number sets per month.

= = + + b. = 1 when . = 1 = + = + 2 17. a. =

= ( ) = 1 b. = 1 when = 1, so = . 18. a. =

= ( ) + 1 = b. is independent of . c. If = 1 the revenue is not affected by the price. If > 1, the revenue is always elastic, whereas if < 1, the revenue is always inelastic. This is an unrealistic model because there should be a price at which the revenue is a maximum. 19. a. =

2 2 + = (2 + ) 2 = 2 + + + + 2 2 + b. = 1 when 2 = 1 2 2 = 2 + + + + ± 2 3 giving = . 3 20. a. =

3 2 + 2 +

= 0,

21. a. We have two data points: ( , ) = (2.00, 3,000) and (4.00, 0). The line through these two points is = 1,500 + 6,000. 1,500 b. = = ( 1,500) = 1,500 + 6,000 1,500 + 6,000 1,500 = 1 when = 1 1,500 = 1,500 + 6,000 = $2 per hamburger 1,500 + 6,000 This gives a total weekly revenue of = = 2( 1,500(2) + 6,000) = $6,000.

22. a. We have two data points: ( , ) = (50.00, 10,000) and (80.00, 1,000). The line through these two points is = 300 + 25,000. 300 b. = = ( 300) = 300 + 25,000 300 + 25,000 300 = 1 when = 1 300 = 300 + 25,000 = $41.67 per book. 300 + 25,000 This gives a total weekly revenue of = = 41.67( 300 41.67 + 25,000) = $520,833.17. 23. a. The data points ( , ) = (3.00, 407) and (5.00, 223) give us the exponential function = 1,000 0.30 . b. = = 300 0.3 = 0.3 1,000 0.3


Solutions Section 12.7 At = $3, = 0.9; at = $4, = 1.2; at = $5, = 1.5. c. = 1 when 0.3 = 1, so = $3.33. d. We first find the price that produces a demand of 200 pounds: 1,000 0.3 = 200 0.3 = 0.2 = (ln .0.2) 0.3 $5.36 Selling at a lower price would increase demand, but you cannot sell more than 200 pounds anyway so your revenue would go down. On the other hand, if you set the price higher than $5.36 the decrease in sales will outweigh the increase in price, which we know because the elasticity at = 5.36 is 1.6 > 1. You should therefore set the price at $5.36 per pound. 24. a. The data points ( , ) = (4.00, 287) and (5.00, 223) give us the exponential function 780 0.25 . b. = = 195 0.25 = 0.25 780 0.25 At = $3, = 0.75; at = $4, = 1; at = $5, = 1.25. c. = 1 when 0.25 = 1, so = $4. d. We first find the price that produces a demand of 200 pounds: 780 0.25 = 200 0.25 = 200 780 = [ln(200 780)] 0.3 $5.44 Selling at a lower price would increase demand, but you cannot sell more than 200 pounds anyway so your revenue would go down. On the other hand, if you set the price higher than $5.44 the decrease in sales will outweigh the increase in price, which we know because the elasticity at = 5.44 is 1.3 > 1. You should therefore set the price at $5.44 per pound.

0.0156 2 + 1.5 = 0.01( 0.0156 + 1.5) = 0.01( 0.0078 2 + 1.5 + 4.1) 0.0078 2 + 1.5 + 4.1 When = 20, 0.77 : At a family income level of $20,000 the fraction of children attending a live theatrical performance is increasing by 0.77% per 1% increase in household income. 25. =

0.0012 2 + 0.38 = 0.01(0.0012 + 0.38) = 0.01(0.0006 2 + 0.38 + 35) 0.0006 2 + 0.38 + 35 When = 30, 0.27 : At a family income level of $30,000 the fraction of children attending a live theatrical performance is increasing by 0.27% per 1% increase in household income. 26. =

1.554 0.021 = 1.554 0.021 = 74 0.021 + 92 74 0.021 + 92 When = 100, 0.23 : At a household income level of $100,000 the percentage of people using broadband in 2010 was increasing by 0.23% per 1% increase in household income. b. To see the effect on of large income, we take the limit as : 1.554 0.021 0 lim = lim = = 0 74 0.021 + 92 0 + 92 The model predicts elasticity approaching zero for households with large incomes. 27. a. =

1.118 0.013 = 1.118 0.021 = 86 0.013 + 92 86 0.013 + 92 When = 60, 0.58 : At a household income level of $60,000 the percentage of people using broadband in 2007 was increasing by 0.58% per 1% increase in household income. b. To see the effect on of large income, we take the limit as : 1.118 0.013 0 lim = lim = = 0 0.013 86 + 92 0 + 92 The model predicts elasticity approaching zero for households with large incomes. 28. a. =

0.3454 0.3454 = = 0.3454 ln . 3.047 0.3454 ln . 3.047 When = 60, 000, 0.46. The demand for computers is increasing by 0.46% per 1% increase in 29. a. =


Solutions Section 12.7 household income. b. decreases as income increases because the denominator of gets larger. c. Unreliable; it predicts a likelihood greater than 1 at incomes of $123,000 and above. In a model appropriate for large incomes, one would expect the curve to level off at or below 1. d. approaches 0 as goes to infinity, so for very large we have 0.

0.2802 0.2802 = = 0.2802 ln . 2.505 0.3454 ln . 2.505 When = 60, 000, 0.48. The demand for computers is increasing by 0.48% per 1% increase in household income. b. decreases as income increases because the denominator of gets larger. c. Because the demand should level off at or below 1 as the income rises, a logistic model would be more appropriate to model the demand. d. would approach 0 as goes to infinity; The derivative approaches 0 exponentially while the term grows only linearly for large . 30. a. =

31. = 0.035 + 6.5 0.035 = = 0.035 + 6.5 0.035(60) When = 60, = 0.24 0.035(60) + 6.5 When the price of oil is $60 per barrel, Saudi production increases at a rate of 0.24% per 1% increase in the price. 32. = 0.34 + 1.2 0.34 = = 0.34 + 1.2 0.34(15) When = 15, = 0.81 0.34(15) + 1.2 When the price of oil is $15 per barrel, Saudi production increases at a rate of 0.81% per 1% increase in the price. 33. The income elasticity of demand is = betaP 1 =

.

34. (D): The price elasticity of demand is 0.4 < 1, so an increase in tuition will result in an increase in revenue. 35. the price is lowered 36. the price is changed in either direction

37. Start with = and differentiate with respect to to obtain . = + For a stationary point, = 0, so + = 0. Rearranging this result gives = and hence


Solutions Section 12.7 = 1, or = 1, showing that stationary points of correspond to points of unit elasticity.

1 1 = = 2 = 0 when = 0 (multiply by 2), i.e., when = 1, so when ! " the income elasticity of demand is 1. 38.

39. The distinction is best illustrated by an example. Suppose that is measured in weekly sales and is the unit price in dollars. Then the quantity measures the drop in weekly sales per $1 increase in price. The elasticity of demand , on the other hand, measures the percentage drop in sales per 1% increase in price. Thus, measures absolute change, while measures fractional, or percentage, change.

40. Technically, the member of your study group may be correct: The argument in Exercise 37 shows that setting = 1 should yield, among its solutions, a value for that results in the maximum revenue. There is, however, nothing to prevent there being several solutions of = 1, of which only one corresponds to the maximum revenue. Try, for example, the demand function = 2 5 + 3 + 100 . Note also that the argument in Exercise 37 makes several (reasonable) assumptions. If these assumptions 10 ( 10) 3 are not met, interesting things can happen. For instance, the demand equation = has the property that the only value of for which = 1 is = 10, and this corresponds to neither a minimum nor a maximum revenue.


Solutions Chapter 12 Review Chapter 12 Review

1. ( ) = 2 3 6 + 1 on [ 2, ) Endpoints: 2 Stationary points: ( ) = 6 2 6, so ( ) = 0 when 6 2 6 = 0 6( 2 1) = 0 = ±1 Singular points: ( ) is defined for all , so no singular points. Thus, we have stationary points at = ±1 and the endpoint = 2. We test a point to the right of 1 as well:

( )

2 3

1 5

1

3

2

5

The graph must increase from = 2 to = 1, decrease to = 1, and increase from then on.

This gives absolute mins at ( 2, 3) and (1, 3) and a relative max: ( 1, 5).

2. ( ) = 3 2 1 on ( , ) Endpoints: None Stationary points: ( ) = 3 2 2 1 = (3 + 1)( 1), so ( ) = 0 when (3 + 1)( 1) = 0, giving = 1 3 or 1. Singular points: None

= 3 2 1

1 3

22 27

0 (Test point)

2

2 (Test point)

1 2 (Test point)

11 1 1

This shows a relative maximum at ( 1 3, 22 27), and a relative minimum at (1, 2).

3. ( ) = 4 4 on [ 1, 1] Endpoints: 1, 1 Stationary points: ( ) = 4 3 4 = 4( 3 1) = 4( 1)( 2 + + 1), so ( ) = 0 when 4( 1)( 2 + + 1) = 0 = 1. However, = 1 does not count as a stationary point (it is an endpoint, and (1) is not defined). Singular points: None


1

Solutions Chapter 12 Review

= 4 4

5

3

1

From the table we see that has an absolute maximum at ( 1, 5) and an absolute minimum at (1, 3).

+ 1 for 2 2; 1 ( 1) 2 Endpoints: 2, 2 ( 1) 2 2( + 1)( 1) 1 2( + 1) 3 Stationary points: ( ) = = = 4 3 ( 1) ( 1) ( 1) 3 ( ) = 0 when = 3, which is outside of the domain. So, no stationary points. Singular points: ( ) is undefined at = 1, which is not in the domain of , so there are no singular points. Thus, there are no critical points in the domain of . We test the endpoints and another point on each side of the point = 1 : 4. ( ) =

2

( ) 0.11

1 0

1.5 10

2

3

The graph must increase from = 2 until the vertical asymptote at = 1. It then decreases on the other side of = 1 until = 2.

This gives an absolute min: ( 2, 1 9) and a relative min: (2, 3). 5. ( ) = ( 1) 2 3 Endpoints: None

2 ( 1) 1 3; ( ) is never 0, so no stationary points. 3 Singular points: ( ) is not defined at the interior point = 1. Thus, we have a single critical point at = 1. We test a point on either side: Stationary points: ( ) =

( )

0

1

1

0

2

1

The graph must decrease until = 1 and then increase.


Solutions Chapter 12 Review

This gives an absolute min: (1, 0).

6. ( ) = 2 + ln . on (0, + ) Endpoints: None Stationary points: ( ) = 2 + 1 , so ( ) = 0 when 2 + 1 = 0, which cannot happen for > 0. Thus, no stationary points. Singular points: ( ) is defined for all in the domain of . Thus, has no critical points in its domain.

has no extrema.

1 1 + 2 Endpoints: The domain of is all 0. Thus, there are no endpoints of the domain. Stationary points: 1 2 ( ) = 2 3 , and so ( ) = 0 when 2 2 = 3 3 + 2 2 = 0 2( + 2) = 0 = 2 ( = 0 is not in the domain). Singular points: ( ) is defined for all in the domain of , so there are no singular points. Thus, has a stationary point at = 2. We test points on either side of 2 and points to the right of = 0: 7. ( ) =

3

2

( ) 0.22 0.25

1 0

1

2

2

0.75

The graph decreases to = 2 and then increases approaching the vertical asymptote at = 0. On the other side of the asymptote it decreases.

has an absolute min: ( 2, 1 4).


2

Solutions Chapter 12 Review

8. ( ) = + 1 Endpoints: None

2

Stationary points: ( ) = 2 , and so ( ) = 0 when = 0. Singular points: ( ) is always defined, so there are no singular points. We test points on either side of the stationary point at = 0 :

1

( ) 3.72

0

2

1

3.72

The graph decreases until = 0 then increases.

has an absolute min: (0, 2).

9. Relative max: = 1, point of inflection: = 1

10. Relative min: = 2, point of inflection: = 1

11. Relative max: = 2 : the derivative goes from positive to negative, so the must go from increasing to decreasing. Relative min: = 1 : goes from decreasing to increasing; point of inflection at = 1 : a min of gives a point of inflection of . 12. Relative min: = 2.5 and = 1.5 : in both cases goes from decreasing to increasing. Relative max: = 3 : goes from increasing to decreasing; point of inflection at = 2 : a max of gives a point of inflection of .

13. One point of inflection, at = 0. Note that is not defined at = 0, but does change from negative to positive there.

14. Points of inflection at = 0, where is not defined but changes sign, and = 1, where the graph of crosses the -axis. 2 1 08 4 1 0, 4 2 ;,= = 3 + 2; 4 = = 4 3 m/sec2 2 0 0 3 3 4 2 b. At time = 1, acceleration is 4 = = 2 m/sec2. (1) 4 (1) 3 15. a. 8 =

3 08 8 3 0, 24 ;,= = 3+ ;4= = 4 m/sec2 4 0 4 0 24 24 b. At time = 2, acceleration is 4 = 4 = = 1.5 m/sec2. 16 2 16. a. 8 =

4

2

17. ( ) = 3 12 on [ 2, + ) Stationary points: ( ) = 0 when 3( 2 4) = 0, = ±2;


Solutions Chapter 12 Review Singular points: None; ( ) is always defined. Possible points of inflection: ( ) = 6 , so ( ) = 0 when = 0; ( ) is always defined. has a relative max: ( 2, 16), an absolute min: (2, 16), and a point of inflection at (0, 0). It has no horizontal or vertical asymptotes. Graph:

18. ( ) = 4 4 on [ 1, 1] Endpoints: 1, 1 Stationary points: ( ) = 4 3 4 = 4( 3 1) = 4( 1)( 2 + + 1), so ( ) = 0 when 4( 1)( 2 + + 1) = 0 = 1 Singular points: None

= 4 4

-1

5

1

-3

From the table we see that has an absolute maximum at ( 1, 5) and an absolute minimum at (1, 3) Possible points of inflection: ( ) = 12 2 = 0 when = 0 However, the second derivative is never negative, meaning that the curve is never concave down. Therefore, there are no points of inflection. Graph:

2 3 3 The domain of includes all numbers except 0. 2 # 3 ( 2 3)(3 2) 4 + 9 2 2 + 9 Stationary points: ( ) = , so ( ) = 0 when = ±3 = = 4 6 6 Singular points: None; ( ) is defined for all in the domain of . 2 # 4 ( 2 + 9)(4 3) 2 5 36 3 2( 2 18) Inflection points: ( ) = = = 8 8 5 ( ) = 0 when = ± 18 = ±3 2; ( ) is defined for all in the domain of . has a relative min: ( 3, 2 9), a relative max: (3, 2 9), points of inflection: ( 3 2, 5 2 36), (3 2, 5 2 36) a vertical asymptote at = 0, and horizontal asymptote: = 0.

19. ( ) =


Solutions Chapter 12 Review Graph:

20. ( ) = ( 1) 2 3 +

2 3

( ) is defined for all . 2 2 ( ) = ( 1) 1 3 + ; ( ) = 0 when ( 1) 1 3 = 1, 1 = ( 1) 3 = 1, = 0; 3 3 ( ) is not defined when = 1. ( ) = ( 1) 4 3; ( ) is never 0; ( ) is not defined when = 1. has a relative max: (0, 1) and a relative min: (1, 2 3). It has no points of inflection and no horizontal or vertical asymptotes.

21. ( ) = ( 3) The domain of is 0. Stationary points: 1 3 3 ( ) = + ( 3) , so ( ) = 0 when = down[ .1.<] down[.1.<] 2 2 2 1 = = 1 Singular points: ( ) is not defined when = 0, but this is not an interior point. No singular points. 3 3 Inflection points: ( ) = down[.1.<] ; ( ) is never 0 and is defined for every in the + 4 4 3 2 interior of the domain. has a relative max: (0, 0) and an absolute min: (1, 2). Graph:

22. ( ) = ( + 3) The domain of is 0.


Solutions Chapter 12 Review 1 3 3 ( ) = + ( + 3) down[.1.<] = down[ .1.<] + down[.1.<] 2 2 2 ( ) is never 0; ( ) defined at all interior points of the domain, so there are no singular points. 3 3 ( ) = down[.1.<] ; ( ) = 0 when = 3 2 = 0 = 1. 4 4 3 2 is defined on every interior point of the domain of . It is difficult to see from the graph, but has a point of inflection where = 1 : we can calculate that (0.5) < 0 and (2) > 0. So, has a relative min: (0, 0) and a point of inflection at (1, 4). Graph:

+ 1 ; = 3, M = 0.005 1 0 0 2 2 20 a. , so 0 = 0 = 0 = . = 2 0 0 ( 1) ( 1) 2 ( 1) 2 b. Take 0 = M = 0.005, so the approximate change in is 23. =

0 =

2(0.005) 20 = = 0.0025. 2 ( 1) (3 1) 2

2 ; = 3, M = 0.0005 + 2 0 4 0 4 40 a. , so 0 = 0 = 0 = . = 2 2 0 0 ( + 2) ( + 2) ( + 2) 2 b. Take 0 = M = 0.0005, so the approximate change in is 24. =

0 =

4( 0.0005) 40 = = 0.002. ( + 2) 2 ( 3 + 2) 2

25. ( ) = 4 3 + 2 8 + 7; 4 = 1 (4) = 4( 1) 3 + ( 1) 2 8( 1) + 7 = 12, ( ) = 12 2 + 2 8, so (4) = 12( 1) 2 + 2( 1) 8 = 2. 7( ) = (4) + (4)( 4) = 12 + 2( + 1) = 2 + 14

1 + 1; 4 = 2 1 1 10 1 1 8 (4) = 2 1 = , ( ) = + 1, so (4) = + 1= . 3 9 2 1 ( 1) 2 ( 2 1) 2 10 8 8 14 7( ) = (4) + (4)( 4) = + ( + 2) = 3 9 9 9 26. ( ) =

2 ; 4 = ln 3 1 + 2 1 2 2 ln 3 3 (4) = , so (4) = , ( ) = = = . 1 + ln 3 2 (1 + ) 2 (1 + ln 3) 2 8 1 3 3 + 3 ln 3 + 4 7( ) = (4) + (4)( 4) = + ( + ln 3) = 2 8 8 27. ( ) =


Solutions Chapter 12 Review 28. ( ) = ; 4 = ln 4 2 ln 4 ln 4 15 + ln 4 + ln 4 17 (4) = , ( ) = , so (4) = . = = 2 8 2 2 8 15 17 17 17 ln 4 + 15 7( ) = (4) + (4)( 4) = ( ln 4) = + 8 8 8

29. Objective: Maximize $ = %! = %( % 2 + 33% + 9) = % 3 + 33% 2 + 9% Endpoints: 18, 28 66 ± 66 2 4( 3)(9) 0$ Stationary points: 0.14 or 22.14 = 3% 2 + 66% + 9 = 0 when % = 0% 2( 3) We reject the negative value and obtain the following table: %

18

22.14

28

$ = % 3 + 33% 2 + 9% 5,022 5,522.61 4,172

So the maximum revenue of $5,522.61 occurs when % = $22.14 per book.

30. Objective: Maximize $ = %! = %( 2% 2 + 5% + 6) = 2% 3 + 5% 2 + 6% Endpoints: 0, 3.3 0$ Stationary points: = 6% 2 + 10% + 6 = 2(3% 2 5% 3) = 0 when % 0.47 or 2.14 0% We reject the negative value and obtain the following table: %

$ = 2% 3 + 5% 2 + 6%

0

2.14

3.3

0

16.14 2.38

So the maximum revenue of $16.14 occurs when % = $2.14 per book. 31. a. Profit: = $ = %! (9! + 100) = %( % 2 + 33% + 9) 9( % 2 + 33% + 9) 100 = % 3 + 42% 2 288% 181 b. = 3% 2 + 84% 288 = 3(% 2 28% + 96) = 3(% 24)(% 4) = 0 when % = 4 or 24. To see which, if either, gives us the maximum profit, we test points on either side: %

0

4

24

30

(%) 181 725 3,275 1,979

decreases to a low at % = 4 and then increases to a maximum at % = 24, after which it decreases again. So, the company should charge $24 per copy for a profit of (24) = $3, 275. c. For maximum revenue, the company should charge $22.14 per copy. At this price, the cost per book is decreasing with increasing price, while the revenue is not decreasing (its derivative is zero). Thus, the profit is increasing with increasing price, suggesting that the maximum profit will occur at a higher price. This is, in fact, what we just found. 32. a. Profit: = $ = %! 3! = %( 2% 2 + 5% + 6) 3( 2% 2 + 5% + 6) = 2% 3 + 11% 2 9% 18 b. = 6% 2 + 22% 9 = 0 when % 0.47 or 3.20. To see which, if either, gives us the maximum profit, we test points on either side:


%

0

0.47

Solutions Chapter 12 Review

3.2

3.3

(%) 18 20.008 0.304 0.216

The company should charge $3.20 per copy, for a profit of (24) = 30¢. c. For maximum revenue, the company should charge $2.14 per copy. At this price, the cost per book is decreasing with increasing price, while the revenue is not decreasing (its derivative is zero). Thus, the profit is increasing with increasing price, suggesting that the maximum profit will occur at a higher price. This is, in fact, what we just found.

33. Let be the width of the south wall and the width of the east and west walls.\\Objective: Maximize = subject to 8 + 24 = 480, or + 3 = 60. Solve the constraint equation for and substitute into the objective: = (60 3 ) = 60 3 2 0 = 60 6 0 Stationary point occurs at = 60 6 = 10, which gives the maximum area. Thus, the width of the east and west walls is = 10 ft, and the width of the south wall is = 60 3(10) = 30 ft. Resulting area = = 30 × 10 = 300 sq ft. 34. Let be the width of the north wall and the width of the east and west walls.\\Objective: Minimize = 12 + 8 subject to = 384. Solve the constraint equation for and substitute into the objective: 3,072 384 384 = = 12 + 8 = 12 + 3,072 0 = 12 0 2 Stationary point occurs at = 256 = 16,which gives the minimum cost. 384 Thus, the width of the north wall is = 16 ft, and the width of the east and west wall is = = 24 ft. 16 Resulting cost = 12 + 8 = 12(16) + 8(24) = $384. 35. Start by labeling the edges of the box:

Objective: Maximize ( = = 2 subject to: 2 + 2 = 36, or = 18 . Substitute into the objective: ( = 2(18 ) = 18 2 3 0( = 36 3 2 = 3 (12 ) 0 Stationary point occurs at = 0, 12. = 12 in gives the maximum volume of ( = 18(12) 2 (12) 3 = 864 cubic inches. The height of the box is = 18 = 18 12 = 6 in.


Solutions Chapter 12 Review 36. Start by labeling the edges of the box:

Objective: Maximize = 4 + 2 subject to: 2 + 2 = 36, or = 18 . Substitute into the objective: = 4 (18 ) + 2 = 72 3 2 0 = 72 6 0 Stationary point occurs at = 12, which gives the maximum surface area. The height of the box is = 18 = 18 12 = 6 in. These are exactly the same dimensions as the box in Exercise 35.

37. a. Weekly sales are growing fastest when the rate of change 8 is a maximum. From the graph of 8 , we see that this occurred at about week 5 (see also the graph of 8 which becomes zero at that point). b. This point (a maximum in the graph of 8 ) corresponds to a point of inflection on the graph of 8. On the graph of 8 , that point is given by the maximum, and by the -intercept in the graph of 8 . c. The graph appears to level of around 8 = 10,500; if weekly sales continue as predicted by the model, they will level off at around 10,500 books per week in the long term. d. The graph of 8 appears to level off at 8 = 0. If weekly sales continue as predicted by the model, the rate of change of sales approaches zero in the long term. 38. a. " ( ) appears to have a maximum around = 4 : 4th quarter of 2010. b. This point (a maximum in the graph of " ) corresponds to a point of inflection on the graph of ". On the graph of " , that point is given by the maximum, and by the -intercept in the graph of " . c. The graph of " appears to level off around 1,100; If quarterly sales continue as predicted by the model, they will level off at around 1,100 oPods per quarter in the long term. d. The graph of " appears to level off at " = 0. If sales continue as predicted by the model, the rate of change of sales approaches zero in the long term. 39. Let be the distance between Marjory Duffin and John O'Hagan. Let be Marjory Duffin's distance from the corner, and let be John O'Hagan's distance. Then 2 + 2 = 2 0 0 0 2 + 2 = 2 0 0 0 0 a. 2(2)(5) + 2(2)(5) = 2(2 2) 0 0 40 = 4 2 0 0 10 ft/sec = 2 0 0 b. 2(1)(5) + 2(1)(5) = 2 2 0 0 20 = 2 2 0 0 10 ft/sec = 2 0


Solutions Chapter 12 Review 0 c. 2( )(5) + 2( )(5) = 2 2 0 0 20 = 2 2 0 0 10 ft/sec = 2 0 d. Since the answer to part (c) is independent of , it also must hold as parts (a) through (c). 40. The combined area is = Given that * = 1,

*+

0* = 0.5, and 0

1 2 ' . 4

= 3, we want to find

0, giving the same answer as

0 . 0

0 0* 1 0 *+ . + ' 0 0 2 0 0 1 0 0 0 Substitute: 0 = + 3(0.5) + '(3) = 1.5 + (1 + 3' 2) , = 1.5 (1 + 3' 2) 0.263 inches 0 2 0 0 0 per second. Using the fact that is constant, we differentiate: 0 =

41. The revenue is given by $ = %! = %( % 2 + 33% + 9) = % 3 + 33% 2 + 9%, so the change in revenue corresponding to a change of M% in the price is approximately M$

0$ M% = ( 3% 2 + 66% + 9)M% 0%

With % = 25 and M% = 0.50, we get

M$ ( 3(25) 2 + 66(25) + 9)(0.50) = 108,

so an increase in price of 50¢ will decrease revenue by about $108. Similarly, with M% = 0.50, M$ ( 3(25) 2 + 66(25) + 9)( 0.50) = 108,

so a decrease in price of 50¢ will increase revenue by about $108.

42. The revenue is given by $ = %! = %( 2% 2 + 5% + 6) = 2% 3 + 5% 2 + 6%, so the change in revenue corresponding to a change of M% in the price is approximately M$

0$ M% = ( 6% 2 + 10% + 6)M% 0%

With % = 1.50 and M% = 0.20, we get

M$ ( 6(1.50) 2 + 10(1.50) + 6)(0.20) = 1.5,

so an increase in price of 20¢ will increase revenue by about $1.50. Similarly, with M% = 0.20, M$ ( 6(1.50) 2 + 10(1.50) + 6)( 0.20) = 1.5,

so a decrease in price of 20¢ will decrease revenue by about $1.50. 43. a. V = ( 2% + 33)

% 2% 2 33% = % 2 + 33% + 9 % 2 + 33% + 9

b. V(20) 0.52 and V(25) 2.03. When the price is $20, demand is dropping at a rate of 0.52% per 1% increase in the price; when the price is $25, demand is dropping at a rate of 2.03% per 1% increase in the price.


Solutions Chapter 12 Review c. V = 1 when 2% 2 33% = % 2 + 33% + 9 3% 2 66% 9 = 0 % 2 22% 3 = 0, so % $22.14 per book (using the quadratic formula; the other solution is negative so we reject it). 44. a. V = ( 4% + 5)

% 4% 2 5% = 2% 2 + 5% + 6 2% 2 + 5% + 6)

b. V(2) = 0.75 and V(3) = 7. When the price is $2, demand is dropping at a rate of 0.75% per 1% increase in the price; when the price is $3, demand is dropping at a rate of 7% per 1% increase in the price. c. V = 1 when 4% 2 5% = 2% 2 + 5% + 6 2(3% 2 5% 3) = 0, so % $2.14 per book (using the quadratic formula; the other solution is negative so we reject it).

2 0! % % # = ( 2% + 1)1,000 + # = ( 2% + 1)% = 2% 2 % 0% ! 1,000 2+ b. V(2) = 2(2) 2 2 = 6 Interpretation: The demand is dropping at a rate of 6% per 1% increase in the price. c. For maximum revenue, V = 1 : 2% 2 % = 1 2% 2 % 1 = 0 (2% + 1)(% 1) = 0 % = $1.00 (reject the negative solution).

45. a. V =

For the revenue, $ = %! = % # 1,000

2+

For the revenue, $ = %! = % # 2,000 3

2+ 2

, so $(1) = 1,000 1+ 1 = $1,000 per month.

2 0! % % # = ( 6% + 2)2,000 3 + 2 # = ( 6% + 2)% = 6% 2 2% 2+ 2 0% ! 3 2,000 b. V(2) = 6(2) 2 2(2) = 20 Interpretation: The demand is dropping at a rate of 20% per 1% increase in the price. c. For maximum revenue, V = 1 : 6% 2 2% = 1 6% 2 2% 1 = 0 % $0.61 (reject the negative solution)

46. a. V =

, so $(0.61) $1,353 per month.


Solutions Chapter 12 Case Study Chapter 12 Case Study

1. We now have " = 2,000,000, = 5,000, . = 0.12 and 3 = 1, so ( ) = 5,000 + 120,000 + 2,000,000. As in the text, the minimum will occur at = ." (2 ) 4.9. To find an integer value of we check C(4) = 2,050,000 and (5) = 2,049,000. So, you should use 5 print runs. 2. If we double both and ", we can see from the formula ." (2 ) for the critical point that the critical point will not change. Hence, the optimal number of print runs will not change.

3. If we increase by a factor of 4 and all else remains constant, ." (2 ) will decrease by a factor of 2, so we should cut the number of print runs in half. 4. With " = 1,200,000 and . = 0.12, we look for such that ." (2 ) = 1 : = ." 2 = 144,000 2 = 72,000. Thus, a setup cost of $72,000 or more will result in the optimal number of print runs being 1.

5. With " = 1,200,000 and = 5,000, we look for . such that ." (2 ) = 12 : . = 288 " = 1,440,000 1,200,000 = 1.2. Thus, if the storage cost was $1.20 per year per book, the optimal number of print runs would be 12, which is one each month. 6. This would not affect the calculation of the optimal number of print runs. The only thing that might change is the storage cost, but the figure in the statement of the exercise shows that the average number of books in stock remains the same, so the storage cost remains the same. 7. We substitute the optimal number of print runs, = ." (2 ), into the cost function ." ( ) = + + "3 to get 2 ." ." 2 ." ." = + + "3 = + + "3 = 2." + "3. W 2 W 2 W 2 2 W ." The average cost per book is (") = " = 2. " + 3.

8. lim (") = lim X 2. " + 3Y = 3. As the number of books increases, the average cost per book approaches the production cost per book alone. The setup costs and storage costs become negligible on a per book basis.


Solutions Section 13.1 Section 13.1

1. We recognize 6 5 as the derivative of 6. Thus,

6 5 = 6 + .

2. We recognize 3 2 as the derivative of 3. Thus,

3 2 = 3 + .

3. We recognize 6 as the derivative of 6 . Thus,

6 = 6 + .

4. We recognize 5 as the derivative of 5 . Thus, 5.

6.

7.

8.

9.

( 5) = 5 + .

+ 1 6 = + ; = 5 5 = + +1 6 + 1 8 = + ; = 7 7 = + +1 8

+ 1 4 4 = + ; = 5 5 = + = + +1 ( 4) 4

+ 1 6 6 = + ; = 7 7 = + = + +1 ( 6) 6 1 2 1 3 = = + = 2 + 3 2 2

10.

11.

12. 13.

14.

15.

16.

1 1 1 2 = = + = + 2 1

4

=

3

=

5 4 4 5 4 1 4 = + = + 5 4 5

4 3 3 4 3 1 3 = + = + 4 3 4

3 2 ( 2 ) = 2 = + 3 2

2 4 ( + 3) = + 3 = + + 2 4 2 (1 + ) = 1 + = + + 2

2 ( + 4) = ( ) + 4 = + 4 = + 4 + 2


Solutions Section 13.1 3.3 0.3 17. ( 2.3 + 1.3) = 2.3 + 1.3 = + 3.3 0.3 18.

19.

20.

21.

22.

23.

24.

25.

26. 27.

28.

29.

30.

31.

0.8 1.2 ( 0.2 0.2) = 0.2 0.2 = + 0.8 1.2 6 2 6 4 5 = 4 5 = 4 + = + 6 3

8 3 8 6 7 = 6 7 = 6 + = + 8 4

1 2 2 2 = 2 2 = 2 + = + 1

2 5 5 3 = 5 3 = 5 + = 2 + 2 2

2 2 2 2 2 2 3 2 3 = = 2 = + = + 3 3 3 3 3 9 3 5 3 5 3 3 6 6 = = 5 = + = + 2 2 2 2 6 4

3 3 2 3 3 1 3 2 = = = + = + 2 2 2 2 2 1 2

2 2 5 2 2 4 1 = = 5 = + = 4 + 3 5 3 3 3 4 6 ( 2

) =

1

2

1

=

3

3

ln | | +

1 ( 2 + 2 1) = 1 + 2 ln | | + = + 2 ln | | + (3 4 2 2 + 5 + 4) = 3 4 2 2 + 5 + 4 5 4 3 = + 2 1 + 4 + 5 4 (4 7 3 + 1) = 4 7 3 + 1 8 8 2 2 4 = + + = + + + 8 2 2 2

2 1 2 + = 2 ln | | + � + � = (2 1 + (1 4) ) = 2 ln | | + 4 4 2

2

8+


2

Solutions Section 13.1 2 1 1 3 32. � 2 + � = (2 2 + (1 4) 2) = + + = 2 1 + 4 4 3 1 33.

34.

35.

36.

37.

2

3

12 +

1 2 1 � = ( 1 + 2 2 3) � + 2 3 1 2 2 1 = ln | | + 2 + = ln | | + 2 + 2 1 2

3 1 1 + � = (3 1 5 + 7) � 5 7 4 6 1 1 = 3 ln | | + + = 3 ln | | + 4 + 4 6 4 6 6 3 1.1 5.3 (3 0.1 4.3 4.1) = 4.1 + 1.1 5.3 2.1 3.1 2.3� = 2.3 + � 2 6.2

3 4 0.9 0.1 0.9 40 4 + = + + � 0.1 1.1 � = (3 0.1 4 1.1) = 3 0.9 0.3 0.1 0.1

1 1 0.1 10 38. � 1.1 � = ( 1.1 1) = ln | | + = 0.1 ln | | + 0.1 39.

40.

41.

42.

43.

44.

1.2 3 5.1 2 0.2 + 1.2 � = (5.1 1.2 1 + 3 1.2) = 1.2 ln | | + 3 + �5.1 2 0.2 15 = 2.55 2 1.2 ln | | 0.2 + 1 1.2 0.1 2.2 + + �3.2 + 0.9 + � = (3.2 + 0.9 + 1.2 3) = 3.2 + 3 0.1 3(2.2) 2.2 = 3.2 + 10 0.1 + + 6.6 (2 + 5| | + 1 4) = 2 +

5 | | + + 2 4

| | | | ( 4 + | | 3 1 8) = 4 + 8 + = 4 + 8+ (2)(3) 6

6.1 0.5 1 0.5 1.5 � = �6.1 0.5 + 0.5 � = 6.1 + + � 0.5 + 6 6 0.5 6 1.5 1.5 = 12.2 0.5 + + 9

4.2 0.4 1 0.6 1.4 2 � = �4.2 0.4 + 0.4 2 � = 4.2 + 2 + � 0.4 + 3 3 0.6 3 1.4


1.4 = 7 0.6 + 2 + 4.2

45.

46.

47.

48.

Solutions Section 13.1

2 3 (2 3 ) = + ln 2 ln 3

1.1 2 (1.1 + 2 ) = + + ln 1.1 ln 2

2| | 100(1.1 ) 2 | | 2 + �100(1.1 ) � = �100(1.1 ) | |� = 3 3 ln(1.1) 3 2 100(1.1 ) | | = + ln(1.1) 3

4| | 1, 000(0.9 ) 4 | | 4 + + �1, 000(0.9 ) + � = �1, 000(0.9 ) + | |� = 5 5 ln(0.9) 5 2 1, 000(0.9 ) 2 | | = + + ln(0.9) 5

49. First multiply inside the integral: 3 3 3 3 5 3 3 5 2� 4 4 � = � 2 6� = + = + + . 2 3 2 5 3 10 2 50. First multiply inside the integral: 2 3 9 9 1 9 1 3 4� 4 + = �6 + 2� = 6 + + = 6 + . � 5 5 1 5 5 6 51. First decompose the integrand into a sum of two fractions: +2 2 1 2 = � 3 + 3 � = � 2 + 3 � = ( 2 + 2 3) 3 1 1 = 1 2 + = 2 + . 52. First decompose the integrand into a sum of two fractions: 2 2 2 2 2 2 = � � = � � = ( 2 1) = 2 ln | | + . 2

53. ( ) = , so ( ) = = (0) = 1. Substituting gives 02 + = 1 = 1. 2 2 So, ( ) = + 1. 2

2 + . To find the value of we use the given information that 2

1 1 54. ( ) = , so ( ) = = ln | | + . To find the value of we use the given information that (1) = 1. Substituting gives ln 1 + = 1 = 1. So, ( ) = ln | | + 1.


55. ( ) = 1, so ( ) = ( 1) = + . To find the value of we use the given information that (0) = 0. Substituting gives 0 0 + = 0 1 + = 0 = 1. So, ( ) = 1. Solutions Section 13.1

56. ( ) = 2 + 1, so ( ) = (2 + 1) = 2 + + . To find the value of we use the given information that (1) = 1. Substituting gives 2 1 + 1 + = 1 2 + 1 + = 1 = 2( + 1). So, ( ) = 2 + 2( + 1).

57. ( ) = 5 a. ( ) =

10,000

2 ( ) = 5 = 5 + 10,000 20,000

200 2 100 2 b. = (200) (100) = 5(200) + 5(100) + 20,000 20,000

[998 + ] [499.5 + ] = $498.50. c. (0) = 20,000, so 0 0 + = 20,000, giving = 20,000. 2 Thus, ( ) = 5 + 20,000. 20,000 2 100,000 2 3 a. ( ) = �10 + + � = 10 + 100,000 300,000 58. ( ) = 10 +

b. = (3,000) (600) = 10(3,000) + 3,000 3 300,000 + 10(600) + 600 3 300,000 + . [120,000 + ] [6,720 + ] = $113,280. c. (0) = 100,000, so 0 + 0 + = 100,000, giving = 100,000. 3 Thus, ( ) = 10 + + 100,000. 300,000 59. ( ) = 5 + 0.002 +

1

1 �5 + 0.002 + � = 5 + 0.001 2 + ln + (Note that > 0, so there is no need to write ln | |.) b. = (600) (500) = [5(600) + 0.001(600) 2 + ln 600 + ] [5(500) + 0.001(500) 2 + ln 500 + ]. [3366.397 + ] [2756.215 + ] $610.18. c. (1) = 1,000, so 5 + 0.001 + ln 1 + = 1,000, giving = 994.999. Thus, ( ) = 5 + 0.001 2 + ln + 994.999 a. ( ) =

1 2 1 2 a. ( ) = �10 + + 2 � = 10 + 1 + 2 60. ( ) = 10 + +

(200) 2 (100) 2 (200) 1 + 10(100) + (100) 1 + 2 2 [21999.995 + ] [5999.99 + ] $16,000.01.

b. = (200) (100) = 10(200) +


Solutions Section 13.1 100 2 1 c. (100) = 10,000, so 10(100) + + = 10,000, giving = 4,000.01. 2 100 2 Thus, ( ) = 10 + 1 + 4,000.01. 2 61. We are given ( ) = ( ) = 2.12 + 141, so

(2.12 + 141) = 1.06 2 + 141 + . To find we use the fact that (0) = 372. Substituting gives 372 = 1.06(0) 2 + 141(0) + = , so ( ) = 1.06 2 + 141 + 372 b. Midway through 2014 ( = 3.5), the number of active users was (3.5) = 1.06(3.5) 2 + 141(3.5) + 372 878 million active users. c. No; ( ) = ( ) = 2.12 + 141 has only positive coefficients and so is never negative. Thus ( ) was always increasing. a.

( ) =

62. We are given ( ) = ( ) = 358 2,040, so

(358 2,040) = 179 2 2,040 + . To find we use the fact that (0) = 11,000. Substituting gives 11,000 = 179(0) 2 2,040(0) + = , so ( ) = 179 2 2,040 + 11,000 b. Midway through 2018 ( = 6.5), the number of shows was (6.5) = 179(6.5) 2 2,040(6.5) + 11,000 5,303 shows. c. Yes: We can look at ( ) = 358 2,040 to see where it is negative and find that ( ) < 0 when 0 < 5.7, so the number of shows was decreasing from the start of 2012 until sometime in September of 2017. a.

( ) =

63. We are given ( ) = ( ) = 32(1.28 ), so 1.28 ( ) = 32(1.28 ) = 32 + . ln 1.28 Total number of cases from February 4 up to day 1.28 1.28 0 = ( ) (0) = 32 + " 32 + " ln 1.28 ! ln 1.28 ! 1.28 32 1.28 1 . = 32 = 32 ln 1.28 ln 1.28 ln 1.28 February 29 is represented by = 25, so the total number of cases from Feb. 4 to Feb. 29 was 1.28 25 1 (25) (0) = 32 62,000 cases. ln 1.28 64. We are given ( ) = ( ) = 157(1.02 ), so 1.02 ( ) = 157(1.02 ) = 157 + . ln 1.02 Total number of cases from October 10 up to day 1.02 1.02 0 = ( ) (0) = 157 + 157 + " "! ln 1.02 ln 1.02 ! 1.02 157 1.02 1 . = 157 = 157 ln 1.02 ln 1.02 ln 1.02


Solutions Section 13.1 October 31 is represented by = 21, so the total number of cases from Oct. 10 to Oct. 31 was 1.02 21 1 (21) (0) = 157 4,088 thousand cases, or around 4,088,000 cases. ln 1.02

65. The rate of change of #( ) is $1,200 per year; # ( ) = 1,200. So, #( ) = 1,200 = 1,200 + . To find we use the fact that in 2000 ( = 0) the median household income was $42,000. Thus, #(0) = 42,000 : 1,200(0) + = 42,000 = 42,000, so #( ) = 42,000 + 1,200 . 2005 income: #(5) = 42,000 + 1,200(5) = $48,000

66. The rate of change of #( ) is $1,500 per year; # ( ) = 1,500. So, #( ) = 1,500 = 1,500 + . To find we use the fact that in 2000 ( = 0) the mean household income was $57,000. Thus, #(0) = 57,000 : 1,500(0) + = 57,000 = 57,000, so #( ) = 57,000 + 1,500 . 2006 income: #(6) = 57,000 + 1,500(6) = $66,000 67. a. $ = (320 200) 8 = 15 so % ( ) = 15 + 200 billion dollars per year b. %( ) = (15 + 200) = 7.5 2 + 200 + %(0) = 3,800 = , so %( ) = 7.5 2 + 200 + 3,800 billion dollars 2027 estimate: %(8) = 7.5(8) 2 + 200(8) + 3,800 = $5,880 billion, or $5.88 trillion.

68. a. $ = (190 100) 10 = 9, so % ( ) = 9 + 100 billion dollars per year b. %( ) = (9 + 100) = 4.5 2 + 100 + %(0) = 1,300 = , so %( ) = 4.5 2 + 100 + 1,300 billion dollars 2015: %(15) = 4.5(15) 2 + 100(15) + 1,300 = $3,812.5 billion, or $3.8 trillion, considerably more than the actual amount.

69. a. From the Quick Example referred to in the hint, the rate of change (velocity) of the percentage is given by ( ) = &( ) = 0.4 = 0.4 + . To find , use the fact that at time = 0, = 1 percentage point per year, so (0) = 1, giving = 1 : ( ) = 0.4 + 1 points per year. b. From the quick example referred to in the hint, the value of the percentage is given by '( ) = ( ) = ( 0.4 + 1) = 0.2 2 + + . To find , use the fact that at time = 0, ' = 13 percentage points, so '(0) = 13, giving = 13 : '( ) = 0.2 2 + + 13. In 2008, = 1, so '(1) = 0.2(1) 2 + 1 + 13 = 13.8%. 70. a. From the Quick Example referred to in the hint, the rate of change (velocity) of the value is given by ( ) = &( ) = 20 = 20 + . To find , use the fact that at time = 0, = 40 billion dollars per year, so (0) = 40, giving = 40 : ( ) = 20 + 40 billion dollars per year. b. From the quick example referred to in the hint, the value of outstanding debt is given by (( ) = ( ) = ( 20 + 40) = 10 2 + 40 + .


Solutions Section 13.1 To find , use the fact that at time = 0, ( = 1,300, so ((0) = 1,300, giving = 1,300 : (( ) = 10 2 + 40 + 1,300. In 2009, = 1, so ((1) = 10(1) 2 + 40(1) + 1,300 = $1,330 billion. 71. ( ) = 2 + 1

a. (( ) = ( ) = ( 2 + 1) =

b. 0 + 0 + = 1 so ( = 72. ( ) = 3 +

3 + +1 3

3 + + 3

a. (( ) = ( ) = (3 + ) = 3 + b. 3 0 + 0 + = 3, = 0 so ( = 3 +

2 2

2 + 2

73. &( ) = 32 ( ) = &( ) = ( 32) = 32 + (0) = 0 is given, so 0 = 0 + = 0, and so ( ) = 32 . After 10 seconds, (10) = 32(10) = 320, so the stone is traveling 320 ft/sec downward.

74. &( ) = 32 ( ) = &( ) = ( 32) = 32 + (0) = 10 is given, so 10 = 0 + = 10, and so ( ) = 32 + 10. After 10 seconds, (10) = 32(10) + 10 = 310, so the stone is traveling 310 ft/sec downward.

75. a. ( ) = &( ) = ( 32) = 32 + At time = 0, = 16 ft/sec, so 16 = 32(0) + = 16, giving ( ) = 32 + 16. b. (( ) = ( ) = ( 32 + 16) = 16 2 + 16 + At time = 0, ( = 185 ft, so 185 = 16(0) 2 + 16(0) + = 185, giving (( ) = 16 2 + 16 + 185. It reaches its zenith when ( ) = 0 32 + 16 = 0 = 16 32 = 0.5 sec. Its height at that moment is ((0.5) = 16(0.5)2 + 16(0.5) + 185 = 189 feet, 4 feet above the top of the tower.

76. a. ( ) = &( ) = ( 32) = 32 + At time = 0, = 24 ft/sec, so 24 = 32(0) + = 24, giving ( ) = 32 + 24. b. (( ) = ( ) = ( 32 + 24) = 16 2 + 24 + At time = 0, ( = 185 ft, so 185 = 16(0) 2 + 24(0) + = 185, giving (( ) = 16 2 + 24 + 185. It reaches its zenith when ( ) = 0 32 + 24 = 0 = 24 32 = 0.75 sec. Its height at that moment is ((0.5) = 16(0.75)2 + 24(0.75) + 185 = 194 feet, 9 feet above the top of the tower.


Solutions Section 13.1 77. a. The ground speed is = Air speed + Tailwind speed = 500 + (25 + 50 ) = 525 + 50 . ( Thus, = 525 + 50 , where ( is the total distance traveled. Thus the total distance traveled is ( = [525 + 50 ] = 525 + 25 2 + . To obtain , use the information that, at time = 0, ( = 0 as well (zero distance was traveled at time = 0). Thus, 0 = 525(0) + 25(0) 2 + = 0, giving ( = 525 + 25 2. b. At the end of a 1,800-mile trip, ( = 1,800 and so, by (a) 1,800 = 525 + 25 2. To obtain we solve the quadratic 25 2 + 525 1,800 = 0. Dividing by 25 gives 2 + 21 72 = 0 ( + 24)( 3) = 0. So, rejecting the negative solution gives = 3 hours. c. The negative solution indicates that, at time = 24, the tailwind would have been large and negative, causing the plane to be moving backward through that position 24 hours prior to departure and arrive at the starting point of the flight at time 0! 78. a. The ground speed is = Air speed − Headwind speed = 500 (25 + 50 ) = 475 50 . ( Thus, = 475 50 , where ( is the total distance traveled. Thus the total distance traveled is ( = [475 50 ] = 475 25 2 + . To obtain , use the information that, at time = 0, ( = 0 as well (zero distance was traveled at time = 0). Thus, 0 = 475(0) 25(0) 2 + = 0, giving ( = 475 25 2. b. For a 1,500-mile trip, ( = 1,500 and so, by (a) 1,500 = 475 25 2. To obtain we solve the quadratic 25 2 475 + 1,500 = 0. Dividing by 25 gives 2 19 + 60 = 0 ( 4)( 15) = 0. So, rejecting the larger solution gives = 4 hours. c. Had the flight continued until time = 15, the headwinds would have become so powerful (they were increasing by 50 mph each hour) that the plane would be flying backward by then and would arrive at the destination a second time at = 15 (11 hours later)! 79. From the formulas at the end of the section in the textbook, ( ) = 32 + 0 . The projectile has zero velocity when ( ) = 0, so that 0 = 32 + 0 . Solving for gives = 0 32. This is when it reaches its highest point. 80. From the formulas at the end of the section in the textbook, (( ) = 16 2 + 0 + (0 . If we take the starting height as 0, (0 = 0, and so


Solutions Section 13.1 (( ) = 16 2 + 0 . From Exercise 69, the projectile reaches its highest point at = 0 32; its height then is (( 0 32) = 16( 0 32) 2 + 0 ( 0 32) = 02 64 feet. 2 0 64 = 20

0 = (1,280)

1 2

2 0 64 = 40

0 = (2,560)

1 2

35.78 ft/sec.

81. By Exercise 70, the ball reaches a maximum height of

51 ft/sec.

82. By Exercise 70, the ball reaches a maximum height of

2 1 2 = 80 ft/sec. 0 64 = 100 0 = (6,400) 2 b. (( ) = 16 + 0 + (0

2 0 64 feet. Thus, 2 0 64 feet. Thus,

83. a. By Exercise 70, the chalk reaches a maximum height of

2 0 64 feet. Thus,

If we take the starting height as 0, (0 = 0, and so (( ) = 16 2 + 100 . The chalk strikes the ceiling when (( ) = 100 16 2 + 100 = 100 16 2 100 + 100 = 0 4(4 5)( 5) = 0 = 1.25 or 5. We take the first solution, which is the first time it strikes the ceiling. Now, ( ) = 32 + 0 = 32 + 100, so the velocity when it strikes the ceiling is (1.25) = 60 ft/sec. c. Start with ((0) = 100 and (0) = 60. So, ( ) = 32 + 0 = 32 60. We have (( ) = 16 2 + 0 + (0 = 16 2 60 + 100. Now we find when (( ) = 0 : 16 2 60 + 100 = 0 4(4 5)( + 5) = 0 = 1.25 or 5. We take the positive solution and say that it takes 1.25 seconds to hit the ground.

84. a. 02 64 = (16,000) 2 64 = 4,000,000 feet b. 0 32 = 16,000 32 = 500 seconds c. (( ) = 16 2 + 0 = 16 2 + 16,000 The projectile hits the ground when ((0) = 0 : 16 2 + 16,000 = 0 16 ( 1000) = 0 = 0 or 1,000. The solution = 0 is the starting point, so we are interested in the other solution, = 1,000. ( ) = 32 + 0 = 32 + 16,000 so (1,000) = 16,000. Thus, the projectile is traveling at a speed of 16,000 ft/sec when it hits the ground. 85. Let 0 be the speed at which Prof. Strong throws and let )0 be the speed at which Prof. Weak throws. We have 2 2 0 64 = 2)0 64, so solving for 0 gives 0 = )0 2. Thus, Prof. Strong throws it

2 1.414 times as fast as Prof. Weak.

Thus, Prof. Weak throws it

3 1.732 times as fast as Prof. Strong.

86. Let 0 be the speed at which Prof. Strong throws and let )0 be the speed at which Prof. Weak throws. We have 3 02 64 = )02 64, so solving for )0 gives )0 = 0 3.


Solutions Section 13.1 87. The term indefinite refers to the arbitrary constant term in the indefinite integral; we do not obtain a definite value for , and hence the integral is "not definite." 88. an antiderivative 89. Constant; since the derivative of a linear function is constant, linear functions are antiderivatives of constant functions. 90. The zero function; since the derivative of a constant function is zero, constant functions are antiderivatives of the zero function. 91. No; there are infinitely many antiderivatives of a given function, each pair of them differing by a constant. Knowing the value of the function at a specific point suffices.

92. Yes; if * ( ) is an antiderivative of ( ), then its derivative is ( ), so that we can obtain the exact function by taking the derivative of * ( ). 93. They differ by a constant, +( ) * ( ) = Constant.

94. If * ( ) is one antiderivative of ( ), then every other antiderivative has the form * ( ) + for some constant . 95. Their changes are the same: If we know * ( ) = + ( ) on an interval, then * and + differ by a constant on that interval, so * (,) * (&) = +(,) +(&). 96. No; the given information gives us * (,) * (&) for every & and ,. But we have seen that if +( ) = * ( ) + for a constant , then * and + have the same change over [&, ,]: +(,) +(&) = * (,) * (&), so we do not know the function * . 97. antiderivative, marginal 98. velocity, acceleration ( ) represents the total cost of manufacturing items. The units of ( ) are the product of the units of ( ) and the units of . 99. Up to a constant,

100. Up to a constant,

-( ) represents the total volume of rocket fuel burned since liftoff.

101. The indefinite integral of the constant 1 is not zero; it is (+ constant). Thus, the correct answer is (3 + 1) = 3 2 2 + + .

102. The indefinite integral of 11 is not zero; it is 11 2 2 (+ constant). Thus, the correct answer is (3 2 11 ) = 3 11 2 2 + . 103. There should be no integral sign (

) in the answer. The correct way to write the calculation is


(12 5 4 ) = 2 6 2 2 + .

Solutions Section 13.1

104. The " " indicates that the variable of integration is , and not . Correct answer: 5 + 105. The integral of a constant times a function is the constant times the integral of the function, not the integral of the constant times the integral of the function. (In general, the integral of a product is not the product of the integrals.) The correct way to write the calculation is: 4( 2 ) = 4 ( 2 ) = 4( 2) + .

106. The integral of 2 is 2 (ln 2) and not 2 + 1 ( + 1). Correct answer: 2 (ln 2) +

107. It is the integral of 1 that equals ln | | + , not 1 itself. Correct answer:

1 = ln | | +

108. 1 3 can be written as 3, and its integral is therefore 2 2 + . (The logarithm comes in only when you are integrating to the 1 power.)

( ( ) + -( )) is, by definition, an antiderivative of ( ) + -( ). Let * ( ) be an antiderivative of ( ), and let +( ) be an antiderivative of -( ). Then, since the derivative of * ( ) + +( ) is ( ) + -( ) (by the rule for sums of derivatives), this means that * ( ) + +( ) is an antiderivative of ( ) + -( ). In 109.

symbols, integrals.

( ( ) + -( )) = * ( ) + +( ) + = ( ) + -( ) , the sum of the indefinite

(. ( )) is, by definition, an antiderivative of . ( ). Let * ( ) be an antiderivative of ( ). Then, since the derivative of .* ( ) is . ( ) (by the rule for constant multiples of derivatives), this means that 110.

.* ( ) is an antiderivative of . ( ). In symbols, multiple of the indefinite integral.

(. ( )) = .* ( ) + = . ( ) , the constant

111. Answers will vary.

1 = = 2 2 + , whereas 2 1 = � + /� ( + 0), 2 which is not the same as 2 2 + , no matter what values we choose for the constants , /, and 0.

( 1) = = 2 2 + , whereas 2 ( ) ( 1 ) = ( 2 + /) ( + 0), which is not the same as 2 2 + , no matter what values we choose for the constants , /, and 0. 112. Answers will vary.

113. If you take the derivative of the indefinite integral of ( ), you obtain ( ) back. On the other hand, if you take the indefinite integral of the derivative of ( ), you obtain ( ) + .


Solutions Section 13.1 114. The only possible antiderivatives of 1 have the form 2 2 + for some constant . Thus, the "classified" Martian function is 1( ) = 2 2 + , and so the Institute of Alien Mathematics is not being truthful.


Solutions Section 13.2 Section 13.2 1.

2.

3.

4. 5.

6.

7.

8. 9.

= 3 5

4 (3 5) 4 1 4 3 1 (3 5) 3 = $ = + = + = + 3 3 4 12 12

= 2 + 5

1 (2 + 5) 1 1 1 2 1 (2 + 5) 2 = $ = + = + = + 2 2 1 2 2

=3 1 = 3 =2 1 = 2

(3 5) 4 1 (3 5) 4 (3 5) 3 = + = + 3 4 12

(2 + 5) 1 1 (2 + 5) 1 (2 + 5) 2 = + = + 2 2 1 =

= 1 = =

2

1 = 2 = 2

= = + = +

2 = 2 + = 2 2 +

1 = + = + 1

1 2 2 = + = 2 2 + 1 2 = ( + 1) 2

= 2( + 1) = 2( + 1)

2 1 ( + 1) ( + 1) = ( + 1) = 2( + 1) 2 2 1 1 = + = ( + 1) + 2 2


10.

11.

12.

13.

14.

15.

16.

17.

= ( 1)

3

= 3( 1) 2 = 3( 1) 2 = 3 + 1

=3 1 = 3

1 ( 1) = ( 1) 2 = 2 3 3( 1) 3 1 1 = + = ( 1) + 3 3 Solutions Section 13.2

2 ( 1) 3

6 (3 + 1) 6 1 6 51 (3 + 1) 5 = = + = + = + 3 3 6 18 18

8

( 1) 8 + 8

= 1

7 ( 1) 7 = = + = 8

= 3 4

7.2 3 4 = 7.2 3 2 = 1.6(3 4) +

= 3 + 4

1 4.4 4.4 ( 3 + 4) 4.4 ( 3 + 4) = 4.4 = + = + 3 3 3

= 0.6 + 2

1 1.2 (0.6 + 2) = 1.2 = 2 + = 2 (0.6 + 2) + 0.6

= 3 + 4

8.1 3 + 4 = 8.1 3 2 = 1.8( 3 + 4) +

= 1 =

=3 1 = 3

= 3 1 = 3

= 0.6 1 = 0.6 = 3 1 = 3

= 3 2 + 3

= 6 1 = 6

3 2 1 1 2 = 2.4 = 2.4 + 3 3 2

3 2 1 = 2.7 + 3 3 2

4 (3 2 + 3) 4 1 1 3 (3 2 + 3) 3 = 3 = = + = + 6 6 12 24


18.

19.

20.

21.

22.

23.

24.

25.

= 1 2

= 2 1 = 2 = 3 2 1

= 6 1 = 6

= 2 + 1

= 2 1 = 2 = 2 + 1

= 2 1 = 2

= 1 + 3

= 3 2 1 = 2 3

= 4 2 1

= 8 1 = 8

= 4 3 + 1

= 12 2 1 = 12 2 = 3.1 2

= 3.1 1 = 3.1

Solutions Section 13.2 4 1 1 3 ( 1) = 3 = = + 2 2 8 2 4 ( 1) = + 8 2

3

2 3 2 1 = 2 2 = (3 2 1) 3 2 + 9

3 2 1 1 1 2 = = + 6 3 9 2

1 3 1 2 3 2 + 1 = 3 = 2 2 3 3 2 = $ + = ( 2 + 1) 3 2 + 2 3 2 1 1 1.3 = = ( 2 + 1) 1.3 1.3 2 2 0.3 ( 2 + 1) 0.3 = + = + 0.6 0.6 2 2 1 1 1.4 = = (1 + 3) 1.4 1.4 3 2 3 0.4 (1 + 3) 0.4 = + = + 1.2 1.2 1 1 1 | | |4 2 1| = | | = | | = + 8 8 8 2 (4 2 1)|4 2 1| = + 16 1 1 2|4 3 + 1| = 2| | = | | 2 12 12 (4 3 + 1)|4 3 + 1| 1 | | = + = + 12 2 24 1 (1 + 9.3 3.1 2) = + 9.3 3.1 2 = + 9.3 3.1 = + 3 + = + 3 3.1 2 +


26.

27.

28.

29.

30.

31.

32.

33.

Solutions Section 13.2 1 4 ) = 3.2 4 = 3.2 + (3.2 4 1.2 1.2 10 1.2 3 = 3.2 + 3

= 1.2 3 = 1.2 1 = 1.2

1.2 3

2 1 1 + 1 = = 2 2 2 1 1 = + = + 1 + 2 2

= 2 + 1

= 2 1 = 2 = 2 2 1

= 4 1 = 4

2 2 1 1 1 1 2 1 = = = + = 2 1 + 4 4 4 4

= ( 2 + 2 )

= (2 + 2) 1 = 2( + 1) = 2 2 2

= (4 2) 1 = 2(2 1) = 2 + + 1

= 2 + 1 1 = 2 + 1

= 3 4 4 3

= 12 3 12 2 1 = 3 12( 2) = 2 3 + 6 5

= 6 2 + 6 5 1 = 2 6( + 5)

2 1 1 ( + 1) ( + 2 ) = ( + 1) = 2( + 1) 2 2 1 1 = + = ( + 2 ) + 2 2 2 1 1 (2 1) 2 2 = (2 1) = 2(2 1) 2 2 1 1 = + = 2 2 + 2 2

2 1

( 2 + + 1) 3 =

3

=

(2 + 1) 1 1 = 3 3 2 + 1

=

2

2

+ =

( 2 + + 1) 2 + 2

3 2 3 2 1 1 1 = = 3 4 4 3 12 12( 3 2) 1 1 ln | | + = ln |3 4 4 3| + = 12 12 2 + 5

2 3 + 6 5 =

=

2 + 5 1 1 1 2 = 2 5 6 6( + )

1 1 2 1 1 + = (2 3 + 6 5) 1 2 + or 6 1 2 3 3

2 3 + 6 5 +


34.

35.

36.

= 5 4 4

= 20 3 20 4 1 = 20( 3 4) = 2 =1 =

= 2

=1 =

37.

38.

39.

5

= +1 =1 =

= +1 =1 =

Solutions Section 13.2 2( 3 4) 2( 3 4) 1 1 5 = = 3 4 5 (5 4 4 5) 5 10 20( ) =

4

40

+ =

1 (5 4 4 5) 4 + 40

( 2) 5 = 5 . To remove the remaining , solve for in terms of in the expression for = 2, so = + 2. The above integral is then 1 7 1 6 5 = ( + 2) 5 = ( 6 + 2 5) = + + 7 3 1 1 = ( 2) 7 + ( 2) 6 + . 7 3

:

( 2) 1 3 = 1 3 . To remove the remaining , solve for in terms of in the expression for : = 2, so = + 2. The above integral is then 7 3 2 4 3 1 3 = ( + 2) 1 3 = ( 4 3 + 2 1 3) = + + 7 3 4 3 3 3 = ( 2) 7 3 + ( 2) 4 3 + . 7 2

2 + 1 = 2 . To remove the remaining , solve for in terms of in the expression for : = + 1, so = 1. The above integral is then 2 = 2( 1) = 2 ( 3 2 4 4 = ( + 1) 5 2 ( + 1) 3 2 + . 5 3

1 2

) = 2

5 2

5 2

2

3 2

3 2

+

= . To remove the remaining , solve for in terms of in +1 the expression for : = + 1, so = 1. The above integral is then 3 2 1 2 1 = = ( 1 2 1 2) = + 3 2 1 2 2 = ( + 1) 3 2 2( + 1) 1 2 + . 3

= 1 0.05

= 0.05 0.05 1 = 0.05 0.05

0.05 0.05 1 1 = = 20 0.05 1 0.05 0.05 = 20 ln | | + = 20 ln |1 0.05 | +


40.

41.

42.

43.

44.

45.

=2+

1.2

= 1.2 1.2 1 = 1.2 1.2 1 1 = 2 = 2 =

2 =

2 = 2 2 = 2 1 =4+ 2 2 = 3 3 = 2 =2

1

1 = 2 = 2

= 2.5 ln |2 + 1.2 | +

3 1 3 2 = 3 = 3 + = 3 1 + = 2 2

2 2 2 2 = = + = 2 + = 2 2 2

3 3 4 (4 + 1 2) 3 3 = = = + 3 2 2 8 3 (4 + 1 2) 4 = + 8

1 1 1 = 2 = = ln | | + = ln |2 1 | + 2(2 1 ) 2

+ = � + . For the integral on the right, take + � = 2 2 2 2 2 =

= 1 =

46.

Solutions Section 13.2 3 1.2 3 1.2 1 1 = 2.5 = 2.5 ln | | + = 1.2 2 + 1.2 1.2

= = + + = + . 2 2 2 2 2 2 2

( 2 + 2) = 2 + 2 . For the first integral, take = 2

=1 2 = 2

2 = 2 = 2 + = 2 2 + .

For the second integral, take

= 2 :

= 2:

= :


Solutions Section 13.2

= 2

2 = 2 = 2 + = 2 2 + .

= 1 2 = 2

So, 47.

48.

49.

50.

( 2 + 2) = 2 2 2 2 + = 2( 2 2) + .

1 1 = = = ln | | + + = ln | + | + or, equivalently, ln( + ) + (because and are nonnegative).

= +

= 1 =

= 2 2 2 + 2 = 2 2 = 2 + 2 = 1 3 1

= 3 3 1 1 = 3 1 3 = 1 1

= 1 1 = 1 2

2 + 2 2 + 2 2 = 2 2 2 + 2 1 = 2 = 2 ln | | + = 2 ln | 2 2| + 1 1 3 1|1 3 1| = 3 1| | 3 1 = | | 3 3 (1 3 1)|1 3 1| 1 | | = + = + 3 2 6 1 | 1 1|( 1) = | |( 1) = | | 1 1 1 | | ( 1)| 1| = + = + 2 2 2

2

2

[(2 1) 2 2 + ] = (2 1) 2 2 + . For the first integral, take 2 = 2 2 : 2 1 1 = 2 2 2 (2 1) 2 2 = (2 1) = 2(2 1) 2 2 = 4 2 1 1 = + = 2 2 + . 2 2 1 = 2(2 1)

51.

= 2

= 2 1 = 2

= 2 :

2 1 1 1 1 2 = = = + = + . 2 2 2 2

For the second integral, take


2 2 2

Solutions Section 13.2 1 2 2 2 1 2 + ] = + + . 2 2 2

So,

[(2 1)

52.

( + 1 + 2 ) = + 1 + 2 . For the first integral, take

2

= 2 + 1

= 2 1 = 2

2

2 1 1 + 1 = = 2 2 2 1 1 = + = + 1 + . 2 2

=2 1 = 2

So, 53.

54.

55.

56.

57.

= 2 :

1 1 1 2 = = + = 2 + . 2 2 2

For the second integral, take = 2

= 2 + 1 :

2 2 1 1 ( + 1 + 2 ) = + 1 + 2 + . 2 2

= & + ,

+ 1 (& + ,) + 1 1 (& + ,) = = + = + & &( + 1) &( + 1)

= & + ,

1 1 1 1 (& + ,) 1 = = ln | | + = ln |& + ,| + & & &

= & + ,

1 11 1 |& + ,| = | | = | | + = (& + ,)|& + ,| + & &2 2&

= & + ,

1 1 1 + = = + = + + & & &

=& 1 = & =& 1 = & =& 1 = & =& 1 = &

1 1 + = + + = + = + & 1


Solutions Section 13.2 1 58. + = + + 1 = 1 + & 59.

60.

1 1 + = + + 2 1 = 2 1 + & 2

1 1 3 1 + = + + 3 = + = 3 + & 3 3

61.

(& + ,) =

62.

(& + ,) =

63.

64.

65.

66.

67.

68.

69.

70.

71.

72.

(& + ,) + 1 (2 + 4) 3 (2 + 4) 3 + (2 + 4) 2 = + = + &( + 1) 2(3) 6 (& + ,) + 1 1 + (3 2) 4 = (3 2) 5 + &( + 1) 15

1 1 1 (& + ,) 1 = ln |& + ,| + = (5 1) 1 = ln |5 1| + 5 1 & 5

1 (& + ,) 1 = ln |& + ,| + ( 1) 1 = ln | 1| + & (& + ,) =

(& + ,) + 1 (1.5 ) 4 1 + (1.5 ) 3 = + = (1.5 ) 4 + &( + 1) 1.5(4) 6

1 1 2.1 + = + + 2.1 = + & 2.1

1 + 1.5 3 . + = . + 1.5 3 = + & ln . 3 ln(1.5) 1 + 4 2 . + = . + 4 2 = + & ln . 2 ln 4

1 1 |& + ,| = (& + ,)|& + ,| + |2 + 4| = (2 + 4)|2 + 4| + 2& 2(2) 1 = (2 + 4)|2 + 4| + 4 1 1 |& + ,| = (& + ,)|& + ,| + |3 2| = (3 2)|3 2| + 2& 2(3) 1 = (3 2)|3 2| + 6

1 + 2 3 + 4 2 3 + 4 2 3 + 4 2 3 + 4 . + = . + (2 3 + 4 + 2 3 + 4) = + = + & ln . 3 ln 2 3 ln 2 3 ln 2

1 + 1.1 + 4 + 1.1 + 4 . + = . + (1.1 + 4 + 1.1 + 4) = + & ln . ln(1.1)


73. ( ) = ( 2 + 1) 3, so ( ) = ( 2 + 1) 3

Solutions Section 13.2

= 2 + 1,

1 = 2 , = 2

1 1 1 4 1 3 ( 2 + 1) 3 = 3 = = + = ( 2 + 1) 4 + 2 2 8 8 1 1 1 1 (0) = 0, so (0 + 1) 4 + = 0 = , and so ( ) = ( 2 + 1) 4 8 8 8 8 74. ( ) =

, so ( ) = 2 2 + 1 +1

= 2 + 1,

1 = 2 , = 2

1 1 1 1 1 = = = ln | | + = ln | 2 + 1| + 2 + 1 2 2 2 2 1 1 1 1 (1) = 0, so ln 2 + = 0 = ln 2, and so ( ) = ln | 2 + 1| ln 2 2 2 2 2 2

2

75. ( ) = 1, so ( ) = 1 = 2 1,

1 = 2, , = 2

2 1 1 1 1 2 1 = = = + = 1 + 2 2 2 2 1 0 1 1 2 1 (1) = 1 2, so + = = 0, and so ( ) = 2 2 2 2

2

76. ( ) = ( 1) 2 , so ( ) = ( 1) 2 = 2 2 ,

1 = (2 2), = 2( 1)

2 1 1 1 1 2 ( 1) 2 = ( 1) = = + = 2 + 2( 1) 2 2 2 1 0 1 1 2 2 (2) = 1, so + = 1 = , and so ( ) = ( + 1) 2 2 2

77. The given integral sixth power:

6

=

-

7

7

(5 2 3) 6 has the form

+

6

, so we use Mike's shortcut rule for the

provided is constant

(5 2 3) 7 (5 2 3) 7 (5 2 3) 6 = + = + . 10 7 70

= (5 2 3) 6 has the form (5 2 3) 6 shortcut rule for the sixth power: 78. The given integral

6

, so we use Mike's


-

Solutions Section 13.2 1 - 6 = provided is constant + 5 5 1 1 (5 2 3) 6 = + = + . 5 2 10 5(5 3) 50(5 2 3) 5

= (3 1) 1 2 has the form 3 1 Mike's shortcut rule for the 1 2 power: 1 2 - 1 2 = provided is constant + (1 2) 2(3 1) 1 2 (3 1) 1 2 (3 1) 1 2 = + = + . 3 (1 2) 3 79. The given integral

1 + 2 = (1 + 2 ) 1 2 has the form Mike's shortcut rule for the 1/2 power: 3 2 - 1 2 = provided is constant + (3 2) (1 + 2 ) 3 2 (1 + 2 ) 3 2 (1 + 2 ) 1 2 = + = + . 2 (3 2) 3 80. The given integral

81. The given integral - =

-

1 2

1 2

, so we use

, so we use

4

3 ( 8) has the form - , so we use Mike's shortcut rule: provided is constant

+

4

4 4 3 ( 8) 3 ( 8) = 3 ( 8) + = + . 4 4

82. The given integral shortcut rule:

- =

-

3

( 4 8)

+

=

4

3 ( 8) has the form - , so we use Mike's

provided is constant 4

4 3 ( 8) ( 4 8) 3 ( 8) = + = + . 4 4 3

83. The given integral

3 = 3 (1 + 2 3 ) 1 has the form 1 + 2 3

Mike's shortcut rule: - 1 = ln | | + provided is constant 3 1 3 (1 + 2 3 ) 1 = 3 ln |1 + 2 3 | + = ln |1 + 2 3 | + . 6 6 84. The given integral

2 = 2 ( 2 3) 1 has the form 2 3

Mike's shortcut rule: - 1 = ln | | +

provided is constant

1

, so we use

1

, so we use


Solutions Section 13.2 2 1 2 ( 2 3) 1 = ln | 2 3| + = ln | 2 3| + . 2 2 2

85. ( ) = 5 + 1 ( + 1) 2, so ( ) = [5 + ( + 1) 2] = 5 + ( + 1) 2 = 5 ( + 1) 1 + (& + ,) + 1 (We used the shortcut formula (& + ,) = + to do the second integral.) &( + 1) (1) = 1, 000, so 5 2 1 + = 1, 000 = 995.5. 1 Thus, ( ) = 5 + 995.5. ( + 1) 86. ( ) = 10 ( 2 + 1) 2, so ( ) = �10

� = 10 2 ( 2 + 1) 2 ( + 1) 2

1 = 2 , = 2 1 1 1 1 2 = = = 1 + = + 2 ( 2 + 1) 2 2 2 2 2 2( + 1) 1 Thus, �10 + . � = 10 + 2 2 2 2( + 1) ( + 1) 1 (2) = 1, 000, so 10(2) + + = 1, 000 = 979.9. 2(5) 1 Thus, ( ) = 10 + + 979.9. 2 2( + 1) = 2 + 1,

87. +( ) = (2,000 480 0.06 ) = 2,000

480 0.06 + (by the shortcut) 0.06

= 2,000 + 8,000 0.06 + +(0) = 0 gives 0 = 2,000(0) + 8,000 0.06(0) + 0 = 8,000 + = 8,000, so +( ) = 2,000 + 8,000 0.06 8,000. Total GDP through June 2014 ( = 4.5) is therefore approximately +(4.5) = 2,000(4.5) + 8,000 0.06(4.5) 8,000 7,107 billion pesos 88. ( ) = (2,400 0.25 200) =

2,400 0.25 200 + (by the shortcut) 0.25

= 9,600 0.25 200 + (0) = 0 gives 0 = 9,600 0.25(0) 200(0) + 0 = 9,600 + = 9,600, so ( ) = 9,600 0.25 200 + 9,600. Total housing starts through June 2009 ( = 3.5) is therefore approximately (3.5) = 9,600 0.25(3.5) 200(3.5) + 9,600 4,898 thousand homes, which rounds to 4.9 million homes, in exact agreement with the actual (rounded) number. 89. Total revenue ( ) =

[(0.64 + 0.85) 2 + 25.5] =

(0.64 + 0.85) 3 + 25.5 + (by the shortcut) 3(0.64)

(0.64 + 0.85) 3 + 25.5 + 1.92 (0.64(0) + 0.85) 3 0.85 3 0.85 3 (0) = 0 gives 0 = 0.32, so + 25.5(0) + 0 = + = 1.92 1.92 1.92 (0.64 + 0.85) 3 ( ) = + 25.5 0.32. 1.92 =


Solutions Section 13.2 (0.64(5) + 0.85) 3 The total revenue up to the start of 2025 is (5) = + 25.5(5) 0.32 $161.8 billion. 1.92

90. Total revenue 0( ) =

[(0.078 + 2.73) 3 1.3] =

(0.078 + 2.73) 4 1.3 + (by the shortcut) 4(0.078)

(0.078 + 2.73) 4 1.3 + 0.312 (0.078(0) + 2.73) 4 2.73 4 2.73 4 0(0) = 0 gives 0 = 178.0, so 1.3(0) + 0 = + = 0.312 0.312 0.312 (0.078 + 2.73) 4 0( ) = 1.3 178.0. 0.312 (0.078(4) + 2.73) 4 The total revenue up to the start of 2024 is 0(4) = 1.3(4) 178.0 $91.3 billion. 0.312 =

91. We are given ( ) = ( ) = 32 0.2469 , so 0.2469 ( ) = 32 0.2469 = 32 + 129.61 0.2469 + . 0.2469 Total number of cases from February 4 up to day = ( ) (0) = �129.61 0.2469 + � �129.61 0 + � = 129.61( 0.2469 1). February 29 is represented by = 25, so the total number of cases from Feb. 4 to Feb. 29 was (25) (0) = 129.61( 0.2469(25) 1) 62,000 cases.

92. We are given ( ) = ( ) = 157 0.0198 , so 0.0198 ( ) = 157 0.0198 = 157 + 7,929.3 0.0198 + . 0.0198 Total number of cases from October 10 up to day = ( ) (0) = �7,929.3 0.0198 + � �7,929.3 0 + � = 7,929.3( 0.0198 1). October 31 is represented by = 21, so the total number of cases from Oct. 10 to Oct. 31 was (21) (0) = 7,929.3( 0.0198(21) 1) 4,088 thousand cases, or around 4,088,000 cases. 93. Cost per day = Daily cost per inmate × Number of inmates 20 0.2 = 75240 + thousand dollars. 0.07 + 0.2 3

Therefore, the total cost function is 20 0.2 20 0.2 ( ) = 75240 + 75 40 75 = + 0.07 + 0.2 0.07 + 0.2 3 0.2 = 3,000 + 1,500 0.07 + 0.2 = 0.07 + 0.2 ,

1 = 0.2 0.2 , = 0.2 0.2

0.2 1 = 3,000 1,500 5 0.2 0.2 = 3,000 7,500 ln | | + = 3,000 7,500 ln(0.07 + 0.2 ) + . Therefore, the total cost from March 1 ( = 0) to March 31 ( = 30) is (30) (0) = [3,000(30) 7,500 ln(0.07 + 0.2(30)) + ] [3,000(0) 7,500 ln(0.07 + 0.2(0)) + ] 109,700 ( 500) 110,000 thousand dollars, or $110 million. = 3,000 1,500


Solutions Section 13.2 94. Cost per day = Daily cost per inmate × Number of inmates 7 0.04 = 150280 + thousand dollars. 0.5 + 0.04 3 Therefore, the total cost function is 7 0.04 7 0.04 ( ) = 150280 + = 150 80 + 150 3 0.04 0.5 + 0.04 0.5 + 0.04 = 12,000 + 1,050 0.5 + 0.04 = 0.5 + 0.04 ,

1 = 0.04 0.04 , = 0.04 0.04

0.04 1 = 12,000 1,050 25 0.04 0.04 = 12,000 26,250 ln | | + = 12,000 26,250 ln(0.5 + 0.04 ) + . Therefore, the total cost from March 1 ( = 0) to March 31 ( = 30) is (30) (0) = [12,000(30) 26,250 ln(0.5 + 0.04(30)) + ] [12,000(0) 26,250 ln(0.5 + 0.04(0)) + ] 365,800 ( 10,600) 376,000 thousand dollars, or $376 million. = 12,000 1,050

95. Using the substitution = 3 + 0.25 , we get 3,600 900 0.25 Total sales 4( ) = = = 3,600 ln(3 + 0.25 ) + . 3 + 0.25 At time = 0, 0 = 3,600 ln(3 + 1) + = 3,600 ln 4, giving 4( ) = 3,600[ln(3 + 0.25 ) ln 4]. After 12 months, 4(12) = 3,600[ln(3 + 0.25(12)) ln 4] 6,310 sets.

96. Using the substitution = 10 + 0.75 , we get 1,800 0.75 2,400 Total sales 4( ) = = 2,400 ln(10 + 0.75 ) + . = 10 + 0.75 At time = 0, 0 = 2,400 ln(10 + 1) + = 2,400 ln 11, giving 4( ) = 2,400[ln(10 + 0.75 ) ln 11]. After 12 months, 4(12) = 2,400[ln(10 + 0.75(12)) ln 11] 15,800 sets. 97. a. ( = ( ) = [ ( 2 + 1) 4 + ] = ( 2 + 1) 4 + For the integral on the left, take

= 2 + 1,

1 2 2

1 = 2 , = 2

1 1 1 5 1 4 ( 2 + 1) 4 = 4 = = + = ( 2 + 1) 5 + . 2 2 10 10 1 1 So, ( = ( 2 + 1) 5 + 2 + . 10 2 1 9 1 1 9 b. ((0) = 1 gives (0 + 1) 5 + 0 + = 1 = . So, ( = ( 2 + 1) 5 + 2 + . 10 10 10 2 10


Solutions Section 13.2 2 1 98. a. ( = ( ) = (3 + ) = 3 + 2 2 2

For the integral on the left, take

= 2,

1 = 2 , = 2

2 2 1 3 3 3 2 3 = 3 = = + = + . 2 2 2 2 3 2 1 2 So, ( = + + . 2 2 3 3 3 2 1 3 b. ((0) = 3 gives 0 + 0 + = 3 = . So, ( = + 2 + . 2 2 2 2 2

99. None; the substitution = simply replaces the letter throughout by the letter and thus does not change the integral at all. For instance, the integral (3 2 + 1) becomes (3 2 + 1) if we substitute = . 100. The purpose of substitution is to introduce a new variable that is defined in terms of the variable of integration. We cannot say = 2 + 1, since is not a new variable. Instead, define ) = 2 + 1 (or any other letter different from ). 101. It may mean that, but it may not; see Example 4. 102. After taking the antiderivative 103. (D): To compute the integral, first break it up into a sum of two integrals, 3 + , and then compute the first using = 2 1 and the second using 2 1 2 + 1 = 2 + 1.

104. Instead of the single term canceling, the substitution would result in an 2 in the denominator, for which further manipulation would be necessary to compute the integral. Easiest is write the integrand as 2 1 = 1 , and then integrate each term. 105. There are several errors: First, the term " " is missing, and this affects all the subsequent steps (since we must substitute for when changing to the variable ). Second, when there is a noncanceling in the integrand, we cannot treat it as a constant. Correct calculation: 3( 2 1) 2 1 3 3 2 3 ( 2 1) = 3 = = + = + = 2 1 2 2 2 2 4 = 2 1 = 2 106. The antiderivative was never taken (the integral sign and the were just dropped in the second line of the calculation). Correct answer: ( 3 1) 3 9 +

107. In the fourth step, was substituted back for before the integral was taken, and was just changed to ; they're not equal. Correct calculation:


= 1 = 3 2 1 = 2 3 3

( 3 1) 2 1 1 1 2 2( 3 1) = 2 = = + = + 3 6 3 2 3 2 Solutions Section 13.2

108. Two errors: In the first step, was just changed to , and in the next step, was treated as though 1 it was constant. Correct answer: ln | 2 1| + 2 2

, which is just the original integral. The substitution 1 1 = , which is no easier to evaluate. The substitution = 2 is = 2 leads to 2 2 similar. 109. The substitution = leads to

1 = , a dead end. (You could try 2 1 2 further substitutions, but they won't lead to an integral that is doable by the methods we've discussed.) The 1 1 substitution = 2 leads to 1 = , another dead end. The substitution 2 2 = 2 is similar. 110. The substitution = 1 2 leads to


Solutions Section 13.3 Section 13.3

1. ( ) = 4 1 over [0, 2]; 5 = 4; 5 = 655

7(6) = 86 9

2 0 = 0.5 4

0

0.5

1

1.5

Total

−1

1

3

5

8

Left Riemann Sum = [ (0) + (0.5) + (1) + (1.5)](0.5) = 8(0.5) = 4

2. ( ) = 1 3 over [ 1, 1]; 5 = 4; 5 = 655

7(6) = 9 :6

1 ( 1) = 0.5 4

−1

−0.5

0

0.5

Total

4

2.5

1

−0.5

7

Left Riemann Sum = [ ( 1) + ( 0.5) + (0) + (0.5)](0.5) = 7(0.5) = 3.5

3. ( ) = 2 over [ 2, 2]; 5 = 4; 5 = 655

7(6) = 6

2 ( 2) =1 4

−2

−1

0

1

Total

4

1

0

1

6

Left Riemann Sum = [ ( 2) + ( 1) + (0) + (1)](1) = 6(1) = 6 5 1 =1 4

2 4. ( ) = over [1, 5]; 5 = 4; 5 =

655

7(6) = 6

1

2

3

4

Total

1

4

9

16

30

Left Riemann Sum = [ (1) + (2) + (3) + (4)](1) = 30(1) = 30 4 ( 1) =1 5

5. ( ) = ( 1) 3 over [ 1, 4]; 5 = 5; 5 = 655

7(6) = (6 9)

−1

0

1

2

3

Total

−8

−1

0

1

8

0

Left Riemann Sum = [ ( 1) + (0) + (1) + (2) + (3)](1) = 0(1) = 0

6. ( ) = 3 over [ 2, 3]; 5 = 5; 5 = 655

7(6) = 6

3 ( 2) =1 5

−2

−1

0

1

2

Total

−8

−1

0

1

8

0

Left Riemann Sum = [ ( 2) + ( 1) + (0) + (1) + (2)](1) = 0(1) = 0

7. ( ) =

1 1 0 over [0, 1]; 5 = 5; 5 = = 0.2 1+ 5


7(6) =

Solutions Section 13.3

655

0

9 9+6

0.2

1

0.4

0.6

0.8

Total

0.83333 0.71429 0.625 0.55556 3.72817

Left Riemann Sum = [ (0) + (0.2) + (0.4) + (0.6) + (0.8)](0.2) 3.72817(0.2) 0.7456

8. ( ) =

1 0 over [0, 1]; 5 = 5; 5 = = 0.2 5 1 + 2 7(6) =

655

0

6 9+6

0.2

0

0.4

0.6

0.8

Total

0.19231 0.34483 0.44118 0.4878 1.46612

Left Riemann Sum = [ (0) + (0.2) + (0.4) + (0.6) + (0.8)](0.2) 1.46612(0.2) 0.2932

9. ( ) =

over [0, 10]; 5 = 5; 5 = 655

7(6) = ;

0 1

10 0 =2 5

2

4

6

8

Total

0.13534 0.01832 0.00248 0.00034 1.15647

Left Riemann Sum = [ (0) + (2) + (4) + (6) + (8)](2) 1.15647(2) 2.3129

10. ( ) = over [ 5, 5]; 5 = 5; 5 = 655

7(6) = ;

−5

5 ( 5) =2 5

−3

−1

1

3

Total

148.41316 20.08554 2.71828 0.36788 0.04979 171.63464

Left Riemann Sum = [ ( 5) + ( 3) + ( 1) + (1) + (3)](2) 171.63464(2) 343.2693 , & 5 0 = =1 5 Left Sum = [ (0) + (1) + (2) + (3) + (4)] [14 + 6 + 2 + 2 + 6](1) = 30

11. =

, & 8 0 = =2 4 Left Sum = [ (0) + (2) + (4) + (6)] [5 + 1 + 1 + 2](2) = 18

12. =

, & 9 1 = =2 4 Left Sum = [ (1) + (3) + (5) + (7)] [0 + 4 + 4 + 3](2) = 22

13. =

, & 2.5 0.5 = = 0.5 4 Left Sum = [ (0.5) + (1) + (1.5) + (2)] [2 + 2.5 + 4 + 2.5](0.5) = 5.5

14. =

, & 3.5 1 = = 0.5 5 Left Sum = [ (1) + (1.5) + (2) + (2.5) + (3)] [ 1 + ( 1) + ( 2) + ( 1) + 1](0.5) = 2

15. = 16. =

, & 3.5 0.5 = =1 3


Solutions Section 13.3 Left Sum = [ (0.5) + (1.5) + (2.5)] [ 1 + ( 1) + ( 1)](1) = 3 , & 3 1 = =1 3 Left Sum = [ (0) + (1) + (2)] [0 + 0 + 0](1) = 0 17. =

, & 3 0.5 = = 0.5 5 Left Sum = [ (0.5) + (1) + (1.5) + (2) + (2.5)] [0 + 0 + 0 + 0 + 0](0.5) = 0

18. =

19. The area is a square of height 1 and width 1, so has area 1 × 1 = 1.

20. The area is a rectangle of height 5 and width 2, so has area 5 × 2 = 10.

1 1 21. The area is a triangle of height 1 and base 1, so has area (1 × 1) = . 2 2

22. Think of the area as the difference of the areas of two triangles: The larger one (above [0, 2] on the -axis) has height 2 and base 2 while the smaller one (above [0, 1] on the -axis) has height 1 and base 1. 1 1 3 So, the area is (2 × 2) (1 × 1) = . 2 2 2


Solutions Section 13.3 1 1 1 1 23. The area is a triangle of height and base 1, so has area < × 1= = . 2 2 2 4

24. Think of the area as the difference of the areas of two triangles: The larger one (above [0, 2] on the 1 -axis) has height 1 and base 2 while the smaller one (above [0, 1] on the -axis) has height and base 1. 2 1 1 1 3 So, the area is (1 × 2) < × 1= = . 2 2 2 4

1 25. The area is a triangle of height 2 and base 2, so has area (2 × 2) = 2. 2

1 9 26. The area is a triangle of height 3 and base 3, so has area (3 × 3) = . 2 2

27. Since the area below the -axis is the same as the area above, the integral is 0.


Solutions Section 13.3 28. Since the area below the -axis is the same as the area above, the integral is 0.

29. If counting grid squares, note that each grid square has an area of 1 × 0.5 = 0.5. Instead of counting grid squares, we average the left and right Riemann sums. Left Sum = (1 + 1 + 1.5 + 2)(1) = 5.5 Right Sum = (1 + 1.5 + 2 + 2)(1) = 6.5 Total Change = Area under graph over [1, 5] = Average of left and right sums = (5.5 + 6.5) 2 = 6 30. If counting grid squares, note that each grid square has an area of 1 × 0.5 = 0.5. Instead of counting grid squares, we average the left and right Riemann sums. Left Sum = (0 + 0.5 + 1 + 1.5)(1) = 3 Right Sum = (0.5 + 1 + 1.5 + 2)(1) = 5 Total Change = Area under graph over [2, 6] = Average of left and right sums = (3 + 5) 2 = 4

31. Left Sum = ( 1 0.5 + 0 + 0.5)(1) = 1 Right Sum = ( 0.5 + 0 + 0.5 + 1)(1) = 1 Total Change = Net area over [2, 6] = Average of left and right sums = ( 1 + 1) 2 = 0

32. Left Sum = ( 0.5 1 0.5 + 0 + 0.5)(1) = 1.5 Right Sum = ( 1 0.5 + 0 + 0.5 + 1)(1) = 0 Total Change = Net area over [0, 5] = Average of left and right sums = ( 1.5 + 0) 2 = 0.75

33. Note that each grid square has an area of 0.5 × 0.5 = 0.25. Note that the areas corresponding to [ 1, 0] and [0, 1] cancel out, so we are left with the area above [1, 2], which is 2 grid squares, or 0.5. 34. Note that each grid square has an area of 0.5 × 0.5 = 0.25. The net number of grid squares is 0.5, giving a total net area of 0.5 × 0.25 = 0.125. 35. Technology formulas: TI-83/84 Plus: 4*√(1-x^2) a. = 10: 3.3045; 55 = 100: 3.1604; 55 = 1,000: 3.1436 b. 3.1416

Excel/Website 4*sqrt(1-x^2)

36. Technology formula: 4/(1+x^2) a. = 10: 3.2399; 55 = 100: 3.1516; 55 = 1,000: 3.1426 b. 3.1416 37. Technology formula: 2*x^1.2/(1+3.5*x^4.7) a. = 10: 0.0275; 55 = 100: 0.0258; 55 = 1,000: 0.0256 b. 0.0256 38. Technology formulas: TI-83/84 Plus/Website: 3*x*e^(1.3*x) Excel/Website: 3*x*exp(1.3*x) a. = 10: 1012.8484; 55 = 100: 1088.5457; 5 = 1,000: 1096.3118 b. 1,097.1769


Solutions Section 13.3 2 0 39. >( ) = 3 2 + 5 over [0, 2]; 5 = 5; 5 = = 0.4 5 ?5

@(?) = :? + A

0

0.4

0.8

1.2

1.6

Total

5

5.48

6.92

9.32 12.68 39.4

Left Riemann Sum = [>(0) + >(0.4) + >(0.8) + >(1.2) + >(1.6)](0.4) = 39.4(0.4) = 15.76

Therefore, total volume 15.76 liters.

40. >( ) = 6 2 + 40 over [0, 3]; = 6; = ?55

@(?) = B? + 8C

3 0 = 0.5 6

0

0.5

1

1.5

2

2.5

Total

40

41.5

46

53.5

64

77.5 322.5

3

4

Left Riemann Sum = [>(0) + >(0.5) + >(1) + >(1.5) + >(2) + >(2.5)](0.5) = 322.5(0.5) = 161.25

Therefore, total volume 161.25 liters. 41. ( ) = 20

5 0 over [0, 5]; 5 = 5; 5 = =1 200 5

D (6) = EC

655

6 ECC

0 20

1

2

Total

19.995 19.99 19.985 19.98 99.95

Left Riemann Sum = [ (0) + (1) + (2) + (3) + (4)](1) = 99.95(1) = 99.95

Therefore, total cost $99.95. 42. ( ) = 25

5 0 over [0, 5]; 5 = 5; 5 = =1 50 5

D (6) = EA

655 6 AC

0 25

1

2

3

4

Total

24.98 24.96 24.94 24.92 124.8

Left Riemann Sum = [ (0) + (1) + (2) + (3) + (4)](1) = 124.8(1) = 124.8

Therefore, total cost $124.80.

43. Marginal cost = 680 0.002 ; Marginal revenue = 1,100 Marginal profit '( ) = Marginal revenue − Marginal cost = 1,100 (680 0.002 ) = 420 + 0.002 Total profit =

20,000

10,000

(420 + 0.002 ) .

'( ) = 420 + 0.002 over [10,000, 20,000]; 5 = 5; =

Riemann sum calculation:

20,000 10,000 = 2,000 5

655 10,000 12,000 14,000 16,000 18,000 Total

F(6) = 8EC + C.CCE6

440

444

448

452

456

2,240

Left Riemann Sum = ['(10,000) + '(12,000) + '(14,000) + '(16,000) + '(18,000)](2,000) = 2,240(2,000) = 4,480,000


Solutions Section 13.3 Therefore, total hourly profit $4,480,000.

44. Marginal cost = 450 + 0.001 ; Marginal revenue = 510 Marginal profit '( ) = Marginal revenue − Marginal cost = 510 (450 + 0.001 ) = 60 0.001 Total profit =

60,000

50,000

(60 0.001 ) .

'( ) = 60 0.001 over [50,000, 60,000]; 5 = 5; 5 =

Riemann sum calculation:

60,000 50,000 = 2,000 5

655 50,000 52,000 54,000 56,000 58,000 Total

F(6) = BC C.CC96

10

8

6

4

2

30

Left Riemann Sum = ['(50,000) + '(52,000) + '(54,000) + '(56,000) + '(58,000)](2,000) = 30(2,000) = 60,000

Therefore, total hourly profit $60,000.

45. The period given corresponds to [0, 5]. Total revenue =

5

( )5 . 0 5 0 ( ) = (0.64 + 0.85) 2 + 25.5 over [0, 5]; 5 = 5; 5 = =1 5 ?55

0

1

2

3

4

Total

G(?) = (C.B8? + C.HA) + EA.A 26.22 27.72 30.04 33.17 37.13 154.28

Left Riemann Sum = [ (0) + (1) + (2) + (3) + (4)](1) 154.28(1) 154

Therefore, total revenue $154 billion.

46. The period given corresponds to [0, 4]. Total revenue =

4

( )5 . 0 4 0 ( ) = (0.078 + 2.73) 3 1.3 over [0, 4]; 5 = 5; 5 = = 0.8 5 ?55

0

0.8

1.6

2.4

3.2

Total

;(?) = (C.CIH? + E.I:) 9.: 19.05 20.47 21.97 23.53 25.15 110.16

Left Riemann Sum = [ (0) + (0.8) + (1.6) + (2.4) + (3.2)](0.8) 110.16(0.8) 88 Therefore, total revenue $88 billion. 6

( )5 . 1 6 1 Interval: [1, 6]; 5 = 5; 5 = =1 5

47. Total number of downloads = ?55

J(?)

1

2

3

4

5

Total

60

65

75

85

110

395

Left Riemann Sum = [ (1) + (2) + (3) + (4) + (5)](1) = 395(1) = 395

Therefore, total number of downloads 395 billion.


6

Solutions Section 13.3

( )5 . 1 6 1 Interval: [1, 6]; 5 = 5; 5 = =1 5 48. Total number of downloads = ?55

J(?)

1

2

3

4

5

Total

25

30

30

30

35

150

Left Riemann Sum = [ (1) + (2) + (3) + (4) + (5)](1) = 150(1) = 150

Therefore, total number of downloads 150 billion.

49. a. Left Sum (3000 + 3000 + 2000 + 2000 + 1000 23000)(1) = 12,000 Right Sum (3000 + 2000 + 2000 + 1000 23000 + 18000)(1) = 3,000 b. Left Sum: Number in 2021 = Number in 2015 + = 148.5 12 = 136.5 million Right Sum: Number in 2021 = Number in 2015 + = 148.5 + 3 = 151.5 million c. The right sum is far more accurate, as the left sum does not take into acount the extremely large jump in the rate of jobs from April 2020 to April 2021.

50. a. Left Sum (2000 + 2000 + 2000 + 3000 + 3000 + 2000)(1) = 14,000 Right Sum (2000 + 2000 + 3000 + 3000 + 2000 + 2000)(1) = 14,000, so their average is also 14,000. b. From Example 4 we can estimate =

18

( )5 as the average of the left and right sums: 14,000, 12 or 14 million additional people employed, for a total of 141.9 + 14 = 155.9 million. c. Your study partner is wrong: the rate of change of the total number of jobs is the same at = 12 and = 18, but that does not mean that the values of are the same at those two values. In fact, they are not, as the rate of change is positive during that period, so the number of jobs grew during that entire period.

51. The period 2013–2020 is represented by the interval [0, 7]. The total change in the number of users is given by the definite integral of the rate of change in the number of users: Total change = 07 ( )5 = Area above interval [0, 7] Left Sum = (35 + 5 25 10 10 10 + 0)(1) = 15 Right Sum = (5 25 10 10 10 + 0 + 5)(1) = 45 15 45 Average = = 30, representing a loss of 30,000 users. 2

52. The period 2018–2022 is represented by the interval [0, 4]. The total change in the number of users is given by the definite integral of the rate of change of the number of users: Total change = 04 ( )5 = Area above interval [0, 4] Left Sum = (20 + 20 + 20 + 40 + 20 + 10 70 10)(0.5) = 25 Right Sum = (20 + 20 + 40 + 20 + 10 70 10 + 10)(0.5) = 20 25 + 20 Average = = 22.5, representing a gain of 22.5 million users. 2 53. The total change in the number of students .( ) from China who took the GRE exams is given by the definite integral of its rate of change: 4

. ( )5 = Area above interval [2, 4] 2 Left Sum = (35 + 32.5 + 30 + 20)(0.5) = 58.75 Right Sum = (32.5 + 30 + 20 + 17.5)(0.5) = 50 58.75 + 50 Average = = 54.375, or about 54,000 students. 2 Total change =


Solutions Section 13.3 54. The total change in the number of students .( ) from India who took the GRE exams is given by the definite integral of its rate of change: 4

( )5 = Area above interval [2, 4] 2 Left Sum = (50 + 52.5 + 50 + 40)(0.5) = 96.25 Right Sum = (52.5 + 50 + 40 + 25)(0.5) = 83.75 58.75 + 50 Average = = 90, or about 90,000 students. 2 Total change =

55. Left Sum = (5 + 10 + 10 + 20)(1) = 45 Right Sum = (10 + 10 + 20 + 30)(1) = 70 45 + 70 Trapezoid Sum = = 57.5 billion dollars 2 The total volume during 2020 is the sum of the amounts traded during the four quarters of that year, which is (8) + (9) + (10) + (11), the right sum. So the trapezoid sum is an underestimation of that volume. 56. Left Sum = (45 + 15 + 5 + 5)(1) = 70 Right Sum = (15 + 5 + 5 + 5)(1) = 30 70 + 30 Trapezoid Sum = = 50 billion dollars 2 The total volume during 2018 is the sum of the amounts traded during the four quarters of that year, which is (0) + (1) + (2) + (3), the left sum. So the trapezoid sum is an underestimation of that volume. 5 0 =1 5 Left Riemann sum = ['(0) + '(1) + '(2) + '(3) + '(4)](1) = [0.83 + 0.71 + 0.67 + 0.61 + 0.62](1) = 3.44 Right Riemann sum = ['(1) + '(2) + '(3) + '(4) + '(5)](1) = [0.71 + 0.67 + 0.61 + 0.62 + 0.63](1) = 3.24 b. Because the right Riemann sum in part (a) is calculated by adding the annual production for = 1 through = 5, it calculates the total production (in billions of barrels) for the corresponding years: 2016 through 2020. Thus, Pemex produced a total of 3.24 billion barrels of crude oil in the period 2016 through 2020. 57. a. =

6 1 =1 5 Left Riemann sum = ['(1) + '(2) + '(3) + '(4) + '(5)](1) = [0.59 + 0.56 + 0.50 + 0.51 + 0.51](1) = 2.67 Right Riemann sum = ['(2) + '(3) + '(4) + '(5) + '(6)](1) = [0.56 + 0.50 + 0.51 + 0.51 + 0.50](1) = 2.58 b. Because the left Riemann sum in part (a) is calculated by adding the annual production for = 1 through = 5, it calculates the total offshore production (in billions of barrels) for the corresponding years: 2016 through 2021. Thus, Pemex produced a total of 2.67 billion barrels of offshore crude oil in the period 2016 through 2021. 58. a. =

59. = (4 0) 5 = 0.8 Left Riemann sum: = [ (0) + (0.8) + (1.6) + (2.4) + (3.2)](0.8) = [30 + 4.4 21.2 46.8 72.4](0.8) = 84.8 ft The answer represents the total change in position, so, after 4 seconds, the stone is about 84.8 ft below where it started.

60. = (1 0) 5 = 0.2 Left Riemann sum = [ (0) + (0.2) + (0.4) + (0.6) + (0.8)](0.2) = [4 + 2.04 + 0.08 1.88 3.84](0.2) = 0.08 m


Solutions Section 13.3 The answer represents the total change in position, so, after 1 second, the stone is about 0.08 m. above where it started. 61. ( ) = 40 2 ft/sec

2

Height of rocket 2 seconds after launch = 0 +

( )5 . 0 a. Using technology, we compute the left Riemann sum with = 100: 105.1 ft. b. Using a numerical integrator, we get 62. ( ) = 600(1 0.5 ) ft/sec

2

0

( )5 106.7 ft. 4

( )5 . 0 a. Using technology, we compute the left Riemann sum with = 100: 1,352 ft. Total distance traveled in first 4 seconds =

b. Using a numerical integrator, we get

4

0

( )5 1,362 ft.

63. a. Using technology, we compute the left Riemann sum with = 150: 892. 10

-( )5 891. 2 c. Using the more accurate answer from part (b): The number of games available at the App Store increased by about 891,000 from mid-2012 to mid-2020. b. Using a numerical integrator, we get

64. a. Using technology, we compute the left Riemann sum with = 150: 526. 8

-( )5 525. 0 c. Using the more accurate answer from part (b): The number of games available at the Play Store increased by about 525,000 from mid-2010 to mid-2018. b. Using a numerical integrator, we get

65. a. Using technology, we compute the left Riemann sum with = 100: 60,100 (to the nearest 100). 25

( )5 61,900. 0 c. Using the more accurate answer from part (b): There were around 61,900 new cases of COVID-19 in the U.S. between February 4 and February 29, 2020. b. Using a numerical integrator, we get

66. a. Using technology, we compute the left Riemann sum with = 100: 4,100 (to the nearest 100). 21

( )5 4,100. 0 c. Using either answer: There were around 4,100,000 new cases of COVID-19 in the U.S. between October 10 and October 31, 2020. b. Using a numerical integrator, we get

67. Yes. The Riemann sum gives an estimated area of (0 + 15 + 18 + 8 + 7 + 16 + 20)(5) = 420 square feet. 68. Using the Riemann sum for the area of the oil slick and multiplying by the thickness, we get an approximation of (0 + 130 + 120 + 130 + 440 + 140 + 80)(50)(0.01) = 520 cubic meters.


Solutions Section 13.3 69. a. Graph:

>( ) = '( )K( ) represents annual total expenditure on oil in the United States, so

20

10

>( )5 represents the

total expenditure on oil in the United States from 1990 to 2000. b. Here is a method of computing the Riemann sum on the Website: At the Website, select the Online Utilities tab, and choose the Numerical Integration Utility and Grapher. Enter (76x+5540)* (0.45x^2-12x+105) for ( ), 10 and 20 for the left and right endpoints, and 200 for the number of subdivisions. Then press "Left Sum" to obtain the left Riemann sum: 2,010,295.5 2,010,000 (rounded 20

>( )5 2,010,000. A total of $2,010,000 million, or $2.01 trillion, 10 was spent on oil in the United States from 1990 to 2000. to 3 significant digits). Therefore,

70. a. Graph:

>( ) = '( )K( ) represents annual total expenditure on oil in China, so

20

10

>( )5 represents the total

expenditure on oil in China from 1990 to 2000. b. Here is a method of computing the Riemann sum on the Website: At the Website, select the Online Utilities tab, and choose the Numerical Integration Utility and Grapher. Enter (82x+221)* (0.45x^2-12x+105) for ( ), 10 and 20 for the left and right endpoints, and 200 for the number of subdivisions. Then press "Left Sum" to obtain the left Riemann sum: 444,240 444,000 (rounded to 3 20

>( )5 444,000. A total of $444,000 million, or $444 billion, was 10 spent on oil in China from 1990 to 2000. significant digits). Therefore,

71. a. '( ) =

1

2

( 72.6) 54.08, so we compute the integral

5.2 2L 99.2% of students obtained between 60 and 100. b.

30

0

'( )5 0 (to at least 15 decimal places)

100

60

'( )5 0.992. Approximately

10.5 2 1 ( 4.5) 2, so we compute the integral '( )5 0.1587. Approximately 2L 5.5 15.87% of customers rated the toothpaste 6 or above.

72. a. '( ) = b.

1.5

0.5

'( )5 0.0013 so approximately 0.13% of customers rated the toothpaste 0 or 1.

73. stays the same: The graph is a horizontal line, and all Riemann sums give the exact area.


Solutions Section 13.3 74. stays the same: The graph is a horizontal line, and all Riemann sums give the exact area. 75. increases: The left sum underestimates the function, by less as increases. 76. decreases: The left sum overestimates the function, by less as increases.

77. The area under the curve and above the -axis equals the area above the curve and below the -axis. 78. Answers may vary. For example:

79. Answers will vary. One example: Let >( ) be the rate of change of net income at time . If >( ) is negative, then the net income is decreasing, so the change in net income, represented by the definite integral of >( ), is negative. 80. Answers may vary. For example:

81. Answers may vary. For example:

82. For positive functions, the midpoint approximation gives, for each subinterval, a rectangle whose height is the average of the heights of the left and right edges. If the function is linear, this rectangle has the same area as the trapezoid under the graph over this subinterval, and hence gives the same result. The argument for negative functions is similar, and for functions that change sign we would need to consider various cases. 83. The total cost is .(1) + .(2) + 61

1

.( )5 with = 60.

+ .(60), which is represented by the Riemann sum approximation of


84. The total cost is .(0) + .(1) + 60

0

.( )5 with = 60.

85. [ ( 1 ) + ( 2 ) +

Solutions Section 13.3 + .(59), which is represented by the Riemann sum approximation of

+ ( )] = M = 1 ( )

86. The difference approaches zero, since both the left and right Riemann sums approach the integral. 87. Answers may vary. For example:

88. If increasing by a factor of 10 does not change the value of the answer when rounded to three decimal places, then the answer is (likely) accurate to three decimal places.


Solutions Section 13.4 Section 13.4 1.

2.

3.

4.

5.

6.

7.

8.

9.

1

1

1

2

( 2 + 2)5 = � ( 2) = �

1

0

1 3 1 14 1 + 2 � = � + 2� � 2� = 1 3 3 3 3

1 2 1 4 15 2 � = � 2� � + 4� = 2 2 2 2 2

(12 5 + 5 4 6 2 + 4)5 = [2 6 + 5 2 3 + 4 ]01 = (2 + 1 2 + 4) (0) = 5

1

0

(4 3 3 2 + 4 1)5 = [ 4 3 + 2 2 ]01 = (1 1 + 2 1) (0) = 1

2

2

1

1

( 3 2 )5 = � (2 3 + )5 = �

16 16 4� � 4� = 0 4 4

4 2 1 1 1 1 1 + � =� + � � + �=0 2 2 1 2 2 2 2

3 3 2 3 2 27 3 40 (2 2 + 3 )5 = � 2 1 + 2� = � + � � 2 + � = � 2 + 3 �5 = 1 2 3 2 2 3 1 1 3

3 1 2 9 5 3 � + �5 = � + ln | |5� = � + ln 53� (2 + ln 52) = + ln� � 2 2 2 2 2 2 3 1

0

10.

11.

12.

13.

14.

15.

(2.1 4.3 1.2)5 = �1.05 2 0

1

(4.3 2 1)5 = �

1

0

4.3 2.2 1 4.3 � = �1.05 � (0) 0.9045 0 2.2 2.2

0 4.3 3 4.3 � = �0 0� � + 1� 0.4333 1 3 3

2 5 = [2 ]01 = 2 2 = 2( 1)

0

1

1

0

0 3 5 = [3 ] 1 = 3 3 1 = 3(1 1)

5 =

1 3

1

1

0

1 2 2 2 1 25 = � 3 2� = 0 = 0 3 3 3 0 1

5 =

2 5 = �

1 3 3 3 1 35 = � 4 3� = = 0 1 4 4 4 1 1

2 1 2 1 1 = � = ln 2 0 ln 52 ln 52 ln 52


Solutions Section 13.4 1 3 3 1 2 16. 3 5 = � = � = ln 3 0 ln 53 ln 53 ln 53 0 1 18(3 + 1) 6 1 17. 18(3 + 1) 55 = � � = [(3 + 1) 6]01 = 4 6 1 6 = 4, 095 0 3 6 0 1

1

0

18.

2

0

20.

+ 15 = [ + 1]02 = 1 ( 1) =

2

0

21.

2 + 15 = �

1

1

22.

3 2 15 = �

0

1

2 + 1 2 2 1 2 3 � = � � � �= 0 ln 52 ln 52 ln 52 2 ln 52

3 2 1 1 3 3 3 40 � =� � � �= 2 ln 53 1 2 ln 53 2 ln 53 27 ln 53

4 1 1 1 | 3 + 4|5 = � ( 3 + 4)| 3 + 4|5� = � ( 8)(8)� � (4)(4)� = 80 6 = 40 3 0 6 6 6

4

23.

4 1 1 1 | 2|5 = � ( 2)| 2|5� = � ( 6)(6)� � (2)(2)� = 40 2 = 20 4 2 2 2 4 4

24.

1 ; when = 0, = 1; when = 1, = 9 = 8 2 + 1, = 16 , = 16

25. 1

9

5 (8 2 + 1) 1 25 =

26. 2

27. 2

2 28. 2

2

1

5 1 2

9 9 1 5 5 5 1 2 5 = 5 = � 2 1 2� = (3 1) = 5 4 1 16 16 16 8 1

1 ; when = 0, = 1; when = = 2 2 + 1, = 4 , = 4

2 2 + 15 =

0

29.

8( + 1) 8 1 � = [ ( + 1) 8]01 = 0 8 ( 1 8) = 1 0 8

1 1 1 1 1 2 15 = � 2 1� = 1 3 = ( 3) 1 2 2 2 2 1 1

19.

0

8( + 1) 75 = �

1 1 5 1 2 1 5 = 5 = � 4 4 1 6

5

1

2, = 5

1 3 2 5 � = (5 3 2 1) 1 6

1 ; when = 2, = 5; when = = 2 2 + 1, = 4 , = 4

3 2 2 + 15 =

5

5

3

1 3 5 1 2 1 5 = 5 = � 4 4 2 5

2, = 5

1 1 3 2 5 � = 5 3 2 5 3 2 = 0 5 2 2

1 ; when = 2, = 3; when = 2, = 3 = 2 + 1, = 2 , = 2 2

+ 15 =

3

3

3 1 1 3 1 1 5 = 5 = � � = ( 3 3) = 0 2 2 2 3 2 3

1 ; when = 0, = 2; when = 1, = 3 = 2 + 2, = 2 , = 2


1

0

5

30. 1

0

Solutions Section 13.4 3 3 1 5 5 5 5 = 5 5 = 5 = � � = ( 3 2) 2 2 2 2 2 2 2

1 ; when = 0, = 2; when = 2, = 6 = 2 + 2, = 2 , = 2 2

5 + 25 =

31. 3

32. 3

2

5 =

1

1

7

5 =

1 ; when = 2, 3 2

= 7; when = 3, = 26

13

3

13 1 1 131 1 1 1 5 = 5 = � ln | |5� = [ln 513 ln 53] or ln(13 3) 3 4 4 4 4 4 3

1 ; when = 0, = 0; when = 1, = 1 = 2, = 2 , = 2

(1.1) 1 1 1 1 5 = (1.1) 5 = � � 2 2 2 ln 51.1 0 0 0 (1.1) 1 1 0.1 = � � � �= 2 ln 51.1 2 ln 51.1 2.2 ln 51.1 2

1

(1.1)

= 3, = 3 2, = 3

2(2.1) =

35.

6 1 3 61 3 3 3 5 = 5 = � ln | |5� = (ln 56 ln 52) = ln 53 2 2 2 2 2 2 2

26 1 1 26 1 1 1 1 5 = 5 = � ln | |5� = [ln 526 ln 57] or ln(26 7) 2 7 3 3 3 3 3 7

26 2

(1.1) 5 =

34. 0

2

3

1 ; when = 2, = 3; when = 3, = 13 = 2 2 5, = 4 , = 4

2 2 2 5

33.

6

= 3 1, = 3 2, =

3 2 1

0

3

2+ 2

1

0

2(2.1)

1 ; when = 0, 3 2

= 0; when = 1, = 1

(2.1) 1 1 1 1 2.1 1 1.1 5 (2.1) 5 = = = � � = 0 3 3 ln 52.1 3 ln 52.1 3 ln 52.1 3 ln(2.1) 3 2 0

= 1 , = 1 2, = 25 ; when = 1, = 1; when = 2, = 1 2

1 2 1 2 2 1 2 5 5 5 = [ ]1 = 1 2 ( ) = 1 2 = = 2 2 1 1 1 2 1

36. 2

1

37.

= ln 5 , = 1 , = 5 ; when = 1, = 0; when = 2, = ln 52

ln 2 ln 2 5 = 5 = � 3 0

2 2 3 2 ln 2 = (ln 52) 3 2 0 = (ln 52) 3 2 � 0 3 3

= 1 + 3 2 , = 6 2 , = 4

1 5 ; when = 0, 6 2

4

= 4; when = 2, = 1 + 3 4

1+ 3 1+ 3 4 2 2 1 1 1+ 3 1 1 5 = | |5� = = � �5 � �5 �ln 2 4 6 6 6 2 0 1 + 3 4 4 1 4 = 6 [ln(1 + 3 ) ln 54] 0.2221 2

38.

= 1 3 2 , = 6 2 , = 12 5 ; when = 0, = 2; when = 2, = 1 3 2 6


1 3 2

2

Solutions Section 13.4

1 3 2 1 1 1 3 1 1 5 | | = = = � �5 � �5 �ln � 2 2 6 6 6 2 0 1 3 2 2 1 2 = 6 [ln |1 3 | ln 52] 0.3932 1

39.

2

2

= + 1, = 1, = ; when = 0, = 1; when = 2, = 3; = 1

3 3 3 3 1 1 5 = 5 = 5 = �1 �5 = � ln | |5� 1 0 +1 1 1 1 = 3 ln 53 (1 ln 51) = 2 ln 3 2

40.

= + 2, = 1, = ; when = 1, = 1; when = 1, = 3; = 2

3 3 3 3 2 2 2 4 4 5 = 5 = 5 = �2 �5 = �2 4 ln | |5� 1 2 1 1 1 1 + = (6 4 ln 53) (2 4 ln 51) = 4 4 ln 3 = 4(1 ln 53) 1

41. 2

1

( 2) 55 =

42. 2

1

55 =

0

( + 2) 55 =

0

1

( 6 + 2 5)5 = �

7

7

6 0

+

� 3 1

= 2, = 1, = ; when = 1, = 1; when = 2, = 0; = + 2

=�

1

0

1 1 4 1 1 = (0 + 0) � + �= 7 3 21

( 2) 1 35 =

43. 0

= 2, = 1, = ; when = 1, = 1; when = 2, = 0; = + 2

3 7

7 3

+

3 2

0

1

1 35 =

0

1

( + 2) 1 35 =

3 3 15 4 3 0 � = (0 + 0) � + � = 1 7 2 14

0

1

( 4 3 + 2 1 3)5

= 2 + 1, = 2, = 12 5 ; when = 0, = 1; when = 1, = 3; = 12 ( 1);

1 3 1 31 1 3 3 2 5 = ( 1) 1 25 = ( 1 2)5 2 2 2 4 1 1 1 1 5 2 1 3 2 3 3 5 2 3 3 2 1 1 3 5 2 3 3 2 1 =� + � =� � � �= 1 10 6 10 6 10 6 10 6 15

2 + 15 =

44. 0

1

= + 1, = 1, = ; when = 1, = 0; when = 0, = 1; = 1

2 + 15 = =�

4 5

5 2

4 3

1

0

2

5 =

1

0

2( 1) 1 25 = 2

4 4 8 3 2 1 � = 0= 0 5 3 15

1

0

( 3 2

1 2

)5

45. The line N = crosses the -axis at = 0, so we can calculate the area as

1

0

5 = �

2 1 1 � = . Graph: 2 0 2


Solutions Section 13.4

46. The line N = 2 crosses the -axis at = 0, so we can calculate the area as 2

1

2 5 = [ 2]12 = 4 1 = 3. Graph:

47. The line N = 4

0

crosses the -axis at = 0, so we can calculate the area as 4 4 2 16 16 5 = 1 25 = � 3 2� = . Graph: 0= 0 3 3 3 0

48. The line N = 2 crosses the -axis at = 0, so we can calculate the area as 16 16 16 4 256 256 2 5 = 2 1 25 = � 3 2� = . Graph: 0= 0 3 3 3 0 0

49. The value of |2 3| is never negative, so we can compute the area as


Solutions Section 13.4 3 1 1 1 9 |2 3|5 = � (2 3)|2 3|5� = (9) ( 9) = . 0 4 4 4 2 0 3

50. The value of |3 2| is never negative, so we can compute the area as 3 3 1 1 1 53 |3 2|5 = � (3 2)|3 2|5� = (49) ( 4) = . 0 6 6 6 6 0

51. The curve N = 2 1 crosses the -axis where 2 1 = 0, so at = ±1. We compute two integrals (see graph below): 1 1 3 1 2 ( 2 1)5 = � � = 1 0 = 0 3 3 3 0 and 4 4 3 64 1 54 ( 2 1)5 = � � = � 4� � 1� = . 1 3 3 3 3 1 The total area is the sum of the absolute values, so 23 + 54 . Graph: = 56 3 3

52. The curve N = 1 2 crosses the -axis where 1 2 = 0, so at = ±1. We compute two integrals (see graph below): 1 3 1 1 1 4 (1 2)5 = � � = �1 � � 1 + � = 1 3 3 3 3 1 and 2 3 2 8 1 4 (1 2)5 = � � = �2 � �1 � = . 1 3 3 3 3 1 The total area is the sum of the absolute values, so 43 + 43 = 83 . Graph:

2

53. The curve N = crosses the -axis at = 0, so we can calculate the area as

(ln 2) 1 2

0

2

5 .


Let

1 ; when = 0, = 0; when = (ln 52) 1 2, = , = 2 , = 2 (ln 2) 1 2 ln 2 2 ln 2 1 1 ln 2 1 5 = 5 = 5 = � � 2 2 2 0 0 0 0

Solutions Section 13.4

2

= ln 52.

1 ln 2 1 1 1 1 = 2 = 2 2 2 2 2

=

2

54. The curve N = 1 crosses the -axis at = 0, so we can calculate the area as Let

= 1, 1

0

2

1 = 2 , = 2 ; when = 0, = 1; when = 1, = 0. 0 1 0 1 1 1 1 1 1

2

15 =

55. >( ) = 3 2 + 5

1

2

5 =

2 1

5 =

2

2

1

0

2

15 .

= (1 ) 2

2

Therefore, total volume = 02(3 2 + 5) = � 3 + 5 � = (2 3 + 5(2)) 0 = 18 liters. 0

56. >( ) = 6 2 + 40

3

Therefore, total volume = 03(6 2 + 40) = �2 3 + 40 � = (2(3) 3 + 40(3)) 0 = 174 liters.

57. ( ) = 5 +

0

2 1,000

100

( )5 =

100

�5 +

10 10 1,000 1 = 500 + �50 + � = $783 3 3

Change in cost =

58. O ( ) = 100 0.001

2 3 100 �5 = �5 + � 1,000 3,000 10

100 0.001 1,000 � 100 0.001 100 100 1 0.1 0.1 1 ) = 100,000( = 100,000 ( 100,000 ) = $53,695.80 1,000

Change in revenue =

O ( )5 =

1,000

100 0.001 5 = �

59. Marginal cost = 680 0.002 ; Marginal revenue = 1,100 Marginal profit '( ) = Marginal revenue − Marginal cost = 1,100 (680 0.002 ) = 420 + 0.002 Total hourly profit =

20,000

10,000

(420 + 0.002 ) = �420 + 0.001 2�

20,000

10,000

= (420(20,000) + 0.001(20,000) 2) (420(10,000) + 0.001(10,000) 2) = 8,800,000 4,300,000 = $4,500,000.

60. Marginal cost = 450 + 0.001 ; Marginal revenue = 510 Marginal profit '( ) = Marginal revenue − Marginal cost = 510 (450 + 0.001 ) = 60 0.001 Total profit =

60,000

50,000

(60 0.001 ) = �60 0.0005 2�

60,000

50,000

= (60(60,000) 0.0005(60,000) ) (60(50,000) 0.0005(50,000) 2) = 1,800,000 1,750,000 = $50,000.

61. ( ) = 60 10 mph

2


Solutions Section 13.4 6

6

Total distance traveled = 1

( )5 =

1

(60 10)5 = [60 + 10 10]16 = 360 + 10 6 10 (60 + 10 1 10) 296miles.

62. ( ) = 100 32 ft/sec

7

( )5 =

7

(100 32 )5 = [100 16 2]17 1 1 = 700 784 (100 16) = 168ft. The ball is 168 feet lower after 7 seconds than it was at the end of the first second.

Total distance traveled =

63. ( ) = | 10 + 40| ft/sec The distance traveled is the displacement: 10 10 1 1 1 | 10 + 40|5 = � ( 10 + 40)| 10 + 40|5� = (60) 2 + (40) 2 = 260ft. 0 20 20 20 0 64. ( ) = 10 + | 5 + 30| ft/sec The distance traveled is the displacement: 10 10 1 (10 + | 5 + 30|)5 = �10 ( 5 + 30)| 5 + 30|5� 0 10 0 1 1 = �100 + (20) 2� �0 (30) 2� = 230ft. 10 10 65.

66.

10

(1 )5 = [ + ]010 = 10 + 10 (0 + 0) 9 gallons

0

1 5 = [ln( + 1)]010 = ln 511 ln 51 2.4 gallons 1 + 0 10

67. Total revenue =

5

0

( )5 =

5

0

[(0.64 + 0.85) 2 + 25.5]5

(0.64 + 0.85) 3 = + 25.5 (by the shortcut) 3(0.64) 0 =2

5

(0.64(5) + 0.85) 3 (0.64(0) + 0.85) 3 + 25.5(5)3 2 + 25.5(0)3 $161.8 billion. 3(0.64) 3(0.64)

68. Total revenue =

4

0

( )5 =

4

0 4

[(0.078 + 2.73) 3 1.3]5

(0.078 + 2.73) 4 = 1.3 (by the shortcut) 0.312 0 =2

69.

10

(0.078(4) + 2.73) 4 (0.078(0) + 2.73) 4 1.3 3 2 1.3 3 $91.2 billion. 0.312 0.312

-( )5 =

2

8 3 [ 8 2 + 90 98]5 = + 45 2 98 " 3 !2 2

10

10

8.(10) 3 8(2) 3 + 45(10) 2 98(10)3 2 + 45(2) 2 98(2)3 891 3 3 The number of games available at the App Store increased by about 891,000 from mid-2012 to mid-2020. = 2

70.

8

0

-( )5 =

7 3 [ 7 2 + 52 + 7]5 = + 26 2 + 7 " 3 !0 0 8

8


Solutions Section 13.4

7(8) + 26(8) 2 + 7(8)3 0 525 . 3 The number of games available at the App Store increased by about 525,000 from mid-2010 to mid-2018. = 2

71.

72.

3

10 0.065 4 3.4 3 + 11 2 + 3.6 = 4 3 8 8 0.065 3.4 0.065 3.4 (10) 4 + (10) 3 11(10) 2 + 3.6(10) (8) 4 + (8) 3 11(8) 2 + 3.6(8) = 4 3 4 3 93.17 ( 161.49) 68 ml 10

( 0.065 3 + 3.4 2 22 + 3.6) = <

22 0.028 4 2.9 3 + 22 2 + 95 = 4 3 20 20 0.028 2.9 0.028 2.9 4 3 2 4 (22) + (22) 22(22) + 95(22) (20) + (20) 3 22(20) 2 + 95(20) = 4 3 4 3 95.3 ( 286.7) 380 ml 22

( 0.028 3 + 2.9 2 44 + 95) = <

12

(0.016 3 0.27 2 + 1.4 2.08) = [0.004 4 0.09 3 + 0.7 2 2.08 ]812 8 = (0.004(12) 4 0.09(12) 3 + 0.7(12) 2 2.08(12)) (0.004(8) 4 0.09(8) 3 + 0.7(8) 2 2.08(8)) = 3.264 ( 1.536) = 4.8. The interval [8, 12] represents the first quarter of 2020 to the first quarter of 2021. The total number of active users of the Coinbase exchange increased by about 4.8 million during the 1-year period starting with the first quarter of 2020. 73.

6 1.3333 4 + 0.48 3 + 30 2 + 229 = 4 3 3 1.3333 4 1.3333 4 3 2 3 (6) + 0.48(6) + 30(6) + 229(6)= < (3) + 0.48(3) + 30(3) 2 + 229(3)= =< 4 4 2,989.67 996.96 1,992.7. The interval [3, 6] represents the period 2017–2020. The total value of online payments through PayPal during the three-year period 2017–2020 was about $1,992.7 billion.

74.

6

(1.3333 3 + 1.44 2 + 60 + 229) = <

1.28 25 32(1.28 ) = 32 ln 1.28 "!0 0 0 25 0 1.28 25 1 P 1.28 S P 1.28 S 32 32 61,950. = R32 = RQ ln 1.28 UUT RQ ln 1.28 UT ln 1.28 There were around 61,950 new cases of COVID-19 in the U.S. between February 4 and February 29, 2020. 75.

76.

25

( ) =

21

0

( )5 =

25

1.02 21 157(1.02 )5 = 157 ln 1.02 !"0 0 21

1.02 21 S P 1.02 0 S 1.02 21 1 P 157 157 4,088. = R157 = RQ ln 1.02 UUT RRQ ln 1.02 UUT ln 1.02 There were around 4,088,000 new cases of COVID-19 in the U.S. between October 10 and October 31, 2020.

480 0.06 77. +(V ) = 0 (2,000 480 0.06 )5 = �2,000 0.06 � (by the shortcut) 0

480 0.06 480 = �2,000V 0.06 � �2,000(0) 0.06 0.06(0)� = 2,000V + 8,000 0.06 8,000.

Total GDP through June 2014 (V = 4.5) is therefore approximately +(4.5) = 2,000(4.5) + 8,000 0.06(4.5) 8,000 7,107 billion pesos


Solutions Section 13.4

2,400 0.25 78. (V ) = 0 (2,400 0.25 200)5 = � 0.25 200 � (by the shortcut) 0

2,400 0.25 2,400 0.25(0) = � 0.25 200V � � 0.25 200(0)� = 9,600 0.25 200V + 9,600

Total housing starts through June 2009 ( = 3.5) is therefore approximately (3.5) = 9,600 0.25(3.5) 200(3.5) + 9,600 4,898 thousand homes, which rounds to 4.9 million homes, in exact agreement with the actual (rounded) number.

79. If 4( ) is the weekly sales rate, we have 4( ) = 50(1 0.05) T-shirts per week after weeks. The total sales over the coming year will be 52 50(0.95) 52 50 50(0.95) 5 = � [(0.95) 52 1] 907 T-shirts. � = 0 ln 50.95 ln 50.95 0 80. If 4( ) is the annual sales rate, we have 4( ) = 4,000(1.1) pens per year after years. The total sales over the next five years will be 5 4,000(1.1) 5 4,000 4,000(1.1) 5 = � [(1.1) 5 1] 25,622 pens. � = 0 ln 51.1 ln 51.1 0

81. Cost per day = Daily cost per inmate × Number of inmates 20 0.2 = 75240 + thousand dollars. 0.07 + 0.2 3 Therefore, the total cost from March 1 ( = 0) to March 31 ( = 30) is

30 30 20 0.2 20 0.2 75 40 75 = + 0.2 0.07 + 0.2 3 0 0 0 0.07 + 30 0.2 = [3,000 ]030 + 1,500 0.2 0 0.07 + 30 0.2 = 90,000 + 1,500 0.2 0 0.07 + We evaluate the last integral using the substitution = 0.07 + 0.2 : 30

75240 +

= 90,000 1,500

0.07+ 6

5

0.07+ = 90,000 7,500[ln ]1.07

1.07 110,000 thousand dollars, or $110 million.

6

82. Cost per day = Daily cost per inmate × Number of inmates 7 0.04 = 150280 + thousand dollars. 0.5 + 0.04 3 Therefore, the total cost from March 1 ( = 0) to March 31 ( = 30) is

30 30 7 0.04 0.04 150 80 1,050 = + 0.04 0.5 + 0.04 3 0 0 0 0.5 + 30 0.04 = [12,000 ]030 + 1,050 0.04 0 0.5 + 30 0.04 = 360,000 + 1,050 0.04 0 0.5 + We evaluate the last integral using the substitution = 0.5 + 0.04 : 30

150280 +

= 360,000 1,050

0.5+ 1.2

25

0.5+ = 360,000 26,250[ln ]1.5

1.5 376,000 thousand dollars, or $376 million.

1.2


Solutions Section 13.4 , , , 83. a. = = = 1 + W, (1 + W, ), , + W W + , = W + , ,

b. Let

1 = (ln 5,), , = (ln ) .

, , 1 1 5 5 = 5 = = 1 + W, W + , (ln 5,), ln , ln(W + , ) ln 5 = + = + ln 5, ln 5, c. Total number of graduates = = �220 +

14

6

�220 +

110 �5 1 + 3.8(1.27)

14 ( ln 1.27� 2,401 thousand students 6 110 ln(3.8 + 1.27 )

= = = 1 + W (1 + W ) + W W + b. Let = W + , = X , = 1 . 84. a.

1 1 5 5 = 5 = = 1 + W W + X X ln(W + ) ln = + = + X X 1 20 c. Total sales = 3�11 + �5 = [11 + 22.22 ln(1,800 + 0.9 )]813 0.9 1 1,800 + 8 = [11(13) + 22.22 ln(1,800 + 0.9(13))] [11(8) + 22.22 ln(1,800 + 0.9(8))] $140 billion 85. Y =

86. a.

0

b. Y =

2 3 2

.2 1

0

=1

Y = =

1 1 1 $5 = � $ 2� 1 = $ 12 $ 02 0 2 2 2

2 1 2 2 1 2 2 2 3 2 (') = �$0 �1 2 � + $0 2 �1 2 � � = $0 �1 2 � . . . .

= $0 �1

Let

1 1 ' 5 = ($ )5 = 0 0

1

1

0 2

$0 .

.

2

.2

+

2

.2

� = $0 �1

2 3 2

.2

1 2 3 2 (')5 = $0 �1 2 � 5 . 0

2

2

, = 2 2 , = 2 ; when 2

$0

2 1 2

�1 12 �

�1

3 2 .

2

2

5 =

$0 . 2

2 1 2

�1 02 �

c. Write $ for $0 and substitute

=

0,

=

2 0 0 = 1 2 and similarly for

$0 . 2 1 3 2 $ .2 2 2 = 0 � 2 1 2� 1 = 0 1 2 0 1 2 0 ( 1 ) ( 0 ) 2 2 0

= 0 in

$0 . 2

2 1 2 �1 2 �

=

1.

to get 0 = $. 2.

87. They are related by the Fundamental Theorem of Calculus, which (briefly) states that the definite


Solutions Section 13.4 integral of a suitable function can be calculated by evaluating the indefinite integral at the two endpoints and subtracting. 88. Calculation of the definite integral always gives a (definite) result: the total change of a function whose rate of change is being integrated, whereas calculation of the indefinite integral gives a whole family of results (antiderivatives) rather than a single, definite result. 89. Computing its definite integral from 5 to /

90. The total sales from time 5 to time / are obtained from the marginal sales by taking its definite integral from a to b. 91. Calculate definite integrals by using antiderivatives. 6( ) , and the FTC tells us that, regardless of the antiderivative of 6 we use, we will obtain this (single) value. 92. Wrong. There is only one possible value for

93. Change in cost = cost.

0

2( ) = ( ) (0) by the FTC, so ( ) = (0) + 0 2( ) . (0) is the fixed

94. Comparing to Exercise 81, we see that the fixed cost is $246.76 and the marginal cost of the th item is 5 . Therefore, the marginal cost of the 10th item is $50. 95. An example is ( ) = 5.

96. An example is 6( ) = 5. 97. An example is 6( ) = .

98. We compute the total change of a quantity over an interval of time using the definite integral of the quantity's rate of change over that interval. If the rate of change of the quantity is negative, the quantity is decreasing, so the change in the total is negative. This corresponds to area below the time axis, so this area should be counted as negative.

6( ) = *( ) *(5), where * is an antiderivative of 6. Hence, 7 ( ) = *( ) *(5). Taking derivatives of both sides, 7 ( ) = * ( ) + 0 = 6( ), as required. The result gives us a formula, in terms of area, for an antiderivative of any continuous function. 99. By the FTC,

2

.( ) = for a suitable choice of 5. However, we require .(0) = 0, and a value for 5 that gives this is 5 = 0. Hence 100. Using Exercise 99, the function we need is 2

.( ) = 0 . The use of technology now gives 2

2

2

.(1) = 01 0.75, .(2) = 02 0.88, .(3) = 03 0.89.


Solutions Chapter 13 Review Chapter 13 Review 1.

2.

3.

4.

3 ( 2 10 + 2) = 5 2 + 2 + 3 2 ( + ) = + 3 2 + 3

4 2 4 4 4 4 3 4 1 2 � = � 2 2� = + = 4 3 15 + 4 (5 ) + � 5 5 5 5 3 5 ( 1) 5

3 3 3 3 3 2 3 ln | | + = 3 2 10 (3 5) ln | | + � � = � 1� = 5 5 5 5 5 2 5

5. By the shortcut,

6. By the shortcut,

7. By the shortcut,

8. By the shortcut, 9.

10.

11.

= 2 + 1

= 2 1 = 2

= 2 + 4

= 2 1 = 2

= 2 7

= 2 1 = 2

1 (2 ) 1 = ln |2 | + 2

( 2 + 2) 1 1 ( 2 + 2) 1 ( 2 + 2) 2 = + = + . 2 ( 1) 2 1 2 +11 2 +11 = + = 2 +11 2 + . 2

(4 3) 1 == + = 1 [4(4 3)] + . (4 3) 2 ( 1)(4)

2.3 1 1 1.3 ( 2 + 1) 1.3 = 1.3 = = + 2 2 4.6 ( 2 + 1) 2.3 = + 4.6

1 1 1 10 ( 2 + 4) 10 = 10 = = 2 2 22 1 = ( 2 + 4) 11 + 22 4

11

+

4 1 1 = 2 = 2 ln | | + 2 = 2 ln | 2 7| +

( 2 7)

=


12.

13.

14.

15.

16.

17.

18.

Solutions Chapter 13 Review 1 1 0.4 = = (3 2 1) 0.4 0.4 6 6 0.6 (3 2 1) 0.6 = + = + 3.6 3.6

= 3 1 = 6 1 = 6 2

= 4 4 + 1

1 ( 3 1) 4 4 + 1 = ( 3 1) 4( 3 1) 1 1 2 3 2 1 1 2 = = + = ( 4 4 + 1) 3 2 + 4 4 3 6

= 3 + 3 + 2

2 + 1 2 + 1 1 1 2 = = ( 3 + 3 + 2) 2 2 3 3( 2 + 1) 1 1 = 1 + = + 3 3 3( + 3 + 2)

= 4 3 4 = 4( 3 1) 1 = 3 4( 1) = 3 2 + 3 = 3( 2 + 1) 1 = 3( 2 + 1) = 2 2

= 1 =

= 2 2

= 1 = = +2

=1 =

= 1 =1 =

2 2 1 ( 2) = ( ) = = + = 2 +

2 2 1 ( 2) = ( ) = = + = 2 +

+1 +1 1 = = + 2 (Solve for in the equation = + 2) 1 = �1 � = ln | | + = ( + 2) ln | + 2| + (or ln | + 2| + if we incorporate the 2 in ).

1 = = ( + 1) (Solve for in the equation = 1.) 2( 1) 5 2 2( 1) 3 2 2 5 2 2 3 2 = ( 3 2 + 1 2) = + + = + + 5 3 5 3

3 0 = = 0.5 6 Left Sum = [ (0) + (0.5) + (1) + (1.5) + (2) + (2.5)] [ 4 + 0 + 3 + 1 + 0 + 2](0.5) = 1 19. =

3 1 = = 0.5 4 Left Sum = [ (1) + (1.5) + (2) + (2.5)] [ 4 1 + 1 + 0](0.5) = 2

20. =


Solutions Chapter 13 Review 1 ( 1) 21. ( ) = 2 + 1 over [ 1, 1]; = 4; = = 0.5 4

( ) = +

−1

−0.5

0

0.5

Total

2

1.25

1

1.25

5.5

Left Riemann Sum = [ ( 1) + ( 0.5) + (0) + (0.5)](0.5) = 5.5(0.5) = 2.75

22. ( ) = ( 1)( 2) 2 over [0, 4]; = 4; =

( ) = ( )( )

4 0 =1 4

0

1

2

3

Total

0

−2

−2

0

-4

Left Riemann Sum = [ (0) + (1) + (2) + (3)](1) = 4(1) = 4

23. ( ) = ( 2 1) over [0, 1]; = 5; =

( ) = ( )

0

0.2

0

1 0 = 0.2 5 0.4

0.6

0.8

Total

−0.192 −0.336 −0.384 −0.288 -1.2

Left Riemann Sum = [ (0) + (0.2) + (0.4) + (0.6) + (0.8)](0.2) = 1.2(0.2) = 0.24

24. ( ) =

1 over [0, 1.5]; = 3; 2 ( ) =

=

1.5 0 = 0.5 3

0

0.5

1

Total

0.5

0.33333

0

0.83333

Left Riemann Sum = [ (0) + (0.5) + (1)](0.5) 0.83333(0.5) 0.4167 2

25. ( ) = over [0, 10]; = 4; =

( ) =

10 0 = 2.5 4

0

2.5

5

7.5

Total

1

0.00193

0

0

1.00193

Left Riemann Sum = [ (0) + (2.5) + (5) + (7.5)](2.5) 1.00193(2.5) 2.5048 2

26. ( ) = over [0, 100]; = 4; =

( ) =

100 0 = 25 4

0

25

50

75

Total

1

0

0

0

1

Left Riemann Sum = [ (0) + (25) + (50) + (75)](25) = 1(25) = 25 27. Tech formulas: TI-83/84: e^(-x2) Excel: EXP(-x^2) Website: e^(-x^2) or exp(-x^2) a. = 10: 0.7778 = 100: 0.7500 = 1,000: 0.7471 b. 0.7468


Solutions Chapter 13 Review 28. Tech formula: x^(-x) a. = 10: 0.7869 = 100: 0.6972 = 1,000: 0.6885 b. 0.6875

29. Left Sum = (0 + 0.5 + 1 + 0 + 0 + 1)(0.5) = 0.25 Right Sum = ( 0.5 + 1 + 0 + 0 + 1 + 1)(0.5) = 0.25 Total Change = Area under graph over [ 1, 2] = Average of left and right sums = ( 0.25 + 0.25) 2 = 0

30. Left Sum = (0 + 0.5 + 1 + 0)(0.5) = 0.25 Right Sum = ( 0.5 + 1 + 0 + 0.5)(0.5) = 0 Total Change = Area under graph over [0, 2] = Average of left and right sums = (0.25 + 0) 2 = 0.125 31.

32.

1

1

( 3 + | |) = �

9

0

( + ) = �

33. By the shortcut,

36.

50

0

0

2 2 3 2 9 81 54 117 + � =� + � 0= 2 3 0 2 3 2

1 3 1 3 = � � = (1 2)( 1 ( 3 7)) = 2 7. 2 2 2 5 1 1 (2 5)

34. By the shortcut,

35.

1

9 1 9 0 + 1 = [ln | + 1|]0 = ln 10 ln 1 = ln 10 2.303.

0.02 1 = �

20

2 4 | | 1 1 1 1 1 1 1 + � =� + � � �=1 2 4 2 1 2 4 2 2 4 2

3 2.2 = �

0.02 1 50 � = 50 2 ( 50 1) = 50( 1 2) 0.02 0

3 2.2 0 3 3 4.4 3(1 44) = = � 2.2 20 2.2 2.2 2.2

1 = 3 2, = 2 ; when = 0, = 1; when = 2, = 9. 3 2 9 1 1 9 1 2 2 2 52 2 3 + 1 = 2 = [ 3 2]19 = (27 1) = = 2 3 1 9 9 9 3 0 1

37. Let

= 3 + 1,

1 = 3 2, = 2 ; when = 0, = 1; when = 2, = 9. 3 2 9 2 2 1 1 9 1 2 2 2 4 = = [ 1 2]19 = (3 1) = = 2 3 3 3 3 0 3 + 1 1 3 1

38. Let

= 3 + 1,

= 8 2 , = 2 ; 8 when = 0, = 1 + 4 = 5; when = ln 2, = 1 + 4 2 ln 2 = 1 + 1 = 2. ln 2 2 5 2 2 1 1 1 1 = = [ln ]25 = [ln 5 ln 2] or ln(2.5) = 2 2 8 8 8 1 + 4 8 0 5 2 8 39. Let

= 1 + 4 2 ,

40. Let

= 1 3 2 ,

when = 0,

= 6 2 , = 2 ; 6 = 1 3 = 2; when = ln 3, = 1 3 2 ln 3 = 1 27 = 26.


Solutions Chapter 13 Review 26 26 2 17, 568 2 2 1 26 2 (1 3 2 ) 2 = = [ 3] 2 = = = 976 2 6 18 18 6 2 2

ln 3

0

41. = 4 2 crosses the -axis when = ±2, so we can compute the area as 2 3 2 8 8 32 (4 2) = �4 � = 8 � 8 + � = . 2 3 3 3 3 2 42. = 4 2 crosses the -axis when = ±2, so we compute two integrals: 2 3 2 8 16 (4 2) = �4 � = 8 (0) = 0 3 3 3 0 and 5 3 5 125 8 (4 2) = �4 � = 20 �8 � = 27. 2 3 3 3 2 16 97 Adding the absolute values gives a total area of . + 27 = 3 3 2

43. = crosses the -axis at = 0, so we can compute the area as Let

5

0

2

.

1 = 2 , = ; when = 0, = 0; when = 5, = 25. So the area is 2 5 25 2 1 1 25 1 1 25 == = = [ ]0 25 = . 2 2 2 2 0 0 0 = 2,

44. Since = |2 | never crosses the -axis, we compute the area as 1

1

|2 | = �

(2 )|2 | 1 4 4 =2 � = 1 4 4 4

1

1

|2 | .

45. a. ( ) = ( ) = (196 + 2 0.16 5) = 196 + 3 3 0.16 6 6 + When = 0, = 0, and substituting gives = 0. So ( ) = 196 + 3 3 0.16 6 6 = 196 + 3 3 0.08 . . b. (6) = 196(6) + (6) 3 3 0.08(6) . . = 3.84 4 books

46. a. ( ) = 20 , so = ( 20 ) = 10 2 + = 100,000 when = 0, so 0 + = 100,000, giving = 100,000, and so = 100,000 10 2 b. = 0 when 100,000 10 2 = 0 2 = 10,000 = $100

47. a. From the formulas at the end of the first section, ( ) = 32 + 0 , where 0 = 100, so ( ) = 32 + 100. By the same formulas, !( ) = 16 2 + 0 + !0 , where !0 = 0, so !( ) = 16 2 + 100 . b. At its highest point the velocity is zero. From above, ( ) = 32 + 100. This is zero when = 100 32 = 3.125 seconds. At that time, the height is !(3.125) = 1" . .125) 2 + 100(3.125) = 156.25 ft. c. The ball returns to Juan's hand when !( ) = 0 : 0 = 16 2 + 100 = ( 16 + 100), which is zero when = 0 or = 100 16 = 6.25 seconds. 48. a. From the formulas at the end of the first section, ( ) = 32 + 0 , where 0 = 60, so ( ) = 32 + 60. By the same formulas, !( ) = 16 2 + 0 + !0 , where !0 = 100, so 16 2 + 60 + 100.


Solutions Chapter 13 Review The book will hit the ground when !( ) = 0 16 2 + 60 + 100 = 0 = 5 seconds. (The other solution is = 1.25 sec, which we reject because it is negative.) b. (5) = 100 ft/sec, so the book is traveling 100 ft/sec when it hits the ground. c. The maximum value of !( ) occurs when ( ) = 0, so when 32 + 60 = 0, giving = 1.875 sec. The height at that time is !(1.875) = 156.25 ft above ground level. 6.2 0.25 +3 !( ) = �6.2 0.25 +3� = + = 24.8 0.25 +3 + 0.25 1 (by the shortcut + = + + ) b. The answer to part (a) tells us that the total projected sales to time is

49. a.

!( ) = 24.8 0.25 +3 + thousand books. As there are no sales on the Web site prior to its launch at ( = 0) the total sales at that time would be zero: 0 = 24.8 0.25(0)+3 + 0 = 24.8 3 + = 24.8 3, and so total usage = 24.8 0.25 +3 24.8 3 = 24.8� 0.25 +3 3� thousand books.

5.1 0.1 ( ) = �5.1 0.1 + 3.3� = + 3.3 + = 51 0.1 + 3.3 + 0.1 1 (by the shortcut + = + + ) b. The answer to part (a) tells us that the total usage to time is 50. a.

( ) = 51 0.1 + 3.3 + terabytes. As Billy-Sean began work at the beginning of January ( = 0) the usage until that time was zero: 0 = 51 0.1(0) + 3.3(0) + 0 = 51 + = 51, and so total usage = 51 0.1 + 3.3 51 terabytes.

5 0 = = 0.5 10 Left Sum = [ (0) + (0.5) + (1) + (1.5) + (2) + + (4.5)] [5 + 5 + 5 + 5 + 5 + 5 + 5 + 5 + 5 + 5](0.5) = 25 Hence, sales were approximately 25,000 copies. 51. =

52. March 1 through June 1 is represented by the interval [2, 5] in the graph. 5

!( ) thousand dollars 2 [ (2) + (3) + (4)](1) [4.5 + 7.5 + 10.5] = $22.5 thousand, or $22,500

Total value of sales =

53. The total number of books sold is given by the definite integral of the function shown over the interval [0, 1.5]; = 0.25. Left Sum = (2 + 1 + 0 1 1 + 0)(0.25) = 0.25 Right Sum = (1 + 0 1 1 + 0 + 0)(0.5) = 0.25 0.25 + ( 0.25) Average = Total net sales amounted to 0. =0 2 54. The total number of books sold is given by the definite integral of the function shown over the interval [0, 4]. 4

( ) =Area above -axis Area below -axis 0 Each grid square on the graph has an area of 0.5 × 0.5 = 0.25. Adding boxes gives us Total change =


Solutions Chapter 13 Review ( 0.25) + ( 0.25) + ( 0.25) + ( 0.125) + 0.125 + 0.125 + ( 0.125) + ( 0.25) = 1. (Alternatively, you can average the left and right Riemann sums to obtain the same answer.) Thus, the total change was 1,000 books.

55. The first 10 days of the period is represented by the interval [0, 10]. = (10 0) 5 = 2 Left Sum = ( (0) + (2) + (4) + (6) + (8))(2) = (0 + 1,968 + 3,904 + 5,856 + 7,872)(2) = 39,200 hits 56.

10

5

( ) [ (5) + (6) + (7) + (8) + (9)](1) = 71.75 72 crashes

1,000( + 3) 2 3 3 2) 5 (8 + ( + 3) ) Put = 8 + ( + 3) 3; = 3( + 3) 2; = [3( + 3) 2]; = 5 = 8 + 83 = 520; = 7 = 8 + 10 3 = 1,008 1,008 1,000 2,000 Cost = [520 1 2 1,008 1 2] 8.237 thousand dollars, or about $8,200. � �= 3 2 3 3 520

57. Total cost =

7

4

( 2) 2[8 ( 2) 3] 3 2 2 Put = 8 ( 2) 3; = 3( 2) 2; = (3( 2) 2); = 2 = 8; = 4 = 8 8 = 0 4 0 8 3 2 ( 2) 2[8 ( 2) 3] 3 2 = ( 2) 2 3 2 = 3 3( 2) 2 2 8 0 2 5 2 8 =# $0 24 thousand dollars. 15 58. Total change in cost =

59. If %( ) is the weekly sales in week , OHaganBooks.com was estimating that %( ) = 6,400(2) 2. The total sales over the first five weeks would be 5 5 12,800 2 5 12,800 5 2 %( ) = 6,400(2) 2 = [2 ]0 = [2 2 0] 86,000 books. ln 2 ln 2 0 0 60. If %( ) is the weekly sales in week , the formula is %( ) = 7,500(2) 3. The total sales over the first five weeks would be 5 5 22,500 3 5 22,500 5 3 %( ) = 7,500(2) 3 = [2 ]0 = [2 2 0] 71,000 books. ln 2 ln 2 0 0 4,474 0.55 0.55 + 14.01 0 0 0 8 0.55 [6,053 + 8,135 ln( + 14.01)]0 35,800 books.

61. Total sales =

5

!( ) =

5

6,053 +

5

(We used the substitution = 0.55 + 14.01 for the second integral.) 900 0.25 3 + 0.25 0 0 0 = [620 + 3,600 ln(3 + 0.25 )]014 16,600 hours.

62. Total usage =

14

&( ) =

14

620 +

14

(We used the substitution = 3 + 0.25 for the second integral.)


Solutions Chapter 13 Case Study Chapter 13 Case Study

1. Spending in Feb = 101.16% of Jan spending = 618.7(1 + .0116) 625.876 $625.9 million Spending in Mar = 625.876(1 + 0.0117) 633.199 $633.2 million Spending in Apr = 633.199(1 + 0.0133) $641.6 million Model predictions: Spending in Feb = '(2) = 0.014941(2) Spending in Mar = '(3) =

0.34505+ 1+ 0.036532(2)+ 6.4060

0.014941(3) 0.34505+ 1+ 0.036532(3)+ 6.4060

0.014941(4) 0.34505+ 1+ 0.036532(4)+ 6.4060

$627.2 million

$632.9 million

Spending in Apr = '(4) = $636.4 million February and March figures are in agreement to two significant digits; April figures agree to only one. Yes, it is acceptable; we do not expect the observed data to coincide with model data; some months the predicted data will be higher; other months lower, depending on random factors that influence month-tomonth construction activity. 2. Z =

18 0.34505+ 1+ 0.036532 + 6.4060 1 0.014941 5 611 million dollars 18 6 6

3. There are at least two advantages. (1) A mathematical model of construction spending makes estimating the monthly spending or the average monthly spending over a given period simple. (2) More importantly, the model shows the trend of the spending rate and thus makes it possible to make projections about future spending figures. 4. Going back to the derivation in the text, replacing 618.7 by '0 would affect the determination of the arbitrary constant : We would get: = ln('0 ) 0.021591,

giving

'( ) = 0.014941

Z =

Hence,

0.34505+ 1+ 0.036532 + ln( ) 0.021591 0.34505+ 1+ 0.036532 0.021591 0 . = '0 0.014941

0.34505+ 1+ 0.036532 0.021591 1 0 '( )5 \ \ \ would be given by 0.014941 5 . ( >

5. ln ' = N5 = (0.00005 2 0.0028 + 0.0158)5 = (0.00005 3) 3 0.0014 2 + 0.0158 + = 1 gives so

ln 618.7 = 0.00005 3 0.0014 + 0.0158 + ,

= ln 618.7 0.00005 3 + 0.0014 0.0158 6.4132.

This gives

3

2

'( ) = (0.00005 3) 0.0014 + 0.0158 + 6.4132,

and hence


Solutions Chapter 13 Case Study 3 2 1 1 Z = '( )5 = (0.00005 3) 0.0014 + 0.0158 + 6.41325 . ( > ( >

6. The quadratic model gives a minimum value of N at = , (2&) = 0.0028 0.0001 = 28, or May 2008, after which time the monthly changes begin to increase, eventually becoming positive and then climbing toward infinity. The model in the text predicts that the monthly changes continue to decrease by a larger percentage each month. This may be reasonable in the near term, but in the long term it predicts housing construction essentially grinding to a halt ('( ) [ 0 as [ +\) whereas the quadratic model, also reasonable in the short term, predicts, in the long term, that housing construction will increase exponentially without bound ('( ) [ +\ as [ +\). Mathematically, there is simply not enough room on the planet to accommodate the long-term predictions of the quadratic model, whereas the model in the text is unreasonably pessimistic (we hope!), so neither model is realistic in the long term.


Solutions Section 14.1 Section 14.1

1.

+ − +

2

0

2

2 = 2 2 + = 2( 1) +

2.

+

3

3

+

0

3 = 3 3 + = 3( + 1) + 3.

+ 3 1 −

3

+

0

(3 1) = (3 1) 3 + = (3 + 2) + 4.

+ 1 − +

1 0

(1 ) = (1 ) + + = (2 ) +


5.

+ 2 1 −

2

+

2

0

Solutions Section 14.1

2

1 2 2 1 2 4 1 2 8

1 1 1 1 ( 2 1) 2 = ( 2 1) 2 2 + 2 + = (2 2 2 1) 2 + 2 2 4 4 6.

+ 2 + 1 −

2

+

2

0

2

12 2 1 2 4

18 2

1 1 1 1 ( 2 + 1) 2 = ( 2 + 1) 2 2 2 + = (2 2 + 2 + 3) 2 + 2 2 4 4 7.

+ 2 + 1 −

2

+

2

0

2 + 4

12 2 + 4 1 2 + 4 4

18 2 + 4

1 1 1 ( 2 + 1) 2 + 4 = ( 2 + 1) 2 + 4 2 + 4 2 + 4 + 2 2 4 1 2 2 + 4 = (2 + 2 + 3) + 4


8.

+ 2 + 1 −

2

+

2

0

Solutions Section 14.1

3 + 1

1 3 + 1 3 1 3 + 1 9

1 3 + 1 27

1 2 2 3 + 1 1 ( 2 + 1) 3 + 1 = ( 2 + 1) 3 + 1 3 + 1 + (9 2 6 + 11) 3 + 1 + + = 3 9 27 27 9.

+ 2 −

1

+

0

2

2 ln 2

2 (ln 2) 2

1 1 2 1 (2 )2 = (2 )2 + 2 + = � + �2 + 2 2 ln 2 ln 2 (ln 2) (ln 2) 10.

+ 3 2 −

3

+

0

4

4 ln 4

4 (ln 4) 2

1 3 3 2 3 (3 2)4 = (3 2)4 4 + = � �4 + 2 ln 4 ln 4 (ln 4) (ln 4) 2 11.

+ 2 1 −

2

+

2

0

3

3 ln 3

3 (ln 3) 2

3 (ln 3) 3

1 2 2 2 ( 2 1)3 = ( 1)3 3 3 + 2 3 ln 3 (ln 3) (ln 3) 2

= � ln 1 + 3

2 + 2 3 �3 + (ln 3) 2 (ln 3)


12.

+ 1 2 − +

2 2 0

Solutions Section 14.1

2

2 ln 2

2 (ln 2) 2

2 (ln 2) 3

1 2 2 (1 2)2 = (1 2)2 + 2 + 2 + 2 3 ln 2 (ln 2) (ln 2) 2

13.

= � ln 1 + 2

2 + 2 3 �2 + (ln 2) 2 (ln 2)

+ 2

− 2 1 +

2

0

2 = ( 2 ) = ( 2 ) (2 1) 2 + = ( 2 + + 1) + 14.

+ 2 + 1 − +

2 0

3

13 3 1 3 9

2 + 1 1 2 1 = (2 + 1) 3 = (2 + 1) 3 3 + = (6 + 5) 3 + 3 3 9 9


15.

+ − +

1

Solutions Section 14.1

( + 2) 6

( + 2) 7 7

0

( + 2) 8 56

1 1 ( + 2) 6 = ( + 2) 7 ( + 2) 8 + 7 56 16.

+ − + −

2

2 2

( 1) 6

( 1) 7 7

( 1) 8 56

0

( 1) 9 504

1 1 1 2( 1) 6 = 2( 1) 7 ( 1) 8 + ( 1) 9 + 7 28 252 17.

+ − +

1 0

( 2) 3

( 2) 2 2 ( 2) 1 2

1 1 = ( 2) 3 = ( 2) 2 ( 2) 1 + ( 2) 3 2 2 1 = + 2( 2) 2 2( 2)

18.

+ − +

1 0

( 1) 2

( 1) 1

ln | 1|

= ( 1) 2 = ( 1) 1 + ln | 1| + = + ln | 1| + ( 1) 2 1


19.

+ ln −

Solutions Section 14.1

3

1

4 4

1 1 3 1 1 4 3 ln = 4 ln = 4 ln + 4 4 4 16 20.

+ ln −

1

2

3 3

1 1 2 1 1 2 ln = 3 ln = 3 ln 3 + 3 3 3 9 21.

+ ln(2 ) −

1

3

2

+ 1

3+

1 1 1 1 ( 2 + 1) ln(2 ) = � 3 + � ln(2 ) � 2 + 1� = � 3 + � ln(2 ) 3 + 3 3 3 9 22.

+ ln( ) 1

3

2

3 2 2

1 1 1 1 ( 2 ) ln( ) = � 3 2� ln( ) � 2 � 3 3 2 2 1 3 1 2 1 3 1 2 = � + + � ln( ) 3 2 9 4 23.

+ ln −

1 3

1

ln =

1 3

3 4 3 4

3 4 3 3 1 3 3 4 3 9 4 3 3 4 3 3 ln = ln + = �ln � + 4 4 4 16 4 4


24.

+ ln − 1 2

1 2

1

2 1 2

ln = 2 1 2 ln

25.

+ −

Solutions Section 14.1

2 1 2 = 2 1 2 ln 4 1 2 + = 2 1 2(ln 2) +

log3

1

1 ( ln 3)

1 log = log3 = log3 + 3 ln 3 ln 3 26.

+ −

log2

1 ( ln 2)

2 2

1 1 2 log2 = 2 log2 = 2 log2 + 2 ln 2 2 2 4 ln 2

4 ( 2 4 3 ) = 2 4 3 = 2 3 . To evaluate the remaining integral 3 we use integration by parts:

27.

+ − +

1 0

2

1 2 2 1 2 4

4 1 1 4 1 1 4 2 3 = 2 2 3 + = � � 2 3 + . 3 2 4 3 2 4 3


Solutions Section 14.1

( 2 + 2 + 1) = 2 + 2 + 1 = 2 2 + 1. To evaluate the remaining integral we use integration by parts: 28.

+ − + −

2

2

0

2

2 2 + 1 = 2 2 2 2 + 1 + = ( 2 + 2 + 2) 2 + 1 + = ( 2 + 2 + 2 + 2 ) + . 2

2

( 2 ) = 2 . To evaluate the first integral we use integration by parts: 29.

+ − + −

2

2

2

0

2 = 2 2 + 2 + = ( 2 2 + 2) + . To evaluate the second integral we use substitution: 1 = 2, = 2 , = 2 2 1 1 1 1 2 = = = + = + . 2 2 2 2

Combining the two integrals, we get 2

2 1 2 ( 2 ) = ( 2 2 + 2) + . 2 2

[(2 + 1) + 2 2 + 1] = (2 + 1) + 2 2 + 1 . To evaluate the first integral we use substitution: = 2 + , = (2 + 1) , = 2 +1 1

30.

2 2 1 (2 + 1) + = (2 + 1) = = + = + + . 2 + 1 To evaluate the second integral we use integration by parts:


+ − + −

2

Solutions Section 14.1

2 + 1

2

1 2 + 1 2

0

1 2 + 1 8

2

1 2 + 1 4

1 1 1 1 1 1 2 2 + 1 = 2 2 + 1 2 + 1 + 2 + 1 + = � 2 + � 2 + 1 + . 2 2 4 2 2 4 Combining the two integrals, we get 2 2 1 1 1 [(2 + 1) + 2 2 + 1] = + � 2 + � 2 + 1 + . 2 2 4

31.

+ 3 4 − +

3

(2 1) 1 2

1 (2 1) 3 2 3

0

1 (2 1) 5 2 15

1 1 (3 4) 2 1 = (3 4)(2 1) 3 2 (2 1) 5 2 + 3 5 32.

+ 2 + 1 − +

2

(3 2) 1 2

2 (3 2) 1 2 3

0

4 (3 2) 3 2 27

2 + 1 2 8 = (2 + 1)(3 2) 1 2 (3 2) 3 2 + 3 2 3 27 33.

+ + 1 − +

1

0

1 0

( + 1) = [( + 1) ]01 = [ ]01 = 0 =


34.

+ 2 +

− 2 + 1

1

1 1 ( 2 + ) = [ ( 2 + ) (2 + 1) 2 ] 1 = [ ( 2 + 3 + 3) ] 1

= 7 1 ( 1) =

35.

+ − + − 1

0

0

− 1

2

+

( + 1) 10

2

2 2 0

7

( + 1) 11 11

( + 1) 12 132

( + 1) 13 1, 716

1 1 2 1 1 ( + 1) 11 ( + 1) 12 + ( + 1) 13� 0 11 66 858 38, 229 1 11 1 12 1 13 1 2 2 + 2 = = 11 66 858 858 286

2( + 1) 10 = �

36.

+

( + 1) 10

3

( + 1) 11 11

− 3 2 + − + 1

0

Solutions Section 14.1

6 6 0

( + 1) 12 132

( + 1) 13 1, 716

( + 1) 14 24, 024

1 1 3 1 2 1 1 ( + 1) 11 ( + 1) 12 + ( + 1) 13 ( + 1) 14� 0 11 44 286 4, 004 471, 041 1 11 1 12 1 13 1 1 2 2 + 2 2 14 � = �= 11 44 286 4, 004 4, 004 4, 004

3( + 1) 10 = �


37.

+ ln(2 ) −

Solutions Section 14.1

1

2 2

2 2 2 2 1 1 1 1 ln(2 ) = � 2 ln(2 )� = � 2 ln(2 )� � 2� 1 1 1 2 2 4 1 1 2 1 1 1 3 7 3 = 2 ln 4 ln 2 �1 � = 4 ln 2 ln 2 = ln 2 2 4 2 4 2 4 2

38.

+ ln(3 ) −

1

2

3 3

2 2 2 2 1 1 2 1 1 2 ln(3 ) = � 3 ln(3 )� = � 3 ln(3 )� � 3� 1 1 1 3 3 9 1 1 3 8 1 8 1 8 1 7 = ln 6 ln 3 � � = ln 6 ln 3 3 3 9 9 3 3 9 2

39.

+ ln( + 1) −

1 ( + 1)

2 2

1 1 1 1 2 1 2 ln( + 1) = � 2 ln( + 1)� = ln 2 0 2 2 0 0 2( + 1) 0 2( + 1) 1

We evaluate this integral using a substitution: = + 1, = ; = 1; when = 0, = 1; when = 1, = 2. 1 2 2 2 ( 1) 2 2 1 2 2 2 + 1 1 2 1 = = = = � 2 + � 2( 1) 2 2 2 2 + 0 1 1 1 1 2 1 2 1 1 1 1 1 = � + ln � = 1 2 + ln 2 � 1 + 0� = + ln 2 1 4 2 2 4 4 2 1 1 1 1 1 Combining with our earlier calculation, we get ln( + 1) = ln 2 � + ln 2� = . 2 4 2 4 0 40.

+ ln( + 1) −

1 ( + 1)

2

3 3

1 1 1 1 3 1 3 2 ln( + 1) = � 3 ln( + 1)� = ln 2 0 3 3 0 0 3( + 1) 0 3( + 1) 1

We evaluate this integral using a substitution:


Solutions Section 14.1 = + 1, = ; = 1; when = 0, = 14; when = 1, = 2 1 2 3 2 ( 1) 3 3 1 2 3 3 2+ 3 1 = = = 3 3 0 3( + 1) 1 3 1 1 2 1 2 2 1 1 1 2 1 = + ln � � 3 + 3 � = � 3 1 3 9 2 3 1 8 1 1 1 5 1 = 2 + 2 ln 2 � + 1 0� = ln 2. 9 3 9 2 18 3 Combining with our earlier calculation, we get 1 1 5 1 2 5 2 ln( + 1) = ln 2 � ln 2� = ln 2 . 3 18 3 3 18 0 41. We calculate the area using

+ − + 10

0

1 0

10

. To evaluate this integral we use integration by parts:

0

= [ ]010 = 10 10 10 (0 0) = 1 11 10.

42. We calculate the area using

+ ln −

1

1

ln . To evaluate this integral we use integration by parts:

2 2

1 1 1 1 ln = � 2 ln � = � 2 ln � � 2� 1 1 1 2 2 4 1 1 2 1 2 1 2 1 1 2 = 0 � � = ( + 1). 2 4 4 4

43. We calculate the area using

+ ln −

1

2

1

( + 1) ln . To evaluate this integral we use integration by parts:

+ 1

2 2 +

2 2 2 2 1 1 1 1 ( + 1) ln = �� 2 + � ln � � + 1� = �� 2 + � ln � � 2 + � 1 1 1 2 2 2 4 1 1 2

= 4 ln 2 0 �1 + 2 � 14 + 1�� = 4 ln 2 74


Solutions Section 14.1 44. The curve crosses the axis at = 1, so we calculate the area as the sum of the absolute values of the two integrals 1

2

( 1) and

( 1) . 0 1 We evaluate each using integration by parts:

+ 1 −

+ 1

0

1 0

( 1) = [( 1) ]01 = [( 2) ]01 = ( 2) = 2

2

( 1) = [( 2) ]12 = 0 ( ) = . 1 The first of these integrals is negative, so the total area is (2 ) + = 2 2.

45.

+ − +

1

| 3|

1 ( 3)| 3| 2

0

1 ( 3) 2| 3| 6

1 1 | 3| = ( 3)| 3| ( 3) 2| 3| + 2 6 46.

+ − +

1 0

| + 4|

1 ( + 2

1 ( + 6

4)| + 4|

4) 2| + 4|

1 1 | + 4| = ( + 4)| + 4| ( + 4) 2| + 4| + 2 6

47.

+ − +

2 2 0

| 3| 3

| 3|

1 ( 3)| 3| 2


Solutions Section 14.1 | 3| 2 = 2 | 3| ( 3)| 3| + 3

48.

+ − +

3

| + 4| + 4

3

| + 4|

1 ( + 2

0

4)| + 4|

| + 4| 3 3 = 3 | + 4| ( + 4)| + 4| + + 4 2 49.

| + 4|

+ 2 2 − + −

4

12 ( + 4)| + 4|

0

1 ( + 4) 3| + 4| 24

1 ( + 6

4

4) 2| + 4|

2 1 2 2| + 4| = 2( + 4)| + 4| ( + 4) 2| + 4| ( + 4) 3| + 4| + 3 6 50.

|2 3|

+ 3 2 − + −

1 (2 3)|2 3| 4

6

1 (2 3) 2|2 3| 24

6

1 (2 3) 3|2 3| 192

0

3 1 1 3 2|2 3| = 2(2 3)|2 3| (2 3) 2|2 3| + (2 3) 3|2 3| + 4 4 32 51.

+ 2 2 + 3 − + −

2 2 2 0

| 4|

1 ( 4)| 4| 2

1 ( 4) 2| 4| 6

1 ( 4) 3| 4| 24


Solutions Section 14.1

( 2 2 + 3)| 4| 1 2 ( 2 + 2

=

52.

1 3)( 4)| 4| 13 ( 1)( 4) 2| 4| + 12 ( 4) 3| 4| +

|2 4|

+ 2 + 1

1 (2 4)|2 4| 4

2 1

1 (2 4) 2|2 4| 24

2

+

1 (2 4) 3|2 4| 192

0

( 2 + 1)|2 4| =

1 2 ( + 4

1 1 1)(2 4)|2 4| 24 (2 1)(2 4) 2|2 4| + 96 (2 4) 3|2 4| +

120

53. We compute the displacement by integrating the velocity over the first two minutes, or 120 seconds: 0

2,000 120 . We evaluate this integral using integration by parts:

+ − + 120

0

1 0

120

120 120

14,400 120

2,000 120 = 2,000[ 120 120 14,400 120]0120

= 2,000[ 14,400 1 14,400 1 ( 14,400 0)] = 28,800,000(1 2 1) ft 20

20

(10 20) = [10 ]020

20 = 200

0 0 We evaluate this last integral using integration by parts: 54.

+ − + 20

0

1 0

20

0

7,610,000 ft.

20

20

20 20 400 20

20 = [ 20 20 400 20]020

= 400 1 400 1 ( 400 0) = 400(1 2 1).

Combining these calculations we get

20

0

(10 20) = 200 400(1 2 1)

94 calculators.


Solutions Section 14.1 ln( 1) + 55. We are given ( ) = 10 + and (0) = 5,000. So ( + 1) 2 ln( + 1) ( ) = ( + 1) 2 ln( + 1) . �10 + � = 10 + ( + 1) 2 To evaluate this integral we use integration by parts:

+ ln( + 1)

( + 1) 2

1 ( + 1)

( + 1) 1

( ) = 10 ( + 1) 1 ln( + 1) +

( + 1) 2 = 10 ( + 1) 1 ln( + 1) ( + 1) 1 + . To determine we substitute (0) = 5,000 : 5,000 = ln 1 1 + = 5,001. So, ln( + 1) 1 ( ) = 10 + 5,001. + 1 + 1

56. ( ) = 10 + 0.001 2 100. The total revenue generated by selling 200 boxes of bulbs is 200

(10 + 0.001 2 100) = [10 ]0200 + 0.001

0

200

= 2,000 + 0.001

2 100 . 0 To evaluate this integral we use integration by parts:

+ − + − 200

2 100

100

2 0

0

2 2

200

100 100

10,000 100

1,000,000 100

2 100 = [ 100 2 100 20,000 100 2,000,000 100]0200 0 = 4,000,000 2 4,000,000 2 2,000,000 2 ( 2,000,000 0) = 2,000,000 10,000,000 2. The total revenue is therefore 2,000 + 0.001(2,000,000 10,000,000 2) = $2,646.65. So,

57. Rate of spending = ( ) ( ), so total spending was 7

( ) ( ) =

7

(2.0 + 1.2 )( 0.76 + 134) . 0 0 We could multiply out to get several integrals we could do directly and one that requires integration by parts, or we could use integration by parts as is:


Solutions Section 14.1

+ 0.76 + 134 0.76

0

+ 7

0

2 + 1.2

2 1.2 2

+ 1.2

(2.0 + 1.2 )( 0.76 + 134) = �( 0.76 + 134)(2 1.2 ) + 0.76( 2 + 1.2 )� 1,998.5

2,000 billion dollars (rounded to the nearest $10 billion).

7

0

58. Rate of spending = ( ) ( ), so total spending was 7

7

( ) ( ) =

(3.5 2.38 0.5 )( 1.2 + 141) . 0 0 We could multiply out to get several integrals we could do directly and one that requires integration by parts, or we could use integration by parts as is:

+ 1.2 + 141 1.2

0

+ 7

0

3.5 2.38 0.5

3.5 + 4.76 0.5

1.75 2 9.52 0.5

(3.5 2.38 0.5 )( 1.2 + 141)

= �( 1.2 + 141)(3.5 + 4.76 0.5 ) + 1.2(1.75 2 9.52 0.5 )� 2,710 billion dollars

7

0

59. Rate of change of area = ( ) ( ) million square feet per year, so total area =

7

0

( ) ( ) = 25

+ 30 + 2,700 30

− + 25

7

0

0

7

0

(0.01 3 0.1 2 + 0.1 + 1)( 30 + 2,700) .

0.01 3 0.1 2 + 0.1 + 1

0.01

0.01

4

4

5

20

0.1

0.1

3

3 4

12

+ 0.1

+ 0.1

(0.01 3 0.1 2 + 0.1 + 1)( 30 + 2,700)

2

2

3

6

+

+

2

2

7

4 3 2 5 4 3 2 = 25 ( 30 + 2,700) 0.01 0.1 + 0.1 + + 30 0.01 0.1 + 0.1 + 4 3 2 20 12 6 2 0 = 264,152 million square feet, or about 264 billion square feet.

60. Rate of change of area = ( ) ( ) million square feet per year, so


total area =

7

0

( ) ( ) = 12

+ 40 + 2,000 + 12

7

0

0

(0.01 3 0.1 2 + 0.1 + 1)(40 + 2,000) .

0.01 3 0.1 2 + 0.1 + 1

40

Solutions Section 14.1

7

0.01

0

0.01

4

4

5

20

0.1

0.1

3

3 4

12

+ 0.1

+ 0.1

(0.01 3 0.1 2 + 0.1 + 1)(40 + 2,000)

2

2

3

6

+

+

2

2

61. Rate of change of revenue = ( ) ( ), so total revenue =

10

5

( ) ( ) =

+ 0.03 2 + 0.7 + 12 − +

0.06 + 0.7 0.06 0

− 10

5

7

4 3 2 5 4 3 2 = 12 (40 + 2,000) 0.01 0.1 + 0.1 + 40 0.01 0.1 + 0.1 + 4 3 2 20 12 6 2 0 101,031 million square feet, or about 101 billion square feet. 10

5

(0.03 2 + 0.7 + 12)(0.1 + 0.02 ) 1 2

(0.1 + 0.02 ) 1 2

100 (0.1 + 0.02 ) 3 2 3 2,000 (0.1 + 0.02 ) 5 2 3 200,000 (0.1 + 0.02 ) 7 2 21

(0.03 2 + 0.7 + 12)(0.1 + 0.02 ) 1 2 = �(0.03 2 + 0.7 + 12) + (0.06)

2,000 100 (0.1 + 0.02 ) 3 2 (0.06 + 0.7) (0.1 + 0.02 ) 5 2 3 3

10 200,000 (0.1 + 0.02 ) 7 2� 5 21

$48 billion

62. Rate of change of revenue = ( ) ( ), so total revenue =

10

5

( ) ( ) =

+ 0.14 2 + 3 + 40 − + −

0.28 + 3 0.28 0

10

5

(0.14 2 + 3 + 40)(0.2 + 0.04 ) 1 2 .

(0.2 + 0.04 ) 1 2

50 (0.2 + 0.04 ) 3 2 3 500 (0.2 + 0.04 ) 5 2 3 25,000 (0.2 + 0.04 ) 7 2 21


Solutions Section 14.1

10

5

(0.14 2 + 3 + 40)(0.2 + 0.04 ) 1 2 = �(0.14 2 + 3 + 40) + (0.28)

50 500 (0.2 + 0.04 ) 3 2 (0.28 + 3) (0.2 + 0.04 ) 5 2 3 3

10 25,000 (0.2 + 0.04 ) 7 2� 5 21

$251 billion

63. Rate of change of revenue = ( ) ( ) million dollars per year, so total revenue to time = = 110

0

0

0

110 0.1 ( 3.7 2 + 37 + 910)

0.1

7.4 + 37

10 0.1

0

1,000 0.1

7.4

+ − Revenue = 110

0.1 ( 3.7 2 + 37 + 910) million dollars.

+ 3.7 2 + 37 + 910 −

( ) ( ) =

0

100 0.1

0.1 ( 3.7 2 + 37 + 910)

= 110� 0.1 [ 10( 3.7 2 + 37 + 910) 100( 7.4 + 37) + 1,000(7.4)]�

( ) = 110� 0.1 (37 2 + 370 5,400) + 5,400� million dollars.

0

64. Rate of change of revenue = ( ) ( ) thousand dollars per year, so total revenue to time = = 110

0

0.1

0

1,000 0.1

0

110 0.1 (4 2 62 + 475)

10 0.1

8

0

8 62

+ Revenue = 110

0

( ) ( ) =

0.1 (4 2 62 + 475) thouand dollars.

+ 4 2 62 + 475 −

100 0.1

0.1 ( 2 62 + 475)

= 110� 0.1 [ 10(4 2 62 + 475) 100(8 62) 1000(8)]�

0

( ) = 110� 0.1 ( 40 2 180 6,550) + 6,550� thousand dollars.

65. If ( ) is the price in week , we are told that ( ) = 10 + 0.5 . If ( ) is the weekly sales, we are told


Solutions Section 14.1 that ( ) = 50 0.02 . The weekly revenue is therefore ( ) = ( ) ( ) = 50(10 + 0.5 ) 0.02 and the total revenue over the coming year is 52

52

50(10 + 0.5 ) 0.02 = 50

(10 + 0.5 ) 0.02 . 0 0 We evaluate the integral using integration by parts:

+ 10 + 0.5 − + 52

0.5 0

0.02

50 0.02

2,500 0.02

(10 + 0.5 ) 0.02 = [ 50(10 + 0.5 ) 0.02 2,500(0.5) 0.02 ]052

0

= 1,800 1.04 1,250 1.04 ( 500 0 1,250 0) = 1,750 3,050 1.04. So, the total revenue is 50(1,750 3,050 1.04) $33,598.

66. If ( ) is the price in week , we are told that ( ) = 10 0.5 . If ( ) is the weekly sales, we are told that ( ) = 50 0.05 . The weekly revenue is therefore ( ) = ( ) ( ) = 50(10 0.5 ) 0.05 and the total revenue over the next six weeks is 6

50(10 0.5 ) 0.05 = 50

6

(10 0.5 ) 0.05 . 0 0 We evaluate the integral using integration by parts:

+ 10 0.5 − + 6

0

0.5 0

0.05

20 0.05

400 0.05

(10 0.5 ) 0.05 = [20(10 0.5 ) 0.05 400( 0.5) 0.05 ]06

= 140 0.3 + 200 0.3 (200 0 + 200 0) = 340 0.3 400. So, the total revenue is 50(340 0.3 400) $2,947.60. 67. a. ( ) ! ( )

if < | | 1 = ( ) + [ ( ) ( )]�1 + � 2 if > if 0 # # 10 0.1 + 3 "( ) = ! 0.05 + 2.5 if 10 # # 20 | 10| 1 = 0.1 + 3 + [( 0.05 + 2.5) ( 0.1 + 3)]�1 + � 2 10 | 10| | 10| = 0.1 + 3 + (0.025 0.25)�1 + � = 0.075 + 2.75 + (0.025 0.25) 10 10 b. The total population change is 20 20 | 10| "( ) = � 0.075 + 2.75 + [0.025 0.25] � 10 0 0


Solutions Section 14.1 | 10| ( 0.075 + 2.75) + (0.025 0.25) . = 10 0 0 The first integral is 20

20

20

( 0.075 + 2.75) = [ 0.0375 2 + 2.75 ]020 = 40. 0 The second integral is done by parts:

| 10| 10

+ 0.025 0.25 − +

0.025 0

1 ( 2

| 10|

10)| 10|

| 10| = [(0.025 0.25)| 10| 0.0125( 10)| 10|]020 10 0 = (2.5(10) 1.25) ( 2.5 1.25) = 2.5. The sum of the integrals gives 40 + 2.5 = 42.5 million people. 20

(0.025 0.25)

68. a. ( ) ! ( )

if < | | 1 = ( ) + [ ( ) ( )]�1 + � 2 if > 0.05 + 2.5 if 0 # # 20 dispaystyle ! 0.075 + 5 if 20 # # 40 | 20| 1 = 0.05 + 2.5 + [( 0.075 + 5) (0.05 + 2.5)]�1 + � 2 20 | 20| | 20| = 0.05 + 2.5 + ( 0.0625 + 1.25)�1 + � = 0.0125 + 3.75 + ( 0.0625 + 1.25) 20 20 b. The total population change is 40 40 | 20| "( ) = � 0.0125 + 3.75 + [ 0.0625 + 1.25] � 20 0 0 40 40 | 20| ( 0.0125 + 3.75) + ( 0.0625 + 1.25) . = 20 0 0 The first integral is 40

( 0.0125 + 3.75) = [ 0.00625 2 + 3.75 ]040 = 140. 0 The second integral is done by parts:

| 20| 20

+ 0.0625 + 1.25 − +

0.0625 0

1 ( 2

| 20|

20)| 20|

| 20| = [( 0.0625 + 1.25)| 20| + 0.03125( 20)| 20|]040 20 0 = ( 25 + 12.5) (25 12.5) = 25. The sum of the integrals gives 140 25 = 115 million people. 40

( 0.0625 + 1.25)


2

2

2

Solutions Section 14.1

69. Answers will vary. Examples are and = 1 $ .

70. the derivative of the first times the integral of the second. 71. Answers will vary. Examples are Exercises 31 and 32, or, more simply, integrals like

72. Answers will vary. Examples are 73. Substitution: = 3 2

( 2 + 1) 1 2 and . ( 2 + 1) 1 2

( + 1) 5 .

74. Parts: Rewrite the integrand as ( 2 3 + 1) (2 3). 75. Parts: Differentiate the first factor and integrate the second. 76. Substitution: = 2 3 + 1 77. Substitution: = ln( + 1) 78. Substitution: = ln( + 1)

79. Parts: Rewrite the integrand as 1 $ ln( 2) or as 2 ln and then integrate the first factor and differentiate the second. 80. Parts: Integrate the first factor and differentiate the second. 81. % + 1 times 82.

+ −

(2 ln )

(ln ) 2

1

(ln ) 2 = (ln ) 2 2 ln . Another integration by parts, as in the text, shows that ln = ln +

so

(ln ) 2 = (ln ) 2 2( ln + ) = (ln ) 2 2 ln + 2 + .

83. If ( ) is a polynomial of degree %, then

0

( ) , we get the following table:

( ) = 0. Using integration by parts to evaluate

( + 1)


+ −

( )

( )

+

( )

±

So, 0

( )

( )

0

pme &

(') + $ $ $ +

84. We can evaluate

+ − + ±

0

( )

( )

( )

( )

(')] + [ (0) +

( )

( ) ]0

( )

(0) + $ $ $ +

(0)] 0 = ( (0) ( (') .

( )

( ) $ $ $

( ) in much the same way as in Exercise 83:

( )

( ) = [ ( )

= [ (') +

Solutions Section 14.1

0

( ) = [ ( ) 0 where this time ( ( ) = ( ) ( ) +

… ( ) +

( ) $ $ $ ±

( ) $ $ $ ±

( ).

( )

( ) ]0 = ( (') ( (0),

( )


Solutions Section 14.2 Section 14.2 1. Area =

2. Area =

[Top Bottom] =

[Top Bottom] =

[Top Bottom] =

2

0

[0 ( 2 4)] = �

2

0

2 3 + 4 � = 8 3 + 8 = 16 3 0 3

[0 ( 3 4 2)] = �

2

[6 ( 3 3 )] =

1 = (18 4) (9 2 1 4) = 39 4 or 9.75

3. Area =

3

2

[Top Bottom] =

4 4 3 2 + � = 4 + 32 3 = 20 3 4 3 0 2

1

[9 3] = �

[Top Bottom] =

= 1 ( 1) = 2

5. Area =

0

[| 1| ( | 1|)] =

2

2

[Top Bottom] =

7. Area = =

[| 1| ( | |)] =

2

[Top Bottom] =

2

[4 + 2 2 2] +

0

3

=

[ 2 + 9 ]

2| 1| = [( 1)| 1|]02

[| 1| + | |]

[5 2 ( 2 2 + 1)] +

[2 2 2 4] = �4 + 2

0 2 = (8 + 4 16 3) + (54 3 9 12) (16 3 4 8) = 31 3 8. Area =

1

2

0

1 1 1 1 2 = [( 1)| 1| + | |] 1 = [(1 + 4) ( 4 1)] = 5 2 2

6. Area =

3

[ 2 + 3 + 12 (12 6 )] =

1 3 2 3 9 = � + � = ( 27 3 + 81 2) ( 1 3 + 9 2) = 82 3 3 2 1

4. Area =

9 2 4 2 � 2 4 1

3

2

[( 2 2 + 1) (5 2)]

3 2 3 2 2 3 2 4 � � + � 2 3 0 3

1

2

[Top Bottom]

[( 2 3 + 3) ( 2 + 3 + 11)] +

1

4

1

[( 2 + 3 + 11) ( 2 3 + 3)]

[ 2 2 + 6 + 8] 2 1 4 1 2 3 2 3 = � 3 2 8 � + � + 3 2 + 8 � 2 1 3 3 = ( 2 3 3 + 8) ( 16 3 12 + 16) + ( 128 3 + 48 + 32) (2 3 + 3 8) = 142 3 =

[2 2 6 8] +

4

9. In these solutions we always take the integral of ( ) )( ) with ( ) * )( ). Remember that, if you reverse the order, you will simply get the negative of that integral and should then take the absolute value. We have 2 * 0 > 1 for all , so the two graphs do not cross. The area is 1 1 1 1 1 1 8 [ 2 ( 1)] = ( 2 + 1) = � 3 + � = + 1 � 1� = . 1 3 3 3 3 1 1


10. 3 = 1 when = 1. The area is 1

1

[ 3 ( 1)] =

1

1

Solutions Section 14.2

1 1 1 1 ( 3 + 1) = � 4 + � = + 1 � 1� = 2. 1 4 4 4

11. = when = 0. The area is 2

0

[ ( )] =

2

0

2 = [ 2]02 = 4 0 = 4.

12. = 2 when = 0. The area is 2 2 3 2 3 = � 2� = 3 0 = 3. � ( )� = 0 2 0 4 0 2 13. Area =

1 3 1 1 1 1 1 [| | 2] = � | | � = � � � + 1 2 3 2 3 2 3 1

1

[Top Bottom] =

1 1 3 [ | | ( 2 2)] = � | | + 2 � 1 2 3 1 1 1 1 1 7 = � + 2� � + 2� = 2 3 2 3 3

1

[Top Bottom] =

14. Area =

15. = 2 when 2 = 0, ( 1) = 0, = 0 or = 1. We calculate the area using two integrals: 0 0 1 1 1 1 5 ( 2 ) = � 3 2� = 0 � � = 1 3 2 3 2 6 1 and 1 1 1 1 1 1 1 ( 2) = � 2 3� = (0) = . 0 2 3 2 3 6 0 5 1 The total area is therefore 6 + 6 = 1.

16. = 3 when 3 = 0, ( 2 1) = 0, = 1, 0, or 1. We calculate the area using two integrals: 0 0 1 1 1 1 1 ( 3 ) = � 4 2� = 0 � � = 1 4 2 4 2 4 1 and 1 1 1 1 1 1 1 ( 3) = � 2 4� = (0) = . 0 2 4 2 4 4 0 1 1 1 The total area is therefore 4 + 4 = 2 . 17. 2 2 = 2 + 4 4 when 2 2 6 + 4 = 0 2( 1)( 2) = 0 = 1, = 2 So, the area is 1

0

[( 2 2 ) ( 2 + 4 4)] +

2

1

[( 2 + 4 4) ( 2 2 )]

1 2 2 3 2 3 3 2 + 4 � + � + 3 2 4 � 0 1 3 3 0 1 = (2 3 3 + 4) 0 + ( 16 3 + 12 8) ( 2 3 + 3 4) = 2.

=

1

(2 2 6 + 4) +

2

( 2 2 + 6 4) = �

18. 2 4 + 2 = 2 + 4 4 when 2 2 8 + 6 = 0


2( 1)( 3) = 0 = 1, = 3 So, the area is

Solutions Section 14.2

1

0

[( 2 4 + 2) ( 2 + 4 4)] +

3

1

[( 2 + 4 4) ( 2 4 + 2)]

1 3 2 3 2 3 4 2 + 6 � + � + 4 2 6 � 0 1 3 3 0 1 = (2 3 4 + 6) 0 + ( 18 + 36 18) ( 2 3 + 4 6) = 16 3.

=

1

3

(2 2 8 + 6) +

( 2 2 + 8 6) = �

19. 2 2 + 10 5 = 2 + 4 + 4 when 3 2 + 6 9 = 0 3( + 3)( 1) = 0 = 3, = 1 So, the area is 1

3

[( 2 + 4 + 4) (2 2 + 10 5)] + 1

2

1

[(2 2 + 10 5) ( 2 + 4 + 4)]

1 (3 2 + 6 9) = [ 3 3 2 + 9 ] 3 + [ 3 + 3 2 9 ]12 1 3 = ( 1 3 + 9) (27 27 27) + (8 + 12 18) (1 + 3 9) = 39.

=

( 3 2 6 + 9) +

2

20. 2 2 + 7 2 = 2 + 4 + 4 when 3 2 + 3 6 = 0 3( + 2)( 1) = 0 = 2, = 1 So, the area is 1

2

[( 2 + 4 + 4) (2 2 + 7 2)] +

2

1

[(2 2 + 7 2) ( 2 + 4 + 4)]

1 2 3 2 3 2 + 6 � + � 3 + 6 � 1 2 2 2 1 2 = ( 1 3 2 + 6) (8 6 12) + (8 + 6 12) (1 + 3 2 6) = 19.

=

1

( 3 2 3 + 6) +

2

(3 2 + 3 6) = � 3

21. > for all . (Examine the graphs, or consider the fact that has its minimum value when its derivative, 1, is 0, which occurs when = 0.) The area is 1 1 1 1 3 ( ) = � 2� = (1) = 1.218. 0 2 2 2 0 22. > 0 > for in [0, 1], so the area is 1 1 1 1 3 1 ( + ) = � + 2� = 1 + ( 1) = 0 2 2 2 0

1.132.

23. ( 1) 2 * 0 * ( 1) 2, so the area is 1

0

[( 1) 2 + ( 1) 2] =

1 2 2 2 2( 1) 2 = � ( 1) 3� = 0 � � = . 0 3 3 3 0 1

24. 2( 3 + 1) 10 * 0 * ( 2 + 1) 10 for in [0, 1], so the area is

0

break this into two integrals so we can use substitution on each: For = 3 + 1, = 3 2 , =

1 ; when = 3 2

0,

1

[ 2( 3 + 1) 10 + ( 2 + 1) 10] . We 1

0

2( 3 + 1) 10 we let

= 1; when = 1,

= 2.


Solutions Section 14.2 1 1 2 10 1 2( 3 + 1) 10 = 2 10 2 = = � 3 33 3 0 1 1 1

For

2

1

0

( 2 + 1) 10 we let

= 2 + 1, = 2 , =

( 2 + 1) 10 =

2

10

2,047 1 11 1 2 = 33 33 33

1 ; when = 2

1 1 2 10 1 = = � 2 2 22 0 1 1 2,047 10,235 The area is therefore 2,047 . + = 33 22 66 1

11 2 � = 1

11 2 � = 1

0,

= 1; when = 0,

2,047 1 11 1 2 = 22 22 22

25. = 4 when 4 = 0, ( 3 1) = 0, = 0 or = 1. So, the area is 1 1 1 1 1 1 3 ( 4) = � 2 5� = . = 0 2 5 2 5 10 0

26. = 4 when 4 + = 0, ( 3 + 1) = 0, = 0 or = 1. So, the area is 0 0 1 1 1 1 3 ( 4 ) = � 5 2� = 0 � � = . 1 5 2 5 2 10 1 27. 3 = 4 when 4 3 = 0, 3( 1) = 0, = 0 or = 1. So, the area is 1 1 1 1 1 1 1 ( 3 4) = � 4 5� = . (0) = 0 4 5 4 5 20 0 28. = 3 when 3 = 0, ( 2 1) = 0, = 0 or = ±1. So, the area is 0 1 0 1 1 1 1 1 ( 3 ) + ( 3) = � 4 2� + � 2 4� 0 1 4 2 2 4 0 1 1 1 1 1 1 = 0 � �+ � �= . 4 2 2 4 2

29. 2 = 4 when 4 2 = 0, 2( 2 1) = 0, = 0 or = ±1. So, the area is 0 1 0 1 1 1 1 1 ( 2 4) + ( 2 4) = � 3 5� + � 3 5� 0 1 3 5 3 5 0 1 1 1 1 1 4 . = 0 � + � + � � 0 = 3 5 3 5 15 (In fact, since 2 * 4 on all of [ 1, 1], we could have used the single integral calculate this area.)

1

1

( 2 4) to

30. 4 2 = 2 4 when 2 4 2 2 = 0, 2 2( 2 1) = 0, = 0 or = ±1. So, the area is 0 1 0 1 2 2 2 2 (2 2 2 4) + (2 2 2 4) = � 3 5� + � 3 5� 0 1 3 5 3 5 0 1 2 2 2 2 8 . = 0 � + � + � � 0 = 3 5 3 5 15 (As in Exercise 29, we could have calculated the area using the single integral 31. 2 2 = 2 + 4 4 when 2 2 6 + 4 = 0 2( 1)( 2) = 0 = 1, = 2 So, the area is

1

1

(2 2 2 4) .)

= 2.


Solutions Section 14.2

2

[( 2 + 4 4) ( 2 2 )] =

2

( 2 2 + 6 4) = �

1 1 = ( 16 3 + 12 8) ( 2 3 + 3 4) = 1 3.

32. 2 4 + 2 = 2 + 4 4 when 2 2 8 + 6 = 0 2( 1)( 3) = 0 = 1, = 3 So, the area is 3

[( 2 + 4 4) ( 2 4 + 2)] =

3

1 1 = ( 18 + 36 18) ( 2 3 + 4 6) = 8 3.

33. 2 2 + 10 5 = 2 + 4 + 4 when 3 2 + 6 9 = 0 3( + 3)( 1) = 0 = 3, = 1 So, the area is 1

[( 2 + 4 + 4) (2 2 + 10 5)] =

3 = ( 1 3 + 9) (27 27 27) = 32.

34. 2 2 + 7 2 = 2 + 4 + 4 when 3 2 + 3 6 = 0 3( + 2)( 1) = 0 = 2, = 1 So, the area is 1

[( 2 + 4 + 4) (2 2 + 7 2)] =

2 = ( 1 3 2 + 6) (8 6 12) = 27 2.

2 2 3 + 3 2 4 � 1 3

( 2 2 + 8 6) = �

1

3

1

2

3 2 3 + 4 2 6 � 1 3

1 ( 3 2 6 + 9) = [ 3 3 2 + 9 ] 3

( 3 2 3 + 6) = � 3

1 3 2 + 6 � 2 2

35. Here are the graphs of + = and + = 2:

The two graphs intersect where = 2, = ln 2. From the graph we can see that the area we want is the area between these two graphs for 0 # # ln 2. So we compute ln 2

0

(2 ) = [2 ]0ln 2 = 2 ln 2 ln 2 (0 1) = 2 ln 2 2 + 1 = 2 ln 2 1.

36. Here are the graphs of + = and + = 3:


Solutions Section 14.2

The two graphs intersect where = 3, = ln 3. From the graph we can see that the area we want is the area between these two graphs for ln 3 # # 0. So we compute 0

ln 3

(3 ) = [3 + ] 0 ln 3 = 0 + 1 ( 3 ln 3 + ln 3) = 3 ln 3 + 1 3 = 3 ln 3 2.

37. Here are the graphs of + = ln and + = 2 ln :

The two graphs intersect where ln = 2 ln , 2 ln = 2, ln = 1, = . From the graph we can see that the area we want is the area between these two graphs for # # 4. So we compute 4

(2 ln 2) = [2( ln ) 2 ] 4

(using the antiderivative of ln we derived in the preceding section) = [2 ln 4 ] 4 = 8 ln 4 16 (2 ln 4 ) = 8 ln 4 + 2 16 0.5269.

38. Here are the graphs of + = ln and + = 1 ln :

The two graphs intersect where ln = 1 ln , ln =

1 , = 2 1 2

area we want is the area between these two graphs for 4

1 2. From the graph we can see that the

# # 4. So we compute

(2 ln 1) = [2( ln ) ] 41 2 = [2 ln 3 ] 41 2 1 2 1 2 1 2 1 2 1 2 = 8 ln 4 12 (2 ln 3 ) = 8 ln 4 + 2 12 2.3878.

39. Formula for Online Utilities → Numerical Integration Utility and Grapher: abs(e^x-(2x+1)). Lower limit: = 1, Upper limit: ' = 1, then press "Integral". Formula for TI-83/84 Plus: fnInt(abs(e^x-(2x+1)),X,-1,1) Answer: 0.9138


Solutions Section 14.2 40. Formula for Online Utilities → Numerical Integration Utility and Grapher: abs(2^x-(x+2)). Lower limt: = 2, Upper limit: ' = 2, then press "Integral". Formula for TI-83/84 Plus: fnInt(abs(2^x-(x+2)),X,-1,1) Answer: 2.667 41. Here are the graphs of + = ln and + =

1 12 : 2

The two graphs intersect at = 1 and at a point somewhere between = 3 and = 4. We cannot solve the equation ln = 12 12 algebraically, but we can use technology to estimate the second intersection point. To four decimal places it is 3.5129. Therefore, the area is approximately 3.5129 1 1 1 1 3.5129 �lnx + � = � ln 2 + � 2 2 4 2 1 1 1 1 1 1 2 = 3.5129 ln(3.5129) 3.5129 (3.5129) + 3.5129 �0 1 + � 0.3222. 4 2 4 2 42. Here are the graphs of + = ln and + = 2:

We need to find the two points of intersection but we cannot solve ln = 2 algebraically. We use technology to estimate that the two points are 0.1586 and 3.1462. Therefore, the area is approximately 3.1462 3.1462 3.1462 1 1 (ln + 2) = � ln 2 + 2 � = � ln 2 + � 0.1586 0.1586 2 2 0.1586 1 1 2 = 3.1462 ln(3.1462) (3.1462) + 3.1462 �0.1586 ln(0.1586) (0.1586) 2 + 0.1586� 2 2 1.9491. 43. Area =

5

0

[ ( ) ( )] =

5

[(100 + 10 ) (90 + 5 )] =

0

5

0

(10 + 5 )

5 2 5 � = 112.5. 2 0 Since area under a curve represents total change, this area represents your total profit for the week, $112.50. = �10 +

44. Area =

3

0

[ ( ) ,( )] =

3

0

[(50,000 + 2,000 ) (45,000 + 1,500 )]


3

Solutions Section 14.2

(5,000 + 500 ) = [5,000 + 250 2]03 = 17,250. 0 This represents the amount you were able to save over the first three years on your job: $17,250. =

12

45. The area is

[(3 2 18 + 750) (6 + 120)] =

12

(3 2 24 + 630) 2 2 = [ 3 12 2 + 630 ]212 = 7,560 1,220 = $6,340 billion, or $6.34 trillion Since the upper curve in the graph represents prescription sales and the lower curve represents R&D spending, the area between them represents the total profit (revenue minus R&D cost) accumulated worldwide by pharmaceutical companies in the development of new drugs from the start of 2012 ( = 2) to the start of 2022 ( = 12). 9

46. The area is 3

[( 4 2 + 4 + 910) (4 2 62 + 480)] =

9

3

( 8 2 + 66 + 430)

9 8 3 + 33 2 + 430 � = 4,599 1,515 = 3,084 million barrels. 3 3 Since the upper curve in the graph represents all crude oil produced and the lower curve crude oil exported to the U.S., the area between them represents the total production of crude oil (in millions of barrels) from the start of 2013 ( = 3) to the start of 2019 ( = 9) not exported to the U.S. Thus, there were about 3,084 million barrels of oil produced from the start of 2013 to the start of 2019 not exported to the U.S.

= �

47. a. Mid-2013 to mid-2020 is represented by the interval [3, 10]. Graph (the curve that is higher over the interval [3, 10] is that of Apple's App Store):

The rate at which game apps were added to the App Store exceeded the rate at which apps were added to the Play Store by ( ) )( ) = 8 2 + 90 98 ( 7 2 + 52 + 7) = 2 + 38 105. Total from mid-2013 to mid-2020 is 10 3 10 1,550 2,009 ( 2 + 38 105) = � + 19 2 105 � = 670 thousand ( 153) = 3 3 3 3 3 apps. b. The integral in part (a) corresponds to the area between the curves + = ( ) and + = )( ) over [3, 10].

48. a. Mid-2010 to mid-2013 is represented by the interval [0, 3]. Graph (the curve that is higher over the interval [03] is that of Google's Play Store):


Solutions Section 14.2

The rate at which game apps were added to the Play Store exceeded the rate at which apps were added to the App Store by )( ) ( ) = 7 2 + 52 + 7 ( 8 2 + 90 98) = 2 38 + 105. Total from mid-2010 to mid-2013 is 3

( 2 38 + 105) = �

3

3

19 2 + 105 � = 153 0 = 153 thousand apps.

0 3 0 b. The integral in part (a) corresponds to the area between the curves + = )( ) and + = ( ) over [0, 3].

49. Area between graphs

[770 0.060 410 0.071 ] 0 (Notice that we need to change the variable of integration so as not to use the letter for two different purposes.) 770 0.060 410 0.071 12,833( 0.060 1) 5,774.6( 0.071 1) = � � 0 0.060 0.071 The total number of wiretaps authorized by federal courts from the start of 1990 up to time was about 12,833( 0.060 1) 5,774.6( 0.071 1). =

23

50. Area between graphs

[770 0.060 410 0.071 ] (Notice that we need to change the variable of integration so as not to use the letter for two different purposes.) 770 0.060 410 0.071 23 12,833(3.9749 0.060 ) 5,774.6(5.1192 0.071 ) = � � 0.060 0.071 The total number of wiretaps authorized by federal courts from time to the start of 2013 was about 12,833(3.9749 0.060 ) 5,774.6(5.1192 0.071 ). =

51. Wrong: It could mean that the graphs of in the textbook.

and ) cross, as shown in the caution at the start of this topic

52. The answer is negative because the curve of ) lies above that of , so the area between the curves should be

[)( ) ( )] , which is the negative of the answer obtained: ( 40) = 40 square units.

53. The area between the export and import curves represents the United States' accumulated trade deficit (that is, the total excess of imports over exports) from 2008 to 2020. 54. The total area between the graphs represents the sum of the accumulated trade deficit from January 1997 to June 1999, and the accumulated trade surplus from June 1999 to December 2001. The integral of


Solutions Section 14.2 Exports Imports represents the net accumulated trade surplus. (A negative value for this integral indicates a deficit.)

55. Up through day 4 the cost curve is above the revenue curve, so the enclosed area over [0, 4] represents accumulated loss. Beyond day 4, the revenue curve is above the cost curve, so the enclosed area over [4, 7] represents accumulated profit. Correct choice: (A). 56. a. (C) because it is the sum of the three pieces of area between the curves. The other two choices each count at least one of those pieces as negative. b. (A) 57. The claim is wrong because the area under a curve can represent income only if the curve is a graph of income per unit time. The value of a stock price is not income per unit time—the income can only be realized when the stock is sold, and it amounts to the current market price. The total net income (per share) from the given investment would be the stock price on the date of sale minus the purchase price of $50. 58. This reasoning is flawed. The total deviation of the measured dosage should be measured by the total area between the curves, which is large. The fact that half of the area between the graphs is above the Specified Dose line and half is below it indicates that the average dose of all the batches is correct.


Solutions Section 14.3 Section 14.3 1. Average =

2. Average =

3. Average =

4. Average =

2 1 2 3 1 1 1 = - 4. = (4 0) = 2 2 0 2 4 2 0

1 1 1 3 1 1 1 1 1 = - 4. = � � = 0 2 2 4 2 4 4 1 1

2 1 2 3 1 1 1 1 ( ) = - 4 2. = (4 2 0) = 1 2 2 4 2 2 0 0

1 1 1 1 1 1 ( 3 ) = - 4 2. = = 4 2 4 2 4 0 0 1


Solutions Section 14.3 1 2 1 1 5. Average = = [ ]02 = (1 2) 0.43 2 2 2 0

6. Average =

1 1 1 1 1 = [ ] 1 = ( 1) 2 1 2 2

7. Average =

1 4| 1 1 1 $ [(2 5)||2 5|]04 = (9 ( 25)) = 34 16 = 17 8 |2 5|| = 4 0 4 4 16

8. Average =

1 3| 1 1 1 3 $ [ ( + 2)| + 2|] 1 + 2| = = (1 ( 9)) = 5 4 4 4 2 8 1

1.18


Solutions Section 14.3 9. / (2) = (3 + 5 + 10) 3 = 6, and so on.

"( )

/ ( )

0

1

2

3

4

5

6

7

3

5

10

3

2

5

6

7

6

6

5

10/3

13/3

6

10. /(2) = (2 + 9 + 7) 3 = 6, and so on.

( )

/( )

0

1

2

3

4

5

6

7

2

9

7

3

2

5

7

1

6

19/3

4

10/3

6

13/3

11. We must have (1 + 2 + "(2)) 3 = / (2) = 3, so "(2) = 6. Working from left to right, we fill in the other missing values similarly.

"( )

/ ( )

0

1

2

3

4

5

6

7

1

2

6

7

11

15

10

2

3

5

8

11

12

9

12. We must have (1 + 5 + "(2)) 3 = / (2) = 5, so "(2) = 9. Working from left to right, we fill in the other missing values similarly.

( )

/( )

0

1

2

3

4

5

6

7

1

5

9

1

5

0

4

2

5

5

5

2

3

2

13. Moving average: 1 3 1 1 1 4 1 / ( ) = 0 5 = - 4. [ ( 5) 4] = (20 3 150 2 + 500 625) = 5 5 4 5 20 20 15 2 125 + 25 = 3 2 4

14. Moving average: 1 1 1 1 1 4 / ( ) = 0 5 ( 3 ) = - 4 2. [ 2 2 ( 5) 4 + 2( 5) 2] = 5 5 4 2 5 20 1 15 2 115 (20 3 150 2 + 480 575) = 3 + 24 = 20 2 4


Solutions Section 14.3

1 2 3 1 3 3 5 3 15. Moving average: / ( ) = 0 5 = - 5 3. [ ( 5) 5 3] = 5 5 5 25 5

1 1 3 1 2 3 5 3 5 16. Moving average: / ( ) = 0 5 ( 2 3 + ) = - 5 3 + [ ( 5) 5 3] + = . 5 5 5 2 5 25 2

1 1 2 17. Moving average: / ( ) = 0 5 0.5 = �2 0.5 � 5 = [ 0.5 0.5( 5)] 5 5 5 2 0.5 2.5 0.5 ) 0.367 = (1 5

18. Moving average: / ( ) = = 10[ 0.1 1] 0.02

1 1 0 0.02 = � 50 0.02 � 5 = 10[ 0.02( 5) 0.02 ] 5 5 5 1.052 0.02


Solutions Section 14.3

1 1 2 1 2 2 3 2 0 5 = - 3 2. [ ( 5) 3 2] = 5 5 3 15 5 (Note that the domain is * 5.)

19. Moving average: / ( ) =

1 1 3 1 3 3 4 3 20. Moving average: / ( ) = 0 5 = - 4 3. [ ( 5) 4 3] = 5 5 4 20 5

||2 1|| 1 1 1 0 5 �1 � = - |2 1||. 5 2 1 5 2 5 1 1 1 1 1 1 = � |2 1|| ( 5) + |2 11||� = �5 |2 1|| + |2 11||� 5 2 2 5 2 2

21. Moving average: / ( ) =

Tech formulas: Function: 1-abs(2x-1)/(2x-1) Moving average: 0.2(5-abs(2x-1)/2+abs(2x-11)/2)


Solutions Section 14.3 ||3 + 1|| 1 1 1 22. Moving average: / ( ) = 0 5 �2 + � = -2 + ||3 + 1||. 5 3 + 1 5 3 5 1 1 1 1| 1 1 = �2 + |3 + 1|| 2( 5) ||3 14||� = �10 + ||3 + 1|| ||3 14||� 5 3 3 5 3 3

Tech formulas: Function: 2+abs(3x+1)/(3x+1) Moving average: 0.2(10+abs(3x+1)/3-abs(3x-14)/3)

1 1 1 1 || 23. Moving average: / ( ) = 0 5 (2 || + 1|| + || ||) = -2 ( + 1)|| + 1|| + | |. 5 5 2 2 5 1 1 1 | | 1 1 | | | | | | = �2 ( + 1)| + 1| + | | 2( 5) + ( 4)| 4| ( 5)| 5|� 5 2 2 2 2

Tech formulas: Function: 2-abs(x+1)+abs(x) Moving average: 0.2(2x-(x+1)*abs(x+1)/2+x*abs(x)/2-2(x-5) +(x-4)*abs(x-4)/2-(x-5)*abs(x-5)/2)

1 1 1 1 24. Moving average: / ( ) = 0 5 (|2 + 1| |2 | 2) = - (2 + 1)|2 + 1| (2 )|2 | 2 . 5 5 4 4 5 1 1 1 1 = � (2 + 1)|2 + 1| (2 )|2 | 2 (2 9)|2 9| 5 4 4 4 1 + (2 10)||2 10|| + 2( 5)� 4


Solutions Section 14.3

Tech formulas: Function: abs(2x+1)-abs(2x)-2 Moving average: 0.2((2x+1)*abs(2x+1)/4-2x*abs(2x)/4-2x -(2x-9)*abs(2x-9)/4+(2x-10)*abs(2x-10)/4+2(x-5)) 25. Plotting the moving average on a TI-83/84 Plus: Y1 = 10X/(1+5*abs(X)) Y2 = (1/3)fnInt(Y1(T),T,X-3,X)

26. Plotting the moving average on a TI-83/84 Plus: Y1 = 1/(1+e^(X)) Y2 = (1/3)fnInt(Y1(T),T,X-3,X)

27. Plotting the moving average on a TI-83/84 Plus: Y1 = ln(1+X^2) Y2 = (1/3)fnInt(Y1(T),T,X-3,X)


Solutions Section 14.3 28. Plotting the moving average on a TI-83/84 Plus: Y1 = e^(1-X^2) Y2 = (1/3)fnInt(Y1(T),T,X-3,X)

29. Plotting the moving average on a TI-83/84 Plus: Y1 = abs(X)-abs(X-1)+abs(X-2)-abs(X-3)+abs(X-4) Y2 = (1/3)fnInt(Y1(T),T,X-3,X)

30. Plotting the moving average on a TI-83/84 Plus: Y1 = abs(X)-2*abs(X-1)+2*abs(X-2)-2*abs(X-3)+abs(X-4) Y2 = (1/3)fnInt(Y1(T),T,X-3,X)

31. Plotting the moving average on a TI-83/84 Plus: Y1 = abs(X)/X-abs(X-1)/(X-1)+abs(X-2)/(X-2)-abs(X-3)/(X-3) Y2 = (1/3)fnInt(Y1(T),T,X-3,X)


Solutions Section 14.3 32. Plotting the moving average on a TI-83/84 Plus: Y1 = abs(X)/X-2*abs(X-1)/(X-1)+2*abs(X-2)/(X-2)-abs(X-3)/(X-3) Y2 = (1/3)fnInt(Y1(T),T,X-3,X)

33. Plotting the moving average on a TI-83/84 Plus: Y1 = abs(X)/X+abs(X-1)/(X-1)+abs(X-2)/(X-2)+abs(X-3)/(X-3) Y2 = (1/3)fnInt(Y1(T),T,X-3,X)

34. Plotting the moving average on a TI-83/84 Plus: Y1 = 4-abs(X)/X-abs(X-1)/(X-1)-abs(X-2)/(X-2)-abs(X-3)/(X-3) Y2 = (1/3)fnInt(Y1(T),T,X-3,X)

35. Average = 36. Average =

10 1 1 � 10 (0.28 + 3.0) = 0.14 2 + 3 �0 = $4.4 million 10 0 10

10 1 1 � 10 (0.14 + 1.1) = 0.07 2 + 1.1 �0 = $1.8 million 10 10 0

4 1 4 1 0.4 3 (0.4 2 0.3 + 0.2) = 0.15 2 + 0.2 . 4 4 3 0 0 (rounded to two significant digits).

37. Average =

4 1 4 1 0.1 3 (0.1 2 + 0.08 + 0.7) = + 0.04 2 + 0.7 . 4 4 3 0 0 (rounded to two significant digits).

38. Average =

1.7333, or $1.7 billion 1.3933, or $1.4 billion


39. Average = 40. Average =

1 1 97.2(1.20) 97.2(1.20) = 10 10 1 ln(1.2) 20 0 10

Solutions Section 14.3 10

1 6 1 230(1.26) 230(1.26) = 6 6 1 ln(1.26) 20 0

6

277 million tons

$498 billion

41. The amount you have in the account at time is 3( ) = 10,000 0.08 , 0 # # 1. The average amount over the first year is 1 10,000 0.08 1 10,000 0.08 = .0 = $10,410.88. 0.08 0 42. The amount you have in the account at time is 3( ) = 10,000 0.12 , 0 # # 1. The average amount over the first year is 1 10,000 0.12 1 10,000 0.12 = .0 = $10,624.74. 0.12 0 43. The amount in the account begins at $3,000 at the beginning of the month and then declines linearly to 0 by the end of the month. So, the amount in the account during the month is 3( ) = 3,000 3,000 , 0 # # 1. The average over one month is therefore 1

1 (3,000 3,000 ) = �3,000 1,500 2�0 = $1,500. 0 Since the average over each month is $1,500, the average over several months is also $1,500.

44. The amount in the account during the first two months is 4,000 3,000 if0 # # 1 3( ) = !8,000 3,000 if1 # # 2. The average amount over the first two months is 1 2 1 (8,000 3,000 ) � � (4,000 3,000 ) + 2 0 1 1 2 1 1 = �4,000 1,500 2� + �8,000 1,500 2� 0 1 2 2 = 1,250 + 1,750 = $3,000. 45. TI-83/84 Plus: On the home screen, enter (1/6)*fnInt(X^3-10X^2+24X+39,X,0,6) and press [ENTER]. (You can find fnInt in the MATH menu (option 9).) Website: Go to

Enter (x^3-10x^2+24x+39)/6 for ( ), 0 and 6 for the left and right endpoints. Then press "Integral". Result: $45,000 Website → On Line Utilities → Numerical Integration Utility and Grapher

46. TI-83/84 Plus: On the home screen, enter (1/6)*fnInt(-0.05X^3+0.3X^2-0.8X+19,X,0,6) and press [ENTER]. (You can find fnInt in the MATH menu (option 9).) Website: Go to

Enter (-0.05x^3+0.3x^2-0.8x+19)/6 for ( ), 0 and 6 for the left and right endpoints. Then Website → On Line Utilities → Numerical Integration Utility and Grapher


Solutions Section 14.3 press "Integral". Result: $1,750 (17.5 hundred dollars) per ounce 47. Year

2012 2013 2014 2015 2016 2017 2018 2019 2020 2021

Stock Price

87

91

97

83

86

82

80

74

45

56

Moving Average (rounded)

76

81

89

90

89

87

83

81

70

64

Each 4-year moving average is computed by averaging that year's figure with those of the preceding three years: 2015: (87 + 91 + 97 + 83) 4 = 89.5 90 2016: (91 + 97 + 83 + 86) 4 = 89.25 89 and so on. The moving average dropped, but to lesser extent, in 2020, and declined further in 2021. 48. Year

2005 2006 2007 2008 2009 2010 2011 2012 2013 2014

Stock Price

18

20

38

16

13

10

5

4

8

8

Moving Average (rounded)

17

18

23

23

22

19

11

8

7

6

Each 4-year moving average is computed by averaging that year's figure with those of the preceding three years: 2008: (18 + 20 + 38 + 16) 4 = 23 2009: (20 + 38 + 16 + 13) 4 = 21.75 22 and so on. The average change in the moving average is lower. 49. a. To obtain the moving averages from January to June, use the fact that the data repeat every 12 months. Graph:

b. The 12-month moving average is constant and equal to the year-long average of approximately 79°. 50. a. To obtain the moving averages from January to June, use the fact that the data repeat every 12 months. Graph:

b. The 12-month moving average is constant and equal to the year-long average of approximately 41°.


Solutions Section 14.3 51. (B); Each four-quarter average includes exactly one high point and three lower points, so there should be no pronounced period peaks or drops. 52. (A): The peaks that occur every two years should produce a pattern of four successive averages being higher (those that contain a peak) alternating with four that are lower (those that do not contain a peak), as in choice (A). 53. a. Average = nearest $ billion).

1 7 1 7 (0.03 2 + 0.7 + 12) = �0.01 3 + 0.35 2 + 12 �0 7 7 0

b. Moving average =

$15 billion per year (to the

1 1 (0.03 2 + 0.7 + 12) = �0.01 3 + 0.35 2 + 12 � 2 2 2 2

1� 0.01( 3 ( 2) 3) + 0.35( 2 ( 2) 2) + 24� 2 c. The function is quadratic because the 3 terms cancel. =

54. a. Average = nearest $ billion).

10 1 10 1 0.14 3 (0.14 2 + 3 + 40) = + 1.5 2 + 40 . 5 5 3 5 5

b. Moving average =

$71 billion per year (to the

1 1 0.14 3 (0.14 2 + 3 + 40) = + 1.5 2 + 40 . 2 2 3 2 2

1 0.14 3 ( ( 2) 3) + 1.5( 2 ( 2) 2) + 80. 2- 3 c. The function is quadratic because the 3 terms cancel. =

55. a. The line through ( , ) = (0, 526) and (11, 977) is = 41 + 526. 1 1 1 b. /( ) = (41 + 526) = �20.5 2 + 526 � 4 = {20.5[ 2 ( 4) 2] + 2, 104 4 4 4 4 1 = (164 + 1, 776) = 41 + 444 4 c. The slope of the moving average is the same as the slope of the original function (because the original is linear). 56. a. The intercept is 290 and the slope is 40, so the line is = 40 + 290. 1 1 1 b. / ( ) = (40 + 290) = �20 2 + 290 � 4 = {20[ 2 ( 4) 2] + 1, 160} 4 4 4 4 1 = (160 + 840) = 40 + 210 4 c. The slope of the moving average is the same as the slope of the original function (because the original is linear). 1 1 4 0 (4 + ') = - 2 + ' . 2 1 4 2 1 4 2 4 = - [ ( ) 2] + ' '( ). = 4 + ' = 4 + ' 2 2 2

57. / ( ) =

1 1 3 0 3 = - . 5 1 3 3 ( ) 3 (1 ) = � �= 5 5 5

58. / ( ) =

59. The moving average "blurs" the effects of short-term oscillations in the price and shows the longer-


Solutions Section 14.3 term trend of the stock price. 60. Sales this month, though lower than last month's sales, were higher than they were a year ago. 61. They repeat every 6 months. 62. No. You would have earned exactly the same amount of money, since the average is computed by summing the monthly salaries and dividing by 12. 63. The area above the -axis equals the area below the -axis. Example: + = on [ 1, 1].

64. No. If / was larger than everywhere, then the area of the rectangle of height / and width ' would be larger than the definite integral of . 65. This need not be the case; for instance, the function ( ) = 2 on [0, 1] has average value 1 3, whereas the value midway between the maximum and minimum is 1 2.

66. Consider, for instance, the 12-month moving average of the temperature. This tends to be almost constant (ignoring global warming!). This moving average conveys no information about the changes of temperature between seasons, as would a shorter-term moving average. 67. (C). A shorter-term moving average most closely approximates the original function, since it averages the function over a shorter period, and continuous functions change by only a small amount over a small period. 68. Larger, since the moving average lags behind the value of the function


Solutions Section 14.4 Section 14.4

1. = 5 2, so / = 5 5 2 = 5 2. The consumers' surplus is 5 2

(10 2 5) =

0

5 2

0

(5 2 ) = [5 2]0

5 2

5 5 2 = 5� � � � (0) = $6.25. 2 2

2. = 100 , so / = 100 20 = 80. The consumers' surplus is 80 80 80 1 1 (100 20) = (80 ) = �80 2� = 80(80) (80) 2 (0) = $3,200. 0 2 2 0 0 3. = (100 ) 2 9, so / = (100 76) 2 9 = 64. The consumers' surplus is 64

0

(100 3 76) =

64

(24 3 ) = [24 2 3 2]064 0 = 24(64) 2(64) 3 2 (0) = $512.

4. = (10 ) 3 8, so / = (10 6) 3 8 = 8. The consumers' surplus is 8 8 8 3 3 (10 2 1 3 6) = (4 2 1 3) = �4 4 3� = 4(8) (8) 4 3 (0) = $8. 0 2 2 0 0

1 1 1 5. = ln( 500), so / = ln(1 5) = ln 5. The consumers' surplus is 2 2 2 (ln 5) 2

(500 2 100) = [ 250 2 100 ]0ln 5 = 250(1 5) 50 ln 5 ( 250) = $119.53.

0

6. = 10 ln(100 ), so / = 10 ln(100 50) = 10 ln 50. The consumers' surplus is 10 ln 50

0

(100 0.1 50) =

10 ln 50

(50 0.1 ) 0 = [50 10 0.1 ]010 ln 50 = 500 ln 50 10(50) ( 10) = $1,466.01.

7. / = 100 2(20) = 60; = 50 2. The consumers' surplus is 60 60 60 1 1 1 1 �50 20� = �30 � = �30 2� = 30(60) (60) 2 (0) = $900. 0 2 2 4 4 0 0 50 1 . The consumers' surplus is 3 3 20 20 20 50 1 20 1 20 1 � 10� = � � = � 2� 0 3 3 3 3 3 6 0 0 20 1 (20) (20) 2 (0) = $66.67. = 3 6

8. / = 50 3(10) = 20; =

9. / = 100 0.25(10) 2 = 75; = 2 100 . The consumers' surplus is

75 4 (2 100 10) = � (100 ) 3 2 10 � 0 3 0 4 4 = (100 75) 3 2 10(75) � (100) 3 2 0� = $416.67. 3 3 75

10. / = 20 0.05(5) 2 = 18.75; =

400 20 . The consumers' surplus is 18.75 1 ( 400 20 5) = � (400 20 ) 3 2 5 � (substitute = 400 20 ) 0 30

18.75

0


Solutions Section 14.4 1 1 3 2 = [400 20(18.75)] 5(18.75) � (400) 3 2 0� = $168.75. 30 30

11. / = 500 0.5 50; = 2 ln� 500 0.5 50

0

1 1 + �. The consumers' surplus is 500 10

500 1 1 + �ln� � 1� = 2 500 10 0

0.5 50

ln�

500 0.5 50

0.5 50 1 1 + � [ ]0500 500 10

1 1 ln� + � 500 0.5 + 50. 500 10 0 We evaluate the remaining integral using substitution (and the integral of ln we found by integration by 1 1 1 parts): Let = + ; = , = 500 . When = 0, = 0.1; when = 500 0.5 50, 500 10 500 = 0.5. = 2

2

500 0.5 50

ln�

0

0.5

1 1 + � = 500 10 0.1

500 ln

0.5

= 500[ ln ]0.1

= 500[ 0.5 ln( 0.5) 0.5 (0.1 ln(0.1) 0.1)] 289.769

The consumers' surplus is therefore 2( 289.769) 500 0.5 + 50 = $326.27.

12. / = 100 ; = 10 ln(100 ). The consumers' surplus is 2

= 100 , = , = ; when = 0, 100 2

2

[10 ln(100 ) 20] . Let 0 = 100; when = 100 2, = 2.

[10 ln(100 ) 20] = 0100 (10 ln 20)

0

100 2

2

= [10 ln 10 20 ]100 2

= [10 ln 30 ]100

= [10 2(2) 30 2 (1,000 ln 100 3,000)] = $1,679.06

13. = 2 5, so / = 20 2 5 = 5. The producers' surplus is 5

0

[20 (10 + 2 )] =

5

0

(10 2 ) = [10 2]05 = 50 25 (0) = $25.

14. = 100, so / = 200 100 = 100. The producers' surplus is 100 100 100 1 [200 (100 + )] = (100 ) = �100 2� 0 2 0 0 10,000 5,000 (0) $5,000. = =

15. = ( 2 5) 3, so / = (12 2 5) 3 = 1. The producers' surplus is 1 1 1 3 3 [12 (10 + 2 1 3)] = (2 2 1 3) = �2 4 3� = 2 (0) = $0.50. 0 2 2 0 0

16. = [( 100) 3] 2, so / = [(124 100) 3] 2 = 64. The producers' surplus is


64

[124 (100 + 3 1 2)] =

0

Solutions Section 14.4 64

0 = 1,536 1,024 (0) = $512.

(24 3 1 2) = [24 2 3 2]064

17. = 2 ln( 500), so / = 2 ln(1,000 500) = 2 ln 2. The producers' surplus is 2 ln 2

(1,000 500 0.5 ) = [1,000 1,000 0.5 ]02 ln 2

0

= 2,000 ln 2 2,000 ( 1,000) = 2,000 ln 2 1,000 = $386.29.

18. = 100 ln( 100), so / = 100 ln 20. The producers' surplus is 100 ln 20

[120 (100 + 0.01 )] =

100 ln 20

(20 0.01 ) = [20 100 0.01 ]0100 ln 20 0 = 2,000 ln 20 2,000 ( 100) = 2,000 ln 20 1900 = $4091.46.

0

19. / = 2(40) 50 = 30; = 2 + 25. The producers' surplus is 30 30 30 1 1 1 �40 � + 25�� = �15 � = �15 2� = 450 225 (0) = $225. 0 2 2 4 0 0

20. / = 4(1,000) 1,000 = 3,000; = 4 + 250. The producers' surplus is 3,000 3,000 1 1 �1,000 � + 250�� = �750 � 4 4 0 0 1 2 3,000 = �750 � = 2,250,000 1,125,000 (0) = $1,125,000. 0 8 21. / = 0.25(10) 2 10 = 15; =

4 + 40 = 2 + 10. The producers' surplus is 15 4 500 4 (10 2 + 10) = �10 ( + 10) 3 2� = 150 � 10 3 2� = $25.50. 0 3 3 3 0 15

22. / = 0.05(50) 2 20 = 105; =

20 + 400. The producers' surplus is 105 1 (50 20 + 400) = �50 (20 + 400) 3 2� 0 30 12,500 800 = 5,250 � � = $1350. 3 3

105

0

23. / = 500 0.05(10) 50 = 500 0.5 50; = 20 ln� 500 0.5 50

0

�10 20 ln�

1 1 + �� 500 10

1 1 + �. The producers' surplus is 500 10

500 0.5 50 1 1 + � + (20 + 1,000)� 0 500 10 1 1 (using the substitution = + as in Exercise 11) 500 10

= �10 (20 + 1,000) ln�

= 5,000 0.5 500 10,000 0.5 ln 0.5 + 10,000 0.5 � 1,000 ln� = $12,684.63.

24. / = 10( 0.5 1); = 10 ln�

1 + 1�. The producers' surplus is 10

1 � + 1,000� 10


10( 0.5 1)

0

10( 0.5 1) 1 + 1� + (10 + 100)� 0 10 0.5 0.5 0.5 0.5 = 50( 1) 100 ln + 100 (100) = $14.87.

10

(7 =

0

30,000 = [30,000 ]010 = $300,000

10

30,000 0.07(10 ) = �

0 5

(7 =

27. 6 7 =

1 + 1�� 10

= �5 (10 + 100) ln�

25. 6 7 =

26. 6 7 =

�5 10 ln�

Solutions Section 14.4

0

40,000 = [40,000 ]05 = $200,000 5

40,000 0.1(5 ) = �

0

10

0

(7 =

30,000 0.07(10 ) 10 � = $434,465.45 0 0.07

40,000 0.1(5 ) 5 � = $259,488.51 0 0.1

(30,000 + 1,000 ) = [30,000 + 500 2]010 = $350,000

10

(30,000 + 1,000 ) 0.07(10 )

0

= �

1,000 0.07(10 ) 10 1 (30,000 + 1,000 ) 0.07(10 ) � 0 0.07 0.07 2

= $498,496.61

(using integration by parts)

28. 6 7 =

5

0

(7 =

(40,000 + 2,000 ) = [40,000 + 1,000 2]05 = $225,000 5

(40,000 + 2,000 ) 0.1(5 ) = [ 10(40,000 + 2,000 ) 0.1(5 ) 200,000 0.1(5 )]05

0

= $289,232.76 29. 6 7 =

(using integration by parts)

10

30,000 0.05 = [600,000 0.05 ]010 = $389,232.76

0

(7 =

10

30,000 0.05 0.07(10 ) =

0

10

0

30,000 0.7 0.02

= [ 1,500,000 0.7 0.02 ]010 = $547,547.16

30. 6 7 =

5

0

(7 =

40,000 0.04 = [1,000,000 0.04 ]05 = $221,402.76 5

40,000 0.04 0.1(5 ) =

5

40,000 0.5 0.06 0 0 40,000 0.5 0.06 5 = � � = $284,879.01 0 0.06


Solutions Section 14.4

5

31. 6 7 =

87 =

0

20,000 = [20,000 ]05 = $100,000 5

20,000 0.08 = [ 250,000 0.08 ]05 = $82,419.99

0

10

32. 6 7 =

87 =

0

50,000 = [50,000 ]010 = $500,000

10

50,000 0.05 = [ 1,000,000 0.05 ]010 = $393,469.34

0 5

33. 6 7 =

0

87 =

(20,000 + 1,000 ) = [20,000 + 500 2]05 = $112,500 5

(20,000 + 1,000 ) 0.08 = [ (250,000 + 12,500 ) 0.08 156,250 0.08 ]05

0

= $92,037.48

(using integration by parts)

10

34. 6 7 =

(50,000 + 2,000 ) = [50,000 + 1,000 2]010 = $600,000

0

87 =

10

(50,000 + 2,000 ) 0.05 = [ (1,000,000 + 40,000 ) 0.05 800,000 0.05 ]010

0

= $465,632.55 35. 6 7 =

87 =

36. 6 7 =

87 =

(using integration by parts)

5

0

20,000 0.03 = � 5

0

20,000 0.03 5 � = $107,889.50 0 0.03

20,000 0.03 0.08 =

10

0

50,000 0.06 = �

10

0

5

0

20,000 0.05 = [ 400,000 0.05 ]05 = $88,479.69

50,000 0.06 10 � = $685,099.00 0 0.06

50,000 0.06 0.05 =

10

0

50,000 0.01 = [5,000,000 0.01 ]010 = $525,854.59

37. To find the equilibrium tuition set demand equal to supply: 20,000 2 = 7,500 + 0.5 , so / = $5,000. The equilibrium supply is thus / = 20,000 2(5,000) = 10,000. To find the consumers' surplus we solve for = 10,000 2 and compute 10,000 10,000 9 = �10,000 5,000� = �5,000 � 2 2 0 0 2 10,000 = �5,000 � = $25,000,000. 4 0 To find the producers' surplus we solve for = 2 15,000 and compute 89 =

10,000

0

[5,000 (2 15,000)] =

= [20,000 2]0

10,000

10,000

0

= $100,000,000.

(20,000 2 )


Solutions Section 14.4 The total social gain is $125 million. 38. To find the equilibrium price we need to set demand equal to supply. We first need to solve the demand equation for in terms of : 128 = 1. : So, 128 1 = 0.5 1 : 128 = 0.5 : 128 = 0.25 2 512 = 3 so / = 8¢ per serving. The equilibrium supply and demand is / = 0.5(8) 1 = 3. The consumers' surplus is 3 3 128 128 9 = 8� = � 8 � = 32 24 ( 128) = 72¢. � 2 0 + 1 0 ( + 1) To find the producer's surplus we solve for = 2 + 2 and compute 89 =

3

[8 (2 + 2)] =

0 The total social gain is 81¢.

3

0

(6 2 ) = [6 2]03 = 18 9 (0) = 9¢.

4

4 0.4 3 0.15 2 + 0.2 . 3 0

6.9333, or $6.9 billion

4

4 0.1 3 + 0.04 2 + 0.7 . 3 0

5.5733, or $5.6 billion

(0.4 2 0.3 + 0.2) = -

39. Total revenue = =

0 (rounded to two significant digits).

(0.1 2 + 0.08 + 0.7) = -

40. Total revenue = =

0 (rounded to two significant digits).

480 0.032 5 480 0.032 = � � 1 0.032 1 revenue of about $2,110 billion. 41. Total revenue =

5

5

1 of about $350 billion. 42. Total revenue =

70 0.070 = �

70 0.070 5 � 1 0.070

2,110 (to the nearest 10). This gives a total 350 (to the nearest 10). This gives a total revenue

4

(0.4 2 0.3 + 0.2) 0.04(4 ) 0 In the Numerical integration utility and grapher, enter(0.4x^2-0.3x+0.2)e^(0.04(4-x)) for ( ) and 0 and 4 for the limits of integration, and press "Integral" to obtain 7.220153442 $7.2 billion to two significant digits. 43. Total revenue =

4

(0.1 2 + 0.08 + 0.7) 0.03(4 ) 0 In the Numerical integration utility and grapher, enter(0.1x^2+0.08x+0.7)e^(0.03(4-x)) for ( ) and 0 and 4 for the limits of integration, and press "Integral" to obtain 5.840213932 $5.8 billion to two significant digits. 44. Total revenue =


45. Total revenue = = �

1

480 0.032 0.05(5 ) =

480 0.25 0.082 5 � 1 0.082

46. Total revenue = = �

Solutions Section 14.4

5

480 0.25 0.082

5

70 0.070 0.03(5 ) =

70 0.15 0.10 5 � 1 0.10

1

1,920 (to the nearest 10). This gives a total value of about $1,920 billion.

5

1

5

1

70 0.15 0.10

330 (to the nearest 10). This gives a total value of about $330 billion.

47. ( ) = 12 × 700 = $8,400 year (7 =

45

8,400 0.06(45 ) = [ 140,000 0.06(45 )]045 = $1,943,162.44

0

48. ( ) = 12 × 400 = $4,800 year (7 =

18

4,800 0.12(18 ) = [ 40,000 0.12(18 )]018 = $306,845.51

0

49. ( ) = 12 × 700 0.03 = 8,400 0.03 (7 =

45

45

8,400 0.03 0.06(45 ) =

8,400 2.7 0.03 0 0 = [ 280,000 2.7 0.03 ]045 = $3,086,245.73

50. ( ) = 12 × 400 0.02 = 4,800 0.02 (7 =

18

4,800 2.16 0.1 0 0 = [ 48,000 2.16 0.1 ]018 = $347,414.80

51. ( ) = 3,125. 8 7 =

30

0

3,125 0.04 = [ 78,125 0.04 ]030 = $54,594.20

52. ( ) = 0.07 × 50,000 = 3,500. 8 7 = 53. ( ) = 100,000 + 5,000 87 =

18

4,800 0.02 0.12(18 ) =

20

0

3,500 0.06 = �

3,500 0.06 20 � = $40,763.67 0 0.06

20

0

(100,000 + 5,000 ) 0.05 = [ 20(100,000 + 5,000 ) 0.05 400(5,000) 0.05 ]020

= $1,792,723.35

54. ( ) = 30,000 + 1,500 87 =

30

0

(30,000 + 1,500 ) 0.06 = �

= $641,168.52

55. total 56. total

(30,000 + 1,500 ) 0.06 1,500 0.06 30 � 0 0.06 0.06 2


1 57. / = ' 4 (' ) /, = 4

Solutions Section 14.4

' 1 2 1 � = � / � � (' ) / 0 4 4 24 0 ' 1 (' 4 (' 4 (' 4 = /) /) 2 / /) (0) 4 24 1 1 (' 4 (' 4 = /) [2' (' 4 /) 24 /] = /) 2 24 24

9 =

58. / = ' + 4 /, =

1 ( ') 4

1 1 2 ' + ( ')� = � + � / 4 24 4 0 1 ' (' + 4 (' + 4 = / /) /) 2 + (' + 4 /) (0) 24 4 1 1 (' + 4 (' + 4 = /) [24 / (' + 4 /) + 2'] = /) 2 24 24

89 =

+

0

/ �

59. She is correct, provided that there is a positive rate of return, in which case the future value (which includes interest) is greater than the total value (which does not). 60. He is wrong. If you invested the present value now, you would have, at the end of the investment period, the same amount as the future value. On the other hand, if you invested the total value now at the same rate of return, you would have more than the future value (since the future value is obtained by investing parts of the total value over time). Therefore, the total value is always larger than the present value, regardless of the rate of return (as long as the rate of return is positive). 61. 8 7 < 6 7 < ( 7

62. a. ( ) ( ) # ( ) # ( ) ( ), since is negative, and ' is positive. b. The inequality in (a) becomes the inequality in Exercise 61 when we integrate.


Solutions Section 14.5 Section 14.5

=

1 diverges 1.

1

=

0

0.5 =

lim [ ]0 = ;lim ( + 1) = 1;

=

lim

2 converges

3.

1 lim [ 2]1 = ;lim � 12 < 2 12 � = =; 2

=

lim

0 converges 2.

lim

2

0.5 =

lim � 2 0.5 � 2 = ;lim ( 2 0.5 + 2 ) = 2 ;

1 1 = lim = lim � 2 0.5�1 = ;lim ( 2< 0.5 + 2) = 2; 1.5 1.5 1 1 converges

4.

2

=

converges

5.

1

1

1 3 diverges

6.

2

lim

=

2 lim [ ] = ;lim ( 2 ) = 2;

= 1

lim

1

1 3

=

1 3 lim � 2 3� = ;lim � 32 32 < 2 3� = =;

2

2 1 1 1 1 1 2 lim = lim � 1� = ;lim � + = �= ; 2 2 2 < 2 converges 2

7.

0

=

diverges 8.

0

lim

=

0 lim [ ] = ;lim ( 1 + ) = =;

1 2 2 6 lim � 2 6 6 � 0 6 36 216 1 2 2 6 2 2 1 < 6 ; = lim � < 2 6 + = �= 6 36 216 216 216 108 converges 9.

2 6 =

0

10.

0 =

lim

0

(2 4) =

lim

5

0

(2 4) =

lim [ (2 4) 2 ]0

lim � (2< 4) 2 (4 2)� = 2;

converges

2

1 3 0 converges

11.

2 6 =

=

lim

5

2

1 3 0+

=

lim �3 2 3� = ;lim 0 + (3 × 5 2 3 3" 2 3) = 3 × 5 2 3;

0+

5


Solutions Section 14.5 2 1 1 1 1 2 12. = lim = lim � 1� = ;lim 0 + � + � = =; 2 2 + + 2 " 0 0 0 diverges 2

2 3 3 3 2 lim = lim [ 3( + 1) 1] = ;lim 1 + � 1 + = � = =; 2 2 + + "+ 1 1 1 ( + 1) 1 ( + 1) diverges 2

13.

2 3 3 2 = lim = lim [6( + 1) 1 2] 1 2 1 2 1 + 1 + ( + 1) 1 ( + 1) = lim 1 + (6 3 6 " + 1) = 6 3; converges 2

14.

0 2 3 3 3 3 lim lim lim = + + 2 2 2 2 + + 1 1 1 1 0 1 1 1 1 3 3 Now, = ln || 2 1|| + by substitution, so 2 1 2 2

15.

0 3 3 | 2 ||. lim ln 1 = | 2 1 + 1 + 2 1 3 = lim > ln ||" 2 1||? = =. 2 1 + 0

lim

Since this one part diverges, the whole integral diverges. (In fact, all three parts diverge.) 2

3

1 3 1

=

lim

3

+

lim

2

3

=

2 9 9 lim - 2 3. + lim - 2 3. 0 2 1 0+ 2

1 3 1 3 0 0+ 1 9 2 3 9 9 2 3 9 2 3 9 = lim > " ? + lim > 2 " ? = (2 2 3 1); + 0 2 2 2 2 2 0 converges

16.

2 1 1 1 = lim + lim 1 5 1 5 1 5 + 1 1 ( + 1) 2 ( + 1) 2 ( + 1) 2 5 5 = lim - ( + 1) 4 5. + lim - ( + 1) 4 5. 1 4 2 1 + 4 5 5 5 5 5 = lim - (" + 1) 4 5 . + lim - 3 4 5 (" + 1) 4 5. = (3 4 5 1); + 1 4 4 4 4 4 1 converges 2

17.

18.

2

2

2 =

4 2

lim @2

2 + 2 +

converges

2

0

lim

+

lim

2

2 2 + 0 4 2 4 2 0 4 2A + lim @2 4 2A0 (use the substitution 2

lim B4 2 4 " 2C +

=

19.

=

lim B2 4 " 2 4C = 4 4 = 0;

2

= 4 2)

0 2 2 2 lim lim = + 2 1 2 1 2 1 + 1 1 0 1 0 = lim �ln | 2 1||� + lim �ln | 2 1||�0 \quad (use the substitution = 2 1) 1

1 +

1


lim � ln |" 1|� +

=

| 2

1 +

lim ln |" 2 1| = = =;

1

Solutions Section 14.5

diverges (Note that the infinities don't cancel. For convergence we need each part of the integral to converge on its own.) 20.

0 2 2 2 2 2 lim lim lim = + + 2 2 2 2 + + 1 1 1 1 0 1 1 1 1 0 2 2 2 = lim �ln || 2 1|� | + lim �ln || 1|� | 0 + lim �ln || 1|� | 2

=

(use the substitution = 1) lim � ln |" 2 1|� + lim ln |" 2 1| +

1 +

diverges 21.

1 2

1 +

1

2

=

0

lim

2

+

1+

lim �ln 3 ln |" 2 1|� = = = + =;

1+

lim

0 0 2 1 1 2 lim . + lim - . - 2 2 0 2

=

2

(use the substitution = ) 2 2 1 1 1 1 1 1 lim = + ? + lim > + ? = + = 0; > 2 2 2 2 2 2 converges 22.

2

1 =

0

lim

2

1 +

lim

0 0 2 1 1 1 1 2 lim . + lim .0 - 2 - 2 2

=

2

1

(use the substitution = 1 ) 2 2 1 1 1 1 1 1 lim = + 1 ? + lim > 1 + ? = + = 0; > 2 2 2 2 2 2 converges

1 2 1 1 1 = lim + lim + 1 ln ln ln 0 0 1 2 1 1 + lim 02 + lim 1 + 0 2 ln ln The first part of the integral is 1 2 1 1 2 lim = lim [ln | ln |] (use the substitution = ln ) + ln 0+ 0 = lim [ln | ln(1 2)| ln | ln "|] = =

23.

0+

Without checking the remaining parts of the integral we can say that the whole integral diverges. 24.

0 = =

ln =

lim

1

ln +

lim

ln

0+ 1 1 lim [ ln ] + lim [ ln ]1 = lim ( 1 " ln " + 0+ 0+

lim <(ln < 1) = =;

") +

lim (< ln < < + 1)

diverges (Note: It is not necessary to know that lim " ln " = 0 to tell that the whole integral diverges, just that lim <(ln < 1) = =.)

0+


Solutions Section 14.5 2 2 2 2 2 25. lim lim lim = + + 2 2 2 2 + 1 1 0 1 0 1 1 2 1 2 = lim �ln | 2 1||�0 + lim �ln | 2 1||� + lim �ln | 2 1||�2

1

lim ln |" 1| +

=

1

| 2

= = + = + =; diverges 26.

1+

lim (ln 3 ln |" 2 1|) +

1+

lim (ln |< 2 1| ln 3)

0 2 2 2 2 2 lim lim lim = + + 2 2 2 2 1 1 1 1 + 1 2 1 0 2 lim �ln || 2 1||� + lim �ln || 2 1||� 2 + lim �ln || 2 1||� = 0

=

1

lim (ln 3 ln |< 1||) + 2

diverges

1 +

lim (ln |" 2 1|| ln 3) +

1

lim ( ln |" 2 1||) = = = + =;

1 +

27. 0.9, 0.99, 0.999, … Converges to 1 28. 0.8862, 0.8862, 0.8862, … Converges to 0.8862 29. 7.602, 95.38, 993.1, … Diverges 30. 1.368, 1.800, 1.937, 1.980, 1.994, 1.998, 1.999, 2.000, … Converges to 2.000 31. 1.368, 1.800, 1.937, 1.980, 1.994, 1.998, 1.999, 2.000, … Converges to 2.000 32. 2.303, 4.605, 6.908, 9.210, … Diverges to = 33. 9.000, 99.00, 999.0, … Diverges to =

34. 49.875, 4999.875, 499999.875, … Diverges to =

1.1 0.26 =

1.1 0.26 0 0 1.1 0.26 1.1 0.26 1.1 ( = lim - 1). = .0 = ; lim - 0.26 0.26 0.26

35. Total revenue =

lim

2.7 0.29 =

2.7 0.29 0 0 2.7 0.29 2.7 0.29 2.7 ( = lim - 1). = .0 = ; lim - 0.29 0.29 0.29

36. Total revenue =

lim

37. Annual sales = 9( ) = 415(0.96) billion cigarettes per year. Total sales = =

0

415(0.96) =

lim �

415

ln 0.96

=

415 ln 0.96

lim

0

(0.96) �

0

$4.23 billion

415(0.96)

= ;lim �

10,200 billion cigarettes

$9.31 billion

415 415 (0.96) � ln 0.96 ln 0.96


Solutions Section 14.5 38. Annual sales = 9( ) = 5,000 0.05 . Total sales = =

5,000 0.05 =

0

lim

0

5,000 0.05

lim � 100,000 0.05 �0 = ;lim � 100,000 0.05 + 100, 000�

= 100,000 copies 39. Annual sales = 9( ) = 200(0.90) . Total sales = =

200(0.90) =

0

lim �

200

ln 0.90

=

200 ln 0.90

0

(0.90) �

1,900

lim

0

200(0.90)

= ;lim �

200 200 (0.90) � ln 0.90 ln 0.90

No, you will not sell more than about 2,000 of them.

40. With annual sales of 9( ) = 200(0.90) and revenue per T-shirt of 8 ( ) = 10 + , the total revenue will be

0

200(0.90) (10 + ) =

lim

0

200(0.90) (10 + )

200 200 (0.90) (10 + ) (0.90) � (by integration by parts) 2 0 ln 0.90 (ln 0.90) 2, 000 200 200 200 (0.90) (10 + <) (0.90) = lim � + � 2 ln 0.90 ln 0.90 (ln 0.90) (ln 0.90) 2 2000 200 $37,000. = + ln 0.90 (ln 0.90) 2

=

lim �

41. Total emissions from 2040 on are given by

20

D( ) =

lim

20

D( ) .

Model 1: The integral is 60 0.027 lim - .20 1,295 thousand metric tons, or about 1.3 million metric tons. 0.027 Model 2: The integral is 680 0.02 lim .20 which diverges to =, thus predicting that total carbon emissions will increase 0.02 without bound.

42. Total emissions (above the baseline) from 2040 on are given by Model 1: The integral is 150 0.067 lim - .20 586 metric tons. 0.067 Model 2: The integral is 370,000 1.9 lim .20 657 metric tons. 1.9 43. Total revenue = =

0

lim � 10(11 +

(11 + 88.3) 0.1 =

lim

20

( ) =

lim

20

(11 + 88.3) 0.1 0 88.3) 0.1 1,100 0.1 �0 (using integration by parts)

( ) .


= 883 + 1,100

44. Total revenue = =

Solutions Section 14.5 1,980 billion (to the nearest $10 billion)

80(9 5) 0.2 =

80(9 5) 0.2 0 (using integration by parts) 5) 0.2 18,000 0.2 �0

0

lim � 400(9

lim

= 2,000 + 18,000 = 16,000 trillion zonars, so the total revenue is Ż16,000 trillion.

45. As 2020 is represented by 0 = 15, the total value is

(0.18 2 + 0.558 + 117) 0.03( 15) =

lim

(0.18 2 + 0.558 + 117) 0.03( 15) 15 15 Tech formula: (0.18x^2 + 0.558x + 117)*e^(-0.03(x-15))

(0.18 2 + 0.558 + 117) 0.03( 15) 15 where < = 50, 100, 1,000, 10,000, …. This gives integrals of approximately 6,662, 16,135, 25,482, 25,482, …, so we conclude that Following the technology note in the textbook, we compute

(0.18 2 + 0.558 + 117) 0.03( 15) 0 or about $25,500 billion

25,482,

46. As 2020 is represented by 0 = 15, the total remittance is

(241.2 ln 213.5) 0.03( 15) =

lim

(241.2 ln 213.5) 0.03( 15) 15 15 Tech formula: (241.2*ln(x) - 213.5)*e^(-0.03(x-15))

(241.2 ln 213.5) 0.03( 15) where 15 < = 50, 100, 1,000, 10,000, …. This gives integrals of approximately 12,764, 20,041, 22,541, 22,541, …, so we conclude that Following the technology note in the textbook, we compute

(241.2 ln 213.5) 0.03( 15) 0 or about $22,500 billion. 47.

0

E( ) =

0

lim

=

80(7) 20 + 7

0

80(7) 20 + 7

80 ln(20 + 7 )� (use the substitution = 20 + 7 ) 0 ln 7 80 80 ln(20 + 7 ) ln(21)� = = = lim � ln 7 ln 7

lim �

=

0

22,541,

E( ) diverges, indicating that there is no bound to the expected total future online sales of mousse. 0

E( )

= =

80(7) 20 + 7 0

80(7) 20 + 7 lim

0


Solutions Section 14.5 0 80 lim � ln(20 + 7 )� = ln 7 80 80 lim � ln(21) ln(20 + 7 )� = ln 7 ln 7 80 80 ln(21) ln(20) 2.006 = ln 7 ln 7

0

E( ) converges to approximately 2.006, indicating that total online sales of mousse prior to the current year amounted to approximately 2 million gallons. 48.

50 2 1 2 1 0 1+ 50 2 1 = lim 2 1 0 1+

( ) =

0

lim �25 ln(1 + 2 1)�0

=

(use the substitution = 1 + 2 1)

lim �25 ln(1 + 2 1) 25 ln(1 + 1)� = =

=

0

( ) diverges, indicating that there is no bound to the expected total future sales of mousse. 0

( )

50 2 1 2 1 1 + 0 50 2 1 lim = 2 1 1+ = = =

0

0

0 lim �25 ln(1 + 2 1)�

lim �25 ln(1 + 1) 25 ln(1 + 2 1)� = 25 ln(1 + 1)

7.832

( ) converges to approximately 7.832, indicating that total past sales of mousse amounted to approximately 7.832 gallons. 49. 1

50. 0.5

51. 0.1587

52. 0.8413

53. The value per bottle is 8 ( ) = 85 0.4 . The annual sales rate is F( ) = 500 . The annual income is ( ) = 8 ( )F( ) = 42,500 0.6 . The total income is

0

42,500 0.6 = =

lim

lim

42,500 0.6

0 0.6 � 70,833 �0 =

;lim � 70,833 0.6 + 70,833� = $70,833.

54. The number of bottles of good wine you will sell per year will be F( ) = 400 0.6 . At $50 per bottle, your annual net income will be ( ) = 20,000 0.6 . Your total income is


0

20,000 0.6 =

lim

0

Solutions Section 14.5

20,000 0.6

lim � 33,333 0.6 �0 = ;lim � 33,333 0.6 + 33,333� = $33,333.

=

55. a.

1 1 5 lim 5 = 1.081 1.081 0.2 5.69975 0.2 5.69975 = lim � 2.1665 0.081�0.2 = ;lim � 2.166< 0.081 + 2.166(0.2) 0.081�

meteors on average 1 1 1 1 b. 5 lim 5 = 1.081 1.081 + 5.69975 5.69975 0 0 1 = lim � 2.1665 0.081� = ;lim 0 + � 2.166 + 2.166" 0.081� = =;

2.468

0+

the integral diverges. We can interpret this as saying that the number of impacts by meteors smaller than 1 megaton is very large. (This makes sense because, for example, this number includes meteors no larger than a grain of dust.)

56. a. The product %(5)G5 E(5 + G5) E(5) approximates the (annual) number of meteors between 5 and 5 + G5. If we multiply this by 5, we get an approximation of the total energy released by these impacts. Thus, 5%(5)G5 approximates the total energy released by impacts of between 5 and 5 + G5 megatons. Finally, summing as 5 ranges from to ' and taking the limit of these Riemann sums gives us the integral 1

0

5%(5) 5, computing the total energy released by meteors with energies between and '

1 1 1 5 lim 5 = 0.081 0.081 + 0 0 5.69975 5.69975 1 lim �0.19095 0.919� = ;lim 0 + �0.1909 0.1909" 0.919� = 0.1909 megatons

megatons. b.

1

5%(5) 5 =

=

0+

This is the total energy released annually by meteors with energies between 0 and 1 megatons. 1 c. 5%(5) 5 = lim 5 0.081 1 1 5.69975 = lim �0.19095 0.919�1 = ;lim �0.1909< 0.919 0.1909� = =;

the integral diverges. Assuming the model is correct for large meteors, the average amount of energy released can be expected to be very large. Of course, the model cannot be expected to hold for arbitrarily large meteors, since the universe is finite, so there is an upper limit to the size of meteors. 57. a. H(1) = =

H(2) =

0

=

lim

0

lim � �0 = ;lim � + 1� = 1

lim

0 = lim � �0 = ;lim � < + 1� = b. H(% + 1) = = lim 0 0 % 1 � = lim � < + = lim �� �0 + 0 0

=

1 0� + %

c. If % is a positive integer, then applying part (b) several times, we get

0

= %H(%)

1


H(%) = (% 1)H(% 1) = (% 1)(% 2)H(% 2) = dots = (% 1)(% 2) $ $ $ 1H(1) = (% 1)!

Solutions Section 14.5

by part (a).

58. a. If ( ) = 1, then

1 lim � � 0 (when integrating with respect to , is treated as a constant) 1 1 1 . = lim � + � =

( ( ) =

0

If ( ) = , then ( ( ) = =

b.

( ( ) = =

c.

( ( ) = =

=

0

lim �

0

lim �

0

lim �

0

=

lim

0

1 % 1 %(% 1) 2 %! %! 2 dots + 1 � = + 1 0 3

=

lim

=

1 1 1 1 1 1 2 � = ;lim � < 2 + 2 � = 2 . 0

0

=

lim

lim

0

( )

1 1 ( ) � = 0

59. The integrand is neither continuous nor piecewise continuous on the interval [ 1, 1], so the FTC does not apply (the integral is imporper). 60. = is not a number.

61. Yes; the integrals converge to 0, and the FTC also gives 0. 62. No; the FTC gives zero, whereas the integrals diverge.

63. a. Not improper. | | is not defined at zero, but lim | | = 1 and lim | | = 1. Since these 0+

0

limits are finite, the integrand is piecewise continuous on [ 1, 1] and so the integral is not improper. b. Improper, since 1 3 has infinite left and right limits at 0. c. Improper, since ( 2) ( 2 4 4) = 1 ( 2), which has an infinite left limit at 2.

64. a. Not improper. | 1| ( 1) is not defined at 1, but lim | 1| ( 1) = 1. Since this limit is 1


Solutions Section 14.5 finite, the integrand is piecewise continuous on [ 1, 1] and so the integral is not improper. b. Improper, since 1 2 3 has an infinite right limit at 0. c. Not improper, since ( 2 4 4) ( 2) = 2, which has a finite left limit at 2. 65. In all cases you need to rewrite the improper integral as a limit and use technology to evaluate the integral of which you are taking the limit. Evaluate for several values of the endpoint approaching the limit. In the case of an integral in which one of the limits of integration is infinite, you may have to instruct the calculator or computer to use more subdivisions as you approach =. 66. The Riemann sums approach 0 as < gets large even though the improper integral is nonzero. The 2

reason for this is that the graph of + = ( 10) is very close to zero except around = 10, where it has a 10,000

2

( 10) using a partition with

0 500 subdivisions, we will miss the small region where the function is non-zero, since the partition widths are much larger than the interval where the function is significantly larger than zero.

maximum of 1. So, if we compute the Riemann sum for, say,

67. Answers will vary. 68. Answers will vary.


Solutions Section 14.6 Section 14.6 1. + = 2. + =

B 2 +

3 2 3 2 + + 3 3

1 � + 3� = ln | | + 3 +

3. + + = ; 4.

C =

+2 2 + + = ; = + 2 2

1 1 1 1 + = ; + = ; ln ||+|| = ln || || + ; ||+|| = ln | ||+ = || ||; + = 3 (where 3 = ± ) + +

2 2 2 1 1 2 + = ; + = ; ln ||+|| = + ; ||+|| = 2+ = 2; + = 3 2 (where + + 2 3 = ± )

5.

3 3 3 1 1 3 + = 2 ; + = 2 ; ln ||+|| = + ; ||+|| = 3+ = 3; + = 3 3 (where + + 3 3 = ± )

6.

( + 1) 2 + 1 1 1 1 2 + ( 1) ; + ( 1) ; ( 1) ; = + = + = + + = + 2 + 2 2 +2 2 += ( + 1) 2 + 7.

8. + 2 + = 9. + + =

+3 1 1 ; + 2 + = ; = ln || + 1|| + ; + = (3 ln || + 1|| + ) 1 3 + 1 + 1 3

+2 1 ln ln ; + + = ; = (ln ) 2 + (substitute 2 2

10. + + = ln ; + 2 = 2 ln

= ln ); + 2 = (ln ) 2 +

+2 2 2 + + = ln ; ln = + (use integration by parts); 2 2 4

2 + ; + = ± 2 ln 2 2 + 2

4 4 2 + ; 1 = 0 0 + ; = 1; + = 2 + 1 4 4

11. + =

( 3 2 ) =

12. + =

(2 ) = 2 + + ; 0 = 0 + 1 + ; = 1; + = 2 + 1

13. + 2 + = 2 ; + = ( 3 + 8) 1 3

+3 3 + 2 + = 2 ; = + ; + 3 = 3 + ; 8 = 0 + ; = 8; + 3 = 3 + 8; 3 3


Solutions Section 14.6 1 1 1 1 1 1 1 1 1+ 14. 2 + = 2 ; + = ; = + ; 2 = 1 + ; = 1; = 1 = ; 2 2 + + + + += 1+ 1 1 1 1 | | | | + = ; + = ; ln |+|| = ln | || + ; |+|| = ln | |+ = ln | | = | ||; + = 3 ; + + 2 = 3 × 1; 3 = 2; + = 2 15.

1 1 1 1 1 1 + = 2 ; + = ; ln ||+|| = + ; 0 = 1 + ; = 1; ln ||+|| = 1 ; ||+|| = 1 1 ; 2 + + + = 1 1 (note that +(1) = 1 > 0) 16.

17.

1 1 2 2 + = ; + = ; ln |+ + 1| = ; + ; ln 1 = 0 + ; = 0; ln |+ + 1| = + + 1 ++ 1 2 2

||+ + 1|| = 2; + + 1 = 2 (note that +(0) + 1 = 1 > 0); + = 2 1 2

2

2

1 1 1 1 + = ; + = ; ln |+ + 1| = ln | | + ; ln 3 = ln 1 + ; = ln 3; + + 1 ++ 1 ln ||+ + 1|| = ln || || + ln 3 = ln ||3 ||; ||+ + 1|| = ||3 ||; + + 1 = 3 (otherwise +(1) = 2, not 2); + = 3 1 18.

1 1 1 1 + = 2 ; + = ; = ln( 2 + 1) + ; 1 = 0 + ; = 1; 2 2 2 + 2 + + 1 + + 1 1 1 2 2 = ln( + 1) + 1; + = + 2 ln( 2 + 1) + 2 19.

1 1 1 + = ; + = ; ln |+|| = + (use the substitution + ( 2 + 1) 2 + 2( 2 + 1) ( 2 + 1) 2 1 1 1 1 2 2 [2( 2+ 1)] ; + (note that = 2 + 1); 0 = + ; = ; ln |+| = + = = 2 2 2( 2 + 1) 2 2( 2 + 1) +(0) = 1 > 0) 20.

21. With ( )\displaystyle {}= monthly sales after months,

= 0.05 ; = 1,000 when = 0.

1 1 = 0.05 ; = ( 0.05) ; ln | || = 0.05 + ; | || = 0.05 + = 0.05 ; = 3 0.05 ; 1,000 = 3 × 1; 3 = 1,000; = 1,000 0.05 quarts per month. 22. With ( ) = monthly profit after months,

1 = 0.1 ; = 15,000 when = 0. = 0.1 ;

1 = 0.1 ; ln | | = 0.1 + ; | | = 0.1 + = 0.1 ; = 3 0.1 ; 15,000 = 3 × 1; 3 = 15,000; = 15,000 0.1 dollars per month. 23. a. By Example 4, I( ) = 6 + (60 6 ) = 75 + (200 75) 0.05 = 75 + 125 0.05 . b. The coffee will have cooled to 80°F when 80 = 75 + 125 0.05 125 0.05 = 5 0.05 = 0.04


0.05 = ln 0.04 = (ln 0.04) 0.05

Solutions Section 14.6

64.4 minutes

24. a. By Example 4, I( ) = 6 + (60 6 ) = 60 + (210 60) 0.08 = 60 + 150 0.08 . b. The coffee will have cooled to 70° when 70 = 60 + 150 0.08 150 0.08 = 10 0.08 = 1 15 0.08 = ln 1 15 = ln 15 = (ln 15) 0.08 33.85 minutes.

25. By Example 4, I( ) = 6 + (60 6 ) = 75 + (190 75) = 75 + 115 . I(10) = 150, so 150 = 75 + 115 10 ; 1 150 75 5 = ln� � 0.04274. 10 115 I( ) = 75 + 115 0.04274 degrees Fahrenheit after minutes 26. By the note after Example 4, I( ) = 6 + (60 6 ) = 350 + (20 350) = 350 330 . I(15) = 80, so 80 = 350 330 15 ; 1 350 80 5 = ln� � 0.01338. 15 330 I( ) = 350 330 0.01338 degrees Fahrenheit after minutes 27. With 9( ) = total sales after months,

9 = 0.1(100,000 9); 9(0) = 0.

28. With 9( ) = total sales after months,

9 = 0.05(100,000 9); 9(0) = 5,000.

1 1 9 = 0.1 ; 9 = 0.1 ; ln(100,000 9) = 0.1 + ; 100,000 9 100,000 9 9( ) = 100,000 3 0.1 . 0 = 100,000 3; 3 = 100,000. 9( ) = 100,000 100,000 0.1 = 100,000(1 0.1 ) monitors after months. 1 1 9 = 0.05 ; 9 = 0.05 ; ln(100,000 9) = 0.05 + ; 100,000 9 100,000 9 9( ) = 100,000 3 0.05 . 5,000 = 100,000 3; 3 = 95,000. 9( ) = 100,000 95,000 0.05 = 5,000 + 95,000(1 0.05 ) monitors after months.

1 1.5 1 1.5 = = 0.05 1.5; = � 0.05 + � ; � 0.05 + � ; ln = 0.05 + 1.5 ln + ; = 3 0.05 1.5. (20) = 20, so 20 = 3 1(20) 1.5; 3 = (20) 0.5 = 0.6078 0.05 1.5 29.

1 0.5 1 0.5 = = 0.02 0.5; = � 0.02 + � ; � 0.02 + � ; ln = 0.02 + 0.5 ln + ; = 3 0.02 0.5. (30) = 30, so 30 = 3 0.6(30) 0.5; 3 = 0.6(30) 0.5 = 9.98 0.02 0.5 30.

31. = 1 and ( ) = + =

( ) =

= 1 = ( + )

0.6078.

9.98.


Solutions Section 14.6 Initial condition: 1 = 0(0 + ); 1 = . So, the solution is + = ( + 1) 32. = 1 and ( ) = 2 + =

( ) = 2 = = ( + ) Initial condition: 2 = (1 + ); = 1. So, the solution is + = ( + 1) or 2 + 33. Rewrite the given equation in the form 1 = and ( ) = . 2 + =

( ) = 2

+ + + =

( ) by dividing both sides by 2:

+ 1 += . 2

2 = 2� 2 2 4 2 + � (using integration by parts)

Initial condition: 1 = ( 4 + ); = 5. So, the solution is + = 2� 2 2 4 2 + 5�. 34. Rewrite the given equation in the form =

1 1 and ( ) = . 2 2

+ + + =

( ) by dividing both sides by 2:

+ 1 1 + += . 2 2

1 1 1 ( ) = 2 > 2? = 2 2 = 2�2 2 4 2 + � 2 2 2 (using integration by parts) Initial condition: 1 = 12 ( 4 + ); = 2. So, the solution is + = 12 2�2 2 4 2 + 2� + =

|| 1|| K K 1 + || 1|| . . 1 and ( ) 5 + K = 7 ( ) with J = = 1 gives + K = 5 1 + = = 1 1 | | | | 1 1 | | | | K = ( ) = 5 1 + = 5 � 1 + ( ) + � (using integration 1 1 by parts) Initial condition: No current flowing at time = 0: K = 0 when = 0; 0 = 5([1 + ( 1)](1 ) + ); = 0. | 1|| So, the solution is K = 5 ( ) 1 + 1 Technology formula for graph: 5*e^(-x)*(e^x-e)*(1+abs(x-1)/(x-1)) 35. J

| 2| K K . = 5 and + K = 7 ( ) with J = 1 and = 5 gives + 5K = 5 1 + 2 || 2|| ( ) = 5 1 + . 2 || 2|| 5 || 2|| 5 10 5 K = ( ) = 5 5 1 + 5 = + �1 �� � + (using 5 2 2 integration by parts) 36. J


Solutions Section 14.6 Initial condition: No current flowing at time = 0: K = 0 when = 0. 0 = 5 ; = 0. So, the solution is | 2| 5 10 | 2| 5 5 K = 5 5 1 + 1 ( 10) = + � � 5 2 2 Technology formula for graph: e^(-5x)*(e^(5x)-e^10)*(1+abs(x-2)/(x-2))

= 5[ ( ) 9( )] = 5(20,000 1,000 ) ln(20,000 1,000 ) 1 1 b. = 5 ; = 5 ; = 5 + ; 20,000 1,000 20,000 1,000 1,000 ( ) = 20 3 1,000 c. (0) = 10 and (1) = 12, so 10 = 20 3; 3 = 10; 12 = 20 10 1,000 ; 20 12 1,0005 = ln� � 0.2231; ( ) = 20 10 0.2231 dollars after months 10 37. a.

= 5[ ( ) 9( )] = 5(2,000 2,000 ) ln(2,000 2,000 ) 1 1 b. = 5 ; = 5 ; = 5 + ; 2,000 2,000 2,000 2,000 2,000 ( ) = 1 3 2,000 c. (0) = 5 and (1) = 3, so 5 = 1 3; 3 = 4; 3 = 1 + 4 ; 2,0005 = ln(2 4) 0.6931; ( ) = 1 + 4 0.6931 dollars after years 38. a.

+ J 2 J , then and = + ( + ) 2 J J J 2 +(J +) = � also, so this + satisfies the differential equation. �� �= + ) + ( + ) 2 39. If + =

+ 1 1 1 + = ; + = ; [ln + ln(J +)] = + ; ln� � = J + ; +(J +) +(J +) J J + + J J = ; + = = J + 1 + +

40.

41. = 1 4 and J = 2, so 9 =

2

0.5 +

for some . 9 = 0.001 when = 0, so 0.001 =

2 1,999 0.001 1 = .9= . Graph: = 2 0.001 1,999 0.5 + 1 1,999

2 ; 1+


Solutions Section 14.6

It will take about 30 months to saturate the market. 42. = 1 10 and J = 20, so 3 =

20

2 +

for some . 3 = 0.02 when = 0, so 0.02 =

20 999 0.02 1 = .3= . Graph: = 20 0.02 999 2 + 1 999

20 ; 1+

3 is growing fastest at about 3.5 months. 20 million people will eventually be affected. 43. a.

1 1 + = ; + = ( ) ; ln[ln(+ ')] = + [use the substitution + ln(+ ') + ln(+ ')

= ln(+ ')]; + = '

b. 5 = 10 , 3 = ln 0.5

, for some constant 3

44. 15 = 10 , 3 = ln 1.5

0.69315; + = 10 0.69315

Graph:

0.40547; + = 10 0.40547 . Graph:

From the graph, we see that the tumor size decreases to ' = 10 cm 3, whereas it increased to this size in Exercise 43. 45. A general solution gives all possible solutions to the equation, using at least one arbitrary constant. A particular solution is one specific function that satisfies the equation. We obtain a particular solution by substituting specific values for any arbitrary constants in the general solution.


Solutions Section 14.6 46. Solving a first-order differential equation involves one integration; hence there will be exactly one arbitrary constant. 47. Example:

48.

2+ = 1 has general solution + = 2

+ = + +

49. Differentiate to get the differential equation

1 2 + 2

+ (integrate twice).

+ = 4 + 3.

50. You will have one of two differential equations: $ = ( ) or $ = )( ). In either case, the differential equation is separable.


Solutions Chapter 14 Review Chapter 14 Review 1.

+ 2 + 2 − + −

2

0

2

( 2 + 2) = ( 2 + 2) 2 + 2 + = ( 2 2 + 4) + 2.

+ 2 − 2 1 + −

2 0

3 + 1

13 3 + 1 1 3 + 1 9

1 3 + 1 27

1 1 1 ( 2 ) 3 + 1 = ( 2 ) 3 + 1 (2 1) 3 + 1 2 3 + 1 + 3 9 27 1 1 ( 9 2 + 9 6 + 3 2) 3 + 1 + = ( 9 2 + 3 + 1) 3 + 1 + = 27 27 3.

+ ln(2 ) −

2

1

3 3

1 ( ln 5)

1 1 2 1 1 2 ln(2 ) = 3 ln(2 ) = 3 ln(2 ) 3 + 3 3 3 9 4.

+ −

log5

1

1 log = log5 = log5 + 5 ln 5 ln 5


5.

+ − +

2 2 0

Solutions Chapter 14 Review

||2 + 1||

1 (2 + 4

1 (2 + 24

1)||2 + 1||

1) 2||2 + 1||

1 1 2 ||2 + 1|| = (2 + 1)||2 + 1|| (2 + 1) 2||2 + 1|| + 2 12

6.

+ − +

3 3 0

| + 5|

12 ( + 5)| + 5| 1 ( + 6

5) 2| + 5|

3 1 3 || + 5|| = ( + 5)|| + 5|| ( + 5) 2| + 5|| + 2 2

7.

+ − +

5 5 0

|| + 3|| + 3

|| + 3||

1 ( + 2

3)| + 3|

|| + 3|| 5 5 = 5 || + 3|| ( + 3)|| + 3|| + 2 + 3

8.

+ − +

2 2 0

|3 + 1| 3 + 1 1 |3 + 1| 3

1 (3 + 18

1)|3 + 1|

|3 + 1| 2 1 2 = ||3 + 1|| (3 + 1)||3 + 1|| + 3 + 1 3 9


9.

+ 3 + 1 −

6

6

10.

0

+ 2

3 2

+

2

Solutions Chapter 14 Review

( 3 + 1) = � ( 3 + 1) 3 2 6 6 � 2 = � ( 3 + 3 2 + 6 + 7) � 2 = 39 2 2 2

+ ln −

12.67

2

2

1

3 3

1 1 2 2 ln = - 3 ln . 3 1 1 1 3 3 1 1 1 1 1 2 3 + 1 = - 3 ln 3. = 3 3 + = 3 9 3 9 9 9 1

11. 3 = 1 3 when 3 = 1 2, = 1 2 1 3. Area = = =

1 2 1 3

0

1 2 1 3

0

1

2 1 3

[(1 3) 3] +

(1 2 3) +

1

1

1 2 1 3

1 2 1 3

(2 3 1) = -

0

1 2

1 3

1 1 + - 4 . 1 3 2 1 2

0.6906

2

( ) = [ + ]02 = 2 + 2 2.

13. 1 2 = 2 when = ±1 Area =

1 4 1 2 2 .0

1 1 1 1 3 1 + 1 + = 1 3 1 3 1 3 1 3 2 2 4$2 4$2 2 2$2

12. * 1 * for in [0, 2], so Area =

[ 3 (1 3)]

2, so

[(1 2) 2] =

1 2 2 1 2 = [ 3] = 2 1 3

1 2

(1 2 2) 1 2 1 2 1 2 4 2 2 . + = = 3 2 6 2 2 6 2 3 2

14. * for in [0, 2] because # 1 in that range. 2 2 1 Area = ( ) = - 2 + + . (using integration by parts) 2 0 0


Solutions Chapter 14 Review = 2 + 2 2 + 2 1 = 1 + 3 2

15. / = 16. / =

2 1 1 4 1 ( 3 1) = = [2 6] = 1 2 ( 2) 4 4 4 2 2 2

1 1 1 1 2 ln( 1) (using = + . 2 1 0 2 0 0 + 1

17. Average =

= 2 + 1) =

1 ln 2 2

1 1 1 2 = � 2 2 + 2 �0 (using integration by parts) = 2 1 0 0

2 1 ( + 1) ln 2 1 1 2 2 1 1 2 1 1 ln = + ? .1 > 2 + 1? (using integration by parts) 2 1 -> 2 2 1 1 2 1 1 1 2 1 2 = �(2 2 + 2 ) ln(2 ) 2 2 + 14 + 1� �� + �ln 4 � = 1 2 1 2 2 1 2( 2 + ) ln 2 + 2 + 5 4 5.10548 = 2 1

18. Average =

19. / =

3( 2) 2 1 1 32 1 3 2 (3 + 1) = + = 1 + � + 2�2 = 3 2 2 2 2 2 2 2 2 2

1 1 1 (6 2 + 12) = �2 3 + 12 � 2 = �2 3 + 12 (2( 2) 3 + 12( 2))� 2 2 2 2 2 = 6 12 + 20

20. / = 21. / =

1 4 3 3 7 3 3 � 7 3 = ( 2) 7 3� = . 2 2 14 14 2

1 1 1 ln = [ ln ] 2 = [ ln ( 2) ln( 2) + 2] 2 2 2 2 1 = [ ln ( 2) ln( 2) 2] 2

22. / =

23. = 100 2 , so / = 100 20 = 80. The consumers' surplus is 80 1 9 = >50 2 10? 0 80 1 = >40 2 ? 0 = �40 14 2�

80

0

= 3,200 1,600 = $1,600.

24. = (10 ) 2, so / = (10 4) 2 = 36. The consumers' surplus is 9 =

36

=

0

B10 1 2 4C

36

0

B6 1 2C

= �6 23 3 2�

36

0

= $72.


Solutions Chapter 14 Review 25. = 2 100, so / = 200 100 = 100. The producers' surplus is 100 1 89 = >100 50 2 ? 0 100 1 = >50 2 ? 0 = �50 14 2�

100

0

= 5,000 2,500 = $2,500.

26. = ( 10) 2, so / = (40 10) 2 = 900. The producers' surplus is 89 =

900

=

0

B40 (10 + 1 2)C

900

0

B30 1 2C

= �30 23 3 2�

27.

28.

29.

900

0

= $9,000.

1 1 1 1 1 1 = lim = lim - 4. = lim - < 4 + . = 5 5 4 4 4 4 1 1 1

1 1 1 1 1 1 1 = lim = lim - 4. = lim � + � = =; diverges 5 5 + + + 4 4 4" 4 0 0 0 0 1

0 = lim + lim 5 3 5 3 5 3 2 2 2 + 1 1 ( 1) 0 ( 1) 1 ( 1) 0 3 3 (use the substitution = 2 1) = lim 1 + + lim 1 1 4( 2 1) 2 3 2 1 4( 2 1) 2 3 20 1

= = =; diverges (Note that the infinities don't cancel. For convergence we need each part of the integral to converge on its own.) 30.

2 = lim + lim 1 3 1 3 1 3 2 2 2 + 1 1 0 ( 1) 0 ( 1) ( 1) 2

31.

2

3( 2 1) 2 3 3( 2 1) 2 3 2 lim + + 1 1 20 2 (use the substitution = 1) 4 4 = 3(3 2 3 1) 4 = lim 1 1

0

2

2 =

= lim [

3

2 6 =

lim

0 2

( 1)] = 1

lim

0 6 2

2

2 = 3

2 6 =

2 lim � �0 (use

= 2)

1 � 6 3� 0 (use 18 lim

= 6 3)

32.

0

33.

1 1 1 3 3 + 3 2 2 + ; + ; ;+= 3 = = = + = + 2 + 3 3 +2 +

1 [ = lim 18

( 1)] =

1 18


Solutions Chapter 14 Review 2 2 1 1 2 34. + = ; + = ; ln |+ + 2|| = + ; + + 2 = 3 2; + = 3 2 2 + + 2 ++ 2 2

+2 +2 1 1 1 1 ; + + = ; 2 ln | | + 1 (note = ln | | + ; = 0 + ; = ln | | + ; + = L 2 2 2 2 that +(1) > 0)

35. + + =

1 1 + = 2 ; + = ; ln ||+|| = 2 + + 1 + + 1

36.

ln 2 = 0 + ; ln + =

1 ln( 2 + 2

+ 45 + 200 45

0

+ 5

0

0

( ) ( ) =

+ 25 + 3,200 25

− + 10

0

5

0

9 0.09 (45 + 200) .

9 0.09

100 0.09

10,000 0.09 9

38. Rate of spending = ( ) ( ), so total spending =

0

5

9 0.09 (45 + 200) = - 0.09 >100(45 + 200) 45 $

1) + (substitute = 2 + 1);

1) + ln 2 = ln B2 2 + 1C; + = 2 2 + 1

37. Rate of spending = ( ) ( ), so total spending =

1 ln( 2 + 2

10

0

10,000 5 9 ?.0

( ) ( ) =

18,238

18,200 dollars

10

0

4 0.04 (25 + 3,200) .

4 0.04

100 0.04

2,500 0.04

10 4 0.04 (25 + 3,200) = � 0.04 (100[25 + 3,200] 25(2,500))�0

163,940

164,000 dollars

39. a. The monthly cost is 2,000 0.01 dollars after months, so the average amount over two years is 24 2,000 200,000 0.24 24 0.01 = 2,000�100 0.01 �0 = ( 1) $2,260 24 0 0 24 2,000 0.01 b. = 500�100 0.01 � 4 = 50,000� 0.01 0.01( 4)� 4 4 = 50,000 0.01 �1 0.04�

1,960.53 0.01 .

40. a. The amount in the account at any given time is 1,000,000 0.06 dollars after years, so the average amount over two years is 2 1,000,000 2 0.06 0.12 1 0.06 = 500,000 500,000 $1,062,500 = 2 0 0.06 0.06 0 0


Solutions Chapter 14 Review 1,000,000 b. 0.06 = 200,000,000� 0.06 � 1 12 = 200,000,000� 0.06 0.06( 1 12)� 1 12 0 1 12 = 200,000,000 0.06 �1 0.06 12�

997,500 0.06 .

41. a. For equilibrium price, 20,000(28 ) 1 3 = 40,000( 19) 1 3 gives, by cubing both sides, 28 = 8( 19) 9 = 180, so = 20 and / = 20,000(28 20) 1 3 = 40,000. / b. Consumers' surplus: Solve the demand equation for : = 20,000(28 ) 1 3 (28 ) 1 3 = = 20,000 2 × 10 4 3 28 = 8 × 10 12 3 = 28 8 × 10 12 40,000 40,000 3 4 9 = 20� 28 20 = = $240,000. �28 1 20 8 × 10 12 32 × 10 12 0 Producers' surplus: Solve the supply equation for : = 40,000( 19) 1 3 ( 19) 1 3 = = 40,000 4 × 10 4 3 19 = 64 × 10 12 3 = 19 + 64 × 10 12 40,000 40,000 3 4 89 = = = $30,000. �20 19 � 1 64 × 10 12 256 × 10 12 20 0 42. a. 1,000 200 2 = 1,000 10 400 200 2 = 10 400 = 50 and / = 1,000 200 2(50) = 10,000 / b. Solve the demand equation for : = 100 2 2,000,000 10,000 10,000 2 2 9 = 50� = �100 �50 � 2,000,000 2,000,000 0 0 3 6,000,000 20 Solve the supply equation for : = 40 + 2 10,000,000

10,000

= 150 89 =

10,000

0

= 110

�50 40

3 30,000,000 20

$333,000.

10,000 2 2 � = �10 � 10,000,000 10,000,000 0

10,000,000

$66,700.

43. The price follows the function ( ) = 40 2 while the quantity sold per week follows ( ) = 5,000 0.1 , where is measured in weeks. The revenue per week is therefore ( ) = ( ) ( ) and the total revenue over the next 8 weeks is


Solutions Chapter 14 Review

8

0

5,000(40 2 ) 0.1 = 5,000� (400 20 ) 0.1 + 200 0.1 �0 (using integration by parts)

= 5,000�(20 200) 0.1 �0 = 5,000[ 40 0.8 + 200]

$910,000.

8

8

44. The monthly investment, in thousands of dollars per year, is (1.7 2 0.5 + 8) 0.05 . So, the total investment is 12

(1.7 2 0.5 + 8) 0.05 thousand dollars

0

= � 20(1.7 2 0.5 + 8) 0.05 400(3.4 0.5) 0.05 27,200 0.05 �0 (using integration by parts) 676.5 thousand dollars, or about $677,000. 12

45. a. The rate at which money is deposited is 100,000 + 10,000(12 ) dollars per month after years; converting to dollars per year gives ( ) = 1,200,000 + 1,440,000 dollars per year after years. The total deposited over 2 years is 2

0

(1,200,000 + 1,440,000 ) 0.06(2 )

1,200,000 + 1,440,000 0.06(2 ) 1,440,000 0.06(2 ) 2 = 0.06 0.06 2 0 b. The principal is given by 2

$5,549,000.

(1,200,000 + 1,440,000 ) = �1,200,000 + 720,000 2�0 = $5,280,000, 0 so the interest is the remaining $269,000. 2

46. a. The rate at which money is deposited is ( ) = 1,000 + 50 dollars per month after months. The interest rate per month is 5 12, so the future value is (7 =

180

180

(1,000 + 50 ) 0.05(180 ) 12 = � 240(1,000 + 50 ) 50 × 57,600) 0.05(180 ) 12�0 0 $1,325,040. b. The total deposited was 180

(1,000 + 50 ) = �1,000 + 25 2�0 = $990,000. 0 The difference, $1,325,040 990,000 = $335,040, is interest. 180

47. The revenue stream is ( ) = 50 0.1 million dollars per year. The present value of next year's revenue is 1

0

50 0.1 0.06 =

3

1

0

50 0.04 = �1,250 0.04 �0

1

$51 million.

48. The present value of the next three years of JungleBooks profit is (in millions of dollars) (40 + 5 ) 0.04 = �( 25(40 + 5 ) 5 × 625) 0.04 �0 0 OHaganBooks should pay no more than $133.86 million.

3

49. The money +( ) in the account satisfies the differential equation equation: 1 + = 0.0001 +2

$133.86 million.

+ = 0.0001+ 2. We solve this


1 + = 0.0001 + 2 1 = 0.0001 + + +(0) = 10,000

Solutions Chapter 14 Review

= 1 10,000 = 0.0001 10,000 1 += . = 0.0001 0.0001 1 The amount in the account would approach infinity one year after the deposit. so

50. If + represents the amount in the account, it satisfies the differential equation

+ = 0.1(1,000,000 +).

Separating variable gives us 1 + = 0.1 . 1,000,000 + Integrating gives ln(1,000,000 +) = 0.1 + so + = 1,000,000 3 0.1 . Because +(0) = 800,000, 3 = 200,000, so + = 1,000,000 200,000 0.1 . As M =, + approaches but never reaches $1 million.


Solutions Chapter 14 Case Study Chapter 14 Case Study

1. 6 ( ) = 0.25 , so the total tax revenue is

E ( )6 ( ) =

0

0

100 0.466 23,000(0.25 )

2. 6 ( ) = 0.45 , so the total tax revenue is

E ( )6 ( ) =

0

0

100 0.4 30,000(0.45 )

$1.851 trillion. $3.108 trillion.

if 0 # < 30,000 N0 Q if 30,000 # < 250,000 3. 6 ( ) = O0.10( 30,000) Q22,000 + 0.80( 250,000) if * 250,000 P | 30,000| | 250,000| = 0.05( 30,000) 250,000 30,000 | 250,000| + [11, 000 + 0.4( 250,000)] 1 + 250,000 We are given E ( ) = 100 0.466 23,000. The total tax revenue is

30,000

E ( )6 ( )

$264.9 billion (using technology).

0 ! 250,000

if < 250,000 if * 250,000 || 250, 000|| 1 = ( 250, 000)R1 + 2 250, 000 S We are given E ( ) = 100 0.4 30,000. The total tax revenue is 4. 6 ( ) =

250,000

5. a.

E ( )6 ( )

E ( ) = E(') E( ) is the number of people earning between $ and $'.

E ( ) =

0 more than $0). b.

$3.394 billion (using technology).

lim

0

E ( ) is the total size of the income earning population (those making

E ( ) represents the total income of the population. One way to see this is to consider a tax 0 function 6 ( ) = , which takes all income as tax. 6.

7. a. E( ) + 8 ( ) = the total population b. Since E( ) + 8 ( ) is constant, E ( ) + 8 ( ) = 0, hence 8 ( ) = E ( ). c. Total tax revenue =

0

E ( )6 ( ) =

0

8 ( )6 ( )


= lim [8 ( )6 ( )]0 +

Solutions Chapter 14 Case Study

8 ( )6 ( ) =

0 0 since 6 (0) = 0 and 8 (<) = 0 for < sufficiently large.

8 ( )6 ( )

8. a. If [ 1 , ] is a subinterval of [ , '], 8 ( 1 ) people will be earning more than 1 dollars, so there will be a contribution of approximately 8 ( 1 )( 1 ) from these people toward the total earned in the bracket. Adding all of these contributions gives a Riemann sum; taking the limit gives the indicated integral. b. If in the bracket < # ' the marginal tax rate is 6 ( ) = 4, then every dollar earned in that bracket will contribute 4 dollars to the total tax revenue. Using part (a), the contribution of this bracket to the total tax revenue will be 4 the stated formula.

8 ( ) =

8 ( )6 ( ) . Adding the contributions of all the brackets gives


Solutions Section 15.1 Section 15.1 1. a. 𝑓(0, 0) = 0 2 + 0 2 − 0 + 1 = 1 b. 𝑓(1, 0) = 1 2 + 0 2 − 1 + 1 = 1 c. 𝑓(0, −1) = 0 2 + (−1) 2 − 0 + 1 = 2 d. 𝑓(𝑎, 2) = 𝑎 2 + 2 2 − 𝑎 + 1 = 𝑎 2 − 𝑎 + 5 e. 𝑓(𝑦, 𝑥) = 𝑦 2 + 𝑥 2 − 𝑦 + 1 f. 𝑓(𝑥 + ℎ, 𝑦 + 𝑘) = (𝑥 + ℎ) 2 + (𝑦 + 𝑘) 2 − (𝑥 + ℎ) + 1 2. a. 𝑓(0, 0) = 0 2 − 0 − 0 + 1 = 1 b. 𝑓(1, 0) = 1 2 − 0 − 0 + 1 = 2 c. 𝑓(0, −1) = 0 2 − (−1) − 0 + 1 = 2 d. 𝑓(𝑎, 2) = 𝑎 2 − 2 − 2𝑎 + 1 = 𝑎 2 − 2𝑎 − 1 e. 𝑓(𝑦, 𝑥) = 𝑦 2 − 𝑥 − 𝑥𝑦 + 1 f. 𝑓(𝑥 + ℎ, 𝑦 + 𝑘) = (𝑥 + ℎ) 2 − (𝑦 + 𝑘) − (𝑥 + ℎ)(𝑦 + 𝑘) + 1 3. a. 𝑓(0, 0) = 0 + 0 − 0 = 0 b. 𝑓(1, 0) = 0.2 + 0 − 0 = 0.2 c. 𝑓(0, −1) = 0 + 0.1(−1) − 0 = −0.1 d. 𝑓(𝑎, 2) = 0.2𝑎 + 0.1(2) − 0.01𝑎(2) = 0.18𝑎 + 0.2 e. 𝑓(𝑦, 𝑥) = 0.2𝑦 + 0.1𝑥 − 0.01𝑥𝑦 = 0.1𝑥 + 0.2𝑦 − 0.01𝑥𝑦 f. 𝑓(𝑥 + ℎ, 𝑦 + 𝑘) = 0.2(𝑥 + ℎ) + 0.1(𝑦 + 𝑘) − 0.01(𝑥 + ℎ)(𝑦 + 𝑘) 4. a. 𝑓(0, 0) = 0 − 0 − 0 = 0 b. 𝑓(1, 0) = 0.4 − 0 − 0 = 0.4 c. 𝑓(0, −1) = 0 − 0.5(−1) − 0 = 0.5 d. 𝑓(𝑎, 2) = 0.4𝑎 − 0.5(2) − 0.05𝑎(2) = 0.3𝑎 − 1 e. 𝑓(𝑦, 𝑥) = 0.4𝑦 − 0.5𝑥 − 0.05𝑥𝑦 = −0.5𝑥 + 0.4𝑦 − 0.05𝑥𝑦 f. 𝑓(𝑥 + ℎ, 𝑦 + 𝑘) = 0.4(𝑥 + ℎ) − 0.5(𝑦 + 𝑘) − 0.05(𝑥 + ℎ)(𝑦 + 𝑘) 5. a. 𝑔(0, 0, 0) = 𝑒 0+0+0 = 1 b. 𝑔(1, 0, 0) = 𝑒 1+0+0 = 𝑒 c. 𝑔(0, 1, 0) = 𝑒 0+1+0 = 𝑒 d. 𝑔(𝑧, 𝑥, 𝑦) = 𝑒 𝑧+𝑥+𝑦 = 𝑒 𝑥+𝑦+𝑧 e. 𝑔(𝑥 + ℎ, 𝑦 + 𝑘, 𝑧 + 𝑙) = 𝑒 𝑥+ℎ+𝑦+𝑘+𝑧+𝑙 6. a. 𝑔(0, 0, 0) = ln(0 + 0 + 0) = ln 0 does not exist. b. 𝑔(1, 0, 0) = ln(1 + 0 + 0) = ln 1 = 0 c. 𝑔(0, 1, 0) = ln(0 + 1 + 0) = ln 0 = 0 d. 𝑔(𝑧, 𝑥, 𝑦) = ln(𝑧 + 𝑥 + 𝑦) = ln(𝑥 + 𝑦 + 𝑧) e. 𝑔(𝑥 + ℎ, 𝑦 + 𝑘, 𝑧 + 𝑙) = ln(𝑥 + ℎ + 𝑦 + 𝑘 + 𝑧 + 𝑙) 0 0 does not exist. b. 𝑔(1, 0, 0) = =0 0+0+0 1+0+0 𝑧𝑥𝑦 𝑥𝑦𝑧 0 c. 𝑔(0, 1, 0) = = 0 d. 𝑔(𝑧, 𝑥, 𝑦) = 2 = 2 2 2 0+1+0 𝑧 +𝑥 +𝑦 𝑥 + 𝑦2 + 𝑧2 (𝑥 + ℎ)(𝑦 + 𝑘)(𝑧 + 𝑙) e. 𝑔(𝑥 + ℎ, 𝑦 + 𝑘, 𝑧 + 𝑙) (𝑥 + ℎ) 2 + (𝑦 + 𝑘) 2 + (𝑧 + 𝑙) 2 7. a. 𝑔(0, 0, 0) =

𝑒0 𝑒0 does not exist. b. 𝑔(1, 0, 0) = =1 0+0+0 1+0+0 𝑒0 𝑒 𝑧𝑥𝑦 𝑒 𝑥𝑦𝑧 c. 𝑔(0, 1, 0) = = 1 d. 𝑔(𝑧, 𝑥, 𝑦) = = 0+1+0 𝑧+𝑥+𝑦 𝑥+𝑦+𝑧 𝑒 (𝑥+ℎ)(𝑦+𝑘)(𝑧+𝑙) e. 𝑔(𝑥 + ℎ, 𝑦 + 𝑘, 𝑧 + 𝑙) = 𝑥+ℎ+𝑦+𝑘+𝑧+𝑙 8. a. 𝑔(0, 0, 0) =

9. a. 𝑓 increases by 2.3 units for every 1 unit of increase in 𝑥. b. 𝑓 decreases by 1.4 units for every 1 unit of increase in 𝑦. c. 𝑓 decreases by 2.5 units for every 1 unit increase in 𝑧. 10. a. 𝑔 decreases by 0.03 units for every 1 unit of increase in 𝑧. b. 𝑔 increases by 0.01 units for every 1 unit of increase in 𝑥. c. 𝑔 increases by 0.02 units for every 1 unit increase in 𝑦. 11. Neither, because of the 𝑦 2 term


Solutions Section 15.1 12. Interaction 13. Linear 14. Neither, because of the 𝑥12 term 15. Linear, because 𝑓(𝑥, 𝑦, 𝑧) = 13 𝑥 + 13 𝑦 − 13 𝑧 16. Linear, because 𝑔(𝑥, 𝑦, 𝑧) = 14 𝑥 − 34 𝑦 + 14 𝑧 17. Interaction, because 𝑔(𝑥, 𝑦, 𝑧) = 14 𝑥 − 34 𝑦 + 14 𝑦𝑧 18. Neither 19. a. 𝑓(20, 10) = 107 b. 𝑓(40, 20) = −14 c. 𝑓(10, 20) − 𝑓(20, 10) = −6 − 107 = −113 20. a. 𝑓(10, 30) = 426 b. 𝑓(20, 10) = 107 c. 𝑓(10, 40) + 𝑓(10, 20) = 558 + 294 = 852 21. 𝑥→ 𝑦 ↓

10

20

30

40

10

52

107

162

217

20

94

194

294

394

30 136

281

426

571

40 178

368

558

748

10

20

30

40

10

−1

−3

−5

−7

20

−6

−14

−22

−30

30 −11

−25

−39

−53

40 −16

−36

−56

−76

22. 𝑥→ 𝑦 ↓

23. Enter the values of 𝑛 in cells B1–K1, and the values of 𝑑 in cells A2–A11. Then enter the formula = FINV(0.05,B$1,$A2) in cell B2 and copy down to K2 and then across to K11. 24. Enter the values of 𝑤 in cells B1–H1, and the values of ℎ in cells A2–A12. Then enter the formula =B$1/$A2^2 in cell B2 and copy down to H2 and then across to H12.


Solutions Section 15.1 70

80

90

100

110

120

130

1.8 21.6

24.7

27.8

30.9

34.0

37.0

40.1

1.85 20.5

23.4

26.3

29.2

32.1

35.1

38.0

1.9 19.4

22.2

24.9

27.7

30.5

33.2

36.0

1.95 18.4

21.0

23.7

26.3

28.9

31.6

34.2

2 17.5

20.0

22.5

25.0

27.5

30.0

32.5

2.05 16.7

19.0

21.4

23.8

26.2

28.6

30.9

2.1 15.9

18.1

20.4

22.7

24.9

27.2

29.5

2.15 15.1

17.3

19.5

21.6

23.8

26.0

28.1

2.2 14.5

16.5

18.6

20.7

22.7

24.8

26.9

2.25 13.8

15.8

17.8

19.8

21.7

23.7

25.7

2.3 13.2

15.1

17.0

18.9

20.8

22.7

24.6

25. Tech formula: TI-83/84 Plus: X^2√(1+X*Y) Result: 18, 4, 0.0965, 47,040 26. Tech formula: TI-83/84 Plus: X^2e^(Y) Result: 0, 148.4132, 23.8777, 980,473.1552

Spreadsheet: =x^2*sqrt(1+x*y)

Spreadsheet: =x^2*exp(y)

27. Tech formula: TI-83/84 Plus: X*ln(X^2+Y^2) Result: 6.9078, 1.5193, 5.4366, 0 28. Tech formula: TI-83/84 Plus: X/(X^2-Y^2) Result: −0.3333, 0, −0.6581, 10

Spreadsheet: =x*ln(x^2+y^2)

Spreadsheet: =x/(x^2-y^2)

29. Let 𝑧 = annual sales of Z (in millions of dollars), 𝑥 = annual sales of X, and 𝑦 = annual sales of Y. A linear model has the form 𝑧 = 𝑎𝑥 + 𝑏𝑦 + 𝑐. We are told that 𝑎 = −2.1 and 𝑏 = 0.4, and that 𝑧(6, 6) = 6. Thus, 6 = −2.1(6) + 0.4(6) + 𝑐, so 𝑐 = 16.2. The model is 𝑧 = −2.1𝑥 + 0.4𝑦 + 16.2. 30. Let 𝑧 = annual sales of Z (in millions of dollars), 𝑥 = annual sales of X, and 𝑦 = annual sales of Y. A linear model has the form 𝑧 = 𝑎𝑥 + 𝑏𝑦 + 𝑐. We are told that 𝑎 = −2.5 and 𝑏 = 23∕2 = 11.5, and that 𝑧(2, 2) = 62. Thus, 62 = −2.5(2) + 11.5(2) + 𝑐, so 𝑐 = 44. The model is 𝑧 = −2.5𝑥 + 11.5𝑦 + 44. 31.

32.


Solutions Section 15.1 33.

34.

35.

36.

37.

38.


Solutions Section 15.1 39.

40.

41. (H): The plane with 𝑧-intercept 1, 𝑥-intercept 1∕3, and 𝑦-intercept −1∕2 42. (E): An inverted cone with apex at 1 on the 𝑧-axis 43. (B): An inverted paraboloid with apex at 1 on the 𝑧-axis 44. (D): A saddle, rising as ||𝑦|| increases, falling as ||𝑥|| increases 45. (F): A hemisphere lying below the 𝑥𝑦-plane 46. (A): A paraboloid with lowest point at 1 on the 𝑧-axis 47. (C): A surface of revolution having the 𝑧-axis as a vertical asymptote. 48. (G): The plane with 𝑧-intercept 1, 𝑥-intercept -1/3, and 𝑦-intercept 1/2 49. 𝑐 = 0 : 2𝑥 2 + 2𝑦 2 = 0 gives 𝑥 2 + 𝑦 2 = 0. This is the circle centered at the origin of radius 0 (that is, the single point (0, 0). 𝑐 = 2 : 2𝑥 2 + 2𝑦 2 = 2, or 𝑥 2 + 𝑦 2 = 1. This is the circle centered at the origin of radius 1. 𝑐 = 18 : 2𝑥 2 + 2𝑦 2 = 18, or 𝑥 2 + 𝑦 2 = 9. This is the circle centered at the origin of radius 3. Sketch:

50. 𝑐 = 0 : 3𝑥 2 + 3𝑦 2 = 0 gives 𝑥 2 + 𝑦 2 = 0. This is the circle centered at the origin of radius 0 (that is, the single point (0, 0). 𝑐 = 3 : 3𝑥 2 + 3𝑦 2 = 3, or 𝑥 2 + 𝑦 2 = 1. This is the circle centered at the origin of radius 1. 𝑐 = 27 : 3𝑥 2 + 3𝑦 2 = 27, or 𝑥 2 + 𝑦 2 = 9. This is the circle centered at the origin of radius 3. Sketch:


Solutions Section 15.1

51. 𝑐 = −2 : 𝑦 + 2𝑥 2 = −2 gives 𝑦 = −2𝑥 2 − 2 (parabola). 𝑐 = 0 : 𝑦 + 2𝑥 2 = 0 gives 𝑦 = −2𝑥 2 (parabola). 𝑐 = 2 : 𝑦 + 2𝑥 2 = 2 gives 𝑦 = −2𝑥 2 + 2 (parabola). Sketch:

52. 𝑐 = −2 : 2𝑦 − 𝑥 2 = −2 gives 𝑦 = (𝑥 2 − 2)∕2 = 12 𝑥 2 − 1 (parabola). 𝑐 = 0 : 2𝑦 − 𝑥 2 = 0 gives 𝑦 = 12 𝑥 2 (parabola). 𝑐 = 2 : 2𝑦 − 𝑥 2 = 2 gives 𝑦 = (𝑥 2 + 2)∕2 = 12 𝑥 2 + 1 (parabola). Sketch:

53. 𝑐 = −1 : 2𝑥𝑦 − 1 = −1 gives 𝑥𝑦 = 0 (the union of the 𝑥- and 𝑦-axes). 𝑐 = 0 : 2𝑥𝑦 − 1 = 0 gives 𝑦 = 1∕(2𝑥) (hyperbola). 𝑐 = 1 : 2𝑥𝑦 − 1 = 2 gives 𝑦 = 3∕(2𝑥) (hyperbola). Sketch:


Solutions Section 15.1 54. 𝑐 = −2 : 2 + 𝑥𝑦 = −2 gives 𝑦 = −4∕𝑥 (hyperbola). 𝑐 = 0 : 2 + 𝑥𝑦 = 0 gives 𝑦 = −2∕𝑥 (hyperbola). 𝑐 = 2 : 2 + 𝑥𝑦 = 2 gives 𝑥𝑦 = 0 (the union of the 𝑥- and 𝑦-axes). Sketch:

55. a. The point (1, 1) appears to lie on the level curve 𝑓(𝑥, 𝑦) = 4, so 𝑓(1, 1) ≈ 4. b. The point (−2, −1) appears to lie about midway between the level curves 𝑓(𝑥, 𝑦) = 4 and 𝑓(𝑥, 𝑦) = 6, so we estimate 𝑓(−2, −1) ≈ 5. c. The point (3, −2.5) appears to lie about midway between the level curves 𝑓(𝑥, 𝑦) = −2 and 𝑓(𝑥, 𝑦) = 0, so we estimate 𝑓(3, −2.5) ≈ −1. 56. a. The point (0, 1) lies on the level curve 𝑓(𝑥, 𝑦) = 2, so 𝑓(0, 1) = 2. b. The point (−1, −0.5) appears to lie about midway between the level curves 𝑓(𝑥, 𝑦) = 2 and 𝑓(𝑥, 𝑦) = 4, so we estimate 𝑓(−1, −0.5) ≈ 3. c. The point (−2, 1) appears to lie about midway between the level curves 𝑓(𝑥, 𝑦) = 0 and 𝑓(𝑥, 𝑦) = −2, so we estimate 𝑓(−2, 1) ≈ −1. 57. If the level curves were contour maps showing the elevations of various parts of a mountain range, the highest points would be around (2, 2) and (−2, −2). 58. If the level curves were contour maps showing the elevations of various parts of a mountain range, the lowest points would be around (2, −2) and (−2, 2). 59.

60.


Solutions Section 15.1 61.

62.

63.

64.

65.

66.

67.

68.


Solutions Section 15.1 69.

70.

71.

72.

73.

74.

75. a. The marginal cost of cars is $6,000 per car, the coefficient of 𝑥. The marginal cost of trucks is $4,000 per truck, the coefficient of 𝑦. b. The graph is a plane with 𝑥-intercept −40, 𝑦-intercept −60, and 𝑧-intercept 240,000. c. The slice 𝑥 = 10 is the straight line with equation 𝑧 = 300,000 + 4,000𝑦. It describes the cost function for the manufacture of trucks if car production is held fixed at 10 cars per week. d. The level curve 𝑧 = 480,000 is the straight line 6,000𝑥 + 4,000𝑦 = 240,000. It describes the number of cars and trucks you can manufacture to maintain weekly costs at $480,000. 76. a. The marginal cost of bicycles is $60 per bicycle, the coefficient of 𝑥. The marginal cost of tricycles is $20 per tricycle, the coefficient of 𝑦. b. The graph is a plane with 𝑥-intercept −400, 𝑦-intercept −1,200, and 𝑧-intercept 24,000. c. The slice by 𝑦 = 100 is the straight line with equation 𝑧 = 26,000 + 60𝑥. It describes the cost function for the manufacture of bicycles if tricycle production is held fixed at 100 tricycles per week. d. The level curve 𝑧 = 72,000 is the straight line 60𝑥 + 20𝑦 = 48,000. It describes the number of bicycles and tricycles you can manufacture to maintain weekly costs at $72,000. 77. 𝐶(𝑥, 𝑦) = 10 + 0.03𝑥 + 0.04𝑦, where 𝐶 is the cost in dollars, 𝑥 is the number of video clips sold per month, and 𝑦 is the number of audio clips sold per month


Solutions Section 15.1 78. 𝐶(𝑥, 𝑦) = 50,000 + 3,000𝑥 + 1,000𝑦, where 𝐶 is the monthly cost in dollars, 𝑥 is the number of performances by the cabaret artist per month, and 𝑦 is the number of hours of jazz per month 79. a. 𝑡 = 0.25 hours, and 𝑥 = 5. Therefore, 𝑐 = 4.25 + 2.2(5) + 100(0.25) = $40.25. b. We are told that 𝑐 = 20 and 𝑥 = 3. Therefore, 𝑐 = 4.25 + 2.2𝑥 + 100𝑡 20 = 4.25 + 2.2(3) + 100𝑡 100𝑡 = 20 − 4.25 − 6.6 = 9.15 𝑡 = 0.0915 hours = 60 × 0.0915 = 5.49 ≈ 5 minutes. c. Since 𝑐 is in dollars and 𝑡 is in hours, the coefficient 100 is measured in dollars per hour. 80. a. 𝑝 = 4 and 𝑥 = 5. Therefore, 𝑐 = 3.5 + 2.2(5) + 0.5(4) = $16.50. b. We are told that 𝑐 = 30 and 𝑝 = 4. Therefore, 𝑐 = 3.5 + 2.2𝑥 + 0.5𝑝 30 = 3.5 + 2.2𝑥 + 0.5(4) 2.2𝑥 = 30 − 3.5 − 2 = 24.5 24.5 𝑥= ≈ 11.1 miles. 2.2 c. Since 𝑐 is in dollars and 𝑥 is in miles, the coefficient 2.2 is measured in dollars per mile. 82 82 ≈ −105, and 𝑥3 -intercept 82. The = 82, 𝑥2 -intercept − 1.0 0.78 slices by 𝑥1 = 𝑐𝑜𝑛𝑠𝑡𝑎𝑛𝑡 are straight lines that are parallel to each other. Thus, the rate of change of Chrome's share as a function of Edge's share does not depend on Safari's share. Specifically, Chrome's share increases by 0.78 percentage points per 1 percentage point increase in Edge's market share, regardless of Safari's share. 81. The graph is a plane with 𝑥1 -intercept

93 93 ≈ 98, 𝑥2 -intercept ≈ 216, and 𝑥3 -intercept 93. The 0.95 0.43 slices by 𝑥2 = 𝑐𝑜𝑛𝑠𝑡𝑎𝑛𝑡 are straight lines that are parallel to each other. Thus, the rate of change of Safari's share as a function of Chrome's share does not depend on Samsung Internet's share. Specifically, Safari's share decreases by 0.95 percentage points per 1 percentage point increase in Chrome's market share, regardless of Samsung Internet's share. 82. The graph is a plane with 𝑥1 -intercept

83. a. Since 𝑥 and 𝑦 must be in thousands, 𝑥 = 1,290 and 𝑦 = 736. So 𝑁(1,290, 736) = 134 − 0.11(1,290) − 0.26(736) + 0.0004(1,290)(736) ≈ 181 thousand prisoners. b. When 𝑦 = 300 : 𝑁(𝑥, 300) = 134 − 0.11𝑥 − 0.26(300) + 0.0004𝑥(300) = 134 − 0.11𝑥 − 78 + 0.12𝑥 = 0.01𝑥 + 56 When 𝑦 = 500 : 𝑁(𝑥, 500) = 134 − 0.11𝑥 − 0.26(500) + 0.0004𝑥(500) = 134 − 0.11𝑥 − 130 + 0.2𝑥 = 0.09𝑥 + 4 Since units of the slope are units of 𝑁 (thousands of prisoners in federal prisons) per unit of 𝑥 (thousands of prisoners in state prisons), we interpret the results as follows: When there are 300,000 prisoners in local jails, the number in federal prisons increases by 10 per 1,000 additional prisoners in state prisons. When there are 500,000 prisoners in local jails, the number in federal prisons increases by 90 per 1,000 additional prisoners in state prisons. 84. a. 𝑁(198, 749) = −540 + 7.5(198) + 2.5(749) − 0.01(198)(749) ≈ 1.3 million prisoners b. 𝑥 = 80 : 𝑁(80, 𝑦) = −540 + 7.5(80) + 2.5𝑦 − 0.01(80)𝑦 = 1.7𝑦 + 60 𝑥 = 100 : 𝑁(100, 𝑦) = −540 + 7.5(100) + 2.5𝑦 − 0.01(100)𝑦 = 1.5𝑦 + 210 Since units of the slope are units of 𝑁 (thousands of prisoners in state prisons) per unit of 𝑦 (thousands of prisoners in local jails), we interpret the results as follows:


Solutions Section 15.1 When there are 80,000 prisoners in federal prisons, the number in state prisons increases by 1,700 per 1,000 additional prisoners in local jails. When there are 100,000 prisoners in federal prisons, the number in state prisons increases by 1,500 per 1,000 additional prisoners in local jails. 85. a. The slices 𝑥 = 𝑐𝑜𝑛𝑠𝑡𝑎𝑛𝑡 and 𝑦 = 𝑐𝑜𝑛𝑠𝑡𝑎𝑛𝑡 are straight lines. b. No. Even though the slices 𝑥 = 𝑐𝑜𝑛𝑠𝑡𝑎𝑛𝑡 and 𝑦 = 𝑐𝑜𝑛𝑠𝑡𝑎𝑛𝑡 are straight lines, the level curves are not, so the surface is not a plane. c. The slice 𝑥 = 10 has a slope of 3,800. The slice 𝑥 = 20 has a slope of 3,600. Manufacturing more cars lowers the marginal cost of manufacturing trucks. 86. a. The slices 𝑥 = 𝑐𝑜𝑛𝑠𝑡𝑎𝑛𝑡 and 𝑦 = 𝑐𝑜𝑛𝑠𝑡𝑎𝑛𝑡 are straight lines. b. No. Even though the slices 𝑥 = 𝑐𝑜𝑛𝑠𝑡𝑎𝑛𝑡 and 𝑦 = 𝑐𝑜𝑛𝑠𝑡𝑎𝑛𝑡 are straight lines, the level curves are not, and so the surface is not a plane. c. The slice 𝑥 = 10 has a slope of 23. The slice 𝑥 = 20 has a slope of 26. Manufacturing more bicycles raises the marginal cost of manufacturing tricycles. 87. a. 𝑅(12,000, 5,000, 5,000) = $9,980 b. 𝑅(𝑧) = 𝑅(5,000, 5,000, 𝑧) = 10,000 − 0.01(5,000) − 0.02(5,000) − 0.01𝑧 + 0.00001(5,000)𝑧 = 9,850 + 0.04𝑧 88. a. 𝑅(12,000, 5,000, 5,000) = 20,000 − 0.02(12,000) − 0.04(5,000) − 0.01(5,000) + 0.00001(5,000)(5,000) = $19,760 b. 𝑅(𝑧) = 𝑅(5,000, 5,000, 𝑧) = 20,000 − 0.02(5,000) − 0.04(5,000) − 0.01𝑧 + 0.00001(5,000)𝑧 = 19,700 + 0.04𝑧 89. 𝑈(11, 10) − 𝑈(10, 10) ≈ 5.75. This means that, if your company now has 10 copies of Macro Publish and 10 copies of Turbo Publish, then the purchase of one additional copy of Macro Publish will result in a productivity increase of approximately 5.75 pages per day. 90. 𝐶(11, 10) − 𝐶(10, 10) = 1,050 while 𝐶(10, 11) − 𝐶(10, 10) = 1,260. This means that, if you are currently using 10 carpenters and 10 electricians to build a house, it would cost you less to hire an additional carpenter ($1,050) than to hire an additional electrician ($1,260). 91. a. (𝑎, 𝑏, 𝑐) = (3, 1∕4, 1∕𝜋) and (𝑎, 𝑏, 𝑐) = (1∕𝜋, 3, 1∕4) both work. In fact, if we take any positive 3 values for 𝑎 and 𝑏 we can take 𝑐 = . 4𝜋𝑎𝑏 4 3 1∕3 b. 𝑉 (𝑎, 𝑎, 𝑎) = 𝜋𝑎 3 = 1 gives 𝑎 = ( ) . The resulting ellipsoid is a sphere with radius 𝑎. 3 4𝜋 92. a. (𝑎, 𝑏, ℎ) = (3, 1, 1∕𝜋) and (𝑎, 𝑏, 𝑐) = (1∕𝜋, 3, 1) both work. In fact, if we take any positive values for 3 𝑎 and 𝑏, we can take ℎ = . 𝜋𝑎𝑏 1 3 1∕3 b. 𝑉 (𝑎, 𝑎, 𝑎) = 𝜋𝑎 3 = 1 gives 𝑎 = ( ) . The resulting cone has a circular base with radius 𝑎 and equal 3 𝜋 height 𝑎. 93. 𝑃 (100, 500,000) = 1,000(100 0.5)(500,000 0.5) ≈ 7,000,000 94. 𝑃 (50, 1,000,000) = 1,000(50 0.5)(1,000,000 0.5) ≈ 7,000,000 95. a. 100 = 𝐾(1,000) 𝑎(1,000,000) 1−𝑎; 10 = 𝐾(1,000) 𝑎(10,000) 1−𝑎 b. Taking logs of both sides of the first equation, we get log 100 = log 𝐾 + 𝑎 log 1,000 + (1 − 𝑎) log 1,000,000;


Solutions Section 15.1 2 = log 𝐾 + 3𝑎 + 6(1 − 𝑎), log 𝐾 − 3𝑎 = −4. From the second equation we get log 𝐾 − 𝑎 = −3 similarly. c. Solving, we get log 𝐾 = −2.5 and 𝑎 = 0.5, so 𝐾 = 10 −2.5 ≈ 0.003162. d. 𝑃 (500, 1,000,000) = 0.003162(500 0.5)(1,000,000 0.5) = 71 pianos (to the nearest piano) 96. a. 100 = 𝐾(1,000) 𝑎(1,000,000) 1−𝑎; 10 = 𝐾(1,000) 𝑎(100,000) 1−𝑎 b. log 𝐾 − 3𝑎 = −4; log 𝐾 − 2𝑎 = −4 c. 𝑎 = 0, log 𝐾 = −4, so 𝐾 = 0.0001 d. 𝑃 (500, 1,000,000) = 0.0001(500 0)(1,000,000 1) = 100 pianos 97. a. We first need to convert 𝑛 into years: 5 days = 5/365 years. So, 1.5 × 10 14 × 5 𝐵(1.5 × 10 14, 5∕365) = ≈ 4 × 10 −3 g/m 2. 14 5.1 × 10 × 365 b. The total weight of sulfates in the Earth's atmosphere 98. We have 𝑥 = 45 × 10 12𝑒 𝑘𝑡, where 𝑘 satisfies 𝑒 20𝑘 = 2, so 𝑘 = (ln 2)∕20 ≈ 0.035. So, 45 × 10 12𝑛𝑒 0.035𝑡 𝐵(𝑡, 𝑛) = . 𝐴 99. a. The value of 𝑁 would be doubled. b. 𝑁(𝑅, 𝑓𝑝 , 𝑛𝑒 , 𝑓𝑙 , 𝑓𝑖 , 𝐿) = 𝑅𝑓𝑝 𝑛𝑒 𝑓𝑙 𝑓𝑖 𝐿, where here 𝐿 is the average lifetime of an intelligent civilization c. Take the logarithm of both sides, since this would yield the linear function ln(𝑁) = ln(𝑅) + ln(𝑓𝑝 ) + ln(𝑛𝑒 ) + ln(𝑓𝑙 ) + ln(𝑓𝑖 ) + ln(𝑓𝑐 ) + ln(𝐿). 100. 𝑁(𝑅, 𝑓𝑝 , 𝑛𝑒 , 𝑓𝑙 , 𝑓𝑖 , 𝑓𝑐 , 𝐿, 𝑓𝑔 , 𝑀) = 𝑅𝑓𝑝 𝑛𝑒 𝑓𝑙 𝑓𝑖 (𝑓𝑐 𝐿 + 𝑓𝑔 𝑀), where 𝑓𝑔 is the fraction of intelligent life-bearing planets on which the intelligent beings develop the means and the will to communicate over intergalactic distances, and 𝑀 is the average lifetime of such technological civilizations (in years). 101. They are reciprocals of each other. 102. They are negatives of each other. 103. For example, 𝑓(𝑥, 𝑦) = 𝑥 2 + 𝑦 2 104. For example, 𝑓(𝑥, 𝑦) = 𝑥 − 𝑦 105. For example, 𝑓(𝑥, 𝑦, 𝑧) = 𝑥𝑦𝑧 106. For example, 𝑓(𝑥, 𝑦, 𝑧) = 𝑥 2𝑦 2𝑧 2 107. For example, take 𝑓(𝑥, 𝑦) = 𝑥 + 𝑦. Then setting 𝑦 = 3 gives 𝑓(𝑥, 3) = 𝑥 + 3. This can be viewed as a function of the single variable 𝑥. Choosing other values for 𝑦 gives other functions of 𝑥. 108. For example, take 𝑓(𝑥) = 3𝑥 − 4. Replacing 𝑥 by (𝑦 − 𝑧) gives 𝑓 = 3(𝑦 − 𝑧) − 4. 109. If 𝑓 = 𝑎𝑥 + 𝑏𝑦 + 𝑐, then fixing 𝑦 = 𝑘 gives 𝑓 = 𝑎𝑥 + (𝑏𝑘 + 𝑐), a linear function with slope 𝑎 and intercept 𝑏𝑘 + 𝑐. The slope is independent of the choice of 𝑘. 110. If 𝑓 = 𝑎𝑥 + 𝑏𝑦 + 𝑐𝑥𝑦 + 𝑑, then fixing 𝑦 = 𝑘 gives 𝑓 = 𝑎𝑥 + 𝑏𝑘 + 𝑐𝑘𝑥 + 𝑑 = (𝑎 + 𝑐𝑘)𝑥 + (𝑏𝑘 + 𝑑), a linear function with slope 𝑎 + 𝑐𝑘 and intercept 𝑏𝑘 + 𝑑. The slope does depend on the choice of 𝑘.


Solutions Section 15.1 111. That CDs cost more than cassettes 112. Renting two DVDs costs less than the price of renting one video game. 113. plane 114. lines 115. (B) Traveling in the direction B results in the shortest trip to nearby isotherms and hence the fastest rate of increase in temperature. 116. (C) Traveling in the direction C results in the shortest trip to nearby level curves, and hence the greatest rate of descent. 117. Agree: any slice through a plane is a straight line. 118. Disagree: The slices 𝑧 = 𝑐𝑜𝑛𝑠𝑡𝑎𝑛𝑡 through the graph are hyperbolas, not straight lines as they would be in the case of a plane. Therefore, the graph is not a plane. 119. The graph of a function of three or more variables lives in four-dimensional (or higher) space, which makes it difficult to draw and visualize. 120. Take 𝑓(𝑥, 𝑦) to be the height of the mountain at (𝑥, 𝑦). The graph of 𝑓 is then the surface of the mountain. 121. We need one dimension for each of the variables plus one dimension for the value of the function. 122. We often project three-dimensional images onto a plane. Photographs are examples. Our brains then interpret them as three-dimensional objects.


Solutions Section 15.2 Section 15.2 1. 𝑓𝑥 (𝑥, 𝑦) = −40; 𝑓𝑦 (𝑥, 𝑦) = 20; 𝑓𝑥 (1, −1) = −40; 𝑓𝑦 (1, −1) = 20 2. 𝑓𝑥 (𝑥, 𝑦) = 5; 𝑓𝑦 (𝑥, 𝑦) = −4; 𝑓𝑥 (1, −1) = 5; 𝑓𝑦 (1, −1) = −4 3. 𝑓𝑥 (𝑥, 𝑦) = 6𝑥 + 1; 𝑓𝑦 (𝑥, 𝑦) = −3𝑦 2; 𝑓𝑥 (1, −1) = 7; 𝑓𝑦 (1, −1) = −3 4. 𝑓𝑥 (𝑥, 𝑦) =

1 −1∕2 1 𝑥 ; 𝑓𝑦 (𝑥, 𝑦) = −8𝑦 3 + 1; 𝑓𝑥 (1, −1) = ; 𝑓𝑦 (1, −1) = 9 2 2

5. 𝑓𝑥 (𝑥, 𝑦) = −40 + 10𝑦; 𝑓𝑦 (𝑥, 𝑦) = 20 + 10𝑥; 𝑓𝑥 (1, −1) = −50; 𝑓𝑦 (1, −1) = 30 6. 𝑓𝑥 (𝑥, 𝑦) = 5 − 3𝑦; 𝑓𝑦 (𝑥, 𝑦) = −4 − 3𝑥; 𝑓𝑥 (1, −1) = 8; 𝑓𝑦 (1, −1) = −7 7. 𝑓𝑥 (𝑥, 𝑦) = 6𝑥𝑦; 𝑓𝑦 (𝑥, 𝑦) = 3𝑥 2; 𝑓𝑥 (1, −1) = −6; 𝑓𝑦 (1, −1) = 3 8. 𝑓𝑥 (𝑥, 𝑦) = 4𝑥 3𝑦 2 − 1; 𝑓𝑦 (𝑥, 𝑦) = 2𝑥 4𝑦; 𝑓𝑥 (1, −1) = 3; 𝑓𝑦 (1, −1) = −2 9. 𝑓𝑥 (𝑥, 𝑦) = 2𝑥𝑦 3 − 3𝑥 2𝑦 2 − 𝑦; 𝑓𝑦 (𝑥, 𝑦) = 3𝑥 2𝑦 2 − 2𝑥 3𝑦 − 𝑥; 𝑓𝑥 (1, −1) = −4; 𝑓𝑦 (1, −1) = 4 10. 𝑓𝑥 (𝑥, 𝑦) = −𝑥 −2𝑦 2 + 𝑦 2 + 𝑦; 𝑓𝑦 (𝑥, 𝑦) = 2𝑥 −1𝑦 + 2𝑥𝑦 + 𝑥; 𝑓𝑥 (1, −1) = −1; 𝑓𝑦 (1, −1) = −3 11. 𝑓𝑥 (𝑥, 𝑦) = 6𝑦(2𝑥𝑦 + 1) 2; 𝑓𝑦 (𝑥, 𝑦) = 6𝑥(2𝑥𝑦 + 1) 2; 𝑓𝑥 (1, −1) = −6; 𝑓𝑦 (1, −1) = 6 12. 𝑓𝑥 (𝑥, 𝑦) = −

2𝑦 2𝑥 ; 𝑓𝑦 (𝑥, 𝑦) = − ; 𝑓𝑥 (1, −1) undefined; 𝑓𝑦 (1, −1) undefined (𝑥𝑦 + 1) 3 (𝑥𝑦 + 1) 3

13. 𝑓𝑥 (𝑥, 𝑦) = 𝑒 𝑥+𝑦; 𝑓𝑦 (𝑥, 𝑦) = 𝑒 𝑥+𝑦; 𝑓𝑥 (1, −1) = 1; 𝑓𝑦 (1, −1) = 1 14. 𝑓𝑥 (𝑥, 𝑦) = 2𝑒 2𝑥+𝑦; 𝑓𝑦 (𝑥, 𝑦) = 𝑒 2𝑥+𝑦; 𝑓𝑥 (1, −1) = 2𝑒; 𝑓𝑦 (1, −1) = 𝑒 15. 𝑓𝑥 (𝑥, 𝑦) = 3𝑥 −0.4𝑦 0.4; 𝑓𝑦 (𝑥, 𝑦) = 2𝑥 0.6𝑦 −0.6; 𝑓𝑥 (1, −1) undefined; 𝑓𝑦 (1, −1) undefined 16. 𝑓𝑥 (𝑥, 𝑦) = −0.2𝑥 −0.9𝑦 0.9; 𝑓𝑦 (𝑥, 𝑦) = −1.8𝑥 0.1𝑦 −0.1; 𝑓𝑥 (1, −1) undefined;𝑓𝑦 (1, −1) undefined 17. 𝑓𝑥 (𝑥, 𝑦) = 0.2𝑦𝑒 0.2𝑥𝑦; 𝑓𝑦 (𝑥, 𝑦) = 0.2𝑥𝑒 0.2𝑥𝑦; 𝑓𝑥 (1, −1) = −0.2𝑒 −0.2; 𝑓𝑦 (1, −1) = 0.2𝑒 −0.2 18. 𝑓𝑥 (𝑥, 𝑦) = (1 + 𝑥𝑦)𝑒 𝑥𝑦; 𝑓𝑦 (𝑥, 𝑦) = 𝑥 2𝑒 𝑥𝑦; 𝑓𝑥 (1, −1) = 0; 𝑓𝑦 (1, −1) = 𝑒 −1 19. 𝑓𝑥𝑥 (𝑥, 𝑦) = 0; 𝑓𝑦𝑦 (𝑥, 𝑦) = 0; 𝑓𝑥𝑦 (𝑥, 𝑦) = 𝑓𝑦𝑥 (𝑥, 𝑦) = 0; 𝑓𝑥𝑥 (1, −1) = 0; 𝑓𝑦𝑦 (1, −1) = 0;𝑓𝑥𝑦 (1, −1) = 𝑓𝑦𝑥 (1, −1) = 0 20. 𝑓𝑥𝑥 (𝑥, 𝑦) = 0; 𝑓𝑦𝑦 (𝑥, 𝑦) = 0; 𝑓𝑥𝑦 (𝑥, 𝑦) = 𝑓𝑦𝑥 (𝑥, 𝑦) = 0; 𝑓𝑥𝑥 (1, −1) = 0; 𝑓𝑦𝑦 (1, −1) = 0;𝑓𝑥𝑦 (1, −1) = 𝑓𝑦𝑥 (1, −1) = 0 21. 𝑓𝑥𝑥 (𝑥, 𝑦) = 0; 𝑓𝑦𝑦 (𝑥, 𝑦) = 0; 𝑓𝑥𝑦 (𝑥, 𝑦) = 𝑓𝑦𝑥 (𝑥, 𝑦) = 10; 𝑓𝑥𝑥 (1, −1) = 0; 𝑓𝑦𝑦 (1, −1) = 0;𝑓𝑥𝑦 (1, −1) = 𝑓𝑦𝑥 (1, −1) = 10


Solutions Section 15.2 22. 𝑓𝑥𝑥 (𝑥, 𝑦) = 0; 𝑓𝑦𝑦 (𝑥, 𝑦) = 0; 𝑓𝑥𝑦 (𝑥, 𝑦) = 𝑓𝑦𝑥 (𝑥, 𝑦) = −3; 𝑓𝑥𝑥 (1, −1) = 0; 𝑓𝑦𝑦 (1, −1) = 0;𝑓𝑥𝑦 (1, −1) = 𝑓𝑦𝑥 (1, −1) = −3 23. 𝑓𝑥𝑥 (𝑥, 𝑦) = 6𝑦; 𝑓𝑦𝑦 (𝑥, 𝑦) = 0; 𝑓𝑥𝑦 (𝑥, 𝑦) = 𝑓𝑦𝑥 (𝑥, 𝑦) = 6𝑥; 𝑓𝑥𝑥 (1, −1) = −6; 𝑓𝑦𝑦 (1, −1) = 0;𝑓𝑥𝑦 (1, −1) = 𝑓𝑦𝑥 (1, −1) = 6 24. 𝑓𝑥𝑥 (𝑥, 𝑦) = 12𝑥 2𝑦 2; 𝑓𝑦𝑦 (𝑥, 𝑦) = 2𝑥 4; 𝑓𝑥𝑦 (𝑥, 𝑦) = 𝑓𝑦𝑥 (𝑥, 𝑦) = 8𝑥 3𝑦; 𝑓𝑥𝑥 (1, −1) = 12; 𝑓𝑦𝑦 (1, −1) = 2;𝑓𝑥𝑦 (1, −1) = 𝑓𝑦𝑥 (1, −1) = −8 25. 𝑓𝑥𝑥 (𝑥, 𝑦) = 𝑒 𝑥+𝑦; 𝑓𝑦𝑦 (𝑥, 𝑦) = 𝑒 𝑥+𝑦; 𝑓𝑥𝑦 (𝑥, 𝑦) = 𝑓𝑦𝑥 (𝑥, 𝑦) = 𝑒 𝑥+𝑦; 𝑓𝑥𝑥 (1, −1) = 1; 𝑓𝑦𝑦 (1, −1) = 1;𝑓𝑥𝑦 (1, −1) = 𝑓𝑦𝑥 (1, −1) = 1 26. 𝑓𝑥𝑥 (𝑥, 𝑦) = 4𝑒 2𝑥+𝑦; 𝑓𝑦𝑦 (𝑥, 𝑦) = 𝑒 2𝑥+𝑦; 𝑓𝑥𝑦 (𝑥, 𝑦) = 𝑓𝑦𝑥 (𝑥, 𝑦) = 2𝑒 2𝑥+𝑦; 𝑓𝑥𝑥 (1, −1) = 4𝑒; 𝑓𝑦𝑦 (1, −1) = 𝑒;𝑓𝑥𝑦 (1, −1) = 𝑓𝑦𝑥 (1, −1) = 2𝑒 27. 𝑓𝑥𝑥 (𝑥, 𝑦) = −1.2𝑥 −1.4𝑦 0.4; 𝑓𝑦𝑦 (𝑥, 𝑦) = −1.2𝑥 0.6𝑦 −1.6; 𝑓𝑥𝑦 (𝑥, 𝑦) = 𝑓𝑦𝑥 (𝑥, 𝑦) = 1.2𝑥 −0.4𝑦 −0.6; 𝑓𝑥𝑥 (1, −1) undefined; 𝑓𝑦𝑦 (1, −1) undefined; 𝑓𝑥𝑦 (1, −1) & 𝑓𝑦𝑥 (1, −1) undefined 28. 𝑓𝑥𝑥 (𝑥, 𝑦) = 0.18𝑥 −1.9𝑦 0.9; 𝑓𝑦𝑦 (𝑥, 𝑦) = 0.18𝑥 0.1𝑦 −1.1; 𝑓𝑥𝑦 (𝑥, 𝑦) = 𝑓𝑦𝑥 (𝑥, 𝑦) = −0.18𝑥 −0.9𝑦 −0.1;𝑓𝑦𝑦 (1, −1) undefined; 𝑓𝑥𝑥 (1, −1) undefined; 𝑓𝑥𝑦 (1, −1) & 𝑓𝑦𝑥 (1, −1) undefined 29. 𝑓𝑥 (𝑥, 𝑦, 𝑧) = 𝑦𝑧; 𝑓𝑦 (𝑥, 𝑦, 𝑧) = 𝑥𝑧; 𝑓𝑧 (𝑥, 𝑦, 𝑧) = 𝑥𝑦; 𝑓𝑥 (0, −1, 1) = −1; 𝑓𝑦 (0, −1, 1) = 0; 𝑓𝑧 (0, −1, 1) = 0 30. 𝑓𝑥 (𝑥, 𝑦, 𝑧) = 𝑦 + 𝑧; 𝑓𝑦 (𝑥, 𝑦, 𝑧) = 𝑥 − 𝑧; 𝑓𝑧 (𝑥, 𝑦, 𝑧) = 𝑥 − 𝑦; 𝑓𝑥 (0, −1, 1) = 0; 𝑓𝑦 (0, −1, 1) = −1; 𝑓𝑧 (0, −1, 1) = 1 4 4 8𝑧 ; 𝑓𝑦 (𝑥, 𝑦, 𝑧) = ; 𝑓𝑧 (𝑥, 𝑦, 𝑧) = ; 𝑓𝑥 (0, −1, 1) 2 2 2 2 (𝑥 + 𝑦 + 𝑧 ) (𝑥 + 𝑦 + 𝑧 ) (𝑥 + 𝑦 + 𝑧 2) 2 undefined; 𝑓𝑦 (0, −1, 1) undefined; 𝑓𝑧 (0, −1, 1) undefined 31. 𝑓𝑥 (𝑥, 𝑦, 𝑧) =

32. 𝑓𝑥 (𝑥, 𝑦, 𝑧) = −

12𝑥 (𝑥 2 + 𝑦 2 + 𝑧 2) 2

; 𝑓𝑦 (𝑥, 𝑦, 𝑧) = −

12𝑦 (𝑥 2 + 𝑦 2 + 𝑧 2) 2

; 𝑓𝑧 (𝑥, 𝑦, 𝑧) = −

12𝑧 (𝑥 2 + 𝑦 2 + 𝑧 2) 2

;

𝑓𝑥 (0, −1, 1) = 0; 𝑓𝑦 (0, −1, 1) = 3; 𝑓𝑧 (0, −1, 1) = −3 33. 𝑓𝑥 (𝑥, 𝑦, 𝑧) = 𝑒 𝑦𝑧 + 𝑦𝑧𝑒 𝑥𝑧; 𝑓𝑦 (𝑥, 𝑦, 𝑧) = 𝑥𝑧𝑒 𝑦𝑧 + 𝑒 𝑥𝑧; 𝑓𝑧 (𝑥, 𝑦, 𝑧) = 𝑥𝑦(𝑒 𝑦𝑧 + 𝑒 𝑥𝑧); 𝑓𝑥 (0, −1, 1) = 𝑒 −1 − 1; 𝑓𝑦 (0, −1, 1) = 1; 𝑓𝑧 (0, −1, 1) = 0 34. 𝑓𝑥 (𝑥, 𝑦, 𝑧) = 𝑦𝑒 𝑧 + 𝑒 𝑦𝑧 + 𝑦𝑧𝑒 𝑥𝑦𝑧; 𝑓𝑦 (𝑥, 𝑦, 𝑧) = 𝑥𝑒 𝑧 + 𝑥𝑧𝑒 𝑦𝑧 + 𝑥𝑧𝑒 𝑥𝑦𝑧; 𝑓𝑧 (𝑥, 𝑦, 𝑧) = 𝑥𝑦𝑒 𝑧 + 𝑥𝑦𝑒 𝑦𝑧 + 𝑥𝑦𝑒 𝑥𝑦𝑧; 𝑓𝑥 (0, −1, 1) = −𝑒 + 𝑒 −1 − 1; 𝑓𝑦 (0, −1, 1) = 0; 𝑓𝑧 (0, −1, 1) = 0 35. 𝑓𝑥 (𝑥, 𝑦, 𝑧) = 0.1𝑥 −0.9𝑦 0.4𝑧 0.5; 𝑓𝑦 (𝑥, 𝑦, 𝑧) = 0.4𝑥 0.1𝑦 −0.6𝑧 0.5; 𝑓𝑧 (𝑥, 𝑦, 𝑧) = 0.5𝑥 0.1𝑦 0.4𝑧 −0.5; 𝑓𝑥 (0, −1, 1) undefined; 𝑓𝑦 (0, −1, 1) undefined, 𝑓𝑧 (0, −1, 1) undefined 𝑦 0.8 𝑥 0.2 36. 𝑓𝑥 (𝑥, 𝑦, 𝑧) = 0.4( ) ; 𝑓𝑦 (𝑥, 𝑦, 𝑧) = 1.6( ) ; 𝑓𝑧 (𝑥, 𝑦, 𝑧) = 2𝑧; 𝑓𝑥 (0, −1, 1) undefined; 𝑥 𝑦 𝑓𝑦 (0, −1, 1) = 0; 𝑓𝑧 (0, −1, 1) = 2


Solutions Section 15.2 37. 𝑓𝑥 (𝑥, 𝑦, 𝑧) = 𝑦𝑧𝑒 , 𝑓𝑦 (𝑥, 𝑦, 𝑧) = 𝑥𝑧𝑒 𝑥𝑦𝑧, 𝑓𝑧 (𝑥, 𝑦, 𝑧) = 𝑥𝑦𝑒 𝑥𝑦𝑧; 𝑓𝑥 (0, −1, 1) = −1; 𝑓𝑦 (0, −1, 1) = 𝑓𝑧 (0, −1, 1) = 0 𝑥𝑦𝑧

38. 𝑓𝑥 (𝑥, 𝑦, 𝑧) = 𝑓𝑦 (𝑥, 𝑦, 𝑧) = 𝑓𝑧 (𝑥, 𝑦, 𝑧) =

39. 𝑓𝑥 (𝑥, 𝑦, 𝑧) = 0; 𝑓𝑦 (𝑥, 𝑦, 𝑧) = −

1 ; 𝑓 (0, −1, 1), 𝑓𝑦 (0, −1, 1) and 𝑓𝑧 (0, −1, 1) undefined 𝑥+𝑦+𝑧 𝑥

600𝑧 𝑦 0.7(1 + 𝑦 0.3) 2

; 𝑓𝑧 (𝑥, 𝑦, 𝑧) =

2, 000 ; 𝑓𝑥 (0, −1, 1) undefined (because 1 + 𝑦 0.3

𝑓(0, −1, 1) is); 𝑓𝑦 (0, −1, 1) undefined; 𝑓𝑧 (0, −1, 1) undefined 0.2𝑒 0.2𝑥 0.1𝑒 0.2𝑥−0.1𝑦 0.2 ; 𝑓𝑦 = ; 𝑓𝑧 (𝑥, 𝑦, 𝑧) = 0; 𝑓𝑥 (0, −1, 1) = ; 1 + 𝑒 −0.1𝑦 1 + 𝑒 0.1 (1 + 𝑒 −0.1𝑦) 2 0.1𝑒 0.1 𝑓𝑦 (0, −1, 1) = ; 𝑓𝑧 (0, −1, 1) = 0 (1 + 𝑒 0.1) 2 40. 𝑓𝑥 (𝑥, 𝑦, 𝑧) =

41. 𝜕𝐶∕𝜕𝑥 = 6,000; the marginal cost to manufacture each car is $6,000. 𝜕𝐶∕𝜕𝑦 = 4,000; the marginal cost to manufacture each truck is $4,000. 42. 𝜕𝐶∕𝜕𝑥 = 60; the marginal cost to manufacture each bicycle is $60. 𝜕𝐶∕𝜕𝑦 = 20; the marginal cost to manufacture each tricycle is $20. 𝜕𝑦 = −0.78. Since units of 𝜕𝑦∕𝜕𝑡 are units of 𝑦 (percentage of articles written 𝜕𝑡 by researchers in the United States) per unit of 𝑡 (years), we conclude that the number of articles written by researchers in the United States was decreasing at a rate of 0.78 percentage points per year. 𝜕𝑦 = −1.02. Since units of 𝜕𝑦∕𝜕𝑥 are units of 𝑦 (percentage of articles written by researchers in the 𝜕𝑥 United States) per unit of 𝑦 (percentage of articles written by researchers in Europe), we conclude that the number of articles written by researchers in the United States was decreasing at a rate of 1.02 percentage points per 1 percentage point increase in articles written in Europe. 43. 𝑦 = 82 − 0.78𝑡 − 1.02𝑥;

𝜕𝑧 = −0.71. Since units of 𝜕𝑧∕𝜕𝑥 are units of 𝑧 (number of articles 𝜕𝑥 written by researchers in the United States) per unit of 𝑥 (number of articles written by researchers in Europe), we conclude that the number of articles written by researchers in the United States was 𝜕𝑧 decreasing at a rate of 0.71 articles per additional article written by researchers in Europe. = 0.50. 𝜕𝑦 Since units of 𝜕𝑧∕𝜕𝑦 are units of 𝑧 (number of articles written by researchers in the United States) per unit of 𝑦 (number written by researchers in other countries excluding Europe), we conclude that the number of articles written by researchers in the United States was increasing at a rate of 0.50 articles per additional article written by researchers in other countries (excluding Europe). 44. 𝑧 = 5,960 − 0.71𝑥 + 0.50𝑦;

45. 𝐶𝑥 (𝑥, 𝑦) = 6,000 − 20𝑦; 𝐶𝑥 (10, 20) = $5,600 per car 46. 𝐶𝑦 (𝑥, 𝑦) = 20 + 0.3𝑥; 𝐶𝑦 (10, 20) = $23 per tricycle 47. a. 𝜕𝑆∕𝜕𝑡 = 4.3 − 0.065ℎ, Evaluating at (59, 58) gives 4.3 − 0.065(58) = 0.53. At the given loyalty levels (59% for Toyota and 58% for Honda), the percentage of Subaru owners who remain loyal increases at a rate of 0.53 percentage points per percentage point increase in the loyalty of Toyota owners. 𝜕𝑆∕𝜕ℎ = 3.6 − 0.065𝑡, Evaluating at (59, 58) gives 3.6 − 0.065(59) = −0.235. At the given loyalty levels (59% for Toyota and 58% for Honda), the percentage of Subaru owners who remain loyal decreases at a rate of 0.235 percentage points per percentage point increase in the loyalty of Honda owners. b. The first calculation in part (a) was 𝜕𝑆∕𝜕𝑡 = 4.3 − 0.065ℎ, which decreases with increasing Honda loyalty.


Solutions Section 15.2 48. a. 𝜕𝐿∕𝜕𝑚 = −0.4 + 0.0043𝑏, Evaluating at (49, 26) gives −0.4 + 0.0043(26) = −0.2882. At the given loyalty levels (49% for Mercedes and 26% for BMW), the percentage of Lexus owners who remain loyal decreases at a rate of 0.2882 percentage points per percentage point increase in the loyalty of Mercedes owners. 𝜕𝐿∕𝜕𝑏 = −0.3 + 0.0043𝑚, Evaluating at (49, 26) gives −0.3 + 0.0043(49) = −0.0893. At the given loyalty levels (49% for Mercedes and 26% for BMW), the percentage of Lexus owners who remain loyal decreases at a rate of 0.0893 percentage points per percentage point increase in the loyalty of BMW owners. b. The first calculation in part (a) was 𝜕𝐿∕𝜕𝑚 = −0.4 + 0.0043𝑏, which increases with increasing BMW loyalty. 49. The marginal cost of cars is $6,000 + 1,000𝑒 −0.01(𝑥+𝑦) per car. The marginal cost of trucks is $4,000 + 1,000𝑒 −0.01(𝑥+𝑦) per truck. Both marginal costs decrease as production rises. 𝑦 50. The marginal cost of a bicycle is 𝐶𝑥 (𝑥, 𝑦) = 60 + 25 . The marginal cost of a tricycle is √𝑥 𝑥 𝐶𝑦 (𝑥, 𝑦) = 20 + 25 . As 𝑥 increases, the marginal cost of a bicycle decreases and the marginal cost of √𝑦 a tricycle increases. As 𝑦 increases, the marginal cost of a bicycle increases and the marginal cost of a tricycle decreases. 51. a. 2015 corresponds to 𝑡 = 5. Single-unit homes (𝑥 = 0): 𝑆(5, 0) = 434 + 54.9(5) − 242(0) − 29.7(0)(5) = 708.5 thousand housing starts. Thus, there were about 708,500 single-unit housing starts in 2015. Multiple-unit homes (𝑥 = 1): 𝑆(5, 1) = 434 + 54.9(5) − 242(1) − 29.7(1)(5) = 318 thousand housing starts. Thus, there were about 318,000 multi-unit housing starts in 2015. b. The rate of change of housing starts is 𝑆𝑡 (𝑡, 𝑥) = 54.9 − 29.7𝑥 Single-unit homes (𝑥 = 0): 𝑆𝑡 (5, 0) = 54.9 − 29.7(0) = 54.9 thousand housing starts/year. Multi-unit homes (𝑥 = 1): 𝑆𝑡 (5, 1) = 54.9 − 29.7(1) = 25.2 thousand housing starts/year. Thus, single-unit housing starts were increasing at a rate of 54,900 per year, and multi-unit housing starts were increasing at a rate of about 25,200 per year in 2015. c. As single-unit housing starts were higher and increasing at a faster rate than multi-unit starts, the difference was widening in 2015. d. Looking at the solution to part (b), we see that the coefficient −29.7 of 𝑥𝑡 is the difference between the rates of change of single- and multi-unit housing starts: the rate at which multi-unit housing starts was outpacing single-unit starts. As that rate is negative, multi-unit housing starts were growing at a slower rate than single-unit starts. 52. a. 2018 corresponds to 𝑡 = 8. Single-unit homes (𝑥 = 1): 𝑆(8, 1) = 62.5 + 11.6(8) + 142(1) + 20.0(1)(8) = 457.3 thousand housing starts. Thus, there were about 457,300 single-unit housing starts in 2018. Multiple-unit homes (𝑥 = 0): 𝑆(8, 0) = 62.5 + 11.6(8) + 142(0) + 20.0(0)(8) = 155.3 thousand housing starts. Thus, there were about 155,300 multi-unit housing starts in 2018. b. The rate of change of housing starts is 𝑆𝑡 (𝑡, 𝑥) = 11.6 + 20.0𝑥 Single-unit homes (𝑥 = 1): 𝑆𝑡 (8, 1) = 11.6 + 20.0(1) = 31.6 thousand housing starts/year. Multi-unit homes (𝑥 = 0): 𝑆𝑡 (8, 0) = 11.6 + 20.0(0) = 11.6 thousand housing starts/year. Thus, single-unit housing starts were increasing at a rate of 31,600 per year, and multi-unit housing starts were increasing at a rate of about 11,600 per year in 2018. c. As single-unit housing starts were higher and increasing at a faster rate than multi-unit starts, the difference was widening in 2018. d. Looking at the solution to part (b), we see that the coefficient 20.0 of 𝑥𝑡 is the difference between the


Solutions Section 15.2 rates of change of single- and multi-unit housing starts: the rate at which single-unit housing starts was outpacing multi-unit starts. As that rate is positive, single-unit housing starts were growing at a faster rate than multi-unit starts. 53. ˉ 𝐶 (𝑥, 𝑦) =

200,000 + 6,000𝑥 + 4,000𝑦 − 100,000𝑒 −0.01(𝑥+𝑦) 𝑥+𝑦

(6,000 + 1,000𝑒 ˉ 𝐶 𝑥 (𝑥, 𝑦) =

=

−0.1(𝑥+𝑦)

)(𝑥 + 𝑦) − (200,000 + 6,000𝑥 + 4,000𝑦 − 100,000𝑒 −0.01(𝑥+𝑦)) (𝑥 + 𝑦) 2

−200,000 + 2,000𝑦 + (1,000𝑥 + 1,000𝑦 + 100,000)𝑒 −0.01(𝑥+𝑦) ; (𝑥 + 𝑦) 2

ˉ 𝐶 𝑥 (50, 50) = −$2.64 per car. This means that, at a production level of 50 cars and 50 trucks per week, the average cost per vehicle is decreasing by $2.64 for each additional car manufactured. (4,000 + 1,000𝑒 ˉ 𝐶 𝑦 (𝑥, 𝑦) =

=

−0.01(𝑥+𝑦)

)(𝑥 + 𝑦) − (200,000 + 6,000𝑥 + 4,000𝑦 − 100,000𝑒 −0.01(𝑥+𝑦)) (𝑥 + 𝑦) 2

−200,000 − 2,000𝑥 + (1,000𝑥 + 1,000𝑦 + 100,000)𝑒 −0.01(𝑥+𝑦) ; (𝑥 + 𝑦) 2

ˉ 𝐶 𝑦 (50, 50) = −$22.64 per truck. This means that, at a production level of 50 cars and 50 trucks per week, the average cost per vehicle is decreasing by $22.64 for each additional truck manufactured. 54. ˉ 𝐶 (𝑥, 𝑦) =

ˉ 𝐶 𝑥 (𝑥, 𝑦) =

=

20,000 + 60𝑥 + 20𝑦 + 50√𝑥𝑦 𝑥+𝑦

(60 + 25√𝑦∕𝑥)(𝑥 + 𝑦) − (20,000 + 60𝑥 + 20𝑦 + 50√𝑥𝑦) (𝑥 + 𝑦) 2

−20,000 + 40𝑦 − 25√𝑥𝑦 + 25𝑦√𝑦∕𝑥 (𝑥 + 𝑦) 2

ˉ 𝐶 𝑥 (5, 5) = −$198 per bicycle. This means that, at a production level of 5 bicycles and 5 tricycles per week, the average cost per cycle is decreasing by $198 for each additional bicycle manufactured. ˉ 𝐶 𝑦 (𝑥, 𝑦) =

=

(20 + 25√𝑥∕𝑦)(𝑥 + 𝑦) − (20,000 + 60𝑥 + 20𝑦 + 50√𝑥𝑦) (𝑥 + 𝑦) 2

−20,000 − 40𝑥 − 25√𝑥𝑦 + 25𝑥√𝑥∕𝑦 (𝑥 + 𝑦) 2

ˉ 𝐶 𝑦 (5, 5) = −$202 per tricycle. This means that, at a production level of 5 bicycles and 5 tricycles per week, the average cost per cycle is decreasing by $202 for each additional tricycle manufactured. 55. No. Your revenue function is 𝑅(𝑥, 𝑦) = 15,000𝑥 + 10,000𝑦 − 5,000√𝑥 + 𝑦, so your marginal revenue from the sale of cars is 2,500 𝑅𝑥 (𝑥, 𝑦) = $15,000 − per car √𝑥 + 𝑦 and your marginal revenue from the sale of trucks is


Solutions Section 15.2 𝑅𝑦 (𝑥, 𝑦) = $10,000 −

2,500 √𝑥 + 𝑦

per truck.

These increase with increasing 𝑥 and 𝑦. In other words, you will earn more revenue per vehicle with increasing sales, so the rental company will pay more for each additional vehicle it buys. 56. The marginal revenue for bicycles is 𝑅𝑥 (𝑥, 𝑦) = $70𝑒 −0.02𝑥−0.01𝑦 per bicycle and the marginal revenue for tricycles is 𝑅𝑦 (𝑥, 𝑦) = $35𝑒 −0.02𝑥−0.01𝑦 per tricycle. Note that these amounts approach 0 as either 𝑥 or 𝑦 approaches +∞. This means that any large dealer will be paying almost nothing per bicycle and tricycle once they have purchased sufficiently many. (For instance, if the dealer buys 50 bicycles and 50 tricycles, it will pay approximately $3 per additional bicycle and half that amount per additional tricycle.) The suggestion is a bad one from your company's point of view. 57. 𝑃𝑧 (𝑥, 𝑦, 𝑧) = 0.016𝑥 0.4𝑦 0.2𝑧 −0.6, and so 𝑃𝑧 (10, 100,000, 1,000,000) ≈ 0.0001010 papers/$, or approximately 1 paper per $10,000 increase in the subsidy. 58. 𝑃𝑥 (𝑥, 𝑦, 𝑧) = 0.09𝑥 −0.7𝑦 0.4𝑧 0.3, and so 𝑃𝑥 (12, 500,000, 40,000) ≈ 72.28 patents/new staff member 59. a. 𝑈𝑥 (10, 5) = 5.18, 𝑈𝑦 (10, 5) = 2.09. This means that, if 10 copies of Macro Publish and 5 copies of Turbo Publish are purchased, the company's daily productivity is increasing at a rate of 5.18 pages per day for each additional copy of Macro purchased and by 2.09 pages per day for each additional copy of Turbo purchased. 𝑈 (10, 5) b. 𝑥 ≈ 2.48 is the ratio of the usefulness of one additional copy of Macro to one of Turbo. Thus, 𝑈𝑦 (10, 5) with 10 copies of Macro and 5 copies of Turbo, the company can expect approximately 2.48 times the productivity per additional copy of Macro compared to Turbo. 60. a. 𝑔𝑡 (𝑡, 𝑥) = 4𝑥 − 0.4𝑡; 𝑔𝑥 (𝑡, 𝑥) = 4𝑡 − 2𝑥; 𝑔𝑡 (10, 3) = 8; 𝑔𝑥 (10, 3) = 34. Thus, if you have studied for 10 hours and have a gpa of 3.0, your score on the examination is increasing by 8 points for each additional hour of study and by 34 points for each additional point of gpa. b. 𝑔𝑡 (10, 3)∕𝑔𝑥 (10, 3) ≈ 0.235. Thus, at a level of 10 hours of study with a gpa of 3.0, one additional hour of study is equivalent to an increase of 0.235 in gpa (as far as your test score is concerned). 61. 𝐹𝑦 (𝑥, 𝑦, 𝑧) = −

2𝐾𝑄𝑞(𝑦 − 𝑏) [(𝑥 − 𝑎) 2 + (𝑦 − 𝑏) 2 + (𝑧 − 𝑐) 2] 2

. With (𝑎, 𝑏, 𝑐) = (0, 0, 0), 𝐾 = 9 × 10 9, 𝑄 = 10, and

𝑞 = 5, 𝐹𝑦 (2, 3, 3) ≈ −6 × 10 9 N/sec. 62. 𝐹𝑧 (𝑥, 𝑦, 𝑧) = −

2𝐾𝑄𝑞(𝑧 − 𝑐) [(𝑥 − 𝑎) 2 + (𝑦 − 𝑏) 2 + (𝑧 − 𝑐) 2] 2

. With (𝑎, 𝑏, 𝑐) = (0, 0, 0), 𝐾 = 9 × 10 9, 𝑄 = 10, and

𝑞 = 5, 𝐹𝑧 (2, 3, 3) ≈ −6 × 10 9 N/sec. 63. a. 𝐴𝑃 (𝑃 , 𝑟, 𝑡) = (1 + 𝑟) 𝑡; 𝐴𝑟 (𝑃 , 𝑟, 𝑡) = 𝑡𝑃 (1 + 𝑟) 𝑡−1; 𝐴𝑡 (𝑃 , 𝑟, 𝑡) = 𝑃 (1 + 𝑟) 𝑡 ln(1 + 𝑟); 𝐴𝑃 (100, 0.1, 10) = 2.59; 𝐴𝑟 (100, 0.1, 10) = 2, 357.95; 𝐴𝑡 (100, 0.1, 10) = 24.72 Thus, for a $100 investment at 10% interest, after 10 years the accumulated amount is increasing at a rate of $2.59 per $1 of principal, at a rate of $2,357.95 per increase of 1 in 𝑟 (note that this would correspond to an increase in the interest rate of 100%), and at a rate of $24.72 per year. b. 𝐴𝑃 (100, 0.1, 𝑡) tells you the rate at which the accumulated amount in an account bearing 10% interest with a principal of $100 is growing per $1 increase in the principal, 𝑡 years after the investment. 64. a. 𝐴𝑃 (𝑃 , 𝑟, 𝑡) = 𝑒 𝑟𝑡; 𝐴𝑟 (𝑃 , 𝑟, 𝑡) = 𝑡𝑃 𝑒 𝑟𝑡; 𝐴𝑡 (𝑃 , 𝑟, 𝑡) = 𝑟𝑃 𝑒 𝑟𝑡; 𝐴𝑃 (100, 0.1, 10) = 2.72; 𝐴𝑟 (100, 0.1, 10) = 2, 718.28; 𝐴𝑡 (100, 0.1, 10) = 27.18 Thus, for a $100 investment at 10% interest invested for 10 years and compounded continuously, the accumulated amount is increasing at a rate of $2.72 per $1 of principal, at a rate of $2,718.28 per increase of 1 in 𝑟, and at a rate of $27.18 per year.


Solutions Section 15.2 b. 𝐴𝑃 (100, 0.1, 𝑡) tells you the rate at which the accumulated amount in an account bearing 10% interest, compounded continuously, with a principal of $100, is growing per $1 increase in the principal, 𝑡 years after the investment. 𝑦 𝑏 𝑥 𝑎 𝑎 𝑥 𝑏 𝑥 𝑎 65. a. 𝑃𝑥 = 𝐾𝑎( ) and 𝑃𝑦 = 𝐾𝑏( ) . They are equal precisely when = ( ) ( ) . Substituting 𝑥 𝑦 𝑏 𝑦 𝑦 𝑎 𝑥 𝑏 = 1 − 𝑎 now gives = . 𝑏 𝑦 b. The given information implies that 𝑃𝑥 (100, 200) = 𝑃𝑦 (100, 200). By part (a) this occurs precisely when 𝑎∕𝑏 = 𝑥∕𝑦 = 100∕200 = 1∕2. But 𝑏 = 1 − 𝑎, so 𝑎∕(1 − 𝑎) = 1∕2, giving 𝑎 = 1∕3 and 𝑏 = 2∕3. 66. 𝐶𝑘 (𝑘, 𝑒) = 100𝑘; 𝐶𝑒 (𝑘, 𝑒) = 120𝑒. Setting these equal gives 100𝑘 = 120𝑒, so 𝑘 = 1.2𝑒. If 𝑒 = 3, then 𝑘 = 1.2(3) = 3.6 ≈ 4 carpenters. 2 1 𝑟2 −𝑟 2∕(4𝐷𝑡) 𝑒 𝑒 −𝑟 ∕(4𝐷𝑡) + 2 2 3 4𝜋𝐷𝑡 16𝜋𝐷 𝑡 Taking 𝐷 = 1, 𝑢𝑡 (1, 3) ≈ −0.0075, so the concentration is decreasing at 0.0075 parts of nutrient per part of water per second.

67. 𝑢𝑡 (𝑟, 𝑡) = −

68. 𝑢𝑡 (4, 4) = 0 : The concentration is neither increasing nor decreasing. 69. f is increasing at a rate of unit of 𝑦 and the value of f is

s units per unit of 𝑥, f is increasing at a rate of r when 𝑥 = a and 𝑦 = b .

t units per

70. 𝑔(𝑎, 𝑏) = 4,000, 𝑔𝑥 (𝑎, 𝑏) = −400, 𝑔𝑦 (𝑎, 𝑏) = −300 71. the marginal cost of building an additional orbicus; zonars per unit 72. the number of additional citizens per additional transportation factory; citizens per lunar vehicle factory 73. Answers will vary. One example is 𝑓(𝑥, 𝑦) = −2𝑥 + 3𝑦. Others are 𝑓(𝑥, 𝑦) = −2𝑥 + 3𝑦 + 9 and 𝑓(𝑥, 𝑦) = 𝑥𝑦 − 3𝑥 + 2𝑦 + 10. 74. Answers will vary. Examples are 𝑓(𝑥, 𝑦, 𝑧) = 𝑥 + 𝑦 + 𝑧 and 𝑓(𝑥, 𝑦, 𝑧) = 2𝑥 − 𝑦 + 0.1𝑧. 75. a. 𝑏 is the 𝑧-intercept of the plane. 𝑚 is the slope of the intersection of the plane with the 𝑥𝑧-plane. 𝑛 is the slope of the intersection of the plane with the 𝑦𝑧-plane. b. Write 𝑧 = 𝑏 + 𝑟𝑥 + 𝑠𝑦. We are told that 𝜕𝑧∕𝜕𝑥 = 𝑚, so 𝑟 = 𝑚. Similarly, 𝑠 = 𝑛. Thus, 𝑧 = 𝑏 + 𝑚𝑥 + 𝑛𝑦. We are also told that the plane passes through (ℎ, 𝑘, 𝑙). Substituting gives 𝑙 = 𝑏 + 𝑚ℎ + 𝑛𝑘.This gives 𝑏 as 𝑙 − 𝑚ℎ − 𝑛𝑘.Substituting in the equation for 𝑧 therefore gives 𝑧 = 𝑙 − 𝑚ℎ − 𝑛𝑘 + 𝑚𝑥 + 𝑛𝑦 = 𝑙 + 𝑚(𝑥 − ℎ) + 𝑛(𝑦 − 𝑘), as required. 76. This follows from the previous exercise by substituting (ℎ, 𝑘, 𝑙) = (𝑎, 𝑏, 𝑓(𝑎, 𝑏)), 𝑚 = 𝑓𝑥 (𝑎, 𝑏), and 𝑛 = 𝑓𝑦 (𝑎, 𝑏).


Solutions Section 15.3 Section 15.3 1. 𝑃 : minimum; 𝑄: none of the above; 𝑅: maximum 2. 𝑃 : maximum; 𝑄: saddle point; 𝑅: maximum 3. 𝑃 : saddle point; 𝑄: maximum; 𝑅: none of the above 4. 𝑃 : none of the above; 𝑄: saddle point; 𝑅: minimum 5. Minimum 6. Saddle point 7. Neither 8. Maximum 9. Saddle point 10. Neither 11. 𝑓𝑥 = 2𝑥; 𝑓𝑦 = 2𝑦; 𝑓𝑥𝑥 = 2; 𝑓𝑦𝑦 = 2; 𝑓𝑥𝑦 = 0. 𝑓𝑥 = 0 when 𝑥 = 0; 𝑓𝑦 = 0 when 𝑦 = 0, so (0, 0) is the only critical point. 𝐻 = 4 and 𝑓𝑥𝑥 > 0, so 𝑓 has a minimum at (0, 0, 1). 12. 𝑓𝑥 = −2𝑥; 𝑓𝑦 = −2𝑦; 𝑓𝑥𝑥 = −2; 𝑓𝑦𝑦 = −2; 𝑓𝑥𝑦 = 0. 𝑓𝑥 = 0 when 𝑥 = 0; 𝑓𝑦 = 0 when 𝑦 = 0, so (0, 0) is the only critical point. 𝐻 = 4 and 𝑓𝑥𝑥 < 0, so 𝑓 has a maximum at (0, 0, 4). 13. 𝑔𝑥 = −2𝑥 − 1; 𝑔𝑦 = −2𝑦 + 1; 𝑔𝑥𝑥 = −2; 𝑔𝑦𝑦 = −2; 𝑔𝑥𝑦 = 0. 𝑔𝑥 = 0 when 𝑥 = −1∕2; 𝑔𝑦 = 0 when 𝑦 = 1∕2, so (−1∕2, 1∕2) is the only critical point. 𝐻 = 4 and 𝑔𝑥𝑥 < 0, so 𝑔 has a maximum at (−1∕2,1/2, 3/2). 14. 𝑔𝑥 = 2𝑥 + 1; 𝑔𝑦 = 2𝑦 − 1; 𝑔𝑥𝑥 = 2; 𝑔𝑦𝑦 = 2; 𝑔𝑥𝑦 = 0. 𝑔𝑥 = 0 when 𝑥 = −1∕2; 𝑔𝑦 = 0 when 𝑦 = 1∕2, so (−1∕2, 1∕2) is the only critical point. 𝐻 = 4 and 𝑔𝑥𝑥 > 0, so 𝑔 has a minimum at (−1∕2, 1∕2, −3∕2). 15. 𝑘𝑥 = 2𝑥 − 3𝑦; 𝑘𝑦 = −3𝑥 + 2𝑦 Critical points: 2𝑥 − 3𝑦 = 0; −3𝑥 + 2𝑦 = 0 Solution: (0, 0), which is the only critical point. 𝑘𝑥𝑥 = 2; 𝑘𝑥𝑦 = −3; 𝑘𝑦𝑦 = 2, so that 𝐻 = (2)(2) − (−3) 2 = −5. Since 𝐻 < 0, 𝑘 has a saddle point at (0, 0, 0). 16. 𝑘𝑥 = 2𝑥 − 𝑦; 𝑘𝑦 = −𝑥 + 4𝑦 Critical points: 2𝑥 − 𝑦 = 0; −𝑥 + 4𝑦 = 0 Solution: (0, 0), which is the only critical point. 𝑘𝑥𝑥 = 2; 𝑘𝑥𝑦 = −1; 𝑘𝑦𝑦 = 4, which gives 𝐻 = (2)(4) − (−1) 2 = 7. Since 𝐻 \gt 0 and 𝑘𝑥𝑥 > 0, 𝑘 has a minimum at (0, 0, 0). 17. 𝑓𝑥 = 2𝑥 + 2𝑦 − 2; 𝑓𝑦 = 2𝑥 + 4𝑦 + 4 Critical points: 2𝑥 + 2𝑦 − 2 = 0 ⇒ 𝑥 + 𝑦 = 1 2𝑥 + 4𝑦 + 4 = 0 ⇒ 𝑥 + 2𝑦 = −2 Solution: (4, −3), which is the only critical point.


Solutions Section 15.3 𝑓𝑥𝑥 = 2; 𝑓𝑥𝑦 = 2; 𝑓𝑦𝑦 = 4, so that 𝐻 = (2)(4) − 2 2 = 4. Since 𝐻 > 0 and 𝑓𝑥𝑥 > 0, 𝑓 has a minimum at (4, −3, −10). (The corresponding 𝑧-coordinate is 𝑓(4, −3) = −10.) 18. 𝑓𝑥 = 2𝑥 + 𝑦 + 3; 𝑓𝑦 = 𝑥 − 2𝑦 − 1 Critical points: 2𝑥 + 𝑦 + 3 = 0 ⇒ 2𝑥 + 𝑦 = −3 𝑥 − 2𝑦 − 1 = 0 ⇒ 𝑥 − 2𝑦 = 1 Solution: (−1, −1), which is the only critical point. 𝑓𝑥𝑥 = 2; 𝑓𝑥𝑦 = 1; 𝑓𝑦𝑦 = −2, which gives 𝐻 = (2)(−2) − 1 2 = −5. Since 𝐻 < 0, 𝑓 has a saddle point at (−1, −1, −1). (The corresponding 𝑧-coordinate is 𝑓(−1, −1) = −1.) 19. 𝑔𝑥 = −2𝑥 − 2𝑦 − 3; 𝑔𝑦 = −2𝑥 − 6𝑦 − 2 Critical points: −2𝑥 − 2𝑦 − 3 = 0 ⇒ −2𝑥 − 2𝑦 = 3 −2𝑥 − 6𝑦 − 2 = 0 ⇒ −2𝑥 − 6𝑦 = 2 Solution: (−7∕4, 1∕4), which is the only critical point. 𝑔𝑥𝑥 = −2; 𝑔𝑥𝑦 = −2; 𝑔𝑦𝑦 = −6, giving 𝐻 = (−2)(−6) − (−2) 2 = 8. Since 𝐻 > 0 and 𝑔𝑥𝑥 < 0, 𝑔 has a maximum at (−7∕4, 1∕4, 19∕8). (The corresponding 𝑧-coordinate is 𝑓(−7∕4, 1∕4) = 19∕8.) 20. 𝑔𝑥 = −2𝑥 − 2𝑦 + 1; 𝑔𝑦 = −2𝑥 + 2𝑦 − 4 Critical points: −2𝑥 − 2𝑦 + 1 = 0 ⇒ −2𝑥 − 2𝑦 = −1 −2𝑥 + 2𝑦 − 4 = 0 ⇒ −2𝑥 + 2𝑦 = 4 Solution: (−3∕4, 5∕4), which is the only critical point. 𝑔𝑥𝑥 = −2; 𝑔𝑥𝑦 = −2; 𝑔𝑦𝑦 = 2, which gives 𝐻 = (−2)(2) − (−2) 2 = −8. Since 𝐻 < 0, 𝑔 has a saddle point at (−3∕4, 5∕4, −23∕8). (The corresponding 𝑧-coordinate is𝑓(−3∕4, 5∕4) = −23∕8.) 21. ℎ𝑥 = 2𝑥𝑦 − 4𝑥; ℎ𝑦 = 𝑥 2 − 8𝑦; ℎ𝑥𝑥 = 2𝑦 − 4; ℎ𝑦𝑦 = −8; ℎ𝑥𝑦 = 2𝑥. ℎ𝑥 = 0 when 𝑥 = 0 or 𝑦 = 2; ℎ𝑦 = 0 when 𝑥 2 = 8𝑦. The two possibilities are 𝑥 = 0, so 𝑦 = 0;or 𝑦 = 2, so 𝑥 2 = 16 or 𝑥 = ±4. This gives three critical points: (0, 0), (−4, 2), and (4, 2). 𝐻(𝑥, 𝑦) = −8(2𝑦 − 4) − 4𝑥 2 = 32 − 16𝑦 − 4𝑥 2; 𝐻(0, 0) = 32 and ℎ𝑥𝑥 (0, 0) = −4 < 0;𝐻(−4, 2) = −64 = 𝐻(4, 2). Hence ℎ has a maximum at (0, 0, 0) and saddle points at (±4, 2, −16). 22. ℎ𝑥 = 2𝑥 − 𝑦 2; ℎ𝑦 = 2𝑦 − 2𝑥𝑦; ℎ𝑥𝑥 = 2; ℎ𝑦𝑦 = 2 − 2𝑥; ℎ𝑥𝑦 = −2𝑦. ℎ𝑥 = 0 when 2𝑥 = 𝑦 2; ℎ𝑦 = 0 when 𝑥 = 1 or 𝑦 = 0. If 𝑥 = 1, then 𝑦 = ±√2;if 𝑦 = 0, then 𝑥 = 0. This gives three critical points: (0, 0), (1, −√2), and (1, √2). 𝐻(𝑥, 𝑦) = 4 − 4𝑥 − 4𝑦 2; 𝐻(0, 0) = 4 and ℎ𝑥𝑥 (0, 0) = 2 > 0; 𝐻(1, ±√2) = −8 < 0. Hence ℎ has a relative minimum at (0, 0, −4) and saddle points at (1, ±√2, −3). 23. 𝑓𝑥 = 2𝑥 + 2𝑦 2; 𝑓𝑦 = 4𝑥𝑦 + 4𝑦 Critical points: 2𝑥 + 2𝑦 2 = 0 ⇒ 𝑥 = −𝑦 2 4𝑥𝑦 + 4𝑦 = 0 ⇒ 4𝑦(𝑥 + 1) = 0 ⇒ 𝑦 = 0 or 𝑥 = −1 𝑦 = 0 gives, using the first equation, 𝑥 = 0. 𝑥 = −1 gives, using the first equation, 𝑦 2 = 1, so 𝑦 = ±1. Critical points: (0, 0, 0), (−1, −1, 1), (−1, 1, 1) (We get the 𝑧-coordinate by substituting for 𝑥 and 𝑦 in the original function.) 𝑓𝑥𝑥 = 2; 𝑓𝑥𝑦 = 4𝑦; 𝑓𝑦𝑦 = 4𝑥 + 4


Solutions Section 15.3 (0, 0, 0) : 𝐻 = (2)(4) − 0 = 8 and 𝑓𝑥𝑥 > 0, giving a minimum at (0, 0, 0) (−1, ±1, 1) : 𝐻 = (2)(0) − (±4) 2 = −16, giving saddle points at (−1, ±1, 1) 2

24. 𝑓𝑥 = 2𝑥 + 2𝑥𝑦; 𝑓𝑦 = 𝑥 2 + 2𝑦 Critical points: 2𝑥 + 2𝑥𝑦 = 0 ⇒ 2𝑥(1 + 𝑦) = 0 ⇒ 𝑥 = 0 or 𝑦 = −1 𝑥 2 + 2𝑦 = 0 ⇒ 𝑥 2 = −2𝑦 𝑥 = 0 gives, using the second equation, 𝑦 = 0. 𝑦 = −1 gives, using the second equation, 𝑥 2 = 2, so 𝑥 = ±√2. Critical points: (0, 0, 0), (±√2, −1, 1) (We get the 𝑧-coordinate by substituting for 𝑥 and 𝑦 in the original function.) 𝑓𝑥𝑥 = 2 + 2𝑦; 𝑓𝑥𝑦 = 2𝑥; 𝑓𝑦𝑦 = 2 (0, 0, 0) : 𝐻 = (2)(2) − 0 2 = 4 and 𝑓𝑥𝑥 > 0, giving a minimum at (0, 0, 0). (±√2, −1, 1) : 𝐻 = (0)(2) − (±2√2) 2 = −8, giving saddle points at (±√2, −1, 1) 2

2

2

2

2

2

2

2

2

2

25. 𝑠𝑥 = 2𝑥𝑒 𝑥 +𝑦 ; 𝑠𝑦 = 2𝑦𝑒 𝑥 +𝑦 ; 𝑠𝑥𝑥 = (2 + 4𝑥 2)𝑒 𝑥 +𝑦 ; 𝑠𝑦𝑦 = (2 + 4𝑦 2)𝑒 𝑥 +𝑦 ; 𝑠𝑥𝑦 = 4𝑥𝑦𝑒 𝑥 +𝑦 . 𝑠𝑥 = 0 when 𝑥 = 0; 𝑠𝑦 = 0 when 𝑦 = 0; so the only critical point is (0, 0). 𝐻(0, 0) = 4 − 0 = 4 and 𝑠𝑥𝑥 (0, 0) = 2 > 0, so 𝑠 has a relative minimum at (0, 0, 1). 2

2

2

2

2

2

2

2

26. 𝑠𝑥 = −2𝑥𝑒 −(𝑥 +𝑦 ); 𝑠𝑦 = −2𝑦𝑒 −(𝑥 +𝑦 ); 𝑠𝑥𝑥 = (−2 + 4𝑥 2)𝑒 −(𝑥 +𝑦 ); 𝑠𝑦𝑦 = (−2 + 4𝑦 2)𝑒 −(𝑥 +𝑦 ); 2

2

𝑠𝑥𝑦 = 4𝑥𝑦𝑒 −(𝑥 +𝑦 ). 𝑠𝑥 = 0 when 𝑥 = 0; 𝑠𝑦 = 0 when 𝑦 = 0; so the only critical point is (0, 0). 𝐻(0, 0) = 4 − 0 = 4 and 𝑠𝑥𝑥 (0, 0) = −2 < 0, so 𝑠 has a relative maximum at (0, 0, 1). 27. 𝑡𝑥 = 4𝑥 3 + 8𝑦 2; 𝑡𝑦 = 16𝑥𝑦 + 8𝑦 3; 𝑡𝑥𝑥 = 12𝑥 2; 𝑡𝑦𝑦 = 16𝑥 + 24𝑦 2; 𝑡𝑥𝑦 = 16𝑦. 𝑡𝑥 = 0 when 𝑥 3 = −2𝑦 2 (notice that 𝑥 ≤ 0 in this case); 𝑡𝑦 = 0 when 𝑦 = 0 or 𝑦 2 = −2𝑥; if 𝑦 = 0, then 𝑥 = 0; if 𝑦 2 = −2𝑥, then 𝑥 3 = 4𝑥,so 𝑥 = 0 (and 𝑦 = 0) or 𝑥 = −2 and 𝑦 = ±2. This gives three critical points: (0, 0) and (−2, ±2). 𝐻(−2, ±2) = 3, 072 ± 32 > 0 and 𝑡𝑥𝑥 (−2, ±2) = 48 > 0. 𝐻(0, 0) = 0 so the second derivative test is inconclusive. We can see from the graph of 𝑡 that the origin is not a max, min, or saddle point. Or we can look at the slice along 𝑥 = 𝑦 (suggested by the graph) where 𝑡 = 3𝑥 4 + 8𝑥 3; this function increases as 𝑥 approaches 0 and then increases again as 𝑥 becomes larger than 0. So, 𝑡 has two minima at (−2, ±2, −16) and (0, 0) is a critical point that is not a relative extremum or saddle point. 28. 𝑡𝑥 = 3𝑥 2 − 3𝑦; 𝑡𝑦 = −3𝑥 + 3𝑦 2; 𝑡𝑥𝑥 = 6𝑥; 𝑡𝑦𝑦 = 6𝑦; 𝑡𝑥𝑦 = −3. 𝑡𝑥 = 0 when 𝑦 = 𝑥 2; 𝑡𝑦 = 0 when 𝑥 = 𝑦 2; substituting gives 𝑥 = 𝑥 4, so 𝑥 = 0 or 1. This gives two critical points, (0, 0) and (1, 1). 𝐻(0, 0) = −9 < 0; 𝐻(1, 1) = 27 and 𝑡𝑥𝑥 (1, 1) = 6 > 0. So, 𝑡 has a saddle point at (0, 0, 0) and a minimum at (1, 1, −1). 29. 𝑓𝑥 = 2𝑥; 𝑓𝑦 = 1 − 𝑒 𝑦; 𝑓𝑥𝑥 = 2; 𝑓𝑦𝑦 = −𝑒 𝑦; 𝑓𝑥𝑦 = 0. 𝑓𝑥 = 0 when 𝑥 = 0; 𝑓𝑦 = 0 when 𝑦 = 0. This gives one critical point, (0, 0). 𝐻(0, 0) = −2, so 𝑓 has a saddle point at (0, 0, −1) and no other critical points. 30. 𝑓𝑥 = 𝑒 𝑦; 𝑓𝑦 = 𝑥𝑒 𝑦; 𝑓𝑥𝑥 = 0; 𝑓𝑦𝑦 = 𝑥𝑒 𝑦; 𝑓𝑥𝑦 = 𝑒 𝑦. 𝑓𝑥 = 0 is impossible. So 𝑓 has no critical points. 2

2

31. 𝑓𝑥 = −(2𝑥 + 2)𝑒 −(𝑥 +𝑦 +2𝑥); 2

2

2

2

𝑓𝑦 = −2𝑦𝑒 −(𝑥 +𝑦 +2𝑥); 2

2

2

2

𝑓𝑥𝑥 = (4𝑥 2 + 8𝑥 + 2)𝑒 −(𝑥 +𝑦 +2𝑥); 𝑓𝑦𝑦 = (4𝑦 2 − 2)𝑒 −(𝑥 +𝑦 +2𝑥); 𝑓𝑥𝑦 = 2𝑦(2𝑥 + 2)𝑒 −(𝑥 +𝑦 +2𝑥). 𝑓𝑥 = 0 when 𝑥 = −1; 𝑓𝑦 = 0 when 𝑦 = 0. This gives one critical point, (−1, 0). 𝐻(−1, 0) = 4𝑒 2 > 0 and 𝑓𝑥𝑥 (−1, 0) = −2𝑒 < 0, so 𝑓 has a relative maximum at (−1, 0, 𝑒).


Solutions Section 15.3 32. 𝑓𝑥 = −(2𝑥 − 2)𝑒

−(𝑥 2+𝑦 2−2𝑥)

;

2

2

2

2

𝑓𝑦 = −2𝑦𝑒 −(𝑥 +𝑦 −2𝑥); 2

2

2

2

𝑓𝑥𝑥 = (4𝑥 2 − 8𝑥 + 2)𝑒 −(𝑥 +𝑦 −2𝑥); 𝑓𝑦𝑦 = (4𝑦 2 − 2)𝑒 −(𝑥 +𝑦 −2𝑥); 𝑓𝑥𝑦 = 2𝑦(2𝑥 − 2)𝑒 −(𝑥 +𝑦 −2𝑥). 𝑓𝑥 = 0 when 𝑥 = 1; 𝑓𝑦 = 0 when 𝑦 = 0. This gives one critical point, (1, 0). 𝐻(1, 0) = 4𝑒 2 > 0 and 𝑓𝑥𝑥 (1, 0) = −2𝑒 < 0, so 𝑓 has a maximum at (1, 0, 𝑒). 2 2 4 4 ; 𝑓𝑦 = 𝑥 − 2 ; 𝑓𝑥𝑥 = 3 ; 𝑓𝑦𝑦 = 3 ; 𝑓𝑥𝑦 = 1 2 𝑥 𝑦 𝑥 𝑦 2 2 1 4 𝑓𝑥 = 0 when 𝑦 = 2 ; 𝑓𝑦 = 0 when 𝑥 = 2 = 𝑥 , 𝑥 = 2 1∕3 2 𝑥 𝑦 (𝑥 = 0 is excluded because it is not in the domain of 𝑓). The corresponding value of 𝑦 is 𝑦 = 2 1∕3. This gives one critical point, (2 1∕3, 2 1∕3). 𝐻(2 1∕3, 2 1∕3) = 3 and 𝑓𝑥𝑥 (2 1∕3, 2 1∕3) = 2 > 0, so 𝑓 has a minimum at (2 1∕3, 2 1∕3, 3(2 2∕3)). 33. 𝑓𝑥 = 𝑦 −

4 2 8 4 ; 𝑓𝑦 = 𝑥 − 2 ; 𝑓𝑥𝑥 = 3 ; 𝑓𝑦𝑦 = 3 ; 𝑓𝑥𝑦 = 1 2 𝑥 𝑦 𝑥 𝑦 4 2 1 4 𝑓𝑥 = 0 when 𝑦 = 2 ; 𝑓𝑦 = 0 when 𝑥 = 2 = 𝑥 ; 𝑥 = 2 8 𝑥 𝑦 (𝑥 = 0 is excluded because it is not in the domain of 𝑓). The corresponding value of 𝑦 is 𝑦 = 1. This gives one critical point (2, 1). 𝐻(2, 1) = 3 and 𝑓𝑥𝑥 (2, 1) = 1 > 0, so 𝑓 has a minimum at (2, 1, 6). 34. 𝑓𝑥 = 𝑦 −

2 2 ; 𝑔𝑦 = 2𝑦 − 2 𝑥 2𝑦 𝑥𝑦 4 4 2 𝑔𝑥𝑥 = 2 + 3 ; 𝑔𝑦𝑦 = 2 + 3 ; 𝑔𝑥𝑦 = 2 2 . 𝑥 𝑦 𝑥𝑦 𝑥 𝑦 1 1 𝑔𝑥 = 0 when 𝑦 = 3 ; 𝑔𝑦 = 0 when 𝑥 = 3 = 𝑥 9, 𝑥 = ±1 (𝑥 = 0 is excluded because it is not in the 𝑥 𝑦 domain of 𝑓). This gives two critical points, (1, 1) and (−1, −1). 𝐻(1, 1) = 32 and 𝑔𝑥𝑥 (1, 1) = 6 > 0; 𝐻(−1, −1) = 32 and 𝑔𝑥𝑥 (−1, −1) = 6 > 0. So, 𝑔 has minima at (1, 1, 4) and (−1, −1, 4). 35. 𝑔𝑥 = 2𝑥 −

3 3 6 6 3 ; 𝑔𝑦 = 3𝑦 2 − 2 ; 𝑔𝑥𝑥 = 6𝑥 + 3 ; 𝑔𝑦𝑦 = 6𝑦 + 3 ; 𝑔𝑥𝑦 = 2 2 2 𝑥 𝑦 𝑥𝑦 𝑥 𝑦 𝑥𝑦 𝑥 𝑦 1 1 16 𝑔𝑥 = 0 when 𝑦 = 4 ; 𝑔𝑦 = 0 when 𝑥 = 4 = 𝑥 , 𝑥 = 1 (𝑥 = 0 is excluded because it is not in the domain 𝑥 𝑦 of 𝑓). This gives one critical point, (1, 1). 𝐻(1, 1) = 135 and 𝑔𝑥𝑥 (1, 1) = 12 > 0, so 𝑔 has a minimum at (1, 1, 5). 36. 𝑔𝑥 = 3𝑥 2 −

37. 𝑓 has an absolute minimum at (0, 0, 1): 𝑥 2 + 𝑦 2 + 1 ≥ 1 for all 𝑥 and 𝑦 because 𝑥 2 + 𝑦 2 ≥ 0. 38. 𝑓 has an absolute maximum at (0, 0, 1): 4 − (𝑥 2 + 𝑦 2) ≤ 4 for all 𝑥 and 𝑦 because 𝑥 2 + 𝑦 2 ≥ 0. 39. The relative maximum at (0, 0, 0) is not absolute. For example, ℎ(10, 10) = 400 > ℎ(0, 0). 40. The relative minimum at (0, 0, −4) is not absolute. For example, ℎ(10, 10) = −804 < ℎ(0, 0). 41. 𝑆𝑥 = 0.2 + 1.6𝑥 − 0.8𝑦; 𝑆𝑦 = −0.4 + 𝑦 − 0.8𝑥; 𝑆𝑥𝑥 = 1.6; 𝑆𝑦𝑦 = 1; 𝑆𝑥𝑦 = −0.8 𝑆𝑥 = 0 and 𝑆𝑦 = 0 when (𝑥, 𝑦) = (0.125, 0.5) 𝐻(0.125, 0.5) = 0.96 and 𝑆𝑥𝑥 (0.125, 0.5) = 1.6 > 0, so 𝑆 has a minimum at (0.125, 0.5), where the value of 𝑆 is 0.2(0.125) − 0.4(0.5) + 0.8(0.125) 2 + 0.5(0.5) 2 − 0.8(0.125)(0.5) + 0.7 = 0.6125 Thus, at least 61.25% of all Subaru owners would choose another new Subaru, and this lowest loyalty would occur if 12.5% of Toyota users and 50% of Honda owners remained loyal to their brands.


Solutions Section 15.3 42. 𝑆𝑥 = −0.1 − 1.6𝑥 + 0.8𝑦; 𝑆𝑦 = 0.5 − 𝑦 + 0.8𝑥; 𝑆𝑥𝑥 = −1.6; 𝑆𝑦𝑦 = −1; 𝑆𝑥𝑦 = 0.8 𝑆𝑥 = 0 and 𝑆𝑦 = 0 when (𝑥, 𝑦) = (0.3125, 0.75) 𝐻(0.3125, 0.75) = 0.96 and 𝑆𝑥𝑥 (0.3125, 0.75) = −1.6 < 0, so 𝑆 has a maximum at (0.3125, 0.75), where the value of 𝑆 is −0.1(0.3125) + 0.5(0.75) − 0.8(0.3125) 2 − 0.5(0.75) 2 + 0.8(0.3125)(0.75) + 0.3 ≈ 0.472 Thus, at most 47.2% of all Subaru owners would choose another new Subaru, and this highest loyalty would occur if 31.25% of Toyota users and 75% of Honda owners remained loyal to their brands. 43. The subsidy is 𝑆(𝑥, 𝑦) = 500𝑥 + 100𝑦, so the net cost is 𝑁(𝑥, 𝑦) = 𝐶(𝑥, 𝑦) − 𝑆(𝑥, 𝑦) = 4,000 + 100𝑥 2 + 50𝑦 2 − 500𝑥 − 100𝑦. 𝑁𝑥 = 200𝑥 − 500; 𝑁𝑦 = 100𝑦 − 100; 𝑁𝑥𝑥 = 200; 𝑁𝑦𝑦 = 100; 𝑁𝑥𝑦 = 0. 𝑁𝑥 = 0 when 𝑥 = 2.5; 𝑁𝑦 = 0 when 𝑦 = 1. 𝐻 = 20,000 and 𝑁𝑥𝑥 = 100 > 0, so 𝑁 has a maximum at (2.5, 1). The firm should remove 2.5 pounds of sulfur and 1 pound of lead per day. 44. The subsidy is 𝑆(𝑥, 𝑦) = 100𝑥 + 500𝑦, so the net cost is 𝑁(𝑥, 𝑦) = 𝐶(𝑥, 𝑦) − 𝑆(𝑥, 𝑦) = 2,000 + 200𝑥 2 + 100𝑦 2 − 100𝑥 − 500𝑦. 𝑁𝑥 = 400𝑥 − 100; 𝑁𝑦 = 200𝑦 − 500; 𝑁𝑥𝑥 = 400; 𝑁𝑦𝑦 = 200; 𝑁𝑥𝑦 = 0. 𝑁𝑥 = 0 when 𝑥 = 0.25; 𝑁𝑦 = 0 when 𝑦 = 2.5. 𝐻 = 80,000 and 𝑁𝑥𝑥 = 400 > 0, so 𝑁 has a maximum at (0.25, 2.5). The firm should remove 0.25 pounds of sulfur and 2.5 pounds of lead per day. 45. The total revenue is 𝑅 = 𝑝1 𝑞1 + 𝑝2 𝑞2 = 100,000𝑝1 − 100𝑝 + 10𝑝1 𝑝2 + 150,000𝑝2 + 10𝑝1 𝑝2 − 100𝑝 = 100,000𝑝1 + 150,000𝑝2 − 100𝑝 + 20𝑝1 𝑝2 − 100𝑝. For convenience, write 𝑅1 for 𝜕𝑅∕𝜕𝑝1 and 𝑅2 for 𝜕𝑅∕𝜕𝑝2 . Then 𝑅1 = 100,000 − 200𝑝1 + 20𝑝2 ; 𝑅2 = 150,000 + 20𝑝1 − 200𝑝2 ; 𝑅11 = −200; 𝑅22 = −200; 𝑅12 = 20. 𝑅1 = 0 and 𝑅2 = 0 when (𝑝1 , 𝑝2 ) = (580.81, 808.08). 𝐻 = 39,600 and 𝑅11 = −200 < 0, so 𝑅 has a maximum at this critical point. You should charge $580.81 for the Ultra Mini and $808.08 for the Big Stack. 46. The total revenue is 𝑅 = 𝑝1 𝑞1 + 𝑝2 𝑞2 = 100,000𝑝1 − 100𝑝 + 𝑝1 𝑝2 + 150,000𝑝2 + 𝑝1 𝑝2 − 100𝑝 = 100,000𝑝1 + 150,000𝑝2 − 100𝑝 + 2𝑝1 𝑝2 − 100𝑝. For convenience, write 𝑅1 for 𝜕𝑅∕𝜕𝑝1 and 𝑅2 for 𝜕𝑅∕𝜕𝑝2 . Then 𝑅1 = 100,000 − 200𝑝1 + 2𝑝2 ; 𝑅2 = 150,000 + 2𝑝1 − 200𝑝2 ; 𝑅11 = −200; 𝑅22 = −200; 𝑅12 = 2. 𝑅1 = 0 and 𝑅2 = 0 when (𝑝1 , 𝑝2 ) = (507.55, 755.08). 𝐻 = 39,996 and 𝑅11 = −200 < 0, so 𝑅 has a maximum at this critical point. You should charge $507.55 for the Ultra Mini and $755.08 for the Big Stack. 47. Let 𝑙 =length, 𝑤 =width, and ℎ =height. We are told that 𝑙 + 𝑤 + ℎ ≤ 62; for the largest possible volume we will want 𝑙 + 𝑤 + ℎ = 62, so 𝑙 = 62 − 𝑤 − ℎ. The volume is 𝑉 = 𝑙𝑤ℎ = (62 − 𝑤 − ℎ)𝑤ℎ = 62𝑤ℎ − 𝑤 2ℎ − 𝑤ℎ 2. 𝑉𝑤 = 62ℎ − 2𝑤ℎ − ℎ 2; 𝑉ℎ = 62𝑤 − 𝑤 2 − 2𝑤ℎ; 𝑉𝑤𝑤 = −2ℎ; 𝑉ℎℎ = −2𝑤; 𝑉𝑤ℎ = 62 − 2𝑤 − 2ℎ. 𝑉𝑤 = 0 when 62 − 2𝑤 − ℎ = 0 (ℎ = 0 is not in the domain); 𝑉ℎ = 0 when 62 − 𝑤 − 2ℎ = 0; this occurs when 𝑤 = ℎ = 62∕3 ≈ 20.67. 𝐻 ≈ 1,280 and 𝑉𝑤𝑤 \lt 0, so 𝑉 has a maximum at this critical point. The largest-volume bag has dimensions 𝑙 = 𝑤 = ℎ ≈ 20.67 in and volume ≈ 8,827 cubic inches. 48. Let 𝑙 =length, 𝑤 =width, and ℎ =height. We are told that 𝑙 + 𝑤 + ℎ ≤ 45; for the largest possible volume we will want 𝑙 + 𝑤 + ℎ = 45, so 𝑙 = 45 − 𝑤 − ℎ. The volume is 𝑉 = 𝑙𝑤ℎ = (45 − 𝑤 − ℎ)𝑤ℎ = 45𝑤ℎ − 𝑤 2ℎ − 𝑤ℎ 2. 𝑉𝑤 = 45ℎ − 2𝑤ℎ − ℎ 2; 𝑉ℎ = 45𝑤 − 𝑤 2 − 2𝑤ℎ; 𝑉𝑤𝑤 = −2ℎ; 𝑉ℎℎ = −2𝑤; 𝑉𝑤ℎ = 45 − 2𝑤 − 2ℎ. 𝑉𝑤 = 0 when 45 − 2𝑤 − ℎ = 0 (ℎ = 0 is not in the domain); 𝑉ℎ = 0 when 45 − 𝑤 − 2ℎ = 0; this occurs when 𝑤 = ℎ = 15. 𝐻 = 675 and 𝑉𝑤𝑤 \lt 0, so 𝑉 has a maximum at this critical point. The largest-volume bag has dimensions 𝑙 = 𝑤 = ℎ = 15 in and volume = 3,375 cubic inches.


Solutions Section 15.3 49. Let 𝑙 =length, 𝑤 =width, and ℎ =height. We are told that 𝑙 + 2(𝑤 + ℎ) ≤ 108; for the largest possible volume we will want 𝑙 + 2(𝑤 + ℎ) = 108, so 𝑙 = 108 − 2(𝑤 + ℎ). The volume is 𝑉 = 𝑙𝑤ℎ = (108 − 2𝑤 − 2ℎ)𝑤ℎ = 108𝑤ℎ − 2𝑤 2ℎ − 2𝑤ℎ 2. 𝑉𝑤 = 108ℎ − 4𝑤ℎ − 2ℎ 2; 𝑉ℎ = 108𝑤 − 2𝑤 2 − 4𝑤ℎ; 𝑉𝑤𝑤 = −4ℎ; 𝑉ℎℎ = −4𝑤; 𝑉𝑤ℎ = 108 − 4𝑤 − 4ℎ. 𝑉𝑤 = 0 when 108 − 4𝑤 − 2ℎ = 0; 𝑉ℎ = 0 when 108 − 2𝑤 − 4ℎ = 0; this occurs when 𝑤 = ℎ = 18. 𝐻 = 3,888 and 𝑉𝑤𝑤 < 0, so 𝑉 has a maximum at this critical point. The corresponding length is 𝑙 = 108 − 2(𝑤 + ℎ) = 36. So, the largest-volume package has dimensions 18 in × 18 in × 36 in and volume = 11,664 cubic inches. 50. Let 𝑙 =length, 𝑤 =width, and ℎ =height. We are told that 𝑙 + 2(𝑤 + ℎ) ≤ 165. For the largest possible volume we will want 𝑙 + 2(𝑤 + ℎ) = 165, so 𝑙 = 165 − 2(𝑤 + ℎ) = 165 − 2𝑤 − 2ℎ. The volume is 𝑉 = 𝑙𝑤ℎ = (165 −2𝑤 − 2ℎ)𝑤ℎ = 165𝑤ℎ − 2𝑤 2ℎ − 2𝑤ℎ 2. 𝑉𝑤 = 165ℎ − 4𝑤ℎ − 2ℎ 2; 𝑉ℎ = 165𝑤 − 2𝑤 2 − 4𝑤ℎ; 𝑉𝑤𝑤 = −4ℎ; 𝑉ℎℎ = −4𝑤; 𝑉𝑤ℎ = 165 − 4𝑤 − 4ℎ 𝑉𝑤 = 0 when 165 − 4𝑤 − 2ℎ = 0. 𝑉ℎ = 0 when 165 − 2𝑤 − 4ℎ = 0. This occurs when 𝑤 = ℎ = 165∕6 = 27.5. The Hessian is 𝐻 = 9,075 and 𝑉𝑤𝑤 < 0, so 𝑉 has a maximum at this critical point. The corresponding length is 𝑙 = 165 − 2(𝑤 + ℎ) = 55. So, the largest-volume package has dimensions 27.5 in × 27.5 in × 55 in and volume = 41,593.75 cubic inches. Note that the length is less than 108 inches, so that constraint is also satisfied. 51.

52.

53. The function graphed below has a relative max at (0, 0, −1), has the unit circle 𝑥 2 + 𝑦 2 = 1 as a vertical asymptote, and takes on values larger than −1 as 𝑥 2 + 𝑦 2 gets large. The particular function 1 graphed is 𝑓(𝑥, 𝑦) = − 2 . | 𝑥 + 𝑦 2 − 1|


Solutions Section 15.3

54.

55. 𝐻 must be positive. 56. 𝑓(𝑥, 𝑦) = 𝑥 4 + 𝑦 4has a minimum at (0, 0) and 𝐻 = 0. 57. No. For there to be a relative maximum at (𝑎, 𝑏), all vertical planes through (𝑎, 𝑏) should yield a curve with a relative maximum at (𝑎, 𝑏). It could happen that a slice by another vertical plane through (𝑎, 𝑏) (such as 𝑥 − 𝑎 = 𝑦 − 𝑏) does not yield a curve with a relative maximum at (𝑎, 𝑏). [An example is 𝑓(𝑥, 𝑦) = √𝑥𝑦 − 𝑥 2 − 𝑦 2, at the point (0, 0). Look at the slices through 𝑥 = 0, 𝑦 = 0 and 𝑦 = 𝑥.] 58. Yes. In fact, for there to be a relative maximum at (𝑎, 𝑏), all vertical planes through (𝑎, 𝑏) should yield a curve with a relative maximum at (𝑎, 𝑏), because 𝑓(𝑎, 𝑏) ≥ 𝑓(𝑥, 𝑦) for (𝑥, 𝑦) in some neighborhood of (𝑎, 𝑏). (𝑥 + 𝑦)𝐶𝑥 − 𝐶 𝜕 𝐶 𝐶 ( )= . If this is zero, then (𝑥 + 𝑦)𝐶𝑥 = 𝐶, or 𝐶𝑥 = 𝐶. =ˉ 𝜕𝑥 𝑥 + 𝑦 𝑥+𝑦 (𝑥 + 𝑦) 2 Similarly, if ˉ 𝐶 𝑦 = 0, then 𝐶𝑦 = ˉ 𝐶 . This is reasonable because if the average cost is decreasing with increasing 𝑥, then the average cost is greater than the marginal cost 𝐶𝑥 . Similarly, if the average cost is increasing with increasing 𝑥, then the average cost is less than the marginal cost 𝐶𝑥 . Thus, if the average cost is stationary with increasing 𝑥, then the average cost equals the marginal cost 𝐶𝑥 . (The situation is similar for the case of increasing 𝑦.) 59. ˉ 𝐶𝑥 =

(𝑥 + 𝑦)𝑃𝑥 − 𝑃 𝜕 𝑃 𝑃 . If this is zero, then (𝑥 + 𝑦)𝑃𝑥 = 𝑃 , or 𝑃𝑥 = 𝑃. ( )= =ˉ 𝜕𝑥 𝑥 + 𝑦 𝑥+𝑦 (𝑥 + 𝑦) 2 Similarly, if ˉ 𝑃 𝑦 = 0, then 𝑃𝑦 = ˉ 𝑃 . This is reasonable because, if the average profit is decreasing with increasing 𝑥, then the average profit is greater than the marginal profit 𝑃𝑥 . Similarly, if the average profit 60. ˉ 𝑃𝑥 =


Solutions Section 15.3 is increasing with increasing 𝑥, the average profit is less than the marginal profit 𝑃𝑥 . Thus, if the average profit is stationary with increasing 𝑥, then the average profit equals the marginal profit 𝑃𝑥 . (Similarly for the case of increasing 𝑦.) 61. The equation of the tangent plane at the point (𝑎, 𝑏) is 𝑧 = 𝑓(𝑎, 𝑏) + 𝑓𝑥 (𝑎, 𝑏)(𝑥 − 𝑎) + 𝑓𝑦 (𝑎, 𝑏)(𝑦 − 𝑏). If 𝑓 has a relative extremum at (𝑎, 𝑏), then 𝑓𝑥 (𝑎, 𝑏) = 0 = 𝑓𝑦 (𝑎, 𝑏). Substituting these into the equation of the tangent plane gives 𝑧 = 𝑓(𝑎, 𝑏), a constant. But the graph of 𝑧 = 𝑐𝑜𝑛𝑠𝑡𝑎𝑛𝑡 is a plane parallel to the 𝑥𝑦-plane. 62. The equation of the tangent plane at the point (𝑎, 𝑏) is 𝑧 = 𝑓(𝑎, 𝑏) + 𝑓𝑥 (𝑎, 𝑏)(𝑥 − 𝑎) + 𝑓𝑦 (𝑎, 𝑏)(𝑦 − 𝑏). If 𝑓 has a saddle point at (𝑎, 𝑏), then 𝑓𝑥 (𝑎, 𝑏) = 0 = 𝑓𝑦 (𝑎, 𝑏). Substituting these into the equation of the tangent plane gives 𝑧 = 𝑓(𝑎, 𝑏), a constant. But the graph of 𝑧 = 𝑐𝑜𝑛𝑠𝑡𝑎𝑛𝑡 is a plane parallel to the 𝑥𝑦-plane.


Solutions Section 15.4 Section 15.4 1. The objective function is 1 − 𝑥 2 − 𝑦 2 − 𝑧 2. Substituting the constraint equation 𝑧 = 2𝑦 gives the objective function as ℎ(𝑥, 𝑦) = 1 − 𝑥 2 − 𝑦 2 − (2𝑦) 2 = 1 − 𝑥 2 − 5𝑦 2 ℎ𝑥 = −2𝑥, ℎ𝑦 = −10𝑦. Critical points: −2𝑥 = 0 −10𝑦 = 0 (𝑥, 𝑦) = (0, 0) is the only critical point. ℎ𝑥𝑥 = −2, ℎ𝑥𝑦 = 0, ℎ𝑦𝑦 = −10 𝐻 = ℎ𝑥𝑥 ℎ𝑦𝑦 − (ℎ𝑥𝑦 ) 2 = 20 > 0 Since ℎ𝑥𝑥 < 0, the critical point is a local maximum. That it is an absolute maximum can be seen by considering the graph of ℎ(𝑥, 𝑦). The corresponding value of the objective function is ℎ(0, 0) = 1 − (0) 2 − 5(0) 2 = 1. When 𝑥 = 0 and 𝑦 = 0, 𝑧 = 2𝑦 = 0 as well, so 𝑓 has an absolute maximum of 1 at the point (0, 0, 0). 2. The objective function is 𝑥 2 + 𝑦 2 + 𝑧 2 − 2. Substituting the constraint equation 𝑥 = 𝑦 gives the objective function as ℎ(𝑥, 𝑦) = 𝑥 2 + 𝑦 2 + 𝑧 2 − 2 = 2𝑥 2 + 𝑧 2 − 2 ℎ𝑥 = 4𝑥, ℎ𝑧 = 2𝑧. Critical points: 4𝑥 = 0 2𝑧 = 0 (𝑥, 𝑧) = (0, 0) is the only critical point. ℎ𝑥𝑥 = 4, ℎ𝑥𝑧 = 0, ℎ𝑧𝑧 = 2 𝐻 = ℎ𝑥𝑥 ℎ𝑧𝑧 − (ℎ𝑥𝑧 ) 2 = 8 > 0 Since ℎ𝑥𝑥 > 0, the critical point is a local minimum. That it is an absolute minimum can be seen by considering the graph of ℎ(𝑥, 𝑦). The corresponding value of the objective function is ℎ(0, 0) = 2(0) 2 + (0) 2 −2 = −2. When 𝑥 = 0 and 𝑧 = 0, 𝑦 = 𝑥 = 0 as well, so 𝑓 has an absolute minimum of −2 at the point (0, 0, 0). 3. The objective function is 1 − 𝑥 2 − 𝑥 − 𝑦 2 + 𝑦 − 𝑧 2 + 𝑧. Substituting the constraint equation 𝑦 = 3𝑥 gives the objective function as ℎ(𝑥, 𝑧) = 1 − 𝑥 2 − 𝑥 − 𝑦 2 + 𝑦 − 𝑧 2 + 𝑧 = 1 − 10𝑥 2 + 2𝑥 − 𝑧 2 + 𝑧 ℎ𝑥 = −20𝑥 + 2, ℎ𝑧 = −2𝑧 + 1. Critical points: ⇒ 𝑥 = 1∕10 −20𝑥 + 2 = 0 ⇒ 𝑧 = 1∕2 −2𝑧 + 1 = 0 (𝑥, 𝑧) = (1∕10, 1∕2) is the only critical point. ℎ𝑥𝑥 = −20, ℎ𝑥𝑧 = 0, ℎ𝑧𝑧 = −2 𝐻 = ℎ𝑥𝑥 ℎ𝑧𝑧 − (ℎ𝑥𝑧 ) 2 = 40 > 0 Since ℎ𝑥𝑥 < 0, the critical point is a local maximum. That it is an absolute maximum can be seen by considering the graph of ℎ(𝑥, 𝑦) = 1 − 10𝑥 2 + 2𝑥 − 𝑧 2 + 𝑧. The corresponding value of the objective function is ℎ(1∕10, 1∕2) = 1 − 10∕100 + 2∕10 − 1∕4 + 1∕2 = 1.35. When 𝑥 = 1∕10 and 𝑧 = 1∕2, 𝑦 = 3𝑥 = 3∕10, so 𝑓 has an absolute maximum of 1.35 at the point (1∕10, 3∕10, 1∕2). 4. The objective function is 2𝑥 2 + 2𝑥 + 𝑦 2 − 𝑦 + 𝑧 2 − 𝑧 − 1. Substituting the constraint equation 𝑧 = 2𝑦 gives the objective function as ℎ(𝑥, 𝑦) = 2𝑥 2 + 2𝑥 + 𝑦 2 − 𝑦 + 𝑧 2 − 𝑧 − 1 = 2𝑥 2 + 2𝑥 + 5𝑦 2 − 3𝑦 − 1 ℎ𝑥 = 4𝑥 + 2, ℎ𝑦 = 10𝑦 − 3.


Solutions Section 15.4 Critical points: 4𝑥 + 2 = 0

𝑥 = −1∕2

10𝑦 − 3 = 0 ⇒ 𝑦 = 3∕10 (𝑥, 𝑦) = (−1∕2, 3∕10) is the only critical point. ℎ𝑥𝑥 = 4, ℎ𝑥𝑦 = 0, ℎ𝑦𝑦 = 10 𝐻 = ℎ𝑥𝑥 ℎ𝑧𝑧 − (ℎ𝑥𝑧 ) 2 = 40 > 0 Since ℎ𝑥𝑥 > 0, the critical point is a local minimum. That it is an absolute minimum can be seen by considering the graph of ℎ(𝑥, 𝑦) = 2𝑥 2 + 2𝑥 + 5𝑦 2 − 3𝑦 − 1. The corresponding value of the objective function is ℎ(−1∕2, 3∕10) = 1∕2 − 1 + 45∕100 − 9∕10 − 1 = −1.95. When 𝑥 = −1∕2 and 𝑦 = 1∕2, 𝑧 = 2𝑦 = 3∕5, so 𝑓 has an absolute minimum of −0.25 at the point (−1∕2, 3∕10, 3∕5). 5. The objective function is 𝑥𝑦 + 4𝑥𝑧 + 2𝑦𝑧. Solving the constraint equation 𝑥𝑦𝑧 = 1 for 𝑧 gives 𝑧 =

1 . 𝑥𝑦

1 1 4 2 + 2𝑦 = 𝑥𝑦 + + 𝑥𝑦 𝑥𝑦 𝑦 𝑥 2 4 𝑆𝑥 = 𝑦 − 2 , 𝑆𝑦 = 𝑥 − 2 𝑥 𝑦 Critical points: 2 𝑦− 2 =0 𝑥 4 𝑥− 2 =0 𝑦 Substituting the first equation in the second gives 4 𝑥− =0 (2∕𝑥 2) 2 𝑥 − 𝑥4 = 0 𝑥(1 − 𝑥 3) = 0 ⇒ 𝑥 = 1. (We reject 𝑥 = 0 because 𝑥 > 0). 2 Substituting 𝑥 = 1 into the equation 𝑦 − 2 = 0 gives 𝑦 = 2. (𝑥, 𝑦) = (1, 2) is the only critical point. 𝑥 𝑆𝑥𝑥 = 4∕𝑥 3, 𝑆𝑥𝑦 = 1, 𝑆𝑦𝑦 = 8∕𝑦 3 𝑆𝑥𝑥 (1, 2) = 4∕1 3 = 4 𝑆𝑥𝑦 (1, 2) = 1, 𝑆𝑦𝑦 (1, 2) = 8∕2 3 = 1 𝐻 = 𝑆𝑥𝑥 𝑆𝑦𝑦 − (𝑆𝑥𝑦 ) 2 = 4 − 1 = 3 > 0 Since 𝑆𝑥𝑥 (1, 2) > 0, the critical point is a local minimum. That it is an absolute minimum can be seen by 4 2 considering the graph of 𝑆(𝑥, 𝑦) = 𝑥𝑦 + + for 𝑥 > 0, 𝑦 > 0. The corresponding value of the objective 𝑦 𝑥 function is 4 2 𝑆(1, 2) = (1)(2) + + = 6. 2 1 When 𝑥 = 1 and 𝑦 = 2, 𝑧 = 1∕𝑥𝑦 = 1∕2, so 𝑆 has an absolute minimum of 6 at (1, 2, 1∕2). 𝑆 = 𝑥𝑦 + 4𝑥

6. The objective function is 𝑥𝑦 + 𝑥𝑧 + 𝑦𝑧. Solving the constraint equation 𝑥𝑦𝑧 = 2 for 𝑧 gives 𝑧 = 2 2 2 2 +𝑦 = 𝑥𝑦 + + 𝑥𝑦 𝑥𝑦 𝑦 𝑥 2 2 𝑆𝑥 = 𝑦 − 2 , 𝑆𝑦 = 𝑥 − 2 𝑥 𝑦 Critical points: 2 𝑦− 2 =0 𝑥 2 𝑥− 2 =0 𝑦 𝑆 = 𝑥𝑦 + 𝑥

2 . 𝑥𝑦


Solutions Section 15.4 Substituting the first equation in the second gives 2 𝑥− =0 (2∕𝑥 2) 2 𝑥 − 𝑥 4∕2 = 0 𝑥(1 − 𝑥 3∕2) = 0 𝑥3 = 2 ⇒ 𝑥 = 2 1∕3 (We reject 𝑥 = 0 because 𝑥 > 0). 2 Substituting 𝑥 = 2 1∕3 into the equation y− 2 = 0 gives 𝑦 = 2 1∕3. 𝑥 (𝑥, 𝑦) = (2 1∕3, 2 1∕3) is the only critical point. 𝑆𝑥𝑥 = 4∕𝑥 3, 𝑆𝑥𝑦 = 1, 𝑆𝑦𝑦 = 4∕𝑦 3 𝑆𝑥𝑥 (2 1∕3, 2 1∕3) = 4∕(2 1∕3) 3 = 4∕2 = 2 𝑆𝑥𝑦 (2 1∕3, 2 1∕3) = 1 𝑆𝑦𝑦 (2 1∕3, 2 1∕3) = 4∕(2 1∕3) 3 = 4∕2 = 2 𝐻 = 𝑆𝑥𝑥 𝑆𝑦𝑦 − (𝑆𝑥𝑦 ) 2 = 4 − 1 = 3 > 0 Since 𝑆𝑥𝑥 (2 1∕3, 2 1∕3) > 0, the critical point is a local minimum. That it is an absolute minimum can be 2 2 seen by considering the graph of 𝑆(𝑥, 𝑦) = 𝑥𝑦 + + for 𝑥 > 0, 𝑦 > 0. The corresponding value of the 𝑦 𝑥 objective function is 2 2 𝑆(2 1∕3, 2 1∕3) = (2 1∕3)(2 1∕3) + + = 3(2 2∕3). 2 1∕3 2 1∕3 When 𝑥 = 2 1∕3 and 𝑦 = 2 1∕3, 𝑧 = 2∕(2 1∕3)(2 1∕3) = 2 1∕3, so 𝑆 has an absolute minimum of 3(2 2∕3) at the point (2 1∕3, 2 1∕3, 2 1∕3). 7. The constraint is 𝑥 + 2𝑦 = 40, or 𝑥 + 2𝑦 − 40 = 0. 𝑓(𝑥, 𝑦) = 𝑥𝑦 𝑔(𝑥, 𝑦) = 𝑥 + 2𝑦 − 4 𝐿(𝑥, 𝑦) = 𝑥𝑦 − 𝜆(𝑥 + 2𝑦 − 40) (1) 𝐿𝑥 = 0 ⇒ 𝑦 − 𝜆 = 0 ⇒ 𝑦 = 𝜆 (2) 𝐿𝑦 = 0 ⇒ 𝑥 − 2𝜆 = 0 ⇒ 𝑥 = 2𝜆 (3) Constraint ⇒ 𝑥 + 2𝑦 = 40 Substitute Equation (1) into (2) to obtain 𝑥 = 2𝑦. Substitute in Equation (3) to obtain 2𝑦 + 2𝑦 = 40 4𝑦 = 40 𝑦 = 10. The corresponding value of 𝑥 is 𝑥 = 2(10) = 20. The corresponding value of the objective is 𝑓(20, 10) = (20)(10) = 200. 8. The constraint is 3𝑥 + 𝑦 = 60, or 3𝑥 + 𝑦 − 60 = 0. 𝑓(𝑥, 𝑦) = 𝑥𝑦 𝑔(𝑥, 𝑦) = 3𝑥 + 𝑦 − 60 𝐿(𝑥, 𝑦) = 𝑥𝑦 − 𝜆(3𝑥 + 𝑦 − 60) (1) 𝐿𝑥 = 0 ⇒ 𝑦 = 3𝜆 (2) 𝐿𝑦 = 0 ⇒ 𝑥 = 𝜆 (3) Constraint ⇒ 3𝑥 + 𝑦 = 60 Substitute Equation (2) into (1) to obtain 𝑦 = 3𝑥. Substitute in Equation (3) to obtain 3𝑥 + 3𝑥 = 60 6𝑥 = 60 ⇒ 𝑥 = 10. The corresponding value of 𝑦 is 𝑦 = 3(1) = 30. The corresponding value of the objective is 𝑓(10, 30) = (10)(30) = 300.


Solutions Section 15.4 9. The constraint is 𝑥 + 𝑦 = 8, or 𝑥 + 𝑦 2 − 8 = 0. 𝑓(𝑥, 𝑦) = 4𝑥𝑦 𝑔(𝑥, 𝑦) = 𝑥 2 + 𝑦 2 − 8 𝐿(𝑥, 𝑦) = 4𝑥𝑦 − 𝜆(𝑥 2 + 𝑦 2 − 8) (1) 𝐿𝑥 = 0 ⇒ 4𝑦 − 2𝜆𝑥 = 0 ⇒ 2𝑦 = 𝜆𝑥 2

2

2

(2) 𝐿𝑦 = 0 ⇒ 4𝑥 − 2𝜆𝑦 = 0 ⇒ 2𝑥 = 𝜆𝑦 (3) Constraint ⇒ 𝑥 2 + 𝑦 2 = 8 Divide Equation (1) by 𝑥 (𝑥 = 0 would give 𝑦 = 0, but (0, 0) does not satisfy the constraint) to obtain 2𝑦 𝜆= . 𝑥 Substitute in Equation (2) to obtain 2𝑦 2𝑦 2 2𝑥 = 𝑦= , or 2𝑥 2 = 2𝑦 2, giving 𝑦 = ±𝑥. 𝑥 𝑥 Substituting into the constraint gives 𝑥 2 + 𝑥 2 = 8, or 2𝑥 2 = 8, giving 𝑥 2 = 4, so 𝑥 = ±2, and 𝑦 = ±2 as well. Thus we have four critical points: (−2, −2), (−2, 2), (2, −2), and (2, 2). Substitute each of these into the objective function: 𝑓(−2, −2) = 4(−2)(−2) = 16 𝑓(−2, 2) = 4(−2)(2) = −16 𝑓(2, −2) = 4(2)(−2) = −16 𝑓(2, 2) = 4(2)(2) = 16. The first and last of these give a maximum of 16. 10. The constraint is 𝑦 = 3 − 𝑥 2, or 𝑥 2 + 𝑦 − 3 = 0. 𝑓(𝑥, 𝑦) = 𝑥𝑦 𝑔(𝑥, 𝑦) = 𝑥 2 + 𝑦 − 3 𝐿(𝑥, 𝑦) = 𝑥𝑦 − 𝜆(𝑥 2 + 𝑦 − 3) (1) 𝐿𝑥 = 0 ⇒ 𝑦 = 2𝜆𝑥 (2) 𝐿𝑦 = 0 ⇒ 𝑥 = 𝜆 (3) Constraint ⇒ 𝑦 = 3 − 𝑥 2 Substitute Equation (2) in Equation (1) to obtain 𝑦 = 2𝑥 2. Substituting into the constraint gives 2𝑥 2 = 3 − 𝑥 2 ⇒ 3𝑥 2 = 3 ⇒ 𝑥 = ±1, so 𝑦 = 2𝑥 2 = 2. Thus we have two critical points: (1, 2), (−1, 2). Substitute each of these into the objective function: 𝑓(1, 2) = (1)(2) = 2 𝑓(−1, 2) = (−1)(2) = −2. The first of these gives a maximum of 2. 11. The constraint is 𝑥 + 2𝑦 = 10, or 𝑥 + 2𝑦 − 10 = 0. 𝑓(𝑥, 𝑦) = 𝑥 2 + 𝑦 2 𝑔(𝑥, 𝑦) = 𝑥 + 2𝑦 − 10 𝐿(𝑥, 𝑦) = 𝑥 2 + 𝑦 2 − 𝜆(𝑥 + 2𝑦 − 10) (1) 𝐿𝑥 = 0 ⇒ 2𝑥 − 𝜆 = 0 ⇒ 2𝑥 = 𝜆 (2) 𝐿𝑦 = 0 ⇒ 2𝑦 − 2𝜆 = 0 ⇒ 𝑦 = 𝜆 (3) Constraint ⇒ 𝑥 + 2𝑦 = 10 Substitute Equation (2) in Equation (1) to obtain 2𝑥 = 𝑦. Substituting into the constraint gives 𝑥 + 4𝑥 = 10 ⇒ 5𝑥 = 10 ⇒ 𝑥 = 2, so 𝑦 = 2𝑥 = 4. Thus we have one critical point: (2, 4). The corresponding value of the objective function is 𝑓(2, 10) = 2 2 + 4 2 = 20.


Solutions Section 15.4 12. The constraint is 𝑥𝑦 = 16, or 𝑥𝑦 − 16 = 0. 𝑓(𝑥, 𝑦) = 𝑥 2 + 𝑦 2 𝑔(𝑥, 𝑦) = 𝑥𝑦 2 − 16 𝐿(𝑥, 𝑦) = 𝑥 2 + 𝑦 2 − 𝜆(𝑥𝑦 2 − 16) (1) 𝐿𝑥 = 0 ⇒ 2𝑥 = 𝜆𝑦 2 2

2

(2) 𝐿𝑦 = 0 ⇒ 2𝑦 = 2𝜆𝑥𝑦 ⇒ 𝑦 = 𝜆𝑥𝑦 (3) Constraint ⇒ 𝑥𝑦 2 = 16 Solve Equation (2) for 𝜆 to obtain 𝜆 = 1∕𝑥 (𝑦 = 0 could not satisfy the constraint). 𝑦2 Substitute in Equation (1) to obtain 2𝑥 = ⇒ 2𝑥 2 = 𝑦 2. 𝑥 Substituting into the constraint gives 𝑥(2𝑥 2) = 16 ⇒ 2𝑥 3 = 16 ⇒ 𝑥 3 = 8 ⇒ 𝑥 = 2. The corresponding values of 𝑦 are 𝑦 2 = 2𝑥 2 = 8 ⇒ 𝑦 = ±2√2. Thus we have two critical points: (2, 2√2) and (2, −2√2). The corresponding values of the objective function are 𝑓(2, ±2√2) = 2 2 + (±2√2) 2 = 12. 13. The constraint is 𝑧 = 2𝑦, or 𝑧 − 2𝑦 = 0. 𝑓(𝑥, 𝑦, 𝑧) = 1 − 𝑥 2 − 𝑦 2 − 𝑧 2 𝑔(𝑥, 𝑦, 𝑧) = 𝑧 − 2𝑦 𝐿(𝑥, 𝑦, 𝑧) = 1 − 𝑥 2 − 𝑦 2 − 𝑧 2 − 𝜆(𝑧 − 2𝑦) (1) 𝐿𝑥 = 0 ⇒ −2𝑥 = 0 (2) 𝐿𝑦 = 0 ⇒ −2𝑦 + 2𝜆 = 0 ⇒ 𝑦 = 𝜆 (3) 𝐿𝑧 = 0 ⇒ −2𝑧 − 𝜆 = 0 ⇒ −2𝑧 = 𝜆 (4) Constraint ⇒ 𝑧 = 2𝑦 The first equation tells us that 𝑥 = 0. Substituting the third equation in the second gives 𝑦 = −2𝑧, or 𝑦 + 2𝑧 = 0. Combining this with the constraint equation 𝑧 − 2𝑦 = 0 gives a system of two equations in two unknowns, whose solution is 𝑦 = 𝑧 = 0. Thus the only critical point is (𝑥, 𝑦, 𝑧) = (0, 0, 0), and the corresponding value of the objective is 𝑓(0, 0, 0) = 1 − 0 2 − 0 2 − 0 2 = 1. That this is an absolute maximum is seen from the fact that 𝑓(𝑥, 𝑦, 𝑧) = 1 − 𝑥 2 − 𝑦 2 − 𝑧 2 can never be larger than 1. 14. The constraint is 𝑥 = 𝑦, or 𝑥 − 𝑦 = 0. 𝑓(𝑥, 𝑦, 𝑧) = 𝑥 2 + 𝑦 2 + 𝑧 2 − 2 𝑔(𝑥, 𝑦, 𝑧) = 𝑥 − 𝑦 𝐿(𝑥, 𝑦, 𝑧) = 𝑥 2 + 𝑦 2 + 𝑧 2 − 2 − 𝜆(𝑥 − 𝑦) (1) 𝐿𝑥 = 0 ⇒ 2𝑥 = 𝜆 (2) 𝐿𝑦 = 0 ⇒ 2𝑦 = −𝜆 (3) 𝐿𝑧 = 0 ⇒ 2𝑧 = 0 (4) Constraint ⇒ 𝑥 = 𝑦 Equation (3) tells us that 𝑧 = 0. Substituting (2) in (1) gives 2𝑥 = −2𝑦, or 𝑥 + 𝑦 = 0. Combining this with the constraint equation 𝑥 − 𝑦 = 0 gives a system of two equations in two unknowns, whose solution is 𝑥 = 𝑦 = 0. Thus the only critical point is (𝑥, 𝑦, 𝑧) = (0, 0, 0), and the corresponding value of the objective is 𝑓(0, 0, 0) = 0 2 + 0 2 + 0 2 − 2 = −2. That this is an absolute minimum is seen from the fact that 𝑓(𝑥, 𝑦, 𝑧) = 𝑥 2 + 𝑦 2 + 𝑧 2 − 2 can never be less than −2.


Solutions Section 15.4 15. The constraint is 3𝑥 = 𝑦, or 3𝑥 − 𝑦 = 0. 𝑓(𝑥, 𝑦, 𝑧) = 1 − 𝑥 2 − 𝑥 − 𝑦 2 + 𝑦 − 𝑧 2 + 𝑧 𝑔(𝑥, 𝑦, 𝑧) = 3𝑥 − 𝑦 𝐿(𝑥, 𝑦, 𝑧) = 1 − 𝑥 2 − 𝑥 − 𝑦 2 + 𝑦 − 𝑧 2 + 𝑧 − 𝜆(3𝑥 − 𝑦) (1) 𝐿𝑥 = 0 ⇒ −2𝑥 − 1 − 3𝜆 = 0 ⇒ −2𝑥 − 1 = 3𝜆 (2) 𝐿𝑦 = 0 ⇒ −2𝑦 + 1 + 𝜆 = 0 ⇒ −2𝑦 + 1 = −𝜆 (3) 𝐿𝑧 = 0 ⇒ −2𝑧 + 1 = 0 (4) Constraint ⇒ 3𝑥 = 𝑦 Equation (3) tells us that 𝑧 = 1∕2. Substituting (2) in (1) gives −2𝑥 − 1 = 6𝑦 − 3, or 2𝑥 + 6𝑦 = 2, that is, 𝑥 + 3𝑦 = 1. Combining this with the constraint equation 3𝑥 − 𝑦 = 0 gives a system of two equations in two unknowns, whose solution is 𝑥 = 1∕10, 𝑦 = 3∕10. Thus the only critical point is (𝑥, 𝑦, 𝑧) = (1∕10, 3∕10, 1∕2), and the corresponding value of the objective is 𝑓(1∕10, 3∕10, 1∕2) = 1 − 1∕100 − 1∕10 − 9∕100 + 3∕10 − 1∕4 + 1∕2 = 1.35. 16. The constraint is 𝑧 = 2𝑦, or 𝑧 − 2𝑦 = 0. 𝑓(𝑥, 𝑦, 𝑧) = 2𝑥 2 + 2𝑥 + 𝑦 2 − 𝑦 + 𝑧 2 − 𝑧 − 1 𝑔(𝑥, 𝑦, 𝑧) = 𝑧 − 2𝑦 𝐿(𝑥, 𝑦, 𝑧) = 2𝑥 2 + 2𝑥 + 𝑦 2 − 𝑦 + 𝑧 2 − 𝑧 − 1 − 𝜆(𝑧 − 2𝑦) (1) 𝐿𝑥 = 0 ⇒ 2𝑥 + 2 = 0 (2) 𝐿𝑦 = 0 ⇒ 2𝑦 − 1 = −2𝜆 (3) 𝐿𝑧 = 0 ⇒ 2𝑧 − 1 = 𝜆 (4) Constraint ⇒ 𝑧 − 2𝑦 = 0 Equation (1) tells us that 𝑥 = −1∕2. Substituting (3) in (2) gives 2𝑦 − 1 = −4𝑧 + 2 or 2𝑦 + 4𝑧 = 3. Combining this with the constraint equation 𝑧 − 2𝑦 = 0 gives a system of two equations in two unknowns, whose solution is 𝑦 = 3∕10, 𝑧 = 3∕5. Thus the only critical point is (𝑥, 𝑦, 𝑧) = (−1∕2, 3∕10, 3∕5), and the corresponding value of the objective is 𝑓(−1∕2, 3∕10, 3∕5) = 2∕4 − 2∕2 + 9∕100 − 3∕10 − 1 + 9∕25 − 3∕5 = −1.95. 17. The constraint is 𝑥𝑦𝑧 = 1, or 𝑥𝑦𝑧 − 1 = 0. 𝑓(𝑥, 𝑦, 𝑧) = 𝑥𝑦 + 4𝑥𝑧 + 2𝑦𝑧, 𝑔(𝑥, 𝑦, 𝑧) = 𝑥𝑦𝑧 − 1, 𝐿(𝑥, 𝑦, 𝑧) = 𝑥𝑦 + 4𝑥𝑧 + 2𝑦𝑧 − 𝜆(𝑥𝑦𝑧 − 1) (1) 𝐿𝑥 = 0 ⇒ 𝑦 + 4𝑧 = 𝜆𝑦𝑧 (2) 𝐿𝑦 = 0 ⇒ 𝑥 + 2𝑧 = 𝜆𝑥𝑧 (3) 𝐿𝑧 = 0 ⇒ 4𝑥 + 2𝑦 = 𝜆𝑥𝑦 (4) Constraint ⇒ 𝑥𝑦𝑧 = 1 Solve Equation (1) for 𝜆:

𝜆=

1 4 + . 𝑧 𝑦

Substituting in (2) gives 1 4 4𝑥𝑧 4𝑥𝑧 2𝑥 𝑥 + 2𝑧 = ( + )𝑥𝑧 ⇒ 𝑥 + 2𝑧 = 𝑥 + ⇒ 2𝑧 = ⇒1= ⇒ 𝑦 = 2𝑥. 𝑧 𝑦 𝑦 𝑦 𝑦 Substituting the expression for 𝜆 in (3) gives 𝑥𝑦 𝑥𝑦 1 4 𝑥 4𝑥 + 2𝑦 = ( + )𝑥𝑦 ⇒ 4𝑥 + 2𝑦 = ⇒ 2 = ⇒ 𝑧 = 𝑥∕2. + 4𝑥 ⇒ 2𝑦 = 𝑧 𝑦 𝑧 𝑧 𝑧 Substituting the expressions we obtained for 𝑦 and 𝑧 in the constraint equation gives: 𝑥(2𝑥)(𝑥∕2) = 1 ⇒ 𝑥 3 = 1 ⇒ 𝑥 = 1. The corresponding values of 𝑦 and 𝑧 are 𝑦 = 2𝑥 = 2 and 𝑧 = 𝑥∕2 = 1∕2. Therefore, the only critical point is (1, 2, 1∕2), and the corresponding value of the objective is 𝑓(1, 2, 1∕2) = (1)(2) + 4(1)(1∕2) + 2(2)(1∕2) = 6. Therefore, the minimum value of the objective function is 6, and occurs at the point (1, 2, 1∕2).


Solutions Section 15.4 18. The constraint is 𝑥𝑦𝑧 = 2, or 𝑥𝑦𝑧 − 2 = 0. 𝑓(𝑥, 𝑦, 𝑧) = 𝑥𝑦 + 𝑥𝑧 + 𝑦𝑧, 𝑔(𝑥, 𝑦, 𝑧) = 𝑥𝑦𝑧 − 2, 𝐿(𝑥, 𝑦, 𝑧) = 𝑥𝑦 + 𝑥𝑧 + 𝑦𝑧 − 𝜆(𝑥𝑦𝑧 − 2) (1) 𝐿𝑥 = 0 ⇒ 𝑦 + 𝑧 = 𝜆𝑦𝑧 (2) 𝐿𝑦 = 0 ⇒ 𝑥 + 𝑧 = 𝜆𝑥𝑧 (3) 𝐿𝑧 = 0 ⇒ 𝑥 + 𝑦 = 𝜆𝑥𝑦 (4) Constraint ⇒ 𝑥𝑦𝑧 = 2 Solve Equation (1) for 𝜆:

𝜆=

1 1 + . 𝑧 𝑦

Substituting in (2) gives 1 1 𝑥𝑧 𝑥𝑧 𝑥 𝑥 + 𝑧 = ( + )𝑥𝑧 ⇒ 𝑥 + 𝑧 = 𝑥 + ⇒𝑧= ⇒ 1 = ⇒ 𝑦 = 𝑥. 𝑧 𝑦 𝑦 𝑦 𝑦 Substituting the expression for 𝜆 in (3) gives 𝑥𝑦 𝑥𝑦 1 1 𝑥 𝑥 + 𝑦 = ( + )𝑥𝑦 ⇒ 𝑥 + 𝑦 = ⇒ 1 = ⇒ 𝑧 = 𝑥. +𝑥⇒𝑦= 𝑧 𝑦 𝑧 𝑧 𝑧 Substituting the expressions we obtained for 𝑦 and 𝑧 in the constraint equation gives: 𝑥(𝑥)(𝑥) = 2 ⇒ 𝑥 3 = 2 ⇒ 𝑥 = 2 1∕3. The corresponding values of 𝑦 and 𝑧 are 𝑦 = 𝑥 = 2 1∕3, 𝑧 = 𝑥 = 2 1∕3. Therefore, the only critical point is (2 1∕3, 2 1∕3, 2 1∕3), and the corresponding value of the objective is 𝑓(2 1∕3, 2 1∕3, 2 1∕3) = (2 1∕3)(2 1∕3) + (2 1∕3)(2 1∕3) + (2 1∕3)(2 1∕3) = 3(2 2∕3). Therefore, the minimum value of the objective function is 3(2 2∕3), and occurs at the point (2 1∕3, 2 1∕3, 2 1∕3). 19. a. 𝑓(𝑥, 𝑦, 𝑧) = (𝑥 − 3) 2 + 𝑦 2 + 𝑧 2 is the square of the distance from the point (𝑥, 𝑦, 𝑧) to (3, 0, 0), and the constraint tells us that (𝑥, 𝑦, 𝑧) must lie on the paraboloid 𝑧 = 𝑥 2 + 𝑦 2. Thus, the problem is to find the point(s) on the paraboloid closest to (3, 0, 0). Because there must be such a closest point (or points), the given problem must have at least one solution. Solution by Lagrange multipliers: 𝑓(𝑥, 𝑦, 𝑧) = (𝑥 − 3) 2 + 𝑦 2 + 𝑧 2, 𝑔(𝑥, 𝑦, 𝑧) = 𝑥 2 + 𝑦 2 − 𝑧, 𝐿(𝑥, 𝑦, 𝑧) = (𝑥 − 3) 2 + 𝑦 2 + 𝑧 2 − 𝜆(𝑥 2 + 𝑦 2 − 𝑧) (1) 𝐿𝑥 = 0 ⇒ 2(𝑥 − 3) − 2𝜆𝑥 = 0 ⇒ 𝑥 − 3 − 𝜆𝑥 = 0 (2) 𝐿𝑦 = 0 ⇒ 2𝑦 − 2𝑦𝜆 = 0 ⇒ 𝑦(1 − 𝜆) = 0 (3) 𝐿𝑧 = 0 ⇒ 2𝑧 + 𝜆 = 0 (4) Constraint ⇒ 𝑦 2 = 𝑧 − 𝑥 2 Equation (2) gives two possibilities: either 𝑦 = 0 or 𝜆 = 1. Case 1: 𝑦 = 0 ⇒ 𝑧 = 𝑥 2 by (4). Substitute this in (3) to eliminate 𝑧 and combine this with Equation (1): 2𝑥 2 + 𝜆 = 0 (Equation (3)) 𝑥 − 3 − 𝜆𝑥 = 0 (Equation (1)). Solving the first of these for 𝜆 and substituting in the second gives 𝑥 − 3 + (2𝑥 2)𝑥 = 0 ⇒ 2𝑥 3 + 𝑥 − 3 = 0, which factors as (𝑥 − 1)(2𝑥 2 + 2𝑥 + 3) = 0, and hence has the single solution 𝑥 = 1. We get 𝑧 by substituting: 𝑧 = 𝑥 2 = 1, so the single critical point is (1, 0, 1). Case 2: 𝜆 = 1. In this case, Equation (1) ⇒ 𝑥 − 3 + 𝑥 = 0 ⇒ −3 = 0, which is absurd, so we reject Case (2). Thus, the only critical point is (1, 0, 1), and since there must be a solution to the given problem, this critical point must give the minimum value of 𝑓, namely 𝑓(1, 0, 1) = (1 − 3) 2 + 0 2 + 1 2 = 5. b. Substitute 𝑧 = 𝑥 2 + 𝑦 2 to get 𝑓 = (𝑥 − 3) 2 + 𝑦 2 + (𝑥 2 + 𝑦 2) 2


Solutions Section 15.4 𝑓𝑥 = 0 ⇒ 2(𝑥 − 3) + 4𝑥(𝑥 + 𝑦 ) = 0 ⇒ 𝑥 − 3 + 2𝑥(𝑥 2 + 𝑦 2) = 0 2

2

𝑓𝑦 = 0 ⇒ 2𝑦 + 4𝑦(𝑥 2 + 𝑦 2) = 0 ⇒ 𝑦(2𝑥 2 + 2𝑦 2 + 1) = 0. The third equation above gives 𝑦 = 0 because the second factor is always strictly positive. Substituting in the equation for 𝑓𝑥 = 0 now gives 𝑥 − 3 + 2𝑥(𝑥 2) = 0 ⇒ 2𝑥 3 + 𝑥 − 3 = 0, which factors as (𝑥 − 1)(2𝑥 2 + 2𝑥 + 3) = 0, and hence has the single solution 𝑥 = 1. We get 𝑧 by substituting: 𝑧 = 𝑥 2 = 1, so the single critical point is (1, 0, 1). We can use the second derivative test on 𝑓 = (𝑥 − 3) 2 + 𝑦 2 + (𝑥 2 + 𝑦 2) 2 to verify that this gives the minimum value of 𝑓, which is 5. c. Substitute 𝑦 2 = 𝑧 − 𝑥 2 in the formula for 𝑓 to get 𝑓 = (𝑥 − 3) 2 + 𝑧 − 𝑥 2 + 𝑧 2 = 𝑥 2 − 6𝑥 + 9 + 𝑧 + 𝑧 2 − 𝑥 2 = −6𝑥 + 9 + 𝑧 2 + 𝑧. This function has no critical points, as its partial derivative with respect to 𝑥 is never zero. Thus, this substitution would seem to indicate that there are no extrema at all. d. The constraint equation 𝑦 2 = 𝑧 − 𝑥 2 tells us that 𝑧 − 𝑥 2 cannot be negative, and thus restricts the domain of 𝑓 to the set of points (𝑥, 𝑦, 𝑧) with 𝑧 − 𝑥 2 ≥ 0. However, this information is lost when 𝑧 − 𝑥 2 is substituted in the expression for 𝑓, and so the substitution in part (c) results in a different optimization problem; one in which there is no requirement that 𝑧 − 𝑥 2 be ≥ 0. If we pay attention to this constraint we can see that the minimum will occur when 𝑧 = 𝑥 2; substituting in 𝑓 = −6𝑥 + 9 + 𝑧 2 + 𝑧 will then lead us to the correct solution. 20. a. 𝑓(𝑥, 𝑦, 𝑧) = 𝑥 2 + (𝑦 + 3) 2 + (𝑧 − 4) 2 is the square of the distance from the point (𝑥, 𝑦, 𝑧) to (0, −3, 4), and the constraint tells us that (𝑥, 𝑦, 𝑧) must lie on the paraboloid 𝑧 = 4 − 𝑥 2 − 𝑦 2. Because there must be such a point (or points) on the paraboloid closest to (0, −3, 4), the given problem must have at least one solution. Solution by Lagrange Multipliers: 𝑓(𝑥, 𝑦, 𝑧) = 𝑥 2 + (𝑦 + 3) 2 + (𝑧 − 4) 2, 𝑔(𝑥, 𝑦, 𝑧) = 4 − 𝑥 2 − 𝑦 2 − 𝑧, 𝐿(𝑥, 𝑦, 𝑧) = 𝑥 2 + (𝑦 + 3) 2 + (𝑧 − 4) 2 − 𝜆(4 − 𝑥 2 − 𝑦 2 − 𝑧) (1) 𝐿𝑥 = 0 ⇒ 2𝑥 + 2𝜆𝑥 = 0 ⇒ 𝑥(1 + 𝜆) = 0 (2) 𝐿𝑦 = 0 ⇒ 2(𝑦 + 3) + 2𝑦𝜆 = 0 ⇒ (𝑦 + 3) + 𝑦𝜆 = 0 (3) 𝐿𝑧 = 0 ⇒ 2(𝑧 − 4) + 𝜆 = 0 (4) Constraint ⇒ 𝑧 = 4 − 𝑥 2 − 𝑦 2 Equation (1) gives two possibilities: either 𝑥 = 0 or 𝜆 = −1. Case 1: 𝑥 = 0 ⇒ 𝑧 = 4 − 𝑦 2 by (4). Substitute this in (3) to eliminate 𝑧 and combine this with Equation (2): (Equation (3)) −2𝑦 2 + 𝜆 = 0 (𝑦 + 3) + 𝑦𝜆 = 0 (Equation (2)). Solving the first of these for 𝜆 and substituting in the second gives 2𝑦 3 + 𝑦 + 3 = 0, which factors as (𝑦 + 1)(2𝑦 2 − 2𝑦 + 3) = 0, and hence has the single solution 𝑦 = −1. We get 𝑧 by substituting: 𝑧 = 4 − 𝑥 2 − 𝑦 2 = 4 − 0 − 1 = 3, so the single critical point is (0, −1, 3). Case 2: 𝜆 = −1. In this case, Equation (2) ⇒ (𝑦 + 3) − 𝑦 = 0 ⇒ 3 = 0, which is absurd, so we reject Case (2). Thus, the only critical point is (0, −1, 3), and since there must be a solution to the given problem, this critical point must give the minimum value of 𝑓, namely 𝑓(0, −1, 3) = 0 2 + (−1 + 3) 2 + (3 − 4) 2 = 5. b. Substitute 𝑧 = 4 − 𝑥 2 − 𝑦 2 to get 𝑓 = 𝑥 2 + (𝑦 + 3) 2 + (−𝑥 2 − 𝑦 2) 2 = 𝑥 2 + (𝑦 + 3) 2 + (𝑥 2 + 𝑦 2) 2 𝑓𝑥 = 0 ⇒ 2𝑥 + 4𝑥(𝑥 2 + 𝑦 2) = 0 ⇒ 𝑥(1 + 2𝑥 2 + 2𝑦 2) = 0


Solutions Section 15.4 𝑓𝑦 = 0 ⇒ 2(𝑦 + 3) + 4𝑦(𝑥 + 𝑦 ) = 0⇒(𝑦 + 3) + 2𝑦(𝑥 2 + 𝑦 2) = 0. The second equation above gives 𝑥 = 0 because the second factor is always strictly positive. Substituting in the equation for 𝑓𝑦 = 0 now gives 2𝑦 3 + 𝑦 + 3 = 0, which factors as (𝑦 + 1)(2𝑦 2 − 2𝑦 + 3) = 0, and hence has the single solution 𝑦 = −1. We get 𝑧 by substituting: 𝑧 = 4 − 𝑥 2 − 𝑦 2 = 4 − 0 − 1 = 3, so the single critical point is (0, −1, 3), as in part (a). We can use the second derivative test to verify that this gives the minimum value of 𝑓, which is 5. 2

2

c. Substitute 𝑥 2 = 4 − 𝑦 2 − 𝑧 in the formula for 𝑓 to get 𝑓 = 4 − 𝑦 2 − 𝑧 + (𝑦 + 3) 2 + (𝑧 − 4) 2 = 6𝑦 + 𝑧 2 − 9𝑧 + 29. This function has no critical points, as its partial derivative with respect to 𝑦 is never zero. Thus, this substitution would seem to indicate that there are no extrema at all. d. The constraint equation 𝑥 2 = 4 − 𝑦 2 − 𝑧 tells us that 4 − 𝑦 2 − 𝑧 cannot be negative, and thus restricts the domain of 𝑓 to the set of points (𝑥, 𝑦, 𝑧) with 4 − 𝑦 2 − 𝑧 ≥ 0. However, this information is lost when 4 − 𝑦 2 − 𝑧 is substituted in the expression for 𝑓, and so the substitution in part (c) results in a different optimization problem; one in which there is no requirement that 4 − 𝑦 2 − 𝑧 be ≥ 0. 21. Let 𝑥 be the length of the east and west sides, and let 𝑦 be the length of the north and south sides. The problem translates to: Maximize 𝐴 = 𝑥𝑦 subject to 8𝑥 + 4𝑦 = 80. The constraint is 8𝑥 + 4𝑦 = 80, or 8𝑥 + 4𝑦 − 80 = 0. 𝑓(𝑥, 𝑦) = 𝑥𝑦, 𝑔(𝑥, 𝑦) = 8𝑥 + 4𝑦 − 80, 𝐿(𝑥, 𝑦) = 𝑥𝑦 − 𝜆(8𝑥 + 4𝑦 − 80) (1) 𝐿𝑥 = 0 ⇒ 𝑦 − 8𝜆 = 0 ⇒ 𝑦 = 8𝜆 (2) 𝐿𝑦 = 0 ⇒ 𝑥 − 4𝜆 = 0 ⇒ 𝑥 = 4𝜆 (3) Constraint ⇒ 8𝑥 + 4𝑦 = 80 Solve Equation (1) for 𝜆 to obtain 𝜆 = 𝑦∕8. Substitute in Equation (2) to obtain 𝑥 = 4𝑦∕8 = 𝑦∕2, or 2𝑥 − 𝑦 = 0. The constraint equation is 8𝑥 + 4𝑦 = 80, or 2𝑥 + 𝑦 = 20. So we have a system of two linear equations in two unknowns: 2𝑥 − 𝑦 = 0 2𝑥 + 𝑦 = 20. The solution is (𝑥, 𝑦) = (5, 10). The corresponding value of the objective function is 𝐴 = 𝑥𝑦 = 5 × 10 = 50sq. ft. 22. Let 𝑥 be the length of the east and west sides, and let 𝑦 be the length of the south side. The problem translates to: Maximize 𝐴 = 𝑥𝑦 subject to 4𝑥 + 4𝑦 = 80. The constraint is 4𝑥 + 4𝑦 = 80, or 4𝑥 + 4𝑦 − 80 = 0. 𝑓(𝑥, 𝑦) = 𝑥𝑦, 𝑔(𝑥, 𝑦) = 4𝑥 + 4𝑦 − 80, 𝐿(𝑥, 𝑦) = 𝑥𝑦 − 𝜆(4𝑥 + 4𝑦 − 80) (1) 𝐿𝑥 = 0 ⇒ 𝑦 = 4𝜆 (2) 𝐿𝑦 = 0 ⇒ 𝑥 = 4𝜆 (3) Constraint ⇒ 4𝑥 + 4𝑦 = 80 Solve Equation (1) for 𝜆 to obtain 𝜆 = 𝑦∕4. Substitute in Equation (2) to obtain 𝑥 = 4𝑦∕4 = 𝑦, or 𝑥 − 𝑦 = 0. The constraint equation is 4𝑥 + 4𝑦 = 80, or 𝑥 + 𝑦 = 20. So we have a system of two linear equations in two unknowns: 𝑥−𝑦=0 𝑥 + 𝑦 = 20. The solution is (𝑥, 𝑦) = (10, 10). The corresponding value of the objective function is 𝐴 = 𝑥𝑦 = 10 × 10 = 100 sq. ft.


Solutions Section 15.4 23. The problem translates to: Maximize 𝑅 = 𝑝𝑞 subject to 𝑞 = 200,000 − 10,000𝑝. The constraint is 𝑞 = 200,000 − 10,000𝑝, or 𝑞 − 200,000 + 10,000𝑝 = 0. 𝑓(𝑝, 𝑞) = 𝑝𝑞, 𝑔(𝑝, 𝑞) = 𝑞 − 200,000 + 10,000𝑝, 𝐿(𝑝, 𝑞) = 𝑝𝑞 − 𝜆(𝑞 − 200,000 + 10,000𝑝) (1) 𝐿𝑝 = 0 ⇒ 𝑞 − 10,000𝜆 = 0 ⇒ 𝑞 = 10,000𝜆 (2) 𝐿𝑞 = 0 ⇒ 𝑝 − 𝜆 = 0 ⇒ 𝑝 = 𝜆 (3) Constraint ⇒ 𝑞 = 200,000 − 10,000𝑝 Substitute Equation (2) in Equation (1) to obtain 𝑞 = 10,000𝑝. The constraint equation is 𝑞 = 200,000 − 10,000𝑝. Equating these two expressions for 𝑞 gives 10,000𝑝 = 200,000 − 10,000𝑝 ⇒ 20,000𝑝 = 200,000 ⇒ 𝑝 = $10. We are not asked for any further information. 24. The problem translates to: Maximize 𝑃 = Revenue − Cost = 𝑝𝑞 − 4𝑞 subject to 𝑞 = 200,000 − 10,000𝑝. The constraint is 𝑞 = 200,000 − 10,000𝑝, or 𝑞 − 200,000 + 10,000𝑝 = 0. 𝑓(𝑝, 𝑞) = 𝑝𝑞 − 4𝑞, 𝑔(𝑝, 𝑞) = 𝑞 − 200,000 + 10,000𝑝, 𝐿(𝑝, 𝑞) = 𝑝𝑞 − 4𝑞 − 𝜆(𝑞 − 200,000 + 10,000𝑝) (1) 𝐿𝑝 = 0 ⇒ 𝑞 = 10,000𝜆 (2) 𝐿𝑞 = 0 ⇒ 𝑝 − 4 = 𝜆 (3) Constraint ⇒ 𝑞 = 200,000 − 10,000𝑝 Substitute Equation (2) in Equation (1) to obtain 𝑞 = 10,000(𝑝 − 4). The constraint equation is 𝑞 = 200,000 − 10,000𝑝. Equating these two expressions for 𝑞 gives 10,000(𝑝 − 4) = 200,000 − 10,000𝑝 ⇒ 20,000𝑝 = 240,000 ⇒ 𝑝 = $12. We are not asked for any further information. 25. We want to maximize 𝑓(𝑥, 𝑦, 𝑧) = 𝑥𝑦𝑧 subject to 𝑥 2 + 𝑦 2 + 𝑧 2 − 1 = 0. Using Lagrange multipliers, we need to solve the system 𝑦𝑧 = 2𝜆𝑥, 𝑥𝑧 = 2𝜆𝑦, 𝑥𝑦 = 2𝜆𝑧, and 𝑥 2 + 𝑦 2 + 𝑧 2 − 1 = 0. We solve the first 𝑦𝑧 𝑦 2𝑧 equation for 𝜆 = and substitute in the second to find 𝑥𝑧 = or 𝑥 2𝑧 = 𝑦 2𝑧, giving 𝑥 2 = 𝑦 2 2𝑥 𝑥 (assuming that 𝑧 ≠ 0, but 𝑧 = 0 makes 𝑥𝑦𝑧 = 0 and we can easily find larger values). Substituting the expression for 𝜆 into the third equation gives 𝑥 2 = 𝑧 2 similarly. From the last equation we then get 3𝑥 2 − 1 = 0, or 𝑥 = ±1∕√3. The corresponding values of 𝑦 and 𝑧 are also ±1∕√3, so we get eight points to check: (±1∕√3, ±1∕√3, ±1∕√3) with all eight choices of signs. Checking, we see that the largest value of 𝑥𝑦𝑧 occurs when all of the signs are positive or exactly two of them are negative; that is, at the points (1∕√3, 1∕√3, 1∕√3), (−1∕√3, −1∕√3, 1∕√3), (1∕√3, −1∕√3, −1∕√3), and (−1∕√3, 1∕√3, −1∕√3). 26. We want to minimize 𝑓(𝑥, 𝑦, 𝑧) = 𝑥 2 + 𝑦 2 + 𝑧 2 subject to 𝑥 2 + 𝑥 + 𝑦 2 + 4 − 𝑧 2 = 0. (Here we have squared both sides of 𝑧 = (𝑥 2 + 𝑥 + 𝑦 2 + 4) 1∕2 first to ease the algebra. We just have to remember that 𝑧 ≥ 0.) Using Lagrange multipliers, we need to solve the system 2𝑥 = 𝜆(2𝑥 + 1), 2𝑦 = 2𝜆𝑦, 2𝑧 = −2𝜆𝑧, and 𝑥 2 + 𝑥 + 𝑦 2 + 4 − 𝑧 2 = 0. From the second equation we see that either 𝜆 = 1 or 𝑦 = 0. We can't have 𝜆 = 1 because the first equation would then simplify to 0 = 1. Therefore, 𝑦 = 0. The third equation tells us that 𝜆 = −1 or 𝑧 = 0. If 𝑧 = 0, then the last equation would be 𝑥 2 + 𝑥 + 4 = 0, which has no real solutions. So, we must have 𝜆 = −1. Substituting in the first equation gives 2𝑥 = −2𝑥 − 1, so 𝑥 = −1∕4. √61 1 1 61 Substituting in the last equation now gives (remember that − + 4 − 𝑧 2 = 0, so 𝑧 2 = , 𝑧 = 16 4 4 4 1 √61 𝑧 ≥ 0). Thus, we have found only one interesting point, (− , 0, ). Now, the surface in question is 4 4 unbounded: We can take 𝑥 and 𝑦 as large as we like and use the equation to get a corresponding (large) value of 𝑧. But making 𝑥, 𝑦, and 𝑧 large clearly makes 𝑓 large. So, there is no maximum value of 𝑓 and the point that we've found must give the minimum value.


Solutions Section 15.4 27. Minimize 𝑑(𝑥, 𝑦, 𝑧) = 𝑥 + 𝑦 + 𝑧 subject to 𝑥 2 + 𝑦 − 1 − 𝑧 = 0. Using Lagrange multipliers, we need to solve 2𝑥 = 2𝜆𝑥, 2𝑦 = 𝜆, 2𝑧 = −𝜆, and 𝑥 2 + 𝑦 − 1 − 𝑧 = 0. From the first equation, either 𝑥 = 0 or 𝜆 = 1. If 𝑥 = 0, substitute 𝑦 = 𝜆∕2 and 𝑧 = −𝜆∕2 in the last equation to get 𝜆∕2 − 1 + 𝜆∕2 = 0, 𝜆 = 1. This gives the point (0, 1∕2, −1∕2). On the other hand, if 𝜆 = 1, then 𝑦 = 1∕2, 𝑧 = −1∕2, and 𝑥 2 + 1∕2 − 1 + 1∕2 = 0, so 𝑥 = 0, giving the same point. Since the distance may get arbitrarily large, but can get no less than 0, there must be a minimum distance and it must be at the point we found, (0, 1∕2, −1∕2). 2

2

2

28. Minimize 𝑑(𝑥, 𝑦, 𝑧) = 𝑥 2 + 𝑦 2 + 𝑧 2 subject to 𝑥 + 𝑦 2 − 3 − 𝑧 = 0. Using Lagrange multipliers, we need to solve 2𝑥 = 𝜆, 2𝑦 = 2𝜆𝑦, 2𝑧 = −𝜆, and 𝑥 + 𝑦 2 − 3 − 𝑧 = 0. From the second equation, either 𝑦 = 0 or 𝜆 = 1. If 𝑦 = 0, then substitute 𝑥 = 𝜆∕2 and 𝑧 = −𝜆∕2 in the last equation to get 𝜆∕2 − 3 + 𝜆∕2 = 0, 𝜆 = 3, so 𝑥 = 3∕2 and 𝑧 = −3∕2. This gives the point (3∕2, 0, −3∕2). On the other hand, if 𝜆 = 1, then 𝑥 = 1∕2, 𝑧 = −1∕2, and 1∕2 + 𝑦 2 − 3 + 1∕2 = 0, 𝑦 = ±√2. Checking the three points (3∕2, 0, −3∕2) and (1∕2, ±√2, −1∕2), we find that the points closest to the origin are (1∕2, ±√2, −1∕2). (The third point gives a relative but not an absolute minimum.) 29. Minimize 𝑑(𝑥, 𝑦, 𝑧) = (𝑥 + 1) 2 + (𝑦 − 1) 2 + (𝑧 − 3) 2 subject to −2𝑥 + 2𝑦 + 𝑧 − 5 = 0. Using Lagrange multipliers, we need to solve 2(𝑥 + 1) = −2𝜆, 2(𝑦 − 1) = 2𝜆, 2(𝑧 − 3) = 𝜆, and −2𝑥 + 2𝑦 + 𝑧 − 5 = 0. Solve the first three equations for 𝑥 = −𝜆 − 1, 𝑦 = 𝜆 + 1, and 𝑧 = 𝜆∕2 + 3. Substitute these in the last equation: 9 −2(−𝜆 − 1) + 2(𝜆 + 1) + (𝜆∕2 + 3) − 5 = 0, 𝜆 + 2 = 0, 𝜆 = −4∕9. 2 This gives 𝑥 = −5∕9, 𝑦 = 5∕9, and 𝑧 = 25∕9. Thus, the point on the plane closest to (−1, 1, 3) is (−5∕9, 5∕9, 25∕9). 30. Minimize 𝑑(𝑥, 𝑦, 𝑧) = (𝑥 − 1) 2 + (𝑦 − 1) 2 + 𝑧 2 subject to 2𝑥 − 2𝑦 − 𝑧 + 1 = 0. Using Lagrange multipliers, we need to solve 2(𝑥 − 1) = 2𝜆, 2(𝑦 − 1) = −2𝜆, 2𝑧 = −𝜆, and 2𝑥 − 2𝑦 − 𝑧 + 1 = 0. Solve the first three equations for 𝑥 = 𝜆 + 1, 𝑦 = −𝜆 + 1, and 𝑧 = −𝜆∕2. Substitute these in the last equation: 9 2(𝜆 + 1) − 2(−𝜆 + 1) − (−𝜆∕2) + 1 = 0, 𝜆 + 1 = 0, 𝜆 = −2∕9. 2 This gives 𝑥 = 7∕9, 𝑦 = 11∕9, and 𝑧 = 1∕9. Thus, the point on the plane closest to (1, 1, 0) is (7∕9, 11∕9, 1∕9). 31. Let the length, width, and height be 𝑙, 𝑤,and ℎ, respectively. We wish to minimize 𝐶 = 20 × 2 × 𝑙𝑤 + 10(2𝑙ℎ + 2𝑤ℎ) = 40𝑙𝑤 + 20𝑙ℎ + 20𝑤ℎ 2 80 40 subject to 𝑙𝑤ℎ = 2. Solve the constraint for 𝑙 = and substitute to get 𝐶(𝑤, ℎ) = + + 20𝑤ℎ, 𝑤ℎ ℎ 𝑤 𝑤 > 0 and ℎ > 0. 40 2 𝐶𝑤 = − 2 + 20ℎ = 0 when ℎ = 2 ; 𝑤 𝑤 80 4 𝐶ℎ = − 2 + 20𝑤 = 0 when 𝑤 = 2 = 𝑤 4, ℎ ℎ giving 𝑤 = 1 (recall that 𝑤 > 0). Thus, ℎ = 2 and 𝑙 = 1. Making either of ℎ or 𝑤 small or both large makes 𝐶 large, so the point we found must be a minimum. Thus, the dimensions of the box of least cost are 𝑙 × 𝑤 × ℎ = 1 × 1 × 2. 32. Minimize 𝐶 = 60𝑙𝑤 + 10𝑙ℎ + 10𝑤ℎ subject to 𝑙𝑤ℎ = 6. 6 Solve the constraint for 𝑙 = and substitute to get 𝑤ℎ 360 60 𝐶(𝑤, ℎ) = + + 10𝑤ℎ, 𝑤 > 0 and ℎ > 0 ℎ 𝑤 60 6 𝐶𝑤 = − 2 + 10ℎ = 0 when ℎ = 2 𝑤 𝑤


Solutions Section 15.4 360 36 𝐶ℎ = − 2 + 10𝑤 = 0 when 𝑤 = 2 = 𝑤 4, so 𝑤 = 1. ℎ ℎ Thus, ℎ = 6 and 𝑙 = 1. The dimensions of the box of least cost are 𝑙 × 𝑤 × ℎ = 1 × 1 × 6. 33. Let 𝑙 =length, 𝑤 =width, and ℎ =height. We want to maximize the volume 𝑉 = 𝑙𝑤ℎ. We are told that 𝑙 + 2(𝑤 + ℎ) ≤ 108; for the largest possible volume we will want 𝑙 + 2(𝑤 + ℎ) = 108. In the solution for Exercise 49 in the preceding section, we showed one way to solve this problem. Here we use the alternative, which is Lagrange multipliers. So, we need to solve 𝑤ℎ = 𝜆, 𝑙ℎ = 2𝜆, 𝑙𝑤 = 2𝜆, and 𝑙 + 2𝑤 + 2ℎ − 108 = 0. Substitute 𝜆 = 𝑤ℎ in the second equation to get 𝑙ℎ = 2𝑤ℎ, so 𝑙 = 2𝑤 (we certainly must have ℎ > 0 for a maximum volume). If we substitute 𝜆 = 𝑤ℎ in the third equation we get 𝑙𝑤 = 2𝑤ℎ, so 𝑙 = 2ℎ, hence 𝑤 = ℎ = 𝑙∕2. Substituting now in the last equation gives 3𝑙 = 108, so 𝑙 = 36, 𝑤 = 18, and ℎ = 18. Thus, the largest-volume package has dimensions 18 in ×18 in × 36 in and volume = 11,664 cubic inches. 34. Let 𝑙 =length, 𝑤 =width, and ℎ =height. We want to maximize the volume 𝑉 = 𝑙𝑤ℎ. We are told that 𝑙 + 2(𝑤 + ℎ) ≤ 165; for the largest possible volume we will want 𝑙 + 2(𝑤 + ℎ) = 165. In the solution for Exercise 50 in the preceding section, we showed one way to solve this problem. Here we use the alternative, which is Lagrange multipliers. So, we need to solve 𝑤ℎ = 𝜆, 𝑙ℎ = 2𝜆, 𝑙𝑤 = 2𝜆, and 𝑙 + 2𝑤 + 2ℎ − 165 = 0. Substitute 𝜆 = 𝑤ℎ in the second equation to get 𝑙ℎ = 2𝑤ℎ, so 𝑙 = 2𝑤 (we certainly must have ℎ > 0 for a maximum volume). If we substitute 𝜆 = 𝑤ℎ in the third equation we get 𝑙𝑤 = 2𝑤ℎ, so 𝑙 = 2ℎ, hence 𝑤 = ℎ = 𝑙∕2. Substituting now in the last equation gives 3𝑙 = 165, so 𝑙 = 55, 𝑤 = 27.5, and ℎ = 27.5. Note that the length is less than 108 inches, so that constraint is also satisfied. Thus, the largest-volume package has dimensions 27.5 in × 27.5 in × 55 in and volume = 41,593.75 cubic inches. 35. Let 𝐿 be the cost of lightweight cardboard and let 𝐻 be the cost of heavy-duty cardboard. Minimize 2 𝐶 = 2𝐻𝑙𝑤 + 2𝐿𝑙ℎ + 2𝐿𝑤ℎ subject to 𝑙𝑤ℎ = 2. Solve the constraint for 𝑙 = and substitute to get 𝑤ℎ 4𝐻 4𝐿 𝐶(𝑤, ℎ) = + + 2𝐿𝑤ℎ. ℎ 𝑤 4𝐿 2 𝐶𝑤 = − 2 + 2𝐿ℎ = 0 when ℎ = 2 ; 𝑤 𝑤 4𝐻 2𝐻 𝐻 4 𝐶ℎ = − 2 + 2𝐿𝑤 = 0 when 𝑤 = 𝑤 , = 2 2𝐿 ℎ 𝐿ℎ 2𝐿 1∕3 𝐻 2∕3 so 𝑤 = ( ) . Substituting in the expressions for 𝑙 and ℎ we find ℎ = 2 1∕3( ) and 𝐻 𝐿 2𝐿 1∕3 𝑙=𝑤=( ) . 𝐻 2𝐿 1∕3 2𝐿 1∕3 𝐻 2∕3 Thus, the box of least cost has dimensions ( ) × ( ) × 2 1∕3( ) . 𝐻 𝐻 𝐿 36. Let 𝐿 be the cost of lightweight cardboard and let 𝐻 be the cost of heavy-duty cardboard. Minimize 2 𝐶 = (𝐻 + 𝐿)𝑙𝑤 + 2𝐿𝑙ℎ + 2𝐿𝑤ℎ subject to 𝑙𝑤ℎ = 2. Solve the constraint for 𝑙 = and substitute to get 𝑤ℎ 2(𝐻 + 𝐿) 4𝐿 𝐶(𝑤, ℎ) = + + 2𝐿𝑤ℎ. ℎ 𝑤 4𝐿 2 𝐶𝑤 = − 2 + 2𝐿ℎ = 0 when ℎ = 2 ; 𝑤 𝑤 2(𝐻 + 𝐿) 𝐻+𝐿 𝐻+𝐿 4 𝐶ℎ = − 𝑤 , + 2𝐿𝑤 = 0 when 𝑤 = = 2 4𝐿 ℎ 𝐿ℎ 2 4𝐿 1∕3 𝐻 + 𝐿 2∕3 so 𝑤 = ( ) ; ℎ = 2( ) , 𝑙 = 𝑤. 𝐻+𝐿 4𝐿 4𝐿 1∕3 4𝐿 1∕3 𝐻 + 𝐿 2∕3 ) ×( ) × 2( ) . Thus, the box of least cost has dimensions ( 𝐻+𝐿 𝐻+𝐿 4𝐿 37. If (𝑥, 𝑦, 𝑧) is one corner of the box (with 𝑥 > 0, 𝑦 > 0, and 𝑧 > 0), the box will have dimensions 2𝑥 × 2𝑦 × 𝑧. We need to maximize 𝑉 = 4𝑥𝑦𝑧 subject to 𝑥 2 + 𝑦 2 + 𝑧 − 1 = 0. Using Lagrange multipliers


Solutions Section 15.4 we must solve 4𝑦𝑧 = 2𝜆𝑥, 4𝑥𝑧 = 2𝜆𝑦, 4𝑥𝑦 = 𝜆, and 𝑥 2 + 𝑦 2 + 𝑧 − 1 = 0. Solve the first equation for 𝑦𝑧 𝑦 2𝑧 2 𝑦𝑧 𝜆=2 and substitute in the second and third equations: 4𝑥𝑧 = 4 , 𝑥 = 𝑦 2, 4𝑥𝑦 = 2 , 2𝑥 2 = 𝑧. 𝑥 𝑥 𝑥 1 1 1 2 2 2 2 2 2 2 Substituting 𝑦 = 𝑥 and 𝑧 = 2𝑥 in the last equation gives 𝑥 + 𝑥 + 2𝑥 = 1, 𝑥 = , 𝑥 = , 𝑦 = , 4 2 2 1 𝑧 = . The dimensions of the box are 1 × 1 × 1∕2. 2 38. If (𝑥, 𝑦, 𝑧) is one corner of the box (with 𝑥 > 0, 𝑦 > 0, and 𝑧 > 0), the box will have dimensions 2𝑥 × 2𝑦 × 𝑧. We need to maximize 𝑉 = 4𝑥𝑦𝑧 subject to 2𝑥 2 + 𝑦 2 + 𝑧 − 2 = 0. Using Lagrange multipliers, we must solve 4𝑦𝑧 = 4𝜆𝑥, 4𝑥𝑧 = 2𝜆𝑦, 4𝑥𝑦 = 𝜆, and 2𝑥 2 + 𝑦 2 + 𝑧 − 2 = 0. Solve the first 𝑦𝑧 𝑦 2𝑧 𝑦𝑧 equation for 𝜆 = and substitute in the second and third equations: 4𝑥𝑧 = 2 , 2𝑥 2 = 𝑦 2, 4𝑥𝑦 = , 𝑥 𝑥 𝑥 1 2 2 2 2 2 2 2 2 4𝑥 = 𝑧. Substituting 𝑦 = 2𝑥 and 𝑧 = 4𝑥 in the last equation gives 2𝑥 + 2𝑥 + 4𝑥 = 2, 𝑥 = , 4 √2 1 𝑥= ,𝑦= , 𝑧 = 1. The dimensions of the box are 1 × √2 × 1. 2 2 39. The objective is to maximize the productivity: 𝑞 = 50𝑛 0.6𝑟 0.4 subject to 150𝑛 + 60𝑟 = 1,500 (𝑛, 𝑟 ≥ 0). So, 𝑔(𝑛, 𝑟) = 150𝑛 + 60𝑟 − 1,500, 𝐿(𝑛, 𝑟) = 50𝑛 0.6𝑟 0.4 − 𝜆(150𝑛 + 60𝑟 − 1,500), and the system we need to solve is: 𝐿𝑛 = 0 : 30𝑛 −0.4𝑟 0.4 = 150𝜆 𝐿𝑟 = 0 : 20𝑛 0.6𝑟 −0.6 = 60𝜆 𝑔=0: 50𝑛 + 60𝑟 = 1,500. Rewrite the first two equations as 𝑟 0.4 𝑛 0.6 30( ) = 150𝜆\qquad 20( ) = 60𝜆. 𝑛 𝑟 Dividing the first by the second: 3 𝑟 0.4 𝑟 0.6 5 ( ) ( ) = ; 2 𝑛 𝑛 2 𝑟 5 that is, 3 = 5, giving 𝑟 = 𝑛. Substituting this result into the constraint equation gives 𝑛 3 150𝑛 + 100𝑛 = 1500 ⇒ 250𝑛 = 1500, 5 so 𝑛 = 6 laborers, 𝑟 = 𝑛 = 10 robots 3 for a productivity of 𝑞 = 50(6) 0.6(10) 0.4 ≈ 368 pairs of socks per day. 40. The objective is to maximize the productivity: 𝑞 = 100𝑥 0.3𝑦 0.7 subject to 60𝑥 + 12𝑦 = 1,200 (𝑥, 𝑦 ≥ 0). So, 𝑔(𝑥, 𝑦) = 60𝑥 + 12𝑦 − 1,200, 𝐿(𝑥, 𝑦) = 100𝑥 0.3𝑦 0.7 − 𝜆(60𝑥 + 12𝑦 − 1,200), and the system we need to solve is: 𝐿𝑥 = 0 : 30𝑥 −0.7𝑦 0.7 = 60𝜆 𝐿𝑦 = 0 : 70𝑥 0.3𝑦 −0.3 = 12𝜆 𝑔=0: 60𝑥 + 12𝑦 = 1,200. Rewrite the first two equations as 𝑦 0.7 𝑥 0.3 30( ) = 60𝜆 \qquad 70( ) = 12𝜆. 𝑥 𝑦 Dividing the first by the second: 3 𝑦 0.7 𝑦 0.3 ( ) ( ) = 5; 7 𝑥 𝑥 3𝑦 35 that is, 𝑥. Substituting this result into the constraint equation gives = 5, giving 𝑦 = 7𝑥 3


Solutions Section 15.4 60𝑥 + 12(

35 )𝑥 = 1,200 3

200𝑥 = 1,200

𝑥 = 6 laborers,

35 𝑥 = $70 thousand 3 for a productivity of 𝑞 = 100(6) 0.3(70) 0.7 ≈ 3,350 automobiles per year. 𝑦=

41. Method 1: Solve 𝑔(𝑥, 𝑦, 𝑧) = 0 for one of the variables, and substitute in 𝑓(𝑥, 𝑦, 𝑧). Then find the maximum value of the resulting function of two variables. Advantage (Answers may vary): We can use the second derivative test to check whether the resulting critical points are maxima, minima, saddle points, or none of these. Disadvantage (Answers may vary): We may not be able to solve 𝑔(𝑥, 𝑦, 𝑧) = 0 for one of the variables.Method 2: Use the method of Lagrange multipliers. Advantage (Answers may vary): We do not need to solve the constraint equation for one of the variables. Disadvantage (Answers may vary): The method does not tell us whether the critical points obtained are maxima, minima, saddle points, or none of these. 42. Yes. The partial derivatives have to vanish at a relative extremum, and (𝑎, 𝑏) is certainly a relative extremum. 43. If the only constraint is an equality constraint and if it is impossible to eliminate one of the variables in the objective function by substitution (solving the constraint equation for a variable or some other method). 44. The method of Lagrange multipliers does not apply when the only constraints are inequality constraints. 45. Answers may vary: Maximize 𝑓(𝑥, 𝑦) = 1 − 𝑥 2 − 𝑦 2 subject to 𝑥 = 𝑦. 46. Answers may vary: Maximize 𝑓(𝑥, 𝑦, 𝑧) = 1 − 2𝑥 2 − 𝑦 2 − 𝑧 2subject to 𝑧 = 𝑦. 47. Yes. There may be relative extrema at points on the boundary of the domain. The partial derivatives of the function need not be 0 at such points. 48. Answers may vary. An example is 𝑓(𝑥, 𝑦, 𝑧) = 1 − √𝑥 2 + 𝑦 2 + 𝑧 2, whose partial derivatives are not defined at its maximum of 1 at (0, 0, 0). 49. In a linear programming problem the objective function is linear, so the partial derivatives can never all be zero. (We are ignoring the simple case in which the objective function is constant.) It follows that the extrema cannot occur in the interior of the domain (since the partial derivatives must be zero at such points). 50. If the solution were located in the interior of one of the line segments making up the boundary of the domain of 𝑓, then the derivative of a certain function would be 0. This function is obtained by substituting the linear equation 𝐶(𝑥, 𝑦) = 0 in the linear objective function. But because the result would again be a linear function, it is either constant, or its derivative is a nonzero constant. In either event, extrema lie on the boundary of that line segment; that is, at one of the corners of the domain.


Solutions Section 15.5 Section 15.5 1

1.

1

∫ 0 ∫ 0 1

2.

3.

4.

5.

1

1 1 1 1 1 1 1 [ 𝑥 2 − 2𝑥𝑦] 𝑑𝑦 = ( − 2𝑦)𝑑𝑦 = [ 𝑦 − 𝑦 2] =− 𝑥=0 𝑦=0 ∫ ∫ 2 2 2 2 0 0

(𝑥 − 2𝑦) 𝑑𝑥 𝑑𝑦 = 2

1

(2𝑥 + 3𝑦) 𝑑𝑥 𝑑𝑦 =

∫ 0 −1 ∫ 1

2

2

4

2

3

∫ −1

[𝑥 2 + 3𝑥𝑦]𝑥2 = 0 𝑑𝑦 = 1

1

∫ −1

1

(4 + 6𝑦) 𝑑𝑦 = [4𝑦 + 3𝑦 2]𝑦 = −1 = 8

1 2 1 2 𝑥 − 𝑥𝑦] 𝑑𝑦 = (𝑒 2𝑦 − 2 − 3𝑦) 𝑑𝑦 𝑥=0 ∫ ∫ ∫ ∫ 2 0 0 0 0 1 2 2 3 2 1 1 2 7 = [ 𝑒 𝑦 − 2𝑦 − 𝑦 ] = 𝑒 − 𝑦=0 2 2 2 2

(𝑦𝑒 𝑥 − 𝑥 − 𝑦) 𝑑𝑥 𝑑𝑦 =

[𝑦𝑒 𝑥 −

2 2 2 1 1 𝑥 3 3 1 3 ( + ) 𝑑𝑥 𝑑𝑦 = [ln 𝑥 + ] 𝑑𝑦 = (ln + )𝑑𝑦 = [(ln )𝑦 + ln 𝑦] 𝑥 2 1 = ∫ ∫ ∫ 𝑦 𝑦 2 𝑦 2 1 ∫ 2 𝑥 1 1 3 = ln + ln 2 = ln 3 2 2

𝑒 𝑥+𝑦 𝑑𝑥 𝑑𝑦 =

∫ ∫ 0 ∫ 0 0 3 2 = (𝑒 − 1)(𝑒 − 1).

2

[𝑒 𝑥+𝑦]𝑥3 = 0 𝑑𝑦 = 2

This may also be found by writing

2

(𝑒 3+𝑦 − 𝑒 𝑦) 𝑑𝑦 = [𝑒 3+𝑦 − 𝑒 𝑦]𝑦 = 0 = 𝑒 5 − 𝑒 2 − 𝑒 3 + 1

∫ 0

3

∫ 0 ∫ 0

𝑒 𝑥+𝑦 𝑑𝑥 𝑑𝑦 =

2

3

3

2

𝑒 𝑥𝑒 𝑦 𝑑𝑥 𝑑𝑦 = ( 𝑒 𝑥 𝑑𝑥)( 𝑒 𝑦 𝑑𝑦) as in ∫ ∫ ∫ 0 ∫ 0 0 0

Exercise 57. 1

6.

1

∫ 0 ∫ 0

1

𝑒 𝑥−𝑦 𝑑𝑥 𝑑𝑦 =

[𝑒 𝑥−𝑦]𝑥1 = 0 𝑑𝑦 =

∫ 0 (𝑒 − 1) 2 −1 . =𝑒−2+𝑒 = 𝑒 This may also be found by writing 1

1

∫ 0 ∫ 0

𝑒 𝑥−𝑦𝑑𝑥 𝑑𝑦 𝑑𝑥 𝑑𝑦 =

1

7.

2−𝑦

1

8.

2−𝑦

∫ 0 ∫ 0 1

9.

1

1

𝑒 𝑥𝑒 −𝑦 𝑑𝑥 𝑑𝑦 = ( 𝑒 𝑥 𝑑𝑥)( 𝑒 −𝑦 𝑑𝑦) as in Exercise 57. ∫ ∫ ∫ 0 ∫ 0 0 0

1 2−𝑦 1 1 1 1 1 8 7 [ 𝑥 2] 𝑑𝑦 = (2 − 𝑦) 2𝑑𝑦 = [− (2 − 𝑦) 3] =− + = 𝑥 0 𝑦 0 = = ∫ ∫ 2 6 6 6 6 0 0 2 1

𝑦 𝑑𝑥 𝑑𝑦 = 𝑥+1

∫ 0

𝑒 𝑥+𝑦 𝑑𝑦 𝑑𝑥 =

1

2

(𝑒 1−𝑦 − 𝑒 −𝑦) 𝑑𝑦 = [−𝑒 1−𝑦 + 𝑒 −𝑦]𝑦 = 0 = −1 + 𝑒 −1 + 𝑒 − 1

1

2−𝑦

[𝑥𝑦]𝑥 = 0 𝑑𝑦 = 1

𝑥+2

1

∫ 0

𝑦(2 − 𝑦) 𝑑𝑦 = [𝑦 2 −

[𝑒 𝑥+𝑦]𝑦𝑥+1 = 𝑥−1 𝑑𝑥 =

∫ ∫ 𝑥−1 −1 ∫ −1 1( 3 −1 −3) 𝑒 −𝑒−𝑒 +𝑒 = 2

10.

∫ 0

1

𝑥 𝑑𝑥 𝑑𝑦 =

∫ 0 ∫ 0

1

1

1

∫ −1

1 3 1 2 𝑦 ] = 𝑦 0 = 3 3

1 1 1 (𝑒 2𝑥+1 − 𝑒 2𝑥−1) 𝑑𝑥 = [ 𝑒 2𝑥+1 − 𝑒 2𝑥−1] 𝑥 = −1 2 2

1 1 𝑥+2 1 𝑑𝑦 𝑑𝑥 = (2√2𝑥 + 2 − 2√2𝑥) 𝑑𝑥 [2√𝑥 + 𝑦]𝑦 = 𝑥 𝑑𝑥 = ∫ ∫ ∫ √𝑥 + 𝑦 0 ∫ 𝑥 0 0 1 2 2 2 2 = [ (2𝑥 + 2) 3∕2 − (2𝑥) 3∕2] = (8 − 2 3∕2 − 2 3∕2) = (8 − 2 5∕2) 𝑥=0 3 3 3 3


Solutions Section 15.5 1

11.

𝑥2

∫ 0 ∫ −𝑥 2 4

12.

1 1 1 𝑑𝑥 = [ 𝑥 4] = 𝑥=0 2 2

∫ 0

1

3

𝑒 𝑦 +1 𝑑𝑥 𝑑𝑦 =

[𝑥𝑒 𝑦 2]

∫ 0

1 𝑦 1 𝑦2 1 1 1 1 𝑦2 𝑥 = 0 𝑑𝑦 = ∫ 𝑦𝑒 𝑑𝑦 = [ 𝑒 ]𝑦 = 0 = 𝑒 − = (𝑒 − 1) 2 2 2 2 0

[𝑥𝑒 𝑦 3+1]

(𝑦 + 𝑥) 1∕3 𝑑𝑥 𝑑𝑦 =

1 𝑦2 1 3 1 1 1 2 𝑦 3+1 𝑑𝑦 = [ 𝑒 𝑦 +1] = 𝑒 2 − 𝑒 𝑥 = 0 𝑑𝑦 = ∫ 𝑦 𝑒 0 3 3 3 0

2

2 2 8−𝑦 3 3 3 45 [ (𝑦 + 𝑥) 4∕3] 𝑑𝑦 = ( (16) − ) 𝑑𝑦 = 𝑑𝑦 𝑥 = 1−𝑦 ∫ ∫ ∫ 4 4 4 4 0 0 0

𝑦2

1−𝑥 2

1

2 𝑑𝑦 𝑑𝑥 =

∫ 0 −1 ∫

∫ −1 2 2 8 =2− +2− = 3 3 3 2−𝑥

2

𝑥 𝑑𝑦 𝑑𝑥 =

∫ 0 ∫ 0

∫ 0

∫ 0 −1 ∫ 1

1

(1 + 𝑦) 𝑑𝑥 𝑑𝑦 =

1

2

𝑑𝑥 = [2𝑦]𝑦1−𝑥 =0

𝑑𝑥 = [𝑥𝑦]𝑦2−𝑥 =0

1−𝑦 2

1

19.

3

2 2 𝑦+1 𝑦+1 𝑦+1 𝑦2 1 1 𝑑𝑥 𝑑𝑦 [− ] 𝑑𝑦 (− = = + 𝑦 + ) 𝑑𝑦 3 2 2 𝑥 1−2𝑦 2 = ∫ ∫ ∫ 2 2 (2𝑦 + 𝑥) 2(2𝑦 + 𝑥) 2(2𝑦 + 𝑦 ) 1 ∫ 1−2𝑦 1 1 1 1 2 1 2 1 1 1 1 115 =[ + 𝑦 + 𝑦] = +1+1− − − = 2 𝑦 = 1 32 12 4 2 96 4(2𝑦 + 𝑦 2) 4

2

18.

1

2

8−𝑦

1

17.

∫ 0

∫ 0 ∫ 1−𝑦 45 2 45 = [ 𝑦] = 𝑦 0 = 4 2 2

16.

1

2

4 𝑦 √ 4 𝑥 4 1 2 𝑑𝑦 𝑑𝑥 = [ ] 𝑑𝑥 𝑑𝑥 = [4√𝑥]𝑥 = 1 = 8 − 4 = 4 = √𝑥 𝑦 = − ∫ ∫ √ 𝑥 𝑥 𝑥 1 1

𝑒 𝑦 𝑑𝑥 𝑑𝑦 =

𝑦2

[𝑥𝑦] 𝑥

𝑑𝑥 = 2𝑥 𝑦 = −𝑥 2 ∫ 0

∫ 0 ∫ 0 1 = 𝑒(𝑒 − 1) 3 2

15.

𝑦

∫ 0 ∫ 0 1

14.

√𝑥

∫ 1 ∫ −√𝑥 1

13.

1

𝑥 𝑑𝑦 𝑑𝑥 =

∫ −1

∫ −1

2(1 − 𝑥 2) 𝑑𝑥 = [2𝑥 −

2

∫ 0

𝑥(2 − 𝑥) 𝑑𝑥 = [𝑥 2 −

1

1−𝑦 2

[𝑥(1 + 𝑦)]𝑥 = 0 𝑑𝑦 =

∫ −1

2 3 1 𝑥 ] 𝑥 = −1 3

1 3 2 8 4 𝑥 ] =4− = 𝑥=0 3 3 3

(1 − 𝑦 2)(1 + 𝑦) 𝑑𝑦

1 2 1 3 1 4 1 𝑦 − 𝑦 − 𝑦 ] 𝑦 = −1 2 3 4 1 1 1 1 1 1 4 =1+ − − +1− − + = 2 3 4 2 3 4 3 =

1

20.

1−𝑦

∫ 0 ∫ 𝑦−1

𝑒 𝑥+𝑦 𝑑𝑥 𝑑𝑦 =

1

[𝑒 𝑥+𝑦]𝑥1−𝑦 𝑑𝑦 = = 𝑦−1

∫ 0 1 1 −1 1 = 𝑒 − 𝑒 + 𝑒 = (𝑒 + 𝑒 −1) 2 2 2 1

21.

(1 + 𝑦 − 𝑦 2 − 𝑦 3) 𝑑𝑦 = [𝑦 +

∫ −1

2 √1−𝑦

∫ −1 ∫ −

2 √1−𝑦

𝑥𝑦 2 𝑑𝑥 𝑑𝑦 =

1

1

∫ 0

2

(𝑒 − 𝑒 2𝑦−1) 𝑑𝑦 = [𝑒𝑦 −

1 √1−𝑦 1 𝑑𝑦 = 0 𝑑𝑦 = 0 [ 𝑥 2𝑦 2] ∫ ∫ 2 𝑥 = −√1−𝑦 2 0 −1

1 2𝑦−1 1 𝑒 ] 𝑦=0 2


Solutions Section 15.5 22.

23.

1

2 √1−𝑦

0

𝑥+1

2

1 √1−𝑦 1 1 2 [ 𝑥 2𝑦 2]𝑥 = 0 𝑑𝑦 = 𝑦 (1 − 𝑦 2)𝑑𝑦 ∫ ∫ ∫ ∫ 2 0 0 −1 −1 2 1 1 5 1 1 1 1 1 2 𝑦 ] = [ 𝑦3 − = − + − = 𝑦 = −1 6 10 6 10 6 10 15

(𝑥 2 + 𝑦 2) 𝑑𝑦 𝑑𝑥 +

∫ −1 ∫ −𝑥−1 0

1

𝑥𝑦 2 𝑑𝑥 𝑑𝑦 =

1

−𝑥+1

∫ 0 ∫ 𝑥−1

(𝑥 2 + 𝑦 2) 𝑑𝑦 𝑑𝑥

1 −𝑥+1 1 3 𝑥+1 1 𝑦 ] 𝑑𝑥 + [𝑥 2𝑦 + 𝑦 3] 𝑑𝑥 𝑦 = −𝑥−1 𝑦 = 𝑥−1 ∫ ∫ 3 3 0 −1 0 1 2 2 (2𝑥 2(𝑥 + 1) + (𝑥 + 1) 3) 𝑑𝑥 + (2𝑥 2(−𝑥 + 1) + (−𝑥 + 1) 3) 𝑑𝑥 = ∫ ∫ 3 3 0 −1 0 1 8 3 2 8 2 ( 𝑥 + 4𝑥 2 + 2𝑥 + )𝑑𝑥 + (− 𝑥 3 + 4𝑥 2 − 2𝑥 + ) 𝑑𝑥 = ∫ ∫ 3 3 3 0 −1 3 2 4 4 3 2 0 2 4 4 3 2 1 2 2 = [ 𝑥 + 𝑥 + 𝑥 + 𝑥] + [− 𝑥 + 𝑥 − 𝑥 + 𝑥] 3 3 3 𝑥 = −1 3 3 3 𝑥=0 2 4 2 2 4 2 2 =− + −1+ − + −1+ = 3 3 3 3 3 3 3

[𝑥 2𝑦 +

=

0

24.

𝑥+2

𝑥 2 𝑑𝑦 𝑑𝑥 +

∫ 𝑥∕2+1 −2 ∫

2

−𝑥+2

∫ 0 ∫ −𝑥∕2+1

𝑥 2 𝑑𝑦 𝑑𝑥 =

0

∫ −2

[𝑥 2𝑦]𝑦𝑥+2 = 𝑥∕2+1 𝑑𝑥 +

2

∫ 0

[𝑥 2𝑦]𝑦−𝑥+2 = −𝑥∕2+1 𝑑𝑥

0

2 1 1 ( 𝑥 3 + 𝑥 2) 𝑑𝑥 + (− 𝑥 3 + 𝑥 2)𝑑𝑥 ∫ ∫ 2 2 0 −2 0 1 8 2 8 2 ( 𝑥 3 + 4𝑥 2 + 2𝑥 + )𝑑𝑥 + (− 𝑥 3 + 4𝑥 2 − 2𝑥 + ) 𝑑𝑥 = ∫ ∫ 3 3 3 3 0 −1 1 4 1 =[ 𝑥 + 3 8 [

=

2

2−𝑥

25.

2

𝑦 𝑑𝑦 𝑑𝑥 =

2 2−𝑥 2 1 1 1 8 4 [ 𝑦 2] 𝑑𝑥 = (2 − 𝑥) 2 𝑑𝑥 = [− (2 − 𝑥) 3] = = 𝑦=0 𝑥=0 ∫ ∫ 2 2 6 6 3 0 0

∫ 0 ∫ 0 4∕3 2 1 𝐴 = (2)(2) = 2 by the geometric formula for the area of a triangle. So, the average value = = . 2 2 3 1

26.

1−𝑥 2

1

(2 + 𝑥) 𝑑𝑦 𝑑𝑥 =

∫ 0 −1 ∫ 1

∫ −1

2

𝑑𝑥 = [(2 + 𝑥)𝑦]𝑦1−𝑥 =0

1

∫ −1

(2 + 𝑥)(1 − 𝑥 2) 𝑑𝑥

1 2 2 3 1 4 1 𝑥 − 𝑥 − 𝑥 ] 𝑥 = −1 2 3 4 1 2 1 1 2 1 8 =2+ − − +2− − + = 2 3 4 2 3 4 3 1 8∕3 1 1 1 1 4 𝐴= (1 − 𝑥 2) 𝑑𝑥 = [𝑥 − 𝑥 3] = 1 − + 1 − = . So, the average value = = 2. −1 ∫ 3 3 3 3 4∕3 −1 =

1

(2 + 𝑥 − 2𝑥 2 − 𝑥 3) 𝑑𝑥 = [2𝑥 +

∫ −1

1−𝑦

1

28.

𝑒 𝑦 𝑑𝑥 𝑑𝑦 =

1

[𝑥𝑒 𝑦]𝑥1−𝑦 𝑑𝑦 = = 𝑦−1

1

2(1 − 𝑦)𝑒 𝑦 𝑑𝑦 = [2(1 − 𝑦)𝑒 𝑦 + 2𝑒 𝑦]𝑦1 = 0 ∫ ∫ ∫ 0 ∫ 𝑦−1 0 0 = 2𝑒 − 4 = 2(𝑒 − 2) 1 𝐴 = (2)(1) = 1 by the formula for the area of a triangle. So, the average value = 2(𝑒 − 2). 2 27.

1−𝑦 2

∫ 0 −1 ∫

𝑦 𝑑𝑥 𝑑𝑦 =

1

∫ −1

1−𝑦 2

[𝑥𝑦]𝑥 = 0 𝑑𝑦 =

1

∫ −1

1 1 1 (𝑦 − 𝑦 3) 𝑑𝑦 = [ 𝑦 2 − 𝑦 4] 𝑦 = −1 2 4


Solutions Section 15.5 1 1 1 1 − − + =0 2 4 2 4 So, the average value = 0∕𝐴 = 0. (We don't need to calculate 𝐴.) =

0

𝑥+1

1

−𝑥+1

2 (𝑥 2 + 𝑦 2) 𝑑𝑦 𝑑𝑥 = , as calculated in Exercise 23. 3 2∕3 1 𝐴 = (√2) 2 = 2 since 𝐴 is the area of a square of side √2. So, the average value = = . 2 3 29.

∫ −1 ∫ −𝑥−1

0

𝑥+2

(𝑥 2 + 𝑦 2) 𝑑𝑦 𝑑𝑥 +

2

∫ 0 ∫ 𝑥−1

−𝑥+2

4 1 𝑥 2 𝑑𝑦 𝑑𝑥 = , as calculated in Exercise 24. 𝐴 = 2 ⋅ (2)(1) = 2, 3 2 4∕3 2 since 𝐴 consists of the area of two triangles. So, the average value = = . 2 3 30.

∫ 𝑥∕2+1 −2 ∫

1

31.

𝑓(𝑥, 𝑦) 𝑑𝑦 𝑑𝑥

∫ 0 ∫ 0

1

∫ 0 ∫ 𝑥−1

√2

33.

34.

∫ 0 ∫ −𝑥∕2+1

1−𝑥

2

32.

𝑥 2 𝑑𝑦 𝑑𝑥 +

𝑓(𝑥, 𝑦) 𝑑𝑦 𝑑𝑥

1

∫ 0

∫ 𝑥 2−1

1

1−𝑥 2

∫ 0 ∫ −1

𝑓(𝑥, 𝑦) 𝑑𝑦 𝑑𝑥

𝑓(𝑥, 𝑦) 𝑑𝑦 𝑑𝑥


Solutions Section 15.5 4

35.

2∕√𝑦

𝑒2

2

36.

𝑓(𝑥, 𝑦) 𝑑𝑥 𝑑𝑦

∫ 1 ∫ 1

𝑓(𝑥, 𝑦) 𝑑𝑥 𝑑𝑦

∫ 0 ∫ 𝑒𝑦

1

2

(1 − 𝑥 2) 𝑑𝑦 𝑑𝑥 =

37. 𝑉 =

∫ 0 ∫ 0 2 4 =2− = 3 3 1

38. 𝑉 =

1−𝑥

∫ 0 ∫ 0 1

=

∫ 0

1

∫ 0

(1 − 𝑥 2) 𝑑𝑦 𝑑𝑥 =

[(1 − 𝑥 2)𝑦]𝑦2 = 0 𝑑𝑥 =

1

∫ 0

1

∫ 0

[(1 − 𝑥 2)𝑦]𝑦1−𝑥 = 0 𝑑𝑥 =

(1 − 𝑥 − 𝑥 2 + 𝑥 3) 𝑑𝑥 = [𝑥 −

2(1 − 𝑥 2) 𝑑𝑥 = [2𝑥 −

2 3 1 𝑥 ] 𝑥=0 3

1

∫ 0

(1 − 𝑥 2)(1 − 𝑥) 𝑑𝑥

1 2 1 3 1 4 1 1 1 1 5 𝑥 − 𝑥 + 𝑥 ] =1− − + = 𝑥=0 2 3 4 2 3 4 12

39. The top face of the tetrahedron is the graph of 𝑧 = 1 − 𝑥 − 𝑦. So, the volume is 1 1−𝑥 1 1 1−𝑥 1 1 𝑉 = (1 − 𝑥 − 𝑦) 𝑑𝑦 𝑑𝑥 = [(1 − 𝑥)𝑦 − 𝑦 2] 𝑑𝑥 = [(1 − 𝑥) 2 − ] 𝑑𝑥 𝑦=0 ∫ ∫ ∫ ∫ 2 2 0 0 0 0 1 1 1 1 1 (1 − 𝑥) 2𝑑𝑥 = [− (1 − 𝑥) 3] = = . 𝑥=0 ∫ 2 6 6 0 𝑥 𝑦 𝑧 𝑥 𝑦 + + = 1, or 𝑧 = 𝑐(1 − − ). So, the volume is 𝑎 𝑏 𝑐 𝑎 𝑏 𝑎 𝑏(1−𝑥∕𝑎) 𝑎 𝑥 𝑦 𝑥 1 2 𝑏(1−𝑥∕𝑎) 𝑉 = 𝑐(1 − − ) 𝑑𝑦 𝑑𝑥 = 𝑐 [(1 − )𝑦 − 𝑦 ] 𝑑𝑥 𝑦=0 ∫ ∫ 𝑎 𝑏 𝑎 2𝑏 0 ∫ 0 0 𝑎 𝑥 2 𝑏 𝑥 2 𝑏𝑐 𝑎 𝑥 2 𝑎𝑏𝑐 𝑥 3 𝑎 𝑎𝑏𝑐 [𝑏(1 − ) − (1 − ) ] 𝑑𝑥 = (1 − ) 𝑑𝑥 = [(1 − ) ] . =𝑐 = 𝑥 0 = ∫ 𝑎 2 𝑎 2 ∫ 𝑎 6 𝑎 6 0 0 40. The top face of the tetrahedron is the graph of

55

12

55

1 0.3 1.7 12 [ 𝑥 𝑦 ] 𝑑𝑥 𝑦=8 ∫ ∫ ∫ 1.7 45 8 45 10,000(12 1.7 − 8 1.7) 55 0.3 10,000(12 1.7 − 8 1.7) 1.3 55 [𝑥 ]𝑥 = 45 𝑥 𝑑𝑥 = = ∫ 1.7 (1.7)(1.3) 45 10,000(12 1.7 − 8 1.7)(55 1.3 − 45 1.3) ≈ 6,471,000 = (1.7)(1.3) 6,471,000 𝐴 = (12 − 8)(55 − 45) = 40. So, the average is approximately ≈ 162,000 gadgets. 40 41.

10,000𝑥 0.3𝑦 0.7 𝑑𝑦 𝑑𝑥 = 10,000


Solutions Section 15.5 55

12

55

1 0.7 1.3 12 [ 𝑥 𝑦 ] 𝑑𝑥 𝑦=8 ∫ ∫ ∫ 1.3 45 8 45 10,000(12 1.3 − 8 1.3) 55 0.7 10,000(12 1.3 − 8 1.3) 1.7 55 [𝑥 ]𝑥 = 45 𝑥 𝑑𝑥 = = ∫ 1.3 (1.3)(1.7) 45 10,000(12 1.3 − 8 1.3)(55 1.7 − 45 1.7) ≈ 12,319,000 = (1.3)(1.7) 12,319,000 𝐴 = (12 − 8)(55 − 45) = 40. So, the average is approximately ≈ 308,000 gadgets. 40 10,000𝑥 0.7𝑦 0.3 𝑑𝑦 𝑑𝑥 = 10,000

42.

43. 𝑅(𝑝, 𝑞) = 𝑝𝑞 with 40 ≤ 𝑝 ≤ 50 and 8,000 − 𝑝 2 ≤ 𝑞 ≤ 10,000 − 𝑝 2. To find the average we first compute 10,000−𝑝 2

50

𝑝𝑞 𝑑𝑞 𝑑𝑝 =

1 50 [ 2] 10,000−𝑝 2 1 50[ 𝑝𝑞 𝑑𝑝 = 𝑝(10,000 − 𝑝 2) 2 − 𝑝(8,000 − 𝑝 2) 2]𝑑𝑝 2 𝑞 8,000−𝑝 = ∫ 2∫ 2 40 40

∫ 40 ∫ 8,000−𝑝 2 1 [ 50 = −(10,000 − 𝑝 2) 3 + (8,000 − 𝑝 2) 3]𝑝 = 40 = 6,255,000,000. 12 Next we compute 10,000−𝑝 2

50

𝐴=

∫ 40 ∫ 8,000−𝑝 2

50

𝑑𝑞 𝑑𝑝 =

∫ 40

[𝑞 ]

50 10,000−𝑝 2 𝑑𝑝 = [10,000 − 𝑝 2 − (8,000 − 𝑝 2)] 𝑑𝑝 2 𝑞 = 8,000−𝑝 ∫ 40

50

=

∫ 40

2,000 𝑑𝑝 = [2,000𝑝]𝑝50= 40 = 100,000 − 80,000 = 20,000.

The average revenue is thus

6,255,000,000 = $312,750. 20,000

44. 𝑅(𝑝, 𝑞) = 𝑝𝑞 with 300 ≤ 𝑝 ≤ 400 and 180,000 − 𝑝 2 ≤ 𝑞 ≤ 200,000 − 𝑝 2. To find the average we first compute 200,000−𝑝 2

400

∫ 300 ∫ 180,000−𝑝 2

𝑝𝑞 𝑑𝑞 𝑑𝑝 =

1 400 [ 2] 200,000−𝑝 2 𝑝𝑞 𝑑𝑝 𝑞 = 180,000−𝑝 2 2∫ 300

1 400[ 𝑝(200,000 − 𝑝 2) 2 − 𝑝(180,000 − 𝑝 2) 2]𝑑𝑝 2∫ 300 1 [ 400 = −(200,000 − 𝑝 2) 3 + (180,000 − 𝑝 2) 3]𝑝 = 300 = 4.55 × 10 13. 12 Next we compute =

400

𝐴=

200,000−𝑝 2

400

𝑑𝑞 𝑑𝑝 =

[𝑞 ]

400 200,000−𝑝 2 𝑑𝑝 = 20,000𝑑𝑝 2 𝑞 = 180,000−𝑝 ∫ 300

∫ ∫ 300 ∫ 180,000−𝑝 2 300 400 = [20,000𝑝]𝑝 = 300 = 8,000,000 − 6,000,000 = 2,000,000.

The average revenue is thus

4.55 × 10 13 = $22,750,000. 2,000,000

45. 𝑅(𝑝, 𝑞) = 𝑝𝑞 with 15,000∕𝑞 ≤ 𝑝 ≤ 20,000∕𝑞 and 500 ≤ 𝑞 ≤ 1,000. To find the average we first compute 1,000 20,000∕𝑞 1 1,000 [ 2 ] 20,000∕𝑞 1 1,000 20,000 2 15,000 2 𝑝𝑞 𝑑𝑝 𝑑𝑞 = 𝑝 𝑞 𝑝 = 15,000∕𝑞 𝑑𝑞 = ( ) 𝑑𝑞 − ∫ 2∫ 2∫ 𝑞 𝑞 500 ∫ 15,000∕𝑞 500 500 175,000,000 1,000 1 175,000,000 175,000,000 𝑑𝑞 = (ln 1,000 − ln 500) = = [ln 𝑞]𝑞1,000 = 500 ∫ 2 𝑞 2 2 500 175,000,000 ln 2 = 2 Next we compute 1,000 20,000∕𝑞 1,000 1,000 1 20,000∕𝑞 𝐴= 𝑑𝑝 𝑑𝑞 = 𝑑𝑞 = 500[ln 𝑞 [ 𝑝]𝑝 = 15,000∕𝑞 𝑑𝑞 = 5,000 ∫ ∫ ∫ ∫ 𝑞 500 15,000∕𝑞 500 500 = 5,000(ln 1,000 − ln 500) = 5,000 ln 2. 175,000,000 ln 2 The average revenue is thus = $17,500. 2 × 5,000 ln 2


Solutions Section 15.5 46. 𝑅(𝑝, 𝑞) = 𝑝𝑞 with 80,000∕𝑞 ≤ 𝑝 ≤ 100,000∕𝑞 2 and 50 ≤ 𝑞 ≤ 100. To find the average we first compute 2

100,000∕𝑞 2

100

1 100 [ 2 ] 100,000∕𝑞 2 𝑝 𝑞 𝑑𝑞 𝑝 = 80,000∕𝑞 2 ∫ 2∫ 50 ∫ 80,000∕𝑞 2 50 100,000 2 − 80,000 2 100 1 1 100 𝑑𝑞 = 1,800,000,000[− 2 ] = = 270,000. 3 ∫ 2 2𝑞 𝑞 = 50 50 𝑞 Next we compute 𝑝𝑞 𝑑𝑝 𝑑𝑞 =

100,000∕𝑞 2

100

𝐴=

100

[ 𝑝]

𝑑𝑝 𝑑𝑞 =

100,000∕𝑞 2

𝑝 = 80,000∕𝑞 ∫ ∫ 50 ∫ 80,000∕𝑞 2 50 1 100 = 20,000[− ] = 200. 𝑞 𝑞 = 50 270,000 The average revenue is thus = $1,350. 200

100

2 𝑑𝑞 = 20,000

1 𝑑𝑞 2 ∫ 50 𝑞

47. Total population 20

=

30

∫ 0 ∫ 0

𝑒 −0.1(𝑥+𝑦) 𝑑𝑦 𝑑𝑥 = −10

20

∫ 0

[𝑒 −0.1(𝑥+𝑦)]𝑦30= 0 𝑑𝑥 = −10

20

∫ 0

(𝑒 −0.1𝑥−3 − 𝑒 −0.1𝑥) 𝑑𝑥

20

20 𝑒 −0.1𝑥 𝑑𝑥 = 100(𝑒 −3 − 1)[𝑒 −0.1𝑥]𝑥 = 0 = 100(𝑒 −3 − 1)(𝑒 −2 − 1) ∫ 0 ≈ 82.16 hundred people, or 8,216 people.

= −10(𝑒 −3 − 1)

20

48. Total population =

30−3𝑥∕2

𝑒 −0.1(𝑥+𝑦) 𝑑𝑦 𝑑𝑥 = −10

∫ 0 ∫ 0

20

[𝑒 −0.1(𝑥+𝑦)] 30−3𝑥∕2𝑑𝑥 𝑦=0

∫ 0

20

= −10

∫ 0

(𝑒 −0.05𝑥−3 − 𝑒 −0.1𝑥) 𝑑𝑥 20

= −10[20𝑒 0.05𝑥−3 + 10𝑒 −0.1𝑥]𝑥 = 0 ≈ 69.36 hundred people, or 6,936 people. 1

1

1 2 3 1 2 1 2 1 𝑦 ] 𝑑𝑥 = (𝑥 2 + ) 𝑑𝑥 = [ 𝑥 3 + 𝑥] = 1. 𝑦=0 ∫ ∫ ∫ ∫ 3 3 3 3 𝑥=0 0 0 0 0 The area of the square is 𝐴 = 1. Hence, the average temperature is 1/1 = 1 degree Celsius. 1

1

(𝑥 2 + 2𝑦 2) 𝑑𝑦 𝑑𝑥 =

49.

1

(𝑥 2 + 2𝑦 2 − 𝑥) 𝑑𝑦 𝑑𝑥 =

50.

[𝑥 2𝑦 +

1

[𝑥 2𝑦 +

1 1 2 3 2 𝑦 − 𝑥𝑦] 𝑑𝑥 = (𝑥 2 + − 𝑥) 𝑑𝑥 𝑦 0 = ∫ 3 3 0

∫ ∫ 0 ∫ 0 0 1 3 2 1 2 1 =[ 𝑥 + 𝑥− 𝑥 ] = 0.5. 𝑥=0 3 3 2 The area of the square is 𝐴 = 1. Hence, the average temperature is 0.5/1 = 0.5 degrees Celsius. 51. The area between the curves 𝑦 = 𝑟(𝑥) and 𝑦 = 𝑠(𝑥) and the vertical lines 𝑥 = 𝑎 and 𝑥 = 𝑏 is given 𝑏

𝑠(𝑥)

by

∫ 𝑎 ∫ 𝑟(𝑥)

𝑑𝑦 𝑑𝑥, assuming that 𝑟(𝑥) ≤ 𝑠(𝑥) for 𝑎 ≤ 𝑥 ≤ 𝑏.

52. The area of the solid above the region 𝑅 and under the surface 𝑧 = 𝑓(𝑥, 𝑦) is given by the double integral

∬ 𝑅

𝑓(𝑥, 𝑦) 𝑑𝑦 𝑑𝑥. 𝑏

53. The first step in calculating an integral of the form

𝑠(𝑥)

∫ 𝑎 ∫ 𝑟(𝑥)

𝑓(𝑥, 𝑦) 𝑑𝑦 𝑑𝑥 is to evaluate the integral

𝑠(𝑥)

∫ 𝑟(𝑥)

𝑓(𝑥, 𝑦)𝑑𝑦, obtained by holding 𝑥 constant and integrating with respect to 𝑦.


Solutions Section 15.5 54. Zonars 55. Paintings per picasso per dali 56. 𝑦 then 𝑥 𝑏

𝑑

𝑏

𝑓(𝑥)𝑔(𝑦) 𝑑𝑥 𝑑𝑦 =

(𝑔(𝑦)

57. The left-hand side is

∫ 𝑎 ∫ 𝑐 constant in the inner integral] 𝑑

𝑏

∫ 𝑎

𝑑

∫ 𝑐

𝑓(𝑥)𝑑𝑥)𝑑𝑦 [because 𝑔(𝑦) is treated as a

𝑑

𝑓(𝑥)𝑑𝑥)( 𝑔(𝑦)𝑑𝑦) [because 𝑓(𝑥)𝑑𝑥 is a constant and can therefore be taken outside the =( ∫ ∫ ∫ 𝑐 𝑎 𝑐 integral with respect to 𝑦]. 1 2 2 1 1 For example, 𝑦𝑒 𝑥 𝑑𝑥 𝑑𝑦 = (𝑒 2 − 𝑒) if we compute it as an iterated integral or as 𝑒 𝑥 𝑑𝑥 𝑦 𝑑𝑦. ∫ ∫ ∫ 2 0 ∫ 1 1 0 𝑏

58. Write

𝑑 𝑓(𝑥)

∫ 𝑎 ∫ 𝑐 𝑔(𝑦)

𝑏

𝑑𝑥 𝑑𝑦 =

𝑑

∫ 𝑎 ∫ 𝑐

𝑓(𝑥)

𝑑 𝑏 1 1 𝑑𝑥 𝑑𝑦 = 𝑓(𝑥) 𝑑𝑥 𝑑𝑦 using Exercise 57. ∫ ∫ 𝑔(𝑦) 𝑐 𝑎 𝑔(𝑦)


Solutions Chapter 15 Review Chapter 15 Review 𝑥 + 𝑥 2𝑦 𝑦 + 𝑥𝑧 0 2 𝑓(0, 1, 1) = 𝑓(2, 1, 1) = + 0 2(1) = 0; + 2 2(1) = 14∕3 1 + (0)(1) 1 + (2)(1) −1 𝑓(−1, 1, −1) = + (−1) 2(1) = 1∕2 1 + (−1)(−1) 𝑧 1 𝑓(𝑧, 𝑧, 𝑧) = + 𝑧 2𝑧 = + 𝑧 3; 2 1 𝑧 + 𝑧+𝑧 𝑥+ℎ 𝑓(𝑥 + ℎ, 𝑦 + 𝑘, 𝑧 + 𝑙) = + (𝑥 + ℎ) 2(𝑦 + 𝑘) 𝑦 + 𝑘 + (𝑥 + ℎ)(𝑧 + 𝑙)

1. 𝑓(𝑥, 𝑦, 𝑧) =

2. 𝑔(𝑥, 𝑦, 𝑧) = 𝑥𝑦(𝑥 + 𝑦 − 𝑧) + 𝑥 2 𝑔(0, 0, 0) = 0; 𝑔(1, 0, 0) = 1; 𝑔(0, 1, 0) = 0; 𝑔(𝑥, 𝑦 + 𝑘, 𝑧) = 𝑥(𝑦 + 𝑘)(𝑥 + 𝑦 + 𝑘 − 𝑧) + 𝑥 2

𝑔(𝑥, 𝑥, 𝑥) = 𝑥 3 + 𝑥 2

3. decreases by 0.32 units; increases by 12.5 units 4. increases by 11 units; decreases by 1.53 units 5.

6. 𝑥→

𝑦 ↓

𝑥→ −1

0

1

−1

4

0

0

0

3

0

1

2

0

𝑦 ↓

−2

2

3

−2

14

2

9

1

2

6

10

21

2

3

4

12

24

7. Answers may vary; two examples are 𝑓(𝑥, 𝑦) = 3(𝑥 − 𝑦)∕2 and 𝑓(𝑥, 𝑦) = 3(𝑥 − 𝑦) 3∕8. 8. 𝑓(𝑥 + 1, 𝑦 − 1) = (𝑥 + 1) 2 + 𝑦 2 = 𝑓(𝑦, 𝑥) 9.

10.


Solutions Chapter 15 Review 11.

12.

13.

14.

15. 𝑓(𝑥, 𝑦) = 𝑥 2 + 𝑥𝑦; 𝑓𝑥 = 2𝑥 + 𝑦; 𝑓𝑦 = 𝑥; 𝑓𝑦𝑦 = 0 16. 𝑓(𝑥, 𝑦) =

𝑥𝑦 𝑦 6 6 6 𝑥 12 + ; 𝑓𝑥 = − 2 + ; 𝑓𝑦 = − 2 + ; 𝑓𝑦𝑦 = 3 𝑥𝑦 6 6 𝑥 𝑦 6 𝑥𝑦 𝑥𝑦

17. 𝑓(𝑥, 𝑦) = 4𝑥 + 5𝑦 − 6𝑥𝑦; 𝑓𝑥 (𝑥, 𝑦) = 4 − 6𝑦, so 𝑓𝑥𝑥 (𝑥, 𝑦) = 0. Therefore 𝑓𝑥𝑥 (1, 0) − 𝑓𝑥𝑥 (3, 2) = 0. 18.

2 2 𝜕 2𝑓 2 2 𝜕𝑓 = 𝑦𝑒 𝑥𝑦 + 6𝑥𝑒 3𝑥 −𝑦 , = (𝑥𝑦 + 1)𝑒 𝑥𝑦 − 12𝑥𝑦𝑒 3𝑥 −𝑦 𝜕𝑥 𝜕𝑥𝜕𝑦

19.

𝜕𝑓 −𝑥 2 + 𝑦 2 + 𝑧 2 ; = 𝜕𝑥 (𝑥 2 + 𝑦 2 + 𝑧 2) 2

𝜕𝑓 2𝑥𝑦 ; =− 2 𝜕𝑦 (𝑥 + 𝑦 2 + 𝑧 2) 2

𝜕𝑓 2𝑥𝑧 ; =− 2 𝜕𝑧 (𝑥 + 𝑦 2 + 𝑧 2) 2

𝜕𝑓 | =1 𝜕𝑥 |(0,1,0)

20. 𝑓𝑥𝑥 + 𝑓𝑦𝑦 + 𝑓𝑧𝑧 = 2 + 2 + 2 = 6 21. 𝑓𝑥 = 2(𝑥 − 1); 𝑓𝑦 = 2(2𝑦 − 3); 𝑓𝑥𝑥 = 2; 𝑓𝑦𝑦 = 4; 𝑓𝑥𝑦 = 0. 𝑓𝑥 = 0 when 𝑥 = 1; 𝑓𝑦 = 0 when 𝑦 = 3∕2, so (1, 3∕2) is the only critical point. 𝐻 = 8 and 𝑓𝑥𝑥 > 0, so 𝑓 has an absolute minimum at (1, 3∕2). 22. 𝑔𝑥 = 2(𝑥 − 1); 𝑔𝑦 = −6𝑦; 𝑔𝑥𝑥 = 2; 𝑔𝑦𝑦 = −6; 𝑔𝑥𝑦 = 0. 𝑔𝑥 = 0 when 𝑥 = 1; 𝑔𝑦 = 0 when 𝑦 = 0, so (1, 0) is the only critical point. 𝐻 = −12 < 0, so 𝑔 has a saddle point at (1, 0). 23. 𝑘𝑥 = 2𝑥𝑦 − 2𝑥 = 2𝑥(𝑦 − 1); 𝑘𝑦 = 𝑥 2 − 2𝑦; 𝑘𝑥𝑥 = 2𝑦 − 2; 𝑘𝑦𝑦 = −2; 𝑘𝑥𝑦 = 2𝑥. 𝑘𝑥 = 0 when 𝑥 = 0 or 𝑦 = 1; 𝑘𝑦 = 0 when 𝑥 2 = 2𝑦. 𝑥 = 0 gives the critical point (0, 0) and 𝑦 = 1 gives two more at (±√2, 1). At (0, 0), 𝐻 = 4 > 0 and 𝑘𝑥𝑥 < 0, so 𝑘 has a maximum at (0, 0). At (±√2, 1), 𝐻 = −8 < 0, so 𝑘 has


Solutions Chapter 15 Review saddle points at (±√2, 1). 24. 𝑗𝑥 = 𝑦 + 2𝑥; 𝑗𝑦 = 𝑥; 𝑗𝑥𝑥 = 2; 𝑗𝑦𝑦 = 0; 𝑗𝑥𝑦 = 1. 𝑗𝑥 = 0 when 𝑦 = −2𝑥; 𝑗𝑦 = 0 when 𝑥 = 0, so (0, 0) is the only critical point. 𝐻 = −1 < 0, so 𝑗 has a saddle point at (0, 0). 25. ℎ𝑥 = 𝑦𝑒 𝑥𝑦; ℎ𝑦 = 𝑥𝑒 𝑥𝑦; ℎ𝑥𝑥 = 𝑦 2𝑒 𝑥𝑦; ℎ𝑦𝑦 = 𝑥 2𝑒 𝑥𝑦; ℎ𝑥𝑦 = (𝑥𝑦 + 1)𝑒 𝑥𝑦. ℎ𝑥 = 0 when 𝑦 = 0; ℎ𝑦 = 0 when 𝑥 = 0, so (0, 0) is the only critical point. 𝐻(0, 0) = −1 < 0, so ℎ has a saddle point at (0, 0). 26. Note that the domain of 𝑓 contains every point except (0, 0). 2𝑦 2𝑥 2 − 2𝑦 2 −2𝑥 2 + 2𝑦 2 2𝑥 𝑓𝑥 = 2 2𝑥; 𝑓 2𝑦; 𝑓 2; 𝑓 − = − = − = − 2; 𝑦 𝑥𝑥 𝑦𝑦 𝑥 + 𝑦2 𝑥2 + 𝑦2 (𝑥 2 + 𝑦 2) 2 (𝑥 2 + 𝑦 2) 2 −4𝑥𝑦 𝑓𝑥𝑦 = . 𝑓𝑥 = 0 if either 𝑥 = 0 or 𝑥 2 + 𝑦 2 = 1; 𝑓𝑦 = 0 if either 𝑦 = 0 or 𝑥 2 + 𝑦 2 = 1; since 2 2 2 (𝑥 + 𝑦 ) (0, 0) is not in the domain, the critical points are all the points on the circle 𝑥 2 + 𝑦 2 = 1. At such a point we can substitute 𝑦 2 = 1 − 𝑥 2 to get 𝑓𝑥𝑥 = (2 − 4𝑥 2) − 2 = −4𝑥 2, 𝑓𝑦𝑦 = 4𝑥 2 − 4; and 𝑓𝑥𝑦 = ±4𝑥(1 − 𝑥 2) 1∕2. Hence 𝐻 = −4𝑥 2(4𝑥 2 − 4) − 16𝑥 2(1 − 𝑥 2) = 0. To resolve what happens along the circle we graph the function:

𝑓 has an absolute maximum at each point on the circle 𝑥 2 + 𝑦 2 = 1. 27. The objective function is 𝑉 = 𝑥𝑦𝑧. Solving the constraint equation 𝑥 + 𝑦 + 𝑧 = 1 for 𝑧 gives 𝑧 = 1 − 𝑥 − 𝑦. Substituting in the objective function gives 𝑉 = 𝑥𝑦(1 − 𝑥 − 𝑦) = 𝑥𝑦 − 𝑥 2𝑦 − 𝑥𝑦 2. 𝑉𝑥 = 𝑦 − 2𝑥𝑦 − 𝑦 2, 𝑉𝑦 = 𝑥 − 𝑥 2 − 2𝑥𝑦 Critical points: 𝑦 − 2𝑥𝑦 − 𝑦 2 = 0 𝑥 − 𝑥 2 − 2𝑥𝑦 = 0 Dividing the first equation by 𝑦 gives 1 − 2𝑥 − 𝑦 = 0 ⇒ 2𝑥 + 𝑦 = 1. Dividing the second equation by 𝑥 gives 1 − 𝑥 − 2𝑦 = 0 ⇒ 𝑥 + 2𝑦 = 1. The system of equations 2𝑥 + 𝑦 = 1 𝑥 + 2𝑦 = 1 has solution (𝑥, 𝑦) = (1∕3, 1∕3). The corresponding value of 𝑧 is 𝑧 = 1 − 𝑥 − 𝑦 = 1 − 2∕3 = 1∕3. 𝑉𝑥𝑥 = −2𝑦, 𝑉𝑥𝑦 = 1 − 2𝑥 − 2𝑦, 𝑉𝑦𝑦 = −2𝑥 Evaluating these at (1/3, 1/3) gives 𝑉𝑥𝑥 = −2∕3, 𝑉𝑥𝑦 = −1∕3, 𝑉𝑦𝑦 = −2∕3 ⇒ 𝐻 = 𝑉𝑧𝑧 𝑉𝑦𝑦 − (𝑉𝑥𝑦 ) 2 = 4∕9 − 1∕9 > 0. 𝑉𝑥𝑥 < 0, so 𝑉 does have a relative maximum at (1∕3, 1∕3, 1∕3). The value of 𝑉 at that point is 𝑉 = 𝑥𝑦𝑧 = 1∕27. That it is the absolute maximum can be seen as interpreting the problems as maximizing the volume of a box whose dimensions add up to 1. 28. The objective function is 𝑥 2 + 𝑦 2 + 𝑧 2 − 1. Substituting the constraint equation 𝑥 = 𝑦 + 𝑧 gives the objective function as ℎ(𝑦, 𝑧) = (𝑦 + 𝑧) 2 + 𝑦 2 + 𝑧 2 − 1 = 2𝑦 2 + 2𝑧 2 + 2𝑦𝑧 − 1. ℎ𝑦 = 4𝑦 + 2𝑧, ℎ𝑦 = 4𝑧 + 2𝑦


Solutions Chapter 15 Review Critical points: 4𝑦 + 2𝑧 = 0 4𝑧 + 2𝑦 = 0 (𝑦, 𝑧) = (0, 0) is the only critical point. ℎ𝑦𝑦 = 4, ℎ𝑦𝑧 = 2, ℎ𝑧𝑧 = 4 ⇒ 𝐻 = ℎ𝑦𝑦 ℎ𝑧𝑧 − (ℎ𝑦𝑧 ) 2 = 14 > 0 Since ℎ𝑦𝑦 > 0, the critical point is a local minimum. That it is an absolute minimum can be seen by considering the graph of ℎ(𝑥, 𝑦) = 2𝑦 2 + 2𝑧 2 + 2𝑦𝑧 − 1. The corresponding value of the objective function is ℎ(0, 0) = 2(0) 2 + 2(0) 2 + 2(0)(0) − 1 = −1. When 𝑦 = 0 and 𝑧 = 0, 𝑥 = 𝑦 + 𝑧 = 0 as well, so 𝑓 has an absolute minimum of −1 at the point (0, 0, 0). 29. We minimize the square of the distance of (𝑥, 𝑦, 𝑧) to the origin: 𝑑(𝑥, 𝑦, 𝑧) = 𝑥 2 + 𝑦 2 + 𝑧 2 subject to 𝑧 = √𝑥 2 + 2(𝑦 − 3) 2, which we rewrite as 𝑥 2 + 2(𝑦 − 3) 2 − 𝑧 2 = 0 (𝑧 ≥ 0). Solving the second equation for 𝑧 2 and substituting in the formula for 𝑑 gives 𝑑 = 𝑥 2 + 𝑦 2 + 𝑥 2 + 2(𝑦 − 3) 2 = 2𝑥 2 + 3𝑦 2 − 12𝑦 + 18 𝑑𝑥 = 4𝑥, 𝑑𝑦 = 6𝑦 − 12. The critical points are 4𝑥 = 0 ⇒ 𝑥=0 6𝑦 − 12 = 0 ⇒ 𝑦 = 2. The corresponding value of 𝑧 is 𝑧 = √𝑥 2 + 2(𝑦 − 3) 2 = √0 2 + 2(2 − 3) 2 = √2. The critical point is therefore (0, 2, √2). Since there must be at least one point on the given surface closest to the origin, the critical point (0, 2, √2) must be that point. 30. The objective function is 𝑆 = 𝑥𝑦 + 𝑥 2𝑧 2 + 4𝑦𝑧. Solving the constraint equation 𝑥𝑦𝑧 = 1 for 𝑧 gives 1 𝑧= . 𝑥𝑦 1 2 1 1 4 Substituting in the objective function gives 𝑆 = 𝑥𝑦 + 𝑥 2( ) + 4𝑦( ) = 𝑥𝑦 + 2 + . 𝑥𝑦 𝑥𝑦 𝑥 𝑦 4 2 𝑆𝑥 = 𝑦 − 2 , 𝑆𝑦 = 𝑥 − 3 𝑥 𝑦 Critical points: 4 𝑦− 2 =0 𝑥 2 𝑥− 3 =0 𝑦 Substituting the first equation in the second gives 2 𝑥− ⇒ 𝑥 − 𝑥 6∕32 = 0 ⇒ 𝑥(1 − 𝑥 5∕32) = 0 ⇒ 𝑥 5 = 32, or 𝑥 = 2. =0 (4∕𝑥 2) 3 4 (We reject 𝑥 = 0 because 𝑥 > 0.) Substituting 𝑥 = 2 into the equation 𝑦 − 2 = 0 gives 𝑦 = 1. 𝑥 (𝑥, 𝑦) = (2, 1) is the only critical point. That it is an absolute minimum can be seen by graphing 1 4 𝑆(𝑥, 𝑦) = 𝑥𝑦 + 2 + for 𝑥 > 0, 𝑦 > 0. (The second derivative test can verify that it is a relative 𝑥 𝑦 minimum.) The corresponding value of the objective function is 𝑆(2, 1) = (2)(1) + 1 + 2 = 5. When 𝑥 = 2 and 𝑦 = 1, 𝑧 = 1∕𝑥𝑦 = 1∕2, so 𝑆 has an absolute minimum of 5 at (2, 1, 1∕2). 31. The constraint is 𝑥𝑦 = 2, or 𝑥𝑦 − 2 = 0. 𝑓(𝑥, 𝑦) = 𝑥 2 + 𝑦 2, 𝑔(𝑥, 𝑦) = 𝑥𝑦 − 2, 𝐿(𝑥, 𝑦) = 𝑥 2 + 𝑦 2 − 𝜆(𝑥𝑦 − 2) (1) 𝐿𝑥 = 0 ⇒ 2𝑥 = 𝜆𝑦 ⇒ 𝜆 = 2𝑥∕𝑦 (2) 𝐿𝑦 = 0

2𝑦 = 𝜆𝑥

(3) Constraint ⇒ 𝑥𝑦 = 2 Substitute Equation (1) in Equation (2) to obtain


Solutions Chapter 15 Review 2𝑥 ∕𝑦 = 2𝑦 ⇒ 𝑥 =𝑦 ⇒ 𝑥 = ±𝑦. Substituting into the constraint gives 𝑥 2 = 2 (we must reject 𝑦 = −𝑥 here) ⇒ 𝑥 = ±√2, so that 𝑦 = 2∕𝑥 = ±√2. Thus we have two critical points: (√2, √2) and (−√2, −√2). The corresponding value of the objective function is 𝑓(±√2, ±√2) = 2 + 2 = 4. 2

2

2

32. The constraint is 𝑥 = 𝑦 + 𝑧, or 𝑥 − 𝑦 − 𝑧 = 0. 𝑓(𝑥, 𝑦, 𝑧) = 𝑥 2 + 𝑦 2 + 𝑧 2 − 1, 𝑔(𝑥, 𝑦, 𝑧) = 𝑥 − 𝑦 − 𝑧, 𝐿(𝑥, 𝑦, 𝑧) = 𝑥 2 + 𝑦 2 + 𝑧 2 − 1 − 𝜆(𝑥 − 𝑦 − 𝑧) (1) 𝐿𝑥 = 0 ⇒ 2𝑥 = 𝜆 (2) 𝐿𝑦 = 0

2𝑦 = −𝜆

(3) 𝐿𝑧 = 0

2𝑧 = −𝜆

(4) Constraint ⇒ 𝑥=𝑦+𝑧 Substituting (1) in (2) gives 2𝑦 = −2𝑥, or 𝑦 = −𝑥. Substituting (1) in (3) gives 2𝑧 = −2𝑥, or 𝑧 = −𝑥. Combining this information with the constraint equation 𝑥 − 𝑦 − 𝑧 = 0 gives 𝑥+𝑥+𝑥=0 ⇒ 𝑥 = 0. Thus the only critical point is (𝑥, 𝑦, 𝑧) = (0, 0, 0), and the corresponding value of the objective is 𝑓(0, 0, 0) = 0 2 + 0 2 + 0 2 − 1 = −1. That this is an absolute minimum is seen from the fact that 𝑓(𝑥, 𝑦, 𝑧) = 𝑥 2 + 𝑦 2 + 𝑧 2 − 1 can never be less than −1. 33. As in Exercise 29, we minimize the square of the distance of (𝑥, 𝑦, 𝑧) to the origin: 𝑑(𝑥, 𝑦, 𝑧) = 𝑥 2 + 𝑦 2 + 𝑧 2 subject to 𝑧 = √𝑥 2 + 2(𝑦 − 3) 2, which we rewrite as 𝑥 2 + 2(𝑦 − 3) 2 − 𝑧 2 = 0 (𝑧 ≥ 0). 𝑑(𝑥, 𝑦, 𝑧) = 𝑥 2 + 𝑦 2 + 𝑧 2, 𝑔(𝑥, 𝑦, 𝑧) = 𝑥 2 + 2(𝑦 − 3) 2 − 𝑧 2, 𝐿(𝑥, 𝑦, 𝑧) = 𝑥 2 + 𝑦 2 + 𝑧 2 − 𝜆[𝑥 2 + 2(𝑦 − 3) 2 − 𝑧 2] (1) 𝐿𝑥 = 0 ⇒ 2𝑥 = 2𝜆𝑥 (2) 𝐿𝑦 = 0

2𝑦 = 4𝜆(𝑦 − 3)

(3) 𝐿𝑧 = 0

2𝑧 = −2𝜆𝑧

(4) Constraint ⇒ 𝑥 2 + 2(𝑦 − 3) 2 − 𝑧 2 = 0 From (1) we see that either 𝑥 = 0 or 𝜆 = 1. Let's try 𝜆 = 1 first: From (2) we get 2𝑦 = 4(𝑦 − 3) = 4𝑦 − 12, so 𝑦 = 6, and from (3) we get 2𝑧 = −2𝑧, so 𝑧 = 0. However, substituting in (4) we get 0 + 2(6 − 3) 2 − 0 = 18 ≠ 0, so 𝜆 = 1 does not work. Therefore, 𝑥 = 0. Now look at (3), which tells us that 𝑧 = 0 or 𝜆 = −1. If we try 𝑧 = 0, Equation (4) then tells us that 0 + 2(𝑦 − 3) 2 − 𝑧 = 0 ⇒ 𝑦 = 3. However, substituting in (2) we get 2𝑦 = 6 ≠ 4𝜆(𝑦 − 3) = 0, so 𝑧 = 0 does not work, and we must have 𝜆 = −1. Substituting in (2) we find 2𝑦 = −4(𝑦 − 3) = −4𝑦 + 12 ⇒ 6𝑦 = 12 ⇒ 𝑦 = 2. Substituting in (4) we get 0 + 2(2 − 3) 2 − 𝑧 2 = 0 ⇒ 𝑧2 = 2 ⇒ 𝑧 = √2. (Remember that 𝑧 ≥ 0.) Therefore, the point on the surface 𝑧 = √𝑥 2 + 2(𝑦 − 3) 2 closest to the origin is (0, 2, √2). 34. The constraint is 𝑥𝑦𝑧 = 1, or 𝑥𝑦𝑧 − 1 = 0. 𝑓(𝑥, 𝑦, 𝑧) = 𝑥𝑦 + 𝑥 2𝑧 2 + 4𝑦𝑧, 𝑔(𝑥, 𝑦, 𝑧) = 𝑥𝑦𝑧 − 1, 𝐿(𝑥, 𝑦, 𝑧) = 𝑥𝑦 + 𝑥 2𝑧 2 + 4𝑦𝑧 − 𝜆(𝑥𝑦𝑧 − 1) (1) 𝐿𝑥 = 0 ⇒ 𝑦 + 2𝑥𝑧 2 = 𝜆𝑦𝑧


Solutions Chapter 15 Review (2) 𝐿𝑦 = 0

𝑥 + 4𝑧 = 𝜆𝑥𝑧

(3) 𝐿𝑧 = 0

2𝑥 2𝑧 + 4𝑦 = 𝜆𝑥𝑦

(4) Constraint

𝑥𝑦𝑧 = 1 1 2𝑥𝑧 Solve Equation (1) for 𝜆 : 𝜆 = + . 𝑧 𝑦 Substituting in (2) gives 1 2𝑥𝑧 2𝑥 2𝑧 2 2𝑥 2𝑧 2 𝑥 2𝑧 𝑥 + 4𝑧 = ( + )𝑥𝑧 ⇒ 𝑥 + 4𝑧 = 𝑥 + ⇒ 4𝑧 = ⇒ 2= ⇒ 𝑧 𝑦 𝑦 𝑦 𝑦 𝑦 = 𝑥 2𝑧∕2. Substituting the expression for 𝜆 in (3) gives 𝑥𝑦 𝑥𝑦 1 2𝑥𝑧 𝑥 2𝑥 2𝑧 + 4𝑦 = ( + )𝑥𝑦 ⇒ 2𝑥 2𝑧 + 4𝑦 = 4𝑦 = ⇒ 4= + 2𝑥 2𝑧 ⇒ 𝑧 𝑦 𝑧 𝑧 𝑧 ⇒ 𝑧 = 𝑥∕4. Substituting the expressions we obtained for 𝑦 and 𝑧 in the constraint equation gives: 𝑥[2𝑥 2(𝑥∕4)∕2](𝑥∕4) = 1 ⇒ 𝑥 5 = 32 ⇒ 𝑥 = 2. The corresponding values of 𝑦 and 𝑧 are 𝑧 = 𝑥∕4 = 2∕4 = 1∕2, 𝑦 = 𝑥 2𝑧∕2 = 1. Therefore, the only critical point is (2, 1, 1∕2) and the corresponding value of the objective is 𝑓(2, 1, 1∕2) = (2)(1) + (2) 2(1∕2) 2 + 4(1)(1∕2) = 5. Therefore, the minimum value of the objective function is 5, and occurs at the point (2, 1, 1∕2). 1

35.

∫ 0 ∫ 0 2

36.

2𝑥𝑦 𝑑𝑥 𝑑𝑦 =

1

1

∫ 0

∫ 0

[𝑥 2𝑦]𝑥2 = 0 𝑑𝑥 𝑑𝑦 =

1

1

4𝑦 𝑑𝑦 = [2𝑦 2]𝑦 = 0 = 2

2

2

∫ ∫ 1 ∫ 0 1 = 2𝑒 2 − 𝑒 2 − (𝑒 − 𝑒) = 𝑒 2

∫ 1

2

37.

2

𝑥𝑦𝑒 𝑥+𝑦 𝑑𝑥 𝑑𝑦 =

[𝑥𝑦𝑒 𝑥+𝑦 − 𝑦𝑒 𝑥+𝑦]𝑥1 = 0 𝑑𝑦 =

𝑦𝑒 𝑦 𝑑𝑦 = [𝑦𝑒 𝑦 − 𝑒 𝑦]𝑦2 = 1

2𝑥

2 2 𝑦 2𝑥 1 2𝑥 2 𝑑𝑦 𝑑𝑥 [ ] 𝑑𝑦 𝑑𝑦 = [ln(𝑥 2 + 1)]𝑥 = 0 = ln 5 == = 2+1 2 + 1 𝑦=0 2+1 ∫ ∫ ∫ ∫ 𝑥 𝑥 𝑥 0 0 0 0 2

1

𝑥𝑦𝑒 𝑥+𝑦 𝑑𝑥 𝑑𝑦 = 𝑒 2. On the other hand, 𝐴 = 1 because the domain is a ∫ ∫ 1 0 square with sides of length 1. Hence the average value is 𝑒 2∕1 = 𝑒 2. 38. We've already computed

39.

40.

1

2−𝑦

1 2−𝑦 1 1 1 [ 𝑥 3 − 𝑥𝑦 2] 𝑑𝑦 = ( (2 − 𝑦) 3 − (2 − 𝑦)𝑦 2 − 𝑦 3 + 𝑦 3) 𝑑𝑦 𝑥=𝑦 ∫ ∫ ∫ ∫ 3 3 3 0 𝑦 0 0 1 1 8 4 8 1 ( − 4𝑦 + 𝑦 3) 𝑑𝑦 = [ 𝑦 − 2𝑦 2 + 𝑦 4] = =1 𝑦=0 ∫ 3 3 3 3 0 1

1−𝑥 2

(𝑥 2 − 𝑦 2) 𝑑𝑥 𝑑𝑦 =

1

1 1 2 1−𝑥 2 1 𝑦 ] 𝑑𝑥 = [(1 − 𝑥 2) − (1 − 𝑥 2) 2] 𝑑𝑥 𝑦=0 ∫ ∫ ∫ ∫ 2 2 −1 0 −1 −1 1 1 1 1 1 1 1 1 4 (1 − 𝑥 4) 𝑑𝑥 = [𝑥 − 𝑥 5] = = (1 − + 1 − ) = 𝑥 = −1 2∫ 2 5 2 5 5 5 −1

(1 − 𝑦) 𝑑𝑦 𝑑𝑥 =

1

[𝑦 −

41. a. ℎ(𝑥, 𝑦) = 5,000 − 0.8𝑥 − 0.6𝑦 hits per day (𝑥 =number of new customers atJungleBooks.com, 𝑦 =number of new customers at FarmerBooks.com) b. Set 𝑥 = 100 and ℎ = 4,770 : 4,770 = 5,000 − 0.8(100) − 0.6𝑦 so that 𝑦 = 250 new customers. c. ℎ(𝑥, 𝑦, 𝑧) = 5,000 − 0.8𝑥 − 0.6𝑦 + 0.0001𝑧 (𝑧 =number of new Internet shoppers) d. Set 𝑥 = 𝑦 = 100 and ℎ = 5,000 : 5,000 = 5,000 − 0.8(100) − 0.6(100) + 0.0001𝑧, so 𝑧 =1.4 million new Internet shoppers.


Solutions Chapter 15 Review 42. a. 𝑝(𝑥, 𝑦) = 15 − 0.3𝑥 − 1.2𝑦 (𝑥\displaystyle {}= temperature in °C, 𝑦\displaystyle {}= number of text messages per hour) b. Substituting the given information gives 3 = 15 − 0.3(20) − 1.2𝑦; ⇒ 1.2𝑦 = 6 ⇒ 𝑦=5 text messages per hour. c. One cup of coffee hour per hour will result in 2.4 additional pages, so the adjusted model is 𝑝(𝑥, 𝑦, 𝑧) = 15 − 0.3𝑥 − 1.2𝑦 + 2.4𝑧 (𝑧\displaystyle {}= number of cups of coffee per hour). d. For 𝑝(𝑥, 𝑦, 𝑧) ≥ 0 we must have 15 − 0.3𝑥 − 1.2𝑦 + 2.4𝑧 ≥ 0, or 𝑧 ≥ (0.3𝑥 + 1.2𝑦 − 15)∕2.4, a linear function whose graph is a plane. Thus, the domain is the region of 3-space in the first octant on or above this plane. 43. a. ℎ(2,000, 3,000) = 2,320 hits per day b. ℎ𝑦 = 0.08 + 0.00003𝑥 hits (daily) per dollar spent on television advertising per month; this increases with increasing 𝑥. c. Set ℎ𝑦 = 1∕5 = 0.2 : 0.08 + 0.00003𝑥 = 0.2 ⇒ 𝑥 = $4,000 per month. 44. a. 𝐶(5, 10) = 20,000 − 100(5) + 600(10) + 300(5)(10) = $40,500 𝜕𝐶 b. = 600 + 300𝑥 dollars per day; increases with increasing 𝑥 𝜕𝑡 𝜕𝐶 c. Rate of increase of cost with respect to time = = 600 + 300𝑥 from part (b). For this to equal 1,200, 𝜕𝑡 we must have 𝑥 = 2 reps. 45. (A) because ℎ𝑥 (𝑥, 0) = 0.05 46. 𝐶(𝑥, 𝑡) = 20,000 − 100𝑥 + 600𝑡 + 300𝑥𝑡, and 𝜕𝐶∕𝜕𝑥 = −100 + 300𝑡 dollars per (additional) sales rep Setting 𝑡 = 10 gives $2,900 dollars per (additional) sales rep (choice (E)). 47. a. 𝑃 (10, 1,000) ≈ 15,800 additional orders per day b. Minimize 𝐶(𝑥, 𝑦) = 150𝑥 + 𝑦 subject to 1,000𝑥 0.9𝑦 0.1 = 15,000. Using Lagrange multipliers, we need to solve the system 150 = 900𝜆𝑥 −0.1𝑦 0.1 1 = 100𝜆𝑥 0.9𝑦 −0.9 1,000𝑥 0.9𝑦 0.1 − 15,000 = 0. 𝑥 0.1 From the first equation we get 𝜆 = 0.1 . 6𝑦 100𝑥 50 Substituting in the second equation gives 1 = , so 𝑦 = 𝑥. 6𝑦 3 50 0.1 3 0.1 Substituting in the last equation gives 1,000( ) 𝑥 = 15,000, so 𝑥 = 15( ) ≈ 11. 3 50 48. a. 𝐶(15, 2,000) ≈ 39,900 fewer orders per day b. Minimize 𝑃 (𝑥, 𝑦) = 200𝑥 + 𝑦 subject to 1,000𝑥 0.8𝑦 0.2 = 20,000. Using Lagrange multipliers, we need to solve the system 200 = 800𝜆𝑥 −0.2𝑦 0.2 1 = 200𝜆𝑥 0.8𝑦 −0.8 1,000𝑥 0.8𝑦 0.2 − 20,000 = 0. 𝑥 0.2 From the first equation we get 𝜆 = 0.2 . 4𝑦 50𝑥 Substituting in the second equation gives 1 = ⇒ 𝑦 = 50𝑥. 𝑦 Substituting in the last equation gives 1,000(50) 0.2𝑥 = 20,000

𝑥 = 20(

1 0.2 ) ≈ 9. 50


Solutions Chapter 15 Review 1,500

49.

2,000

∫ 1,200 ∫ 1,800

(3𝑥 + 10𝑦) 𝑑𝑦 𝑑𝑥 =

1,500

∫ 1,200

[3𝑥𝑦 + 5𝑦 2]𝑦2,000 = 1,800 𝑑𝑥

1,500

1,500 (600𝑥 + 3,800,000) 𝑑𝑥 = [300𝑥 2 + 3,800,000𝑥]𝑥 = 1,200 = 1,383,000,000. ∫ 1,200 𝐴 = (1,500 − 1,200)(2,000 − 1,800) = 60,000, so the average profit is 1,383,000,000∕60,000 = $23,050.

=

120

1,000

(𝑥 2 + 2𝑦) 𝑑𝑦 𝑑𝑥 =

120

[𝑥 2𝑦 + 𝑦 2]𝑦1,000 = 800 𝑑𝑥 =

120

(200𝑥 2 + 360,000) 𝑑𝑥 ∫ ∫ ∫ 100 ∫ 800 100 100 120 200𝑥 3 ≈ 55,733,333. =[ + 360,000𝑥] 𝑥 = 100 3 𝐴 = (120 − 100)(1,000 − 800) = 4,000, so the average cost is 55,733,333∕4,000 ≈ $13,933. 50.


Solutions Chapter 15 Case Study Chapter 15 Case Study 1. We use (almost) the same formula for the regression as in the case study, except that we omit the last column (F) in the data: =LINEST(A2:A61,B2:E61,TRUE,TRUE) This leads to the following result: 𝑎4

𝑎3

𝑎2

𝑎1

𝑎0

-84.11222222 -2.046745652 3307.201667 60.37137541 -30747.77776 4.410141356 0.350127879 183.4689102 6.891384805 1895.128638 0.968329151 80.35263187

#N/A

#N/A

#N/A

420.4031818

55

#N/A

#N/A

#N/A

10857409

355109.9997

#N/A

#N/A

#N/A

giving the model 𝑧(𝑥, 𝑡) = 𝑎0 + 𝑎1 𝑡 + 𝑎2 𝑥 + 𝑎3 𝑡 2 + 𝑎4 𝑥 2 = −30,748 + 60.371𝑡 + 3,307.2𝑥 − 2.0467𝑡 2 − 84.112𝑥 2. 2. The same calculations as in the text with the new coefficients from Exercise 1 give 𝑥→ 𝑡 ↓

19

20

21

20 2,113

2,140

1,999

21 2,090

2,116

1,975

22 2,062

2,089

1,947

23 2,030

2,057

1,916

The new predictions agree with those in the text to three significant digits, with the largest differences being 3,000 students in three of the predictions. 3. For critical points, set the partial derivatives equal to zero. 𝜕𝑧 = 3,308.9 − 168.224𝑥 − 0.18244𝑡 = 0 𝜕𝑥 𝜕𝑧 = 64.1417 − 4.0934𝑡 − 0.18244𝑥 𝜕𝑡 Thus we solve the system of two linear equations in two unknowns: 168.224𝑥 + 0.18244𝑡 = 3,308.9 0.18244𝑥 + 4.0934𝑡 = 64.1417 and obtain the solution 𝑥 = 19.6536, 𝑡 = 14.7936. For the Hessian, 𝑧𝑥𝑥 = −168.224, 𝑧𝑡𝑡 = −4.0934, 𝑧𝑥𝑡 = 𝑧𝑡𝑥 = −0.18244. This gives 𝐻 > 0 so, as 𝑧𝑥𝑥 < 0, we have a maximum at (𝑥, 𝑡) = (19.6536, 14.7936), seen to be an absolute maximum. Thus, college enrollments were highest for 19 and a half-year olds and around three-quarters of the way through 2014. 4. 𝑧(𝑥, 𝑡) = −30,748 + 60.371𝑡 + 3,307.2𝑥 − 2.0467𝑡 2 − 84.112𝑥 2 gives 𝑧𝑥 = 3,307.2 − 168.224𝑥, which does not depend on 𝑡, so the curves 𝑡 = constant are all parallel (parabolas), unlike the situation in the original model where the level curves 𝑡 = constant are not parallel.


Solutions Chapter 15 Case Study 5. 𝑧(𝑥, 𝑡) = −30,748 + 60.371𝑡 + 3,307.2𝑥 − 2.0467𝑡 2 − 84.112𝑥 2 gives 𝑧𝑡 = 60.371 − 4.0934𝑡, which does not depend on 𝑥, so the curves 𝑥 = constant are all parallel (parabolas), unlike the situation in the original model where the level curves 𝑥 = constant are not parallel. 6. Answers will vary.


Solutions Section 16.1 Section 16.1 1.

2.

3.

4.

5.

6.

7.

8.


Solutions Section 16.1 9.

10.

11.

12.

13. From the graph, 𝐶 = 1, 𝐴 = 1, 𝛼 = 0, and 𝑃 = 1, so 𝜔 = 2𝜋. Thus, 𝑓(𝑥) = sin(2𝜋𝑥) + 1.

14. From the graph, 𝐶 = 0, 𝐴 = 1, 𝛼 = −1∕4, and 𝑃 = 1, so 𝜔 = 2𝜋. Thus, 𝑓(𝑥) = sin[2𝜋(𝑥 + 1∕4)]. 15. From the graph, 𝐶 = 0, 𝐴 = 1.5, 𝛼 = 0.25, and 𝑃 = 0.5, so 𝜔 = 4𝜋. Thus, 𝑓(𝑥) = 1.5 sin[4𝜋(𝑥 − 0.25)].

16. From the graph, 𝐶 = 0.5, 𝐴 = 1.5, 𝛼 = 0.25, and 𝑃 = 0.5, so 𝜔 = 4𝜋. Thus, 𝑓(𝑥) = 1.5 sin[4𝜋(𝑥 − 0.25)] + 0.5. 17. From the graph, 𝐶 = −50, 𝐴 = 50, 𝛼 = 5, and 𝑃 = 20, so 𝜔 = 𝜋∕10. Thus, 𝑓(𝑥) = 50 sin[𝜋(𝑥 − 5)∕10] − 50.

18. From the graph, 𝐶 = 35, 𝐴 = 35, 𝛼 = 25, and 𝑃 = 100, so 𝜔 = 𝜋∕50. Thus, 𝑓(𝑥) = 35 sin[𝜋(𝑥 − 25)∕50] + 35.

19. From the graph, 𝐶 = 0, 𝐴 = 1, 𝛽 = 0, and 𝑃 = 1, so 𝜔 = 2𝜋. Thus, 𝑓(𝑥) = cos(2𝜋𝑥).

20. From the graph, 𝐶 = 1, 𝐴 = 1, 𝛽 = 1∕4, and 𝑃 = 1, so 𝜔 = 2𝜋. Thus, 𝑓(𝑥) = cos[2𝜋(𝑥 − 1∕4)] + 1. 21. From the graph, 𝐶 = 0, 𝐴 = 1.5, 𝛽 = 0.375, and 𝑃 = 0.5, so 𝜔 = 4𝜋. Thus, 𝑓(𝑥) = 1.5 cos[4𝜋(𝑥 − 0.375)].Alternatively, 𝛽 = −0.125, so 𝑓(𝑥) = 1.5 cos[4𝜋(𝑥 + 0.125)]. 22. From the graph, 𝐶 = 0.5, 𝐴 = 1.5, 𝛽 = 0.25, and 𝑃 = 0.5, so 𝜔 = 4𝜋. Thus, 𝑓(𝑥) = 1.5 cos[4𝜋(𝑥 − 0.25)] + 0.5. 23. From the graph, 𝐶 = 40, 𝐴 = 40, 𝛽 = 10, and 𝑃 = 20, so 𝜔 = 𝜋∕10. Thus, 𝑓(𝑥) = 40 cos[𝜋(𝑥 − 10)∕10] + 40.

24. From the graph, 𝐶 = −50, 𝐴 = 50, 𝛽 = 10, and 𝑃 = 20, so 𝜔 = 𝜋∕10. Thus, 𝑓(𝑥) = 50 cos[𝜋(𝑥 − 10)∕10] − 50.


25. 𝑓(𝑡) = 4.2 sin(𝜋∕2 − 2𝜋𝑡) + 3

Solutions Section 16.1

26. 𝑓(𝑡) = 3 − sin(𝜋∕2 − 𝑡 + 4)

27. 𝑔(𝑥) = 4 − 1.3 sin[𝜋∕2 − 2.3(𝑥 − 4)]

28. 𝑔(𝑥) = 4.5 sin[𝜋∕2 − 2𝜋(3𝑥 − 1)] + 7

29. sin 2 𝑥 + cos 2 𝑥 = 1; (sin 2 𝑥 + cos 2 𝑥)∕ cos 2 𝑥 = 1∕ cos 2 𝑥; tan 2 𝑥 + 1 = sec 2 𝑥 30. sin 2 𝑥 + cos 2 𝑥 = 1; (sin 2 𝑥 + cos 2 𝑥)∕ sin 2 𝑥 = 1∕ sin 2 𝑥; 1 + cot 2 𝑥 = csc 2 𝑥 31. sin(𝜋∕3) = sin(𝜋∕6 + 𝜋∕6) = sin(𝜋∕6) cos(𝜋∕6) + cos(𝜋∕6) sin(𝜋∕6) = (1∕2)(√3∕2) + (√3∕2)(1∕2) = √3∕2

32. cos(𝜋∕3) = cos(𝜋∕6 + 𝜋∕6) = cos(𝜋∕6) cos(𝜋∕6) − sin(𝜋∕6) sin(𝜋∕6) = (√3∕2) 2 − (1∕2) 2 = 1∕2 33. sin(𝑡 + 𝜋∕2) = sin 𝑡 cos(𝜋∕2) + cos 𝑡 sin(𝜋∕2) = cos 𝑡 because cos(𝜋∕2) = 0 and sin(𝜋∕2) = 1 34. cos(𝑡 − 𝜋∕2) = cos 𝑡 cos(𝜋∕2) + sin 𝑡 sin(𝜋∕2) = sin 𝑡 because cos(𝜋∕2) = 0 and sin(𝜋∕2) = 1 35. sin(𝜋 − 𝑥) = sin 𝜋 cos 𝑥 − cos 𝜋 sin 𝑥 = sin 𝑥 because sin 𝜋 = 0 and cos 𝜋 = −1

36. cos(𝜋 − 𝑥) = cos 𝜋 cos 𝑥 + sin 𝜋 sin 𝑥 = − cos 𝑥 because sin 𝜋 = 0 and cos 𝜋 = −1 37. tan(𝑥 + 𝜋) =

38. cot(𝑥 + 𝜋) =

sin(𝑥 + 𝜋) sin 𝑥 cos 𝜋 + cos 𝑥 sin 𝜋 − sin 𝑥 = = = tan 𝑥 cos(𝑥 + 𝜋) cos 𝑥 cos 𝜋 − sin 𝑥 sin 𝜋 − cos 𝑥

cos(𝑥 + 𝜋) cos 𝑥 cos 𝜋 − sin 𝑥 sin 𝜋 − cos 𝑥 = = = cot 𝑥 sin(𝑥 + 𝜋) sin 𝑥 cos 𝜋 + cos 𝑥 sin 𝜋 − sin 𝑥

2𝜋 = 10.8 years. b. Maximum: 65 + 60 = 125; minimum: 65 − 60 = 5 𝜋∕5.4 c. January 2040 corresponds to 𝑡 = 31. High points occur every 𝑃 = 10.8 years starting at 𝑃 𝛼 + = 2.7 + 2.7 = 5.4; that is, 𝑡 = 5.4, 16.2, 27, 37.8, the last of which is the first value ≥ 31, and 4 corresponds to 2047 (to the nearest year). 39. a. 𝑃 =

2𝜋 = 12.2 years. b. Maximum: 85 + 80 = 165; minimum: 85 − 80 = 5 𝜋∕6.1 c. January 2030 corresponds to 𝑡 = 33.5. High points occur every 𝑃 = 12.2 years starting at 𝑃 𝛼 + = 3.05 + 3.05 = 6.1; that is, 𝑡 = 6.1, 18.3, 30, 5, 42.7, the last of which is the first value ≥ 33.5 and 4 corresponds to 2039 (to the nearest year). 40. a. 𝑃 =


Solutions Section 16.1 41. a.

Sales were highest when 𝑡 ≈ 0, 4, 8, and 12, which correspond to the end of the last quarter or beginning of the first quarter of each year. Sales were lowest when 𝑡 ≈ 2, 6, and 10, which correspond to the beginning of the third quarter of each year. b. The maximum quarterly sales were approximately 21 million iPods per quarter; minimum quarterly sales were approximately 9 million iPods per quarter. c. The maximum and minimum values are 𝐶 ± 𝐴, so the maximum is 15 + 6 = 21 and the minimum is 15 − 6 = 9. 42. a.

Housing starts were highest when 𝑡 ≈ 5, 17, and 29, which correspond to June of each year. Housing starts were lowest when 𝑡 ≈ 11, 23, and 35, which correspond to December of each year. b. The maximum number of housing starts was approximately 118 thousand housing starts per month; the minimum number was approximately 92 thousand housing starts per month. c. The maximum and minimum values are 𝐶 ± 𝐴, so the maximum is 105 + 13.1 = 118.1 and the minimum is 105 − 13.1 = 91.9. 43. Amplitude = 6.00, vertical offset = 15, phase shift = −1.85∕1.51 ≈ −1.23, angular frequecy = 1.51, period = 2𝜋∕1.51 ≈ 4.16. From 2008 through 2010, APPLE's sales of iPods fluctuated in cycles of 4.16 quarters about a baseline of 15 million iPods per quarter. Every cycle, sales peaked at 15 + 6 = 21 million iPods per quarter and dipped to a low of 15 − 6 = 9 million iPods per quarter. Sales first peaked at 𝑡 = −1.23 + (5∕4) × 4.16 = 3.97, the end of 2008. 44. Amplitude = 13.1, vertical offset = 105, phase shift = 1.42∕0.524 ≈ 2.71, angular frequecy = 0.524, period = 2𝜋∕0.524 ≈ 12.0. From January 2017 through January 2020, housing starts fluctuated in cycles of 12 months about a baseline of 105 thousand housing starts per month. Every cycle, housing starts peaked at 105 + 13.1 = 118.1 thousand housing starts per month and dipped to a low of 105 − 13.1 = 91.9 thousand housing starts per month. Housing starts first peaked at 𝑡 = 1.42∕0.524 + (1∕4) × 12 ≈ 5.7, toward the end of June 2017. 45. 𝐶 = 12.5, 𝐴 = 7.5, period = 52 weeks, so 𝜔 = 𝜋∕26, 𝛼 = 52∕4 = 13. So, 𝑃 (𝑡) = 7.5 sin[𝜋(𝑡 − 13)∕26] + 12.5.


Solutions Section 16.1 46. 𝐶 = 35, 𝐴 = 25, period = 12 months, so 𝜔 = 𝜋∕6, 𝛼 = 5 + 12∕4 = 8. So, 𝑃 (𝑡) = 25 sin[𝜋(𝑡 − 8)∕6] + 35.

47. Since the high point is 82 and the low point 75, the baseline is their average: 𝐶 = 78.5. The amplitude is therefore: 𝐴 = 82 − 78.5 = 3.5. The period (low point to low point) is 𝑃 = 12 months, so: 𝜔 = 2𝜋∕12 = 𝜋∕6. The phase shift is the number of months to the high point: 𝛽 = 7 Thus, the model is 𝑇 (𝑡) = 𝐴 cos[𝜔(𝑡 − 𝛽)] + 𝐶 = 3.5 cos[𝜋(𝑡 − 7)∕6] + 78.5.

48. Since the high point is 50 and the low point about 32.5, the baseline is their average: 𝐶 = 41.25. The amplitude is therefore: 𝐴 = 50 − 41.25 = 8.75. The period (low point to low point) is 𝑃 = 12 months, so: 𝜔 = 2𝜋∕12 = 𝜋∕6. The phase shift is the number of months to the high point: 𝛽 = 7. Thus, the model is 𝑇 (𝑡) = 𝐴 cos[𝜔(𝑡 − 𝛽)] + 𝐶 = 8.75 cos[𝜋(𝑡 − 7)∕6] + 41.25. 49. 𝐶 = (4.5 + 7)∕2 = 5.75, 𝐴 = (7 − 4.5)∕2 = 1.25, 𝑃 = 4, so 𝜔 = 𝜋∕2, 𝛼 + 𝑃 ∕4\displaystyle {}= distance to first high = 2. So, 𝛼 = 2 − 1 = 1. 𝑛(𝑡) = 1.25 sin[𝜋(𝑡 − 1)∕2] + 5.75 50. 𝐶 = 20, 𝐴 = 15, period = 12 months, so 𝜔 = 𝜋∕6, 𝛼 = 1 + 12∕4 = 4. So, 𝑠(𝑡) = 15 sin[𝜋(𝑡 − 4)∕6] + 20.

51. 𝐶 = (4.5 + 7)∕2 = 5.75, 𝐴 = (7 − 4.5)∕2 = 1.25, 𝑃 = 4, so 𝜔 = 𝜋∕2, 𝛽 = distance to first high = 2. So, 𝑛(𝑡) = 1.25 cos[𝜋(𝑡 − 2)∕2] + 5.75 52. 𝐶 = 20, 𝐴 = 15, period = 12 months, so 𝜔 = 𝜋∕6, 𝛽 = 7. So, 𝑠(𝑡) = 15 cos[𝜋(𝑡 − 7)∕6] + 20. 53. 𝐶 = 10, 𝐴 = 5, period = 13.5 hours, so 𝜔 = 2𝜋∕13.5, 𝛼 = 5 − 13.5∕4 = 1.625. So, 𝑑(𝑡) = 5 sin[2𝜋(𝑡 − 1.625)∕13.5] + 10.

54. 𝐶 = 8, 𝐴 = 2, period = 16 hours, so 𝜔 = 𝜋∕8, 𝛼 = 4 + 16∕4 = 8. So, 𝑑(𝑡) = 2 sin[𝜋(𝑡 − 8)∕8] + 8. 55. a. 𝐶 = 7.5, 𝐴 = 2.5, period = 1 year, so 𝜔 = 2𝜋, 𝛼 = 0.75. So, 𝑢(𝑡) = 2.5 sin(2𝜋(𝑡 − 0.75)) + 7.5. b. 𝑐(𝑡) = 1.04 𝑡[2.5 sin(2𝜋(𝑡 − 0.75)) + 7.5] 56. a. 𝐶 = 7, 𝐴 = 3, period = 1 year, so 𝜔 = 2𝜋, 𝛼 = 0. So, 𝑢(𝑡) = 3 sin(2𝜋𝑡) + 7. b. 𝑠(𝑡) = (0.88 𝑡)3 sin(2𝜋𝑡) 57. a. Graph:

Rough estimates: 𝐶 = 60, 𝐴 = 10, 𝑃 = 12, 𝛽 = 6


Solutions Section 16.1 b. 𝑓(𝑡) = 7.771 cos[2𝜋(𝑡 − 6.206)∕11.93] + 61.97. Graph:

c. Domestic air travel on U.S. air carriers showed a pattern that repeats itself every 12 months, from a low of 54 to a high of 70 billion revenue passenger miles. (Rounding answers to the nearest whole number.) 58. a. Graph:

Rough estimates: 𝐶 = 26, 𝐴 = 4, 𝑃 = 12, 𝛽 = 6 b. 𝑓(𝑡) = 3.748 cos[2𝜋(𝑡 − 6.319)∕11.27] + 25.32. Graph:

c. International air travel on U.S. air carriers showed a pattern that repeats itself every 12 months, from a low of 22 to a high of 29 billion revenue passenger miles. (Rounding answers to the nearest whole number.)


Solutions Section 16.1 59. a.

2 2 2 2 2 2 cos 𝑥 + cos 3𝑥 + cos 5𝑥 + cos 7𝑥 + cos 9𝑥 + cos 11𝑥 𝜋 3𝜋 5𝜋 7𝜋 9𝜋 11𝜋 Graph of all four functions: b. 𝑦11 =

Graph of 𝑦11 alone:

c. Multiply the amplitudes by 3 and change 𝜔 to 1/2: 6 𝑥 6 3𝑥 6 5𝑥 6 7𝑥 6 9𝑥 6 11𝑥 𝑦11 = cos + cos cos cos cos cos . + + + + 𝜋 2 3𝜋 2 5𝜋 2 7𝜋 2 9𝜋 2 11𝜋 2 60. a.

2 2 2 2 2 2 sin 𝑥 + sin 3𝑥 + sin 5𝑥 + sin 7𝑥 + sin 9𝑥 + sin 11𝑥 𝜋 3𝜋 5𝜋 7𝜋 9𝜋 11𝜋 Graph of all four functions: b. 𝑦11 =


Solutions Section 16.1

Graph of 𝑦11 alone:

c. Multiply the amplitudes by 3 and change 𝜔 to 1/2: 6 𝑥 6 3𝑥 6 5𝑥 6 7𝑥 6 9𝑥 6 11𝑥 𝑦11 = sin + sin sin sin sin sin . + + + + 𝜋 2 3𝜋 2 5𝜋 2 7𝜋 2 9𝜋 2 11𝜋 2 61.

The period is approximately 12.6 units. 62.

The period is approximately 63 units. 63. Lows: 𝐵 − 𝐴; Highs: 𝐵 + 𝐴

64. The model is not realistic because it predicts lows of a negative number (2.3 − 4 = −1.7) of items in stock. 65. He is correct. The other trigonometric functions can be obtained from the sine function by first using the formula cos 𝑥 = sin(𝑥 + 𝜋∕2) to obtain cosine, and then using the formulas sin 𝑥 cos 𝑥 1 1 tan 𝑥 = , cot 𝑥 = , sec 𝑥 = , and csc 𝑥 = cos 𝑥 sin 𝑥 cos 𝑥 sin 𝑥


Solutions Section 16.1 to obtain the rest. 66. She is correct. The other trig functions can be obtained from the cosine function by first using the formula sin 𝑥 = cos(𝑥 − 𝜋∕2) to obtain sine, and then using the formulas sin 𝑥 cos 𝑥 1 1 tan 𝑥 = , cot 𝑥 = , sec 𝑥 = , csc 𝑥 = cos 𝑥 sin 𝑥 cos 𝑥 sin 𝑥 to obtain the rest.

67. The largest 𝐵 can be is 𝐴. Otherwise, if 𝐵 is larger than 𝐴, the low figure for sales would have the negative value of 𝐴 − 𝐵.

68. If the cost of an item is given by 𝑐(𝑡) = 𝐴 + 𝐵 cos[𝜔(𝑡 − 𝛼1)], then the cost fluctuates by pmB with a period of 2𝜋 ∕𝜔 about a base of 𝐴, peaking at time 𝑡 =α.


Solutions Section 16.2 Section 16.2

1. 𝑓 ′(𝑥) = cos 𝑥 + sin 𝑥

2. 𝑓 ′(𝑥) = sec 2 𝑥 − cos 𝑥

3. 𝑔 ′(𝑥) = (cos 𝑥)(tan 𝑥) + (sin 𝑥)(sec 2 𝑥) = sin 𝑥(1 + sec 2 𝑥)

4. 𝑔 ′(𝑥) = −(sin 𝑥 cot 𝑥 + cos 𝑥 csc 2 𝑥) = − cos 𝑥(1 + csc 2 𝑥) 5. ℎ ′(𝑥) = −2 csc 𝑥 cot 𝑥 − sec 𝑥 tan 𝑥 + 3 6. ℎ ′(𝑥) = 2 sec 𝑥 tan 𝑥 + 3 sec 2 𝑥 + 3 7. 𝑟 ′(𝑥) = cos 𝑥 − 𝑥 sin 𝑥 + 2𝑥

8. 𝑟 ′(𝑥) = 2 sin 𝑥 + 2𝑥 cos 𝑥 − 2𝑥

9. 𝑠 ′(𝑥) = (2𝑥 − 1) tan 𝑥 + (𝑥 2 − 𝑥 + 1) sec 2 𝑥

10. 𝑠 ′(𝑥) = [(𝑥 2 − 1) sec 2 𝑥 − 2𝑥 tan 𝑥]∕(𝑥 2 − 1) 2

11. 𝑡 ′(𝑥) = −[csc 2 𝑥(1 + sec 𝑥) + cot 𝑥 sec 𝑥 tan 𝑥]∕(1 + sec 𝑥) 2 12. 𝑡 ′(𝑥) = sec 𝑥 tan 𝑥(1 − cos 𝑥) + (1 + sec 𝑥) sin 𝑥 13. 𝑘 ′(𝑥) = −2 cos 𝑥 sin 𝑥 14. 𝑘 ′(𝑥) = 2 tan 𝑥 sec 2 𝑥 15. 𝑗 ′(𝑥) = 2 sec 2 𝑥 tan 𝑥

16. 𝑗 ′(𝑥) = −2 csc 2 𝑥 cot 𝑥

17. 𝑓 ′(𝑥) = cos(3𝑥 − 5)(3) = 3 cos(3𝑥 − 5)

18. 𝑓 ′(𝑥) = − sin(2𝑥 + 7)(2) = −2 sin(2𝑥 + 7)

19. 𝑓 ′(𝑥) = − sin(−2𝑥 + 5)(−2) = 2 sin(−2𝑥 + 5)

20. 𝑓 ′(𝑥) = cos(−4𝑥 − 5)(−4) = −4 cos(−4𝑥 − 5) 𝜋 21. 𝑝 ′(𝑥) = 𝜋 cos [ (𝑥 − 4)] 5 22. 𝑝 ′(𝑥) =

𝜋 𝜋 sin [ (𝑥 + 3)] 2 6


23. 𝑢 (𝑥) = −(2𝑥 − 1) sin(𝑥 − 𝑥) ′

2

Solutions Section 16.2

24. 𝑢 ′(𝑥) = (6𝑥 + 1) cos(3𝑥 2 + 𝑥 − 1)

25. 𝑣 ′(𝑥) = (2.2𝑥 1.2 + 1.2) sec(𝑥 2.2 + 1.2𝑥 − 1) tan(𝑥 2.2 + 1.2𝑥 − 1) 26. 𝑣 ′(𝑥) = (2.2𝑥 1.2 + 1.2) sec 2(𝑥 2.2 + 1.2𝑥 − 1)

27. 𝑤 ′(𝑥) = sec 𝑥 tan 𝑥 tan(𝑥 2 − 1) + 2𝑥 sec 𝑥 sec 2(𝑥 2 − 1)

28. 𝑤 ′(𝑥) = − sin 𝑥 sec(𝑥 2 − 1) + 2𝑥 cos 𝑥 sec(𝑥 2 − 1) tan(𝑥 2 − 1)

29. 𝑦 ′(𝑥) = 𝑒 𝑥[− sin(𝑒 𝑥)] + 𝑒 𝑥 cos 𝑥 − 𝑒 𝑥 sin 𝑥 = 𝑒 𝑥[− sin(𝑒 𝑥) + cos 𝑥 − sin 𝑥] 30. 𝑦 ′(𝑥) = 𝑒 𝑥 sec(𝑒 𝑥) tan(𝑒 𝑥) 31. 𝑧 ′(𝑥) = 32. 𝑧 ′(𝑥) = 33.

34.

35.

36.

37.

38.

39.

40.

sec 𝑥 tan 𝑥 + sec 2 𝑥 sec 𝑥(tan 𝑥 + sec 𝑥) = = sec 𝑥 sec 𝑥 + tan 𝑥 sec 𝑥 + tan 𝑥 − csc 𝑥 cot 𝑥 − csc 2 𝑥 = − csc 𝑥 csc 𝑥 + cot 𝑥

𝑑 𝑑 1 sin 𝑥 1 sin 𝑥 sec 𝑥 = = = = sec 𝑥 tan 𝑥 𝑑𝑥 𝑑𝑥 [ cos 𝑥 ] cos 2 𝑥 cos 𝑥 cos 𝑥

𝑑 𝑑 cos 𝑥 1 − sin 2 𝑥 − cos 2 𝑥 cot 𝑥 = = = − 2 = − csc 2 𝑥 [ ] 2 𝑑𝑥 𝑑𝑥 sin 𝑥 sin 𝑥 sin 𝑥 𝑑 𝑑 1 cos 𝑥 1 cos 𝑥 csc 𝑥 = =− 2 =− = − csc 𝑥 cot 𝑥 𝑑𝑥 𝑑𝑥 [ sin 𝑥 ] sin 𝑥 sin 𝑥 sin 𝑥

𝑑 𝑑 1 𝑑 − sin 𝑥 ln | sec 𝑥| = ln = − ln | cos 𝑥| = − = tan 𝑥 𝑑𝑥 𝑑𝑥 | cos 𝑥 | 𝑑𝑥 cos 𝑥

𝑑 −2𝑥 [𝑒 sin(3𝜋𝑥)] = −2𝑒 −2𝑥 sin(3𝜋𝑥) + 3𝜋𝑒 −2𝑥 cos(3𝜋𝑥) = 𝑒 −2𝑥[−2 sin(3𝜋𝑥) + 3𝜋 cos(3𝜋𝑥)] 𝑑𝑥 𝑑 5𝑥 [𝑒 sin(−4𝜋𝑥)] = 5𝑒 5𝑥 sin(−4𝜋𝑥) − 4𝜋𝑒 5𝑥 cos(−4𝜋𝑥) = 𝑒 5𝑥[5 sin(−4𝜋𝑥) − 4𝜋 cos(−4𝜋𝑥)] 𝑑𝑥 𝑑 [sin(3𝑥)] 0.5 = 0.5[sin(3𝑥)] −0.53 cos(3𝑥) = 1.5[sin(3𝑥)] −0.5 cos(3𝑥) 𝑑𝑥

2𝑥(𝑥 − 1) − 𝑥 2 𝑑 𝑥 2 − 2𝑥 ⎛ 𝑥2 ⎞ ⎛ 𝑥2 ⎞ ⎛ 𝑥2 ⎞ cos ⎜ sin sin = − = − ⎜⎝ 𝑥 − 1 ⎟⎠ ⎜⎝ 𝑥 − 1 ⎟⎠ 𝑑𝑥 ⎝ 𝑥 − 1 ⎟⎠ (𝑥 − 1) 2 (𝑥 − 1) 2


41.

3𝑥 (𝑥 − 1) − 2𝑥 4 𝑑 𝑥3 𝑥3 𝑥3 sec ( 2 sec tan = ) ( ) ( 𝑑𝑥 𝑥 −1 𝑥2 − 1 𝑥2 − 1) (𝑥 2 − 1) 2 2

=

42.

43.

44.

2

Solutions Section 16.2

𝑥 4 − 3𝑥 2 𝑥3 𝑥3 sec tan (𝑥2 − 1) (𝑥2 − 1) (𝑥 2 − 1) 2

(2 + 𝑒 𝑥) tan 𝑥 sec 2 𝑥 − 𝑒 𝑥 tan 2 𝑥 𝑑 tan 𝑥 2 tan 𝑥 (2 + 𝑒 𝑥) sec 2 𝑥 − 𝑒 𝑥 tan 𝑥 2 2 = = (2 + 𝑒𝑥) 𝑑𝑥 ( 2 + 𝑒 𝑥 ) (2 + 𝑒 𝑥) 2 (2 + 𝑒 𝑥) 3 cot(2𝑥 − 1) 𝑑 ([ln |𝑥|][cot(2𝑥 − 1)]) = − 2 ln |𝑥| csc 2(2𝑥 − 1) 𝑑𝑥 𝑥 cos 𝑥 − 2𝑒 −𝑥(1 − 𝑥) 𝑑 | ln sin 𝑥 − 2𝑥𝑒 −𝑥| = 𝑑𝑥 sin 𝑥 − 2𝑥𝑒 −𝑥

45. a. Not differentiable at 0: | sin ℎ| − | sin 0| sin ℎ lim = lim =1 + + ℎ ℎ ℎ→0 ℎ→0 but | sin ℎ| − | sin 0| − sin ℎ lim = lim− = −1. ℎ → 0− ℎ→0 ℎ ℎ b. sin(1) ≈ 0.84 > 0 so | sin 𝑥| = sin 𝑥 for 𝑥 near 1; hence 𝑓 ′(1) exists, and sin 1.0001 − sin 0.9999 𝑓 ′(1) ≈ ≈ 0.5403. 0.0002

46. a. sin(1) ≈ 0.84 > 0 so | sin(1 − 𝑥)| = sin(1 − 𝑥) for 𝑥 near 0; hence 𝑓 ′(0) exists, and sin 0.9999 − sin 1.0001 𝑓 ′(0) ≈ ≈ −0.5403. 0.0002 b. Not differentiable at 1: | sin(−ℎ)| − | sin 0| sin ℎ lim = lim =1 + + ℎ ℎ ℎ→0 ℎ→0 but | sin(−ℎ)| − | sin 0| − sin ℎ lim = lim− = −1. ℎ → 0− ℎ→0 ℎ ℎ 47. a. 0: Write 𝑓(𝑥) = (sin 2 𝑥)∕𝑥. 𝑥

−0.1

−0.01 −0.001 −0.0001

𝑓(𝑥) −0.0997 −0.01 −0.001 −0.0001

0

0.0001 0.001 0.01

0.1

0.0001 0.001 0.01

0.1

0.0001 0.001 0.01 0.0997

sin 2 𝑥 2 sin 𝑥 cos 𝑥 = lim = 0. 𝑥→0 𝑥 𝑥→0 1

b. By L'Hospital's rule: lim

48. a. Does not exist: Write 𝑓(𝑥) = (sin 𝑥)∕𝑥 2.

𝑥 −0.1 −0.01 −0.001 −0.0001

𝑓(𝑥) −9.98 −100 −1000 −10,000

0

sin 𝑥 cos 𝑥 does not exist. = lim 2 𝑥→0 𝑥 𝑥 → 0 2𝑥

b. By L'Hospital's rule: lim

10,000 1000

100

9.98


49. a. 2: Write 𝑓(𝑥) = (sin 2𝑥)∕𝑥. 𝑥

−0.1

−0.01

Solutions Section 16.2

−0.001

−0.0001

𝑓(𝑥) 1.98669 1.99986667 1.99999867 1.999999 b. By L'Hospital's rule: lim

𝑥→0

−0.01

−0.001

𝑓(𝑥) 0.995 0.99995 0.9999995 b. By L'Hospital's rule: lim

sin 𝑥

𝑥 → 0 tan 𝑥

= lim

−0.0001

cos 𝑥

1

𝑥 → 0 sec 2 𝑥

−0.1

−0.01

−0.001

−0.0001

𝑓(𝑥) 4.99583 49.999583 499.999958 4999.99997 b. By L'Hospital's rule: lim

𝑥→0

−0.1

−0.01

𝑓(𝑥) −0.4995835 −0.4999958 b. By L'Hospital's rule: lim 53. 1 = (sec 2 𝑦)

0.001

0.1

0

1

0.01

0.1

0.9999995 0.99995 0.995

0 0.0001

0.001

0.01

0.1

−5000 −499.99996 −49.999583 −4.99583

−0.001 −0.0001 −0.5

−0.5

0

0.0001 0.001 −0.5

0.01

0.1

−0.5 −0.4999958 −0.4995835

cos 𝑥 − 1 1 − sin 𝑥 − cos 𝑥 = lim = lim =− . 𝑥 → 0 2𝑥 𝑥→0 2 2 𝑥2

𝑑𝑦 𝑑𝑦 , so = 1∕ sec 2 𝑦 𝑑𝑥 𝑑𝑥

54. 1 = (− sin 𝑦) 55. 1 +

𝑥→0

0.0001

0.01

cos 𝑥 − 1 − sin 𝑥 − cos 𝑥 does not exist. = lim = lim 𝑥 → 0 3𝑥 2 𝑥→0 6𝑥 𝑥3

52. a. −0.5 : Write 𝑓(𝑥) = (cos 𝑥 − 1)∕𝑥 2. 𝑥

0.001

= 1. (Or, note that 𝑓(𝑥) = cos 𝑥.)

51. a. Does not exist: Write 𝑓(𝑥) = (cos 𝑥 − 1)∕𝑥 3. 𝑥

0.0001

1.999999 1.99999867 1.99986667 1.98669

sin 2𝑥 2 cos 2𝑥 = lim = 2. 𝑥→0 𝑥 1

50. a. 1: Write 𝑓(𝑥) = sin 𝑥∕ tan 𝑥. 𝑥 −0.1

0

𝑑𝑦 𝑑𝑦 , so = −1∕ sin 𝑦 𝑑𝑥 𝑑𝑥

𝑑𝑦 𝑑𝑦 𝑑𝑦 𝑑𝑦 1 + 𝑦 cos(𝑥𝑦) + (𝑦 + 𝑥 )cos(𝑥𝑦) = 0, [1 + 𝑥 cos(𝑥𝑦)] = −[1 + 𝑦 cos(𝑥𝑦)], so =− 𝑑𝑥 𝑑𝑥 𝑑𝑥 𝑑𝑥 1 + 𝑥 cos(𝑥𝑦)

56. 𝑦 + 𝑥

𝑑𝑦 𝑑𝑦 𝑑𝑦 𝑑𝑦 1 − 𝑦 − cos 𝑦 + cos 𝑦 − 𝑥 sin 𝑦 = 1, (𝑥 − 𝑥 sin 𝑦) = 1 − 𝑦 − cos 𝑦, so = 𝑑𝑥 𝑑𝑥 𝑑𝑥 𝑑𝑥 𝑥 − 𝑥 sin 𝑦

57. 𝑐 ′(𝑡) = 7𝜋 cos[2𝜋(𝑡 − 0.75)]; 𝑐 ′(0.75) ≈ $21.99 per year ≈ $0.42 per week

58. 𝑐 ′(𝑡) = −(800𝜋∕7) sin[2𝜋(𝑡 − 2)∕7]; 𝑐 ′(3) ≈ −281. Therefore, daily sales are decreasing at a rate of 281 cartons per day.


Solutions Section 16.2 𝜋 𝜋 𝜋 59. 𝑁(𝑡) = 65 + 60 sin [ (𝑡 − 2.7)], so 𝑁 ′(𝑡) = 60 cos [ (𝑡 − 2.7)], 𝑁 ′(6) ≈ −11.94. At the 5.4 5.4 5.4 beginning of January 2015, the number of sunspots was decreasing at a rate of about 11.94 sunspots per year.

𝜋 𝜋 𝜋 (𝑡 − 3.05)], so 𝑁 ′(𝑡) = 80 cos [ (𝑡 − 3.05)], 𝐹 ′(2.5) ≈ 39.56. At the end 6.1 6.1 6.1 of 1998 (or the start of 1999), the number of sunspots was increasing at a rate of about 39.56 per year. 60. 𝑁(𝑡) = 85 + 80 sin [

61. 𝑐 ′(𝑡) = 1.035 𝑡[ln(1.035)(0.8 sin(2𝜋𝑡) + 10.2) + 1.6𝜋 cos(2𝜋𝑡)]; 𝑐 ′(1) = 1.035[10.2 ln(1.035) + 1.6𝜋] ≈ $5.57 per year, or $0.11 per week

62. 𝑠 ′(𝑡) = 𝑒 −0.2𝑡[9𝜋 cos(2𝜋𝑡) − 0.9 sin(2𝜋𝑡)]; 𝑠 ′(1) = 𝑒 −0.2[9𝜋] ≈ 23.15. Thus, sales of the wine were increasing at a rate of 23.15 bottles per day each year.

63. a. 𝑑(𝑡) = 5 cos(2𝜋𝑡∕13.5) + 10 b. 𝑑 ′(𝑡) = −(10𝜋∕13.5) sin(2𝜋𝑡∕13.5); 𝑑 ′(7) ≈ 0.270. At noon the tide was rising at a rate of 0.270 feet per hour. 64. a. 𝑑(𝑡) = 2 cos[𝜋(𝑡 − 7)∕8] + 8 b. 𝑑 ′(𝑡) = −(𝜋∕4) sin(𝜋(𝑡 − 7)∕8); 𝑑 ′(7) = 0. At noon, the tide is stationary (neither rising nor falling). 65. 𝑉 (𝑡) = 110| sin(100𝜋𝑡)| | sin(100𝜋𝑡)| | sin(100𝜋𝑡)| 𝑑𝑉 (100𝜋) cos(100𝜋𝑡) = 11,000𝜋 cos(100𝜋𝑡) = 110 𝑑𝑡 sin(100𝜋𝑡) sin(100𝜋𝑡) Graph:

The sudden jumps in the graph are due to the nondifferentiability of 𝑉 at the times 0, ±0.01, ±0.02, …. The derivative is negative immediately to the left and positive immediately to the right of these points. 66. 𝑉 (𝑡) = 55[| sin(100𝜋𝑡)| + sin(100𝜋𝑡)] | sin(100𝜋𝑡)| 𝑑𝑉 (100𝜋) cos(100𝜋𝑡) + (100𝜋) cos(100𝜋𝑡) = 55 [ sin(100𝜋𝑡) ] 𝑑𝑡 | sin(100𝜋𝑡)| = 5,500𝜋 cos(100𝜋𝑡) +1 [ sin(100𝜋𝑡) ] Graph:


Solutions Section 16.2 The sudden jumps in the graph are due to the nondifferentiability of 𝑉 at the times 0, ±0.01, ±0.02, …. At some of these points the derivative is negative immediately to the left and 0 to the right; at others it is 0 immediately to the left and positive to the right. 67. a. 𝑝(0) = 1.2 cos(5𝜋(0) + 𝜋) = −1.2 cm; 1.2 cm above the rest position b. Velocity = 𝑝 ′(𝑡) = −6𝜋 sin(5𝜋𝑡 + 𝜋) cm/sec 𝑡 = 0 : 𝑝 ′(0) = −6𝜋 sin(5𝜋(0) + 𝜋) = 0 cm/sec; not moving 𝑡 = 0.1 : 𝑝 ′(0.1) = −6𝜋 sin(5𝜋(0.1) + 𝜋) = 6𝜋 ≈ 18.85 cm/sec; moving downward at 18.85 cm/sec c. Period = 𝑃 = 2𝜋∕𝜔 = 2𝜋∕(5𝜋) = 0.4 sec. Frequency = 1∕𝑃 = 2.5 cycles per second

68. a. 𝑝(0) = 4.2 sin(2𝜋(0) + 𝜋∕2) = 4.2 cm; 4.2 cm above the rest position b. Velocity = 𝑝 ′(𝑡) = 8.4𝜋 cos(2𝜋𝑡 + 𝜋∕2) cm/sec 𝑡 = 0 : 𝑝 ′(0) = 8.4𝜋 cos(2𝜋(0) + 𝜋∕2) = 0 cm/sec; not moving 𝑡 = 0.1 : 𝑝 ′(0.25) = 8.4𝜋 cos(2𝜋(0.25) + 𝜋∕2) = −8.4𝜋 cm/sec; moving downward at 26.39 cm/sec c. Period = 𝑃 = 2𝜋∕𝜔 = 2𝜋∕(2𝜋) = 1 sec. Frequency = 1∕𝑃 = 1 cycle per second 69. a. Velocity = 𝑝 ′(𝑡) = 1.2𝑒 −0.1𝑡[−0.1 cos(5𝜋𝑡 + 𝜋) − 5𝜋 sin(5𝜋𝑡 + 𝜋)] cm/sec 𝑡 = 0 : 𝑝 ′(0) = 1.2𝑒 −0.1(0)[−0.1 cos(5𝜋(0) + 𝜋) − 5𝜋 sin(5𝜋(0) + 𝜋)] = 0.12 cm/sec; moving downward at 0.12 cm/sec 𝑡 = 0.1 : 𝑝 ′(0.1) = 1.2𝑒 −0.1(0.1)[−0.1 cos(5𝜋(0.1) + 𝜋) − 5𝜋 sin(5𝜋(0.1) + 𝜋)] ≈ 18.66 cm/sec; moving downward at 18.66 cm/sec b. Graphs: 𝑝:

𝑝′ :

By zooming in on the graph of 𝑝 ′, we can see that the slope of 𝑝 is greatest when 𝑡 ≈ 0.1.

70. a. Velocity = 𝑝 ′(𝑡) = 4.2𝑒 −0.5𝑡[−0.5 sin(2𝜋𝑡 + 𝜋∕2) + 2𝜋 cos(2𝜋𝑡 + 𝜋∕2)] cm/sec 𝑡 = 0 : 𝑝 ′(0) = 4.2𝑒 −0.5(0)[−0.5 sin(2𝜋(0) + 𝜋∕2) + 2𝜋 cos(2𝜋(0) + 𝜋∕2)] = −2.1; moving downward at 2.1 cm/sec 𝑡 = 0.1 : 𝑝 ′(0.25) = 4.2𝑒 −0.5(0.25)[−0.5 sin(2𝜋(0.25) + 𝜋∕2) + 2𝜋 cos(2𝜋(0.25) + 𝜋∕2)] ≈ −23.29 cm/sec;


b. Graphs: 𝑝:

Solutions Section 16.2 moving downward at 23.29 cm/sec

𝑝′ :

By zooming in on the graph of 𝑝 ′, we can see that the slope of 𝑝 is greatest when 𝑡 ≈ 0.72.

71. a. (III): From what we are told we must have 𝛼 = −500 and 𝑃 = 40, so 𝜔 = 2𝜋∕40. b. 𝐴 ′(𝑡) = −(𝜋∕20) sin[2𝜋(𝑡 + 500)∕40], 𝐴 ′(−150) ≈ 0.157. The tilt was increasing at a rate of 0.157 degrees per thousand years. 72. 𝐸(−200) = 0.05. 𝐸 ′(𝑡) = −0.025(𝜋∕200) sin[2𝜋(𝑡 + 200)∕400] + (𝜋∕100) sin[2𝜋(𝑡 + 200)∕100]; 𝐸 ′(−200) = 0. The eccentricity was 0.05 and (instantaneously) stationary 200,000 years ago. 73. −6; 6 74. −1; 5

75. Answers will vary. Examples: 𝑓(𝑥) = sin 𝑥; 𝑓(𝑥) = cos 𝑥

76. Answers will vary. Examples: 𝑓(𝑥) = sin(2𝑥); 𝑓(𝑥) = cos(2𝑥) 77. Answers will vary. Examples: 𝑓(𝑥) = 𝑒 −𝑥; 𝑓(𝑥) = −2𝑒 −𝑥

78. Answers will vary. Examples: 𝑓(𝑥) = sin 𝑥; 𝑓(𝑥) = cos 𝑥; 𝑓(𝑥) = 𝑒 𝑥, 𝑓(𝑥) = 𝑒 −𝑥


Solutions Section 16.2 79. The graph of cos 𝑥 slopes down over the interval (0, 𝜋), so its derivative is negative over that interval. The function − sin 𝑥, and not sin 𝑥, has this property. 80. 𝑓 ″(𝑥) = −𝑓(𝑥)

81. The velocity is 𝑝 ′(𝑡) = 𝐴𝜔 cos(𝜔𝑡 + 𝑑), which is a maximum when its derivative, 𝑝 ″(𝑡) = −𝐴𝜔 2 sin(𝜔𝑡 + 𝑑), is zero. But this occurs when sin(𝜔𝑡 + 𝑑) = 0, so that 𝑝(𝑡) is zero as well, meaning that the stock is at yesterday's close.

82. The acceleration is 𝑝 ″(𝑡) = −𝐴𝜔 2 cos(𝜔𝑡 + 𝑑), which has the greatest value when cos(𝜔𝑡 + 𝑑) has the lowest value; that is, when 𝑝(𝑡) is lowest.

83. The derivative of sin 𝑥 is cos 𝑥. When 𝑥 = 0, this is cos(0) = 1. Thus, the tangent to the graph of sin 𝑥 at the point (0, 0) has slope 1, which means that it slopes upward at 45°. 84. The derivative of cos 𝑥 is − sin 𝑥. When 𝑥 = 0, this is − sin 0 = 0. Thus, the tangent to the graph of cos 𝑥 at the point (0, 1) has slope 0, which means it has angle 0°.


Solutions Section 16.3 Section 16.3 1.

(sin 𝑥 − 2 cos 𝑥) 𝑑𝑥 = − cos 𝑥 − 2 sin 𝑥 + 𝐶 ∫

2.

(cos 𝑥 − sin 𝑥) 𝑑𝑥 = sin 𝑥 + cos 𝑥 + 𝐶 ∫

3.

(2 cos 𝑥 − 4.3 sin 𝑥 − 9.33) 𝑑𝑥 = 2 sin 𝑥 + 4.3 cos 𝑥 − 9.33𝑥 + 𝐶 ∫

4.

(4.1 sin 𝑥 + cos 𝑥 − 9.33∕𝑥) 𝑑𝑥 = −4.1 cos 𝑥 + sin 𝑥 − 9.22 ln |𝑥| + 𝐶 ∫

5.

6.

7.

8.

9.

cos 𝑥 sin 𝑥 (3.4 sec 2 𝑥 + − 3.2𝑒 𝑥) 𝑑𝑥 = 3.4 tan 𝑥 + − 3.2𝑒 𝑥 + 𝐶 ∫ 1.3 1.3 ( ∫

3 sec 2 𝑥 𝑒𝑥 3 𝑒𝑥 ) 𝑑𝑥 = tan 𝑥 − 1.3 cos 𝑥 − + 1.3 sin 𝑥 − +𝐶 2 3.2 2 3.2

7.6 7.6 cos(3𝑥 − 4) 𝑑𝑥 = sin(3𝑥 − 4) + 𝐶 ∫ 3

4.4 4.4 sin(−3𝑥 + 4) 𝑑𝑥 = cos(−3𝑥 + 4) + 𝐶 ∫ 3

1 𝑥 sin(3𝑥 2 − 4) 𝑑𝑥 = − cos(3𝑥 2 − 4) + 𝐶 : Substitute 𝑢 = 3𝑥 2 − 4. ∫ 6

10.

11.

12.

13.

14.

15.

1 𝑥 cos(−3𝑥 2 + 4) 𝑑𝑥 = − sin(−3𝑥 2 + 4) + 𝐶 : Substitute 𝑢 = −3𝑥 2 + 4. ∫ 6

(4𝑥 + 2) sin(𝑥 2 + 𝑥) 𝑑𝑥 = −2 cos(𝑥 2 + 𝑥) + 𝐶 : Substitute 𝑢 = 𝑥 2 + 𝑥. ∫

1 1 (𝑥 + 1)[cos(𝑥 2 + 2𝑥) + (𝑥 2 + 2𝑥)] 𝑑𝑥 = sin(𝑥 2 + 2𝑥) + (𝑥 2 + 2𝑥) 2 + 𝐶 : Substitute 𝑢 = 𝑥 2 + 2𝑥. ∫ 2 4 1 (𝑥 + 𝑥 2) sec 2(3𝑥 2 + 2𝑥 3) 𝑑𝑥 = tan(3𝑥 2 + 2𝑥 3) + 𝐶 : Substitute 𝑢 = 3𝑥 2 + 2𝑥 3. ∫ 6 (4𝑥 + 2) sec 2(𝑥 2 + 𝑥) 𝑑𝑥 = 2 tan(𝑥 2 + 𝑥) + 𝐶 : Substitute 𝑢 = 𝑥 2 + 𝑥. ∫ 1 (𝑥 2) tan(2𝑥 3) 𝑑𝑥 = − ln | cos(2𝑥 3)| + 𝐶 : Substitute 𝑢 = 2𝑥 3. ∫ 6


Solutions Section 16.3

16.

(4𝑥) tan(𝑥 2) 𝑑𝑥 = −2 ln | cos(𝑥 2)| + 𝐶 : Substitute 𝑢 = 𝑥 2. ∫

17.

6 sec(2𝑥 − 4) 𝑑𝑥 = 3 ln | sec(2𝑥 − 4) + tan(2𝑥 − 4)| + 𝐶 : Substitute 𝑢 = 2𝑥 − 4. ∫

18.

3 csc(3𝑥) 𝑑𝑥 = − ln | csc(3𝑥) + cot(3𝑥)| + 𝐶 : Substitute 𝑢 = 3𝑥. ∫

19.

20.

21.

22.

23.

24.

25.

1 𝑒 2𝑥 cos(𝑒 2𝑥 + 1) 𝑑𝑥 = sin(𝑒 2𝑥 + 1) + 𝐶 : Substitute 𝑢 = 𝑒 2𝑥 + 1. ∫ 2 𝑒 −𝑥 sin(𝑒 −𝑥) 𝑑𝑥 = cos(𝑒 −𝑥) + 𝐶 : Substitute 𝑢 = 𝑒 −𝑥. ∫ 0

∫ −𝜋

0 sin 𝑥 𝑑𝑥 = [− cos 𝑥]−𝜋 = − cos(0) + cos(−𝜋) = −1 − 1 = −2

𝜋

∫ 𝜋∕2

𝜋 cos 𝑥 𝑑𝑥 = [sin 𝑥]𝜋∕2 = sin(𝜋) − sin(𝜋∕2) = 0 − 1 = −1

𝜋∕3

∫ 0

tan 𝑥 𝑑𝑥 = −[ln | cos 𝑥|]0

𝜋∕3

𝜋∕2

∫ 𝜋∕6

= − ln | cos(𝜋∕3)| + ln | cos(0)| = − ln(1∕2) + ln(1) = ln(2)

cot 𝑥 𝑑𝑥 = [ln | sin 𝑥|]𝜋∕6 = ln | sin(𝜋∕2)| − ln | sin(𝜋∕6)| = ln(1) − ln(1∕2) = ln(2) 𝜋∕2

√𝜋+1

∫ 1

𝑥 cos(𝑥 2 − 1) 𝑑𝑥 =

1 1 1 cos 𝑢 𝑑𝑢 (substitute 𝑢 = 𝑥 2 − 1) = [sin 𝑢]0𝜋 = [sin(𝜋) − sin(0)] = 0 ∫ 2 2 2 0 𝜋

1 𝜋 sin 𝑢 𝑑𝑢 (substitute 𝑢 = 2𝑥 − 1) 2∫ 0

(𝜋+1)∕2

sin(2𝑥 − 1) 𝑑𝑥 =

∫ 1∕2 1 1 = [− cos 𝑢]0𝜋 = [− cos(𝜋) + cos(0)] = 1 2 2

26.

27.

28.

29.

30.

2∕𝜋 sin(1∕𝑥)

∫ 1∕𝜋

𝜋∕3

∫ 0

𝑥2

𝑑𝑥 = −

𝜋∕2

∫ 𝜋

sin 𝑢 𝑑𝑢 (substitute 𝑢 = 1∕𝑥) = [cos 𝑢]𝜋

𝜋∕2

= cos(𝜋∕2) − cos(𝜋) = 1

1∕2 sin 𝑥 1 1 1∕2 𝑑𝑥 = − 𝑑𝑢 (substitute 𝑢 = cos 𝑥) = [ ] =2−1=1 2 2 ∫ 𝑢 1 cos 𝑥 𝑢 1

1 1 1 cos(𝑎𝑥 + 𝑏) 𝑑𝑥 = cos 𝑢 𝑑𝑢 (substitute 𝑢 = 𝑎𝑥 + 𝑏) = sin 𝑢 + 𝐶 = sin(𝑎𝑥 + 𝑏) + 𝐶 ∫ ∫𝑎 𝑎 𝑎

1 1 1 sin(𝑎𝑥 + 𝑏) 𝑑𝑥 = sin 𝑢 𝑑𝑢 (substitute 𝑢 = 𝑎𝑥 + 𝑏) = − cos 𝑢 + 𝐶 = − cos(𝑎𝑥 + 𝑏) + 𝐶 ∫ ∫𝑎 𝑎 𝑎


cos 𝑥 1 cot 𝑥 𝑑𝑥 = 𝑑𝑥 = 𝑑𝑢 (substitute 𝑢 = sin 𝑥) = ln |𝑢| + 𝐶 = ln | sin 𝑥| + 𝐶 ∫ ∫ sin 𝑥 ∫𝑢 Solutions Section 16.3

31.

32.

csc 𝑥 + cot 𝑥 1 csc 𝑥 𝑑𝑥 = csc 𝑥 𝑑𝑥 = − 𝑑𝑢 ∫ ∫ ∫𝑢 csc 𝑥 + cot 𝑥 [substitute 𝑢 = csc 𝑥 + cot 𝑥, so 𝑑𝑢 = − csc 𝑥(csc 𝑥 + cot 𝑥) 𝑑𝑥] = − ln |𝑢| + 𝐶 = − ln | csc 𝑥 + cot 𝑥| + 𝐶

33.

35.

1 sin(4𝑥) 𝑑𝑥 = − cos(4𝑥) + 𝐶 ∫ 4

cos(−𝑥 + 1) 𝑑𝑥 = − sin(−𝑥 + 1) + 𝐶 ∫

1 sin(−1.1𝑥 − 1) 𝑑𝑥 = cos(−1.1𝑥 − 1) + 𝐶 ∫ 1.1 37.

39.

41.

42.

1 cot(−4𝑥) 𝑑𝑥 = − ln | sin(−4𝑥)| + 𝐶 ∫ 4 𝜋∕2

∫ −𝜋∕2 𝜋

∫ 0

34.

36.

38.

40.

1 cos(5𝑥) 𝑑𝑥 = sin(5𝑥) + 𝐶 ∫ 5 1 1 sin( 𝑥) 𝑑𝑥 = −2 cos( 𝑥) + 𝐶 ∫ 2 2 1 cos(4.2𝑥 − 1) 𝑑𝑥 = sin(4.2𝑥 − 1) + 𝐶 ∫ 4.2 1 tan(6𝑥) 𝑑𝑥 = − ln | cos(6𝑥)| + 𝐶 ∫ 6

sin 𝑥 𝑑𝑥 = 0 because, by symmetry, there is as much area above the 𝑥-axis as below.

cos 𝑥 𝑑𝑥 = 0 because, by symmetry, there is as much area above the 𝑥-xis as below.

2𝜋

(1 + sin 𝑥) 𝑑𝑥 = 2𝜋 because, by symmetry, the average value of 1 + sin 𝑥 over [0, 2𝜋] is 1; hence ∫ 0 the area under the curve is the same as the area under the line of height 1, which is 2𝜋. 43.

2𝜋

(1 + cos 𝑥) 𝑑𝑥 = 2𝜋 because, by symmetry, the average value of 1 + cos 𝑥 over [0, 2𝜋] is 1; hence ∫ 0 the area under the curve is the same as the area under the line of height 1, which is 2𝜋. 44.

𝐷

45.

+ − +

𝑥 1 0

𝐼

sin 𝑥

− cos 𝑥 − sin 𝑥

𝑥 sin 𝑥 𝑑𝑥 = −𝑥 cos 𝑥 + sin 𝑥 + 𝐶 ∫


𝐷

46.

+ − + −

𝐼

Solutions Section 16.3

𝑥2

cos 𝑥

2

− cos 𝑥

2𝑥 0

sin 𝑥

− sin 𝑥

𝑥 2 cos 𝑥 𝑑𝑥 = 𝑥 2 sin 𝑥 + 2𝑥 cos 𝑥 − 2 sin 𝑥 + 𝐶 ∫ 𝐷

47.

+ − + −

𝑥2

2𝑥 2

𝐼

cos(2𝑥)

1 sin(2𝑥) 2

− 14 cos(2𝑥) − 18 sin(2𝑥)

0

𝑥 ⎛𝑥2 1⎞ 𝑥 2 cos(2𝑥) 𝑑𝑥 = ⎜ − ⎟sin(2𝑥) + cos(2𝑥) + 𝐶 ∫ 4⎠ 2 ⎝2 48.

𝐷

+ 2𝑥 + 1 − +

2 0

𝐼

sin(2𝑥 − 1)

− 12 cos(2𝑥 − 1) − 14 sin(2𝑥 − 1)

1 1 (2𝑥 + 1) sin(2𝑥 − 1) 𝑑𝑥 = −(𝑥 + )cos(2𝑥 − 1) + sin(2𝑥 − 1) + 𝐶 ∫ 2 2 𝐷

49.

+ − +

sin 𝑥

cos 𝑥

− sin 𝑥

𝐼

𝑒 −𝑥

−𝑒 −𝑥 𝑒 −𝑥

𝑒 −𝑥 sin 𝑥 𝑑𝑥 = −𝑒 −𝑥 sin 𝑥 − 𝑒 −𝑥 cos 𝑥 − 𝑒 −𝑥 sin 𝑥 𝑑𝑥, so ∫ ∫ −𝑥 −𝑥 −𝑥 2 ∫ 𝑒 sin 𝑥 𝑑𝑥 = −𝑒 sin 𝑥 − 𝑒 cos 𝑥 + 𝐶; 1 1 𝑒 −𝑥 sin 𝑥 𝑑𝑥 = − 𝑒 −𝑥 cos 𝑥 − 𝑒 −𝑥 sin 𝑥 + 𝐶 ∫ 2 2


𝐷

50.

+

𝐼

cos 𝑥

− − sin 𝑥 +

− cos 𝑥

Solutions Section 16.3

𝑒 2𝑥

1 2𝑥 𝑒 2 1 2𝑥 𝑒 4

1 1 1 𝑒 2𝑥 cos 𝑥 𝑑𝑥 = 𝑒 2𝑥 cos 𝑥 + 𝑒 2𝑥 sin 𝑥 − 𝑒 2𝑥 cos 𝑥 𝑑𝑥 so ∫ 2 4 4∫ 5 ∫ 𝑒 2𝑥 cos 𝑥 𝑑𝑥 = 12 𝑒 2𝑥 cos 𝑥 + 14 𝑒 2𝑥 sin 𝑥 + 𝐶; 4

2 1 𝑒 2𝑥 cos 𝑥 𝑑𝑥 = 𝑒 2𝑥 cos 𝑥 + 𝑒 2𝑥 sin 𝑥 + 𝐶 ∫ 5 5 𝐷

51.

+ − + − 𝜋

∫ 0

sin 𝑥

2𝑥

− cos 𝑥

0

cos 𝑥

2

− sin 𝑥

𝑥 2 sin 𝑥 𝑑𝑥 = [−𝑥 2 cos 𝑥 + 2𝑥 sin 𝑥 + 2 cos 𝑥]0𝜋 = 𝜋 2 − 4 𝐷

52.

+ − + 𝜋∕2

∫ 0

𝑥2

𝐼

𝐼

𝑥

cos 𝑥

0

− cos 𝑥

1

sin 𝑥

𝑥 cos 𝑥 𝑑𝑥 = [𝑥 sin 𝑥 + cos 𝑥]0

53. Average =

𝜋∕2

= 𝜋∕2 − 1

1 𝜋 1 2 sin 𝑥 𝑑𝑥 = [− cos 𝑥]0𝜋 = 𝜋∫ 𝜋 𝜋 0


54. Average =

55.

56.

4 𝜋∫ 0

𝜋∕4

cos(2𝑥) 𝑑𝑥 =

sin 𝑥 𝑑𝑥 = lim

∫ 0

𝑀

𝑀 →∞ ∫ 0

cos 𝑥 𝑑𝑥 = lim

∫ 0

𝐷

57.

cos 𝑥

+

− − sin 𝑥

− cos 𝑥

+

𝜋∕4 4 1 4 1 2 sin(2𝑥) ]0 = 𝜋 [ 2 ] = 𝜋 𝜋 [2

Solutions Section 16.3

sin 𝑥 𝑑𝑥 = lim [− cos 𝑥]0𝑀 = lim (1 − cos 𝑀); diverges

𝑀

𝑀 →∞ ∫ 0

𝑀 →∞

𝑀 →∞

cos 𝑥 𝑑𝑥 = lim [sin 𝑥]0𝑀 = lim sin 𝑀; diverges 𝑀 →∞

𝐼

𝑀 →∞

𝑒 −𝑥

−𝑒 −𝑥 𝑒 −𝑥

𝑒 −𝑥 cos 𝑥 𝑑𝑥 = −𝑒 −𝑥 cos 𝑥 + 𝑒 −𝑥 sin 𝑥 − 𝑒 −𝑥 cos 𝑥 𝑑𝑥, so ∫ ∫ −𝑥 −𝑥 −𝑥 2 ∫ 𝑒 cos 𝑥 𝑑𝑥 = −𝑒 cos 𝑥 + 𝑒 sin 𝑥 + 𝐶 1 1 𝑒 −𝑥 cos 𝑥 𝑑𝑥 = − 𝑒 −𝑥 cos 𝑥 + 𝑒 −𝑥 sin 𝑥 + 𝐶 ∫ 2 2 ∞

∫ 0

𝑒 −𝑥 cos 𝑥 𝑑𝑥 = lim

𝑀

𝑀 →∞ ∫ 0

1 1 𝑒 −𝑥 cos 𝑥 𝑑𝑥 = lim [− 𝑒 −𝑥 cos 𝑥 + 𝑒 −𝑥 sin 𝑥]0𝑀 𝑀 →∞ 2 2

1 1 1 1 = lim (− 𝑒 −𝑀 cos 𝑀 + 𝑒 −𝑀 sin 𝑀 + ) = ; converges 𝑀 →∞ 2 2 2 2

58.

1 1 𝑒 −𝑥 sin 𝑥 𝑑𝑥 = − 𝑒 −𝑥 sin 𝑥 − 𝑒 −𝑥 cos 𝑥 + 𝐶 from Exercise 49. So, 2 2 ∞ 𝑀 1 1 𝑒 −𝑥 sin 𝑥 𝑑𝑥 = lim 𝑒 −𝑥 sin 𝑥 𝑑𝑥 = lim [− 𝑒 −𝑥 sin 𝑥 − 𝑒 −𝑥 cos 𝑥]0𝑀 ∫ ∫ 𝑀 → ∞ 𝑀 → ∞ 2 2 0 0 ∞

∫ 0

1 1 1 1 = lim (− 𝑒 −𝑀 sin 𝑀 − 𝑒 −𝑀 cos 𝑀 + ) = ; converges 𝑀 →∞ 2 2 2 2

𝜋 2.6 𝜋 (0.04 − 0.1 sin[ (𝑡 − 25)]) 𝑑𝑡 = 0.04𝑡 + cos[ (𝑡 − 25)] + 𝐾; 𝐶(12) = 1.50 gives ∫ 26 𝜋 26 2.6 𝜋 𝐾 = 1.02, so 𝐶(𝑡) = 0.04𝑡 + cos[ (𝑡 − 25)] + 1.02 𝜋 26 59. 𝐶(𝑡) =


𝜋 2.4 𝜋 (0.05 + 0.4 cos[ (𝑡 − 11)]) 𝑑𝑡 = 0.05𝑡 + sin[ (𝑡 − 11)] + 𝐾; 𝐶(5) = 5 gives 𝐾 = 4.75, ∫ 6 𝜋 6 2.4 𝜋 so 𝐶(𝑡) = 0.05𝑡 + sin[ (𝑡 − 11)] + 4.75 𝜋 6 Solutions Section 16.3

60. 𝐶(𝑡) =

61. Position = 62. Position =

10 𝜋 𝜋 9𝜋 𝜋 3𝜋 cos[ (𝑡 − 1)] 𝑑𝑡 = [6 sin( (𝑡 − 1))] = 6 sin − 6 sin(− ) = 12 feet 0 2 2 2 2

10

∫ 0

10 𝜋 𝜋 𝜋 𝜋 (− sin[ (𝑡 − 2)]) 𝑑𝑡 = [2 cos( (𝑡 − 2))] = 2 cos 2𝜋 − 2 cos(− ) = 2 feet 0 2 4 4 2

10

∫ 0

7 1 7 𝜋 1 60 ⋅ 5.4 𝜋 65 60 sin (𝑡 2.7) 𝑑𝑡 65𝑡 cos (𝑡 2.7) + − = − − [ ] ( [ 5.4 ]) [ 5.4 ] 5 2∫ 2 𝜋 5 ≈ 118 sunspots

63. Average =

9 1 9 𝜋 1 80 ⋅ 6.1 𝜋 85 + 80 sin [ (𝑡 − 3.05)])𝑑𝑡 = [85𝑡 − cos [ (𝑡 − 3.05)]] ( ∫ 5 4 6.1 5 𝜋 6.1 4 ≈ 143 sunspots

64. Average =

65. 𝑃 (𝑡) = 7.5 sin[𝜋(𝑡 − 13)∕26] + 12.5 (see Exercise 45 in Section 1 of this chapter). 13 1 1 Average = 7.5 sin[𝜋(𝑡 − 13)∕26] + 12.5 𝑑𝑡 = [−62.07 cos[𝜋(𝑡 − 13)∕26] + 12.5𝑡]013 ≈ 7.7% ∫ 13 0 13 66. 𝑃 (𝑡) = 25 sin[𝜋(𝑡 − 8)∕6] + 35 (see Exercise 46 in Section 1 of this chapter). 1 2 1 Average = 25 sin[𝜋(𝑡 − 8)∕6] + 35 𝑑𝑡 = [−47.75 cos[𝜋(𝑡 − 8)∕6] + 35𝑡]02 ≈ 46.9% 2∫ 2 0

67. a. Average voltage over [0, 1∕6] is 6 ∫0 165 cos(120𝜋𝑡) 𝑑𝑡 = 2.63[sin(120𝜋𝑡)]0 = 0. In one second the voltage goes through 60 periods of the cosine wave, hence reaches its maximum 60 times; hence the electricity has a frequency of 60 cycles per second. b. 1∕6

1∕6

c. We can use technology to estimate the average ˉ 𝑆 ≈ 13,612.5; hence the RMS voltage is approximately √13,612.5 ≈ 116.673 volts. Or, we can notice that the graph in (b) appears to be a sinusoid with an average value of ˉ 𝑆 = 165 2∕2 = 13,612.5, so that the RMS voltage is 165∕√2 ≈ 116.673 volts. 68. 𝑑(𝑡) = 5 sin[2𝜋(𝑡 − 1.625)∕13.5] + 10 (see Exercise 53 in Section 9.1). Average depth 1 14 1 14 5 sin[2𝜋(𝑡 − 1.625)∕13.5] + 10 𝑑𝑡 = [−10.743 cos[2𝜋(𝑡 − 1.625)∕13.5] + 10𝑡]10 ≈ 5.72 feet = 4∫ 4 10 69. 𝑇 𝑉 = ∫01[50,000 + 2,000𝜋 sin(2𝜋𝑡)] 𝑑𝑡 = [50,000𝑡 − 1,000 cos(2𝜋𝑡)]01 = $50,000


70. 𝑇 𝑉 = ∫01.5[100,000 − 2,000𝜋 sin(𝜋𝑡)] 𝑑𝑡 = [100,000𝑡 − 2,000 cos(𝜋𝑡)]01.5 = $148,000 Solutions Section 16.3

71. The integral over a whole number of periods is always zero, by symmetry. 72. One is the negative of the other.

73. 1, because the average value of 2 cos 𝑥 will be approximately 0 over a large interval. 74. 3, because the average value of cos 𝑥 will be approximately 0 over a large interval. 75. Integrate twice to get 𝑠 = −

𝐾 sin(𝜔𝑡 − 𝛼) + 𝐿𝑡 + 𝑀 for constants 𝐿 and 𝑀. 𝜔2

76. Answers will vary. One example is sin(√2𝑥).


Solutions Chapter 16 Review Chapter 16 Review

1. 𝐶 = 1, 𝐴 = 2, 𝛼 = 0, and 𝑃 = 2𝜋, so 𝜔 = 1. Thus, 𝑓(𝑥) = 2 sin 𝑥 + 1.

2. 𝐶 = 0, 𝐴 = 10, 𝛼 = −𝜋∕4, and 𝑃 = 𝜋, so 𝜔 = 2. Thus, 𝑓(𝑥) = 10 sin[2(𝑥 + 𝜋∕4)] = 10 sin(2𝑥 + 𝜋∕2). 3. 𝐶 = 2, 𝐴 = 2, 𝛼 = 1, and 𝑃 = 2, so 𝜔 = 𝜋. Thus, 𝑓(𝑥) = 2 sin[𝜋(𝑥 − 1)] + 2. We could also take 𝛼 = −1, getting 𝑓(𝑥) = 2 sin[𝜋(𝑥 + 1)] + 2.

4. 𝐶 = −2, 𝐴 = 3, 𝛼 = 0.25, and 𝑃 = 0.5, so 𝜔 = 4𝜋. Thus, 𝑓(𝑥) = 3 sin[4𝜋(𝑥 − 0.25)] − 2. We could also take 𝛼 = −0.25, getting 𝑓(𝑥) = 3 sin[4𝜋(𝑥 + 0.25)] − 2. 5. 𝐶 = 1, 𝐴 = 2, 𝛽 = 𝜋∕2, and 𝑃 = 2𝜋, so 𝜔 = 1. Thus, 𝑓(𝑥) = 2 cos(𝑥 − 𝜋∕2) + 1. 6. 𝐶 = 0, 𝐴 = 10, 𝛽 = 0, and 𝑃 = 𝜋, so 𝜔 = 2. Thus, 𝑓(𝑥) = 10 cos(2𝑥).

7. 𝐶 = 2, 𝐴 = 2, 𝛽 = 3∕2, and 𝑃 = 2, so 𝜔 = 𝜋. Thus, 𝑓(𝑥) = 2 cos[𝜋(𝑥 − 3∕2)] + 2. We could also take 𝛽 = −1∕2, getting 𝑓(𝑥) = 2 cos[𝜋(𝑥 + 1∕2)] + 2.

8. 𝐶 = −2, 𝐴 = 3, 𝛽 = 0.375, and 𝑃 = 0.5, so 𝜔 = 4𝜋. Thus, 𝑓(𝑥) = 3 cos[4𝜋(𝑥 − 0.375)] − 2. We could also take 𝛽 = −0.125, getting 𝑓(𝑥) = 3 cos[4𝜋(𝑥 + 0.125)] − 2. 9. −2𝑥 sin(𝑥 2 − 1)

10. 2𝑥[cos(𝑥 2 + 1) cos(𝑥 2 − 1) − sin(𝑥 2 + 1) sin(𝑥 2 − 1)] 11. 2𝑒 𝑥 sec 2(2𝑒 𝑥 − 1) 12.

(2𝑥 − 1) sec √𝑥 2 − 𝑥 tan √𝑥 2 − 𝑥 2√𝑥 2 − 𝑥

13. 4𝑥 sin(𝑥 2) cos(𝑥 2)

14. 4 cos(2𝑥) cos[1 − sin(2𝑥)] sin[1 − sin(2𝑥)] 15.

16.

17.

4 cos(2𝑥 − 1) 𝑑𝑥 = 2 sin(2𝑥 − 1) + 𝐶 (substitute 𝑢 = 2𝑥 − 1) ∫

1 (𝑥 − 1) sin(𝑥 2 − 2𝑥 + 1) 𝑑𝑥 = − cos(𝑥 2 − 2𝑥 + 1) + 𝐶 (substitute 𝑢 = 𝑥 2 − 2𝑥 + 1) ∫ 2 4𝑥 sec 2(2𝑥 2 − 1) 𝑑𝑥 = tan(2𝑥 2 − 1) + 𝐶 (substitute 𝑢 = 2𝑥 2 − 1) ∫

1 cos ( ) 𝑥 18. 𝑑𝑥 = − ln | sin(1∕𝑥)| + 𝐶 [substitute 𝑢 = sin(1∕𝑥)] ∫ 2 1 𝑥 sin ( ) 𝑥


1 𝑥 tan(𝑥 2 + 1) 𝑑𝑥 = − ln | cos(𝑥 2 + 1)| + 𝐶 (substitute 𝑢 = 𝑥 2 + 1) ∫ 2 Solutions Chapter 16 Review

19.

20.

21.

22.

𝜋

∫ 0

cos(𝑥 + 𝜋∕2) 𝑑𝑥 = [sin(𝑥 + 𝜋∕2)]0𝜋 = sin(3𝜋∕2) − sin(𝜋∕2) = −2

ln(𝜋)

∫ ln(𝜋∕2) 2𝜋

∫ 𝜋

𝑒 𝑥 sin(𝑒 𝑥) 𝑑𝑥 =

𝜋

∫ 𝜋∕2

𝜋 sin 𝑢 𝑑𝑢 (substitute 𝑢 = 𝑒 𝑥) = [− cos 𝑢]𝜋∕2 = − cos 𝜋 + cos(𝜋∕2) = 1

tan(𝑥∕6) 𝑑𝑥 = [−6 ln | cos(𝑥∕6)|]𝜋2𝜋 = −6 ln | cos(𝜋∕3)| + 6 ln | cos(𝜋∕6)|

= −6 ln(1∕2) + 6 ln(√3∕2) = 3 ln 3 𝐷

23.

+ − + −

𝐼

sin 𝑥

𝑥2

2𝑥

− cos 𝑥

0

cos 𝑥

2

− sin 𝑥

𝑥 2 sin 𝑥 𝑑𝑥 = −𝑥 2 cos 𝑥 + 2𝑥 sin 𝑥 + 2 cos 𝑥 + 𝐶 ∫ 𝐷

24.

+ − +

𝐼

sin 2𝑥

𝑒𝑥

−4 sin 2𝑥

𝑒𝑥

2 cos 2𝑥

𝑒𝑥

𝑒 𝑥 sin 2𝑥 𝑑𝑥 = 𝑒 𝑥 sin 2𝑥 − 2𝑒 𝑥 cos 2𝑥 − 4 𝑒 𝑥 sin 2𝑥 𝑑𝑥, so ∫ ∫ 𝑥 𝑥 𝑥 5 ∫ 𝑒 sin 2𝑥 𝑑𝑥 = 𝑒 sin 2𝑥 − 2𝑒 cos 2𝑥 + 𝐶, 1 2 𝑒 𝑥 sin 2𝑥 𝑑𝑥 = 𝑒 𝑥 sin 2𝑥 − 𝑒 𝑥 cos 2𝑥 + 𝐶 ∫ 5 5

25. 𝐶 = 10,500, 𝐴 = 1,500, 𝛼 = 52∕2 = 26, and 𝑃 = 52, so 𝜔 = 2𝜋∕52. Thus, 𝑠(𝑡) = 1,500 sin[(2𝜋∕52)(𝑡 − 26)] + 10,500 ≈ 1,500 sin(0.12083𝑡 − 3.14159) + 10,500.

26. 𝐶 = (50 + 15)∕2 = 32.5, 𝐴 = (50 − 15)∕2 = 17.5, 𝛽 = 12, and 𝑃 = 12, so 𝜔 = 2𝜋∕12 = 𝜋∕6. Thus, 𝑠(𝑡) = 17.5 cos[(𝜋∕6)(𝑡 − 12)] + 32.5 outbursts/week. 27. 𝑅 ′(𝑡) = (−0.05)20,000𝑒 −0.05𝑡 sin[(𝜋∕6)(𝑡 − 2)] + 20,000𝑒 −0.05𝑡(𝜋∕6) cos[(𝜋∕6)(𝑡 − 2)] = 20,000𝑒 −0.05𝑡−0.05 sin[(𝜋∕6)(𝑡 − 2)] + (𝜋∕6) cos[(𝜋∕6)(𝑡 − 2)] dollars/month 𝑅 ′(20) = 20,000𝑒 −1−0.05 sin[(𝜋∕6)(18)] + (𝜋∕6) cos[(𝜋∕6)(18)] ≈ −$3,852∕month; decreasing at a rate of $3,852 per month


28. 𝑅 (𝑡) = 15,000−0.12𝑒 month ′

−0.12𝑡

Solutions Chapter 16 Review cos[(𝜋∕6)(𝑡 − 4)] − (𝜋∕6)𝑒 −0.12𝑡 sin[(𝜋∕6)(𝑡 − 4)]; 𝑅 ′(10) ≈ $542 per 20

𝑅(𝑡) 𝑑𝑡 : ∫ 0 100000+20000*e^(-0.05*x)*sin((pi/6)*(x-2)) Answer: $2,029,700 29. Using technology to evaluate

10

𝑅(𝑡) 𝑑𝑡 : ∫ 0 20000 + 15000e^(-0.12*x)*cos((pi/6)*(t-4)) Answer: $222,300 30. Using technology to evaluate

𝑡

150 + 50 sin[(𝜋∕2)(𝑥 − 1)] 𝑑𝑥 ∫ 0 = [150𝑥 − (100∕𝜋) cos[(𝜋∕2)(𝑥 − 1)]]0𝑡 100 𝜋 cos [ (𝑡 − 1)] grams = 150𝑡 − 𝜋 2

31. Total consumption =

32. 𝐶 = 8,750, 𝐴 = 750, period = 12 months, so 𝜔 = 𝜋∕6, 𝛼 = 6 + 12∕4 = 9. So, 𝑠(𝑡) = 750 sin[𝜋(𝑡 − 9)∕6] + 8,750. Total expenditure in a year =

12

∫ 0

(750 sin[𝜋(𝑡 − 9)∕6] + 8,750) 𝑑𝑡 = [(750(6∕𝜋) cos[𝜋(𝑡 − 9)∕6] + 8,750𝑡]012 = $105,000


Solutions Chapter 16 Case Study Chapter 16 Case Study 1. The amplitude of the cyclical term is

𝐴 = √𝑃 2 + 𝑄 2 ≈ √0.5269 2 + 1.289 2 ≈ 1.393.

Therefore, the volume of empty seat miles fluctuates from 1.393 billions seat miles below the secular line to 1.393 miles above it. 2. Actual: January 2019 corresponds to 𝑡 = 67, so the actual total was

20 + 17 + 15 + 16 + 15 + 12 + 13 + 15 + 18 + 17 + 18 + 15 = 191 billion seat miles.

The total predicted by the model is 3. The function to graph is 𝑦 =

𝑡

𝑉 (𝑥) 𝑑𝑥 =

79

∫ 67

𝑉 (𝑡) 𝑑𝑡 ≈ 198 billion seat miles.

𝑡

[𝑃 sin(𝜋𝑥∕6) + 𝑄 cos(𝜋𝑥∕6) + 𝐵𝑥 + 𝐶] 𝑑𝑥 ∫ ∫ 0 0 6𝑄 6𝑃 [1 − cos(𝜋𝑡∕6)] + sin(𝜋𝑡∕6) + 𝐵𝑡 2∕2 + 𝐶𝑡 = 𝜋 𝜋 6(−0.5269) 6(−1.289) [1 − cos(𝜋𝑡∕6)] + sin(𝜋𝑡∕6) + 0.03331𝑡 2∕2 + 14.10𝑡. = 𝜋 𝜋

Graph of this function superimposed with 𝑦 = 1,000 : (6*0.5269/pi)*(cos(pi*x/6) - 1)-(6*1.289/pi)*sin(pi*x/6)+ 0.03331x^2/2 + 14.10x

The graphs cross at 𝑡 ≈ 66, corresponding to December 2018.


Solutions Chapter 16 Case Study 4. The data are arranged as follows:

and the regression output is

giving the model

𝑉 (𝑡) = −0.2024 sin(𝜋𝑡∕6) − 0.4434 cos(𝜋𝑡∕6) − 0.03337𝑡 + 2.329𝑥1 − 1.640𝑥2 + 14.18.


Solutions Chapter 16 Case Study 5. Setup for regression:

Regression output:

Resulting model:

𝑉 (𝑡) = −0.5533 sin(𝜋𝑡∕6) − 1.060 cos(𝜋𝑡∕6) + 0.03981𝑡 + 13.95

Graph:


Turn static files into dynamic content formats.

Create a flipbook
Issuu converts static files into: digital portfolios, online yearbooks, online catalogs, digital photo albums and more. Sign up and create your flipbook.